You are on page 1of 1691

SBI Clerk & RRB PO Mains PDF Course 2023

Reasoning Ability Day -1 (Eng)

Reasoning Ability

Directions (1-5): Study the following information a) Tuesday-C- Prabhas


carefully and answer the given questions. b) Friday- D- Mohanlal
Seven persons – A, B, C, D, E, F and G are c) Wednesday-B-Kamal
going to meet their favorite actors on seven d) Thursday-E-Mohanlal
different days from Sunday to Saturday of the e) Saturday-D-Prabhas
same week. Each person likes different actor -
Ajith, Vijay, Surya, Prabhas, Kamal, Mohanlal, 3. If each person bought a gift for their favorite
and Mohan. actor and the cost of the gift bought by the
The one who likes Mohan goes three days after person who goes on the first day is Rs.30000,
C, who goes one of the days before Wednesday. second day is Rs.34000 such that the cost is
Only three persons go between C and D. The increased by Rs.4000 on each day, then what is
number of persons going before D is one more the total cost of the gift bought by C, E and D?
than the number of persons going after F. B goes a) Rs.136000
three days before the one who likes Prabhas. b) Rs.148000
Only one person goes between G and the one c) Rs.138000
who likes Prabhas. The one who likes Mohanlal d) Rs.126000
goes immediately before G. B and A like neither e) Rs.128000
Surya nor Mohanlal. The number of persons
going between A and the one who likes Ajith is 4. Which of the following statements is/are not
one more than the number of persons going true as per the given arrangement?
between the one who likes Ajith and the one who a) As many persons go between F and D as
likes Kamal. E goes one of the days before the before E
one who likes Vijay and after the one who likes b) G goes three days after the one who likes Ajith
Surya. F doesn’t like Surya. c) No one goes between F and the one who likes
1. Who among the following person likes Ajith? Kamal
a) The one who goes on Sunday d) Both a and c
b) E e) Both b and c
c) D
d) G 5. Who among the following person goes three
e) The one who goes on Monday. days before E?
a) The one who goes on Tuesday
2. Which of the following combination is true? b) B

Click Here For Bundle PDF Course | support@guidely.in Page 1 of 8


SBI Clerk & RRB PO Mains PDF Course 2023
Reasoning Ability Day -1 (Eng)

c) G 6. Who among the following person doesn’t eat


d) A Crinkle cookie?
e) The one who likes Ajith a) The one who works in Accenture
b) The one who eats the same cookie as the one
Directions (6-10): Study the following information who works in CTS
carefully and answer the given questions. c) H
Nine persons – A, B, C, D, E, F, G, H and I are d) D
eating three different cookies viz. Parle-G, Sun e) The one who works in Zoho
feast, Crinkle. At-least two persons but not more
than four persons eat the same cookie. Each of 7. F works in which of the following company?
them works in different company viz. TCS, a) Reliance
Infosys, Wipro, CTS, Accenture, HDFC, Zoho, b) Accenture
HCL and Reliance. c) Infosys
Note: If it is said that X eats the same cookie as d) HCL
the one who works in “Y” company, then it e) HDFC
means both of them eat the same cookie but X
doesn’t work in “Y” company. 8. Which of the following pair of persons eat the
The one who works in Accenture eats the same same cookie?
cookie as H. The one who works in Wipro eats I. A and the one who works in CTS
Crinkle cookie. H didn’t work in Wipro. F neither II. F and E
eats the same cookie as the one who works in III. D and the one who works in Accenture
Accenture nor eats Sun feast. Neither F nor C IV. I and B
works in Wipro. The one who works in Zoho eats a) Only II and III
the same cookie as F. C eats the same cookie as b) Only III
the one who works in Reliance. The one who c) Only I and IV
works in Reliance eat the same cookie with only d) Only II and IV
two persons. I works in TCS and eats the same e) Only I, II and III
cookie as C. I didn’t eat Parle-G. A eats the
same cookie as I. The one who works in Infosys 9. Who among the following person works in
eats the same cookie as B, who either works in HDFC?
CTS or in Reliance. The one who works in CTS a) F
doesn’t eat Parle-G. Neither E nor G works in b) H
Wipro. The one who works in HCL eats the same c) C
cookie as E. G didn’t work in Zoho. d) I

Click Here For Bundle PDF Course | support@guidely.in Page 2 of 8


SBI Clerk & RRB PO Mains PDF Course 2023
Reasoning Ability Day -1 (Eng)

e) D a) A
b) H
10. Which of the following statement(s) is/are c) B
FALSE as per the given arrangement? d) D
a) C and A eat the same cookie e) E
b) E works in Accenture
c) D eats the same cookie as the one who works 12. If point U is 4m south of point A, then what is
in CTS the shortest distance between points U and F
d) H works in Infosys (approximately)?
e) All the statements are false a) 11m
b) 13m
Direction (11-15): Study the following information c) 14m
carefully and answer the given questions. d) 17m
Person X starts walking from point A towards the e) 18m
north and walks for 5m to reach point B, from
where he takes a right turn and walks for 8m to 13. If Person X want to reach Person Y’s starting
reach point C. Then, again he takes a right turn point from point C, then which among the
and walks for 7m to reach point D. From, point D following option will be the shortest distance?
he takes a left turn and walks for 3m to reach I. Go to point B and go to point K and then go to
point E. Finally, he takes a left turn and walks for point I via shortest distance, then reach point H.
12m to reach point F. II. Go to point A via shortest distance and then
Person Y starts walking from Point H towards the go to point I via shortest distance and then reach
west and walks for 4m to reach point I, where he point H.
takes a right turn and walks for 9m to reach point III. Go to point J through point K and then reach
J. Then, he again takes a right turn and walks for point H via shortest distance.
7m to reach point K. At, point K he takes a left a) Both I and II
turn and walks for 5m to reach point L. Finally, he b) Only II
takes a right turn and walks for 13m to reach c) Both II and III
point F. d) Only III
11. Four of the following five points are alike in a e) Both I and III
certain way based on the directions with respect
to point J in the given arrangement and thus form 14. Which of the following statements is/are
a group. Which one of the following does not TRUE as per the given arrangement?
belong to the group?

Click Here For Bundle PDF Course | support@guidely.in Page 3 of 8


SBI Clerk & RRB PO Mains PDF Course 2023
Reasoning Ability Day -1 (Eng)

I. Person X initial position is North-west of the 16. Which of the following code is wrongly
person Y Initial position represented in the bolded sentence?
II. The shortest distance between L and B is a) %^∞
√29m b) ®^∞
III. The distance between the points IH is half of c) ®βα
the distance between the points BC d) Ωβ∞
IV. J, K, B and C are not in a straight line e) All are true
a) Only I, II and IV
b) Only II and IV 17. Which among the following code comes in
c) Only II, III and IV the place of _ (?) _.
d) Only II and III a) +^∞
e) Only I, II and III b) Ωβ^
c) +βα
15. If point V is 5m east of point L, then the d) %^∞
distance between F and V is twice that of which e) Ωβ∞
of the following points?
a) BC/IH 18. Which of the following phrase represent the
b) DE code “+βα @β∞” in the given code language?
c) LF+JI a) Handsome war
d) CD b) Juncture are
e) JK- DE c) Faithful ear
d) The Dwindle
Directions (16-20): Study the following e) Typical Day
information carefully and answer the given
questions. 19. What does the code “+^α ®β∞” represent in
In a certain code language, the given coded language?
“Bundle Quantity Sales fast” is coded as a) Chipotle Thunder
“Ωβ∞ ∑^∞ %β$ +βα” b) Distance Trouble
“Truss Rope Narrow Sales” is coded as c) Umbrella Snowman
“∑β∞ Ωβ∞ Ωβ$ %^∞” d) Presence Habitue
“Chariot Quarrel Based rope” is coded as e) Both a and c
“%^∞ ®^∞ Ωβ∞ ®βα”
“Carbon Bundle Bunch Comet” is coded as 20. What is the code for “Import Share Loss”?
“∑^∞ (?) ∑β∞ Ωβ$” a) Ωβ∞ ∑β∞ +^α

Click Here For Bundle PDF Course | support@guidely.in Page 4 of 8


SBI Clerk & RRB PO Mains PDF Course 2023
Reasoning Ability Day -1 (Eng)

b) %β$ +^α ∑^$ d) ∑β∞ Ω^∞ %β$


c) +βα Ωβ∞ %β$ e) Ω^∞ ®β∞ ∑βα
Click Here to Get the Detailed Video Solution for the above given Questions
Or Scan the QR Code to Get the Detailed Video Solutions

Answer Key with Explanation


Directions (1-5):  The number of persons going before D is
1. Answer: E one more than the number of persons
2. Answer: D going after F.
3. Answer: C From the above conditions, we have three
4. Answer: B possibilities:
5. Answer: E
Final arrangement:

Again we have,
 B goes three days before the one who
likes Prabhas.
 Only one person goes between G and the
We have,
one who likes Prabhas.
 The one who likes Mohan goes three
 The one who likes Mohanlal goes
days after C, who goes one of the days
immediately before G.
before Wednesday.
 B and A like neither Surya nor Mohanlal.
 Only three persons go between C and D.

Click Here For Bundle PDF Course | support@guidely.in Page 5 of 8


SBI Clerk & RRB PO Mains PDF Course 2023
Reasoning Ability Day -1 (Eng)

While applying the above conditions, case 2 gets 8) Answer: A


eliminated, because B doesn’t like Mohanlal. 9) Answer: C
10) Answer: B
Final arrangement

We have,
 The one who works in Accenture eats the
Again we have,
same cookie as H.
 The number of persons going between A
 The one who works in Wipro eats Crinkle
and the one who likes Ajith is one more
cookie.
than the number of persons going
 H didn’t work in Wipro.
between the one who likes Ajith and the
From the above condition, there are three
one who likes Kamal.
possibilities
 E goes one of the days before the one
who likes Vijay and after the one who
likes Surya.
 F doesn’t like Surya.
From the above conditions, case 3 gets
eliminated, because we cannot place the one
Again, we have
who likes Ajith. Thus, case 1 gives the final
 F neither eats the same cookie as the one
arrangement.
who works in Accenture nor eats Sun
feast.
 Neither F nor C works in Wipro.
 The one who works in Zoho eats the
same cookie as F.
 C eats the same cookie as the one who
works in Reliance.
 The one who works in Reliance eat the
Directions (6-10): same cookie with only two persons.
6) Answer: E  I works in TCS and eats the same cookie
7) Answer: D as C.

Click Here For Bundle PDF Course | support@guidely.in Page 6 of 8


SBI Clerk & RRB PO Mains PDF Course 2023
Reasoning Ability Day -1 (Eng)

 I didn’t eat Parle-G. 11) Answer: C (All the points are south-east of
 A eats the same cookie as I. point J, except option c)
After applying the above conditions case-2 gets 12) Answer: E
eliminated because there is no possibility to 13) Answer: B
place A. From statement I, the distance covered is -
>8+2+11 (approx.) +4=25m
From statement II, the distance covered is ->9
(approx.) +10 (approx) +4=23m
From statement II, the distance covered is -
>8+2+7+10 (approx)=27m
14) Answer: D
15) Answer: E
Again, we have
Final arrangement
 The one who works in Infosys eats the
same cookie as B, who either works in
CTS or in Reliance.
 The one who works in CTS doesn’t eat
Parle-G.
 Neither E nor G works in Wipro.
 The one who works in HCL eats the same
cookie as E.
 G didn’t work in Zoho.
After applying the above conditions case-1 gets
eliminated because G works in Zoho, hence Directions (16-20):

case-3 shows the final arrangement. 16) Answer: B


17) Answer: E
18) Answer: C
19) Answer: E
20) Answer: D
The logic for the coded language is,
The logic for the first code for each word is
based on the number of letters in each word,
Direction (11-15):
If the word has three letters, then the first code
of the word is “@”

Click Here For Bundle PDF Course | support@guidely.in Page 7 of 8


SBI Clerk & RRB PO Mains PDF Course 2023
Reasoning Ability Day -1 (Eng)

If the word has four letters, then the first code of If a word ends with a vowel, then the second
the word is “%” code of the word is “^”
If the word has five letters, then the first code of If a word ends with a consonant, then the
the word is “Ω” second code of the word is “β”
If the word has six letters, then the first code of The logic for the third code for each word is
the word is “∑” based on the number of vowels in each word,
If the word has seven letters, then the first code If the word has one vowel, then the third code of
of the word is “®” the word is “$”
If the word has eight letters, then the first code of If the word has two vowels, then the third code of
the word is “+” the word is “∞”
The logic for the second code for each word is If the word has three vowels, then the third code
based on the ending letter of each word, of the word is “α”

Click Here For Bundle PDF Course | support@guidely.in Page 8 of 8


SBI Clerk & RRB PO Mains PDF Course 2023
Quantitative Aptitude Day -1 (Eng)

Quantitative Aptitude

Directions [1–4]: Read the following information carefully and answer the questions based on it.
The Pie chart given below shows the % distribution of people visited in five different malls – P, Q, R, S
and T on Monday. Average number of people visited in all malls together is 80.

Note:
a) Ratio of people visited in mall P and Q together to that of mall R, S and T together is 11:9
respectively.
b) Value of Y, N, and Z are in increasing arithmetic progression with common difference of A0.
1) Different number of Male, female and children c) II only
visited in mall Q. If the ratio of number of male d) I and II only
and female visited in mall Q is Z: (M – 5) e) None of these
respectively then the difference between number
of children visited mall Q and the total number of 2) The number of females visited in mall S is
people visited in mall T is ____. half of the males visited in mall P, and the
Find which of the following value cannot be fit in difference between the males visited in mall S
the blank. and females visited in mall P is (2Y + 4). Find the
I. 42 total number of people visited in mall S is how
II. 29 much % more or less than the females visited in
III. 11 mall P? (Note: Only males and females visited in
a) I and III only mall P and S)
b) III only a) 500/9%

Click Here For Bundle PDF Course | support@guidely.in Page 1 of 12


SBI Clerk & RRB PO Mains PDF Course 2023
Quantitative Aptitude Day -1 (Eng)

b) 200/9% b) II and III only


c) 250/9% c) I and II only
d) 350/9% d) I, II, and III
e) 550/9% e) None of these

3) For mall R, if entry fee for Male is Rs. N and 4) People visited the mall U is 40% more than
for female is Rs. (N – 3), and the total revenue that of T, and ratio of male and females visited in
generated on Monday by mall R is Rs. B. The U is 5:2 respectively. Males visited in U is B%
difference between number of male and female more/less than that of people visited in mall Q.
visited mall R is Z. Find difference between B and M?
Find which of the following is/are possible value a) 40/3
of B/2. b) 10
I. 440 c) 15
II. 420 d) 5
III. 444 e) None of these
a) I only

Directions [5–8]: Read the following information carefully and answer the questions based on it.
The chart given below shows the number of employees in two department (A and B) of five companies –
P, Q, R, S and T.

Click Here For Bundle PDF Course | support@guidely.in Page 2 of 12


SBI Clerk & RRB PO Mains PDF Course 2023
Quantitative Aptitude Day -1 (Eng)

5) For department A, Average number of 7) Number of males in department B of company


employees in company R, T and U together is Q is thrice as that of females, while number of
60, while number of males and females in males in department A of company S is one
company U is Z and Z/3 respectively and number fourth as that of males in both the department of
of females in company T is 40% less than that of company Q. Find the minimum possible value of
males. Find number of males in company T is Z, if the number of females in department B of
how much % more or less than ((Z/3) + 5)? company S is twice of females in department A
a) 500/9% of same company and females in department A
b) 400/9% of company Q is not less than that of males in
c) 350/9% department A of company Q.
d) 420/9% Z = [(females in department B of company
e) None of these S)/(females in department B of company Q)] x
100
6) For department A, females in P are eight a) 25
times as that of females in R, while total females b) 40
in both the companies together > 30 and the total c) 60
males in both the companies together can be d) 37.5
_____. e) None of these
I. 33
II. 44 8) For company R, females in both department is
III. 24 same, while males in department A are 3Z(>
III. 19 females in both department), and males in
a) I and II only department B are 5Y. Find the maximum
b) II, III and IV only possible males in company R.
c) III and I only a) 72
d) IV and I only b) 66
e) None of these c) 69
d) 60
e) None of these

Directions [9–12]: Read the following information carefully and answer the questions based on it.
The table given below shows the total items (Burgers + Pizzas) sold by a shopper on five different days
of a week from Monday to Friday.

Click Here For Bundle PDF Course | support@guidely.in Page 3 of 12


SBI Clerk & RRB PO Mains PDF Course 2023
Quantitative Aptitude Day -1 (Eng)

Note:
a) Total items sold on Tuesday is L less than that on Wednesday, while total items sold on Thursday is
more than that on Tuesday.
b) All variables in table are natural numbers.
9) Quantity I = Burger sold on Tuesday a) Rs.26980
Quantity II = Pizzas sold on Wednesday b) Rs.27360
Find correct relation between Quantity I and c) Rs.26360
Quantity II. d) Rs.26860
a) Quantity I > Quantity II e) None of these
b) Quantity I < Quantity II
c) Quantity I = Quantity II 12) Find the average of total items(burger +
d) Quantity I + Quantity II > 240 pizza) sold on Monday and Wednesday
e) Can’t be determined together?
a) 275
10) On Saturday, total items (Burgers + pizzas) b) 265
sold are (L + 5D), and ratio of Burgers and c) 285
Pizzas sold is 4Y: C . Find the difference d) 245
between pizzas and burger sold on Saturday. e) None of these
a) 150
b) 100 13) Which of the following is/are definitely true.
c) 120 I. Z:L = 3:2
d) 125 II. 3Y% of 2D = perfect square
e) None of these III. L – M = 2D
a) II and III only
11) If each burger is sold at Rs. 12 and each b) I and III only
pizza is sold at Rs. 50, then find the total c) I and II only
revenue generated by shopper for all five days d) I, II and III
together? e) None of these

Click Here For Bundle PDF Course | support@guidely.in Page 4 of 12


SBI Clerk & RRB PO Mains PDF Course 2023
Quantitative Aptitude Day -1 (Eng)

Directions [14–17]: Read the following I. 41


information carefully and answer the questions II. 45
based on it. III. 49
A survey conducted among 360 people in Delhi a) I and II only
about liking of three brands among P, Q, and R. b) I and III only
Number of people those who likes only R is twice c) II and III only
of those who likes only P (M). Number of people d) I, II, and IIII
those who likes all three products is 8.33% of the e) None of these
people surveyed, while number of people those
who likes both P and Q but not R is same as 16) Find the maximum number of people those
those who likes both P and R but not Q. Out of likes brand P?
total number of people those who likes R, total a) 150
people those who likes exactly one of the other b) 120
brands is same as people those who likes c) 180
exactly other two brands. Number of people d) 136
those who do not likes any of the given product is e) None of these
same as number of people those who likes both
Q and R but not P. Number of people those who 17) Number of people those don’t like any
likes only Q is M/2 less than that of those who product is twice of those likes both P and Q but
likes only R. not R, then find the unit digit of (K + M/5)M, where
14) Find the difference between the number of K is the number of people those likes at least
people those likes only Q and only R? one brand?
a) 20 a) 1
b) 15 b) 4
c) 30 c) 8
d) 24 d) 9
e) None of these e) None of these

15) People don’t like any of the product is two- Directions [18–20]: Read the following
digit prime number (< 20), and the number of information carefully and answer the questions
people those likes exactly two products based on it.
are_____. There are two shoppers P and Q sold some pens
Find which of the following value is/are fit in the and pencils. Ratio of number of pens and pencils
blank. sold by P is Z: Y, while the number of pens sold

Click Here For Bundle PDF Course | support@guidely.in Page 5 of 12


SBI Clerk & RRB PO Mains PDF Course 2023
Quantitative Aptitude Day -1 (Eng)

by Q is twice as that of P. The total number of 19) If the selling price of each Pen and Pencil is
items sold by Q (pens + pencils) is 120, while the Rs. 12 and Rs.5 respectively then find the total
ratio of number of pens sold by P to that of revenue generated by shopper Q?
pencils sold by Q is Z:10 respectively. Number of a) Rs. 1090
pencils sold by P is 120/ (5 + Z) more than that of b) Rs. 1065
pens. c) Rs. 1060
Note: Y is the largest single digit number. d) Rs. 1080
18) Find the difference between the total number e) None of these
of pens sold by both the shoppers together and
pencils sold by both the shoppers together? 20) Find the difference between the total number
a) 20 of pens and pencils sold by both the shoppers?
b) 15 a) 30
c) 10 b) 40
d) 12 c) 20
e) Cannot be determined d) 60
e) None of these
Click Here to Get the Detailed Video Solution for the above given Questions
Or Scan the QR Code to Get the Detailed Video Solutions

Answer Key with Explanation


Directions [1–4]: So, number of people visited in (R + S + T) =
Total number of people visited in all malls 400 - 220 = 180
together = 80 x 5 = 400 Y, N, and Z are in increasing AP, with common
Number of people visited in (P + Q) = 11/20 x difference of = A0 = 1
400 = 220 So, 1% of 400 = 4
Let number of people visited in T = a

Click Here For Bundle PDF Course | support@guidely.in Page 6 of 12


SBI Clerk & RRB PO Mains PDF Course 2023
Quantitative Aptitude Day -1 (Eng)

Now, So, number of males visited in mall P = 2 x a =


(a – 4 + a + a + 4) = 180 2a
People visited mall T = 180/3 = 60 Number of Females in mall P = (140 – 2a)
Value of N = 60/400 x 100 = 15% Number of Males in mall S = (56 – a)
So, value of Y = 15 – 1 = 14% Now,
Value of Z = 15 + 1 = 16% (140 – 2a – 56 + a) = 2 x 14 + 4
Also, Value of a = 52
M + 15 + M = 55 56 - a - 140 + 2a = 2 x 14 + 4
Value of M = 20 a = 116 (this is not possible)
Now, we can find the people visited in all given So, number of females in P = 140 – 2 x 52 = 36
malls. Required % change = (56 – 36)/36 x 100 =
500/9%
Hence answer is option A

3) Answer: B
1) Answer: C For mall R,
According to question, Difference between Male and female = 16
Total number of people visited mall Q = 80 Sum of male and female = 64
Ratio of number of male and female visited in Number of males = (64 + 16)/2 OR (64 – 16)/2 =
mall Q = 16: (20 – 5) = 16:15 40 or 24
That means, number of male and females must Number of females = 24 or 40
be multiple of = (16 + 15) = 31 Required revenue = (40 x 15 + 24 x 12) or (24 x
Possible values of (male + female) in Q = 31 or 15 + 40 x 12) = 888 or 840
62 Required value = Rs. 444 or Rs. 420
So, number of children visited in Q = (80 – 31) or Only II and III is possible,
(80 – 62) = 49 or 18 Hence answer is option B
Required difference = (60 – 49) or (60 – 18) = 11
or 42 4) Answer: D
Only II is not possible, For mall U
Hence answer is option C Total number of people visited = 60 x 140% = 84
Number of Males visited = 84 x 5/7 = 60
2) Answer: A Number of People visited in Q = 80
Let number of females visited in mall S = a Value of B = (80 – 60)/80 x 100 = 25%

Click Here For Bundle PDF Course | support@guidely.in Page 7 of 12


SBI Clerk & RRB PO Mains PDF Course 2023
Quantitative Aptitude Day -1 (Eng)

Required difference = 25 – 20 = 5 Number of males in R = 24 – 4 = 20


Hence answer is option D Number of males in (P + R) = 22 + 20 = 42
If a = 5
5) Answer: B Number of males in P = 54 – 40 = 14
For department A, Number of males in R = 24 – 5 = 19
Number of employees in (R + T + U) = 60 x 3 = Number of males in (P + R) = 14 + 19 = 33
180 If a = 6
So, number of employees in U = 180 – 24 - 32 = Number of males in P = 54 – 48 = 6
124 Number of males in R = 24 – 6 = 18
Now, Number of males in (P + R) = 18 + 6 = 24
Z + Z/3 = 124 Further values of a is not possible.
Value of Z = 93 So, I and III is possible
Value of Z/3 = 93/3 = 31 Hence answer is option C
Value of ((Z/3) + 5) = 31 + 5 = 36
Employees in T = 32 7) Answer: C
Ratio of males and females in T = 5:3 Employees in department B of company Q = 80
So, males in T = 5/8 x 32 = 20 Ratio of males and females in department B of
Required % change = (36 – 20)/36 x 100 = company Q = 3:1
400/9% So, females in department B of company Q = 1/4
Hence answer is option B x 80 = 20
Males in department B of company Q = 80 – 20
6) Answer: C = 60
For department A, Males in department A of company Q ≤ females
Number of employees in P = 54 in same department of same company
Number of employees in R = 24 So, maximum possible males in department A of
Let the number of females in R = a company Q = 120/2 = 60
So, number of females in P = 8 x a = 8a So, maximum possible males in department A of
Now, company S = 1/4 x (60 + 60) = 30
8a + a > 30 So, minimum possible females in department A
Value of a > 10/3 of company S = 36 – 30 = 6
First possible value of a = 4 So, minimum possible females in department B
If a = 4 of company S = 2 x 6 = 12
Number of males in P = 54 – 32 = 22 Minimum value of Z = 12/20 x 100 = 60

Click Here For Bundle PDF Course | support@guidely.in Page 8 of 12


SBI Clerk & RRB PO Mains PDF Course 2023
Quantitative Aptitude Day -1 (Eng)

Hence answer is C but Burgers – Pizzas = 40


ratio units’ difference between burgers and
8) Answer: B Pizzas = 3, 2, and 1
For company R, difference of ratio units cannot be 3, that means
Let the number of females in both department = burgers and pizzas got non-integral value.
a If Burgers: Pizzas = 4:2
Now, That means, burger = 4/(4 – 2) x 40 = 80
3Z + a = 24………… (1) Pizzas = 80/2 = 40
5Y + a = 48 Value of L = 80 – 60 = 20
5Y – 3Z = 24 Total items sold on Wednesday = 80 + 40 = 120
Z = 5Y/3 – 8 So, total items sold on Tuesday = 120 – 20 =
When Y is maximum, also Z is maximum. 100
Maximum possible value of Y = 9 So, Burgers sold on Tuesday = (100 + 60)/2 = 80
So, maximum possible value of Z = 5 x 9/3 – 8 = So, 4Y2 + 20 = 80
7 Y = √15 (Y must be natural number)
So, maximum possible males in company = 3 x 7 So, that means Burgers: Pizzas on Wednesday
+ 5 x 9 = 66 = 4:3
Hence answer is option B Value of C = 5
Burgers sold = 40 x 4 = 160
Directions [9–12]: Pizzas sold = 40 x 3 = 120
For Monday, Total items sold = 160 + 120 = 280
Burgers: Pizza = 3:2 L + 60 = 160
Burgers – Pizza = 50 Value of L = 100
So, burgers sold = 3/1 x 50 = 150 For Tuesday,
Pizzas sold = 50 x 2/1 = 100 Total items sold on Tuesday = 280 – 100 = 180
Total items sold = 150 + 100 = 250 Burgers sold = (180 + 60)/2 = 120
Value of Z = 150 So, 4Y2 + 20 = 120
For Wednesday Value of Y = 5
(Burgers – Pizzas) = 150/2 – 35 = 40 Pizzas sold = 180 - 60 = 60
Burgers: Pizzas = 4: (C – 2) So, A:B = 120:60 = 2:1
C must be greater than 2. For Friday
So, value of C = 3, 4, 5 Burgers – Pizzas = 64
so, burgers: Pizzas = 4:1, 4:2, and 4:3 Burgers: Pizzas = 5: 1 = 5:1

Click Here For Bundle PDF Course | support@guidely.in Page 9 of 12


SBI Clerk & RRB PO Mains PDF Course 2023
Quantitative Aptitude Day -1 (Eng)

So, burgers = 5/4 x 64 = 80 Total burgers sold = 630


Pizzas = 80 – 64 = 16 Total pizzas sold = 386
Total items sold = 80 + 16 = 96 Required revenue = 630 x 12 + 386 x 50 = 7560
Value of M = 80/2 = 40 + 19300 = Rs. 26860
For Thursday, Hence answer is option D
Burgers sold = 120
Burgers: Pizzas = 2 x 2: E = 4:E 12) Answer: B
Ratio can be = 4:1, 4:2, 4:3 Required average = (250 + 280)/2 = 265
Total items sold in Thursday > 180 Hence answer is option B
So, pizzas sold = 120/4 x 3 = 90
Total items = 120 + 90 = 210 13) Answer: D
Burgers: Pizzas = 120:90 = 4:3 I. Z:L = 3:2
Value of E = 3 Required ratio = 150:100 = 3:2 (true)
Value of D = 120 – 90 = 30 II. 3Y% of 2D = perfect square
3 x 5% x 2 x 30 = 9 (True)
III. L – M = 2D
100 – 40 = 2 x 30
60 = 60
This statement is true.
9) Answer: C
So, I, II and III are true.
According to the question,
Hence answer is option D
Quantity I = 120
Quantity II = 120
Directions [14–17]:
So, Quantity I = Quantity II
Total number of people surveyed = 360
Hence answer is option C
Total number of who likes all three products =
360 x 1/12 = 30
10) Answer: A
Number of People likes only P = M
For Saturday,
So, people number of like only R = 2 x M = 2M
Total items sold = 100 + 5 x 30 = 250
So, the number of People likes only Q = 2M –
Ratio of burgers and Pizzas = 4 x 5: 5 = 4:1
M/2 = 3M/2
Required difference = 250/5 x 3 = 150
Let the number of people likes both P and Q but
Hence answer is option A
not R = Y

11) Answer: D

Click Here For Bundle PDF Course | support@guidely.in Page 10 of 12


SBI Clerk & RRB PO Mains PDF Course 2023
Quantitative Aptitude Day -1 (Eng)

So, the number of people likes both P and R but


not Q = Y 15) Answer: B
Let the number of people likes both Q and R but People don’t like any of the product = Z (Two-
not P = Z digit prime number)
So, the number of people do not like any of the People those likes exactly two products = Y + Y
given product = Z + Z = Y + 30
Out of total number of people those likes R, Also,
people those likes exactly one of the other Y + Z = 30
brands is same as people those likes exactly Possible value of Z = 11, 13, 17, 19
other two brands Possible value of Y = 19, 17, 13, 11
So, Y + Z = 30………………… (1) Required possible values = 49, 47, 43, 41
Also, So, I and III are possible
M + 2M + 3M/2 + Y + Y + Z + Z + 30 = 360 Hence answer is option B
9M/2 + 2(Y + Z) = 330………………… (2)
From (1) and (2), we get 16) Answer: A
9M/2 + 2 x 30 = 330 Number of people those likes only P = 60 + 30 +
Value of M = 60 2Y = 90 + 2Y
We have the following data Y should be maximum, so Z must be minimum
Y + Z = 30
Minimum value of Z = 0,
So, value of Y = 30
Required value = 90 + 2 x 30 = 150
Hence answer is option A

17) Answer: E
We know that,
Y + Z = 30
According to question,
14) Answer: C
Z = 2Y
According to the question,
So,
Difference between the number of people those
2Y + Y = 30
who likes only R to those likes only Q = 120 – 90
Value of Y = 10
= 30
Value of Z = 20
Hence answer is option C

Click Here For Bundle PDF Course | support@guidely.in Page 11 of 12


SBI Clerk & RRB PO Mains PDF Course 2023
Quantitative Aptitude Day -1 (Eng)

Value of K = 360 – 20 = 340


(K + M/5) = 340 + 60/5 = 352
Required unit digit = (352)60 = 260 = 6
Hence answer is option E

18) Answer: C
Directions [18–20]: According to question,
Let the number of pens sold by P = Za Total pens sold = 105
Y = largest single digit number = 9 Total pencils sold = 95
So, let the number of pencils sold by P = 9a Required difference = 105 – 95 = 10
So, the number of pens sold by Q = 2 x Za = 2Za Hence answer is option C
Total number of items sold by Q = 120
So, the number of pencils sold by Q = 120 –2Za 19) Answer: A
Now, Required revenue = 70 x 12 + 50 x 5 = Rs. 1090
Za/ (120 – 2Za) = Z/10 Hence answer is option A
Value of a = 60/ (5 + Z)
Also, 20) Answer: B
9a – Za = 120/ (5 + Z) Required difference = 120 – 80 = 40
(9 – Z) x 60 / (5 + Z) = 120/ (5 + Z) Hence answer is option B
Value of Z = 7
So, value of a = 60/12 = 5

Click Here For Bundle PDF Course | support@guidely.in Page 12 of 12


SBI Clerk & RRB PO Mains PDF Course 2023
ENGLISH Day -1

English Language

Directions (1-7): Read the passage carefully and as per BS-IV norms and spew less toxicity into
answer the following questions. the air. In a lower to middle income country like
The surge in automobile sales in March, with the India, many households buy cars and use them
country seeing 14,000 more cars being sold in sparingly, but the policy does not differentiate
comparison with the same period last year, between a 10-year-old car that may have run
suggests that the policy to scrap old cars, 20,000 km and a two-year-old car that may have
introduced in many states with enthusiasm, is run 50,000 km.
beginning to yield results. While market leader The lack of enforcement, to ensure all vehicles
Maruti Suzuki has seen almost a flat sales curve, on roads meet emission standards, is a hallmark
other manufacturers including Hyundai, Tata of most transport departments in the country, and
Motors, Mahindra, Toyota and MG have contributes to the problem of pollution. Several
recorded impressive growth in numbers. suggestions have been offered over the years to
The objectives of the policy are to provide a address the issue without being in the sort of
boost to the economy through increased sales, hurry we seem to be in, to scrap vehicles.
reduce pollution because older vehicles are These include making it mandatory for vehicle
presumed to emit more toxic fumes and improve manufacturers to retrofit older vehicles so that
the fuel-efficiency of vehicles allowed on the they meet more stringent emission standards.
roads. These are all laudable objectives, but Such vehicles can then be used in the country, or
must rekindle the debate on whether citizens of a exported. Of course, manufacturers would not be
relatively poor country can afford the indulgence happy with such a policy, but the government’s
of changing cars every 10 or 15 years. aim cannot be to please a Tata or a Mahindra.
For, while three-fourths of the material in a Other suggestions are for older vehicles to be
scrapped vehicle can be recycled, the rest ends sent to rural areas to reduce the use of tractors,
up in landfills, adding to pollution. Secondly, the which consume a lot of fuel and pollutants during
question that must be answered by policy- the transportation of goods and people. One
makers is whether the age of a vehicle or the aspect of the vehicle scrapping policy is the
emission standards its manufacturer follows are government’s expectation that new cars will be
necessarily linked with actual emissions. cheaper, both at the point of acquisition, and
Surely, poorly maintained three-year-old vehicles over the long run. Evidence suggests this is not
manufactured as per the latest BS-VI emission true, with prices of the same car having gone up
standards have less right to be on the roads than by at least 25 per cent over the past five years.
well-maintained six-year old cars that were made

Click Here For Bundle PDF Course | support@guidely.in Page 1 of 13


SBI Clerk & RRB PO Mains PDF Course 2023
ENGLISH Day -1

As with vehicles, the government must consider a) Only 1


hybrid options that best suit our needs. b) Only 2
1. According to the context of the passage, what c) Only 3
is/are effect (s) of the vehicle scrap in policy? d) Both 1 and 2
• Scraping old and unfit vehicles will create e) Both 2 and 3
demand in the automobile industry, as the old
one will now be replaced with the new vehicles. 3. In context to the passage, which of the
• This policy will hit hard the poor section of the following options do not support vehicle
society, as it will be very difficult for them to buy scrapping policy?
a new vehicle. • If you own any refund from your vehicle
• The policy will provide a major push to the insurance provider for the premium that has
recycling industry; this will not only make the already been paid, ahead of the scrapping, it will
recycling sector more active but also create jobs be calculated on a pro-rata basis.
in the industry. • The ambiguity between the old and newly
a) Only 1 manufactured vehicles under the standardized
b) Only 2 emission level could create inconvenience for
c) Only 3 working class people.
d) Both 1 and 2 • If there is no follow up to check all the
e) All of the above vehicles running on road, to meet the emission
standards, it will not lessen the pollution level.
2. What was the intention of the policy makers a) Only 1
behind the vehicle scrapping policy? b) Only 2
• In order to cater the demand of the policy c) Both 1 and 2
makers, the manufacturers come up with d) Both 2 and 3
technological update and environment friendly e) All of the above
vehicles.
• The idea behind the scrapping policy is to 4. Which of the following options correctly
gradually develop an Ecosystem that phases out mentioned the suggestion for vehicle scrapping
vehicle contribution to pollution; most of these policy discussed in the passage?
are old vehicles. • Except private vehicles, government should
• The scrapping policy is expected to consider only commercial vehicle under vehicle
significantly and effectively reduce the carbon scrapping policy.
footprint caused by old and unfit vehicle • It was suggested that vintage vehicles, which
emissions. are older than the average vehicles running on

Click Here For Bundle PDF Course | support@guidely.in Page 2 of 13


SBI Clerk & RRB PO Mains PDF Course 2023
ENGLISH Day -1

road should be sent to rural areas as these 6. In context to the passage, which of the
vehicles require less maintenance. following options is synonym of ‘sparingly’?
• To check vehicles' emission level pollution a) Lavishly
under control test unit should be installed through b) Frugally
authorized automated testing stations. c) Opulently
a) Only 1 d) Pompously
b) Only 2 e) None of these
c) Only 3
d) Both 2 and 3 7. In context to the passage, which of the
e) None of these following options is antonym of ‘rekindle’?
a) Resurrect
5. The question that must be answered by policy- b) Call forth
makers is whether the age of a vehicle or the c) Quench
emission standards its manufacturer follows are d) Evoke
necessarily linked with actual emissions. e) None of these
What do you understand by the ab ove
highlighted statement? Directions (8-12): In the following questions, a
• This policy will provide a major push to the sentence is given with two blanks, which is
recycling industry, as it will create further followed by some options. You have to choose
improvement in waste and recycling appropriate option to fill the blank to make the
management sentence meaningful.
• The policy makers should direct the 8. Vogue and social media stardom prove that
manufacturers to assemble the vehicles that __________ exists far beyond youth and that it’s
meet the actual emission standards of road frankly __________ that society sidelines older
• The manufacturers should ensure the women, given their style, wit and energy
emission level of the vehicle should be in line Column I Column II
with the actual emission standards 1) vitality 4) absurd
a) Only 1 2) vapidity 5) ludicrous
b) Only 2 3) vivacity 6) prudent
c) Only 3 a) Only (1-6)
d) Both 2 and 3 b) Only (2-5)
e) None of these c) Only (3-4)
d) Both (1-5) and (3-4)

Click Here For Bundle PDF Course | support@guidely.in Page 3 of 13


SBI Clerk & RRB PO Mains PDF Course 2023
ENGLISH Day -1

e) Both (2-6) and (3-5) 11. Singapore’s robust defense of its drug laws
that prescribe punishments deemed ___________
9. What makes things worse is the intense in other jurisdictions is in focus once again as the
competition prompted by a ___________ republic prepares to hang a man convicted of
economy and widespread unemployment. One _________ an attempt to smuggle cannabis
can hardly be friendly with colleagues if he Column I Column II
constantly seen them as _____________ in a job- 1) mellow 4) adulating
insecure environment 2) savagely 5) infringing
Column I Column II 3) harsh 6) abetting
1) freed 4) guards a) Only (1-5)
2) beleaguered 5) threats b) Only (2-4)
3) liberated 6) agitator c) Only (3-6)
a) Only (1-6) d) Both (1-6) and (2-5)
b) Only (2-5) e) Both (2-4) and (3-5)
c) Only (3-4)
d) Both (1-5) and (2-4) 12. When governments or powerful entities
e) Both (2-6) and (3-5) exercise control over what information is
___________ (disseminated) through mainstream
10. Several deaths were attributed to the media channels, it leads to a lack of diversity in
overcrowding in 2019, as inexperienced climbers viewpoints and a/an ____________ (narrowing) of
__________ the route to the peak; causing the range of opinions represented
interminable delays that forced many to Column I Column II
__________ to high-altitude sickness. 1) promulgated 4) restricting
Column I Column II 2) disseminated 5) obstruct
1) drained 4) succumb 3) conversed 6) narrowing
2) clogged 5) conquer a) Only (1-5)
3) congested 6) surrender b) Only (2-4)
a) Only (1-5) c) Only (3-6)
b) Only (2-4) d) Both (1-4) and (2-6)
c) Only (3-6) e) Both (2-5) and (3-4)
d) Both (1-6) and (2-5)
e) Both (2-6) and (3-4) Directions (13-16): In the following questions,
three sentences are given; in each sentence one

Click Here For Bundle PDF Course | support@guidely.in Page 4 of 13


SBI Clerk & RRB PO Mains PDF Course 2023
ENGLISH Day -1

highlighted word is mentioned. You have to 15.


identify the highlighted word is correctly used or • Quantitative forecasting techniques are used
not, if not, mark it as your answer to a lead astray and augment the quantitative
13. forecasting process by making accurate
• Knowledge is a power when it comes to predictions for the organization
knowing what diet advice to listen and what to • When discussing reasons for discontent, the
cast aside law of diminishing utility is relevant. Following
• He says law enforcement fact turn the tide this logic, satiable desires are to blame.
upon those who seek to asperse the country’s • Lazy and slothful, Jim had a reputation of
good name. never holding down a job and sleeping all of the
• For a moment, he was taken aback as he time
realized she was so sorry for him, she was a) Only 1
arranging a date b) Only 2
a) Only 1 c) Only 3
b) Only 2 d) Both 1 and 2
c) Both 1 and 2 e) All are correct
d) Both 2 and 3
e) All are correct 16.
• The patient had been so debilitate by the
14. fever that he lacked the strength to sit up.
• Environmental results showed that the robot • A person, who is imperious and tries to gloss
can move out pipe and can call forth obstacles over his fault, is bound to suffer in the end.
such as flanges, T-joint, L-joint • You can’t change what has already
• Unless your goal is to watch an insipid movie happened. So, don’t waste your time thinking
that will bore you to death, do not go and see about it. Move on, clampdown and get over it
Hank Wolf’s latest film. a) Only 1
• Frequent face-to-face meetings iron out any b) Only 2
problems and come up with interface and c) Only 3
process improvements d) Both 1 and 2
a) Only 1 e) All are correct
b) Only 2
c) Only 3 Directions (17-21): In the following questions,
d) Both 2 and 3 three statements are given in each question. In
e) All are correct which some statements are correct and some

Click Here For Bundle PDF Course | support@guidely.in Page 5 of 13


SBI Clerk & RRB PO Mains PDF Course 2023
ENGLISH Day -1

are not, you have to identify the correct c) Only 3


statement, and mark it as your answer. d) Both 1 and 2
17. e) Both 2 and 3
• In the diverse fabric of Indian literature,
Adivasi literature is a genre left untouched by 19.
most intellectual critics and writers • It has published 400 books by renowned
• In spite of winning such a prestigious award, scientists and science writers, books that
the book remained almost unnoticed through all command readership far beyond the borders,
these years and not only in English.
• The coal mines in the suburbs of Jharkhand • Republicans, who were stunned by the
has evicted people from their villages in the midterm results, must realize that even those
name of development and painted everything women who otherwise supports them may be
black swayed away by the ban on abortions
a) Only 1 • Wilfred Rhodes was the “heaviest” wicket-
b) Only 2 taker, with 4187, in cricket’s history who also
c) Only 3 rose from No 11 to become an opener and finish
d) Both 1 and 2 with a career aggregate of 39,802 runs
e) Both 2 and 3 a) Only 1
b) Only 2
18. c) Only 3
• Our architecture was both original and d) Both 1 and 2
socially progressive. Furthermore, these projects e) Both 1 and 3
were also testimony to enlighten bureaucrats and
polity. 20.
• We are destine to live in private enclaves, • Nations around the world have been impacted
drive private vehicles, study in private in different ways by the war, and for them an end
universities and so on; the public sphere is to hostilities is of greater importance than
dammed apportioning blame
• The ban on the abortion pill would have • Already, warnings have been issued by
come as an additional blow to women, and experts and several states have cautioned
forced many to return to quackery, argue citizens not to step outdoors in the hours
abortion-rights advocates between noon and 4 p.m.
a) Only 1 • The conflation of professional media, for all its
b) Only 2 weaknesses, with social media, is one of the

Click Here For Bundle PDF Course | support@guidely.in Page 6 of 13


SBI Clerk & RRB PO Mains PDF Course 2023
ENGLISH Day -1

most dangerous development emanating out of • Up in the Himalayas, a physically violated


the digital revolution landscape is wreaking a terrible revenge by
a) Only 1 destroying a town and displacing thousands of
b) Only 2 inhabitants
c) Only 3 • Reversible fuel cells produce electricity from
d) Both 1 and 2 hydrogen and oxygen and generate heat and
e) Both 2 and 3 water as by products, just like other fuel cells
a) Only 1
21. b) Only 2
• The profoundest message of Hampi does not c) Only 3
come sole from the monuments but equally from d) Both 1 and 2
the rocks among which they are set e) Both 2 and 3
Click Here to Get the Detailed Video Solution for the above given Questions
Or Scan the QR Code to Get the Detailed Video Solutions

Answer Key with Explanation

1. Answer: D comparison with the same period last year,


Option 3 - the only information mentioned in the suggests that the policy to scrap old cars,
passage is about the ratio of recycled vehicle by introduced in many states with enthusiasm, is
the recycling industry. It is not mentioned in the beginning to yield results)
passage that the policy will help or create jobs in Option 2 - it is mentioned in the passage. So,
recycling industry. So, this is an incorrect option this is the correct option (These are all laudable
Option 1 - it is mentioned in the passage. So, objectives, but must rekindle the debate on
this is the correct option (The surge in whether citizens of a relatively poor country can
automobile sales in March, with the country afford the indulgence of changing cars every 10
seeing 14,000 more cars being sold in or 15 years)

Click Here For Bundle PDF Course | support@guidely.in Page 7 of 13


SBI Clerk & RRB PO Mains PDF Course 2023
ENGLISH Day -1 (Eng)

20,000 km and a two-year-old car that may have


2. Answer: E run 50,000 km)
Option 1 - the option twisted the context of the Option 3 - it is mentioned in the passage. So,
sentence, it is not the demand of the public to this is the correct option (The lack of
use environment friendly vehicles, it is a policy enforcement, to ensure all vehicles on roads
made by the government to lessen the carbon meet emission standards, is a hallmark of most
footprint or pollution on roads. So, this is an transport departments in the country, and
incorrect option. contributes to the problem of pollution)
Options 2 and 3 - it is mentioned in the passage.
So, both the options are correct. (The objectives 4. Answer: E
of the policy are to provide a boost to the Options 1 and 3 - both the options are out of
economy through increased sales, reduce context. No such information is mentioned in the
pollution because older vehicles are presumed passage. So, both the options are eliminated
to emit more toxic fumes and improve the fuel- Option 2 - it twisted the context of the passage. It
efficiency of vehicles allowed on the roads) is mentioned in the passage that older vehicles
will be sent to the rural area to reduce the use of
3. Answer: D tractors, it is not mentioned in the passage that
Option 1 - the option is out of context, no such vintage vehicles will be sent to rural area for
information about the claim of old vehicles is maintenance purpose. So, this is an incorrect
mentioned in the passage. So, this is an option
incorrect option (Other suggestions are for older vehicles to be
Option 2 - it is mentioned in the passage. So, sent to rural areas to reduce the use of tractors,
this is the correct option (Surely, poorly which consume a lot of fuel and pollutants during
maintained three-year-old vehicles manufactured the transportation of goods and people)
as per the latest BS-VI emission standards have
less right to be on the roads than well- 5. Answer: C
maintained six-year old cars that were made as The context of the sentence says that
per BS-IV norms and spew less toxicity into the policymakers should ensure that the age of the
air. In a lower to middle income country like vehicle or the emission standards that is
India, many households buy cars and use them followed by the manufacturers should be linked
sparingly, but the policy does not differentiate with the actual emission standard of the road
between a 10-year-old car that may have run Option 1 - the option is completely out of context,
the context of the sentence is not talking about

Click Here For Bundle PDF Course | support@guidely.in Page 8 of 13


SBI Clerk & RRB PO Mains PDF Course 2023
ENGLISH Day -1 (Eng)

to recycle the vehicles, it talks about the The context of the sentence says that voice and
emission standard of the vehicle. So, the option social media proves that the liveliness and
is eliminated energy exists far beyond the age, and it is foolish
Option 2 - it twisted the context of the statement, to sideline an older woman because of her style,
the context of the sentence talks about to ensure wit and energy
the emission standards; it is not talking about 1st blank - the context of the first Frank talks
how the manufactures should assemble the about the liveliness and energy that exists far
vehicles. So, this is an incorrect option beyond the age. In that context, options (1) and
Option 3 - it supports the context of the (3) both support the context of the blank.
sentence. So, this is the correct option 'Vitality' – the state of being strong and active
'Vivacity' – high spirit and full of life
6. Answer: B Option (2) - 'vapidity' is opposite to the context of
Meaning of ‘sparingly’ – in an economical or the blank, the context of the bank is not talking
limited way about dull or lack in Spirit. The context of the
Option (2) ‘frugally’ is synonym of it sentence talks about the energy and liveliness
‘Frugally’ – in an economical way; not wastefully that exists beyond an age. So, this is an
Meaning of other words ‘lavishly’ – in a very incorrect option
generous or extravagant way ‘Vapidity’ – dull in uninspiring
‘Opulently’ – in an expensive and luxurious way 2nd blank - the context of the second blank says
‘Pompously’ – in a very serious way that the way society sideline an older woman is
totally foolish. In that context, options (4) and (5)
7. Answer: C both support the context of the blank.
Meaning of ‘rekindle’ – to start or stir up ‘Absurd’ – unreasonable and illogical
something again ‘Ludicrous’ – foolish and unreasonable
Option (3) ‘quench’ is the antonym of it Option (6) - 'prudent' is opposite to the context of
‘Quench’ – to bring to an end the blank, the context of the blank is not saying
Meaning of other words ‘resurrect’ – bring back that sideline and older woman is wise. So, this is
to life an incorrect option
‘Call forth’ – to cause something to exist ‘Prudent’ – careful and avoiding risk
‘Evoke’ – bring a feeling, memory into the mind
9. Answer: B
8. Answer: D The context of the sentence says that when
intense competition is attacked by troubled

Click Here For Bundle PDF Course | support@guidely.in Page 9 of 13


SBI Clerk & RRB PO Mains PDF Course 2023
ENGLISH Day -1 (Eng)

economy and widespread unemployment, it


makes the situation worse, and in this situation 10. Answer: E
one can hardly become a friend with his/her The context of the sentence says that the peak
colleagues, he/she always sees his colleagues was overcrowded because of the inexperienced
as a threat climbers, and it causes unbroken delays, which
1st blank - the context of the first planned talks forces many climbers to die with high altitude
about troubled economy and widespread sickness
unemployment, in that case, only option (2) 1st blank - the context of the first blank talks
supports the context of the blank. about the jammed route, because of the high
‘Beleaguered’ – cause problem or difficulty number of inexperienced climbers. In that case,
Options (1) and (3) - 'freed' and 'liberated' both options (2) and (3) support the context of the
are opposite to the context of the blank. The blank
context of the blank is not talking about safe and ‘Clogged’ – blocked or become blocked
free economy. So, both the options are ‘Congested’ – so crowded with traffic or people
eliminated Option (1) - 'drained' is opposite to the context of
‘Freed’ – to set free the blank, the context of the blank is not saying
‘Liberated’ – no longer having any limits or that the route of the peak was empty, the context
control of the sentence says that the route was
2nd blank - the context of the blank says that in overcrowded. So, this is an incorrect option
a job insecure environment, one always sees his ‘Drained’ – to cause the gradual disappearance
colleagues as a threat. In that context, option (5)
supports the context of the bank. 2nd blank - the context of the blank says that
Option (4) - 'guards' is opposite to the context of because of never-ending delays many climbers
the blank, the context of the blank symbolizes were forced to die. In that context, options (4)
colleagues as a threat. So, this is an incorrect and (6) support the context of the blank.
option ‘Succumb’ – die from the effect of a disease or
Option (6) - 'agitator' is contextually correct but injury
grammatically incorrect, the noun agitator is ‘Surrender’ – give up
used to symbolize 'colleagues', which is plural, Option (5) - 'conquer' is opposite to the context
so we also need plural form of noun to qualify of the blank, the context of the blank talks about
'colleagues'. 'Agitators' is the correct word. So, the high altitude sickness that forced the
this is an incorrect option climbers to die, the context of the blank is not
‘Agitator’ – trouble maker

Click Here For Bundle PDF Course | support@guidely.in Page 10 of 13


SBI Clerk & RRB PO Mains PDF Course 2023
ENGLISH Day -1 (Eng)

saying that the climbers overcome the high ‘Abetting’ – encourage or assist someone to do a
altitude sickness. So, this is an incorrect option crime
‘Conquer’ – climb a mountain successfully Option (4) – ‘adulating’ is opposite to the context
of the sentence, according o the context of the
11. Answer: C sentence, the man is not praising the smuggling
the context of the sentence says that Singapore of cannabis, in fact the man is breaking the law
Drug law punishment is seen harsh in others by smuggle the cannabis. So, this is an incorrect
jurisdiction, it comes under focus when a man option.
breaks the law by attempting to smuggle ‘Adulating’ – praise someone excessively
cannabis.
1st blank – the context of the sentence talks 12. Answer: D
about harsh punishment under drug laws of the context of the sentence says that when
Singapore. In that context, option (3) ‘harsh’ powerful entities try to control the circulation of
supports the context of the sentence information through media, it narrow down the
Option (1) – ‘mellow’ is opposite to the context of view points of the public.
the sentence, according to the context of the 1st blank – the context of the sentence talks
sentence the drug law of Singapore is not about the circulation of information through
pleasant, it is harsh/strict. So, this is an incorrect media channels, in that context, options (1) and
option (2) support the context of the sentence.
‘Mellow’ – smooth and soft ‘Promulgated’ – promote or make known widely
Option (2) – ‘savagely’ is contextually correct but ‘Disseminated’ – spread information widely
grammatically incorrect. We need an adjective to Option (3) – ‘conversed’ doesn’t make any sense
fill the blank, ‘savagely’ is an adverb. So, this is here, the context of the sentence is not talking
an incorrect option about any kind of conversation, it talks about the
‘Savagely’ – very severe circulation of information. So, this is an incorrect
option
2nd blank – the context of the blank talks about a ‘Conversed’ – engage in conversation
man, who tries to smuggle cannabis by breaking 2nd blank – the context of the sentence says that
the drug laws of Singapore. In that context, control over the information will narrow down the
options (5) and (6) support the context of the viewpoint of public. In that context, options (4)
sentence. and (6) support the context of the sentence
‘Infringing’ – actively break the terms of law ‘Restricting’ – to confine with bounds
‘Narrowing’ – become limited in extent or scope

Click Here For Bundle PDF Course | support@guidely.in Page 11 of 13


SBI Clerk & RRB PO Mains PDF Course 2023
ENGLISH Day -1 (Eng)

Option (5) – ‘obstruct’ is grammatically incorrect, Option (2) – the context of the sentence says
we need a noun to fill the blank, ‘obstruct’ is a that unless someone wants to become bore or
verb. So, this is an incorrect option feel dull don’t go to see the movie ‘Hank Wolf’s
‘Obstruct’ – keep under control; put a limit on latest movie’. ‘Insipid’ supports the context of the
something sentence
‘Insipid’ – lacking flavour; dull
13. Answer: E Option (3) – the context of the sentence says
option (1) – the context of the sentence says that that because of the frequent face-to-face
knowledge becomes a power when it comes to meeting the problems are settled down. ‘Iron out’
know what diet advice should one consider and supports the context of the sentence.
what advice to be discarded. ‘Cast aside’ ‘Iron out’ – solve or settle difficulties
support the context of the sentence.
‘Cast aside’ – discard or reject something 15. Answer: D
Option (2) – the context of the sentence says option (1) – the context of the sentence says that
that the law enforcement fact has changed the quantitative forecasting helping to make
situation for those who try to destroy the name of accurate predictions, ‘lead astray’ doesn’t
the country. ‘Turn the tide’ supports the context support the context of the sentence. The context
of the sentence of the sentence is not talks about in a positive
‘Turn the tide’ – reverse the trend of events sense about the quantitative forecasting, it is not
Option (3) – the context of the sentence says saying that quantitative forecasting took the
that for a moment he was shocked, when he organization is a wrong direction. So, this is an
realized she was arranging a date incorrect option
‘Taken aback’ – shock or surprise ‘Lead astray’ – give someone wrong direction
Option (2) – the context of the sentence talks
14. Answer: A about the reasons for discontent, ‘satiable’
option (1) – the context of the sentence says that doesn’t support the context of the sentence, the
the robot pass out all the problems related to the context of the sentence is not talking about
pipe. ‘Call forth’ doesn’t support the context of satisfaction. So, this is an incorrect option
the sentence, the context of the sentence talks ‘Satiable’ – satisfied
about to pass the problem, it is not talking about Option (3) – the context of the sentence talks
to evoke the problems about a lazy person who spends all his time in
‘Call forth’ – to cause something to come into sleeping, ‘slothful’ supports the context of the
existence sentence.

Click Here For Bundle PDF Course | support@guidely.in Page 12 of 13


SBI Clerk & RRB PO Mains PDF Course 2023
ENGLISH Day -1 (Eng)

‘Slothful’ – lazy; inactive


18. Answer: C
16. Answer: C option (1) – ‘enlighten’ will be replaced by
options (1) – the context of the sentence talks ‘enlightened’, ‘enlighten’ is a verb, whereas we
about the lackness of a patient that he is not need an adjective to qualify the noun
able to sit, ‘debilitate’ supports the context of the ‘bureaucrats’. So, ‘enlighten’ is incorrect here
sentence Option (2) – ‘destine’ will be replaced by
‘Debilitate’ – weak; infirm ‘destined’, the structure of the sentence is in
Option (2) – the context of the sentence talks passive voice of present continuous tense, so
about an arrogant person, who suffer a lot in the the verb will be used according to the tense.
end, ‘imperious’ supports the context of the ‘Destined’ is the correct word here
sentence
‘Imperious’ – arrogant and domineering 19. Answer: E
Option (3) – the context of the sentence talks option (2) – ‘supports’ will be replaced by
about to move on in the life, it is not talking about ‘support’, the subject of the verb is ‘women’,
to suppress anyone, so ‘clampdown’ is incorrect which is plural, so the verb will be used
here according to its subject. So, ‘support’ is incorrect
‘Clampdown’ – sudden restriction on an activity here

17. Answer: A 20. Answer: D


option (2) – ‘through’ will be replaced by option (3) – ‘development’ will be replaced by
‘throughout’, ‘throughout’ is used to signify the ‘developments’, according to the rule, ‘one of
duration of time period, the context of the the’ is always followed by plural noun, so
sentence talks about a duration of time period, ‘development’ is incorrect here
so ‘through’ is incorrect here
Option (3) – ‘has’ will be replaced by either 21. Answer: E
‘have’ or ‘had’, the subject of the verb is ‘coal option (1) – ‘sole’ will be replaced by ‘solely’,
mines’, which is plural, so the verb will be used ‘come’ is a verb and we need an adverb to
according to its subject. So, ‘has’ is incorrect qualify the verb, ‘sole’ is a noun. So, ‘sole’ is
here incorrect here

Click Here For Bundle PDF Course | support@guidely.in Page 13 of 13


SBI Clerk & RRB PO Mains PDF Course 2023
Reasoning Ability Day -2 (Eng)

Reasoning Ability

Direction (1-5): Study the following information 1. Who among the following pair of persons
carefully and answer the given questions. faces the same direction?
Fourteen persons are sitting in two parallel rows a) NE
containing seven persons each in such a way b) KM
that there is equal distance between adjacent c) FO
persons. In row 1: J, K, L, M, N, O and P are d) CJ
seated and some of them facing north and some e) PB
of them facing south. In row 2: A, B, C, D, E, F
and G are seated and some of them facing north 2. Who among the following person sits third to
and some of them facing south. Row 1 is in north the right of the one who sits opposite to A?
of row 2. a) The one who faces B
Note I: Only six persons (three pairs) are facing b) N
each other. c) The one who sits opposite to G
Note II: Not more than two persons facing the d) No one
same direction sit together. e) The one who sits second to the left of O
D sits fifth to the left of the one who sits opposite
to M, who does not sit at any of the extreme 3. If one of the neighbours of G is F, then who
ends. Only two persons sit between M and K. C among the following is the second neighbour?
sits to the immediate right of the one who sits a) The one who faces K
opposite to K. The number of persons sitting to b) A
the right of C is one more than the number of c) D
persons sitting to the left of P, who doesn’t sit at d) The one who sits second to the right of B
the end of the row. A and one of the immediate e) C
neighbours of P are facing each other. G sits
second to the left of A. L and G are facing the 4. What is the position of N with respect to the
opposite direction of B. Only three persons sit one who sits opposite to E?
between B and F. F sits second to the right of the a) Fourth to the left
one who sits opposite to O. The number of b) Fifth to the right
persons sitting between L and J is one more than c) Fourth to the right
the number of persons sitting between J and N, d) Second to the right
who faces north. Immediate neighbours of J are e) Third to the left
facing the same direction as G.

Click Here For Bundle PDF Course | support@guidely.in Page 1 of 8


SBI Clerk & RRB PO Mains PDF Course 2023
Reasoning Ability Day -2 (Eng)

5. As many persons sit to the right of K as to the e) Both conclusions I and III are true
left of ___.
a) M 8. Statements:
b) E A<S=P≥D; I<V=O≥P<Y; C≥V≤B=M<N
c) F Conclusions:
d) A I. Y<N
e) C II. S<M
III. C≥D
Directions (6-10): In the given questions, the a) Only conclusion I is true
relationship between different elements is shown b) Only conclusion II is true
in the statements followed by some conclusions. c) Only conclusion III is true
Find the conclusion which is definitely true. d) Both conclusions I and II are true
6. Statements: e) Both conclusions I and III are true
T≤Y>O=P≤L; K>Y≤J=N>M; Z≥V>N≤W
Conclusions: 9. Statements:
I. P<Z F<G≤H>P=Q; I=U<Q≥N=L; W≥H<E=R<T
II. W≥T Conclusions:
III. M<L I. F>L
a) Only conclusion I is true II. U<T
b) Only conclusion II is true III. N≥F
c) Only conclusion III is true a) Only conclusion I is true
d) Both conclusions I and II are true b) Only conclusion II is true
e) Both conclusions I and III are true c) Only conclusion III is true
d) Either conclusion I or III is true
7. Statements: e) Both b and d are true
S<G≤H=J>K; A≥Y<C≤V; U≥M>J<N=Y;
Conclusions: 10. Statements:
I. S<V R≥E=W>Q≤T; Y≤Q≥U=I<O; F≤I>P=K
II. U≥G Conclusions:
III. K>A I. P<R
a) Only conclusion I is true II. Y<K
b) Only conclusion II is true III. T≥F
c) Only conclusion III is true a) Only conclusion I is true
d) Both conclusions I and II are true b) Only conclusion II is true

Click Here For Bundle PDF Course | support@guidely.in Page 2 of 8


SBI Clerk & RRB PO Mains PDF Course 2023
Reasoning Ability Day -2 (Eng)

c) Only conclusion III is true d) YT


d) Both conclusions I and II are true e) KN
e) Both conclusions I and III are true
14. Which of the following statements is/are
Direction (11-15): Study the following information TRUE as per the given arrangement?
carefully and answer the given questions. a) If A%T(6m), then the shortest distance
A@B (23m) means “B is 19m south of A” between P and A is 5m
A#B (35m) means “B is 41m east of A” b) Y&#W
A&B (11m) means “B is 16m north of A” c) If Z is the mid-point of H and K, then the
A%B (42m) means “B is 39m west of A” distance between H and Z is same as the
R&W (25m), Y#T (1m), H&O (17m), N@K (17m), distance between T and W.
P@T (8m), R%M (16m), O#P (11m), N#W (4m), d) Both a and b
K%H (11m) e) Both a and c
11. What is the shortest distance between M and
T? 15. Four of the following five pairs of points are
a) 35m alike in a certain way based on the directions in
b) 27m the given arrangement and thus form a group.
c) 37m Which one of the following does not belong to
d) 39m the group?
e) 33m a) HR
b) OK
12. What is the direction of N with respect to Y? c) NR
a) @# d) WP
b) #& e) OY
c) &%
d) @% Direction (16-20): Study the following information
e) Can’t be determined carefully and answer the given questions.
Seven persons – P, Q, R, S, T, U, and V are
13. If O#Y (6m) and P@Z (16m), then the working in an IT company at different
shortest distance between Y and Z is same as designations such as MD, CEO, CFO, COO,
the shortest distance between __ and __ CTO, HR and TL. The designations are given in
a) HK decreasing order such that MD is the senior-most
b) WR and TL is the junior-most designation. Each of
c) OP them uses different types of OS in the office-

Click Here For Bundle PDF Course | support@guidely.in Page 3 of 8


SBI Clerk & RRB PO Mains PDF Course 2023
Reasoning Ability Day -2 (Eng)

Linux, MSDos, UNIX, iOS, Solaris, Symbian and c) Only two persons are designated between S
Ubuntu. and the one who uses UNIX
Q is four persons senior to the one who uses d) Both a and b
Solaris. The number of persons senior to Q is e) Both a and c
one less than the number of persons junior to U. 18. Who among the following person is three
The one who uses iOS is two persons senior to persons junior to the one who uses Linux?
U. P is three persons junior to the one who uses a) The one who is designated as COO
Symbian. Neither P nor T is a juniormost b) P
designated person. The number of persons c) The one who uses Solaris
designated between P and T is one more than d) V
the number of persons designated between T e) The one who is two persons senior to the one
and the one who uses Ubuntu. V does not use who uses UNIX
Ubuntu. The one who uses MSDos is three
persons senior to R. S is immediately junior to 19. Which of the following combination is true?
the one who uses Linux. The one who uses Unix a) MD-V-Linux
is not designated as CTO. b) CTO-T-Ubuntu
16. Who among the following person uses c) TL-R-MSDos
UNIX? d) CFO-S-iOS
a) P e) COO-P-Solaris
b) R
c) S 20. As many persons designated between V and
d) T __ as between then one who uses Solaris and
e) Q ___ respectively.
a) The one who uses iOS, R
17. Which of the following statements is/are not b) The one who uses UNIX, Q
false as per the given arrangement? c) The one who uses MSDos, S
a) As many persons senior to Q as junior to the d) The one who uses Linux, P
one who uses Solaris e) The one who uses Ubuntu, U
b) Only four persons are senior to P

Click Here For Bundle PDF Course | support@guidely.in Page 4 of 8


SBI Clerk & RRB PO Mains PDF Course 2023
Reasoning Ability Day -2 (Eng)

Click Here to Get the Detailed Video Solution for the above given Questions
Or Scan the QR Code to Get the Detailed Video Solutions

Answer Key with Explanation


Directions (1-5):
1. Answer: C
2. Answer: E
3. Answer: D
Again we have,
4. Answer: E
 C sits to the immediate right of the one
5. Answer: C
who sits opposite to K.
Final arrangement:
 The number of persons sitting to the right
of C is one more than the number of
persons sitting to the left of P, who
doesn’t sit at the end of the row.
 A and one of the immediate neighbours of
P are facing each other.
 G sits second to the left of A.
We have,
 D sits fifth to the left of the one who sits
opposite to M, who does not sit at any of
the extreme ends.
 Only two persons sit between M and K.
From the above conditions, we have two
Again we have,
possibilities:
 L and G are facing the opposite direction
of B.
 Only three persons sit between B and F.

Click Here For Bundle PDF Course | support@guidely.in Page 5 of 8


SBI Clerk & RRB PO Mains PDF Course 2023
Reasoning Ability Day -2 (Eng)

 F sits second to the right of the one who II. U<T -> T>R=E>H>P=Q>U -> True
sits opposite to O. III. N≥F -> F<G≤H>P=Q≥N=L -> False
 The number of persons sitting between L 10. Answer: E
and J is one more than the number of I. P<R -> R≥E=W>Q≥U=I>P -> True
persons sitting between J and N, who II. Y<K -> Y≤Q≥U=I>P=K -> False
face north. III. T≥F -> F≤I=U≤Q≤T -> True
 Immediate neighbours of J are facing the
same direction as G. Directions (11-15):
While applying the above conditions, case 1 gets 11. Answer: C
eliminated, because not more than two persons 12. Answer: D
facing the same direction should sit together. 13. Answer: E
Thus, case 2 gives the final arrangement. 14. Answer: A
15. Answer: D (In the given option, the first point
is northwest of second point, except option D)

6. Answer: D
I. P<Z -> Z≥V>N=J≥Y>O=P -> True
II. W≥T -> T≤Y≤J=N≤W -> True
III. M<L -> M<N=J≥Y>O=P≤L -> False

7. Answer: A
I. S<V -> S<G≤H=J<N=Y<C≤V -> True
II. U≥G -> G≤H=J<M≤U -> False
III. K>A -> A≥Y=N>J>K -> False Directions (16-20):
16. Answer: B
8. Answer: C 17. Answer: A
I. Y<N -> Y>P≤O=V≤B=M<N -> False 18. Answer: E
II. S<M -> S=P≤O=V≤B=M -> False 19. Answer: D
III. C≥D -> C≥V=O≥P≥D -> True 20. Answer: C
9. Answer: E Final arrangement:
I. F>L -> F<G≤H>P=Q≥N=L -> False

Click Here For Bundle PDF Course | support@guidely.in Page 6 of 8


SBI Clerk & RRB PO Mains PDF Course 2023
Reasoning Ability Day -2 (Eng)

number of persons designated between T


and the one who uses Ubuntu.

We have,
 Q is four persons senior to the one who
uses Solaris.
 The number of persons senior to Q is one
less than the number of persons junior to
U.
 The one who uses iOS is two persons
senior to U.
From the above conditions, we have three
possibilities:

Again we have,
 V does not use Ubuntu.
 The one who uses MSDos is three
persons senior to R.
 S is immediately junior to the one who
uses Linux.
 The one who uses Unix is not designated
Again we have,
as CTO.
 P is three persons junior to the one who
While applying the above conditions, case 1, 3
uses Symbian.
and 3a get eliminated. Because can’t place
 Neither P nor T is a juniormost designated
Linux and V does not use Ubuntu. Thus, case 2
person.
gives the final arrangement.
 The number of persons designated
between P and T is one more than the

Click Here For Bundle PDF Course | support@guidely.in Page 7 of 8


SBI Clerk & RRB PO Mains PDF Course 2023
Reasoning Ability Day -2 (Eng)

Click Here For Bundle PDF Course | support@guidely.in Page 8 of 8


SBI Clerk & RRB PO Mains PDF Course 2023
Quantitative Aptitude Day -2 (Eng)

Quantitative Aptitude

Directions [01 – 04]: Read the following information carefully and answer the questions based on it.
The chart given below shows the number of employees in five different companies (P, Q, R, S, and T).
Each company has two departments – A and B. Number of employees in department A of company R, is
Z.

Note: a) Number of employees in department B of company P is Y, and number of employees in


department A of company Q is Y% of total employees number of in that company.
b) Out of total number of employees in company S, (Z + 10) % are in department A, and ratio of number
of employees in department A of company P and R is in ratio of 8:5.
c) 5/9 of total number of employees in company T are in department B, while number of employees in
department B of company R is 25% less than that of same department in company P. All the values in
graph is multiple of 5.
1) For company Q, 40 employees likes tea, out c) 10
of which 25% are from department A. Find d) 5
difference between number of employees in e) None of these
department A those likes coffee and number of
employees in department B those who likes tea. 2) For company S, Z% of total number of
Consider each employee either likes coffee or employees in department B are males, while 5/6
Tea. of total number of employees in department A
a) 20 are males. Find number of female employees in
b) 15

Click Here For Bundle PDF Course | support@guidely.in Page 1 of 12


SBI Clerk & RRB PO Mains PDF Course 2023
Quantitative Aptitude Day -2 (Eng)

company S is what % of total number of e) None of these


employees in the company? 4) Which of the following is/are true.
a) 40% I. Number of employees in department A of
b) 30% company P and T is same.
c) 25% II. Number of employees in department B of
d) 60% company S is twice as that of company Q.
e) None of these III. Difference between number of employees in
department A and department B of company Q is
3) Find difference between number ofemployees half as that of R.
in department A and department B of all a) I only
companies together? b) II and III only
a) 65 c) I and II only
b) 95 d) I, II and III
c) 55 e) None of these
d) 75

Directions [05 – 08]: Read the following information carefully and answer the questions based on it.
ARUN PANCHAL is Quant content developer in GUIDELY for Mains Exam. The first piechart given below
shows the % distribution of Totalnumber of questions developed by Arun in four months from March to
June, and the second pie chart shows the% distribution ofnumber of DI question developed by Arun in
these four Months.

Click Here For Bundle PDF Course | support@guidely.in Page 2 of 12


SBI Clerk & RRB PO Mains PDF Course 2023
Quantitative Aptitude Day -2 (Eng)

Note: a) Out of Total number of DI question developed by Arun, half of them developed in month of April.
b) Number of Arithmetic Question developed in month of March are 20 less than that of number of DI
questions developed in same month.
c) Number of Arithmetic questions developed in April are 80.
Total number of questions =Number of DI questions + Number of Arithmetic Questions
5) For June, totalnumber of DI questions b) 8.33%
developed are of three type sets – A (set of five c) 13.33%
questions), B (set of three questions), C (set of d) 12.5%
six questions). Number of set B is 50% more e) None of these
than that of set A, while maximum possible and
minimum possible number of set C is P and Q 7) P = Number of arithmetic questions in March
respectively. Find the value of (P – Q). Q = Number of arithmetic questions in May
a) 27 Find correct relation between P and Q.
b) 19 a) P > Q
c) 21 b) P < Q
d) 17 c) P ≥ Q
e) None of these d) P ≤ Q
e) P = Q
6) Number of arithmetic questions in June is
what % of total questions developed in March? 8) If company P pays Rs. 180 for each DI
a) 16.66% question, and Rs. 120 for each Arithmetic

Click Here For Bundle PDF Course | support@guidely.in Page 3 of 12


SBI Clerk & RRB PO Mains PDF Course 2023
Quantitative Aptitude Day -2 (Eng)

question. Find total earnings of Arun in month of 11) A toy is formed by mounted cone on a
June? hemisphere, each have base radius of R cm.
a) Rs. 40800 curved surface area of each conical and
b) Rs. 31300 hemispherical part is Z cm2. If H is the height of
c) Rs. 38400 the cone, then find the value of (3R2 + 2H2)/R2.
d) Rs. 32800 a) 4
e) None of these b) 9
c) 9/2
9) Arun purchased two types of balls – Tennis d) 5
balls and plastic balls and pays Rs. Z. Number of e) None of these
plastic balls purchased by him is 62.5% as that
of tennis balls, while cost price of a tennis ball is 12) Pipe P alone can fill tank in Z hours, while Q
2.5 times as that of plastic ball. He sold plastic can fill it in Y hours. Time taken by Pipe R alone
balls at 60% profit and tennis balls at 10% profit to empty the tank is 3Y/2 hours. When P and Q
thus earning overall profit of (Y – 10) %. Find the opened together tank can be filled in T/2 hours,
value of Y. when pipe P and R opened together tank can be
a) 30% filled in 4T/3 hours. Find Z is how much % more
b) 20% or less than that of Y?
c) 35% a) 40%
d) 25% b) 66.66%
e) None of these c) 37.5%
d) 60%
10) Car P and Car Q started from point A and e) None of these
point Brespectively, running towards each other.
Car P reaches point B in 20 minutes, while car Q 13) Quantity I. Present age of P is 50% more
takes 18 minutes to reach point A after meeting than that of Q, while Q is 6 years younger than
car P. Find the total time taken by car Q to reach R. Age of R after 9 years will be 31 years. Find
point A? the present age of S, who is six years younger
a) 20 minutes than P.
b) 24 minutes Quantity II. P started a business with initial
c) 30 minutes investment of Rs. Z, after 4 months Q entered
d) 12 minutes with initial investment of Rs. (Z + 6), after another
e) None of these 4 months R entered with initial investment of Rs.

Click Here For Bundle PDF Course | support@guidely.in Page 4 of 12


SBI Clerk & RRB PO Mains PDF Course 2023
Quantitative Aptitude Day -2 (Eng)

48. At the end of one year, profit share of Q and vessel Q is poured in to vessel P, such that milk
R is same. Find the value of Z. in vessel P is not less than water. Every time
a) Quantity I > Quantity II amount of milk and water is an integral value.
b) Quantity I < Quantity II Find maximum value of Z
c) Quantity I = Quantity II Quantity II. Z2 – 12Z + 35 = 0
d) Quantity I ≥ Quantity II Find the value of 4Z.
e) Quantity I ≤ Quantity II a) Quantity I > Quantity II
b) Quantity I < Quantity II
14)If, P2 + Q + PQ = 7 and Q2 + PQ + Q = 5, c) Quantity I = Quantity II or No relation
then d) Quantity I ≥ Quantity II
Quantity I. Find the value of 2P e) Quantity I ≤ Quantity II
Quantity II. Find the value of (2Q + P)/Q
Where P and Q arenatural numbers 16)If(√64)5/3 = (Z + Y) M – 2, Where Z, M and Y are
a) Quantity I > Quantity II distinct natural Numbers.
b) Quantity I < Quantity II Quantity I. Find minimum possible value of (MZ –
c) Quantity I = Quantity II or No relation 4).
d) Quantity I ≥ Quantity II Quantity II. Find unit digit of (M + Y)43.
e) Quantity I ≤ Quantity II a) Quantity I > Quantity II
b) Quantity I < Quantity II
15) Quantity I.Vessel P contains 60 ml mixture of c) Quantity I = Quantity II or No relation
milk and water in ratio of 3:1 and vessel Q d) Quantity I ≥ Quantity II
contains mixture of milk and water in 1:2. Z ml e) Quantity I ≤ Quantity II
mixture from vessel P is removed and 45 ml from

Directions [17 – 20]: Read the following information carefully and answer the questions based on it.
The chart given below shows the partial information about income of company from 2015 to till 2019, and
% of expenses of company for a particular year out of total income in that year.

Click Here For Bundle PDF Course | support@guidely.in Page 5 of 12


SBI Clerk & RRB PO Mains PDF Course 2023
Quantitative Aptitude Day -2 (Eng)

Note: a) Savings in 2019 is thrice as that in 2016, while expenses in 2017 to that in 2019 is in ratio of
7:10
b) Savings of company in 2015 is M crores.
c) Expenses of company in 2016 = Savings of company in 2016
17) Savings in 2018 is what % more or less than 19) Three statements I, II, and III given below. If
Income in 2016? a statement is true then score is 5, if false then
a) 9.09% score is 0. Find the total score.
b) 10% I. Ratio of income in 2018 and 2019 is 4:9
c) 11.11% respectively.
d) 8.33 II. Difference between savings and expenses in
e) None of these 2018 is N. unit digit of NM is 4.
III. Savings in 2019 is four times of M.
18) Income of company in 2020 is five times of a) 0
income in 2017, and ratio of savings and b) 5
expenses of company in 2020 is M:10. Find c) 10
difference between expenses and savings of d) 15
company in 2020? e) None of these
a) 160
b) 210 20) Find total savings of company in all the given
c) 140 years together?
d) 168 a) 195
e) None of these b) 185
c) 205
d) 175
e) None of these

Click Here For Bundle PDF Course | support@guidely.in Page 6 of 12


SBI Clerk & RRB PO Mains PDF Course 2023
Quantitative Aptitude Day -2 (Eng)

Click Here to Get the Detailed Video Solution for the above given Questions
Or Scan the QR Code to Get the Detailed Video Solutions

Answer Key with Explanation


Directions [01 – 04]: Number of employees in department A of
Ratio of number of employees in department A company S = (50 + 10) % x 200 = 120
of company P and R = 8:5 (8a and 5a) Number of employees in department B of
Ratio of number of employees in department B company S = 200 – 120 = 80
of company P and R = 4:3 (4b, 3b) Number of employees in department B of
Now, company T = 5/9 x 180 = 100
8a + 4b = 120 Number of employees in department A of
OR, company T = 180 – 100 = 80
2a + b = 30………………. (1)
Also
5a + 3b = 80……………... (2)
On solving both, we get
Value of a = 10, and b = 10 1) Answer: C
So, employees in department B of company P = According to question,
4 x 10 = 40 (Y) For company Q,
Number of employees in department A of Number of employeeswho likes tea = 40
company R = 5 x 10 = 50 (Z) Number of employees who likes coffee = 75 – 40
Number of employees in department A of = 35
company Q = 40% x 75 = 30 Number of employees in department A who likes
Number of employees in department B of tea = 1/4 x 40 = 10
company Q = 75 – 30 = 45 Number of employees in department B who likes
tea = 40 – 10 = 30

Click Here For Bundle PDF Course | support@guidely.in Page 7 of 12


SBI Clerk & RRB PO Mains PDF Course 2023
Quantitative Aptitude Day -2 (Eng)

Number of employees in department A who likes Number of employees in department A of


coffee = 30 – 10 = 20 company P = Number of employees in
Number of employees in department B who likes department B of company T = 80
coffee = 35 – 20 = 15 This statement is true.
Required difference = 30 – 20 = 10 II. Number of employees in department B of
Hence answer is option C company S is twice as that of company Q.
Number of employees in department B of
2) Answer: B company S = 80
For company S, Number of employees in department B of
Number of males in department B = 50% x 80 = company Q = 45
40 Required ratio = 80:45 = 16:9
Number of females in department B = 80 – 40 = This statement is false.
40 III. Difference between number of employees in
Number of females in department A = 1/6 x 120 department A and department B of company Q
= 20 is half as that of R.
Required % = 60/200 x 100 = 30% Difference between number of employees in
Hence answer is option B department A and department B of company Q =
45 – 30 = 15
3) Answer: A Difference between number of employees in
Total number of employees in department A of department A and department B of company R =
all given companies together = 80 + 30 + 50 + 50 – 30 = 20
120 + 80 = 360 Required ratio = 15:20 = 3:4
Totalnumber of employees in department B of all This statement is false.
given companies together = 40 + 45 + 30 + 80 + So, only I is true
100 = 295 Hence answer is option A
Required difference = 360 – 295 = 65
Hence answer is option A Directions [05 – 08]:
Number of DI questions made by Arun in April =
4) Answer: A 800 x ½ = 400
I. Number of employees in department A of Now,
company P and T is same. 2Z + 10 = 400/800 x 100
2Z = 40%
Z = 20%

Click Here For Bundle PDF Course | support@guidely.in Page 8 of 12


SBI Clerk & RRB PO Mains PDF Course 2023
Quantitative Aptitude Day -2 (Eng)

Total number of questions made by Arun in There is no common factor between 19 and 6,
March (DI + AM) = (20 + 5) % x 1200 = 300 so next number which is exactly divisible by 6 =
Difference betweennumber of DI and Arithmetic (162 – 19 x 6) = 162 – 114 = 48
questions made by Arun in March = 20 Now, further values are possible.
So,number of DI questions made by Arun in So, value of Q = 48/6 = 8
March = (300 + 20)/2 = 160 Required difference = 27 – 8 = 19
Number of arithmetic questions made by Arun in Hence answer is option B
March = 160 – 20 = 140
Total number of questions made by Arun in April 6) Answer: C
= 400 + 80 = 480 Number of arithmetic questions in June = 40
So, value of Y = 480/1200 x 100 = 40% Total number of questions in March = 300
So, value of M = 100 – 25 – 40 – 20 = 15% Required % = 40/300 x 100 = 13.33%
Value of N = 160/800 x 100 = 20% Hence answer is option C
So, value of K = 100 – 50 – 20 – 5 = 25%
Now we can calculate all data. 7) Answer: E
Value of P = 140
Value of Q = 140
So, P = Q.
Hence answer of option E

5) Answer: B 8) Answer: A
According to question, Total earnings of Arun in month of June = 200 x
For June, 180 + 40 x 120 = Rs. 40800
Let number of set A = 2a Hence answer is option A
So, number of set B = 1.5 x 2a = 3a
Let number of set C = b 9) Answer: A
Now, Number of units of tennis balls and plastic balls =
5 x 2a + 3 x 3a + 6 x b = 200 8:5
Value of b = (200 – 19a)/6 Ratio of cost price of tennis ball and plastic balls
(200 – 19a) must be even (divisible by 6). = 5:2
Minimum possible value of a = 2, so value of b = So, ratio of total cost price of tennis ball and
(200 – 19 x 2)/6 = 162/6 = 27 plastic balls = (8 x 5: 5 x 2) = 4:1
Value of P = 27 Total amount spend by Arun = (4 + 1) = 5 units

Click Here For Bundle PDF Course | support@guidely.in Page 9 of 12


SBI Clerk & RRB PO Mains PDF Course 2023
Quantitative Aptitude Day -2 (Eng)

Total selling price = 4 x 1.1 + 1 x 1.6 = 6 units H2 + R2 = 4R2


Required % profit = (6 – 5)/5 x 100 = 20% H2 = 3R2
So, value of Y = 20 + 10 = 30 So, required value = (3R2 + 6R2)/R2 = 9
Hence answer is option A Hence answer is option B

10) Answer: C 12) Answer: A


Let speed of carP and Q = a and b According to question,
Let distance travelled by P while meets Q, and 1/Z + 1/Y = 2/T
distance travelled by Q while meets P = Z and Y ZY/ (Z + Y) = T/2……………. (1)
respectively. Also,
Time taken by P to travel Z km = time taken by Q 1/Z – 2/3Y = 3/4T
to travel Y km = T minutes 3ZY/ (3Y – 2Z) = 4T/3……………... (2)
Now, On dividing both equations, we get
a x T = b x 18……………… (1) (3Y – 2Z) / (Z + Y) = 9/8
Also, So, Y/Z = 5/3
a x (20 - T) = b x T………………. (2) Required % change = (5 – 3)/5 x 100 = 40%
on dividing both we get, Hence answer is option A
T/ (20 – T) = 18/T
T2 + 18T – 360 = 0 13) Answer: C
T2 + 30T – 12T – 360 = 0 Quantity I.
T = 12, - 30 Present age of R = 31 – 9 = 22 years
So, possible value of T = 12 Present age of Q = 22 – 6 = 16 years
So, required time = 12 + 18 = 30 minutes Present age of P = 1.5 x 16 = 24 years
Hence answer is option C So, present age of S = 24 – 6 = 18 years
Quantity II.
11) Answer: B Ratio of profit share of P, Q, and R = Z x 12: (Z +
Let slant height of cone = L cm 6) x 8: 48 x 4
Now, Now,
Π x R x L = 2 x π x R2 (Z + 6) x 8 = 48 x 4
L = 2R…………………... (1) Value of Z = 18
Also, So, Quantity I = Quantity II
H2 + R2 = L2…………. (2) Hence answer is option C
So, from equation (1) and (2), we get

Click Here For Bundle PDF Course | support@guidely.in Page 10 of 12


SBI Clerk & RRB PO Mains PDF Course 2023
Quantitative Aptitude Day -2 (Eng)

14) Answer: E (Z – 7) (Z – 5) = 0
P2 + Q + PQ = 7………………. (1) Z = 7, 5
Q2 + PQ + P = 5………………. (2) Required value = 7 x 4 and 5 x 4 = 28 and 20
On adding both, we get So, quantity I ≥ Quantity II
(P + Q)2 + (P + Q) = 12 Hence answer is option D
(P + Q) x (P + Q + 1) = 12…………………. (3)
On subtracting both, we get 16) Answer: D
(P2 – Q2) + (P – Q) = 3 (√64)5/3 = (Z + Y) M – 2
(P – Q) x (P + Q + 1) = 3………………… (4) 43 x 5/3 = (Z + Y) M – 2
On dividing both equations, we get 45 = (Z + Y) M – 2 OR 210 = (Z + Y) M – 2
(P + Q) / (P – q) = 12/3 If Z + Y = 2, that means Z = Y = 1, but both Z
P/Q = 2/1 and Y are distinct.
Values of P and Q are multiple of 2 and 1. So, (Z + Y) = 3
Quantity I = 2 x 2 = 4 Z = 1 or 3 and Y = 3 or 1
Quantity II = (2 x 1 + 2)/1 = 4 M–2=5
So, Quantity I = Quantity II M=7
Hence answer is option C Quantity I.
Required minimum value = 7 x 1 – 4 = 3
15) Answer: D Quantity II.
Quantity I. Value of (M + Y) = 7 + 3 OR 7 + 1 = 10 or 8
When Z ml is removed from vessel P, ratio of Required unit digit = 1043 = 0
milk and water will remain same = 3:1 [3a, a] Also, required unit digit = 843 = 83 = 2
Now, So, Quantity I ≥ Quantity II
3a + 1/3 x 45 ≥ a + 2/3 x 45 Hence answer is option D
3a + 15 ≥ a + 30
2a ≥ 15 Directions [17 – 20]:
Value of a ≥ 7.5 Income of company in 2016 = (Z – 60)
Value of Z = (60 – 4a) Savings of company in 2016 = 1/2 x (Z – 60)
When a is minimum, Z is maximum. Savings of company in 2019 = 1/3 x (500 – Y)
So minimum possible value of Z a = 8 Now,
So, maximum value of Z = 60 – 4 x 8 = 28 3 x 1/2 (Z – 60) = 1/3 x (500 – Y)
Quantity II. 9/2 x (Z – 60) = (500 – Y) ……………. (1)
Z2 – 12Z + 35 = 0 Expenses in 2017 = 60% x (240 – Z)

Click Here For Bundle PDF Course | support@guidely.in Page 11 of 12


SBI Clerk & RRB PO Mains PDF Course 2023
Quantitative Aptitude Day -2 (Eng)

Expenses in 2019 = 2/3 x (500 – Y) Income in 2020 = 140 x 5 = 700


Now, Ratio of savings and expenses of company =
10 x 60% x (240 – Z) = 7 x 2/3 x (500 – Y) 15:10 = 3:2
9/7 x (240 – Z) = (500 – Y) ……………... (2) Required difference = 1/5 x 700 = 140
From (1) and (2) Hence answer is option C
9/7 x (240 – Z) = 9/2 x (Z – 60)
480 – 2Z = 7Z – 420 19) Answer: C
900 = 9Z I. Ratio of income in 2018 and 2019 is 4:9
Value of Z = 100 respectively.
On putting value of Z in equation (1), we get Respective ratio = 80:180 = 4:9
Value of Y = 320 This statement is true and score is = 5
Savings in 2015 = 25% x 60 = 15 II. Difference between savings and expenses in
So, Expenses in 2018 = 45% x 80 = 36 2018 is N. unit digit of NM is 4.
Now we can find all data for a particular year Value of N = 44 – 36 = 8
Required unit digit = 815 = 83 = 2
This statement is false and score is = 0
III. Savings in 2019 is four times of M.
Required ratio = 60:15 = 4:1
This statement is true and score is = 5
17) Answer: B So, total score = 5 + 5 = 10
According to question, Hence answer is option C
Savings in 2018 = 44
Income in 2016 = 40 20) Answer: A
Required % change = (44 – 40)/40 x 100 = 10% Total savings = 195 Crores
Hence answer is option B Hence answer is option A

18) Answer: C

Click Here For Bundle PDF Course | support@guidely.in Page 12 of 12


SBI Clerk & RRB PO Mains PDF Course 2023
ENGLISH Day -2 (Eng)

English Language

Directions (1-6): In the following passage some a) Commendable


blanks are given. You have to choose the b) Subtle
appropriate word (s) from the given options to c) Sudden
make the passage meaningful. d) Rapid
The _______________ (1) growth in India’s road e) Retrograde
infrastructure is well known. It enables reduction a) Both 1 and 4
in transportation costs and enhances economic b) Both 2 and 3
growth. Closer to the borders, it _____________ c) All 1, 4 and 5
(2) military logistics and adds to national security. d) All 2, 3 and 4
But when a National Highway (NH) is utilized to e) All 3, 4 and 5
25 per cent capacity because of a single bridge
being in ____________ (3) in vulnerable border 2. Which of the following word is/are appropriate
areas, exactly the reverse happens for blank 2?
The huge investment is under-utilized leading to a) Complicates
_____________ (4) economic losses each day. b) Boosts
Military logistics are affected making us more c) Reduces
vulnerable along our borders. It is a fact that this d) Streamlines
is happening because of a lack of capability to e) Increases
respond to distressed bridges adequately. a) Both 1 and 3
New technology allows it to be rectified in days. b) Both 2 and 4
We need to understand and accept the problem c) All 1, 3 and 4
first. The solution is then just round the corner d) All 2, 4 and 5
and available at a _____________ (5) fraction of e) All 3, 4 and 5
the cost of what we are losing. Let me put the
problem in perspective and examine how we can 3. Which of the following word is/are appropriate
resolve it before the next monsoons, when the for blank 3?
incidence of distressed bridges increases and a) Distress
the losses to the nation _____________ (6) b) Maintenance
exponentially, in addition to adding to the woes of c) Repair
the soldiers on the front line. d) Service
1. Which of the following word is/are appropriate e) Operations
for blank 1? a) Both 1 and 3

Click Here For Bundle PDF Course | support@guidely.in Page 1 of 15


SBI Clerk & RRB PO Mains PDF Course 2023
ENGLISH Day -2 (Eng)

b) Both 2 and 4 b) Stabilize


c) All 1, 2 and 3 c) Multiply
d) All 2, 3 and 4 d) Decrease
e) All 3, 4 and 5 e) Plateau
a) Both 1 and 3
4. Which of the following word is/are appropriate b) Both 3 and 5
for blank 4? c) All 1, 2 and 4
a) Negligible d) All 2, 3 and 4
b) Recurring e) All 3, 4 and 5
c) Significant
d) Moderate Directions (7-11): In the following questions, a
e) Catastrophic table is given with three columns. A divided
a) Both 1 and 3 sentence is mentioned in three columns. You
b) Both 2 and 5 have to combine the divided parts of the
c) All 1, 3 and 4 sentence, so that a meaningful word can be
d) All 2, 3 and 4 formed.
e) All 3, 4 and 5 7.
Column I Column II Column III
5. Which of the following word is/are appropriate 1) To 4) the army 7) the order
for blank 5? understand how would like to of the losses
a) Substantial huge the call some of thereof is
b) Minuscule economic these roads close to Rs
c) Exorbitant impact is let us 1000 crore
d) Reasonable per annum
e) Negligible 2) Because of 5) the capacity 8) which
a) Both 1 and 4 the weak bridge of the entire carries 20 to
b) Both 2 and 5 in Irang, NH 37 is 30 per cent of
c) All 1, 2 and 3 affected the traffic
d) All 2, 3 and 5
e) All 3, 4 and 5 3) A similar 6) take the 9) it means
situation example of faster
6. Which of the following word is/are appropriate prevails on NH 37 from logistics
for blank 6? other National Silchar to build-up at a
a) Mount

Click Here For Bundle PDF Course | support@guidely.in Page 2 of 15


SBI Clerk & RRB PO Mains PDF Course 2023
ENGLISH Day -2 (Eng)

highways near Imphal, one of lesser cost b) Only (2-5-8)


the LAC the lifelines of c) Both (1-6-7) and (2-4-9)
the state of d) Both (2-6-9) and (3-4-8)
Manipur, e) None of these
a) Only (1-5-7)
b) Only (3-4-9) 9.
c) Both (1-6-8) and (2-5-7) Column I Column II Column III
d) Both (2-4-7) and (3-6-9) 1) Ironically, 4) to control of 7) ever
e) None of these both the resources and growing
Generals RSF integration demand for
8. claimed that holding
Column I Column II Column III they were democratic
1) In the 4) $2.7bn in 7) will be trying to bring elections
extreme, AI 110 deals in positive and back Sudan
can become 2022 their risks will be 2) The time 5) undermining 8) and power
so clever as to manageable frame to carry institutions like that he has
outwit out this the judiciary, gained with
humanity itself integration resulting in the their support
2) Powerful AI 5) it is very 8) Indeed, the AI has been a forceful and
systems much race has just thorny issue
should be plausible begun. But the 3) His tenure 6) would mean 9) and the
developed risks are too was marred losing control army
profound to be by human over his army, dispensed with
ignored rights abuses, the militias and the civilian-led
3) The 6) only once 9) an corruption all the wealth transitional
generative-AI we are unscrupulous and government in
startups alone confident actor will October 2021
collectively that their sabotage its a) Only (1-6-9)
raised effects capability to b) Only (2-4-8)
cause c) Only (3-5-7)
unspeakable d) Both (1-4-8) and (2-6-9)
harm to society e) None of these
a) Only (1-5-9)

Click Here For Bundle PDF Course | support@guidely.in Page 3 of 15


SBI Clerk & RRB PO Mains PDF Course 2023
ENGLISH Day -2 (Eng)

10. also be dependency on required to


Column I Column II Column III produced from the import of drive towards
1) These 4) cement its 7) would have algae, crude oil and net zero
strategists position strengthened natural gas targets
have been strongly as US power in the 2) While India 5) for energy 8) fuel
thinking more the most world by is a net security and blending
in terms of powerful winning more exporter of low carbon programs and
country friends and petroleum mobility demand from
providing a products, the medical,
credible liquor and
framework cosmetic
2) With vision 5) who can 8) thus the industry
and wisdom, challenge US potential path of 3) The 6) but it is 9) creates a
this was the dominance peace which objective is to currently not persistent
time for the would have push future considered sense of
USA to strengthened fuels into the economically vulnerability
US power in the mainstream viable
world narrative
3) The 6) in which 9) with no real a) Only (1-6-8)
concept of several competitors and b) Only (2-5-7)
dominance of hundred no big enemies c) Both (1-4-9) and (2-6-8)
the world by thousand d) Both (2-4-9) and (3-5-7)
any single people died e) None of these
country
a) Only (1-5-8) Directions (12-16): In the following questions, a
b) Only (2-4-9) sentence is given in which two parts of the
c) Only (3-6-7) sentence is highlighted. You have to identify the
d) Both (2-5-8) and (3-4-9) highlighted part of the sentence is correct or not,
e) None of these if not replace it with the correct part mentioned in
the options.
11. 12. It is time such third-generation bridges are
Column I Column II Column III introduced in larger numbers. At a small cost the
1) Ethanol can 4) the 7) which is

Click Here For Bundle PDF Course | support@guidely.in Page 4 of 15


SBI Clerk & RRB PO Mains PDF Course 2023
ENGLISH Day -2 (Eng)

value obtained will far higher, the results b) being a silent spectator, diplomatic isolation
immediate could worked
a) have been introduced, will be obtained far c) be a silent spectating, work diplomatic
higher isolation would
b) were introduced, obtained will be far higher d) silent be a spectator, isolation diplomatic work
c) will be introduced, obtained will far highest would
d) be introduced, obtained will be far more higher e) no replacement required
e) no replacement required
15. A great new beginning by the USA to openly
13. Much more resources need to be allocated to and clearly renounce a of role dominance and
researchabout AI alignment for societal needs, with it replacing a leadership role, guided by
its safety and governance, to devise standards peace and soft power, to one of the best things
and to create a bureaucracy to administer the happen would be to our deeply troubled world
standards for governing AI development a) replacing a leadership with a role of
a) with AI alignment towards societal needs, the dominance and it a, would happen to be one of
standards governing AI development and its the best things
administration b) a role of dominance and replacing it with a
b) within AI alignment of societal needs, the leadership, would be one of the best things to
governance of AI development through happen
standards c) replacing a role of dominance with a
c) for AI alignment against societal needs, the leadership and of it a, the best things would be to
standards for AI development and governing happen one of
d) on AI alignment to societal needs, the d) Leadership a replacing with and role of
standards and govern AI development dominance it, happen would be one of the best
e) no replacement required things to
e) no replacement required
14. The international community cannot sit on the
fence and be a silent spectator. To what extent 16. Thanks to the shift to fossil fuel based
imposing sanctions and diplomatic isolation technologies, coal and natural gas the preferred
would work, is difficult to say, but it can be mainstream sources are not, processed to which
considered first produce are syngas and then methanol.
a) be a silenting spectator, would work diplomatic a) are the preferred mainstream sources, which
isolation are processed to first produce

Click Here For Bundle PDF Course | support@guidely.in Page 5 of 15


SBI Clerk & RRB PO Mains PDF Course 2023
ENGLISH Day -2 (Eng)

b) the sources mainstream preferred are not, to b) In order to, similarly


first produce which are processed c) Instead of, as a result
c) mainstream sources are not the preferred, a) Only 1
which processed are to first produce b) Only 2
d) are not preferred mainstream sources the, c) Only 3
produce to which first are processed d) Both 1 and 2
e) no replacement required e) None of these

Directions (17-20): In the following questions, a 19.


sentence is divided into three parts. You have to a) I didn't want to go to the party,
choose the correct connector to combine the b) I promised my friend I would,
sentence, so that a meaningful sentence can be c) I went and ended up having a great time
formed. a) So, but
17. b) But, so
a) Although I love living in the city, sometimes I c) But, that
miss the peace quiet of the countryside, a) Only 1
b) I make an effort to take weekend trips to the b) Only 2
mountains or the beach c) Only 3
a) And, so d) Both 1 and 2
b) So, and e) None of these
c) So, that
a) Only 1 20.
b) Only 2 a) I'm not a big fan of horror movies,
c) Only 3 b) I'll watch one occasionally
d) Both 2 and 3 c) it's with a group of friends
e) None of these a) but, if
b) if, but
18. c) so, that
a) Because I'm trying to save money, I've been a) Only 1
bringing my lunch to work eating out, and b) Only 2
b) I've been able to pay off some of my debt c) Only3
d) Both 1 and 2
a) Similarly, in order to e) None of these

Click Here For Bundle PDF Course | support@guidely.in Page 6 of 15


SBI Clerk & RRB PO Mains PDF Course 2023
ENGLISH Day -2 (Eng)

Click Here to Get the Detailed Video Solution for the above given Questions
Or Scan the QR Code to Get the Detailed Video Solutions

Answer Key with Explanation

1. Answer: C growth has happened unexpectedly or quickly


The context of the sentence suggests that it is without warning, which may not be accurate
referring to the growth of India's road given that the sentence suggests that the growth
infrastructure and its impact on the economy. is well known. So, the option is incorrect
The sentence mentions that the growth in road Option (5) - "Retrograde" growth would be an
infrastructure is well known and that it leads to a incorrect option because it implies that the
reduction in transportation costs and enhances growth has been backwards or declining, which
economic growth. is the opposite of what the sentence is trying to
Options (1) and (4) support the context of the convey. So, the option is incorrect
sentence. So, these are the correct options ‘Retrograde’ – moving backward
‘Commendable’ – deserving praise
‘Option (2) - "Subtle" growth would be an 2. Answer: D
incorrect option because it implies that the The blank in the sentence requires a verb to
growth has been minimal or not noticeable, describe the impact of a factor near the borders
which does not fit with the sentence's emphasis on military logistics and national security
on the impact of the growth on transportation Options (2), (4) and (5) support the context of
costs and economic growth. So, the option is the sentence. So, these are the correct options
incorrect ‘Streamlines’ – to make simpler or more efficient
‘Subtle’ – noticeable or obvious in a way Option (1) - "Complicates" would be an incorrect
Option (3) - "Sudden" growth would be an option because it implies that the factor makes
incorrect option because it implies that the military logistics more difficult and complex,

Click Here For Bundle PDF Course | support@guidely.in Page 7 of 15


SBI Clerk & RRB PO Mains PDF Course 2023
ENGLISH Day -2 (Eng)

which would harm national security. So, the effectively, leading to economic losses and
option is incorrect vulnerabilities along the country's borders. The
Option (3) - "Reduces" would be an incorrect cause of this underutilization is a lack of
option because it implies that the factor capability to respond to distressed bridges
decreases military logistics, which would harm adequately
national security, it is opposite to the context of Options (2), (3) and (4) support the context of
the sentence. So, the option is incorrect the sentence. So, these are the correct option
‘Recurring’ – happening many times
3. Answer: C Option (1) – “negligible” is an incorrect word.
The context of the sentence suggests that it is "Negligible economic losses" would imply that
referring to the impact of a single bridge in the losses are small and insignificant, which
vulnerable border areas on the capacity contradicts the context of the sentence. So, the
utilization of a National Highway (NH). The option is eliminated
sentence implies that when the capacity Option (5) – “catastrophic” is an incorrect word.
utilization of the NH is reduced due to a single "Catastrophic economic losses" would imply that
bridge, something unexpected happens the losses are disastrous and potentially ruinous,
Options (1), (2) and (3) support the context of which is not supported by the context of the
the sentence. So, these are the correct options sentence. So, the option is eliminated
‘Distress’ – a state of suffering or pain ‘Catastrophic’ – involving or causing sudden
Option (4) - "Service" would be an incorrect great damage
option because it implies that the bridge needs
routine servicing, but does not address the 5. Answer: B
underlying issue of reduced capacity utilization the context of the sentence says that
of the NH. So, the option is incorrect understanding and accepting a problem is the
Option (5) - "Operation" would be an incorrect first step in finding a solution that is just around
option because it implies that the bridge is the corner and available at a negligible fraction
operational, but does not address the issue of of the cost of what is currently being lost. The
reduced capacity utilization of the NH due to the context implies that the problem is related to a
single bridge. So, the option is incorrect financial cost of some kind
Options (2) and (5) support the context of the
4. Answer: D sentence. So, both the options are correct
the sentence describes a situation in which a ‘Minuscule’ – extremely small
significant investment is not being utilized

Click Here For Bundle PDF Course | support@guidely.in Page 8 of 15


SBI Clerk & RRB PO Mains PDF Course 2023
ENGLISH Day -2 (Eng)

Option (1) – “substantial” is an incorrect word. exponential increase. So, the option is
"Substantial fraction of the cost" would imply that eliminated
the cost of the solution is significant and may not ‘Stabilize’ – make or become unlikely to change
be feasible, which contradicts the context of the Option (4) – “decrease” is an incorrect word.
sentence. So, the option is eliminated "Decrease exponentially" would imply that the
‘Substantial’ – of a considerable size or value losses are decreasing at an increasingly fast
Option (3) – “exorbitant” is an incorrect word. rate, which contradicts the context of the
"Exorbitant fraction of the cost" would imply that sentence. So, the option is eliminated
the cost of the solution is prohibitively high, Option (5) – “plateau” is an incorrect word.
which is not supported by the context of the "Plateau exponentially" also doesn't make sense
sentence. So, the option is eliminated in the context of the sentence, as a plateau
‘Exorbitant’ – unreasonably high would suggest a stable period of no change in
Option (4) – “reasonable” is an incorrect word. the losses, not an exponential increase. So, the
"Reasonable fraction of the cost" would imply option is eliminated
that the cost of the solution is acceptable but ‘Plateau’ – reach a state of little or no change
does not convey the sense of the cost being very
low, which is suggested by the context of the 7. Answer: C
sentence. So, the option is eliminated (1-6-8) – To understand how huge the economic
impact is let us take the example of NH 37 from
6. Answer: A Silchar to Imphal, one of the lifelines of the state
the context of the sentence says that there is a of Manipur, which carries 20 to 30 per cent of the
problem related to distressed bridges that needs traffic
to be addressed before the next monsoons (2-5-7) - Because of the weak bridge in Irang,
arrive. If the problem is not resolved, the losses the capacity of the entire NH 37 is affected the
to the nation will multiply exponentially, and order of the losses thereof is close to Rs 1000
soldiers on the front line will be further impacted. crore per annum
Options (1) and (3) support the context of the
sentence. So, both the options are correct 8. Answer: D
‘Mount’ – move up (2-6-9) – Powerful AI systems should be
Option (2) – “stabilize” is an incorrect word. developed only once we are confident that their
"Stabilize exponentially" doesn't make sense in effects will be positive and their risks will be
the context of the sentence, as stabilization manageable
would suggest a leveling off of the losses, not an

Click Here For Bundle PDF Course | support@guidely.in Page 9 of 15


SBI Clerk & RRB PO Mains PDF Course 2023
ENGLISH Day -2 (Eng)

(3-4-8) - The generative-AI startups alone that should have been done in the past.
collectively raised $2.7bn in 110 deals in 2022. “Obtained will far higher” - This changes the
Indeed, the AI race has just begun. But the risks word order of the phrase "obtained will be far
are too profound to be ignored higher" by removing the auxiliary verb "be". The
correct phrase is "obtained will be far higher. So,
9. Answer: C the highlighted part of the sentence is incorrect.
(3-5-7) - His tenure was marred by human rights Option (a) – “have been introduced” - This
abuses, corruption and undermining institutions changes the verb tense to the present perfect
like the judiciary, resulting in the forceful and tense "have been introduced" which doesn't
ever growing demand for holding democratic make sense in the context of the sentence. The
elections. correct verb tense is the past simple "were
introduced”. “Will be obtained far higher” - This
10. Answer: B changes the word order of the phrase "obtained
(2-4-9) - With vision and wisdom, this was the will be far higher" by putting the auxiliary verb
time for the USA to cement its position strongly "will be" after the object "the value". The correct
as the most powerful country with no real phrase is "obtained will be far higher”. So, the
competitors and no big enemies option is incorrect.
Option (c) – “will be introduced” - This changes
11. Answer: D the verb tense to the future tense "will be
(2-4-9) - While India is a net exporter of introduced" which doesn't make sense in the
petroleum products, the dependency on the context of the sentence. The correct verb tense
import of crude oil and natural gas creates a is the past simple "were introduced”. “Obtained
persistent sense of vulnerability will far highest” - This changes the correct
(3-5-7) - The objective is to push future fuels into adjective "far higher" to the incorrect form "far
the mainstream narrative for energy security and highest". The correct form is "far higher”. So, the
low carbon mobility, which is required to drive option is incorrect.
towards net zero targets Option (d) – “be introduced” - This changes the
verb tense to the present subjunctive "be
12. Answer: B introduced" which doesn't make sense in the
“are introduced” - This changes the verb tense context of the sentence. The correct verb tense
from past to present tense. The correct verb is the past simple "were introduced”. “Obtained
tense is the past simple "were introduced" will be far more higher” - This changes the
because the sentence is talking about something correct phrase "far higher" to the incorrect

Click Here For Bundle PDF Course | support@guidely.in Page 10 of 15


SBI Clerk & RRB PO Mains PDF Course 2023
ENGLISH Day -2 (Eng)

phrase "far more higher". The correct phrase is changes the meaning of the sentence to suggest
"far higher”. So, the option is eliminated. that the standards themselves are responsible
Option (b) – “were introduced” and “obtained will for both governing and administering AI
be far higher” is contextually and grammatically development. So, the option is incorrect
correct. So, this is the correct option Option (b) - "within AI alignment of societal
needs" - This changes the preposition "to" to
13. Answer: D "of," which alters the meaning of the phrase to
"about AI alignment for societal needs" - This suggest that societal needs are already included
changes the preposition "on" to "about," which within AI alignment. Additionally, "within"
suggests a more casual discussion of the topic suggests that the alignment is happening within
rather than a formal exploration of it. "The the context of societal needs, rather than
standards for governing AI development" - This aligning AI to meet those needs. "The
rephrases the sentence to suggest that the governance of AI development through
standards are for governing AI development, standards": This changes the order of the
rather than being separate from the act of sentence to put "governance" before
governing. The new phrase "for governing" "standards," which changes the emphasis of the
implies that the standards themselves are sentence to suggest that governance is more
responsible for governing, rather than being important than the standards themselves.
used to regulate the process. So, the highlighted Additionally, "through standards" implies that
part of the sentence is incorrect. governance is being done by the standards,
Option (a) - "with AI alignment towards societal rather than the standards being used to regulate
needs" - This changes the preposition "on" to the governance process. So, the option is
"with," which changes the meaning of the phrase incorrect
to suggest that AI is already aligned with societal Option (c) - "for AI alignment against societal
needs. Additionally, "towards" changes the needs" - This changes the preposition "to" to
direction of the alignment to suggest that AI is "against," which completely reverses the
moving towards societal needs, rather than meaning of the phrase to suggest that AI is
being aligned with them. "The standards being aligned to work against societal needs,
governing AI development and its administration" rather than meeting them. "The standards for AI
- This change adds the possessive pronoun "its" development and governing": This changes the
before "administration," which can be unclear order of the words to suggest that the standards
since it's not clear what "its" is referring to. are for both AI development and governing,
Additionally, adding "and its administration" which could be ambiguous. Additionally,

Click Here For Bundle PDF Course | support@guidely.in Page 11 of 15


SBI Clerk & RRB PO Mains PDF Course 2023
ENGLISH Day -2 (Eng)

"governing" by itself doesn't fully convey the "spectatorship", but this is an uncommon word
meaning of regulating AI development, and and would sound awkward in this phrase. A
could imply that the standards are being used for better option would be to use "spectating" as a
a broader range of governance activities. So, the gerund, making the correct form. Work
option is incorrect. diplomatic isolation would - This form changes
Option (d) – “on AI alignment to societal needs” the order of the words in the original phrase,
and “the standards and govern AI development” making it grammatically incorrect. In English, the
is contextually and grammatically incorrect subject usually comes before the verb, so
"diplomatic isolation" should come before "would
14. Answer: E work." So, the option is incorrect
option (a) - Be a silenting spectator- "Silenting" is Option (d) - Silent be a spectator - This form
not a word in the English language, so this form changes the order of the words in the original
is incorrect. Would work diplomatic isolation - phrase, making it grammatically incorrect. In
This form switches the positions of the verb English, adjectives usually come before the
"work" and the noun phrase "diplomatic nouns they modify, so "silent" should come
isolation", which makes the sentence before "spectator." The correct form would be
grammatically incorrect. The correct form would "Be a silent spectator”. Isolation diplomatic work
be "Diplomatic isolation would work”. So, the would: This form also changes the order of the
option is incorrect words in the original phrase, making it
Option (b) - Being a silent spectator - This form grammatically incorrect. The correct order
is grammatically correct but changes the tense. should be "diplomatic isolation would work”. So,
The original phrase is in the imperative form, the option is incorrect
which is used to give commands or make Highlighted part – ‘be a silent spectator’ and
requests. This form changes the sentence to ‘diplomatic isolation would work’ is contextually
present continuous tense. Diplomatic isolation and grammatically incorrect. So, no replacement
could worked - This form changes the verb required.
"work" to "worked", which makes it the past
tense form. However, in this context, "would 15. Answer: B
work" is the correct form as it suggests a ‘a of role dominance and with it replacing a
hypothetical situation. So, the option is incorrect leadership’ - This option is grammatically
Option (c) - Be a silent spectating - "Spectating" incorrect because it mixes up the order of the
is a verb form, which is incorrect in this context. words and phrases in the original sentence. ‘To
The correct noun form of this word is one of the best things happen would be’ - This

Click Here For Bundle PDF Course | support@guidely.in Page 12 of 15


SBI Clerk & RRB PO Mains PDF Course 2023
ENGLISH Day -2 (Eng)

option is grammatically incorrect because it which is generally considered poor grammar. So,
changes the order of the words and phrases, the option is incorrect.
making it unclear what the intended meaning is. Option (b) – ‘a role of dominance and replacing it
So, the highlighted part of the sentence is with a leadership’ and ‘would be one of the best
incorrect. things to happen’ is contextually and
Option (a) – ‘replacing a leadership with a role of grammatically correct. So, this is the correct
dominance and it a’ - This option is option
grammatically incorrect because it includes an
unnecessary word "it" and switches the order of 16. Answer: A
the phrases "replacing a leadership" and "a role ‘The preferred mainstream sources are not’ -
of dominance. ‘Would happen to be one of the This option is grammatically incorrect because it
best things’ - This option is grammatically omits the object of the sentence, leaving it
incorrect because it changes the order of the unclear what is not the preferred mainstream
words and phrases in the original sentence, sources. ‘Processed to which first produce are’ -
making it unclear what the intended meaning is. This option is grammatically incorrect because it
So, the option is incorrect changes the order of the words and phrases in
Option (c) – ‘Replacing a role of dominance with the original sentence, making it unclear what the
a leadership and of it a’ - This option is intended meaning is. So, the highlighted part of
grammatically incorrect because it includes an the sentence is incorrect
unnecessary phrase "of it a”. ‘The best things Option (b) – ‘The sources mainstream preferred
would be to happen one of’ - This option is are not’ - This option is grammatically incorrect
grammatically incorrect because it changes the because it changes the order of the words and
order of the words and phrases, making it phrases, making it unclear what the intended
unclear what the intended meaning is. So, the meaning is. ‘To first produce which are
option is incorrect processed’ - This option is grammatically
Option (d) – ‘Leadership a replacing with and incorrect because it changes the order of the
role of dominance it’ - This option is words and phrases, making it unclear what the
grammatically incorrect because it mixes up the intended meaning is. So, the option is incorrect
order of the words and phrases in the original Option (c) – ‘Mainstream sources are not the
sentence and includes an unnecessary "it" at the preferred’ - This option is grammatically incorrect
end. ‘Happen would be one of the best things to’ because it omits the object of the sentence,
- This option is grammatically incorrect because leaving it unclear what is not the preferred
it ends the sentence with a preposition ("to"), mainstream sources. ‘Which processed are to

Click Here For Bundle PDF Course | support@guidely.in Page 13 of 15


SBI Clerk & RRB PO Mains PDF Course 2023
ENGLISH Day -2 (Eng)

first produce’ - This option is grammatically countryside, so I make an effort to take weekend
incorrect because it changes the order of the trips to the mountains or the beach
words and phrases, making it unclear what the
intended meaning is. So, the option is eliminated 18. Answer: C
Option (d) – ‘Are not preferred mainstream “Instead of" is a transitional phrase that shows
sources the’ - This option is grammatically the speaker's decision to do something different
incorrect because it changes the order of the than what was originally planned. "As a result" is
words and phrases, making it unclear what the a transitional phrase that shows the
intended meaning is. ‘Produce to which first are consequence of the speaker's decision to bring
processed’ - This option is grammatically their lunch to work.
incorrect because it changes the order of the The sentence is - Because I'm trying to save
words and phrases, making it unclear what the money, I've been bringing my lunch to work
intended meaning is. So, the option is incorrect instead of eating out, and as a result, I've been
Option (a) – ‘are the preferred mainstream able to pay off some of my debt.
sources’ and ‘which are processed to first
produce’ is contextually and grammatically 19. Answer: B
correct. In this sentence, "but" is a coordinating
conjunction that connects the two independent
17. Answer: A clauses "I didn't want to go to the party" and "I
In this sentence, "although" is a subordinating promised my friend I would.""So" is a
conjunction that introduces the subordinate coordinating conjunction that connects the two
clause "Although I love living in the independent clauses "I promised my friend I
city.""Sometimes" is an adverb that indicates the would" and "I went and ended up having a great
speaker's occasional feeling of missing the time”.
countryside. "And" is a coordinating conjunction The sentence is - I didn't want to go to the party,
that connects the two nouns "mountains" and but I promised my friend I would, so I went and
"beach.""So" is a coordinating conjunction that ended up having a great time
connects the two independent clauses "I miss
the peace and quiet of the countryside" and "I 20. Answer: A
make an effort to take weekend trips”. In this sentence, "but" is a coordinating
The sentence is - Although I love living in the conjunction that connects the two independent
city, sometimes I miss the peace and quiet of the clauses "I'm not a big fan of horror movies" and
"I'll watch one occasionally if it's with a group of

Click Here For Bundle PDF Course | support@guidely.in Page 14 of 15


SBI Clerk & RRB PO Mains PDF Course 2023
ENGLISH Day -2 (Eng)

friends.""If" is a subordinating conjunction that preference for movies and the exception they
introduces the conditional clause "if it's with a make for social situations.
group of friends." These two connectors show a The sentence is - I'm not a big fan of horror
contrast between the speaker's general movies, but I'll watch one occasionally if it's with
a group of friends

Click Here For Bundle PDF Course | support@guidely.in Page 15 of 15


SBI Clerk & RRB PO Mains PDF Course 2023
Reasoning Ability Day -3 (Eng)

Reasoning Ability

Directions (1-5): Study the following statements No table is a den


and then decide which of the given conclusions Only a few dens are cave
will definitely false from the given statements Conclusions:
disregarding the commonly known facts. a) Some seats are table
1. Statements: b) Some caves are table is a possibility
Only a few abacuses are calculator c) All seats can never be cave
Some boxes are tins d) All chairs are den is a possibility
No Calculator is an engine e) No den is Chair
Only a few engines are tins
Conclusions: 4. Statements:
a) All boxes can never be calculators Only rat is a rabbit
b) All abacuses can be engines No rat is a bee
c) Some calculators being tin is a possibility Only a few bees are hen
d) Some engines are definitely not boxes At least some hens are Peacocks
e) Both b and d All peacocks are parrots
Conclusions:
2. Statements: a) All bees can never be parrot
Only a few beats are music b) All rabbits are hen is not a possibility
At least some music is melody c) Some rabbits can be bee
All Melodies are tune d) All peacocks are bee
No tune is a theme e) Some hens are definitely not rat
Only a few themes are colour
Conclusions: 5. Statements:
a) No theme is Melody Only a few nails are hammer
b) Some melodies are definitely not beats All knives are hammer
c) Some music can be colour No knife is a blade
d) All music being theme is not a possibility Only a few blades are cutter
e) Some melodies are not themes is a possibility Conclusions:
a) Some nails are not knife
3. Statements: b) All hammers can be cutters
All seats are chair c) No blade is nail is a possibility
Only a few chairs are table d) Some hammers are not cutters is a possibility

Click Here For Bundle PDF Course | support@guidely.in Page 1 of 12


SBI Clerk & RRB PO Mains PDF Course 2023
Reasoning Ability Day -3 (Eng)

e) All cutters can be knife 7. If the incentive of G’s daughter-in-law is


increased by 20% and the average incentive of
Directions (6-10): Study the following information G’s daughter-in-law after the increment and F’s
carefully and answer the given questions. daughter is Rs.32000, then what is the sum of
Eight persons – A, B, C, D, E, F, G and H are the incentive of F’s daughter and D?
there in the family. Each person got an incentive a) Rs. 80000
from their company. No two persons got the b) Rs. 50000
same amount of incentives. c) Rs. 72000
B is the only daughter of H and got more d) Rs. 90000
incentive than G, who is the daughter-in-law of F. e) Rs. 100000
F has only two children. C is the son of G and got
less incentive than his uncle. E is not C’s father’s 8. How is F related to the one who got the third
sibling. G has only one sibling. A, who is the lowest incentive?
daughter-in-law of D, got more incentive than B. a) Grandmother
C, who is the nephew of E and B, got more b) Mother
incentive than G. C did not get more incentive c) Son-in-law
than B and he is a married person. The one who d) Grandfather
is E’s sister’s husband got the highest incentive. e) Either a or d
The one who is the mother of B got less incentive
than her daughter-in-law. The one who got the 9. Which of the following statement(s) is required
second highest incentive got Rs.35000. H got to find the incentive of the one who got the fourth
more incentive than his daughter-in-law and E. H lowest incentive?
neither got 35000 nor got more than B. The one I. B got two times the sum of the incentive of C’s
who got the second lowest incentive got spouse and B’s sibling and the difference
Rs.10000, which is one-fifth of the one who got between the incentives of B and C’s uncle is Rs.
the highest incentive. 90000.
6. As many persons got more incentive than G’s II. The average of the incentive of H’s daughter-
sister-in-law as less than__. in-law and C’s grandmother is Rs.7500 and the
a) G’s daughter-in-law sum of the incentives of G’s brother's incentive
b) A’s Father-in-law and F is Rs.45000.
c) H’s granddaughter III. The sum of the incentive of D’s son and H’s
d) A’s wife son is Rs.64000 and the sum of the incentives of
e) None of these A’s spouse and G’s brother is Rs.36000.
a) Either I or II

Click Here For Bundle PDF Course | support@guidely.in Page 2 of 12


SBI Clerk & RRB PO Mains PDF Course 2023
Reasoning Ability Day -3 (Eng)

b) Only II immediate right of the one who writes sorry. Only


c) Either II or III two persons sit between the one who writes Day
d) Only I and the one who writes Ego, who faces the one
e) Only III who sits second to the right of T. The number of
persons sitting to the right of the one who writes
10. If the incentive of the grandfather of C is Class is one more than the number of persons
8000 less than C’s spouse and the sum of the sitting to the left of E, who sits third to the right of
incentive of G and D’s mother incentive is 16000, A. A doesn’t write Class. The one who faces A
then who among the following person's incentive sits second to the right of the one who writes
is more than the sum of the incentive of the D’s Nature. Only two persons sit between C and the
father and D’ mother? one who faces P. The one who writes Friend
a) D faces R. The one who writes Topic sits
b) A immediate right of Q. D neither writes Standard
c) C nor writes Friend.
d) Both a and b 11. Who among the following person writes
e) Both b and c Nature in the note?
a) The one who sits second to the right of R
Directions (11-15): Study the following b) A
information carefully and answer the given c) The one who faces the one who writes Hate
questions. d) C
Ten persons are sitting in two parallel rows e) The one who sits immediate left of E
containing five persons each in such a way that
there is an equal distance between adjacent 12. Who among the following person sits second
persons. In row 1: A, B, C, D and E are seated to the left of the one who writes Day?
and all of them are facing south. In row 2: P, Q, a) The one who writes Date
R, S and T are seated and all of them are facing b) S
north. Each person in row 1 faces another c) The one who writes Friend
person in row 2. Each of them writes different d) C
word in a note viz. Sorry, Topic, Hate, Ego, e) The one who sits immediate right of B
Nature, Friend, Class, Day, Standard and Date.
Only one person sits between the one who faces 13. As many persons sit between C and the one
the one who writes Date in a note and B, who who writes Ego as between __ and __.
doesn’t sit at the end. S sits immediate left of the a) The one who writes Date and Q
one who writes date. The one who faces S sits b) A and the one who writes Sorry

Click Here For Bundle PDF Course | support@guidely.in Page 3 of 12


SBI Clerk & RRB PO Mains PDF Course 2023
Reasoning Ability Day -3 (Eng)

c) The one who faces the one who writes Day O is 4m west of point I, which is the midpoint of
and E points A and Y.
d) Q and the one who writes Nature Statement II: Point B is 8m north of point Y. Point
e) The one who writes Standard and D T is 5m east of point Y. The distance between
points Y and T is half of the distance between
14. Which of the following combination is true? points V and W, which is south-west of point B.
a) Q-Nature a) If the data given in statement I alone is
b) A-Ego sufficient to answer the question.
c) S-Topic b) If the data given in statement II alone is
d) C-Standard sufficient to answer the question.
e) T-Hate c) If the data given in either statement I alone or
statement II alone is sufficient to answer the
15. If the position of the one who writes Class question.
and the one who writes Topic are interchanged, d) If the data given in both statement I and
then who among the following person faces the statement II together are necessary to answer
one who writes Class? the question.
a) Q e) If the data given in both statements I and II
b) P together are not sufficient to answer the
c) C question.
d) B
e) E 17. Eight persons – A, B, C, D, E, F, G and H are
sitting around a circular table facing the centre.
Directions (16-20): Each of the questions below Who among the person sits opposite to F?
consists of a question and three statements Statement I: B sits third to the right of A. One
numbered I and II given below it. You have to person sits between D and B. The one who faces
decide whether the data provided in the D sits immediate left of F. Two persons sit
statements are sufficient to answer the between F and H. E sits third to the left of H. E
questions. doesn’t sit adjacent to D.
16. What is the direction of point Y with respect Statement II: C sits immediate right of E. Two
to point W? persons sit between C and G. D sits third to the
Statement I: Point U is 9m north of point W and right of G. As many persons sit between E and G
point I is 18m south of point U. Point A is 5m as between D and A. Two persons sit between A
west of point L, which is 3m east of point O. Point and F.

Click Here For Bundle PDF Course | support@guidely.in Page 4 of 12


SBI Clerk & RRB PO Mains PDF Course 2023
Reasoning Ability Day -3 (Eng)

a) If the data given in statement I alone is d) If the data given in both statement I and
sufficient to answer the question. statement II together are necessary to answer
b) If the data given in statement II alone is the question.
sufficient to answer the question. e) If the data given in both statements I and II
c) If the data given in either statement I alone or together are not sufficient to answer the
statement II alone is sufficient to answer the question.
question.
d) If the data given in both statement I and 19. Seven persons – P, Q, R, S, T, U and V are
statement II together are necessary to answer reading books on seven different days from
the question. Monday to Sunday of the same week. Who
e) If the data given in both statements I and II reads the book immediately before P?
together are not sufficient to answer the Statement I: Only four persons read between S
question. and P. T reads two days after P. The number of
persons read before T is one less than the
18. A family consists of eight members with three number of persons read after V. U reads on one
generations. who among the following person is of the days after V.
the uncle of M? Statement II: U reads on one of the days after
Statement I: G is the mother of P, who is the only Friday. Only one person reads between U and T.
son of L. G has only two children. R is the wife of As many persons read between T and V as
P. T is the sister-in-law of R, who doesn’t have between Q and T. P reads immediately before V.
any siblings. M is the niece of T, who is an a) If the data given in statement I alone is
unmarried person in the family. sufficient to answer the question.
Statement II: R is the daughter-in-law of G, who b) If the data given in statement II alone is
is the grandmother of M. M is the son of U, who sufficient to answer the question.
doesn’t have any siblings. X is the brother-in-law c) If the data given in either statement I alone or
of U. G has only one son. X is not the brother of statement II alone is sufficient to answer the
R. question.
a) If the data given in statement I alone is d) If the data given in both statement I and
sufficient to answer the question. statement II together are necessary to answer
b) If the data given in statement II alone is the question.
sufficient to answer the question. e) If the data given in both statements I and II
c) If the data given in either statement I alone or together are not sufficient to answer the
statement II alone is sufficient to answer the question.
question.

Click Here For Bundle PDF Course | support@guidely.in Page 5 of 12


SBI Clerk & RRB PO Mains PDF Course 2023
Reasoning Ability Day -3 (Eng)

20. Eight persons – L, M, N, O, P, Q, R and S as below M, where no one lives to the west of M.
are living in a four storey building from 1 to 4 O lives immediately above M in different type of
from bottom to top respectively. Two flats are flats.
there on each floor viz. Flat P and Flat Q. Flat P a) If the data given in statement I alone is
is west of Flat Q. Only one person lives in each sufficient to answer the question.
flat. Who among the person lives immediately b) If the data given in statement II alone is
above the flat of L? sufficient to answer the question.
Statement I: O lives in flat Q of an odd numbered c) If the data given in either statement I alone or
floor. Only one floor is between O and L but both statement II alone is sufficient to answer the
of them are living in the same type of flat. As question.
many floors above L as below M. S lives d) If the data given in both statement I and
immediately below M but both of them are living statement II together are necessary to answer
in different type of flats. the question.
Statement II: N lives three floors above S but e) If the data given in both statements I and II
both are not living in the same type of flat. Only together are not sufficient to answer the
one floor is between S and L but both are living question.
in different type of flats. As many floors above L
Click Here to Get the Detailed Video Solution for the above given Questions
Or Scan the QR Code to Get the Detailed Video Solutions

Answer Key with Explanation


1. Answer: B 2. Answer: E

Click Here For Bundle PDF Course | support@guidely.in Page 6 of 12


SBI Clerk & RRB PO Mains PDF Course 2023
Reasoning Ability Day -3 (Eng)

3. Answer: D 8. Answer: A

9. Answer: E
From statement I, B=2 (35000+50000) -
>B=170000 ->170000-E=90000 ->E=80000,
which is higher than the one who gets the
highest incentive.
From statement II, 10000+F/2=7500 ->F=5000 -
4. Answer: C >E+5000=45000 ->E=40000, which is higher
than the one who gets the second highest
incentive.
From statement III, C+50000=64000 ->C=14000
->14000+E=36000 ->E=22000

10. Answer: D
A=35000 ->H=27000, 10000+F=16000 -
5. Answer: E >F=6000 ->27000+6000 ->33000, which is less
than the incentive of D and A.
We have,
 B is the only daughter of H and got more
incentive than G, who is the daughter-in-
law of F.
 F has only two children.
 C is the son of G and got less incentive
Directions (6-10): than his uncle.
6. Answer: D  E is not C’s father’s sibling.
 G has only one sibling.
7. Answer: C  A, who is the daughter-in-law of D and got
120/100*35000=42000 - more incentive than B.
>42000+B/2=3200042000+B=64000  C, who is the nephew of E and B and got
64000-42000=22000, so the incentive of F’s more incentive than G.
daughter=22000, then the required sum=  C did not get more incentive than B and
50000+22000=72000 he is a married person.

Click Here For Bundle PDF Course | support@guidely.in Page 7 of 12


SBI Clerk & RRB PO Mains PDF Course 2023
Reasoning Ability Day -3 (Eng)

Directions (11-15):
Again, we have
11. Answer: C
 The one who is E’s sister’s husband got
12. Answer: E
the highest incentive.
13. Answer: B
 The one who is the mother of B got less
14. Answer: A
incentive than her daughter-in-law.
15. Answer: D
 The one who got the second highest
Final arrangement
incentive is Rs.35000.
 H got more incentive than his daughter-in-
law and E.
 H neither got 35000 nor got more than B.
 The one who got the second lowest
incentive is Rs.10000, which is one-fifth of
We have,
the one who got the highest incentive.
 Only one person sits between the one
who faces the one who writes Date in a
note and B, who doesn’t sit at the end.
 S sits immediate left of the one who writes
date.
From the above conditions, there are three
possibilities

Click Here For Bundle PDF Course | support@guidely.in Page 8 of 12


SBI Clerk & RRB PO Mains PDF Course 2023
Reasoning Ability Day -3 (Eng)

Again, we have
 The one who faces S sits immediate right
Again, we have
of the one who writes sorry.
 The one who faces A sits second to the
 Only two persons sit between the one
right of the one who writes Nature.
who writes Day and the one who writes
 Only two persons sit between C and the
Ego, who faces the one who sits second
one who faces P.
to the right of T.
 The one who writes Friend faces R.
 The number of persons sitting to the right
 The one who writes Topic sits immediate
of the one who writes Class is one more
right of Q.
than the number of persons sitting to the
 D neither writes Standard nor writes
left of E, who sits third to the right of A.
Friend.
 A doesn’t write Class.
After applying the above conditions case-2 gets
After applying the above conditions case-1 gets
eliminated because there is no possibility to
eliminated because there is no possibility to
place the one who writes Topic, hence case-3
place the one who writes Day.
shows the final arrangement.

Click Here For Bundle PDF Course | support@guidely.in Page 9 of 12


SBI Clerk & RRB PO Mains PDF Course 2023
Reasoning Ability Day -3 (Eng)

From statement II, we cannot determine the


direction of point Y.
16. Answer: A
Hence, statement I is sufficient to answer the
From statement I:
question.

17. Answer: E
From statement I:
B sits third to the right of A. One person sits
between D and B. The one who faces D sits
immediate left of F. Two persons sit between F
and H. E sits third to the left of H. E doesn’t sit
adjacent to D.

From statement I, clearly shows that we can


determine point Y is south-east of point W.
From statement II:

From statement I, Clearly shows that we cannot


determined who is facing F.
From statement II:
C sits immediate right of E. Two persons sit
between C and G. D sits third to the right of G.
As many persons sit between E and G as

Click Here For Bundle PDF Course | support@guidely.in Page 10 of 12


SBI Clerk & RRB PO Mains PDF Course 2023
Reasoning Ability Day -3 (Eng)

between D and A. Two persons sit between A 19. Answer: D


and F. From statement I:

From statement II, Clearly shows that we cannot


determine who is facing F.
Hence, Statement I and II is not sufficient to
answer the question.
From statement I, Clearly shows that we cannot
find the person who reads immediately before P.
18. Answer: B
From statement II:
From statement I:

From statement I, Clearly shows that we cannot


find the uncle of M.
From statement II:

From statement II, Clearly shows that we cannot


find the person who reads immediately before P.
Combining statement I and II:

From statement II, Clearly shows that we can


find X is the uncle of M.
Hence, statement II alone is sufficient to answer
the question.

Click Here For Bundle PDF Course | support@guidely.in Page 11 of 12


SBI Clerk & RRB PO Mains PDF Course 2023
Reasoning Ability Day -3 (Eng)

From statement I, we cannot find who lives


immediately above the flat of L.
From statement II:

After combining statements I and II, we clearly


find that R reads immediately before P.
Hence, both statement I and II is sufficient to
answer the question.
From statement II, clearly we can find N lives
immediately above L’s flat
20. Answer: B
Hence, the data given in statement II alone is
From statement I:
sufficient to answer the question.

Click Here For Bundle PDF Course | support@guidely.in Page 12 of 12


SBI Clerk & RRB PO Mains PDF Course 2023
Quantitative Aptitude Day -3 (Eng)

Quantitative Aptitude

Directions [01 – 04]: Read the following e) None of these


information carefully and answer the questions
based on it. 2) Find difference between gold coins and
There are three types of coins – Gold, silver, and copper coins in bag R?
copper in each of the three bags – P, Q, and R. a) 3
• Number of copper coins in bag Q is 37.5% b) 11
more than that of bag P. c) 13
• Number of silver coins in bag R (Prime number) d) 16
is more than that of bag P, while gold coins in e) None of these
bag Q is half as that of bag P.
• Difference between total coins in bag Q and 3) Cost of each copper coin is Rs. 3000 which is
bag R is 50% more than difference total coins in same as that of silver coin, which is 600 more
bag P and bag Q. than that of each gold coin. Find total cost of
• Silver coins in bag Q is 14 less than that of coins in bag P?
copper coins in same bag, while total coins in all a) Rs. 1.02 lakhs
three bags together is 130. b) Rs. 1.38 lakhs
• Gold coins in bag R is 25% more than copper c) Rs. 1.09 lakhs
coins in bag P, while total coins in bag R is 25Z d) Rs. 1.29 Lakhs
(Z is an integer). e) None of these
• Number of coins in bag Q is less than other two
bags, while gold coins in bag Q is less than any 4) Silver coins in bag P and bag Q together is
type of coins in any bag. what % of total coins in bag R?
1) Find total number of silver coins in all three a) 30%
bags together? b) 54%
a) 50 c) 58%
b) 45 d) 64%
c) 35 e) None of these
d) 40

Directions [05 – 07]: Read the following information carefully and answer the questions based on it.

Click Here For Bundle PDF Course | support@guidely.in Page 1 of 12


SBI Clerk & RRB PO Mains PDF Course 2023
Quantitative Aptitude Day -3 (Eng)

The chart given below shows the partial information about the number of employees in five department –
P, Q, R, S, and T of a company and % by which male employees is more than that of female employees
in a particular department.

Note: a) Number of female employees in R is 5Y which is 35 more than number of female employees in
Q. Total number of employees in R, S and T together is 635.
b) All the given values in graph are multiple of 5, and male employees in R is 50% more than that of
female employees in P.
5) Find difference between total number of male c) 50%
and female employees in all companies d) 48%
together? e) 54%
a) 175
b) 195 7) A = Male employees in Q
c) 185 B = Female employees in P
d) 205 Find which of the following option is true?
e) None of these a) A > B
b) A = B
6) The number of female employees in R and S c) A < B
together is approx. what % more than the d) A + B > 180
number of male employees in company T? e) None of these
a) 46%
b) 45%

Click Here For Bundle PDF Course | support@guidely.in Page 2 of 12


SBI Clerk & RRB PO Mains PDF Course 2023
Quantitative Aptitude Day -3 (Eng)

Directions [08 – 10]: In each of the following Statement II: When half of the mixture is
question, two statements I and II given. You removed from the vessel and replaced by (3Z +
must decide whether data provided in the 30) ml of milk and Y ml of water, then the amount
statements is sufficient to answer the question or of milk in the final mixture is twice as that of
not. water.
8) The radius of the circle is (3Z + 1) cm and the a) Statement I alone is sufficient to answer the
side of the square is Z cm. Find the area of the question.
circle. b) Statement II alone is sufficient to answer the
Statement I: The length and breadth of the question.
rectangle is L cm and M cm (both are single digit c) Either Statement I alone or II alone is sufficient
composite numbers) respectively. The side of the to answer the question
square and breadth of the rectangle are d) Both statements together is sufficient to
consecutive integers, where the breadth of the answer the question.
rectangle is more than the side of square. e) None of these
Statement II: The perimeter of the rectangle is
twice as that of square. 10) Find the speed of boat P in still water is how
a) Statement I alone is sufficient to answer the much % more or less than that of the boat Q.
question. The speed of the stream for both boats is same,
b) Statement II alone is sufficient to answer the and A = B.
question. Statement I: The time taken by boat Q to travel
c) Either Statement I alone or II alone is sufficient (A + B) km downstream is the same as the time
to answer the question taken by boat P to travel 2B km with the speed of
d) Both statements together is sufficient to the boat in still water.
answer the question. Statement II: The time taken by boat P to travel
e) None of these 90 km downstream is twice the time taken by
boat Q to travel 18 km upstream.
9) A vessel contains 30Z ml mixture of milk and a) Statement I alone is sufficient to answer the
water in ratio of 3:2. Find the value of Y (> Z). question.
Statement I: When one third of the mixture is b) Statement II alone is sufficient to answer the
removed from the vessel and replaced by 4Z ml question.
of milk and (6.5Z + Y) ml of water, then the c) Either Statement I alone or II alone is sufficient
amount of milk and water in the final mixture to answer the question
becomes equal. The difference between the d) Both statements together is sufficient to
value of Z and Y is 5 answer the question.

Click Here For Bundle PDF Course | support@guidely.in Page 3 of 12


SBI Clerk & RRB PO Mains PDF Course 2023
Quantitative Aptitude Day -3 (Eng)

e) None of these more than that of train P. Find the sum of the
lengths of two trains.
11) The number of boys in school P is twice as a) 540 meters
that of Q, while the number of girls in school Q is b) 640 meters
20 less than that of P. If the difference between c) 580 meters
the total number of students in P and Q is same d) 620 meters
as the that of the total number of students in Q, e) None of these
then find the total number of girls in both the
schools. 14) The cost price of article P is Rs. M and it is
a) 60 sold after a discount of 10% thus earning a profit
b) 80 of Z%. The cost price of article Q is Rs. N sold
c) 90 after a discount of 15% thus earning a profit of
d) 40 (2Z + 30)%. The marked price of both articles is
e) None of these same and the cost price of P is 50% more than
that of Q. Find the selling price of another article
12) The base area of a cone is Y cm2 and its if its cost price is Rs. 1200 and sold at (2Z +
volume is (Y + 192) cm2. The height of the 10)% profit.
cylinder is (Z – 1) cm and its radius is (M + 4) cm. a) Rs. 1600
Find the volume of cylinder, if the radius of the b) Rs. 1800
cone is M cm and the height of the cone is Z cm. c) Rs. 1500
The base area of the cone is numerically equal d) Rs. 1680
to 24 times its radius. Use π = 3. e) None of these
a) 960π cm3
b) 1080π cm3 15) Arun invested Rs. Z in scheme P for two
c) 1440π cm3 years at (Y + 15)% per annum compounded
d)720π cm3 annually, while he invested Rs. M in scheme Q
e) None of these for three years at 25% per annum compounded
annually. The amounts received by Arun from
13) The length of train P is Z meters running at a scheme P and Q after respective time period is
speed of Y m/s, and it crosses a platform which in the ratio of 72:25. Find simple interest earned
is 25% less than that of its length, in 14 seconds. by Arun on Rs. 40000 for three years at (Y – 3)%
Train P also overtakes train Q which is running in per annum, if money invested in scheme P is 2.5
the same direction at a speed of 90 km/h in 108 time as that of Q.
seconds and the length of train Q is 60 meters a) Rs. 32400

Click Here For Bundle PDF Course | support@guidely.in Page 4 of 12


SBI Clerk & RRB PO Mains PDF Course 2023
Quantitative Aptitude Day -3 (Eng)

b) Rs. 45600 not elder than S but elder than R. The present
c) Rs. 38400 age of P is 37.5% more than that of Q. All
d) Rs. 57600 persons have their birthday today. Find sum of
e) None of these all possible values of M.
a) 54
16) The present age of P is Z years and that of Q b) 51
is Y years. P is (M + 4) years elder than Q and c) 63
the age of S after 4 years is 68 years. The ratio d) 57
of the present ages of R and S is 23:32, and Q is e) None of these

Directions [17 – 20]: Read the following information carefully and answer the questions based on it.
The chart given below shows the % distribution of number of total vehicles (Cars + Trucks) manufactured
by five companies – P, Q, R, S, and T, and % distribution or numbers of trucks manufactured by these
five companies in 2020.

Click Here For Bundle PDF Course | support@guidely.in Page 5 of 12


SBI Clerk & RRB PO Mains PDF Course 2023
Quantitative Aptitude Day -3 (Eng)

Note: ‘a’ is a prime number < 20, and the total number of trucks manufactured by T is less than any other
company.
17) Find the average number of cars number of cars manufactured by R in 2021 is
manufactured by Q and T together? 25% less than that in 2020. Find the total number
a) 105 of cars manufactured by Q and R together in
b) 125 2021?
c) 115 a) 219
d) 95 b) 249
e) None of these c) 199
d) 247
18) The number of cars manufactured by S is (Z e) None of these
– 5) % more or less than the number of trucks
manufactured by P. Find the value of Z? 20) For company U, the number of cars
a) 45 manufactured in 2020 is 22 more than that of S
b) 40 and the ratio of the number of trucks and cars
c) 55 manufactured by company U is 5:4. Find the total
d) 50 number of vehicles (Cars + Trucks)
e) None of these manufactured by U is how many times as that of
company R?
19) The number of cars manufactured by Q in a) 1.2 times
2021 is 50% more than that in 2020, while b) 1.8 times

Click Here For Bundle PDF Course | support@guidely.in Page 6 of 12


SBI Clerk & RRB PO Mains PDF Course 2023
Quantitative Aptitude Day -3 (Eng)

c) 2.5 times e) None of these


d) 1.25 times
Click Here to Get the Detailed Video Solution for the above given Questions
Or Scan the QR Code to Get the Detailed Video Solutions

Answer Key with Explanation


Directions [01 – 04]: Total coins in bag R = 50
Let total coins in bag P and Q = a and b Total coins in bag Q = 130 – 45 – 50 = 35
respectively Copper coins in bag Q is 37.5% more than that
Now, of copper coins in bag P.
(a – b) x 3/2 = (25Z – b) So, ratio of copper coins in bag Q and bag P =
3a – 3b = 50Z – 2b 11:8 (11K, 8K)
So, b = 3a – 50Z………………... (1) Silver coins in bag Q = 11K – 14
Also, That means copper coins in bag Q > 11
(a + b + 25Z) = 130……………... (2) So, copper coins in bag Q = 22 or 33
From (1) and (2), we get But if copper coins in Q = 33
a + 3a – 50Z + 25Z = 130 So, silver coins in bag Q = 33 – 14 = 19
4a – 25Z = 130 But total coins in bag Q = 35
Value of a = (130 + 25Z)/4 So, that means copper coins in bag Q = 22
(130 + 25Z) must be even and exactly divisible So, silver coins in bag Q = 22 – 14 = 8
by 4. So 25Z must be even. So, gold coins in bag Q = 35 – 22 – 8 = 5
So, total coins in bag R = 50 or 100 (Z = 2 or 4) Copper coins in bag P = 8 x 2 = 16
If Z = 2, value of a = (130 + 50)/4 = 45 Gold coins in bag P = 2 x 5 = 10
If Z = 4, value of a = (130 + 100)/4 = 57.5 So, silver coins in bag P = 45 – 16 - 10 = 19
So, possible value of a = 45 and Z = 2 Gold coins in bag R = 16 x 5/4 = 20
So, total coins in bag P = 45 Silver coins in bag R is prime number > 19

Click Here For Bundle PDF Course | support@guidely.in Page 7 of 12


SBI Clerk & RRB PO Mains PDF Course 2023
Quantitative Aptitude Day -3 (Eng)

(silver + copper) coins in bag R = 50 – 20 = 30 Female employees in P = 180 x 4/9 = 80


Silver coins in bag R = 23 or 29 So, male employees in R = 80 x 1.5 = 120
But also, in each bag number of any type of coin Ratio of male and female employees in S = 7:5
>5 [7a, 5a]
That means silver coins in bag R = 23 Now,
Copper coins in bag R = 30 – 23 = 7 200 + 7a + 5a + 120 + 5Y = 635
12a + 5Y = 315
Y = 63 – 12a/5
Male employees in R = 120
So, female employees in R < 120
5Y < 120
1) Answer: A
Y < 24
According to question,
Value of Y obtained on subtracting multiple of 12
Total silver coins in all bags together = 50
from 63.
Hence answer is option A
So, 63 – 12 * 20/5= 15
So, when a = 20, value of Y = 15
2) Answer: C
When a = 25, value of Y = 3
Gold coins in bag R = 20
But also, female employees in Q = 5Y – 35
Copper coins in bag R = 7
That means only value of Y = 15 and a = 20
Required difference = 20 – 7 = 13
So female employees in R = 5 x 15 = 75
Hence answer is option C
Total employees in R = 120 + 75 = 195
So, total employees in S = 315 – 75 = 240
3) Answer: D
Male employees in S = 7 x 20 = 140
Total cost of coins in bag P = 10 x 2400 + (19 +
Female employees in S = 5 x 20 = 100
16) x 3000 = Rs. 1.29 lakhs
Male employees in Q = 2 x 40 = 80
Hence answer is option D
Total employees in Q = 80 + 40 = 120

4) Answer: B
Silver coins in bag (P + Q) = 19 + 8 = 27
Total coins in bag R = 50
Required % = 27/50 x 100 = 54%
Hence answer is option B 5) Answer: C
According to question,
Directions [05 – 07]:

Click Here For Bundle PDF Course | support@guidely.in Page 8 of 12


SBI Clerk & RRB PO Mains PDF Course 2023
Quantitative Aptitude Day -3 (Eng)

Total male employees in all companies together If Z = 3, L = 8 and M = 4 (possible)


= 560 If we further put values of Z, we get L as double
Total female employees in all companies digit.
together = 375 So, value of Z = 3
Required difference = 560 – 375 = 185 Radius of the circle = 3 x 3 + 1 = 10 cm
Hence answer is option C Required area = π x 10 x 10 = 100π cm2
So, both statements I and II together is sufficient
6) Answer: A to answer the question.
Female employees in (R + S) = 75 + 100 = 175 Hence answer is option D
Male employees in T = 120
Required % change = (175 - 120)/120 x 100 = 9) Answer: C
46% Amount of mixture initially = 30Z ml
Hence answer is option A Amount of milk initially = 60% x 30Z = 18Z ml
Amount of water initially = 30Z – 18Z = 12Z ml
7) Answer: B From I:
Value of A = 80 (2/3 x 18Z + 4Z) = 2/3 x 12Z + 6.5Z + Y
Value of B = 80 16Z = 14.5Z + Y
So, A = B 1.5Z = Y
Hence answer is option B Z/Y = 2/3
So, value of Y = [3/ (3 – 2)] x 5 = 15
8) Answer: D This statement alone is sufficient to answer the
It is clearly observed that none of the single question.
statement is sufficient to answer the question. From II:
From I. (1/2 x 18Z + 3Z + 30) = 2 x (1/2 x 12Z + Y)
M=Z+1 12Z + 30 = 12Z + 2Y
From II. Y = 15
4Z x 2 = 2 x (L + M) This statement alone is sufficient to answer the
4Z = L + M question.
From I and II, we get Hence answer is option C
4Z = L + Z + 1
3Z = L + 1 10) Answer: D
If Z = 1, L = 2 (not possible) A=B
If Z = 2, L = 5 (not possible)

Click Here For Bundle PDF Course | support@guidely.in Page 9 of 12


SBI Clerk & RRB PO Mains PDF Course 2023
Quantitative Aptitude Day -3 (Eng)

Let speed of boat in still water of boat P and Q = So, total number of girls in P and Q together =
P and Q km/h 20 + 20 + 20 = 60
Let speed of stream = Y km/h Hence answer is option A
From I.
(A + A)/ (Q + Y) = 2A/P 12) Answer: D
P–Q=Y Y = 24M
This statement alone is not sufficient to answer Now
the question. Π x M2 = Y
From II. 3 x (Y/24)2 = Y
90 / (P + Y) = 2 x 18 / (Q – Y) Y = 192 …………. (1)
5Q – 2P = 7Y Also,
This statement alone is not sufficient to answer 1/3 x (π x M2) x Z = (Y + 192) …………... (2)
the question. Put equation (1) in equation (2), we get
From (1) and (2), we get Y/3 x Z = 2Y
5Q – 2P = 7 x (P – Q) Z = 6 cm
9P = 12Q M = 192/24 = 8 cm
P/Q = 4/3 Radius of cylinder = 8 + 4 = 12 cm
So, required % change = (4 – 3)/3 x 100 = Height of the cylinder = 6 – 1 = 5 cm
33.33% Volume of cylinder = π x 12 x 12 x 5 = 720π cm3
So, both statements together is sufficient to Hence answer is option D
answer the question,
Hence answer is option D 13) Answer: A
Ratio of length of train P and platform = 4:3 =
11) Answer: A [4a, 3a]
Let number of boys in Q = Z Now,
So, number of boys in P = 2 x Z 4a + 3a = 14 x Y
Let number of girls in Q = Y So, a = 2Y……………... (1)
So, number of girls in P = (Y + 20) Length of train Q = 4a + 60
Now, Now,
Now, 4a + 4a + 60 = (Y – 90 x 5/18) x 108
2Z + Y + 20 – (Z + Y) = (Z + Y) 2a + 15 = 27Y– 675
Z + 20 = Z + Y 27Y = 2a + 690……………... (2)
So, value of Y = 20 From (1) and (2), we get

Click Here For Bundle PDF Course | support@guidely.in Page 10 of 12


SBI Clerk & RRB PO Mains PDF Course 2023
Quantitative Aptitude Day -3 (Eng)

27Y = 2 x 2Y + 690 So, the present age of R = 23/32 x 64 = 46


23Y = 690 Ratio of present age of P and Q = 11:8
Value of Y = 30 Age of Q is > 46 and ≤ 64
So, value of a = 30 x 2 = 60 Possible age of Q = 48, 56 and 64
Required sum = 8 x 60 + 60 = 540 meters Possible age of P = 66, 77, and 88
Hence answer is option A So, possible values of M = 14, 17, 20
Required sum = 14 + 17 + 20 = 51
14) Answer: B Hence answer is option B
Ratio of cost price of P and Q = 3:2
Marked price of P and Q is same, they sold at a Directions [17 – 20]:
discount of 10% and 15% respectively. From second pie chart
So, ratio of the selling price of P and Q = 64 + a% = 100% - 25% - 20% - 10%
90%:85% = 18:17 64 = (45 – a) %
Now, Number of trucks manufactured by T is least,
[3 x (100 + Z)]% / [2 x (100 + 2Z + 30)%] = 18/17 and ‘a’ is prime number less than 20.
1700 + 17Z = 1560 + 24Z Possible values of a = 11, 13, 17, 19
7Z = 140 On putting a = 13, we get integral value.
Value of Z = 20% So,
Required selling price = 1200 x 1.5 = Rs. 1800 (45 – 13) % = 64
Hence answer is option B So, total trucks manufactured = 64/32 x 100 =
200
15) Answer: C Now we can find all the related data.
According to question,
5/2 x [1 + (Y + 15)/100]2 = 1 x (5/4)3 x 72/25
1 + (Y + 15)/100 = 3/2
Y + 15 = 50
Y = 35
17) Answer: C
Required simple interest = 40000 x 32% x 3 =
According to question,
Rs. 38400
Number of cars manufactured by Q = 118
Hence answer is option C
Number of cars manufactured by Q = 112
Required average = (118 + 112)/2 = 115
16) Answer: B
Hence answer is option C
Present age of S = 68 – 4 = 64 years

Click Here For Bundle PDF Course | support@guidely.in Page 11 of 12


SBI Clerk & RRB PO Mains PDF Course 2023
Quantitative Aptitude Day -3 (Eng)

18) Answer: D Hence answer is option A


Cars manufactured by S = 58
Trucks manufactured by P = 40 20) Answer: E
Required % change = (58 – 40)/40 x 100 = 45% Number of cars manufactured by U = 58 + 22 =
So, value of Z = 45 + 5 = 50% 80
Hence answer is option D Number of trucks manufactured by U = 5/4 x 80
= 100
19) Answer: A Total number of vehicles manufactured by U =
Cars manufactured by Q in 2021 = 118 x 1.5 = 80 + 100 = 180
177 Total vehicles manufactured by R = 120
Cars manufactured by R in 2021 = 56 x 75% = Required ratio = 180/120 = 1.5 times
42 Hence answer is option E
Required sum = 177 + 42 = 219

Click Here For Bundle PDF Course | support@guidely.in Page 12 of 12


SBI Clerk & RRB PO Mains PDF Course 2023
ENGLISH Day -3 (Eng)

English Language

Directions (1-5): Given below are a few his arch rival opponents in the government and
questions based on the paragraph with an the military establishment.
omitted sentence. You have to choose the E. None of the above
sentence from the options to fill in the blank
given. If none of the options are correct then 2. In April, Marin's Social Democrats fell to third
choose option E as your answer. place with 43 seats in the 200-member
1. Imran Khan has been offensive in his parliament, behind the conservative National
statements and stance against the new military Coalition with 48 seats and the anti-immigration
establishment, levelling grave accusations of Finns Party with 46. ___________. The now 37-
conspiracy, masterminding assassination year-old Marin became the world's youngest
attempts on his life and teaming up with the elected head of government in 2019, running a
current coalition government to deny Imran Khan centre-left coalition with five women party
and his political movement under PTI the right to leaders, four under the age of 35.
freedom of expression. According to reliable A. The National Coalition is currently negotiating
sources, going against the army, has mounted to form a government with the Republican Party.
trouble for him and this is the reason why his B. The International Coalition is currently
days are numbered. __________________. negotiating to form a party with Finns Party.
A. His political past, public glare and his party’s C. The National Coalition is currently negotiating
political persistence is now going to be decided to form a government with the Finns Party.
by his arch rival opponents in the government D. The National Coalition is currently negotiating
and the military establishment. to form a government with the Democrats Party.
B. His social future, public glare and his party’s E. None of the above
social existence is now going to be decided by
his arch rival opponents in the government and 3. Brother of JeM chief Masood Azhar, Abdul
the military establishment. Rauf, born in 1974 in Pakistan, has been
C. His economical future, public glare and his involved in planning and executing numerous
party’s economical existence is now going to be terror strikes in India including the hijacking of
decided by his arch rival opponents in the Indian Airlines aircraft IC814 in 1999, the attack
government and the military establishment. on the Parliament in 2001 and the targeting of
D. His political future, public glare and his party’s the IAF base in Pathankot in 2016. It is learned
political existence is now going to be decided by that China objected to the proposal from India to
add Abdul Rauf of the JeM to the UN Security

Click Here For Bundle PDF Course | support@guidely.in Page 1 of 8


SBI Clerk & RRB PO Mains PDF Course 2023
ENGLISH Day -3 (Eng)

Council’s 1267 ISIL and Al Qaida Sanctions list. embraced the chador, a cloak that covers the
________________. body from head to toe, except for the face.
A. Tamim was sanctioned by the US in Images of armed women encompassed in black
December 2010. cloth became a familiar sight for Americans
B. Osama Bin laden was sanctioned by the US in during the U.S. _______________________. But
December 2010. other women protested a decision by Grand
C. Martha was sanctioned by the US in Ayatollah Ruhollah Khomeini ordering the hijab
December 2010. to be worn in public. In 1983, it became the law,
D. Rauf Azhar was sanctioned by the US in enforced with penalties including fines and two
December 2010. months in prison.
E. None of the above A. Refugees were given shelter that year.
B. No embassy takeover and hostage crisis
4. Jemima Goldsmith, a British screenwriter, happened later that year.
television, film and documentary producer, was C. There was widespread happiness on the
Imran Khan's first wife. The couple married in decision mentioned above.
1995 and divorced in 2004. They have two sons D. Embassy takeover and hostage crisis later
— Sulaiman Isa (born in 1996) and Kasim (born that year.
in 1999). ____________________. However, it has E. None of the above
been widely reported that the couple faced
challenges due to the pressures of Imran Khan's Directions (6-10): Given below are a few
political career and the cultural differences questions with a blank in each question to be
between the couple. filled from the options given. You have to fill in
A. The exact reasons for their divorce have not the blanks with appropriate options in order to
been extensively disclosed. make the sentence contextually correct. If none
B. The exact reasons for their love have not been of the options are correct then choose option E
extensively disclosed. as your answer.
C. The exact reasons for their incarnation have 6. While Prime Minister Modi has travelled to the
not been extensively disclosed. US numerous times after ___________ office in
D. The exact reasons for their fallout have not 2014, this would be the first ‘State Visit’ - the
been extensively disclosed. highest level of diplomatic reception which
E. None of the above involves a state dinner and official public
ceremonies.
5. By the 1979 Islamic Revolution, some of the A. take-off
women who helped overthrow the shah B. resuming

Click Here For Bundle PDF Course | support@guidely.in Page 2 of 8


SBI Clerk & RRB PO Mains PDF Course 2023
ENGLISH Day -3 (Eng)

C. assuming through Mexico or other countries en route to the


D. presuming US or failed to use other US legal pathways.
E. None of the above A. sweeping
B. reaping
7. As the sun beat down on a Turkish tent camp C. creeping
for earthquake survivors, Bahattin Kar emerged D. deepening
from his makeshift shelter to complain about the E. None of the above
lack of electricity and water, the unfair
distribution of aid and the surging cost of living. 10. Though the confrontation and Imran Khan's
But the 54-year-old remains __________ loyal to ouster jolted Pakistan's political rhythm at the
President Tayyip Erdogan ahead of national time, the country with a populace of over 238
elections that are shaping up to be the toughest million has a history of interventions by its
of the Turkish leader's two-decade rule. powerful military and __________ of the top
A. precariously executive offices.
B. turbidly A. incorporation
C. steadfastly B. categorization
D. stunningly C. incarnation
E. None of the above D. incarceration
E. None of the above
8. While Biden has kept Title 42 in place until
now, not all migrants caught crossing the border Directions (11-15):Given below are a few
illegally have been expelled, with more than half questions with two sentences each containing a
in recent months allowed into the US to few words to be swapped between the
__________ their immigration cases. sentences in order to make the sentence
A. staunch contextually correct. If no swapping is required
B. paddle then choose no swapping as your answer.
C. issue 11.
D. pursue a. The fifth prime minister of Pakistan who held
E. None of the above office for over 13 months, Suhrawardy had
refused to (a) endorse General Ayub Khan's (b)
9. To discourage illegal crossings, the Biden seizure of the government.
administration plans to issue a __________ new b. He was banned from the country's politics and
regulation this week that would deny asylum to was held liable for (c) breaching Pakistan's
most migrants caught crossing if they passed Elective Bodies Disqualification Order (EBDO). In

Click Here For Bundle PDF Course | support@guidely.in Page 3 of 8


SBI Clerk & RRB PO Mains PDF Course 2023
ENGLISH Day -3 (Eng)

the ordinance, EBDO, Ayub Khan, who was the accepting kickbacks from an international entity
first military president of the country, (d) to (d) conceal customs fraud.
disqualified over 70 elected representatives of A. a-c
the Assembly. B. a-d
A. a-c C. b-c
B. a-d D. b-d
C. b-c E. No swapping required
D. b-d
E. No swapping required 14.
a. After being (a) cast to exile by General Pervez
12. Musharraf in 1999, Nawaz Sharif (b) arrested to
a. Benazir Bhutto (a) funeral as Pakistani prime Pakistan.
minister for two (b) terms, between December b. As per reports, he was (c) returned within
1998 and August 1990 and October 1993 and hours of his return and was sent to Jeddah,
November 1996. Saudi Arabia to complete his remaining years of
b. She was a leader of the opposition under Ziaul his decade-long (d) exile.
Haq's (c) dictatorship in Pakistan. She was A. a-c
placed under house arrest for 90 days in 1985 B. a-d
when she arrived in Pakistan to attend her C. b-c
brother's (d) served. D. b-d
A. a-c E. No swapping required
B. a-d
C. b-c 15.
D. b-d a. Emirates Nuclear Energy Corporation (ENEC),
E. No swapping required the body (a) responsible for developing the
United Arab Emirates' nuclear energy sector, has
13. signed three agreements with Chinese nuclear
a. In August 1986, Bhutto was arrested for (a) energy organisations as it looks to (b) boost low-
disqualified the government at a public (b) rally carbon nuclear power.
on Pakistan's Independence Day. b. The UAE, which is (c) hosting the COP28
b. In April 1999, Benazir Bhutto was sentenced climate summit this year and wants to get 6% of
to five years of jail term and was (c) condemning its energy needs from nuclear as part of its 2050
from holding public office on the charges of net zero plan, has previously said China would
be a key partner in its energy (d) transition plan.

Click Here For Bundle PDF Course | support@guidely.in Page 4 of 8


SBI Clerk & RRB PO Mains PDF Course 2023
ENGLISH Day -3 (Eng)

A. a-c as supremo of the Shiv Sena after crisis and


B. a-d dissidence within the party.
C. b-c
D. b-d 16. Which of the following sentence will be last
E. No swapping required after the rearrangement?
A. b
Directions (16-20): Given below are a few B. a
questions based on the jumbled sentence to be C. d
rearranged in order to make a contextually D. c
correct sentence. There is a sentence with a E. None of the above
blank to be filled in order to make it contextually
correct and a sentence that itself is jumbled to be 17. Which of the following sentence will be
rearranged to make it contextually correct. If second after the rearrangement?
none of the options are correct then choose A. c
option E as your answer. B. d
a. When Nationalist Congress Party (NCP) chief C. b
Sharad Pawar, on May 2, burst a bombshell by D. a
announcing his intent to step down as the party’s E. None of the above
national president, it was seen by many as a
move to stomp out any rebellion and strengthen 18. Which of the following sentence will be
his command over the party. second last after the rearrangement?
b. Thackeray emerged stronger from these A. a
episodes. It remains to be seen whether things B. b
will work out similarly for Pawar. C. c
c. Three days on (a)/, after protests and mass D. d
(b)/ resignations by NCP (c)/ Pawar withdrew his E. None of the above
intent to resign (d)/ functionaries and workers (e).
d. This shock therapy and Thackeray’s 19. Which of the following word fits best in the
charismatic personality and pull over the party blank given in sentence e?
faithful saw him prevail upon frenzied cadre and A. designation
stay on at the helm. B. resignation
e. The ________ drama in the NCP revives C. supremacy
memories of three instances when Pawar’s friend D. leniency
and political foe Bal Thackeray similarly resigned E. None of the above

Click Here For Bundle PDF Course | support@guidely.in Page 5 of 8


SBI Clerk & RRB PO Mains PDF Course 2023
ENGLISH Day -3 (Eng)

20. Which of the following is the correct C. ACBED


rearrangement of sentence c? D. ABCED
A. ACBDE E. None of the above
B. ACDBE
Click Here to Get the Detailed Video Solution for the above given Questions
Or Scan the QR Code to Get the Detailed Video Solutions

Answer Key with Explanation


1. Answer: D context of the sentence. So, option D is the
Here, the political future of Imran Khan is being correct answer.
talked and option D is giving information about
the political future and existence of his party 4. Answer: A
whereas all other options are giving information Here, the context of the sentence is about one of
about social, personal and economical future the wives of the Imran Khan and their divorce
which is contextless. So, option D is the correct and option A is the only option giving information
answer. about the same. So, option A is the correct
answer.
2. Answer: B
Here, the context of the paragraph is about 5. Answer: D
negotiation by National Coalition to form a party Here, some situation of chaos is presented in the
with Finns Party and same is given in option B. starting lines and here from the options the
So, option B is the correct answer. takeover of embassy and hostages matches the
context. So, option D is the correct answer.
3. Answer: D
Here, Azhar brothers is being talked and from 6. Answer: C
the options Rauf Azhar is only correct as per the

Click Here For Bundle PDF Course | support@guidely.in Page 6 of 8


SBI Clerk & RRB PO Mains PDF Course 2023
ENGLISH Day -3 (Eng)

Here, the assuming fits best in the blank given in from the given options. So, option A is the
the above sentence as assuming office means to correct answer.
take upon oneself or to take on or over a position Creeping means the style of reptile’s walk.
and here the context of office is given. So, option Reaping means to gather or to take.
C is the correct answer.
10. Answer: D
7. Answer: C Here, incarceration fits best in the blank given as
Here, steadfastly fits best in the blank as it it means imprisonment and here as per the
means faithful and loyal and here loyalty of the context of the sentence which is about the
mentioned person with President is being talked condition of top executives in Pakistan and how
so it is apt to use steadfastly. So, option C is the they are imprisoned and here it fits best from the
correct answer. given options. So, option D is the correct
Precariously means dangerously. answer.
Turbidly means confusingly.
Stunningly means in an extreme attractive way. 11. Answer: E
Here, no swapping is required as the highlighted
8. Answer: D words are appropriately used. So, option E is the
Here, pursue fits best in the blank as it means to correct answer.
try to achieve something or to continue to do
something over a period of time and here prior is 12. Answer: A
apt as the context of the sentence is about Here, funeral and served need swapping as
migrants crossing the border. So, option D is the brother’s funeral is the correct phrase as well as
correct answer. served as. So, option A is the correct answer.
Staunch means very loyal and committed to So, option A is the correct answer.
attitude.
Paddle means a kind of mechanical force 13. Answer: A
producing object. Here, disqualified and condemning need
swapping in order to make contextually correct
9. Answer: A sentences. So, option A is the correct answer.
Here, sweeping fits best in the blank as it means
having a great and important effect and here 14. Answer: C
new regulations are being talked and it fits best Here, returned and arrested need swapping as
returned to Pakistan is the correct phrase as well

Click Here For Bundle PDF Course | support@guidely.in Page 7 of 8


SBI Clerk & RRB PO Mains PDF Course 2023
ENGLISH Day -3 (Eng)

as arrested within hours. So, option C is the with b which gives information about the
correct answer. comparison with past to present situation. So,
option A is the correct answer.
15. Answer: E
Here, no swapping is required as highlighted 18. Answer: D
words are used appropriately. So, option E is the Here, the first sentence after rearrangement will
correct answer. be a which gives the introductory part of the
paragraph about Shharad pawar intending to
16. Answer: A give resignation followed by c which gives
Here, the first sentence after rearrangement will information about protests after the resignation
be a which gives the introductory part of the was tendered followed by e which connects
paragraph about Shharad pawar intending to contextually, followed by d which gives
give resignation followed by c which gives information about personality and finally ends
information about protests after the resignation with b which gives information about the
was tendered followed by e which connects comparison with past to present situation. So,
contextually, followed by d which gives option d is the correct answer.
information about personality and finally ends
with b which gives information about the 19. Answer: B
comparison with past to present situation. So, Here, resignation fits best in the blank given in
option A is the correct answer. part e as the context of the situation is about the
resignation of Sharad Pawar. So, option B is the
17. Answer: A correct answer.
Here, the first sentence after rearrangement will
be a which gives the introductory part of the 20. Answer: D
paragraph about Shharad pawar intending to Here, the first part after rearrangement will be an
give resignation followed by c which gives as it gives an introductory part with a time frame
information about protests after the resignation followed by b which connects contextually
was tendered followed by e which connects followed by c then e, and finally ends with d
contextually, followed by d which gives which gives the end part of the sentence. So,
information about personality and finally ends option D is the correct answer.

Click Here For Bundle PDF Course | support@guidely.in Page 8 of 8


SBI Clerk & RRB PO Mains PDF Course 2023
Reasoning Ability Day -4 (Eng)

Reasoning Ability

Directions (1-5): Study the following information 2) Who among the following person joined three
carefully and answer the given questions. persons before C?
Twelve persons – A, B, C, D, E, F, G, H, I, J, K, a) The one who joined two persons after J
and L joined a company on three different dates b) D
viz.- 3rd, 6th, and 9th in four different months viz.- c) The one who joined six persons before K
March, June, August, and September during the d) Either a or b
same year 2022. Only one person joined on each e) Both a and c
date.
Only four persons joined between G and L, who 3) Four of the following five are alike in a certain
joined on a prime numbered date of a month way based on the given arrangement and thus
having an even number of days. The number of form a group. Which one of the following does
persons joined before L is one more than the not belong to the group?
number of persons joined after B. Only four a) The one who joined two persons after L
persons joined between D and the one who b) C
joined two persons after E, who joined in a month c) The one who joined two persons before J
having an even number of days. The number of d) F
persons joined between E and G is one more e) The one who joined three persons before B
than the number of persons joined before J.
Neither D nor H joined in September. As many 4) Who among the following persons joined on
persons joined between J and I as between I and the 3rd of the given months?
K. Neither K nor A joined at last. The number of I. B
persons joined between H and L is two less than II. The one who joined four persons before F
the number of persons joined between A and K. III. The one who joined immediately after J
C joined in one of the months before F. a) Only III
1) How many persons joined between F and the b) Only I and II
one who joined two persons after I? c) Only I
a) Three d) Only I and III
b) One e) Only II
c) Two
d) More than three 5) If all the persons joined in alphabetical order
e) None starting from 3rd March, then the position of how
many persons remains unchanged?

Click Here For Bundle PDF Course | support@guidely.in Page 1 of 9


SBI Clerk & RRB PO Mains PDF Course 2023
Reasoning Ability Day -4 (Eng)

a) Three 7) How many persons sit between P and T?


b) One a) Two
c) Two b) Three
d) More than three c) Seven
e) None d) Five
e) Six
Directions (6-10): Study the following information
carefully and answer the given questions. 8) Four of the following five are alike in a certain
Nine persons – P, Q, R, S, T, U, V, W, and X are way based on the given arrangement and thus
sitting in a linear row of twelve chairs marked 1 to form a group. Which one of the following does
12 from the left end and all the chairs are facing not belong to the group?
north. a) The one who sits immediate left of U
Note: Neither the chairs at the end of the row nor b) V
the adjacent chairs are vacant. One chair at the c) The one who sits second to the left of P
odd numbered position and two chairs at the d) W
even numbered positions are vacant. e) The one who sits fourth from the right end
V sits fifth from the left end. Only two persons sit
between V and R, who sits to the right of V. At 9) Who among the following persons sits to the
least one chair to the right of R is vacant. X sits right of S?
three places away from V but doesn’t sit adjacent I. X
to R. As many chairs to the left of X as to the II. The one who sits immediate left of V
right of W. Only three persons sit between Q and III. The one sits second to the left of W
S but neither of them sits on the adjacent seat V a) Only III
nor R. Only one person sits between Q and P, b) Only I and II
who sits to the left of S. The number of chairs c) Only I
between U and T is same as the number of d) Only I and III
chairs to the left of P. T doesn’t sit adjacent to S. e) All I, II and III
6) What is the position of P with respect to R?
a) Third to the right 10) If X is related to V and U is related to W in a
b) Third to the left certain way, then who among the following
c) Second to the right person is related to S?
d) Fifth to the left a) Q
e) Second to the left b) The one who sits immediate left of V
c) T

Click Here For Bundle PDF Course | support@guidely.in Page 2 of 9


SBI Clerk & RRB PO Mains PDF Course 2023
Reasoning Ability Day -4 (Eng)

d) The one who sits third from the left end. c) Only I and II
e) None of these d) Only II
e) All I, II, and III
Directions (11-15): Study the following
information carefully and answer the given 13) If all the letters in the given words are
questions. arranged in alphabetical order, then which of the
11) If all vowels in the following words are following word can be formed (meaningful/non-
replaced by the second preceding letters and all meaningful) from the fourth letter of each word
the consonants are replaced by the third from the right end?
succeeding letters as per the English I. COMPUTER
alphabetical series, then which of the following II. DERIVATIVE
options have at most three vowels? (consider the III.MAGNITUDE
immediate preceding letter of A is Z and the IV. HOSPITAL
immediate succeeding letter of Z is A) a) SROP
I. End, Broken, Dark, Field b) PRMN
II. Enjoy, Life, Nature, Air c) ROPN
III. Newly, Open, Cashew, Bag d) PRMO
a) Only I e) PSRO
b) Only II
c) Only III 14) Which of the following word has the highest
d) Both I and II number of pairs of letters in the word each of
e) Both I and III which has as many letters between them in the
word as in the English alphabetical series (both
12) If ‘1’ is added to the odd positioned digits and forward and backward directions)?
‘2’ is subtracted from the even positioned digits I. COUNTER
from the left end of all the numbers, then all the II. EUPHORIA
odd digits from newly formed numbers are III. MAGNITUDE
dropped, then the sum of the digits of which of a) Only I and II
the following numbers are divided by four? b) Only II
I. 36596323 c) Only I and III
II. 895473358 d) Only III
III. 53857856 e) Only I
a) Only I and III
b) Only I

Click Here For Bundle PDF Course | support@guidely.in Page 3 of 9


SBI Clerk & RRB PO Mains PDF Course 2023
Reasoning Ability Day -4 (Eng)

15) If all the consonants in the word 17) Which of the following statement(s) is/are
“EDUCATIONAL” are replaced by the next letter NOT TRUE as per the given arrangement?
and the vowels which come after M in the I. T is the brother-in-law of Z.
English alphabetical series are replaced by the II. H is the wife of K.
third succeeding letter. Then the unchanged III. M is the Aunt of Z.
vowels are removed from the word, and the a) Only I and II
remaining letters are arranged in alphabetical b) Only I and III
order from left to right. Then how many letters c) Only III
are there in the English alphabetical series d) All I, II, and III
between the letters which are third from the left e) Only II
end and third from the right end?
a) Four Directions (18-20): Study the following
b) Seven information carefully and answer the given
c) Three questions.
d) Five Ten members are there in a family of three
e) One generations K is the daughter-in-law of R and is
not the spouse of T. P is the sister-in-law of J,
Directions (16-17): Study the following who is the son of R. M is the mother-in-law of K,
information carefully and answer the given who is the only daughter of S. Q is the brother-in-
questions. law of J, who is the father of C. S is the father-in-
Nine members are there in a family of three law of T’s brother and is the father of Q. G is the
generations. T is the only son-in-law of R’s sister. son of P and G’s gender is same as the gender
D is the brother-in-law of K and vice-versa. K is of C.
the father of M, who has two children. S is the 18) How M is related to G’s mother?
father of U, who is the brother-in-law of T. Z is a) Sister
the granddaughter of H, who is the only sister of b) Aunt
D. R is not the sibling of S and of the same c) Niece
gender as D. d) Mother-in-law
16) How R is related to Z? e) None of these
a) Brother
b) Uncle 19) Which of the following statements is/are not
c) Nephew TRUE as per the given arrangement?
d) Father-in-law I. C is the nephew of S.
e) None of these II. K is the wife of J

Click Here For Bundle PDF Course | support@guidely.in Page 4 of 9


SBI Clerk & RRB PO Mains PDF Course 2023
Reasoning Ability Day -4 (Eng)

III. M is the aunt of C. 20) How many male members are in the family?
a) Only I and II a) Four
b) Only I and III b) Five
c) Only III c) Six
d) All I, II, and III d) Seven
e) Only II e) Cannot be determined

Click Here to Get the Detailed Video Solution for the above given Questions
Or Scan the QR Code to Get the Detailed Video Solutions

Answer Key with Explanation


Directions (1-5):
1) Answer: C
2) Answer: E
3) Answer: B (all the persons joined on an odd
numbered date, except option B)
4) Answer: D
5) Answer: C

We have:

Click Here For Bundle PDF Course | support@guidely.in Page 5 of 9


SBI Clerk & RRB PO Mains PDF Course 2023
Reasoning Ability Day -4 (Eng)

 Only four persons joined between G and Since, the number of persons joined
L, who joined on a prime numbered date between E and G is one more than the
of a month having an even number of number of persons joined before J, thus
days. at least one person must joined between
 The number of persons joined before L is E and G.
one more than the number of persons That means, in case (1) E joined on 9
joined after B. June, in case (1a) E joined on 6 June,
That means, in case (1) L joined the case (2) is not valid because we cannot
company on 3 June, in case (2) L joined place J.
the company on 3 September. Based on the above given information we have:
Based on the above given information we have:

Again, we have:
Again, we have:
 As many persons joined between J and I
 Only four persons joined between D and
as between I and K.
the one who joined two persons after E,
 Neither K nor A joined at last.
who joined in a month having an even
That means, in case (1) I joined on 6
number of days.
June, in case (1a) I joined on 3 August.
 The number of persons joined between E
 The number of persons joined between H
and G is one more than the number of
and L is two less than the number of
persons joined before J.
persons joined between A and K.
 Neither D nor H joined in September.

Click Here For Bundle PDF Course | support@guidely.in Page 6 of 9


SBI Clerk & RRB PO Mains PDF Course 2023
Reasoning Ability Day -4 (Eng)

That means, in case (1a) H joined on 3  R sits to the right of V.


March, and A joined on 9 June, case (1)  At least one chair to the right of R is
is not valid because we cannot place L. vacant.
 C joined in one of the months before F. Since, none of the chairs at the end of the
That means, C must join on 6 August. rows are vacant, thus R can’t sit second
Based on the above given information we have: from the right end.
That means, in case (1) R sits fifth from
the right end, in case (2) R sits fourth from
the right end, in case (3) R sits third from
the right end.
Based on the above given information we have:

Again, we have:
 X sits three places away from V but
doesn’t sit adjacent to R.
 As many chairs to the left of X as to the
Direction (6-10): right of W.
6) Answer: D That means, X sits second from the left
7) Answer: D end.
8) Answer: C(all the persons seated at an odd  Only three persons sit between Q and S
position from the left end, except option C) but neither of them sits adjacent seat of V
9) Answer: A nor R.
10) Answer: B That means, Q must sit to the left of S.
 Only one person sits between Q and P,
who sits to the left of S.
That means, in case (2) S sits second to
We have:
the right of V, in case (2a) S sits at the
 V sits fifth from the left end.
right end, case (1) & case (3) are not valid
 Only two persons sit between V and R.

Click Here For Bundle PDF Course | support@guidely.in Page 7 of 9


SBI Clerk & RRB PO Mains PDF Course 2023
Reasoning Ability Day -4 (Eng)

Since, one seat at odd and two seats at II. Enjoy, Life, Nature, Air -> CQMMB
even positions are vacant, thus chair OGIC QYWSUC YGU
number 3, 6, and 10 are vacant. III. Newly, Open, Cashew, Bag -> QCZOB
Based on the above given information we have: MSCQ FYVKCZ EYJ

12) Answer: D
I. 36596323  44677131  446  14
II. 895473358 976281439  6284  20
III. 53857856 61938664  68664  30

13) Answer: D
We have:
Case (1) is not valid as no two adjacent chairs
I. COMPUTER  CEMOPRTU
are vacant, case (3) is not valid as none of the
II. DERIVATIVE  ADEEIIRTVV
chair at the ends are vacant.
III.MAGNITUDE ADEGIMNTU
Again, we have:
IV. HOSPITAL AHILOPST
 The number of chairs between U and T is
Thus, words formed  PRMO
same as the number of chairs to the left of
P.
14) Answer: D
 T doesn’t sit adjacent to S
I. COUNTER  TR
That means, in case (2a) T sits at the
II. EUPHORIA  EH
right end, case (2) is not valid because we
III. MAGNITUDE  GI, IM, TU, EI, EM and DE
cannot place T and U.
Based on the above given information we have:
15) Answer: A
EDUCATIONAL  EEXDAUIROAM
After omitting the unchanged vowels:
EXDUROM  DEMORUX
Thus, four letters are between M and R.

Directions (16-17):
11) Answer: C
16) Answer: B
I. End, Broken, Dark, Field ->CQG
17) Answer: B
EUMNCQ GYUN IGCOG

Click Here For Bundle PDF Course | support@guidely.in Page 8 of 9


SBI Clerk & RRB PO Mains PDF Course 2023
Reasoning Ability Day -4 (Eng)

Direction (18-20):
18) Answer: D
19) Answer: B
20) Answer: D

Click Here For Bundle PDF Course | support@guidely.in Page 9 of 9


SBI Clerk & RRB PO Mains PDF Course 2023
Quantitative Aptitude Day -4 (Eng)

Quantitative Aptitude

Directions [01 – 04]: Read the following information carefully and answer the questions based on it.
The chart given below shows the % distribution of number of cars manufactured by four different
companies – P, Q, R, and S.

Note: a) Difference between number of cars manufactured by company P and Q (Q > P) is same as that
of cars manufactured by company R.
b) Number of cars manufactured by company S is 60 more than that of R, while number of cars
manufactured by Q is twice as that of S.
c) Out of total number of vehicles (Cars + Buses) manufactured by all companies together, Z% are
buses.
1) Find total number of buses manufactured by buses manufactured by company P, R, and S
all companies together? together?
a) 2000 a) 120
b) 1200 b) 140
c) 1600 c) 180
d) 1500 d) 160
e) None of these e) None of these

2) Total number of vehicles manufactured by 3) For another company T, total number of


company Q is 800, find average number of vehicles (Cars + buses) manufactured is thrice of
number of cars manufactured by company S.

Click Here For Bundle PDF Course | support@guidely.in Page 1 of 11


SBI Clerk & RRB PO Mains PDF Course 2023
Quantitative Aptitude Day -4 (Eng)

Out of the total number of vehicle manufactured • Number of people those who use only FB is Y,
by company T, 75% are cars. 80% of the number which is half of number of people those who use
of buses manufactured by company T are sold. only WP.
Find number of unsold buses of company T. • Number of people those whouse both IM and
a) 36 WP but not FB and number ofpeople those who
b) 44 likes both FB and WP but not IM is in ratio of 2:3
c) 54 respectively.
d) 40 • Number of people those who likes both FB and
e) None of these IM but not WP is 4 more than number of people
who likes all three apps.
4) A = Difference between number of cars • Number of people those who likes only IM is 16
manufactured by P and Q less than thrice the people those who likes only
B = Difference between number of cars FB.
manufactured by R and S. • Difference between number of people who
Find A is how many times of B those who likes only FB and number of people
a) 2 times those who likes both FB and IM but not WP is
b) 2.5 times Z/3. People those who likes IM is same as those
c) 3.2 times likes WP.
d) 3 times 5) Find the number of people those who likes
e) None of these exactly two app?
a) 16
Directions [05 – 08]: Read the following b) 24
information carefully and answer the questions c) 20
based on it. d) 18
A surveyis conducted among certain number of e) None of these
people about uses of social media apps among
three apps – FACEBOOK (FB), INSTAGRAM 6) Number of people those who likes all three
(IM), and WHATS APP (WP). Each surveyed apps is what % of people those who likes only
people use at least one of the given app. IM?
• Number of people those who use all three apps a) 30%
are Z, which is same as that of those who use b) 25%
both FB and WP but IM.Number of people those c) 40%
who likes only FB is more than that of those who d) 45%
likes all three apps e) None of these

Click Here For Bundle PDF Course | support@guidely.in Page 2 of 11


SBI Clerk & RRB PO Mains PDF Course 2023
Quantitative Aptitude Day -4 (Eng)

7) Find the total number of surveyed people? water then mixture T formed in which water is 32
a) 132 ml.
b) 92 9) Find the value of N?
c) 112 a) 240
d) 82 b) 160
e) None of these c) 220
d) 280
8) Number of people those who likes only WP is e) None of these
what % more than number of people those who
likes only IM? 10) When 2Z ml water and 100 ml milk added to
a) 20% mixture S, now find the milk is K% more/less
b) 25% than that of water. Find the value of K
c) 16.66% a) 60
d) 12.5% b) 40
e) None of these c) 62.5
d) 37.5
Directions [09 – 12]: Read the following e) None of these
information carefully and answer the questions
based on it. 11) For initial mixture P, 25% of mixture is
Drum A contains 6Z ml milk and drum B contains removed and replaced by same amount of milk,
10Z ml water. M ml from A mixed with Y ml from then one third of mixture is removed and
B to form new mixture P. Rest M ml from A replaced by same amount of milk. Now find
mixed with rest quantity from B to form new difference between amount of mixture in P and
mixture Q. When 3Z ml water added to mixture amount of water in mixture Q (initially)?
Q, then amount of water in mixture Q becomes a) 120 ml
twice as that of milk. Drum C contains N ml milk, b) 140 ml
while drum D contains (8Z – Y ml) water. 160 ml c) 260 ml
milk from C mixed with and drum D mixed d) 160 ml
together to form new mixture R. Total quantity of e) None of these
water in P, Q, and R together initially is 440 ml.
Rest quantity from drum D mixed with drum E 12) Which of the following is/are true.
(contains half volume of water as that in drum D) I. Total amount of milk in mixture P and mixture T
to form new mixture S. When 20% from mixture initially is 168 ml.
S is removed and replaced by same amount of

Click Here For Bundle PDF Course | support@guidely.in Page 3 of 11


SBI Clerk & RRB PO Mains PDF Course 2023
Quantitative Aptitude Day -4 (Eng)

II. Amount of milk in mixture Q initially is same as b) II and III only


that of water. c) I and II only
III. Milk in mixture T is (Z + 10) % more than that d) I, II and III
of water. e) None of these
a) I and III only
Directions [13 – 16]: Read the following information carefully and answer the questions based on it.
There are three websites – P, Q, and R those sold IPL 2023 tickets. Tickets categorized into three
categories – Gold, Platinum, and Silver. The table given below shows the partial information about the
number of tickets sold by each website.

Note: a) Number of platinum tickets sold by P is same as that of silver, while total number of gold tickets
sold by all websites together is twice that of gold tickets sold by website R.
b) Each website sold at least one ticket of each category, and difference between total number of tickets
sold by P and Q is half of difference between total number of tickets sold by Q and R.
c) Number of tickets sold by P, Q, and R is in order P < Q < R.
13) Number of gold tickets sold by Q is same as e) None of these
that of P, which of the following can be the
possible value (s) of number of platinum ticket 14) Find maximum number of platinum tickets
sold by R. sold by all websites together?
I. 13 a) 38
II. 5 b) 36
III. 15 c) 40
IV. 17 d) 34
a) I, and II only e) None of these
b) I, II and III only
c) III and IV only
d) I, II, and IV only

Click Here For Bundle PDF Course | support@guidely.in Page 4 of 11


SBI Clerk & RRB PO Mains PDF Course 2023
Quantitative Aptitude Day -4 (Eng)

15) Number of gold tickets sold by P is same as Find difference between total number of gold
that of silver tickets sold by Q, find number of tickets and total number of platinum tickets sold
silver tickets sold by P? by all websites together?
a) 4 a) 0
b) 3 b) 4
c) 1 c) 8
d) 2 d) 2
e) None of these e) Cannot be determined

16) Number of gold tickets sold by Q is 50%


more than that of platinum tickets sold by him.
Directions [17 – 20]: Read the following information carefully and answer the questions based on it.
The chart given below shows the % of people those are non-vegetarian (NV) out of total number of
people in a particular city and number of males non – vegetarians (MNV) in that city.
Total number of people = Number of Vegetarians (VG) + Number of Non – Vegetarians (NVG) = Number
of Males + Number of Females

17) For city R, Number of female NVG is twice c) 160


as that of male, find number of female VG in city d) 140
if total number of males in city are 100. e) None of these
a) 120
b) 180

Click Here For Bundle PDF Course | support@guidely.in Page 5 of 11


SBI Clerk & RRB PO Mains PDF Course 2023
Quantitative Aptitude Day -4 (Eng)

18) Difference between NVG and VG of city P d) 90


are 180, then find number of female NVG in city e) None of these
P?
a) 50 20) For city S, number of female VG are Y, which
b) 40 is same as total number of NVG in city. Find
c) 45 number of female NVG in city, if difference
d) 60 between number of male VG and female NVG in
e) None of these city is same as that of number of female
vegetarians in city?
19) For city Q, if ratio of number of female NVG a) 160
to that of male NVG is 7:5, find number of b) 40
Vegetarians in the city? c) 70
a) 120 d) 100
b) 60 e) Can’t be determined
c) 80
Click Here to Get the Detailed Video Solution for the above given Questions
Or Scan the QR Code to Get the Detailed Video Solutions

Answer Key with Explanation


Directions [01 – 04]: 100 = 25 + Z + M + Y
Difference between number of cars Y = 75 – (Z + M) ………………… (2)
manufactured by company P and Q (Q > P) is Form (1) and (2), we get
same as that of cars manufactured by company 75 – (Z + M) = Z – 25
R. 100 = 2Z + M
Y = Z – 25…………………… (1) It is given that Z = 2M
Also, So,

Click Here For Bundle PDF Course | support@guidely.in Page 6 of 11


SBI Clerk & RRB PO Mains PDF Course 2023
Quantitative Aptitude Day -4 (Eng)

100 = 2 x 2M + M Hence answer is option D


Value of M = 100/5 = 20%
So, value of Z = 2 x 20% = 40% 3) Answer: A
Value of Y = 40 – 25 = 15% Number of vehicles manufactured by company T
Now, = 3 x 240 = 720
(20 – 15) % x total number of cars manufactured Number of buses manufactured by company T =
by company = 60 25% x 720 = 180
So, total number of cars manufactured by all Number of unsold buses of company T = 20% x
companies = 60/5 x 100 = 1200 180 = 36
so, 60% x (total number of vehicles Hence answer is option A
manufactured by company) = 1200
so, total number of vehicles manufactured by 4) Answer: D
company = 1200/60 x 100 = 2000 Value of A = 40 – 25 = 15%
Number of buses manufactured by company = Value of B = 20 – 15 = 5%
2000 – 1200 = 800 A: B = 15:5 = 3:1
Hence answer is option D

Directions [05 – 08]:


Number of people those who likes only FB = Y
1) Answer: E So, Number of people those who likes only WP =
According to question, 2 x Y = 2Y
Total number ofbuses manufactured by all So, people those who likes only IM = 3 x Y - 16=
companies together = 800 3Y – 16
Hence answer is option E People those who likes all three apps = People
those who likes both FB and WP but not IM = Z
2) Answer: D So, people those who likes both IM and WP but
Total number of vehicles manufactured by not FB = Z/3 x 2 = 2Z/3
company Q = 800 So, people those who likes both FB and IM but
So, number of buses manufactured by Q = 800 – not WP = Z + 4
480 = 320 Now,
So, total number of buses manufactured by (P + Z + 4 + Z + 2Z/3 + 3Y – 16 = Z + 2Z/3 + Z + 2Y
R + S) = 800 – 320 = 480 3Y – 12 = 2Y
Required average = 480/3 = 160 Value of Y = 12

Click Here For Bundle PDF Course | support@guidely.in Page 7 of 11


SBI Clerk & RRB PO Mains PDF Course 2023
Quantitative Aptitude Day -4 (Eng)

Also, The total number of Surveyed people = 24 + 6 +


Difference between people those who likes both 6 + 4 + 12 + 10 + 20 = 82
FB and IM but not WP = Z/3 Hence answer is option D
So, (Z + 4) – Y = Z/3
2Z/3 = 12 – 4 8) Answer: A
Z = 12 (not possible because Y > Z) Required % change = (24 – 20)/20 x 100 = 20%
So, Hence answer is option A
Y – Z – 4 = Z/3
4Z/3 = 8 Directions [09 – 12]:
Z=6 Drum A contains 6Z ml milk. M ml milk from A is
Now we can find all the related data. in mixture P and rest M ml milk from A is from
mixture Q.
So, 2M = 6Z
M = 3Z ml
Amount of milk and water in mixture P = 3Z and
Y ml
Amount of milk and water in mixture Q = 3Z and
(10Z – Y) ml
When 3Z ml water added to Q then amount of
5) Answer: C water in Q becomes twice as that of milk.
According to question, So,
People those who likes exactly two app = 10 + 6 2 x 3Z = 3Z + 10Z – Y
+ 4 = 20 Y = 7Z………………. (1)
Hence answer is option C Amount of milk in R = 160 ml
Amount of water in R = (8Z – Y)
6) Answer: A Now,
Number of people those who likes all three apps 10Z + (8Z – Y) = 440 ml
=6 18Z – Y = 440 ml……………. (2)
Number of people those who likes only IM = 20 From (1) and (2), we get
Required % = 6/20 x 100 = 30% 18Z – 7Z = 440
Hence answer is option A Z = 440/11 = 40 ml
Value of Y = 7 x 40 = 280 ml
7) Answer: D Amount of milk in mixture S = (N – 160)

Click Here For Bundle PDF Course | support@guidely.in Page 8 of 11


SBI Clerk & RRB PO Mains PDF Course 2023
Quantitative Aptitude Day -4 (Eng)

Amount of water in mixture S = (8 x 40 – 280)/2 Hence answer is option B


= 20 ml
Now, 12) Answer: D
20% x [(N – 160) + 20] + 80% x 20 = 32 ml I. Total amount of milk in mixture P and mixture
N – 140 = 80 T initially is 168 ml.
N = 220 ml Required value = 120 + 60 x 80% = 168 ml
So, amount of milk in S = 220 – 160 = 60 ml This statement is true.
When 20% from mixture S is removed and II. Amount of milk in mixture Q initially is same
replaced by same amount of water then mixture as that of water.
T formed in which water is 32 ml. Amount of milk in Q initially = Amount of water in
So, amount of milk in T = 60 - 60 x 20% = 48 ml. Q initially = 120 ml
9) Answer: C This statement is true
According to question, III. Milk in mixture T is (Z + 10) % more than that
Value of N = 220 of water.
Hence answer is option C Milk in T = 48 ml
Water in T = 32 ml
10) Answer: A Required % change = (48 – 32)/32 x 100 = 50%
Ratio of milk and water in mixture S, final = (60 + = (Z + 10) %
100): (20 + 2 x 40) = 8:5 Hence answer is option D
Required % change = (8 – 5)/5 x 100 = 60%
K = 60 Directions [13 – 16]: Difference between number
Hence answer is option A of tickets sold by P and Q = ½ x difference
between number of tickets sold by Q and R
11) Answer: B 2 x (Q – 10) = (R – Q)
Amount of mixture in P initially = 400 ml 2Q – 20 = R – Q
Amount of water in P initially = 280 ml R = 3Q – 20……………… (1)
Now, Also,
Amount of water in final mixture P = 280 x 3/4 x 10 + Q + R = 70
2/3 = 140 ml Q + R = 60……………… (2)
So, amount of milk in final mixture P = 400 – 140 From (1) and (2), we get
= 260 ml Q + 3Q – 20 = 60
Amount of water in Q = 120 ml So, Q = 20
Required difference = 260 – 120 = 140 ml R = 40

Click Here For Bundle PDF Course | support@guidely.in Page 9 of 11


SBI Clerk & RRB PO Mains PDF Course 2023
Quantitative Aptitude Day -4 (Eng)

Let number of platinum tickets sold by P = silver Hence answer is option D


tickets sold by P = Z
So, Number of gold tickets sold by P = 10 – (Z + 14) Answer: B
Z) = 10 – 2Z We need to find maximum value of 42 – 2Y
Let Number of gold tickets sold by R = Y Number of platinum tickets sold by P = Z = 4
So, Number of gold tickets sold by all websites (maximum possible)
together = 2 x Y = 2Y Y + 2Z – 10 = 1 (minimum possible number of
So, Number of gold tickets sold by Q = 2Y – Y – gold tickets sold by Q)
10 + 2Z = Y + 2Z – 10 Y–2=1
Number of silver tickets sold by Q = 28 – 19 – Z Y=3
=9–Z So, maximum number of platinum tickets sold =
Number of platinum tickets sold by R = 40 – 19 – 42 – 2Y = 42 – 2 x 3 = 36
Y = 21 - Y Hence answer is option B
Number ofplatinum tickets sold by all websites
together = 70 – 2Y – 28 = 42 – 2Y 15) Answer: C
Number of platinum tickets sold by Q = 42 – 2Y – According to question,
Z – 21 + Y = 21 – (Y + Z) 10 – 2Z = 9 – Z
Value of Z = 1
Hence answer is option C

16) Answer: D
According to question,
13) Answer: D
Ratio of number ofgold to platinum tickets sold
According to question,
by Q = 3:2 (3a and 2a)
Number of gold tickets sold by P = Number of
Now,
gold tickets sold by Q
3a + 2a + 9 – Z = 20
So,
5a – Z = 11
10 – 2Z = Y + 2Z – 10
Value of a = (11 + Z)/5
Y = 20 – 4Z
Only one value of Z is possible = 4
Possible values of Z= 1, 2, 3, and 4
So, value of a = 3
Possible value of Y = 16, 12, 8, 4
So, number of gold tickets sold by P = 10 – 2 x 4
Required value = 21 – Y
=2
So possible required values = 5, 9, 13, and 17
Number of old tickets sold by Q = 3 x 3 = 9
So, I, II and IV is possible

Click Here For Bundle PDF Course | support@guidely.in Page 10 of 11


SBI Clerk & RRB PO Mains PDF Course 2023
Quantitative Aptitude Day -4 (Eng)

So, total number of gold tickets sold = 2 x (9 + 2) Number of female NVG = 90 – 40 = 50


= 22 Hence answer is option A
Total number of platinum tickets sold = 42 – 22 =
20 19) Answer: D
Required difference = 22 – 20 = 2 For city Q
Hence answer is option D Number male NVG = 25
Number of female NVG = 7/5 x 25 = 35
17) Answer: C Total number of NVG = 35 + 25 = 60
For city R So, number of Vegetarians in city = 60/40 x 60 =
Number of female NVG = 2 x 20 = 40 90
So, total number of NVG = 40 + 20 = 60 Hence answer is option D
So, totalpopulation of city = 60/20 x 100 = 300
Total number of VG in city = 80% x 300 = 240 20) Answer: B
Number of male VG in city = 100 – 20 = 80 Number NVG = 30%
So, Number of female VG in city = 240 – 80 = Number of VG = 70%
160 Number of female VG = 30%
Hence answer is option C So, number of male VG = 70% - 30% = 40%
Difference between Male VG and Female NVG =
18) Answer: A 30%
For city P So, number of female NVG = 40 – 30 = 10%
Number of NVG = 25% Now,
Number of VG = 75% 20% = 80
Now, So, 10% = 40
(75% - 25) % x total population = 180 Hence answer is option B
Total population of city = 180 x 2 = 360
Number of NVG = 25% x 360 = 90

Click Here For Bundle PDF Course | support@guidely.in Page 11 of 11


SBI Clerk & RRB PO Mains PDF Course 2023
ENGLISH Day -4

English Language

Directions (1-7): Given below are a few that some are planning to carve Mumbai out of
questions based on the passage. You have to Maharashtra. But, if such attempts are made, we
read the passage carefully and answer the must stay alert and oppose it.” This led to fears
questions accordingly. If none of the options are among Marathi-speakers in Mumbai, who felt
correct then choose option E as your answer. they would be marginalised further. In the
In 1978, the Shiv Sena was in the throes of a subsequent BMC polls, the Shiv Sena captured
crisis. Three years ago, the party had capitulated India’s richest civic body on the back of massive
to pressure and threats of a ban to support the linguistic polarisation. The victory rejuvenated the
then Prime Minister Indira Gandhi’s declaration Shiv Sena, which gradually managed to establish
of nationwide emergency. In the Brihanmumbai a ‘reward-economy’ network for its organisational
Municipal Corporation (BMC) elections held in rank and file using its control over the BMC.
1978, the Shiv Sena fielded around 120 In 1991, the Shiv Sena was beset with another
candidates, but only 21 managed to win. Mindful crisis as senior leader and former Mumbai Mayor
of the growing murmurs in the party, Thackeray Chhagan Bhujbal walked out of the party. In the
did something unthinkable. At a rally in the BMC polls held the next year, the Shiv Sena
Shivaji Park grounds in Dadar, he announced his decided against forging an alliance with new-
resignation as party president but withdrew it found ally BJP, but lost to the Congress.
after being similarly ‘requested’ by diehard Shiv After the Shiv Sena’s defeat in the 1992 BMC
Sainiks. In 1985, the Shiv Sena was still licking polls, Madhav Deshpande, a former Shiv Sena
its wounds after a string of electoral defeats— functionary, went public with allegations against
including one in the 1984 Lok Sabha polls, where Thackeray. He accused Thackeray of holding the
the party’s alliance with the BJP had come a Shiv Sena hostage to his personality and alleged
cropper and led to the two parting ways. But it that his pro-Hindutva stance smacked of
was given an unexpected lifeline by then chief opportunism. He also charged Thackeray’s son
minister Vasantdada Patil, who was locked in a Uddhav and nephew Raj, then rising stars in the
cold war with Bombay Pradesh Congress party, with “interfering” in the Shiv Sena’s affairs.
Committee (BPCC) strongman Murli Deora. In Confronted with a challenge to his leadership,
the state legislative council, Shiv Sena leader Thackeray used his ‘dhakka tantra’ or shock
Pramod Navalkar alleged that the Union therapy. On July 18, 1992, the Shiv Sena
government was planning to administer Mumbai mouthpiece Saamana, which had Bal Thackeray
centrally by severing it from Maharashtra. as the editor, carried a cryptic two-line
Replying to this, Patil said: “There is a chance announcement on its front page: ‘Akhercha Jai

Click Here For Bundle PDF Course | support@guidely.in Page 1 of 9


SBI Clerk & RRB PO Mains PDF Course 2023
ENGLISH Day -4

Maharashtra’ (The Final Goodbye). The A. In 1978, the Shiv Sena was in the throes of a
announcement said Thackeray and his family crisis.
members had said “a final Jai Maharashtra to the B. In the Brihanmumbai Municipal Corporation
Shiv Sena”. Thousands of shocked and upset (BMC) elections held in 1978, the Shiv Sena
Shiv Sainiks rushed to Matoshri, the Thackeray fielded around 120 candidates, but only 25
family residence at Kalanagar in suburban managed to win.
Bandra, and were told that the party supremo C. At a rally in the Shivaji Park grounds in Dadar,
was firm on his decision. The next day, he announced his resignation as party president
Thackeray wrote an editorial but withdrew it after being similarly ‘requested’ by
in Saamana explaining his stance and lashed out diehard Shiv Sainiks.
at Deshpande. Implored by Shiv Sainiks, D. Both A&C
Thackeray addressed them in a public meeting E. None of the above
on July 20 outside Shiv Sena Bhawan at Dadar
West in pouring rain. He announced he was 3. Which of the following options give the reason
going back on his word after being ‘persuaded’ for shivsena’s beset with another crisis in 1991?
by hysterical Sainiks, some of whom had even A. Because Thackeray’s son Uddhav and
threatened to set themselves ablaze. In the nephew Raj, then rising stars in the party, with
public meeting, frenzied Shiv Sainiks refused to “interfering” in the Shiv Sena’s affairs.
allow party leaders and public representatives to B. Because Thackeray wrote an editorial
get on the stage and some like Manohar Joshi in Saamana explaining his stance and lashed out
were even heckled. In 1995, the Shiv Sena-BJP at Deshpande. Implored by Shiv Sainiks,
alliance came to power in Maharashtra and Thackeray addressed them in a public meeting
Thackeray chose Joshi to become the chief on July 20 outside Shiv Sena Bhawan at Dadar
minister. West in pouring rain.
1. Which of the following is the antonym of C. Because a senior leader and former Mumbai
“STANCE”? mayor Chhagan Bhujbal walked out of the party.
A. attitude D. Both A&B
B. carriage E. None of the above
C. posture
D. station 4. Which of the following phrase is correct as per
E. None of the above the given passage?
A. In 1985, the Shiv Sena was still licking its
2. Which of the following phrase is incorrect as wounds after a string of electoral defeats—
per the given passage? including one in the 1984 Lok Sabha polls, where

Click Here For Bundle PDF Course | support@guidely.in Page 2 of 9


SBI Clerk & RRB PO Mains PDF Course 2023
ENGLISH Day -4

the party’s alliance with the BJP had come a D. Both A&C
cropper and led to the two parting ways. E. None of the above
B. After the Shiv Sena’s defeat in the 1992 BMC
polls, Madhav Deshpande, a former Shiv Sena 7. Which of the following is the synonym of
functionary, went public with allegations against “LASHED”?
Thackeray. A. unravel
C. In the Brihanmumbai Municipal Corporation B. undo
(BMC) elections held in 1978, the Shiv Sena C. uncoil
fielded around 120 candidates, but only 21 D. whack
managed to win. E. None of the above
D. All of the above
E. None of the above
Directions (8-12): Given below are a few
questions with three highlighted words each
5. Which of the following was the reason for
which may or may not need to be replaced in
parting ways of Shivsena and BJP in 1985?
order to make the sentence contextually correct.
A. it was due to continuous defeat in elections
B. it was due to bitter relationship with If no replacement is required then choose option

Vasantdada Patil E as your answer.

C. it was due to other tie up ideas coming in the


8. Cox's Bazar (a) encompasses the world’s
minds of Shivseniks
largest refugee camp, (b) sheltering over 9 lakh
D. it was due to different ideology of Uddhav and
Rohingya Muslims who fled the (c) persecution
raj thackarey
and violence Myanmar in August 2017.
E. None of the above
A. Only a
B. Only b
6. Why did Thackeray address the public
C. Only c
meeting on July 20 outside Shiv Sena Bhawan at
D. Both A&C
Dadar West in pouring rain?
E. No replacement required
A. Because shivseniks were not happy with
Deshpande.
9. Caste has always been a key (a) variable in
B. Because shivseniks were not happy with the
electoral politics in India. The 2023 Assembly
election performance of their leaders.
elections in Karnataka are no different -- the
C. Because shivseniks were not happy with the
three major parties in the state have a strong (b)
decision made by Thackarey family of saying the
last goodbye to Maharashtra.

Click Here For Bundle PDF Course | support@guidely.in Page 3 of 9


SBI Clerk & RRB PO Mains PDF Course 2023
ENGLISH Day -4

lookup from at least one (c) dominant caste or 12. Truss' premiership (a) distributed to bringing
community. relations between Britain and China to a low
A. Only a point, though her successor Rishi Sunak is
B. Only b aiming to (b) engage where possible with China
C. Only c while also raising (c) concerns over areas of
D. Both A&C disagreement.
E. No replacement required A. Only a
B. Only b
10. (a) Barring Coastal Karnataka and the Old C. Only c
Mysuru region, Lingayats are spread across the D. Both A&C
state and highly (b) concreted in northern E. No replacement required
Karnataka, (c) especially in Mumbai-Karnataka.
A. Only a Directions (13-17): Given below are a few
B. Only b questions with jumbled phrases in a sentence.
C. Only c You have to rearrange the phrases to make a
D. Both A&C contextually correct sentence. If none of the
E. No replacement required options are correct then choose option E as your
answer.
11. Imran Khan, who was (a) ousted as 13. Two criminal investigations (a)/ Trump
Pakistan's prime minister last year and leads the separately faces (b)/ overseen by a U.S. Justice
Opposition, faces more than legal 100 cases, Department (c)/ special counsel into his keeping
most involving allegations that he incited violence classified documents (d)/ after leaving office and
and threatened police and government his efforts to overturn his 2020 election loss (e).
officials. Imran Khan, who was ousted as A. adebc
Pakistan's prime minister last year and leads the B. abced
Opposition, faces more than legal 100 cases, C. bacde
most involving (c) allegations that he (c) excited D. badce
violence and threatened police and government E. No rearrangement required
officials.
A. Only a 14. CNN is facing a backlash over its town hall
B. Only b (a)/ featuring former President Donald Trump
C. Only c (b)/, an event that swiftly turned chaotic (c)/ in a
D. Both A&C stark display of the tightrope facing journalists
E. No replacement required

Click Here For Bundle PDF Course | support@guidely.in Page 4 of 9


SBI Clerk & RRB PO Mains PDF Course 2023
ENGLISH Day -4

covering (d)/ a leading 2024 Republican local (c)/ and Chinese navies in drills which it
candidate who refuses to play by the rules (e). said at the time would (d)/ "strengthen the
A. adebc already flourishing relations between South
B. abced Africa, Russia and China" (e).
C. bacde A. adebc
D. badce B. abced
E. No rearrangement required C. bacde
D. badce
15. A Ukrainian brigade commander (a)/ fighting E. No rearrangement required
in the ruins of Bakhmut said Russian mercenary
forces have stepped up (b)/ shelling and artillery Directions (18-22): Given below are a few
attacks (c)/ despite its chief's claims to the questions divided into five parts which may or
contrary (d)/ in recent days and were not facing a may not contain errors in them. You have to find
munitions shortage (e). the part containing the error and mark your
A. adebc answer accordingly. If there is no error then
B. abced choose option E as your answer.
C. bacde 18. The China-Canada relationship has been
D. badce cold (a)/ for several years (b)/, especially after
E. No rearrangement required the detention of Huawei Technologies executive
Meng Wanzhou in 2018, China then arrested two
16. South Africa (a)/ against a UN resolution Canadians on spying charges (c)/, All three were
decrying Russia’s attack on Ukraine (b)/ has free in 2021 (d). No error (e).
consistently refused to take sides in the war (c)/, A. a
which was among the first (d)/ of a few countries B. b
to abstain from voting (e). C. c
A. adebc D. d
B. abced E. e
C. bacde
D. badce 19. Last year (a)/, Beijing lifted a three-year ban
E. No rearrangement required on imports of canola (b)/, Canada's largest crop
(c)/, from trading companies Richardson
17. Russian Foreign Minister Sergei Lavrov for International and Viterra that was imposed in
talks in January (a)/ South Africa hosted (b), and 2018 (d). No error (e).
a few weeks later Russian warships joined the A. a

Click Here For Bundle PDF Course | support@guidely.in Page 5 of 9


SBI Clerk & RRB PO Mains PDF Course 2023
ENGLISH Day -4

B. b ago (b)/, the Argentine government wanted "to


C. c cut my head off" by backing false accusations
D. d (c)/ that he had collaborated to the military
E. e dictatorship of the 1970s (d). No error (e).
A. a
20. Imran Khan, his wife Bushra Bibi, and other B. b
leaders of his party Pakistan Tehreek-e-Insaf are C. c
accused in the Al-Qadir Trust case (a)/, which D. d
revolves around the creation of Al-Qadir E. e
University (b)/. The case involves a supposed
agreement between Imran Khan's government 22. When Francis were elected pope in 2013
and a real estate magnate (c)/, which resulted in (a)/, an Argentine journalist accused Francis of
a loss of 50 billion rupees to Pakistan's already having betrayed the two priests (b)/ when he was
vulnerable economy (d). No error (e). Father Jorge Mario Bergoglio and the superior of
A. a Argentine Jesuits (c)/ during the military's "dirty
B. b war" against leftists (d). No error (e).
C. c A. a
D. d B. b
E. e C. c
D. d
21. Pope Francis said that (a)/ when he was E. e
archbishop of Buenos Aires more than a decade
Click Here to Get the Detailed Video Solution for the above given Questions
Or Scan the QR Code to Get the Detailed Video Solutions

Answer Key with Explanation

Click Here For Bundle PDF Course | support@guidely.in Page 6 of 9


SBI Clerk & RRB PO Mains PDF Course 2023
ENGLISH Day -4

1. Answer: E Here, from the lines, in 1985, the Shiv Sena was
Here, the stance means a way of standing or still licking its wounds after a string of electoral
being placed or an intellectual or emotional defeats—including one in the 1984 Lok Sabha
attitude and here all of the options given are polls, where the party’s alliance with the BJP had
synonyms. So, option E is the correct answer. come a cropper and led to the two parting ways.
We can say that option A is the correct answer.
2. Answer: B
6. Answer: C
Here, from the lines In the Brihanmumbai From the lines, On July 18, 1992, the Shiv Sena
Municipal Corporation (BMC) elections held in mouthpiece Saamana, which had Bal Thackeray
1978, the Shiv Sena fielded around 120 as the editor, carried a cryptic two-line
candidates, but only 21 managed to win. We can announcement on its front page: ‘Akhercha Jai
say that only 21 managed to win not 25. So, Maharashtra’ (The Final Goodbye). The
option B is the correct answer. announcement said Thackeray and his family
members had said “a final Jai Maharashtra to
the Shiv Sena”. Thousands of shocked and
3. Answer: C upset Shiv Sainiks rushed to Matoshri, the
Here, from the lines, in 1991, the Shiv Sena was Thackeray family residence at Kalanagar in
beset with another crisis as senior leader and suburban Bandra, and were told that the party
former Mumbai Mayor Chhagan Bhujbal walked supremo was firm on his decision. The next day,
out of the party. We can say that option C is the Thackeray wrote an editorial
correct answer. in Saamana explaining his stance and lashed
out at Deshpande. Implored by Shiv Sainiks,
4. Answer: C Thackeray addressed them in a public meeting
Here, as per the passage given, all of the on July 20 outside Shiv Sena Bhawan at Dadar
options given are correct in the first options it is West in pouring rain. We can say from the lines
said about the defeat of shivesena in 1985 then that option C is the correct answer.
in the second option it is mentioned about the
defeat in 1992, and a BMC election is mentioned 7. Answer: D
in the third option and all of the phrases are Here, the lash means to make a verbal attack or
correct. So, option C is the correct answer. retort and here whack means the same whereas
all other options are antonyms. So, option D is
5. Answer: A the correct answer.

Click Here For Bundle PDF Course | support@guidely.in Page 7 of 9


SBI Clerk & RRB PO Mains PDF Course 2023
ENGLISH Day -4

Here, first part after rearrangement will be b as it


8. Answer: E gives introductory part of the sentence about
Here, no replacement is required as highlighted number of investigation followed by b which
words are appropriately used. So, option E is the gives information about on whom it is being
correct answer. investigated and then cde which is in same
sequence in which they already are. So, option
9. Answer: B C is the correct answer.
Here, lookup needs replacement as it means to
look after something whereas as per the context 14. Answer: E
of the sentence which is about Karnataka Here, no rearrangement is required for this
election, backup should have been used. So, question. So, option E is the correct answer.
option B is the correct answer.
15. Answer: B
10. Answer: B Here, the first part after rearrangement will be a
Here, concentrated should have been used in which is the introductory part starting with the
place of concreted as per the context of the Ukrainian commander followed by b which
sentence. So, option B is the correct answer. connects contextually, followed by c which gives
information about shelling and then e which
11. Answer: C gives reason for all these happening and finally
Here, excited should have been used in place of ends with d. So, option B is the correct answer.
incited as per the context of the sentence.
Excited means eagerly waiting for something 16. Answer: A
good to be happened while incited means to Here, the first part after rearrangement will be a
rouse or to poke someone or something. So, which gives an introductory part with the subject
option C is the correct answer. as South Africa followed by d which connects
contextually followed by e and then b and finally
12. Answer: A ends with C which is ending part. So, option A is
Here, contributed should have been used in the correct answer.
place of distributed as per the context of the
sentence. So, option A is the correct answer. 17. Answer: C
Here, the first part after rearrangement will be b
13. Answer: C which gives an introductory part starting with
South Africa hosting followed by a which gives

Click Here For Bundle PDF Course | support@guidely.in Page 8 of 9


SBI Clerk & RRB PO Mains PDF Course 2023
ENGLISH Day -4

information about whom it is hosting and then


followed in the same sequence as they are given 20. Answer: E
as cde. So, option C is the correct answer. Here, there is no error in the given question.
Therefore, option E is the correct answer.
18. Answer: D
Here, the error lies in part d as the use of free is 21. Answer: D
inappropriate as the sentence is in past tense Here, the error lies in part d as the use of to after
and so freed should have been used here in collaborated is inappropriate and here should
place of free in part d as freed is the third form of have been used instead. Collaborate is always
free which is apt here. So, option D is the correct followed by with and the same should have been
answer. used here. So, option D is the correct answer.

19. Answer: D 22. Answer: A


Here, the error lies in part d as the use of was is Here, the error lies in part a as the use of were is
inappropriate and here had been should have inappropriate as Francis is a singular subject
been used instead. As the sentence is in past and therefore was should have been used
perfect continuous tense and therefore had been because were is used with plural subject and
should have been used here. So, option D is the was with singular. So, option A is the correct
correct answer. answer.

Click Here For Bundle PDF Course | support@guidely.in Page 9 of 9


SBI Clerk & RRB PO Mains PDF Course 2023
Reasoning Ability Day -5 (Eng)

Reasoning Ability

Directions (1-5): Study the following information a) Match


carefully and answer the given questions. b) 64
Input: 39 96 Launch Satellite 55 Orbit 48 Payload c) Inaugural
82 Astronaut Dock 42 d) Team
Step I: 82Payload 39 96 Launch Satellite 55 e) 28
Orbit 48 Astronaut Dock 42
Step II: 42 Satellite 82 Payload 39 96 Launch 55 3) What is the sum of the second number from
Orbit 48 Astronaut Dock the left end in Step II and the third number from
Step III: 96 Launch 42 Satellite 82 Payload 39 55 the right end in Step VI?
Orbit 48 Astronaut Dock a) 147
Step IV: 39 Dock 96 Launch 42 Satellite 82 b) 128
Payload 55 Orbit 48 Astronaut c) 116
Step V: 48 Orbit 39 Dock 96 Launch 42 Satellite d) 92
82 Payload 55 Astronaut e) 157
Step VI: 55 Astronaut 48 Orbit 39 Dock 96
Launch 42 Satellite 82 Payload 4) How many elements are between the “Team”
Step VI is the last step of the given arrangement. and “Franchise” in the penultimate step?
Based on the given logic, rearrange the given a) Four
input. b) Six
Input: 28 Inaugural Between 64 Team 93 52 c) Seven
Franchise 36 League 75 Match d) Five
1) In which of the following step, “93 Franchise e) None
Between 36” appear in the same order?
a) Step II 5) Which of the following element is exactly
b) Step III placed between “52” and “36” in step III?
C) Step IV a) 28
d) Step V b) Between
e) Both B and D c) League
d) 36
2) Which of the following element is fourth to the e) Cannot be determined
left of the element which is immediate right of
League in step IV?

Click Here For Bundle PDF Course | support@guidely.in Page 1 of 12


SBI Clerk & RRB PO Mains PDF Course 2023
Reasoning Ability Day -5 (Eng)

Directions (6-10): Study the following information c) Air Commodore


carefully and answer the given questions. d) Group Captain
Seven people – K, L, M, N, O, P and Q are e) Air Marshal
working in the Indian Air Force at different
designations such as Marshal, Air Chief Marshal 7) Which of the following statement(s) is/are
(ACM), Air Marshal (AM), Air Vice Marshal NOT TRUE as per the given arrangement?
(AVM), Air Commodore, Group Captain and I) Only three people are designated between the
Wing Commander. The designations are given in one who likes Longan and P
decreasing order such thatMarshal is the II) Q doesn’t like Durian
seniormost designation and Wing Commander is III) N is designated as Marshal
the juniormost designation. They like seven IV) The one who likes Pitaya is designated as the
different fruits – Durian, Lychee, Pitaya, Ackee, juniormost person
Longan, Pomelo and Santol. a) Only I
Only three people are designated between the b) Only II and IV
one who likes Santol and Q, who is not junior to c) Only I, II and III
the AVM. As many people senior to the one who d) Only IV
likes Santol as junior to the one who likes Durian. e) Only III
No one is designated between the one who likes
Durian and N, who does not like Pomelo. The 8) Four of the following five are alike in a certain
one who likes Pitaya is three people junior to N. way based on the given arrangement and thus
The number of people senior to the one who form a group. Which one of the following does
likes Pitaya is one more than the number of not belong to the group?
people designated between K and the one who a) Q - The one who likes Ackee
likes Pomelo. Only one person is designated b) K – The one who likes Longan
between O and the one who likes Pomelo. As c) N – The one who likes Durian
many people senior to O as junior to P. Only one d) L – The one who likes Ackee
person is designated between P and the one e) P – The one who likes Pitaya
who likes Lychee. The one who likes Ackee is
junior to the one who likes Lychee. L is three 9) Who among the following person is
people senior to the one who likes Longan. designated as Air Vice Marshal?
6) What is the designation of the person who a) N
likes Santol? b) The one who likes Ackee
a) Air Vice Marshal c) Q
b) Marshal d) K

Click Here For Bundle PDF Course | support@guidely.in Page 2 of 12


SBI Clerk & RRB PO Mains PDF Course 2023
Reasoning Ability Day -5 (Eng)

e) The one who is immediately junior to the one ii) If a team’s number is a perfect square, then it
who likes Pomelo must be selected in the red group.
Blue group:
10) If the Indian army gave three star rating to The remaining teams are selected in the blue
the people who are junior to L while it gave four group.
star rating to the remaining people, then what is Quarter-finals:
the sum of the star rating of the one who likes In each group, only two matches are conducted
Lychee, O and the one who is the Air chief and each team is playing exactly one match and
marshal? two winning teams in each group get qualified for
a) 12 the semi-finals.
b) 10 In each group, the consecutive alphabetically
c) 11 named teams are not playing against each other
d) 9 in the quarter-finals. In the Red group, the sum of
e) None of these the numbers of the two teams which are playing
against each other is a prime number.
Directions (11-15): Study the following Semi Finals:
information carefully and answer the given The team with the second lowest even number is
questions. qualified for the semifinals. Only one team with a
Eight cricket teams – A, B, C, D, E, F, G and H vowel as a team name is qualified for the
are playing in a tournament and the teams are semifinals. The team with the highest number is
numbered from 1 to 8 from A to H respectively. not selected for the semifinals. In the Blue group,
Initially, the teams are divided into two groups i.e. the team which has the lowest number is not
Red and Blue. The teams are playing quarter- selected.
finals, semi-finals and finals based on some In the semifinals, the teams which are in the
conditions given below. same group do not play against each other. The
Note I: There are four teams in each group consecutive alphabetically named teams are not
Note II: If the name of two teams is a vowel, then playing against each other in the semi-finals.
the two teams are not selected in the same Finals:
group. In the finals, the teams which are in the same
Red Group: group do not play against each other. The sum of
In the Red group, the four teams are selected the number of the two teams which are qualified
based on the below condition. and eliminated is equal. The team with a vowel
i) The sum of the numbers of the four teams in as the name is not qualified for the finals. The
the red group should be a perfect square.

Click Here For Bundle PDF Course | support@guidely.in Page 3 of 12


SBI Clerk & RRB PO Mains PDF Course 2023
Reasoning Ability Day -5 (Eng)

team which belongs to red group does not win e) 3


the tournament.
11) Which of the following teams are selected in 15) What is the sum of the numbers of all the
the Red group? teams which are present in the blue group?
I. H a) 20
II. C b) 18
III. F c) 14
a) Only I d) 12
b) Only II e) 16
c) Only I and III
d) Only I and II Directions (16-17): Study the following
e) All I, II and III information carefully and answer the question
given below:
12) Which of the following team played in the The Reserve Bank of India announced that it has
semi-final with team C? decided to withdraw the Rs 2,000 currency notes
a) F from circulation. It asked all to exchange these
b) D notes by September 30, 2023. About 89 per cent
c) E of the 2000 denomination banknotes were issued
d) H before March 2017 and are at the end of their
e) None of these estimated life span of 4-5 years.The three
13) Which of the following teams played against segments which will be affected by this move will
each other in the semifinals? be gold &jewellery, real estate and political
a) F and D parties.RBI said all bank branches will provide
b) D and E the facility of exchange of ₹ 2000 banknotes into
c) E and C banknotes of other denominations up to a limit of
d) both a and b ₹ 20,000 at a time and provide the exchange
e) both a and c facilities for 2000 banknotes until September 30,
2023.
14) What is the number of the team which won The Government then said the SBNs were being
the tournament? demonetised to fight against corruption, black
a) 7 money and fake currency. RBI said members of
b) 6 the public may deposit ₹ 2000 banknotes into
c) 4 their bank accounts and/or exchange them for
d) 5 banknotes of other denominations at any bank

Click Here For Bundle PDF Course | support@guidely.in Page 4 of 12


SBI Clerk & RRB PO Mains PDF Course 2023
Reasoning Ability Day -5 (Eng)

branch. The central bank said the facility for the c) This move is significant, coming as it does just
exchange of 2000 banknotes up to the limit of after the state assembly elections in Karnataka,
20,000 at a time will also be provided at its 19 where the Congress won, and in the run-up to
Regional Offices (ROs) having Issue elections in other states and the 2024 general
Departments. elections.
16) Which of the following can be a d) About 89 per cent of the ₹ 2000 denomination
probable Course of Action to reduce the public banknotes were issued before March 2017 and
burden in the banks while exchanging the are at the end of their estimated life span of 4-5
currency notes? years.
a) The central bank said the facility for the e) None of these
exchange of 2000 banknotes up to the limit of
20,000 at a time will also be provided at its 29 18) Statement:
Regional Offices (ROs) having Issue "You are hereby appointed as an officer with a
Departments. probation period of one year and your
b) RBI said some particular bank branches will performance will be reviewed at the end of the
provide the facility of exchange of ₹ 2000 period for confirmation." - A line in an
banknotes into banknotes of other denominations appointment letter.
up to a limit of ₹ 20,000 in a day. Assumptions:
c) RBI said all bank branches will provide the I. The performance of an individual generally is
exchange facilities for ₹ 2000 banknotes until known at the time of the appointment offer.
September 30, 2023. II. Generally, an individual tries to prove his worth
d) RBI said members of the public may deposit during the probation period.
2000 banknotes into their bank accounts and/or a) Only I is implicit.
exchange them for banknotes of other b) Only II is implicit
denominations at any bank branch c) Either I or II is implicit
e) None of these d) Neither I nor II is implicit
17) Which of the following can e) Both I and II are implicit.
be concluded from the above paragraphs?
a) The three segments which will be affected by Directions (19-20): A statement is followed by
this move will be Gold &Jewellery, Real estate three inferences numbered I, II, and III. Consider
and Political parties. the statement to be true even if it is at variance
b) The Government then said the SBNs were with commonly known facts. You have to decide
being demonetised to fight against corruption, which of the inferences, if any, follow from the
black money and fake currency given statement.

Click Here For Bundle PDF Course | support@guidely.in Page 5 of 12


SBI Clerk & RRB PO Mains PDF Course 2023
Reasoning Ability Day -5 (Eng)

Statement: d) Russia seek the convenant from the US, that


Russia-Ukraine War, war between Russia and Ukraine should be moved away from NATO
Ukraine that began in February 2014 with the e) All of them follow
covert invasion of the Ukrainian autonomous
republic of Crimea by disguised Russian troops. 20) What could be the most possible reason for
On February 24, 2022, Russia launched a full- the Russian aggression in the above-mentioned
scale invasion of Ukraine. Although Russian statement?
forces made significant gains in the first days of I. Russia, considering the economic significance
combat, Ukrainian defenders rebuffed attempts of Ukraine, sought Ukraine’s membership in the
to seize Kyiv and other major cities and were Eurasian Economic Community (EAEC), which is
soon launching counterattacks at Russian a free trade agreement that came into being in
positions. Ukraine has become the bone of 2015, with its huge market and advanced
contention between Washington and Moscow. agriculture and industrial output, Ukraine was
Ukraine wants to join NATO which has not gone supposed to play an important role. But Ukraine
well with Russia and refused to join the some refused to join the agreement.
important agreement with Russia. II. NATO, led by the U.S., has planned to install
19) Which of the following cannot be inferred missile defence systems in Eastern Europe in
from the given statement? countries like Poland and the Czech Republic to
a) Russia thinks if Ukraine is allowed to join counter Russia’s intercontinental-range missiles.
NATO, the group would move closer to Russia's III. Maintaining strong relations with Ukraine
borders. serves India’s national interests. If not, there is
b) Russia wants the West to keep Ukraine and also an impact on the strong Indian diaspora
other former Soviet nations out of NATO, halt present in the region, threatening the lives of
weapons deployments near Russian borders and thousands of Indian students.
roll back forces from Eastern Europe. a) Only I follows
c) If Ukraine joins NATO, it is eligible to get b) Only II and III follow
support from the group's members in case of c) Only I and II follow
external attacks. d) Only III follow
e) All I, II and III

Click Here For Bundle PDF Course | support@guidely.in Page 6 of 12


SBI Clerk & RRB PO Mains PDF Course 2023
Reasoning Ability Day -5 (Eng)

Click Here to Get the Detailed Video Solution for the above given Questions
Or Scan the QR Code to Get the Detailed Video Solutions

Answer Key with Explanation


Directions (1-5):
1) Answer: D
2) Answer: B
3) Answer: E
4) Answer: C The words are arranged in alphabetical order

5) Answer: A based on the last letter of each word, if two

Input: 28 Inaugural Between 64 Team 93 52 words have the same last letter, then the words

Franchise 36 League 75 Match are arranged based on the second last letter of

Each number is divided by the lowest digit in the word.

each number, then based on the resultant In each step first, the number is arranged from

number, The numbers are arranged in the left end followed by the word is arranged

descending order from the left end, if both digits immediately next to that number.

are same then the number is divided by any one Step I: 93 Franchise 28 Inaugural Between 64

of the digits within the number. Team 52 36 League 75 Match

Ex) 28 here the lowest digit is two, so 28 is Step II: 52 League 93 Franchise 28 Inaugural

divided by two and the resultant is 14 Between 64 Team 36 75 Match

Similarly for other numbers, we get, 64/4=16, Step III: 64 Match 52 League 93 Franchise 28

93/3=31, 52/2=26, 36/3=12, 75/5=15 Inaugural Between Team 36 75

The resultants are 14, 16, 31, 26, 12, 15 Step IV: 75 Inaugural 64 Match 52 League 93

Based on the above resultant, the original Franchise 28 Between Team 36

numbers are arranged in descending order Step V: 28 Team 75 Inaugural 64 Match 52


League 93 Franchise Between 36

Click Here For Bundle PDF Course | support@guidely.in Page 7 of 12


SBI Clerk & RRB PO Mains PDF Course 2023
Reasoning Ability Day -5 (Eng)

Step VI: 36 Between 28 Team 75 Inaugural 64


Match 52 League 93 Franchise

Directions (6-10):
6) Answer: D
7) Answer: B
8) Answer: A (The first person is immediately
senior to the second person except “Q - The one
who likes Ackee”)
9) Answer: E
10) Answer: C
Again we have,
Final arrangement
 No one is designated between the one
who likes Durian and N, who does not like
Pomelo.
 The one who likes Pitaya is three persons
junior to N.
 The number of people senior to the one
who likes Pitaya is one more than the
number of people designated between K
and the one who likes Pomelo.
 Only one person is designated between O
and the one who likes Pomelo.
 As many people senior to O as junior to
We have, P.
 Only three people are designated After applying the above condition, Case1 gets
between the one who likes Santol and Q, eliminated. Because there is no possibility to
who is not junior to the AVM. place P. There is another possibility that is
 As many people senior to the one who Case2a.
likes Santol as junior to the one who likes
Durian.
From the above condition, there are three
possibilities

Click Here For Bundle PDF Course | support@guidely.in Page 8 of 12


SBI Clerk & RRB PO Mains PDF Course 2023
Reasoning Ability Day -5 (Eng)

 L is three people senior to the one who


likes Longan.
After applying the above condition, Case2a and
Case3 get eliminated. Because in Case2a there
is no possibility to place L and in case3 there is
no possibility to place Lychee. Hence, Case 2
shows the final arrangement.

Directions (11-15):
11) Answer: D
12) Answer: C
13) Answer: E
14) Answer: B
15) Answer: A
Teams are numbered from 1 to 8 respectively,
A=1, B=2, C=3, D=4, E=5, F=6, G=7 and H=8
Red Group selection:
Teams A and D are in the red group because we
Again we have,
cannot place these teams in the blue group. We
 Only one person is designated between P
need to select two teams from the other six
and the one who likes Lychee.
teams.
 The one who likes Ackee is junior to the
one who likes Lychee.

Click Here For Bundle PDF Course | support@guidely.in Page 9 of 12


SBI Clerk & RRB PO Mains PDF Course 2023
Reasoning Ability Day -5 (Eng)

 The sum of the numbers of the four teams playing against each other in the quarter-
in the red group should be a perfect finals.
square. From the above condition, in red group the
When we add H and C along with A and D,so possible combinations of teams which are
that we get the perfect square number. playing against each other are A and D, A and
A + D+ H + C = 1+4+8+3 = 16 H, A and C, D and H, H and C.
So, teams A, D, H and C are selected in the red Now consider the below condition,
group and the remaining teams are selected in  In Red group, the sum of the numbers of
the blue group. the two teams which are playing against
each other is a prime number.
A+D=5, A+H=9, A+C=4, D+H=12, H+C=11
So, A and D, H and C are playing against each
other.
Quarter finals:
Semi Finals:
In each group only two matches are conducted
 The team with the second lowest even
and each team is playing exactly one match and
number is qualified for the semifinals.
two winning teams in each group get qualified for
 Only one team with a vowel as a team
the semi-finals.
name is qualified for the semifinals.
 In each group the consecutive
From the above conditions,
alphabetically named teams are not
In red group D is qualified
playing against each other in the quarter-
In blue group E is qualified
finals.
 The team with the highest number is not
From the above condition,
selected for the semifinals.
In blue group the possible combinations of
 In the Blue team the team which has the
teams which are playing against each other are
lowest number is not selected.
B and E, E and G, B and G, B and F.
From the above conditions, C and F are selected
B is placed in three pairs. We need to select one
for the semifinals.
from it in order that the other two teams are
Finally, teams D, E, C and F are selected for the
different from the team which is playing against
semifinals.
B.
 In the semifinals, the teams which are in
So, E and G, B and F are playing against each
the same group do not play against each
other.
other.
 In each group the consecutive
alphabetically named teams are not

Click Here For Bundle PDF Course | support@guidely.in Page 10 of 12


SBI Clerk & RRB PO Mains PDF Course 2023
Reasoning Ability Day -5 (Eng)

 The consecutive alphabetically named banknotes of other denominations at any bank


teams are not playing against each other branch, it can be reduced the public burden.
in the semi-finals. Option (a): The central bank said the facility for
The possible combinations are C and E, D and the exchange of 2000 banknotes up to the limit
F. of 20,000 at a time will also be provided at its 19
From the above combinations, only CE and DF Regional Offices (ROs) having Issue
are possible pairs to play in the semifinals. Departments. (19 ROs only not 29) so does not
Finals: follow.
 In the finals, the teams which are in the Option (b): RBI said some particular bank
same group do not play against each branches will provide the facility of exchange of
other. 2000 banknotes into banknotes of other
 The sum of the number of the two teams denominations up to a limit of 20,000 in a day.
which are qualified and eliminated is (All the bank branches not some particular bank
equal. branches and exchange of other denominations
C+F=9; E+D=9 up to a limit of 20,000 at a time not in a day) so
 The team with a vowel as the name is not does not follow.
qualified for the finals. Option (c): RBI said all bank branches will
So, the pairs E and D are not qualified for the provide the exchange facilities for 2000
finals. banknotes until September 30, 2023. (it is a
C and F played against each other in the finals. deadline not an action to reduce the burden) so
 The team which belongs to red group does not follow.
does not win the tournament.
C belongs to red group. Hence, it is eliminated. 17) Answer: B
Thus, F won the tournament. A conclusion means an interpretation or a logical
judgment that can be reached after knowing the
16) Answer: D facts.
A course of action is a step taken to counter a After reading the passage, the government then
given situation, problem or circumstance, so that said the SBNs were being demonetised to fight
either it may completely solve the problem or it against corruption, black money and fake
may reduce the complexity of the problem and currency is the conclusion, so option (b) is
improve the situation. correct.
If the public may deposit 2000 banknotes into Option (a): It is a fact mentioned so that can’t be
their bank accounts and/or exchange them for a conclusion

Click Here For Bundle PDF Course | support@guidely.in Page 11 of 12


SBI Clerk & RRB PO Mains PDF Course 2023
Reasoning Ability Day -5 (Eng)

Option (c): It is not mentioned in the passage so 19) Answer: D


rule out All the options A, B, and C can be inferred.
Option (d): It is a fact mentioned so that can’t be Option D only cannot be inferred because as per
a conclusion the above passage, Ukraine not yet joined into
NATO. But the option mentioned, Ukraine
18) Answer: B already joined into NATO.
The performance of the individual has to be
tested over a span of time as the statement 20) Answer: C
mentions. So, I is not implicit. Statements I and II can be the possible reason
The statement mentions that the individual's for the war. Statement III explained about India
worth shall be reviewed (during probation period) and Ukraine relationship, it could not be the
before confirmation. So, II is implicit. possible reason for war.

Click Here For Bundle PDF Course | support@guidely.in Page 12 of 12


SBI Clerk & RRB PO Mains PDF Course 2023
Quantitative Aptitude Day -5 (Eng)

Quantitative Aptitude

Directions (01–04): Read the following e) Cannot be determined


information carefully and answer the questions
2) Find sale of Q on Thursday?
based on it.
a) 9
The data given below gives information about
b) 11
the sale of laptops in two outlets – P and Q of a
c) 10
company in five different days of a week (from
d) 8
Monday to Friday). Total sale of laptops in five
e) None of these
days together by outlet Q is Y.
• Sale of P on Monday is thrice as that Q on
3)
Monday, while sale of P on Friday is 8Z (Z is an
A = Difference between sale of P and Q on
integer).
Monday
• Sale of Q on Friday is 12 and sale of Q on
B = Difference between sale of P and Q on
Tuesday is 25% as that of P on Friday.
Wednesday
• Total sale on Monday is K which is same as
a) A > B
that of sale of P on Thursday. Sale of P on
b) A < B
Wednesday is prime number and total sale of P
c) A = B
is 130.
d) A + B < 27
• Sale of Q on Wednesday is same as average
e) None of these
sale of Q on five days together.
• Sale on Monday of each outlet is lowest
4) Which of the following is/are true?
among all days, while sale on Friday of each
I. sale of Q on Wednesday and Thursday is
day is highest among all days.
same
• Sale of P on each day is distinct and also total
II. Y = 45
sale on each day is also distinct. Sale on any
III. Difference between sale of P and Q on all
day by outlet P is more than 20 but not more
days together = 85
than 50.
a) II only
1) Find total sale of laptops on all five days
b) I and III only
together?
c) II and III only
a) 185
d) I, II, and III
b) 180
e) None of these
c) 175
d) 190

Click Here For Bundle PDF Course | support@guidely.in Page 1 of 12


SBI Clerk & RRB PO Mains PDF Course 2023
Quantitative Aptitude Day -5 (Eng)

Directions (05–07): Read the following information carefully and answer the questions based on it.
Arun Panchal is a Quant Content developer in GUIDELY for PO MAINS exams. The chart given below
shows the % distribution of total questions developed by Arun in four months April, May June, and July.
He developed three type of sets P, Q, and R (each set consists different number of question). Second
pie chart given below shows the % distribution of number of questions in one set of a particular type.

Note: It is not necessary that how many sets he made of each type in a particular month.
5) For July, number of type R sets developed are (7Y + 8Z), if he made at least one set of each
Z which is half as that of type Q set and number type.
of type P sets developed by Y. Find the value of a) 48

Click Here For Bundle PDF Course | support@guidely.in Page 2 of 12


SBI Clerk & RRB PO Mains PDF Course 2023
Quantitative Aptitude Day -5 (Eng)

b) 22 each company. Total employees in all five


c) 23 companies together are 800.
d) 30
e) None of these

6) For April, he made distinct number of sets of


each type (at least one set of each type) then
find total number of sets prepared in April month
if number of sets of type R is 6.
a) 14
Note:
b) 12
a) Total employees in HR department are 300,
c) 16
while total employees in company R and T is in
d) 10
ratio of 9:10 respectively.
e) None of these
b) Employees in IT department of company P is
50% more than that of T, while employees in IT
7) For June, number of type Q, R and P sets are
department of company Q is more any other
in arithmetic progression in same order, find
company in same department.
number of type P sets in month of June, if he
c) D is exact multiple of 10 and in each
developed at least one set of each type.
department of each company there is at least
a) 6
one employee. All variables in table are integers.
b) 4
8) Find difference between employees in HR
c) 7
department of company Q and R?
d) 5
a) 8
e) Can’t be determined
b) 14
c) 6
Directions (08–11): Read the following
d) 12
information carefully and answer the questions
e) None of these
based on it.
There are five companies – P, Q, R, S, and T.
9) For company T, Ratio of male and female
Each company has two departments – HR and
employees in HR department is (D + E): D, while
IT. The table given below shows the information
male employees in IT department is 8A% more
about the number of employees in each
than that of female employees. Find difference
company and employees in department B of

Click Here For Bundle PDF Course | support@guidely.in Page 3 of 12


SBI Clerk & RRB PO Mains PDF Course 2023
Quantitative Aptitude Day -5 (Eng)

between total male and female employees in the The table given below shows the partial
company? information about the articles (4 articles) – A, B,
a) 50 C, and D sold by a shopkeeper.
b) 70
c) 40
d) 60
e) None of these

10) For HR department, in company S, ratio of


Note: Discount given on article A is Q% while on
employees ages more than 30 years to that of
article D is 3P%.
less than 30 years is 25:16 respectively. Number
12) For article C, discount given is Rs. M and
of employees aged exactly 30 years in company
profit earned is Rs. 600. Find profit% earned on
R and S is same, find number of employees in
article C is if ratio of profit and discount earned is
company R aged not exactly 30 years?
4Q:5P?
a) 8
a) 60%
b) 5
b) 40%
c) 7
c) 50%
d) 9
d) 45%
e) Can’t be determined
e) None of these

11)
13) Ratio of discount offered on article D to that
A = difference between employees in HR and IT
of profit earned on article A is 15:4, find the value
department of company P and Q together.
of Z?
B = Difference between employees in HR and IT
a) Rs.200
department of company R and S together.
b) Rs.180
Find A:B?
c) Rs.300
a) 41:9
d) Rs.240
b) 41:19
e) None of these
c) 37:9
d) 37:17
14) Discount given on article B is Rs. 60, and
e) None of these
marked price of article C is 50% more than that
of B. if article C sold after two successive
Direction (12–15): Read the following information
carefully and answer the questions based on it.

Click Here For Bundle PDF Course | support@guidely.in Page 4 of 12


SBI Clerk & RRB PO Mains PDF Course 2023
Quantitative Aptitude Day -5 (Eng)

discounts of 30% and Rs. 130 respectively. Find 17) Bag P and Q contains black and white balls.
the selling price of article C. Bag P contains 3 black balls and Z white balls,
a) Rs. 500 while bag Q contains 5 white balls and (2Z + 1)
b) Rs. 600 black balls. If one ball chosen at random from
c) Rs. 540 any bag, then the probability that it is of black
d) Rs. 640 color is 11/ (4Z + 12). Find the value of Z.
e) None of these a) 3
b) 2
15) Selling price of article D is half as that cost c) 4
price of article E. Article E sold after profit % of d) 1
(2Q – 5)%. Find difference between selling price e) Can’t be determined
of article E and article A, if cost price of D is
same as that of B? 18) Arun invested Rs. Z under compound
a) Rs. 576 interest for two years at 50% per annum, and
b) Rs. 516 invested Rs. Y under simple interest for five
c) Rs. 408 years at 20% per annum. Interest earned on Rs.
d) Rs. 508 Z is Rs. 3000 more than that of Rs. Y. If he
e) None of these invested (Z + 2Y) under simple interest at 30%
per annum for two years, then he received total
16) P started a business with initial investment of amount of Rs. 12800 after two years.
Rs. Z and after Y months Q joined him with initial Which of the following is/are true.
investment of Rs. (Z + 300). After 2 months I. Z: Y = 2:1
(when Q joined) R joined them with initial II. Z + Y > 5000
invested of Rs. 400. At the end of the year total III. On dividing (difference between Z and Y)/10
profit is Rs. 87500, and profit share of P is Rs. by 13 remainder is 7.
30000, which is 50% more than that of R. Find a) II only
the value of (3Z + 800 + 200Y)? b) I and III only
a) Rs.2720& Rs.2450 c) II and III only
b) Rs.2660& Rs.2560 d) I and II only
c) Rs.2660& Rs.2520 e) None of these
d) Rs.2600& Rs.2650
e) Rs.2520& Rs.2660 19) P alone can do a job in Z days, while Q alone
can do it in Y days. If they working in alternate
manner starting with P, so task completed in

Click Here For Bundle PDF Course | support@guidely.in Page 5 of 12


SBI Clerk & RRB PO Mains PDF Course 2023
Quantitative Aptitude Day -5 (Eng)

14.5 days. If they working in alternate manner 20) A bag contains Rs. 74, in denominations of
(starting with Q) such that after every two days P Rs. 2, Rs. 5, and Rs. 10 notes. Total notes in
will work, so work completed in 17 days. If they bag are M and Rs.2 notes are twice as that of
work continues together, they can complete 6 Rs. 5 notes (Z), and Rs. 10 notes are Y.
times of original job in M days. Find the value of Which of the following is/are true.
M? I. M = 20
a) 44 days II. Y = prime number
b) 44/3 days III. (2Z + 5Y) = 64
c) 66 days a) I and II only
d) 22 days b) II and III only
e) None of these c) III and I only
d) I, II, and III
e) None of these
Click Here to Get the Detailed Video Solution for the above given Questions
Or Scan the QR Code to Get the Detailed Video Solutions

Answer Key with Explanation


Directions (01–04): So, sale of P on Thursday = 32
Sale of P on Friday on any outlet is highest. (8Z)
Sale of P and Q on Monday = 3:1
So, minimum possible sale of P on Friday = 8 x 5
Sale on any day > 20 and ≤ 50
= 40
Sale on Monday on each outlet is lowest as
So, sale of P on (Tuesday + Wednesday) = 130
compared on any other day.
– 40 – 32 – 24 = 34
So, sale on Monday can be = 21, 24……
This is not possible, because sale on Tuesday
If sale of P on Monday = 24
and Wednesday each > 24
So, sale of Q on Monday = 24/3 = 8
This is sure, sale of P on Monday = 21
So, total sale on Monday = 24 + 8 = 32

Click Here For Bundle PDF Course | support@guidely.in Page 6 of 12


SBI Clerk & RRB PO Mains PDF Course 2023
Quantitative Aptitude Day -5 (Eng)

So, sale of Q on Monday = 21/3 = 7 M = 50 – 27 – 10 = 13 (> 12, not possible)


So, sale of P on Thursday = 21 + 7 = 28 So, Y = 45
So, sale of P on (Tuesday + Wednesday) = 130 And M = 9
– 21 – 28 – 8Z = 81 – 8Z Now we are able to find all the data
If sale of P on Friday = 40
So, sale of P on Wednesday and Tuesday
together = 41
But sale of P on Wednesday is prime number (>
21). So minimum sale in Wednesday of P = 23
1) Answer: C
So, sale of P on Tuesday = 41 – 23 = 18 (< 21)
Total sale of laptops on all five days together =
So, sale of P on Friday = 32
175
Sale of P on (Tuesday + Wednesday) = 81 – 32
Hence answer is option C
= 49
So, sale of P on Wednesday must be = 23
2) Answer: A
Sale of P on Tuesday = 49 – 23 = 26
Sale of Q on Thursday = 9
Sale of Q on Tuesday = 32 x 25% = 8
Hence answer is option A
Sale of Q on Wednesday = Y/5
Let sale of Q on Thursday = M
3) Answer: C
Value of A = 21 – 7 = 14
Value of B = 23 – 9 = 14
So, A = B
Hence answer is option C
Sale of Q on Friday is maximum (12) and on
Monday is minimum. 4) Answer: D
So, minimum possible sale of Q on Wednesday I. sale of Q on Wednesday and Thursday is
and Thursday each = 7 same.
So, Y ≥ 12 + 8 + 7 + 7 + 7 = 41 (should be Sale of Q on Wednesday = Thursday = 9
multiple of 5) This statement is true.
So, if total sale of Q = 45 II. Y = 45
Sale of Q on Wednesday = 45/5 = 9 Required value = 45
So, M = 45 – 27 – 9 = 9 (possible) This statement is true.
If Y = 50 III. Difference between sale of P and Q on all
So, sale of Q on Wednesday = 50/5 = 10 days together = 85

Click Here For Bundle PDF Course | support@guidely.in Page 7 of 12


SBI Clerk & RRB PO Mains PDF Course 2023
Quantitative Aptitude Day -5 (Eng)

Required difference = 130 – 45 = 85


This statement is true 6) Answer: C
All statements are true. For April,
Hence answer is option D Let number of type P and

Q sets = Z and Y
Directions (05–07):
Now,
Number of questions in one set of type P = 20%
3Z+ 5Y + 2 x 6 = 40
x 10 = 2
3Z + 5Y = 28
Number of questions in one set of type Q = 50%
Z = (28 – 5Y)/3
x 10 = 5
If Y = 2, Z = 6 (not possible because type R sets
Number of questions in one set of type R = 30%
are 6)
x 10 = 3
If Y = 5, Z = 1 (only possible value)
Number of questions developed in April = 25% x
So, required sum = 6 + 5 + 1 = 12
120 = 30
Hence answer is option C
Number of questions developed in May = 20% x
120 = 24
7) Answer: D
Number of questions developed in June = 40% x
Let number of type Q, R, and P sets is a, b and c
120 = 48
respectively
Number of questions developed in July = 15% x
When a, b and c are in AP, then
120 = 18
(a + c) = 2b……………… (1)
5) Answer: B
Also,
According to question,
5a + 2b + 3c = 48……………... (2)
For July
From (1) and (2), we get
Number of type R sets = Z
5a + 3c + a + c = 48
So, number of type Q sets = 2 x Z = 2Z
6a + 4c = 48
So, number of type P sets = Y
3a + 2c = 24
Now,
Value of a = 8 – 2c/3
5 x 2Z + 2 x Z + 3 x Y = 18
If c = 3, a = 6. So, value of b = (6 + 3)/2 = 4.5
12Z + 3Y = 18
(not possible)
4Z + Y = 6
Both c and a must be even.
Only possible value of Z = 1, then Y = 2
On putting c = 12, a = 0 (not possible)
Required value = (7 x 2 + 8 x 1) = 22
So, only possible value of c = 6
Hence answer is option B
So, value of a = 4

Click Here For Bundle PDF Course | support@guidely.in Page 8 of 12


SBI Clerk & RRB PO Mains PDF Course 2023
Quantitative Aptitude Day -5 (Eng)

Value of b = (6 + 4)/2 = 5 So, that means value of C and B = 20 and 18


So, number of type R sets = 5 Value of A = 63 – 38 = 25
Hence answer is option D Value of D = 30 and E = 20
Now, we are able to calculate all data.
Directions (08–11):
Total employees in all companies together = 800
Employees in HR department of all companies
together = 300
So, employees in IT department of all companies
together = 800 – 300 = 500
Total employees in (Q + R + T) = 100 – 22 – 15 8) Answer: C
= 63% According to question,
So, Employees in HR department of Q = 50
Ratio of employees in R and T = 9:10 Employees in HR department of R = 44
Employees in IT department of company T = Required difference = 50 – 44 = 6
24% x 2/3 = 16% Hence answer is option C
So, employees in IT department of company T =
16% x 500 = 80 9) Answer: D
So, employees in IT department of company (Q For company T,
+ R) = 100 – 24 – 16 – 10 = 50% Ratio of male and female in HR = (30 + 20): 30 =
D + E = 50 5:3
Employees in IT department of Q (> 24%, and Ratio of male and females in IT = (100 + 25 x 8):
multiple of 10. 100 = 3:1
Possible value of D and E are = 30 and 20 , 40 So, total male employees = 5/8 x 80 + 3/4 x 80 =
and 10 respectively. 110
Employees in IT department of company T = 80 So, female employees = 160 – 110 = 50
So, 80/800 x 100 = 10% Required difference = 110 – 50 = 60
So, value of C > 10 Hence answer is option D
So, value of C = 20, 30
So, value of B = 18, 27 10) Answer: B
But minimum possible employees in IT For HR department
department of Q = 30% x 500 = 150 Ratio of employees in company S ages > 30
150/800 x 100 = 18.75% years and < 30 years = 25:16

Click Here For Bundle PDF Course | support@guidely.in Page 9 of 12


SBI Clerk & RRB PO Mains PDF Course 2023
Quantitative Aptitude Day -5 (Eng)

So, employees in S aged exactly 30 years = 70 –


41 = 39
So, employees in company R (in HR
department) aged not exactly 30 years = 44 – 39
=5
Hence answer is option B
12) Answer: C
According to question,
11) Answer: A For article C
Value of A = (120 + 150) - (56 + 50) = 164 Ratio of profit and discount = 4 x 25: 5 x 20 = 1:1
Value of B = (100 + 50) – (44 + 70) = 36 So, discount offered = profit earned
Required ratio = 164:36 = 41:9 Marked price of article = 2 x CP = 2CP
Hence answer is option A So, 2CP – CP = 600 + 600
So, cost price = 1200
Direction (12–15): Required % profit = 600/1200 x 100 = 50%
For Article A, Hence answer is option C
Ratio of cost price and marked price of article =
100:160 = 5:8 13) Answer: A
Discount % = Q% Selling price of A = 240 x 1.6 x .75 = 288
Profit % = P% Profit earned on A = 288 – 240 = 48
Now, Discount offered on D = 48/4 x 15 = 180
5 x (100 + P) = 8 x (100 – Q) So, marked price of D = 180/60 x 100 = 300
8Q + 5P = 300………………... (1) So, value of Z = 300 x 2/3 = Rs. 200
For Article D, Hence answer is option A
Ratio of Cost price and marked price = 100:150
= 2:3 14) Answer: A
Discount % = 3P% Selling price of B = 400 x 1.35 = Rs. 540
Loss % = (P + Q – 5)% Marked price of B = 540 + 60 = Rs. 600
Now, Marked price of C = 600 x 1.5 = Rs. 900
2 x (100 – P – Q + 5) = 3 x (100 – 3P) Cost price of C = 900/2 = Rs. 450
210 – 2P – 2Q = 300 – 9P Selling price of C = 900 x 70% - 130 = Rs. 500
7P – 2Q = 90……………. (1) Hence answer is option A
On solving both we get, P = 20 and Q = 25

15) Answer: C

Click Here For Bundle PDF Course | support@guidely.in Page 10 of 12


SBI Clerk & RRB PO Mains PDF Course 2023
Quantitative Aptitude Day -5 (Eng)

Cost price of B = 400 White balls in bag Q = 5


Selling price of D = 400 x 60% = 240 Now,
Cost price of E = 240 x 2 = 480 1/2 x [3/ (Z + 3) + (2Z + 1) / (2Z + 6)] = 11/ 2 x
Selling price of E = 480 x 1.45 = 696 (2Z + 6)
Selling price of A = 240 x 6/5 = 288 [3 x (2Z + 6) + (2Z + 1)(Z + 3)] / [(Z + 3)(2Z + 6)]
Required difference = 696 – 288 = Rs.408 = 11/ (2Z + 6)
Hence answer is option C Z2 + Z – 6 = 0
Z = - 3 and 2
16) Answer: D So, value of Z = 2
Ratio of profit share of P, Q, and R = Z x 12: (Z + Hence answer is option B
300) x (12 –Y): (400) x (10 – Y)
Profit share of P = 30000/87500 = 12/35 18) Answer: D
Profit share of R = 8/35 Compound interest earned on Rs. Z = Z x (3/2 x
Profit share of Q = 15/35 3/2 – 1) = 5Z/4
So, ratio of profit share of P, Q, and R = 12:15:8 Simple interest earned on Rs. Y = Y x 20% x 5 =
Now, Rs. Y
12 x Z / 400 x (10 – Y) = 3/2 Now,
Z = 50 x (10 – Y)……………… (1) 5Z/4 – Y = 3000…………... (1)
Also, Also,
12 x Z / (Z + 300) (12 – Y) = 12/15 (Z + 2Y) x (100 + 30 x 2) % = 12800
15Z = (Z + 300) x (12 – Y)………… (2) Z + 2Y = 8000…………. (2)
From (1) and (2), we get From (1) and (2), we get
Y2 – 13Y + 42 = 0 Z = 4000 and Y = 2000
Y = 6 and 7 I. Z: Y = 2:1
So, Z = 150 and 200 Required ratio = 4000:2000 = 2:1
Required value = (3 x 150 + 800 + 200 x 7) and This statement is true.
(3 x 200 + 800 + 200 x 6) = 2650 and 2600 II. Z + Y > 5000
Hence answer is option D Z + Y = 6000 > 5000
This statement is true.
17) Answer: B III. On dividing (difference between Z and Y)/10
Black balls in bag P = 3 by 13 remainder is 7.
White balls in bag P = Z Required remainder = 200/13 = 5
Black balls in bag Q = (2Z + 1) This statement is false

Click Here For Bundle PDF Course | support@guidely.in Page 11 of 12


SBI Clerk & RRB PO Mains PDF Course 2023
Quantitative Aptitude Day -5 (Eng)

So, only I and II is true. Hence answer is option A


Hence answer is option D
20) Answer: A
19) Answer: A According to question,
Let efficient of P and Q = P and Q respectively 2 x 2Z + 5Z + 10Y = 74
Now, 9Z + 10Y = 74
7.5 x P + 7 x Q = total work…………. (1) Possible value of Z = 6 and Y = 2
Also, I. M = 20
5 x P + 12 x Q = Total work…………... (2) M = 12 + 6 + 2 = 20 (true)
From (1) and (2), we get II. Y = prime number
7.5P + 7Q = 5P + 12Q Y = 2 = prime = True
2.5P = 5Q III. (2Z + 5Y) = 34
P/Q = 2/1 2 x 6 + 5 x 2 = 22 (false)
Total work = 2 x 5 + 12 x 1 = 22 units Only I and II is true.
Required time = 22 x 6/3 = 44 days Hence answer is option A

Click Here For Bundle PDF Course | support@guidely.in Page 12 of 12


SBI Clerk & RRB PO Mains PDF Course 2023
ENGLISH Day -5

English Language

Directions (1-5): Given below are a few A. knock


questions based on the blanks given in the B. perturb
paragraph. You have to choose from the options, C. mask
the correct word which fits in the blank. If none of D. detect
the options are correct then choose option E as E. None of the above
your answer.
An international team of researchers has 3. Which of the following options fits best in the
__________ a whopping list of 62 new moons that third blank?
had remained hidden in the past. They used the A. sequential
shift and stack technique to comb Saturn's B. consequential
surroundings to ___________ these previously C. trajectory
unseen objects. The method has been used for D. momentary
moon searches around Neptune and Uranus in E. None of the above
the past. Researchers from The University of
British Columbia said that shifting a set of 4. Which of the following options fits best in the
__________ images at the rate that the moon is fourth blank?
moving across the sky results in the A. faint
enhancement of the moon's signal when all the B. majestic
data is combined, allowing moons that were too C. echoic
________ to be seen in individual images to D. mystic
become visible when the images are _________. E. None of the above
1. Which of the following options fits best in the
first blank? 5. Which of the following options fits best in the
A. bundled fifth blank?
B. eschewed A. maintained
C. compiled B. perused
D. prevented C. marked
E. None of the above D. stacked
E. None of the above
2. Which of the following options fits best in the
second blank? Directions (6-10): Given below are a few
questions with three highlighted words that may

Click Here For Bundle PDF Course | support@guidely.in Page 1 of 7


SBI Clerk & RRB PO Mains PDF Course 2023
ENGLISH Day -5

or may not be inappropriate as per the context of allegations by the US Ambassador that the
the sentence. You have to mark that word as country (b) supplied weapons to Russia, it was
your answer which is inappropriately used as per announced on Thursday.
the context of the sentence. If none of the This comes after Brigety earlier (c) exclaimed at
options are correct then choose option E as your a media conference in Pretoria that a Russian
answer. ship had collected arms from South Africa in
6. The populist approach of Pheu Thai and its (a) December last year.
successor has been so successful that rival A. Only a
forces that once (b) derided it as vote-buying - B. Only b
military-backed Palang Pracharat and Prayuth's C. Only c
United Thai Nation - now offer (c) strikingly D. Both a&c
similar policies. E. None of the above
A. Only a
B. Only b 9. India is planning a (a) repatriation campaign
C. Only c for (b) artificial dating back to the colonial era,
D. Both a&b including the controversial Kohinoor diamond
E. None of the above and other idols and (c) sculptures in museums
across the UK, according to a British media
7. Erdogan, 69, is a powerful (a) orator and report on Saturday.
master campaigner who has pulled out all the A. Only a
stops on the campaign trail as he battles to B. Only b
survive his toughest political test. He commands C. Only c
(b) fierce loyalty from pious Turks who once felt D. Both a&c
(c) disenfranchised in secular Turkey and his E. None of the above
political career has survived an attempted coup
in 2016, and numerous corruption scandals. 10. The Kohinoor, also known as Koh-i-Noor or
A. Only a mountain of light in Persian, was in the spotlight
B. Only b at last week's (a) oration with Queen Camilla
C. Only c averting a diplomatic row by choosing alternative
D. Both a&b diamonds for her (b) consort’s crown. The 105-
E. None of the above carat diamond was held by rulers in India before
landing in the hands of the East India Company
8. South African President Cyril Ramaphosa will from Maharaja Ranjit Singh’s treasury and then
institute an independent inquiry into (a)

Click Here For Bundle PDF Course | support@guidely.in Page 2 of 7


SBI Clerk & RRB PO Mains PDF Course 2023
ENGLISH Day -5

being presented to Queen Victoria following the D. no approach toward its territorial claims
(c) annexation of Punjab. E. None of the above
A. Only a
B. Only b 13. The Thai prime minister first came to power
C. Only c in a 2014 coup, ousting an elected government
D. Both a&b and suspending the country's Constitution
E. None of the above sparking________________.
A. with colours.
Directions (11-15): Given below are a few B. the new lesson of his life.
questions with each question containing a blank C. the dispute over Nile river.
to be completed in order to complete the D. nationwide unrest and violence
sentence and make it a contextually correct E. None of the above
sentence. If none of the options are correct then
choose option E as your answer. 14. _____________________for about three hours
11. John Shing-Wan Leung, who holds at the French presidential Elysee Palace — an
permanent residency in Hong Kong, was encounter kept under wraps until shortly before
detained on April 15, 2021, by the the Ukrainian leader’s arrival in Paris from
counterintelligence ______________. Germany on a French government jet, extending
A. made in the southeastern city of Suzhou. his multi-stop European tour.
B. performed in the southeastern city of Suzhou. A. Ukraine’s Volodymyr Zelenskyy and France’s
C. mastered in the southeastern city of Suzhou. Emmanuel Macron met
D. agency in the southeastern city of Suzhou. B. Ukraine’s capital kiev
E. None of the above C. The President of France and the president of
Ukraine which were sought to be responsible
12. Relations between Washington and Beijing D. The Russian tyranny over Ukraine
have hit a historical low amid disputes over E. None of the above
trade, technology, human rights and
China’s___________________________. 15. _______________________from a trove of
A. increasingly non aggressive approach toward U.S. intelligence leaks suggesting that Zelenskyy
its territorial claims has considered trying to capture areas in Russia
B. decreasingly aggressive approach toward its proper for possible use as bargaining chips in
territorial claims peace negotiations to end the war launched by
C. increasingly aggressive approach toward its Moscow in February 2022.
territorial claims A. The Washington Post cited previously sued

Click Here For Bundle PDF Course | support@guidely.in Page 3 of 7


SBI Clerk & RRB PO Mains PDF Course 2023
ENGLISH Day -5

B. The Washington Post cited previously C. contenders, paving, reign


undisclosed documents D. rivals, bestowed, monarch
C. The Washington Post cited previously which E. None of the above
were not
D. The Washington Post referred as 18. A nearly 150-year-old ____________-glass
E. None of the above church window that ____________ a dark-skinned
Jesus Christ interacting with women in New
Directions (16-20): Given below are a few _____________ scenes has stirred up questions
questions with three blanks each to be filled from about race, Rhode Island’s role in the slave trade
the given options in the same sequence as and the place of women in 19th century New
given. You have fill in the blanks with appropriate England society.
options. If none of the options are correct then A. sustained, depicts, obscene
choose option E as your answer. B. sustained, predicts, obscene
16. Khartoum and the ___________ cities of Bahri C. stained, predicts, testament
and Omdurman across the Nile's two branches D. stained, depicts, testament
have been the main theatre of ___________ along E. None of the above
with western Darfur __________ since the army
and the Rapid Support Forces paramilitary 19. The historic Roosevelt Hotel in midtown
started fighting on April 15. Manhattan ____________ three years ago, but it
A. undivided, conflict, monastery will soon be bustling again — reopening to
B. undivided, joy, monastery accommodate an __________ influx of asylum
C. adjoining, joy, province ____________ just as other New York City hotels
D. adjoining, conflict, province are being converted to emergency shelters.
E. None of the above A. shuttered, participated, provided
B. shattered, participated, provided
17. Thailand's main Opposition parties bested C. shattered, anticipated, seekers
other ______, including military parties, in the D. shuttered, anticipated, seekers
recently held general election, ___________ the E. None of the above
way for a ballot-led change in the nine-year
_____________ of the incumbent Prime Minister 20. Pakistan's ___________ prime minister Imran
Prayuth Chan-ocha. Khan hit out at the country's powerful army in his
A. rivals, bestowed, reign first ____________ after an Islamabad court set
B. contenders, paving, monarch him free, saying it should be ___________ of

Click Here For Bundle PDF Course | support@guidely.in Page 4 of 7


SBI Clerk & RRB PO Mains PDF Course 2023
ENGLISH Day -5

jumping into politics and could form its own C. farmer, address, ashamed
political party. D. farmer, recluse, ashamed
A. former, address, ashamed E. None of the above
B. former, recluse, ashamed
Click Here to Get the Detailed Video Solution for the above given Questions
Or Scan the QR Code to Get the Detailed Video Solutions

Answer Key with Explanation


1. Answer: C 3. Answer: A
Here, compiled fits best in the first blank as it Here, sequential fits best in the third blank as it
means to collect information and arrange it in a means following in order of time or place and
list and here it is said for researches. So, option here it is said in the context of set of images. So,
C is the correct answer. option A is the correct answer.
Eschewed means deliberately avoid using or
abstain from something. 4. Answer: A
Here, faint fits best in the fourth blank as it
2. Answer: D means lacking brightness as per the context of
Here, detect fits best in the second blank as it the sentence as it is about the appearance of
means to notice or discover something that is moon. So, option A is the correct answer.
difficult to see, feel etc. and here it is used as the Majestic means beautiful and powerful.
context of the sentence is about previously
unseen objects mentioned. So, option D is the 5. Answer: D
correct answer. Here, stacked fits best in the fifth blank as it
Perturb means to make somebody worried or means full of piles or things and here piles of
upset. images are being talked. So, option D is the
correct answer.

Click Here For Bundle PDF Course | support@guidely.in Page 5 of 7


SBI Clerk & RRB PO Mains PDF Course 2023
ENGLISH Day -5

given on ceremonial occasion whereas


6. Answer: A coronation means the formal investiture of a
Here, successor used is inappropriate as use of monarch with their regal powers and here latter
predecessor will be appropriate because the fits best. So, option A is the correct answer.
person here being asked were present in past
and predecessor means people or previous 11. Answer: D
generations. So, option A is the Here, the person mentioned would have been
correct answer. detained by a agency and the word agency is
used only in option D. So, option D is the correct
7. Answer: E answer.
Here, none of the words highlighted are
inappropriate. So, option E is the correct answer. 12. Answer: C
Orator means person with good speaking skills. Here, the rising dispute is being talked here as
Disenfranchised means to take away per the context of the sentence and as per this,
somebody’s rights. increasingly aggressive approach fits best rather
8. Answer: C than other options. So, option C is the correct
Here, exclaimed used is inappropriate and answer.
claimed should have been used here. Exclaimed
means cry out suddenly in surprise where claim 13. Answer: D
means to say something is true and here latter Here, the context of the sentence is about a Thai
fits best. So, option C is the correct answer. PM and how he ousted an elected government
and as per the options nationwide unrest and
9. Answer: B violence fits best after ousting a elected
Here, artificial used is inappropriate and government. So, option D is the correct answer.
artefacts should have been used here as
artificial is the adjective for man made whereas 14. Answer: A
artefacts means an object that is made by Here, the meeting of Zelensky and Emmanual
person. So, option B is the correct answer. macron will be correct option to be used in the
blank as per the context of the sentence. So,
10. Answer: A option A is the correct answer.
Here, oration used is inappropriate and
coronation should have been used here as 15. Answer: B
oration means a formal speech especially one

Click Here For Bundle PDF Course | support@guidely.in Page 6 of 7


SBI Clerk & RRB PO Mains PDF Course 2023
ENGLISH Day -5

Here, as per the context of the sentence the means to leave a mark on something, depicts
undisclosed documents cited by the Washington should have been used in the second blank, and
post will be correct as per the context of the testament in the final blank as it means
sentence as the effort to end the war is being something that shows that something exists or is
talked. So, option B is the correct answer. true. So, option D is the correct answer.

16. Answer: D 19. Answer: D


Here, as per the context of the sentence which is Here, shuttered fits best in the first blank as it
about conflict the first blank should be filled with means to close and here a hotel that was closed
adjoining which means next to something, and three years ago is being talked about,
the second blank should be filled with conflict anticipated fits best in the second blank as it
and the final blank with the province as it means means expect or predict and seekers fits best in
a country or region brought under the control of the final blank as it means someone who is
the government. So, option D is the correct trying to find something. So, option D is the
answer. correct answer.

17. Answer: C 20. Answer: A


Here, contenders should be used in the first Here, the former fits best in the first blank as it
blank as it means a person who may win the means holding position in the past and the
competition, and here election is the context, the second blank should be filled with the address
second blank should be filled with paving and which means to give a speech to the public or
the final blank with reign which means the period nation and the final blank should be filled with
of rule. So, option C is the correct answer. ashamed as it is already given in all options and
also the feeling or shame can also be assumed
18. Answer: D as per the context of the sentence. So, option A
Here, as per the context of the sentence, stained is the correct answer.
should have been used before glass as stained

Click Here For Bundle PDF Course | support@guidely.in Page 7 of 7


SBI Clerk & RRB PO Mains PDF Course 2023
Reasoning Ability Day - 6 (Eng)

Reasoning Ability
Directions (1-5): Study the following information b. Only II
carefully and answer the given questions. c. Only III
A certain number of persons are sitting in a linear d. Only I and II
row facing north. Only two persons sit between I e. Only II and III
and U, who sits five places away from Q. Q sits
fourth to the right of G. The number of persons 3) What is the position of G with respect to M?
sitting between I and Q is one more than the a. 12th to the left
number of persons sitting between I and D, who b. 10th to the right
does not sit to the right of I. The one who sits c. 14th to the left
third to the right of T sits sixth to the right of G, d. 13th to the right
who sits immediate left of S. Only two persons sit e. 19th to the left
between T and O. The number of persons sitting
between F and O is one less than the number of 4) As many persons sit between G and F as
persons sitting between Q and S. Only five between ____ and ___.
persons sit between F and M. The number of a. D, U
persons sitting between U and T is one more b. The one who sits second to the right of V, O
than the number of persons sitting between V c. The one who sits immediate left of S, M
and M, who sits to the right of V. d. I, The one who sits second to the left of O
1) If D and M sit at the extreme ends of the row, e. U, Q
then how many persons are sitting in the row?
a. 24 5) How many persons sit between F and the one
b. 20 who sits second to the right of Q?
c. 32 a. One
d. 23 b. Two
e. 26 c. Four
d. Three
2) Which of the following statement(s) is/are e. Six
TRUE as per the given arrangement?
I. Q sits immediate right of T. Directions (6-10): Study the following information
II. U sits to the left of I. carefully and answer the given questions.
III. As many persons sit to the left of U as to the Eight persons – P, Q, R, S, T, U, V and W are
right of O. attending the concert in eight different months –
a. Only I January, February, March, April, May, July,

Click Here For Bundle PDF Course | support@guidely.in Page 1 of 11


SBI Clerk & RRB PO Mains PDF Course 2023
Reasoning Ability Day - 6 (Eng)

September and December of the same year. b. S


Each of them likes different musical genre viz. c. The one who likes Jazz
Folk, Pop, Jazz, Punk, Disco, Techno, Metal and d. V
Reggae. e. The one who likes Techno
R attends the concert in the month having less
than 31 days. One month gap between the 8) How many months between W and the one
months in which R and the one who likes Pop who likes Metal attend the concert?
attend the concert. As many persons attend a. Three
before the one who likes Pop as after W. The b. Five
one who likes Reggae attends two months c. Two
before W. The one who likes Reggae and T d. One
attend in the month having different number of e. None
days. The one who likes Disco attends
immediately before T. The number of persons 9) P likes which of the following musical genre?
attending after the one who likes Disco is one a. Metal
more than the number of persons attending b. Punk
before the one who likes Folk, which is not liked c. Jazz
by R. The one who likes Jazz attends three d. Reggae
months after U. Only three persons attend e. Pop
between U and S. Either S or R likes punk. V
attends immediately after the one who likes 10) Who among the following person attends the
punk. Q, who likes Techno attends before P. V concert at first and last month?
doesn’t like metal. a. V, the one who likes Disco
6) Which of the following combination is correct? b. U, the one who likes Techno
a. U-Jazz c. R, the one who likes Folk
b. S-Punk d. U, the one who likes Metal
c. V-Pop e. T, the one who likes Jazz
d. W-Techno
e. P-Disco Directions (11-15): Study the following
information carefully and answer the given
7) The number of months between Q and the questions.
one who likes Reggae is three more than the Twelve persons - A, B, C, D, E, F, G, H, I, J, K
number of months between T and __. and L are sitting around a square table in such a
a. The one who likes Folk way that four of them are sitting at the corners

Click Here For Bundle PDF Course | support@guidely.in Page 2 of 11


SBI Clerk & RRB PO Mains PDF Course 2023
Reasoning Ability Day - 6 (Eng)

and facing the centre while two of them are neither stored 3l nor stored more quantity of milk
sitting in the middle of each side and facing away than G. A stored the milk in a quantity, which is
from the centre. multiple of 11. F stored less quantity of milk than
Arrangement 1 L. The total quantity of milk stored by B and F is
I sit third to the left of D. F sits second to the right 63l. C and J stored 3l less than and 3l more than
of I and sits at the corner of the table. Four F respectively.
persons sit between I and H. J sits immediate 11) Who among the following person stored 30l
right of C, who doesn’t sit at the corner. Four of milk?
persons sit between B and C, who does not sit a. A
opposite to B. H sits second to the left of J. The b. G
number of persons sitting between A and B is c. H
one less than the number of persons sitting d. I
between E and L, when counted from the right of e. F
A and left of E. A doesn’t sit adjacent to D. Four
persons sit between L and K, who does not sit in 12) Who among the following person sits second
the middle of the sides. to the left of the one who faces F in arrangement
Arrangement 2 3?
The persons sitting opposite to each other in a. L
arrangement 1 are interchanged their positions b. A
with each other. c. H
Arrangement 3 d. I
The persons stored different quantities of milk. e. B
Note-1: All the persons stored milk, which
quantity is a multiple of 3. 13) What is the position of L with respect to E in
Note-2: The quantity of milk stored by a person arrangement 3?
does not exceed 50l. a. Immediate left
K stored milk in a quantity, which is a multiple of b. Immediate right
7. The difference between the quantity of milk c. Second to the right
stored by K and H is 9l. I stored 6l more than H. I d. Third to the left
neither stored 48l of milk nor stored 39 l of milk. e. Second to the left
The total quantity of milk stored by D and G is
36l. G stored less quantity of milk than D, who 14) Four of the following five are alike in a certain
stored 9l less than I. The difference between the way based on arrangement 3 and thus form a
quantity of milk stored by G and L is 15l. E

Click Here For Bundle PDF Course | support@guidely.in Page 3 of 11


SBI Clerk & RRB PO Mains PDF Course 2023
Reasoning Ability Day - 6 (Eng)

group. Which one of the following does not Note-III:Area of each flat on each floor is equal.
belong to the group? Note-IV: Only three persons live on each floor
a. H, B and only one person lives in each flat.
b. D, G J lives to the west of K, who doesn’t live in fat Y.
c. F, K Two floors are there between K and P, who lives
d. L, C on an odd numbered floor. P does not live in the
e. A, J same type of flat as J. I lives immediately above
T’s flat. N lives to the east of T. O lives to the
15) Who among the following person stored the immediate west of I, who lives in the same type
highest and lowest quantity of milk respectively? of flat as S. N doesn’t live on an odd numbered
a. B, E floor. As many persons live to the west of R as
b. B, D to the east of S. M and Q live in the same type of
c. E, L flat. As many floors below M as above R.
d. C, H 16) L lives to the south of _____.
e. K, J a. The one who lives on the 3rd floor
b. The one who lives to the west of T
Directions (16-20): Study the following c. S
information carefully and answer the given d. The one who lives to the east of R
questions. e. M
Twelve persons – I, J, K, L, M, N, O, P, Q, R, S
and T live on four different floors of a four storey 17) How many floors are there above the one
building where the lowermost floor is numbered who lives to the south of T?
one and the floor immediately above it is a. None
numbered two and so on. b. One
Note-I: Each floor has three flats viz., Flat-X, c. Three
Flat-Y and Flat-Z, where Flat X is to the west of d. Two
Flat Y, which is west of Flat Z. e. Cannot be determined
Note-II:Flat Z of floor 2 is immediately above Flat
Z of floor 1 and immediately below Flat Z of floor 18) Who among the following person lives in the
3 and so on. Flat Y of floor 2 is immediately same flat as K?
above Flat Y of floor 1 and immediately below a. I
Flat Y of floor 3 and so on. Similarly, Flat X of b. Q
floor 2 is immediately above Flat X of floor 1 and c. S
immediately below Flat X of floor 3 and so on. d. R

Click Here For Bundle PDF Course | support@guidely.in Page 4 of 11


SBI Clerk & RRB PO Mains PDF Course 2023
Reasoning Ability Day - 6 (Eng)

e. L I. R lives in Flat Z.
19) O lives on ____ floor in flat ___. II. S lives on the same floor as P.
a. 3rd, X III. I lives to the southeast of J.
b. 2nd, Y a. Only I and II
c. 3rd, Z b. Only II and III
d. 1st, Y c. Only II
e. 4th, Y d. Only III
e. Only I
20) Which of the following statement(s) is/are
TRUE as per the given arrangement?
Click Here to Get the Detailed Video Solution for the above given Questions
Or Scan the QR Code to Get the Detailed Video Solutions

Answer Key with Explanation

Directions (1-5) From the above condition, there are four


1) Answer: E possibilities
2) Answer: A
3) Answer: C
4) Answer: D
5) Answer: A
Final arrangement

We have
 Only two persons sit between I and U,
who sits five places away from Q.
Again we have,

Click Here For Bundle PDF Course | support@guidely.in Page 5 of 11


SBI Clerk & RRB PO Mains PDF Course 2023
Reasoning Ability Day - 6 (Eng)

 Q sits fourth to the right of G.


 The number of persons sitting between I
and Q is one more than the number of
persons sitting between I and D, who
does not sit to the right of I.
From the above condition, the possibilities are

Directions (6-10):
6) Answer: B
7) Answer: E
8) Answer: C
9) Answer: A
10) Answer: D
Again we have, Final arrangement
 The one who sits third to the right of T sits
sixth to the right of G, who sits immediate
left of S.
 Only two persons sit between T and O.
 The number of persons sitting between U
and T is one more than the number of
persons sitting between V and M, who sits
to the right of V.
 Only five persons sit between F and M.
 The number of persons sitting between F
and O is one less than the number of
persons sitting between Q and S.
We have,
From the above conditions, case 1a, case 2
 R attends the concert in the month having
and case 2a get eliminated because we
less than 31 days.
cannot place O in case 1a, S in case 2 and in
 One month gap between the months in
case 2a M should sit to the right of V. Hence
which R and the one who likes Pop
case 1 shows the final arrangement
attends the concert.
From the above conditions, there are three
possibilities

Click Here For Bundle PDF Course | support@guidely.in Page 6 of 11


SBI Clerk & RRB PO Mains PDF Course 2023
Reasoning Ability Day - 6 (Eng)

Again, we have
 The one who likes Jazz attends three
months after U.
 Only three persons attend between U and
S.
 Either S or R likes punk.
 V attends immediately after the one who
likes punk.
Again, we have
 Q, who likes Techno attends before P.
 As many persons attend before the one  V doesn’t like metal.
who likes Pop as after W. After applying the above condition case-3 gets
 The one who likes Reggae attends two eliminated because V likes Metal, hence case-2
months before W. shows the final arrangement.
 The one who likes Reggae and T attend
in the month having different number of
days.
 The one who likes Disco attends
immediately before T.
 The number of persons attending after the
one who likes Disco is one more than the
number of persons attending before the
one who likes Folk, which is not liked by
R.
After applying the above conditions case-1 gets
eliminated because R likes Folk.
Directions (11-15)
11) Answer: C
12) Answer: B
13) Answer: D
14) Answer: E (In the given pair of persons
sitting opposite to each other except option e)
15) Answer: A
Final arrangement

Click Here For Bundle PDF Course | support@guidely.in Page 7 of 11


SBI Clerk & RRB PO Mains PDF Course 2023
Reasoning Ability Day - 6 (Eng)

Again we have,
 The number of persons sitting between A
and B is one less than the number of
Arrangement 1
persons sitting between E and L, when
We have
counted from the right of A and left of E.
 I sit third to the left of D.
 Four persons sit between L and K, who
 F sits second to the right of I and sits at
does not sit in the middle of the sides.
the corner of the table.
 A doesn’t sit adjacent to D.
 Four persons sit between I and H.
From the above condition, case 2 gets
From the above condition, there are two
eliminated because we cannot place A and E.
possibilities
Hence Case-1 shows the final arrangement.

Again we have,
 H sits second to the left of J. Arrangement 1

 J sits immediate right of C, who doesn’t sit


at the corner.
 Four persons sit between B and C, who
does not sit opposite to B.
From the above condition, the possibilities are

Arrangement 2

Click Here For Bundle PDF Course | support@guidely.in Page 8 of 11


SBI Clerk & RRB PO Mains PDF Course 2023
Reasoning Ability Day - 6 (Eng)

 The persons sitting opposite to each other  G stored less quantity of milk than D, who
in arrangement 1 are interchanged in their stored 9l less than I.
position.  The difference between the quantity of
From the above condition, the persons changed milk stored by G and L is 15l.
their position  E neither stored 3l nor stored more
quantity of milk than G.
From the above condition, case 1 gets
eliminated because G stored more quantity of
milk than D.

Arrangement 3
We have,
Again we have,
 K stored milk in a quantity, which is a
 A stored milk in a quantity, which is
multiple of 7.
multiple of 11.
 The difference between the quantity milk
 F stored less quantity of milk than L.
stored by K and H is 9l.
 Total quantity of milk stored by B and F is
 I stored 6l more than H.
63l.
 I neither stored 48l of milk nor stored 39 l
 C and J stored 3l less than and 3l more
of milk.
than F respectively.
From the above condition, there are two
From the above condition, case 1a shows the
possibilities
final arrangement.

Again we have,
 Total quantity of milk stored by D and G is
36l.

Directions (16-20)

Click Here For Bundle PDF Course | support@guidely.in Page 9 of 11


SBI Clerk & RRB PO Mains PDF Course 2023
Reasoning Ability Day - 6 (Eng)

16) Answer: D
17) Answer: C
18) Answer: E
19) Answer: A
20) Answer: B
Final arrangement

We have,
 J lives to the west of K, who doesn’t live in
fat Y.
 Two floors are there between K and P,
who lives on an odd numbered floor.
 P does not live in the same type of flat as
Again we have,
J.
 I live immediately above T’s flat.
From the above condition, there are four
 N lives to the east of T.
possibilities
 O lives to the immediate west of I, who
lives in the same type of flat as S.
 N doesn’t live on an odd numbered floor.
From the above condition, the possibilities are

Click Here For Bundle PDF Course | support@guidely.in Page 10 of 11


SBI Clerk & RRB PO Mains PDF Course 2023
Reasoning Ability Day - 6 (Eng)

From the above condition, case 1, case 1a, case


2a get eliminated because we cannot place R.
Hence Case-2 shows the final arrangement.

Again we have,
 As many persons lives to the west of R as
to the east of S.
 M and Q live in the same type of flat.
 As many floors below M as above R.

Click Here For Bundle PDF Course | support@guidely.in Page 11 of 11


SBI Clerk & RRB PO Mains PDF Course 2023
Quantitative Aptitude Day - 6 (Eng)

Quantitative Aptitude

Directions (01 - 05): Study the following information carefully and answer the questions given below.
The given bar graphs show the total number of people who participated in three different sports [High,
long and triple jump] from six different complexes such as A,B,C,D,E and F.

The given table shows the percentage of the number of people who participated in the long jump out of
total number of people who participated from respective complex and the number of people who
participated in the high jump from those complexes

Note –The number of people who participated in the long jump from C is 60 and the ratio of the number of
people who participated in the long jump from A to in the high jump from B is 6:5.
1) The number of people who participated in the from A is 48 kg, 54 kg, and 62 kg, respectively,
high jump from A is 50% more than the number and the weight of each participants in the long,
of people who participated in the long jump from high, and triple jump from B is 52 kg, 56 kg, and
A. The number of people who participated in the 58 kg, respectively. Find the approximate
triple jump from B is 8x. The weight of each average weight of all the people who participated
participants in the long, high, and triple jump

Click Here For Bundle PDF Course | support@guidely.in Page 1 of 11


SBI Clerk & RRB PO Mains PDF Course 2023
Quantitative Aptitude Day - 6 (Eng)

in the all sports from complexes A and B total number of people who participated in the
together? high jump from G and D?
a) 57.36 a) 7
b) 58.24 b) 6
c) 54.51 c) 8
d) 49.35 d) 9
e) 51.33 e) None of these

2) The ratio of the number of people who 4) From D, 45% of the people participated, from
participated in the long jump to triple jump from C F 25% of the people participated and from C,
is 12:5. The ratio of the number of male to 50% of the people participated. If the total
female who participated in the long jump and number of people who did not participate from D,
triple jump from C is 3:2 and 3:2 respectively. C, and F together is [Ax+y] then find the value of
The average number of male who participated in A?
the long jump and high jump in C is 24. The total a) 233.5
number of male who participated from C is what b) 234.5
percent of total number of people who c) 258.3
participated from C? d) 267.2
a) 44.32% e) None of these
b) 54.32%
c) 57.32% 5) 37.5% of the total number of people
d) 52.50% participated in the triple jump from F. The ratio of
e) 59.32% the number of people who participated in the
long jump to discuss throw from F is 2:3. Find the
3) The number of people who participated in the number of people who participated in the high
triple jump from D is 10y. The number of people jump from F is what percent of the number of
who participated in the long jump from G is 25% people who participated in the discus throw from
more than the number of people who participated F?
from D in the long jump, and the number of a) 37.5%
people who participated in the triple jump from G b) 32.5%
is 20% more than that of D. If the total number of c) 36.5%
people who participated from G is 10% more d) 45.3%
than that of D, find the difference between the e) None of these

Click Here For Bundle PDF Course | support@guidely.in Page 2 of 11


SBI Clerk & RRB PO Mains PDF Course 2023
Quantitative Aptitude Day - 6 (Eng)

Directions (06 - 10): Study the following 7) In shop A, the marked the price of headphone
information carefully and answer the questions is ______% more than its cost price and sold at
given below. ______% discount. The selling price of
The ratio between the cost price of headphone in headphone in A is _______.
shops A and B is 11:13. The ratio of the cost Find which of the given option is true?
price of the charger and fitness band in shop B is a) 80, 20, Rs.722.25
1:2. The ratio of the cost price of the charger in b) 40, 5, Rs.792.25
shops A and B is 7:6. The cost price of the c) 75, 20, Rs.729.35
headphone in shop C is 33.33% less than the d) 60, 10, Rs.792
cost price of the charger in shop B. The cost e) 40,10, Rs.728
price of the charger in shop C is 75% more than
the cost price the headphone in shop C. 8) The cost price of the smart watch in shop B is
Difference of cost price of headphone and double the cost price of the headphone in the
charger(charger – headphone) in shop A is same shop. If each of the four products is sold at
Rs.290 and sum of the cost price of headphone a 25% profit in shop B, then find the difference
and charger in shop B is Rs.1370. The cost Price between the selling price of headphone and
of the fitness band in shop C is double the cost fitness band together and the selling price of the
price of the charger in the same shop. The cost charger and smart watch together?
price of the fitness band in shops C and A is the a) Rs.82.5
same. b) Rs.87.5
6) Find which of the following case profit c) Rs.69.5
percentage is more? d) Rs.72.5
I. When the headphone of shop A sold at e) None of these
Rs.680.
II. When the charger of shop B sold at Rs.860. 9) The selling price of headphone, charger, and
III. When the fitness band of shop C sold at fitness band in shop C is Rs. 840, Rs.1240, and
Rs.1980. Rs.1840 respectively. Find the overall profit
a) I and II are more percentage of shop C by selling all the products?
b) III is more a) 30.66%
c) I is more b) 32.55%
d) II is more c) 38.33%
e) All are the same d) 39.65%
e) 34.65%

Click Here For Bundle PDF Course | support@guidely.in Page 3 of 11


SBI Clerk & RRB PO Mains PDF Course 2023
Quantitative Aptitude Day - 6 (Eng)

10) Shop A marked up the price of the fitness a) 10:9


band by 60% and then offered a 25% discount, b) 9:10
while Shop B marked up the price of the fitness c) 12:13
band by 40% and then offered a 10% discount. d) 13:12
Find the ratio between the selling prices of the e) None of these
fitness band in A to B.

Directions (11 - 15): Study the following information carefully and answer the questions given below.
The given table shows the number of orders received for pizza and information about the number of
orders received for burgers by different shops [A, B, C, D, and E] on Saturday.

Note –
I. The number of orders received for sandwiches in shops A, B, and C is 10%, 20%, and 30% more than
the orders received for burgers in the same shop on Saturday.
The ratio of orders received for sandwiches in shops D and B is 3:2, and the ratio of orders received for
sandwiches in shops C and E is 7:4 on Saturday.
11) The average number of orders received for d) 43
pizza in shops F and E is 70 and the average e) 41
number of orders received for burgers in shops D
and F is 75 on Saturday. The average number of 12) The ratio of the number of orders received
orders received for burgers and sandwiches in for pizza, burgers, and sandwiches in the shop
shop F is 55 on Saturday. Find the difference on Saturday and Sunday is 5:6, 4:5, and 3:4,
between the total number of orders received in respectively, in shop B. The price of pizza,
shop F and A on Saturday? burgers, and sandwiches on Saturday in shop B
a) 42 is Rs. 160, Rs. 55, and Rs. 70, and the price of a
b) 49 pizza, burger, and sandwich on Sunday in shop
c) 40 B is Rs. 175, Rs. 60, and Rs. 75, respectively.

Click Here For Bundle PDF Course | support@guidely.in Page 4 of 11


SBI Clerk & RRB PO Mains PDF Course 2023
Quantitative Aptitude Day - 6 (Eng)

Find the total revenue earned by shop B on both e) None of these


days together.
a) 32540 15) The ratio of thin crust and cheese crust pizza
b) 36520 sold in shop A is 5:3 out of the total number of
c) 35482 pizzas sold, and the ratio of the same in shop B
d) 38540 is 3:2. So, the total number of thin crust pizzas
e) 36540 sold in shops A and B together is what percent of
the total number of cheese crust pizzas sold in
13) Out of the total number of orders received for shops A and B together?
a burger in shops A, B, D, C, and E on Saturday, a) 157.142%
40%, 70%, 30%, 60%, and 60% are for veg b) 154.23%
burgers, and the rest are for nonveg burgers. c) 125.32%
Find the ratio between the total number of veg d) 142.33%
burgers to the total number of nonveg burgers e) None of these
sold in all shops together on Saturday
respectively? 16)
a) 163:147 Quantity I: A and B together complete work in 20
b) 158:147 days, B and C complete work in 30 days, and A
c) 162:143 and C complete work in 40 days. Find the
d) 147:158 number of days taken A, B, and C together to
e) None of these complete the work they work on alternative days
start with A, then B, and then C?
14) Number of sandwiches sold in Shops A and Quantity II: A complete the work in 20 days. The
B together is _____ less than the number of ratio of efficiency of A and B is 5:4. Ratio of
sandwiches sold in Shop C and Shop _____ efficiency of B and C is 7:5. Find the time taken
together on Saturday. to complete the work if B works with the help of A
I. 16, E and C on an alternative day start with A?
II. 26, D a) Quantity: I 3*Quantity: II
III.72, D b) Quantity: I > Quantity: II
Find which one is true? c) Quantity: I <3*Quantity: II
a) I and III are true d) Quantity: I ≤ Quantity: II
b) I and II are true e) Quantity I = Quantity II
c) only II is true
d) all true

Click Here For Bundle PDF Course | support@guidely.in Page 5 of 11


SBI Clerk & RRB PO Mains PDF Course 2023
Quantitative Aptitude Day - 6 (Eng)

17) Ratio of the speed of boat P and boat Q in km/hr. The speed of train A is 6/5th of the speed
still water is 5:3. Ratio of the speed of boat Q of train B.
and Rin still water is 4:7.Boat Q covers 36 km in d) None
3 hours of still water. The speed of the stream of e) More than one option is correct
the river A is 1/3rdof the speed of boat Q in still
water. The speed of the stream of River B is 25% 19) Find the age of A?
more than the speed of the stream of River A. a)The Age of A is 20% more than the age of B.
Quantity I: Find the distance travel by boat P in 5 The Age of C is 30% more than the age of D.
hours downstream in river A and 4 hours in river Sum of the age of A and D is 50 years old. The
B upstream. age of E is 5 more than the age of B.
Quantity II: Find the distance travel by boat R in b)The Age of A is 10 years less than the age of D
4 hours downstream in river A and 5 hours in who is 9 years younger than C. The Age of B is
river B upstream. 24 years. The age of E is 1 year younger than C.
a) Quantity: I Quantity: II c) None
b) Quantity: I > Quantity: II d) Sum of the age of A and B is 44 years. The
c) Quantity: I < Quantity: II average age of B and C is 27 years. The ratio of
d) Quantity: I ≤ Quantity: II age of A and C is 2:3. Difference between the
e) Quantity I = Quantity II age of A and E is 9 years.
e) More than one option is correct
Directions (18-20): Study the information
carefully and find from which of the given options 20) Find the rate of interest?
we can calculate the answer to the question. [ a) Difference between compound interest and
Answer may differ from each option] simple interest amount of the same interest rate
18) Find the length of train A? for 2 years is 50. The principle amount is
a)The Speed of train A is 20% higher than the Rs.1250.
speed of train B. The Ratio of the lengths of train b) If Rs.1640 invests for n years at some rate
A and B is 2:3. Train B covers a 140 m long simple interest then the interest amount is
bridge in 15 sec. Rs.656 and if the same amount invests for (n+1)
b)The Ratio of speed of train A and B is 4:5. years for 5% more interest rate then the interest
Train B crosses a tree in 5.33 sec and a bridge of amount is Rs.1230.
180 m in 11.33 sec. The length of train B is 160 c) Rs.p invest for 3 years compound interest at
m. some rate of interest is Rs.11438. Rs.12500
c) Train A crosses a tree in 5.33 sec and a bridge invest for the same interest rate for some years
of 180 m in 11.33 sec. The speed of train B is 90 then SI is 6250.

Click Here For Bundle PDF Course | support@guidely.in Page 6 of 11


SBI Clerk & RRB PO Mains PDF Course 2023
Quantitative Aptitude Day - 6 (Eng)

d) None e) More than one option is correct


Click Here to Get the Detailed Video Solution for the above given Questions
Or Scan the QR Code to Get the Detailed Video Solutions

Answer Key with Explanation

Directions (01 - 05): The number of people who participated in the


The number of participants in the long jump from triple jump from B = 8*2=16
C is 60. So, 120*25x/100=60 The number of people who participated in the
Or, x=2 triple jump from A = 80-24-36=20
So, the number of people participating in the The number of people who participated in the
high jump from B is 10*2=20 long jump from B = 60-16-20=24
So, the number of people participating in the So, average weight
long jump from A = 20*6/5=24 =[24*48+54*36+20*62+24*52+56*20+16*58]/140
So, 80*10y/100=24, y=3 =[1152+1944+1240+1248+1120+928]/140=54.5
1

2) Answer: D
The total number of people who participated in
the triple jump from C = 60*5/12=25
From C -
The number of male who participated in the long
jump = 60*3/5=36
1) Answer: C The number of female who participated in the
The number of people who participated in the long jump = 60*2/5=24
High Jump from A = 24*150/100=36 The number of male who participated in the triple
jump = 25*3/5=15

Click Here For Bundle PDF Course | support@guidely.in Page 7 of 11


SBI Clerk & RRB PO Mains PDF Course 2023
Quantitative Aptitude Day - 6 (Eng)

The number of female who participated in the 5) Answer: A


triple jump is = 25*2/5=10 Number of people participated in the triple jump
The number of male who participated in the high from F =[80*37.5/100]=30
jump = 24*2-36=12 The number of people participated in the long
Required percentage = jump from F = 80-30-18=32
[(36+15+12)/120]*100=52.5% The number of people participated in the discus
throw from F = 32*3/2=48
3) Answer: B Required percentage =[18/48]*100=37.5%
The number of people who participated in the
triple jump from D = 10*3=30 Directions (06 - 10):
The number of people who participated in the Let the cost price of headphone in shop A and B
long jump from D =90-30-24=36 is 11x and 13x respectively.
The number of people who participated in the The cost price of the charger in shops A and B is
long jump from G = 36*125/100=45 7y and 6y respectively.
The number of people who participated in the 7y-11x=290 and 13x+6y=1370
triple jump from G =30*120/100=36 By solving two equations we get x=50 and y=120
The number of people who participated in the The cost price of headphone in shops A and B is
high jump of G = 90*110/100-45-36=18 11*50=Rs.550 and 13*50=Rs.650 respectively.
So, the required difference = 24-18=6 The cost price of the charger in shops A and B is
7*120=Rs.840 and 6*120= Rs.720 respectively.
4) Answer: A The cost price of headphone in C is 720*(100-
The number of people who did not participate 33.33)/100= Rs.480
from D = [90/45]*55=110 The cost price of the charger in shop C =
The number of people who did not participate 480*175/100= Rs.840
from F = [80/25]*75=240 The cost price of the fitness band in shop C =
The number of people who did not participate 840*2= Rs.1680
from C =[120/50]*50=120 The cost price of the fitness band in shop A =
The total number of people who did not Rs.1680
participate from D, F, and C together is = The cost price of the fitness band in shop B =
110+240+120=470=Ax+y 720*2/1= Rs.1440
Or, 2A+3=470 6) Answer: C
Or, A=233.5 I. When the headphone of shop A sold at
Rs.680.

Click Here For Bundle PDF Course | support@guidely.in Page 8 of 11


SBI Clerk & RRB PO Mains PDF Course 2023
Quantitative Aptitude Day - 6 (Eng)

So, profit percentage = [680- 9) Answer: A


550]*100/550=23.63% Total selling price of all the products in C =
II. When the charger of shop B sold at Rs.860. [840+1240+1840]=3920
So, profit percentage = [860- Total cost price is = [480+840+1680]=3000
720]*100/720=19.44% So, profit percentage = [3920-
III. When the fitness band of shop C sold at 3000]*100/3000=30.66%
Rs.1980.
So, profit percentage = [1980- 10) Answer: A
1680]*100/1680=17.85% Selling price of fitness band is =
So, the option I is more 1680*160/100*75/100=2016
Selling price of fitness band is =
7) Answer: D 1440*140/100*90/100=1814.4
If we check option D, So, the required ratio =
The marked price of the headphone is 2016:1814.4=20160:18144=10:9
=550*160/100= Rs.880 Directions (11 - 15):
Selling price of headphone is =880*90/100=
Rs.792
So, it is true

8) Answer: B
The cost price of the smart watch in shop B =
650*2= Rs.1300
So, the selling price of headphones = 11) Answer: C

650*125/100= Rs.812.5 Orders received for pizza in shop F is = 70*2-

The selling price of charger = 720*125/100= 65=140-65=75

Rs.900 Orders received for burgers in shop F is = 75*2-

The selling price of fitness band = 60=90

1440*125/100= Rs.1800 Orders received for the sandwich in shop F is =

The selling price of smartwatch = 55*2-90=20

1300*125/100= Rs.1625 So, required difference = [75+90+20]-

So, difference = [1800+812.5]-[900+1625]= [40+50+55]=40

Rs.87.5
12) Answer: D

Click Here For Bundle PDF Course | support@guidely.in Page 9 of 11


SBI Clerk & RRB PO Mains PDF Course 2023
Quantitative Aptitude Day - 6 (Eng)

So, the number of orders received for pizza on Total crust pizza sold is =
Sunday is = 50*6/5=60 40*5/8+50*3/5=25+30=55
For burger is = 60*5/4=75 Total cheese crust pizza sold is =
Foor sandwich is = 72*4/3=96 40*3/8+50*2/5=15+20=35
So, the total revenue earned by shop B on both So, required percentage =
days together is [55/35]*100=157.142%
=50*160+60*55+72*70+60*175+60*75+96*75
=38540 16) Answer: B
Quantity I
13) Answer: B Let the total work is 120
The total number of veg burgers sold in all shops The total efficiency of [A and B], [B and C], and [
together is A and C] is 6,4 and 3 respectively.
=50*40/100+60*70/100+70*30/100+60*60/100+6 The efficiency of A is [13/2]-4=2.5
5*60/100 Of B=6.5-3=3.5
=20+42+21+36+39=158 Of C =6.5-6=.5
The total number of nonveg burgers sold in all So, in three days they did = [2.5+3.5+.5]=6.5 unit
shops together is work
= [50+60+70+60+65]-158=147 After 54 days they did =6.5*18=117 unit work
So, the required ratio = 158:147 On 55th day = 117 + 2.5 = 119.5
They finishes the work in = 55+[.5/3.5]=55.14
14) Answer: A days
Total sandwiches sold in shops A and B together Quantity II
is = 55+72=127 (i) B takes = 20*5/4=25 days
Total sandwiches sold in shops C and D C takes = 25*7/5=35 days
together is =108+91=199 (ii) Total work is = lcm of 20,25 and 35 = 700
Total sandwiches sold in shops C and E together The efficiency of A, B, and C is 35,28, and 20
is = 91+52=143 (iii) respectively.
So, the two difference between (i) and (ii) and In two days they did = [35+28]+[28+20]=111 unit
(iii) and (i) is 72 and 16 respectively. of work
So, I and III are true After 12 days they did = 666 units of work
Total work will complete in 12+[34/63]=12.53
15) Answer: A days
Quantity I > Quantity II

Click Here For Bundle PDF Course | support@guidely.in Page 10 of 11


SBI Clerk & RRB PO Mains PDF Course 2023
Quantitative Aptitude Day - 6 (Eng)

17) Answer: E 19) Answer: D


Speed of boat Q in still water = 36/3=12 km/hr From D, we can get the answer.
Speed of boat P is = 12*5/3=20 km/hr Let the age of A and C is 2x and 3x.
The speed of boat R is = 15*7/4=21 km/hr So, the age of B is 44-2x
The speed of the stream of river A is 12/3=4 So, 44-2x+3x=27*2=54
km/hr Or, x=10
The speed of a stream of river B is = Age of A is 20 years
4*125/100=5 20) Answer: E
Quantity I From a)
So, total distance = [20+4]*5+[20-5]*4=180 km We can get by formula difference of SI and CI =
Quantity II principle [R/100]2
Total distance = [21+4]*4+[21-5]*5=180 km From b)
Quantity I = Quantity II 1640*n*r/100=656
nr = 40
18) Answer: C And 1640*(n+1)*(r+5)/100 =1230
In option A, we cannot calculate the length of (n+1)(r+5) = 75
train B so we cannot calculate the length of train nr + r + 5 + 5n = 75
A r + 5n = 75 – 45 = 30
In option B, no information is given about the nr = 40
relation between train A and train b. 40/n + 5n = 30
In option C, we can get the answer. 5n2 - 30n + 40 =0
Speed of train A is 90*6/5=108 km/hr= N=4,2 so r= 10,20
108*5/18=30 m/sec We can get the answer by solving the equations
So, length/30 =5.33, length = 30*5.33=159.9 = So, option E is true.
160 m

Click Here For Bundle PDF Course | support@guidely.in Page 11 of 11


SBI Clerk & RRB PO Mains PDF Course 2023
ENGLISH Day - 6

English Language

Directions (1-7): Given below are a few a person puts mud in their eyes and starts
questions based on the passage. You have to rubbing it in. It can cause your eyes then become
read the passage carefully and answer the given red, irritated and watery," said Dr Gopal Pillai. To
questions accordingly. If none of the options are prevent dry eyes, wash your eyes frequently,
correct then choose option E as your answer. suggested Dr Pillai. "After about every few
Summer has its own set of infections that are minutes, voluntarily, close your eyes tightly and
spread rampantly if a proper lifestyle is not then open them. Watch your eyes more often as
followed. Our eyes go through a great deal of it will help lubricate the eyeballs," he said. To
stress and pressure, especially due to the combat dryness, use tear substitutes like eye
summer sun that is bright and scorching. Cases drops as well. Dr Sarang Goel, Medical Director,
of ocular diseases like conjunctivitis are Ayu Health Network of Hospitals, told
reportedly heavily in the hot months and taking IndiaToday.in that rehydration is important for
necessary precautions is of utmost importance. eye health.
Eyes are quite sensitive to the ultraviolet (UV) "Rehydration is extremely important in summer.
rays of the sun. In summer, prolonged exposure Avoid any chance of dehydration as it can affect
to UV modifies the lens proteins, leading to the eyes. While applying moisturiser, rub it
various problems like cataract formation and around your eyes. Wear a hat when out in the
worsening eyesight. UV can also damage the sun or use an umbrella. Most importantly do not
retina with a high risk of cancer like basal cell rub your eyes," said Dr Sarang Goel.
carcinoma and squamous cell carcinoma. Dr Another major problem in summer is ocular
Gopal S Pillai, Clinical Professor, Head of diseases like conjunctivitis, keratitis,
Opthalmology, Amrita Hospital, Kochi, told endophthalmitis, cellulitis and stye, among
IndiaToday.in people should wear UV others. Dr Pillai said that conjunctivitis is one of
sunglasses to protect their eyes. This will also the most common eye problems in the hot
prevent dryness in the eyes, caused by the sun months.
and more screen exposure. "Conjunctivitis spreads via human touch.
"Dryness is a major problem in eye health these Someone could use an infected towel or even a
days, especially among children who study for a door knob that has been touched by an infected
long time and those who are in front of the person. In a classroom, one child could infect
screens. Many people look at their screens for 8- another child if they have conjunctivitis. It's a viral
14 hours a day which can cause extreme infection, so it spreads rampantly," the expert
dryness. In summer, dryness is a feeling in which said.

Click Here For Bundle PDF Course | support@guidely.in Page 1 of 9


SBI Clerk & RRB PO Mains PDF Course 2023
ENGLISH Day - 6

Ideally, a healthy well balanced diet is essential D. restrained


for eye health along with that we must have E. None of the above
antioxidants like beta carotene, vitamins C and
E, Omega 3 and zinc. These help in preventing 2. Which of the following option is incorrect as a
the development of serious eye conditions. Avoid result of prolonged exposure to UV in the
unprocessed foods on a daily basis," Dietician summer?
Jyoti Khaniojh, Dietetics, Nutrition And Dietetics, A. UV can also damage hair follicles and causes
Max Super Speciality Hospital, Patparganj, New hairfall.
Delhi, told IndiaToday.in. B. UV can also damage the retina with a high risk
When we eat a meal plate, which is colourful like of cancer like basal cell carcinoma and
a rainbow, we get all sorts of antioxidants in our squamous cell carcinoma
diet. C. prolonged exposure to UV modifies the lens
Since our childhood, we have been taught that proteins, leading to various problems like
an apple a day keeps the doctor away. Likewise, cataract formation and worsening eyesight
carrots are known to be best for the eyes. It is D. Both C&D
rich in beta-carotene and vitamin, which protects E. None of the above
the eye against infections," Jyoti Khaniojh said.
Besides this, add lemon and citrus fruits to your 3. Which of the following option is the synonym
diet for vitamin C intake. Nuts and seeds that of “VOLUNTARILY”?
contain vitamin E prevent cataracts and age- A. willingly
related macular degeneration. B. reluctantly
Salmon fish is a rich source of omega-3 and this C. unwillingly
healthy fat helps in prevent eye dryness as it is D. unintentionally
great for retina health. E. None of the above
Egg yolks contain vitamin A, zinc, lutein and
zeaxanthin and zinc, which help in age-related 4. Which of the following is not one of the
degeneration of eyes. Zinc helps in night reasons given by Dr. Pillai to prevent dry eyes?
improvement of night vision. A. You should wash your eyes frequently
B. After about every few minutes, voluntarily,
1. Which of the following option is the antonym of close your eyes tightly and then open them
RAMPANTLY? C. Watch your eyes more often as it will help
A. unbridled lubricate the eyeballs
B. unhindered D. Wash your eyes with hygienic soap frequently
C. unhampered E. None of the above

Click Here For Bundle PDF Course | support@guidely.in Page 2 of 9


SBI Clerk & RRB PO Mains PDF Course 2023
ENGLISH Day - 6

5. Which of the following is not an example of World Bank chief David Malpass (c)/, an
ocular diseases? economist and former US Treasury official during
A. conjunctivitis the Trump administration (d). No error (e).
B. cellulitis A. a
C. endophthalmitis B. b
D. orthopthalmitis C. c
E. None of the above D. d
E. e
6. Why does an infected towel or even a door
knob is susceptible to conjunctivitis? 9. While the rapidly growing Indian aviation
A. Because it spreads only via human touch sector offers ample opportunities (a)/ for private
B. Because it spreads only via clothes carriers to launch operations (b)/, he remains an
C. Because it spreads only via saliva unforgiving space (c)/, considering the number of
D. Because it is a viral infection and it spreads airlines that have been forced to shut shop due
rampantly to financial troubles (d). No error (e).
E. None of the above A. a
B. b
7. Which of the food item is not mentioned by Dr. C. c
jyoti khanioj for a healthier eye? D. d
A. carrot E. e
B. lemon juice
C. selmon fish 10. In November 2019 (a)/, the bankruptcy court
D. zinga fish ordered a corporate insolvency resolution
E. None of the above process for the grounded regional low-cost air
carrier, Air Costa (b)/, aviation regulator
Directions (8-12): Given below are a few Directorate General of Civil Aviation (DGCA) in
questions divided into five parts which may or June 2017 suspended the flying license for Air
may not contain errors in them. You have to find Costa (c)/, making it permanently stop operations
the part containing errors in them. If none of the (d). No error (e).
parts contains error then choose option E as A. a
your answer. B. b
8. Banga, 63, was nominated for the post by US C. c
President Joe Biden (a)/ in late February and D. d
was the sole contender (b)/ to replace departing E. e

Click Here For Bundle PDF Course | support@guidely.in Page 3 of 9


SBI Clerk & RRB PO Mains PDF Course 2023
ENGLISH Day - 6

11. Sahara Airlines was first established on Sept 13. Disney+ lost a (a) whopping 4 million
20, 1991 (a)/, and later begun its operation after subscribers in just the second quarter. The
2 years on December 3, 1993 (b)/. Two Boeing company (b) indicated that it could lose more
737-200 aircraft served as Sahara Airlines (c)/. subscribers in the current quarter.
On October 2, 2000, the airline was rebranded to One of the reasons behind growing losses could
Air Sahara, it controlled 12 per cent of India's be the sharp fall in subscribers in the South Asia-
domestic flight market at its peak (d). No error focused Disney+ Hotstar after it lost the (c)
(e). streaming rights to the popular Indian Premier
A. a League (IPL) cricket matches.
B. b A. Only a
C. c B. Only b
D. d C. Only c
E. e D. Both a&b
E. None of the above
12. The World Bank has been led by an
American (a)/ since its founding at the end of 14. While Go First had (a) blamed P&W for its
World War Two (b)/, while the International financial troubles and bankruptcy filing, the US
Monetary Fund has been led by an European engine maker said earlier that the airline’s (b)
(c)/. Banga, who was born in India and spent his claim was “unfounded” as it had not paid for the
early career there, has been a US citizen since maintenance and lease charges for years, which
2007 (d). No error (e). (c) penultimately led to the necessary
A. a suspension of services.
B. b A. Only a
C. c B. Only b
D. d C. Only c
E. e D. Both a&c
E. None of the above
Directions (13-16):Given below are a few
questions with three words highlighted which 15. While a (a) surge in traffic in India, the
may or may not used appropriately as per the world’s third-biggest aviation market in the world,
context of the sentence. You have to find the led to record jet orders, the (b) failure of two
word which is used out of the context of the major airlines – Kingfisher Airlines in 2012 and
sentence. If all words are appropriately used Jet Airways in 2019 – have made lessors (c)
then choose option E as your answer. comprehensive about the market.

Click Here For Bundle PDF Course | support@guidely.in Page 4 of 9


SBI Clerk & RRB PO Mains PDF Course 2023
ENGLISH Day - 6

A. Only a B. who expect lower food prices to aid the rise


B. Only b C. who expect lower food prices to add the rise
C. Only c D. who expect lower food prices to aid the
D. Both a&b decline
E. None of the above E. No improvement is required

16. The Go First bankruptcy decision has also 18. Taiwanese electronics giant Foxconn has
made lessors nervous about another financially- bought a huge tract of land on the outskirts of
troubled airline, SpiceJet. While SpiceJet said it Indian tech hub Bengaluru, the key Apple
has no plans of filing for insolvency, lessors are supplier said in a filing as it looks to diversify
worried that a similar move by the airline could production towards China.
(a) spoil their (b) respect of (c) repossessing A. he looks to diversify production away from
planes. China
A. Only a B. she looks to diversify production away from
B. Only b China
C. Only c C. them looks to diversify production toward
D. Both b&c China
E. None of the above D. it looks to diversify production away from
China
Directions (17-20): Given below are few question E. No improvement is required
with a highlighted phrase which may or may not
need improvement. You have to find the correct 19. Apple have making its own push into India
replacement of the phrase if required. If no and chief executive Tim Cook last month opened
improvement is required then choose option E as its first two retail stores in the world's most
your answer. populous country.
17. India's headline retail inflation in May is likely The California-based firm is betting big on the
to fall further towards 4% — the midpoint of the nation of 1.4 billion people -- home to the
central bank's target and a level last seen in second-highest number of smartphone users in
January 2021, according to a few economists, the world, after China.
which expect lower food prices to add the A. Apple have been making its own push into
decline. India
A. who expect lower food prices to add the B. Apple have been made its own push into India
decline C. Apple has been making its own push into
India

Click Here For Bundle PDF Course | support@guidely.in Page 5 of 9


SBI Clerk & RRB PO Mains PDF Course 2023
ENGLISH Day - 6

D. Apple has been made its own push into India shares were pummelled earlier this year by a US
E. No improvement is required short-seller's report.
A. Adani Ports has been floated a tender of up to
20. Adani Ports had floated a tender of up to B. Adani Ports had been floated a tender of up to
$130 million of 3.375% 2024 maturity dollar- C. Adani Ports was floated a tender of up to
denominated bonds late last month, as it seeks D. Adani Ports were floated a tender of up to
to boost investor confidence after the group's E. No improvement is required

Click Here to Get the Detailed Video Solution for the above given Questions
Or Scan the QR Code to Get the Detailed Video Solutions

Answer Key with Explanation

1. Answer: D carcinoma. Dr Gopal S Pillai, Clinical Professor,


Here, rampant means showing no sign of being Head of Opthalmology, Amrita Hospital, Kochi,
under control and restrained means just the told IndiaToday.in people should wear UV
opposite to it whereas all other options are sunglasses to protect their eyes. This will also
synonyms of rampant. So, option D is the correct prevent dryness in the eyes, caused by the sun
answer. and more screen exposure. We can say from the
options that hair fall is nowhere mentioned in the
2. Answer: A passage. So, option A is the correct answer.
Here, from the lines, In summer, prolonged
exposure to UV modifies the lens proteins, 3. Answer: A
leading to various problems like cataract Here, voluntarily means of one’s own free will
formation and worsening eyesight. UV can also and here willingly is the only synonym whereas
damage the retina with a high risk of cancer like all other options are antonyms. So, option A is
basal cell carcinoma and squamous cell the correct answer.

Click Here For Bundle PDF Course | support@guidely.in Page 6 of 9


SBI Clerk & RRB PO Mains PDF Course 2023
ENGLISH Day - 6

could infect another child if they have


4. Answer: D conjunctivitis. It's a viral infection, so it spreads
Here, from the lines, Dryness is a major problem rampantly," the expert said. We can say that
in eye health these days, especially among option D is the correct answer.
children who study for a long time and those who
are in front of the screens. Many people look at 7. Answer: D
their screens for 8-14 hours a day which can Here, as per the lines, When we eat a meal
cause extreme dryness. In summer, dryness is a plate, which is colourful like a rainbow, we get all
feeling in which a person puts mud in their eyes sorts of antioxidants in our diet. Since our
and starts rubbing it in. It can cause your eyes childhood, we have been taught that an apple a
then become red, irritated and watery," said Dr day keeps the doctor away. Likewise, carrots are
Gopal Pillai. To prevent dry eyes, wash your known to be best for the eyes. It is rich in beta-
eyes frequently, suggested Dr Pillai. "After about carotene and vitamin, which protects the eye
every few minutes, voluntarily, close your eyes against infections," Jyoti Khaniojh said. Besides
tightly and then open them. Watch your eyes this, add lemon and citrus fruits to your diet for
more often as it will help lubricate the eyeballs," vitamin C intake. Nuts and seeds that contain
he said. We can say that option D is the correct vitamin E prevent cataracts and age-related
answer. macular degeneration. Salmon fish is a rich
source of omega-3 and this healthy fat helps in
5. Answer: D prevent eye dryness as it is great for retina
Here, as per the lines, Another major problem in health. Egg yolks contain vitamin A, zinc, lutein
summer is ocular diseases like conjunctivitis, and zeaxanthin and zinc, which help in age-
keratitis, endophthalmitis, cellulitis and stye, related degeneration of the eyes. Zinc helps in
among others. Dr Pillai said that conjunctivitis is the night improvement of night vision. We can
one of the most common eye problems in the hot say that option D is the correct answer.
months. We can say that option D is the correct
answer. 8. Answer: E
Here, in this question, there is no error. So,
6. Answer: D option E is the correct answer.
Here, as per the lines, Conjunctivitis spreads via
human touch. Someone could use an infected 9. Answer: C
towel or even a door knob that has been touched Here, the error lies in part c as the use of he is
by an infected person. In a classroom, one child inappropriate as here in this question there is no

Click Here For Bundle PDF Course | support@guidely.in Page 7 of 9


SBI Clerk & RRB PO Mains PDF Course 2023
ENGLISH Day - 6

gender-specific thing so he or she cannot be 14. Answer: C


used as per the context, instead it should have Here, ultimately should have been used in place
been used. So, option C is the correct answer. of penultimately as ultimately means in the end,
and here ultimately fits best whereas
10. Answer: C penultimately means next to the last which is
Here, the error lies in part c as the use of for is inappropriate as per the context of the sentence.
inappropriate before Air Costa as of should have So, option C is the correct answer.
been used instead of for. Of shows possession
and here license was possessed by the 15. Answer: C
company so it should have been used. So, Here, apprehensive should have been used here
option C is the correct answer. in place of comprehensive as comprehensive
means including everything or nearly everything
11. Answer: B that is connected to a particular subject whereas
Here, the error lies in part b as the use of begun apprehensive means worried or afraid that
is inappropriate, and began should have been something bad may happen and here failure of
used here because the sentence is in past airlines is the main focus of the sentence. So,
simple tense and here the second form of the option C is the correct answer.
verb should have been used. So, option B is the
correct answer. 16. Answer: B
Here, prospects should have been used in place
12. Answer: C of respects which is contextless here as per the
Here, the error lies in part c as the use of an context of the sentence. Prospects mean a
before European is inappropriate as an is used possibility that something might happen whereas
before words which begins with a vowel sound, respect means polite behavior or care towards
and European sounds like European and somebody and here former fits as per the
therefore an cannot be used before it, instead a context of the sentence. So, option B is the
should have been used. So, option C is the correct answer.
correct answer.
17. Answer: D
13. Answer: E Here, which should be replaced with who as
Here, none of the highlighted words are here economists word is used which is a living
inappropriately used. So, option E is the correct being and for them who should have been used
answer. there and add should have been replaced with

Click Here For Bundle PDF Course | support@guidely.in Page 8 of 9


SBI Clerk & RRB PO Mains PDF Course 2023
ENGLISH Day - 6

aid as add means to put something together and 19. Answer: C


aid means help and here latter fits best. So, Here, have should be replaced with has been as
option D is the correct answer. the sentence is in present perfect continuous
tense and Apple is a singular subject and
18. Answer: D therefore we will use a singular helping verb
Here, towards should have been replaced with which is has been. So, option C is the correct
away from as land has been bough in India answer.
which means the company has bought land
away from China and therefore option D fits 20. Answer: E
best. So, option D is the correct answer. Here, there is no need for any improvement. So,
option E is the correct answer.

Click Here For Bundle PDF Course | support@guidely.in Page 9 of 9


SBI Clerk & RRB PO Mains PDF Course 2023
Reasoning Ability Day - 7 (Eng)

Reasoning Ability
Directions (1-5): Study the following information b) The difference between the ages of Y and K is
carefully and answer the given questions. equal to the age of U
Ten persons – A, C, G, H, K, P, R, T, U and Y c) P was born in 1974
were born in different years. The age of each d) Y was born in one of the years before T
person is calculated from the base year 2023. e) All the statements are true
Age of each person is not less than 7 years and
not more than 75 years. No two persons have 3. Four of the following five are alike in a certain
the same age. way based on the given arrangement and thus
A was born in 1969. The difference between the form a group. Which one of the following does
ages of R and A is 19 years. The difference not belong to the group?
between the ages of R and H is 27 years. No one a) Y
was born between H and P. Two years gap b) T
between the years in which H and P were born. c) A
U was born eighteen years before P. P was born d) H
in neither 1958 nor 1980. The difference between e) G
the ages of G and U is 17 years. K was born two
years before G. K’s age is not less than 45 years. 4. What is the sum of the ages of H and the one
The sum of the ages of K and T is 91 years. The who was born two persons after A?
average age of T and C is 52 years. The a) 54
difference between the ages of C and Y is 13 b) 128
years. c) 112
1. Who among the following person was born d) 78
immediately after T? e) 63
a) The one who was born in 1957
b) H 5. In which of the following option first person
c) The one who was born in 1977 born three years after the second person?
d) A I. UC
e) The one who was born in 1988 II. TG
III. GK
2. Which of the following statement(s) is/are IV. PH
TRUE as per the given arrangement? a) Only I, II and IV
a) More than two persons were born after U b) Only III and IV
c) Only II

Click Here For Bundle PDF Course | support@guidely.in Page 1 of 13


SBI Clerk & RRB PO Mains PDF Course 2023
Reasoning Ability Day - 7 (Eng)

d) Only IV d) It is immediately preceded as well as


e) All I, II, III and IV immediately followed by a number
e) All the statements are true
Directions (6-10): Study the following information
carefully and answer the given questions. 7. What is the difference between the place
S5@TH3#2A%R$Q6?M^B&D76* values (as per the alphabetical series) of the
P9=V4βLα8X letters which is the second vowel from the left
Step 1: Two is subtracted from the numbers each end and the third consonant from the right end in
of which is immediately preceded by a letter but step 2?
not immediately followed by another number. a) 10
Step 2: All such letters each of which is either b) 12
immediately succeeded by a symbol or c) 18
immediately preceded by a number but not both d) 5
are replaced by the corresponding e) 7
complementary pair as per the English
alphabetical series. 8. What is the position of the first symbol towards
Step 3: All such symbols each of which is the left of R in Step 1?
immediately succeeded by a consonant are a) Fifth from R
interchanging their position with respect to the b) Second from R
element immediately before it. c) Fourth from R
Note: Step 1 is followed by step 2 and step 2 is d) Immediate left of R
followed by step 3. e) Third from R
6. If the positions of the second odd number from
the left end and the second consonant from the 9. If every alternate element starting from @ in
right end in the final step are interchanged and step 3 is dropped, then which of the following
then the alphabets which are immediately element is exactly in the middle of the third
followed by a symbol is dropped, then which of consonant from the left end and the second
the following statement is true with respect to “^” symbol from the right end?
in the final arrangement? a) 6
a) It is 6th to the left of the = b) 7
b) $ is 5th to the right of ^ c) Y
c) It is 7th to the left of the 8th element from the d) N
right end e) Q

Click Here For Bundle PDF Course | support@guidely.in Page 2 of 13


SBI Clerk & RRB PO Mains PDF Course 2023
Reasoning Ability Day - 7 (Eng)

10. If every letter in step 2 is replaced by the lowermost step is numbered 1 and the step
succeeding letter (as per English alphabetical immediately above it is numbered 2 and so on.
series), then how many such symbols are in the II) Initially, the persons are standing in the steps
series each of which is immediately preceded by which are the multiples of 7 i.e., the person in the
a number but not immediately followed by a lowermost floor stands on step 7 and so on. Only
vowel? one person stands on each step.
a) Three III) All the persons move from their respective
b) Five steps based on the dice. The persons living on
c) Four the odd numbered floor move downward
d) One whereas the persons living on the even
e) Two numbered floor move upward.
IV) Each person is allowed to throw the dice
Directions (11-15): Study the following twice and the sum of the two outcomes of the
information carefully and answer the given dice is the number they need to move, i.e., if a
questions. person gets the outcome as (6,4), then that
Seven persons – L, M, N, O, P, Q and R are person needs to move 10 steps either up or
living on different floors of a seven-storey down.
building where the lowermost floor is numbered V) All the persons throw the dice in the reverse
one and the floor immediately above it is alphabetical order. After the final arrangement,
numbered two and so on. No two persons live on more than one person may or may not stand on
the same floor. the same step.
O lives on a prime numbered floor. L lives two VI) The following shows the final outcome of the
floors below O. Only two persons live between L dice thrown by all the persons in the reverse
and P. The number of floors above P is two more alphabetical order respectively.
than the number of floors below R. Only two (3, 4), (4, 1), (6, 2), (1, 2), (2, 2), (2, 4), (5, 3)
floors are between R and M. N lives on one of 11. Which of the following statements is/are not
the floors above Q. N doesn’t live on the topmost true with respect to the final arrangement?
floor. I. Both O and N stand on the same staircase
Based on the below conditions they are standing II. Q stands on the odd numbered step
on the steps: III. No one stands above staircase 46
Special conditions: IV. There is an odd number of steps between N
I) There are 50 staircases in the building in which and R
each person stands on different steps. The a) Only I, II and IV
b) Only III and IV

Click Here For Bundle PDF Course | support@guidely.in Page 3 of 13


SBI Clerk & RRB PO Mains PDF Course 2023
Reasoning Ability Day - 7 (Eng)

c) Only I, II and III b) R


d) Only II and IV c) The one who lives immediately above P
e) All I, II, III and IV d) L
e) The one who lives on the first floor
12. Which of the following pair of persons live on
the adjacent floors of Q? Directions (16-20): Study the following
a) Both M and O information carefully and answer the given
b) Only P questions.
c) Both R and P Eight persons – A, B, C, D, E, F, G and H are
d) Both N and Q sitting in a linear row and facing towards the
e) Both L and R north. Each of them likes different numbers.
B sits fourth to the left of the one who likes 98. B
13. R lives on which of the following floor and doesn’t sit at the extreme end of the row. Only
stands on which of the following step one person sits between the one who likes 98
respectively? and G. C sits fourth to the left of G. The number
a) 6th floor, 10th step of persons sitting to the right of C is two more
b) 4th floor, 30th step than the number of persons sitting to the left of
c) 3rd floor, 46th step the one who likes 120. Only two persons sit
d) 5th floor, 40th step between C and H, who doesn’t sit at the extreme
e) 4th floor, 35th step ends. The sum of the numbers liked by H and the
one who sits third from the right end is 161. Only
14. What is the sum of the steps in which O, R three persons sit between A and E, who sits
and M stand (after the rearrangement of all the immediate right of the one who likes 79. G
persons)? doesn’t like 79. The difference between the
a) 84 numbers liked by C and A is 46. A didn’t like the
b) 82 number which is more than 40. The sum of the
c) 101 numbers liked by the persons sitting at both ends
d) 134 is 100. The sum of the numbers liked by D and E
e) 98 is 90. One of the persons likes 53.
16. What is the average of the numbers liked by
15. Who among the following person stands on G, F and B?
the same staircase along with O (after the a) 53
rearrangement of all the persons)? b) 22
a) The one who lives on the Sixth floor c) 40

Click Here For Bundle PDF Course | support@guidely.in Page 4 of 13


SBI Clerk & RRB PO Mains PDF Course 2023
Reasoning Ability Day - 7 (Eng)

d) 64 c) 126m
e) 10 d) 60m
e) 6m
17. Which among the following pairs of persons
like a prime number? 19. Who among the following person sits third to
I. BC the left of the one who sits immediate right of H?
II. DH a) The one who likes 98
III. CG b) The one who sits at the extreme left end
IV. DB c) The one who sits immediate right of C
a) Only I, II and III d) The one who sits second to the left of A
b) Only I, III and IV e) None of these
c) Only II and III
d) Only I and IV 20. The number of persons sitting to the left of
e) All I, II, III and IV the one who likes 120 is twice that of the number
of persons sitting to the right of __.
18. If the distance between adjacent persons is a a) The one who likes 53
consecutive multiple of 6m from the left end. b) E
Then what is the distance between the one who c) D
likes 79 and D? d) The one who sits immediate right of H
a) 90m e) The one who likes 98
b) 36m
Click Here to Get the Detailed Video Solution for the above given Questions
Or Scan the QR Code to Get the Detailed Video Solutions

Answer Key with Explanation

Directions (1-5): 1) Answer: E

Click Here For Bundle PDF Course | support@guidely.in Page 5 of 13


SBI Clerk & RRB PO Mains PDF Course 2023
Reasoning Ability Day - 7 (Eng)

2) Answer: D
3) Answer: B(in the given option, all the persons
were born in on odd numbered year except
option b)
4) Answer: A
5) Answer: C Again, we have

Final arrangement  No one was born between H and P.


 Two years gap between the years in
which H and P were born.
 U was born eighteen years before P.
 P was born in neither 1958 nor 1980.
 The difference between the ages of G and
U is 17 years.
 K was born two years before G.
 K’s age is not less than 45 years.
Age of each person is not less than 7 years and
not more than 75 years. No two persons have
the same age.
We have,
 A was born in 1969.
After applying the above conditions case-3 gets
 The difference between the ages of R and
eliminated because U’s age is more than 75
A is 19 years.
years.
 The difference between the ages of R and
H is 27 years.
The age of each person is calculated from the
base year 2023. Age of each person is not less
than 7 years and not more than 75 years. No two
persons have the same age.
From the above conditions, there are three
Again, we have
possibilities
 The sum of the ages of K and T is 91
years.
 The average age of T and C is 52 years.
 The difference between the ages of C and
Y is 13 years.

Click Here For Bundle PDF Course | support@guidely.in Page 6 of 13


SBI Clerk & RRB PO Mains PDF Course 2023
Reasoning Ability Day - 7 (Eng)

Age of each person is not less than 7 years and @3TH#2%$I?4^&YD7*6=712βα8


not more than 75 years. No two persons have C.
the same age.
In case-1 Y’s age is either 78 or 52 no person 7) Answer: E
has age more than 75 so, there is no possibility Step 2: S 3 @ T H 1 # 2 A % I $ Q 4 ? N ^ Y & D
to place 75 and K’s age is 52 years so, there is 76*P7=V2βOα8C
no possibility to place 52. The Second vowel from the left end is I and the
After applying the above conditions case-1 gets third consonant from the right end is P
eliminated because there is no possibility to The Place value of I is 9 and the place value of P
place Y, hence case-2 shows the final is 16
arrangement. 16-9=7

8) Answer: D
Step 1: S 3 @ T H 1 # 2 A % R $ Q 4 ? M ^ B &
D76*P7=V2βLα8X

9) Answer: B
Step 3: S @ 3 T H 1 # 2 A % $ I Q ? 4 ^ N & Y D
7 * 6 P = 7 V 2 β O α 8 C.
Every alternate element starting from @ is
dropped from the given series
Directions (6-10): Required series:
6) Answer: C S3H#A$Q4NY76=VβαC
Step 3: S @ 3 T H 1 # 2 A % $ I Q ? 4 ^ N & Y D S3H#A$Q4NY76=VβαC
7 * 6 P = 7 V 2 β O α 8 C. The third consonant from the left end is Q and
The Second odd number from the left end and the second symbol from the right end is β
the second consonant from the right end are 1 Q 4 N Y 7 6 = V β.
and V respectively.
S@3THV#2A%$IQ?4^N&YD7*6P 10) Answer: C
= 7 1 2 β O α 8 C. Step 2: S 3 @ T H 1 # 2 A % I $ Q 4 ? N ^ Y & D
And then the alphabets which are immediately 76*P7=V2βOα8C
followed by a symbol is dropped Every letter is replaced by a succeeding letter as
Required series: per the alphabetical series

Click Here For Bundle PDF Course | support@guidely.in Page 7 of 13


SBI Clerk & RRB PO Mains PDF Course 2023
Reasoning Ability Day - 7 (Eng)

Required series: Step 3: S @ 3 T H 1 # 2 A % $ I Q ? 4 ^ N & Y D


T3@UI1#2B%J$R4?O^Z&E76*Q 7 * 6 P = 7 V 2 β O α 8 C.
7=W2βPα8D
Step 1: Two is subtracted from the numbers Directions (11-15):
each of which is immediately preceded by a 11) Answer: D
letter but not immediately followed by another 12) Answer: C
number. 13) Answer: E
Given series: 14) Answer: B
S5@TH3#2A%R$Q6?M^B&D76* 15) Answer: A
P9=V4βLα8X Final arrangement
Required series:
Step 1: S 3 @ T H 1 # 2 A % R $ Q 4 ? M ^ B &
D76*P7=V2βLα8X
From Step 1,
Step 2: All such letters each of which is either
immediately succeeded by a symbol or
immediately preceded by a number but not both
are replaced with the corresponding
complementary pair as per the English
alphabetical series.
Step 1: S 3 @ T H 1 # 2 A % R $ Q 4 ? M ^ B &
D76*P7=V2βLα8X
Required series:
Step 2: S 3 @ T H 1 # 2 A % I $ Q 4 ? N ^ Y & D
76*P7=V2βOα8C
From Step 2,
Step 3: All such symbols each of which is
immediately followed by a consonant are
interchanged their position with respect to the
element immediately before it.
Step 2: S 3 @ T H 1 # 2 A % I $ Q 4 ? N ^ Y & D
76*P7=V2βOα8C
Required series:

Click Here For Bundle PDF Course | support@guidely.in Page 8 of 13


SBI Clerk & RRB PO Mains PDF Course 2023
Reasoning Ability Day - 7 (Eng)

We have,
 O lives on a prime numbered floor.
 L lives two floors below O.
From the above conditions, there are three
possibilities

Again, we have
 Only two persons live between L and P.
 The number of floors above P is two more
than the number of floors below R.

Click Here For Bundle PDF Course | support@guidely.in Page 9 of 13


SBI Clerk & RRB PO Mains PDF Course 2023
Reasoning Ability Day - 7 (Eng)

 Only two floors are between R and M.


After applying the above conditions case-2 gets
eliminated because there is no possibility to
place R.

Again, we have
 N lives on one of the floors above Q.
 N doesn’t live on the topmost floor.
After applying the above conditions case-1 gets
eliminated because N lives on the topmost floor,
hence case-3 shows the final arrangement.

Special conditions:
Before arrangement, the position of each person
on the staircase is as follows.

Click Here For Bundle PDF Course | support@guidely.in Page 10 of 13


SBI Clerk & RRB PO Mains PDF Course 2023
Reasoning Ability Day - 7 (Eng)

M=(2,4) =6 steps Downward, L=(5,3) =8 steps


Downward.
After arrangement, the position of each person in
the staircase is as follows.

The following shows the final outcome of the


dice thrown by all the persons in the reverse
alphabetical order respectively.
R =(3,4) =7 steps Upward, Q=(4,1) =5 steps
Downward, P=(6,2) =8 steps Upward, O=(1,2)
=3 steps Downward, N=(2,2) =4 steps Upward,

Click Here For Bundle PDF Course | support@guidely.in Page 11 of 13


SBI Clerk & RRB PO Mains PDF Course 2023
Reasoning Ability Day - 7 (Eng)

Final arrangement

. We have,
 B sits fourth to the left of the one who
likes 98.
 B doesn’t sit at the extreme end of the
row.
 Only one person sits between the one
who likes 98 and G.
 C sits fourth to the left of G.
From the above conditions, there are three
possibilities

Again, we have
 The number of persons sitting to the right
of C is two more than the number of
persons sitting to the left of the one who
likes 120.
 Only two persons sit between C and H,
who doesn’t sit at the extreme ends.
 The sum of the numbers liked by H and
the one who sits third from the right end is
Directions (16-20): 161.
16) Answer: D  Only three persons sit between A and E,
17) Answer: B who sits immediate right of the one who
18) Answer: E likes 79.
19) Answer: C  G doesn’t like 79.
20) Answer: D

Click Here For Bundle PDF Course | support@guidely.in Page 12 of 13


SBI Clerk & RRB PO Mains PDF Course 2023
Reasoning Ability Day - 7 (Eng)

 The difference between the numbers liked  The sum of the numbers liked by the
by C and A is 46. persons sitting at both ends is 100.
 A didn’t like the number which is more  The sum of the numbers liked by D and E
than 40. is 90.
After applying the above conditions case-2 gets  One of the persons likes 53.
eliminated because the difference between the After applying the above conditions case-3 gets
numbers liked by C and A is 41. eliminated because there is no possibility to
place D and F, hence case-1 shows the final
arrangement.

Again, we have

Click Here For Bundle PDF Course | support@guidely.in Page 13 of 13


SBI Clerk & RRB PO Mains PDF Course 2023
Quantitative Aptitude Day - 7 (Eng)

Quantitative Aptitude

Directions [01 – 04]: Read the following information carefully and answer the questions based on it.
A company manufactured perfumes of three brands P, Q and R. The table given below shows the partial
information about the number perfumes of different brands manufactured by company from April to till
July.

1) Find number of brand P perfume 3) In May, 80% of manufactured perfumes are


manufactured in June? sold, and all perfumes of brand Q and brand R
a) 104 sold, find number of brand P perfumes which are
b) 86 not sold?
c) 76 a) 30
d) 96 b) 32
e) None of these c) 16
d) 24
2) In April, number of brand R perfume e) None of these
manufactured is 50% more than that of brand Q
perfumes manufactured, then find difference 4) Number of brand Q perfume manufactured in
between brand Q and brand P perfumes July is 10% as that of brand P perfume
manufactured? manufactured, and number of brand Q perfume
a) 98 manufactured in June is twice as that of R
b) 88 manufactured in July. Find brand R perfume sold
c) 72 in June is approx. what percent of total perfumes
d) 82 sold in that month?
e) None of these a) 47%
b) 45%
c) 50%

Click Here For Bundle PDF Course | support@guidely.in Page 1 of 11


SBI Clerk & RRB PO Mains PDF Course 2023
Quantitative Aptitude Day - 7 (Eng)

d) 42% e) 55%

Direction [05 - 08]: Read the following information carefully and answer the questions based on it.
The given line graph shows the data about the total words typed by four different typewriters named
typewriter A ,B, C and D in books, magazines and articles and also gives the data about the words typed
by typewriter A in those books, magazines and articles. The table shows the data about words typed by
typewriters B, C, D in those books, magazines and articles.

5) Number of words typed by typewriter D for 6) Find the number ofwords typed for magazines
Books is 40% of those for magazines. Find the and articles together by 4 typewriters is what
total words typed for books by typewriters B and percentage more than the number of words
C? typed for books by 4 typewriters?
a) 140 a) 35(1/3) %
b) 100 b) 23(1/3) %
c) 150 c) 73(1/3) %
d) 260 d) 40%
e) None of these e) None of these

Click Here For Bundle PDF Course | support@guidely.in Page 2 of 11


SBI Clerk & RRB PO Mains PDF Course 2023
Quantitative Aptitude Day - 7 (Eng)

7) Number of words typed by typewriter D for 8) Number of words typed by typewriter B for
articles and books is in the ratio 4:5. Find the books is 62.5% less than the total words typed
difference of number of words typed for articles by typewriter A for all the given threeand 25%
and magazines for typewriter D, if the total words less than the words typed by typewriter D for
typed by typewriter D, is 140? articles. Find the total words typed by typewriter
a) 15 B for books and typewriter D for articles
b) 30 together?
c) 20 a) 70
d) 10 b) 40
e) None of these c) 20
d) 25
e) None of these

Directions [9 - 12]: Read the following information carefully and answer the questions based on it.
Given bar-graph depicts the success of three schemes A, B and C implemented in the states Goa, Delhi,
Gujarat and Orissa.
Success of schemes (in Rs.)= Return on investment (ROI) (in 000’s)

9) Find the ratio of ROI for schemes A and C in d) 8:15


Orissa? e) None of these
a) 7:15
b) 28:15 10) What is the average ROI for scheme C in all
c) 14:15 the states together?

Click Here For Bundle PDF Course | support@guidely.in Page 3 of 11


SBI Clerk & RRB PO Mains PDF Course 2023
Quantitative Aptitude Day - 7 (Eng)

a) Rs.31250 d) Orissa
b) Rs.65000 e) Noneof these
c) Rs.62500
d) Rs.31500 12) Find the difference between total ROI in
e) None of these Delhi and Gujarat for all the three schemes?
a) Rs.55500
11) ROI for schemes A and B together is b) Rs.53000
maximum for which state? c) Rs.45000
a) Goa d) Rs.50000
b) Gujarat e) None of these
c) Delhi

Directions [13 – 16]: Read the following information carefully and answer the questions based on it.
RAJPAL whois a psychiatric patient, needs electric shocks every day, so he admitted in RAJENDRAS
Hospital and treated by four top most doctors of India (treating patients free of cost) – Arun, Varun, Geeta
and Kabir. The chart given below shows(in X axis)% distribution of number of levels of electric shock
given by a doctor for a patient to his/her treatment out of total number of levels of treatment of all doctors
together and (in Y axis) % of level of electric shocks required by RAJPAL from a particular doctor. Each
level of treatment of each doctor required 40 minutes electric shock.

• Number of levels of treatment of all doctors together for a patient is 384 more than the average levels of
treatment required by Rajpal from Arun and Kabir together.

Click Here For Bundle PDF Course | support@guidely.in Page 4 of 11


SBI Clerk & RRB PO Mains PDF Course 2023
Quantitative Aptitude Day - 7 (Eng)

• Rajpal required electric shock of 70 minutes, 50 minutes, 36 minutes, and 42 minutes respectively in a
particular day.
• Every day Rajpal visited doctors in order of Arun, Varun, Geeta and Kabir. He first visited to Arun,
completes first level of electric shock and then he starts second level of electric shock from same doctor
on same day only after completing first level of all doctors or completing his requirement of electric shock
from other doctors.
NOTE: It is just a question, so don’t get personalized. Kahani k SaarePatrkalpanik h, eskavastavikJeevan
se koi lenadenanahi h.
13) Find total number of levels completed by 15) Which level of treatment is going on from
Rajpal in his first two days of treatment? doctor Kabir at the end of day 23 for the
a) 7 treatment of Rajpal?
b) 10 a) 24
c) 8 b) 25
d) 9 c) 26
e) None of these d) 23
e) Can’t be determined
14) If there is another Psychopath patient Sapna,
whose requirement of electric shock per day 16) Find minimum possible number of levels of
from each doctor is same as that of Rajpal, and treatment completed by Rajpal of any of the
she needs 25 levels of electric shock from Varun doctor in 4 days?
and 20 levels of electric shock from Geeta. Find a) 3
the difference between the number of days (on b) 2
which day) to complete all levels of treatment c) 4
from doctor Varun and Geeta, if she first d) 5
completed the treatment from doctor Varun and e) None of these
goes to Geeta?
a) 5 Directions [17 – 20]: Read the following
b) 6 information carefully and answer the questions
c) 4 based on it.
d) 3 A survey conducted among Z number of people
e) None of these about their liking of laptops of three brands
among – Lenovo(L), Acer (A), and HP (H). (Z –
31) people likes at least one of the three given
brands. People those likes both A and H but not

Click Here For Bundle PDF Course | support@guidely.in Page 5 of 11


SBI Clerk & RRB PO Mains PDF Course 2023
Quantitative Aptitude Day - 7 (Eng)

L is 12 less than thrice of the people those likes a) 21


both L and A but not H. Number of people those b) 19
likes both L and H but not A is Twice of people c) 17
those likes only H and three less than those likes d) 18
only L. Number of people those likes all three e) None of these
brands are 9, which is 16 less than those likes
only A 19) M = number of people those likes only H
Note: N = number of people those likes all three brands
a) Number of people those likes only L + number Find the value of (4N + 6M)?
of people those likes both L and A but not H = a) 77
Number of people those likes only A b) 81
b) Number of people those likes only H > number c) 85
of people those likes both A and H but not L d) 76
17) Find the value of (2Z + 48)? e) None of these
a) 226
b) 248 20) Find the number of people those likes brand
c) 288 A?
d) 298 a) 46
e) None of these b) 50
c) 48
18) Find the difference between number of d) 52
people those likes only A and people those likes e) None of these
both A and L but not H?
Click Here to Get the Detailed Video Solution for the above given Questions
Or Scan the QR Code to Get the Detailed Video Solutions

Answer Key with Explanation

Click Here For Bundle PDF Course | support@guidely.in Page 6 of 11


SBI Clerk & RRB PO Mains PDF Course 2023
Quantitative Aptitude Day - 7 (Eng)

Directions [01 – 04]: According to question,


For April Number of brand P perfume manufactured in
Total Perfumes manufactured = 200 June = 76
Number of brand P manufactured = 60% x 200 = Hence answer is option C
120
Number of (Q + R) manufactured = 40% x 200 = 2) Answer: B
80 In April,
Till May Respective ratio of brand Q and brand R
Number of (Q + R) manufactured till May = 96 perfume manufactured = 2:3
Total perfumes manufactured till May = 96/30 x So, number of brand Q perfume manufactured =
100 = 320 2/5 x 80 = 32
So, Brand P perfumes manufactured till May = Required difference = 120 – 32 = 88
320 x 70% = 224 Hence answer is option B
Till June
Total perfume manufactured = 600 3) Answer: D
Number of (Q + R) manufactured = 300 In May
So, number of P manufactured = 600 – 300 = Total number of perfumes sold = 80% x 120 = 96
300 Number of brand P perfumes unsold = 120 – 96
Till July = 24
Number of (Q + R) manufactured = 360 Hence answer is option D
Total perfume manufactured = 360/40 x 100 =
900 4) Answer: A
Number of brand Q perfume manufactured = Number of brand R perfumes manufactured in
60% x 900 = 540 July = 60 – 10% x 240 = 36
So, number of brand R perfumes manufactured
in June = 204 – 2 x 36 = 132
Required percentage change = 132/280 x 100 =
47% (Approx)
Hence answer is option A

5) Answer: B
According to question,

1) Answer: C

Click Here For Bundle PDF Course | support@guidely.in Page 7 of 11


SBI Clerk & RRB PO Mains PDF Course 2023
Quantitative Aptitude Day - 7 (Eng)

Words typed by typewriter D for books = 40% x 8) Answer: A


Words typed by typewriter D for magazines According to question,
Words typed by typewriter D for books = 40/100 Words typed by typewriter B for books = 37.5% x
* 50 = 20 total words typed by typewriter A
Total words typed by 4 typewriters for books = Words typed by typewriter B for books = 37.5% x
150 80 = 30
Words typed by typewriter B for books = 150 – Words typed by typewriter B for books = 75% x
30 – 20 – 40 = 60 words typed by typewriter D for articles
Thus, Words typed by typewriter D for articles = 4/3 x
Words typed by typewriter B and C for books = 30 = 40
60 + 40 = 100 Total words typed by typewriter B for books and
Hence, answer is option B typewriter D for articles = 30 + 40 = 70
Hence, answer is option A
6) Answer: C
According to question, 9) Answer: D
Words typed for magazines and articles together According to question,
by 4 typewriters = 160 + 100 = 260 Ratio of ROI for schemes A and C in Orissa =
Words typed for books by 4 typewriters = 150 40:75 = 8:15
Required percentage = (260-150)/150 x 100 = Hence, answer is option D
73(1/3) %
Hence, answer is option C 10) Answer: C
According to question,
7) Answer: D Average ROI for scheme C in all the states
According to question, together = (75 + 25 + 75 + 75)/4 =
Words typed by typewriter D for articles: Words 62.5=>Rs.62500
typed by typewriter D for books = 4:5 Hence, answer is option C
Total words typed by typewriter D = 140
140 = 9x + 50 11) Answer: B
Thus, x = 10 According to question,
Difference of number of words typed for articles ROI for Goa = 30 + 50 = 80
and magazines = 50 - 40 = 10 ROI for Delhi = 45 + 60 = 105
Hence, answer is option D ROI for Gujarat = 30 + 80 = 110
ROI for Orissa = 40 + 30 = 70

Click Here For Bundle PDF Course | support@guidely.in Page 8 of 11


SBI Clerk & RRB PO Mains PDF Course 2023
Quantitative Aptitude Day - 7 (Eng)

ROI for schemes A and B together is maximum 13) Answer: C


for Gujarat According to question,
Hence, answer is option B Rajpal takes electric shock of 40 minutes from
Arun, then 40 minutes from Varun and then 36
12) Answer: E minutes from Geeta (because his requirement is
According to question, 36 minutes, then 40 minutes from Kabir. Now he
Total ROI in Delhi for all the three schemes= come back to Arun takes 30 minutes from Arun,
Rs.130000 10 minutes from Varun and then directly goes to
Total ROI in Gujaratfor all the three schemes = Kabir (because he already completed his
Rs.185000 requirement from Geeta) and takes 2 minutes
Difference between total ROI in Delhi and from Varun. So, he ended with the day.
Gujarat = Rs.55000 At end of day 1
Hence, answer is option E Arun = 40 minutes (level 1 completed) + 30
minutes (level 2)
Directions [13 – 16]: Varun = 40 minutes (level 1 completed) + 10
Let total number of levels of treatment of all minutes (level 2)
doctors together = a Geeta = 36 minutes (no level completed)
Levels of treatment of Arun = 20% x a Kabir = 40 minutes (level 1 completed) + 2
Number of levels of treatment required by Rajpal minutes (level 2)
from Arun = 25% x (20% x a) = 5% x a At the end of day 2
Levels of treatment of Kabir = 10% x a Arun = 10 minutes (level 2 completed) + 40
Number of levels of treatment required by Rajpal (minutes level 3 completed) + 20 minutes (level
from Kabir = 30% x (10% x a) = 3%a 4)
Now, Varun = 30 minutes (level 2 completed) + 20
a = (5%a + 3%a)/2 + 384 minutes (level 3)
96%a = 384 Geeta = 4 minutes (level 1) completed + 32
Value of a = 4 minutes (level 2)
Kabir = 38 minutes (level 2 completed) + 4
minutes (level 3)
So, at the end of day 2, total levels of treatment
completed by Rajpal = 3 + 2 + 1 + 2 = 8
Hence answer is option C

Click Here For Bundle PDF Course | support@guidely.in Page 9 of 11


SBI Clerk & RRB PO Mains PDF Course 2023
Quantitative Aptitude Day - 7 (Eng)

14) Answer: D Day 4 = 12 minutes (level 3 completed) + 24


Time taken by Sapna to complete the treatment minutes (level 4)
of doctor Arun = (25 x 40)/50 = 20 days So, he completed three levels of treatment from
Time taken by Sapna to complete the treatment doctor Geeta.
of doctor Geeta = (20 x 40)/36 = 23 days Hence answer is option A.
Required difference = 23 – 20 = 3 days
Hence answer is option D Directions [17 – 20]:
Let the number of people those likes only H = a
15) Answer: B So, number of people those likes both L and H
Requirement of Rajpal per day from Kabir = 42 but bot A = 2 x a = 2a
minutes So, number of people those likes only L = 2a + 3
Day 1 = 40 minutes (Level 1 completed) + 2 Let the number of people those likes both L and
(minutes level 2) A but not H = b
Day 2 = 38 minutes (level 2 completed) + 4 So, number of people those likes both A and H
(minutes level 3) but not L = 3 x b – 12 = 3b – 12
: : Number of people those likes only A = 9 + 16 =
: : 25
: : Also, Number of people those likes only L +
Day 20 = 2 minutes (level 20 completed) + 40 number of people those likes both L and A but
minutes (level 21 completed) not H = Number of people those likes only A
So, on 23 days = 24 levels completed and 25 Now,
going on 2a + 3 + b = 25
Hence answer is option B 2a + b = 22
Value of a = 11 – b/2
16) Answer: A Also, 3b – 12 must be more than zero, so
Daily requirement of Rajpal from doctor Geetais minimum possible value of b is 6
minimum, so he completed the minimum If b = 6, then a = 8 (possible)
possible levels in 4 days If b = 8, then a = 7
Day 1 = 36 minutes Number of people those likes only H > number of
Day 2 = 4 minutes (level 1 completed) + 32 people those likes both A and H but not L
minutes (level 2) So, number of people those likes only H = a = 7
Day 3 = 8 minutes (level 2 completed) + 28 Number of people those likes both A and H but
minutes (level 3) not L = 3b – 12 = 3 x 6 – 12 = 6

Click Here For Bundle PDF Course | support@guidely.in Page 10 of 11


SBI Clerk & RRB PO Mains PDF Course 2023
Quantitative Aptitude Day - 7 (Eng)

This pair of value is not possible. If we further Hence, the answer is option C
increase the value of b, then value of ‘a’
becomes lesser, so we have only one pair of 18) Answer: B
values. Number of people those likes only A = 25
Value of a = 8, and b = 6 People those likes both A and L but not H = 6
Now we can find all the data Required difference = 25 – 6 = 19
Hence, the answer is option B

19) Answer: E
Value of M = 8
Value of N = 9
Required value = (4 x 9 + 6 x 8) = 84
Hence, the answer is option E

20) Answer: A
17) Answer: C
Number of people those likes brand A = 25 + 6 +
Total number of people surveyed = 89 + 31 =
9 + 6 = 46
120 = Z
Hence, the answer is option A
According to the question,
Value of (2Z + 48) = 120 x 2 + 48 = 288

Click Here For Bundle PDF Course | support@guidely.in Page 11 of 11


SBI Clerk & RRB PO Mains PDF Course 2023
ENGLISH Day - 7

English Language

Directions (1-5): Given below are a few For India, the G20 Presidency also marks the
questions with a blank in each paragraph to be beginning of “Amritkaal”, the 25-year period
filled from the options given in order to make the beginning from the ___________.
paragraph contextually correct. If none of the A. 73th anniversary of its independence on 15
options are correct then choose option E as your August 2022.
answer. B. 74th anniversary of its independence on 15
1. Bangladesh Prime Minister Sheikh Hasina is August 2022.
likely to participate in the 18th G20 summit in C. 75th anniversary of its independence on 15
New Delhi later this year. Obaidul Quader, a August 2022.
minister in the Sheikh Hasina-led government, D. 79th anniversary of its independence on 15
on Friday 19th may she said she will be August 2022.
participating as the representative of a 'guest E. None of the above
country' in the summit. Quader, who is also the
General Secretary of the Bangladesh Awami 3. Diplomatic passports, also known as 'Type D'
League, said, “India has invited PM Hasina to passports, are issued to Indian diplomats,
attend the G20 summit taking place in government officials, and select individuals who
September. Ahead of that, we will hold a party- are authorised to undertake official travel on
to-party meeting." He also stated that behalf of the Indian government. Diplomatic
the_______. passports are maroon in colour. A diplomatic
A. the final decision solely depends on the PM. passport has 28 pages. Unlike a normal passport
B. party officers are non trustworthy. (which has a dark blue cover) and is valid for 10
C. ruling party members may consult with the years for adults and five years for minors,
supreme court judge. ______________________.
D. ruling party members may visit India in June A. diplomatic passports are issued for a period of
as well. five years or less.
E. None of the above B. diplomatic passports are issued in the name of
PM.
2. India holds the presidency of the G20 nations C. diplomatic passports are always used for the
till November 30. Other than Bangladesh, India journey made for the diplomatic purposes.
has also invited Mauritius, the Netherlands, D. diplomatic passports are never used for
Nigeria, Oman, Singapore, Spain, Egypt, and the personal purposes.
UAE to attend the G20 Leader’s Summit. E. None of the above

Click Here For Bundle PDF Course | support@guidely.in Page 1 of 8


SBI Clerk & RRB PO Mains PDF Course 2023
ENGLISH Day - 7

4. Diplomatic passports are issued to individuals C. none of the challenges in January.


based on their official designation and roles D. 'Soul of Steel' challenge on January 14.
within the government. They are not available for E. None of the above
general public use and cannot be obtained for
personal or leisure travel. Directions (6-10): Given below are a few
The issuance and usage of diplomatic passports questions based on the jumbled paragraph given
are_________________________________. to be rearranged in order to make a contextually
A. which are not for the educational purposes but correct paragraph. Also, some of the sentences
for diplomatic purpose. have blanks and some are jumbled to be
B. governed by the Ministry of External Affairs rearranged if required. If none of the options are
and are subject to specific regulations and correct or no rearrangement is required then
protocols choose option E as your answer.
C. governed by governor of the state and not by a. Kerala transport minister Antony Raj has
the country. _____________ to allow a child below the age of
D. which may or may not require the same 12 to sit on a bike with two adults without
E. None of the above attracting fines.
b. Over seven hundred (a) AI cameras have
5. A team of the 'Soul of Steel', consisting of 23 been installed (b) to bring down traffic violations
youths from across the country, instructors from (c) in Kerala in a bid (d) and enhance safety on
the Indian Army and CLAW (Conquer Land Air the roads (e).
and Water) Global and a team of ex-military c. A meeting was chaired on May 24, where this
commandos from the Indian special forces are issue was discussed and the decision was taken
receiving rigorous training in the hills of to exempt a child under 12 with two adults.
Uttarakhand, for what is called the world's first d. This will be in _________________ until further
self-sustained high altitude skill and endurance, notice from the Union government.
at such glaciated and snowbound terrains. The e. According to the Motor Vehicles Act, more
initiative, aimed at promoting adventure sports in than two people cannot travel on a two-wheeler.
the mountains of Uttarakhand, is being backed 6. Which of the following will be last after the
by the Indian Army. rearrangement?
The Army had also signed a Memorandum of A. a
Understanding (MoU) with CLAW Global, which B. b
held the first edition of the___________________. C. c
A. on January 14. D. d
B. participation of the person in the challenge. E. e

Click Here For Bundle PDF Course | support@guidely.in Page 2 of 8


SBI Clerk & RRB PO Mains PDF Course 2023
ENGLISH Day - 7

7. Which of the following sentence will be middle Directions (11-15): Given below are a few
after the rearrangement? questions with two blanks in each of them to be
A. a filled from the options given below. You have to
B. b fill the the blanks as per the context and mark
C. c your answer accordingly. If none of the options
D. d are correct then choose option E as your answer.
E. e 11. Sengol gets its name from the Tamil word
'semmai', meaning righteousness. The sceptre is
8. Which of the following option fits in the blank a historical _____________ of Independence as it
given in the sentence which will be second after ______________ the transfer of power from the
the rearrangement? British to the Indians.
A. place A. signifies, symbol
B. side B. restrain, testimony
C. accordance C. symbol, signifies
D. agreed D. testimony, restrain
E. None of the above E. None of the above

9. Which of the following option is the correct 12. The rate-setting Monetary Policy Committee
rearrangement of the jumbled sentence given? has raised the policy repo rate by 250 basis
A. abdce points since May last year to ____________
B. acdeb inflationary _____________. The panel kept the
C. bacde repo rate steady at its meeting last month and is
D. badce expected to pause again when it meets in June.
E. No rearrangement is required A. pressures, quell
B. quench, rise
10. Which of the option fits best in the sentence C. rise, quench
which will be first after the rearrangement? D. quell, pressures
A. agreed E. None of the above
B. place
C. tyranny 13. In a bid to ___________ tourism and
D. held __________ ties, India on Tuesday presented 20
E. None of the above Broad Gauge (BG) locomotives to Bangladesh.
The diesel loco engines en route to Dhaka were

Click Here For Bundle PDF Course | support@guidely.in Page 3 of 8


SBI Clerk & RRB PO Mains PDF Course 2023
ENGLISH Day - 7

flagged off by Union Railway Minister Ashwini or may not require swapping in order to make
Vaishnaw through a video conference. sentences contextually correct. If no
A. bolster, bilateral replacement is required then choose option E as
B. bilateral, bolster your answer.
C. unilateral, revamp 16. The Kartarpur Corridor (a) province
D. revamp, unilateral Gurdwara Darbar Sahib in Pakistan's Punjab (b)
E. None of the above links, the final resting place of Sikhism founder
Guru Nanak Dev, with the Dera Baba Nanak (c)
14. While ______________ national and state shrine in Gurdaspur district in India’s Punjab
parties contest elections on their registered party state. Indian Sikh (d) pilgrims can access the 4
symbols, Independents and candidates of km-long corridor and visit the Darbar Sahib
unrecognised parties have to choose their poll without visas.
symbols from the list __________ by the poll A. Only a-b
panel from time to time. B. Only b-c
A. issued, recognised C. Only c-d
B. recognised, issued D. Only d-a
C. termed, provided E. No swapping is required
D. provided, termed
E. None of the above 17. The 414 Army Service Corps Battalion
Marketing (Territorial Army) is one of the (a)
15. A series of assembly polls are battalions raised in 1983 by the government to
_________________ for this year, with the (b) cater to handling internal and external
______________ of the legislative assemblies of emergencies. The battalion is (c) affiliated with
Mizoram, Chhattisgarh, Madhya Pradesh, the marketing (d) division and has more than 100
Rajasthan and Telangana ending between employees of Indian Oil.
December this year and January 2024. A. Only a-b
A. scheduled, terms B. Only b-c
B. prepared, terms C. Only c-d
C. masked, prepared D. Only d-a
D. usurped, prepared E. No swapping is required
E. None of the above
18. On May 9, violent protests (a) erupted after
Directions (16-20): Given below are few paramilitary Rangers arrested Khan from the
questions with four highlighted words which may Islamabad High Court (IHC) (b) premises. His

Click Here For Bundle PDF Course | support@guidely.in Page 4 of 8


SBI Clerk & RRB PO Mains PDF Course 2023
ENGLISH Day - 7

party workers (c) including a dozen military C. Only c-d


installations, (d) vandalised the Lahore Corps D. Only d-a
Commander's House, the Mianwali airbase and E. No replacement is required
the ISI building in Faisalabad in response to
Khan's arrest. 20. In the last weeks since May 7, the Chinese
A. Only a-b research ship, at times (a) venture by a dozen
B. Only b-c vessels, has been moving largely across gas
C. Only c-d block 04-03, (b) operated by Vietsovpetr, a joint
D. Only d-a (c) flanked between Zarubezhneft and
E. No replacement is required PetroVietnam, according to vessel-tracking data
shared with Reuters by South China Sea
19. The Chinese vessel Xiang Yang Hong 10 (a) Chronicle Initiative (SCSCI), an (d) independent
significant (b) operating in Vietnam's EEZ on non-profit.
May 7, (c) representing the most (d) began A. Only a-b
incursion since 2019, according to Ray Powell, B. Only a-c
who leads Stanford University's Project Myoushu C. Only b-d
on the South China Sea. D. Only d-a
A. Only a-b E. No replacement is required
B. Only b-c
Click Here to Get the Detailed Video Solution for the above given Questions
Or Scan the QR Code to Get the Detailed Video Solutions

Answer Key with Explanation

1. Answer: D summit to be held in India and here from the


Here, the context of the paragraph is the options the the ruling party members may visit
participation of Sheikh Haseena in the G20

Click Here For Bundle PDF Course | support@guidely.in Page 5 of 8


SBI Clerk & RRB PO Mains PDF Course 2023
ENGLISH Day - 7

India is the contextually correct part to be filled in Here, the first sentence after rearrangement will
the blank. So, option D is the correct answer. be a which gives information about the fine
related statement given by the minister followed
2. Answer: C by d which connects contextually, followed by c
Here, as per the context of the paragraph which which gives further information about the
is about amritkaal declaration on the 75th meeting held followed by e which gives
anniversary of the Independence Day and here information about the act and then finally ends
option C is the only correct option to fit in the with b which gives information about the
blank. So, option C is the correct answer. preparations done after the law was passed. So,
option B is the correct answer.
3. Answer: A
Here, as per the context of the sentence which is 7. Answer: C
about the details of diplomatic passports and the Here, the first sentence after rearrangement will
whole details of the same is given and here the be a which gives information about the fine
issuance is described in the first option and so it related statement given by the minister followed
fits in the blank given. So, option A is the correct by d which connects contextually, followed by c
answer. which gives further information about the
meeting held followed by e which gives
4. Answer: B information about the act and then finally ends
Here, as per the context of the sentence which is with b which gives information about the
about the diplomatic passports and here the preparations done after the law was passed. So,
governing authority is being talked and the same option C is the correct answer.
has been explained in the second option. So,
option B is the correct answer. 8. Answer: A
Here, the sentence d will be at second place
5. Answer: D after the rearrangement and as per the context
Here, as per the context of the sentence which is of the sentence which gives information about
about the challenge named “soul of steel” and the notice and here place fits best as per the
here option D is the contextually correct. So, context. So, option A is the correct answer.
option D is the correct answer.
9. Answer: A
6. Answer: B Here, the first part after the rearrangement will
be a as it gives information about the number of

Click Here For Bundle PDF Course | support@guidely.in Page 6 of 8


SBI Clerk & RRB PO Mains PDF Course 2023
ENGLISH Day - 7

AI installed followed by b then c followed by d contextually correct. So, option A is the correct
and finally ends with e which is the end part. So, answer.
option A is the correct answer. Revamp means to change something in order to
make it more attractive or more modern.
10. Answer: A
Here, sentence a will be first after the 14. Answer: B
rearrangement and as per the context of the Here, the recognised fits best in the first blank
sentence, option A fits best. So, option A is the and issued in the second blank as both are
correct answer. contextually correct. So, option B is the correct
answer.
11. Answer: C
Here, the symbol fits best in the first blank as the 15. Answer: A
symbol of Independence is the correct phrase Here, the scheduled fits best in the first blank as
and signifies fits best in the second blank as it it means a plan of things that will happen or of
means to be a sign of something and here the work that must be done and here the same is
same is required as per the context of the said for the elections and terms should be there
sentence. So, option C is the correct answer. in the second blank as for the polls terms is used
as it means the mechanism by which people can
12. Answer: D choose their representatives for times. So,
Here, the quell fits best in the first blank as it option A is the correct answer.
means to end something and here to end
inflation is being talked and in the second blank 16. Answer: A
pressures fits best as per the context of the Here, the province and links require swapping as
sentence. So, option D is the correct answer. links between corridor and darbar sahib is the
Quench means to put out or to extinguish. correct phrase and Punjab province is the
correct one too. So, option A is the correct
13. Answer: A answer.
Here, the bolster fits best as it means to support
or encourage something and here 17. Answer: D
encouragement of tourism is being talked about Here, none of the highlighted words need
and in the second blank bilateral fits best as it swapping. So, option E is the correct answer.
means the ties between two which is
18. Answer: C

Click Here For Bundle PDF Course | support@guidely.in Page 7 of 8


SBI Clerk & RRB PO Mains PDF Course 2023
ENGLISH Day - 7

Here, the swapping is required between significant incursion are the correct phrases. So,
vandalised and including as vandalization of option D is the correct answer.
military bases and including house are the 20. Answer: B
correct phrases. So, option C is the correct Here, the swapping is required between flanked
answer. and venture as flanked means be on each or on
one side of something and here ship flanked is
19. Answer: D the correct phrase and venture means a project
Here, the swapping is required between began which is new and possibly dangerous and here
and significant as began operating and the same is required. So, option B is the correct
answer.

Click Here For Bundle PDF Course | support@guidely.in Page 8 of 8


SBI Clerk & RRB PO Mains PDF Course 2023
Reasoning Ability Day - 8 (Eng)

Reasoning Ability
1) If a meaningful word is formed by using the e) N, A, and I
third letter from the first word, fourth letter from
the second word, fifth letter from the third word 3) If in the word “INTERVAL” all the letters which
and seventh letter from the fourth word (from the are coming after “M” in the alphabetical series
left end of each word) in each set of words, then are replaced by the reverse alphabet and the
which of the following set of words formed more remaining letters will be replaced by the third
than one meaningful word? succeeding letter, and then all the letters are
I) STAR, COMMON, COMPETE, EVIDENT arranged in alphabetical order from the left, then
II) COMMON, APPEAR, POWERFUL, what is the product of the place value(as per the
SENTENCE alphabetical order) of the third letter from the left
III) STREAM, EVIDENCE, LETTER, CHAMPION and the fourth letter from the right end of the
a) Only I word thus formed?
b) Both II and III a) 72
c) Only III b) 63
d) Both I and II c) 96
e) Both I and III d) 108
e) None of these
2) If in the word “SPECTECULAR”, all the odd
positioned letters (except vowels) are replaced 4) If “1” is added to the odd digits and “2” is
by the second succeeding letter and all the odd subtracted from the even digits of the given
positioned vowels are replaced by the immediate numbers then add all the digits within the number
succeeding vowel, all the even positioned letters thus formed, then which of the following number
(except vowels) replaced by the second yields resultant as the second-highest?
preceding letter and all the even positioned a) 48635745
vowels are replaced by the immediate b) 53481862
succeeding letter, then which of the following c) 65184537
letters will be repeated more than once in the d) 18367824
new arrangement?(all the replacements are e) 78318562
done as per the alphabetical series)
a) E, N, and V 5) If the vowels of the following words are
b) I, E, and V arranged first followed by the consonants as per
c) V and N the alphabetical order, then all vowels are
d) Only N changed to the immediate next letter and the

Click Here For Bundle PDF Course | support@guidely.in Page 1 of 12


SBI Clerk & RRB PO Mains PDF Course 2023
Reasoning Ability Day - 8 (Eng)

consonants are changed to the immediate c) Fifth to the left


previous letter as per the alphabetical series, d) Third to the left
then which will be the third letter from the right e) Immediate left
end of each newly formed word in the same
order? 7) Who among the following person sits third to
I. COUNTER the right of Binod?
II. DIAMOND a) Jack
III. FAMOUS b) The one who sits immediate right of Elly
a) BML c) Deep
b) CQV d) The one who sits second to the left of Harry
c) ECQ e) Ishita
d) MCE
e) None of these 8) How many persons are sitting between Ishita
and the one who sits immediate right of Harry
Directions (6-10): Study the following information when counted from the left of Ishita?
carefully and answer the given questions. a) One
Ten persons – Abhay, Binod, Cherry, Deep, Elly, b) Three
Fida, Gayle, Harry, Ishita, and Jack are sitting c) Two
around a circular table facing the centre. The d) None
consecutive alphabetically named persons are e) More than three
not sitting together.
Two persons are sitting between Cherry and the 9) If all the persons are made to sit in
one who sits second to the left of Ishita. Three alphabetical order starting from Abhay in an
persons are sitting between Jack and Harry, who anticlockwise direction, then the position of how
sits second to the right of Cherry. Binod sits third many persons remains unchanged?
to the right of Deep. As many persons sit a) Three
between Deep and Fida as between Elly and b) None
Abhay, when counted to the left of both Fida and c) Two
Abhay. Elly sits adjacent to neither Jack nor d) More than three
Binod. Gayle doesn’t sit adjacent to Abhay. e) One
6) What is the position of Gayle with respect to
the one who sits immediate left of Ishita? 10) Which of the following statement is not true
a) Fourth to the right as per the given arrangement?
b) Second to the right a) Fida sits immediate left of Binod

Click Here For Bundle PDF Course | support@guidely.in Page 2 of 12


SBI Clerk & RRB PO Mains PDF Course 2023
Reasoning Ability Day - 8 (Eng)

b) Only four persons are sitting between Fida Ghevar and P as between the one who likes
and Elly Dosa and Q. The one who likes Ghevar and the
c) Deep sits second to the left of Harry one who likes Peda were not born in the same
d) Ishita sits immediate right of Deep year. W was not born in a month having an odd
e) None of the statements are true number of days.
11) Who among the following person was not
Directions (11-15): Study the following born in 2018?
information carefully and answer the given a) The one who likes Chowmin
questions. b) S
Eight persons – P, Q, R, S, T, U, V, and W were c) U
born in four different months viz.- March, June, d) The one who likes Peda
August, and November of two different years e) W
viz.- 2018 and 2021. Each person likes different
food items viz.- Jalebi, Samosa, Maggie, Laddo, 12) Who among the following person likes Dosa?
Peda, Chowmin, Dosa, and Ghevar. Only one a) P
person was born in each month of the same b) The one who was born immediately after V
year. c) R
The one who likes Chowmin was born in a month d) The one who was born in November 2018
having an odd number of days. V, who likes e) W
Jalebi and was born in an odd numbered year.
As many persons born before the one who likes 13) How many persons were born before the one
Chowmin as after V. R and the one who likes who likes Samosa?
Samosa were born in the same month but in a) Two
different years. Only one person was born b) As many persons born after the one who likes
between the one who likes Jalebi and R. Only Maggie
two persons were born between the one who c) Four
likes Samosa and T, who was born two persons d) As many persons born after T
after the one who likes Maggie. Q, who likes e) None
Maggie but was not born in the same month as
the one who likes Laddo. Only one person was 14) Four of the following five are alike in a certain
born between U and the one who likes Laddo. way based on the given arrangement and thus
Neither R nor T likes Laddo. U and the one who form a group. Which one of the following does
likes Peda was born in the same month. As not belong to the group?
many persons born between the one who likes a) S

Click Here For Bundle PDF Course | support@guidely.in Page 3 of 12


SBI Clerk & RRB PO Mains PDF Course 2023
Reasoning Ability Day - 8 (Eng)

b) T Book D is placed on an even numbered unit and


c) U placed immediately below the unit which has 4
d) Q boxes. The unit which has a height of 48cm is
e) P three units away from the book D. Book E is
placed adjacent to the unit whose height is 48cm.
15) Which of the following statement is TRUE as The number of books above B is one less than
per the given arrangement? the number of books below the unit which has a
a) Only two persons were born before V height of 39cm. Book B is placed immediately
b) U likes Peda below the unit which has 5 boxes. As many units
c) The one who likes Dosa was born in August between book C and the unit which has 7 boxes
2021 as between book F and book A. Book A is placed
d) Q was born immediately before the one who immediately above the unit whose height is
likes Peda 46cm. None of the units has eight boxes. The
e) None of the statements are true height of the unit having book D is 23cm less
than the height of the unit having book F. The
Directions (16-20): Study the following height of one of the units is 52cm, which is less
information carefully and answer the given than the height of the unit which has book F.
questions. 16) Which of the following book is placed in unit
There are six units placed one above the other in number 5?
a single stack. The lowermost unit is numbered 1 a) The unit whose height is 52cm
and immediately above it is numbered 2, and so b) E
on. Six books viz.- A, B, C, D, E, and F are c) The one which is placed two books above D
placed in each unit. There are certain number of d) A
boxes which are placed one above the other in e) None of these
each unit and the total height of each unit is
between 35cm to 75cm with the height of each 17) Height of which of the following unit is 42cm?
box in each unit is same. No unit has same a) The unit which has book A
height and same number of boxes. b) The unit which has book B
Note: I. If the total height of the unit is 10cm and c) The unit which has book E
the number of boxes in the same unit is 2, then d) The unit which has book D
the height of each box is 5cm. e) None of these
II. At least two and not more than nine boxes are
placed in each unit. 18) What is the sum of the heights of units 6 and
3?

Click Here For Bundle PDF Course | support@guidely.in Page 4 of 12


SBI Clerk & RRB PO Mains PDF Course 2023
Reasoning Ability Day - 8 (Eng)

a) 104cm d) B-65
b) 61cm e) A-48
c) 107cm
d) 94cm 20) Which of the following statement is not TRUE
e) None of these as per the given arrangement?
a) Book A is placed at an even numbered unit
19) Which of the following combinations of books b) The height of unit 5 is 48cm
and unit height is correct? c) Only two books are placed between C and A
a) C-46 d) Book F is placed immediately below book B
b) D-52 e) None of the statements are true
c) A-39
Click Here to Get the Detailed Video Solution for the above given Questions
Or Scan the QR Code to Get the Detailed Video Solutions

Answer Key with Explanation

1) Answer: E 3) Answer: B
I) STAR, COMMON, COMPETE, INTERVAL  LMGHIEDO  DEGHILMO
EVIDENTAMET  MATE, TEAM, MEAT, and Thus required product = (7 x 9) = 63
TAME
II) COMMON, APPEAR, POWERFUL, 4) Answer: E
SENTENCE  MERCNo meaningful word a) 48635745  26446826  38
III) STREAM, EVIDENCE, LETTER, b) 53481862  64262640  30
CHAMPIONRDEO REDO, RODE c) 65184537  46262648  38
d) 18367824 26448602  32
2) Answer: C e) 78318562  86426640 36
SPECTECULAR  UNIAVFEVNBT

Click Here For Bundle PDF Course | support@guidely.in Page 5 of 12


SBI Clerk & RRB PO Mains PDF Course 2023
Reasoning Ability Day - 8 (Eng)

5) Answer: D
I. COUNTER  EOUCNRT  FPVBMQS
II. DIAMOND  AIODDMN  BJPCCLM
III. FAMOUS  AOUFMS  BPVELR

Directions (6-10):
6) Answer: C Again, we have:

7) Answer: D  Binod sits third to the right of Deep.

8) Answer: A That means, in case (1) Deep sits second

9) Answer: E to the left of Ishita, in case (1a) Deep sits

10) Answer: C immediate left of Ishita, in case (2) Deep


sits fifth to the right of Ishita, in case (2a)
Deep sits sixth to the right of Ishita.
 As many persons sit between Deep and
Fida as between Elly and Abhay, when
counted to the left of both Fida and
Abhay.
 Elly sits adjacent to neither Jack nor
Binod.
That means, in case (1a) & case (2a) Fida
We have:
sits second to the right of Deep, in case
 Two persons are sitting between Cherry
(2) Fida sits fourth to the right of Deep,
and the one who sits second to the left of
case (1) is not valid.
Ishita.
Based on the above given information we have:
 Three persons are sitting between Jack
and Harry, who sits second to the right of
Cherry.
Since, the consecutive alphabetically
named persons are not sitting together,
thus Ishita and Jack can’t sit together.
That means, in case (1) Ishita sits third to
the right of Harry, in case (2) Ishita sits
third to the left of Harry.
Based on the above given information we have:

Click Here For Bundle PDF Course | support@guidely.in Page 6 of 12


SBI Clerk & RRB PO Mains PDF Course 2023
Reasoning Ability Day - 8 (Eng)

12) Answer: B
13) Answer: B
14) Answer: C (All of them were born in the
month having an odd number of days except the
one given in option c)
15) Answer: C
Case (1) is not valid as Elly neither sits adjacent
to Binod nor Jack.
Again, we have:
 Gayle doesn’t sit adjacent to Abhay.
That means, case (2) & case (2a) are not
valid.
Based on the above given information we have:
We have:
 The one who likes Chowmin was born in
a month having an odd number of days.
 As many persons born before the one
who likes Chowmin as after V.
 V, who likes Jalebi and was born in an
odd numbered year.
That means, in case (1) V was born in
November 2021, in case (2) V was born in
June 2021.
Based on the above given information we have:

Case (2) is not valid as Gayle and Harry can’t sit


together, case (2a) is not valid as Gayle doesn’t
sit adjacent to Abhay.

Directions (11-15):
11) Answer: D

Click Here For Bundle PDF Course | support@guidely.in Page 7 of 12


SBI Clerk & RRB PO Mains PDF Course 2023
Reasoning Ability Day - 8 (Eng)

 Q, who likes Maggie but was not born in


the same month as the one who likes
Laddo.
That means, in case (1) T was born in
March 2021, in case (2a) T was born in
August 2021, case (2) is not valid.
Based on the above given information we have:

Again, we have:
 Only one person was born between the
one who likes Jalebi and R.
 R and the one who likes Samosa were
Case (2) is not valid as because we cannot
born in the same month but in different
place the one who likes Maggie.
years.
Again, we have:
That means, in case (1) R was born in
 Only one person was born between U and
June 2021, in case (2) R was born in
the one who likes Laddo.
November 2018, in case (2a) R was born
 Neither R nor T likes Laddo.
in November 2021.
That means, in case (1) the one who likes
 Only two persons were born between the
Laddo was born in November 2018, in
one who likes Samosa and T, who was
case (2a) the one who likes Laddo was
born two persons after the one who likes
born in June 2018.
Maggie.
 U and the one who likes Peda was born in
the same month.

Click Here For Bundle PDF Course | support@guidely.in Page 8 of 12


SBI Clerk & RRB PO Mains PDF Course 2023
Reasoning Ability Day - 8 (Eng)

 As many persons born between the one  W was not born in a month having an odd
who likes Ghevar and P as between the number of days.
one who likes Dosa and Q. Thus, W was born in June.
 The one who likes Ghevar and the one Based on the above given information we have:
who likes Peda were not born in the same
year.
That means, in case (2a) T likes Dosa,
and P likes Chowmin, case (1) is not
valid.
Based on the above given information we have:

Directions (16-20):
16) Answer: A
17) Answer: D
18) Answer: A
19) Answer: C
20) Answer: E

Case (1) is not valid as the one who likes


Ghevar and the one who likes Peda were not
born in the same year.
Again, we have:

Click Here For Bundle PDF Course | support@guidely.in Page 9 of 12


SBI Clerk & RRB PO Mains PDF Course 2023
Reasoning Ability Day - 8 (Eng)

We have:
 Book D is placed on an even numbered
unit and placed immediately below the
unit which has 4 boxes.
 The unit which has a height of 48cm is
three units away from book D.
That means, in case (1) Book D is placed Again, we have:
at unit number 2, in case (2) Book D is  The number of books above B is one less
placed at unit number 4. than the number of books below the unit
 Book E is placed adjacent to the unit which has a height 39cm.
whose height is 48cm. Since, the number of boxes in each unit is
That means, in case (1) book E is placed of same height, thus only possible
at unit number 4, in case (1a) book E is combination is (3 x 13). But number of
placed at unit number 6, in case (2) book boxes can’t be more than 9, thus 3 boxes
E is placed at unit number 2. of each height 13cm are placed in unit.
Based on the above given information we have:  Book B is placed immediately below the
unit which has 5 boxes.

Click Here For Bundle PDF Course | support@guidely.in Page 10 of 12


SBI Clerk & RRB PO Mains PDF Course 2023
Reasoning Ability Day - 8 (Eng)

That means, in case (2) book B is placed  As many units between book C and the
at unit number 5, case (1) & case (1a) are unit which has 7 boxes as between book
not valid. F and book A.
Based on the above given information we have:  Book A is placed immediately above the
unit whose height is 46cm.
That means, in case (2) book F is placed
immediately above book B, in case (2a)
book F is placed immediately below book
E.
Based on the above given information we have:

Again, we have:
 The height of the unit having book D is
23cm less than the height of the unit
having book F.
Since, the height of units is more than
35cm and less than 75cm.

Case (1) & case (1a) are not valid as Book B is Thus only possible combinations are (42,

placed immediately below the unit which has 5 65), and (63, 40).

boxes.  The height of one of the units is 52cm,

Again, we have: which is less than the height of the unit


which has book F.

Click Here For Bundle PDF Course | support@guidely.in Page 11 of 12


SBI Clerk & RRB PO Mains PDF Course 2023
Reasoning Ability Day - 8 (Eng)

Thus, the height of the unit of book F


must be 65cm.
 None of the units has eight boxes.
Thus, number of boxes at unit one must
be 6.
Based on the above given information we have:

Case (2a) is not valid as the height of the unit of


book F can’t be 48cm.

Click Here For Bundle PDF Course | support@guidely.in Page 12 of 12


SBI Clerk & RRB PO Mains PDF Course 2023
Quantitative Aptitude Day - 8 (Eng)

Quantitative Aptitude

Directions [01 – 04]: Read the following b) 4800


information carefully and answer the questions c) 3200
based on it. d) 3000
The funnel chart given below gives information e) None of these
about the number of people those who are
interest to take loans. Number of people whose 2) Number of people those are interested to take
application accepted, they all got the loan from loan but don’t apply for loan is 7200, while 30%
bank. of total people apply for loan, then find number of
people those are not eligible for loan?
a) 22000
b) 16000
c) 25000
d) 28000
e) None of these

3) If respective ratio of total population to that of


number of people those apply for loan is 40:13,
find number of people those are interested to
take loan. if 17500 people got the loan.
a) 31800
b) 32200
c) 28400
d) 31200
e) Can’t be determined

Note: Q/R = 5/4


4) Number of people those are eligible for loan is
1) Number of people those loan application
16000, and number of people those are interest
rejected is 10% of total population, and the
to take loan is 60% more than those apply for
number of people those are eligible for loan is
loan, then find the total population?
8/3 times of those got loan, then find number of
a) 40000
people those are eligible for loan but are not
b) 20000
interested to take the loan.
c) 16000
a) 2400

Click Here For Bundle PDF Course | support@guidely.in Page 1 of 9


SBI Clerk & RRB PO Mains PDF Course 2023
Quantitative Aptitude Day – 8 (Eng)

d) 24000 e) Can’t be determined

Directions (05 - 09): Study the following information carefully and answer the questions given below.
There are five youtube channels. Each channel contains only four subjects [Math, Reasoning, English,
and GK] video. The given two pie chart shows the percentage distribution of the number of Math videos
on five youtube channels A,B,C,D and E respectively, and the percentage distribution of the number of
GK's video on the same five youtube channels.

Note:

Click Here For Bundle PDF Course | support@guidely.in Page 2 of 9


SBI Clerk & RRB PO Mains PDF Course 2023
Quantitative Aptitude Day – 8 (Eng)

Total number of videos on Math’s subject in five channels together is 300 and the number of videos on
GK's Subject is 400. The ratio of the number of videos from Math and reasoning's subject on all each
channels is 3:2 and the ratio of the number of videos for GK's and English subjects is 2:3.
5) Find the difference between the total number 8) Find the difference between the total number
of videos of Reasoning subjects in channels C of videos in channels A and B together and the
and D together and the number of videos of number of videos in channels C and D together?
English subjects in channels A and E together? a) 105
a) 135 b) 115
b) 145 c) 108
c) 190 d) 110
d) 185 e) None of these
e) 168
9) The Total number of videos in channel F is 10
6) Find the ratio between the total number of more than the number of videos on channel A.
videos of English subject on channels A, B, and Ratio of the number of math, English, reasoning,
C together and the total number of videos of and GK is 3:2:2:1. Find the sum of the total
Reasoning subject on channels C, D, and E videos of the Math and Reasoning subject?
together? a) 200
a) 97:38 b) 250
b) 79:83 c) 240
c) 88:37 d) 220
d) 93:35 e) None of these
e) 91:89
Directions (10 - 14): Study the following
7) Average duration of each video subject math, information carefully and answer the questions
English, reasoning, and GK is 55 min, 40 min, 50 given below.
min, and 30 min respectively in channel C. Find The amount of milk in container E is 45 liters and
the total number of hours of videos in channel C? the amount of water in containers C is 30 liters.
a) 245.5 hour The ratio of milk and water in container A is 2:1.
b) 282.5 hour Ratio of milk in containers A and B is 4:5.
c) 285.1 hour Amount of water in container B is 200% more
d) 268.2 hour than the amount of water in container A. Amount
e) None of these of milk in container C is 10% less than the
amount the milk in container B. Ratio of milk and

Click Here For Bundle PDF Course | support@guidely.in Page 3 of 9


SBI Clerk & RRB PO Mains PDF Course 2023
Quantitative Aptitude Day – 8 (Eng)

water in container D is 11:8. The amount of milk b) 40%


in containers C and E is the same. The amount c) 44%
of water in container A is 66.66% less than the d) 49%
amount of water in container E. Amount of water e) None of these
in container D is 33.33% less than that of
incontainer E. 13) If containers A and E are mixedtogether then
10) 40% of the mixture from container B is taken find the difference between milk and water in the
out then 15 L milk and 14 liters of water is added new mixture?
into the container B. Then again 20% of mixture a) 5 litres
is taken out. Find the difference between the final b) 4 litres
amount of milk and water in container B? c) 3 litres
a) 3liters d) 2 litres
b) 5liters e) None of these
c) 4liters
d) 2liters 14) If15 liters of milk and 25 liters of water are
e) 1liters added to container D and then 20% mixture is
taken out, then Find the final amount of water in
11) 10 liters of milk and 20 liters of water are container D?
added to the mixture of container A. Now 60% a) 52 litres
mixture is taken out and added in container C. b) 54 litres
Find the final ratio of milk and water in container c) 59 litres
C? d) 62 litres
a) 33:29 e) None of these
b) 21:37
c) 23:29 Directions (15-19): Study the following
d) 25:18 information carefully and answer the questions
e) 31:25 given below. There are three rows, calculate the
roots of the equation and compare them with
12) Some amount of the mixture from container other columns. Check which one matches.
B and 20% mixture from container E is mixed in 15)
container F. If the ratio of milk and water in Equation Statements
container F is 29:36, then find the percentage of i) X2-25X+150=0 a) Both roots are
the amount of mixture from container B taken? positive.
a) 50%

Click Here For Bundle PDF Course | support@guidely.in Page 4 of 9


SBI Clerk & RRB PO Mains PDF Course 2023
Quantitative Aptitude Day – 8 (Eng)

ii) X2+21X+108=0 b) Both roots are are the positive prime


negative. number.
iii) X2+13X+40=0 c) Both of the roots a) ii)-a)
are the positive prime b) i)-b)
number. c) ii)-a) & iii) -b)
a) ii)-c) d) iii)-c)
b) i)-b) e) None of these
c) i)-a) & ii) -b)
d) iii)-c) 18)
e) None of these Equation Statements
i) X2-22X+112=0 a) Both the roots are
16) positive.
Equation Statements ii) X2+13X-90=0 b) Sum of roots is
i) X2-22X+85=0 a) Sum of the roots is negative.
positive. iii) X2-9X+8=0 c) Both of the roots are
ii) X2+11X-80=0 b) Sum of roots is the positive prime
negative. number.
iii) X2+2X-48=0 c) Both of the roots are a) i)-c)
the positive prime b) ii)-a)
number. c) i)-a) & ii) -b)
a) ii)-a) d) iii)-c)
b) i)-b) e) None of these
c) ii)-a) &i) -c)
d) i)-c) 19)
e) None of these Equation Statements
i) X2-21X+90=0 a) Sum of the roots is
17) positive.
Equation Statements ii) X2+23X+132=0 b) Both roots are
i) X2-13X+36=0 a) Sum of the roots is negative.
positive. iii) X2+7X-98=0 c) Both of the roots
ii) X2+19X+84=0 b) Sum of roots is are the positive prime
negative. number.
iii) X2+15X-100=0 c) Both of the roots a) ii)-a)

Click Here For Bundle PDF Course | support@guidely.in Page 5 of 9


SBI Clerk & RRB PO Mains PDF Course 2023
Quantitative Aptitude Day – 8 (Eng)

b) i)-b) d) ii)-b)
c) iii)-b) & ii) -b) e) None of these

Click Here to Get the Detailed Video Solution for the above given Questions
Or Scan the QR Code to Get the Detailed Video Solutions

Answer Key with Explanation

1) Answer: C 2) Answer: B
According to Question, According to question,
Number of people those loan application is 30% x P = P/2% of Q
rejected is 10% of total population. Q/2 = 30
So, total population (P) = 2000/10 x 100 = 20000 Value of Q = 60
Now, So, value of R = 60/5 x 4 = 48
Q% of P = 8/3 x Z R% of P – P/2% of Q = 7200
75Q = Z…………... (1) 48% of P – 30% of P = 7200
Also, So, value of P = 7200/18 x 100 = 40000
P/2% of Q = 2000 + Z So, number of people eligible for loan = Q% of P
100Q = 2000 + Z……………. (2) = 60% of P
From (1) and (2) we get, So, number of people not eligible for loan = 40%
25Q = 2000 of P = 40% x 40000 = 16000
Value of Q = 80 Hence answer is option B
So, value of R = 4/5 x 80 = 64%
Number of people those are eligible for loan but 3) Answer: D
not interested to take loan = 80% of P – 64% of Total number of people apply for loan = 2000 +
P = 16% of P = 16% x 20000 = 3200 17500 = 19500
Hence answer is option C So, Total population = 19500/13 x 40 = 60000

Click Here For Bundle PDF Course | support@guidely.in Page 6 of 9


SBI Clerk & RRB PO Mains PDF Course 2023
Quantitative Aptitude Day - 8 (Eng)

So, P/2% of Q = 19500


So, value of Q = 65
Value of R = (65/5) x 4 = 52%
So, number of people interested to take loan =
R% of P = 52% x 60000 = 31200
Hence answer is option D

4) Answer: E
According to question, 5) Answer: C

R% of P / (P/2% of Q) = 8/5 Required difference = [132+168]-[60+50]=190

2R/Q = 8/5
R/Q = 4/5 [already given] 6) Answer: D
Total number of videos of English subject =
We don’t know the value of R or Q so we
132+90+150=372
can’tfind the value of P.
Hence answer is option E Total number of videos of Reasoning’s subject =

So, I1<I2<I3 60+50+30=140


Required ratio = 372:140=93:35

Directions (05 - 09): 7) Answer: B

The number of videos of math's subject in A is = Total duration is =

18*300/100=54 90*55+40*150+60*50+30*100=16950 min

The number of videos of Reasoning's subject in =282.5 hours

A is = 54*2/3=36
The number of videos of GK's subject in A is 8) Answer: A

=22*400/100=88 The total number of videos in channels A and B

The number of videos of English's subject in A is together is

= 88*3/2=132 = [54+36+36+24+88+60+132+90] =520


The total number of videos in channels C and D
Similarly, we can calculate the value of the other
also. together is
= [90+75+60+50+100+40+150+60] =625
Required difference = 625-520=105

9) Answer: A

Click Here For Bundle PDF Course | support@guidely.in Page 7 of 9


SBI Clerk & RRB PO Mains PDF Course 2023
Quantitative Aptitude Day - 8 (Eng)

The total number of videos in channel F is = So milk taken out from mixture E =45*20/100 =
54+36+88+132+10=320 9litres
So, the number of Math and Reasoning subject water taken out from mixture E = 60*20/100
videos is 320*5/8=200 =12litres
Let x amount of mixture from container B is
Directions (10 - 14): taken out
The amount of milk in C is 45 liters. So, [x*5/11]+9/[x*6/11]+12=29/36
The amount of milk in B is 45*100/90=50 liters Or, [(36*5*x)/11]-[(29*6*x)/11]=29*12-36*9=24
The amount of milk in A is 50*4/5=40 liters Or, 6x=24*11, or, x=44
The amount of water in A is = 40/2=20 liters So, required percentage is = [44/110]*100=40%
Amount of water in B is = 20*300/100=60 liters
Amount of water in E is =20*100/(100- 13) Answer: A
66.66)=60liters The total amount of milk is = 40+45=85
Amount of water in D is = 60*(100- The total amount of water is =60+20=80
33.33)/100=40liters So, the difference is = 85-80=5 litres
The amount of milk in D is = 40*11/8=55 liters
14) Answer: A
10) Answer: C So, final amount of water is =
The final amount of milk is [40+25]*80/100=65*80/100=52litres
= [50*(60/100)+15]*(80/100)=36liters
The final amount of water is 15) Answer: C
= [60*(60/100)+14]*(80/100)=40liters X2-25X+150=0
The required difference is 40-36=4liters Or, X2-15X-10X+150=0
Or, (X-15)(X-10)=0, X=15,10
11) Answer: D X2+21X+108=0
The amount of milk in container C is Or, X2+12X+9X+108=0
=45+[40+10]*60/100=45+30=75 Or, (X+12)(X+9)=0, X=-12,-9
The amount of water in container C is X2+13X+40=0
=30+[20+20]*60/100=30+24=54 Or, X2+8X+5X+40=0
Required ratio is = 75:54=25:18 Or, (X+8)(X+5)=0, X=-5,-8

12) Answer: B 16) Answer: D


20% mixture from container E is taken out X2-22X+85=0

Click Here For Bundle PDF Course | support@guidely.in Page 8 of 9


SBI Clerk & RRB PO Mains PDF Course 2023
Quantitative Aptitude Day - 8 (Eng)

Or, X2-17X-5x+85=0 X2-22X+112=0


Or, (X-17)(X-5)=0, X=17,5 Or, X2-14X-8x+112=0
X2+11X-80=0 Or, (X-14)(X-8)=0, X=14,8
Or, X2+16X-5X-80=0 X2+13X-90=0
Or, (X+16)(X-5)=0, X=-16,5 Or, X2+18X-5X-90=0
X2+2X-48=0 Or, (X+18)(X-5)=0
Or, X2+8X-6X-48=0 Or, X=-18,5
Or, (X+8)(X-6)=0, X=-8,+6 X2-9X+8=0
So, only i)-c) is matched. Or, X2-8X-X+8=0
Or, (X-8)(X-1)=0, X=8,1
17) Answer: C
X2-13X+36=0 19) Answer: D
Or, X2-9X-4X+36=0 X2-21X+90=0
Or, (X-9)(X-4)=0, X=9,4 Or, X2-15X-6X+90=0
X2+19X+84=0 Or, (X-15)(X-6)=0, X=15,6
Or, X2+12X+7X+84=0 X2+23X+132=0
Or, (X+12)(X+7)=0, X=-12,-7 Or, X2+12X+11X+132=0, or, (X+12)(X+11)=0,
X2+15X-100=0 X=-11,-12
Or, X2+20X-5X-100=0 X2+7X-98=0
Or, (X+20)(X-5)=0, X=-20,5 Or, X2+14X-7X-98=0
Or, (X+14)(X-7)=0, X=-14,7
18) Answer: C

Click Here For Bundle PDF Course | support@guidely.in Page 9 of 9


SBI Clerk & RRB PO Mains PDF Course 2023
ENGLISH Day - 8

English Language

Directions (1-5): Given below are a few Column 1 Column 2


questions with a table given with two columns 1. Russia said a. while the Kremlin
with highlighted connectors to be matched in Tuesday it had demanded the military
order to make sentences contextually correct as deployed jets and prevent any repeat
per the rules of connectors. If none of the options artillery to destroy an attack.
are correct then choose option E as your answer. armed group that
1. penetrated the border
Column 1 Column 2 from Ukraine,
1. The new outbreak a. since China 2. Moscow reported b. which has previously
could be the largest scrapped its zero- Russian forces killed faced shelling attacks
wave of infections Covid policy in more than 70 that have killed dozens
December 2022. Ukrainian fighters and of people since
2. Aleksandr b. who the SBU destroyed four Moscow launched its
Skachkov, arrested in 2020 during armored vehicles, offensive last year.
a raid on people selling 3. Belgorod's c. but AFP was unable
translated versions of Governor Vyacheslav to independently verify
the Christchurch Gladkov said civilians the claims.
Shooter’s Manifesto. were evacuated from
nine border villages in
3. Iran has expanded c. despite opposition the region,
its missile programme, from the United States A. 1-a, 2-b, 3-c
particularly its ballistic and expressions of B. 1-a, 2-c, 3-b
missiles, concern by European C. 1-c, 2-b, 3-a
countries. D. 1-b, 2-c, 3-a
E. None of the above
A. 1-a, 2-b, 3-c
B. 1-a, 2-c, 3-b 3.
C. 1-c, 2-b, 3-a Column 1 Column 2
D. 1-b, 2-c, 3-a 1. The annual a. but Ireland's
E. None of the above assembly in Geneva regulations are intended
decided not to to be more
2. extend Taiwan an comprehensive.

Click Here For Bundle PDF Course | support@guidely.in Page 1 of 11


SBI Clerk & RRB PO Mains PDF Course 2023
ENGLISH Day - 8

invitation to the usually treated as


event, collectibles and are
2. Ireland will b. which runs from May usually not released as
introduce mandatory 21-30. legal tender,
health labelling of 3. The Battlegrounds c. However, users will
alcoholic drinks, Mobile India, which only be able to play
warning of the links has been developed the game from May
to cancer, liver by Kraft, can now be 29.
disease and the risk preloaded by all
of drinking Android users starting
3. Other countries c. while pregnant. from today, May 27.
include warnings on A. 1-a, 2-b, 3-c
alcohol products, B. 1-a, 2-c, 3-b
A. 1-a, 2-b, 3-c C. 1-c, 2-b, 3-a
B. 1-a, 2-c, 3-b D. 1-b, 2-c, 3-a
C. 1-c, 2-b, 3-a E. None of the above
D. 1-b, 2-c, 3-a
E. None of the above 5.
Column 1 Column 2
4. 1. The previous version a. which generally
Column 1 Column 2 of the game, PUBG lowers global
1. The question for a. especially given Mobile, continues to be temperatures slightly,
investors is whether to mounting concern banned in India for the world will
jump on the AI train amongst regulators similar security experience a return to
now, or exercise about the technology's reasons and its El Nino, the warmer
caution, potentially disruptive associations with counterpart, later this
impact. China. year.
2. Higher b. On the other hand, 2. Further research on b. In addition, certain
denomination lower denomination X-rays from lawmakers expressed
commemorative coins, commemorative coins supernovae is valuable their opposition to the
which have precious usually stay in not just for availability of the
metals like gold and circulation for a limited understanding the life game in India, citing
silver in them, are time. cycle of stars, concerns about its

Click Here For Bundle PDF Course | support@guidely.in Page 2 of 11


SBI Clerk & RRB PO Mains PDF Course 2023
ENGLISH Day - 8

impact on children. c. Microsoft said the Chinese hacking group has


been active since at least 2021 and was targeted
3. Climate models c. but also has several industries including communications,
suggest that after three implications for fields manufacturing, utility, transportation,
years of the La Nina like astrobiology, construction, maritime, government, information
weather pattern in the paleontology, and the technology, and education.
Pacific Ocean, earth and planetary A. Only a
sciences. B. Only b
A. 1-a, 2-b, 3-c C. Only c
B. 1-a, 2-c, 3-b D. Both A&B
C. 1-c, 2-b, 3-a E. No error
D. 1-b, 2-c, 3-a
E. None of the above 7.
a. Under the new Twitter Blue program rolled out
Directions (6-10): Given below are a few last month, individuals can pay $8 a month for a
questions with three sentences in each of them blue check mark.
which may or may not contain errors in them. b. Under its new owner, Elon Musk, Twitter has
You have to find the sentence which has or have allowed paid subscriptions for anyone to obtain a
errors in them and mark your answers verified account in exchange for a monthly
accordingly. If none of the sentence contain charge.
errors then choose option E as your answer. c. The finance ministry said the new Rs 75 coin
6. will bear the inscription of the Parliament
a. Gripped by economic turmoil and suffering a complex and have the image of the new
balance of payments crisis, Pakistan is trying to Parliament building.
reach agreement with the International Monetary A. Only a
Fund (IMF) to disburse the stalled final $1.1 B. Only b
billion from a $6.5 billion bailout agreed in 2019. C. Only c
b. As China has stepped up military and D. Both A&C
diplomatic pressure in its claim to democratically E. No error
governed Taiwan, US President Joe Biden has
said he would be willing to use force to defend 8.
Taiwan. a. Life on Earth in itself is fragile and was a result
of millions of coincidences coming together on
the planet for us to emerge and survive.

Click Here For Bundle PDF Course | support@guidely.in Page 3 of 11


SBI Clerk & RRB PO Mains PDF Course 2023
ENGLISH Day - 8

b. A team of explorers announced it found a C. Only c


sunken Japanese ship that was transporting D. Both A&B
Allied prisoners of war when it was torpedoed off E. No error
the coast of the Philippines in 1942, resulting in
Australia’s largest maritime wartime loss with a 10.
total of 1,080 lives. a. The world's hot year on record so far was
c. The launch pad on the Satish Dhawan Space 2016, coinciding with a strong El Nino - although
Centre is set to be in a busy mode for the next climate change has fuelled extreme
few months. The Indian Space Research temperatures even in years without the
Organisation (Isro) is all set to conduct several phenomenon.
big launches in a year it has decided to not just b. Europe experienced its hottest summer on
speed up its operations but also its turnaround record in 2022, while climate change-fuelled
time for rockets. extreme rain caused disastrous flooding in
A. Only a Pakistan, and in February, Antarctic sea ice
B. Only b levels hit a record low.
C. Only c c. Despite most of the world's major emitters
D. Both A&B pledging to eventually slash their net emissions
E. No error to zero, global CO2 emissions last year
continued to rise.
9. A. Only a
a. Isro said that it is gearing up for several bug B. Only b
missions in mid-2023. The missions will not only C. Only c
explore deep space but also the Sun and the D. Both A&B
Moon. E. No error
b. ISRO will launch the much-awaited
Chandrayaan-3 mission to the Moon with the Directions (11-15): Given below are a few
next couple of months. question with two columns given and both
c. The world could breach a new average contains parts of sentences to be matched in
temperature record in 2023 or 2024, fuelled by order to make the sentence contextually correct.
climate change and the anticipated return of the If none of the options are correct then choose
El Nino weather phenomenon, climate scientists option E as your answer.
say. 11.
A. Only a
B. Only b

Click Here For Bundle PDF Course | support@guidely.in Page 4 of 11


SBI Clerk & RRB PO Mains PDF Course 2023
ENGLISH Day - 8

Column 1 Column 2 Catholics past year, from


1. Drugs are often a. while in Europe, persistent pain in his
identified with a symbol captured leaders right knee to sciatica to
representing stood trial for crimes his recent hospital stay
against humanity. for bronchitis.

2. Peru and Colombia b. the group which 3. For about a year, c. due to recurring pain
are the largest makes or moves the the pope has had to in his knee that he has
producers of coca leaf product. rely on a wheelchair said cannot be treated
through surgery.
3. After World War II, c. and cocaine in the
thousands of high- world, according to A. 1-a, 2-b, 3-c
ranking Nazi officers the United Nations. B. 1-a, 2-c, 3-b
sought refuge in South C. 1-c, 2-b, 3-a
America, D. 1-b, 2-c, 3-a
A. 1-a, 2-b, 3-c E. None of the above
B. 1-a, 2-c, 3-b
C. 1-c, 2-b, 3-a 13.
D. 1-b, 2-c, 3-a Column 1 Column 2
E. None of the above 1. Suffering a massive a. of nationalism that
economic crisis and has impacted the fate
12. surrounded by of its biggest election of
Column 1 Column 2 countries at war, modern times.
1. The pope's morning a. heads of states, Turkey is witnessing a
audiences are usually associations and resurgence
reserved for clerics, while his 2. Often referred to as b. illustrating the
afternoons are devoted the ‘world’s most continued demand
to work and private notorious’ spyware, among governments
meetings. and the lack of joint
international efforts to
2. Francis, who has b. for a decade, has limit the use of such
been the leader of the suffered increasing tools.
world's 1.3 billion health issues over the

Click Here For Bundle PDF Course | support@guidely.in Page 5 of 11


SBI Clerk & RRB PO Mains PDF Course 2023
ENGLISH Day - 8

3. It was found to c. Pegasus often found including Forbidden that her iPhone was
have been used in its traces in the circles Stories, under attack by a
another dozen of journalists, foreign government.
countries since 2021, oppositions and critics.
A. 1-a, 2-b, 3-c
A. 1-a, 2-b, 3-c B. 1-a, 2-c, 3-b
B. 1-a, 2-c, 3-b C. 1-c, 2-b, 3-a
C. 1-c, 2-b, 3-a D. 1-b, 2-c, 3-a
D. 1-b, 2-c, 3-a E. None of the above
E. None of the above
15.
14. Column 1 Column 2
Column 1 Column 2 1. WhatsApp, which is a. to defend itself, and
1. Sometime in mid- a. the Pegasus Project owned by Meta, is he has cast the
2021, Anna initiative investigated among a number of Ukraine war as a battle
Naghdalyan who is the the use of spyware by for the survival of
spokesperson governments on Russia against an
journalists, opposition aggressive West.
politicians, activists
and business people. 2. Russian President b. tech companies
Vladimir Putin says the pursuing legal action
2. Citizen Lab, a b. suspected Pegasus United States and its against the NSO group
Toronto-based operators in allies since 2019.
research organisation Azerbaijan that they
identified at least two call “BOZBASH” and 3. Putin has repeatedly c. are fighting an
“YANAR” by their warned that Russia, expanding proxy war
ongoing internet which has more against Russia after
scanning and DNS nuclear weapons than the Kremlin chief sent
cache probing. any other country, will troops into Ukraine 15
use all means months ago.
3. In a collaborative c. for Armenia’s
investigation by 17 foreign affairs agency, A. 1-a, 2-b, 3-c
media agencies was alerted by Apple

Click Here For Bundle PDF Course | support@guidely.in Page 6 of 11


SBI Clerk & RRB PO Mains PDF Course 2023
ENGLISH Day - 8

B. 1-a, 2-c, 3-b C. the NSO Group introduce their military-grade


C. 1-c, 2-b, 3-a spyware
D. 1-b, 2-c, 3-a D. the NSO Group introduced its military-grade
E. None of the above spyware
E. No replacement is required
Directions (16-20): Given below are a few
questions with a highlighted phrase in each of 18. For years, Armenia and Azerbaijan has been
them which may or may not require replacement. at loggerheads over the dispute Nagorno-
If replacement is required then choose a proper Karabakh region. While it’s internationally
replacement from the options. If no replacement recognised as part of Azerbaijan, many of its
is required then choose option E as your answer. residents are Armenian nationals.
16. NSO Group’s infamous Pegasus spyware, A. Armenia or Azerbaijan have been at
which has been earlier in the news for its misuse loggerheads over the disputed
against political and civil rights individuals, has B. Armenia and Azerbaijan has been at
found its use in a war zone. loggerheads over the disputed
A. which had been earlier in the news for its C. Armenia and Azerbaijan have been at
misuse against loggerheads over the disputed
B. which have been earlier in the news for its D. Armenia and Azerbaijan had been at
misuse against loggerheads over the disputed
C. who has been earlier in the news for its E. No replacement is required
misuse against
D. who had been earlier in the news for its 19. Russia had been an huge numerical
misuse against superiority over the United States and the NATO
E. No replacement is required military alliance when it comes to tactical nuclear
weapons: the United States believes Russia has
17. Global Intelligence and cyberwarfare in the around 2,000 such working tactical warheads.
digital age altered forever when an Israeli A. Russia have a huge numerical superiority over
company, the NSO Group introduce its military- the United States and
grade spyware Pegasus in 2011 to the global B. Russia has an huge numerical superiority over
market. the United States and
A. the NSO Group introduced his military-grade C. Russia had a huge numerical superiority over
spyware the United States and
B. the NSO Group introduced her military-grade D. Russia has a huge numerical superiority over
spyware

Click Here For Bundle PDF Course | support@guidely.in Page 7 of 11


SBI Clerk & RRB PO Mains PDF Course 2023
ENGLISH Day - 8

the United States and A. in a territorial dispute over a potentially


E. No replacement is required energy-rich stretch in a South China Sea
B. in a territorial dispute over a potentially
20. Vietnam and China have long been energy-rich stretch at the South China Sea
embroiled in a territorial dispute over a potentially C. in a territorial dispute over a potentially
energy-rich stretch in South China Sea, a energy-rich stretch in the South China Sea
strategic waterway through which more than $3 D. in a territorial dispute over an potentially
trillion of commerce passes annually. energy-rich stretch in the South China Sea
E. No replacement is required

Click Here to Get the Detailed Video Solution for the above given Questions
Or Scan the QR Code to Get the Detailed Video Solutions

Answer Key with Explanation

1. Answer: A Here, the first part of column 1 will be matched


Here, the first part of column 1 will be matched with a and second part with c and the final
wthe ith first part of column 2 as it is connected leftover part will be matched with b. So, option B
by since as we use it to give a reason for is the correct answer.
something and here the same can be seen, the
second part of column 1 will be matched with 3. Answer: D
second part of column 2 and finally the leftover Here, the first part of column 1 will be matched
will be matched. So, option A is the correct with the second part of column 2 as it is
answer. connected by which, the second part of column 1
will be connected with the third part of column 2
2. Answer: B and the final part of the first column will be

Click Here For Bundle PDF Course | support@guidely.in Page 8 of 11


SBI Clerk & RRB PO Mains PDF Course 2023
ENGLISH Day - 8

matched with the first part of column 1. So, Here, none of the sentence contains error. So,
option D is the correct answer. option E is the correct answer.

4. Answer: A 8. Answer: A
Here, all of the parts of column 1 will be matched Here, the error lies in part a as use of was before
with the parts which are just in opposite of them a result is inappropriate as sentence is in
means with the same parts of column 2. So, present tense and therefore is should have been
option A is the correct answer. used. So, option A is the correct answer.

5. Answer: D 9. Answer: B
Here, the first part of column 1 will be matched Here, the error lies in sentence b as use of with
with the second part of column 2 as both are before the next is inappropriate as within should
connected by in addition to as the same context have been used as within is used in the context
with further information is given, the second part of time as it means under certain limitations of
of column 1 will be matched with the third part of time given and here the same can be seen. So,
column 2 as both are connected with but also option B is the correct answer.
which is used to add emphasis when writing
about two related things and finally the third part 10. Answer: A
of column 1 will be matched with the first part of Here, the error lies in sentence a as use of hot is
column 2 which is connected by which. So, inappropriate because as per the context of the
option D is the correct answer. sentence the superlative degree should have
been used which is hottest. The superlative
6. Answer: C degrees are used when the noun in a particular
Here, the error lies in part c as use of was before context has the greatest or least degree of the
targeted is inappropriate as here sentence is in quality and here greatest quality is there. So,
perfect continuous tense from starting and option A is the correct answer.
second part is in present perfect tense and
therefore has should have been used here due 11. Answer: D
to perfect tense and singular subject. So, option Here, the first part of column 1 which is about
C is the correct answer. drugs should be matched with second part of
column 2 and second part of column 1 which is
7. Answer: E about the peru and Colombia being largest
producer should be matched with third part of

Click Here For Bundle PDF Course | support@guidely.in Page 9 of 11


SBI Clerk & RRB PO Mains PDF Course 2023
ENGLISH Day - 8

column 2 and finally the third part of column one Here, the first part of column 1 which is about
will be matched with first part of column 2. So, whatsapp should be matched with the second
option D is the correct answer. part of column 2, the second part of column 1
which is about putting and his statement about
12. Answer: A US should be matched with the third part of
Here, all of the parts given in first column will be column 2 and finally the third part of column 1
matched with the part given same in the column which is about warning given by Putin should be
2. So, option A is the correct answer. matched with the first part of column 2. So,
option D is the correct answer.
13. Answer: B
Here, the first part of column 1 which is about 16. Answer: E
turkey experiencing economic crisis should be Here, the phrase highlighted is grammatically
matched with first part of column 2, the second correct and no replacement is required. So,
part of column 1 which is about notorious option E is the correct answer.
spyware should be matched with the third part of
column 2 and finally the third part of column 3 17. Answer: D
which is about the use of something by countries Here, there should be introduced in place of
should be matched with the second part of introduce as the sentence is in past tense and so
column 2. So, option B is the correct answer. the second form of verb will be used in case of
past simple tense. So, option D is the correct
14. Answer: C answer.
Here, the first part of column 1 which is about
some spokesperson should be matched with the 18. Answer: C
third part of column 2, the second part of column Here, the use of has been is inappropriate as
1 which is about some research being done subject is plural and therefore have been will be
should be matched with the second part of used and also the dispute should be changes
column 2 and finally the third part of column 1 into disputed as the second part of the sentence
which is about some investigations should be is in past simple tense and therefore it will use
matched with the first part of column 2. So, second form of the verb. So, option C is the
option C is the correct answer. correct answer.

15. Answer: D 19. Answer: D

Click Here For Bundle PDF Course | support@guidely.in Page 10 of 11


SBI Clerk & RRB PO Mains PDF Course 2023
ENGLISH Day - 8

Here, the use of had been is inappropriate as 20. Answer: C


has should have been used as sentence is in Here, there should be use of ‘the’ in between in
present perfect continuous tense and also the and South China sea as the article the is used
use of an is also inappropriate as huge doesn’t before seas and here the sea is mentioned and
sound like a vowel and also h is not a vowel and therefore, the is required as an article. "The" is
therefore, a should have been used. So, option used in these case before the names of deserts,
D is the correct answer. oceans and seas. So, option C is the correct
answer.

Click Here For Bundle PDF Course | support@guidely.in Page 11 of 11


SBI Clerk & RRB PO Mains PDF Course 2023
Reasoning Ability Day - 9 (Eng)

Reasoning Ability
Direction (1-5): Each of the questions below Statement I: Point P is 4m to the west of point Q.
consists of a question and two statements Point R is 8m to the south of point Q. Point U is
numbered I and II given below it. You have to 7m to the east of point T.
decide whether the data provided in the Statement II: Point R is 5m to the west of point S.
statements are sufficient to answer the question: Point T is 3m to the north of point S. Point U is
1. Six persons - A, B, C, D, E and F are sitting in 8m to the north of point V.
a linear row facing the north direction. Who a) Only Statement I alone is sufficient to answer
among the person sits at the extreme left end of the questions
the row? b) Only Statement II alone is sufficient to answer
Statement I: Only two persons sit between D and the questions
E. C and E are immediate neighbours. A sits to c) Either statement I or statement II is sufficient
the immediate left of C. Only two persons sit to answer the questions
between A and F, who does not sit at any of the d) Neither statement I nor statement II is
extreme ends. sufficient to answer the questions
Statement II: B sits second to the left of A. C sits e) Both statements I and statement II together
second to the right of the one who sits to the are necessary to answer the questions
immediate right of B. E sits to the immediate left
of F, who sits at one of the extreme ends of the 3. What is the code for “Vision Contact” in the
row. given code language?
a) Only Statement I alone is sufficient to answer Statement I: In a code language, “Wants come
the questions contact harmful” is coded as “pk rd af jh”, “Vision
b) Only Statement II alone is sufficient to answer are contact beautiful” is coded as “af pr zm dc”,
the questions “specs awesome vision ultra” is coded as “ld as
c) Either statement I or statement II is sufficient pr dq”.
to answer the questions Statement II: In a code language, “Doctor says
d) Neither statement I nor statement II is contact useful” is coded as “af ct mt lq”, “Vision
sufficient to answer the questions must contact yourself” is coded as “af pr ab zl”,
e) Both statements I and statement II together “Care your vision must” is coded as “wa ab pr
are necessary to answer the questions qb”.
a) Only Statement I alone is sufficient to answer
2. Point P is in which direction with respect to the questions
point V? b) Only Statement II alone is sufficient to answer
the questions

Click Here For Bundle PDF Course | support@guidely.in Page 1 of 11


SBI Clerk & RRB PO Mains PDF Course 2023
Reasoning Ability Day - 9 (Eng)

c) Either statement I or statement II is sufficient Statement I: Q lives on an even numbered floor.


to answer the questions R lives two floors above Q. Only three persons
d) Neither statement I nor statement II is live between Q and V. S lives immediately above
sufficient to answer the questions V.
e) Both statements I and statement II together Statement II: R lives on an odd numbered floor.
are necessary to answer the questions S lives three floors below R. As many persons
live above S as below Q. Only two persons live
4.How is F related to E? between Q and V.
Statement I: D is the son-in-law of A. E is the son a) Only Statement I alone is sufficient to answer
of B, who is married to A. C is the only sister of the questions
E. B is the grandmother of F, who is the daughter b) Only Statement II alone is sufficient to answer
of C. the questions
Statement II: D is the son of C. C is the wife of F, c) Either statement I or statement II is sufficient
who has only two children. B is the only son-in- to answer the questions
law of F. E is the sister-in-law of A. A is married d) Neither statement I nor statement II is
to B, who has no siblings. sufficient to answer the questions
a) Only Statement I alone is sufficient to answer e) Both statements I and statement II together
the questions are necessary to answer the questions
b) Only Statement II alone is sufficient to answer
the questions Directions (6-10): Study the following information
c) Either statement I or statement II is sufficient carefully and answer the given questions.
to answer the questions Twelve boxes - A, B, C, D, E, F, G, H, I, J, K and
d) Neither statement I nor statement II is L are kept on six different shelves where the
sufficient to answer the questions bottommost shelf is numbered 1 and the topmost
e) Both statements I and statement II together shelf is numbered 6. Each box has different
are necessary to answer the questions words viz., Ion, Dish, Beans, Tom, Corn, Car,
Peas, Kin, Chilli, Jam, Flow and Drum.
5. Seven persons - P, Q, R, S, T, U and V are Note-I: Each shelf has two stacks viz., Stack-1
living on different floors of a seven storey and Stack-2, where Stack-1 is to the west of
building where the lowermost floor is numbered Stack-2.
one and the floor immediately above it is Note-II: Stack-2 of shelf 2 is immediately above
numbered two and so on. Who lives on the stack-2 of shelf 1 and immediately below stack-2
topmost floor? of shelf 3 and so on. Similarly, stack-1 of shelf 2

Click Here For Bundle PDF Course | support@guidely.in Page 2 of 11


SBI Clerk & RRB PO Mains PDF Course 2023
Reasoning Ability Day - 9 (Eng)

is immediately above stack-1 of shelf 1 and e)The box which is kept to the east of box C
immediately below stack-1 of shelf 3 and so on.
Note-III: Area of each stack on each shelf is 7. Four of the following five are alike in a certain
equal. way based on the given arrangement and thus
Note-IV: Only two boxes are kept on each shelf form a group. Which one of the following does
and only one box is kept in each stack. not belong to the group?
Box G is kept four shelves below Box F, where a) A
both are kept in different stacks. Only one shelf is b) B
between box G and box J, which is kept to the c) C
west of the box which has Beans. Box G and box d) D
J are kept in different stacks. Box I, which has e) G
Drum, is kept two shelves above Box J. Box B is
kept immediately below Box I, where both are 8. As many shelves above box J as below___
kept in the same stack. As many shelves above I. B
B as below L. Only two shelves are between L II. I
and the box which has Car, where both are kept III. H
in different stacks. Box C, which has Peas, is a) Only I
kept to the west of box E. The box which has Ion b) Only II
is kept immediately below E, where both are kept c) Only I and III
in different stacks. Box H, which has Tom, is kept d) Only I and II
above the box which has Ion in the same stack. e) All I, II and III
Box D, which has Jam, is kept in an even
numbered stack. The box which has Dish is kept 9. Which of the following statement is true as per
immediately above the box which has Chilli in the the given arrangement?
same stack. Box F does not have Kin. Only two a) Box B is kept in stack 1
shelves are between the box which has Flow and b) Box J is kept on an even numbered shelf
K. Corn is not kept on an even numbered shelf. c) The box which has Peas is kept on the
Box J doesn’t have Flow. bottommost shelf
6. Which of the following box has Kin? d) Two shelves are between box I and box C
a) A e) None is true
b) B
c) The box which is kept immediately above C in 10. Box F has which of the following word?
the same stack a) Car
d) L b) Corn

Click Here For Bundle PDF Course | support@guidely.in Page 3 of 11


SBI Clerk & RRB PO Mains PDF Course 2023
Reasoning Ability Day - 9 (Eng)

c) Dish lives three floors below V, who lives above 9th


d) Chilli floor. Q lives on a prime numbered floor above V
e) Ion but not immediately above and both live in
different type of flats. At most 4 floors are
Directions (11-15): Study the following between V and Q, who lives on the topmost floor.
information carefully and answer the given As many floors above Q as below Y, who lives to
questions. the east of P.
Certain number of floors are there in a building 11. How many floors are between the flat of C
where the lowermost floor is numbered one, the and E?
floor immediately above it is numbered two and a) Two
so on. Only some of the flats are occupied by b) Four
some persons. c) Six
Note-I: Each floor has two type of flats viz., Flat- d) Eight
A and Flat-B, where Flat A is to the west of flat B. e) Ten
Note-II: Flat B of floor 2 is immediately above
Flat B of floor 1 and immediately below Flat B of 12.How many floors are there in the building?
floor 3 and so on. Similarly, Flat A of floor 2 is a) 11
immediately above Flat A of floor 1 and b) 17
immediately below Flat A of floor 3 and so on. c) 13
Note-III: Area of each flat on each floor is equal. d) 19
X lives on an odd numbered floor but below sixth e) 23
floor. Four floors are between X and C, where
both of them live in different type of flats. D lives 13. Four of the following five are alike in a certain
two floors above C and both live in the same type way based on the given arrangement and thus
of flat. No one lives to the east of D. As many form a group. Which one of the following does
floors between X and D as between C and M, not belong to the group?
who lives below the fifth floor. E lives to the east a) V
of M. R lives on the floor whose number is two b) R
more than thrice the floor number of E, where c) X
both live in different type of flats. U lives three d) M
floors above R in the same type of flat. As many e) D
floors between U and D as between M and L,
who lives four floors below H in the same type of 14. Who among the following person does not
flat. Both L and M live in different type of flats. H live above X?

Click Here For Bundle PDF Course | support@guidely.in Page 4 of 11


SBI Clerk & RRB PO Mains PDF Course 2023
Reasoning Ability Day - 9 (Eng)

I. The one who lives on the same floor as E * means either the hour or minute hand of the
II. C clock is at 7
III. The one who lives on the same floor as V % means either the hour or minute hand of the
a) Only I clock is at 10
b) Only II £ means either the hour or minute hand of the
c) Only I and III clock is at 11
d) Only I and II © means either the hour or minute hand of the
e) All I, II and III clock is at 4
∑ means either the hour or minute hand of the
15. L lives on which of the following floor and clock is at 1
flat? Ω means either the hour or minute hand of the
a) Flat B, floor 3 clock is at 2
b) Flat A, floor 2 > means PM
c) Flat B, floor 4 < means AM
d) Flat B, floor 5 Note: The first symbol represents an hour and
e) None of these the second symbol represents minutes. (Ex: %@
means 10 hours 15 minutes)
Directions (16-20): Study the following 16.Peter takes 20 minutes to reach the office
information carefully and answer the given from his home while coming back to home he
questions takes 15 minutes more due to traffic. If Peter
The following symbols represent time in a clock: started walking from his office at ©!>, then when
$ means either the hour or minute hand of the did he reach home?
clock is at 12 a) #@>
@ means either the hour or minute hand of the b) ?@>
clock is at 3 c) *?<
& means either the hour or minute hand of the d) ©%>
clock is at 6 e) Cannot be determined.
# means either the hour or minute hand of the
clock is at 9 17.If Ajay started walking from his home at ?*>
? means either the hour or minute hand of the and reached the beach at &!>, then what is the
clock is at 5 total time taken by him to reach the beach?
! means either the hour or minute hand of the a) 1 hour 5 minutes
clock is at 8 b) 1 hour 45 minutes
c) 1 hour 35 minutes

Click Here For Bundle PDF Course | support@guidely.in Page 5 of 11


SBI Clerk & RRB PO Mains PDF Course 2023
Reasoning Ability Day - 9 (Eng)

d) 1 hour 25 minutes destination which is 300km from the starting


e) None of these point?
a) &%>
18. Kumar started his daily routine, he spends 2 b) ©$>
hours for walking, 1 hour for exercise, 30 c) £#<
minutes for fresh up, 15 minutes for breakfast d) @$>
and he reached the office at #£<. If he takes 40 e) Cannot be determined
minutes for travelling to the office, then when did
he start the daily routine? 20.Ram and Vishnu started running at ‘?$<’. If
a) $#< Ram is twice as faster as Vishnu and reached
b) ?#< the destination at ‘&©<’, then when did Vishnu
c) ?&< reach the destination?
d) ??< a) ?#<
e) Cannot be determined b) £*<
c) *!<
19. A bus can travel 180km in 3 hours. If the bus d) @&<
started at #$< and it has taken 15 minutes break e) Cannot be determined
in every hour, then when did the bus reach the
Click Here to Get the Detailed Video Solution for the above given Questions
Or Scan the QR Code to Get the Detailed Video Solutions

Answer Key with Explanation

Directions (1-5):
1. Answer: B
Statement 1

Click Here For Bundle PDF Course | support@guidely.in Page 6 of 11


SBI Clerk & RRB PO Mains PDF Course 2023
Reasoning Ability Day - 9 (Eng)

From Statement II we can’t get the answer

From Statement I we can’t get the answer 4. Answer: C


Statement 2 Statement 1

From statement II we get B as the answer.

2. Answer: E
From Statement I we can get the answer
Statement 2

From Statement II we can get the answer


So, either Statement I or Statement II is
By combining statements I and II, we can get the
sufficient to answer
answer

5. Answer: A
3. Answer: A
Statement 1
Statement 1

From Statement I we can get the answer


Statement 2

Click Here For Bundle PDF Course | support@guidely.in Page 7 of 11


SBI Clerk & RRB PO Mains PDF Course 2023
Reasoning Ability Day - 9 (Eng)

We have,
 Box G is kept four shelves below Box F,
From Statement I we can get the answer
where both are kept in different stacks.
Statement 2
 Only one shelf is between box G and box
J, which is kept to the west of the box
which has Beans.
 Box I, which has Drum, is kept two
shelves above Box J.
 Box G and box J are kept in different
stacks.
From the above conditions, we have two
possibilities,

From Statement II we can’t get the answer

Directions (6-10):
6. Answer: C
7. Answer: E(All the boxes are kept on an even
numbered shelf except option e)
Again we have,
8. Answer: C
 Box B is kept immediately below Box I,
9. Answer: D
where both are kept in the same stack.
10. Answer: B
 As many shelves above B as below L.
Final arrangement:
 Only two shelves are between L and the
box which has Car, where both are kept in
different stacks.

Click Here For Bundle PDF Course | support@guidely.in Page 8 of 11


SBI Clerk & RRB PO Mains PDF Course 2023
Reasoning Ability Day - 9 (Eng)

Final arrangement:

Again we have,
 The box which has Dish is kept
immediately above the box which has
Chilli in the same stack.
 Box F does not have Kin.
 Only two shelves are between the box
which has Flow and K.
 Corn is not kept on an even numbered
shelf.
 Box J doesn’t have Flow.
We cannot place the box which has corn in case
2. Hence it is eliminated. Thus, case 1 gives the
final arrangement.
We have,
 X lives on an odd numbered floor but
below the sixth floor
 Four floors are between X and C, where
both of them live in different type of flats.
 D lives two floors above C and both live in
the same type of flat.
 No one lives to the east of D.
Directions (11-15):
From the above conditions, we have three
11. Answer: C
possibilities,
12. Answer: B
13. Answer: E (All the persons live in flat A
except option e)
14. Answer: A
15. Answer: D

Click Here For Bundle PDF Course | support@guidely.in Page 9 of 11


SBI Clerk & RRB PO Mains PDF Course 2023
Reasoning Ability Day - 9 (Eng)

 As many floors between U and D as


between M and L, who lives four floors
below H in the same type of flat.
 Both L and M live in different type of flats.
 H lives three floors below V, who lives
above 9th floor.

Again we have,
 As many floors between X and D as
between C and M, who lives below the
fifth floor.
 E lives to the east of M.
 R lives on the floor whose number is two
more than thrice the floor number of E,
where both live in different type of flats.
 U lives three floors above R in the same
type of flat.
We cannot place M in case 3. Hence it is
eliminated.

Again we have,
 Q lives on a prime numbered floor above
V but not immediately above and both live
in different type of flats.
 As many floors above Q as below Y, who
lives to the east of P.
 At most 4 floors are between V and Q,
who lives on the topmost floor.
We cannot place Y and P in case 2. Hence it is
eliminated. Thus, case 1 gives the final
Again we have, arrangement.

Click Here For Bundle PDF Course | support@guidely.in Page 10 of 11


SBI Clerk & RRB PO Mains PDF Course 2023
Reasoning Ability Day - 9 (Eng)

18. Answer: C
Total time taken for daily routine is 3 hours 45
minutes.
When we include the 40 minutes travel time, he
should start the daily routine 4:25 minutes before
the time he reached the office.
So, he should have started the daily routine at
5:30 AM.

19. Answer: D
The speed of the bus is 60 km per hour. So it
travels for 60km and stops for 15 minutes.
The bus starts at 9 AM.
The bus will take 5 hours to reach the
destination without stoppage.
It will stop 4 times during the travel. So the
stoppage time is 1 hour.
16. Answer: B So, the total journey takes 6 hours.
Peter started from his office by 4:40 PM The bus will reach the destination at 3 PM.
It takes 35 minutes while coming back to home. So, the answer is @$>
So, Peter will reach the home by 5:15 PM. The
answer is ?@> 20. Answer: C
Both of them started running at 5:00 AM
17. Answer: A Ram reached the destination at 6:20 AM
Ajay started walking at 5:35 PM Ram is twice as faster as Vishnu. So Vishnu will
He reached the beach at 6:40 PM reach the destination at 7:40 AM.
So, the total journey time is 1 hour 5 minutes. So, *!A< is the answer.

Click Here For Bundle PDF Course | support@guidely.in Page 11 of 11


SBI Clerk & RRB PO Mains PDF Course 2023
Quantitative Aptitude Day - 9 (Eng)

Quantitative Aptitude

Directions (1-5): Study the following information 3) Find the total time taken by boat A to travel
carefully and answer the given questions. 180 km in upstream and 150 km in downstream
Speed of boat A in downstream is (s + 21) km/hr together?
and ratio of speed of boat A in downstream and A.5 hours
upstream is 5:3. Boat B can cover a distance of B.4.5 hours
360 km in downstream in 6 hours and ratio of C.6 hours
speed of boat A and B in still water is 4:3, D.8 hours
respectively. Speed of boat C in still water is (s – E.7 hours
14) km/hr and it can catch boat D, which is 50 km
ahead of it in 5 hours, when both the boats are 4) If downstream speed of another boat M is
travelling in the same direction. 120% of that of same of boat A, then find the
Note: All the boats (A, B, C and D) are rowing in time taken by boat M to travel 240 km in
the same river/stream. upstream direction given that speed of stream is
1) What is the ratio of speed of boat B in still 15km/hr?
water to speed of boat D in still water? A.5 hours
A.2:3 B.3.5 hours
B.5:3 C.4.5 hours
C.3:2 D.4 hours
D.4:3 E. None of these
E. None of these
5) If speed of boat Q in still water is equal to the
2) Speed of boat C in still water is how much sum of speeds of boats A and boat B in still
percent of the speed of the stream? water, then find the time taken by boat Q to
A.266.67% travel 300 km downstream given that speed of
B.166.67% stream is 15 km/hr?
C.233.67% A.2 hours
D.133.67% B.2.5 hours
E.366.67% C.1 hour
D.3 hours
E.4 hours

Click Here For Bundle PDF Course | support@guidely.in Page 1 of 10


SBI Clerk & RRB PO Mains PDF Course 2023
Quantitative Aptitude Day - 9 (Eng)

Directions (6-10): Study the following information carefully and answers the questions given below.
The Radar graph given below shows the income (in Rs. Hundreds) of the two companies X and Y.

Note:- Profit Percentage = (Income – Expenditure)/Expenditure × 100


6) Average income of Company Y over all the C.17% more
given years is approximately how much D.11% less
percentage more or less than the average E.None of these
income of Company X over all the given years?
A.15% more 8) Income of Y and income of X in 2019
B.12% less increases by 15% and 32% over the previous
C.9 % more year. Income of X in 2019 and 2020 are in the
D.4% less ratio of 8 : 9 then find the income of Y in 2019 is
E.None of these how much percentage more or less than the
income of X in 2020?
7) If profit percentage earned by both the A.20% more
companies in the year 2013 are equal and the B.17% less
expenditure of X in 2013 is Rs. 10,000, then C.15%more
expenditure of Company Y in 2013 is how much D.12% less
percentage more or less than the expenditure of E.None of these
X in 2013? (Approximately).
A.21% more 9) Profit percentage for Company Y in 2013 is
B.14% less 20% and profit percentage for Company X in

Click Here For Bundle PDF Course | support@guidely.in Page 2 of 10


SBI Clerk & RRB PO Mains PDF Course 2023
Quantitative Aptitude Day - 9 (Eng)

2017 is 25%. If the profit percentage is 10) Find the ratio of the average of the total
calculated on the income then find what will be expenditure of the Company X in all the given
the ratio of expenditure for Company Y in 2013 years to the average of the total expenditure of
and Company X in 2017? the Company Y in all the given years together?
A.55 : 43 A.11 : 15
B.25 : 81 B.15 : 22
C.64 : 55 C.17 : 9
D.36 : 25 D.5 : 12
E.None of these E.Cannot be determined

Directions (11-15): Study the following data carefully and answer the questions:
5 machines A, B, C, D and E produce different numbers of units of an article in 1 minute.
Bar graph given below shows the following information:
A + B = Sum of number of units produced by machines A and B in 1 minute.
B + C = Sum of number of units produced by machines B and C in 1 minute.
C + D = Sum of number of units produced by machines C and D in 1 minute.
D + E = Sum of number of units produced by machines D and E in 1 minute.

Note:
1). Number of units produced by machine D is as more than that produced by machine A as the number
of units produced by machine A is more than that produced by machine E.

Click Here For Bundle PDF Course | support@guidely.in Page 3 of 10


SBI Clerk & RRB PO Mains PDF Course 2023
Quantitative Aptitude Day - 9 (Eng)

11) If efficiency of machine A were 30% less number of units produced by machines C and F
than of its original efficiency and efficiency of together in 30 minutes?
machine D were 33(1/3) % less than of its A.910
original efficiency, then what would be the ratio B.830
of number of units produced by machine A in 25 C.900
minutes to that produced by machine D in 15 D.990
minutes? E.870
A.7:6
B.14:13 14) Find the difference between number of units
C.5:4 produced by machines A and B together in 25
D.10:9 minutes and that produced by machines D and E
E.21:20 together in 20 minutes?
A.10
12) If efficiency of machine B were 33(1/3) % B.0
more than of its original efficiency and efficiency C.20
of machine C were 16(2/3) % less than of its D.5
original efficiency, then what would be the total E.15
number of units produced by machines B and C
together in 20 minutes? 15) Number of units produced by machines A, B
A.640 and E together in 1 minute is what percent of that
B.600 produced by machines C and D together in 1
C.620 minute?
D.680 A.83.33%
E.660 B.90%
C.87.5%
13) If number of units produced by machines D D.84%
and F together in 15 minutes is 675, then find the E.80%

Directions (16-20): Study the following data carefully and answer the questions:
A panel of Economists, was studying three parameters GDP, Population Density and GDP per capita for
5 cities of a state (Chennai, Salem, Coimbatore, Trichy and Madurai). GDP per capita (Size of bubble) is
defined as the ratio of GDP of that city to the population of that city. Population Density (y - axis) of a city
is defined as the ratio of the population of city to the total area of city, bubble chart shows GDP (USD
million – x axis) and population density (persons per km – y axis).

Click Here For Bundle PDF Course | support@guidely.in Page 4 of 10


SBI Clerk & RRB PO Mains PDF Course 2023
Quantitative Aptitude Day - 9 (Eng)

Note:
The colour of each bubble represents each city. The Green, blue, grey, yellow and brown represents
Chennai, Coimbatore, Salem, Trichy and Madurai respectively.
16) Find the ratio of total population of city 18) If all cities got ranks as per the increasing
Chennai to city Trichy? order of their area. Then find the 2nd rank city?
A.5:6 A.Chennai
B.4:5 B.Coimbatore
C.6:5 C.Salem
D.5:4 D.Madurai
E.5:7 E.Both a and d

17) Area of city Coimbatore is how much percent 19) Find the average number of population in
more than that of City Salem? (Approx) Salem, Coimbatore and Madurai?
A.150% A.28*10⁴
B.50% B.30*10⁴
C.100% C.32*10⁴
D.200% D.33*10⁴
E.225% E.36*10⁴

Click Here For Bundle PDF Course | support@guidely.in Page 5 of 10


SBI Clerk & RRB PO Mains PDF Course 2023
Quantitative Aptitude Day - 9 (Eng)

20) Find the population of the cities with the least C.125000
total area? D.200000
A.150000 E. Both a and d
B.320000
Click Here to Get the Detailed Video Solution for the above given Questions
Or Scan the QR Code to Get the Detailed Video Solutions

Answer Key with Explanation

Directions (1-5): Speed of boat A in upstream = 3x = 3 × 15 = 45


Let speed of boat A in upstream and km/hr
downstream be ‘3x’ km/hr and ‘5x’ km/hr, Speed of stream = x = 15 km/hr
respectively. Speed of boat A in still water = 4x = 4 × 15 = 60
Speed of boat A in still water = (5x + 3x)/2 = ‘4x’ km/hr
km/hr Since, speed of boat A in downstream = (s + 21)
So, speed of stream = (5x – 3x)/2 = ‘x’ km/hr km/hr
Speed of boat B in downstream = 360/6 = 60 So, s + 21 = 75
km/hr Or, s = 54
Speed of boat B in still water = (60 – x) km/hr Speed of boat B in still water = 60 – 15 = 45
ATQ; km/hr
4x/(60 - x) = 4/3 Speed of boat B in upstream = 45 – 15 = 30
12x = 240 – 4x km/hr
16x = 240 Since, speed of boat C in still water = s – 14 = 54
x = 15 – 14 = 40km/hr
So, speed of boat A in downstream = 5x = 5 × 15 Speed of boat C in upstream = 40 – 15 = 25
= 75 km/hr km/hr

Click Here For Bundle PDF Course | support@guidely.in Page 6 of 10


SBI Clerk & RRB PO Mains PDF Course 2023
Quantitative Aptitude Day - 9 (Eng)

Speed of boat C in downstream = 40 + 15 =


55km/hr 3) Answer: C
Let speed of boat D in still water is ‘p’km/hr Total time taken = 180/45 + 150/75 = 4 + 2 = 6
Since, boat C can catch boat D which is 50 km hours.
ahead of it in 5 hours, when both the boats are
travelling in the same direction. 4) Answer: D
Relative speed of boat C w.r.t. boat D = (40 – p) Downstream speed of boat M = 1.20 × 75 = 90
km/hr km/hr
So, 50/(40 – p) = 5 Upstream speed of boat M = 90 – 2 × 15 = 60
40 – p = 10 km/hr
p = 30 Required time = 240/60 = 4 hours.
So, speed of boat D in still water = p = 30km/hr
Speed of boat D in upstream = 30 – 15 = 5) Answer: B
15km/hr Downstream speed of boat Q = 60 + 45 + 15 =
Speed of boat D in downstream = 30 + 15 = 120 km/hr
45km/hr Time taken = 300/120 = 2.5 hours.
6) Answer: D
Average income of Company Y over all the
years is,
= [100 × (120 + 100 + 140 + 80 + 120 + 160)]/6
= (100 × 720)/6
= 72,000/6 => Rs.12,000
Average income of Company X over all the
years is,
1) Answer: C
= [100 × (140 + 120 + 100 + 130 + 110 + 150)]/6
According to question,
= (100 × 750)/6
Speed of the boat B in still water = 45km/hr
= 75,000/6 =>Rs. 12,500
Speed of the boat D in still water = 30km/hr
So, the required percentage = (12,500 –
Required ratio = 45:30 = 3:2
12,000)/12,500 × 100
= 500/12,500 × 100
2) Answer: A
= 4%
Speed of the stream = 15km/hr
Hence, the required answer = 4% less.
Speed of boat C in still water = 40km/hr
Required percentage = 40/15 × 100 = 266.67%.

Click Here For Bundle PDF Course | support@guidely.in Page 7 of 10


SBI Clerk & RRB PO Mains PDF Course 2023
Quantitative Aptitude Day - 9 (Eng)

7) Answer: B x = 120 - 24
Let the expenditure of Company Y in 2013 = Rs. x = 96
x Similarly, for Company X,
Then, according to the question, Let the expenditure of Company X in 2017 = y
(14,000 – 10,000)/10,000 = (12,000 – x)/x Then, according to the question,
4000/10,000 = (12,000 – x)/x y + 25/100 × 110 = 110
2/5 = (12,000 – x)/x y + 55/2 = 110
2x = 60,000 –5x y = 110 – 55/2
7x = 60,000 y = 165/2
x ≈ 8571 So, the required ratio = x : y = 96 : 165/2
So, the required percentage = (10,000 – = 192 : 165 => 64 : 55
8571)/10,000 × 100 Hence, the required answer = 64 : 55.
= 1429/10,000 × 100 ≈ 14% less
Hence, the required answer = 14% less(approx.) 10) Answer: E
Since we don’t have any information about the
8) Answer: B expenditure and the profit percentage.
Income of Y in year 2019 = 16,000 × 115/100 = So, we cannot find the solution of this question.
18,400
Income of X in year 2019 = 15,000 × 132/100 = Directions (11-15):
19,800 A + B = 32 --------------(1)
So, the income of X in year 2020 = (19,800/8) × B + C = 30 --------------(2)
9 = 22,275 C + D = 48 --------------(3)
Thus, the required percentage = (22,275 – D + E = 40 --------------(4)
18,400)/22,275 × 100 Sincenumber of units produced by machine D is
= 3875/22,275 × 100 ≈ 17% less as more than that produced by machine A as the
Hence, the required answer = 17% less. number of units produced by machine A is more
than that produced by machine E.
9) Answer: C So,
Let the expenditure of Company Y in 2013 = Rs. D–A=A–E
x D + E = 2A --------------(5)
Then, according to the question, From equations (4) and (5):
x + 20/100 × 120 = 120 2A = 40
x + 24 = 120 A = 20

Click Here For Bundle PDF Course | support@guidely.in Page 8 of 10


SBI Clerk & RRB PO Mains PDF Course 2023
Quantitative Aptitude Day - 9 (Eng)

From equation (1): And number of units produced by machine D in


B = 12 15 minutes with 66(2/3) % of its original
From equation (2): efficiency = 20 * 15 = 300
C = 18 Required ratio = 350:300 = 7:6
From equation (3):
D = 30 12) Answer: C
From equation (4): Number of units produced by machine B in 1
E = 10 minute with its original efficiency = 12
Number of units produced by machine A in 1 So, number of units produced by machine B in 1
minute = 20 minute with 133(1/3) % of its original efficiency =
Number of units produced by machine B in 1 133(1/3) % of 12 = 16
minute = 12 Number of units produced by machine C in 1
Number of units produced by machine C in 1 minute with its original efficiency = 18
minute = 18 So, number of units produced by machine B in 1
Number of units produced by machine D in 1 minute with 83(1/3) % of its original efficiency =
minute = 30 83(1/3) % of 18 = 15
Number of units produced by machine E in 1 Total number of units produced by machines B
minute = 10 and C together in 20 minutes with their new
11) Answer: A efficiencies = (16 + 15) * 20 = 620
According to question,
Number of units produced by machine A in 1 13) Answer: D
minute with its original efficiency = 20 Number of units produced by machine D in 1
So, number of units produced by machine A in 1 minute = 30
minute with 70% of its original efficiency = 70% So, number of units produced by machine F in 1
of 20 = 14 minute = (675/15) – 30 = 15
And number of units produced by machine A in Number of units produced by machine C in 1
25 minutes with 70% of its original efficiency = minute = 18
14 * 25 = 350 So, number of units produced by machines C
Number of units produced by machine D in 1 and F together in 30 minutes:
minute with its original efficiency = 30 (18 + 15) * 30 = 990
So, number of units produced by machine D in 1
minute with 66(2/3) % of its original efficiency = 14) Answer: B
66(2/3) % of 30 = 20

Click Here For Bundle PDF Course | support@guidely.in Page 9 of 10


SBI Clerk & RRB PO Mains PDF Course 2023
Quantitative Aptitude Day - 9 (Eng)

Number of units produced by machines A and B


together in 25 minutes:
(20 + 12) * 25 = 800
Number of units produced by machines D and E
together in 20 minutes:
(30 + 10) * 20 = 800
Required difference = 800 – 800 = 0 16) Answer: C
Required ratio = (15*10⁴) : (12.5*10⁴) = 6 : 5

15) Answer: C
Number of units produced by machines A, B and 17) Answer: C

E together in 1 minute: Required % = (860 - 427)/427]*100 = 101.4% ~

20 + 12 + 10 = 42 100%

Number of units produced by machines C and D


together in 1 minute: 18) Answer: E

18 + 30 = 48 Increasing order area = 180, 250, 250, 427, 860

Required percentage = (42/48) * 100 = 87.5% 2nd rank = 250 i.e. Chennai and Madurai.

Directions (16-20): 19) Answer: E

In chennai, Required average = [(32+56+20)/3]*10⁴ = 36*10⁴

Population = (120*10⁶)/ 800 = 15*10⁴


Area = (15*10⁴)/ 600 = 250 20) Answer: C

Similarly we can calculate other cities. The least area is Trichy.


The population of Trichy is 125000.

Click Here For Bundle PDF Course | support@guidely.in Page 10 of 10


SBI Clerk & RRB PO Mains PDF Course 2023
ENGLISH Day - 9

English Language

Directions (1-7): Given below are a few redeeming them by using money he obtained
questions based on the passage. You have to from pawning other pieces of temple jewellery. In
read the passage carefully and answer the 2016, Senapathi pawned 66 pieces of gold
questions accordingly. If none of the options are jewellery from the temple on 172 occasions, the
correct then choose option E as your answer. report said. He continued this practice between
An Indian chief priest of Singapore's Hindu 2016 and 2020, redeeming all the jewellery and
temple was sentenced to six years in jail on 30th returning it to the temple before the audit was
june for misappropriating over 2 million scheduled. Once the audit was completed, he
Singapore Dollars ($1.5 million) of jewellery would pawn the jewellery again to return the
repeatedly from a prominent temple, according to borrowed money. Senapathi got SGD 2,328,760
media reports. Kandasamy Senapathi was from pawn shops between 2016 to 2020, of
appointed as a priest at Sri Mariamman Temple which he remitted about SGD 141,000 to India
by the Hindu Endowments Board in the and deposited the remaining amount into his
downtown Chinatown district from December bank account. In March 2020, at the height of the
2013 until he resigned on March 30, 2020. He Covid-19 pandemic in Singapore, the audit was
was found guilty of criminal breach of trust by delayed due to the "circuit breaker" measures
dishonest misappropriation and two charges of forbidding non-essential activity in Singapore.
remitting criminal proceedings out of the country, Later during the June 2020 audit, Senapathi
with six other charges also considered during the misled members of the temple finance team that
conviction, according to the reports of Channel he did not have the key to the safe and said he
News Asia. had likely forgotten the key in India while visiting
Senapathi, an Indian national, was caught during family.
the Covid-19 pandemic which disturbed the However, when the staff member insisted that
regular audit timing and revealed the missing the audit be done, Senapathi eventually
jewellery. confessed that he had taken the jewellery for
In 2014, keys and combination numbers for the pawning. Later, all the jewellery was returned to
safe in the temple's holy sanctum were handed the temple, and the temple suffered no loss, the
to Senapathi, which contained 255 pieces of gold prosecutor said. A police report was filed against
jewellery owned by the temple, with a book value the temple priest by a member of the temple
of about SGD 1.1 million. committee.
Senapathi started pawning pieces of jewellery in According to the prosecutor, Senapathi had
2016, taking them to pawn shops and later resigned after the incident. A seven-year jail

Click Here For Bundle PDF Course | support@guidely.in Page 1 of 10


SBI Clerk & RRB PO Mains PDF Course 2023
ENGLISH Day - 9

sentence was also demanded by the prosecutor handed to Senapathi, which contained 255
for pointing to the high pawn value of the pieces of gold jewellery owned by the temple,
jewellery involved. Senapathi, however, in his with a book value of about SGD 1.1 million.
defence, said that he wanted to help a friend B. In 2017, Senapathi pawned 66 pieces of gold
raise funds for cancer and to help schools and jewellery from the temple on 172 occasions, the
temples in India. While pronouncing the verdict, report said.
the judge said he could not ignore that the case C. While pronouncing the verdict, the judge said
involved about SGD 2 million, a significant he could not ignore that the case involved about
amount and higher than any previous similar SGD 2 million, a significant amount and higher
cases. In a statement following the hearing, the than any previous similar cases.
Hindu Endowments Board (HEB) said it had D. Both A&B
commissioned a gold audit after the incident at E. None of the above
its four temples - Sri Mariamman, Sri Srinivasa
Perumal, Sri Sivan and Sri Vairavimada 3. Which of the following information about
Kaliamman. The audit confirmed that all jewellery Senapati given in the passage is incorrect?
was adequately accounted for. An expert A. An Indian chief priest of Singapore's Hindu
goldsmith also certified that the jewellery temple was sentenced to six years in jail on 30th
Senapathi had returned was authentic. "HEB has june for misappropriating over 2 million
further tightened its governance and internal Singapore Dollars ($1.5 million) of jewellery
controls to ensure its charitable assets remain repeatedly from a prominent temple.
protected," the report quoted the board as B. Kandasamy Senapathi was appointed as a
saying. priest at Sri Mariamman Temple by the Hindu
1. Which of the following is the synonym of Endowments Board in the downtown Chinatown
PROMINENT? district from December 2013 until he resigned on
A. subtle March 30, 2020.
B. shrouded C. Senapathi started pawning pieces of jewellery
C. obscure in 2016, taking them to pawn shops and later
D. salient redeeming them by using money he obtained
E. None of the above from pawning other pieces of temple jewellery.
D. Senapathi got SGD 2,328,760 from pawn
2. Which of the following information is incorrect shops between 2016 to 2020, of which he
as per the given passage? remitted about SGD 141,000 to India and
A. In 2014, keys and combination numbers for deposited the remaining amount into his bank
the safe in the temple's holy sanctum were

Click Here For Bundle PDF Course | support@guidely.in Page 2 of 10


SBI Clerk & RRB PO Mains PDF Course 2023
ENGLISH Day - 9

account. 6. Which of the following is not one of the


E. None of the above temples in which gold audit had been
commissioned by HEB?
4. Which of the following is the antonym of A. Sri Mariamman
EVENTUALLY? B. Sri Srinivsasa Perumal
A. momentarily C. Sri sivanapata
B. ultimately D. Sri vairamvimada kaliamman
C. never E. None of the above
D. forthwith
E. none of the above 7. Which of the following phrase about charges
imposed on Senapathi is correct?
5. From the line, “According to the prosecutor, A. He was found guilty of criminal breach of trust
Senapathi had resigned after the incident”. by dishonest misappropriation and six charges of
Which incident is being talked here? remitting criminal proceedings out of the country,
A. He misled the numbers of the temple finance with six other charges also considered during the
team that he did not have the key to the safe and conviction
said he had likely forgotten the key in USA while B. He was found guilty of criminal breach of trust
visiting family. by dishonest misappropriation and six charges of
B. He misled the numbers of the temple finance remitting criminal proceedings out of the country,
team that he did not have the key to the safe and with two other charges also considered during
said he had likely forgotten the key in pakistan the conviction
while visiting family. C. He was found guilty of criminal breach of trust
C. He misled the numbers of the temple finance by dishonest misappropriation and two charges
team that he did not have the key to the safe and of remitting criminal proceedings out of the
said he had likely forgotten the key in India while country, with two other charges also considered
visiting family. during the conviction
D. He misled the numbers of the temple finance D. He was found guilty of criminal breach of trust
team that he did not have the key to the safe and by dishonest misappropriation and two charges
said he had likely forgotten the key in Australia of remitting criminal proceedings out of the
while visiting family. country, with six other charges also considered
E. None of the above during the conviction
E. None of the above

Click Here For Bundle PDF Course | support@guidely.in Page 3 of 10


SBI Clerk & RRB PO Mains PDF Course 2023
ENGLISH Day - 9

Directions (8-12): Given below are a few 10. During the demonetisation (a) exercise in
questions with four highlighted words that may or 2016, when the Centre announced a (b) ban on
may not need rearrangement to make a the old Rs 500 and Rs 1,000 currency notes,
contextually correct sentence. If no there were reports of people (c) queues while
rearrangement is required then choose option E waiting in (d) fainting to exchange the notes.
as your answer. A. Only a-b
8. Earlier this month, the RBI (a) circulation Rs B. Only b-c
2,000 notes from (b) withdrew but requested C. Only c-d
people not to panic as it will continue to be legal D. Only d-a
tender. The central bank has, however, asked E. No rearrangement is required
citizens to get all such currency notes (c)
exchanged or (d) deposited at banks before 11. Quick approval by both the House and
September 30, 2023. Senate would (a) ensure government checks will
A. Only a-b continue to go out to Social Security recipients,
B. Only b-c veterans and others, and (b) upheaval financial
C. Only c-d (c) prevent worldwide by (d) allowing Treasury to
D. Only d-a keep paying U.S. debts.
E. No rearrangement is required A. Only a-b
B. Only b-c
9. Reserve Bank of India (RBI) Governor C. Only c-d
Shaktikanta Das said it would not be (a) D. Only d-a
surprising if India's GDP surpasses the 7 per E. No rearrangement is required
cent mark this year as he (b) asserted
confidence in the country's economic growth. 12. Russia (a) allies a full-scale invasion of
The Governor's (c) remarks came during the Ukraine in February 2022, (b) devastating cities,
annual (d) session of the Confederation of Indian forcing millions of people to (c) flee their homes
Industry (CII) in Mumbai. and costing thousands of lives. Moscow calls the
A. Only a-b war a "special military operation" to "denazify"
B. Only b-c Ukraine and protect Russian speakers. Kyiv and
C. Only c-d its (d) launched say it is an unprovoked land
D. Only d-a grab.
E. No rearrangement is required A. Only a-b
B. Only b-c
C. Only c-d

Click Here For Bundle PDF Course | support@guidely.in Page 4 of 10


SBI Clerk & RRB PO Mains PDF Course 2023
ENGLISH Day - 9

D. Only d-a A. violent protests erupted after the arrest Khan


E. No rearrangement is required by paramilitary Ranger
B. violent protests erupted after arrest of Khan
Directions (13-17): Given below are a few paramilitary Rangers
questions with a phrase highlighted which may or C. violent protests erupted after the arrest of
may not require improvement. You have to find Khan paramilitary Rangers
the correct improved part from the given options. D. violent protests erupted after arrest of Khan by
If no improvement is required then choose option paramilitary Rangers
E as your answer. E. No improvement is required
13. Nevada’s Joe Lombardo on 30th may 2023
became one of the first Republican governors to 15. The recent skirmishes come amid rising
enshrine protections for out-of-state abortion tensions among the two countries over the flow
patients and in-state providers, adding the of water from the Helmand River in Afghanistan
western swing state to the list of those passing to the eastern region of Iran. It is being reported
new laws to solidify here status as safe havens that the eastern region has been plagued by
for abortion patients. severe drought conditions.
A. to solidify their status as safe havens for A. amid rising tensions between the two
abortion patients countries on the flow of water
B. to solidify her status as safe havens for B. amid rising tensions among the two countries
abortion patients on the flow of water
C. to solidify his status as safe havens for C. amid rising tensions between the two
abortion patients countries at the flow of water
D. to solidify its status as safe havens for D. amid rising tensions between the two
abortion patients countries over the flow of water
E. No improvement is required E. No improvement is required

14. On May 9, violent protests erupted after the 16. Taliban in Afghanistan seeks recognition on
arrest of Khan by paramilitary Rangers in the global platform, but United States had stated
Islamabad. His party workers vandalised over 20 that it will not recognize the Taliban government
military installations and government buildings, unless it meets specific conditions, which include
including the Lahore Corps Commander House, respecting human rights and preventing
Mianwali airbase and the ISI building in Afghanistan from becoming a safe haven for
Faisalabad. terrorists.

Click Here For Bundle PDF Course | support@guidely.in Page 5 of 10


SBI Clerk & RRB PO Mains PDF Course 2023
ENGLISH Day - 9

A. but United States has stated that it will not (iii) Despite the government figures
recognize the Taliban government (a) PM Modi dedicated the National War
B. but the United States have stated that it will Memorial, a tribute to the martyrs of India, to the
not recognize the Taliban government nation. This War Memorial is a tribute to all the
C. but United States have stated that it will not soldiers of the army who have lost their lives for
recognize the Taliban government the country since independence.
D. but the United States has stated that it will not (b) this memorial is for those 26,000 martyrs of
recognize the Taliban government independent India who attained martyrdom in
E. No improvement is required one war or the other.
A. Only i
17. Belarus, led by President Alexander B. Only ii
Lukashenko since 1994, is Russia's staunchest C. Only iii
ally among ex-Soviet states and allowed its D. Both A&B
territory to be used to launch the Kremlin E. None of the above
invasion of Ukraine in February 2022.
A. its territory to be used launch the Kremlin's 19.
invasion of Ukraine in February 2022 (i) While Musk can go on
B. its territory to be used to launch Kremlin's (ii) Furthermore
invasion of Ukraine in February 2022 (iii) Until the Sam Altman
C. territory to be used to launch the Kremlin's (a) Twitter boss Elon Musk has often shared his
invasion of Ukraine in February 2022 thoughts about living on Mars. The tech mogul
D. its territory to be used to launch the Kremlin's has even said in the past that the hopes to have
invasion of Ukraine in February a ‘self-sustaining city’ on Mars in the next two
2022 decades.
E. No improvement is required (b) He and on about the idea of living on another
planet, his former business partner, OpenAI CEO
Directions (18-20): Given below are a few Sam Altman, has opposite views.
questions with two sentence and starters and we A. Only i
have to find which of the starters fits best as per B. Only ii
the context. If none of the options are correct C. Only iii
then choose option E as your answer. D. Both A&C
18. E. None of the above
(i) Hence the government figures
(ii) According to government figures 20.

Click Here For Bundle PDF Course | support@guidely.in Page 6 of 10


SBI Clerk & RRB PO Mains PDF Course 2023
ENGLISH Day - 9

(i) According to the AI company (b) Once installed, the malware can steal
(ii) Until and unless the company sensitive information from the victim's device,
(iii) Adding to the statement such as contacts, messages, and banking
(a) Hence CloudSEK, DogeRAT malware is credentials.
disguised as legitimate mobile applications such A. Only i
as a game, productivity tool, or entertainment B. Only ii
app like Netflix, YouTube, and so on. It is further C. Only iii
distributed through social media and messaging D. Both A&B
apps like Telegram. E. None of the above
Click Here to Get the Detailed Video Solution for the above given Questions
Or Scan the QR Code to Get the Detailed Video Solutions

Answer Key with Explanation

1. Answer: D 3. Answer: E
Here, the prominent means likely to attract Here, none of the options are correct as the
attention and here salient means the same information given in the passage is exactly same
whereas all other options are the antonyms of as the information given in the options about
prominent. So, option D is the correct answer. Senapathi. So, option E is the correct answer.

2. Answer: B 4. Answer: C
Here, as per the passage, the 66 pieces of gold Here, the eventually means at a later time, and
jewellery from the temple on 172 occasions were here never is the antonym whereas all other
pawned in 2016 not in 2017 and therefore given options are synonyms. So, option C is the
information given in option second is incorrect. correct answer.
So, option B is the correct answer.
5. Answer: C

Click Here For Bundle PDF Course | support@guidely.in Page 7 of 10


SBI Clerk & RRB PO Mains PDF Course 2023
ENGLISH Day - 9

Here, from the lines, Later during the June 2020 News Asia. We can say that from the above
audit, Senapathi misled members of the temple lines that option D is the correct answer.
finance team that he did not have the key to the
safe and said he had likely forgotten the key in 8. Answer: A
India while visiting family. Here, the rearrangement is required between
However, when the staff member insisted that circulation and withdrew as RBI withdrew Rs two
the audit be done, Senapathi eventually thousand notes from circulation is the correct
confessed that he had taken the jewellery for phrase. So, option A is the correct answer.
pawning. Later, all the jewellery was returned to
the temple, and the temple suffered no loss, the 9. Answer: E
prosecutor said. A police report was filed against Here, no rearrangement is required in any of the
the temple priest by a member of the temple words highlighted. So, option E is the correct
committee. According to the prosecutor, answer.
Senapathi had resigned after the incident. We
can say that option C is the correct answer. 10. Answer: C
Here, rearrangement is required between c and
6. Answer: C d as fainting of people is the correct one and
Here, from the lines, In a statement following the waiting in queues too. Fainting means near to
hearing, the Hindu Endowments Board (HEB) unconsciousness and queues means standing in
said it had commissioned a gold audit after the an orderly manner and here swapping among
incident at its four temples - Sri Mariamman, Sri them is required. So, option C is the correct
Srinivasa Perumal, Sri Sivan and Sri answer.
Vairavimada Kaliamman. We can say that option
C is the correct answer. 11. Answer: B
Here, upheaval and prevent needs to rearrange
7. Answer: D as upheaval means a sudden big change that
Here, from the lines, He was found guilty of cause a lot of trouble and financial upheaval is
criminal breach of trust by dishonest the correct phrase and preventing it is another
misappropriation and two charges of remitting correct one. So, option B is the correct answer.
criminal proceedings out of the country, with six
other charges also considered during the 12. Answer: D
conviction, according to the reports of Channel Here, rearrangement is required between
launched and allies as allies means friendly

Click Here For Bundle PDF Course | support@guidely.in Page 8 of 10


SBI Clerk & RRB PO Mains PDF Course 2023
ENGLISH Day - 9

relation and here Kyiv and its allies is the correct 17. Answer: D
phrase and Russia launched is another correct Here, improvement is required as Kremlin’s
one. So, option D is the correct answer. should have been used in place of Kremlin as
Kremlin’s invasion is the correct phrase,
13. Answer: A apostrophe is placed after noun to show that
Here, the improvement is needed as the use of noun owns something. So, option D is the
here is inappropriate as it is contextless and their correct answer.
should have been used because it shows
possession and here possession is needed as 18. Answer: B
per the context. So, option A is the correct Here, according to the government figures
answer. should be used in the second sentence as it fits
contextually. So, option B is the correct answer.
14. Answer: E The sentence made will be
Here, no improvement is required in the part PM Modi dedicated the National War Memorial,
highlighted. So, option E is the a tribute to the martyrs of India, to the nation.
correct answer. This War Memorial is a tribute to all the soldiers
of the army who have lost their lives for the
15. Answer: D country since independence. According to the
Here, improvement is required as use of among government figures this memorial is for those
is inappropriate because it is used for 26,000 martyrs of independent India who
comparison of more than two whereas between attained martyrdom in one war or the other.
is used for comparison between exactly two. So,
option D is the correct answer. 19. Answer: A
Here, while Musk can go on should be used in
16. Answer: D the second sentence as it fits contextually. So,
Here, improvement is required as use as there option A is the correct answer.
should be use of the before United States as The sentence made will be
article the is used before United States and also Twitter boss Elon Musk has often shared his
the use of had is inappropriate and here has thoughts about living on Mars. The tech mogul
should have been used to make it contextually has even said in the past that the hopes to have
and grammatically correct. So, option D is the a ‘self-sustaining city’ on Mars in the next two
correct answer. decades. While Musk can go on and on about
the idea of living on another planet, his former

Click Here For Bundle PDF Course | support@guidely.in Page 9 of 10


SBI Clerk & RRB PO Mains PDF Course 2023
ENGLISH Day - 9

business partner, OpenAI CEO Sam Altman, has According to the AI company CloudSEK,
opposite views. DogeRAT malware is disguised as legitimate
mobile applications such as a game, productivity
20. Answer: A tool, or entertainment app like Netflix, YouTube,
Here, according to the AI company should be and so on. It is further distributed through social
used in the first sentence as it fits contextually. media and messaging apps like Telegram. Once
So, option A is the correct answer. installed, the malware can steal sensitive
The sentence made will be information from the victim's device, such as
contacts, messages, and banking credentials.

Click Here For Bundle PDF Course | support@guidely.in Page 10 of 10


SBI Clerk & RRB PO Mains PDF Course 2023
Reasoning Ability Day - 10 (Eng)

Reasoning Ability
Direction (1-5): Study the following information d) C, E
carefully and answer the given questions. e) J, B
Ten persons - A, B, C, D, E, F, G, H, I and J are
sitting around a circular table facing the centre. 3. Which of the following pair of persons, the
There are twelve seats, where two seats are left second person sits to the immediate right of the
vacant. The vacant seats are not adjacent to person who faces the first person?
each other. a) BH
F sits fifth to the left of B. Two seats are between b) DJ
F and G. One person sits between G and J, who c) FB
doesn’t sit adjacent seat of B. H sits third to the d) IG
left of J. Three persons sit between H and C. e) CE
None of the vacant seats is adjacent to A, who
sits third to the right of C. A neither faces nor sits 4. What is the position of I with respect to J?
adjacent seat to B. The number of persons sitting a) Third to the left
between A and D is one more than the number b) Seventh to the left
of persons sitting between D and I when counted c) Fourth to the left
from the right of D. More than three persons sit d) Sixth to the right
between E and I when counted from both sides e) Fifth to the left
of E.
1. Who among the following person sits third to 5. Who among the following pair of persons are
the right of the one who faces E? sitting in the adjacent seat of the vacant seats?
a) B a) BA
b) Vacant b) GH
c) The one who sits to the immediate left of J c) CF
d) The one who sits opposite to D d) DH
e) Both c and d e) AG

2. As many persons sitting between D and __ as Direction (6-10): Study the following information
between F and __, when counted from the left of carefully and answer the given questions.
both D and F respectively. Seven boxes are kept one above the other in a
a) B, H single stack. Each box has different number of
b) G, A pins in it, which is a two digit number.
c) E, D

Click Here For Bundle PDF Course | support@guidely.in Page 1 of 9


SBI Clerk & RRB PO Mains PDF Course 2023
Reasoning Ability Day - 10 (Eng)

Box G, which has 7 pins more than C, is kept 8. If the rate of each pin is Rs.5, then what is the
four boxes below the box which has prime total cost of pins in boxes B, E and C?
number of pins. Only one box is kept between G a) Rs.385
and the box which has 20 pins. The number of b) Rs.425
boxes kept above the box which has 20 pins is c) Rs.415
one more than the number of boxes kept below d) Rs.430
E, which has 5 pins less than D. The box which e) Rs.400
is kept immediately above E has an even number
of pins. A, which has an odd number of pins, is 9. As many boxes kept between A and ____ as
kept three boxes above the box which has 33 between E and ___.
pins. B is kept immediately above F and has the a) F, the box which is kept two boxes above F
number of pins which is a multiple of 9. F has the b) C, The box which has 33 pins
least number of pins and three pins less than A. c) G, the box which has 20 pins
D has 13 pins more than C. The difference d) E, The box which has 23 pins
between the number of pins in B and D is 11. e) None of these
6. If Box C is kept below box F, then which of the
following box is kept two boxes above box B? 10. How many boxes are kept between G and
a) The box which has 20 pins the box which has 4 pins less than B?
b) C a) One
c) G b) Two
d) The box which has 38 pins c) Three
e) E d) None
e) More than three
7. Which of the following statements is/are not
false as per the given arrangement? Direction (11-15): Study the following information
a) The sum of the number of pins in boxes A and carefully and answer the given questions.
G is an even number Eight persons - S, D, G, V, K, Q, Z and N belong
b) Box E has more pins than C but less pins than to the family sitting in a linear row facing north.
B Only three generations are in the family.
c) Only two boxes are kept between F and C Only two persons sit between V and her father-
d) All the given statement is true in-law, who sits third from one of the extreme
e) All the given statement is false ends. The number of persons sitting to the right
of V is one more than the number of persons
sitting to the left of G. The grand-son of G sits

Click Here For Bundle PDF Course | support@guidely.in Page 2 of 9


SBI Clerk & RRB PO Mains PDF Course 2023
Reasoning Ability Day - 10 (Eng)

three places away from him. Q and V don’t sit 14. If X is married to K and sits between Q and
together. N sits fourth to the left of her only D, then which of the following statement is not
nephew. Z, who is the grandfather of K, sits false?
second to the right of N. The number of persons a) X is the son-in-law of the one who sits third
sitting between Q and D is one less than the from the left end
number of persons sitting between D and his b) X sits exactly in the middle of the row
grandmother, who doesn’t sit at any of the c) X sits fourth to the right of the one who is the
extreme ends. G has only one child. V is the aunt of K
sister-in-law of N, who is not married. S is d) Both a and b
married to Z. e) Both b and c
11. Who among the following person sits second
to the left of the one who is the niece of N? 15. Four of the following five are alike in a certain
a) S way based on the given arrangement and thus
b) The one who is the father of D form a group. Which one of the following does
c) G not belong to the group?
d) The one who sits third to the right of Z a) NQ
e) None of these b) ZV
c) GD
12. How is the one who is the daughter-in-law of d) QS
S related to the one who sits at the extreme left e) GQ
end?
a) Sister Direction (16-20): Study the following information
b) Mother-in-law carefully and answer the given questions.
c) Sister-in-law Seven persons - I, J, K, L, M, N and O are in a
d) Mother family of three generations went to the party in
e) Can’t be determined different months viz., January, March, April, July,
September, October and December of the same
13. What is the position of S with respect to the year.
one who is the son-in-law of G? M went to the party three months before her
a) Third to the left sister-in-law. Five months are between the
b) Third to the right months in which M and O’s niece went to the
c) Second to the left party. The number of persons went to the party
d) Fifth to the right before O’s niece is one less than the number of
e) Fifth to the left persons went to the party after O. Only two

Click Here For Bundle PDF Course | support@guidely.in Page 3 of 9


SBI Clerk & RRB PO Mains PDF Course 2023
Reasoning Ability Day - 10 (Eng)

persons went to the party between O and N. L spent Rs.1700 for gifts, then what is the total
went to the party two persons before his amount spent by J, Nephew of I and mother of
grandfather, who is not M. No one went to the L?
party between K and her mother-in-law. I, who is a) Rs.4100
not married, is the sister-in-law of the mother of b) Rs.5100
L. N and J are of the same gender.M is the c) Rs.4600
grandmother of N’s son. d) Rs.3600
16. Who among the following person went to the e) Rs.5600
party in March?
a) The one who went to the party two months 19. The number of persons went to the party
before I before the one who is the mother-in-law of K is
b) The one who is the daughter of J two less than the number of persons went to the
c) O party after ___.
d) The one who went to the party immediately a) J
after M b) The one who is the sister-in-law of K
e) None of these c) N
d) The one who went to the party two months
17. How many months are there between the after I
one who is married to K and the one who is the e) O
aunt of I?
a) One 20. If all the persons went to the party as per the
b)Two alphabetical order from January, then how many
c) Three persons do not go to the party in the same
d) Four month?
e) More than four a) One
b) Six
18. If the persons went to the party in the month c) Three
which has an even number of days spent d) No one
Rs.1200 for gifts and the remaining persons e) Five

Click Here For Bundle PDF Course | support@guidely.in Page 4 of 9


SBI Clerk & RRB PO Mains PDF Course 2023
Reasoning Ability Day - 10 (Eng)

Click Here to Get the Detailed Video Solution for the above given Questions
Or Scan the QR Code to Get the Detailed Video Solutions

Answer Key with Explanation

Directions (1-5):
1. Answer: B
2. Answer: C
3. Answer: D
4. Answer: E Again we have,
5. Answer: B  H sits third to the left of J.
Final arrangement:  Three persons sit between H and C.
 None of the vacant seats is adjacent to A,
who sits third to the right of C.
 A neither faces nor sits adjacent seat to
B.

We have,
 F sits fifth to the left of B. Again we have,
 Two seats are between F and G.  The number of persons sitting between A
 One person sits between G and J, who and D is one more than the number of
doesn’t sit adjacent seat of B. persons sitting between D and I when
From the above conditions, we have three counted from the right of D.
possibilities:  More than three persons sit between E
and I, when counted from both sides of E.

Click Here For Bundle PDF Course | support@guidely.in Page 5 of 9


SBI Clerk & RRB PO Mains PDF Course 2023
Reasoning Ability Day - 10 (Eng)

While applying the above conditions, case 1 and  The box which is kept immediately above
3 gets eliminated, because can’t place E and I. E has an even number of pins.
Thus, case 2 gives the final arrangement. From the above conditions, we have two
possibilities:

Directions: (6-10):
6. Answer: D
7. Answer: C
8. Answer: B
9. Answer: D Again we have,
10. Answer: C  A, which has an odd number of pins, is
Final arrangement: kept three boxes above the box which has
33 pins.
 B is kept immediately above F and has
the number of pins which is a multiple of
9.
 F has the least number of pins and three
pins less than A.
 D has 13 pins more than C.
From the above conditions, case 1a gets
eliminated because C has a prime number of
We have, pins.
 Box G, which has 7 pins more than C, is
kept four boxes below the box which has
prime number of pins.
 Only one box is kept between G and the
box which has 20 pins.
 The number of boxes kept above the box
which has 20 pins is one more than the
Again we have,
number of boxes kept below E, which has
5 pins less than D.

Click Here For Bundle PDF Course | support@guidely.in Page 6 of 9


SBI Clerk & RRB PO Mains PDF Course 2023
Reasoning Ability Day - 10 (Eng)

 The difference between the number of  Only two persons sit between V and her
pins in boxes B and D is 11. father-in-law, who sits third from one of
While applying the above conditions, case 1 gets the extreme ends.
eliminated, because box B has pins, which is a  The number of persons sitting to the right
multiple of 9. Thus, case 2 gives the final of V is one more than the number of
arrangement. persons sitting to the left of G.
 The grandson of G sits three places away
from him.
From the above conditions, we have three
possibilities:

Directions (11-15)
11. Answer: D Note: G.son means Grand-son; FIL means
12. Answer: C Father-in-law; G.ma means Grand-mother
13. Answer: B Again we have,
14. Answer: E  Q and V don’t sit together.
15. Answer: E (In the given option, two persons  N sits fourth to the left of her only
sit between the given pair of persons, except in nephew.
option e)  Z, who is the grandfather of K, sits second
Final arrangement: to the right of N.
 The number of persons sitting between Q
and D is one less than the number of
persons sitting between D and his
grandmother, who doesn’t sit at any of the
extreme ends.
While applying the above conditions, case 3 gets
eliminated, because the grand-mother of D can’t
We have,
sit at extreme ends.

Click Here For Bundle PDF Course | support@guidely.in Page 7 of 9


SBI Clerk & RRB PO Mains PDF Course 2023
Reasoning Ability Day - 10 (Eng)

Again we have,
 G has only one child.
 V is the sister-in-law of N, who is not
married.
 S is married to Z.
While applying above conditions, case 2 gets
eliminated, because N’s nephew and V’s father-
in-law can’t be a same person. Thus, case 1 We have,

gives the final arrangement.  M went to the party three months before
her sister-in-law.
 Five months are between the months in
which M and O’s niece went to the party.
 The number of persons went to the party
before O’s niece is one less than the
number of persons went to the party after
O.
From the above conditions, we have three
possibilities:
Note: sil means sister-in-law; grandpa means
Directions (16-20): grandfather.
16. Answer: D
17. Answer: E
18. Answer: C
19. Answer: D
20. Answer: B
Final arrangement:

Again we have,

Click Here For Bundle PDF Course | support@guidely.in Page 8 of 9


SBI Clerk & RRB PO Mains PDF Course 2023
Reasoning Ability Day - 10 (Eng)

 Only two persons went to the party  I, who is not married, is the sister-in-law of
between O and N. the mother of L.
 L went to the party two persons before his  N and J are of the same gender.
grandfather, who is not M.  M is the grandmother of N’s son.
Case 2 gets eliminated, because N can’t be the
sister-in-law of M and case 3 gets eliminated,
because can’t place K. Thus, case 1 gives the
final arrangement.

Again we have,
 No one went to the party between K and
her mother-in-law.

Click Here For Bundle PDF Course | support@guidely.in Page 9 of 9


SBI Clerk & RRB PO Mains PDF Course 2023
Quantitative Aptitude Day - 10 (Eng)

Quantitative Aptitude

Direction (1- 5): Study the given information carefully and answer the given questions.
There are six washermen – A, B, C, D, E, and F. The line graph given below shows the time taken by
these washermen to wash 100 kg of clothes individually. The table given below shows the rate charges
of washing different types of clothes per kg.

Notes:
i. The time taken by C to wash 100 kg of clothes is 75% of the time taken by A to wash 100 kg of clothes.
ii. The ratio of time taken by B to wash 100 kg of clothes to the time taken by D to wash 100 kg of clothes
is 4: 5.
1. Washerman E washes ‘d’ kg clothes which
Shirts, Pants, and Coats are there. He gets Rs.

Click Here For Bundle PDF Course | support@guidely.in Page 1 of 16


SBI Clerk & RRB PO Mains PDF Course 2023
Quantitative Aptitude Day - 10 (Eng)

7950 for the wash in which 10 kg are Coats. The b. 132 hours
number of Shirts is 15 kg more than the number c. 128 hours
of Pants. Find the value of d. d. 124 hours
a. 65 kg e. 136 hours
b. 78 kg
c. 75 kg 4. A, B, and D start to wash ___ kg of clothes.
d. 77 kg They started together, but after ___ hours, A left
e. None of these the work, and after ___ hours, D left the work. B
completes the remaining work in 12 hours.
2. Washerman F takes a tenderto wash 76.5 kg i. 2250, 24, 40
of Jeans and Woollen clothes in the ratio 5: ii. 1620, 36, 52
4.Find the revenue collected by washerman F on iii. 2900, 60, 100
this tender. a. All statements i, ii, and iii are correct
a. Rs.13225 b. Statements i and iii are correct
b. Rs.12325 c. Statements i and ii are correct
c. Rs.15345 d. Statement ii and iii are correct
d. Rs.12225 e. None of these
e. None of these
5. The efficiency of washerman C is increased by
3. A, C, and F take a tender to wash 3750 kg of ‘J’%, then C and D together wash 240 kg of
clothes together. They start working together, but clothes in 10 hours. Find the value of J.
after working 72 hours, C left the work and after a. 26
working 108 hours F left the work. The remaining b. 25
work is completed by A alone. Find the total c. 29
number of hours took by them to complete the d. 20
work together. e. 28
a. 138 hours

Direction (6 –10): Study the given information carefully and answer the given questions.

Click Here For Bundle PDF Course | support@guidely.in Page 2 of 16


SBI Clerk & RRB PO Mains PDF Course 2023
Quantitative Aptitude Day - 10 (Eng)

Notes:
i. Cost price of cabbage per kg is Rs. 4 more than the selling price of tomato per kg.
ii. Onions are sold at a profit of 50% per kg.
iii. The cost price of cabbage per kg is 250% of the cost price of potato per kg.
6. If potatoes are sold at a profit of Rs. ‘d’ and 8. Hotel Fine Dine makes a cuisine of onion,
the onions are sold at a discount of d% then the potato, and tomato together. It takes 25 kg of
selling price of potatoes and onions are the potatoes, 15 kg of tomatoes, and 10 kg of
same. Find the value of d. onions, and adds some spice mixtures to this
a. 28 cuisine. After making the cuisine, sold it for Rs.
b. 24 51 per kg at a profit of 65%. If the quantity of
c. 20 spices mixed in the cuisine is 5 kg, find the cost
d. 25 price of the per kg spice mixture.
e. 16 a. Rs.55
b. Rs.65
7. A shopkeeper Vaishali bought x kg of c. Rs.50
tomatoes at the Cost price. 15 kg of tomatoes d. Rs.60
gets rotten after sometime. Rotten tomatoes e. None of these
were sold at Rs. 20 per kg to a Sauce factory
and the rest of the tomatoes are sold at Rs. 50 9.
per kg to gain a profit of 6.25 % on thetotal Quantity I: Profit on cauliflower is what percent of
tomatoes. Find the value of x. the discount on the potatoes.
a. 60 Quantity II: If the selling price of cauliflower is
b. 65 increased by Rs. 10 then the profit percentage
c. 45 on cauliflower is what percent of the discount
d. 50 percentage on potatoes.
e. 55 a. Quantity I > Quantity II
b. Quantity I ≥ Quantity II

Click Here For Bundle PDF Course | support@guidely.in Page 3 of 16


SBI Clerk & RRB PO Mains PDF Course 2023
Quantitative Aptitude Day - 10 (Eng)

c. Quantity II > Quantity I 11. The ratio of a to b is 12: 13. Investment of


d. Quantity II ≥ Quantity I Abhishek in Q3 is Rs. 6500 less than the
e. None of these investment of Rahul in Q3 and Rs. 4000 less
than his investment in Q2. Find the percentage
10. If the marked price of cauliflower is ___% of of profit share of Chandan at the end of Q3 if
the cost price of cauliflower and sold at a Karan invested 25% less than Abhishek in Q2.
discount of ___% then there is a profit of Rs. ___ a. 4900 / 259 %
on selling the onions. b. 4900 / 251 %
i. 50, 25, 3 c. 4900 / 255 %
ii. 25, 10, 3 d. 4900 / 257 %
iii. 30, 20, 2 e. None of these
a. Only ii and iii are correct
b. Only i and ii are correct 12. If at the end of Q4 ratio of profit of Karan,
c. Only i and iii are correct Abhishek, Rahul, and Chandan is 25:33:37: 31.
d. All i, ii, iii are correct Find the ratio of a, b, and c.
e. None of these a. 3: 5: 7
b. 3: 5: 2
Direction (11-15): Study the given information c. 7: 5: 2
carefully and answer the given questions. d. 2: 3: 5
In a business, there are four partners – Karan, e. None of these
Abhishek, Rahul, and Chandan. Initially, Karan,
Abhishek, Rahul, and Chandan invested Rs. x + 13.The ratio of profit of Karan, Abhishek, Rahul,
3000, Rs. y + 4000, Rs. y + 7000, and Rs. x + and Chandan is 23:31:34: 26.If business ends
7000 respectively. At the end of the 1 st quarter, after the 3rd quarter. Rahul got a profit of Rs.
they had invested some additional amount in the 25500, then find the profit share of Karan.
ratio 3:a:2: 1 respectively. At the end of the 2 nd i. y – ((y/10) + (x/20))
quarter, they invested some additional amount in ii. x + (ab)2 10
the ratio 2:4:b: 3 respectively. And at the end of iii. x + 0.25 y – 10 ab
the 3rd quarter, they invested some additional a. Only i and iii are correct
amount in the ratio c: 2: 3: 5 respectively. b. Only ii and iii are correct
The initial investment of Karan is 75% of c. Only i and ii are correct
Abhishek’s initial investment. The ratio of d. All I, ii, and iii are correct
Abhishek’s initial investment to Rahul’s initial e. None of these
investment is 8: 9.

Click Here For Bundle PDF Course | support@guidely.in Page 4 of 16


SBI Clerk & RRB PO Mains PDF Course 2023
Quantitative Aptitude Day - 10 (Eng)

14. Find the profit share of Karan at the end of 1 c. Only statement iii alone is sufficient to answer
year, if Chandan gets a profit of Rs. 15500 at the the question
end of 1 year. d. All statement i, ii, and iii together are sufficient
i. Amount invested by Abhishek at the end of 1st to answer the question
quarter is double the amount invested by Rahul e. Neither statement i, ii, and iii together are
at the end of 1st quarter. sufficient to answer the question
ii. Amount invested by Karan at the end of the 3 rd
quarter is the same as the amount invested by 15. If the business ends after 6 months and
Rahul at the end of the 3rd quarter. Rahul gets a profit of Rs. 14500 out of the total
iii. Amount invested by Rahul at the end of the profit of Rs.51000. b is 50% more than a. Find
2nd quarter is 25% more than the amount the value of b.
invested by Abhishek at the end of the 2 nd a. 8.5
quarter. b. 7.5
a. Only statement i alone is sufficient to answer c. 5.5
the question d. 6.5
b. Only statement ii alone is sufficient to answer e.5.8
the question

Direction (16- 20): Study the given information carefully and answer the given questions.
In a car showroom, there are five different brands of cars: Honda, Maruti Suzuki, Tata, Hyundai, and
Mahindra. The first pie chart given below shows the total number of sales of cars by different brands. The
second pie chart given below shows the percentage of sales of SUV cars of different brands out of total
SUV cars.

Click Here For Bundle PDF Course | support@guidely.in Page 5 of 16


SBI Clerk & RRB PO Mains PDF Course 2023
Quantitative Aptitude Day - 10 (Eng)

i. Ratio of the total number of sales of SUV cars to the total number of sales of Hatchback cars is 31 : 32.
ii. Value of K is 10.
iii. 2N2 + 4N – 30 = 0, Value of N is the positive root.
iv. Ratio of sales of total Honda cars in 2022 to the sales of total Maruti Suzuki cars in 2022 is 9 :
16. v. O = M - N
16. Honda claims the average speed of its SUV to the same point in ‘N’ hours. Find the value of
car is 140 kmph. The SUV of Honda starts from N if the average speed of Hyundai’s SUV is 180
point P at 9: 30 am, and after 2 hours SUV of kmph.
Hyundai starts chasing it from the same point. a. 12
After being chased it takes a U-turn and returns b. 10

Click Here For Bundle PDF Course | support@guidely.in Page 6 of 16


SBI Clerk & RRB PO Mains PDF Course 2023
Quantitative Aptitude Day - 10 (Eng)

c. 18 b. Statement iii alone is sufficient to answer the


d. 14 question while statement i and ii are not sufficient
e.16 to answer the question.
c. Statement ii alone is sufficient to answer the
17. Tata’s hatchback claims a mileage of 22 question while statement i and iii are not
km/litre with a tank capacity of 40 litres, whereas sufficient to answer the question.
Mahindra’s hatchback claims a mileage of 20 d. Statement i and ii together are sufficient to
km/litre with a tank capacity of 45 litres. What is answer the question while statement iii is not
the difference between the distance covered by sufficient to answer the question.
both hatchbacks when their tanks are filled? e. None of these
i. O2M
ii. MN + O 19. In 2022, Honda sells its SUV cars at a 12%
iii. M2– M profit and hatchback cars at an f% profit to gain a
a. Only i and ii are correct 13.66% profit on all cars. Find the approximate
b. Only ii and iii are correct value of f.
c. Only i and iii are correct a. 15
d. All i, ii, and iii are correct b. 12
e. None of these c. 18
d. 10
18. Find the sale of both Maruti cars in 2018. e.13
i. Sales of Hyundai cars increased 10% every
year compared to the previous year, and the 20. If Mahindra’s SUVs are in three colour
ratio of Hyundai cars sold in 2019 to Maruti cars variants: blue, black, and white, in the ratio 3:4:2,
sold in 2019 is 3:4. Lalit randomly selects an SUV from Mahindra.
ii. Sales of Tata cars in 2018 were 20% more Find the probability that the selected car is either
than sales of Maruti cars in 2018. black or white.
iii. Sales of Maruti cars are always 250 more a. 1/3
than the sales of Mahindra cars in every year, b.3/4
and sales of Mahindra cars in 2022 will be 10% c. 1/4
more than its sales in 2018. d. 2/3
a. Statement I alone is sufficient to answer the e. None of these
question while statements ii and iii are not
sufficient to answer the question.

Click Here For Bundle PDF Course | support@guidely.in Page 7 of 16


SBI Clerk & RRB PO Mains PDF Course 2023
Quantitative Aptitude Day - 10 (Eng)

Click Here to Get the Detailed Video Solution for the above given Questions
Or Scan the QR Code to Get the Detailed Video Solutions

Answer Key with Explanation

Direction (1-5): Time taken by E to wash 100 kg clothes = 14


Time taken by C to wash 100 kg clothes = P2 hours
Time taken by A to wash 100 kg clothes = 4P Time taken by F to wash 100 kg clothes = 6
Then, P2/ 4P = 3 / 4 hours
P=3
And, Time taken by B to wash 100 kg clothes =
Q+1
Time taken by D to wash 100 kg clothes = P + Q
Then, (Q + 1) / (P + Q) = 4 / 5
5Q + 5 = 4P + 4Q
Q = 4P – 5
1. Answer: C
By putting the value of P.
Let, the washerman E washes x kg Pants.
Q=4*3–5
Then, Washerman E washes x + 15 kg Shirts.
Q=7
Amount get by Washerman E by washing Shirts
Time taken by A to wash 100 kg clothes = 12
= (x + 15) * 80
hours
Amount got by Washerman E by washing Pants
Time taken by B to wash 100 kg clothes = 8
= x * 130
hours
Amount got by Washerman E by washing Coats
Time taken by C to wash 100 kg clothes = 9
= 10 * 150
hours
Then, (x + 15) * 80 + x * 130 + 1500 = 7950
Time taken by D to wash 100 kg clothes = 10
80x + 1200 + 130x + 1500 = 7950
hours
210x = 5250

Click Here For Bundle PDF Course | support@guidely.in Page 8 of 16


SBI Clerk & RRB PO Mains PDF Course 2023
Quantitative Aptitude Day - 10 (Eng)

x = 25 Remaining quantity of clothes washed by A


Weight of Shirts wash by washerman E = 25 + alone = 3750 – (2600 + 900)
15 = 40 kg = 250 kg
Weight of Pants washed by washerman E = 25 Total time taken by A alone to wash remaining
kg clothes = 250/(100/12)
Weight of Coats washed by washerman E = 10 = 30 hours
kg Total time taken by A, C, and F together to wash
d = 40 + 25 + 10 all clothes = 72 + 36 + 30
The value of d = 75 kg = 138 hours

2. Answer: B 4. Answer: D
Total weight of Jeans to wash by Washerman F From statement i,
= 76.5 * (5/9) Quantity of clothes washed by A, B, and D in 24
= 42.5 kg hours = ((100/12) + (100/8) + 10) * 24
Total weight of Woollen clothes to wash by = (3700/120) * 24
Washerman F = 76.5 – 42.5 = 740 kg
= 34 kg Quantity of clothes washed by B and D in 16
Revenue collected from Jeans = 42.5 * 130 = hours = ((100/8) + 10) * 16
Rs.5525 = (900/40) * 16
Revenue collected from Woollen clothes = 34 * = 360 kg
200 = Rs.6800 Quantity of clothes washed by B in 12 hours =
Required revenue = 5525 + 6800 =Rs.12,325 (100/8) * 12
= 150 kg
3. Answer: A Total quantity of clothes = 740 + 360 + 150
Quantity of clothes washed by A, C, and F = 1250 kg
together in 72 hours = ((100/12) + (100/9) + Statement i is incorrect.
(100/6)) * 72 From statement ii,
= (1300/36) * 72 Quantity of clothes washed by A, B, and D in 36
= 2600 kg hours = ((100/12) + (100/8) + 10) * 36
Quantity of clothes washed by A and F in 36 = (740/24) * 36
hours = ((100/12) + (100/6)) * 36 = 1110 kg
= (300/12) * 36 Quantity of clothes washed by B and D in 16
= 900 kg hours = ((100/8) + 10) * 16

Click Here For Bundle PDF Course | support@guidely.in Page 9 of 16


SBI Clerk & RRB PO Mains PDF Course 2023
Quantitative Aptitude Day - 10 (Eng)

= (180/8) * 16 (100 + J)/9 = 14


= 360 kg 100 + J = 126
Quantity of clothes washed by B in 12 hours = J = 26
(100/8) * 12
= 150 kg Direction (6-10):
Total quantity of clothes = 1110 + 360 + 150 Selling price of tomato per kg = c2a – a2
= 1620 kg The cost price of cabbage per kg = c2a
Statement ii is correct. Then, c2a – (c2a – a2) = 4
From statement iii, a2 = 4
Quantity of clothes washed by A, B, and D in 60 a=2
hours = ((100/12) + (100/8) + 10) * 60 The cost price of onions = abc
= (740/24) * 60 The selling price of onions = b2c
= 1850 kg Then, b2c / abc = 3 / 2
Quantity of clothes washed by B and D in 40 b/a=3/2
hours = ((100/8) + 10) * 40 b=3
= (180/8) * 40 The cost price of cabbage per kg = c2a
= 900 kg The cost price of potato per kg = a2c
Quantity of clothes washed by B in 12 hours = Then, c2a / a2c = 5 / 2
(100/8) * 12 c/a=5/2
= 150 kg c=5
Total quantity of clothes = 1850 + 900 + 150
= 2900 kg
Statement iii is correct.
Statements ii and iii are correct.

5. Answer: A
The efficiency of D = 10 kg/hour
Let the new efficiency of C be x kg.
Then, 240 / (10 + x) = 10
24 = 10 + x
x = 14 6. Answer: C
Increased efficiency of C = 14 kg/hour New selling price of potatoes = 20 + d
(100/9) ((100 + J)/100) = 14

Click Here For Bundle PDF Course | support@guidely.in Page 10 of 16


SBI Clerk & RRB PO Mains PDF Course 2023
Quantitative Aptitude Day - 10 (Eng)

New selling price of tomatoes = 50 ((100 – x = 60


d)/100) The cost price of the spices mixture is Rs. 60 per
Then, 20 + d = 50 ((100 – d)/100) kg.
40 + 2d = 100 – d
3d = 60 9. Answer: C
The value of d = 20 Quantity I:
Profit on Cauliflower = 50 – 24 = 26
7. Answer: A Discount on Potato = 32 – 28 = 4
Amount got by Vaishali after selling rotten Then, (26/4) * 100
tomatoes = 15 * 20 = Rs.650
= 300 Quantity II:
Amount got by Vaishali after selling remaining New selling price of Cauliflower = 50 + 10 = 60
tomatoes = 50 * (x – 15) New profit percentage on Cauliflower = ((60 –
Total amount got by Vaishali after selling all 24)/24) * 100
tomatoes = 40 (106.25/100) * x = 150 %
Then, 300 + 50 * (x – 15) = 42.5x Discount percentage on Potato = ((32 – 28)/32) *
300 + 50x – 750 = 42.5x 100
7.5x = 450 12.5%
The value of x = 60 Reqd. percentage = (150/12.5) * 100
= Rs.1200
8. Answer: D Quantity II > Quantity I
Total quantity of cuisine = 25 + 15 + 10 + 5
= 55 kg 10. Answer: B
Total amount gets by hotel selling the cuisine = For i,
55 * 51 The cost price of cauliflower = 24
= 2805 New marked price of cauliflower = 24 (150/100)
Let the cost price of the spices mixture be x. = 36
Total cost price of the cuisine = 25 * 20 + 15 * 40 New selling price of cauliflower = 36 ((100 –
+ 10 * 30 + x * 5 25)/100)
= 1400 + 5x = 27
Then, (1400 + 5x) (165/100) = 2805 New profit on cauliflower = 27 – 24
1400 + 5x = 1700 =3
5x = 300 Statement i is correct

Click Here For Bundle PDF Course | support@guidely.in Page 11 of 16


SBI Clerk & RRB PO Mains PDF Course 2023
Quantitative Aptitude Day - 10 (Eng)

For ii, The initial investment of Abhishek = Rs.24000


The cost price of cauliflower = 24 The initial investment of Rahul = Rs.27000
New marked price of cauliflower = 24 (125/100) The initial investment of Chandan = Rs.22000
= 30 11. Answer: D
New selling price of cauliflower = 30 ((100 – According to the question,
10)/100) The ratio of investment of Karan, Abhishek,
= 27 Rahul, and Chandan in 2nd Quarter = 3:4:2: 1
New profit on cauliflower = 27 – 24 Let the investment of Abhishek in the 2nd quarter
=3 be 4p and his investment in the 3rd quarter be
Statement ii is correct 4q.
For iii, Investment of Rahul in 3rd quarter = 4p (13/12)
The cost price of cauliflower = 24 = 13p/3
New marked price of cauliflower = 24 (130/100) Then 4p – 4q = 4000
= 31.2 p – q = 1000 …. (1)
New selling price of cauliflower = 31.2 ((100 – (13p/3) - 4q = 6500
20)/100) 13p – 12q = 19500
= 24.96 Or, p + 12p – 12q = 19500
New profit on cauliflower = 24.96 – 24 By putting the value from eqn. 1
= .96 p + 12000 = 19500
Statement iii is incorrect. p = 7500
q = 6500
Directions (11-15): The ratio of investment of Karan, Abhishek,
Initial investment of Karan = x + 3000 Rahul, and Chandan = (18000 + 22500 + 13000)
Initial investment of Abhishek = y + 4000 : (24000 + 30000 + 26000) : (27000 + 15000 +
Initial investment of Rahul = y + 7000 32500) : (22000 + 7500 + 19500)
Then, (x + 3000) / (y + 4000) = 3 / 4 = 53500 : 80000 : 74500 : 49000
4x + 12000 = 3y + 12000 = 107 : 160 : 149 : 98
x/y=3/4 …. (1) Reqd. percentage = (98/ (107 + 160 + 149 + 98))
And, (y + 4000) / (y + 7000) = 8 / 9 * 100
9y + 36000 = 8y + 56000 = (98/514) * 100
Y = 20000 = 4900/257 %
X = 15000
The initial investment of Karan = Rs.18000 12. Answer: B

Click Here For Bundle PDF Course | support@guidely.in Page 12 of 16


SBI Clerk & RRB PO Mains PDF Course 2023
Quantitative Aptitude Day - 10 (Eng)

The ratio of investment of Karan, Abhishek, = 17250


Rahul, and Chandan = (18 + 3 + 2 + c) : (24 + a Statement i is correct.
+ 4 + 2) : (27 + 2 + b + 3) : (22 + 1 + 3 + 5) From statement ii,
= (23 + c) : (30 + a) : (32 + b) : (31) x + (ab)2 10
The ratio of profit of Karan, Abhishek, Rahul, and = 15000 + (3 * 5)2 * 10
Chandan = 25:33:37: 31 = 15000 + 2250
Then, 23 + c = 25 = 17250
c=2 Statement ii is correct.
30 + a = 33 From statement iii,
a=3 x + 0.25y – 10ab
32 + b = 37 = 15000 + 0.25 * 20000 – 10 * 3 * 5
b=5 = 15000 + 5000 – 150
a:b: c = 3:5: 2 = 19850
Statement iii is incorrect.
13. Answer: C
The ratio of profit of Karan, Abhishek, Rahul, and 14. Answer: D
Chandan = 23:31:34: 26 From statement i,
Profit of Rahul = 25500 The ratio of the amount invested by Rahul to the
Profit of Karan = (25500/34) * 23 amount invested by Abhishek at the end of 1st
= 750 * 23 quarter = 1 : 2
= 17250 The ratio of investment of Karan, Abhishek,
And, the Ratio of investment of Karan, Abhishek, Rahul, and Chandan at the end of 1st quarter = 3
Rahul, and Chandan till 3rd quarter = (18 + 3 + 2) : 4 :2 : 1
: (24 + a + 4) : (27 + 2 + b) : (22 + 1 + 3) Statement i alone is not sufficient to answer the
= 23 : 28 + a : 29 + b : 26 question.
28 + a = 31 From statement ii,
a=3 The ratio of the amount invested by Karan at the
29 + b = 34 end of 3rd quarter to the amount invested by
b=5 Rahul at the end of 3rd quarter = 1 : 1
From statement i, The ratio of investment of Karan, Abhishek,
y – ((y/10) + (x/20)) Rahul, and Chandan at the end of 3rd quarter = 3
= 20000 – ((20000/10) + (15000/20)) :2:3:5
= 20000 – (2000 + 750)

Click Here For Bundle PDF Course | support@guidely.in Page 13 of 16


SBI Clerk & RRB PO Mains PDF Course 2023
Quantitative Aptitude Day - 10 (Eng)

Statement ii alone is not sufficient to answer the b = 5 (150/100)


question. b = 7.5
From statement iii,
The ratio of the amount invested by Rahul at the Directions (16-20):
end of 2nd quarter to the amount invested by 2N2 + 4N – 30 = 0
Abhishek at the end of 2nd quarter = 5 : 4 2N2 + (10 – 6) N – 30 = 0
The ratio of investment of Karan, Abhishek, 2N2 + 10N – 6N – 30 = 0
Rahul, and Chandan at the end of 2nd quarter = 2N (N + 5) – 6 (N + 5) = 0
2:4:5:3 (2N – 6) ((N + 5) = 0
Statement iii alone is not sufficient to answer the N = -5, 3
question. Sales of total Honda cars in 2022 = N2MK
From statements i, ii, and iii together, Sales of total Maruti Suzuki cars in 2022 = (M +
The ratio of the amount invested by Karan, N) K2
Abhishek, Rahul, and Chandan in 1 year = (18 + Then, (N2MK) / ((M + N) K2) = 9/16
3 + 2 + 3) : (24 + 4 + 4 + 2) : (27 + 2 + 5 + 3) : By putting the value of N and K,
(22 + 1 +3 + 5) 16(90M) = 9 (100M + 300)
= 26:34:37: 31 1440M = 900M + 2700
Profit share of Karan = (15500/31) * 26 540M = 2700
= Rs.13000 M=5
All statements i, ii, and iii together are sufficient According to the question,
to answer the question. O=M–N
O=5–3
15. Answer: B O=2
Profit of Rahul = 14500 Total sales of Honda cars in 2022 = 450
Profit share of Rahul =14500/51000 Total sales of Maruti Suzuki cars in 2022 = 800
= 29/102 Total sales of Tata cars in 2022 = 650
The ratio of investment of Karan, Abhishek, Total sales of Hyundai cars in 2022 = 700
Rahul, and Chandan = (18 + 3) : (24 + a) : (27 + Total sales of Mahindra cars in 2022 = 550
2) : (22 + 1) Total sales of all cars together = 450 + 800 +
= 21 : 24 + a : 29 : 23 650 + 700 + 550
Then, 21 + 24 + a + 29 + 23 = 102 = 3150
97 + a = 102 Total no. of sales of SUV cars = 3150 (31/63)
a=5 = 1550

Click Here For Bundle PDF Course | support@guidely.in Page 14 of 16


SBI Clerk & RRB PO Mains PDF Course 2023
Quantitative Aptitude Day - 10 (Eng)

Total no. of sales of hatchback cars = 3150 – Distance covered by Tata’s hatchback when its
1550 = 1600 tank is full = 22 * 40
No. of sales of Honda SUV cars = 1550 = 880 km
(400/3100) Mileage of Mahindra’s hatchback = 20 km/litre
No. of sales of Maruti Suzuki SUV cars = 1550 Tank capacity of Mahindra’s hatchback = 45
(400/3100) litres
No. of sales of Tata SUV cars = 1550 (800/3100) Distance covered by Mahindra’s hatchback
No. of sales of Hyundai SUV cars = 1550 when its tank is full = 20 * 45
(600/3100) = 900 km
No. of sales of Mahindra SUV cars = 1550 Difference between the distance covered by both
(900/3100) hatchbacks = 900 – 880
= 20 km
From i,
O2M = (2)25
=4*5
= 20
16. Answer: D Statement i is correct
The average speed of Honda’s SUV = 140 kmph From ii,
The average speed of Hyundai’s SUV = 180 MN + O = 5 * 3 + 2
kmph = 15 + 2
Distance travel by Honda’s SUV in 2 hours = 140 = 17
* 2= 280 km Statement ii is incorrect
Time taken by Hyundai’s SUV to catch Honda’s From iii,
SUV = 280/ (180 – 140) M2 – M = 52 – 5
= 7 hours = 25 – 5
Time taken by Hyundai’s SUV to return to the = 20
point P = 2 * 7 Statement iii is correct
= 14 hours
The value of N = 14 hours 18. Answer: B
From statement I,
17. Answer: C Sales of total Hyundai cars in 2019 = 700 (9/10)
Mileage of Tata’s hatchback = 22 km/litre (9/10) (9/10)
Tank capacity of Tata’s hatchback = 40 litres = 5103/10

Click Here For Bundle PDF Course | support@guidely.in Page 15 of 16


SBI Clerk & RRB PO Mains PDF Course 2023
Quantitative Aptitude Day - 10 (Eng)

Maruti cars in 2019 = (5103/10) (4/7)


Statement i alone is not sufficient to answer the
question.
From statement ii,
Let the sales of Maruti Suzuki cars in 2018 be
5x.
Then sales of Tata cars in 2018 = 5x (120/100)
= 6x
Then, (13.66 – 12) / (f – 13.66) = 5 / 4
Statement ii alone is not sufficient to answer the
1.66 * 4 = 5f – 68.3
question.
5f = 68.3 + 6.64
From statement iii,
5f = 74.94
Sales of Mahindra cars in 2022 = 550
f ≈ 15
Sales of Mahindra cars in 2018 = 550 (10/11)
f = 15
= 500
Sales of Maruti Suzuki cars in 2018 = 500 + 250
20. Answer: D
= 750
Blue SUVs of Mahindra = 450 (3/9)
Statement iii alone is sufficient to answer the
= 150
question.
Black SUVs of Mahindra = 450 (4/9)
= 200
19. Answer: A
White SUVs of Mahindra = 450 (2/9)
Profit on Honda’s SUV cars = 12%
= 100
Profit on Honda’s hatchback cars = f%
Probability of selected car is Black = (200/450)
Overall profit on Honda’s hatchback cars =
= 4/9
13.66%
Probability of selected car is White = (100/450)
= 2/9
Probability of selected car is either Black or
White = (4/9) + (2/9)= 2/3

Click Here For Bundle PDF Course | support@guidely.in Page 16 of 16


SBI Clerk & RRB PO Mains PDF Course 2023
ENGLISH Day - 10

English Language

Directions (1-5): In the following passage, some and retaining those already employed, it
of the words have been left out, each of which is observed. The tech workforce has found new
indicated by a number. Find the suitable word love with remote working, giving them freedom
from the options given against each number and and productivity that requires HR to be more
fill up the blanks with appropriate words to make trusting, it said adding that unengaged or
the paragraph meaningful. (4)___________ employees will find alternatives
With the COVID-19 pandemic bringing faster and exit faster than pre-pandemic times."It
unprecedented changes in work life, a study has is interesting to observe how the fast
revealed that 82% of (1)__________ admitted that (5)__________ 'Future of Work' is getting past the
they prefer working from home to going back to world of conventional HR and transforming into
the office.The remote work trend was initially an ecosystem that now involves people beyond
forced on employees due to the pandemic, the confines of company payroll and office
however, after two years remote working has boundaries.
become a new normal and as things (2)________ 1. Fill in the blank (1)
down new habits have formed.The Talent Tech a) Responses
Outlook 2022 study is an analysis of inputs from b) Respondents
100 plus C-suite and human capital leaders c) Senators
across four continents, done by surveys, social d) Employee
media inputs, interviews and panel e) Spectators
discussions.The study further (3)_________ that
64% employees said they are more productive 2. Fill in the blank (2)
working from home and feel less stressed.The a) Settled
new normal for HR shifted from engaging face to b) Drowned
face to engaging with remote employees whom c) Fell
they meet only virtually most of the time, it d) Sorted
stated.More than 80% of the HR managers e) Resolved
admitted that hiring employees for full-time office
presence is getting more difficult, the study 3. Fill in the blank (3)
said.Working from home is no longer a choice a) Said
but a new normal that every tech talent expects b) Left out
from their employer and the employers not willing c) Revealed
to adapt will face challenges in hiring good talent d) Concealed

Click Here For Bundle PDF Course | support@guidely.in Page 1 of 9


SBI Clerk & RRB PO Mains PDF Course 2023
ENGLISH Day - 10

e) Wrote 7. INCUMBENT
I. George Federick is the present incumbent and
4. Fill in the blank (4) he has three more years to serve.
a) Impolite II. She felt it incumbent on her to be there for her
b) Undeserving cousin during the surgery.
c) Inappropriate III.The incumbent has been awarded a new
d) Involved contract despite the heavy competition.
e) Underutilised a) Only I
b) Only II
5. Fill in the blank (5 ) c) Only III
a) Running d) Both II and III
b) Forwarding e) All are correct
c) Moving
d) Evolving 8. AMBIVALENT
e) Proceeding I. She remained ambivalent on her promotion as
she loves both her career and family.
Directions (6-10): In each of the following II. Ambivalent decisions always lead to disaster
questions, a highlighted word is given followed or failure because of the lack of confidence.
by three sentences. Choose the sentence(s) that III.There is ambivalency among men when a
has/have the wrong usage of the highlighted woman or girl rides a gear bike.
word. a) Only I
6. DISPARATE b) Only II
I. The people in the community are so disparate c) Only III
that their ideologies are very distinct and difficult d) Both I and III
for any other common man to understand. e) All are correct
II. Ravi became disparate after the thief stole his
wallet and ran away. 9. LIGHTNING
III.She has always got disparate ideas for which I. It was a lightning strike by Sir Ravindra Jadeja
she has recently been awarded by the corporate. with the last two balls.
a) Only I II. The thunder lightning was so heavy and bright
b) Only II that the huge neighbourhood tree was struck and
c) Only III burst into flames.
d) Both I and II
e) All are correct

Click Here For Bundle PDF Course | support@guidely.in Page 2 of 9


SBI Clerk & RRB PO Mains PDF Course 2023
ENGLISH Day - 10

III.E-commerce site Amazon offers lightning the player hails from Chennai — the Titans
deals that have a pre-set time limit to place the finished with 214 for four before the intervention.
order. (C) At Ahmedabad’s Narendra Modi Stadium,
a) Only I CSK pulled off a last-over heist against
b) Only II defending champion Gujarat Titans with
c) Only I and II Ravindra Jadeja thumping the winning runs in a
d) Both II and III final that was spread over three days.
e) All are correct (D) The weather may have been fickle with a
summer night yielding rain but there was no
10. CALLED mistaking the inevitable air around Chennai
I. The marriage was called off after the dowry Super Kings (CSK) lifting the Indian Premier
issue. League (IPL) Trophy on a Monday night that
II. I called her around 9:30 AM in the morning but spilled over to Tuesday.
she was busy and told me to send a text (E) The target was revised to 171 following a
message. rain-break but CSK proved equal to the task
III.I was called down by my mother for not putting (even though Dhoni fell for a duck) as Jadeja’s
the clothes to wash. long-handle did the trick.
a) Both I and II 11. Which of the following is the
b) Both II and III INTRODUCTORY sentence after the
c) Both I and III rearrangement ?
d) All are wrong a) A
e) All are correct b) B
c) C
Directions (11-15): Rearrange the following five d) D
sentences (A), (B), (C), (D) and (E) in the proper e) E
sequence to form a coherent paragraph and then
answer the questions given below. 12. Which of the following is the FOURTH
(A) Sunday was a washout and on Monday, the sentence after the rearrangement ?
rains intervened and delayed the climax a) A
into the early hours of Tuesday. b) B
(B) CSK prised out Shubman Gill for 39 with c) C
Dhoni’s lightning stumping delivering the fatal d) D
blow, but once Sai Sudharsan hammered a 96 — e) E

Click Here For Bundle PDF Course | support@guidely.in Page 3 of 9


SBI Clerk & RRB PO Mains PDF Course 2023
ENGLISH Day - 10

13. Which of the following is the SECOND a) A


sentence after the rearrangement ? b) B
a) A c) C
b) B d) D
c) C e) No error
d) D
e) E 17. Sita told Ram that/A she felt ignored as she
was counting on him/B to be there at her 25th
14. Which of the following is the CONCLUDING birthday party/C that was held last thursday/D.
sentence after the rearrangement ? a) A
a) A b) B
b) B c) C
c) C d) D
d) D e) No error
e) E
18. A large number of sim cards have/A never
15. Which of the following is the THIRD sentence being used and people stopped recharging
after the rearrangement ? them/B, so, the company has decided to call
a) A off/C its business as a service provider/D.
b) B a) A
c) C b) B
d) D c) C
e) E d) D
e) No error
Directions (16-20): Read the sentence to find out
whether there is an error in it. The error, if any, 19. Because there is no support by/A the society
will be in one part of the sentence. The letter to eliminate the most inhuman of professions/B,
corresponding to that part will be your answer. If manual scavenging still persist/C which is really
the given sentence is correct as it is, mark the saddening/D.
answer as ‘No Error’. a) A
16. The Indian cricket player was injured b) B
heavily/A while the bowler threw the ball/B c) C
straight on his face/C but he claimed it to be an d) D
accident/D. e) No error

Click Here For Bundle PDF Course | support@guidely.in Page 4 of 9


SBI Clerk & RRB PO Mains PDF Course 2023
ENGLISH Day - 10

20. I have recently used/A the services of his b) B


travel agency to/B book a cruise ship for a family c) C
trip to Atlanta/C, which was started two decades d) D
before/D. e) No error
a) A
Click Here to Get the Detailed Video Solution for the above given Questions
Or Scan the QR Code to Get the Detailed Video Solutions

Answer Key with Explanation

1. Answer: B
The blank needs a collective noun. 2. Answer: A
With the COVID-19 pandemic bringing Settled down is a phrasal verb with meaning ‘to
unprecedented changes in work life, a study has get comfortable’ or ‘to get used to’
revealed that 82% of respondents admitted that The remote work trend was initially forced on
they prefer working from home to going back to employees due to the pandemic, however, after
the office. two years remote working has become a new
Respondent - a person who answers a question normal and as things settled down new habits
or provides a response for a survey. have formed.
Other options don’t fit the blank for the following Other options don’t fit the blank for the following
reasons : reasons :
Responses - is not the right pick because the Drowned - to die in water/to sink
sentence has ‘they’ referring to a set of people. Fell down - to collapse
Senator - person working for the government Sorted - organised/dealt correctly
Employee - is in singular form(singular noun) Resolved - to fix something
and hence doesn’t fit the blank.
Spectator - a person who is watching an event

Click Here For Bundle PDF Course | support@guidely.in Page 5 of 9


SBI Clerk & RRB PO Mains PDF Course 2023
ENGLISH Day - 10

All these words have a meaning that is different Other options don’t fit the blank for the following
from the context of the sentence which has the reasons :
blank. Impolite cannot fit the blank because the context
here is about the employees contribution and not
3. Answer: C about their qualities/character.
Revealed fits the blank correctly. Deserving, inappropriate, involved all these three
The study further revealed that 64% employees words will change the original meaning of the
said they are more productive working from sentence.
home and feel less stressed.
Revealed - is to let out some information, here it 5. Answer: D
means the study further provided the information Fast-evolving is the right way to put.
that 64% of employees said they are comfortable It is interesting to observe how the fast evolving
with wfh concept. 'Future of Work' is getting past the world of
Other options don’t fit the blank for the following conventional HR and transforming into an
reasons : ecosystem that now involves people beyond the
Using said will cause redundancy in the confines of company payroll and office
sentence and it is not the best option too. boundaries.
Left out - to leave something/fail to include Running, moving, proceeding and forwarding are
Concealed - to hide something actions of motion but fast evolving fits the blank
Wrote is an inappropriate word and makes the perfectly.
sentence meaningless. Evolving - to develop/to grow

4. Answer: E 6. Answer: B
The blank (4) needs a negative word because it Disparate means ‘to be different’. The word is
is preceded by a negative word unengaged. correctly used in sentence I and III.But usage of
The tech workforce has found new love with the word ‘disparate’ is completely wrong in
remote working, giving them freedom and sentence II because the disparate should be
productivity that requires HR to be more trusting, replaced with desperate. ‘Ravi became
it said adding that unengaged or underutilised desperate’ would be the correct way instead the
employees will find alternatives faster and exit word disparate is used - which does not add
faster than pre-pandemic times. meaning to the sentence.
Underutilised - to not use something/someone
up to the level 7. Answer: E

Click Here For Bundle PDF Course | support@guidely.in Page 6 of 9


SBI Clerk & RRB PO Mains PDF Course 2023
ENGLISH Day - 10

All the sentences are correct and have the lifting the IPL trophy, the sentence gives more
correct usage of the word incumbent. information on the weather and also mentions
Incumbent - an official/a person who holds a the details of the days.The second sentence is
position/a business ‘C’ because it elaborates the cricket match that
Be incumbent - to feel that it is necessary to be took place providing the opponent team
with someone/to do something at the right time. information and it also adds the message that
the final was delayed for a period of three
8. Answer: C days.The third sentence is ‘A’ as it is revisiting
The sentence III is absolutely wrong because the context of the introductory statement and
‘ambivalency’ must be replaced with explains the reason for delay.This is followed by
‘ambivalence’. sentence ‘B’ as it has a clue ‘intervention’ - from
Sentence I and II are correct as it is.The that we may conclude that the Gujarat Titans
meaning of the word ambivalent/ambivalence - was the first to bat and set a target for CSK.The
to have a mixed feeling or emotion towards last sentence is ‘E’ which talks about the target
something or someone. and the rain break(which is a continuation of
sentence ‘B’) and how the team still managed to
9. Answer: E lift the trophy.
All the given sentences are right as it is.The
word lightning is used in a correct way and the 12. Answer: B
sentences are contextually and grammatically The correct sequence after rearrangement is
correct and meaningful. DCABE.
The first or the introductory sentence should be
10. Answer: E ‘D’ which provides the first information of CSK
All the given sentences are correct and lifting the IPL trophy, the sentence gives more
meaningful. information on the weather and also mentions
Called off - to cancel the details of the days.The second sentence is
Called down - to be angry or be severe ‘C’ because it elaborates the cricket match that
took place providing the opponent team
11. Answer: D information and it also adds the message that
The correct sequence after rearrangement is the final was delayed for a period of three
DCABE. days.The third sentence is ‘A’ as it is revisiting
The first or the introductory sentence should be the context of the introductory statement and
‘D’ which provides the first information of CSK explains the reason for delay.This is followed by

Click Here For Bundle PDF Course | support@guidely.in Page 7 of 9


SBI Clerk & RRB PO Mains PDF Course 2023
ENGLISH Day - 10

sentence ‘B’ as it has a clue ‘intervention’ - from The correct sequence after rearrangement is
that we may conclude that the Gujarat Titans DCABE.
was the first to bat and set a target for CSK.The The first or the introductory sentence should be
last sentence is ‘E’ which talks about the target ‘D’ which provides the first information of CSK
and the rain break(which is a continuation of lifting the IPL trophy, the sentence gives more
sentence ‘B’) and how the team still managed to information on the weather and also mentions
lift the trophy. the details of the days.The second sentence is
‘C’ because it elaborates the cricket match that
13. Answer: C took place providing the opponent team
The correct sequence after rearrangement is information and it also adds the message that
DCABE. the final was delayed for a period of three
The first or the introductory sentence should be days.The third sentence is ‘A’ as it is revisiting
‘D’ which provides the first information of CSK the context of the introductory statement and
lifting the IPL trophy, the sentence gives more explains the reason for delay.This is followed by
information on the weather and also mentions sentence ‘B’ as it has a clue ‘intervention’ - from
the details of the days.The second sentence is that we may conclude that the Gujarat Titans
‘C’ because it elaborates the cricket match that was the first to bat and set a target for CSK.The
took place providing the opponent team last sentence is ‘E’ which talks about the target
information and it also adds the message that and the rain break(which is a continuation of
the final was delayed for a period of three sentence ‘B’) and how the team still managed to
days.The third sentence is ‘A’ as it is revisiting lift the trophy.
the context of the introductory statement and
explains the reason for delay.This is followed by 15. Answer: A
sentence ‘B’ as it has a clue ‘intervention’ - from The correct sequence after rearrangement is
that we may conclude that the Gujarat Titans DCABE.
was the first to bat and set a target for CSK.The The first or the introductory sentence should be
last sentence is ‘E’ which talks about the target ‘D’ which provides the first information of CSK
and the rain break(which is a continuation of lifting the IPL trophy, the sentence gives more
sentence ‘B’) and how the team still managed to information on the weather and also mentions
lift the trophy. the details of the days.The second sentence is
‘C’ because it elaborates the cricket match that
14. Answer: E took place providing the opponent team
information and it also adds the message that

Click Here For Bundle PDF Course | support@guidely.in Page 8 of 9


SBI Clerk & RRB PO Mains PDF Course 2023
ENGLISH Day - 10

the final was delayed for a period of three


days.The third sentence is ‘A’ as it is revisiting 18. Answer: B
the context of the introductory statement and The word ‘being’ must be replaced with
explains the reason for delay.This is followed by ‘been’.The sentence is in passive voice whereas
sentence ‘B’ as it has a clue ‘intervention’ - from ‘being’ is used only for continuous tense.
that we may conclude that the Gujarat Titans The correct sentence is : A large number of sim
was the first to bat and set a target for CSK.The cards have never been used and people stopped
last sentence is ‘E’ which talks about the target recharging them, so, the company has decided
and the rain break(which is a continuation of to call off its business as a service provider.
sentence ‘B’) and how the team still managed to 19. Answer: C
lift the trophy. Replace persist with persists.
The correct sentence is : Because there is no
16. Answer: B support by the society to eliminate the most
While must be replaced with when. inhuman of professions, manual scavenging still
The correct sentence is : The Indian cricket persists which is really saddening.
player was injured heavily when the bowler
threw the ball straight on his face but he claimed 20. Answer: D
it to be an accident. The error is in part ‘D’ - he started the agency
When - at what time two decades before is wrong, replace ‘before’
While - during the time with ‘ago’.
The correct sentence is : I have recently used
17. Answer: E the services of his travel agency to book a cruise
The given sentence is correct as it is.There is no ship for a family trip to Atlanta, which was started
error. two decades ago.

Click Here For Bundle PDF Course | support@guidely.in Page 9 of 9


SBI Clerk & RRB PO Mains PDF Course 2023
Reasoning Ability Day - 11 (Eng)

Reasoning Ability
Directions (1-5): Study the following information a) T
carefully and answer the given questions. b) The one who joined on June 7th
Nine persons – O, P, Q, R, S, T, U, V, and, W c) The one who joined two persons after S
joined the bank in three different months viz.- d) O
March, June, and August on three different dates e) V
– 4th, 7th, and 9th. Only one person joined on one
date of each month. All persons like different 2) Who among the following person joined on 7th
shoe brands viz.- Mochi, Puma, Reebok, Bata, August?
Relaxo, Nike, Red Chief, Liberty, and Adidas. a) The one who likes Bata
The one who likes Bata joined on an even b) The one who joined two persons after U
numbered date in a month having an odd c) The one who likes Puma
number of days. Only three persons joined d) V
between the one who likes Bata and W, who e) P
joined immediately after the one who likes Nike.
R joined four persons before the one who likes 3) Four of the following five are alike in a certain
Red Chief. At least one but not more than three way based on the given arrangement and thus
persons joined between the one who likes Bata form a group. Which one of the following does
and R. Both R and O like neither Nike nor not belong to the group?
Adidas. S joined three persons before the one a) T
who likes Adidas. Both O and S like neither Bata b) Q
nor Nike. As many persons joined before the one c) W
who likes Adidas as after O. As many persons d) R
joined between the one who likes Red Chief and e) S
O as between P and the one who likes Relaxo.
The one who likes Relaxo joined immediately 4) How many persons joined between U and the
after U, who joined on an even numbered date. P one who likes Reebok?
and the one who likes Mochi joined in the same a) As many persons joined before the one who
month. Only three persons joined between T and likes Nike
the one who likes Mochi. More than two persons b) Three
joined between the one who likes Liberty and Q, c) As many persons joined between V and the
who likes neither Red Chief nor Reebok. one who likes Reebok
1) Who among the following person likes d) Four
Relaxo? e) None

Click Here For Bundle PDF Course | support@guidely.in Page 1 of 11


SBI Clerk & RRB PO Mains PDF Course 2023
Reasoning Ability Day - 11 (Eng)

digital sum of the outcome from top to bottom


5) Which of the following statements is/are not respectively.
true as per the given arrangement? Arrangement 1:
a) T joined in August D, I, R, Z, T, E, O, F, K, U, G, P, and V are to be
b) The one who likes Puma joined immediately filled against the outcomes. Only one letter is
after V placed along with each outcome.
c) Three persons joined before U I. All vowels are placed in reverse alphabetical
d) Either B or C order along with each outcome whose digital
e) All the above statements are true sum is a prime number from top to bottom
respectively.

Directions (6-10): Study the following information II. All consonants which come before M are

carefully and answer the given questions. placed in reverse alphabetical order along with

There are nine levels marked one to nine from each outcome whose digital sum is an even

bottom to top respectively. There are thirteen number from top to bottom respectively.

outcomes which are placed in these nine levels. III. All the remaining letters are placed with the

At least one and at most two outcomes are remaining outcomes from bottom to top in

placed on each level. No two levels have the alphabetical order. (If more than two outcomes

same outcome. Two dice are thrown and their are at the same level, then the digital sum of the

outcomes are given below: outcome is highest given first preference)

(6, 4), (6, 3), (4, 5), (1, 2), (1, 5), (1, 1), (3, 2), (5, Arrangement 2:

3), (3, 3), (5, 2), (4, 4), (6, 6), and (5, 5). All the letters are placed in a row from the left

Condition: end based on the descending order of their

I) If the digital product of the outcome is digital sum of the outcome and facing north. (If

even(more than 16), then the respective the outcome of letters are same, then give

outcomes are arranged in descending order and preference in alphabetical order).

placed at an even numbered level from top to 6) Which of the following letter is placed second

bottom respectively. to the right of the letter which is fifth to the left of

II) If the digital product of the outcome is odd, G in arrangement 2?

then the respective outcomes are arranged in a) Z

ascending order and placed at an odd numbered b) The letter which is placed immediate left of E

level from top to bottom respectively. c) The letter which is placed fourth from the left

III) The remaining outcomes are placed on an end

even numbered level in ascending order of the d) O


e) F

Click Here For Bundle PDF Course | support@guidely.in Page 2 of 11


SBI Clerk & RRB PO Mains PDF Course 2023
Reasoning Ability Day - 11 (Eng)

a) U, 8, K
7) ______ is the outcome of the letter which is b) F, 6, D
_______ to the right of V in arrangement 2. c) G, 7, I
a) (1, 1), Fourth d) Both B and C
b) (4, 5), Second e) I, 7, O
c) (1, 5), Third
d) (6, 4), Fifth Directions (11-15): Study the following
e) None of these information carefully and answer the given
questions.
8) Which of the following letters are placed at Sixteen persons from A to P are sitting around
even numbered levels? the two concentric square tables in such a way
I. TRD that eight persons are sitting at the inner table
II. RUF facing away from the centre whereas eight
III. FRT persons are sitting at the outer table facing
IV. OTK towards the centre. One person sits at each
a) Only III and IV corner whereas one person sits in the middle of
b) Only III each side of both tables.
c) Only I, III, and IV Note: I. Consecutive alphabetically named
d) Only I, II and IV persons are neither sitting at the same table nor
e) None facing each other.
II. If A is facing B, then both are facing each
9) Four of the following five are alike in a certain other at different tables, if A sits opposite to B,
way based on the given arrangement 2 and thus then both are sitting at the same table.
form a group. Which one of the following does H sits third to the right of the one who is facing C,
not belong to the group? who sits at the corner of the inner table. One
a) The letter which is placed third to the right of G person sits between C and E, who sits opposite
b) F to G. The one who is facing G sits third to the left
c) The letter which is placed second to the right of N. Only three persons sit between F and D,
of P who sits adjacent to H. As many persons sit
d) E between F and J as between K and M, when
e) O counted from the left of both F and K. K sits
opposite to O, who neither sits adjacent to C nor
10) _____ letter is placed at level _____ and E. Both M and I don’t sit adjacent to E. Only
immediately above ______.

Click Here For Bundle PDF Course | support@guidely.in Page 3 of 11


SBI Clerk & RRB PO Mains PDF Course 2023
Reasoning Ability Day - 11 (Eng)

three persons sit between A and I. The number 14) If K is related to F, similarly D is related to O,
of persons sitting between I and O is one less then who among the following person is related
than the number of persons sitting between B to E?
and J, when counted from the right of both B and a) N
I. Two persons sit between B and P. b) The one who sits third to the left of D
11) What is the position of C with respect to the c) The one who faces I
one who faces J? d) Both A and B
a) Immediate right e) Either B or C
b) Second to the left
c) Third to the left 15) Who among the following persons are sitting
d) Second to the right between the one who faces I and B, when
e) Immediate left counted from the left of B?
I. O
12) How many persons are sitting between A II. The one who sits second to the left of L
and the one who faces N, when counted from the III. The one who faces C
right of A? a) Only II
a) Two b) Only I and III
b) One c) Only III
c) Three d) All I, II, and III
d) More than three e) Only II and III
e) None

Directions (16-20): Study the following


13) Four of the following five are alike in a certain information carefully and answer the given
way based on the given arrangement and thus questions.
form a group. Which one of the following does In a certain code language following statements
not belong to the group? are coded as follows:
a) L “Burn Are More Empty Close” is coded as “#%+
b) The one who sits second to the right of A @%¥ #$¥ @^* @^¥”.
c) N “Went All File Above Equal” is coded as “@^¥
d) The one who sits third to the left of B @^© #$£ #%£ #%¥”.
e) O “Blue Copy Age Down Today” is coded as “@%+
#$¥ @^+ @^* @^¥”.

Click Here For Bundle PDF Course | support@guidely.in Page 4 of 11


SBI Clerk & RRB PO Mains PDF Course 2023
Reasoning Ability Day - 11 (Eng)

Note: (All the given codes are three symbols a) @^¥ @^+
codes only) b) @^* #$¥
16) How “went” is coded in the given code c) @^¥ #$¥
language? d) Either A or B
a)#$¥ e) Either B or C
b) @^©
c)@^¥ 19) What may be the code of “Run Today
d)@%¥ Away”?
e) @^* a) @%* @%+ #^+
b)@$* #%* #^+
17) Which of the following phrase is coded as c)@$* @%+ #^+
“#$¥ #%£”? d) Either A or B
a) Age Equal e) None of these
b) Down Empty
c) Are Equal 20) What is the code of “Empty File”?
d) Both A and C a) #*^ @^£
e) None of these b) #%+ @^¥
c) #*^ @^+
18) How “More Copy” is coded in the given code d) @%^ #^¥
language? e) None of these
Click Here to Get the Detailed Video Solution for the above given Questions
Or Scan the QR Code to Get the Detailed Video Solutions

Answer Key with Explanation

Direction (1-5): 2. Answer: C


1. Answer: B

Click Here For Bundle PDF Course | support@guidely.in Page 5 of 11


SBI Clerk & RRB PO Mains PDF Course 2023
Reasoning Ability Day - 11 (Eng)

3. Answer: D(All the persons joined in the month


having 31 days except option D)
4. Answer: A
5. Answer: B

Again, we have:
 R joined four persons before the one who
likes Red Chief.
 At least one and not more than three
persons joined between the one who likes
Bata and R.
We have:  Both R and O like neither Nike nor
 The one who likes Bata joined on an even Adidas.
numbered date in a month having an odd That means, in case (1) R joined on 9
number of days. March, in case (2) R joined on 7 June, in
 Only three persons joined between the case (2a) R joined on 4 June.
one who likes Bata and W, who joined Based on the above given information we have:
immediately after the one who likes Nike.
That means, in case (1) the one who likes
Bata joined on 4 March, in case (2) the
one who likes Bata joined on 4 August.
Based on the above given information we have:

Again, we have:

Click Here For Bundle PDF Course | support@guidely.in Page 6 of 11


SBI Clerk & RRB PO Mains PDF Course 2023
Reasoning Ability Day - 11 (Eng)

 S joined three persons before the one That means, W likes Mochi and T likes
who likes Adidas. Bata.
 Both O and S like neither Bata nor Nike. Based on the above given information we have:
 As many persons joined before the one
who likes Adidas as after O.
Since, R doesn’t like Adidas.
That means, in case (2) S joined on 4
March, in case (2a) R joined on 9 June,
case (1) is not valid.
Based on the above given information we have:

Case (2a) is not valid as the one who likes


Relaxo joined immediate after U.
Again, we have:
Case (1) is not valid as O doesn’t like Bata.  More than two persons joined between
Again, we have: the one who likes Liberty and Q, who likes
 As many persons joined between the one neither Red Chief nor Reebok.
who likes Red Chief and O as between P Thus, S likes Liberty and O likes Reebok.
and the one who likes Relaxo. Based on the above given information we have:
 The one who likes Relaxo joined
immediately after U, who joined on an
even numbered date.
That means, in case (2) U joined on 4
June, case (2a) is not valid.
 P and the one who likes Mochi joined in
the same month.
 Only three persons joined between T and
the one who likes Mochi.

Click Here For Bundle PDF Course | support@guidely.in Page 7 of 11


SBI Clerk & RRB PO Mains PDF Course 2023
Reasoning Ability Day - 11 (Eng)

Arrangement 2:

We have:
Outcome: (6, 4), (6, 3), (4, 5), (1, 2), (1, 5), (1, 1),
(3, 2), (5, 3), (3, 3), (5, 2), (4, 4), (6, 6), and (5,
5).
Even digital products: 36, 24, 20, 18,
Odd Digital products: 1, 5, 9, 15, 25
Digital sum: 3, 5, 7, 8

Direction (6-10):
6. Answer: B
7. Answer: C
8. Answer: A
9. Answer: A(All the letters are placed in the
middle of the row except option A)
10. Answer: D
Arrangement 1: Arrangement 1:
D, I, R, Z, T, E, O, F, K, U, G, P, and V are to be
filled against the outcomes. Only one letter is
placed along with each outcome.
I. All vowels are placed in reverse alphabetical
order along with each outcome whose digital
sum is a prime number from top to bottom
respectively.
Thus, U, O, I, and E are placed.
II. All consonants which come before M are
placed in reverse alphabetical order along with

Click Here For Bundle PDF Course | support@guidely.in Page 8 of 11


SBI Clerk & RRB PO Mains PDF Course 2023
Reasoning Ability Day - 11 (Eng)

each outcome whose digital sum is an even 12. Answer: B


number from top to bottom respectively. 13. Answer: A(All the persons are sitting in the
Thus, K, G, F, and D are placed. middle of the table except option a)
III. All the remaining letters are placed with the 14. Answer: D
remaining outcomes from bottom to top in 15. Answer: C
alphabetical order. (If more than two outcomes
are at the same level then the digital sum of the
outcome is highest given first preference)
Thus, the remaining letters: P, R, T, V, and Z are
placed
Based on the above conditions we have:

We have:
 H sits third to the right of the one who is
facing C, who sits at the corner of the
inner table.
 One person sits between C and E, who
sits opposite to G.
That means, in case (1) E sits second to
the left of C, in case (2) E sits second to
the right of C.
Arrangement 2:
 The one who is facing G sits third to the
All the letters are placed in a row from the left
left of N.
end based on the descending order of their
Based on the above given information we have:
digital sum of the outcome and facing north. (If
the outcome of letters are same, then give
preference in alphabetical order).
Based on the above given condition we have:

Again, we have:

Direction (11-15):  Only three persons sit between F and D,

11. Answer: C who sits adjacent to H.

Click Here For Bundle PDF Course | support@guidely.in Page 9 of 11


SBI Clerk & RRB PO Mains PDF Course 2023
Reasoning Ability Day - 11 (Eng)

Since, the persons in alphabetical order


are neither sitting at the same table nor
sits facing each other.
That means, in case (1) & case (2) D sits
immediate right of H.
 As many persons sit between F and J as Case (1) & case (2) are not valid as no place

between K and M, when counted from the available for I and A.

left of both F and K. Again, we have:

 K sits opposite to O, who neither sits  Two persons sit between B and P.

adjacent to C nor E. That means, P sits immediate left of F.

 Both M and I don’t sit adjacent to E. Based on the above given information we have:

Since, we have to consider the number of


persons between F and J, thus J and K
can’t sit at the same table.
That means, in case (1a) M is facing D, in
case (1) M sits immediate right of C, in
case (2) M sits immediate left of C.
Based on the above given information we have:

Direction (16-20):
16. Answer: B
17. Answer: D
18. Answer: A
Again, we have: 19. Answer: C
 Only three persons sit between A and I. 20. Answer: B
 The number of persons sitting between I We have:
and O is one less than the number of For the first symbol: If the word starts with a
persons sitting between B and J, when consonant then it is coded as ‘@’, else it is
counted from the right of both B and I. coded as ‘#’.
That means, in case (1a) I sits immediate For the second symbol: the second symbol
right of C, case (1) & case (2) are not represents the number of letters in the word.
valid. For 3  $, 4  ^, and 5  %
Based on the above given information we have: For the third symbol: the third symbol represents
the last letter of the word.

Click Here For Bundle PDF Course | support@guidely.in Page 10 of 11


SBI Clerk & RRB PO Mains PDF Course 2023
Reasoning Ability Day - 11 (Eng)

Y  +, N  *, E  ¥, and L  £

Click Here For Bundle PDF Course | support@guidely.in Page 11 of 11


SBI Clerk & RRB PO Mains PDF Course 2023
Quantitative Aptitude Day - 11 (Eng)

Quantitative Aptitude

Directions (01 - 05): Study the following b) 29.65%


information carefully and answer the questions c) 27.50%
given below. d) 15.35%
The table given below shows the number of e) 10.33%
mobiles [4G+5G] sold in shop A in five months
and also given the Percentage of the number of 2) Total number of mobile phones sold in shop B
4G mobiles sold in each month in shop A. in March is [(2M-N+O)/2] % more than that of
same month in shop A. If the ratio of the number
of 4G and 5G mobiles sold in shop B in March is
3:2 then find the total number of 5G mobiles sold
in shop A and shop B together in March?
a) 1452
b) 2952
c) 2150
d) 1664
e) 1894

3)
I. Number of 5G mobiles sold in April is ______
more than the number of 4G mobiles sold in
May.
Note: II. Number of 4G mobiles sold in February is ____
1. M+N+120=220 more than the number of 5G mobiles sold in
2. O% of 120 + 60% of 240=204 January.
3. 2M+O=50% of 260 Find which of the given option satisfy the above
1) If the number of 4G mobiles sold in June is blank?
increased by (N/4) % than the previous month a) 360,150
and the number of total mobiles sold in June is b) 360,120.
increased by (M/2) % than the previous month. c) 120,360
Find the percentage of increase in 5G mobiles d) 150,360.
sold in June than the previous month? e) None of these
a) 24.35%

Click Here For Bundle PDF Course | support@guidely.in Page 1 of 10


SBI Clerk & RRB PO Mains PDF Course 2023
Quantitative Aptitude Day – 11 (Eng)

4) If the total number of 4G and 5G mobiles sold covered by car A in 6 hours. Car C can cover a
in all months is represented in the pie chart then distance of 255 km in 8.5 hours.
find the central angle made by the total number 6) Car A covers L km in (t+2) hours and car B
of 4G and 5G mobiles sold in May? covers M km in (t+1) hours. The value of L is 40
a) 600 km less than the distance between M and N.
b) 960 Then the distance covered by car C in 5t hours is
c) 1080 how much more than M km?
d) 720 a) 220km
e) None of these b) 260km
c) 250km
5) Number of 4G and 5G mobiles sold in June is d) 210km
840 and 680 respectively and the Number of 4G e) 280km
and 5G mobiles sold in July is 940 and 780
respectively. Find the difference between the 7) Car B covers (D+120) km in 5 hours and Car
total number of 4G mobiles sold in the first six E covers (2D+140) km in 15 hours. Find the
months and the total number of 5G mobiles sold average speed of car B and car E?
in the last six months till July? a) 145/3 km/hr
a) 20 b) 401/3 km/hr
b) 30 c) 332/3 km/hr
c) 50 d) 115/3km/hr
d) 80 e) 135/3 km/hr
e) None of these
8) Speed of car F is 20% more than the speed of
Directions (06 - 10): Study the following car D. Car F and Car C together covered some
information carefully and answer the questions distance in 5 hours. The total distance covered
given below. by these two cars is covered by car A in t hours.
The speed of car A is 33.33% more than the Find the value of t?
speed of car C. Speed of car C is 40% less than a) 11.75 hours
the speed of car B. A certain distance is covered b) 12.75 hours
by car D in 2a hours and same distance is c) 19.35 hours
covered by car A in 3a hours. Distance between d) 21.35 hours
two points P and Q is covered by car D in 3 e) None of these
hours. Distance between two points M and N is

Click Here For Bundle PDF Course | support@guidely.in Page 2 of 10


SBI Clerk & RRB PO Mains PDF Course 2023
Quantitative Aptitude Day – 11 (Eng)

9) Car A start travelling at point P and car B at ____(n)____ days. They together complete 50%
point Q at 9 am towards Q and P respectively. of work in ___(o)___ days.
After some hours, they meet each other then car Find in which of the given options value of m, n,
B reached P and car A reached Q but car A and o satisfy the above condition.
returns to P. Find the difference between the I. m=1/4, n=8 and o=24/5
total time (in minutes) when both cars reached II. m=1/5, n=10 and o= 12
P? III. m=1/3, n=15 and o=10
a) 324 min a) Only I
b) 321 min b) Only III
c) 334 min c) Only I and III
d) 384 min d) Only II and III
e) None of these e) Only I and II

10) Point O and N is in the East and south 12) Cost price of the item is Rs.240. Shop
direction from point M. Distance between MN keeper marked up the item 60% above the cost
and MO is the same. Car A and D start their price then gave a discount D% and earn P%
journey with their original speed at the same time profit. Find the correct combination of values D
from M and they reached N and O respectively. and P in the given Column.
After reached their respective points, again they
start their journey from N and O and reached M
but both of them reached at same time. Find the
increased percent speed of car A when coming
back to M?
a) II-A
a) 50%
b) III-A
b) 100%
c) II-B
c) 200%
d) I-C
d) 75%
e) More than one option is correct
e) None of these

13) Speed of the boat in still water is m km/hr


Directions (11 - 15): Study the following
and the Speed of the stream is 4 km/hr. Boat can
information carefully and answer the questions
cover 48 km in 3 hours in upstream.
given below.
Find which of the given option is true from the
11) A can complete ___(m)____ part of work in 4
above data?
days and B can complete 1/3rd of work in

Click Here For Bundle PDF Course | support@guidely.in Page 3 of 10


SBI Clerk & RRB PO Mains PDF Course 2023
Quantitative Aptitude Day – 11 (Eng)

I. The Speed of the car is (m+28) km/hr and car d) 40, 25


cover 144 km in 3 hours. e) More than one option is true.
II. The boat covers 116 km in 4.5 hours
downstream. Directions (16-20): Study the following
III. The distance covered by boat in 5 hours information carefully and answer the questions
downstream is the same as the distance covered given below. There are three rows, calculate the
by bus with the speed of 32 km/hour in 3 hours. roots of the equation and compare them with
a) Only II other columns. Check which one matches.
b) Only I 16)
c) Only III Equations Statements
d) Only I and II i) 5x2-18x+9=0 a) Sum of the roots is
e) None of these positive.
ii) 2x2-12x+18=0 b) Sum of roots is
14) Ratio of milk and water in a mixture is 3:2. negative.
Total quantity of the mixture is _____ liters and iii) 2x2+15x+13=0 c) Both roots are
40% mixture is taken out from the mixture then integers.
amount of milk in the remaining mixture is _____ a) ii)-b)
litres. b) i)-b)
Find which of the option is true which fill the c) i)-a) & iii) -b)
blank in same sequence? d) iii)-c)
a) 1050,378 e) None of these
b) 1430,512.8
c) 1200,430 17)
d) 1800,640 Equations Statements
e) None of these i) x2+16x+39=0 a) Sum of the roots is
positive.
15) Average age of A, B and C is ____ years. ii) x2+15x+56=0 b) Sum of roots is
Ratio of age of B and C is 3:2. Age of A after 5 negative.
years is 29 years. The difference in age between iii) x2-9x+14=0 c) Both the roots are
B and C is ______. positive.
Find which given option satisfies the blank? a) ii)-a)
a) 30, 15 b) i)-c)
b) 25, 18 c) i)-a) & ii) -c)
c) 28, 12

Click Here For Bundle PDF Course | support@guidely.in Page 4 of 10


SBI Clerk & RRB PO Mains PDF Course 2023
Quantitative Aptitude Day – 11 (Eng)

d) iii)-c) negative.
e) None of these iii) x2+20x+75=0 c) Both the roots are
positive.
18) a) ii)-a)
Equations Statements b) i)-b)
i) x2 -56x+784=0 a) Sum of the roots is c) i)-b) & ii) -b)
positive. d) i)-c)
ii) x2+10x-171=0 b) Sum of roots is e) None of these
negative.
iii) 2x2+12x+16=0 c) Highest root is a 20)
perfect square Equations Statements
number. i) 4x2+2x-20=0 a) Sum of the roots is
a) ii)-a) positive.
b) i)-b) ii) 3x2-10x-13=0 b) Sum of roots is
c) ii)-c) & ii) -b) negative.
d) iii)-c) iii) 10x2-8x-2=0 c) Both the roots are
e) None of these positive prime
numbers.
19) a) ii)-a)
Equations Statements b) i)-a)
i) 2x2-19x+17=0 a) Sum of the roots is c) i)-a) & ii) -b)
positive. d) iii)-c)
ii) x2+16x+60=0 b) Both the roots are e) None of these

Click Here For Bundle PDF Course | support@guidely.in Page 5 of 10


SBI Clerk & RRB PO Mains PDF Course 2023
Quantitative Aptitude Day - 11 (Eng)

Click Here to Get the Detailed Video Solution for the above given Questions
Or Scan the QR Code to Get the Detailed Video Solutions

Answer Key with Explanation

Directions (01 - 05):


2. O% of 120 + 60% of 240=204
Or, O=[204-144]*100/120=50
3. 2M+O=50% of 260
So, 2M=130-50=80, M=40
1. M+N+120=220
So, N=220-120-40=60
The Number of mobile phones sold in January is 1) Answer: C

1200. The total number of mobiles sold in June is

The Number of 4G mobile phones sold in 1200*120/100=1440

January is 1200*40/100=480 Number of 4G mobiles sold in June is

The number of 5G mobile phones sold in 720*[100+(N/4)]/100=828

January is 1200-480=720 The Number of 5G mobiles sold in June is 1440-

The Number of mobile phones sold in February 828=612

is 2600-1200=1400 So, increase percentage = [(612-

The Number of 4G mobiles sold in February is 480)/480]*100=27.5%

1400*60/100=840
The Number of 5G mobiles sold in February is 2) Answer: D

1400-840=560 Number of mobile phone sold in shop B in March

Similarly, we can calculate forother months also. is 1600*[100+(80-


60+50)/2]=1600*135/100=2160

Click Here For Bundle PDF Course | support@guidely.in Page 6 of 10


SBI Clerk & RRB PO Mains PDF Course 2023
Quantitative Aptitude Day - 11 (Eng)

The number of 5G mobiles sold in shop B in So, if the speed of car B is 100 then the speed of
March is 2160*2/5=864 car C is 60.
The total number of 5g Mobile sold in March in So, The ratio of the speed of cars B and C is 5:3.
shop A and B together is 864+800=1664 So, The speed of cars A, B, C, and D is 4:5:3:6.
The Speed of car C is 255/8.5=30 km/hr.
3) Answer: B So, The Speed of car A, B, and D is 4*10=40
I. Number of 5G mobiles sold in April is __(1080- km/hr, 5*10=50 km/hr, 6*10=60 km/hr.
720)=360___ more than the number of 4G Distance betweenP and Q is 60*3= 180 km
mobiles sold in May. Distance between M and N is 40*6=240 km
II. Number of 4G mobiles sold in February is 6) Answer: C
__(840-720)=120_ more than the number of 5G 40*(t+2)=L=240-40=200km
mobiles sold in January. Or, 40t=120, t=3
So, the answer is 360,120. M=50*4=200km
C cover distance in 5*3=15 hours is
4) Answer: A =15*30=450km
The central angle made by total mobiles sold in So, the difference is 450-200=250 km
May
= 7) Answer: D
[1200/(1200+1400+1600+1800+1200)]*360=600 So, D+120=5*50=250
Or, D=250-120=130
5) Answer: A So, the speed of car E is
Required difference [130*2+140]/15=400/15km/hr
= [480+840+800+720+720+840] – So, average speed is = (400/15 + 50)/2 =230/6 =
[560+800+1080+480+680+780] 115/3km/hr
= 20
8) Answer: B
Directions (06 - 10): The speed of car F is 60*120/100=72 km/hr.
The speed of car A is 33.33% more than the Total distance covered by car F and Car C is
speed of car of C. 5*(72+30)=510km
So, the speed ratio of car A and car C is 4:3. So, value of t= 510/40=12.75 hours
The Speed ratio of cars A and D is 2:3.
The Speed of car C is 40% less than the speed 9) Answer: A
of car B. So, car A cover 180*2=360 Km distance.

Click Here For Bundle PDF Course | support@guidely.in Page 7 of 10


SBI Clerk & RRB PO Mains PDF Course 2023
Quantitative Aptitude Day - 11 (Eng)

Total time taken by car A is = 360/40=9 Similarly, we can check for other values also. II-
hours=540 min A also satisfies the given condition.
Car B cover only 180 km. 13) Answer: B
So, time taken by car B is 180/50=3.6 hours=216 The Speed of the boat upstream is =48/3=16
min km/hr
The difference in time is 540-216=324 min The Speed of the boat in still water is
16+4=20km/hr=m.
10) Answer: C I. The Speed of the car is (m+28) km/hr and the
D cover full journey in (240/60) * 2 =8 hrs car cover 144 km in 3 hours.
A cover first half journey in (240/40) =6 hrs So, [20+28]*3=144km
Remaining hrs = 2 So, it is true.
Increase in speed of car A = 240/2 = 120km/hr II. The boat covers 116 km in 4.5 hours
Required increase in speed = (120-40)/40 * 100 downstream.
= 200% [20+4]*4.5=108 km, so it is false.
III. The distance covered by boat in 5 hours in
11) Answer: A downstream is the same as the distance covered
Now check the value of I, by bus with the speed of 32 km/hour in 3 hours.
So, m=1/4, A can complete in 4*4=16 days, Boat covers in 5 hours in downstream is
B can complete in 8*3=24 days 5*24=120 km
So, the total work is LCM of 16, and 24 is 48 Bus covers in 3 hours is 32*3=96 km
units. So, it is false,
The efficiency of A and B is 3 and 2 units/day Only I is true.
respectively.
A and B together complete 50% of work in 24/5 14) Answer: A
days. Let's check option a,
Options I is true. Similarly, we can check others The total quantity of the mixture is 1050 liters,
options so other two options are false. The amount of milk is 1050*3/5=630 liters
Left milk is 630*60/100=378 liters
12) Answer: E So, it is true. Similarly, we check other options
From the given options E is true. also. So, only A is true.
Marked price is 240*160/100=Rs.384
Selling price 384*75/100=288 15) Answer: C
Profit percentage = [(288-240)/240] *100=20% Let's check the option C,

Click Here For Bundle PDF Course | support@guidely.in Page 8 of 10


SBI Clerk & RRB PO Mains PDF Course 2023
Quantitative Aptitude Day - 11 (Eng)

The average age of A,B, and C is 28years, total or, x2-2*28x+282=0


age is 28*3=84years or, (x-28)2=0, x=28, 28
The Sum of the age of B and C is 84-[29- x2+10x-171=0
5]=60years or, x2+19x-9x-171=0
So, difference in age of B and C is 60/5=12years or, (x+19)(x-9)=0, x=-19,9
So, it is true. 2x2+12x+16=0
2x2+8x+4x+16=0
16) Answer: C Or, (x+4)(2x+4)=0, x=-4,-2
5x2-18x+9=0 So, option C is true.
Or, 5x2-15x-3x+9=0
Or, (x-3)(5x-3)=0, x=3,5/3 19) Answer: D
2x2-12x+18=0 2x2-19x+17=0
Or, 2x2-6x-6x+18=0 Or, 2x2-17x-2x+17=0
Or, (x-3)(2x-6)=0, x=3,3 Or, (x-1)(2x-17)=0, x=1,17/2
2x2+15x+13=0 x2+16x+60=0
Or, 2x2+13x+2x+13=0 or, x2+10x+6x+60=0
Or, (2x+13)(x+1)=0, x=-1,-13/2 or, (x+10)(x+6)=0, x=-10,-6
So, option c is true. x2+20x+75=0
or, x2+15x+5x+75=0
17) Answer: D or, (x+15)(x+5)=0, x=-15,-5
x2+16x+39=0 so, option D is true.
or, x2+13x+3x+39=0
or, (x+13)(x+3)=0, x=-13,-3 20) Answer: A
x2+15x+56=0 4x2+2x-20=0
or, x2+7x+8x+56=0 Or, 4x2+10x-8x-20=0
or, (x+7)(x+8)=0, x=-7,-8 Or, (2x+5)(2x-4)=0, x=-5/2,2
x2-9x+14=0 3x2-10x-13=0
or, x2-7x-2x+14=0 Or,3x2-13x+3x-13=0
or, (x-7)(x-2)=0, x=7,2 Or, (3x-13)(x+1)=0, x=13/3,-1
so, option D is true. 10x2-8x-2=0
Or, 10x2-10x+2x-2=0
18) Answer: C Or, (x-1)(10x+2)=0, x=1,-1/5
x2-56x+784=0 So, option A is true.

Click Here For Bundle PDF Course | support@guidely.in Page 9 of 10


SBI Clerk & RRB PO Mains PDF Course 2023
Quantitative Aptitude Day - 11 (Eng)

Click Here For Bundle PDF Course | support@guidely.in Page 10 of 10


SBI Clerk & RRB PO Mains PDF Course 2023
ENGLISH Day - 11

English Language

Directions (1-7): Read the given passage currency gyrates in an unpredictable manner. In
carefully and answer the following questions. such a scenario, many countries are forced to
The International Monetary Fund (IMF) last week seek help from the IMF to meet their external
confirmed a $3 billion bailout plan for Sri Lanka’s debt and other obligations, to purchase essential
struggling economy. IMF officials are also in imports, and also to prop up the exchange value
negotiations with Pakistan for a $1.1 billion of their currencies.Meanwhile, a country’s
bailout plan as the country faces a severe domestic economic policies can also have an
economic crisis marked by a falling currency and adverse impact on its currency’s exchange rate
price rise.Countries seek help from the IMF and foreign exchange reserves. For example,
usually when their economies face a major economic policy that imperils productivity can
macroeconomic risk, mostly in the form of a affect a country’s ability to attract the necessary
currency crisis. For instance in the case of Sri foreign exchange for its survival. Bad luck can
Lanka and Pakistan, both countries have also contribute to a crisis. In the case of Sri
witnessed domestic prices rise rapidly and the Lanka, a decrease in foreign tourists visiting the
exchange value of their currencies drop steeply country led to a steep fall in the flow of U.S.
against the U.S. dollar. Such currency crises are dollars into the nation.The IMF basically lends
generally the result of gross mismanagement of money, often in the form of special drawing rights
the nation’s currency by its central bank, often (SDRs), to troubled economies that seek the
under the covert influence of the ruling lender’s assistance. SDRs simply represent a
government. Central banks may be forced by basket of five currencies, namely the U.S. dollar,
governments to create fresh money out of thin air the euro, the Chinese yuan, the Japanese yen,
to fund populist spending. Such spending and the British pound. The IMF carries out its
eventually results in a rapid rise of the overall lending to troubled economies through a number
money supply, which in turn causes prices to rise of lending programs such as the extended credit
across the economy and the exchange value of facility, the flexible credit line, the stand-by
the currency to drop. A ______(A), unpredictable agreement, etc. Countries receiving the bailout
fall in the value of a currency can destroy can use the SDRs for various purposes
confidence in said currency and affect economic depending on their individual circumstances.
activity as people may turn hesitant to accept the Currently, both Sri Lanka and Pakistan are in
currency in exchange for goods and urgent need for U.S. dollars to import essential
services.Foreigners may also be unwilling to items and also to pay their foreign debt. So any
invest in an economy where the value of its

Click Here For Bundle PDF Course | support@guidely.in Page 1 of 13


SBI Clerk & RRB PO Mains PDF Course 2023
ENGLISH Day - 11

money that they receive from the IMF is likely to lending to troubled economies may turn out to be
go towards addressing these urgent issues. a wasted effort because these economies have
The IMF was set up in 1945 out of the Bretton poor institutions and suffer from high corruption.
Woods conference. The primary goal of the IMF In other words, these countries are most likely to
back then was to bring about international squander the bailout money.
economic coordination to prevent competing 1. From the given passage above, what do you
currency devaluation by countries trying to understand by the term ‘bailout plan’ that is used
promote their own exports. Eventually, the IMF to recover a country’s failing economy ?
evolved to be a lender of last resort to a) Bailout occurs when a third party government
governments of countries that had to deal with or agency steps in to save a country or a
severe currency crises.The IMF usually imposes company by providing them with capital, credit
conditions on countries before it lends any and other forms of support.
money to them. For example, a country may b) The countries collect a fund for bailout and
have to agree to implement certain structural use it for a country when its economy is failing or
reforms as a condition to receive IMF loans. The if it has a currency crisis.
IMF’s conditional lending has been controversial c) Bailout is when the government prints extra
as many believe that these reforms are too tough currencies to induce liquidity but keeping inflation
on the public. Some have also accused the IMF’s in mind, because excess generation of money
lending decisions, which are taken by officials may lead to currency devaluation.
appointed by the governments of various d) Bailout is performed either by the World bank
countries, to be influenced by international or the International Monetary Fund as they were
politics.Supporters of the IMF’s lending policies, established with this agenda.
however, have argued that conditions are e) None of these
essential for the success of IMF lending. For one,
countries that seek an IMF bailout are usually in 2. According to the given passage, which of the
a crisis due to certain policies adopted by their following sentence(s) is/are definitely true?
governments that turned out to be inimical to a) The primary goal of the World bank back then
economic growth and stability. It may thus not was to bring about international economic
make sense for the IMF to throw money at a coordination and to reduce currency devaluation
country when the policies that caused its crisis by countries.
remain _________(B). So, for instance, the IMF b) Sri Lanka and Malaysia, both countries have
may demand a country affected by high price witnessed domestic prices rise rapidly and the
inflation to ensure the independence of its central exchange value of their currencies drop steeply
bank. Corruption is another issue. The IMF against the Euros.

Click Here For Bundle PDF Course | support@guidely.in Page 2 of 13


SBI Clerk & RRB PO Mains PDF Course 2023
ENGLISH Day - 11

c) The IMF lends money, often in the form of as there is excess money in the hands of the
special drawing rights to the troubled economies politicians.
that seek assistance and support. d) The IMF lending to troubled economies may
d) Sri Lanka is the country which has opted for turn out to be a wasted effort because of poor
bailout options for more than 10 times from IMF institutions and suffer from high corruption.
and World Bank e) Corruption and poor institutions do not affect
e) Even Though the economy is in an the lending activities because the country has to
unpredictable state, the foreigners are willing to recover first and the country’s people and the
invest and take risks because they believe that government understand it.
currency devaluation will be followed by
revaluation. 5. According to the given passage, which of the
following sentence(s) is/are definitely false?
3. Which of the following are the reasons for the a) Economic activity is affected by the value of
currency crises that a country faces? the currency as people lose the confidence in
a) Governments may create pressure on the spending or to accept money in exchange for
central bank to create fresh money for spending goods and services.
for education. b) Domestic policies of a country also affect the
b) A serious mismanagement by the central bank rates.
under the influence of the government. c) Countries receiving the bailout can use the
c) Too much spending by the government for special drawing rights facility for various
sectors that are already developed. purposes depending on the situation of the crisis.
d) The investors are not able to assess the risk d) The IMF is the lender of last resort to
factor of the company as there are no proper governments of countries.
sources and agencies to help them. e) All the given statements are true.
e) None of these
6. Fill in the blanks A and B with the suitable
4. How does corruption affect the lending words.
activities of the IMF to a country? a) Known, same
a) Corruption is the major reason for a currency b) Clear, changed
crisis in a country. c) Slow, Unchanged
b) The lending activities are strongly influenced d) Rapid, Untouched
by the corrupt ministers of a country. e) Same, alike
c) The bailout money that is provided by the IMF
to deal with the crisis, actually triggers corruption

Click Here For Bundle PDF Course | support@guidely.in Page 3 of 13


SBI Clerk & RRB PO Mains PDF Course 2023
ENGLISH Day - 11

7. Choose from the given options the exact Schedule has failed to fulfil the purpose for which
OPPOSITE of the word ‘hesitant’ as used in the it was ________.
passage. a) By, directly, enacted
a) Unsure b) For, informally, formed
b) Doubtful c) Through, altogether, enacted
c) Indecisive d) With, instantly, made
d) Uncertain e) Of, altogether, introduced
e) confident
10. In the midst of navigating the tumultuous
Directions (8-12): Each of the following questions hardships of _________, 12-year-old mathematics
has three blanks, each blank indicating that a genius Prem Patel discovers his recently
word has been omitted. Choose the set of words __________father was a famous rapper and
that best fits the given blanks according to the immediately sets out to learn more about his
context of the sentence. father’s life and passions, while his recent
8. In most of the cases dealing with political actions may appear ________and the quickest
violence, killings and sexual assault, the court way for him to lose everything.
has ruled _______the State government, it has a) Adolescence, deceased, reckless
often _________the necessity for an investigation b) Teen, dead, ruthless
by central agencies or a special investigation c) Boy, passed out, meaningless
team set up by the court, to _______the people’s d) Student, died, innocent
faith. e) Adult, deceased, strange
a) Out, stressed, get back
b) For, ignored, win 11. To face the continuing decline in the quality of
c) Of, avoided, let go our environment, we will need to ____more and
d) Against, imposed, loss more on solutions that draw upon biodiversity or
e) Against, underlined, restore nature, also called nature-based solutions to
_______our future and it is biodiversity that will
9. Anti-defection law introduced ________a restore our ________lands and polluted rivers
constitutional amendment in 1985, the Tenth and oceans and sustain our agriculture in the
Schedule penalises disobedience of the party face of climate change.
whip with disqualification from the House a) Depend, save, precious
_________and ironically, as recent events have b) Concentrate, getback, disowned
more than amply demonstrated, the Tenth c) Rely, protect, destroying
d) Rely, secure, degraded

Click Here For Bundle PDF Course | support@guidely.in Page 4 of 13


SBI Clerk & RRB PO Mains PDF Course 2023
ENGLISH Day - 11

e) Fund, improve, clean


A. If around ₹1.55 lakh D. for monthly GST
crore is likely to be the revenues, the
12. When the stock of ₹100 notes was quickly
new normal government must
_________, banks demanded more from the RBI.
seize this window to
The RBI had no option but to supply old and expedite the
________notes that banks had returned to the resolution of policy-
RBI earlier. Soiled notes often ________the level anomalies.
ATMs, which only added to the chaos.
a) Over, torn, stuck B. Report does not E. with a digitised

b) Exhausted, soiled, jammed clarify if Coromandel voting system and

c) Diminished, sandy, pass Express crossed well-engineered

d) Exhausted, bad, jamming acoustics.

e) Empty, soiled, free

C. The new Parliament F. have been banned


Directions (13-17): In the following questions two
building is centrally air- by the Central
columns are given containing three
conditioned and Government
Sentences/phrases each. In the first column,
closed,
sentences/phrases are A, B and C and in the
second column the sentences/phrases are D, E
a) A-D
and F. A sentence/phrase from the first column
b) B-F and C-E
may or may not connect with another
c) A-D and C-E
sentence/phrase from the second column to
d) B-D and A-E
make a grammatically and contextually correct
e) None of these
sentence. Each question has five options, four of
which display the sequence(s) in which the
14.
sentences/phrases can be joined to form a
grammatically and contextually correct sentence. COLUMN 1 COLUMN 2
If none of the options given forms a correct
A. With temperatures D. which will be
sentence after combination, mark option (E), i.e.
crossing the 42 supervised by the
“None of these” as your answer.
degree mark, central zoo authorities
13.
and forest team.
COLUMN 1 COLUMN 2

Click Here For Bundle PDF Course | support@guidely.in Page 5 of 13


SBI Clerk & RRB PO Mains PDF Course 2023
ENGLISH Day - 11

B. A detailed E. include the daily C. Vidyut’s grandfather F. but soon branched


investigations wage workers, the who was seriously ill out into the production
regarding the train beggars and other for the past couple of and release of short
accident will be done contract workers from months with severe movies
by the team different states. breathing issues

C.While industry F. the city records the a) A-E

experts claim that hottest day so far. b) C-F and B-D

some of these c) A-D

combinations aren’t d) A-E and C-D

available in the market e) None of these

currently, the banned


combinations 16.

a) C-D COLUMN 1 COLUMN 2

b) B-D A. Markets have been D. people can


c) A-E and B-D outperforming the continue to travel in
d) A-F majority of their global one of the routes that
e) None of these peers in recent weeks was immediately
however further repaired and fixed.
15.
deterioration of the
COLUMN 1 COLUMN 2 trend,

A.The Kamakshi D.breathed his last B. While the RBI has E. especially in the
temple, ancient shrines and died in his called on the public to US markets, might
existed in these hometown. exchange/deposit these change the entire
locations prior to 600 notes in banks by situation but put it
AD but the September 30, upside down.

B.Going by the past E. current construction C. Nonetheless, most F. who may get
year's trends, UPSC was built by the LGBT people in India promoted with work
result may be declared Pallavas and the remain closeted, fearing experience.
within 17 days from the Cholas and the temple discrimination from their
date of the examination covers an area of over families,
40 acres.

Click Here For Bundle PDF Course | support@guidely.in Page 6 of 13


SBI Clerk & RRB PO Mains PDF Course 2023
ENGLISH Day - 11

a) A-E statements are in logical order and form a


b) A-E and C-D coherent paragraph/passage. From the given
c) B-F options, choose the odd one out that does not fit
d) B-E and A-F into the theme of the passage.
e) None of these 18. Find the odd one out
A. For the second year in a row, the top three
17. ranks of the Civil Services Examination, 2022,
conducted by the Union Public Service
COLUMN 1 COLUMN 2
Commission have been secured by women.
A. Excel is a powerful D. help people buy B. In a spectacular achievement, results
tool that has become their groceries at a announced on Tuesday show that 12 women are
entrenched in business lowest price instantly. in the top 20.
processes worldwide C. The 26-year-old topper, Ishita Kishore, who
graduated from Delhi University’s Sri Ram
B. The effective E. and the army
College of Commerce and is a resident of
extraction and proper came to a head when
Greater Noida, reached the pinnacle in her third
assessment of military buildings and
attempt.
antioxidants from food property were
D. This clearly shows that the men take a
and medicinal plants ransacked last month
backseat when it comes to competitive exams.
are crucial
E. The young women’s reflections on success
C. With the pace F. to explore the and failure, the need to persevere against all
bowlers likely to do a lot potential antioxidant odds and the importance of an empathetic
of heavy lifting during sources and promote ecosystem are significant pointers for future
the one-off Test its application students, teachers and parents.
a) A-E a) A

b) B-F b) B

c) C-F and A-E c) C

d) B-D and A-E d) D

e) None of these e) E

Directions (18-20): In each of the following 19. Find the odd one out

questions, five statements are given below, A. Later this year, India will have a new

labelled A, B, C, D and E. Among these. Four ‘supercomputer’ or, more correctly, an upgraded

Click Here For Bundle PDF Course | support@guidely.in Page 7 of 13


SBI Clerk & RRB PO Mains PDF Course 2023
ENGLISH Day - 11

‘high performance computing (HPC)’ system that A. Mr. Modi presented the aesthetics of the new
will arguably be its fastest. parliament building as a representation of India’s
B. The Narendra Modi government signed a deal myriad diversity, its rich cultural heritage and its
in December 2018 with France to procure high- soaring aspirations.
performance computers worth ₹4,500 crore by B. India is a diverse nation because it is made up
2025. of Hindus (82.41%), Muslims (11.6%), Christians
C. These HPC systems will run at two (2.32%), Sikhs (1.99%), Buddhists (0.77%), and
institutions, the Indian Institute of Tropical Jains (0.41%), in addition to the tribal societies,
Meteorology, Pune, and the National Centre for many of which continue to practise animism and
Medium Range Weather Forecasting, Noida, that magic.
currently host two of India’s most powerful such C. A multi-religious prayer was a part of the
machines, Mihir and Pratyush. ceremony, but there was no mistaking that Hindu
D. Recently Indian Institute of tropical ritualism overshadowed all else.
meteorology launched a High-resolution global D. By weaving an artful tale around a Sengol, a
forecast model for better forecast of extreme sceptre gifted to the first Prime Minister of India
climatic episodes. by a Shaivite sect of Tamil Nadu, the current
E. Like their predecessors, the Atos machines dispensation has sought to reimagine the
will be used primarily to run sophisticated founding principles of India’s republican
weather models that, for some years now, are sovereignty.
being used to prepare a range of forecasts, from E. A Sengol symbolised divine right and is now
long-term monsoon to fortnightly as well as daily installed in the Assembly of people’s
weather changes. representatives and this strengthens Tamil
a) A Nadu’s connection to the political centre of India,
b) B and the Bharatiya Janata Party (BJP) is trying to
c) C make political gains from it.
d) D a) A
e) E b) B
c) C
20. Find the odd one out d) D
e) E

Click Here For Bundle PDF Course | support@guidely.in Page 8 of 13


SBI Clerk & RRB PO Mains PDF Course 2023
ENGLISH Day - 11

Click Here to Get the Detailed Video Solution for the above given Questions
Or Scan the QR Code to Get the Detailed Video Solutions

Answer Key with Explanation

1. Answer: A The last paragraph clearly states the content of


The first two lines give a clear idea about the Option ‘d’.Even Though IMF is ready to lend but
‘bailout’ concept. Option ‘a’ is the right definition. there is always an uncertainty if the offering is
Other options are wrong as there is no not wasted and utilised properly for proper
information regarding them in the given passage. recovery and well-being of the country.

2. Answer: C 5. Answer: E
Only option c is true according to the given All the five statements given are true according
passage, Other four options are either false or to the passage.
they are not mentioned in the passage and
hence tend to be false. 6. Answer: D
A rapid, unpredictable fall - rapid is the correct
3. Answer: B pick here because it is followed by the word
The main reason for the currency crises that is unpredictable.And the second blank should be
mentioned in the passage is “A serious filled with untouched.It may thus not make sense
mismanagement by the central bank under the for the IMF to throw money at a country when
influence of the government”. Other given the policies that caused its crisis remain
options are not mentioned as reasons for crises untouched. Option ‘d’ has the correct pair of
in the passage. words to fit both the blanks A and B respectively.

4. Answer: D 7. Answer: E

Click Here For Bundle PDF Course | support@guidely.in Page 9 of 13


SBI Clerk & RRB PO Mains PDF Course 2023
ENGLISH Day - 11

Hesitant - not sure to speak or do something and


afraid of the results. 9. Answer: C
Unsure,Doubtful,Indecisive and uncertain are the The correct option is ‘c’ as it has the correct
synonyms of Hesitant whereas Confident is the words that can fill the three blanks and make the
antonym and hence Option ‘e’ is correct. sentence contextually correct and meaningful.
First blank - for, with and of are wrong
8. Answer: E prepositions to fill the first blank, the law was
The correct option is ‘e’ as it has the correct introduced by an amendment or through an
words that can fill the three blanks and make the amendment(eliminate options b,d and e)
sentence contextually correct and meaningful. Second blank - altogether means completely or
First blank - ruled for and ruled of are wrong. by including everything/everybody. Whereas
Ruled out and ruled against are the possible directly is not an appropriate fit.
answers (eliminate the options b and c) By eliminating the options we can conclude
Second blank - stressed and underlined both are option ‘c’ as our answer.
applicable here The correct sentence :Anti-defection law
Impose is not a right fit because, the meaning of introduced through a constitutional amendment
impose is to make a law/ask somebody to in 1985, the Tenth Schedule penalises
perform a particular action.(eliminate option d) disobedience of the party whip with
Third blank - get back will not be a right option disqualification from the House altogether and
because get back - is to return to. Whereas ironically, as recent events have more than
‘restore’ would be the best fit as it has the exact amply demonstrated, the Tenth Schedule has
meaning to complete the sentence, meaning of failed to fulfil the purpose for which it was
restore - re-establish or bring back something to enacted.
its original form.
By eliminating the options we have arrived at the 10. Answer: A
right option as ‘e’. The correct option is ‘a’ as it has the correct
The correct sentence : In most of the cases words that can fill the three blanks and make the
dealing with political violence, killings and sexual sentence contextually correct and meaningful.
assault, the court has ruled against the State First blank - Clearly a 12 year old can be a
government, it has often underlined the adolescent, boy or student ( eliminate option b
necessity for an investigation by central and e - teen and adult are wrong words to
agencies or a special investigation team set up address a 12yr old)
by the court, to restore the people’s faith.

Click Here For Bundle PDF Course | support@guidely.in Page 10 of 13


SBI Clerk & RRB PO Mains PDF Course 2023
ENGLISH Day - 11

Second blank - deceased is the right word to fit draw upon biodiversity or nature, also called
the second blank, died is a wrong usage, passed nature-based solutions to secure our future and
out is to become unconscious - also a wrong it is biodiversity that will restore our degraded
usage. lands and polluted rivers and oceans and sustain
By eliminating the options, the answer is option our agriculture in the face of climate change.
‘a’.
The correct sentence : In the midst of navigating 12. Answer: B
the tumultuous hardships of adolescence, 12- The correct option is ‘b’ as it has the correct
year-old mathematics genius Prem Patel words that can fill the three blanks and make the
discovers his recently deceased father was a sentence contextually correct and meaningful.
famous rapper and immediately sets out to learn First blank - diminished and empty are wrong
more about his father’s life and passions, while fits(because diminished is to decrease or go
his recent actions may appear reckless and the smaller in size and empty is used for a
quickest way for him to lose everything. container/storage- hence eliminate options c and
e)
11. Answer: D Second blank - all words may fit the blank
The correct option is ‘d’ as it has the correct Third blank - ‘jammed the atm’ is the right way to
words that can fill the three blanks and make the put and complete the sentence in a meaningful
sentence contextually correct and meaningful. way.
First blank - all the words given as options for Hence, option b is the right pick.
first blank are suitable in one way or the other. The correct sentence : When the stock of ₹ 100
Second blank - all words are suitable in one way notes was quickly exhausted, banks demanded
or the other more from the RBI. The RBI had no option but to
Third blank - restore our ____ lands and polluted supply old and soiled notes that banks had
rivers. Here we can clearly have an idea that returned to the RBI earlier. Soiled notes often
blank should have a word with negative meaning jammed the ATMs, which only added to the
because we have a hint “polluted rivers” so the chaos.
lands must also be in a poor state. The only right
fit is degraded, hence we go for option ‘d’ as our 13. Answer: C
answer. The sentence after combining A and D is “If
The correct sentence : To face the continuing around ₹ 1.55 lakh crore is likely to be the new
decline in the quality of our environment, we will normal for monthly GST revenues, the
need to rely more and more on solutions that

Click Here For Bundle PDF Course | support@guidely.in Page 11 of 13


SBI Clerk & RRB PO Mains PDF Course 2023
ENGLISH Day - 11

government must seize this window to expedite of their global peers in recent weeks however
the resolution of policy-level anomalies.” further deterioration of the trend, especially in
The sentence after combining C and E is “The the US markets, might change the entire
new Parliament building is centrally air- situation but put it upside down”
conditioned and closed,with a digitised voting Other options do not combine to form a
system and well-engineered acoustics.” meaningful sentence and hence not the right
Other combinations are meaningless and answer.
incorrect.
17. Answer: B
14. Answer: D The sentence is “The effective extraction and
Option ‘d’ is the right answer. After combining A proper assessment of antioxidants from food and
and F, With temperatures crossing the 42 medicinal plants are crucial to explore the
degree mark, the city records the hottest day so potential antioxidant sources and promote its
far. application”.
Other given combinations are wrong and
meaningless. 18. Answer: D
The sentence D is inappropriate to the context of
15. Answer: D the passage and hence it is the odd one out. The
The sentences after combining the phrases are context of the passage here revolves around the
“The Kamakshi temple, ancient shrines existed women and their progress towards the
in these locations prior to 600 AD but the competitive exam.There is no comparison
current construction was built by the Pallavas between the men and women.So, D sounds
and the Cholas and the temple covers an area of completely different and doesn’t fit in the
over 40 acres” and “Vidyut’s grandfather who passage.
was seriously ill for the past couple of months
with severe breathing issues breathed his last 19. Answer: D
and died in his hometown “ The sentence ‘D’ is the odd one out here as the
Other combinations do not form any meaningful context of the passage is about High
sentence and hence discarded, performance computers and not weather
forecasting. Even Though HPCs are used in
16. Answer: A weather reporting and forecasting, sentence ‘D’
After combing A and E, the complete sentence is doesn’t connect with other sentences as it talks
“Markets have been outperforming the majority about the institute’s launch which is

Click Here For Bundle PDF Course | support@guidely.in Page 12 of 13


SBI Clerk & RRB PO Mains PDF Course 2023
ENGLISH Day - 11

inappropriate and different from the idea of the building’. All the other four sentences provide
passage. information about the building and its
inauguration, whereas ‘B’ is totally out of context
20. Answer: B providing factual data about the diversity of our
Sentence ‘B’ is clearly the odd one out here as nation which is not required to be included in the
the given passage context is ‘the new parliament passage and hence it is the odd one out.

Click Here For Bundle PDF Course | support@guidely.in Page 13 of 13


SBI Clerk & RRB PO Mains PDF Course 2023
Reasoning Ability Day - 12 (Eng)

Reasoning Ability

Directions (1-5): Each of the questions below viz. January, March, May, June, August,
consists of a question and three statements September, October and December of the same
numbered I and II given below it. You have to year. Who among the person attends the
decide whether the data provided in the function three months before G?
statements are sufficient to answer the Statement I: G attends the function in the month
questions. having 30 days. Only two persons attend
1. The conclusion “No calendar being agenda is between G and A. C attends immediately before
a possibility” is definitely followed by which of the A. Only one month between the months in which
following statements? C and F attend the function. The number of
Statement I: Only a few notes are pens; Few persons attending before F is one less than the
pens are staplers; All wallets are staplers; At number of persons attending after H.
least some wallets are agenda; No calendar is a Statement II: A attends the function in the month
wallet. having 31 days but after August. C attends two
Statement II: Only pin is a mouse; At least some months before A. The number of persons
pins are chargers; only a few chargers are attending before C is one less than the number
agenda; All calendars are chargers. of persons attending after E. Only three months
a) If the data given in statement I alone is between the months in which E and H attend the
sufficient to answer the question. function. Only two persons attend the function
b) If the data given in statement II alone is between G and H.
sufficient to answer the question. a) If the data given in statement I alone is
c) If the data given in either statement I alone or sufficient to answer the question.
statement II alone is sufficient to answer the b) If the data given in statement II alone is
question. sufficient to answer the question.
d) If the data given in both statement I and c) If the data given in either statement I alone or
statement II together are necessary to answer statement II alone is sufficient to answer the
the question. question.
e) If the data given in both statements I and II d) If the data given in both statement I and
together are not sufficient to answer the statement II together are necessary to answer
question. the question.
e) If the data given in both statements I and II
2. Eight persons – A, B, C, D, E, F, G and H are together are not sufficient to answer the
attending the function in eight different months question.

Click Here For Bundle PDF Course | support@guidely.in Page 1 of 15


SBI Clerk & RRB PO Mains PDF Course 2023
Reasoning Ability Day - 12 (Eng)

3. What is the code for “Time Value Nothing” in Statement II: Only four persons sit between U
the given coded language? and T. F sits third to the right of T. A is an
Statement I: In a certain code language, immediate neighbour of only F. Only one person
“Problem arrives nothing duty” is coded as “po se sits between U and J. Q sits sixth to the left of J
df vc”, “Nothing Value duty dignity” is coded as and sits at one of the extreme ends of the row.
“se yt df ab”, “Value Nothing Problem Time” is a) If the data given in statement I alone is
coded as “yt se po ui”. sufficient to answer the question.
Statement II: In a certain code language, “Value b) If the data given in statement II alone is
Morals life level” is coded as “qw kl yt mn”, sufficient to answer the question.
Morals Nothing life lessons” is coded as “kl se jh c) If the data given in either statement I alone or
mn”, “Nothing Morals seat Time” is coded as “se statement II alone is sufficient to answer the
kl we ds”. question.
a) If the data given in statement I alone is d) If the data given in both statement I and
sufficient to answer the question. statement II together are necessary to answer
b) If the data given in statement II alone is the question.
sufficient to answer the question. e) If the data given in both statements I and II
c) If the data given in either statement I alone or together are not sufficient to answer the
statement II alone is sufficient to answer the question.
question.
d) If the data given in both statement I and 5. Eight persons – L, M, N, O, P, Q, R and S are
statement II together are necessary to answer living on four different floors of a four storey
the question. building, where the lowermost floor is numbered
e) If the data given in both statements I and II one and the floor immediately above it is
together are not sufficient to answer the numbered two and so on, who among the person
question. lives immediately above the flat of O?
Note-I: Each floor has two type of flats viz., Flat-
4. A certain number of persons is sitting in a A and Flat-B, where Flat A is to the west of flat B.
linear row and facing the north, how many Note-II: Flat B of floor 2 is immediately above
persons are sitting in the row? Flat B of floor 1 and immediately below Flat B of
Statement I: E sits third from one of the extreme floor 3 and so on. Similarly, Flat A of floor 2 is
ends of the row. Only three persons sit between immediately above Flat A of floor 1 and
E and J. U sits second to the left of J. Neither J immediately below Flat A of floor 3 and so on.
nor U sits at the extreme end of the row. Note-III: The area of each flat on each floor is
same.

Click Here For Bundle PDF Course | support@guidely.in Page 2 of 15


SBI Clerk & RRB PO Mains PDF Course 2023
Reasoning Ability Day - 12 (Eng)

Note-IV: Only two persons live on each floor and Note:


only one person lives in each flat. I. The age of the youngest person is 18 years.
Statement I: S lives three floors above Q, both II. No two persons have the same age.
are living in different type of flats. No one lives to III. If it’s given that X and the one whose age is
the west of P, who lives on the adjacent floor of “Y” lives in the same state, then X’s age is not
Q. The number of floors above P is one more “Y”
than the number of floors below O. G and the one whose age is 64 years live in the
Statement II: N lives on an odd numbered floor same state. Both C and E live in the same state.
and lives two floors below M. N lives east of Q. C neither lives with G nor lives in Manipur. B and
Only two floors are between Q and S, both are the one whose age is 58 years live in the same
living in different type of flats. P lives two floors state but both of them live neither in Assam nor
above O. R lives above O in different type of flat. in Tripura. B’s age is 47 years, which is 12 years
No one lives to the east of L. more than E’s age. The sum of the ages of A and
a) If the data given in statement I alone is G is 74 years and the age of A is less than 25
sufficient to answer the question. years. A lives with neither G nor B. A doesn’t live
b) If the data given in statement II alone is in Assam. The sum of the ages of B and F is 65
sufficient to answer the question. years. H lives with F but not lives with A and the
c) If the data given in either statement I alone or age of H is 7 years more than F. The youngest
statement II alone is sufficient to answer the person doesn’t live with G. D doesn’t live with H.
question. The age of I is 16 years more than C’s age. C’s
d) If the data given in both statement I and age is 5 years less than G.
statement II together are sufficient to answer the 6. What is the sum of the ages of the persons
question. living in Tripura?
e) If the data given in both statements I and II a) 138 years
together are not sufficient to answer the b) 104 years
question. c) 128 years
d) 135 years
Directions (6-10): Study the following information e) 125 years
carefully and answer the questions given below.
Nine persons – A, B, C, D, E, F, G, H and I live in 7. Who among the following person lives in
three different states- Assam, Tripura and Assam?
Manipur. Each of them is of different age. Not a) C
less than two and more than four persons live in b) E
the same state. c) D

Click Here For Bundle PDF Course | support@guidely.in Page 3 of 15


SBI Clerk & RRB PO Mains PDF Course 2023
Reasoning Ability Day - 12 (Eng)

d) G Step I: Consider the above series, all the letters


e) H in the series are changed to the fourth
succeeding letter as per the English alphabetical
8. Which of the following statements is/ are true series.
based on the given information? Step II: After completing step I, every fourth letter
a) D lives in Assam only with G from the left end is changed to its immediate next
b) The age difference between I and D is 11 letter as per the English alphabetical series
years whereas the letters in the odd numbered position
c) The sum of the age of the one who lives with from the left end are changed to its immediate
H and A is 39 years previous letter as per the English alphabetical
d) A lives in Manipur series.
e) All the given statements are true Step III: After completing step II, the letter which
has its corresponding number as the least prime
9. E lives in which of the following state? number in the English alphabetical series is
a) Tripura added immediately before every letter which has
b) Manipur its corresponding number as the multiple of 4 in
c) Assam the English alphabetical series.
d) Either a or b Step IV: After completing step III, every fourth
e) Either b or c letter from the left end is dropped and then the
remaining letters are changed to the immediate
10. Four of the following five are alike in a certain previous letter as per the English alphabetical
way based on the given arrangement and thus series.
form a group. Which one of the following does Step IV is the last and final step and the outcome
not belong to the group? in this step is the final series. As per the steps
a) C given above, find out the final alphabet series
b) F and answer the following questions.
c) I 11. If all the letters in the final series which are in
d) E the first half of the English alphabetical series are
e) D dropped and then the remaining letters are
changed to its corresponding complementary
Directions (11-15): Study the following pair in the alphabetical series and then arrange
information carefully and answer the given all the letters in the alphabetical order from the
questions. left end, then which of the following letter is
KHFMVLJACHIDBTRCULSE second to the left of fifth letter from the left end?

Click Here For Bundle PDF Course | support@guidely.in Page 4 of 15


SBI Clerk & RRB PO Mains PDF Course 2023
Reasoning Ability Day - 12 (Eng)

a) C 14. If all the B’s which are immediately preceded


b) J by a consonant are interchanged with the
c) F immediate next letter in step III then how many
d) D consonants are immediately preceded as well as
e) L immediately followed by the same letter?
a) Three
12. If a four letter meaningful word can be b) Two
formed by using the fourth letter from the left and c) One
then first, second and seventh letters from the d) More than four
right end (using each letter only once) of the final e) None
series, then what is the second letter from the
right end of the newly formed word? Mark X as 15. If all the vowels are dropped from step I, then
your answer if more than one word is formed, which among the following letter has equal
Mark Z if no meaningful word can be formed. number of letters towards its left as well its right?
a) X a) L
b) W b) Z
c) U c) H
d) Q d) M
e) Z e) F

13. If the first ten letters from step II are reversed Directions (16-20): Study the following
and the letters which are immediately followed by information carefully and answer the given
a vowel are changed to the immediate preceding questions.
letter as per the English alphabetical series and Five persons are taking therapy sessions on 3
then all the repeated letters are dropped, then different days in a week from Monday to Sunday
what is the sum of the place value of the letters at three different timings – 8:30 am to 9:30 am,
(as per English alphabetical series) which are 10:30 am to 11:30 am and 7 pm to 8 pm. Each of
third from both ends? them takes the therapy session at least once but
a) 22 not more than twice in a week. No one takes the
b) 41 therapy twice in a day. Each of them takes the
c) 54 therapy at different timings on all three days.
d) 25 There must be three persons or no persons
e) 71 taking the therapy in a day.
Note:

Click Here For Bundle PDF Course | support@guidely.in Page 5 of 15


SBI Clerk & RRB PO Mains PDF Course 2023
Reasoning Ability Day - 12 (Eng)

I. Two days gap between the days on which G 18. Which of the following statement is true with
and J take the sessions, and both are taking the respect to the final arrangement?
therapy session at different timings. a) H takes the therapy only before I
II. J does not take the therapy after 4.00 pm b) F takes the therapy only in the morning
III. I takes the therapy only in the morning but c) J takes the therapy only once in a week
does not take adjacent timing with G. d) As many days before H as after J
The persons may have the session only on the e) All the given statements are false
below-mentioned days:
19. Four of the following five are alike in a certain
way based on the given arrangement and thus
form a group. Which one of the following does
not belong to the group?
a) F
b) J
c) H
16. Who among the following person doesn’t
d) I
take the therapy on Sunday?
e) G
a) F
b) I
20. On which of the following days all the
c) J
persons take the therapy?
d) Both a and b
I. Monday
e) Both b and c
II. Thursday
III. Saturday
17. J takes the therapy session on which of the
following time? IV. Sunday
a) Only III
a) 8:30am-9:30am
b) Only I, II and III
b) 10:30am-11:30am
c) Only I, III and IV
c) 7:00pm-8:00pm
d) Only I, II and IV
d) Both b and c
e) All I, II, III and IV
e) Both a and c

Click Here For Bundle PDF Course | support@guidely.in Page 6 of 15


SBI Clerk & RRB PO Mains PDF Course 2023
Reasoning Ability Day - 12 (Eng)

Click Here to Get the Detailed Video Solution for the above given Questions
Or Scan the QR Code to Get the Detailed Video Solutions

Answer Key with Explanation

1. Answer: C Hence, either statement I or statement II is


From statement I: sufficient.

2. Answer: E
From statement I:
G attends the function in the month having 30
days. Only two persons attend between G and
A. C attends immediately before A. Only one
month between the months in which C and F
attend the function. The number of persons
Clearly, shows that “No calendar being agenda attending before F is one less than the number
is a possibility” is definitely follows. of persons attending after H.
From statement II: From the above conditions,

Clearly, shows that “No calendar being agenda


is a possibility” is definitely follows.

Click Here For Bundle PDF Course | support@guidely.in Page 7 of 15


SBI Clerk & RRB PO Mains PDF Course 2023
Reasoning Ability Day - 12 (Eng)

From statement I, clearly shows that we cannot From statement II, clearly shows that we cannot
determine the person who attends three months determine the person who attends three months
before G. before G.
From statement II: Hence, both statement I and II together is not
A attends the function in the month having 31 sufficient to answer the question.
days but after August. C attends two months
before A. The number of persons attending 3. Answer: A
before C is one less than the number of persons From statement I:
attending after E. Only three months between
the months in which E and H attend the function.
Only two persons attend the function between G
and H.
From the above conditions, we have

From statement I, clearly shows that “Time


Value Nothing” is coded as “ui yt se”
From statement II:

Click Here For Bundle PDF Course | support@guidely.in Page 8 of 15


SBI Clerk & RRB PO Mains PDF Course 2023
Reasoning Ability Day - 12 (Eng)

neighbour of only F. Only one person sits


between U and J. Q sits sixth to the left of J and
sits at one of the extreme ends of the row.
From the above conditions, there are three
possibilities

From statement II, clearly we cannot determine


the code for time and value.
Hence, Statement I is sufficient to answer the
question.

From statement II, clearly we cannot determine


4. Answer: D
how many persons are sitting in the row.
From statement I:
From statement I and II:
E sits third from one of the extreme ends of the
E sits third from one of the extreme ends of the
row. Only three persons sit between E and J. U
row. Only three persons sit between E and J. U
sits second to the left of J. Neither J nor U sits at
sits second to the left of J. Neither J nor U sits at
the extreme end of the row.
the extreme end of the row. Only four persons sit
From the above conditions, there are two
between U and T. F sits third to the right of T. A
possibilities
is an immediate neighbour of only F. Only one
person sits between U and J. Q sits sixth to the
left of J and sits at one of the extreme ends of
the row.
From the above conditions, there are two
possibilities

From statement I, clearly we cannot determine


how many persons are sitting in the row.
From statement II:
From both I and II, clearly we can determine 14
Only four persons sit between U and T. F sits
persons are sitting in the row.
third to the right of T. A is an immediate

Click Here For Bundle PDF Course | support@guidely.in Page 9 of 15


SBI Clerk & RRB PO Mains PDF Course 2023
Reasoning Ability Day - 12 (Eng)

Hence, both statement I and II is sufficient to


answer the question.

5. Answer: B
From statement I:
S lives three floors above Q, both are living in
different type of flats. No one lives to the west of
From statement II, clearly we can determine M
P, who lives on the adjacent floor of Q. The
lives immediately above the flat of O.
number of floors above P is one more than the
Hence, statement II is sufficient to answer the
number of floors below O.
question.
From the above conditions, there are two
possibilities
Directions (6-10):
6. Answer: B
7. Answer: E
8. Answer: C
9. Answer: A
10. Answer: D ((In the given option, the age of all
the persons is even, except in option d)
From statement I, clearly we cannot determine Final arrangement
who lives immediately above the flat of O.
From statement II:
N lives on an odd numbered floor and lives two
floors below M. N lives east of Q. Only two floors
We have,
are between Q and S, both are living in different
 G and the one whose age is 64 years live
type of flats. P lives two floors above O. R lives
in the same state.
above O in different type of flats. No one lives to
 Both C and E live in the same state.
the east of L.
 C neither lives with G nor lives in Manipur.
From the above conditions, we have
From the above conditions, there are four
possibilities

Click Here For Bundle PDF Course | support@guidely.in Page 10 of 15


SBI Clerk & RRB PO Mains PDF Course 2023
Reasoning Ability Day - 12 (Eng)

Again, we have
 B and the one whose age is 58 years live
in the same state but both of them lives
neither in Assam nor in Tripura.
Again, we have
 B’s age is 47 years, which is 12 years
 D doesn’t live with H.
more than E’s age.
 The age of I is 16 years more than C’s
 The sum of the ages of A and G is 74
age.
years and the age of A is less than 25
 C’s age is 5 years less than G.
years.
After applying the above conditions case-1 gets
 A lives with neither G nor B.
eliminated because the sum of the ages of A and
 A doesn’t live in Assam.
G is 74 if G’s age is 47 then A’s age is 27 which
 The sum of the ages of B and F is 65
is more than 25, hence case-3 shows the final
years.
arrangement.
 H lives with F but not with A and the age
of H is 7 years more than F.
 The youngest person doesn’t live with G.
In case-3 and case-4 there is no possibility to
place G’s age as 58 years because the sum of
ages of A and G is 74 if G’s age is 58 then A’s
age must be 16 years. But it is given that the age
of the youngest person is 18 years. 11. Answer: C
After applying the above conditions case-2 and Given series:
case -4 get eliminated because A doesn’t live in KHFMVLJACHIDBTRCULSE
Assam in case 2. And in case-4 less than two Required series:
persons live in the same state. Step I: O L J Q Z P N E G L M H F X V G Y P W
I
Step II: N L I R Y P M F F L L I E X U H X P V J

Click Here For Bundle PDF Course | support@guidely.in Page 11 of 15


SBI Clerk & RRB PO Mains PDF Course 2023
Reasoning Ability Day - 12 (Eng)

Step III: N B L I R Y B P M F F B L B L I E B X U Second letter from the right end is O


BHBXBPVJ Seventh letter from the right end is W
Step IV: M A K Q X A L E E K A K D A W A G A By using Q, U, O and W letters no meaningful
AOU word is formed.
All the letters in the final series which are in the
first half of the English alphabetical series are 13. Answer: B
dropped Given series:
Step IV: Q X W O U KHFMVLJACHIDBTRCULSE
Remaining letters are changed to its Required series:
corresponding complementary pair in the Step I: O L J Q Z P N E G L M H F X V G Y P W
alphabetical series I
Step IV: Q X W O U Step II: N L I R Y P M F F L L I E X U H X P V J
JCD LF Step III: N B L I R Y B P M F F B L B L I E B X U
All the letters are arranged in the alphabetical BHBXBPVJ
order from the left end Step IV: M A K Q X A L E E K A K D A W A G A
Step IV: J C D L F AOU
CDFJL First ten letters from step II are reversed
Fifth letter from the left end is L and then second Step II: L F F M P Y R I L N L I E X U H X P V J
to the left of L is F. The letters which are immediately followed by a
vowel are changed to the immediate preceding
12. Answer: E letter as per the English alphabetical series
Given series: Step II: L F F M P Y R I L N L I E X U H X P V J
KHFMVLJACHIDBTRCULSE Step II: L F F M P Y Q I L N K H E W U H X P V
Required series: J
Step I: O L J Q Z P N E G L M H F X V G Y P W All the repeated letters are dropped
I Step II: L F F M P Y Q I L N K H E W U H X P V
Step II: N L I R Y P M F F L L I E X U H X P V J J
Step III: N B L I R Y B P M F F B L B L I E B X U Step II: M Y Q I N K E W U X V J
BHBXBPVJ Q-17; X-24 ->17+24=41
Step IV: M A K Q X A L E E K A K D A W A G A
AOU 14. Answer: A
Fourth letter from the left end is Q Given series:
First letter from the right end is U KHFMVLJACHIDBTRCULSE

Click Here For Bundle PDF Course | support@guidely.in Page 12 of 15


SBI Clerk & RRB PO Mains PDF Course 2023
Reasoning Ability Day - 12 (Eng)

Required series: 16. Answer: E


Step I: O L J Q Z P N E G L M H F X V G Y P W 17. Answer: A
I 18. Answer: C
Step II: N L I R Y P M F F L L I E X U H X P V J 19. Answer: B (In the given options, each of
Step III: N B L I R Y B P M F F B L B L I E B X U them take the therapy twice in a week, except
BHBXBPVJ option b)
Step IV: M A K Q X A L E E K A K D A W A G A 20. Answer: D
AOU Final arrangement
All the B’s which are immediately preceded by a
consonant are interchanged with the immediate
next letter
Step III: N B L I R Y B P M F F B L B L I E B X U
BHBXBPVJ
Step III: N L B I R Y P B M F F L B L B I E B X U
BHXBPBVJ We have,
 Each of them takes the therapy session at
15. Answer: D least once but not more than twice in a
Given series: week.
KHFMVLJACHIDBTRCULSE  No one takes the therapy twice in a day.
Required series:  Each of them takes the therapy at
Step I: O L J Q Z P N E G L M H F X V G Y P W different timings on all three days.
I  There must be three persons or no
Step II: N L I R Y P M F F L L I E X U H X P V J persons taking the therapy in a day.
Step III: N B L I R Y B P M F F B L B L I E B X U
BHBXBPVJ
Step IV: M A K Q X A L E E K A K D A W A G A
AOU
All the vowels are dropped from step I
Step I: L J Q Z P N G L M H F X V G Y P W
8 letters are there to the left of M as well as to its
right  Two days gap between the days on which
G and J take the sessions, and both are

Directions (16-20):

Click Here For Bundle PDF Course | support@guidely.in Page 13 of 15


SBI Clerk & RRB PO Mains PDF Course 2023
Reasoning Ability Day - 12 (Eng)

taking the therapy session at different


timings.
 J does not take the therapy after 4.00 Pm.
Only two days between G and J and the only
day for J takes therapy is Thursday because if
we place on Wednesday and Friday we have to
place G on Saturday and Tuesday respectively
Case-4:
but in given table G takes therapy on Monday,
Friday and Sunday. If J takes therapy on
Thursday then G takes the therapy on Monday
and Sunday.
From the above conditions, there are four
possibilities.
Case-1:
Again, we have
 I takes the therapy only in the morning but
does not take adjacent timing with G.
Case-1: I must take the therapy on Monday and
Thursday as well I must take therapy in morning
time. On Monday, I should take therapy from
8.30 am-9.30 am because he does not take
Case-2:
therapy at adjacent timing with G. On Thursday,
I should take 10.30 am-11.30 am (since each of
them takes the therapy at different timings on all
three days).
F must take the therapy on Thursday and
Sunday. H must take the therapy Sunday,
Monday but we have the only option that H takes
Case-3: the therapy on Monday in a similar way we must
place F on Thursday. Each of them takes the
therapy at different timings on all three days, so
F should take the therapy on 8.30 am-9.30 am

Click Here For Bundle PDF Course | support@guidely.in Page 14 of 15


SBI Clerk & RRB PO Mains PDF Course 2023
Reasoning Ability Day - 12 (Eng)

on Sunday and in the remaining place we can fill not take adjacent timing with G). On Thursday
H. we have only option to place I at 8.30 am-9.30
am we have, each of them takes the therapy at
different timings on all three days, due to this it’s
get eliminated

Case-2: Possibly I take therapy Monday,


Tuesday, Thursday, but according to the below
diagram I must take the therapy on Monday and
Thursday as well I must take therapy in morning Case-4: Possibly I take therapy Monday,
time. On Monday I should take therapy from 8.30 Tuesday, Thursday, but according to the below
am-9.30 am but according to the point, I does diagram I must take therapy on Monday and
not take adjacent timing with G, it gets Thursday as well I must take therapy on morning
eliminated. time. On Monday I should take therapy on 10.30
am-11.30 am but according to the point, I does
not take adjacent timing with G, so its gets
eliminated.

Case-3: Possibly I take therapy Monday,


Tuesday, Thursday, but according to the below
diagram I must take therapy on Monday and
Thursday as well I must take therapy in morning
time. On Monday I should take therapy from 8.30
am-9.30 am (since according to the point, I does

Click Here For Bundle PDF Course | support@guidely.in Page 15 of 15


SBI Clerk & RRB PO Mains PDF Course 2023
Quantitative Aptitude Day - 12 (Eng)

Quantitative Aptitude

Direction (1-4): Study the following data carefully and answer the questions:
The data given below is related to the marks obtained by three students, P, Q, and R, in five different
subjects: A, B, C, D, and E. The maximum marks for each subject are different.
The bar graph given below shows the following data:

Note:
1: Only in subjects B and C, the marks obtained by Q is less than those obtained by R.
2: The maximum marks for subject A are 80% of subject B and 66(2/3) % of subject D, which is 480. The
maximum marks of subject C are 62.5% of the maximum marks of subject B, and the average of the
maximum marks of all five subjects is 350.
1)If R obtained M% marks in subject A and N% 2)If the average marks obtained by Q in the
marks in subject C, then find which of the subjects C, E, and F are ___, then the average
following statements is/are true? marks obtained by Q in the subjects A, B, D and
X: Ratio of M to N is 125: 176. F will be 253.25.
Y: Value of (2M – N) is 37. Find the value that can be filled in the blank?
Z: Average of M and N is 75.25. a) 161
a) All are true b) 151
b) Only Y and Z c)181
c)Only X and Z d) 171
d) Only X and Y e) 191
e) None is true

Click Here For Bundle PDF Course | support@guidely.in Page 1 of 16


SBI Clerk & RRB PO Mains PDF Course 2023
Quantitative Aptitude Day - 12 (Eng)

3)The total marks obtained by R in all five 4)If the average marks obtained by P, Q, R, and
subjects together are approximately what S in subject A are 225 and the average marks
percent more or less than those obtained by P in obtained by them in subject D are 245, then find
all five subjects together? that the total marks obtained by S in subjects A
a) 9% and D together are what percent of the maximum
b) 15% marks of both subjects together?
c)3% a) 55.5%
d) 6% b) 50.5%
e) 12% c) 45.5%
d) 60.5%
e) 40.5%

Direction (5-8): Study the following data carefully and answer the questions:
A survey is conducted on the someemployees of a company to find out their preferred methods of
communication, which are email, text message, social media, and direct mail.
The pie chart given below shows the percentage distribution of total number ofemployees, who preferred
the given methods of communication

The pie chart given below shows the percentage distribution of number ofmale employees who preferred
the given methods of communication.

Click Here For Bundle PDF Course | support@guidely.in Page 2 of 16


SBI Clerk & RRB PO Mains PDF Course 2023
Quantitative Aptitude Day - 12 (Eng)

Note:
1: The difference between the number of employees who like communication through text message and
direct mail is 24.
2: The number of employees who like communication through email is 18 more than those who like
communication through text message.
3: 75% of the total number of employees who like communication through text messages are male
employees.
5) Find the average number of male employees, degree distribution of the number of female
who like communication through email, those employees who like communication through
who like communication through text message, email?
and those who like communication through social a) 168°
media? b) 72°
a) 27 c)120°
b) 24 d) 192°
c)30 e) 96°
d) 21
e) 25 7)The total number of female employees who
like communication through social media and
6) If the number of female employees who like direct mail together is what percentage of the
communication through the given methods are total number of male employees who like
described in a circle, then what will be the

Click Here For Bundle PDF Course | support@guidely.in Page 3 of 16


SBI Clerk & RRB PO Mains PDF Course 2023
Quantitative Aptitude Day - 12 (Eng)

communication through social media and direct 10) Two series (I) and (II) are given below with
mail together? the wrong term P in series (I) and the missing
a) 62.5% term Q in series (II). Both the series follow the
b) 75% same pattern.
c)60% (I): 18, 42, 92, 173, 294, 463
d) 50% (II): 185, …, …, …, …,?
e) 66.67% Find the relation between P and Q?
a) (Q – P + 36)1/2 = 25
8) Find the percentage of number of female b) Both (a) and (c)
employees in the company out of total number of c)(Q – P – 12)1/2 = 24
employees in company? d) (P + Q + 4)1/2 = 26
a) 37.5% e) Both (c) and (d)
b) 25%
c) 30% 11) Two series (I) and (II) are given below with
d) 33.33% missing term P in series (I) and missing term Q
e) 40% in series (II). Both the series follow the same
pattern.
9) Two series (I) and (II) are given below with the (I): 22, 26, 32, 42, 56, P
wrong term P in series (I) and the wrong term Q (II): 7, 11, 17, 27, 41, Q
in series (II). Find that which of the following statement/s
(I): 3, 7, 15, 31, 60, 127 is/are true?
(II): 8, 26, 124, 360, 1330 A: P and Q are co-prime with each other?
Find that which of the following statements is/are B: Sum of P and Q is a prime number.
true? C: Difference of P and Q is divisible by 5.
A: Value of is 3. a) Only B
B: Ratio of P to Q is 1: 4. b) Only A and B
C: Difference of P and Q is multiple of 11. c) Only C
a) Only B d) Only B and C
b) Only A and B e) Only A
c)Only C
d) Only B and C 12) Two series (I) and (II) are given below with
e) Only A one wrong term in each series.
(I): 213, 184, 161, 142, 123, 112
(II): 20, 24, 32, 43, 56, 73

Click Here For Bundle PDF Course | support@guidely.in Page 4 of 16


SBI Clerk & RRB PO Mains PDF Course 2023
Quantitative Aptitude Day - 12 (Eng)

If P is the number, which should come in place of bought only item A and 5 more than those who
the wrong term of series (I) and Q is the number, bought both items A and C but not B. The total
which should come in place of the wrong term in number of customers in Mall P is 115.
series (II), then find that which of the following In Mall Q:
is/are true? The number of customers who bought both items
A: P1/3 = Q1/2 A and B but not C is 83(1/3) % of those who
B: (P + 19)1/2 = Q - 13 bought only item A, which is 20% more than
C: those who bought only item A in mall P. The

a) Only B number of customers who bought only item B is

b) Only A and B 30% of those who bought only item C, and the

c) Only C total number of customers who bought only one

d) Only B and C item is 95. The number of customers who bought

e) Only A all three items is equal to those who bought both


items A and B but not C and also 25% more than

Direction (13-16): Study the following data those who bought both items A and C but not B.

carefully and answer the questions: The total number of customers who bought only

Two malls, P and Q, sell three types of items: A, two items is 55.

B, and C. The data given below shows the 13) The total number of customers who bought

number of customers who bought these products item A from both malls together is approximately

from malls P and Q. Some bought only one what percentage of the total number of

product, some bought only two products, some customers in both malls together?

bought both items, and the remaining bought all a) 57%

three products. b) 49%

In Mall P: c)72%

The number of customers who bought only item d) 61%

A is 83 (1/3) % of those who bought only item B. e) 45%

The total number of customers who bought only


one item is 65. The number of customers who 14) Find the ratio of the total number of

bought both items B and C but not item A is customers who bought item C from mall P to

equal to those who bought only item C and also those who bought item C from mall Q?

double that of those who bought all three items. a) 20: 51

The number of customers who bought both items b) 8: 21

A and B but not C is 5 less than those who c)10: 31


d) 4: 9

Click Here For Bundle PDF Course | support@guidely.in Page 5 of 16


SBI Clerk & RRB PO Mains PDF Course 2023
Quantitative Aptitude Day - 12 (Eng)

e) 2: 3 Statement I: The ratio of B’s efficiency to D’s


efficiency is 4:15, and they together can
15) The total number of customers who bought complete the one-day work is 180/19 hours.
at most 2 items in mall P is what percentage Statement II: The ratio of A’s wages to B’s wages
more or less than those, who bought at least 2 is 3:1.
items in mall Q? Statement III: B’s wages are ₹ 1750 less than
a) 40% more C’s wages.
b) 30% less a) Statements I and II together are sufficient to
c)37.5% more answer the question.
d) 25% more b) Statements I and II together or statement III
e) 33.33% less alone is sufficient to answer the question.
c)Statements II and III together are sufficient to
16) If the ratio of male to female customers in answer the question.
mall P is 15:8 and the ratio of male to female d) Statement II alone is sufficient to answer the
customers in mall Q is 4:3, then find the question.
difference between total number of male e) All the statements together are not sufficient to
customers in both malls together and total answer the question.
number of female customers in both malls
together. 18) If train A crosses a 220 m long platform in 24
a) 55 seconds, then in what time train B will cross the
b) 50 same platform?
c) 45 Statement I: Train A can cross train B in
d) 60 seconds while running in opposite direction.
e) 40 Statement II: Speed of train A is 12 km/h less
than that of train B.
Direction (17-20): In each question, some Statement III: Ratio of speed of train A to that of
statements are given. You have to decide train B is 21: 25.
whether the data provided in the statements is a) Statements I alone is sufficient to answer the
sufficient to answer the question. question.
17) A, B, C, and D are hired to complete a work b) All the statements together are not sufficient to
and they are paid in proportion to their answer the question.
efficiencies. A, B, and C together can complete c)Statements II alone is sufficient to answer the
the work in 7.2 hours. In what time can B, C, and question.
D together complete the work?

Click Here For Bundle PDF Course | support@guidely.in Page 6 of 16


SBI Clerk & RRB PO Mains PDF Course 2023
Quantitative Aptitude Day - 12 (Eng)

d) All the statements together are necessary to e) Statement II alone is sufficient to answer the
answer the question question.
e) Statement III alone is sufficient to answer the
question. 20) Article A is marked up by ₹ 600 and sold at a
discount of 10% on its marked price. Find the
19) The difference between compound interest profit percent on the cost price of article A.
and simple interest on ₹ X at a 5R% rate after 2 Statement I: If article B is marked up by 25% and
years is ₹ 405. Find the value of R. sold at a 12% discount on its marked price, then
Statement I: The ratio of simple interest received there will be a total profit of ₹ 460 on both
on ₹ (X – 3000) at a 6R% rate in after 1 year to articles together.
that received on ₹ (X + 2000) at a 4R% rate Statement II: Marked price of both articles
after 1 year is 9: 8. together is ₹ 5000.
Statement II: The ratio of the difference between Statement III: If article B is marked up by 50%
compound interest and simple interest received and sold at ₹ 400 discount on its marked price,
on ₹ 2X at a 10% rate after 2 years and that on there will be a 25% profit on its cost price.
₹ (2X/3) at a 15% rate after 2 years is 4: 3. a) All the statements together are not sufficient to
Statement III: The difference between 15% of X answer the question.
and 50% of X is 6300. b) Statements I and II together or statements I
a) All the statements together are necessary to and III together are sufficient to answer the
answer the question. question.
b) Statements I and II together are sufficient to c)Statements I and II together are sufficient to
answer the question. answer the question.
c)Statements I alone is sufficient to answer the d) All the statements together are necessary to
question. answer the question.
d) All the statements together are not sufficient to e) Statements I and III together are sufficient to
answer the question. answer the question.

Click Here For Bundle PDF Course | support@guidely.in Page 7 of 16


SBI Clerk & RRB PO Mains PDF Course 2023
Quantitative Aptitude Day - 12 (Eng)

Click Here to Get the Detailed Video Solution for the above given Questions
Or Scan the QR Code to Get the Detailed Video Solutions

Answer Key with Explanation

Direction (1-4): From Y:


Maximum marks of subject D = 480 Value of (2M – N) = 2 * 62.5 – 88 = 37
Maximum marks of subject A = = 320 So, Y is true.
Maximum marks of subject B = = 400 From Z:

Maximum marks of subject C = 62.5% of 400 = Average of M and N = = 75.25

250 So, Z is true.


Maximum marks of subject E = (5 * 350) – (480 Hence, all X, Y and Z are true.
+ 320 + 400 + 250) = 300
2) Answer: D
Since, the average marks obtained by Q in the
subjects A, B, D and F will be = 253.25.
So, the marks obtained by Q in subject F = (4 *
253.25) – 300 – 270 – 288 = 155
And the value that can be filled in the blank =
1) Answer: A = 171
Marks obtained by R in subject A = M =
= 62.5%
3) Answer: C
Marks obtained by R in subject C = N = Total marks obtained by P in all 5 subjects
= 88% together:
From X: 240 + 180 + 200 + 240 + 210 = 1070
Ratio of M to N = 62.5: 88 = 625: 880 = 125: 176 Total marks obtained by R in all 5 subjects
So, X is true. together:

Click Here For Bundle PDF Course | support@guidely.in Page 8 of 16


SBI Clerk & RRB PO Mains PDF Course 2023
Quantitative Aptitude Day - 12 (Eng)

200 + 340 + 220 + 208 + 138 = 1106 So, the number of employees, who like
Required percentage = = 3% communication through social media:
(approx.)

Number of male employees, who like


4) Answer: B communication through text message = 75% of
Since, the average marks obtained by P, Q, R, 36 = 27
and S in subject A = 225 Number of male employees, who like
So, the marks obtained by S in subject A = (4 * communication through email:
225) – 240 – 300 – 200 = 160
Since, the average marks obtained by P, Q, R,
Number of male employees, who like
and S in subject D = 245
communication through social media:
So, the marks obtained by S in subject D = (4 *
245) – 240 – 288 – 208 = 244
Total marks obtained by S in subjects A and D Number of male employees, who like
together = 160 + 244 = 404 communication through direct mail:
Total maximum marks of subjects A and D
together = 320 + 480 = 800
Required percentage = = 50.5%

Direction (5-8):
Since, the difference between the number of
employees, who like communication through text
message and direct mail, is 24.
5) Answer: B
So, the number of employees, who like
Number of male employees, who like
communication through text message:
communication through email = 30
Number of male employees, who like
The number of employees, who like communication through text message = 27
communication through direct mail: Number of male employees, who like
36 – 24 = 12 communication through social media = 15
The number of employees, who like Required average = = 24
communication through email = 36 + 18 = 54
So, the value of X = = 45 6) Answer: D
And the value of Y = 100 – 45 – 30 – 10 = 15

Click Here For Bundle PDF Course | support@guidely.in Page 9 of 16


SBI Clerk & RRB PO Mains PDF Course 2023
Quantitative Aptitude Day - 12 (Eng)

Total number of female employees = 24 + 9 + 3 33 – 1 = 26


+ 9 = 45 53 – 1 = 124
Number of female employees, who like 73 – 1 = 342(360)
communication through email = 24 113 – 1 = 1330
So, required degree distribution = = So, the wrong term in series (II) = Q = 360
192° From A:
Value of =
7) Answer: E So, A is true.
Total number of male employees, who like From B:
communication through social media and direct Ratio of P to Q = 60: 360 = 1: 6
mail together = 15 + 3 = 18 So, B is not true.
Total number of female employees, who like From C:
communication through social media and direct Difference between P and Q = 360 – 60 = 300.
mail together = 3 + 9 = 12 Since, 300 is not a multiple of 11.
Required percentage = = 66.67% So, C is not true.
Hence, only A is true.
8) Answer: A
Total number of employees in the company = 54 10) Answer: E
+ 36 + 18 + 12 = 120 Logic in series (I):
Number of female employees in the company = 18 + 52 = 43(42)
24 + 9 + 3 + 9 = 45 43 + 72 = 92
Required percentage = = 37.5% 92 + 92 = 173
173 + 112 = 294

9) Answer: E 294 + 132 = 463

Logic in series (I): So, the wrong term in series (I) = P = 42

3*2+1=7 Now, series (II):

7 * 2 + 1 = 15 185 + 52 = 210

15 * 2 + 1 = 31 210 + 72 = 259

31 * 2 + 1 = 63(60) 259 + 92 = 340

63 * 2 + 1 = 127 340 + 112 = 461

So, the wrong term in series (I) = P = 60 461 + 132 = 630

Logic in series (II): So, the missing term in series (II) = Q = 630

23 – 1 = 7

Click Here For Bundle PDF Course | support@guidely.in Page 10 of 16


SBI Clerk & RRB PO Mains PDF Course 2023
Quantitative Aptitude Day - 12 (Eng)

The relation between P and Q = (Q – P – 12)1/2 = Difference of P and Q = 78 – 63 = 15


24 Since, 15 is divisible by 5.
And also, relation between P and Q = (P + Q + So, C is true.
4)1/2 = 26 Hence, only C is true.
But not = (Q – P + 36)1/2 = 25
12) Answer: B
11) Answer: C Logic in series (I):
Series (I): 213 – 29 = 184
22 + 2 * 2 = 26 184 – 23 = 161
26 + 2 * 3 = 32 161 – 19 = 142
32 + 2 * 5 = 42 142 – 17 = 125(123)
42 + 2 * 7 = 56 125 – 13 = 112
56 + 2 * 11 = 78 So, the value of P = 125
So, the missing term in series (I) = P = 78 Logic in series (II):
Series (II): 20 + 5 = 25(24)
7 + 2 * 2 = 11 25 + 7 = 32
11 + 2 * 3 = 17 32 + 11 = 43
17 + 2 * 5 = 27 43 + 13 = 56
27 + 2 * 7 = 41 56 + 17 = 73
41 + 2 * 11 = 63 So, the value of Q = 25
So, the missing term in series (II) = Q = 63 From A:
From A: P1/3 = Q1/2 => 1251/3 = 251/2 => 5 = 5
Factors of P = 78 = 2 * 3 * 13 So, A is true.
Factors of Q = 63 = 3 * 3 * 7 From B:
Since, P and Q has a common factor, so P and (P + 19)1/2 = Q – 13 =>(125 + 19)1/2 = 25 – 13
Q are not co-prime with each other. =>12 = 12
So, A is not true. So, B is true.
From B: From C:
Sum of P and Q = 78 + 63 = 141
Since, 141 is divisible by 3 and 47, so the sum of
So, C is not true.
P and Q is not a prime number.
Hence, only A and B are true.
So, B is not true.
From C:
Direction (13-16):

Click Here For Bundle PDF Course | support@guidely.in Page 11 of 16


SBI Clerk & RRB PO Mains PDF Course 2023
Quantitative Aptitude Day - 12 (Eng)

In mall P: The number of customers who bought only item


Let the number of customers who bought only A = 120% of 25 = 30
item B = 6x The number of customers who bought both
So, the number of customers who bought only items A and B but not C = 83(1/3) % of 30 = 25
item A = 83(1/3) % of 6x = 5x Let the number of customers who bought only
Let the number of customers who bought only item C = 10x
item C = y So, the number of customers who bought only
So, item B = 30% of 10x = 3x
6x + 5x + y = 65 So,
11x + y = 65 -------------(1) 10x + 3x = 95 – 30
Now, the number of customers who bought both x=5
items B and C but not A = y The number of customers who bought all three
The number of customers who bought all three items = 25
items = (y/2) The number of customers who bought both
The number of customers who bought both items A and C but not B = = 20
items A and B but not C = (5x – 5) The number of customers who bought both
The number of customers who bought both items B and C but not A = 55 – (25 + 20) = 10
items A and C but not B = (5x – 5) – 5 = (5x – 10)
Since, the total number of customers = 115
So,

3y + 20x = 130 ------------(2)


By equation (1) * 3 – equation (2):
33x + 3y – 3y – 20x = 195 – 130
x = 5, y = 10 Total number of customers in mall Q = 30 + 25 +
15 + 20 + 25 + 10 + 50 = 175
13) Answer: A
Total no. of customers who bought item A from
mall P = 25 + 20 + 15 + 5 = 65
Total no. of customers who bought item A from
mall Q = 30 + 25 + 25 + 20 = 100
Total number of customers in mall P = 115
In mall Q: Total number of customers in mall Q = 175

Click Here For Bundle PDF Course | support@guidely.in Page 12 of 16


SBI Clerk & RRB PO Mains PDF Course 2023
Quantitative Aptitude Day - 12 (Eng)

Required percentage = = 57% So, number of male customers in mall Q =


(approx.) = 100
And the number of female customers in mall Q =
14) Answer: B 175 – 100 = 75
Total number of customers who bought item C Required difference = (75 + 100) – (40 + 75) =
from mall P = 15 + 5 + 10 + 10 = 40 60
Total number of customers who bought item C
from mall Q = 20 + 25 + 10 + 50 = 105 17) Answer: A
Required ratio = 40: 105 = 8: 21 Since, A, B and C together can complete the
work in 7.2 hours.
15) Answer: C So,
The total number of customers who bought at- ------------(1)
most 2 items in mall P = Total number of From statements I and II together:
customers in mall P – The number of customers Let the time taken by B alone and D alone to
who bought all the 3 items in mall P complete the work is 15t hours and 4t hours.
Total number of customers who bought at-most So,
2 items in mall P = 115 – 5 = 110
Total number of customers who bought at-least
t=3
2 items in mall Q = Total number of customers in
Time taken by B alone to complete the work = 45
mall Q – The number of customers who bought
hours
only one item
Time taken by D alone to complete the work =
Total number of customers who bought at-least
12 hours
2 items in mall Q = 175 – (30 + 15 + 50) = 80
Since, ratio of A’s wages to B’s wages = 3: 1.
Required percentage = = 37.5%
So, time taken by A alone to complete the work
more
= = 15 hours
From equation (1):
16) Answer: D
Total number of customers in mall P = 115
So, number of male customers in mall P = Time taken by C alone to complete the work =

= 75 20 hours

And the number of female customers in mall P = Work done by B, C and D together in 1 hour =

115 – 75 = 40
Total number of customers in mall Q = 175

Click Here For Bundle PDF Course | support@guidely.in Page 13 of 16


SBI Clerk & RRB PO Mains PDF Course 2023
Quantitative Aptitude Day - 12 (Eng)

So, time taken by B, C and D together to ---------------(2)


complete the work = hours Since, we can’t solve the equation.
Statements I and II together are sufficient to So, statement I alone is not sufficient to answer
answer the question. the question.
From statement III alone: From statement II alone:
B’s wages is ₹ 1750 less than C’s wages. Speed of train B =
Since, we can’t determine the time taken by B Since, the data is insufficient.
alone, C alone and D alone to complete the So, statement II alone is not sufficient to answer
work. the question.
So, statement III alone is not sufficient to answer From statement III alone:
the question. Let the speeds of trains A and B are ‘21x’ m/s
From statement II and III together: and ‘25x’ m/s respectively.
The ratio of A’s wages to B’s wages is 3:1. Since, the data is insufficient.
B’s wages are ₹ 1750 less than C’s wages. So, statement III alone is not sufficient to answer
Since, we can’t determine the time taken by B the question.
alone, C alone and D alone to complete the From statements I, II and III together:
work. Let the speeds of trains A and B are ‘21x’ m/s
So, statements II and III together are not and ‘25x’ m/s respectively.
sufficient to answer the question. So,
Hence, Statements I and II together are
sufficient to answer the question.
x=
Speed of train A =
18) Answer: D
Speed of train B =
Let the speed of train A is ‘x’ m/s and the length
of train A is ‘d’ m. From equation (1):

So,
--------------(1) d = 200 m
From statement I alone: From equation (2):
Let the speed of train B is ‘y’ m/s and the length
of train B is ‘s’ m.
s = 280
Since, train A can cross train B in seconds
The time, in which train B will cross the same
while running in opposite direction.
platform:
So,

Click Here For Bundle PDF Course | support@guidely.in Page 14 of 16


SBI Clerk & RRB PO Mains PDF Course 2023
Quantitative Aptitude Day - 12 (Eng)

The MP of article B = ₹ 125y


And the SP of article B = 88% of 125y = ₹ 110y
Hence, all the statements together are
So,
necessary to answer the question.
(90x + 540 + 110y) = 100x + 100y + 460
x – y = 8 ------------(1)
19) Answer: C
And,
--------------(1)
100x + 600 + 125y = 5000
From statement I alone:
100x + 125y = 4400
4x + 5y = 176 ------------(2)
X = 18000 After solving equations (1) and (2):
From equation (1): x = 24, y = 16
Now the CP of article A = ₹ 2400
And the SP of article A = ₹ 2700
R=3
Required profit percentage = =
So, statement I alone is sufficient to answer the
12.5%
question.
So, statements I and II together are sufficient to
From statement II alone:
answer the question.
From statements I and III together:
The MP of article B = ₹ 125y
There are two variables and from this equation
And the SP of article B = 88% of 125y = ₹ 110y
we can’t find the value of X and without X we
So,
can’t find the value of R.
(90x + 540 + 110y) = 100x + 100y + 460
So, statement II alone is not sufficient to answer
x – y = 8 ------------(1)
the question.
Now MP of article B = ₹ 150y
Hence, statement I alone is sufficient to answer
New SP of article B = ₹ (150y – 400)
the question.
So,
150y – 400 = 125y
20) Answer: B
y = 16
Let the CP of article A = ₹ 100x
So, x = 24
So, the MP of article A = ₹ (100x + 600)
Now the CP of article A = ₹ 2400
And the SP of article A = 90% of (100x + 600) =
And the SP of article A = ₹ 2700
₹ (90x + 540)
Required profit percentage = =
Also let the CP of article B = ₹ 100y
12.5%
From statements I and II together:

Click Here For Bundle PDF Course | support@guidely.in Page 15 of 16


SBI Clerk & RRB PO Mains PDF Course 2023
Quantitative Aptitude Day - 12 (Eng)

So, statements I and III together are sufficient to Hence, statements I and II together or
answer the question. statements I and III together are sufficient to
answer the question.

Click Here For Bundle PDF Course | support@guidely.in Page 16 of 16


SBI Clerk & RRB PO Mains PDF Course 2023
ENGLISH Day - 12

English Language

Directions (1-5): Given below are a few 2. Which of the following option fits best in the
questions based on the paragraph given with a second blank?
few blanks to be filled from the appropriate A. convinced
options in order to make a meaningful sentence. B. perceived
If none of the options are correct then choose C. conceived
option E as your answer. D. treated
The Good Book is being (a)_____________ like a E. None of the above
bad book in Utah after a parent frustrated by
efforts to ban materials from schools 3. Which of the following option fits best in the
(b)___________ a suburban district that some third blank?
Bible verses were too vulgar or violent for A. secondary
younger children. And the Book of Mormon could B. senior secondary
be next. C. elementary
The 72,000-student Davis School District north of D. college
Salt Lake City removed the Bible from its E. None of the above
(c)___________ and middle schools while keeping
it in high schools after a committee reviewed the 4. Which of the following option fits best in the
(d)_____________ in response to a parental fourth blank?
complaint. The district has removed other titles, A. scripture
including Sherman Alexie’s “The Absolutely True B. sculpture
Diary of a Part-Time Indian” and John Green’s C. caricature
“Looking for Alaska,” following a 2022 state law D. despotic
requiring districts to include parents in decisions E. None of the above
over what (e)______________ “sensitive material.”
1. Which of the following option fits best in the 5. Which of the following option fits best in the
first blank? fifth blank?
A. fractioned A. meddled
B. termed B. constitutes
C. pasted C. dismantle
D. treated D. desecrate
E. None of the above E. None of the above

Click Here For Bundle PDF Course | support@guidely.in Page 1 of 7


SBI Clerk & RRB PO Mains PDF Course 2023
ENGLISH Day - 12

Directions (6-10): Given below are a few White House nomination (e)/, papers filed with
questions with jumbled parts in order to the US Federal Election Commission (f).
rearrange them and make a contextually correct A. abcdf
sentence. One of the part is fixed and rest to be B. acdfb
rearranged if needed. If no rearrangement is C. bcdfa
required then choose option E as your answer. D. fadcb
6. The ties between India and Nepal (a)/ came E. No rearrangement is required
under severe strain after Kathmandu published
(b)/ a new political map in 2020 (c)/ Indian 9. Years of financial mismanagement (a)/ have
territories Limpiyadhura (d) that showed three pushed Pakistan's economy to the limit (b)/,
(e), Kalapani and Lipulekh as part of Nepal (f). exacerbated by a global energy crisis (c)/ that
A. abced submerged a third of the country (d)/ and
B. acdeb devastating floods (e)/ in 2022 (f).
C. bcdae A. abced
D. bedca B. acdeb
E. No rearrangement is required C. adebc
D. acedb
7. Under international law (a)/, ships are allowed E. No rearrangement is required
to sail through foreign EEZs (b)/, but
unauthorised surveys are not permitted and 10. Pakistan needs billions of dollars (a)/ in
China's operations (c)/ in the South China Sea financing to service (b)/ staggering levels of
have long been problematic (d)/ for countries in external debt (C)/, and foreign exchange
the region, as Beijing claims most of the energy- reserves (d)/ barely enough for a month of
rich sea (e)/, including foreign EEZs (f). imports (e)/ have dwindled to just $4.2 billion (f).
A. bcedf A. acdfe
B. cdefb B. adfec
C. bdcef C. cedfa
D. edcab D. cdeaf
E. No rearrangement is required E. No rearrangement is required

8. showed Monday (a)/ Republican former vice Directions (11-15): Given below are a few
president Mike Pence (b)/ has launched his questions with three highlighted words in each of
hotly-anticipated challenge (c)/ to his one-time them. You have to find the synonyms of the
boss Donald Trump (d)/ for the party's 2024 words highlighted in the same sequence as they

Click Here For Bundle PDF Course | support@guidely.in Page 2 of 7


SBI Clerk & RRB PO Mains PDF Course 2023
ENGLISH Day - 12

are given in the question. If none of the options A. accord, compeer, bodacious
are correct then choose option E as your answer. B. compeer, accord, bodacious
11. If we mark spots around the globe where the C. compeer, bodacious, accord
militaries of the world’s superpowers could run D. bodacious, accord, compeer
into each other, both the South China Sea and E. None of the above
the Black Sea would top the chart. Earlier it was
rare that the world's most powerful nations would 14. In response to Russia's invasion of Ukraine,
engage in assertive manoeuvring over Western powers imposed a price cap on Russian
international waters, but now that is a common crude of $60 a barrel.
sight. While non-EU countries can import seaborne
A. involve, conspire, enterprising Russian crude, Western shipowners and insurers
B. enterprising, conspire, involve are prohibited from handling such cargoes
C. conspire, involve, enterprising unless they are sold at or below that price.
D. involve, enterprising, conspire A. count, levy, interdicted
E. None of the above B. interdicted, levy, count
C. levy, count, interdicted
12. While US-China tensions may have been D. interdicted, count, levy
given new impetus under the Trump E. None of the above
administration which levied tariffs broadly and
imposed sanctions on Huawei, the friction has 15. The aggressive push by Reliance to increase
continued unabated under President Joe Biden popular content on its streaming platform, in
as both countries vie for global tech pre- addition to premium sports content, could lead to
eminence. a massive disruption in the OTT streaming
A. goad, incessant, paramountcy space, which has been dominated mostly by
B. goad, paramountcy, incessant foreign players.
C. paramountcy, incessant, goad A. overpower, fierce, disrangement
D. incessant, goad, paramountcy B. overpower, disrangement, fierce
E. None of the above C. fierce, overpower, disrangement
D. fierce, disrangement, overpower
13. Reliance’s JioCinema has signed another E. None of the above
major streaming deal to take on rivals Netflix,
Disney+ Hotstar and Amazon in India after Directions (16-20): Given below are a few
striking a partnership with Warner Bros questions with blank in each of them in the last to
Discovery in April. be filled with an appropriate option in order to

Click Here For Bundle PDF Course | support@guidely.in Page 3 of 7


SBI Clerk & RRB PO Mains PDF Course 2023
ENGLISH Day - 12

make the sentence meaningfully correct. If none India, fintech incubator Rainmatter, and
of the options are correct then choose option E Rainmatter Foundation,
as your answer. _________________________.
16. The BRICS brings together five of the largest A. which is still the smallest curated thing.
developing countries of the world, representing B. which is known for its sports adventure.
41 per cent of the global population, C. which supports climate-related non-profits
____________________________. D. which is known for its mouth-watering taste.
A. pertaining to the risk involved they just denied E. None of the above
doing that.
B. the following problem is the main cause of the 19. Musk's net worth dropped below $200 billion
rift. last year as investors dumped Tesla's shares on
C. also the BRICS is the one of the toughest worries the top executive and largest
ruled organisations. shareholder of the world's most valuable electric-
D. 24 per cent of the global GDP, and 16 per vehicle maker is more preoccupied with Twitter,
cent of the global trade. ____________________.
E. None of the above A. which he dumped at some billion dollars.
B. which he bought for $44 billion.
17. Zerodha co-founder Nikhil Kamath became C. which he took as a loan from Jake Dorsey.
the youngest and fourth Indian to join 'The Giving D. which is one of the most important factor in
Pledge', where the wealthiest families and deciding the future of the company.
individuals _________________________________. E. None of the above
A. doesn’t provide information to the needy and
use the cyber data to manipulate election of the 20. While the RBI acknowledged that the
developed nations. domestic economy faces some challenges due
B. doesn’t provide shelter to the needy. to the sluggish global economic outlook, it said
C. is country’s largest broking company and still strong macroeconomic factors, financial
bootstrapped. conditions and expected dividends
D. globally commit to giving the majority of their from______________________.
fortune to charitable causes. A. past reforms put the country in an
E. None of the above advantageous position
B. that the repo and reverse repo rates are rising
18. Kamath founded Zerodha in 2010, as well as from the last five monetary policy meetings.
Gruhas for private investments, hedge fund True C. that the last meeting was not fruitful due to
Beacon that manages wealth for ultra HNIs in less convincing environment.

Click Here For Bundle PDF Course | support@guidely.in Page 4 of 7


SBI Clerk & RRB PO Mains PDF Course 2023
ENGLISH Day - 12

D. future prediction of inflation cannot be done as E. None of the above


situation is changing everyday.

Click Here to Get the Detailed Video Solution for the above given Questions
Or Scan the QR Code to Get the Detailed Video Solutions

Answer Key with Explanation

1. Answer: D
Here, treated fits best in the first blank as per the 4. Answer: A
context of the sentence because here book is Here, scripture fits best in the fourth blanks as it
being treated as bad and therefore it fits best. means the sacred writings of Christianity
So, option D is the correct answer. contained in the Bible and here the same is
being talked about. So, option A is the correct
2. Answer: A answer.
Here, convinced fits best in the second blank as Sculpture means a statue made out of hands.
it means to be completely sure about something Caricature means a picture or description of
and here it fits perfectly as per the context of the somebody that makes his or her appearance or
sentence. So, option A is the correct answer. behaviour funnier and more extreme than it
Perceived means to notice or realize something. really is.
Conceived means create by fertilizing an egg.
5. Answer: B
3. Answer: C Here, constitutes fits best in the fifth blank as it
Here, elementary fits best as it is said in the means to contain and here it fits best as the
context of a type of school and elementary content of book is being talked with respect to
schools are the first school for a child. So, option the sensitivity or sensitive content. So, option B
C is the correct answer. is the correct answer.

Click Here For Bundle PDF Course | support@guidely.in Page 5 of 7


SBI Clerk & RRB PO Mains PDF Course 2023
ENGLISH Day - 12

Desecrate means to damage a place of religious followed by d and finally ends with f which is
importance or treat it without respect. fixed. So, option A is the correct answer.

6. Answer: A 10. Answer: A


Here, the first part after rearrangement will be a Here, the first part after rearrangement will be a
which is introductory part starting with the ties which gives introductory part as need of billions
between India and Nepal followed by b which of dollars for Pakistan followed by b which is
gives the reason for the stress mentioned fixed followed by c which gives reason for need
followed by c and then d and finally ends with e of money followed by d then f and finally ends
and f in which f is already fixed. So, option A is with e which shows how much amount is left with
the correct answer. them. So, option A is the correct answer.

7. Answer: E 11. Answer: D


Here, no rearrangement is required and Here, engage means to provide occupation for
therefore, option E is the correct answer. and here involve is the synonym, assertive
means disposed to or characterized by bold or
8. Answer: C confident statements and behaviour and here
Here, the first part after rearrangement will be b enterprising means same, manoeuvring means
as it gives introductory part as former vice to move to a different position using skill and
president followed by c which gives information here conspire means the same. So, option D is
about a challenge launched by him followed by d the correct answer.
and then e which is fix followed by f which gives
information about details submitted by him and 12. Answer: C
finally ends with a which is ending part. So, Here, impetus means a driving force and here
option C is the correct answer. goad is the synonym, unabated means being at
full strength or force and here incessant means
9. Answer: A the same, eminence means a position of
Here, the first part after rearrangement will be a prominence or superiority and here paramountcy
which is introductory part as it gives information means the same. So, option C is the correct
about some mismanagement from years answer.
followed by b which gives information about
whom it is referring followed by c then e which 13. Answer: A
gives information about the floods and then

Click Here For Bundle PDF Course | support@guidely.in Page 6 of 7


SBI Clerk & RRB PO Mains PDF Course 2023
ENGLISH Day - 12

Here, deal means to take action with regard to 16. Answer: D


someone or something and here accord, rivals Here, the context of the sentence is about
means one of two or more striving to reach or BRICS and its importance and here option D
obtain something that only one can possess, which is giving information about the contribution
striking means attracting attention or notice of BRICS in global economy is the correct
through unusual or conspicuous qualities and option. So, option D is the correct answer.
here bodacious means the same. So, option A is
the correct answer. 17. Answer: D
Here, the context of the sentence is about the
14. Answer: E giving pledge and option D explains the use of
Here, imposed means to establish or apply by fund donated under this pledge. So, option D is
authority and here levy means the same, import the correct answer.
means to bring from a foreign or external source
and here count is somewhat near to the meaning 18. Answer: C
but not exact synonym as per the context of the Here, the blank should be filled with the
sentence, prohibited means not permitted or information about rainmatter foundation and
forbidden by the authority and here interdicted option C gives the information correctly as per
means the same. So, option E is the correct the context. So, option C is the correct answer.
answer.
19. Answer: B
15. Answer: D Here, the context of the sentence is about twitter
Here, aggressive means marked by combative and in the end as per the context it should end
readiness or tending toward or exhibiting and with the price at which it is bought and option B
here fierce means the same, disruption means a explains it correctly. So, option B is the correct
break or interruption in the normal course or answer.
continuation of some activity and here
derangement means the same, dominated 20. Answer: A
means to exert the supreme determining or Here, the context of the sentence is about
guiding influence on something and here domestic economic situations as per the RBI and
overpower means the same. So, option D is the here option A is contextually fitting in the blank.
correct answer. So, option A is the correct answer.

Click Here For Bundle PDF Course | support@guidely.in Page 7 of 7


SBI Clerk & RRB PO Mains PDF Course 2023
Reasoning Ability Day - 13 (Eng)

Reasoning Ability

Direction (1-5): Study the following information floor is between K and P, who has a square
carefully and answer the given questions number of shoes. No one lives to the east of K,
Eight persons - I, J, K, L, M, N, O and P are living who has 4 shoes less than L. The difference
on four different floors of a four storey building between the number of shoes with I and J is 3. J
where the lowermost floor is numbered one and has the number of shoes which is reverse of the
the floor immediately above it is numbered two number of shoes with M. The difference between
and so on. the number of shoes with O and K is 15. No one
Note-I: Each floor has two type of flats viz., Flat- lives to the east of the one who has the number
A and Flat-B, where Flat A is to the west of Flat of shoes which is a multiple of 5.
B. 1. What is the sum of the number of shoes with
Note-II: Flat B of floor 2 is immediately above L, N and P?
Flat B of floor 1 and immediately below Flat B of a) 110
floor 3 and so on. Similarly, Flat A of Floor 2 is b) 96
immediately above Flat A of floor 1 and c) 133
immediately below Flat A of floor 3 and so on. d) 109
Note-III: Area of each flat on each floor is equal. e) 113
Note-IV: Only two persons live on each floor and
only one person lives in each flat. 2. Which of the following statements is/are not
Note V: Each person has different number of false as per the given arrangement?
shoes with them, which is a two digit number. No a) The difference between the number of shoes
one has more than 50 shoes. with J and O is a prime number
The one who has 27 shoes lives two floors above b) The one who lives immediately above the flat
the flat of N, who lives on an odd numbered floor, of I has an odd number of shoes
has 8 shoes less than L. The number of floors c) M has 20 shoes less than K
below N is one less than the number of floors d) All the given statement is true
above I, who has the number of shoes which is a e) All the given statement is false
multiple of seven. Only one floor is between I and
the one who has 17 shoes less than P, and both 3. Who among the following person lives two
are living in the same type of flat. M, who has a floors above J?
prime number of shoes, lives two floors above a) The one who lives immediately below the flat
the flat of L. Both M and P neither live on the of M
same floor nor in the same type of flat. Only one b) P

Click Here For Bundle PDF Course | support@guidely.in Page 1 of 10


SBI Clerk & RRB PO Mains PDF Course 2023
Reasoning Ability Day - 13 (Eng)

c) I Q@W$E%R&T; U&E@I%O$P; H#O@G%L#A


d) The one who has 46 shoes Conclusions:
e) Both a and c I. H$T
II. R@L
4. Which of the following combination is true? III. Q$P
a) Floor number 4, Flat B - M a) Only conclusion I is true
b) Floor number 2, Flat A - P b) Only conclusion II is true
c) Floor number 3, Flat A - I c) Both conclusions I and II are true
d) Floor number 1, Flat B - J d) Only conclusion III is true
e) Floor number 4, Flat A - K e) Both conclusions II and III are true

5. If the rate of each shoe bought by the person 7. Statements:


who lives in flat A is Rs.1000 and Flat B is A@B&C%D$E; G%F$D#H$I; L$H&J&K%M
Rs.500, then who among the following person Conclusions:
spent the highest amount to buy shoes? I. K@B
a) Both N and K II. L$G
b) O III. E#M
c) Both P and M a) Only conclusion I is true
d) L b) Only conclusion II is true
e) Both J and I c) Both conclusions I and II are true
d) Only conclusion III is true
Directions (6-10): In the given questions, the e) None is true
relationship between different elements is shown
in the statements followed by some conclusions. 8. Statements:
Find the conclusion which is definitely true. K@L%J&H#G; A&S%Y$D@F; E#L$T%Y@O
A#B means B is neither greater than nor equal to Conclusions:
A I. F&K
A$B means B is neither smaller than nor equal to II. H$A
A III. J#O
A%B means B is neither greater than nor less a) Only conclusion I is true
than A b) Only conclusion II is true
A&B means B is not greater than A c) Both conclusions I and II are true
A@B means B is not smaller than A d) Only conclusion III is true
6. Statements: e) Both conclusions II and III are true

Click Here For Bundle PDF Course | support@guidely.in Page 2 of 10


SBI Clerk & RRB PO Mains PDF Course 2023
Reasoning Ability Day - 13 (Eng)

Row 1 contains numbers which is a consecutive


9. Statements: multiple of 17 from left to right starting from 34.
Z$X%K&O#V; S&H%K$L@C; A#S@D#F$G Row 2 contains numbers which is a consecutive
Conclusions: multiple of 19 from left to right starting from 57
I. X$G Row 3 contains numbers which is a consecutive
II. A#V even numbers from left to right starting from 28
III. K&G Row 4 contains numbers which is a consecutive
a) Only conclusion I is true multiple of 13 from left to right starting from 13
b) Only conclusion II is true Row 5 contains numbers which is a consecutive
c) Both conclusions I and II are true prime numbers from left to right starting from 41
d) Either conclusion I or III is true Conditions:
e) Both b and d are true I. If the outcome is less than 500, we get Rs.90
as cashback
10. Statements: II. If the outcome is between 501 and 1300, we
P@L@M%N&B; S@W#A%E@R; A%V$N#C$X get Rs.200 as cashback
Conclusions: III. If the outcome is between 1301 and 2200, we
I. P@R get Rs.450 as cashback
II. W#C IV. If the outcome is between 2201 and 3500, we
III. E$X get Rs.700 as cashback
a) Only conclusion I is true V. If the outcome is between 3501 and 5000, we
b) Both conclusions I and II are true get Rs.1500 as cashback
c) None is true Note: The amount of money we receive will differ
d) Only conclusion II is true depending on the outcome we receive from the
e) Both conclusions I and III are true string
For the outcome of the string:
Direction (11-15): Study the following information A. If an odd number is followed by a non-prime
carefully and answer the given questions. number, then the sum of both numbers is
There is a 5x5 matrix that is used to get cash multiplied by 3
back amount while using credit card in POS. The B. If an even number is followed by a prime
row of the matrix is denoted as 1 to 5 from top to number, then find the difference between both
bottom and the column of the matrix is denoted numbers and cube the sum of the digits of the
as @, #, $, % and & from left to right resultant.
respectively.

Click Here For Bundle PDF Course | support@guidely.in Page 3 of 10


SBI Clerk & RRB PO Mains PDF Course 2023
Reasoning Ability Day - 13 (Eng)

C. If an odd number is followed by a prime II. 5& 1$ 5@


number, then the sum of the unit digits of both a) 3
numbers is multiplied by 7 b) 5
D. If an even number is followed by a non-prime c) 4
odd number, then the difference between the unit d) 8
digits of both numbers is multiplied by 9. e) 9
E. If an even number is followed by an even
number, then multiply the unit digit of both 14. How much cash back will get from the
numbers and square the resultant. outcome of the given string?
11. How much cashback will get from the sum of 1% 4@ 3& 5$
the outcome of both strings? a) Rs.90
I. 2# 5% 1& b) Rs.200
II. 3@ 2$ 4% c) Rs.450
a) Rs.90 d) Rs.700
b) Rs.200 e)Rs.1500
c) Rs.450
d) Rs.700 15. Find the difference between the cash back
e) Rs.1500 we get from the outcome of each string given
below.
12. What is the product of the outcome of the I. 1%5#2&
below strings? II. 2@3#4$
I. 2% 3& 1# a) Rs.90
II. 4@ 3% 5$ b) Rs.210
a) 2420 c) Rs.110
b) 2320 d) Rs.150
c) 2220 e) Rs.100
d) 2520
e) 2120 Direction (16-20): Study the following information
carefully and answer the given questions.
13. If the rate of one chocolate is Rs.40, then Ten persons – A, B, C, D, E, F, G, H, I and J are
how many chocolates can buy using the cash sitting around a circular table, where some of
back will get from the sum of the outcome of both them are facing towards the centre while some of
strings? them are facing away from the centre.
I. 1@ 3$ 4#

Click Here For Bundle PDF Course | support@guidely.in Page 4 of 10


SBI Clerk & RRB PO Mains PDF Course 2023
Reasoning Ability Day - 13 (Eng)

Note: Consecutive alphabetically named persons d) I


are not sitting opposite to each other. e) The one who sits second to the right of E
G sits fourth to the right of B, where both are
facing opposite directions. Immediate neighbours 18. What is the position of E with respect to I in
of B are facing the opposite direction to him. the given arrangement?
Two persons sit between G and H, where both a) Third to the left
are facing the same direction. J and H are sitting b) Seventh to the right
immediate right of each other. The number of c) Fourth to the left
persons sitting between J and A is one less than d) Third to the right
the number of persons sitting between A and C, e) Fifth to the left
when counted from the left of both J and A
respectively. Three persons sit between I and D. 19. If J and B interchanged their position,
E sits to the immediate right of D. Both I and F similarly D and H interchanged their position,
are facing the opposite direction of E. The one then who among the following person sits third to
who sits to the immediate right of F is facing the left of D?
away from the centre. a) F
16. Who among the following pair of persons b) C
facing the same direction? c) B
a) GJ d) J
b) HI e) G
c) CE
d) DJ 20. As many persons sit between G and __ as
e) FH between C and __, when counted left of both G
and C.
17. Who among the following person sits fourth a) J, H
to the right of F? b) I, A
a) E c) A, E
b) The one who sits opposite to H d) H, F
c) A e) D, J

Click Here For Bundle PDF Course | support@guidely.in Page 5 of 10


SBI Clerk & RRB PO Mains PDF Course 2023
Reasoning Ability Day - 13 (Eng)

Click Here to Get the Detailed Video Solution for the above given Questions
Or Scan the QR Code to Get the Detailed Video Solutions

Answer Key with Explanation

Directions (1-5): From the above conditions, we have two


1. Answer: C possibilities:
2. Answer: A
3. Answer: E
4. Answer: D
5. Answer: D
Final arrangement:

Again we have,
 Only one floor is between I and the one
who has 17 shoes less than P, and both
are living in the same type of flat.
 M, who has prime number of shoes, lives
We have, two floors above the flat of L.
 The one who has 27 shoes lives two  Both M and P neither live on same floor
floors above the flat of N, who lives on an nor in the same type of flat.
odd numbered floor, has 8 shoes less  Only one floor is between K and P, who
than L. has a square number of shoes.
 The number of floors below N is one less  No one lives to the east of K, who has 4
than the number of floors above I, who shoes less than L.
has the number of shoes which is a
multiple of seven.

Click Here For Bundle PDF Course | support@guidely.in Page 6 of 10


SBI Clerk & RRB PO Mains PDF Course 2023
Reasoning Ability Day - 13 (Eng)

 If K has 12 shoes, then L may have 16


shoes and if K has 42 shoes then L may
have 46 shoes (L=K+4)
 If L has 46 shoes then N may have 38
shoes and if L has 16 shoes, then N may
have 8 shoes, which is not possible (Only
Again we have, two digit number)
 The difference between the number of  Thus, we may conclude, O=27, K=42,
shoes with I and J is 3. L=46, N=38
 J has the number of shoes which is While applying above conditions, case 2 gets
reverse of the number of shoes with M. eliminated, because no one should live to the
 The difference between the number of east of the one who has the number of shoes
shoes with O and K is 15. which is a multiple of 5. Thus, case 1 gives the
 No one lives to the east of the one who final arrangement.
has the number of shoes which is a
multiple of 5.
Calculations:
 M (Prime) – Taking all the prime numbers
from 11 to 50, we can find the number of
shoes had by J, who has the reverse of
the shoes had by M.
 Then we can find the number of shoes
6. Answer: E
had by P by using J (P=J+17, P has
I. H$T -> T>H (T≤R=E≤I=O<H) -> False
square number of shoes)
II. R@L -> L≥R (R=E≤I=O≤G=L) -> True
 By this, we can conclude: M=23, J=32
III. Q$P -> Q<P (P>O=I≥E>W≥Q) -> True
and P=49.
 Next, I-J is 3 (i.e. I=35/29), but I has
7. Answer: D
number of shoes which is multiple of 7, so
I. K@B -> B≥K (K≤J≤H<D=C≤B) -> False
I=35.
II. L$G -> G>L (G=F<D>H>L) -> False
 We know O has 27 shoes, so K may have
III. E#M -> M<E ( E>D>H≥J≥K=M) -> True
12 or 42 shoes (K-O is 15)

8. Answer: B
I. F&K -> F≥K (K≤L<T=Y<D≤F) -> False

Click Here For Bundle PDF Course | support@guidely.in Page 7 of 10


SBI Clerk & RRB PO Mains PDF Course 2023
Reasoning Ability Day - 13 (Eng)

II. H$A -> A>H (H≤J=L<T=Y=S≤A) ->True 13. Answer: B


III. J#O -> O<J (J=L<T=Y≤O) -> False I. 1@3$4# -> 34 32 26
By condition E: square of (4*2) = 64
9. Answer: E By condition E: square of (4*6) = 576
I. X$G -> X<G (X=K=H≤S≤D>F<G) -> False II. 5&1$5@ -> 59 68 41
II. A#V -> V<A (V<O≤K=H≤S<A) -> True By condition A: (59+68)*3 = 381
III. K&G -> K≥G (X=K=H≤S≤D>F<G) -> False By condition C: (1+1)*7 = 14
Combining I and III, we can say either Sum of both the strings= 590. Thus, cashback
conclusion I or III is true we get is Rs.200, so we can buy 5 chocolates.

10. Answer: C 14. Answer: E


I. P@R -> P≤R (P≤L≤M=N>V=A=E≤R) -> False 1%4@3&5$ -> 85 13 36 47
II. W#C -> W>C (C<N>V=A<W) -> False By condition C: (5+3)*7 = 56
III. E$X -> E<X (X>C<N>V=A=E) -> False By condition E: square of (6*6) = 1296
By condition B: (1296-47) = 1249 =Cube of
11. Answer: B (1+2+4+9) = 4096
I. 2#5%1& -> 76 53 102
By condition B: 76-53=23 -> 2+3 -> cube of 5 = 15. Answer: C
125 I. 1%5#2& -> 85 43 133
By condition A: (125+102)*3 = 681 By condition C: (5+3)*7 = 56
II. 3@2$4% -> 28 95 52 By condition D: (6-3)*9 = 27
By condition D: (8-5)*9 =27 We get Rs.90 as cashback.
By condition A: (27+52)*3 =237 II. 2@3#4$ -> 57 30 39
Sum of both strings=681+237 = 918 By condition A: (57+30)*3 = 261
By condition A: (261+39)*3 = 900
12. Answer: D We get Rs.200 as cashback.
I. 2%3&1# -> 114 36 51 Hence, difference between the two cashback is
By condition E: square of (6*4) = 576 Rs.110
By condition D: (6-1)*9 = 45
II. 4@3%5$ -> 13 34 47
By condition A: (13+34)*3 = 141
By condition C: (1+7)*7 = 56
Product of both strings: 45*56=2520

Click Here For Bundle PDF Course | support@guidely.in Page 8 of 10


SBI Clerk & RRB PO Mains PDF Course 2023
Reasoning Ability Day - 13 (Eng)

Again we have,
 The number of persons sitting between J
and A is one less than the number of
Directions (16-20): persons sitting between A and C, when
16. Answer: C counted from the left of both J and A
17. Answer: E respectively.
18. Answer: D  Three persons sit between I and D.
19. Answer: D  E sits to the immediate right of D.
20. Answer: B
Final arrangement:

Again we have,
 Both I and F are facing the opposite
direction of E.
 The one who sits to the immediate right of
F is facing away from the centre.
We have, While applying the above condition, case 1 gets
 G sits fourth to the right of B, where both eliminated, because the one who sits to the
are facing opposite directions. immediate right of F should face away from the
 Immediate neighbours of B are facing the centre. Thus, case 2 gives the final arrangement.
opposite direction to him.
 Two persons sit between G and H, where
both are facing the same direction.
 J and H are sitting immediate right of
each other.
From the above conditions, we have two
possibilities:

Click Here For Bundle PDF Course | support@guidely.in Page 9 of 10


SBI Clerk & RRB PO Mains PDF Course 2023
Reasoning Ability Day - 13 (Eng)

Click Here For Bundle PDF Course | support@guidely.in Page 10 of 10


SBI Clerk & RRB PO Mains PDF Course 2023
Quantitative Aptitude Day - 13 (Eng)

Quantitative Aptitude

Directions (1-5): Study the following information carefully and answer the questions given below.
The given below bar graph shows the number of mobile phones manufactured by five different
companies [A, B, C, D and E] in the month of June and also given the number of mobile phones
manufactured in the month of July is how much more or less then the number of mobile phones
manufactured by the same companies in June.

The table given below shows the ratio of 4G to 5G mobile manufactured by the company in each month
out of total number of mobile phones
manufactured by the company.

1) Out of the total number of 4G mobile phones 40% has 128GB memory and the remaining
manufactured by company A in the month June, mobile phones has 64GB memory and out of the

Click Here For Bundle PDF Course | support@guidely.in Page 1 of 12


SBI Clerk & RRB PO Mains PDF Course 2023
Quantitative Aptitude Day - 13 (Eng)

total number of 4G mobile phones manufactured companies together and the total 5G phone
by company A in the month July, 60% has manufactured in the month of July?
128GB memory and the remaining mobile A.420
phones has 64GB memory. Find the total B.450
number of 64GB memory 4G mobile phones C.460
manufactured by company A in the months of D.440
June and July together. E.None of these
A.920
B.940 4) Manufacturing cost of each 4G phone of
C.930 company C and D is Rs.4800 and Rs.6400
D.960 respectively and manufacturing cost of each 5G
E.990 phone of company C and D is Rs.5800 and
Rs.7000 respectively in both the months. Find
2) Ratio of snapdragon and Exynos processor in the difference (in thousand) of total
the 4G manufacturing phone of company B in the manufacturing cost in both months of company C
month of June is 3:2 respectively and Ratio of and D?
snapdragon and Exynos processor in the 4G A.10100
manufacturing phone of company B in the month B.14200
of July is 1:4 respectively. Find the difference C.15200
between the total number of snapdragon D.13500
processor 4G phone and total number of exynos E.None of these
processor 4G phone manufactured by company
B in both months together?(Consider only 5) Number of defective mobile manufactured in
snapdragon and exynos processor is month of June and July in company E is 20%
manufactured) and 10% of total mobile manufactured in the
A.20 company respectively and Number of defective
B.30 mobile manufactured in month of June and July
C.40 in company F is 10% and 10% of total mobile
D.0 manufactured in the company respectively. Find
E.10 the total defective mobile manufactured by
company E and F?
3) Find the difference between the total 5G A.870
phone manufactured in the month of June by all B.820
C.840

Click Here For Bundle PDF Course | support@guidely.in Page 2 of 12


SBI Clerk & RRB PO Mains PDF Course 2023
Quantitative Aptitude Day - 13 (Eng)

D.850 C.Rs. 3240


E.None of these D.Rs. 4240
E.None of these
6) A 720ml solution of sugar and water is
prepared in the ratio 7:2 respectively. Another 8) Certain distance is covered by a boat in
800ml solution of sugar and water in the ratio of downstream in 9 hours and in upstream in 11
1:7, the two solutions are mixed to form a new hours. Find the percentage of distance covered
solution. Now 20% solution is taken out from the the boat in 27 hours rowing downstream over the
new solution. Then some amount of sugar is distance coveredin55 hours while rowing
added which equals to 20% of the sugar content upstream?
in the first solution initially and 32ml water is A.45%
added. Water in the final mixture is how much B.60%
more or less than the sugar content? C.50%
A.80 more D.40%
B.50 more E.None of these
C.20 less
D.60 more 9) Half of the sum of the ages of Rohit, his wife
E.None of these and his daughter after 6 years will be 41 years.
After 8 years, the age of his daughter will be half
7) Three friends A, B and C invested in a firm the present age of his wife. Rohit’s age after 18
6:9:8, respectively. One year later, A increased years will be equal to his wife’s age 10 years
his investment by 25%. After 2 years, later. What will be his daughter’s age after 3
investments of B and C become twice and 1.5 years?
times of their initial investments respectively. A.11 years
Profit of Rs. 7650 is distributed among them after B.13 years
3 years. Find the profit amount received by B? C.21 years
A.Rs. 3245 D.31 years
B.Rs. 3540 E.None of these

Directions (10-13): Study the following data carefully and answer the given questions:
Data given below is related to the number of boys and girls studying in under-graduation (UG) and post-
graduation (PG) in 5 different colleges A, B, C, D and E.
Table given below shows the following information:

Click Here For Bundle PDF Course | support@guidely.in Page 3 of 12


SBI Clerk & RRB PO Mains PDF Course 2023
Quantitative Aptitude Day - 13 (Eng)

Note:
1: Total number of boys studying UG in all the 5 colleges together = 800
2: Total number of boys studying PG in all the 5 colleges together = 500
3: Some data in the table are missing, which needs to be calculated in the questions.
10) In college B, if the number of girls studying E.112
PG is 56 less than those in
UG, then find the average of number of boys 12) If the number of girls studying UG in college
studying PG in colleges A, B and E? E is equal to the number of boys studying UG in
A.100 college B, and ratio of boys to girls studying UG
B.80 in college E is 5: 6, then find the difference
C.75 between the total number of boys studying UG
D.120 and PG together in college E and the total
E.90 number of girls studying UG and PG together in
college E?
11) If the number of boys studying UG in college A.2
C is 168 less than those studying UG in college B.8
D and number of girls studying UG in college C C.4
is 16(2/3)% more than those studying UG in D.10
college A, then find the average number of boys E.6
and girls studying UG in college A?
A.102 13) If the average of number of girls studying PG
B.110 in colleges A and C is 115, then find the ratio of
C.108 the total number of students studying UG in
D.104 college C to those studying PG in college C?

Click Here For Bundle PDF Course | support@guidely.in Page 4 of 12


SBI Clerk & RRB PO Mains PDF Course 2023
Quantitative Aptitude Day - 13 (Eng)

A.24: 25 C.2834
B.4: 5 D.2323
C.16: 17 E.2258
D.8: 9
E.14: 15 15) In Village C, the number of people in the age
of below 50 years is 80 more than the number of
Directions (14-17): Study the following people in the age of above 50 years. What is the
information carefully and answer the given percentage of the number of people in the age of
questions. above 50 years in total population of Village C?
There are four different Villages – A,B,C and D. A.38.38%
The following facts are known about number B.54.54%
male & female population and number of people C.45.45%
below & above the age of 50 years. In Village B, D.64.56%
the number of male population is 100 more than E.68.87%
female population. Male population of Village B is
20% more than that of in Village A and female 16) Find the difference between the female
population of Village B is 20% less than that of in population of Villages A & C together and the
Village A. Total population in Village A is 1000. In male population of Villages B & D together?
Village C, Male population is 50 less than female A.70
population in Village A and female population is B.90
70 less than male population of Village A. In C.80
Village D, male population is 4% less than that of D.75
in Village C and female population is 10% more E.65
than male population of Village C. The ratio of
number of people in the age of below 50 years 17) Total number of people in the age of below
and above 50 years in Village A and D is same 50 years in villages A and D together is how
and its ratio is 3:2. much more than total number of people in the
14) If the ratio between the number of people in age of above 50 years in Villages A&D together?
the age of below 50 years and above 50 years in A.406
Villages B&C is 4:3 & 6:5 respectively. Find the B.687
total number of people in the age of below 50 C.577
years for all given Villages together? D.605
A.3232 E.543
B.3032

Click Here For Bundle PDF Course | support@guidely.in Page 5 of 12


SBI Clerk & RRB PO Mains PDF Course 2023
Quantitative Aptitude Day - 13 (Eng)

18) In the question two equations numbered I One of the roots of equations I and II are
and II are given. You have to solve both the common while value of A is a positive number.
equations and mark the answer. A.x < y
I: 2x2 – 3x – A = 0 B.x ≤ y
II: y2 – 20y + B = 0 C.x ≥ y
One root of equation I is the largest single digit D.x = y or relationship cannot be determined.
prime number and ratio of A to B is 7: 9. E.x > y
A.x < y
B.x ≤ y 20) In the question two equations numbered I
C.x ≥ y and II are given. You have to solve both the
D.x = y or relationship cannot be determined. equations and mark the answer.
E.x > y I: 2x – 525/x = 29
II: y2/ (36 – y) = 6
19) In the question two equations numbered I A.x < y
and II are given. You have to solve both the B.x ≤ y
equations and mark the answer. C.x ≥ y
I: 2x2 + 9x – 18 = 0 D.x = y or relationship cannot be determined.
II: 2 (y2 + 9) = Ay E.x > y
Click Here to Get the Detailed Video Solution for the above given Questions
Or Scan the QR Code to Get the Detailed Video Solutions

Answer Key with Explanation

Directions (1-5): Total number of mobile phones manufactured by


Total number of mobile phones manufactured by company A in the month of July is 1200 + 800 =
company A in the month of June is 1200. 2000.

Click Here For Bundle PDF Course | support@guidely.in Page 6 of 12


SBI Clerk & RRB PO Mains PDF Course 2023
Quantitative Aptitude Day - 13 (Eng)

Number of 4G mobile phones manufactured by 3) Answer: B


company A in the month of June is 1200 * 5/8 = Required difference = [(450 + 400 + 1000 + 800
750 + 800 + 600) - (800 + 300 + 400 + 500 + 1000 +
Number of 5G mobile phones manufactured by 600)] = 450
company A in the month of June is 1200 - 750 =
450 4) Answer: A
Number of 4G mobile phones manufactured by Total cost of company C = 900 * 4800 + 1400 *
company A in the month of July is 2000 * 3/5 = 5800 = 12440000
1200 Total cost of company D = 2100 * 6400 + 1300 *
Number of 5G mobile phones manufactured by 7000 = 22540000
company A in the month of July is 2000 - 1200 = So, difference = 22540000 – 12440000 =
800 10100000 = 10100 thousands
Similarly, we can calculate the others value also.
5) Answer: A
Total defective mobile
= 1800 * 20/100 + 2500 * 10/100 + 1500 *
10/100 + 1100 * 10/100
= 870

6) Answer: A
According to question,
Total quantity of solution I = 720ml
Quantity of sugar: water in the mixture = 7:2
Sugar content in solution I = 560ml
1) Answer: C
Water content in solution I = 160ml
Total number of 64GB memory 4G phone
Total quantity of solution II = 800ml
manufactured by company A in the month June
Quantity of sugar: water in the mixture = 1:7
and July together = 750 * 60/100 + 1200 *
Sugar content in solution II = 100ml
40/100 = 930
Water content in solution II = 700ml
After combining the solutions,
2) Answer: D
Total sugar content = 660ml
Required difference = [600 * 3/5 + 200 * 1/5] -
Total water content = 860ml
[600 * 2/5 + 200 * 4/5] = 0
20% solution is taken out

Click Here For Bundle PDF Course | support@guidely.in Page 7 of 12


SBI Clerk & RRB PO Mains PDF Course 2023
Quantitative Aptitude Day - 13 (Eng)

Sugar added = 20% x sugar content in the first Certain distance is covered by a boat in
solution initially downstream in 9 hours and in upstream in 11
So, 112ml sugar is added hours
32ml water is added Downstream speed of boat = D/9 kmph
Thus, final sugar content = 660 – 132 + 112 = Upstream speed of boat = D/11 kmph
640ml Distance covered in 27 hours rowing
Final water content = 860 – 172 + 32 = 720ml downstream = D/9 x 27 = 3D km
Required difference = 720 - 640= 80 more Distance covered in 55 hours rowing upstream =
Hence, answer is option A D/11 x 55 = 5D km
Percentage of distance covered by the boat in
7) Answer: C 27 hours rowing downstream over the distance
According to question, covered in55 hours rowing upstream = 3D/5D x
Let the initial amount invested by A, B and C = 100 = 60%
6x:9x:8x Hence, answer is option B
One year later, A increased his investment by
25% 9) Answer: A
After 2 years, investment of B becomes twice Let the present age of Rohit, daughter and wife =
After 2 years, investment of C becomes 1.5 R, D and W years
times According to question,
Total profit after 3 years = Rs. 7650 After 6 years, [1/2 x (Rohit + wife + daughter +
So, total time period= 3 years 18) = 41 years]
Ratio of profit shares = [(6x * 1) + (125% * 6x * 1/2 x (R + W + D + 18) = 41
2)]: [(9x * 2) + (2 * 9x * 1)]: [(8x * 2) + (1.5x * 8 * R + W + D = 82 – 18
1)] = 21:36:28 R + W + D = 64 ……………….. (1)
Profit amount received by B = Rs. 36/85 x 7650 After 8 years the age of his daughter = 1/2 x
= Rs. 3240 present age of his wife
Hence, answer is option C D + 8 = 1/2 x W
W – 2D = 16 ……………….. (2)
8) Answer: B Rohit’s age after 18 years =Rohit’s wife’s age 10
Let the distance covered = ‘D’ km years later
Let the speed of boat = ‘B’ kmph R + 18 = W + 10
Let the speed of stream = ‘S’ kmph W – R = 8 ……………….. (3)
According to question, From three equations we get,

Click Here For Bundle PDF Course | support@guidely.in Page 8 of 12


SBI Clerk & RRB PO Mains PDF Course 2023
Quantitative Aptitude Day - 13 (Eng)

W = 32 years
R = 24 years 12) Answer: A
D = 8 years Number of girls studying UG in college E =
Rohit’s daughter age after 3 years = 11 years Number of boys in UG in college B = 24% of 800
Hence, answer is option A = 192
Number of boys studying UG in college E = 192
10) Answer: A * (5/6) = 160
Number of boys studying UG in college B = 24% Number of boys studying PG in college E = 30%
of 800 = 192 of 500 = 150
So, number of girls studying UG in college B = (2 Number of girls studying PG in college E = (2 *
* 168) – 192 = 144 135) – 150 = 120
Number of girls studying PG in college B = 144 – Required difference = (192 + 120) – (160 + 150)
56 = 88 =2
And number of boys studying PG in college B =
(2 * 74) – 88 = 60 13) Answer: B
Number of boys studying PG in college A = 18% Number of boys studying PG in college A = 18%
of 500 = 90 of 500 = 90
Number of boys studying PG in college E = 30% Number of girls studying PG in college A = (2 *
of 500 = 150 105) – 90 = 120
Required average = (60 + 90 + 150)/3 = 100 Number of girls studying PG in college C = (2 *
115) – 120 = 110
11) Answer: C Number of boys studying PG in college C = 26%
Number of boys studying UG in college D = 31% of 500 = 130
of 800 = 248 Total number of students studying PG in college
So, number of boys studying UG in college C = C = 130 + 110 = 240
248 – 168 = 80 Since, average of boys and girls studying UG in
Number of girls studying UG in college C = (2 * college C = 96
96) – 80 = 112 So, total number of students studying UG in
And number of girls studying UG in college A = college C = 2 * 96 = 192
112 * (300/350) = 96 Required ratio = 192: 240 = 4: 5
Since, number of boys studying UG in college A
= 15% of 800 = 120 Directions (14-17):
So, required average = (120 + 96)/2 = 108

Click Here For Bundle PDF Course | support@guidely.in Page 9 of 12


SBI Clerk & RRB PO Mains PDF Course 2023
Quantitative Aptitude Day - 13 (Eng)

Let the male population of village A is ‘x’ and The number of people in the age of below 50
female population is ‘y’ years in C = 880*6/11 = 480
x + y = 1000 -------(1) The total number of people in the age of below
In village B, 50 years for all given Villages= 600 + 618 + 560
120*x/100 – 80*y/100 = 100 ---------(2) + 480 = 2258
By solving (1) and (2),
x = 450 15) Answer: C
y = 550 Let number of people in the age of above 50
In village B, years is ‘x’
Male population = 540 The number of people in the age of below 50
Female population = 440 years = 80 + x
In Village C, x + x + 80 = 880
Male population = 550 – 50 = 500 x = 400
Female population = 450 – 70 = 380 Required percentage = 400*100 / 880 = 45.45%
In Village D,
Male population = 500*96/100 = 480 16) Answer: B
Female population = 500*110/100 = 550 Female population of Villages A& C = 550+380 =
Number of people in the age of below 50 years 930
in Village A = 1000*3/5 = 600 Male population of Villages B&D = 540+480 =
Number of people in the age of above 50 years 1020
in Village A = 1000*2/5 = 400 Required difference = 1020 – 930 = 90
Number of people in the age of below 50 years
in Village D = 1030*3/5 = 618 17) Answer: A
Number of people in the age of above 50 years Total number of people in the age of below 50
in Village D = 1030*2/5 = 412 year in Villages A and D = 600 + 618 = 1218
Total number of people in the age of above 50
years in Villages A and D = 400 + 412 = 812
Required difference = 1218 – 812 = 406

14) Answer: E 18) Answer: A


The number of people in the age of below 50 I: 2x2 – 3x – A = 0
years in B = 980*4/7 = 560 We know that: Largest single digit prime number
=7

Click Here For Bundle PDF Course | support@guidely.in Page 10 of 12


SBI Clerk & RRB PO Mains PDF Course 2023
Quantitative Aptitude Day - 13 (Eng)

2 * 72 – 3 * 7 – A = 0 A = 15
98 – 21 = A Case 2: When ‘-6’ is the common root.
A = 77 2 * [(-6)2 + 9] = A * (-6)
2x2 – 3x – 77 = 0 90 = - 6A
2x2 – 14x + 11x – 77 = 0 A = -15 (Invalid)
2x * (x – 7) + 11 * (x – 7) = 0 Hence, 2 (y2 + 9) = 15y
(x – 7) * (2x + 11) = 0 2y2 + 18 – 15y = 0
x = 7 and -11/2 2y2 – 12y – 3y + 18 = 0
II: y2 – 20y + B = 0 2y * (y – 6) – 3 * (y – 6) = 0
A: B = 7: 9 (y – 6) * (2y – 3) = 0
77: B = 7: 9 y = 6 and 3/2
B = 99 When x = 3/2, y = 6; x < y
y2 – 20y + 99 = 0 When x = 3/2, y = 3/2; x = y
y2 – 11y – 9y + 99 = 0 When x = -6, y = 6; x < y
y (y – 11) – 9 (y – 11) = 0 When x = -6, y = 3/2; x < y
(y – 11) (y – 9) = 0 Hence, x ≤ y.
y = 11 and 9
When x = 7, y = 11; x < y 20) Answer: D
When x = 7, y = 9; x < y I: 2x – 525/x = 29
When x = -11/2, y = 11; x < y 2x2 – 525 = 29x
When x = -11/2, y = 9; x < y 2x2 – 29x – 525 = 0
Hence, x < y. 2x2 – 50x + 21x – 525 = 0
2x * (x – 25) + 21 * (x – 25) = 0
19) Answer: B (x – 25) * (2x + 21) = 0
I: 2x2 + 9x – 18 = 0 x = 25 and -21/2
2x2 + 12x – 3x – 18 = 0 II: y2/ (36 – y) = 6
2x (x + 6) – 3 (x + 6) = 0 y2 = 216 – 6y
(x + 6) (2x – 3) = 0 y2 + 6y – 216 = 0
x = 3/2 and -6 y2 – 12y + 18y – 216 = 0
II: 2 (y2 + 9) = Ay y (y – 12) + 18 (y – 12) = 0
Case 1: When ‘3/2’ is the common root. (y – 12) (y + 18) = 0
2 [(3/2)2 + 9] = A * (3/2) y = 12 and -18
45/2 = 3A/2 When x = 25, y = 12; x > y

Click Here For Bundle PDF Course | support@guidely.in Page 11 of 12


SBI Clerk & RRB PO Mains PDF Course 2023
Quantitative Aptitude Day - 13 (Eng)

When x = 25, y = -18; x > y Hence, relationship between ‘x’ and ‘y’ cannot be
When x = -21/2, y = 12; x < y determined.
When x = -21/2, y = -18; x > y

Click Here For Bundle PDF Course | support@guidely.in Page 12 of 12


SBI Clerk & RRB PO Mains PDF Course 2023
ENGLISH Day - 13

English Language

Directions (1-5): Given below are a few A. a


questions based on the jumbled paragraph given B. b
in which sentences are to be rearranged in order C. c
to make a contextually and meaningfully correct D. d
paragraph. If none of the options are correct then E. e
choose option E as your answer.
a. The finance ministry said the new Rs 75 coin 3. Which of the following sentence will be exactly
will bear the inscription of the Parliament in middle after the rearrangement?
complex and have the image of the new A. a
Parliament building. B. b
b. One side of the coin will feature the Lion C. c
Capital of Ashoka Pillar, with the words D. d
“Satyamev Jayate” below it. E. e
c. It will be circular in shape with a diameter of 44
millimeters and have 200 serrations along its 4. Which of the following sentence will be second
edges. last after the rearrangement?
d. The 35-gram coin will be made from a four- A. a
part alloy, comprising 50 per cent silver, 40 per B. b
cent copper, 5 per cent nickel and 5 per cent C. c
zinc. D. d
e. Below the image of the Parliament complex, E. e
2023 will be inscribed to mark the year.
1. Which of the following sentence will be last 5. Which of the following sentence will be first
after the rearrangement? after the rearrangement?
A. a A. a
B. b B. b
C. c C. c
D. d D. d
E. e E. e

2. Which of the following sentence will be second Directions (6-10): Given below are a few
after the rearrangement? questions divided in five parts which may or may

Click Here For Bundle PDF Course | support@guidely.in Page 1 of 10


SBI Clerk & RRB PO Mains PDF Course 2023
ENGLISH Day - 13

not contain error in them. You have to find the B. b


part in which there is an error. If none of the C. c
parts have error then choose option E as your D. d
answer. E. e
6. A new-generation ballistic missile 'Agni Prime'
was successfully flight tested(a)/ by the Defence 9. The telecom market in India was upended by
Research and Development Organisation Jio's launch(a)/ in 2016(b)/, when it offered free
(DRDO)(b)/ on June 7, 2023,(c)/at nearly 7.30 calls and cut-price data plans(c)/, eroding the
pm, from Dr APJ Abdul Kalam Island off the profit and the revenue of rivals and leading to
coast of Odisha(d). No error (e). consolidation(d). No error (e).
A. a A. a
B. b B. b
C. c C. c
D. d D. d
E. e E. e

7. After Chinese aggression in areas bordering 10. Astronauts have been visit space for decades
Sikkim(a)/, Ladakh and Arunachal Pradesh(b)/, now (a)/, and there has been at least one
China’s People’s Liberation Army (PLA) have astronaut in the Space Station constantly for the
shifted focus to(c)/ ramp up military infrastructure last years (b)/, now, the plan is to push further on
in locations across Uttrakhand(d). No error (e). the Moon and Mars (c)/, which require long haul
A. a space flights and that could have its toll on the
B. b body and mind (d). No error (e).
C. c A. a
D. d B. b
E. e C. c
D. d
8. Repo rate is the rate at which the RBI E. e
lendmoney to banks(a)/, and any change in it
affects bank loans and EMIs(b). The RBI raised Directions (11-15): Given below are a few
the repo rate by a total of 250 basis points(c)/ questions with two columns to be matched with
since May 2022 to keep inflation in check(d). No each other to make a contextually correct
error (e). sentence based on the connectors. If none of the
A. a

Click Here For Bundle PDF Course | support@guidely.in Page 2 of 10


SBI Clerk & RRB PO Mains PDF Course 2023
ENGLISH Day - 13

options are correct then choose option E as your ambit of the scheme
answer. and more are likely to
11. be included in view of
Column 1 Column 2 the stellar results,
1.Though some a. Not only has the A. 1-a, 2-b, 3-c
aspects of India’s scheme reduced B. 1-b, 2-c, 3-a
economy are being dependency on C. 1-c, 2-a, 3-b
impacted by the global imports, but it has also D. 1-a, 2-c, 3-b
slowdown such as played a crucial role in E. None of the above
slower exports, impact creating more
of high interest rates employment 12.
and inflation, the opportunities in labour- Column 1 Column 2
factors mentioned intensive sectors. 1.Lack of job creation a. On the other hand,
above have played the is another problem that lower denomination
role of an equalizer, could hinder India’s commemorative coins
smoothing the track for growth, usually stay in
India to be the fastest- circulation for a limited
growing economy, time.
2.For instance, the b. despite a slight slow
government has been down in growth 2.Higher denomination b. However, for higher
aggressively promoting estimates for FY24. commemorative coins, denomination coins
investments across which have precious (those exclusively for
multiple important metals like gold and collections), you need
sectors through its silver in them, are to order them directly
successful production- usually treated as from any of the Indian
linked incentive (PLI) collectibles and are Government Mints
scheme, which has led usually not released as during a specific time
to a sharp jump in legal tender. period.
output, and
3.From electronics to c. therefore, reduced 3.The lower c. according to
automobiles, several India’s dependency on denomination coins economists, a majority
sectors have been imports. can be seen often in of whom said private
included under the general circulation and investments are not

Click Here For Bundle PDF Course | support@guidely.in Page 3 of 10


SBI Clerk & RRB PO Mains PDF Course 2023
ENGLISH Day - 13

you may have come enough to raise the Manipur Assembly


across them in the employment levels. and government.
past. A. 1-a, 2-b, 3-c
B. 1-b, 2-c, 3-a
A. 1-a, 2-b, 3-c C. 1-c, 2-a, 3-b
B. 1-b, 2-c, 3-a D. 1-a, 2-c, 3-b
C. 1-c, 2-a, 3-b E. None of the above
D. 1-a, 2-c, 3-b
E. None of the above 14.
Column 1 Column 2
13. 1. It has been more a. while scores of others
Column 1 Column 2 than a month since the have been displaced.
1.Government a. However, the same violence between the However, there seems
infrastructure spending has been extended on Meitei and Kuki to be no end to the
will bolster steel and a yearly basis. communities has been violence in sight.
cement, raging. Nearly 100
2. August 22, 2008, the b. while India's net- people have so far
suspension of zero commitment will been killed,
operations (SoO) drive investment in 2. The WSJ report b. However, it is only
agreement was sealed renewable energy. states that Instagram useful when you lose
with the aim of starting is allowing pedophiles something at the same
a political dialogue with to share sexual place within 20-50
the militant groups. content that contains meters of range.
3. The Suspension of c. Since the Kuki minors.
Operation agreement organisations, who had 3. JioTag has a limited c. Not only has the
has a one-year earlier called for the range, but it is useful platform failed to restrict
duration. creation of a separate to locate lost items like child porn, but its
Kuki state, settled on a small purse or algorithm is also
the creation of a wallet, a key ring, car promoting such content
"Kukiland territorial key, and more. It is and recommending it to
council," with financial also pretty affordable. pedophiles.
and administrative A. 1-a, 2-b, 3-c
authority separate from

Click Here For Bundle PDF Course | support@guidely.in Page 4 of 10


SBI Clerk & RRB PO Mains PDF Course 2023
ENGLISH Day - 13

B. 1-b, 2-c, 3-a


C. 1-c, 2-a, 3-b 3. Beyond science, the c. While the loss of
D. 1-a, 2-c, 3-b Chandrayaan-3 Chandrayaan-2
E. None of the above mission will also act as dampened the spirits,
a confidence boost for engineers and the Isro
15. Isro, which has plans to science team worked
Column 1 Column 2 explore the Sun and day in and night out to
1. The Space Force's a. While the SLS meets Venus in the coming get Chandrayaan-3
Pentecost said Boeing Pentagon months and years. ready for launch.
has been in talks with requirements, its slow
U.S. defense officials production rate A. 1-a, 2-b, 3-c
to explore offering SLS "probably doesn't," B. 1-b, 2-c, 3-a
in the Pentagon's Pentecost added. C. 1-c, 2-a, 3-b
Phase 3 National D. 1-a, 2-c, 3-b
Security Space Launch E. None of the above
competition - a
procurement program Directions (16-20): Given below are a few
expected to buy billions questions with a highlighted part in each of them
of dollars in launches which may or may not need replacement. You
from multiple have to find the appropriate replacement in order
companies. to make the sentence grammatically correct. If
2. India has been b. However, there was no replacement is required then choose option E
exploring the lunar a brief pause when the as your answer.
world for over a decade space agency lost the 16. While US-China tensions may have been
now as it continues to Chandrayaan-2 given new impetus under the Trump
push its boundaries. It mission, but with its administration which levied tariffs broadly and
all began with next successor, Isro imposed sanctions on Huawei, the friction had
Chandrayaan when wants to go beyond the continued unabated under President Joe Biden
Isro launched the orbit of the Moon and as both countries vie for global tech pre-
maiden Moon mission, touch the Moon, eminence.
which eventually ended literally. A. friction has continued unabated under
up finding water on the President Joe Biden
Moon.

Click Here For Bundle PDF Course | support@guidely.in Page 5 of 10


SBI Clerk & RRB PO Mains PDF Course 2023
ENGLISH Day - 13

B. the friction have continued unabated under same-sex partnerships


President Joe Biden D. South Korea does not recognises civil same-
C. friction have continued unabated under sex partnerships
President Joe Biden E. No replacement is required
D. the friction has continued unabated under
President Joe Biden 19. South Korea is facing a impending
E. No replacement is required demographic crisis, including a world-record low
birthrate.
17. The number of increasingly risky aerial A. South Korea is facing an impending
intercepts conducted by Chinese aircraft is demograph crisis
alarming as China has been getting more B. South Korea is facing an impend demographic
aggressive which could inculcate a mishap or crisis
potential unsafe incident. C. South Korea is facing a impending demograph
A. The number of increasingly risky aerial crisis
intercept conducted D. South Korea is facing an impending
B. The number of increasingly risky aerial demographic crisis
intercepts conducting E. No replacement is required
C. The number of increasingly risky aerial
interceptconducting 20. The BRICS countries proposed to hold the
D. The number of increasing risk aerial intercepts next summit in South Africa in August that year,
conducted media reports say several countries have
E. No replacement is required evinced interest in joining the grouping of
emerging markets.
18. South Korea does not recognised civil same- A. The BRICS countries proposeto hold the next
sex partnerships but the constitution stipulates summit in South Africa in August this year
that marriage and family shall be established on B. The BRICS countries proposed to hold next
the basis of individual dignity and equality of the summit in South Africa in August that year
sexes. C. The BRICS countries proposed to hold the
A. South Korea does not recognise civil same- next summit in South Africa at August that year
sex partnerships D. The BRICS countries proposed to hold the
B. The South Korea does not recognises civil next summit in South Africa in August this year
same-sex partnerships E. No replacement is required
C. The South Korea does not recognise civil

Click Here For Bundle PDF Course | support@guidely.in Page 6 of 10


SBI Clerk & RRB PO Mains PDF Course 2023
ENGLISH Day - 13

Click Here to Get the Detailed Video Solution for the above given Questions
Or Scan the QR Code to Get the Detailed Video Solutions

Answer Key with Explanation

1. Answer: B of the coin from back and front. So, option C is


Here, the first sentence after the rearrangement the correct answer.
will be a which gives introductory part as the
launch of special coin and it will be followed by c 3. Answer: D
which gives the looking of the coin and its Here, the first sentence after the rearrangement
dimensions and it will be followed by d which will be a which gives introductory part as the
gives information about the metal from which it is launch of special coin and it will be followed by c
made and it will be followed by e and then finally which gives the looking of the coin and its
ends with b which gives end part as the final look dimensions and it will be followed by d which
of the coin from back and front. So, option B is gives information about the metal from which it is
the correct answer. made and it will be followed by e and then finally
ends with b which gives end part as the final look
2. Answer: C of the coin from back and front. So, option D is
Here, the first sentence after the rearrangement the correct answer.
will be a which gives introductory part as the
launch of special coin and it will be followed by c 4. Answer: E
which gives the looking of the coin and its Here, the first sentence after the rearrangement
dimensions and it will be followed by d which will be a which gives introductory part as the
gives information about the metal from which it is launch of special coin and it will be followed by c
made and it will be followed by e and then finally which gives the looking of the coin and its
ends with b which gives end part as the final look dimensions and it will be followed by d which
gives information about the metal from which it is

Click Here For Bundle PDF Course | support@guidely.in Page 7 of 10


SBI Clerk & RRB PO Mains PDF Course 2023
ENGLISH Day - 13

made and it will be followed by e and then finally which is inappropriate instead lends should have
ends with b which gives end part as the final look been used. So, option A is the correct answer.
of the coin from back and front. So, option E is
the correct answer. 9. Answer: D
Here, the error lies in the part d as use of the
5. Answer: A before revenue is inappropriate as it has been
Here, the first sentence after the rearrangement already used before profit and here the revenue
will be a which gives introductory part as the and profit of rivals is being talked and therefore
launch of special coin and it will be followed by c one article should be used as if used before both
which gives the looking of the coin and its revenue and profit then it will be deemed as both
dimensions and it will be followed by d which are indicating for different but here it is being
gives information about the metal from which it is talked for the one. So, option D is the correct
made and it will be followed by e and then finally answer.
ends with b which gives end part as the final look
of the coin from back and front. So, option A is 10. Answer: A
the correct answer. Here, the error lies in part a as use of visit is
inappropriate as sentence is in present perfect
6. Answer: E continuous and here visiting is the correct form
Here, there is no error in the given question. So, for this tense as it always uses v1+ing form of
option E is the correct answer. verb. So, option A is the correct answer.

7. Answer: C 11. Answer: B


Here, the error lies in the part c as use of have is Here, the first part of column one should be
inappropriate as PLA is a singular subject and matched with second part of column two as both
therefore singular helping verb will be used. So, are connected by despite as a connector which
has should have been used here in place of is used when contradiction is seen in situations,
have. So, option C is the correct answer. second part of first column should be matched
with third part of second column as both are
8. Answer: A connected by therefore which is used to connect
Here, the error lies in the part a as RBI is a the further information of a sentence and finally
singular subject and therefore it will be followed the third part of first column should be matched
by singular noun but here lend is a plural noun with first part of second column as both are
connected with not only as it is used to emphasis

Click Here For Bundle PDF Course | support@guidely.in Page 8 of 10


SBI Clerk & RRB PO Mains PDF Course 2023
ENGLISH Day - 13

when writing about two related things and here 14. Answer: D
same can be seen. So, option B is the correct Here, the first part of column one should be
answer. matched with first part of column two as both are
connected with while and second part of column
12. Answer: C one should be matched with third part of column
Here, the first part of column one should be two as both are connected with not only and
matched with third part of column two as both finally the third part of column one should be
are connected with according and it is used to matched with second part of column two as both
connect the further information about the context are connected with however. So, option D is the
being used and second part of column one correct answer.
should be matched with the first part of column
two as both are connected with on the other 15. Answer: A
hand which is used to connect parts showing Here, the first part of column one should be
contradictions and third part of column one matched with the first part of column two as both
should be matched with second part of column are connected with while which is used to
two as both are connected with however which is connect two longer events or activities
used to connect sentences showing happening at the same time and second part of
contradictions. So, option C is the correct column one should be matched with second part
answer. of column two as both are connected with
however which is used to show contrast and
13. Answer: B finally the third part of column one should be
Here, the first part of column one should be matched with third part of column two as both
matched with second part of column two as both are connected with while. So, option A is the
are connected by while and second part of correct answer.
column one should be matched with third part of
column tow as both are connected by since 16. Answer: D
which is used to denote the time period and third Here, has should have been used in place of
part of column one should be matched with first had as sentence is in present perfect tense and
part of column two as both are connected with this tense uses has or have as helping verb and
however which is used to show contrast and here subject is singular therefore has should
here same can be seen. So, option B is the have been used. So, option D is the correct
correct answer. answer.

Click Here For Bundle PDF Course | support@guidely.in Page 9 of 10


SBI Clerk & RRB PO Mains PDF Course 2023
ENGLISH Day - 13

17. Answer: E Here, an should have been used before


Here, no replacement is required for this impending as impending starts with vowel and
highlighted phrase. So, option E is the correct an is used before those words sounding like
answer. vowel and here impending sounds like vowel.
So, option D is the correct answer.
18. Answer: A
Here, recognised should be replaced with 20. Answer: D
recognise as sentence is in present indefinite Here, "this "should be used instead "that".
tense and here second form of verb will not be Because sentence is indicating something to
appropriate. So, option A is the correct answer. happen in the future this year and here
"this"should have been used. That year indicates
19. Answer: D past. So, option D is the correct answer.

Click Here For Bundle PDF Course | support@guidely.in Page 10 of 10


SBI Clerk & RRB PO Mains PDF Course 2023
Reasoning Ability Day - 14 (Eng)

Reasoning Ability
Directions (1-5): Study the following information d) Paternal aunt
carefully and answer the e) Cannot be determined
given questions.
The family consists of ten persons – E, F, G, H, I, 2) Which of the following statement(s) is/are
U, V, W, X and Y. They are sitting in TRUE as per the given arrangement?
two parallel rows containing five persons each, in I) Only one person sits between U’s spouse and
such a way that there is an equal X’s brother
distance between adjacent persons. In row-1 E, II) No one sits to the right of G’s daughter-in-law
F, G, H and I are seated and all of III) W sits immediate left of E’s mother
them are facing north. In row-2 U, V, W, X and Y IV) X doesn’t face the one who is the brother-in-
are seated and all of them are facing law of I
south. Each person in row 1 faces exactly one a) Only I
person in row 2. b) Only I and II
X sits second to the left of I’s father-in-law. H’s c) Only I, II and IV
brother-in-law is an immediate neighbour of the d) Only III
one who faces I’s father-in-law. X’s brother e) All are true
doesn’t face the same direction as X. As many
people sit to the left of H’s brother-in-law as to 3) If G is related to U and F’s father is related to
the right of X’s brother. U’s father-in-law sits third Y, in the same pattern who among the following
to the left of F, who faces the one who is the person is related to V?
mother of U. H is an unmarried male. E’s mother a) H
sits second to the right of U’s mother. U is the b) G’s son
only sister of H but is not married to F. W is not c) F
an immediate neighbour of E’s sister-in-law. E is d) Y’s grandson
U’s sister-in-law and sits second to the left of the e) I’s sister
one who faces G’s grandson. U sits third to the
right of V’s spouse. V is the grandfather of F. G’s 4) What is the position of H with respect to the
only son sits third to the left of H. one who faces G’s spouse?
1) How E is related to the one who sits second to a) Third to the right
the left of H’s father? b) Immediate left
a) Mother c) Second to the right
b) Paternal uncle d) Immediate right
c) Nephew e) Third to the left

Click Here For Bundle PDF Course | support@guidely.in Page 1 of 10


SBI Clerk & RRB PO Mains PDF Course 2023
Reasoning Ability Day - 14 (Eng)

5) Who among the following person sits second III) If the triangle has a consonant letter, then the
to the left of the one who is the letter is replaced by its third
brother-in-law of I? succeeding letter of its reverse alphabet
a) Y according to alphabetical series.
b) X IV) If the triangle has an even value, then the
c) E value is subtracted from its immediate
d) V preceding prime number.
e) None of these 6) What is the square value of the difference
between the highest and the
Directions (6-10): Study the following information second highest value from the final output?
carefully and answer the a) 49
given questions. b) 324
Five triangles numbered 1 to 5 are inscribed in a c) 361
pentagon box. Each triangle has a letter and d) 289
value (positive integers). The value for each e) None of these
triangle is the difference between the first five
two digit composite numbers and the square 7) What is the final output of triangle 4?
value of the first five odd prime numbers a) 3V
respectively, then the resultant values are b) 64W
arranged in ascending order from triangles 1 to 5 c) 16Z
respectively. The letter for each triangle is the d) 81Q
corresponding letter of the sum of the digits of e) 1Y
the resultant value of each triangle, then the
resultant letters are arranged in alphabetical 8) If all the letters are written together without
order from triangles 1 to 5 respectively. any gap in the same order
Conditions: of triangle 1 to 5 in the final output, then how
I) If the triangle has a vowel letter, then the letter many such pairs of letters are there in the word
is replaced by the immediate each of which has as many letters between them
preceding letter of its reverse alphabet according as there in the English alphabetical series (both
to alphabetical series. forward and backward directions)?
II) If the triangle has an odd value, then the value a) One
is replaced by its square value of b) Two
the sum of the digits. c) Four
d) Three

Click Here For Bundle PDF Course | support@guidely.in Page 2 of 10


SBI Clerk & RRB PO Mains PDF Course 2023
Reasoning Ability Day - 14 (Eng)

e) None direction to each other.


Note II: Not more than two adjacent persons
9) What is the corresponding letter (as per the facing the same direction.
alphabetical order) of the Only one person sits between J and B, who sits
difference between the place value(as per the third from the extreme end of the row. D
alphabetical order) of the letters which are sits third to the right of J, who faces north. The
obtained from triangle 2 and triangle 5 in the final number of people sitting to the right of D is one
output? more than the number of people sitting to the left
a) H of A. C sits second to the right of A, who doesn’t
b) I sit adjacent to D. Only two people sit between C
c) D and E and both of them are facing the same
d) J direction. The immediate neighbours of E are
e) F facing the opposite direction to each other. As
many people sit between E and H as to the right
10) If all the letters are obtained in the final of G. H and I are not sitting adjacent to each
output arranged in alphabetical order starting other. The immediate neighbours of C are facing
from triangle 1 in the final output, then the the same direction of I. At most four people sit
position of which of the following letter will between G and I.
remain unchanged? 11) If the persons who are facing the north
a) Z direction are married, then how
b) V many persons are unmarried?
c) Q a) One
d) Y b) Two
e) W c) Three
d) Four
Directions (11-15): Study the following e) More than four
information carefully and answer the
given questions. 12) Which of the following statement(s) is/are
Nine people – A, B, C, D, E, G, H, I and J are NOT TRUE as per the given arrangement?
sitting in a linear row in such a way that I) Only one person sits between I and E
some of them are facing north while some of II) H sits immediate left of C
them are facing south. III) J sits exactly between A and D
Note I: The persons who are sitting at the IV) D sits at one of the extreme ends of the row
extreme end of the row facing opposite a) Only II

Click Here For Bundle PDF Course | support@guidely.in Page 3 of 10


SBI Clerk & RRB PO Mains PDF Course 2023
Reasoning Ability Day - 14 (Eng)

b) Only I and IV Direction (16-20): Study the following information


c) Only I, III and IV carefully and answer the given questions
d) Only III and IV Eight persons - A, B, C, D, E, F, G and H are
e) None is true living on different floors of an eight storey
building where the lowermost floor is numbered
13) How many people sit between C and the one one and the floor immediately above it is
who sits immediate left of B? numbered two and so on. Each of them uses
a) One different flavour of perfumes- Rose, Vanilla,
b) As many people sit to the right of G Citrus, Jasmine, floral, Fruity, Fresh and Aquatic.
c) Two D lives five floors below the one who uses Citrus.
d) More than four The number of floors above D is one more than
e) As many people sit to the left of J the number of floors below B, where neither of
them uses rose and aquatic. Only three floors
14) The immediate neighbours of which among are between B and the one who uses Fruity. H
the following persons are not facing the same lives three floors below the one who uses Rose.
direction? The number of floors between H and F, where
I) D neither of them uses Fruity is one more than the
II) J number of floors between F and the one who
III) C uses Aquatic. F and C are not living on the
a) Only II adjacent floors. E lives immediately below C,
b) Only II and III where neither of them uses Citrus. Only one floor
c) Only III is between E and the one who uses Vanilla, who
d) All I, II and III lives on the adjacent floor of the one who uses
e) None Floral. A lives above the one who uses Fresh
and below G. At-most one floor is between G and
15) In which among the following pair of persons the one who uses Jasmine.
the first person sits adjacent to the one who sits 16. Who among the following person lives three
second to the left of the second person? floors below the one who uses Jasmine?
a) A-C a) The one who uses Fruity
b) H-J b) H
c) G-D c) D
d) C-B d) The one who lives two floors above F
e) None of these e) G

Click Here For Bundle PDF Course | support@guidely.in Page 4 of 10


SBI Clerk & RRB PO Mains PDF Course 2023
Reasoning Ability Day - 14 (Eng)

17. Which of the following combination is true? 19. The number of floors between G and ___ is
a) A-Jasmine one more than the number of floors between E
b) H-Rose and ___ respectively
c) D-Vanilla a) The one who uses citrus, D
d) B-Fruity b) H, the one who uses Jasmine
e) E-Aquatic c) The one who uses aquatic and A
d) The one who uses Floral and B
18. Which of the following statements is/are not e) None of these
false as per the given arrangement?
a) The one who uses Floral lives on an odd 20. How many floors are between A and the one
numbered floor who uses Vanilla?
b) Odd number of floors between B and the one a) Six
who uses Vanilla b) Four
c) As many floors above the one who uses fruity c) One
as below H d) Two
d) Both a and b e) Five
e) Both b and c
Click Here to Get the Detailed Video Solution for the above given Questions
Or Scan the QR Code to Get the Detailed Video Solutions

Answer Key with Explanation

Directions (1-5): 5) Answer: C


1) Answer: D Final arrangement:
2) Answer: B
3) Answer: E
4) Answer: A

Click Here For Bundle PDF Course | support@guidely.in Page 5 of 10


SBI Clerk & RRB PO Mains PDF Course 2023
Reasoning Ability Day - 14 (Eng)

 H is an unmarried male.
 E’s mother sits second to the right of U’s
mother.
 U is the only sister of H but is not married
to F.
After applying the above condition, Case3 gets
We have,
eliminated. Because F is not the brother-
 X sits second to the left of I’s father-in-
in-law of H.
law.
 H’s brother-in-law is an immediate
neighbour of the one who faces I’s father-
in-law.
 As many people sit to the left of H’s
brother-in-law as to the right of X’s
brother.
 X’s brother doesn’t face the same
direction as X.
From the above condition, there are four
possibilities

Again we have,
 E is U’s sister-in-law and sits second to
the left of the one who faces G’s
grandson.
 U sits third to the right of V’s spouse.
 V is the grandfather of F.
 G’s only son sits third to the left of H.
 W is not an immediate neighbour of E’s
sister-in-law.
After applying the above condition, Case1 and
Case2 get eliminated. Hence, Case4
shows the final arrangement. Case1 was
Again we have,
eliminated because not possible to
 U’s father-in-law sits third to the left of F.
 F faces the one who is the mother of U.

Click Here For Bundle PDF Course | support@guidely.in Page 6 of 10


SBI Clerk & RRB PO Mains PDF Course 2023
Reasoning Ability Day - 14 (Eng)

place “U” in case1. Case 2 was eliminated Final arrangement:


because there is no possibility to place “E” Five triangles are numbered 1 to 5 are inscribed
in case 2 in a pentagon box.
Each triangle has a letter and value(positive
integer).
The value for each triangle is the difference
between the first five two digit composite
numbers and the square value of the first five
odd prime numbers respectively, then the
resultant values are arranged in ascending order
from triangles 1 to 5 respectively. The square
value of the first five odd prime numbers are – 9,
25, 49, 121 and 169
The first five two digit composite numbers are –
10, 12, 14, 15 and 16
The difference between the square value of the
first five odd prime numbers and the
first five two digit composite numbers are,
Triangle 1 = 10-9 = 1
Triangle 2 = 25-12 = 13
Triangle 3 = 49-14 = 35
Triangle 4 = 121-15 = 106
Triangle 5 = 169-16 = 153
The letter for each triangle is the corresponding
Directions (6-10): letter of the sum of the digits of the resultant
6) Answer: D value of each triangle, then the resultant letters
7) Answer: A are arranged in alphabetical order from triangles
8) Answer: C 1 to 5 respectively.
1 = A; 1+3=4 = D; 3+5=8 = H; 1+0+6= G;
1+5+3= I

9) Answer: B
10) Answer: E

Click Here For Bundle PDF Course | support@guidely.in Page 7 of 10


SBI Clerk & RRB PO Mains PDF Course 2023
Reasoning Ability Day - 14 (Eng)

 Only two people sit between C and E and


both of them are facing the same direction
 The immediate neighbours of E are facing
the opposite direction to each other
After applying the above condition, Case 1 gets
eliminated Because we cannot place A. And
Directions (11-15): there is another possibility Case 3a.
11) Answer: D
12) Answer: C
13) Answer: E
14) Answer: A
15) Answer: B Again we have,
Final arrangement  As many people sit between E and H as
to the right of G
 The immediate neighbors of C are facing
We have, the same direction of I
 Only one person sits between J and B,  At most four people sit between G and I
who sits third from the extreme end of the  H and I are not sitting adjacent to each
row other
 D sits third to the right of J, who faces After applying the above condition, Case 2 and
north Case 3 get eliminated. Because in
From the above condition, there are three Case 2 H and I are sitting adjacent to each
possibilities other. In case 3, more than four people sit
between G and I. Hence, Case 3a shows the
final arrangement.

Again we have,
 The number of people sitting to the right
of D is one more than the number of
people sitting to the left of A Directions (16-20):
 C sits second to the right of A, who 16. Answer: D
doesn’t sit adjacent to D 17. Answer: C

Click Here For Bundle PDF Course | support@guidely.in Page 8 of 10


SBI Clerk & RRB PO Mains PDF Course 2023
Reasoning Ability Day - 14 (Eng)

18. Answer: B more than the number of floors between F


19. Answer: C and the one who uses Aquatic.
20. Answer: B  F and C are not living on the adjacent
Final arrangement: floors.
 E lives immediately below C, where
neither of them uses Citrus.
From the above conditions, case 1 gets
eliminated because D does not use Aquatic.

We have,
 D lives five floors below the one who uses
Citrus.
 The number of floors above D is one
Again we have,
more than the number of floors below B,
 Only one floor is between E and the one
where neither of them uses rose and
who uses Vanilla, who lives on the
aquatic.
adjacent floor of the one who uses Floral.
 Only three floors are between B and the
 A lives above the one who uses Fresh
one who uses Fruity.
and below G.
From the above conditions, we have three
 At-most one floor is between G and the
possibilities:
one who uses Jasmine.
While applying the above conditions, case 2 gets
eliminated, because more than one floor is
between G and the one who uses Jasmine.
Hence, case 3 gives the final arrangement.

Again we have,
 H lives three floors below the one who
uses Rose.
 The number of floors between H and F,
where neither of them uses Fruity is one

Click Here For Bundle PDF Course | support@guidely.in Page 9 of 10


SBI Clerk & RRB PO Mains PDF Course 2023
Reasoning Ability Day - 14 (Eng)

Click Here For Bundle PDF Course | support@guidely.in Page 10 of 10


SBI Clerk & RRB PO Mains PDF Course 2023
Quantitative Aptitude Day - 14 (Eng)

Quantitative Aptitude

Directions (1-5): Read the following information carefully and answer the questions based on it.
There are five educational YouTube channels. In each channel, Video is available only for two subjects
maths and reasoning. The duration of each video is either below 45 mins or above 45 mins. Percentage
distribution of video available for reasoning in all the channels together is given in the pie chart. In the
table chart, the percentage of math video out of the total videos and the ratio of the video duration below
45 min and above 45 min is given.
The total number of reasoning video in all channels together is 2200

1) Quantity I: Total above 45 mins video in what percent of the total below 45 mins video of
channel A and B together of both subjects is both subjects of the same channel?

Click Here For Bundle PDF Course | support@guidely.in Page 1 of 13


SBI Clerk & RRB PO Mains PDF Course 2023
Quantitative Aptitude Day - 12 (Eng)

Quantity II: Total above 45 mins video in channel B.11.05% less


D and E together of math is what percent of total C.11.68% less
below 45 mins video of reasoning on the same D.15% less
channel? E.19% less
A. Quantity I < Quantity II
B. Quantity I ≥ Quantity II 4) 25% of videos on channel C have a total of
C. Quantity II ≥ Quantity I 100k views and 20% of videos on channel D
D. Quantity I > Quantity II have 100k views. Find the total number of 100k
E. Quantity I = Quantity II or relation can't be views of these two channels?
established A.476
B.530
2) Find the difference between the total above 45 C.543
mins video of math in all channels together and D.523
the total below 45 mins video of reasoning in all E.556
channels together?
A.383 5) Quantity I: find the total number of above 45
B.348 mins videos in channel B and C together?
C.492 Quantity II: Find total number of below 45 mins
D.392 videos in channel A and E together?
E.302 A. Quantity I < Quantity II
B. Quantity I ≥ Quantity II
3) Total number of math’s video in channel A, B C. Quantity II ≥ Quantity I
and C together is what percent more or less than D. Quantity I > Quantity II
the total number of reasoning videos in channel E. Quantity I = Quantity II or relation can't be
C,D and E together? established
A.12.01% less

Direction (6 – 8): Read the following information carefully and answer the questions based on it.
The given bar graph shows the percentage of the Laptops, Desktops and Macs sold by company A, B
and C out of the products manufactured by the companies.

Click Here For Bundle PDF Course | support@guidely.in Page 2 of 13


SBI Clerk & RRB PO Mains PDF Course 2023
Quantitative Aptitude Day - 12 (Eng)

6) The difference between the number of Macs unsold by company B. Where, K = 800 + 11.5%
manufactured by company A and B is same as of the number of laptops manufactured by
that of the company B and C. If the number of company A.
Macs sold by company B and C is same, then a) 1200
find the number of Macs manufactured by the b) 4000
company A is what percentage of the company c) 3500
B. (A manufactured highest number of Macs and d) 2800
C manufactured the lowest number of Macs) e) Can’t be determined
a) 69.23%
b) 133.33% 8) If the sum of the desktops sold by company A
c) 144.44% and B is 2586 and the sum of the desktops sold
d) 125% by company A and C is 3930. If the number of
e) None of these desktops sold by company C is 87.5% more than
that of company B, then find the number of
7) If the number of Macs manufactured by the desktops manufactured by company C?
company A and B is 40% more than the number a) 4200
of Laptops manufactured by each company A b) 2560
and B. The average number of Macs sold by c) 1536
company A and B together is K more than the d) 1050
average number of laptops sold by company A e) None of these
and B together. Then find the number of Laptops

Click Here For Bundle PDF Course | support@guidely.in Page 3 of 13


SBI Clerk & RRB PO Mains PDF Course 2023
Quantitative Aptitude Day - 12 (Eng)

Directions (9 - 14): Study the following information carefully and answer the questions given below.
The below funnel graph shows the total number of populations, registered voters, cast votes, valid votes
and percentage of the vote got by party A. In each village party, A, B and C contest the election.

Note – i. (m + n + o) = 110
ii. m% of 360 - 62 = 0,
iii. o + 120 + 300 – 260 = 200, number of invalid votes in villages X, Y, and Z is 500, 1000, and 1000
respectively. The percentage of votes by party A depends on the valid vote.
9) Number of non registered voters of villages X 10) Find the difference in the total votes got
and Y together is what percent of the number of parties A and B together in village X and parties
non registered voters of villages Y and Z A and B in village Y together? [If ratio of votes
together? get by parties A and B is 5:3 in village X and 5:2
a) 12m% in village Y]
b) 13m% a) 5Q + 1960
c) 15m% b) 2P + 1960
d) 19m% c) R/2 + 1960
e) 20m% d) 2Q/5 + 1960
e) 3Q + 1960

Click Here For Bundle PDF Course | support@guidely.in Page 4 of 13


SBI Clerk & RRB PO Mains PDF Course 2023
Quantitative Aptitude Day - 14 (Eng)

11) Ratio of the number of eligible voters who did a) 20000


not cast their vote in villages X and Y together b) 22000
and the number of eligible voters who did not c) 25000
cast their vote in villages Y and Z together? d) 18000
a) 2:3 e) None of these
b) 4:5
c) 1:9 Directions (15-20): Study the following data
d) 7:5 carefully and answer the questions given below:
e) None of these 15) Two inlet pipes P and Q and an outlet pipe R
is used to fill a tank. Find the time taken by all
12) Total invalid votes of villages X and Y the three pipes together to fill the tank?
together is what percent of the total votes got by Statement I: Time taken by pipe P alone and
parties B and C together in village Z? pipes P and R together to fill the tank is 6 hours
a) 16.66% and 12 hours respectively.
b) 15.35% Statement II: Time taken by pipes P and Q
c) 10.32% together to fill the tank is 24/7 hours.
d) 18.33% Statement III: Pipe R alone can empty the tank in
e) None of these 12 hours.
a) Statements I and II together are sufficient.
13) Total number of people in village D is [P + Q] b) Statements I and III together are sufficient.
and the population of village E is m% more than c) Statements II and III together are sufficient.
that of village D then find the population in E? d) Either statements I and II together or
a) 23650 Statements II and III together are sufficient.
b) 22450 e) Either statements I and III together or
c) 25680 Statements II and III together are sufficient.
d) 20140
e) None of these 16) Find the value of ‘D’?
Statement I: A boat can go ‘D + 15’ in
14) The ratio of the number of male to female downstream in 2.4 hours.
population in village A is m:n and the total Statement II: Time taken by boat to go ‘D – 20’
population in village A is 8000 more than that of km in downstream speed is 1 hour.
village Z. Find the difference between the Statement III: Speed of boat in still water is 4
number of male and female population in village times of the speed of stream of the river.
A.

Click Here For Bundle PDF Course | support@guidely.in Page 5 of 13


SBI Clerk & RRB PO Mains PDF Course 2023
Quantitative Aptitude Day - 14 (Eng)

a) Either statements I and II together or Statement II: Respective interests earned in both
Statements II and III together are sufficient. SI & CI after 2 years and 3 years are Rs. 13800
b) Statements I and III together are sufficient. and Rs. 22450.
c) Statements II and III together are sufficient. a) The data in statement I alone is sufficient to
d) Either statements I and III together or answer the question.
Statements II and III together are sufficient. b) The data in statements I and II together are
e) Statements I and II together are sufficient. required to answer the question.
c) The data in statement neither I nor II are
17) Three persons P, Q, and R enter into a sufficient to answer the question.
partnership. Find the ratio of profit share d) The data in statement II alone is sufficient to
received by them? answer the question.
Statement I: Ratio of initial investment of P, Q, e) The data in statement either I or II are required
and R is 4: 2: 5 respectively. to answer the question.
Statement II: Ratio of time period of the
investment of P and R is 5: 6. 19) Two ships M and N begin to move towards
Statement III: Time period of investment of Q is 2 each other. Initial distance between the ships is
months more than the time period of investment 300km. then what is the approximate time taken
of R. by ships for collision?
a) Statements I and III together are sufficient. Statement I: Respective ratio of upstream speed
b) All the statements I, II, and III together are not of ship M and speed of ship N in still water is 4:1.
sufficient. Respective ratio of speed of ship N in still water
c) Statements I and II together are sufficient. and speed of stream is 3:2.
d) Statements II and III together are sufficient. Statement II: Average of downstream speed of
e) All the statements I, II, and III together are boat M and downstream speed of boat N is
sufficient. 21kmph.
Statement III: Downstream speed of boat M is
18) Priya invested some amount in two different 12kmph.
schemes, P and Q. Scheme P offered 20%p.a. in a) The data in statement I alone is sufficient to
SI and scheme Q offered 25%p.a. in CI. How answer the question.
much amount was invested? b) The data in statements I and III together or I
Statement I: Difference of interest earned in 2 and II togetheris required to answer the question.
years is Rs. 4200. c) The data in statement III alone is sufficient to
answer the question.

Click Here For Bundle PDF Course | support@guidely.in Page 6 of 13


SBI Clerk & RRB PO Mains PDF Course 2023
Quantitative Aptitude Day - 14 (Eng)

d) The data in statements II alone is required to Statement III: Probability of drawing a grey ball is
answer the question. 3/10.
e) The data in statements III and II together are a) The data in statement I alone is sufficient to
required to answer the question. answer the question.
b) The data in statements I, II and III together are
20) A bag contains balls of 4 different colours required to answer the question.
i.e., pink, yellow, green and grey. Total no. of c) The data in statement II alone is sufficient to
balls are (m - 10). Respective ratio of number of answer the question.
pink, green and grey balls is 10:5:9. If a ball is d) The data in statements III aloneis sufficient to
randomly drawn, then find the total no. of balls. answer the question.
Statement I: Probability of drawing a yellow ball e) The data in any two statements together are
is 1/5. required to answer the question.
Statement II: Number of green balls are 2 less
than number of yellow balls.
Click Here to Get the Detailed Video Solution for the above given Questions
Or Scan the QR Code to Get the Detailed Video Solutions

Answer Key with Explanation

Directions (1-5): Number of above 45 mins video of math=484-


Reasoning video in channel 121=363
A=2200*18/100=396 Number of below 45 mins video of
So, the math video in channel A= reasoning=396*2/3=264
[396/45]*55=484 Number of above 45 mins video of
Number of below 45 mins video of math reasoning=396-264=132
=484*1/4=121 Similarly, we can calculate for other channels
also.

Click Here For Bundle PDF Course | support@guidely.in Page 7 of 13


SBI Clerk & RRB PO Mains PDF Course 2023
Quantitative Aptitude Day - 14 (Eng)

So required percentage ={[1540-


1360]/1540}*100=11.68% less

4) Answer: B
Total 100k views video in C =2*660*25/100=330
Total 100k views video in
D=450*20/100+550*20/100=90+110=200
So total number of 100k views =330+200=530

5) Answer: D
1) Answer: D
Quantity I:
Quantity I:
Total above 45 min video in B and C together
Required percentage
=120+198+264+396=978
=[(363+132+120+198)/(121+96+264+66)]*100
Quantity II:
=813/547*100
Total below 45 min video in A and E together
=148.62%
=121+198+264+198=781
Quantity II:
Quantity I > Quantity II
Required
percentage=[(300+297)/(250+198)]*100 =
Directions (6-8):
597/448*100
Tabulate the Data for further Convenience,
=133.25%
Quantity I >Quantity II

2) Answer: E
Required difference = [363+120+264+300+297]-
[264+66+264+250+198]
6) Answer: C
= 1344 - 1042
According to the Question,
=302
A–B=B–C
A + C = 2B ------- (1)
3) Answer C
Also, 50% of B = 90% of C
Total number of math video in A, B and C
B/C = 9x/5x--------- (2)
=484+216+660=1360
From (1) and (2),
Total number of reasoning video in C, D and E
A + 5x = 2*9x
=660+550+330=1540

Click Here For Bundle PDF Course | support@guidely.in Page 8 of 13


SBI Clerk & RRB PO Mains PDF Course 2023
Quantitative Aptitude Day - 14 (Eng)

A = 13x Desktops sold by company C – Desktops sold by


The required % = 13x/9x * 100 = 144.44% company B
Hence, the answer is option C = 1344
Also, Desktops sold by company C = 187.5% of
7) Answer: D desktop sold by the company B
According to the question, So, 187.5 of the desktops sold by the company
Let, the number of Laptops manufactured by the B – 100 of the desktop sold by company B =
company A and B is 100a and 100b respectively. 1344*100
The number of Macs manufactured by company Desktops sold by the company B = 1344/87.5 x
A and B is 140a and 140b respectively. 100 = 1536
Now according to the question, Desktops sold by the company C = 187.5% of
(45% x 140a + 50% of 140b)/2 – (40% x 100a + 1536 = 2880
30% x 100b)/2 = 800 + 11.5% of 100a Desktops manufactured by the company C =
63a + 70b – 40a – 30b = 1600 + 23a 2880/80 x 100 = 3600
23a + 40b = 1600 + 23a Hence, the answer is option E
40b = 1600 Directions (9 - 14):
So, the value of b = 40 m% of 360 - 62 = 0, so, m = 36 * 10/36 = 10
Number of laptops manufactured by company B o + 120 + 300 – 260 = 200
= 100b Or, o = 200 + 260 – 300 – 120 = 40.
Number of laptops sold by company B = 30b Or, m + n + o = 110, n = 110 – 40 – 10 = 60
Number of laptops unsold by company B= 100b P = 13000 – 500 = 12500, Q = 10000 – 1000 =
– 30b = 70b 9000, R = 16000 – 1000 = 15000
= 70 x 40 = 2800
Hence, the answer is option D

8) Answer: E
According to the question,
Sum of the Desktops sold by company (A + B) =
2586 ----- (1)
Sum of the Desktops sold by company (A + C) =
3930 ----- (2)
9) Answer: C
From (1) and (2),
Required percentage = [6000/4000] * 100 =
150% = 15m%

Click Here For Bundle PDF Course | support@guidely.in Page 9 of 13


SBI Clerk & RRB PO Mains PDF Course 2023
Quantitative Aptitude Day - 14 (Eng)

From equation (1):


10) Answer: D
Number of vote party B get in village x = 1250 *
3/5 = 750
Number of vote party B get in village y = 5400 * Pipe R alone can empty the tank = 12 hours

2/5 = 2160 From II,

So, required difference = [5400 + 2160] - [1250 +


750] = 5560 = 2Q/5 + 1960 Time taken by all the three pipes together to fill
the tank =
11) Answer: B
= 4.8 hours
Required ratio = [1000 + 3000]: [3000 + 2000] =
Statements I and II together are sufficient.
4000:5000 = 4:5
From I and III together:
From I,
12) Answer: A
According to the question:
Required percentage = [(500 + 1000)/ (15000 -
Time taken by P alone = 6 hours ……….. (1)
6000)] * 100 = 16.66%

13) Answer: A From equation (1):


Total number of people in village E = [12500 +
9000] * 110/100 = 23650

Pipe R alone can empty the tank = 12 hours


14) Answer: A
We did not have information about pipe Q.
Total population in A = 20000 + 8000 = 28000
Hence, we cannot determine the time taken by
m = 10 and n = 60
all the three pipes together to fill the tank.
Required difference = [60 – 10]/[60 + 10] * 28000
Statements I and III together are not sufficient.
= 20000
From II and III together:
From II,
15) Answer: D
According to the question:
From I and II together:
From I,
According to the question: Pipe R alone can empty the tank = 12 hours
Time taken by P alone = 6 hours ……….. (1) Time taken by all the three pipes together to fill
the tank =

Click Here For Bundle PDF Course | support@guidely.in Page 10 of 13


SBI Clerk & RRB PO Mains PDF Course 2023
Quantitative Aptitude Day - 14 (Eng)

= 4.8 hours 17) Answer: B


Statements II and III together are sufficient. From I, II, and III together:
Let the initial investment of P, Q, and R is ‘4x’,
16) Answer: E ‘2x’, and ‘5x’ respectively.
From I and II together: Let the time period of investment of P and R is
Let speed of boat in still water and speed of ‘5y’ and ‘6y’ respectively.
stream is ‘x’ km/hr and ‘y’ km/hr respectively. Time period of investment of Q = (6y + 2)
Ratio of profit share of P, Q, and R = (4x * 5y):
[2x * (6y + 2)]: (5x * 6y)
= 10y: (6y + 2): 15y
We cannot determine the desired ratio.
Statements I, II, and III together are not
D – 20 = x + y ………. (2)
sufficient.
From (1) and (2):
D + 15 = 2.4 * (D - 20)
18) Answer: D
D = 45 km
Let the amount invested on SI = Rs. ‘a’
Value of D = 45 km
Let the amount invested on CI = Rs. ‘b’
Statements I and II together are sufficient.
Scheme P offered 20% SI
From I and III together:
Scheme Q offered 25% CI
Let speed of boat in still water and speed of
Statement I: Difference of interest earned in 2
stream is ‘x’ km/hr and ‘y’ km/hr respectively.
years = Rs. 4200
Principle amount is different for both schemes
x = 4y ………… (4) So, Statement I alone is not sufficient to answer
Two equations are not sufficient to calculate the the question.
value of 3 variables. Statement II: Interest earned after 2 years = Rs.
Statements I and III together are not sufficient. 13800
From II and III together: (20% x a x 2) + [b * (1.25)² – 1] = 13800
Let speed of boat in still water and speed of 32a + 45b = 1104000------ 1
stream is ‘4x’ km/hr and ‘x’ km/hr respectively. Interest earned after 3 years = Rs. 22450
(20% x a x 3) + [b * (1.25)³ – 1] = 22450

D – 20 = 4x + x = 5x 192a + 305b = 7184000---- 2

Statements II and III together are not sufficient. From both the equations,
a = Rs. 12000 and b = Rs. 16000

Click Here For Bundle PDF Course | support@guidely.in Page 11 of 13


SBI Clerk & RRB PO Mains PDF Course 2023
Quantitative Aptitude Day - 14 (Eng)

Total Rs. 28000 was invested M + S = 12


Statement II alone is sufficient to answer the 14x + 2x = 12
question. So, x = 3/4
Hence, the answer is option d) Thus, M = 42/4 and N = 9/4
Time taken by ships for collision = 300/(51/4) =
19) Answer: B 23.5 hours
Let the speed of ship M = ‘M’ kmph Statements I and III together are sufficient to
Let the speed of ship N = ‘N’ kmph answer the question.
Let the speed of stream = ‘S’ kmph Combining statements, I and II
Distance between two ships = 300km N/S = 3/2
Statement I: Speed of ship N in still water: speed N:S = 3x:2x
of stream = 3:2 (M - S): N = 4: 1
N/S = 3/2 So, (M – 2x): 3x = 4: 1
Upstream speed of ship M: speed of ship N in So, M = 14x
still water = 4: 1 Downstream speed of boat M + downstream
(M - S): N = 4: 1 speed of boat N = 42kmph
So, (M – 2x): 3x = 4: 1 =16x + 5x = 21x = 42 , x=2
So, M = 14x From that 300/(28+6)= 8.8hours
So, statement I alone is not sufficient to answer Hence, the answer is option b)
the question.
Statement II: Downstream speed of boat M 20) Answer: E
+downstream speed of boat N = 42kmph Total no. Of balls = (m - 10)
300 / (M + S + N + S) = 42 Number of pink balls: green balls: grey balls =
Time taken by ships for collision = 7.14 hours 10:5:9
Hence, the statement II alone is sufficient to A ball is randomly drawn
answer the question. Let the total no. Of pink balls = 10x
Statement III: Downstream speed of boat M = Let the total no. Of green balls = 5x
12kmph Let the total no. Of grey balls = 9x
M + S = 12 So, total no. Of yellow balls = m – 10 – 24x
Hence, statement III alone is not sufficient to Statement I: Probability of drawing a yellow ball
answer the question. = 1/5
Combining statements, I and III (m – 10 – 24x)/ (m – 10) = 1/5
Since, S = 2x and M = 14x 5m – 50 – 120x = m - 10

Click Here For Bundle PDF Course | support@guidely.in Page 12 of 13


SBI Clerk & RRB PO Mains PDF Course 2023
Quantitative Aptitude Day - 14 (Eng)

4m – 120x = 40 9x /(m – 10) = 3/10


Hence, the statement I alone is not sufficient to m - 30x = 10
answer the question. Hence, the statement III alone is not sufficient to
Statement II: yellow balls - green balls = 2 answer the question.
(m – 10 – 24x) – 5x = 2 Combining any two statements
So, m – 29x = 12 x = 2 and m = 70
Hence, the statement II alone is not sufficient to Total balls = 60
answer the question. If we combine any two statements, then we get
Statement III: Probability of drawing a grey ball = our answer
3/10 Hence, the answer is option e)

Click Here For Bundle PDF Course | support@guidely.in Page 13 of 13


SBI Clerk & RRB PO Mains PDF Course 2023
ENGLISH Day - 14

English Language

Directions (1-7): Read the given passage stringent policies or due to less stringent policies
carefully and answer the following questions. in those countries. This, the EU worries, would
The European Union’s (EU) key climate law, the put its industries at a disadvantage. To tackle
Carbon Border Adjustment Mechanism (CBAM), this, the impacted industries in the EU had so far
has spooked India. New Delhi fears that CBAM been receiving free allowances or permits under
will cripple (A) the export of its carbon-intensive the ETS. Furthermore, the EU also apprehends
products to the EU. While India’s exports may be the phenomenon of ‘carbon leakage’, that is, due
limited to aluminium, iron, and steel, and affect to the application of ETS, European firms
only 1.8% of its total exports to the EU, India has operating in carbon-intensive sectors might
reportedly decried CBAM as being protectionist possibly shift to those countries that have less
and discriminatory. There is also talk of stringent GHG emission norms.CBAM is aimed
challenging the CBAM at the World Trade at addressing this quagmire, and, thus, levelling
Organization (WTO)’s dispute settlement body. the playing field for the EU industries . Under the
This debate brings to the forefront the inter- CBAM, imports of certain carbon-intensive
linkages between trade and the environment. products, namely cement, iron and steel,
While the international trade regime allows electricity, fertilisers, aluminium, and hydrogen,
countries to adopt unilateral measures for will have to bear the same economic costs borne
safeguarding the environment, environmental by EU producers under the ETS. The price to be
protection should not become a smokescreen for paid will be linked to the weekly average of the
trade protectionism. The CBAM needs to be emissions priced under the ETS. However,
viewed from this standpoint.In 2005, the EU where a carbon price has been explicitly paid for
adopted an important climate change policy the imported products in their country of origin, a
known as the Emissions Trading System (ETS). reduction can be claimed.
Now in its fourth stage, the ETS is a market- A cornerstone principle of WTO law is non-
based mechanism that aims at reducing discrimination. Thus, countries are required to
greenhouse gas (GHG) emissions by allowing accord (B) equal treatment to ‘like’ products
bodies emitting GHG to buy and sell these irrespective of their country of origin (most-
emissions amongst themselves. However, the favoured nation treatment) and to treat foreign-
EU’s concern is that while it has a mechanism for made ‘like’ products as they treat domestic ones
its domestic industries, emissions embedded in (national treatment principle). While the CBAM’s
products imported from other countries may not design is origin-neutral in appearance, it may, in
be priced in a similar way due to a lack of its application, discriminate between goods from

Click Here For Bundle PDF Course | support@guidely.in Page 1 of 12


SBI Clerk & RRB PO Mains PDF Course 2023
ENGLISH Day - 14

different countries on account of an inadequate under this category. However, it is doubtful if it


carbon pricing policy, or due to onerous reporting would satisfy the chapeau, which inter alia
requirements that importers would be subject requires that countries do not apply measures in
to.A key consideration in all this may be whether a manner that results in arbitrary or unjustifiable
the carbon-intensive products to which the discrimination between countries where the
CBAM applies are ‘like’. While steel products same conditions prevail. As commentators
may appear similar, the process by which electric argue, the CBAM only considers ‘explicit’ carbon
arc furnaces produce steel is less carbon- prices, not ‘implicit’ costs (non-price-based costs)
intensive than the steel produced in blast borne by products originating in certain countries.
furnaces, for instance. Being products that are Accordingly, it arbitrarily or unjustifiably
not ‘like’, the rules on non-discrimination would discriminates between countries where the same
have little application in such a case. environmental conditions exist.Finally, CBAM is
Accordingly, the debate on CBAM reignites a also an important issue in the ongoing India-EU
long-standing debate in international trade law free trade agreement negotiations. India should
circles: must processes and production methods work with the EU to secure gains on CBAM and
be relevant for comparing products? ___________________________________________
Traditionally, the answer to that in WTO __, even as the possibility of a WTO challenge
jurisprudence has been no, and, on that account, remains open.
as commentators have noted, the CBAM violates 1) How is India affected by Europe's key climate
WTO law for discriminating between EU and policy, the CBAM ?
foreign products covered by CBAM based on the I. India will not be affected by the new policy that
embedded emissions.However, even if the EU’s has been introduced by Europe because the
CBAM is discriminatory, there could be a claim country’s export that involves carbon is only
for justifying it under the General Exceptions 1.8% of the total exports.
clause given in Article XX of the General II. The capital fears that the policy will eventually
Agreement on Tariffs and Trade (GATT). Under ban the imports of carbon intensive products
Article XX, measures taken by countries that from India.
otherwise violate GATT obligations are permitted III. India considers the Carbon Border
if, first, they fall under one of the listed policy Adjustment Mechanism to impose trade
grounds, and second, if they satisfy the protectionism by protecting Europe’s own
requirements of the introductory clause of Article industry.
XX, known as the chapeau. One of the listed IV. India sees Europe’s CBAM as an act of
policy grounds in Article XX is ‘conservation of discrimination just to favour the domestic players
exhaustible natural resources’. CBAM would fall and impose burden on foreign ones.

Click Here For Bundle PDF Course | support@guidely.in Page 2 of 12


SBI Clerk & RRB PO Mains PDF Course 2023
ENGLISH Day - 14

V. There is no direct effect on India due to the I. The European firms are trying to move to
policy because India does not have any stringent other countries which have laws which are less
policy like CBAM. strict in terms of greenhouse emissions, this will
a) Only I impact the European economy.
b) Both I and II II. The imports of the emissions embedded
c) Both III and IV products from other countries may not be priced
d) I,III and IV in a similar way when compared to the price fixed
e) Both I and V for the local firms and this may be
disadvantageous for the domestic market.
2) According to the given passage, which of the III.Exports of certain carbon-intensive products
following statements are true regarding the like cement, iron and steel, electricity, fertilisers,
Emissions Trading System (ETS) ? aluminium, and hydrogen have been banned
I. The system aims to reduce greenhouse gas because of the implementation of ETS.
(GHG) emissions by allowing buying and selling IV.The domestic market has many advantages
the emissions only between the bodies that emit than the disadvantages after the ETS
GHG. implementation.
II. ETS was introduced a decade ago and now it a) Only I
is replaced by Carbon Border Adjustment b) Only II
Mechanism (CBAM). c) Both I and II
III.Under ETS all the domestic players are given d) Both III and IV
free allowances and permits. e) Only IV
IV. Due to ETS implementation, chances are
there for the European players in the carbon- 4) According to the passage, Article XX of the
intensive sectors to shift to other countries with General Agreement on Tariffs and Trade has two
less stringent GHG rules. clauses by which Europe can justify CBAM.
a) Both I and II There are five statements given below regarding
b) Both II and III the two clauses and their usage, select the
c) Both III and IV option which comprises the statement(s) which
d) Both I and III are true and valid as per the passage?
e) Both I and IV I. First clause of Article XX has a listed policy
which is ‘conservation of exhaustible natural
3) How is Europe's domestic market affected by resources’.
its own ETS policy? II. CBAM clearly falls under the clause I category
and can be justified.

Click Here For Bundle PDF Course | support@guidely.in Page 3 of 12


SBI Clerk & RRB PO Mains PDF Course 2023
ENGLISH Day - 14

III.The CBAM cannot possibly prove the second d) ensure smooth onboarding for European
clause which involves a chapeau. importers to maximise the benefits of a bilateral
IV. The chapeau requires that countries do not deal
apply measures in a manner that results in e) None of these
arbitrary or unjustifiable discrimination between
countries where the same conditions prevail. 7) From the given options select a suitable title
V. Europe’s CBAM is in a way an unjustifiable for the passage.
discrimination. a) Europe’s ETS vs Europe’s CBAM
a) Both I and II b) Trade and stringent norms
b) Both I and III c) Greenhouse gases
c) I, II and III d) Climate law by Europe
d) III, IV and V e) All of these
e) All I, II, III, IV and V
Directions (8-12): In each of the questions given
5) Which of the following pairs can possibly below two phrases in the sentence have been
replace the words (A) and (B) respectively? highlighted. It may or may not need
a) Boost, disagree improvement. Read the question carefully and
b) Enable, remove choose an option that would best replace the
c) Damage, give highlighted parts to make a grammatically correct
d) Disable, grant sentence. In case, the sentence is correct in its
e) Hurt, agree current form and there is no improvement
needed, mark (E) as your answer.
6) Complete the given blank to make the 8) “We have requested that all FIRs against
sentence grammatically and contextually correct wrestlers should be taken off and the minister
and meaningful. has agreed to it and if no action is taken within a
a) ensure smooth onboarding for Indian week’s time, we will continue our protest”, told
importers to maximise the benefits of a bilateral the wrestler in the press meeting which started
deal around 11AM in the morning.
b) ensure smooth onboarding for Indian a) If no action is taken in a week time, we shall
exporters to maximise the benefits of a bilateral continue our protest
deal b) Against wrestlers should be taken back
c) ensure smooth onboarding for European c) Against wrestlers should be taken out
exporters to maximise the benefits of a bilateral d) If any action is taken within a week’s time, we
deal will continue our protest

Click Here For Bundle PDF Course | support@guidely.in Page 4 of 12


SBI Clerk & RRB PO Mains PDF Course 2023
ENGLISH Day - 14

e) No improvement needed b) Whatever the criticisms are against national


cricket boards
9) The paramilitary forces has created a security c) Spread of the game is important to the growth
grid and are patrolling “buffer areas” among the of the players
Imphal valley and adjoining hill areas, where the d) Spreading of the game is as important as the
Kuki people live, to prevent any violence, can growth of the players
only be of limited help. e) No improvement needed
a) The paramilitary forces that have created a
security grid 12) On a hot June day, Phuphee had gone to
b) Between the Imphal valley and adjoining hill inspect the paddy fields when she bumped into
areas the only teacher who worked at the school and
c) The paramilitary forces that has created a he informed her that attendance had fallen in the
security grid past few months and if things carried on like this,
d) Both a and b the authorities would probably close the school.
e) No improvement needed a) When she bumped onto the teacher who
worked at the school
10) This means that if a consumer who has an b) Informed her that attendance has fallen in the
average limit of 100 units uses around 150 units past few months
of power in a month, then they would have to pay c) When she bumped into the only teacher who
their bill for the additional 50 units. used to work at the school
a) If consumers who have an average limit of d) Let her knew that hr attendance has fell in the
b) The he/she would have to pay their bill past few months
c) If consumer who has an average limit of e) No improvement needed
d) Then they have to pay their bill
e) No improvement needed Directions (13-16): In the following questions,
three statements and five possible pairs of
11) Whatever the criticism are against national connectors to combine the sentences are given.
cricket boards like the BCCI, the fact remains Only one of the pairs from those given can be
that cricket is their main concern; the used to combine the given three statements into
development, advancement and spread of the one sentence without changing the meaning.
game is as important as the growth of the Choose that option which has the correct pair of
players for their respective countries. connectors as your answer.
a) Whatever the criticisms is against national 13)
cricket boards

Click Here For Bundle PDF Course | support@guidely.in Page 5 of 12


SBI Clerk & RRB PO Mains PDF Course 2023
ENGLISH Day - 14

I. Great batsmen play to their strengths and e) But, but


compensate for their weaknesses
II. Gavaskar did in ODI, and Kohli is doing in T20 16)
III.Root could too, playing his own game I. Any relationship, let it be friendship,
a) As, because companionship is stronger if it is bound by love
b) Like, though II. By anger, as genuine love and affection can
c) As, so heal and change a person completely
d) Although, and III.She/he has an unpleasant past.
e) Like, because a) Than, with
b) Rather than, even if
14) c) Rather, even though,
I. The BJP wants itself to be a Hindutva straight- d) Instead, although
jacket into which every other community identity e) And, even
must be subsumed, under national leadership
II. giving representation to a wide range of Directions (17-21): In the questions below, a
communities sentences is given with two blanks. The given
III. what it abhors is autonomous caste blanks will be filled by different word(s), you have
assertions of all kinds to find the correct word(s) which can fill both
a) While, and blanks to make the sentence contextually and
b) When, but grammatically correct. Choose the option which
c) With, because has the correct pair of words as your answer.
d) By, also 17) Self-interest is not such a bad thing if it
e) And, as _________with universal interest, but it
_______does and a decade after the Big Three’s
15) hijack bid, the ICC has proposed a similar
I. I won't go to her retirement party surplus revenue-sharing model.
II. She invited me personally and also made a a) Matches, never
phone call to remind me of the date b) Coincides, seldom
III.I have an important client interview on the c) Aligned, rarely
same date. d) Goes, not
a) Even though, because e) Abides, occasionally
b) As, as
c) Yet, and 18) The current dispensation has sought to
d) When, while ________the founding principles of India’s

Click Here For Bundle PDF Course | support@guidely.in Page 6 of 12


SBI Clerk & RRB PO Mains PDF Course 2023
ENGLISH Day - 14

republican sovereignty, through the installation of ________subaltern communities. Mr. Modi plays
the _________symbol in Parliament. the emotional and material angles very well, and
a) Imagine, old his speeches invariably have an __________tone.
b) Reconsider, ancient a) Targeted, empowering
c) Reimagine, religious b) Focus, alluring
d) Thought, unique c) avoid,harsh
e) Revisit, same d) skipped,soft
e) Diverged, disqualifying
19) Like last year, in China, the perennial
________to find employment after graduation is 21) But on August 17, 2022, the State
especially dire this year and it is of crucial government wrote to the committee about its
importance to ________that the hundreds of __________to provide the land for the national
millions of college and university graduates last park as “the legal status of the land in question is
year and this year are also the ones forming the unclassed forest or community forest on which
first and second cohorts of this century. the local people have been enjoying customary
a) Ease, forget rights since time ___________”.
b) Fight, overlook a) Ability, memorial
c) Burden, ignore b) incapable,old
d) Struggle, remember c) Inability, immemorial
e) calm,neglect d) Capability, history
e) Capacity, history
20) The Union Budget recently was sharply
focused on welfare schemes that
Click Here to Get the Detailed Video Solution for the above given Questions
Or Scan the QR Code to Get the Detailed Video Solutions

Answer Key with Explanation

Click Here For Bundle PDF Course | support@guidely.in Page 7 of 12


SBI Clerk & RRB PO Mains PDF Course 2023
ENGLISH Day - 14

1) Answer: C 3) Answer: C
Statements III and IV are the effects of Europe’s Both statements I and II are the effects of
CBAM on India as mentioned in the given implementation of ETS on the domestic market
passage. of Europe.
The first few lines of the passage clearly states III and IV are definitely false as there is no such
that even though India’s exports which involve information provided in the passage.
carbon is only 1.8% of the total exports to
Europe, there will still be an impact on India’s 4) Answer: E
carbon intensive products. And India is worried All of the given statements are inferred from the
that this act will protect trade from other given passage above, they are true and valid
countries and favour the domestic players alone. according to the information provided in the
Statement I is wrong because even though the passage.
percentage of export is low, India is still affected
by the policy. 5) Answer: C
Statement II is false because the passage does The right words that can replace cripple and
not provide any information about the ban of accord are damage and give respectively.
imports. Meaning of cripple - to damage something
Statement V is false because CBAM policy Meaning of accord (verb as used in passage) -
directly affects the imports from the other to give, to agree to so something
countries, it does not depend on individual All the rest of the options have at least one
country’s laws. antonym each of the bold words.

2) Answer: E 6) Answer: B
Both the statements I and IV are true and valid India must ensure smooth onboarding of Indian
according to the given passage. Exporters to Europe to have the maximum
Statement II is wrong because ETS was benefit from the bilateral deal.
introduced in 2015 and it is not replaced by Other options are wrong because India is
CBAM. concerned about the Indian exporters to Europe
Statement III is wrong because not all the who are working in carbon intensive units.
domestic firms were given free allowances and
permits. Only the affected firms were treated 7) Answer: D
with benefits. Out of the given options, ‘d’ would be the best
suitable title because the first few sentences

Click Here For Bundle PDF Course | support@guidely.in Page 8 of 12


SBI Clerk & RRB PO Mains PDF Course 2023
ENGLISH Day - 14

have the introductory part which talks about the sentence to make it grammatically and
new climate law. contextually correct and meaningful.
Option a is not a right fit because the passage The paramilitary forces is a plural form and thus
does not compare ETS and CBAM has to be followed by ‘have’. Also we must add
Option b cannot be the right answer because it is
‘that’ because the sentence is explaining a
very general and not specific
scenario and later is followed by a comment on
Option c cannot be the answer because
it(can only be of limited help). So, pointing out
greenhouse gases are just a part of the passage
and not the whole context. the scenario is a must here. “ The paramilitary

forces that have created a security grid” is the


8) Answer: B correct phrase.
The FIRs given must be taken back is the In the second phrase, among must be replaced
correct way to put the phrase. with ‘between’, because among is used only to
To take back is to ‘undo’ something. address more than two entities. Between is used
The second phrase that has been highlighted is to address exactly two(Imphal and the adjoining
already correct as it is and requires no hill areas).
improvement. ( so we can eliminate the options The correct sentence :The paramilitary forces
a and d) that have created a security grid and are
Take back is the right phrasal verb that has to be patrolling “buffer areas” between the Imphal
used in the sentence, valley and adjoining hill areas, where the Kuki
Take off - to leave/to start, Take out - to remove people live, to prevent any violence, can only be
something. (eliminate option c) of limited help.
The correct sentence : “We have requested that
all FIRs against wrestlers should be taken back 10) Answer: E
and the minister has agreed to it and if no action The given sentence is correct and meaningful
is taken within a week’s time, we will continue without any errors. Hence, no correction is
our protest”, told the wrestler in the press needed.
meeting which started around 11AM in the The first phrase is correct so leave it as it is,
morning. coming to the second phrase the confusion is
regarding the pronoun that is used (they), the
9) Answer: D pronoun ‘they’ is completely fine to address the
Option d is the right answer, both a and b are noun ‘consumer’ which can either be a singular
required corrections that have to be made to the noun or plural noun.

Click Here For Bundle PDF Course | support@guidely.in Page 9 of 12


SBI Clerk & RRB PO Mains PDF Course 2023
ENGLISH Day - 14

For these reasons there is no correction that has community identity must be subsumed, under
to be performed for the given sentence. national leadership, while giving representation
to a wide range of communities and what it
11) Answer: B abhors is autonomous caste assertions of all
‘Whatever the criticisms are’ is the right way to kinds.
put it. In the other options either a single connector or
Criticism is singular and must be followed by ‘is’ both the connectors given is wrong and they
Criticisms is plural and must be followed by ‘are’ make the sentence completely wrong.
This simple rule is applied to the first phrase and For eg : consider option c (with, because), The
the second phrase is correct as it is. BJP wants itself to be a Hindutva straight-jacket
into which every other community identity must
12) Answer: E be subsumed, under national leadership with
No improvement is needed. The given sentence giving representation to a wide range of
is correct as it is and the two highlighted phrases communities.
need no improvement. As you read the given above sentence, one can
clearly identify that ‘with’ is not the right
13) Answer: C connector and it is inappropriate in a way even
The connectors ‘as’ and ‘so’ are the best to though it is not changing the complete meaning
connect the three statements to form a complete of the sentence.
sentence with meaning.
The sentence: Great batsmen play to their 15) Answer: A
strengths and compensate for their weaknesses The right connectors are Even though and
as Gavaskar did in ODI, and Kohli is doing in because. Other connectors if fit between the
T20, so, Root could too, playing his own game. given statements alter the original
Other options have inappropriate connectors like meaning/context of the sentence
though, because,like which does not make any Even though - despite the fact that
sense. Despite the fact that she invited me, I cannot
attend because of my interview.
14) Answer: A The sentence : I won't go to her retirement party
The connectors ‘while and and’ are the correct even though he invited me personally and also
ones to connect the given statements. made a phone call to remind me of the date
The sentence : The BJP wants itself to be a because I have an important client interview on
Hindutva straight-jacket into which every other the same date.

Click Here For Bundle PDF Course | support@guidely.in Page 10 of 12


SBI Clerk & RRB PO Mains PDF Course 2023
ENGLISH Day - 14

18) Answer: C
16) Answer: B Reimagine and religious is the correct pair of
Rather than and even is the right pair of words that are required to fill the blanks.
connectors. The sentence : The current dispensation has
The sentence :Any relationship, let it be sought to reimagine the founding principles of
friendship, companionship is stronger if it is India’s republican sovereignty, through the
bound by love rather than by anger, as genuine installation of the religious symbol in Parliament.
love and affection can heal and change a person Other options are wrong for the following reasons
completely even if she/he has an unpleasant : Imagine and old are not the right fits, because
past. one reimagines the past and old symbol does not
Rather than - instead of make any sense (even though the symbol is old
Even if - although, even in the case of but the sentence here talks about the authenticity
in a religious way)
17) Answer: B Reconsider is a wrong word to be used,
Coincides and seldom fits the two given blanks reconsider meaning you consider something for
correctly. the second time
Coincides - to be exactly the same of something Option d - thought does not fit the blank
else Option e - same does not add meaning to the
Seldom - rarely sentence
The sentence : Self-interest is not such a bad
thing if it coincides with universal interest, but it 19) Answer: D
seldom does and a decade after the Big Three’s Struggle and remember is the right pair of words.
hijack bid, the ICC has proposed a similar The correct sentence : Like last year, in China,
surplus revenue-sharing model. the perennial struggle to find employment after
The other options are not right because : graduation is especially dire this year and it is of
Option a - never changes the whole meaning of crucial importance to remember that the
the sentence and hence not correct hundreds of millions of college and university
Option b - aligned is wrong form of usage, aligns graduates last year and this year are also the
would have been a correct word instead ones forming the first and second cohorts of this
Option d - not does not fit the second blank century.
Option e - abides with is completely wrong, The other options are wrong because :
instead abides by would be correct. The first blank - among the given option struggle
is the right fit. Calm and ease are just opposite to

Click Here For Bundle PDF Course | support@guidely.in Page 11 of 12


SBI Clerk & RRB PO Mains PDF Course 2023
ENGLISH Day - 14

the word that is needed for the blank.Fight and the PM should also be in a positive way.Other
burden has got a similar meaning to struggle but than empowering all the given tones of voice
are not the best options. take a negative form and hence doesn't go with
The second blank - all the four words ignore, the meaning of the sentence.
neglect, overlook and forget are just the exact
opposite words to the word ‘remember’ and they 21) Answer: C
don’t fit the blank because they alter the entire Inability and immemorial is the right pair of
meaning of the sentence. words.
The sentence : But on August 17, 2022, the
20) Answer: A State government wrote to the committee about
Targeted and empowering are the words that are its inability to provide the land for the national
apt for the given sentence. park as “the legal status of the land in question is
The sentence : The Union Budget recently was unclassed forest or community forest on which
sharply focused on welfare schemes that the local people have been enjoying customary
targeted subaltern communities. Mr. Modi plays rights since time immemorial”.
the emotional and material angles very well, and Immemorial - that existed longer than history
his speeches invariably have an empowering Other given options are wrong because :
tone. The words ability, capacity and capability are just
The other options are not right because: the opposite of what that the blank needs to be
First blank - targeted is the right fit, other given filled with ( so eliminate options a,d and e)
words are not in the right form and meaning and Option b - incapable although same meaning of
hence cannot fit the blank. inability, but the blank needs a noun not an
Second blank - The sentence is conveying a adjective.
message that is positive and hence the tone of

Click Here For Bundle PDF Course | support@guidely.in Page 12 of 12


SBI Clerk & RRB PO Mains PDF Course 2023
Reasoning Ability Day - 15 (Eng)

Reasoning Ability
Directions (1-5): Study the following information All the persons except senior to Additional
carefully and answer the given questions. General Manager are transferred to three
Twenty one persons are working in an different branches viz.- Head Branch, Urban
organization at different designations viz.- Brach, and Rural Branch as per below
Chairman, Director, Additional General Manager conditions:
(AGM), Manager, Assistant Manager (AM), and Condition I: The persons whose name comes
Clerk. The designations are given in descending before ‘M’ in alphabetical series and also even
order such as the Chairman is the seniormost number of persons working along with them are
designation whereas Clerk is the juniormost transferred to Head Brach.
designation. Condition II: The persons whose name comes
Note: I. The number of persons working in each after ‘M’ in alphabetical series and also odd
designation is one less than the number of number of persons working along with them are
persons working in their immediate junior transferred to Urban Brach.
designation. Condition III: All the remaining persons are
II. If A works with B then, both A and B are transferred to Rural Brach.
working in the same designation. The persons senior to Additional General
Odd number of persons are working along with Manager are appointed as a supervisor of all
K, who is immediately senior to H. The number of three branches viz.- Head Branch, Urban Brach,
persons junior to E is one more than the number and Rural Branch in alphabetical order
of persons designated between L and C. L is respectively.
designated senior to C and odd number of 1) ___ persons are transferred to Rural Branch
persons working in L’s designation. G and P are and supervised by ___.
working together, but neither with C nor with E. P a) 10, D
is two positions junior to F. Even number of b) 10, L
persons are senior to F. I and Q are working c) 8, L
together but immediately senior to O. The d) 6, J
number of persons senior to Q is the same as e) None of these
the number of persons working in Q’s
designation. U, B and S are working together but 2) Who among the following person is not
not works with E. Even number of persons supervised by J?
working between B and D, who is immediately a) S
senior to A. N and R working together. R is b) U
senior to M but junior to J. M is junior to T. c) P

Click Here For Bundle PDF Course | support@guidely.in Page 1 of 11


SBI Clerk & RRB PO Mains PDF Course 2023
Reasoning Ability Day - 15 (Eng)

d) K Directions (6-10): Study the following information


e) O carefully and answer the given questions.
A School organised an activity sessions for the
3) _____ persons are designated senior to students in a summer vacation. The sessions
_______. were held in between 10am and 6pm. six
a) 10, T sessions were held, two sessions for Yoga
b) 3, O activity, two sessions for dance activity and two
c) 16, N sessions for painting. Each session was held for
d) 6, A 1 hr. There are two breaks between the session
e) None of these viz. 1-2 pm lunch break and 4-5 pm Snacks
break.
4) Who among the following persons are working Note: No two continuous sessions were held for
in the same designation? the same activity.
I. PGS One of the Painting sessions was held after the
II. AQJ lunch break. One of the dance sessions was held
III. HEF 180 minutes before the Yoga session. Dance
a) Only I session was held neither immediately before nor
b) Only II and III immediately after the Lunch break. One of the
c) Only III Painting sessions was held 120 minutes after
d) Only I and III one of the Yoga sessions. Dance sessions were
e) Only II not held after the snacks breaks.
6) What is the time (Hrs) gap between both Yoga
5) The monthly salary of each designation is sessions?
decreased by 10K from chairman to Clerk, and if a) 2hrs
the monthly salary of the Chairman is 100K, then b) 4hrs
what is the sum of the monthly salaries of C, Q c) 3hrs
and B? d) 5hrs
a) 140K e) 6hrs
b) 220K
c) 200K 7) Which among the following session(s) is/are
d) 180K held after 3pm?
e) None of these a) Yoga
b) Painting
c) Dance

Click Here For Bundle PDF Course | support@guidely.in Page 2 of 11


SBI Clerk & RRB PO Mains PDF Course 2023
Reasoning Ability Day - 15 (Eng)

d) Both (a) and (b) Eight persons – P, Q, R, S, T, U, V, and W joined


e) Both (b) and (c) ALPHA company in 2020 in four different months
viz.- January, April, August and November on
8) Which of the following session was held from three different dates viz.- 9, 12, and 15. Only one
12 pm – 1 pm? person joined on each date of any month. At
a) Dance least one person joined in each month. Each
b) Either a or c person already has different experience between
c) Yoga 5 years and 20 years.
d) Painting Only two persons joined before V, who was born
e) Either c or d on 9th of the month. Only three persons joined
between V and Q, whose experience is 15 years.
9) Painting session was held at which of the Only three persons joined between the one who
following time? joined on 12th January and S, whose experience
a) 10-11 am is a perfect square. The difference between the
b) 12-1 pm experience of S and Q is equal to the experience
c) 2-3 pm of R, who joined two persons after the one
d) 5-6 pm whose experience is 12 years. The number of
e) Both (b) and (d) persons joined between R and W is one more
than the number of persons joined before the
10) Which of the following statement(s) is/are one whose experience is 14 years. W joined in
true with respect to the final arrangement? November and joined two persons after the one
a) Dance session was held immediately before who joined on 15th August. The one whose
Yoga session experience is 14 years joined either immediately
b) As many sessions held before 1st Dance after or immediately before P. The number of
session as after 2nd Yoga session. persons joined between S and P is one less than
c) Yoga session was held 120 minutes before the number of persons joined before the one who
Dance session. joined on 15th April. The experience of T is a
d) Two sessions were held between Snack break prime number and joined on 12th January. The
and Yoga session experience of U is ten years more than the
e) None is true experience of T. One of the persons whose
experience is a perfect cube number joined on
Directions (11-15): Study the following 12th of any of the months. No one joined on 9
information carefully and answer the given August.
questions.

Click Here For Bundle PDF Course | support@guidely.in Page 3 of 11


SBI Clerk & RRB PO Mains PDF Course 2023
Reasoning Ability Day - 15 (Eng)

Based on their working experience they are a) QVU


selected for a project following certain conditions: b) RQU
I. Project is offered to those who have completed c) PVU
minimum 3 years in Alpha (calculate their d) PVW
experience as on 31st August 2023). e) None of these
II. If two persons who joined in the same month
are selected then, the person whose total 15) _____ joined on 12 August and joined _____
experience is more is to be selected. persons after _____.
11) Experience of ______ is 12 years and joined a) U, Two, R
on _______. b) S, Three, P
a) U, 15 AUG c) U, Four, R
b) V, 9 JAN d) S, Two, P
c) Q, 12 NOV e) None of these
d) P, 15 JAN
e) None of these 16) If “1” is added to the odd digits and “2” is
subtracted from the even digits of the given
12) Who among the following person joined two numbers, then add all the digits within the
persons after U? number thus formed, then the resultant of which
a) The one whose experience is 6 years of the following number is a multiple of four?
b) W I. 45862745
c) The one whose experience is 15 years II.78437617
d) V III. 18467624
e) None of these IV. 57136182
a) Only III
13) Which of the following combination of person b) Only I and IV
and their experience is correct? c) Only I, II, and IV
a) V – 9 d) Only III and IV
b) U – 17 e) None
c) S – 16
d) T – 11 17) If the vowels in the following words are
e) None of these arranged first followed by the consonants as per
the alphabetical order and the vowels are
14) Who among the following persons got the changed to the immediate next letter and the
project? consonants are changed to the immediate

Click Here For Bundle PDF Course | support@guidely.in Page 4 of 11


SBI Clerk & RRB PO Mains PDF Course 2023
Reasoning Ability Day - 15 (Eng)

previous letter in the alphabetical series, then followed by the second half digits in descending
which will be the fourth letter from the left end of order, now what is the resultant if the sum of the
each newly formed word in the same order? fourth digits from both ends is divided by the third
I. UNIFORM digit from the left end after the arrangement?
II. CLIMATE a) 2.2
III. KINGDOM b) 1.25
a) EBJ c) 2.75
b) LBK d) 1.5
c) LKM e) None of these
d) EBF
e) None of these 20) If in the word “SPECTACULAR”, all the odd
positioned letters (except vowels) are replaced
18) If the fourth letter from the right end of each by the second succeeding letter and all the odd
word is taken to form a meaningful word, then positioned vowels are replaced by the immediate
which of the following can’t form a meaningful succeeding vowel, all the even positioned letters
word? (except vowels) replaced by the second
I. Handed, Branch, Debate, Taking preceding letter and all the even positioned
II. Silver, Submit, Vision, Winter vowels are replaced by the immediate
III. Health, Dealer, Closed, Tagging succeeding letter, then which of the following
IV. Corner, Mentor, Lawyer, Cricket letters will be repeated more than once in the
a) Only IV new arrangement?(all the replacements are
b) Only II, III, and IV done as per the alphabetical series)
c) Only I and III a) NIT
d) None b) Only B
e) Only I, III, and IV c) VBA
d) BNV
19) If in a number “792515836427” the first half e) None
of digits are arranged in ascending order

Click Here For Bundle PDF Course | support@guidely.in Page 5 of 11


SBI Clerk & RRB PO Mains PDF Course 2023
Reasoning Ability Day - 15 (Eng)

Click Here to Get the Detailed Video Solution for the above given Questions
Or Scan the QR Code to Get the Detailed Video Solutions

Answer Key with Explanation

Directions (1-5):  Odd number of persons are working along


1. Answer: B with K, who is immediately senior to H.
2. Answer: D Since, odd number of persons working
3. Answer: A with K, thus the number of persons
4. Answer: A working on K’s designation must be even.
5. Answer: C That means, in case (1) K works as the
Director, in case (2) K works as Manager.
 The number of persons junior to E is one
more than the number of persons
designated between L and C.
 L is designated senior to C and odd
number of persons working in L’s
designation.
That means, in case (1) & case (2) E
works as an assistant Manager, in case
(1a) & case (2a) E works as the additional
general manager.
Based on the above given information we have:

We have:

Click Here For Bundle PDF Course | support@guidely.in Page 6 of 11


SBI Clerk & RRB PO Mains PDF Course 2023
Reasoning Ability Day - 15 (Eng)

 U,B and S are working together but not


works with E.
Thus, U,B and S are working as clerk.
 Even number of persons working between
B and D, who is immediately senior to A.
That means, in case (2) A is working as
Again, we have:
additional general manager, case (1) is
 G and P are working together, but neither
not valid.
with C nor with E.
 N and R are working together.
 P is two positions junior to F.
 R is senior to M but junior to J.
 Even number of persons are senior to F.
 M is junior to T.
That means, only possible position for F is
That means, T must work as an assistant
manager.
Manager.
Thus, case (1a) & case (2a) are not valid.
Based on the above given information we have:
Based on the above given information we have:

Case (1a) & case (2a) are not valid as only


Case (1) is not valid as Even number of persons
possible position of both G and P is clerk, but P
working between B and D, who is immediately
can’t work with C.
senior than A.
Again, we have:
All the persons except senior to Additional
 I and Q are working together but
General Manager are transferred to three
immediately senior to O.
different branches viz.- Head Branch, Urban
 The number of persons senior to Q is the
Brach, and Rural Branch as per below
same as the number of persons working
conditions:
in Q’s designation.
Condition I: The persons whose name comes
Since, maximum six persons work on any
before ‘M’ in alphabetical series and also even
designation, thus only such possible
number of persons working along with them are
position for Q is additional general
transferred to Head Brach.
manager.

Click Here For Bundle PDF Course | support@guidely.in Page 7 of 11


SBI Clerk & RRB PO Mains PDF Course 2023
Reasoning Ability Day - 15 (Eng)

Condition II: The persons whose name comes


after ‘M’ in alphabetical series and also odd
number of persons working along with them are
transferred to Urban Brach.
Condition III: All the remaining persons are
transferred to Rural Brach.
The persons senior to Additional General
Manager are appointed as a supervisor of all
three branches viz.- Head Branch, Urban Brach,
and Rural Branch in alphabetical order
We have,
respectively.
 One of the Painting sessions was held
after the lunch break.

Directions (6-10):
6. Answer: C
7. Answer: E
8. Answer: D
9. Answer: E
10. Answer: E
Final arrangement

Again we have,
 One of the dance sessions was held 180
minutes before the Yoga session.
 Dance session was held neither
immediately before nor immediately after the
Lunch break.

Click Here For Bundle PDF Course | support@guidely.in Page 8 of 11


SBI Clerk & RRB PO Mains PDF Course 2023
Reasoning Ability Day - 15 (Eng)

From the above conditions case 1 gets


eliminated because dance session was held
immediately before lunch break.

Again we have,
We have:
 One of the Painting sessions was held
 Only two persons joined before V, who
120 minutes after one of the Yoga sessions.
was born on 9th of the month.
 Dance sessions were not held after the
 Only three persons joined between V and
snacks breaks.
Q, whose experience is 15 years.
From the above conditions case 2 gets
 Only three persons joined between the
eliminated because Dance session was held
one who joined on 12th January and S,
after snack break. Hence case 3 shows the final
whose experience is a perfect square.
arrangement.
Since, only possible perfect square is 9 or
16.
That means, in case (1) no person joined
before the one who joined on 12 January,
in case (2) only one person joined before
the one who joined on 12 January.
Based on the above given information we have:

Direction (11-15):
11. Answer: D
12. Answer: B
13. Answer: B
14. Answer: C
15. Answer: B

Click Here For Bundle PDF Course | support@guidely.in Page 9 of 11


SBI Clerk & RRB PO Mains PDF Course 2023
Reasoning Ability Day - 15 (Eng)

 The one whose experience is 14 years


joined either immediately after or
immediately before P.
 The number of persons joined between S
and P is one less than the number of
persons joined before the one who joined
on 15th April.
 The experience of T is a prime number
and joined on 12th January.
That means, in case (1a) R joined three
persons before Q, case 1 and case (2) is
not valid.
Based on the above given information we have:
Again, we have:
 The difference between the experience of
S and Q is equal to the experience of R,
who joined two persons after the one
whose experience is 12 years.
Since the experience of S is either 9 or 16
years, and each person already has some
experience between 5 years and 20
years.
Thus, only possible combination of
experience must be: S = 9 years, and R =
6 years.
Case (1) and Case (2) are not valid as the
 The number of persons joined between R
experience of T is a prime number and joined on
and W is one more than the number of
12th January.
persons joined before the one whose
Again, we have:
experience is 14 years.
 The experience of U is ten years more
 W joined in November and joined two
than the experience of T.
persons after the one who joined on 15th
Since, only possible prime numbers are 7,
August.
11, 13, 17, and 19.

Click Here For Bundle PDF Course | support@guidely.in Page 10 of 11


SBI Clerk & RRB PO Mains PDF Course 2023
Reasoning Ability Day - 15 (Eng)

Now, U’s experience is ten years more II. If two persons who joined in the same month
than T, thus only possible combinations is are selected then, the person whose total
(7, 17). experience is more is to be selected.
 One of the persons whose experience is a Based on the above conditions: PVU
perfect cube number joined on 12th of any 16) Answer: D
of the months. I. 45862745 26640826  34
Now, Since only possible perfect cube II.78437617 86248428  42
experience is 8 years, and only possible III. 1846762426248402  28
experience of P must be 12 years. IV. 57136182 68244260  32
 No one joined on 9 August.
Based on the above given information we have: 17) Answer: D
I. UNIFORM  IOUFMNR  JPVELMQ
II. CLIMATE AEICLMT  BFJBKLS
III. KINGDOM IODGKMN  JPCFJLM
Thus, required letters  EBF

18) Answer: B
I. Handed, Branch, Debate, TakingNABK 
BANK
II. Silver, Submit, Vision, WinterLBSN
III. Health, Dealer, Closed, TaggingAAOG
IV. Corner, Mentor, Lawyer, Cricket RNWC
Thus, All II, III, and IV doesn’t form a meaningful
word.
Based on their working experience they are
selected for a project following certain 19) Answer: A
conditions: 792515836427 125579876432
I. Project is offered to those who have completed Thus, required number  (5 + 6)/5 = 2.2
minimum 3 years in Alpha (calculate their
experience as on 31st August 2023). 20) Answer: D
SPECTACULAR  UNIAVBEVNBT

Click Here For Bundle PDF Course | support@guidely.in Page 11 of 11


SBI Clerk & RRB PO Mains PDF Course 2023
Quantitative Aptitude Day - 15 (Eng)

Quantitative Aptitude

Directions (01 - 05): Study the following information carefully and answer the questions given below.
There are five companies A, B, C, D and E. Each company has two offices in the city Mumbai and Delhi.
Total number of employees of Mumbai and Delhi office of each company and difference of employees of
Mumbai and Delhi office are given in bar graphs. There are three posts [HR, IT, and Marketing] in each
branch. Ratio of number of in HR, IT and Marketing of each office of each branch is given in table.

Notes – Number of employees in Mumbai office of company A and B is more than that of Delhi office and
Number of employees in Delhi office of company C, D and E is more than that of Mumbai office.

Click Here For Bundle PDF Course | support@guidely.in Page 1 of 11


SBI Clerk & RRB PO Mains PDF Course 2023
Quantitative Aptitude Day - 15 (Eng)

1) Ratio of total number of male and female c) Difference between the number of employees
employees of Mumbai office of company A is 3:2, in marketing post of company E in Mumbai and
3:5 and 3:1 in HR, IT and Marketing posts. Ratio the Delhi office is 18.
of total number of male and female employees of d) Difference of average number of employees in
the Delhi office of company A is 2:3, 1:3 and 3:2 IT post all companies together in Mumbai office
in HR, IT and Marketing posts. Find the and average number of employees in Marketing
difference between the total number of males post all companies together in Mumbai office is
and total number of females of company A? 15 .
a) 16 e) None of these
b) 19
c) 12 4)
d) 18 Quantity I: Total number of employees in
e) 24 Marketing post in Delhi office of companies A, B
and C together is what percent of total number of
2) Find which one is true? employees in IT post of companies B, C and D
I. Difference in total employees in IT posts in together in Mumbai office?
Mumbai of all companies together and total Quantity II: Total number of employees in IT post
employees in IT posts In Delhi of all companies in Delhi office of companies A, B and C together
together is 14. is what percent of total number of employees In
II. Total employees in the Marketing post of all HR post of companies B, C and D together in
companies together are 306. Mumbai?
III. Average number of employees in the Delhi a) Quantity I > Quantity II
office of all companies together is 120. b) Quantity I ≥ Quantity II
a) Only I and II are true c) Quantity II > Quantity I
b) Only I is true d) Quantity II ≥ Quantity I
c) Only I and are III true e) Quantity I = Quantity II or Relation cannot be
d) All are true establish
e) All are false ed

3) Find which one is true? 5) Find the difference between the total number
a) Ratio of number of employees in HR post of of employees in companies A, B and C together
company C in Mumbai and Delhi office is 2:3. in the Mumbai office and the total number of
b) Total number of employees in HR post in employees in companies C, D and E together in
Mumbai office of all companies is 300. the Delhi office?

Click Here For Bundle PDF Course | support@guidely.in Page 2 of 11


SBI Clerk & RRB PO Mains PDF Course 2023
Quantitative Aptitude Day - 15 (Eng)

a) 0 d) 3
b) 1 e) None of these
c) 2

Directions (06 - 10): Study the following information carefully and answer the questions given below.
The line graph shows the percentage of registered voters who cast their vote, the percentage of invalid
votes out of total cast votes, and the percentage of party A's votes out of a valid vote in five cities. The
total number of registered voters in each city is 30000. There is only party A and Party B contest in the
election.

6) Find the sum of the difference of votes got by and the average number of valid votes of all five
parties A and B in all cities together? cities together?
a) 20152 a) 2150
b) 26530 b) 2650
c) 23820 c) 2840
d) 21300 d) 2370
e) 26520 e) 2850

7) Find the difference between the average 8) In city F, the total number of populations is
number of cast votes of all five cities together 35000, and the number of registered voters is
the same as mentioned in the question. The ratio

Click Here For Bundle PDF Course | support@guidely.in Page 3 of 11


SBI Clerk & RRB PO Mains PDF Course 2023
Quantitative Aptitude Day - 15 (Eng)

of valid votes of cities F and R is 4:3 and party A respectively. The speed of boat P, Q, and R in
got 55% of the valid vote then find the number of still water is x km/hr, y km/hr, and z km/hr
votes party B got in city F? respectively. On Monday, boat P goes 66 km
a) 11250 downstream and 56 km upstream in 7 hours and
b) 10800 88 km downstream and 28 km upstream in 6
c) 14200 hours. On Tuesday, boat Q goes 44 km
d) 13000 downstream and 60 km upstream in 8 hours and
e) None of these 88 km downstream and 70 km upstream in 11
hours. On Wednesday, boat R goes 64 km
9) Find the ratio of the total number of people downstream and 32 km upstream in 4 hours and
who did not cast their vote in all cities together 96 km downstream and 64 km upstream in 7
and the number of invalid votes in all cities hours.
together? 11) On Tuesday, boat P goes 5z km downstream
a) 200:79 and boat R goes 8c km upstream. Find the
b) 212:25 difference in time taken by two boats?
c) 241:26 a) 5(4/9)hrs
d) 241:22 b) 3(4/9)hrs
e) None of these c) 1(4/9)hrs
d) 2(4/9)hrs
10) Find the difference between the percentage e) 4(4/9)hrs
of party B's votes out of the total registered
voters in city P and the percentage of party A's 12) Find the ratio of the total distance covered by
votes out of the total registered voters in city Q? boat Q on Monday in b hours downstream and
a) 17.75% the total distance covered by boat R on Monday
b) 15.2% in c hours upstream?
c) 17.8% a) 2:3
d) 16.5% b) 4:5
e) None of these c) 5:6
d) 3:4
Directions (11 - 15): Study the following e) 1:2
information carefully and answer the questions
given below. 13) On Monday,Boat P goes from point M to N
The speed of the stream of river on Monday, downstream of the river. O is the middle point of
Tuesday, and Wednesday is a, b, and c km/hr M and N. From M to O, speed of the stream is

Click Here For Bundle PDF Course | support@guidely.in Page 4 of 11


SBI Clerk & RRB PO Mains PDF Course 2023
Quantitative Aptitude Day - 15 (Eng)

25% less than the original speed. If the distance 16)


between point M and N is 88 km, then find the Quantity I: Total surface area of a sphere is 2464
approximate total time taken by the boat? m2. Find the volume of the sphere?
a) 5 hrs Quantity II: The sum of the height and radius of a
b) 4 hrs cone is 28 m and the ratio of radius and height of
c) 3 hrs the cone is 11:3. Find the volume of the cone?
d) 2 hrs Quantity III: The ratio of the side of the cube and
e) None of these length of the cuboid is 2:5. Volume of the cube is
64 m. Ratio of length and breadth and the ratio of
14) Speed of boat S is 25% more than the speed length and height of the cuboid is 5:3 and 5:7.
of boat of Q in still water. Find the distance cover Find the volume of the cuboid?
by boat S on Wednesday in (a+2) hours a) Quantity I <Quantity II >Quantity III
downstream? b) Quantity I> Quantity II =Quantity III
a) 168 km c) Quantity I< Quantity II< Quantity III
b) 172 km d) Quantity I> Quantity II> Quantity III
c) 180 km e) Quantity I> Quantity II <Quantity III
d) 182 km
e) None of these 17)
Quantity I: The ratio of age of A and B is 2:3.
15) Boat Q and Boat R are travelling in opposite After 5 years age of B is 32 years. Age of C is 5
direction towards each other from a certain years more than A. Find the average age of A, B,
distance on Tuesday. If boat Q goes downstream and C?
and boat R goes upstream and they meet after 7 Quantity II: The sum of age of A and B is 38
hours then find the distance between boat Q and years and age of B is 5 years older than C whose
R initially? age is 13 years. Find the age of A?
a) 280 km Quantity III: The age of the son is 1/3rd of the
b) 220 km age of the father and the age of the mother is
c) 256 km thrice of her daughter. If the sum of the age of
d) 250 km the father and mother is 84 years and both of
e) None of these their ages are equal. Find the sum of the age of
the daughter and son?
Direction (16-20) – Each question has three a) Quantity I >Quantity II >Quantity III
quantities. Calculate the value of each quantity b) Quantity I>Quantity II< Quantity III
and compare it. c) Quantity I =Quantity II< Quantity III

Click Here For Bundle PDF Course | support@guidely.in Page 5 of 11


SBI Clerk & RRB PO Mains PDF Course 2023
Quantitative Aptitude Day - 15 (Eng)

d) Quantity I< Quantity II <Quantity III Quantity II: An item isbought at Rs.1250 then
e) Quantity I >Quantity II= Quantity III marked the price 50% above the buying price
and then gave a 10% discount on the marked
18) price, find the profit percentage?
Quantity I: A person has Rs.12400. He invests Quantity III: The selling price of the item is
40% in the scheme which provides a 10% rate of Rs.1580. The shopkeeper is given a 20%
interest and the rest invests in another scheme discount on the marked price and the marked up
which provides a 5% rate of interest. Find the percentage is 40%. Find the profit percentage?
total interest earned by the person after 5 years if a) Quantity I <Quantity II <Quantity III
both are invested in SI? b) Quantity I= Quantity II >Quantity III
Quantity II: Rs.4852.85 c) Quantity I <Quantity II> Quantity III
Quantity III: A person invests Rs.1200 at 20% d) Quantity I >Quantity II> Quantity III
rate of compound interest for 2 years and e) Quantity I >Quantity II <Quantity III
Rs.6000 invests at 25% rate of compound
interest for 2 years. Find the total interest earned 20) f(x)=x3-3x2+3x-4
by the person? Quantity I: Find the value of f(x) when x=3?
a) Quantity I =Quantity II =Quantity III Quantity II: Find the value of f(x) when x=-2.5?
b) Quantity I >Quantity II >Quantity III Quantity III: Find the value of f(x) when x=1.5?
c) Quantity I <Quantity II< Quantity III a) Quantity I <Quantity II >Quantity III
d) Quantity I >Quantity II =Quantity III b) Quantity I< Quantity II< Quantity III
e) Quantity I < Quantity II >Quantity III c) Quantity I <Quantity II =Quantity III
d) Quantity I >Quantity II< Quantity III
19) e) Quantity I =Quantity II <Quantity III
Quantity I: The ratio of the cost price and selling
price of the item is 7:11. Find the profit
percentage of the item?

Click Here For Bundle PDF Course | support@guidely.in Page 6 of 11


SBI Clerk & RRB PO Mains PDF Course 2023
Quantitative Aptitude Day - 15 (Eng)

Click Here to Get the Detailed Video Solution for the above given Questions
Or Scan the QR Code to Get the Detailed Video Solutions

Answer Key with Explanation

Directions (01 - 05):


Number of employee in Mumbai office of
company A is [220+60]/2=140
Number of employee in Delhi office of company
A is [220-60]/2=80
Number of employees in HR post of Mumbai
Office =140*3/7=60
Number of employees in IT post of Mumbai
office =140*2/7=40
Number of employees in Marketing post of
Mumbai office =140*2/7=40
Number of employees in HR post of Delhi Office
=80*3/8=30
Number of employees in IT post of Delhi office
=80*2/8=20
Number of employees in Marketing post of Delhi
office =80*3/8=30
Similarly, we can calculate other values also.
1) Answer: C
Total male in company A is
=60*3/5+40*3/8+40*3/4+30*2/5+20*1/4+30*3/5=
116

Click Here For Bundle PDF Course | support@guidely.in Page 7 of 11


SBI Clerk & RRB PO Mains PDF Course 2023
Quantitative Aptitude Day - 15 (Eng)

Total female in company A is 4) Answer: A


=60*2/5+40*5/8+40*1/4+30*3/5+20*3/4+30*2/5= Quantity I
104 Required percentage = [(30+40+20) /
So, difference is = 116-104 = 12 (60+20+40)]*100 = 75%
Quantity II
2) Answer: D Required percentage =
I. Difference of total employees in IT post in [(20+20+60)/(80+60+60)]*100 = 50%
Mumbai of all companies together and total
employees in IT post in Delhi of all companies 5) Answer: A
together is Required difference = [140+160+100] –
= [40+60+20+40+30]-[20+20+60+40+36]=14. [120+160+120] =0
II. Total employees in Marketing post of all
companies together Directions (06 - 10):
=40+20+20+40+20+30+40+20+40+36=306. In city P,
III. Average number of employees in Delhi office The number of total registered voters is 30000.
of all companies together is = The total number of cast votes is
[80+120+120+160+120]/5=120. 30000*80/100=24000
The total number of people who did not cast vote
3) Answer: E is 30000-24000=6000
a) Ratio of number of employees in HR post of The total number of valid votes is
company C in Mumbai and Delhi office is 3:2. 24000*90/100=21600
b) Total number of employees in HR post in the The total number of invalid votes is 24000-
Mumbai office of all companies is 320. 21600=2400
c) Difference between the number of employees The number of votes gets by party A is
in marketing post of company E in Mumbai and 21600*60/100=12960
the Delhi office is 16. The number of votes gets by party B is 21600-
d) Difference of average number of employees in 12960=8640
IT post all companies together in Mumbai office Similarly, we can calculate other cities also.
and average number of employees in Marketing
post all companies together in Mumbai office is
10.
So, all are false.

Click Here For Bundle PDF Course | support@guidely.in Page 8 of 11


SBI Clerk & RRB PO Mains PDF Course 2023
Quantitative Aptitude Day - 15 (Eng)

The speed of the stream of river R on Monday,


Tuesday, and Wednesday is a, b, and c km/hr
respectively. The speed of boats P, Q, and R in
still water is x km/hr, y km/hr, and z km/hr
So, we can say,
66/(a+x) + 56/(x-a) =7 --------I
88/(a+x) + 28/(x-a) =6 --------II
By solving I and II we get, x=18 km/hr and a=4
km/hr.
6) Answer: C
44/(y+b) + 60/(y-b) =8 --------III
Required sum
88/(y+b) + 70/(y-b) =11 --------IV
=[12960-8640]+[13965-5985]+[9900-
By solving III and IV we get, y=16 km/hr and b=6
8100]+[13770-9180]+[15390-10260]= 23820
km/hr.
64/(z+c) +32/(z-c) =4 --------V
7) Answer: D
96/(z+c) + 64/(z-c) =7 --------VI
Required difference
By solving V and VI we get, z=24 km/hr and c=8
= [24000+21000+22500+25500+27000]/5 –
km/hr.
[21600+19950+18000+22950+25650]/5
11) Answer: C
=24000-21630=2370
Required difference = 120/(18+6)-64/(24-
6)=1(4/9)hrs
8) Answer: B
B’s party vote is [18000*4/3]*45/100=10800
12) Answer: D
Required ratio =[16+4]*6 : [24-4]*8=120:160=3:4
9) Answer: A
Required ratio=[6000+9000+7500+4500+3000]:
13) Answer: B
[2400+1050+4500+2550+1350]
The speed stream from M to O is 4*75/100=3
=30000:11850=200:79
So, required time is =
[44/(18+3)]+[44/(18+4)]=4.09=4 hours.
10) Answer: A
Required difference =[13965/30000]*100–
14) Answer: A
[8640/30000]*100=17.75%
The speed of boat S is =16*125/100=20 km/hr.
Required distance = [20+8]*[4+2]=28*6=168 km
Directions (11 - 15):

Click Here For Bundle PDF Course | support@guidely.in Page 9 of 11


SBI Clerk & RRB PO Mains PDF Course 2023
Quantitative Aptitude Day - 15 (Eng)

15) Answer: A 18) Answer: E


Total distance of boat Q and R initially is = Quantity I:
[16+6]*7+[24-6]*7=280 km Total interest earned
= [12400*40/100]* 5*10
16) Answer: D /100+[12400*60/100]*5*5/100
Quantity I: =Rs.4340
So, 4*22/7*r*r=2464, r=14m Quantity II: Rs.4852.5
So, the volume is 22/7*14*14*14*4/3=11498.66 Quantity III:
m3 Total interest earned
Quantity II: 1200*[1+20/100]2+6000*[1+25/100]2-
Radius is 28*11/14=22m, height is 28-22=6m 7200=Rs.3903
So, volume is =1/3*22/7*22*22*6=3042.28m3 Quantity I < Quantity II >Quantity III
Quantity III:
The side of the cube is 4m 19) Answer: D
So, the length of the cuboid is 4*5/2=10m Quantity I:
The breadth of the cuboid is 10*3/5=6m Profit percentage = [4/7]*100=57.14%
The height of the cuboid is 10*7/5=14m Quantity II:
So, volume is 10*6*14=840m3 Selling price =
Quantity I> Quantity II> Quantity III 1250*150*90/(100*100)=Rs.1687.5
Profit percentage = [(1687.5-
17) Answer: B 1250)/1250]*100=35%
Quantity I: Quantity III:
Age of B is 32-5=27 years. Cost price of item is
Age of A is 27*2/3=18 years 1580*[100/80]*[100/140]=1410.71
Age of C is 18+5=23 years So, profit percentage [1580-
So, the average age is [27+18+23]/3=22.66 1410]*100/1410=12.05%
years Quantity I >Quantity II> Quantity III
Quantity II:
Age of Ais 38-[13+5]=20 years 20) Answer: D
Quantity III: Quantity I:
The Sum of the age of Son and daughter is So, f(x)=3*3*3-3*(3)2+3*3-4 when x=3,
84/3=28 years Or, f(x)=5
Quantity I>Quantity II< Quantity III Quantity II:

Click Here For Bundle PDF Course | support@guidely.in Page 10 of 11


SBI Clerk & RRB PO Mains PDF Course 2023
Quantitative Aptitude Day - 15 (Eng)

So, f(x)=(-2.5)3-3(-2.5)2+3*(-2.5)-4=-45.875 when So, f(x)=(1.5)3-3*(1.5)2+3*(1.5)-4=-2.875


x=-2.5 So, Quantity I >Quantity II < Quantity III
Quantity III:

Click Here For Bundle PDF Course | support@guidely.in Page 11 of 11


SBI Clerk & RRB PO Mains PDF Course 2023
ENGLISH Day - 15

English Language

Directions (1-6): Read the following passage (SDG) targets for under-five and neonatal
carefully and answer the questions given below. mortality. Between 2000 and 2020, the region
Few alphabets and numbers are given to help achieved a 34% decline in tuberculosis incidence
you assist in answering those questions. rates, and by the end of 2020, had met each of
On World Health Day (April 7) this year, the the Global Technical Strategy for Malaria
World Health Organization (WHO) celebrates 75 milestones for mortality and morbidity. Since
years of improving public health and well-being in 2016, six countries of the region have eliminated
the South-East Asia region. Globally, WHO is at least one neglected tropical disease and all
fully committed to achieving ‘Health for All’ countries continue to strengthen PHC services to
through universal health coverage (UHC) — prevent, detect, control and treat non-
when all people can access ________(A) health communicable diseases.(B)Impressively, the
services, without financial hardship.For decades region has maintained its polio-free status and
— and even before the 1978 Declaration of continues to have controlled(1) maternal and
Alma-Ata — high-level leaders and policymakers neonatal tetanus as a public health problem, five
from across the region have recognised the countries of the region have eliminated measles,
critical role that access for all to quality, many(2) have eliminated rubella, and four have
affordable and comprehensive primary health eliminated(3) hepatitis B through immunisation
care (PHC) can play in achieving UHC, and but the region has two(4) more mountains to
therefore ‘Health for All’. This has been reflected climb. Today, in the shadow of the COVID-19
in the region’s renewed, decade-long push to crisis, around 40% of people are unable to
achieve UHC, which, since 2014, has been one access essential health services. In 2017, around
of eight flagship priorities of the region.Between 299 million people faced catastrophic health
2010 and 2019, the region increased its UHC spending, and an estimated 117 million people
service coverage index from 47 to 61. Between were pushed or further pushed below the
2000 and 2017, the region reduced the number purchasing power parity poverty line of $1.90 a
of households that have been impoverished or day — a figure that has since been
further impoverished from __________(X) exacerbated.As emphasised (Y) by the region’s
spending on health, from 30% to 6%. Since 2021 Strategy for Primary Health Care, as well
2014, the density of doctors, nurses and as the region’s vision to build back better from
midwives in the region has improved by over the COVID-19 pandemic, action by the whole-of-
30%.Five countries of the region have already government and whole-of-society is needed to
achieved the Sustainable Development Goal

Click Here For Bundle PDF Course | support@guidely.in Page 1 of 13


SBI Clerk & RRB PO Mains PDF Course 2023
ENGLISH Day - 15

drive rapid and sustained progress towards UHC I. It is quite __________ for old people to know
and ‘Health for All’. We must all contribute to this. how to use mobile phones particularly when they
High-level leaders should maintain and are living away from their children.
strengthen political and financial commitments to II. Recently the drug prices have seen a rise and
achieve UHC, accelerating the momentum ahead hence the public is finding it difficult to purchase
of the second United Nations General Assembly even the ________ medicines.
meeting on UHC in September this year. The a) Important
COVID-19 crisis has shown that investments in b) Various
UHC and health system resilience underpin not c) Difficult
just health but also social and economic security d) Essential
as well as the achievement of an array of e) All the above
SDGs.Policymakers and programme managers
must continue to implement the region’s Strategy 2) The sentence given as (B) has four words that
for PHC. Of specific focus should be a are given in bold namely (1),(2),(3) and (4). The
strengthening of the public health infrastructure, bold words may or may not be in the right place.
workforce and financing, while at the same time Swap the words if it is necessary to make the
increasing equity for those at risk of or who are sentence (B) grammatically and contextually
already being __________ (C). Shared learning correct and meaningful.
must continue to be a core priority, _________ (D) a) 1-3
the full power of the region’s new forum for PHC- b) 2-4
oriented health systems.On WHO’s 75th c) 1-2 and 3-4
founding anniversary, let us unite in purpose and d) 1-3 and 2-4
be driven by action to achieve UHC and ‘Health e) No swapping required
for All’, ensuring that all people have good health
for a fulfilling life in a peaceful, prosperous and 3) Choose from the below phrases that would
sustainable world. best fit the blank X in order to make the
1) Which of the following words given in the respective sentence correct and meaningful.
options should fit the blank marked as (A) in the a) Out-from-pocket
above passage to make the sentence b) Out-of-pocket
grammatically correct and meaningful. Also, the c) Deep-pocketed
same word should fill in the two blanks in the d) Fat-cat
sentences given below. e) Well-off

Click Here For Bundle PDF Course | support@guidely.in Page 2 of 13


SBI Clerk & RRB PO Mains PDF Course 2023
ENGLISH Day - 15

4) Which of the following words given in the b) Prioritising


options can replace the bold word marked as (Y) c) Using
in the above passage without changing the d) Leaving
meaning or the context of the sentence. Also, the e) Leveraging
same word should fill in the two blanks in the
sentences given below. Directions (7-11): In the following questions three
I. After reviewing the user manual document the columns are given containing three
manager had sent a mail mentioning the Sentences/phrases each. In the first column,
information or points that had to be ________ in sentences/phrases are A, B and C, in the second
order to enhance readability. column the sentences/phrases are D, E and F
II. Our recent outing to a hill station was and in the third column the sentences/phrases
_________ by a unique fire show that was hosted are G,H and I are given. A sentence/phrase from
by the hotel we stayed at. one column may or may not connect with
a) Highlighted another sentence/phrase from the other two
b) Entitled columns to make a grammatically and
c) Given contextually correct sentence. Each question has
d) Written five options, four of which display the
e) All the above sequence(s) in which the sentences/phrases can
be joined to form a grammatically and
5) Which of the following phrasal verbs given in contextually correct sentence. If none of the
the options can fill the blank (C) in the above options given forms a correct sentence after
passage making the sentence grammatically and combination, mark option (E), i.e. “None of
contextually correct and meaningful. these” as your answer.
a) Leave off 7)
b) Left over COLUMN 1 COLUMN 2 COLUMN 3
c) Left behind
d) Let out A. Chat GPT is D. and thereby G. and other

e) Hold down a natural encourage the parts of the


language private sector to agriculture`

6) Which of the following words given in the processing invest in

options can fill the blank (C) in the above tool driven by storage,

passage making the sentence grammatically and AI technology transportation

contextually correct and meaningful.


a) Utilise

Click Here For Bundle PDF Course | support@guidely.in Page 3 of 13


SBI Clerk & RRB PO Mains PDF Course 2023
ENGLISH Day - 15

B. According E. that allows H. this A.A bad film D. G. in


to the you to have language has dangerous conflagrations particular,alar
government, human-like model can effects on the resulted in ming it all
conversations answer human mind scores of deaths began as
and questions and as it deals with and the protests.
assist you cheap displacement of
with tasks, entertainment people in
such as and vulgar Churachandpur
composing scenes, and Imphal
emails,
B. Violence E. Mughal H. and
essays, and
has flared dynasty and he improvise
code.
again in ruled the their direction
C.A species F. but the nation I. such Manipur, Mughal empire skills.
which was was fighting an species are merely three from 1605
native to a existential battle known are weeks after
region and its endangered
C. Jahagir was F. it also I. till the
population species
the fourth misleads watchman fell
strength
Mughal particularly asleep at
reduced from
emperor and young minds midnight.
50 percent to 5
one of the with is a gross
percent;
most sensation
a) A-E-H prominent
b) B-D-G rulers of the
c) C-F-I
a) A-F-H
d) B-F-G
b) C-E-G
e) None of these
c) B-D-G
d) A-D-H
8)
e) None of these
COLUMN 1 COLUMN 2 COLUMN 3

9)

COLUMN 1 COLUMN 2 COLUMN 3

Click Here For Bundle PDF Course | support@guidely.in Page 4 of 13


SBI Clerk & RRB PO Mains PDF Course 2023
ENGLISH Day - 15

10)
A. The G20 D. the interest G. resulting in
group of 19 on deposits equal COLUMN 1 COLUMN 2 COLUMN 3
countries and and distributes opportunities
A. A study D. felt that the G.which means
the EU was the amount for all the
carried out by patient was to avoid
established in citizens of the
Aalto very serious wastage and
1999 as a nation.
university,Finla and had to be over-
platform for
nd this year discharge and capitalisation
Finance
admitted in situations.
Ministers and
Central Bank B. Two years E. financial H. the new
Governors later after the management parliament
protest in Nov that estimates building which
B. When E. is slowed H. as well as
1985 precise was
central banks down and due to the
requirement of inaugurated by
increase liquidity is changes in
funds the honourable
reserve sucked out of levels of
minister.
requirements, the system, productivity.
the speed of C. The F. on I. factors like
credit creation financial increasing user homophily
planning is the polarisation on and algorithmic
C. The main F. to discuss I. and
key element of social media filtering have
business of international together, the
the found that enforced belief
the banking economic and G20 countries
systems.
industry is to financial issues account for
give almost two- a) A-F-I

thirds of the b) C-E-G

global c) B-D-H

population. d) Both A-F-I and C-E-G


e) None of these
a) A-F-I
b) C-E-H
11)
c) B-D-G
d) A-F-H COLUMN 1 COLUMN 2 COLUMN 3

e) None of these

Click Here For Bundle PDF Course | support@guidely.in Page 5 of 13


SBI Clerk & RRB PO Mains PDF Course 2023
ENGLISH Day - 15

I. He was very successful in his struggle against


A. China is the D. for India to G. that risk is
the British and became a prominent leader in the
major reason have bilateral higher in the
freedom struggle.
for hesitation ties with latter
II. For India, which imports more than 80% of its
and confusion pakistan approach as it
concentrates crude oil requirements, the announcements of

more on dry supply curtailment are a cause for some concern

lands. given the potential they have to push up global


oil prices.
B. It is only of E. acquire H. as India is III. The birds here camouflage their nests with
justice and some worried about moss and leaves
humanitarianis knowledge on the a) Only I
m that a leader discovering the consequences b) Only II
should path hole after that. c) Both I and II
d) Only III
C. Students F. people’s I. because the
e) All are correct
were tested on obsession with country is
South korea’s eductaion cautious to
13) VANDALISM, DESOLATE, MULTIFARIOUS
culture take a step
I. Mr. Modi repeated his concerns over vandalism
forward
and attacks defacing community centres and
keeping in
temples with pro-Khalistani, anti-India and anti-
mind its
Modi graffiti.
citizens.
II. These snow-fed rivers will first swell and then
a) A-D-H
run dry, triggering off devastating floods which is
b) A-D-I
followed by a desolativeness.
c) C-F-G and A-D-H
III. From a loose arrangement, it has become a
d) Both A-D-H and A-D-I
tight bureaucratic organisation with its
e) None of these
jurisdictions extending to multifarious activities.
a) Only I
Directions (12-15): In each of the following
b) Only II
questions, three sentences are given with a
c) Only III
highlighted word. Choose the sentence(s) that
d) Both II and III
has/have the wrong usage of the highlighted
e) All are correct
word.
12) PROMINENT, CURTAIL, CAMOUFLAGE
14) CRITERIA, PURPORT, LEAK

Click Here For Bundle PDF Course | support@guidely.in Page 6 of 13


SBI Clerk & RRB PO Mains PDF Course 2023
ENGLISH Day - 15

I. Rainfall is not the only criterion for the IMD to the sentences are correct then choose ‘option e’
declare the monsoon’s advent as your answer.
II. The company has alleged that TLB lenders 16)
issued a notice demanding immediate payment A) The king began laughing loudly and
of the entire amount under the TLB, despite asked,”Have you lost all sense, Haridra ?”
knowing that the purported acceleration was B) We live in a gated community which
under challenge before the court. comprises of six blocks with 24 flats and 8
III. A leakage of kerosene has polluted the water individual villas, in an area of about two thousand
supply all over the city. square metres.
a) Only I C) There are just a few trains for the ever
b) Only II growing number of passengers in Mumbai, the
c) Only III local trains are always crowded with people.
d) Both I and III D) I missed the last bus which I usually board
e) All are correct and had to stay at the bus stop until my father
came to pick me up.
15) ENSURE, HINDRANCE, RELUCTANT a) A
I. Month-long safety drive to be launched to b) B
ensuring cabins which house such equipment c) C
are provided with a dual-lock access system. d) D
II. It also helps in reducing uncertainties which e) All are correct
can be a hindrance to growth of the company.
III. Many state governments are reluctance to 17)
issue new ration cards beyond the central quota. A) Our manager do not like being interrupted in
a) Only I the middle of the PI planning.
b) Only II B) The theatre play was fantastic, the whole
c) Only III crowd gave a standing ovation to the play and
d) Both I and III appreciated the performers.
e) All are correct C) Shanta always prefers coffee to tea as she is
used to coffee since her childhood.
Directions (16-19): In the questions given below, D) To increase the selling of the banking
four statements are given labelled as (A),(B),(C) products in rural areas, the bank will recruit more
and (D), with one statement being grammatically salespersons this year.
and contextually incorrect.You are required to a) A
choose that sentence which is erroneous. If all b) B

Click Here For Bundle PDF Course | support@guidely.in Page 7 of 13


SBI Clerk & RRB PO Mains PDF Course 2023
ENGLISH Day - 15

c) C e) All are correct


d) D
e) All are correct 19)
A) According to the media the police are trying
18) hard to find out the root cause of the problem.
A) All this comes at a time while fund allocation B) Because of her prolonged illness and back to
has been doubled. back surgeries she could not concentrate on her
B) Arnav lives in the house rented by a 80 year studies although he has good desire.
old lady who’s husband had died a decade ago. C) As India moves beyond selling cars and
C) Neither Shyam nor I like to travel along those becomes a manufacturing and innovation hub, it
routes, neither of the roads are good. gives a lot of opportunities to the youth.
D) After having been friends for more than two D) Ravindran has seldom if ever been to India
decades, they had a fight last year and haven't since his graduation in the states.
seen each other ever since. a) A
a) A b) B
b) B c) C
c) C d) D
d) D e) All are correct
Click Here to Get the Detailed Video Solution for the above given Questions
Or Scan the QR Code to Get the Detailed Video Solutions

Answer Key with Explanation

1) Answer: D Important - although the word important may be


‘Essential’ is the right word that fits all the three partially correct but the word ‘essential’ is more
blanks correctly. appropriate to the context of all the three
Other options don’t fit because : sentences.

Click Here For Bundle PDF Course | support@guidely.in Page 8 of 13


SBI Clerk & RRB PO Mains PDF Course 2023
ENGLISH Day - 15

Various may only fit blank (A) and does not give the sentence completely wrong and
any meaning to the sentences I and II. meaningless. The word usage causes
Difficult does not fit all the blanks, for eg : put the redundancy and hence it is a wrong option to
word ‘difficult’ in sentence II and note that the choose
sentence is contextually incorrect and Option ‘e’ is wrong as swapping is a must here.
meaningless. Because words ‘controlled’ and ‘two’ are clearly
And hence, the correct word choice is ‘essential’ in the wrong place and need swapping.
Globally, WHO is fully committed to achieving
‘Health for All’ through universal health coverage 3) Answer: B
(UHC) — when all people can access essential Out-of-pocket is the best fit.
health services. (From passage)
It is quite essential for old people to know how Between 2000 and 2017, the region reduced the
to use mobile phones particularly when they are number of households that have been
living away from their children.(Sentence I) impoverished or further impoverished from out of
Recently the drug prices have seen a rise and pocket spending on health, from 30% to 6%.
hence the public is finding it difficult to purchase Out of pocket spending means to spend wildly or
even the essential medicines.(Sentence II) extremely even though you have less money in
hand.
2) Answer: D Other options are wrong because:
Swap words (1) - (3) and (2) - (4). ‘Out from pocket’ is not even a correct phrase
After swapping : Impressively, the region has and does not exist.
maintained its polio-free status and continues to Deep-pocketed, fat-cat and well-off are the exact
have eliminated maternal and neonatal tetanus opposite of the phrase ‘out of pocket’. They
as a public health problem, five countries of the change the meaning of the sentence completely
region have eliminated measles, two have and hence not the right fit.
eliminated rubella, and four have controlled
hepatitis B through immunisation but the region 4) Answer: A
has many more mountains to climb. Highlighted is the correct word that fits in all the
Other options other than ‘d’ are wrong because : three given blanks.
Option ‘a’ and ‘b’ are partially correct as they Highlighted - to make something clear and
interchange only two words. distinct so as to grab attention/to mark a part of
Option ‘c’ is wrong - because the words if text in a different style or colour.
interchanged as mentioned in option c makes

Click Here For Bundle PDF Course | support@guidely.in Page 9 of 13


SBI Clerk & RRB PO Mains PDF Course 2023
ENGLISH Day - 15

As highlighted by the region’s 2021 Strategy for Shared learning must continue to be a core
Primary Health Care.(From passage) priority, leveraging the full power of the region’s
After reviewing the user manual document the new forum for PHC-oriented health systems.
manager had sent a mail mentioning the Leverage - to use something to the maximum
information or points that had to be highlighted in Other options do not fit the blank because :
order to enhance readability.(Sentence I) Utilise is not a gerund(gerund is the right form
Our recent outing to a hill station was highlighted here that should be used)
by a unique fire show that was hosted by the Prioritising, leaving and using don’t fit the blank
hotel we stayed at.(Sentence II) as they alter the context of the sentence.
Other given options entitled, given and written do
not fit all the blanks and hence are not right 7) Answer: A
options. A-E-H is the right sequence that forms a
5) Answer: C meaningful sentence.
‘Left behind’ is the correct phrasal verb that The sentence after combining the phrases :
should be used. ChatGPT is a natural language processing tool
Left behind - is to leave/forget someone or driven by AI technology that allows you to have
something/not include human-like conversations and this language
Of specific focus should be a strengthening of model can answer questions and assist you with
the public health infrastructure, workforce and tasks, such as composing emails, essays, and
financing, while at the same time increasing code.
equity for those at risk of or who are already The above sentence is the only proper one that
being left behind. can be formed from the phrases given in the
Other options are wrong because : column.
Leave off - discontinue The other phrases when matched don’t make a
Left over - something that is remaining grammatically correct and meaningful sentence.
Let out - utter a cry
Hold down - prevent someone from moving 8) Answer: C
The meaning of the above phrasal verbs don’t fit B-D-G is the right sequence which forms a
into the context of the sentence and hence are meaningful sentence.
wrong. The sentence after combining the phrases :
Violence has flared again in Manipur, merely
6) Answer: E three weeks after conflagrations resulted in
Leveraging is the right pick here. scores of deaths and the displacement of people

Click Here For Bundle PDF Course | support@guidely.in Page 10 of 13


SBI Clerk & RRB PO Mains PDF Course 2023
ENGLISH Day - 15

in Churachandpur and Imphal in polarisation on social media found that factors


particular,alarming it all began as protests. like user homophily and algorithmic filtering have
The other phrases when matched don’t make a enforced belief systems.
grammatically correct and meaningful sentence. Sentence II: The financial planning is the key
Even though A-F and C-E form a partial element of the financial management that
sentence, there are no matching phrases in estimates precise requirement of funds which
column 3 to form a complete meaningful means to avoid wastage and over-capitalisation
sentence. And hence option c is the right situations.
answer. The left out three phrases B,D,H are distinct and
have different context and hence cannot be
9) Answer: A combined.
A-F-I is the only right sequence that forms a 11) Answer: D
correct and meaningful sentence among the Both A-D-H and A-D-I form two distinct correct
given options. and meaningful sentences.
The sentence after combining the phrases :The The pair A-D forms a sentence with the phrase
G20 group of 19 countries and the EU was ‘H’ and another sentence with ‘I’. Other given
established in 1999 as a platform for Finance options are wrong as they do not form any
Ministers and Central Bank Governors to discuss proper sentence.
international economic and financial issues and The sentences after combining the phrases :
together, the G20 countries account for almost Sentence I: China is the major reason for
two-thirds of the global population. hesitation and confusion for India to have
B-E forms a partially correct sentence but does bilateral ties with Pakistan as India is worried
not have a continuation phrase in column 3. about the consequences after that.
Similarly C-D forms a partially correct sentence Sentence II: China is the major reason for
but cannot be completed as there is no hesitation and confusion for India to have
appropriate phrase to connect in column 3. bilateral ties with Pakistan because the country
is cautious to take a step forward keeping in
10) Answer: D mind its citizens.
Both the sequences A-F-I and C-E-G form
correct and meaningful sentences. 12) Answer: E
The sentences after combining the phrases : All the given three sentences are correct and
Sentence I: A study carried out by Aalto meaningful. The highlighted words are used
university,Finland this year on increasing accurately in the sentences.

Click Here For Bundle PDF Course | support@guidely.in Page 11 of 13


SBI Clerk & RRB PO Mains PDF Course 2023
ENGLISH Day - 15

Meaning of the highlighted words: be replaced with ensure, similarly reluctance


Prominent - important and famous must be replaced with reluctant to make the
Curtailment - cutting short on something sentence meaningful and correct. The
Camouflage - materials or colours that soldiers prepositions ‘to and are’ give us a clue here. ‘To
use to hide ensure’ is the right way and ‘are reluctant’ is the
right form.
13) Answer: B The second sentence is correct as it is and the
Desoltiveness must be replaced with usage of the word is also right.
desolateness to give a proper meaning to that Meaning of the highlighted words :
particular sentence. Desolate has other forms of Ensure - to make sure
usage which are desolation, desolating and Hindrance - something that makes it difficult, an
desolateness. obstacle
The other two sentences are correct as it is. Reluctant - not wanting to do something
Meaning of the highlighted words:
Vandalism - act of willful destruction of a 16) Answer: B
property Sentence ‘b’ is erroneous. ‘Comprises of’ is
Desolate - empty in a sad way completely wrong; it becomes redundant in
Multifarious - having great diversity nature. Replace it with just the word ‘comprises’
The sentence after correction : We live in a
14) Answer: E gated community which comprises six blocks
All the given three sentences are correct and with 24 flats and 8 individual villas, in an area of
have used the highlighted word in the right way. about two thousand square metres.
The sentences are grammatically and All the other three sentences are correct as it is.
contextually correct and meaningful.
Meaning of the highlighted words: 17) Answer: A
Criterion - Singular form of the word criteria with Our manager doesn’t like being interrupted in the
meaning ‘a standard’ middle of the PI planning. Use ‘does not’ for the
Purport - intending or claiming something singular form of a third person - here the
Leakage - act of coming out manager is a third person.
All the other sentences are grammatically correct
15) Answer: D and meaningful.
The words ensure and reluctant are used
wrongly in the given sentences.Ensuring must 18) Answer: A

Click Here For Bundle PDF Course | support@guidely.in Page 12 of 13


SBI Clerk & RRB PO Mains PDF Course 2023
ENGLISH Day - 15

Who’s expansion is ‘who is’ or ‘who has’ but All the other sentences are grammatically correct
here we are talking about the lady’s husband and meaningful.
who belongs to her. So replace ‘who’s’ with 19) Answer: E
‘whose’. ‘Whose’ is used for belongings. All the given sentences are grammatically and
contextually correct and are error free.

Click Here For Bundle PDF Course | support@guidely.in Page 13 of 13


SBI Clerk & RRB PO Mains PDF Course 2023
Reasoning Ability Day - 16 (Eng)

Reasoning Ability
Directions (1-5): Study the following information a) 10
carefully and answer the given questions. b) 18
The first seventeen consecutive even numbers c) 14
are arranged in ascending order from top to d) 20
bottom. The word “BRAIN” is arranged against e) 8
the multiple of 6’s position from the top in
alphabetical order. 3. How many letters are placed between Y and V
Q is placed above B, where atleast one letter is in the arrangement?
placed between them. S is placed three letters a) As many letters placed between F and B
below Q. As many letters placed above S as b) Three
below V. Only one letter is placed between R and c) As many letters placed below S
M, which is placed below R. The number of d) Two
letters placed between M and V is two more than e) Five
the number of letters placed between N and C,
which is not placed below N. Only two letters are 4. Which of the following letters are placed above
placed between C and X. D, which is neither I?
placed adjacent to S nor placed adjacent to R, is I. Q
placed three positions below Y. The number of II. S
letters placed between D and Z is two more than III. V
the number of letters placed between I and Y. a) Only I
Only one letter is placed between A and F. The b) Only II
number of letters placed below F is one more c) Only I and III
than the number of letters placed above T. J is d) Only I and II
one of the letters in the arrangement. e) All I, II and III
1. What is the sum of the positions of J and C in
the arrangement? 5.If Y is related to D and F is related to J in a
a) 16 certain way, then in the same way who among
b) 28 the following person is related to M?
c) 20 a) T
d) 32 b) X
e) 24 c) J
d) A
2. What is the position of ‘X’ in the arrangement? e) Z

Click Here For Bundle PDF Course | support@guidely.in Page 1 of 10


SBI Clerk & RRB PO Mains PDF Course 2023
Reasoning Ability Day - 16 (Eng)

Directions (6-10): In the following questions, the d) Both conclusions I and II follow
symbols #, @, $, %, &and ! are used with the e) All conclusion follows
following meaning as illustrated below. study the
following information and answer the given 7. Statements:
question. In each of the question given below Ant & Mosquito * Fly $ Bug # Beetle$ Bees &
statements are followed by some conclusions. Ladybugs
You have to take the given statements to be true Conclusions:
even if they seem to be at variance from I. Beetle &? Mosquito
commonly known facts. Read the conclusions II. Fly % Beetle
and then decide which of the given conclusions III. Mosquito &? Fly
logically follows from the given statements, a) Both conclusions II and III follow
disregarding commonly known facts b) Only conclusion I follows
A$B means Some A is B c) Both conclusions I and III follow
A#B means No A is B d) Both conclusions I and II follow
A*B means Only a few A is B e) All conclusions follow
A@B means Only A is B
A&B means All A is B 8. Statements:
A%B means Some A is not B Fever * Cold % Cough & Piles $ Dengue
?means Possibility Conclusions:
Note: If ‘?’ is placed after any of the symbols I. Cough $? Dengue
mentioned above then it will be considered as a II. Piles % Cold
possible case of the symbol. III. Dengue &? Fever
For example A$?B means Some A being B is a a) Both conclusions II and III follow
possibility. b) Only conclusion I follows
6. Statements: c) Both conclusions I and III follow
Asia $ Europe # Australia & Africa * Antartic; d) Both conclusions I and II follow
Australia $ America e) All conclusions follow
Conclusions:
I. Antartic % Europe 9. Statements:
II. Asia % Australia Chocolate @Vannila; Chocolate $ Cherry;
III. Antartic $? Asia Caramel # Cherry; Caramel & Strawberry
a) Both conclusions II and III follow Conclusions:
b) Only conclusion I follows I. Chocolate &? Cherry
c) Both conclusions I and III follow II. Vannila $? Strawberry

Click Here For Bundle PDF Course | support@guidely.in Page 2 of 10


SBI Clerk & RRB PO Mains PDF Course 2023
Reasoning Ability Day - 16 (Eng)

III. Strawberry % Cherry D is 38 years old. Only two persons were born
a) Both conclusions II and III follow between D and G. G is four years elder than F
b) Only conclusion III follows and no one was born between them. The
c) Both conclusions I and III follow difference between the ages of D and F is 9
d) Both conclusions I and II follow years. A was born four persons after F. Less
e) All conclusions follow than two persons born after A, whose age is
equal to the last two digits of the birth year of G.
10. Statements: As many persons born after A as before H. B
Boat & Skullcandy; Boat $ Bose * Sony; JBL # was born either immediately before or
Sony immediately after H. The difference between the
Conclusions: ages of B and F is 11 years. The difference
I. Bose % JBL between the ages of H and B is two years. As
II. Skullcandy $ Bose many persons born between B and F as between
III. JBL &? Boat A and I. The sum of the ages of H and I is 100
a) Both conclusions II and III follow years. E was born before C. The age of E is five
b) Only conclusion I follows more than the sum of all the digits of the birth
c) Both conclusions I and III follow year of I. The age of C is two less than the last
d) Both conclusions I and II follow two digits of the birth year of G.
e) All conclusions follow 11. E was born in which of the following year?
a) 1979
Direction (11-15): Study the following information b) 1985
carefully and answer the given questions c) 1996
Nine persons - A, B, C, D, E, F, G, H and I were d) 1992
born in different years. The age of each person is e) 1998
calculated from the base year 2023.
Note: If it is given that A’s age is equal to the last 12. What is the sum of the ages of F and A?
two digits of the birth year of B, then the age of A a) 88
will be in any sequence. For example: If B was b) 74
born in 1973, A’s age is equal to the last two c) 95
digits of the birth year of B, then the age of A d) 68
may be either 37 or 73 years. The age of all the e) 85
persons is not more than 60 and not less than
25.

Click Here For Bundle PDF Course | support@guidely.in Page 3 of 10


SBI Clerk & RRB PO Mains PDF Course 2023
Reasoning Ability Day - 16 (Eng)

13. If the persons were born in reverse A family consist of eight members – P, Q, R, S,
alphabetical order from 1965, then how many T, U, V and W consumed different quantities of
persons remain in the same position? petrol in a particular month.
a) One Q is the daughter-in-law of P, who consumed
b) Two more petrol than S and V. P has only two
c) Four children. R is the daughter of P and consumed
d) Three more petrol than W but less than U. U is married
e) None to Q and is the brother-in-law of V, who has no
siblings. The son-in-law of P consumed more
14. Four of the following five are alike in a certain petrol than the daughter-in-law of P. W is the
way based on the given arrangement and thus niece of U and consumed more petrol than her
form a group. Which one of the following does grandfather. S is the mother-in-law of Q and
not belong to the group? consumed less petrol than her son-in-law. T is
a) D the child of the brother-in-law of V and consumed
b) E less petrol than the sister-in-law of R. The child
c) A of the son of S and the spouse of V are of
d) C opposite genders. The grandmother of W
e) G consumed 5 litres more petrol than her daughter-
in-law. The grandfather of T consumed 20 litres
15. Who among the following person is younger of petrol, which is 8 litres more than the petrol
than D? consumed by his spouse.
I. A 16. Who among the following persons address
II. E the spouse of P as their grandmother?
III. C I. T
a) Only I II. W
b) Only II III. Q
c) Only I and III a) Only I
d) Only I and II b) Only II
e) All I, II and III c) Only III
d) Both I and II
Direction (16-20): Study the following information e) Both I and III
carefully and answer the given questions
17. What is the sum of the quantity of petrol
consumed by both S and Q?

Click Here For Bundle PDF Course | support@guidely.in Page 4 of 10


SBI Clerk & RRB PO Mains PDF Course 2023
Reasoning Ability Day - 16 (Eng)

a) 21 litres 19. If the average of the quantity of petrol


b) 19 litres consumed by P, V, S and Q is 14, then what is
c) 25 litres the quantity of petrol consumed by V?
d) 27 litres a) 17 litres
e) Cannot be determined b) 24 litres
c) 13 litres
18. How many male members are there in the d) 15 litres
family? e) Cannot be determined
a) Six
b) Two 20. Which of the following option shows that the
c) Four first person is the parent of the second person?
d) Three a) WR
e) Five b) SW
c) PQ
d) TQ
e) SR
Click Here to Get the Detailed Video Solution for the above given Questions
Or Scan the QR Code to Get the Detailed Video Solutions

Answer Key with Explanation

Directions (1-5): Final arrangement:


1. Answer: B
2. Answer: C
3. Answer: A
4. Answer: D
5. Answer: E

Click Here For Bundle PDF Course | support@guidely.in Page 5 of 10


SBI Clerk & RRB PO Mains PDF Course 2023
Reasoning Ability Day - 16 (Eng)

Again we have,
 Only one letter is placed between R and
M, which is placed below R.
 The number of letters placed between M
and V is two more than the number of
letters placed between N and C, which is
not placed below N.
Only two letters are placed between C and X.

We have,
 The word “BRAIN” is arranged against the
multiple of 6’s position from the top in
alphabetical order.
 Q is placed above B, where atleast one
letter is placed between them.
 S is placed three letters below Q.
 As many letters placed above S as below Again we have,

V.  D, which is neither placed adjacent to S


nor placed adjacent to R, is placed three
positions below Y.
 The number of letters placed between D
and Z is two more than the number of
letters placed between I and Y.

Click Here For Bundle PDF Course | support@guidely.in Page 6 of 10


SBI Clerk & RRB PO Mains PDF Course 2023
Reasoning Ability Day - 16 (Eng)

Again we have,
6. Answer: A
 Only one letter is placed between A and
F.
 The number of letters placed below F is
one more than the number of letters
placed above T.
 J is one of the letters in the arrangement.
We cannot place T in case 1 and case 3. Hence
7. Answer: D
it is eliminated. Thus, case 2 gives the final
arrangement.

8. Answer: C

Click Here For Bundle PDF Course | support@guidely.in Page 7 of 10


SBI Clerk & RRB PO Mains PDF Course 2023
Reasoning Ability Day - 16 (Eng)

9. Answer: B

We have,
 D is 38 years old.
10. Answer: E  Only two persons were born between D
and G.
 G is four years elder than F and no one
was born between them.
 The difference between the ages of D and
F is 9 years.
From the above conditions, we have two
possibilities,
Directions (11-15):
11. Answer: D
12. Answer: B
13. Answer: C
14. Answer: A (All the persons were born in an
even numbered year except D)
15. Answer: E
Again we have,
Final arrangement:
 A was born four persons after F.
 Less than two persons born after A,
whose age is equal to the last two digits
of the birth year of G.
 As many persons born after A as before
H.

Click Here For Bundle PDF Course | support@guidely.in Page 8 of 10


SBI Clerk & RRB PO Mains PDF Course 2023
Reasoning Ability Day - 16 (Eng)

 B was born either immediately before or  The age of E is five more than the sum of
immediately after H. all the digits of the birth year of I.
We cannot place A’s age in case 1. Hence it is  The age of C is two less than the last two
eliminated. digits of the birth year of G.
We cannot fix the age of E in case 2a. Hence it
is eliminated. Thus, case 1 gives the final
arrangement.

Again we have,
 The difference between the ages of B and
F is 11 years.
 The difference between the ages of H and
B is two years.
 As many persons born between B and F
as between A and I.
Directions (16-20):
 The sum of the ages of H and I is 100
16. Answer: D
years.
17. Answer: B
18. Answer: C
19. Answer: A
20. Answer: E
Final arrangement:

Again we have,
 E was born before C.
We have,

Click Here For Bundle PDF Course | support@guidely.in Page 9 of 10


SBI Clerk & RRB PO Mains PDF Course 2023
Reasoning Ability Day - 16 (Eng)

 Q is the daughter-in-law of P, who


consumed more petrol than S and V.
 P has only two children.
 R is the daughter of P and consumed
more petrol than W but less than U.
 U is married to Q and is the brother-in-law
of V, who has no siblings.

Again we have,
 T is the child of the brother-in-law of V
and consumed less petrol than the sister-
in-law of R.
 The child of the son of S and the spouse
Again we have, of V are of opposite genders.
 The son-in-law of P consumed more  The grandmother of W consumed 5 litres
petrol than the daughter-in-law of P. more than her daughter-in-law.
 W is the niece of U and consumed more  The grandfather of T consumed 20 litres
petrol than her grandfather. of petrol, which is 8 litres more than the
 S is the mother-in-law of Q and consumed petrol consumed by his spouse.
less petrol than her son-in-law.

Click Here For Bundle PDF Course | support@guidely.in Page 10 of 10


SBI Clerk & RRB PO Mains PDF Course 2023
Quantitative Aptitude Day - 16 (Eng)

Quantitative Aptitude

Directions (1-4): Study the following data 1) Find the average number of items sold by
carefully and answer the questions. company B in each of the given days.
Data given below shows the number of items a) 112
produced and the number of items sold (out of b) 142
total produced items) by two companies A and B c) None of these
in 4 different days Monday, Tuesday, d) 122
Wednesday and Thursday. e) 132
Table given below shows the following data.
2)If company A sold its items at ₹60 per item and
company B sold its items at ₹75 per item, then
find the difference between the amount received
by companies A and B on Wednesday.
a) ₹1060
b) ₹1050
c) ₹1080
d) ₹1070
e) ₹1090
Note:
1: Total number of produced items = Number of
3) Which of the following is/are true?
(sold + unsold) items
P: On Thursday, ratio of number of unsold items
2: Number of items produced by company A on
of company A to that of company B is 3: 11.
Thursday, is 75% of that produced on Monday
Q: Average number of unsold items of company
and also 5 more than that produced on Tuesday.
A on Monday, Tuesday and Wednesday is 65.
Ratio of number of items produced by company
R: On Tuesday, total number of items sold by
A on Tuesday to that produced on Wednesday is
companies A and B together is 68.33% of the
5: 4. Total number of items produced by
total number of item produced by them together.
company A in all the 4 days together is 735.
a) Only P and R
3: On Monday, 62.5% of total produced items of
b) All are true
company A were sold, which is equal to that on
c) Only Q
Thursday. Number of items sold by company A
d) Only Q and R
on Tuesday is 70% of that sold on Monday and
e) None is true
also equal to that sold on Wednesday.

Click Here For Bundle PDF Course | support@guidely.in Page 1 of 15


SBI Clerk & RRB PO Mains PDF Course 2023
Quantitative Aptitude Day – 16 (Eng)

4) Number of items produced by companies A Number of sold units of the product was 500,
and B on Friday is respectively 95 and 25 more which was 8(1/3)% of the total cost of raw
than that produced by them on Tuesday. If material to manufacture the sold units. Ratio of
company A sold 60% of total produced items on selling cost per unit to the cost of labouring per
Friday and company B sold 80% of total unit was 3: 1. Selling cost per unit was 4.8% of
produced items on Friday, then find the ratio of the number of sold units.
number of items sold by company A to company 5) Profit of the company in 2015 was what per
B on Friday. cent more/less than that of the company in
a) 9: 7 2016?
b) 81: 70 a) 70%
c) 27: 20 b) 83.33%
d) 54: 35 c) 75%
e) 3: 2 d) 80%
e) 87.5%
Directions (5-7): Study the following data
carefully and answer the questions. 6) Which of the following is/are not true?
A certain product manufacturing company A: Ratio of sales in 2015 to that in 2016 is 5: 6.
analysed its sales data over two different years B: Ratio of total cost of raw material to
2015 and 2016. The company used following manufacture the sold units in 2015 to that in
equations to analyse its sales data. 2016 is 3: 4.
1: Sales = Selling cost per unit * Number of units C: Selling cost per unit in 2016 was 92% of that
sold in 2015.
2: Profit = Sales – Total cost a) Only C
3: Total cost = (Cost of raw material + Coat of b) Only A and C
labouring) * Number of units sold c) Only B
In 2015: d) Only B and C
Number of sold units of the product was 400 at e) Only A
the cost of ₹25 per unit. The cost of raw material
to manufacture one unit of the product was 45% 7) If profit per cent in 2015 was M% and that in
of the selling cost per unit and ratio of the cost of 2016 was N%, then find the value of (13M –
raw material to manufacture one unit to the cost 15N)?
of labouring per unit was 9: 4. a) 450
In 2016: b) 200
c) 350

Click Here For Bundle PDF Course | support@guidely.in Page 2 of 15


SBI Clerk & RRB PO Mains PDF Course 2023
Quantitative Aptitude Day – 16 (Eng)

d) 400 e) 250

Directions (8-12): Study the following data carefully and answer the questions.
Data given below is related to the monthly salary and monthly expenditure of 5 different persons A, B, C,
D and E.
Pie chart given below shows the percentage distribution of monthly salary of each of the given person.

Pie chart given below shows the percentage distribution of monthly expenditure of each of the given
person.

Note:

Click Here For Bundle PDF Course | support@guidely.in Page 3 of 15


SBI Clerk & RRB PO Mains PDF Course 2023
Quantitative Aptitude Day – 16 (Eng)

1: Difference between A’s monthly salary and C’s monthly salary is ₹12000 and A’s monthly salary is
₹6000 more than B’s monthly salary.
2: C’s monthly expenditure is of his monthly salary.
8) Total monthly expenditure of A and C together b) Only P
is what per cent of total monthly salary of A and c) Only Q and R
C together? d) Only R
a) e) Only P and R
b)
c) 11) Find the average monthly savings of each of

d) the given person.


a) ₹12500
e)
b) ₹15000
c) ₹13500
9) Find the ratio of total monthly savings of B, C
d) ₹12000
and D together to the monthly savings of E.
e) ₹14000
a) 1: 1
b) 28: 29
12) If E spends 43.75% of his monthly
c) 55: 58
expenditure on his wife, 25% of his monthly
d) 5: 6
expenditure on his child and the remaining at
e) 131: 145
himself, then what per cent of his monthly salary
does he spend on himself?
10)If B saves M% of his monthly salary and D
a) 9.09%
saves N% of his monthly salary, then the value
b) 16.67%
of 3(M – N) lies between which of the following?
c) 12.5%
P: 35 and 42
d) 13.33%
Q: 38 and 45
e) 11.11%
R: 32 and 38
a) Only P and Q

Directions (13-16): Study the following data carefully and answer the questions.
Bar graph given below shows the percentage distribution of sales generated by different product
categories i,e; Electronics, Clothing, Sporting goods and Food in three different malls A, B and C on a
particular day.

Click Here For Bundle PDF Course | support@guidely.in Page 4 of 15


SBI Clerk & RRB PO Mains PDF Course 2023
Quantitative Aptitude Day – 16 (Eng)

Note:
1: In each mall, sales are generated by the given categories only.
2: Electronics sales in mall B is ₹3400 more than that in mall A and double of clothing sales in mall B is
₹7000 more than the clothing sales in mall A.
3: Food sales in mall A is ₹900 less than the Food sales in mall C.
13)If clothing sales in mall A is M% more than c) Ratio of clothing sales in mall B to that in mall
that in mall B and clothing sales in mall C is N% C is 16: 19.
more than that in mall B, then find the difference d) Both (a) and (b)
between M and N. e) Both (a) and (c)
a) 15.75
b) 21.75 15) Total sales in mall A is what per cent more or
c) 24.75 less than that in mall B?
d) 18.75 a) 12.5%
e) 12.75 b) 15%
c) 16.67%
14) Which of the following is/are true? d) 10%
a) Food sales in mall A is 80% of Food sales in e) 20%
mall C.
b) Average of electronic sales in mall A and that 16)If electronic sales in mall D is 75% of that in
in mall B is ₹14500. mall A, clothing sales in mall D is 120% of that in
mall B, sporting goods sales in mall D is 125% of

Click Here For Bundle PDF Course | support@guidely.in Page 5 of 15


SBI Clerk & RRB PO Mains PDF Course 2023
Quantitative Aptitude Day – 16 (Eng)

that in mall C and total sales in mall D is ₹36000, 18) Three inlet pipes A, B and C and one outlet
then find the food sales in mall D? pipe D are connected to a tank. If ratio of time
a) ₹8450 taken by pipes A, B and C together to fill the tank
b) ₹10450 to that time taken by pipe D alone to empty the
c) ₹9450 tank is 1: 9, then in what time pipes A and D
d) ₹7450 together will fill the tank?
e) ₹11450 Statement I: Pipes C and D together can fill the
tank in 12 minutes, ratio of B’s efficiency to C’s
Directions (17-20): In each question, some efficiency is 3: 4 and ratio of ratio of B’s efficiency
statements are given. You have to decide to D’s efficiency is 3: 1.
whether the data provided in the statements are Statement II: Ratio of A’s efficiency to C’s
sufficient to answer the question. (Level: efficiency is 1: 2, pipe B alone can fill the tank in
Moderate) 12 minutes and time taken by pipe C alone to fill
17) Find the simple interest received on ₹(X + the tank is 3 minutes less than that time taken by
500) at 4R% rate after 3 years? pipe B alone to fill the tank.
Statement I: Ratio of simple interest received on Statement III: Pipes B and D together can fill the
₹X at 3R% rate after 2 years to that simple tank in 18 minutes.
interest received on ₹(X – 800) at 2R% rate after a) Statements I alone is sufficient to answer the
5 years is 9: 7. question.
Statement II: Difference between compound b) All the statements together are not sufficient to
interest and simple interest on ₹(X – 300) at 15% answer the question.
rate after 2 years is ₹27. c)Statements II alone is sufficient to answer the
a) Statements I and II together are necessary to question.
answer the question. d) Either statement I alone or statement II alone
b) Both the statements together are not sufficient is sufficient to answer the question.
to answer the question. e) Statement I and III together are sufficient to
c) Statement II alone is sufficient to answer the answer the question.
question.
d) Either statement I alone or statement II alone 19) In mixture A, quantity of milk is 18 litres more
is sufficient to answer the question. than that of water and cost of the pure milk is
e) Statement I alone is sufficient to answer the ₹42 per litre. Find the cost of mixture A?
question. Statement I: In mixture B, quantity of milk is 32
litre more than that of water and cost of mixture B
is ₹37.8 per litre.

Click Here For Bundle PDF Course | support@guidely.in Page 6 of 15


SBI Clerk & RRB PO Mains PDF Course 2023
Quantitative Aptitude Day – 16 (Eng)

Statement II: Ratio of milk to water in mixture B is in family B is 6: 5. Find the average age of family
9: 1 and ratio of milk in mixture A to that in B.
mixture B is 2: 3. Statement I: Ratio of wife’s age in family A to the
a) Both the statements together are not sufficient wife’s age in family B is 7: 6 and child’s age in
to answer the question. family B is 1 year less than the child’s age in
b) Either statement I alone or statement II alone family A. In family B, husband’s age is 4 years
is sufficient to answer the question. more than wife’s age.
c) Statements I alone is sufficient to answer the Statement II: In family A, average age of
question. husband and wife is 45 years.
d) Both the statements together are necessary to a) Both the statements together are not sufficient
answer the question. to answer the question.
e) Statement II alone is sufficient to answer the b) Both the statements together are necessary to
question. answer the question.
c)Statements I alone is sufficient to answer the
20) There are two families A and B and each question.
family has a husband, a wife and a child. d) Either statement I alone or statement II alone
Average age of family A is 35 years and ratio of is sufficient to answer the question.
husband’s age in family A to the husband’s age e) Statements II alone is sufficient to answer the
question.

Click Here to Get the Detailed Video Solution for the above given Questions
Or Scan the QR Code to Get the Detailed Video Solutions

Answer Key with Explanation

Directions (1-4): Let the number of items produced by company A


in Monday = 4x

Click Here For Bundle PDF Course | support@guidely.in Page 7 of 15


SBI Clerk & RRB PO Mains PDF Course 2023
Quantitative Aptitude Day - 16 (Eng)

So, the number of items produced by company A


on Thursday = 75% of 4x = 3x
The number of items produced by company A on
Tuesday = 3x – 5
And the number of items produced by company
A on Wednesday =
So,

62x – 45 = 3675
x = 60
Number of items produced by company A on
Monday = 4 * 60 = 240 1) Answer: D
Number of items produced by company A on Number of items sold by company B on Monday
Tuesday = 3 * 60 - 5 = 175 = 180
Number of items produced by company A on Number of items sold by company B on Tuesday
Wednesday = = 140 = 100
Number of items produced by company A on Number of items sold by company B on
Thursday = 3 * 60 = 180 Wednesday = 98
Number of items sold by company A on Monday Number of items sold by company B on
= 62.5% of 240 = 150 Thursday = 110
Number of items sold by company A on Required average =
Thursday = 150
Number of items sold by company A on Tuesday 2) Answer: B
= 70% of 150 = 105 Number of items sold by company A on
Number of items sold by company A on Wednesday = 105
Wednesday = 105 Number of items sold by company B on
Wednesday = 98
Total amount received by company A on
Wednesday = 105 * 60 = ₹6300
Total amount received by company B on
Wednesday = 98 * 75 = ₹7350
Required difference = 7350 – 6300 = ₹1050

Click Here For Bundle PDF Course | support@guidely.in Page 8 of 15


SBI Clerk & RRB PO Mains PDF Course 2023
Quantitative Aptitude Day - 16 (Eng)

Number of items sold by company A on Friday =


3) Answer: B 60% of 270 = 162
From P: Number of items sold by company B on Friday =
Number of unsold items of company A on 80% of 150 = 120
Thursday = 180 – 150 = 30 Required ratio = 162: 120 = 27: 20
Number of unsold items of company B on
Thursday = 220 – 110 = 110 Directions (5-7):
Required ratio = 30: 110 = 3: 11 In 2015:
So, P is true. Number of sold units = 400
From Q: Selling cost per unit = ₹25
Number of unsold items of company A on The cost of raw material to manufacture one unit
Monday = 240 – 150 = 90 = 45% of 25 = ₹11.25
Number of unsold items of company A on And the cost of labouring per unit = =
Tuesday = 175 – 105 = 70 ₹5
Number of unsold items of company A on Total cost = (11.25 + 5) * 400 = ₹6500
Wednesday = 140 – 105 = 35 Total sales = 25 * 400 = ₹10000
Required average = Profit = 10000 – 6500 = ₹3500
So, Q is true. In 2016:
From R: Number of sold units = 500
Total number of items produced by companies A Selling cost per unit = 4.8% of 500 = ₹24
and B together on Tuesday = 175 + 125 = 300 Total cost of raw material to manufacture the
Total number of items sold by companies A and sold units = = ₹6000
B together on Tuesday= 105 + 100 = 205 And the cost of labouring per unit = = ₹8
Required percentage = Total cost = 6000 + 500 * 8 = ₹10000
So, R is true. Sales = 24 * 500 = ₹12000
Hence, all are true. Profit = 12000 – 10000 = ₹2000
5) Answer: C
4) Answer: C Profit of the company in 2015 = ₹3500
Number of items produced by company A on Profit of the company in 2016 = ₹2000
Friday = 175 + 95 = 270 Required percentage = = 75%
Number of items produced by company B on
Friday = 125 + 25 = 150 6) Answer: A
From A:

Click Here For Bundle PDF Course | support@guidely.in Page 9 of 15


SBI Clerk & RRB PO Mains PDF Course 2023
Quantitative Aptitude Day - 16 (Eng)

Sales of the company in 2015 = ₹10000 So, A’s monthly salary = =


Sales of the company in 2016 = ₹12000 ₹36000
Required ratio = 10000: 12000 = 5: 6 C’s monthly salary = 36000 – 12000 = ₹24000
So, A is true. D’s monthly salary = = ₹15000
From B: Since, B’s monthly salary = 36000 – 6000 =
Total cost of raw material to manufacture the ₹30000
sold units in 2015 = 11.25 * 400 = ₹4500 So, the value of X = = 20
Total cost of raw material to manufacture the And the value of Z = 60 – 20 – 10 = 30
sold units in 2016 = ₹6000 So, E’s monthly salary = = ₹45000
Required ratio = 4500: 6000 = 3: 4
Since, C’s monthly expenditure is of his
So, B is true.
monthly salary.
From C:
So, C’s monthly expenditure = of 24000 =
Selling cost per unit in 2015 = ₹25
₹8000
Selling cost per unit in 2016 = ₹24
A’s monthly expenditure = = ₹24000
Required percentage = = 96%
B’s monthly expenditure = = ₹20000
So, C is not true.
D’s monthly expenditure = = ₹12000
Hence, only C is not true.
E’s monthly expenditure = = ₹16000

7) Answer: D
Profit per cent in 2015 = M =
Profit per cent in 2016 = N =
So, the value of 13M – 15N = 700 – 300 = 400
Directions (8-12):
From the 1st pie chart:
X + Y + Z = 100 – (24 + 16)
X + Y + Z = 60 -------------(1)
From the 2nd pie chart:
(X – 5) + (Z – 5) = 40
8) Answer: B
X + Z = 50 -------------(2)
Total monthly salary of A and C together =
By equation (1) – equation (2):
36000 + 24000 = ₹60000
Y = 10
Total monthly expenditure of A and C together =
Since, the difference between A’s and C’s
24000 + 8000 = ₹32000
monthly salaries = ₹12000

Click Here For Bundle PDF Course | support@guidely.in Page 10 of 15


SBI Clerk & RRB PO Mains PDF Course 2023
Quantitative Aptitude Day - 16 (Eng)

Required percentage = E’s monthly salary = ₹45000


Required percentage = = 11.11%

9) Answer: A Directions (13-16):


Total monthly savings of B, C and D together = Let the electronics sales, clothing sales, sporting
10000 + 16000 + 3000 = ₹29000 sales and food sales in mall A are ₹35a, ₹25a,
Monthly savings of E = ₹29000 ₹30a and ₹10a respectively.
Required ratio = 29000: 29000 = 1: 1 Let the electronics sales, clothing sales, sporting
sales and food sales in mall B are ₹40b, ₹20b,
10) Answer: A ₹25b and ₹15b respectively.
B’s monthly salary = ₹30000 Let the electronics sales, clothing sales, sporting
B’s monthly savings = ₹10000 sales and food sales in mall C are ₹30c, ₹35c,
So, M = ₹20c and ₹15c respectively.
D’s monthly salary = ₹15000 Since, the electronics sales in mall B is ₹3400
D’s monthly savings = ₹3000 more than that in mall A.
So, N = So,
Now, the value of 3(M – N) = 40b – 35a = 3400
8b – 7a = 680 -----------(1)

So, the value of 3(M – N) lies between 35 and 42 Since, double of clothing sales in mall B is ₹7000

and also between 38 and 45. more than the clothing sales in mall A.
40b – 25a = 7000

11) Answer: E 8b – 5a = 1400 -----------(2)

Average monthly expenditure of each of the By equation (2) – equation (1):

given person: 8b – 5a – 8b + 7a = 1400 – 680


a = 360
From equation (1):
8b – 2520 = 680
12) Answer: E
b = 400
E’s expenditure on his wife = 43.75% of 16000 =
Since, the food sales in mall A is ₹900 less than
₹7000
that in mall C.
E’s expenditure on his child = 25% of 16000 =
So,
₹4000
15c – 10a = 900
So, E’s expenditure on himself = 16000 – 7000 –
c = 300
4000 = ₹5000

Click Here For Bundle PDF Course | support@guidely.in Page 11 of 15


SBI Clerk & RRB PO Mains PDF Course 2023
Quantitative Aptitude Day - 16 (Eng)

15) Answer: D
Total sales in mall A = 12600 + 9000 + 10800 +
3600 = ₹36000
Total sales in mall B = 16000 + 8000 + 10000 +
6000 = ₹40000
Required percentage = = 10%

16) Answer: C
Electronic sales in mall D = 75% of 12600 =
₹9450
Clothing sales in mall D = 120% of 8000 = ₹9600
Sporting sales in mall D = 125% of 6000 =
₹7500
13) Answer: D
So, food sales in mall D = 36000 – (9450 + 9600
Clothing sales in mall A = ₹9000
+ 7500) = ₹9450
Clothing sales in mall B = ₹8000
Clothing sales in mall C = ₹10500
17) Answer: B
So, M = = 12.5%
From statement I alone:
And N = = 31.25%
Required difference = 31.25 – 12.5 = 18.75
14) Answer: A
From option (a): 21X = 45X – 36000
Food sales in mall A = ₹3600 X = 1500
Food sales in mall C = ₹4500 Since, we can’t find the value of R.
Required percentage = So, statement I alone is not sufficient to answer
From option (b): the question.
Average of electronic sales in mall A and that in From statement II alone:
mall B = = ₹14300
From option (c):
X – 300 = 1200
Ratio of clothing sales in mall B to that in mall C
X = 1500
= 8000: 10500 = 16: 21
Since, we can’t find the value of R.
Hence, option (a) is correct.
So, statement II alone is not sufficient to answer
the question.

Click Here For Bundle PDF Course | support@guidely.in Page 12 of 15


SBI Clerk & RRB PO Mains PDF Course 2023
Quantitative Aptitude Day - 16 (Eng)

From statements I and II together: Part of tank filled by A and D together in 1


---------------(1) minute =

-----------------(2) The time taken by pipes A and D together to fill


the tank = 36 minutes
Since, we can’t find the value of R by solving
So, statement I alone is sufficient to answer the
both the equations.
question.
Hence, statements I and II are not sufficient to
answer the question. From statement II alone:
Time taken by pipe B alone to fill the tank = 12
minutes
18) Answer: D
Time taken by pipe C alone to fill the tank = 12 –
Since, ratio of time taken by pipes A, B and C
3 = 9 minutes
together to fill the tank to that taken by pipe D
alone to empty the tank is 1: 9. Time taken by pipe A alone to fill the tank =
= 18 minutes
So,
-------------(1) From equation (1):

From statement I alone:


Let the time taken by pipe B alone and pipe C D = 36
alone to fill the tank is 4t minutes and 3t minutes Part of tank filled by A and D together in 1
respectively. minute =
So, the time taken by pipe D alone to empty the The time taken by pipes A and D together to fill
tank = 3 * 4t = 12t minutes the tank = 36 minutes
And, So, statement II alone is sufficient to answer the
question.

t=3 From statement III alone:

Time taken by pipe B alone to fill the tank = 12 Since we are given the time taken by B and D

minutes together to fill the tank and we cannot calculate

Time taken by pipe C alone to fill the tank = 9 the time taken by pipes B and D alone to fill the

minutes tank. Hence, we cannot determine the time

Time taken by pipe D alone to empty the tank = taken by A and D together to fill the tank.

36 minutes So, statement III alone is not sufficient to answer

From equation (1): the question.


Hence, either statement I alone or statement II
alone is sufficient to answer the question.
A = 18

Click Here For Bundle PDF Course | support@guidely.in Page 13 of 15


SBI Clerk & RRB PO Mains PDF Course 2023
Quantitative Aptitude Day - 16 (Eng)

So, the cost of mixture A = ₹33.6 per L


19) Answer: D Hence, both the statements together are
Let the quantity of water in mixture A = ‘a’ litre necessary to answer the question.
So, the quantity of milk in mixture A = (a + 18)
litre 20) Answer: B
Let the cost of mixture A is ₹’y’ per litre Let the ages of husband, wife and child in family
So, A are ‘6x’ years, ‘7y’ years and ‘z’ years.
---------------(1) So,
From statement I alone: 6x + 7y + z = 3 * 35
Let the quantity of water in mixture B = ‘b’ litre 6x + 7y + z = 105 --------------(1)
So, the quantity of milk in mixture A = (b + 32) Now, the husband’s age in family B = =
litre ‘5x’ years
And the cost of mixture B = ₹37.8 per litre From statement I alone:
So, The wife’s age in family B = = ‘6y’ years
And the child’s age in family B = (z – 1) years
So,
b=4
5x – 6y = 4
Since, there is no relation between ‘a’, ‘b’ and ‘y’.
Since, we can’t solve the equation.
So, statement I alone is not sufficient to answer
So, statement I alone is not sufficient to answer
the question.
the question.
From statement II alone:
From statement II alone:
Since, we don’t know the quantities of milk and
Let the wife’s age in family B is ‘w’ years.
water in mixture B and we can’t determine the
So,
quantity of milk and water in mixture A.
5x + w = 90
So, statement II alone is not sufficient to answer
Since, we can’t solve the equation.
the question.
So, statement II alone is not sufficient to answer
From both the statements together:
the question.
Quantity of water in mixture B = 4 litre
From both the statements together:
Quantity of milk in mixture B = (4 + 32) = 36 litre
The wife’s age in family B = ‘6y’ years
Quantity of milk in mixture A = = 24 litre
The child’s age in family B = (z – 1) years
Quantity of water in mixture A = 24 – 18 = 6 litre
Since, in family A, average age of husband and
From equation (1):
wife is 45 years.
So,

Click Here For Bundle PDF Course | support@guidely.in Page 14 of 15


SBI Clerk & RRB PO Mains PDF Course 2023
Quantitative Aptitude Day - 16 (Eng)

6x + 7y = 90 ---------- (2) x = 8, y = 6
And, z = 105 – 90 = 15 Husband’s age in family B = 5 * 8 = 40 years
Since, in family B, husband’s age is 4 years Wife’s age in family B = 6 * 6 = 36 years
more than wife’s age. Child’s age in family B = 15 – 1 = 14 years
So, Average age of family B = = 30 years
5x – 6y = 4 ----------(3) Hence, both the statements together are
By equation (2) * 6 + equation (3) * 7: necessary to answer the question.
36x + 42y + 35x – 42y = 540 + 28

Click Here For Bundle PDF Course | support@guidely.in Page 15 of 15


SBI Clerk & RRB PO Mains PDF Course 2023
ENGLISH Day - 16

English Language

Directions (1-5): The given questions carry two E. No swapping needed


sentences in which three words each have been
highlighted. These words might not be in their 3.
correct positions. You must choose an option Experts (A) noted that the dam, about 70
which carries the correct pairs of words that kilometres (44 miles) to the east of the city of
when interchanged would form correct and Kherson, was (B) believed to be in disrepair and
meaningful sentences. (C) vulnerable to collapse
1. Besides a vigilant (D) approach like being
High global oil prices have (A) measure mindful of unusual odours, discolouration, or (E)
consequences for India that (B) imports 85% of changes in food texture, cutting-edge technology
its (C)transactions. can play a pivotal role in (F) ensuring food
The current (D) account—which is the safety.
broadest (E) seriousof India’s goods and A. A-D, B-F
services (F) requirements—slipped into a deficit B. B-E, A-D, C-F
A. A-E, B-D C. A-E, C-D, B-F
B. B-D, C-F D. B-D, A-C
C. A-E, C-F E. No swapping needed
D. A-E, B-D, C-F
E. No swapping needed 4.
Govt can’t do the (A) heavy lifting on recovery
2. alone, but the (B)transactionsfor business (C)
The external affairs minister also (A)dangerthat needs to get easier.
India is making significant economic (B) impact The current (D) account—which is the
that has been recognised (C)rang. broadest (E) measure of India’s goods and
Amid the disaster response, artillery shelling services (F) environment.
(D) globallyout as people scrambled to get out of A. A-D
the (E) saidzone, climbing onto military (F) trucks B. C-E
or rafts. C. B-F
A. B-D, A-E, C-F D. B-E
B. A-F, C-E E. No swapping needed
C. C-D, B-F
D. A-E, C-D 5.

Click Here For Bundle PDF Course | support@guidely.in Page 1 of 11


SBI Clerk & RRB PO Mains PDF Course 2023
ENGLISH Day - 16

Punjab and Haryana farmers are (A) worsento C. Our teachers are unpaid and do not get any
paddy and a (B) importscycle that makes stubble job assistance
(C) hookedan imperative. D. Poor the learning outcomes are for students in
High global oil prices have serious rural areas
consequences for India that (D) cultivation85% of E. Some measures have been taken, but these
its requirements and will (E)burningthe current have created more problems
account (F) situation in FY22.
A. A-E, B-D, C-F 7. This should have been obvious but, perhaps, it
B. Only A-E is not especially to the esteemed Brihanmumbai
C. B-D, C-E Municipal Corporation — there is a correlation
D. A-F, B-D, C-E between Mumbai’s open spaces, which includes
E. No swapping needed green cover, and the high temperatures as well
as water-logging or floods that the city routinely
Directions (6-10): The given questions carry a deals with now. The greater the number of open
paragraph from which the last line has been spaces and the wider their spread across the
deleted. You must choose an option that can 482 square kilometres, the lower the impact of
complete the paragraph contextually and the Urban Heat Island effect in which certain
grammatically in the most logical manner as your areas or zones register 2 to 4 degrees Celsius
answer. higher than their nearest neighbourhoods.
6. UNESCO’s recent State of Education report Similarly, the more the open spaces, the rain
for India paints a bleak picture of India’s strength water is absorbed faster into underground
of skilled teachers. The country is short of 11 aquifers and_________________________.
lakh skilled teachers, with the rural parts of the A. the city combats climate change impact
country bearing the larger share of the deficit— affecting the lives and well-being of millions
69% of the vacancies are in schools in these B. not all public open spaces are public open
areas. How acute the problem is for those spaces
lacking privilege is spelled out in the fact that C. there is evidentially less water-logging around
89% of rural schools in the country are single- during the monsoon
teacher schools. No wonder, then, _________. D. the BMC might put out,that Mumbai has 291
A. Globally teaching is not an attractive gardens or parks
profession E. None of these
B. India had a mediocre showing in the UN’s
education index 8. It is NOT for the environment. It is also not for
other species that we coexist with on this planet.

Click Here For Bundle PDF Course | support@guidely.in Page 2 of 11


SBI Clerk & RRB PO Mains PDF Course 2023
ENGLISH Day - 16

It is for us, the human race. Celebration of United 10. The online gaming industry in India has
Nations events like World Environment Day and witnessed tremendous growth in recent years,
World Oceans Day every year on June 5 and with millions of users and significant revenue
June 8, respectively, acts as a reminder for us generation.This exponential growth prompted a
that apart from going about with our day-to-day need for effective regulation to ensure consumer
activities, we should be______________. protection, maintain ethical standards, and
A. exacerbating all the issues like plastic address potential societal concerns. To address
pollution these concerns and shape the regulatory
B. considering the magnitude of the problem framework of the growing sector, the ministry of
C. thinking out of the box based on current electronics and IT recently ___________.
realities to make our world a better place to live A. banned all the gaming websites for safety
in reasons
D. mindful of changes around us that impact our B. addressed a conference discussing the issue
collective well-being C. notified rules for the online gaming industry
E. Both (a) and (d) D. mandated the use of AI in the gaming devices
E. Both (a) and (c)
9. The market share of microfinance companies
has risen 330 basis points (bps) in FY23 at the Directions (11-15): In each question below, a
cost of banks, whose share has fallen by 430 word has been
bps.Credit growth at microfinance companies is given followed by three sentences. One or more
driven by an increase in active loan accounts of these
and higher ticket-size for individual lending.Last sentences can take the given word correctly. You
year, the RBI removed the caps on the pricing of must choose the
small loans given by NBFC-MFIs, option that carries the sentence/(s) that correctly
which________________________. fits the given wordin bold.
A. provides a level playing field to all the players 11. Prompted
in microfinancing activities (i) This exponential growth __________ a need for
B. works wellto make the bankers realise they effective regulation.
have healthy competition all around (ii) This new case has _________ concern that
C. helps the micro-finance institutions gain the virus will spread
momentum in the tough credit market (iii) “Did you hear me?” he_________ when his
D. made them vulnerable to the changing interest friend did not respond to his first question.
rates by the RBI A. Only (ii)
E. None of these B. Both (i) and (ii)

Click Here For Bundle PDF Course | support@guidely.in Page 3 of 11


SBI Clerk & RRB PO Mains PDF Course 2023
ENGLISH Day - 16

C. All (i), (ii) and (iii) (ii) ________ the ruler and the middle of the
D. Both (i) and (iii) paper.
E. Only (iii) (iii) Every afternoon, the teacher has to ________
the desks that her students have pushed out of
12. Resilience place.
(i) The company that measures TV ratings prides A. Both (ii) and (i)
itself on the _________ of its calculations. B. Only (iii)
(ii) Figures released yesterday showed the C. Both (iii) and (i)
__________ of consumer spending last month. D. None
(iii) "Don't miss a tour of the Berlin Experience, E. Both (a) and (b)
which traces its _________history.
A. Only (i) 15. Bonhomie
B. Both (ii) and (iii) (i) When the storms ________, the uncertainties
C. Only (iii) do not.
D. Both (i) and (iii) (ii) Companies sought to avoid _________ on the
E. Only (ii) grid by producing power ‘in house’.
(iii) There was a casual _________ between the
13. Distress actors at rehearsals.
(i) The constable received a _______ call, and A. Only (iii)
saw two youths attacking his colleague. B. Both (i) and (ii)
(ii) The commission gave the case a _________ C. All (i), (ii) and (iii)
which it might otherwise have escaped. D. Only (ii)
(iii) That leaves the question of how immediate or E. None
________ the threatened violence would be.
A. Both (ii) and (iii) Directions (16-20): Five sentences have been
B. Only (i) given below. Read the sentences carefully
C. All (i), (ii) and (iii) before the questions and sequence the given
D. Only (iii) details into a meaningful paragraph to answer
E. None the questions that follow:
P. In the climate negotiations for instance, areas
14. Align of interest to developing countries are not
(i) Here is my first test-fitting of the front panel - covered or sparsely covered, while other areas
the buttons _______ perfectly! are over-regulated.

Click Here For Bundle PDF Course | support@guidely.in Page 4 of 11


SBI Clerk & RRB PO Mains PDF Course 2023
ENGLISH Day - 16

Q. Equitable sustainable development is not not independent silos but reflect their urban
even discussed. lifestyles.
R. At COP27, the policy debate was no longer
legitimised by science. There seems to be a 17. Which of the given sentences should come
concerted effort to fraudulently change the basic before P in the given sequence?
structure of the Climate Treaty. A. The foundation of the Climate Treaty in
S. There are three problems with the current international environmental law is questionable.
negotiating process. B. Carbon budgets are robust as they can be
T. Second, the process (A) comparable that estimated accurately from
global well-being will also follow urbanisation of climate models.
the developing country’s (B) population, requiring C. Fraud in public law is the deliberate attempt to
fossil fuels for (C) infrastructure and energy to avoid provisions of the statute.
achieve (D) ignores levels. D. India’s thrust on LiFE (or “Lifestyle for
16. Which of the given sentences can come Environment”), with the individual shifting from
between S and T in the given arrangement? wasteful consumption of natural resources goes
A. First, Power play framed natural resource use back to the original science.
around risk management rather than technology E. Asia’s emissions with half the world’s
transfer and the well-being of all within ecological population will rise to 40% remaining within its
limits. carbon budget.
B. Carbon budgets are robust as they can be
estimated accurately from climate models. 18. Which of the given options would come after
C. First, the process adopted the structure of T in the sequence above?
international law in a manner that rejected A. Third, the agenda was set around globalised
historical responsibility for a continuing problem, material flows described as global warming (the
and steadily shifted the burden to China and symptom), and not wasteful use of energy.
India. B. Third, the need for vast quantities of cement
D. With one-sixth of the global population, the and steel in developing countries for
developed country share in 2035 will still be 30%. infrastructure — constituting essential emissions,
E. First, citizens in developed countries are not as they urbanise — is not being considered.
even aware that two-thirds of their national C. Power play framed natural resource use
emissions of carbon dioxide come from their diet, around risk management rather than technology
transport, and residential and commercial transfer and the well-being of all within ecological
sectors, which together constitute the major limits.
share of their GDP; the consumption sectors are

Click Here For Bundle PDF Course | support@guidely.in Page 5 of 11


SBI Clerk & RRB PO Mains PDF Course 2023
ENGLISH Day - 16

D. Third, public finance is used as a means to C. A-D


secure a political objective, and not to solve the D. B-C
problem itself. E. All correct
E. As late urbanisers, developing countries
account for more than half the annual emissions 20. Which of the given options provides the
and most emissions growth. correct synonym of the word highlighted in R?
A. Crooked
19. Which of the highlighted words in T must be B. Genuine
swapped to make the sentence meaningful? C. Candid
A. B-D D. Forthright
B. A-C E. Blunt
Click Here to Get the Detailed Video Solution for the above given Questions
Or Scan the QR Code to Get the Detailed Video Solutions

Answer Key with Explanation

1. Answer: C when the words in C and F are interchanged as


The word serious should come before well.
consequences as it acts as a perfect adjective This makes option (c) the most logical answer
that defines the nature of the consequences. choice.
This makes A-E one good pair to be swapped.
The options carrying the same are (a), (c) and 2. Answer: D
(d). The words that should be swapped between
The wordrequirements should come in C as 85% themselves are given in option (d) as, the word
of the required oil is imports is being mentioned said should come after the external affairs
in the line. This meaning can only be understood minister which makes A-E one correct pair to be
interchanged.

Click Here For Bundle PDF Course | support@guidely.in Page 6 of 11


SBI Clerk & RRB PO Mains PDF Course 2023
ENGLISH Day - 16

The wordglobally should come after recognised.


The phrase rang out means to be heard loudly 6. Answer: B
and clearly… This means that the word in C The paragraph talks about the state of education
should come in D. as per the mentioned report which means that
This makes option (d) the most logical answer the conclusion should also be related to the
choice. same. The first option talks about a new idea ie;
teaching as a profession not being attractive.
3. Answer: E This doesn’t carry the main idea forward or even
The given words are in their correct positions conclude the same. So, (a) can be ruled out.
and need no interchange. Even (c) can be eliminated for being out of
This makes option (e) the most logical answer context.
choice. The last sentence starts with no wonder which
means that the paragraph acts as a
4. Answer: C justificationto thepoor performance of India in the
The word environment should come in C given report. So, (a) can be taken as the
because the environment for business is being appropriate answer.
talked about. (d) is grammatically inappropriate and (e) is
This tells us that the words in B and F need to be contextually incorrect.
interchanged. This makes option (b) the most relevant answer
Therefore, the correct answer is option (c). choice.

5. Answer: A 7. Answer: C
The hints: The above paragraph throws light on how
 Stubble – burning(is a method of important the green and open spaces are when
removing paddy crop residues from the it comes to climate mitigation and dealing with
field to sow more). water logging or related issues.
 Hooked to - very interested and The first option if filled in the blank will make it
enthusiastic about something look incomplete as some connector is needed in
 Cultivation – cycle (one crop being sown the beginning of the option. The second option is
after the other) disconnected and can be ruled out. The fourth
These hints tell us that the pairs given in the first option brings in data which does not continue the
option are correct and (a) should be marked the given sentence.
answer.

Click Here For Bundle PDF Course | support@guidely.in Page 7 of 11


SBI Clerk & RRB PO Mains PDF Course 2023
ENGLISH Day - 16

The third option is perfect because the last Three, the removal of the upper limit set by the
sentence talks about how open spaces are RBI on small loans pricing
important and the option continues the idea. We need to continue the third point with the help
Thus, the best would be to mark option (c) as the of one of the options. The first option makes
answer. complete sense as the removal of the limit
(removing the caps on pricing) will now make it a
8. Answer: D level playing field for everyone.
The first option talks about worsening the issues The second option talks about competition which
like pollution etc which doesn’t make sense as makes it an extra addition. The paragraph does
the paragraph talks about the realisation that we not tell us anything about the credit market being
should have as humans about the environment. tough or so which makes the third option
It says we must understand that before irrelevant. Interest rates also have nowhere been
anything/anyone else, we are doing it for mentioned in the paragraph making the fourth
ourselves. option irrelevant.
The second option talks about the size of the The correct answer thus, should be (a).
problem but no problem as such has been
mentioned in the paragraph so this will be 10. Answer: C
mismatch. The paragraph talks about the growth of the
The third option talks about thinking out of the online gaming industry which is the reason why
box but in what sense? We do not get an answer there is a need for regulation in the area.
to this. So, (c) can be ruled out as well. The first option is too extreme as banning the
The fourth option states that we must be games has not been referred to and that doesn’t
sensitive towards the environment and monitor seem to be a solution as well. The second option
our actions as these could impact our well-being. could have been taken but a conference is a
The best would hence be to mark option (d) as new subject and we have a better option than
the answer. this one to continue the line which mentions
what the ministry of IT has done… It hasnotified
9. Answer: A the rules (as there was a need for regulation). (d)
The paragraph mentions the following about the happens to be irrelevant and can be eliminated.
microfinance companies: The best would be to mark option (c) as the
One, that their share has increased in the FY 23. answer.
Two, what has been the reason behind this
increase. 11. Answer: C

Click Here For Bundle PDF Course | support@guidely.in Page 8 of 11


SBI Clerk & RRB PO Mains PDF Course 2023
ENGLISH Day - 16

Prompted means to cause something to happen; the policeman is in trouble and need of help. So,
to make somebody decide to do something. this is correct.
The first sentence needs a word that would The second sentence cannot take this word as it
mean indicated in a sense for which we can use contextually needs a certain word that would
the word given above. The second sentence mean importance or so…
also correctly takes the given word for the same The third sentence explains that the threatened
reasons. violence looks unavoidable now.
The second meaning explained above makes Thus, the only sentence that takes the given
the given word fit for the third sentence as well. word correctly is (b).
This makes option (c) the most logical answer
choice. 14. Answer: E
Align means brought into line or proper
12. Answer: E arrangement…
Resilience means the ability to bounce back; the The first sentence talks about the fitting of the
process and the outcome of successfully buttons (that are placed properly) so;
adapting to difficult or challenging life alignedperfectly. So, (i) is correct.
experiences… The second sentence talks about putting the
The first sentence talks about the perfection of ruler in the centre of the paper which also means
its calculationsor the quality of the calculations arranging or bringing something in a proper line.
so, the word resilience does not fit in. The third sentence discusses the arrangement of
The second sentence talks about the data that the desks pushed by the students which means
shows that the consumer the given word is correct here as well.
spending has rebounded. The above word would The best answer would hence, be option (e).
fit correctly in this sentence. For the third
sentence, we need a word that would define 15. Answer: A
History. History can be turbulent but not resilient. Bonhomie stands for cheerful friendliness;
The correct answer hence, is option (e). geniality.The word fits well in the third sentence
as it talks about the friendly behaviour between
13. Answer: B the actors during rehearsals.
The word distress means the state of being in The first sentence must take a verb like ‘become
great danger and needing immediate help. The less strong’ or ‘abate’ but the highlighted word is
first sentence portrays one such situation where a noun. The second sentence talks about power
generation which again cannot take the given

Click Here For Bundle PDF Course | support@guidely.in Page 9 of 11


SBI Clerk & RRB PO Mains PDF Course 2023
ENGLISH Day - 16

word contextually. It can take words like reliance developing countries. So, before this line an
or dependence. introduction should talk about conveniently
The only sentence that should be marked as the moulding or changing laws or the course of the
answer thus, is in (a). discussion.
The first option says the law is questionable
16. Answer: E which makes it extreme because it is not so. It is
S states that there are three problems. The just convenient. Also, calling the foundation itself
sentence that should come in between S and T questionable should not be correct. (b) talks
should talk about the first problem ideally. We about budget but coming to P after that would
have three options of the sort to choose from. not be possible. We can say the same thing for
We can eliminate (c) as it brings in a new options (d) and (e) as well.
dimension of India and China that might be true The only option that provides the appropriate
but doesn’t fit between the given paragraph for opening to the idea in P, is (c).
sure.
The first option talks about the power play being 18. Answer: B
framed around a certain thing rather than the So, like T talks about the second problem that is
other. This aspect does not fit between S and T being faced in negotiations, the sentence that
as, the problems being mentioned should comes after T should discuss either the same
surround the negotiation process. issue in T or provide the third reason.
The first reason an ‘equitable sustainable We check the options and find that no option
discussion’ is getting difficult is that the people discusses the same problem in T so, the former
do not even realise that it is all happening is not true. The latter which says the third issue
because of them as well (which can be the should be discussed holds true. We can clearly
reason why they feel others should do more than eliminate options (c), (d) and (e) getting clarity
us not equitable here). from the discussion above.
So, if we put sentence in (e) in between S and T, Out of (a) and (b), (b) is more appropriate as it
we would be able to completely understand the further elaborates the infrastructure and energy
flow of the paragraph. point being made in T. (b) better continues T
than (a).
17. Answer: C So, the most appropriate answer should be
If we read P we find that it provides an example option (b).
of how the entire process of climate negotiations
conveniently ignores the interests of the 19. Answer: C

Click Here For Bundle PDF Course | support@guidely.in Page 10 of 11


SBI Clerk & RRB PO Mains PDF Course 2023
ENGLISH Day - 16

‘Ignores the well-being’ and ‘comparable levels’ The highlighted word in R is fraudulently. It
should be the correct usages. means in a way that involves deception,
This makes the words in A and D interchange especially criminal deception; something that is
their places. not genuinely or honestly done.
So, option (c) is the correct answer. Crooked means not straight or even; fraudulent.
The other options are opposite of the given word
20. Answer: A so, can be ruled out.
This makes option (a) the best answer.

Click Here For Bundle PDF Course | support@guidely.in Page 11 of 11


SBI Clerk & RRB PO Mains PDF Course 2023
Reasoning Ability Day - 17 (Eng)

Reasoning Ability
Directions (1-5): Study the following information 2. How many persons are sitting to the left of the
carefully and answer the below questions. one who likes 13?
Eight persons – A, B, C, D, E, F, G, and H are a) As many persons sit to the right of B
sitting in a row in such a way that some of them b) Four
are facing north whereas some of them are c) As many persons sit between D and H
facing south. Each person likes different d) As many persons sit to the right of G
numbers viz.- 8, 13, 14, 15, 16, 23, 25, and 28. e) None of these
All the information is not necessarily in the same
order. 3. What is the difference between the numbers
Note: Not more than two persons facing the liked by A and F?
same direction are sitting together. The a) 5
consecutive alphabetically named persons are b) 2
not sitting together. c) 8
E sits third from one of the extreme ends and sits d) 11
fourth to the left of the one who likes 25. The one e) None of these
who likes 25 sits second to the left of A and faces
north. Only two persons are sitting between A 4. Who among the following persons are sitting
and the one who likes 28. F sits third to the left of between the one who likes 13 and F?
H, both are facing in the same direction. H, who I. The one who likes 28
likes 16, sits immediate left of E. F likes a II. G
number which is seven more than the number III. The one who likes 16
liked by D. E likes a number which is nine more a) Both I and III
than the number liked by C. G sits immediate b) Only III
right of C. B and the one who likes 14 are facing c) Both I and II
in the same direction. At least one person sits d) Only I
between the one who likes 8 and A. e) None of these
1. What is the position of E with respect to the
one who likes 15? 5. Which of the following statement is not true?
a) Fifth to the right a) Only three persons sit to the left of the one
b) Immediate left who likes 13
c) Second to the right b) B likes 28
d) Third to the left c) More than two persons sit to the right of C
e) None of these d) More than two persons sit to the left of G

Click Here For Bundle PDF Course | support@guidely.in Page 1 of 11


SBI Clerk & RRB PO Mains PDF Course 2023
Reasoning Ability Day - 17 (Eng)

e) All the above statements are true All these persons in arrangement-2 are having
different number of files viz., 13, 26, 29, 18, 21,
Directions (6-10): Study the following information 47, 40, 31, 42 and 35.
carefully and answer the given questions. The one who sits third to the right of Ryan has 21
Arrangement 1: files. The one who sits immediate right of Aadhira
Ten persons – Aadhira, Cade, Felix, Joel, Lucy, has twice the files with Cade. Joel has 16 files
Macy, Pablo, Ryan, Vanya and Yael are sitting less than Pablo. Pablo has two files more than
around a pentagon table for an office meeting in that of Yael. The sum of the number of files with
such a way that five of them are sitting at the Joel and Cade is equal to the files with Macy.
corners of the table while others are sitting in the Ryan has three files more than that of Joel. The
middle of the sides of the table. difference between the number of files with Macy
The persons who sit at the corner are facing the and Ryan is equal to the files with Lucy. Felix has
opposite direction of the persons who sit in the half of the number of files with Joel. The
middle of the sides. Ryan sits third to the right of difference between the number of files with Joel
Joel, who does not sit in the middle of the table. and Vanya is 9.
Two persons sit between Ryan and Macy. Felix 6. Which among the following pair of persons,
neither sits at the corner of the table nor sits the first person sits third to the right of the
adjacent to Macy and Ryan. Lucy sits second to second person with respect to arrangement 1?
the right of Felix. Cade sits immediate right of a) Aadhira and Pablo
Lucy. Vanya sits second to the left of Aadhira, b) Lucy and Macy
who sits adjacent to Macy. Pablo neither faces c) Vanya and Yael
inside the table nor sits adjacent to Aadhira. d) Felix and Cade
Arrangement 2: e) All of the above
All the persons in arrangement 1 are sitting
around a circular table facing the centre. Initially, 7. How many persons are sitting between Ryan
the persons in arrangement 1 are sitting around and Macy in arrangement 2?
the circular table in a clockwise direction from the a) One
position of Yael (right to left). Then the persons, b) Three
who sit on the sides of the pentagonal table, are c) Two
moved two places to their right in the circular d) Four
table. The persons who sit at the corner of the e) Five
pentagon table remain in the same position in the
circular table.
Arrangement 3:

Click Here For Bundle PDF Course | support@guidely.in Page 2 of 11


SBI Clerk & RRB PO Mains PDF Course 2023
Reasoning Ability Day - 17 (Eng)

8. The difference between the number of files of Five persons viz - Q, R, S, T and U are standing
which among the following pair of persons will be in a linear row in alphabetical order from right to
the same? left such that three of them are facing south while
I. Cade and Joel the remaining are facing north. The distance
II. Pablo and Macy between the adjacent persons is 12m.
III. Vanya and Yael Note 1: All the persons are initially moving in
IV. Lucy and Joel opposite direction with respect to the direction in
a) Only II, III and IV which they are facing.
b) Only I, II and III Note 2: Not more than two adjacent persons are
c) Only I, III and IV facing the same direction.
d) Only II and III Q faces south and moves for 13m towards a
e) All I, II, III and IV certain direction, then turns to the left and moves
some distance, which is north of the initial
9. What is the position of Ryan with respect to position of S. S faces the same direction as Q
Vanya in arrangement 1? and moves for 9m towards a certain direction,
a) 6th to the right and 4th to the left then turns to the left and moving for 6m, which is
b) 5th to the left and 6th to the right north-east of the final destination of T. U is
c) 4th to the right and 6th to the left moving for 8m towards a certain direction to
d) 3rd to the left and 4th to the right reach his final destination. R is moving for 17m
e) 5th to the right and 7th to the left towards a certain distance and turns to the right
and moving some distance to reach his final
10. Four of the following five are alike in a certain destination, which is south of the final destination
way based on the arrangement-3 and thus form of U. The persons who stand at the end of the
a group. Which one of the following does not row are facing opposite directions. T moves 1/3rd
belong to the group? of the initial distance of S to reach his final
a) Macy-13 destination, which is south-west of S’s final
b) Joel-42 destination.
c) Pablo-40 11. If Q is moving a certain distance for 6m from
d) Vanya-18 his final destination to reach his new final
e) Felix-29 destination, which is north of the initial position of
T, then what is the shortest distance between the
Directions (11-15): Study the following new final destination of Q and the final
information carefully and answer the questions destination of S?
given below. a) 3√5m

Click Here For Bundle PDF Course | support@guidely.in Page 3 of 11


SBI Clerk & RRB PO Mains PDF Course 2023
Reasoning Ability Day - 17 (Eng)

b) 31m a) North-East
c) 2√13m b) South-West
d) 5√8m c) North-West
e) 3√13m d) South
e) South-East
12. If T is moving a certain distance from his final
destination, then turns right and moves a certain 15. Four of the following five are alike in a certain
distance to reach his new final destination, which way based on their directions of the given
is the final destination of S, then what is the total arrangement and thus form a group. Which one
distance between the new final destination of T of the following does not belong to the group?
and the final destination of U? a) The final destination of Q and the initial
a) 7m position of S
b) 18m b) The final destination of T and the final
c) 27m destination of R
d) 35m c) The final destination of S and the final
e) 61m destination of U
d) The initial position of T and the final
13. What is the direction and shortest distance of destination of U
the final destination of R with respect to the final e) The initial position of Q and the final
destination of Q? destination of R
a) 8√42m North-east
b) 12√23m South-east Directions (16-18): Read the given passage
c) 12√23sm South-west carefully and answer the questions based on the
d) 4√193m North-east same respectively
e) 4√61m South-west Newly launched Blessful Sleep Gummies are
perfect bliss to induce restful sleep. With a hectic
14. If P is exactly between R and S and facing schedule and long tiring day, sound sleep is all
the opposite direction of T, then moving 8m we need to wake up refreshed. Take two beauty
towards a certain distance, from there he turns to sleep gummies 30 minutes before bedtime and
the right and moves some distance to reach his dive into your dreamland. Additionally, it also
final destination, which is south of the final helps in rejuvenating your skin health overnight.
destination of T, then find the direction of the You can easily manage your sleep-wake cycle
final destination of P with respect to the final with wholesome ingredients; Vitamin B6 to fall
destination of S? asleep faster, Passiflora Extract that induces

Click Here For Bundle PDF Course | support@guidely.in Page 4 of 11


SBI Clerk & RRB PO Mains PDF Course 2023
Reasoning Ability Day - 17 (Eng)

restful sleep by relaxing your body and mind, and 18. Choose the correct conclusion from the
Melatonin to improve your sleep routine. These options given below.
non-habit-forming gummies calm your body and a) Sleeping disorders and skin health go hand-in-
mind by coping with daily stress. Passion Fruit hand
Flavour gummies will reverse the skin damage b) Deficiency or unavailability of vital ingredients
caused by stress and rejuvenate the skin from might disrupt sleep cycle
within, while you sleep tight. These Gummies will c) Skin health improves well during sleep so
help you in getting your dream sleep and good sleep is important
managing your sleep disorders. It is a 100 per d) Sleep wake cycle of an individual improves
cent clinically tested & doctor recommended when proper intake of ingredients is taken care of
product for managing sleep disorders. e) All of the above
16. Which of the given options can be ‘inferred’
from the paragraph given above? 19. STATEMENT: Higher disposal costs
a) Stress is the main reason people have a encourage those who produce waste to look for
disturbed sleep cycle cheaper way to get rid of it. The term cost of
b) Sleep disorders are a common place in disposal is used to describe the incremental
today’s times. expense directly attributed to the disposal of an
c) All people want after a tiring day is a blissful asset, contract, or cash-generating entity. Cost of
stress-free sleep disposal is oftentimes a future liability that flows
d) People generally accept medicines that don’t as an expense to the income statement as it is
cause dependency incurred.
e) All options except (c) Course of Action:
1. The disposal cost should be made higher.
17. Which of the following ‘assumptions’ is 2. The disposal cost should be brought down.
correct as per the contents of the paragraph? 3. A committee should be set up to study the
a) The people with sleep disorders continuously details in this respect.
keep looking for remedies a) Only 1 follows
b) Natural ingredients do not have side effects b) Only 2 follows
c) People with sleep disorders are concerned c)Only 2 & 3 follow
about their skin health d) None follows
d) The gummies will be accepted by people as e) All follow
they are innocuous
e) All the given statements are correct 20. Statement: Flavonoids are a common
component of almost all plants, but a specific

Click Here For Bundle PDF Course | support@guidely.in Page 5 of 11


SBI Clerk & RRB PO Mains PDF Course 2023
Reasoning Ability Day - 17 (Eng)

variety of flavonoid in apples has been found to b) Flavonoids are essential to preventing heart
be an antioxidant. Antioxidants are known to be a disease.
factor in the prevention of heart disease. c) Eating at least one apple each day will prevent
Which one of the following can be properly heart disease.
inferred from the passage? d) At least one type of flavonoid helps to prevent
a) A diet composed largely of fruits and heart disease.
vegetables will help to prevent heart disease. e) A diet deficient in antioxidants is a common
cause of heart disease.
Click Here to Get the Detailed Video Solution for the above given Questions
Or Scan the QR Code to Get the Detailed Video Solutions

Answer Key with Explanation

Directions (1-5):  The one who likes 25 sits second to the


1. Answer: C left of A and faces north.
2. Answer: C  Only two persons are sitting between A
3. Answer: B and the one who likes 28.
4. Answer: A That means, in case (1) the one who likes
5. Answer: C 28 sits third to the left of A, in case (2) the
one who likes 28 sits third to the right of
A.
Based on the above given information we have:

We have:
 E sits third from one of the extreme ends
and sits fourth to the left of the one who
likes 25.

Click Here For Bundle PDF Course | support@guidely.in Page 6 of 11


SBI Clerk & RRB PO Mains PDF Course 2023
Reasoning Ability Day - 17 (Eng)

Since, the possible number liked by D is


8.
Thus, in case (2) C sits at the end, in case
(2a) C sits immediate left of A, case (1) is
not valid.
Based on the above given information we have:

Again, we have:
 F sits third to the left of H, both are facing
in the same direction.
 H, who likes 16, sits immediate left of E.
 F likes a number which is seven more Case (1) is not valid D and E can’t sit together.
than the number liked by D. Again, we have:
Since, only such possible (8, 15), and (16,  B and the one who likes 14 are facing in
23), thus F must like 15. the same direction.
That means, F sits at the left end and  At least one person sits between the one
likes 15. who likes 8 and A.
Based on the above given information we have: That means, case (2) is not valid.
Based on the above given information we have:

Again, we have: Case (2) is not valid as at least one person sits
 E likes a number which is nine more than between the one who likes 8 and A.
the number liked by C.
Since, only such possible combination is Directions (6-10):
(14, 23). 6) Answer: E
Thus, E likes 23. 7) Answer: D
 G sits immediate right of C. 8) Answer: B
9) Answer: C

Click Here For Bundle PDF Course | support@guidely.in Page 7 of 11


SBI Clerk & RRB PO Mains PDF Course 2023
Reasoning Ability Day - 17 (Eng)

10) Answer: A (All the persons are sitting second  The person who sits at the corner is
to the right of the given number of files except facing opposite direction of the person
option a) who sits in the middle of the table.
Final arrangement  Ryan sits third to the right of Joel, who
Arrangement 1: does not sit in the middle of the table.
 Two persons sit between Ryan and Macy.
 Felix neither sits at the corner of the table
nor sits adjacent to Macy and Ryan.
 Lucy sits second to the right of Felix.

Arrangement 2:

Arrangement 3:
Again we have,
 Cade sits immediate right of Lucy.
From the above condition case-1a and case-2a
get eliminated, because Macy sits immediate
right of Lucy.
 Vanya sits second to the left of Aadhira,
who sits adjacent to Macy.
 Pablo neither faces inside the table nor
sits adjacent to Aadhira.
Arrangement 1:
From the above conditions, case 2 gets
We have,
eliminated because Pablo faces inside the table.

Click Here For Bundle PDF Course | support@guidely.in Page 8 of 11


SBI Clerk & RRB PO Mains PDF Course 2023
Reasoning Ability Day - 17 (Eng)

All these persons in arrangement-2 are having


different number of files viz., 13, 26, 29, 18, 21,
47, 40, 31, 42 and 35.
 The one who sits third to the right of Ryan
has 21 files. The one who sits immediate
right of Aadhira has twice the files with
Cade.
 Joel has 16 files less than Pablo. Pablo
has two files more than that of Yael.
 The sum of the number of files with Joel
and Cade is equal to the files with Macy.
Ryan has three files more than that of
Joel.
 The difference between the number of
Arrangement 2: files with Macy and Ryan is equal to the
We have, files with Lucy.
All the persons in arrangement 1 are sitting  Felix has half of the number of files with
around a circular table facing the centre. Initially, Joel. The difference between the number
the persons in arrangement 1 are sitting around of files with Joel and Vanya is 9.
the circular table in a clockwise direction from
the position of Yael (right to left). Then the
persons, who sit on the sides of the pentagonal
table, are moved two places to their right in the
circular table. The persons who sit at the corner
of the pentagon table remain in their same
position.

Direction (11-15):

Arrangement 3: 11) Answer: C

We have, 12) Answer: D


13) Answer: E

Click Here For Bundle PDF Course | support@guidely.in Page 9 of 11


SBI Clerk & RRB PO Mains PDF Course 2023
Reasoning Ability Day - 17 (Eng)

14) Answer: B the sellers is a proof enough that it is people’s


15) Answer: A(first point is northeast of second tendency to trust a non-addictive drug.
point except option a) Therefore, the correct answer is option (e).
Final arrangement 17. Answer: C
The entire paragraph focusses on addressing
the issue of sleep disorders and at multiple
places, the skin has also been mentioned. It
gives us a hint that the author assumes that
people are affected by their sleep cycle and they
are concerned as it directly affects their skin. If
we assume that the people look for good skin
health, we will be able to understand the
analogy.
Options (a) and (b) are superfluous as; (a) looks
more like a fact and not an assumption and (b) is
not necessarily true. It cannot be assumed as
16. Answer: E nothing about the quality etc of the ingredients
The statement given in the third option cannot be has been discussed. (d) is illogical as no
considered as an inference as it has directly connection about the same can be made
been given in the paragraph. An inference is through the paragraph about the same.
something that can be deduced based on the Thus, the best would be to go with option (c) as
available facts and is not directly given. the answer.
(a) can be inferred from the following quoted
lines; ‘These non-habit-forming gummies calm 18. Answer: E
your body and mind by coping with daily stress.’ A conclusion is something that can be a
(b) can also be inferred because the entire judgement or a decision reached by reasoning.
paragraph discusses a kind of medicine that has All the above given options can be logically
been trusted to work well on the sleep related deduced from the contents of the paragraph and
disorders. hence, the correct answer should be all of the
(d) can be inferred from the following quoted above ie; option (e).
lines; ‘These non-habit-forming gummies calm
your body and mind by coping with daily stress.’ 19. Answer: C
The fact that this point has been highlighted by

Click Here For Bundle PDF Course | support@guidely.in Page 10 of 11


SBI Clerk & RRB PO Mains PDF Course 2023
Reasoning Ability Day - 17 (Eng)

Course of action 2nd and 3rd are feasible and This answer is the closest to our pre-phrase, and
effective to combat the problem. this is the correct answer. Notice how the
language of this answer choice—“helps to
20. Answer: D prevent”—matches the stimulus language—
“factor in the prevention.”

Click Here For Bundle PDF Course | support@guidely.in Page 11 of 11


SBI Clerk & RRB PO Mains PDF Course 2023
Quantitative Aptitude Day - 17 (Eng)

Quantitative Aptitude

Directions (1- 4): Study the following information carefully and answer the questions given below.
The given pie chart shows the percentage distribution of the number of five different items [ice cream,
pizza, burger, sandwich, and cake] sold in shop A.

Note –I.The number of eachitems sold in shop B is 75% of the number of respectiveitems sold in shop A.
II. Difference between 50% number of burgers and 25% of the number of sandwiches sold in shop A is
44.
1) Ratio of the number of vanilla and chocolate 2) Out of the total burgers sold in shop A, 55%
ice creams sold in shop A is 3:1, and the ratio of are veg burgers and the rest are non-veg
the number of vanilla and chocolate ice creams burgers. If the number of veg burgers sold in
sold in shop B is 6:5. If average number of shops A and B is the same, then find the
vanilla ice creams sold in shops A, B and C is percentage of non-veg burgers sold in shop B
84, and the average number of chocolate ice out of the total number of burgers and pizzas
creams sold in shop A, B and C is 62, Find the sold in shop B?
total number of ice creams sold in shop C? a) 11.11%
a) 145 b) 19.33%
b) 167 c) 10.25%
c) 130 d)16.66%
d) 152 e) 15.33%
e) 197

Click Here For Bundle PDF Course | support@guidely.in Page 1 of 10


SBI Clerk & RRB PO Mains PDF Course 2023
Quantitative Aptitude Day – 17 (Eng)

3) In shop A, 75% of the veg sandwiches are the number of cakes sold in shop B is m more
sold and the rest are non-veg sandwiches. In than the original number of cake sold in shop B,
shop B, 50% of the veg sandwiches are sold and then the ratio of the number of cake sold in shop
the rest are non-veg sandwiches. The Price of A (original) and B (new) is 28:25. Find the value
the veg sandwich in shops A and B is Rs.60 and of n+m=?
Rs.75 respectively, and the price of the non-veg a) 48
sandwich in shops A and B is Rs.80 and Rs.95 b) 42
respectively. Find the difference between the c) 65
total revenue earned by shops A and B by selling d) 55
sandwiches? e) None of these
a) Rs.124
b) Rs.180 5) Series I and Series II follow the same logic.
c) Rs.158 Find the value of d?
d) Rs.167 Series I – 145, 158, 192, 249, a
e) None of these Series II – a, b, c, d, e
a) 445
4) If the number of cakes sold in shop A is n b) 454
more than the original number of cakes sold in c) 494
shop A, then the ratio of the number of cakes d) 432
sold in shop A (new) and B (original) is 10:7. If e) None of these

Directions (6 -10): Study the following information carefully and answer the questions given below.
The table shows the number of candidates selected in BANK, SSC and Railway in five different, [A, B, C,
D, and E] institutes.

Note- Total number of candidates selected from A and B is 432 and 465 respectively and x:z=3:2.
6) The ratio of the number of candidates selected candidates selected as bank PO from B is 20%
as a bank PO and clerk from A is 5:3. Number of more than the number of candidates selected as

Click Here For Bundle PDF Course | support@guidely.in Page 2 of 10


SBI Clerk & RRB PO Mains PDF Course 2023
Quantitative Aptitude Day – 17 (Eng)

bank PO from A. Find the percentage of number III. Difference between the number of candidates
of candidates selected asbank clerk from A to selected in the bank from D and E is 4.
that from B? a) only I is true
a) 74% b) only III is true
b) 61% c) only I and II is true
c) 75% d) only I and III are false
d) 66% e) None of these
e) 84%
9) Find the value of 2 * (X + Y + Z)
7) 60% and 65% of the candidates selected in a) 70
SSC from C and D are selected in the b) 78
CGL exam, and the rest are selected in the c) 88
CHSL exam. If the average number of d) 90
candidates selected in the SSC CGL exam from e) None of these
C, D and E is 95 then find the number of
candidates selected in the CHSL exam from E, C 10) The ratio of boys and girls selected in railway
and D together? from A is 5:2, and the ratio of boys and girls
a) 241 selected in railway from E is 7:4, If ratio of the
b) 252 number of boys and girls selected in railway from
c) 255 A, E and B together is 3:1, Find the difference
d) 285 between boys and girls selected in railway from
e) 197 B?
a) 152
8) Find which of the following is true? b) 142
I. Difference between the number of candidates c) 165
selected in the railway from A and B is 24. d) 187
II.The sum of candidates selected in all three e) None of these
exams together in E is 624.

Directions (11 -13): Study the following information carefully and answer the questions given below.
The line graph shows the average runs scored by five batsmen after thirty matches.

Click Here For Bundle PDF Course | support@guidely.in Page 3 of 10


SBI Clerk & RRB PO Mains PDF Course 2023
Quantitative Aptitude Day – 17 (Eng)

11) In the next five matches, A scored 35, 42, 5, e) 8


85, and 22 runs, and B scored 112, 88, 2, 8, and
30 runs. If the difference between the average 13) Find the difference between 55% of total
runs of A and B after the 35th match is r, then find runs scored by A and E inthirty matches together
the value of 2r+37.5(upto two decimal)? and 47% of total runs scored by B and D
a) 61.33 together in thirty matches?
b) 68.22 a) 254.3
c) 60.99 b) 234.6
d) 49.32 c) 215.3
e) 49.24 d) 211.6
e) None of these
12) After the 31st match, the average of C is 36,
if in the 31st match C takes, 24 singles, 10 14) Find the sum of the missing number of two
doubles and the rest of the runs are score in the series?
boundary(4’s and 6’s), if he hit two 6's then find Series I – 85,87, 177, 535,?
the number of 4's he hit? Series II – 56,65.8,85.2,114,?
a) 12 a) 2297
b) 14 b) 2456
c) 16 c) 2415
d) 10 d) 2150

Click Here For Bundle PDF Course | support@guidely.in Page 4 of 10


SBI Clerk & RRB PO Mains PDF Course 2023
Quantitative Aptitude Day – 17 (Eng)

e) None of these and dairy milk but not kitkat. The number of
people who like both KitKat and dairy milk but not
15) A and B is the wrong number of the series. milky bar is 38.
Find the relation between A and B? 16) If x is the difference between the total
Series I – 74,198,415,758,1270 number of people like KitKat and dairy milkand y
Series II – 85,88.6,93.2,98.8,104.5 is the difference between the total number of
a) 2A+3B=745 people like dairy milk and milky bar(dairy milk-
b) 2A-B=56 milky bar), Find the value of x*y=?
c)A-B=93.5 a) 2450
d) 3A+7B=745 b) 2410
e) None of these c) 2000
d) 2260
Directions (16 -20): Study the following e) 2750
information carefully and answer the questions
given below. 17) The ratio of the number of males and
A survey conducted on the number of people like females like only dairy milk is 5:4, and the ratio of
KitKat, dairy milk and milky bar chocolate. The the number of males and females like only KitKat
number of people who like only KitKat is four is 3:1. Find the difference between the total
times the number of people who like both dairy number of males who like dairy milk and KitKat
milk and KitKat but not a milky bar. The number together and the total number of females who
of people who like both KitKat and dairy milk but like dairy milk and KitKat together?
not milky bar is 5.55% more than the number of a) 84
people who like dairy milk and milky bar but not b) 82
KitKat. The number of people who like dairy milk c) 83
and milky bar but not kitkat is 5.88% more than d) 88
the number of people who like all three e) 81
chocolates. The number of people who like only
milky bars is double the number of people who 18)
like both KitKat and milky bars but not dairy milk. Quantity I: The number of people who like both
The number of people who like KitKat and milky KitKat and dairy milk but not the milky bar is what
bar but not diary milk is 5.88% less than the percent of the number of people who like both
number of people who like all three. The number KitKat and milky bar but not dairy milk?
of people who like only dairy milk is three times
the number of people who like both milky bars

Click Here For Bundle PDF Course | support@guidely.in Page 5 of 10


SBI Clerk & RRB PO Mains PDF Course 2023
Quantitative Aptitude Day – 17 (Eng)

Quantity II: Total number of people who like dairy c) 67


milk What percent of the total number of people d) 61
like milky bar? e) None of these
a) Quantity: I Quantity: II
b) Quantity: I > Quantity: II 20) The number of people who like only Perk is
c) Quantity: I < Quantity: II 25% more than the number of people who like
d) Quantity: I ≤ Quantity: II only milky bar. The number of people who like
e) Quantity I = Quantity II only 5 stars is 30% more than the number of
people who like only the perk. Find the number
19) Out of the number of people who like only of people like only the 5-star?
dairy milk, 25% like only silk, and out of the total a) 104
number of people who like only KitKat, 25% like b) 125
KitKat Hazelnut. Find the total number of people c) 124
who like the dairy milk silk and KitKat hazelnuts? d) 156
a) 65 e) None of these
b) 62
Click Here to Get the Detailed Video Solution for the above given Questions
Or Scan the QR Code to Get the Detailed Video Solutions

Answer Key with Explanation

Directions (1-4): 10x-4.5x=44


Differencebetween50% of the number of burgers 5.5x=44
and 25% of the number of sandwiches sold in x=44*10/55=8
shop A is 44. So, the number of items sold in shop A is
Let the number of items sold in shop A be 100x. 100*8=800.
So, 20x*50/100-18x*25/100=44

Click Here For Bundle PDF Course | support@guidely.in Page 6 of 10


SBI Clerk & RRB PO Mains PDF Course 2023
Quantitative Aptitude Day - 17 (Eng)

Number of Ice creams sold in shop A = 800 * 4. Answer: A


22/100 = 176 The new number of cakes sold in A is =
Number of items sold in shop B = 176 * 75/100 = 168*10/7=240
132 So, 240-224=16=n
The new number of cakes in B is =
224*25/28=200
So, 200-168=32=m
So, n+m=16+32=48

5. Answer: A
Series I – 145, 158, 192,
1. Answer: C 249, a=341
The total number of ice creams sold in shop C is +13*1 +17*2 +19*3 +23*4
= [84*3-176*3/4-132*6/11] + [62*3-176*1/4- Series II – a=341, b=341+13*1=354,
132*5/11] c=354+17*2=388, d=388+19*3=445
= 48+82=130 Directions (6 -10):
So, 10x+12x+14x=432
2. Answer: D Or, x=432/36=12
The number of veg burgers sold in A is = 10y+9y+12y=465
160*55/100=88 Or, 31y=465, y=465/31=15
The number of non-veg burgers sold in A is = So, z=12*2/3=8
160-88=72
The number of the veg burger sold in B is = 88
The number of the non-veg burger sold in B is =
120-88=32
So, required percentage = [32/192]*100=16.66%

3. Answer: B
Required difference 6. Answer: C

=[[144*75/100]*60+[144*25/100]*80]- The number of candidates selected as PO from

[{108*50/100}*75+{108*50/100}*95 A is = 120*5/8=75

= [6480+2880]-[4050+5130]=Rs.180 Number of candidates selected as PO from B is


=75*120/100=90

Click Here For Bundle PDF Course | support@guidely.in Page 7 of 10


SBI Clerk & RRB PO Mains PDF Course 2023
Quantitative Aptitude Day - 17 (Eng)

The number of candidates selected as clerks The number of girls selected in B is =142-
from A is =120-75=45 168*2/7-220*4/11=14
The number of candidates selected as clerks So, difference =166-14=152
from B is =150-90=60
Required percent = [45/60]*100=75% 11. Answer: C
The average of A after the 35th match is
7. Answer: D =[30*44+35+42+5+85+22]/35=43.11
The total number of candidates selected in the The average of B after the 35th match is
SSC CHSL exam from E,C and D is =[30*56+112+88+2+8+30]/35=54.85
={220-[95*3-(170*60/100)- So, difference = 54.85-43.11=11.74
(180*65/100)]}+[(170*40/100)]+[(180*35/100)] So, 2r+37.5=2(11.74)+37.5=23.49 + 37.5 =
=154+68+63=285 60.99

8. Answer: B 12. Answer: D


I. Difference between the number of candidates Runs score in 31th match is = 36*31-34*30=96
selected in railway from A and B is = 180- Runs score in 4’s is = 96-24*1-10*2-2*6=40
168=12. The number of 4's he hit =40/4=10
II. The sum of candidates selected in all three
exams together in E is =144+220+220=584. 13. Answer: B
III. Difference between the number of candidates Total runs score by A and E is
selected in the bank from D and E is 144-140=4. =44*30+42*30=2580
Total runs scored by B and D is =
9. Answer: A 28*30+56*30=2520
2*(12 + 15 + 8) = 70 So, 55*2580/100 – 47*2520/100 = 234.6

10. Answer: A 14. Answer: A


The total number of boys selected from A, E and
B is
= [168+180+220]*3/4=568*3/4=426
The total number of girls selected from A, E and
B is = 568*1/4=142 So, the required sum = 2145+152=2297
Number of boys selected in B = 426-168*5 /7-
220*7/11=166 15. Answer: C

Click Here For Bundle PDF Course | support@guidely.in Page 8 of 10


SBI Clerk & RRB PO Mains PDF Course 2023
Quantitative Aptitude Day - 17 (Eng)

Directions (16-20):
Let the number of people who like all three be
17x.
So, the number of people who like both dairy
16. Answer: C
milk and milky bar but not kitkat is
So, x=152+38+34+32-108-38-36-34=40
17x*105.88/100=18x
Y=108+38+36+34-64-32-34-36=50
The number of people who like both dairy milk
So, x*y=50*40=2000
and KitKat but not milky bar is =
18x*105.55/100=19x
17. Answer: D
The number of people who like both KitKat and
So, required difference = [108*5/9]+[152*3/4]-
milky bar but not diary milk is =17x*[100-
[108*4/9]-[152*1/4]=174-48-38=88
5.88]/100=16x
So, 19x=38, x=2
18. Answer: C
The number of people who like all three is
Quantity I
17x=34
Required percentage = [38/32]*100=118.75%
So, the number of people who like both dairy
Quantity II
milk and milky bar but not kitkat =18*2=36
Required
The number of people who like both dairy milk
percentage=[108+38+34+36]*100/[64+32+34+36
and KitKat but not milky bar = 38
] =130.12%
The number of people who like both KitKat and
Quantity I < Quantity II
milky bar but not diary milk is =16x=32
The number of people who like only KitKat =
19. Answer: A
38*4=152
The Total number of people who like silk and
The number of people who like only dairy milk =
Hazelnuts is
36*3=108
=152*25/100+108*25/100=65
The number of people who like only milky bar =
32*2=64
20. Answer: A

Click Here For Bundle PDF Course | support@guidely.in Page 9 of 10


SBI Clerk & RRB PO Mains PDF Course 2023
Quantitative Aptitude Day - 17 (Eng)

Number of people who like only 5star = 64*(125/100)*(130/100)=104

Click Here For Bundle PDF Course | support@guidely.in Page 10 of 10


SBI Clerk & RRB PO Mains PDF Course 2023
ENGLISH Day - 17

English Language

Directions (1-7): Read the passage given below Specifically, Mr. Das flagged the spatial and
carefully and answer the questions that follow. temporal distribution of rainfall during this
The Monetary Policy Committee (MPC)’s latest monsoon in the wake of El Niño conditions,
decision, to extend the pause in the Reserve unabated geopolitical tensions, uncertainty over
Bank of India (RBI)’s monetary tightening while international commodity prices including those of
staying focused on the withdrawal of sugar, rice and crude oil, and the volatility in
accommodation, reflects the rate setting panel’s global financial markets as upside risks to the
reassuring resolve to keep inflation front and MPC’s inflation projections. Another key factor
centre of its approach to policy. RBI Governor feeding into the RBI’s policy approach is it’s
Shakti Kanta Das was unequivocal in asserting conviction that macroeconomic fundamentals
that “the best contribution of monetary policy to have strengthened after the unrelenting focus on
the economy’s ability to realise its potential is by preserving price and financial stability. To be
ensuring price stability”. The MPC’s recent sure, the increase in credit costs since the RBI
unwavering focus on price stability is informed started raising its benchmark interest rates in
largely by its mandate to achieve the Consumer May 2022 appears to have retarded investment
Price Index (CPI) inflation target of 4%, a goal and consumption activity last year. Bank credit
that it has struggled to actualise right since data show the pace of growth in loans to
January 2021 — a period during which inflation industry, particularly the MSME and medium
remained stuck above or close to the upper sectors, slowed appreciably last year. The
tolerance band of 6% in 20 of the 27 months. Mr. sequential contraction in estimated private
Das acknowledged that even as headline consumption spending in the fourth quarter of the
inflation had eased appreciably in March and last fiscal year is also likely to have been, to
April, slowing to 4.7% in the first month of the some degree, a fallout of the higher borrowing
current fiscal year from the bruising 6.7% costs. Still, as Mr. Das emphasised,
average pace in 2022-23, retail price gains were policymakers can ill afford to take their eyes off
‘still above the target and expected to remain so inflation. Price stability is after all a public good
according to the RBI’s projections for 2023-24’. and achieving durable disinflation must remain a
The MPC, which has forecast CPI inflation to non-negotiable goal, especially amid widening
average 5.1% over the 12 months ending in income inequality and high levels of joblessness.
March 2024, is cognisant of the continuing 1) Which of the following has helped curb the
challenges in aligning inflation with the target, high borrowing costs?
given the global uncertainties.

Click Here For Bundle PDF Course | support@guidely.in Page 1 of 11


SBI Clerk & RRB PO Mains PDF Course 2023
ENGLISH Day - 17

(i) The RBI MPC’s decision to extend the pause 4) Why does Mr Das feel that inflation cannot be
(ii) Easing of the headline inflation taken lightly yet?
(iii) A reduction in the private consumption (a) Because of the growing income inequality
spending and joblessness
(a) Only (i) (b) As the macro-economic fundamentals should
(b) Both (ii) and (i) further be strengthened
(c) Only (ii) (c) It is the most important aim of the monetary
(d) Both (ii) and (iii) policy to maintain price stability
(e) Only (iii) (d) Both (a) and (c)
(e) To achieve a Consumer Price Inflation target
2) Which of the given options are being observed of about 4%
as a threat to the MPC’s inflation projections?
(a) The global uncertainties and volatility in the 5) Which of the given options can be the central
commodity prices idea of the given passage?
(b) The MPC’s failure in achieving the CPI (a) MPC’s unwavering focus on controlling the
inflation targets CPI inflation
(c) The pattern of rainfall over time and space (b) Price stability must continue to be an
during this monsoon uncompromising objective
(d) All (a), (b) and (c) (c) The constant commitment of the RBI to keep
(e) Both (a) and (c) the headline inflation high
(d) Cutting the rates of interest to increase the
3) Choose an option that carries a sentence in frequency of loans
which the word ‘cognisant’ cannot fit correctly. (e) None of these
(a) Everyone very tolerant, very ________ of
everybody's privacy. 6) Choose the option that carries the correct tone
(b) She received the award in ________ of her of the passage.
human rights work. (a) Pessimistic
(c) Then I wondered if the Prince were _________ (b) Informative
at all of what I had done to him. (c) Critical
(d) Be _________ of your surroundings, and you (d) Curious
can make them work for you. (e) None of these
(e) All are correct
7) Choose the part of the sentence that carries
an error in the same.

Click Here For Bundle PDF Course | support@guidely.in Page 2 of 11


SBI Clerk & RRB PO Mains PDF Course 2023
ENGLISH Day - 17

(a) Another key factor feeding into (a) Emergency, aiming


(b) the RBI’s policy approach is it’s conviction (b) Passion, seeming
(c) that macroeconomic fundamentals (c) Agony, planning
(d) have strengthened after the unrelenting (d) Zeal, charging
(e) focus on preserving price and financial (e) No replacement needed
stability.
11) The CM is likely to explain in detail his
Directions (8-12): The given questions carry volunteering of the third front and the freefall
words that have been highlighted as they might meetings with the leaders of political parties.
have been used incorrectly in the given (a) Apprehension, submissive
sentence. You must choose an option that (b) Curiosity, impending
carries words that can replace the words (c) Hawkish, upcoming
highlighted in the sentence given above. (d) Initiation, subsequent
8) Individual rights are being focussed upon by (e) No replacement needed
hooligans who claim to subdue community
interests. 12) At these meetings, seeds of relief of India
(a) Held, expedite were reaped, the results of which were visible in
(b) Trampled, protect today’s India and the 75-year journey of the
(c) Bolstered, harm nation.
(d) Crushing, safeguard (a) Upsurge, sewn
(e) No replacement needed (b) Reconstruction, sowed
(c) Segregation, planned
9) Initial estimates for India’s external trade (d) Support, sown
performance in May are a indication of even (e) No replacement needed
challenging times ahead.
(a) Harbinger, tougher Directions (13-17): The given sentences have
(b) Forthcoming, appease been divided into five parts each. Out of these
(c) Upcoming, better parts, one part might not fit the given sentence.
(d) Adherent, resilient Identify the same and rearrange the remaining
(e) No replacement needed parts to form a meaningful sentence. Choose the
most appropriate option as your answer.
10) This is a crisis of performing nothing but the 13) all States, is necessary and till that (A)/
best, rushing to win a seemingly endless race. happens the Bill should not be allowed to come
into force (B)/ no comparison between mobile

Click Here For Bundle PDF Course | support@guidely.in Page 3 of 11


SBI Clerk & RRB PO Mains PDF Course 2023
ENGLISH Day - 17

phones and electricity distribution (C)/ We have 16) Opposition members have rallied around the
demanded that wider consultation (D)/ among all President (A)/ the papers sought by the FBI
stakeholders, including (E). agents during the raid (B)/ close to the
(a) CAED; B investigation as saying that classified (C)/
(b) DEAB; C documents relating to nuclear weapons were
(c) BEAD; C among (D)/ the Washington Post cited
(d) CDBE; A anonymous sources (E).
(e) No rearrangement needed; E (a) ABED; C
(b) ECDB; A
14) People require nutrition, health and (c) BDAE; C
education services (A)/ policies for that... Courts (d) DECB; A
are not the fora for this (B)/ especially the State (e) No rearrangement needed
governments, to bring out (C)/ in order to
inculcate patriotism among students from an 17) Chinese state media have accused the (A)/
early age (D)/ and it is for the political class and U.S. of building an Indo-Pacific alliance (B)/ to
governments, (E). limit China’s military and (C)/ diplomatic influence
(a) ABED; C in the region (D)/ The joint combat exercises will
(b) DEBA; C end (E).
(c) AECB; D (a) AEDB; C
(d) BECD; A (b) BECD; A
(e) No rearrangement needed (c) ABCD; E
(d) BDCA; E
15) by some sections of traders and stockists to (e) CEDB; A
push the (A)/ the Centre has invoked the
Essential Commodities Act of 1955 (B)/ The Directions (18-22): The given sentences have
government hopes the move will rein in attempts been divided into parts. Two of the given parts
(C)/ price for tur dal upwards, by resorting to (D)/ need a grammatical improvement in them. You
‘restricted sales’ and creating an artificial scarcity must choose an option that carries the erroneous
(E). parts as your answer.
(a) ABED; C 18) After painstakingly sifting through (A)/ the
(b) DEAB; C facts, I narrowed from (B)/ on details that would
(c) AECB; D suit me and (C)/ make significant modifications to
(d) CADE; B (D)/ my regular food intake and physical activity
(e) No rearrangement needed (E).

Click Here For Bundle PDF Course | support@guidely.in Page 4 of 11


SBI Clerk & RRB PO Mains PDF Course 2023
ENGLISH Day - 17

(a) AE (d) BC
(b) CB (e) No Correction Needed
(c) BD
(d) AB 21) The national capital region (NCR) (A)/ thus
(e) No Error must brace of another (B)/ season of air quality
reached (C)/ very poor or even severe levels,
19) Art bridge that gap, (A)/ allowing us to (B)/ (D)/ compounded by the shift in air-flow
communicate and convey (C)/ our innermost dynamics (E).
feelings (D)/ with limitations (E). (a) BC
(a) BD (b) AE
(b) AE (c) AB
(c) AC (d) CD
(d) BE (e) No correction required
(e) No Corrections Needed
22) In an era marked by (A)/ expanding global
20) Dr. Koll and colleagues pointed out the (A)/ horizons, (B)/ affluent Indians have embarking
increasing prominence of Arabian Sea (B)/ (C)/ on overseas adventures (D)/ more then ever
cyclones with a 52% rise in such cyclones (C)/ before (E).
from 2001-2019 and an 8% decrease (D)/ in (a) AB
those over the Bay of Bengal (E). (b) CE
(a) AB (c) BE
(b) ED (d) DC
(c) EB (e) No correction required

Click Here For Bundle PDF Course | support@guidely.in Page 5 of 11


SBI Clerk & RRB PO Mains PDF Course 2023
ENGLISH Day - 17

Click Here to Get the Detailed Video Solution for the above given Questions
Or Scan the QR Code to Get the Detailed Video Solutions

Answer Key with Explanation

1) Answer: E the RBI governor as risks to the MPC’s inflation


The first statement has been given in the first projections. The first and the third reasons can
paragraph as the latest decision that has been be found being clearly discussed but the second
taken by the RBI with an aim to control inflation. statement is unrelated to the question asked.
The question does not ask the same so, it can So, the correct answer is option (e).
be left out.
The second statement is not the direct reason 3) Answer: B
that has helped limit the borrowing cost so, this The one who is cognisant has awareness and
can be eliminated as well. knowledge of the things going around… The first
The third statement is correct as it has been sentence talks about being aware of everyone’s
discussed in the last paragraph that a sequential privacy which means the given word fits the
contraction in the consumer spending has been sentence correctly.
observed leading to a reduction in the borrowing The second sentence needs the word
costs. recognition so this is incorrect.
Of the given statements only the third one Third sentence also reflects the same meaning
answers the question well. of the Prince being aware of the things being
So, the correct answer is option (e). done by someone for him so the word fits
correctly.
2) Answer: E The fourth sentence also takes the given word
The answer to the above question can be found correctly so this can be taken to be correct as
in the first few lines of the second paragraph. well.
These lines clearly mention the reasons cited by

Click Here For Bundle PDF Course | support@guidely.in Page 6 of 11


SBI Clerk & RRB PO Mains PDF Course 2023
ENGLISH Day - 17

The only option that should hence, be marked The third option is irrelevant as keeping the
the answer is option (b). headline inflation high goes against the context
of the passage. The fourth option is not the sole
4) Answer: D objective of the RBI as per the passage so this
Refer to the lines given below from the last can be ruled out as well.
paragraph; Therefore, the best would be to mark option (b)
Still, as Mr. Das emphasised, policymakers can as the answer.
ill afford to take their eyes off inflation. Price
stability is after all a public good and achieving 6) Answer: B
durable disinflation must remain a non- The passage carries a lot of data about the
negotiable goal, especially amid widening current trends of the economy. It provides
income inequality and high levels of joblessness. information on the way inflation etc is being
The lines explain that achieving the Price controlled and the steps being taken in that
Stability goal should be non-negotiable which direction.
means there shouldn’t be a second thought The first option does not fit as it means negative.
about it. The second thing being explained There is nothing negative in the tone of the
above is that it becomes even more important author. Critical can also be cancelled for the
due to the widening income inequality and same reason. It is used to define the author’s
joblessness. tone when the author is finding faults or being
The two points being discussed can be found in negative. This is also not being done so can be
the first and the third options. ruled out. Out of the remaining options, curious
Therefore, the correct answer is option (d). can be eliminated as the author is providing
information and is not curious or asking
5) Answer: B questions.
The second option defines the central idea of the This makes option (b) the most logical answer
passage as the entire content revolves around choice.
achieving just one goal that is ‘Price Stability’
The entire passage discusses various aspects 7) Answer: B
related to the MPC’s actions which have been The error lies in the second part of the given
aimed at achieving the common goal of price sentence as the word it’s has wrongly been used
stability. in the same. The given word is an acronym for it
The first option is one of the aspects that should is clubbed together and is not needed. What is
be undertaken to achieve the main objective.

Click Here For Bundle PDF Course | support@guidely.in Page 7 of 11


SBI Clerk & RRB PO Mains PDF Course 2023
ENGLISH Day - 17

needed in its place is its which is a pronoun would look for a complete option that helps us
being used to refer to the RBI. justify either way.
The correct answer hence, is option (b). Harbinger means a sign that shows that
something is going to happen soon, often
8) Answer: B something bad. This makes the sentence correct
The given words in the sentence make it with tougher being the word fit for the second
confusing as the ‘said hooligans’ (hooligans  blank. This option can be taken.
violent and unruly people) cannot be expected to The word appease means to please or make
focus on Individual rights. Held upon means to someone happy. This makes no sense in the
endure or continue to do something despite given sentence. upcoming cancels the third
difficulty… Even this makes no sense so the first option. Adherent means somebody who
option can be eliminated. Expedited means supports a particular idea. Resilient means
hurried… strong enough to bounce back from hardships.
Trampled (upon) means to harm or inflict an The best combination of words can hence, be
injury upon someone or something. This word found in option (a).
fits the context of the sentence and the second
word also goes well in the context. 10) Answer: E
Bolstered means supported or encouraged The word emergency would be grammatically
someone… crushing does not fit the sentence incorrect in the first blank which makes the first
grammatically. option irrelevant. The word seeming in the
The best would hence be to mark option (b) as second option if used will create redundancy as,
the answer. seemingly has already been used in the
sentence once. The word agony means great
9) Answer: A pain or suffering. Zeal is great energy or
The word indication in the sentence is preceded enthusiasm.
by article ‘a’ which makes it incorrect The given words are correct in the context of the
grammatically even when the word is sentence and need no change. So, the correct
contextually correct. answer is option (e).
The sentence intends to state that the May
performance hints at an even more difficult time 11) Answer: D
ahead. Or, better times ahead may be as we are A freefall is a downward movement of something
not sure about the tone of the sentence. So, we that too consistent downward movement. This

Click Here For Bundle PDF Course | support@guidely.in Page 8 of 11


SBI Clerk & RRB PO Mains PDF Course 2023
ENGLISH Day - 17

word is incorrect so replacement is hence, related part but it does not fit in the sequence of
needed. We can eliminate (e). the sentence.
In a first glance, all words except ‘hawkish: So, the correct answer is option (b).
hostile and aggressive’ doesn’t fit the first blank.
So, (c) can be ruled out. 14) Answer: C
‘Impending’ are things that are just about to The sentence says that it is the duty of the
happen. ‘Submissive meetings’ makes no sense. governments, especially the state governments
Looking at all the options, we can conclude that to make policies that ensure nutrition, health,
the words in the fourth option are all correct and and education services to the people. If we want
hence, (d) is the correct answer. this meaning to come out from the above, we
would need to rearrange the given sentences in
12) Answer: B the sequence given in the third option that starts
The sentence carries a positive tone as it talks the sentence with A.
about the visible results of change in today’s We can easily find that the part in D talks about
India. So, words like ‘segregation’ which means students and patriotism; an idea that does not fit
separation will be incorrect as per the context. in the given sentence.
‘sewn’ will be incorrect as it means to stich which So, the correct answer is option (c).
is not the context here. This cancels option (a).
‘seeds of support’ will not be as correct as 15) Answer: D
‘seeds of reconstruction’ so, this clarifies our Taking hint from the options we can see that, out
correct option ie; (b) as the answer. of A, D and C, only C can start the sentence
Sentence Rearrangement (Eliminate the Odd): because A and D reflect a broken meaning and
such parts cannot be taken as an introduction.
13) Answer: B After this step, we see that C starts and further
The sentence should start with D. No other part checking the sequence, we find that A continues
in the given sentence is as complete as D and it the idea in the sentence.
provides a good introduction. E continues the B talks about some act which doesn’t fit the
idea as D says consultation and E says among context of the given sentence. So, the best is to
the stakeholders… which makes it a good link eliminate that as an odd part.
DE. So, the correct answer is option (d).
We can find the given link in only the second 16) Answer: B
option and on checking the sequence given we
find that it forms a meaningful sentence. C is a

Click Here For Bundle PDF Course | support@guidely.in Page 9 of 11


SBI Clerk & RRB PO Mains PDF Course 2023
ENGLISH Day - 17

The first part i.e.; A given in the sentence above verb. The correct usage should be; Art bridges
does not fit the context of the sentence so, it the gap…
should be taken as the odd part. Part E needs a correction as well because the
We can see that CD is a clear link that can be word with makes the sentence ambiguous.
found in the second option. On checking the Contextually, art is bridging the gaps by allowing
second option, we get a meaningful sentence. people to communicate. This is possible only
So, we can mark the same as the answer. when it is done freely and not with limitations as
Therefore, option (b) is correct. given. With should be replaced by without.
Thus, the correct answer is option (b).
17) Answer: C
The parts of the given sentence are already in 20) Answer: E
their correct sequence and do not need to be The sentence is correct as given and needs no
rearranged. We can see that part E is extra in improvement.
the sequence as it does not fit in the context This makes option (e) the most logical answer
being discussed in the previous parts. choice.
So, the best is to mark option (c) as the answer.
21) Answer: A
18) Answer: C The phrase brace for should be the correct
The second part uses the phrase ‘narrowed usage as it means to prepare yourself for
from’ incorrectly. The correct usage is ‘narrowed something unpleasant. The preposition of is
down (on/upon)’ which means to reduce the incorrect in the context of the sentence. This
number of possibilities or choices… makes incorrect.
In the fourth part; the verb made should be used The sentence is in the continuous form as it is
as the verbs in the sentence should follow a being said that NCR must be ready as the air
parallel construction and the sentence needs to quality is worsening from very poor to severe
take verbs as participles. So, narrowed --- made levels. The correct usage should hence, be
etc should be correct. reaching.
This makes option (c) the most logical answer This makes BC the correct pair of the needed
choice. erroneous parts.
Thus, the correct answer is option (a).
19) Answer: B
The first part uses the subject Art which is 22) Answer: B
singular so it should be followed by a singular

Click Here For Bundle PDF Course | support@guidely.in Page 10 of 11


SBI Clerk & RRB PO Mains PDF Course 2023
ENGLISH Day - 17

Part C contains an error as the helping verb We use than for comparisons and then to
have has wrongly been used in the same. The answer the question when. In the sentence
correct verb needed is are as the continuous above; more than should be correct.
form should take the same. This makes option (b) the most logical answer
choice.

Click Here For Bundle PDF Course | support@guidely.in Page 11 of 11


SBI Clerk & RRB PO Mains PDF Course 2023
Reasoning Ability Day - 18 (Eng)

Reasoning Ability
Directions (1-5): Study the following information
carefully and answer the given questions. 4) Which of the following code is wrongly
In a certain code language, represented in the bolded sentence?
“Traditional Culture Would Sustain” is coded as a) 0VΩ
“5X& 11U* 7V@ 7V#” b) 12Q@
“Telecast Common Issues Policy” is coded as c) 16U%
“16U% 12Zπ 0VΩ 12Q@” d) 12Zπ
“Education Become Special Priority” is coded as e) All are true
“7T* 16Zπ 9V@ 0P#
“Government Upheld Labour Rights” is coded as 5) What does the code “15TΩ” represents in the
“40W% 24UΩ (?) 0V$” given coded language?
1) What does the code “32U& 9V#” represent in I) Cards
the given code language? II) Sales
a) Stand Proper III) Balls
b) Infinite Popular a) Only I
c) Standard Procedure b) Only I and II
d) Mandate Critical c) Only III
e) None of these d) Only I and III
e) Only II
2) Which among the following code comes in the
place (?). Directions (6-10): Study the following information
a) 12T% carefully and answer the given questions.
b) 16V& Sixteen persons – C, D, E, F, G, H, I, J, K, L, M,
c) 32U* N, O, P, Q and R are sitting in three rows namely
d) 12XΩ Row1, Row2 and Row 3 such that four persons
e) 12V& are sitting in each Row1 and Row3 and eight
persons are sitting in row 2. Persons sitting in
3) What is the code for “Higher Enrollment”? Row1 are facing towards the south direction and
a) 12S$ 40U% the persons sitting in Row 3 are facing towards
b) 16T% 44U@ the north direction. The first four persons from
c) 12S@ 11T# the left end of Row 2 are facing north direction
d) 16S$ 32U% and the remaining four persons are facing south
e) 40S$ 12U& direction. Thus the first four persons of Row 2

Click Here For Bundle PDF Course | support@guidely.in Page 1 of 10


SBI Clerk & RRB PO Mains PDF Course 2023
Reasoning Ability Day - 18 (Eng)

are facing the persons sitting in Row1 and the 7) Which among the following pair of persons,
persons sitting in Row 3 are facing the remaining the second person sits immediate right of the
persons of Row 2, who are facing the south one who faces the first person?
direction. The distance between adjacent I) K – I
persons in each row is same. II) F – N
Note: The consecutive alphabetically named III) J – C
persons are not facing towards each other. a) Only I
J sits third to the right of O, who doesn’t sit in b) Only I and II
Row1. O sits at one of the extreme ends of the c) Only III
row. The one who sits second to the left of K is d) Only II and III
facing J. As many persons sit to the left of K as e) All I, II and III
to the right of C. L sits fourth to the right of the
one who faces C. D sits immediate right of L. P is 8) How many persons are sitting between E and
an immediate neighbour of the one who faces D. the one who faces R?
Only two persons sit between the one who faces a) As many persons sit to the right of D
P and the one who faces I. F sits third to the right b) Two
of the one who faces I, who doesn’t sit in Row1. c) One
H sits second to the right of the one who faces F. d) As many persons sit to the left of Q
The number of persons sitting to the right of H is e) None
two less than the number of persons sitting to the
left of Q. At least two persons sit between Q and 9) What is the position of K with respect to the
G, who faces the one who sits second to the right one who faces H?
of N. As many persons sit to the left of N as to a) Fourth to the left
the left of E, who doesn’t sit immediate right of R. b) Immediate right
6) Who among the following person sits third to c) Fourth to the right
the right of the one who faces F? d) Immediate left
a) H e) Third to the right
b) The one who sits immediate right of K
c) C 10) Four of the following five are alike in a certain
d) The one who sits immediate right of the one way based on the given arrangement and thus
who faces L form a group. Which one of the following does
e) P not belong to the group?
a) G
b) J

Click Here For Bundle PDF Course | support@guidely.in Page 2 of 10


SBI Clerk & RRB PO Mains PDF Course 2023
Reasoning Ability Day - 18 (Eng)

c) The one who sits third to the left of R 12. Which of the following box has a maximum
d) M number of roses?
e) The one who sits immediate right of K a) The box which is kept in the topmost rack
b) P
Direction (11-15): Study the following information c) T
carefully and answer the given questions. d) R
Seven boxes are kept one above the other in a e) The box which is kept immediately below Q
single rack. Each box has different number of
roses. 13. Which of the following statements is/are not
The difference between the number of roses in true with respect to the final arrangement?
boxes V and Q is 12. Q is kept four boxes above a) The difference between the roses in boxes S
the box which has 56 roses. The number of and Q is an odd number
boxes kept above Q is one less than the number b) V has 31 roses less than R
of boxes kept below V, which has the least c) P has more roses than Q but less than T
number of roses. The difference between the d) Both a and b
number of roses in boxes P and T is an odd e) Both a and c
number. Only one box is kept between V and the
box which has a prime number of roses. S is kept 14. What is the sum of all the roses in the boxes
four boxes below P, which doesn’t have a prime which has an even number of roses?
number of roses. R has 22 roses less than U, a) 126
which has 11 roses more than T. Only two boxes b) 176
are kept between T and S, which has a square c) 156
number of roses. P has 15 roses more than Q d) 163
and 7 roses less than S. The sum of roses in any e) 192
two boxes will not exceed 150. The number of
roses in the topmost box is not an even number. 15. Which of the following box is kept three
11. What is the difference between the number boxes above R?
of roses in box V and the box which is kept two a) Q
boxes below U? b) The box which has 67 roses
a) 47 c) S
b) 36 d) The box which has 78 roses
c) 52 e) V
d) 11
e) 29

Click Here For Bundle PDF Course | support@guidely.in Page 3 of 10


SBI Clerk & RRB PO Mains PDF Course 2023
Reasoning Ability Day - 18 (Eng)

Directions (16-20): Study the following immediately below R. Only two floors are
information carefully and answer the given between E and Z, who doesn’t live adjacent floor
questions. of W. Z lives two floors above N, who lives in
In a building, there are certain number of floors different type of flat as F. As many floors below N
where the ground floor is numbered as above C.
one and the floor immediately above it is 16) How many flats are above N?
numbered two and so on. a) 9
Note I: Each floor has two type of flats viz., Flat-A b) 12
and Flat-B, where Flat-A is to the c) 16
west of Flat-B. d) 18
Note II: Flat-B of floor 2 is immediately above the e) 8
Flat-B of floor 1 and immediately
below the Flat-B of floor 3 and so on. Similarly, 17) Which of the following statement(s) is/are
Flat-A of floor 2 is immediately above NOT TRUE as per the given arrangement?
the Flat-A of floor 1 and immediately below the I) W lives immediately above the flat of R
Flat-A of floor 3 and so on. II) No one lives to the east of M
Note III: Area of each flat on each floor is equal. III) E and V are not living on the adjacent floors
Note IV: Atmost two persons live on each floor a) Only I and III
and Atmost one person lives in each flat. b) Only I
Note V: Consecutive alphabetically named c) Only II and III
persons are not living on the adjacent floors of d) Only III
the same flat. e) All are true
Only two floors are between V and K, who lives
on the fourth floor. C lives three floors above the 18) Four of the following five are alike in a certain
flat of V. No one lives to the west of C. C lives way based on the given arrangement and thus
four floors away from Q, who lives two floors form a group. Which one of the following does
above M. Q lives west of T. M doesn’t live on the not belong to the group?
floor number which is a multiple of 3. No one a) EZ
lives between T and F, who lives two floors b) FK
below the flat of S. Only two floors are between S c) NT
and I but both of them are not living in the same d) ZE
type of flat. As many floors between I and F as e) KB
below B. Only four floors are between W and B,
who lives four floors below the flat of R. E lives

Click Here For Bundle PDF Course | support@guidely.in Page 4 of 10


SBI Clerk & RRB PO Mains PDF Course 2023
Reasoning Ability Day - 18 (Eng)

19) As many floors between Z and ___ as the persons living on an even numbered floor
between ___ and I respectively. paid 25000/month, then what is the sum of the
a) Q and W rents paid by Q, N and S?
b) E and S a) Rs.50000
c) S and Q b) Rs.55000
d) M and C c) Rs.45000
e) F and E d) Rs.80000
e) Rs.65000
20) If the persons living on an odd numbered
floor paid Rs.10000/month as the flat rent and
Click Here to Get the Detailed Video Solution for the above given Questions
Or Scan the QR Code to Get the Detailed Video Solutions

Answer Key with Explanation

Directions (1-5):
1) Answer: C
2) Answer: E
3) Answer: A
4) Answer: B
5) Answer: D

Click Here For Bundle PDF Course | support@guidely.in Page 5 of 10


SBI Clerk & RRB PO Mains PDF Course 2023
Reasoning Ability Day - 18 (Eng)

If the last letter of the word is L, then the symbol


is “*”
If the last letter of the word is E, then the symbol
is “#”
If the last letter of the word is D, then the symbol
is “&”
If the last letter of the word is N, then the symbol
is “@”
If the last letter of the word is S, then the symbol
is “Ω”
If the last letter of the word is Y, then the symbol
is “π”
If the last letter of the word is R, then the symbol
is “$”

Directions (6-10):
6) Answer: D
The number for each word represents the
7) Answer: B
difference between the square value of the
8) Answer: A
number of consonants and the square value of
9) Answer: C
the number of vowels.
10) Answer: E (All the persons are sitting at one
Ex: Traditional; 6 consonants and 5 vowels = 36-
of the extreme ends of the
25 = 11
row except “The one who sits immediate right of
The letter for each word represents the
K”)
immediate succeeding letter of the highest place
Final arrangement
valued letter (as per the alphabetical series) of
each word
Ex: Traditional; highest place valued letter is T,
then the immediate succeeding letter of T is U
The symbol for each word represents based on
the last letter of each word
If the last letter of the word is T, then the symbol We have,
is “%”

Click Here For Bundle PDF Course | support@guidely.in Page 6 of 10


SBI Clerk & RRB PO Mains PDF Course 2023
Reasoning Ability Day - 18 (Eng)

 J sits third to the right of O, who doesn’t  H sits second to the right of the one who
sit in Row1. faces F
 O sits at one of the extreme ends of the
row.
 The one who sits second to the left of K is
facing J.
From the above condition, there are three
possibilities.

Thus, case (2) is not valid as I doesn’t sit in


row1.
Again we have,
 The number of persons sitting to the right
of H is two less than the number of
persons sitting to the left of Q
Again we have,
 At least two persons sit between Q and G,
 As many persons sit to the left of K as to
who faces the one who sits second to
the right of C
the right of N
 L sits fourth to the right of the one who
 As many persons sit to the left of N as to
faces C
the left of E, who doesn’t sit immediate
 D sits immediate right of L
right of R
 P is an immediate neighbour of the one
After applying the above condition, Case1 gets
who faces D
eliminated. Because there is
 Only two persons sit between the one
no possibility to place “E” . Hence, Case3 shows
who faces P and the one who faces I
the final arrangement.
 F sits third to the right of the one who
faces I, who doesn’t sit in Row1

Click Here For Bundle PDF Course | support@guidely.in Page 7 of 10


SBI Clerk & RRB PO Mains PDF Course 2023
Reasoning Ability Day - 18 (Eng)

Again we have,
Directions (11-15)  The difference between the number of
11. Answer: C roses in boxes P and T is an odd number.
12. Answer: E  Only one box is kept between V and the
13. Answer: D box which has a prime number of roses.
14. Answer: B  S is kept four boxes below P, which
15. Answer: D doesn’t have a prime number of roses.
Final arrangement:  R has 22 roses less than U, which has 11
roses more than T.
 Only two boxes are kept between T and
S, which has a square number of roses.
While applying the above conditions, case 2 gets
eliminated, because can’t place S and P.

We have,
 The difference between the number of
roses in boxes V and Q is 12.
Again we have,
 Q is kept four boxes above the box which
 P has 15 roses more than Q and 7 roses
has 56 roses.
less than S.
 The number of boxes kept above Q is one
 The sum of roses in any two boxes will
less than the number of boxes kept below
not exceed 150.
V, which has the least number of roses.
 The number of roses in the topmost box is
From the above conditions, we have three
not an even number.
possibilities:
Calculation:
First taking S- Squares- 1, 4, 9, 16 and goes on

Click Here For Bundle PDF Course | support@guidely.in Page 8 of 10


SBI Clerk & RRB PO Mains PDF Course 2023
Reasoning Ability Day - 18 (Eng)

 S cannot 1, 4, 9, 16, 25 because if S is


between 1-25, then the value of V will
goes in negative.
 Also S cannot have more than 65 roses,
because if S is 81, then P will be 74, and
their sum of roses will become more than
150.
 So S can be 36/49/64
From S, we can get P (S-7), then Q (P-15), than
finally V (Q-12)
If R is 56, then U is 78 and T is 67; If U is 56,
then T is 45 and R is 34
While applying the value of R and U, case 3 gets
We have,
eliminated. The topmost box should have an odd
 Only two floors are between V and K, who
number of roses, thus case 3a gets eliminated.
lives on the fourth floor.
Thus, case 1 gives the final arrangement.
 C lives three floors above the flat of V.
 No one lives to the west of C.
From the above condition, there are three
possibilities

Directions (16-20):
16) Answer: D
17) Answer: A
18) Answer: E (Only two floors are between the
given pair of persons except option E)
19) Answer: B
20) Answer: C
Final arrangement Again we have,
 C lives four floors away from Q, who lives
two floors above M.
 Q lives west of T

Click Here For Bundle PDF Course | support@guidely.in Page 9 of 10


SBI Clerk & RRB PO Mains PDF Course 2023
Reasoning Ability Day - 18 (Eng)

 M doesn’t live on the floor number which  Only two floors are between E and Z, who
is a multiple of 3. doesn’t live adjacent floor of W
 No one lives between T and F, who lives  Z lives two floors above N, who lives in
two floors below the flat of S. different type of flat as F.
After applying the above condition, Case1 gets  As many floors below N as above C
eliminated. Because there is no After applying the above condition, Case 3 gets
possibility to place M. eliminated. Because there is no
possibility to place N. Hence, Case 2 shows the
final arrangement.

Again we have,
 As many floors between I and F as below
B
 Only two floors are between S and I but
both of them are not living in the same
type of flat.
 Only four floors are between W and B,
who lives four floors below the flat of R
 E lives immediately below R.

Click Here For Bundle PDF Course | support@guidely.in Page 10 of 10


SBI Clerk & RRB PO Mains PDF Course 2023
Quantitative Aptitude Day - 18 (Eng)

Quantitative Aptitude

Direction (1-4): Study the following data carefully e) 3 hours 20 minutes


and answer the questions:
Six workers A, B, C, D, E, and F are hired to 3) If the ratio of A’s efficiency to B’s efficiency is
complete a work, and they are paid in the ratio of p: q andthe ratio of B’s efficiency to F’s efficiency
their efficiencies. The efficiency of each person is is r: s, then the value of lies between?
different. a) 1 and 7
The time in which A alone can complete the work b) 3 and 10
is ‘t’ hours, the time, in which B alone can c)5 and 12
complete the work is (t + 8) hours and the time, d) Both (a) and (b)
in which A and B together can complete the work e) Both (a) and (c)
is 9.6 hours. The time, in which C alone can
complete the work in (t – 1) hours. Total wages
received by C, E, and F together is ₹2250, in 4) Find the difference between B’s wages and
which is E’s wages, and C’s wages is D’s wages?
₹500 more than F’s wages. D and E together can a) ₹1050
complete the work in hours. b) ₹950
1) Find the average of wages received by A, B c) ₹1250
and C? d) ₹850
a) ₹857.22 e) ₹1150
b) ₹854.16
c)₹867.33 Direction (5-7): In each of the following
d) ₹887.16 questions, two quantities I and II are given.
e) ₹864.33 Compare the quantity I and quantity II on its
basis: (Only quantity is to be considered)
2) If A works with 200% of its original efficiency 5)
and, D and E work with their original efficiencies, Quantity I: A solid cube is melted down to form a
then in what time will A, D and E together cuboid, whose length is 200% more than its
compete the work? breadth and whose breadth is 200% more than
a) 2 hours 45 minutes its height. If the side of the cube is 12 cm, then
b) 3 hours 15 minutes find the surface area of the cuboid (in cm2)?
c)2 hours 40 minutes Quantity II: A, B and C are hired to complete a
d) 3 hours 30 minutes work and they are paid in the ratio of their

Click Here For Bundle PDF Course | support@guidely.in Page 1 of 14


SBI Clerk & RRB PO Mains PDF Course 2023
Quantitative Aptitude Day – 18 (Eng)

efficiencies.A and B together can complete the c) Quantity I ≥ Quantity II or relation can’t be
work in 4 hours and their wages are ₹2496 and determined
₹4992 respectively. If B and C together can d) Quantity I ≤ Quantity II
complete the work in 4.8 hours, then find C’s e) Quantity I = Quantity II
wages (in ₹)?
a) Quantity I > Quantity II 7) Two boats, A and B are sailing in a river and
b) Quantity I < Quantity II the ratio of speed in still water of boat A to that of
c) Quantity I ≥ Quantity II boat B is 3: 4. The ratio of the downstream
d) Quantity I ≤ Quantity II distance covered by boat A in 10 seconds to the
e) Quantity I = Quantity II or relation can’t be upstream distance covered by boat B in 15
determined seconds is 8: 9 and the upstream distance
6) covered by boat A in 45 seconds is 360 m.
Quantity I: The difference between compound Quantity I: If the speed of boat A in still water
interest and simple interest on ₹24000 at a R% were 2 m/s less, then what would be the
rate after 2 years is ₹540 and after 3 years is downstream distance covered by boat A in 15
₹1701. Find the value of R? seconds?
Quantity II: An article is marked up by 25% and Quantity II: Find the downstream distance
sold at 12.5% profit after giving ₹X discount on covered by boat B in 10 seconds?
its marked price. If the cost of the article were a) Quantity I ≤ Quantity II
₹24 more than its original cost and sold at ₹(X + b) Quantity I ≥ Quantity II
3) discount on its marked price, then the profit c)Quantity I < Quantity II
per cent would be the same as initially. Find the d) Quantity I > Quantity II
original discount percentage? e) Quantity I = Quantity II or relation can’t be
a) Quantity I > Quantity II determined
b) Quantity I < Quantity II

Direction (8-12): Study the following data carefully and answer the questions:
The data given below shows the number of orders received and the number of orders confirmed (out of
total orders received) by an e-commerce company on five different days Monday, Tuesday, Wednesday,
Thursday and Friday.
Bar graph given below shows the following data:

Click Here For Bundle PDF Course | support@guidely.in Page 2 of 14


SBI Clerk & RRB PO Mains PDF Course 2023
Quantitative Aptitude Day – 18 (Eng)

Some other information is also known:


The ratio of the number of orders received on Monday to that received on Tuesday is 16: 9, the number
of orders received till Tuesday is 62.5% of that received till Wednesday, which is 200. The number of
orders received till Fridayis 20% more than that received till Thursday, which is 140% more than that
received till Tuesday.
8) If the company delivered 96% of the number c)21
of orders confirmed on Wednesday and d) 24
delivered 80% of the number of orders confirmed e) 23
on Thursday, then find the ratio of the number of
orders delivered on Wednesday to that delivered 10) If the number of orders received on Saturday
on Thursday? is of that received on Wednesday and the
a) 6: 5 number of orders confirmed on Saturday is
b) 12: 11 of that confirmed on Monday, then find
c) 8: 7 that the number of orders not confirmed on
d) 16: 15 Saturday, is what percentage of orders not
e) 1: 1 confirmed on that day, in which the number of
orders received is maximum?
9) Find the average of the number of orders not a) 50%
confirmed on Tuesday, Wednesday and Friday? b) 75%
a) 20 c)60%
b) 22 d) 80%

Click Here For Bundle PDF Course | support@guidely.in Page 3 of 14


SBI Clerk & RRB PO Mains PDF Course 2023
Quantitative Aptitude Day – 18 (Eng)

e) 40% The number of person who like only channel A, is


80% of those who like channel B only, which is
11) Which of the following is/are true? double that of those who like channel C only. The
A: Average of number of orders confirmed on ratio of number of persons who like both
Tuesday, Wednesday and Friday is 38. channels A and B but not C to those who like
B: The ratio of the number of orders confirmed both channels A and C but not B is 5: 9. Number
on Thursday to the number of orders not of persons who like all the three channels is 65
confirmed on Thursday is 3: 2. more than those who like both channels B and C
C: of the total orders received on but not A. The total number of persons who like
Wednesday, were not confirmed. channel A, is 275. The number of persons who
a) All are true like both channels B and C but not A, is 5 more
b) Only A and B than those who like both channels A and B but
c)Only B and C not C. One-third of the total number of persons
d) Only A and C who like channel B, like only channel B.
e) None is true 13) Find the difference between the total number
of persons who like only one channel and those
12) What per cent of total number of orders who like only two channels?
received Thursday and Friday together, were not a) 69
confirmed? b) 89
a) 36.5% c) 49
b) 47.5% d) 79
c) 50.5% e) 59
d) 40.5%
e) 32.5% 14) What percent of total number of persons in
society, like channel B?
Direction (13-16): Study the following data a) 75%
carefully and answer the questions: b) 50%
A survey is conducted to know the preferred TV c) 60%
channels (among A, B, and C) of the persons d) 80%
living in a society. Some like only one TV e) 40%
channel, some like only two TV channels, and
remaining like all three TV channels. 15)If the number of persons who like all three
channels, is M% of the total number of persons
who like channel A and also N% of the total

Click Here For Bundle PDF Course | support@guidely.in Page 4 of 14


SBI Clerk & RRB PO Mains PDF Course 2023
Quantitative Aptitude Day – 18 (Eng)

number of persons in the society, then find the e) Only P, Q and R


value of (11M – 9N) divisible by which of the
following? 16) What percent of the total number of persons
P: 7 in society like at-least two channels?
Q: 3 a) 46%
R: 21 b) 54%
a) Only P and Q c) 48%
b) Only R d) 52%
c) Only Q e) None of these
d) Only P and R

Direction (17-20): Study the following data carefully and answer the questions:
Students from four different colleges A, B, C, and D applied for an exam. The table given below shows
the number of students who applied for the exam, number of students who appeared for the exam and
number of students who passed that exam.

Note:
1. Number of students applied = Number of students appeared + Number of students did not appear.
2. Number of students appeared = Number of students passed + Number of students failed

Click Here For Bundle PDF Course | support@guidely.in Page 5 of 14


SBI Clerk & RRB PO Mains PDF Course 2023
Quantitative Aptitude Day – 18 (Eng)

17) The table given below shows the ratio of c) Only B and C
male to female students who did not appear from d) All A, B, and C
college C, ratio of male to female students who e) Only C
failed from college C, and the ratio of male to
female students who passed from college C. 19) Out of the total passed and failed students
Then find the ratio of male to female students from college D, X% and Y% respectively, are
who applied from college C. freshers and the table given below shows the
possible values of X and Y.

a) 5: 2 Find the difference between number of appeared


b) 7: 4 fresher students from college D in both the
c) 7: 3 cases.
d) 5: 3 a) 150
e) 9: 5 b) 60
c) 120
18) Out of the total number of students who d) 180
appeared for the exam and did not appear for the e) 80
exam from college A, ___ percent and ___
percent respectively, are male. If difference 20)What is the ratio of the sum of the number of
between male and female students who applied students who did not appear from colleges B and
to college A is 700 (male > female), then which C together to the sum of number of students who
of the following value(s) can fill the blank in the failed from colleges B and D together?
same order? a) 2: 1
A: 60 and 75 b) 5: 4
B: 65 and 60 c) 5: 2
C: 70 and 45 d) 7: 3
a) Only B e) 5: 3
b) Only A and B

Click Here For Bundle PDF Course | support@guidely.in Page 6 of 14


SBI Clerk & RRB PO Mains PDF Course 2023
Quantitative Aptitude Day - 18 (Eng)

Click Here to Get the Detailed Video Solution for the above given Questions
Or Scan the QR Code to Get the Detailed Video Solutions

Answer Key with Explanation

Direction (1-4): E’s wages = of 2250 = ₹750


The time, in which A alone can complete the So,
work = ‘t’ hours x + (x + 500) + 750 = 2250
The time, in which B alone can complete the x = 500
work = (t + 8) hours Now, ratio between efficiencies of C, E and F
So, respectively:
(500 + 500): 750: 500 = 4: 3: 2
The time, in which E alone can complete the
work = = 20 hours
t2 + 8t = 19.2t + 76.8
The time, in which F alone can complete the
t2 – 11.2t – 76.8 = 0
work = = 30 hours
t2 – 16t + 4.8t – 76.8 = 0
Now, the time, in which D and E together can
t(t – 16) + 4.8(t – 16) = 0
complete the work:
t = 16
The time, in which A alone can complete the
work = 16 hours Let the time, in which D alone can complete the
The time, in which B alone can complete the work is ‘d’ hours.
work = 16 + 8 = 24 hours So,
The time, in which C alone can complete the
work = 16 – 1 = 15 hours d=8
Now, let F’s wages is ₹’x’. So, the time, in which D alone can complete the
So, C’s wages = ₹(x + 500) work = 8 hours

Click Here For Bundle PDF Course | support@guidely.in Page 7 of 14


SBI Clerk & RRB PO Mains PDF Course 2023
Quantitative Aptitude Day - 18 (Eng)

1. Answer: B Ratio of B’s efficiency to C’s efficiency = 15: 24 =


C’s wages = ₹1000 5: 8
Ratio of A’s efficiency to C’s efficiency = 15: 16 So, B’s wages = = ₹625
So, A’s wages = Ratio of D’s efficiency to C’s efficiency = 15: 8
Ratio of B’s efficiency to C’s efficiency = 15: 24 = So, D’s wages = = ₹1875
5: 8 Required difference = 1875 – 625 = ₹1250
So, B’s wages = = ₹625
Total wages received by A, B and C together = 5. Answer: E
Quantity I:
Required average = = ₹854.16 Since, the side of the cube = 12 cm
So, the volume of the cuboid = the volume of the
2. Answer: E cube = 123 = 1728 cm3
The time, in which A alone can complete the Let the height of the cuboid = ‘x’ cm
work with 200% of its original efficiency = So, the breadth of the cuboid = 300% of ‘x’ = ‘3x’
= 8 hours cm

The time, in which D alone can complete the And the length of the cuboid = 300% of 3x = ‘9x’

work = 8 hours cm

The time, in which E alone can complete the So,

work = 20 hours 9x × 3x × x = 1728

Work done by A, D and E together in 1 hour = x=4


Length of the cuboid = 36 cm

So, the time, in which A, D and E together can Breadth of the cuboid = 12 cm

complete the work = 3.33 hours = 3 hours 20 Height of the cuboid = 4 cm

minutes So, the surface area of the cuboid = 2 ×[(36 ×


12) + (12 × 4) + (36 × 4)] = 1248 cm2

3. Answer: D Quantity II:

Ratio of A’s efficiency to B’s efficiency = p: q = Ratio of A’s efficiency to B’s efficiency = 2496:

24: 16 = 3: 2 4992 = 1: 2

Ratio of B’s efficiency to F’s efficiency = r: s = Let the time, in which A alone and B alone can

30: 24 = 5: 4 complete the work, are ‘2t’ hours and ‘t’ hours

So, the value of respectively.


So,

4. Answer: C

Click Here For Bundle PDF Course | support@guidely.in Page 8 of 14


SBI Clerk & RRB PO Mains PDF Course 2023
Quantitative Aptitude Day - 18 (Eng)

t=6 And the new SP of the article = 112.5% of (4a +


Since, B and C together can complete the work 24) = ₹(4.5a + 27)
in 4.8 hours. So,
So, 5a + 30 – X – 3 = 4.5a + 27
X = 0.5a
We can’t find the original discount per cent.
C = 24
Hence, relation can’t be determined.
Ratio of B’s efficiency to C’s efficiency = 24: 6 =
4: 1
7. Answer: D
So, C’s wages = = ₹1248
Let the speeds in still water of boats A and B are
Hence, Quantity I = Quantity II
‘3x’ m/s and ‘4x’ m/s respectively.
Also let the speed of the stream is ‘y’ m/s.
6. Answer: C
So,
Quantity I:
Since,
------------------(1) 9x + 3y = 16x – 4y

And, x = y ----------(1)
And,
---------------(2)
(3x – y) × 45 = 360
By equation (2) ÷ equation (1):
3x – y = 8 --------------(2)
From equation (1) and (2):
R = 15% x=y=4
Quantity II: Speed of boat A in still water = 12 m/s
Let the CP of the article = ₹‘4a’ Speed of boat B in still water = 16 m/s
So, the MP of the article = 125% of ‘4a’ = ₹‘5a’ Speed of the stream = 4 m/s
And the SP of the article = 112.5% of ‘4a’ = Quantity I:
₹‘4.5a’ New speed of boat A in still water = 12 – 2 = 10
So, m/s
5a – X = 4.5a And the downstream distance covered by boat A
X = 0.5a in still water would be:
Now, the new CP of the article = ₹(4a + 24) (10 + 4) × 15 = 210 m
The new MP of the article = 125% of (4a + 24) = Quantity II:
₹(5a + 30) The downstream distance covered by boat B in
10 seconds = (16 + 4) × 10 = 200 m

Click Here For Bundle PDF Course | support@guidely.in Page 9 of 14


SBI Clerk & RRB PO Mains PDF Course 2023
Quantitative Aptitude Day - 18 (Eng)

Hence, Quantity I > Quantity II

Direction (8-12):
Number of orders received till Wednesday = 200
Let the number of orders received on Monday
and Tuesday are 16x and 9x respectively.
So,
16x + 9x = 62.5% of 200
x=5
8. Answer: E
Number of orders received on Monday = 16 × 5
Number of orders confirmed on Wednesday = 50
= 80
So, the number of orders delivered on
Number of orders received on Tuesday = 9 × 5 =
Wednesday = 96% of 50 = 48
45
Number of orders confirmed on Thursday = 60
Number of orders received till Tuesday = 80 + 45
So, the number of orders delivered on Thursday
= 125
= 80% of 60 = 48
Number of orders received till Thursday = 240%
Required ratio = 48: 48 = 1: 1
of 125 = 300
9. Answer: B
Number of orders received till Friday = 120% of
Number of orders not confirmed on Tuesday =
300 = 360
45 – 40 = 5
Number of orders not confirmed on Wednesday
= 75 – 50 = 25
Number of orders not confirmed on Friday = 60 –
24 = 36
Required average = = 22

10. Answer: C
Number of orders received on Wednesday = 75
So, the number of orders received on Saturday =
of 75 = 40
Number of orders confirmed on Monday = 60
So, the number of orders confirmed on Saturday
= of 60 = 16

Click Here For Bundle PDF Course | support@guidely.in Page 10 of 14


SBI Clerk & RRB PO Mains PDF Course 2023
Quantitative Aptitude Day - 18 (Eng)

Number of orders not confirmed on Saturday = Required percentage = = 47.5%


40 – 16 = 24
The day, in which the number of orders received Direction (13-16):
is maximum, is Thursday. Let the number of persons who like only channel
So, the number of orders not confirmed on B = 10x
Thursday = 100 – 60 = 40 So, the number of persons who like only channel
Required percentage = = 60% A = 80% of 10x = 8x
And the number of persons who like only
11. Answer: A channel C = 5x
From A: Let the number of persons who like both
Average of number of orders confirmed on channels A and B but not C = 5y
Tuesday, Wednesday and Friday: So, the number of persons who like both
channels A and C but not B = 9y
Let the number of persons who like both
So, A is true.
channels B and C but not A = z
From B:
So, the number of persons who like all the three
Ratio of number of orders confirmed to the
channels = (z + 65)
number of orders not confirmed on Thursday =
Since, the total number of persons who like
60: (100 – 60) = 60: 40 = 3: 2
channel A, is 275.
So, B is true.
So,
From C:
8x + 5y + 9y + (z + 65) = 275
Number of orders received on Wednesday = 75
8x + 14y + z = 210 ----------------(1)
Number of orders not confirmed on Wednesday
Since, the number of persons who like both
= 75 – 50 = 25
channels B and C but not A, is 5 more than
Required percentage =
those who like both channels A and B but not C.
So, C is true.
So,
Hence, all are true.
z – 5y = 5 ----------------(2)
Since, one-third of the total number of persons
12. Answer: B
who like channel B, likes only channel B.
Total number of orders received on Thursday
So,
and Friday together = 100 + 60 = 160
10x + 5y + z + (z + 65) = 3 × 10x
Number of orders not confirmed on Thursday
20x – 5y – 2z = 65 ----------------(3)
and Friday together = (100 – 60) + (60 – 24) = 76
From equations (1) and (2):

Click Here For Bundle PDF Course | support@guidely.in Page 11 of 14


SBI Clerk & RRB PO Mains PDF Course 2023
Quantitative Aptitude Day - 18 (Eng)

8x + 14y + 5y + 5 = 210 15. Answer: E


8x + 19y = 205 ----------------(4) Total number of persons who like channel A =
By equation (1) × 2 + equation (3): 275
16x + 28y + 2z + 20x – 5y – 2z = 420 + 65 Total number of persons in the society = 450
36x + 23y = 485 ----------------(5) The number of persons who like all the three
By equation (4) × 9 – equation (5) × 2: channels = 105
72x + 171y – 72x – 46y = 1845 – 970 So, M = 105/275 ×100= 420/11%
y=7 And, N = 105/450 ×100= 70/3%
From equation (2): Now, the value of (11M – 9N) = 420 – 210 = 210
z = 40 210 is divisible by3, 7 and 21.
From equation (1):
x=9 16. Answer: B
Number of persons who like at-least two
channels = Number of persons who like only two
channels + Number of persons who like all the
three channels
Number of persons who like at-least two
channels = (35 + 40 + 63) + 105 = 243
Total number of persons in the society = 450
13. Answer: A Required percentage = = 54%
Total number of persons who like only one
channel = 72 + 90 + 45 = 207
Direction (17-20):
Total number of persons who like only two
Number of students applied from college A =
channels = 35 + 40 + 63 = 138
2500
Required difference = 207 – 138 = 69
Number of students appeared from college B =
2500 + 2000 = 4500
14. Answer: C
Number of students passed from college B =
Total number of persons in the society = 72 + 90
4000
+ 45 + 35 + 40 + 63 + 105 = 450
Number of students applied from college B =
Total number of persons who like channel B = 90
= 6000
+ 35 + 40 + 105 = 270
Number of students appeared from college A =
Required percentage = = 60%
4500 – 2500 = 2000

Click Here For Bundle PDF Course | support@guidely.in Page 12 of 14


SBI Clerk & RRB PO Mains PDF Course 2023
Quantitative Aptitude Day - 18 (Eng)

Number of students passed from college A = Number of students who did not appeared from
60% of 2000 = 1200 college C = 4000 – 3000 = 1000
Total number of students applied from colleges Number of male students who did not appeared
A, B, and C together = 12500 from college C = = 600
Number of students applied from college C = Number of female students who did not
12500 – 2500 – 6000 = 4000 appeared from college C = = 400
Number of students appeared from college C = Number of male students who applied from
3000 college C = 700 + 1500 + 600 = 2800
Number of students passed from college C = Number of female students who applied from
= 2000 college C = 300 + 500 + 400 = 1200
Number of students applied from college D = Required ratio = 2800: 1200
4500 = 7: 3
Number of students appeared from college D = 18. Answer: A
2000 × 2 = 4000 Total number of students who appeared for the
Number of students passed from college D = exam from college A = 2000
62.5% of 4000 = 2500 Total number of students who did not appeared
for the exam from college A = 2500 – 2000 = 500
Let the values that can fill the blank are P% and
Q% respectively.
Difference between male and female students
who applied from college A = 700
Number of male students who applied from
17. Answer: C college A = = 1600
Number of students who failed from college C =
P% of 2000 + Q% of 500 = 1600
3000 – 2000 = 1000
20P + 5Q = 1600
Number of male students who failed from college
4P + Q = 320
C= = 700
Q = 320 – 4P
Number of female students who failed from
Since cannot be more than 100.
college C = = 300 Q < 100
Number of male students who passed from 320 – 4P < 100
college C = = 1500 220 < 4P
Number of female students who passed from P > 55
college C = = 500 When P = 60, Q = 80

Click Here For Bundle PDF Course | support@guidely.in Page 13 of 14


SBI Clerk & RRB PO Mains PDF Course 2023
Quantitative Aptitude Day - 18 (Eng)

When P = 65, Q = 60 20. Answer: B


When P = 70, Q = 40 Number of students who did not appeared from
Hence, only B is TRUE. college B = 6000 – 4500 = 1500
Number of students who did not appeared from
19. Answer: E college C = 4000– 3000 = 1000
Total passed students from college D = 2500 Sum of number of students who did not
Total failed students from college D = 4000 – appeared from colleges B and C together = 1500
2500 = 1500 + 1000 = 2500
Number of fresher students appeared from Number of students who failed from college B =
college D in case 1 = 60% of 2500 + 72% of 4500 – 4000 = 500
1500 = 1500 + 1080 = 2580 Number of students who failed from college D =
Number of fresher students appeared from 4000 – 2500 = 1500
college D in case 1 = 64% of 2500 + 60% of Sum of number of students who failed from
1500 = 1600 + 900 = 2500 colleges B and D together = 500 + 1500 = 2000
Required difference = 2580 – 2500 Required ratio = 2500: 2000
= 80 = 5: 4

Click Here For Bundle PDF Course | support@guidely.in Page 14 of 14


SBI Clerk & RRB PO Mains PDF Course 2023
ENGLISH Day - 18

English Language

Directions (01-05): In the passage given below India introduced a New Economic Policy in 1991,
there are 5 blanks, each followed by a under the ____________ (tyrannical) (3)
word/phrase given in bold. Each blank has four management of Prime Minister Narsimha Rao
alternative words/phrases given in options A, B, and Finance Minister Dr. Manmohan Singh, who
C and D. You have to tell which word will best incorporated three significant objectives –
suit the respective blank. Mark E as your answer liberalization, privatization, and globalization.
if the word given in bold after the blank is your Well- known and well-established corporations
answer i.e. “No change required”. from the West started realizing the __________
Indian Economy holds great significance for its growth (4) of the Indian market and led to the
_________ (1) (conservative) transformation. It establishment of millions of jobs, which boosted
attained its forty-year-long old economic change the Purchasing Power Parity index in India. India
from the Mixed Economy in the year 1991. The reached its peak as the fastest-growing economy
change occurred with the Central Government in its 2000’s with the _________ boost (5) of the
introducing the New Economic Policy (NEP) to IT revolution. With the establishment of the
benefit the economy. internet revolution and remote Technology,
India was fundamentally an agrarian economy millions of jobs were formed at the ITES and IT
when the British government left the country in sectors. India became the nerve center of all the
1947. Most of the trade activities occurred non-core activities of the investment banks and
through selling, cultivating, consuming, and IT giants around the world.
producing livestock and agricultural products. The immense growth of the Indian Economy has
Industrialization slowly began with the visionary improved India’s political and social platform.
leadership of eminent leaders like Jawaharlal 1) Which of the following options best suit the
Nehru, Indira Gandhi, and Rajiv Gandhi. blank (1) in the given passage?
Before the 1990s, most of India’s industries were a. superficial
managed by the government-run-Public sector b. radical
units. India’s Economy hit it big in 1992, which c. extrinsic
created an _________ (efflux) (2) of better d. accidental
economic activities and finance for the people. e. No change required
The reflective and idealistic attributions of
Finance Minister Manmohan Singh led to a 2) Which of the following options best suit the
significant impact on the Indian Economy. blank (2) in the given passage?
a. influx

Click Here For Bundle PDF Course | support@guidely.in Page 1 of 15


SBI Clerk & RRB PO Mains PDF Course 2023
ENGLISH Day - 18

b. approach 6) The leader of the party was accused of


c. outflow playing cat and mouse with his supporters and
d. expansion the media ________________
e. No change required a. by starting a committee to specially study
about the growing unemployment rate in his
3) Which of the following options best suit the constituency.
blank (3) in the given passage? b. by delivering no clarifications on whether he
a. despotic would be resigning his post after being involved
b. autocratic in a controversy.
c. democratic c. by promising to bring in welfare schemes for
d. practical the people of his constituency.
e. No change required d. by supporting the ongoing farmer strike and
ensuring that the demands of the farmers would
4) Which of the following options best suit the be met.
blank (4) in the given passage? e. None of the above.
a. development
b. evolution 7) The textile industry, besides facing an acute
c. potential shortage of raw materials, ____________
d. emergence a. is required to comply with the newly
e. No change required implemented orders by the government to
increase taxes for the sale of textiles within and
5) Which of the following options best suit the outside India.
blank (5) in the given passage? b. is likely to meet up with new suppliers of raw
a. acceleration materials to compensate the shortage.
b. inception c. is come up with a decision to charge higher
c. progress prices for the produced goods.
d. betterment d. is required to report to the government about
e. No correction required the ongoing crisis within the next few weeks.
e. None of the above.
Directions (06-10): Complete the sentences with
the phrases that result in meaningful and 8) The issue of environmental conservation is so
grammatically correct sentences when overwhelming that ________________
combined. a. students from a young age are taught about
how to make the world a better place to live in

Click Here For Bundle PDF Course | support@guidely.in Page 2 of 15


SBI Clerk & RRB PO Mains PDF Course 2023
ENGLISH Day - 18

through small but significant actions like tree been reduced due to the increase in the number
planting, reduced plastic consumption, etc. of vacancies.
b. it has become an integral part of literature and d. Both b and c
research in the times. e. None of the above
c. humans continue to give zero attention to the
environment around them. Directions (11-15): The following passage
d. Both a and b contains 3 blanks. Following the passage is a
e. None of the above table that contains 3 columns. Each column
contains a list of options. The first column has a
9) ____________, considering that the negotiation list of words for the first blank, the second
spanned over 11 months since both sides column for the second blank, and the third
debated strongly to ensure that they settled with column for the third blank. Choose the right set
nothing but what they required and intended. of options for the three blanks from the columns
a. The leader of one of the two countries has given below to complete the passage such that it
withdrawn his attendance midway during the is grammatically and contextually right.
crucial negotiation that would have brought in 11) After refusing to participate in multiple
large amounts of profits to his country negotiations, Pakistan has finally come forward
b. The leaders are yet to reach a unanimous to talk about the ongoing chaos at the regions of
conclusion regarding their collaboration in areas the Indo-Pakistan border. The negotiation was
like agriculture, employment, and tourism. __________ (1) since the leader of Pakistan,
c. The two countries have finally decided on the despite ________ (2) orders to the Pakistan
prospects of the memorandum that is about to be troops to invade the Indian Territory in the first
officially signed in the upcoming week place, refused to accept his mistake and walked
d. Both a and b out of the room midway through the negotiation
e. None of the above with a ________ (3) exit.
Column 1 Column 2 Column 2
10) I was in the low drums ____________ A. successful D. issuing G. fierily
a. after knowing that my admission to the armed B. short-lived E. H. urgent
forces was denied siting that I was medically unfit implementation
to hold such a responsibility. C. resumed F. withdrawing I. dramatic
b. after knowing that my parents have purchased a. A-D-G
a new car for my 20th birthday. b. B-D-I
c. after knowing that the cutoff required for c. C-E-G
admission to government medical colleges has

Click Here For Bundle PDF Course | support@guidely.in Page 3 of 15


SBI Clerk & RRB PO Mains PDF Course 2023
ENGLISH Day - 18

d. A-F-G of the city. This murder has triggered the public


e. B-F-G to question their safety.
Column 1 Column 2 Column 3
12) Reducing the prices of petrol and cooking A. looking D. sparked G. about
gas, which were the highest of all time in the last B. interrogating E. motivated H. now
four years, _________ (1) to be a strategy to C. investigating F. backed I. almost
amass votes from the public in the upcoming a. B-D-G
election. Political experts __________ (2) that the b. C-E-H
leader of the ruling party reduced the cost of the c. A-F-I
said commodities shortly before the upcoming d. B-F-I
election, despite witnessing the people of his e. C-D-G
state ________ (3) with frequent price hikes in the
last four years. 14) The National Youth Mission is set up with the
Column 1 Column 2 Column 3 __________ (1) to unite the youth of today's
A. appears D. complaint G. experience generation for a common cause. The
B. seem E. criticized H. suffer commission aims to _______ (2) the idea of
C. considered F. claimed I. meeting solidarity and oneness amongst today's youth. It
a. A-E-I also plans to achieve increased Indian youth
b. A-E-H __________ (3) on global platforms.
c. B-E-H Column 1 Column 2 Column 3
d. C-D-G A. command D. infuse G. representation
e. B-F-I B. goal E. address H. dominance
C. mandate F. direct I. portrayal
13) The police have been __________ (1) the a. A-E-H
murder case for the last three months. The police b. C-E-I
gathered evidence and recorded testimonies c. B-D-G
from multiple witnesses to identify and locate the d. C-D-G
suspect, but no such luck. This murder has e. A-F-I
__________ (2) the public to question the
efficiency of the police department. It's _______ 15) In 1986, a nuclear reactor at the Chernobyl
(3) time the public gets clear information about Nuclear power plant in the erstwhile Soviet Union
the criminal and the murder that was committed was __________ (1) to improper testing at
during broad daylight in one of the busiest parts unfavorable conditions resulting in loss of control.

Click Here For Bundle PDF Course | support@guidely.in Page 4 of 15


SBI Clerk & RRB PO Mains PDF Course 2023
ENGLISH Day - 18

The _______ (2) fire and explosion destroyed the a. A


building where the reactor was kept, which led to b. B
the release of radiation into the atmosphere. It is c. C
said that uranium fuel in the reactor overheated d. D
and melted through the ________ (3) barriers e. All words are appropriate
since the safety measures were ignored.
Column 1 Column 2 Column 3 18) To my horror (A), I realized (B) at the
A. run D. ensuing G. metal restaurant that I forgot (C) to carry my wallet, but
B. induced E. incoming H. protective my friend offered (D) to help me with the bill at
C. subjected F. developing I. created the right time.
a. B-E-I a. A
b. C-D-H b. B
c. B-F-H c. C
d. A-F-G d. D
e. C-E-H e. All words are appropriate

Directions (16-20): In the following sentence, 19) On (A) hindsight, Meena should have
four words are given in bold, out of which one of accepted (B) her last job offer since she is
them is contextually or grammatically currently finding (C) it hard to make ends meet
inappropriate. Find the inappropriate word (D) due to a lack of steady income.
16) The singer was deceived (A) by his a. A
unscrupulous (B) manager, who altered (C) the b. B
financial records of the former to loot (D) money. c. C
a. A d. D
b. B e. All words are appropriate
c. C
d. D 20) Suja panicked (A) and rushed out of her flat
e. All words are appropriate after she woke (B) up to the sound of a fire alarm
going (C) on (D) in the basement of her building.
17) The rapid (A) falling economic figures a. A
indicate (B) that the earning sectors of the b. B
country are not performing (C) well, specifically c. C
post the pandemic that broke (D) out in 2019. d. D
e. All words are appropriate

Click Here For Bundle PDF Course | support@guidely.in Page 5 of 15


SBI Clerk & RRB PO Mains PDF Course 2023
ENGLISH Day - 18

Click Here to Get the Detailed Video Solution for the above given Questions
Or Scan the QR Code to Get the Detailed Video Solutions

Answer Key with Explanation

1) Answer: B significant and essential in benefitting the Indian


The first paragraph of the passage states that economy. Thus, options a and c are incorrect.
the Indian economy changed with the The word ‘radical’ (adjective) describes
implementation of the New Economic policy. The something that highly favors change and breaks
change is described to be of great significance. the usual customs to achieve something great.
The change is also said to have occurred after This word best describes the transformation that
40 years. Thus, the transformation of the Indian has been mentioned in the first paragraph of the
economy should have broken the routine and passage. Thus, option b is correct.
changed drastically to hit historical numbers with The word ‘accidental’ (adjective) means
the implementation of the New Economic policy. something that has occurred unplanned. The
The word conservative (adjective) means transformation described here was not
resistant to change. This cannot fit the blank accidental and out of the blue since the central
because the transformation witnessed a forty- government planned to implement the national
year old long passage according to the first economic policy to bring about this change.
paragraph of the passage. Thus, the word Thus, option d is incorrect.
‘conservative’ is incorrect.
The word ‘extrinsic’ (adjective) denotes 2) Answer: A
something that is not essential. Also, the word The third paragraph of the passage said that the
‘superficial’ (adjective) describes something that Indian economy hit big after the implementation
is of less importance. These words do not fit the of the New Economic Policy in 1992. This means
blank because the transformation is seen to be that the country witnessed an increase in its

Click Here For Bundle PDF Course | support@guidely.in Page 6 of 15


SBI Clerk & RRB PO Mains PDF Course 2023
ENGLISH Day - 18

economy. The country is also described to have The word after the blank – growth (adjective) and
witnessed better economic activities and finance. options a – development (adjective) and
The word ‘efflux’ (noun) denotes something that evolution (adjective) indicate the process of
is flowing out and escaping. This does not fit the becoming larger and larger. The words may fit
context of the passage since the escaping of the blank right grammatically. But the context of
economic activities and finance should have led the passage speaks about how organizations
to economic losses. Thus, the word is incorrect. from the west witnessed and realized that the
Also, option c - ‘outflow’ (noun) denotes Indian market is doing well. But it does not make
something that is escaping. Again, this does not sense for the organizations from the west to
fit the context of the passage. Thus, option c is realize how the Indian market has grown,
incorrect. evolved, or developed since they are not going
to be benefitted just from realizing the growth of
3) Answer: C the market in the country. Thus, the word after
The word after the blank – tyrannical (adjective), the blank and options a and c are incorrect.
autocratic (adjective), and despotic (adjective) The same goes for the word ‘emergence’ as
are synonymous to each other. They mean a well. The word emergence indicates an advent,
country or a system that is ruled by a single onset, beginning of something. The
individual with inherited authority. This does not organizations from the west do not get benefitted
fit the system that prevails in India. Thus, the by realizing the emergence of the Indian market.
word after the blank and options a and b are Thus, option d is incorrect.
incorrect. The word potential (adjective) indicates the
The word democratic (adjective) means a something’s abilities or qualities that help with
system that is ruled by the representatives who succeeding or achieving goals. This fit the
are elected by its citizens. This is the system that context of the passage since the organizations,
prevails in India. Thus, option c is correct. upon realizing the potential of the Indian market,
The word practical (adjective) means realistic. can understand how beneficial it could turn out
Also, this word may fit the sentence to be when choosing to associate with the Indian
grammatically, it is less suitable for the blank market. Thus, option c is correct.
when compared to the word democratic. Thus,
option d is incorrect. 5) Answer: B
The sentence after the blank speaks about the
4) Answer: C establishment (which means to form or build
something) of internet revolution and remote

Click Here For Bundle PDF Course | support@guidely.in Page 7 of 15


SBI Clerk & RRB PO Mains PDF Course 2023
ENGLISH Day - 18

technology that led to increased employment schemes. Option d speaks about how the leader
opportunities of the IT sector. From this it can be supported the farmer-strike and decided to pay
inferred that the IT Revolution has just been heed to the demands of the customers. These
established. options do not elaborate on how the leader tried
The word after the blank – boost (verb/noun) to confuse or mislead someone. Thus, these
means to contribute to the progress of options are incorrect.
something that has already been established. Option b speaks about how the leader has
Thus, the word after the blank is incorrect. refrained to open up about his resignation after
The option a - acceleration (noun) means to being involved in a controversy. The leader
cause something to occur faster. Option c – seems to evade or trick his supporters and the
progress means the development of something. media by not speaking about his resignation.
Option d – betterment means an improvement. This suits the context of the passage. Thus,
All these words indicate the development of option b is correct.
something that has already been established. The right sentence is:
Thus, options a, d, and c is incorrect. The leader of the party was accused of playing
The word inception (noun) indicates the cat and mouse with his supporters and the
beginning or start of something. This word fits media by delivering no clarifications on whether
the context of the passage because it clearly he would be resigning his post after being
indicates and matches the context of the line involved in a controversy.
after the blank. Thus, option b is correct.
7) Answer: A
6) Answer: B The sentence in the passage end with a not that
The sentence in the question uses the idiom – ‘to indicates that the textile industry is facing
play cat and mouse’ which means to do or say another major challenge in addition to facing a
something to deliberately deceive, mislead, shortage of raw materials. The use of the word
confuse, or control something/someone. Thus, besides state that there is another major
the blank should be filled with sentences that challenge faced by the textile industry apart from
convey that the leader tried to confuse or toy the one discussed in the question.
with something/someone. Option a speaks about a challenge wherein the
Option a speaks about how the leader is textile industry is obligated to pay increased
planning to set up a committee to study about taxes for the textiles in and outside India. This is
unemployment. Option c speaks about how the definitely concerning for the textile industry.
leader has planned to introduce welfare Thus, option a is right.

Click Here For Bundle PDF Course | support@guidely.in Page 8 of 15


SBI Clerk & RRB PO Mains PDF Course 2023
ENGLISH Day - 18

Option b speaks about the possibility of reaching become so intense that it has been brought to
out to new suppliers of raw materials to make up the notice of the students and included in their
for the shortage. This statement is not a curriculum to bring in changes. Thus, option a is
challenge but a resolution to the ongoing crisis of correct.
raw material shortage. Also, option c speaks Option b speaks about how this issue is
about how the textile industry has come up with becoming a part of literature and research
a decision to sell textiles at higher prices. This recently. The issue has become so intense that
does not concern the textile industry but authors/poets and researchers are finding ways
provides an alternative to make more money. to get this issue incorporated in their line of work
Thus, options b and c are incorrect. and bring awareness to the people. Thus, option
Option c speaks about how the textile industry is b is correct.
required to report to the government about the Option c speaks about how humans continue to
ongoing crisis. This is not a challenge. Moreover, give zero attention to the environment around
this could be helpful since the government can them. This does not fit the context of the
recommend resolutions for the ongoing crisis. sentence become an overwhelming issue that is
Thus, option d is incorrect. severely disturbing would not bring in this
The right answer is: reaction from the people. Thus, option c is
The textile industry, besides facing an acute incorrect.
shortage of raw materials, is required to comply Thus, options a and b are right.
with the newly implemented orders by the The right sentence is:
government to increase taxes for the sale of The issue of environmental conservation is so
textiles within and outside India. overwhelming that students from a young age
are taught about how to make the world a better
8) Answer: B place to live in through small but significant
The sentence contains the word overwhelming actions like tree planting, reduced plastic
which means to feel something strongly that it consumption, etc.
becomes irresistible. The sentence speaks about (or)
environmental conservation and how much this The issue of environmental conservation is so
issue has grown to become irresistible or made overwhelming that it has become an integral part
people worry to eventually give rise to an event of literature and research in the times.
that should necessarily fill the blank.
Option a speaks about children being taught 9) Answer: C
about environment conservation. The issue has

Click Here For Bundle PDF Course | support@guidely.in Page 9 of 15


SBI Clerk & RRB PO Mains PDF Course 2023
ENGLISH Day - 18

The sentence after the blank states that the The first part of the sentence includes an idiom –
negotiation spanned over 11 months. Also, it is ‘in the lowdrums’ which means to be depressed
given that the negotiation was successful and and low spirited. Thus, options b and c which
that both the countries debated strongly to speak about elated and joyful events do not
achieve what they intended. Thus, last part of match the blank.
the sentence indicates that the countries Option c indicates how the speaker was denied
debated till the achieved what they intended. admission to the armed forces due to lack of
Option a speaks about how one of the leaders medical fitness. This could have probably put the
withdrew his participation in the negotiation speaker in low spirit. Thus, option a is correct.
midway. This signifies that the negotiation is The right sentence is:
unsuccessful. Thus, option a is incorrect. I was in the lowdrums after knowing that my
Option b speaks about how the two leaders are admission to the armed forces was denied siting
yet to reach a unanimous conclusion regarding that I was medically unfit to hold such a
their collaboration. This signifies that the responsibility.
negotiation is unsuccessful. Thus, option b is
incorrect. 11) Answer: B
Option c speaks about how the leaders have The part of the sentence with the first blank
finally decided the prospects of the speaks about how the leader of Pakistan walked
memorandum and how they are about to sign out of the room during the negotiation. Thus, the
this memorandum officially next week. This negotiation is seen to have ended non-
signifies that the negotiation is successful. Thus, successfully. Thus, the blank (1) should indicate
option c is correct. the how the negotiation ended before reaching
The right sentence is: final conclusion.
The two countries have finally decided on the The word resumed (verb) means to begin
prospects of the memorandum that is about to something that has been paused for quite some
be officially signed in the upcoming week, time. Thus, the word resumed in incorrect since
considering that the negotiation spanned over 11 it does not fit the context of the passage.
months since both sides debated strongly to The word successful (adjective) indicates a
ensure that they settled with nothing but what favorable outcome. This word does not fit the
they required and intended. context of the passage.
The word short-lived (adjective) denotes
10) Answer: A something that did not extend over a long time.

Click Here For Bundle PDF Course | support@guidely.in Page 10 of 15


SBI Clerk & RRB PO Mains PDF Course 2023
ENGLISH Day - 18

This word correctly fit the context the passage. negotiation. Thus, the word dramatic better fits
Thus, B is correct. the context of the passage.
The word ‘implementation’ means to bring The right passage is:
something to effect and it is a noun. The word After refusing to participate in multiple
does not fit the blank grammatically because this negotiations, Pakistan has finally come forward
word should be followed by the preposition ‘of’ to talk about the ongoing chaos at the regions of
mandatorily. Thus, the word is incorrect. the Indo-Pakistan border. The negotiation was
The word ‘withdrawing’ means to take back short-lived since the leader of Pakistan, despite
something that is said. The context of the issuing orders to the Pakistan troops to invade
passage speaks about how the leader was at the Indian Territory in the first place, refused to
fault for ordering the Pakistan troops to invade accept his mistake and walked out of the room
the Indian Territory. If he had withdrawn, he midway through the negotiation with a dramatic
would not have been blamed for causing the exit.
ongoing chaos. Thus, this word is contextually
incorrect. 12) Answer: B
The word ‘issuing’ means to put forth something The word in blank (1) should be a singular verb
officially. This word fits the sentence contextually to adhere to the requirements of subject-verb
and grammatically. Thus, the word issuing is agreement. The subject is singular since it talks
correct. about the act of reducing the prices of petrol and
The third blank should describe the word ‘exit’ cooking gas. Thus, the word ‘seem’ is correct.
which is a noun. Thus, the word in the blank The word ‘appears’ fit the blank grammatically
should be an adjective. The word fierily is an and contextually. Thus, this word is correct.
adverb. Thus, it does not fit the blank. The word considered should be followed by the
The words urgent and dramatic are adjectives word ‘is’ to make the sentence grammatically
and thus, both are grammatically correct. But correct. Thus, this word is incorrect.
according to the passage, the word dramatic The word in blank 2 should include a verb that
better suits the blank than urgent since the should signify the action of the political experts.
passage does not speak explicitly about the The word ‘complaint’ is a noun. Thus, this word
leader of Pakistan having to leave the is incorrect.
negotiation immediately due to several other The words ‘criticized’ and ‘claimed’ are both
circumstances. The leader was at fault for verbs. They fit the passage grammatically and
causing the chaos, yet he did not accept his fault contextually. But the word criticized fit the
but walked out of the room midway through the passage better when compared to claimed

Click Here For Bundle PDF Course | support@guidely.in Page 11 of 15


SBI Clerk & RRB PO Mains PDF Course 2023
ENGLISH Day - 18

because the political experts are pointing out the ‘into’ to make sense and make the sentence
wrongdoings of the leader of the ruling party. grammatically correct.
Thus, the word criticized is correct. The word ‘interrogating’ does not fit the context
The word ‘experience’ does not fit the passage of the passage. Because ‘interrogating’ means to
grammatically. The word experience need not be pose a number of questions to a person. Thus,
followed by prepositions. The sentence ‘people the word is incorrect.
of his state experience with’ is wrong. The The word ‘investigating’ means to examine and
sentence should have been right when it is study about something. This word fits the
framed as ‘people of the state experience sentence grammatically and contextually. The
frequent’. Thus, this word is incorrect. sentence with the word investigating would
The word ‘meeting’ does not fit the passage. The speak about how the police examined and
sentence ‘people of his state meeting with’ is studied about the developments of the case.
incorrect. The word in the blank should not be a Thus, this word is correct.
gerund or present participle. Thus, the word For the second blank, the words motivated and
‘meeting’ which is in the verb in the gerund form backed are incorrect. The words backed and
is incorrect. motivated means to provide support for the
The blank 3 should be filled with a verb of the growth and approval of something. These words
simple present tense in plural form. Thus, the convey meanings in a positive sense. Thus,
word suffer is correct. these words are incorrect.
The right passage is: The word sparked means to initiate something in
Reducing the prices of petrol and cooking gas, a negative way. When inserted, the sentence
which were the highest of all time in the last four would convey that the murder triggered the
years, appears to be a strategy to amass votes public to question the efficiency of the police
from the public in the upcoming election. Political department. Thus, the word sparked is correct.
experts criticized that the leader of the ruling For the third blank, the word about fits the
party reduced the cost of the said commodities sentence grammatically and contextually. Also,
shortly before the upcoming election, despite the phrase ‘It’s about time’ is an idiom which
witnessing the people of his state suffer with means that the time has passed for something to
frequent price hikes in the last four years. happen. Thus, the words ‘now’ and ‘almost’ are
incorrect.
13) Answer: E Here, the first part of the sentence says that the
The word ‘looking’ does not fit the first blank. police have been looking into this murder case
Since it should be followed by the preposition for the last three months but are yet to find the

Click Here For Bundle PDF Course | support@guidely.in Page 12 of 15


SBI Clerk & RRB PO Mains PDF Course 2023
ENGLISH Day - 18

suspect. The public would have expected that implanting ideas like solidarity and oneness
the police would have revealed the details of the amongst Indian youth. Thus, the word infuse fits
murder sooner. Thus, the word ‘about’ is correct. blank 2.
The right passage is: The word dominance does not fit blank 3
The police have been investigating the murder because dominance means to have a strong
case for the last three months. The police control over something. This is conceptually
gathered evidence and recorded testimonies wrong because global platforms should not
from multiple witnesses to identify and locate the promote dominance or control. Thus, the word
suspect, but no such luck. This murder has dominance does not fit the passage. Also, the
sparked the public to question the efficiency of word portrayal does not fit blank 3 because
the police department. It's about time the public portrayal is the act of depicting
gets clear information about the criminal and the something/someone. It does not make sense to
murder that was committed during broad daylight say that global platforms should have increased
in one of the busiest parts of the city. Indian youth portrayal. Thus, the word portrayal
is incorrect. The word representation means to
14) Answer: C stand as representatives for
The word ‘command’ means to issue instructions something/someone. This word best suits blank
in a stern way. Also, the word ‘mandate’ means 3. Thus, the word ‘portrayal’ is correct.
to give someone the authority to act in a certain The right passage is:
way. These words do not fit the first blank The National Youth Mission is set up with the
because these words express stern behavior. goal to unite the youth of today's generation for a
The word that should be included here should be common cause. The commission aims to infuse
more like a motive or intention behind setting up the idea of solidarity and oneness amongst
the commission. Thus, the word goal fits blank 1. today's youth. It also plans to achieve increased
The word ‘address’ means to give attention to Indian youth representation on global platforms.
something. The committee would not get
benefitted by just giving attention to the ideas 15) Answer: B
mentioned above. Also, the word ‘direct’ means The word run (which means to operate
to intend or instruct something to go in a certain something) does not fit the first blank because it
way. The commission cannot direct ideas. Thus, does not make sense to say that the nuclear
these words are incurred. The word ‘infuse’ reactor was run to improper testing. Thus, the
means to implant something. It makes better word run is incorrect. The word induced means
sense to say that the commission focuses on to cause something. It does not make sense to

Click Here For Bundle PDF Course | support@guidely.in Page 13 of 15


SBI Clerk & RRB PO Mains PDF Course 2023
ENGLISH Day - 18

say that the nuclear reactor was induced to the atmosphere. It is said that uranium fuel in the
improper testing. But the word subjected, which reactor overheated and melted through the
means to make something/someone experience protective barriers since the safety measures
a certain action, best fits the context of the were ignored.
passage resulting in a sentence that speaks
about how the nuclear reactor was made to 16) Answer: E
experience improper testing. Thus, the word All the words fit the sentence grammatically and
subjected is correct. contextually. Thus, no change is required and
The word ensuing indicates something that the given sentence is correct by itself.
follows immediately or as a result of something.
The fire was caused due to the destruction of the 17) Answer: A
reactor and thus, the word ensuing best fits the The word marked A is position to describe the
context of the passage. The words ‘incoming’ word falling which is a verb. Thus, the word
and ‘developing’ does not fit blank 2 although marked A should be an adverb. But rapid is an
they fit the passage grammatically. The context adjective. Hence the right word should be
of the passage is altered while inserting rapidly. The other words B, C, and D are correct.
incoming or developing in blank 2. Thus, the The right sentence is:
word ensuing is correct. The rapidly falling economic figures indicate that
Blank 3 should include an adjective since it has the earning sectors of the country are not
to describe barriers which is a noun. The word performing well, specifically post the pandemic
metal is a noun and the word created is a verb in that broke out in 2019.
the past tense. Thus, they are incorrect. The
word protective is an adjective and fits the 18) Answer: C
passage grammatically and contextually. Thus, The sentence talks about incidents that
the word protective is incorrect. happened in the past. If there are two or more
The right passage is: sentences from the past, the event that
In 1986, a nuclear reactor at the Chernobyl happened first should be described in past
Nuclear power plant in the erstwhile Soviet perfect sense. Thus, forgot (simple past) should
Union was subjected to improper testing at be replaced with had forgotten (past perfect).
unfavorable conditions resulting in loss of The others words A, B, and D are right.
control. The ensuing fire and explosion The right sentence is:
destroyed the building where the reactor was
kept, which led to the release of radiation into

Click Here For Bundle PDF Course | support@guidely.in Page 14 of 15


SBI Clerk & RRB PO Mains PDF Course 2023
ENGLISH Day - 18

To my horror, I realized at the restaurant that I to make ends meet due to a lack of steady
had forgotten to carry my wallet, but my friend income.
offered to help me with the bill at the right time. 20) Answer: D
The right answer is option d. The phrase ‘alarm
19) Answer: A goes off’ is used to describe the sound that
The word hindsight is commonly preceded by comes when the alarm begins to ring at the
the preposition ‘in’ and not ‘on’. Thus, A is specified time. Thus, the use of the preposition
incorrect. The other words B,C, and D are ‘on’ is incorrect here. The other words A, B, and
grammatically and contextually right. C are right.
The right sentence is” The right sentence is:
In hindsight, Meena should have accepted her Suja panicked and rushed out of her flat after
last job offer since she is currently finding it hard she woke up to the sound of a fire alarm going
off in the basement of her building.

Click Here For Bundle PDF Course | support@guidely.in Page 15 of 15


SBI Clerk & RRB PO Mains PDF Course 2023
Reasoning Ability Day - 19 (Eng)

Reasoning Ability
Directions (1-5): Study the following information among the following persons uploaded the
carefully and answer the given questions. project in the same month?
Eight persons - A, B, C, D, E, F, G and H a) Only D
uploaded their projects online in eight different b) No one
months of the year 2022. Four of the months are c) D and G
January, February, September and October. d) Only G
B uploaded the project four months before D, e) Only F
where both of them uploaded the project in the
month having an odd number of days. Only one
person uploaded the project between D and A, 3. Who among the following persons uploaded
who uploaded in the month having the number of immediately before or immediately after A?
days which is a multiple of five. A didn’t upload I) H
the project before June. The number of persons II) The one who uploaded in July
uploaded the project before B is one less than III) G
the number of persons uploaded the project after IV) The one who uploaded in August
F. C uploaded the project before F, where both of a) Only I and II
them uploaded the project in the month having b) Only II and IV
the same number of days. As many persons c) Only I and III
uploaded the project before C as after E. G didn’t d) Only III and IV
upload the project in the month having an even e) Only I and IV
number of days.
1. Four of the following five are alike in a certain 4. Who among the following person uploaded the
way based on the given arrangement and thus project in the month having the least number of
form a group. Which one of the following does days?
not belong to the group? a) A
a) C and The one who uploaded in February b) C
b) The one who uploaded in September and G c) The one who uploaded immediately after G
c) The one who uploaded in August and D d) D
d) F and The one who uploaded in July e) The one who uploaded immediately before B
e) B and The one who uploaded in January
5. How many persons uploaded the project
2. If all persons uploaded the project in between D and E?
alphabetical order from January, then who a) As many persons uploaded after F

Click Here For Bundle PDF Course | support@guidely.in Page 1 of 11


SBI Clerk & RRB PO Mains PDF Course 2023
Reasoning Ability Day - 19 (Eng)

b) One whereas the persons living on the odd numbered


c) None floors move upward.
d) As many persons uploaded between E and F IV) Each person is allowed to throw the dice
e) Four twice and the sum of the two outcomes of the
dice is the number they need to move, i.e., if a
Directions (6-10): Study the following information person gets the outcome as (6, 4), then that
carefully and answer the given questions. person needs to more 10 steps either up or
Nine persons – A, B, C, D, E, F, G, H and I are down.
living on different floors of a nine-storey building V) All the persons throw the dice in the reverse
where the lowermost floor is numbered one and alphabetical order. After the final arrangement,
the floor immediately above it is numbered two more than one person may or may not stand on
and so on. No two persons live on the same the same step.
floor. VI) The following shows the final outcome of the
H lives on a prime numbered floor. Only four dice thrown by all the persons in the reverse
persons live between H and A. The number of alphabetical order respectively from the left side.
floors below A is two less than the number of (1, 5), (3, 4), (6, 2), (2, 2), (5, 2), (6, 5), (1, 1), (4,
floors above F. D lives two floors below G. As 2), (1, 4)
many floors above D as below C. More than two 6. Who among the following person stands on
persons live between B and C. I neither lives the same staircase along with I (after the
below B nor lives on an even numbered floor. rearrangement of all the persons)?
Based on the below conditions they are standing a) The one who lives on the second floor
on the steps: b) D
Special conditions: c) The one who lives immediately above F
I) There are 40 staircases in the building in which d) G
each person stands on different steps. The e) No one
lowermost step is numbered 1 and the step
immediately above it is numbered 2 and so on. 7. What is the average of the staircase number in
II) Initially, the persons are standing in the steps which G, F and H stand (after the rearrangement
which are the multiples of 4 i.e., the person in the of all the persons)?
lowermost floor stands on step 4 and so on. Only a) 48
one person stands on each step. b) 18
III) All the persons move from their respective c) 25
steps based on the dice. The persons living on d) 100
the even numbered floors move downward e) 82

Click Here For Bundle PDF Course | support@guidely.in Page 2 of 11


SBI Clerk & RRB PO Mains PDF Course 2023
Reasoning Ability Day - 19 (Eng)

Ten persons –L, M, N, O, P, Q, R, S, T and U are


8. Who among the following person lives four sitting around a circular table facing the centre.
floors below F? The circumference of the circle is 440cm. The
a) The one who lives immediately above D distance between adjacent persons is equal.
b) E O sits 88m away from S. L sits 44m to the right of
c) The one who lives two floors above B O. The sum of the distance between L and S is
d) B the same as the sum of the distance between Q
e) H and M. Q sits 44m to the right of R. The sum of
the distance between R and M is the same as
9. How many persons are standing at an odd the sum of the distance between S and U, when
prime numbered step (after movement)? counted from the right of R and left of U. T sits
a) As many persons live above D 264m away from U. N sits 44m away from T.
b) Three Both P and O are not immediate neighbours.
c) As many persons live between I and E 11. Who among the following person sits 220m
d) As many persons live below A to the left of P?
e) No one a) The one who sits 44m to the right of U
b) S
10. Which of the following statements is/are true c) The one who sits 88m to the right of Q
with respect to the final arrangement? d) M
I. Both G and F stand on the same staircase e) None of these
II. No one stands above staircase 35
III. The difference between the staircase number 12. What is the distance between N and Q?
of C and D is 23 a) 44m
IV. There is an even number of staircases b) 110m
between E and D c) 88m
a) Only I, II and IV d) 55m
b) Only I and III e) 132m
c) Only I, II and III
d) Only III and IV 13. How many persons are sitting between O
e) All I, II, III and IV and P, when counted from the right of O?
a) Two
Directions (11-15): Study the following b) Three
information carefully and answer the given c) Four
questions. d) Five

Click Here For Bundle PDF Course | support@guidely.in Page 3 of 11


SBI Clerk & RRB PO Mains PDF Course 2023
Reasoning Ability Day - 19 (Eng)

e) Six % means either the hour or minute hand of the


clock is at 9
14. Who among the following person sits exactly ! means either the hour or minute hand of the
between T and U? clock is at 1
a) The one who sits immediate right of R > means either the hour or minute hand of the
b) The one who sits 44m to the right of M clock is at 3
c) The one who sits 220m to the left of Q ? means either the hour or minute hand of the
d) Both a and c clock is at 2
e) Both a and b # means either the hour or minute hand of the
clock is at 8
15. Which among the following statement(s) < means either the hour or minute hand of the
is/are true as per the given arrangement? clock is at 5
I. M sits 88m to the left of U ^ means PM
II. Both N and L are not immediate neighbours ϖ means AM
III. S sits 264m to the right of N Note: If two symbols are given, then the first
a) Only I and III symbol will be considered as the hour hand and
b) Only II the second symbol will be considered as the
c) Only I and II minute hand.
d) Only III For example: #> ϖ ->8:15 AM
e) All I, II and III 16. The distance between Alex’s college to his
native is 360km. He started his journey from the
Directions (16-20): Study the following college by his bike at a speed of 50km/hr at
information carefully and answer the given “$@^”, then at what time did he reach his native
questions. (approximately)?
The following symbols represent time in a clock: a) %!ϖ
& means either the hour or minute hand of the b) ?$^
clock is at 12 c) <#^
@ means either the hour or minute hand of the d) !$ϖ
clock is at 4 e) None of these
$ means either the hour or minute hand of the
clock is at 6 17. Ram books in Rapido from his house to go to
* means either the hour or minute hand of the the hospital and person X from Rapido took a
clock is at 11 bike to pick up Ram at &<^ and it takes 120
minutes to reach Ram’s house but Ram went to

Click Here For Bundle PDF Course | support@guidely.in Page 4 of 11


SBI Clerk & RRB PO Mains PDF Course 2023
Reasoning Ability Day - 19 (Eng)

a medical shop, then at what time ram should 19. If A and B together can do a piece of work in
reach his house if he wants to reach there 5 8 hours and A alone can do a piece of work in 16
minutes before X’s reaching time? hours and B starts to do the work alone at
a) ?*ϖ “@>ϖ”, then at what time did B alone will
b) ?<^ complete his/her work?
c) <@ϖ a) #>^
d) #!^ b) @*ϖ
e) ?@^ c) $%^
d) %@ϖ
18. Mary went to the tuition centre, she spends e) <@^
“?” hours for writing, “!” hour for reading, “!” for
test and 30 minutes for snacks time and then she 20. The bus departed at “ * $^ ”, it takes 3 hours
will reach her father’s office at #$^, if she takes 50 minutes(without break) to reach the
“!” hour for travelling from tuition centre to her destination, if the bus takes 60 minutes break in
father’s office, then at what time she went to the between the journey, then at what time it will
tuition centre? reach the destination?
a) *&ϖ a) @$ϖ
b) >&^ b) $!^
c) ?$^ c) @@ϖ
d) >$^ d) <*^
e) <%ϖ e) Cannot be determined
Click Here to Get the Detailed Video Solution for the above given Questions
Or Scan the QR Code to Get the Detailed Video Solutions

Answer Key with Explanation

Directions (1-5):

Click Here For Bundle PDF Course | support@guidely.in Page 5 of 11


SBI Clerk & RRB PO Mains PDF Course 2023
Reasoning Ability Day - 19 (Eng)

1) Answer: E(no person uploaded between the


given month and person except option e)
2) Answer: C
3) Answer: D
4) Answer: E
5) Answer: A
Final arrangement:

 The number of persons uploaded the


project before B is one less than the number of
persons uploaded the project after F.

We have,
 B uploaded the project four months before
D, where both of them uploaded the
project in the month having an odd
number of days.
 Only one person uploaded the project
between D and A, who uploaded in the
month having the number of days which is  C uploaded the project before F, where
a multiple of five. both of them uploaded in the month
 A didn’t upload the project before June. having the same number of days.
 As many persons uploaded the project
before C as after E.
 G didn’t upload the project in the month
having an even number of days.
After applying the above conditions, case-1 and
case-2 get eliminated because there are not the
same number of persons uploaded their project
before and after C and E and F can’t be in any
month which has the same number of days as

Click Here For Bundle PDF Course | support@guidely.in Page 6 of 11


SBI Clerk & RRB PO Mains PDF Course 2023
Reasoning Ability Day - 19 (Eng)

that of C respectively. Hence case-3 gives the


final arrangement.

Directions (6-10):
6) Answer: E
7) Answer: C
G and F stand in step 20, H stands in step 35
20+20+35=75/3=25
8) Answer: A
9) Answer: D
10) Answer: B
Final arrangement

Click Here For Bundle PDF Course | support@guidely.in Page 7 of 11


SBI Clerk & RRB PO Mains PDF Course 2023
Reasoning Ability Day - 19 (Eng)

Again, we have
 The number of floors below A is two less
than the number of floors above F.
 D lives two floors below G.
 As many floors above D as below C.
After applying the above conditions case-2 gets
eliminated because there is no possibility to
place F.

We have,
 H lives on a prime numbered floor.
 Only four persons live between H and A.
From the above conditions, there are three Again, we have
possibilities  More than two persons live between B
and C.
 I neither lives below B nor lives on an
even numbered floor.

Click Here For Bundle PDF Course | support@guidely.in Page 8 of 11


SBI Clerk & RRB PO Mains PDF Course 2023
Reasoning Ability Day - 19 (Eng)

After applying the above conditions case-1 gets


eliminated because there is no possibility to
place I, hence case-3 shows the final
arrangement.

Special conditions:
Before arrangement, the position of each person
on the staircase is as follows.

The following shows the final outcome of the


dice thrown by all the persons in the reverse
alphabetical order respectively.
I = (1, 5) =6 steps Upward, H = (3, 4) =7 steps
Upward, G = (6, 2) =8 steps Upward, F = (2, 2) =
4 steps downward, E = (5, 2) =7 steps
downward, D = (6, 5) =11 steps Upward, C = (1,
1) =2 steps Upward, B = (4, 2) =6 steps
downward, A = (1, 4) =5 steps downward.
After arrangement, the position of each person in
the staircase is as follows.

Click Here For Bundle PDF Course | support@guidely.in Page 9 of 11


SBI Clerk & RRB PO Mains PDF Course 2023
Reasoning Ability Day - 19 (Eng)

11) Answer: C
12) Answer: E
13) Answer: A
14) Answer: D
15) Answer: B
Final arrangement

We have
 O sits 88m away from S.
 L sits 44m to the right of O.
From the above conditions, there are two
possibilities

Again, we have
 The sum of the distance between L and S
is the same as the sum of the distance
between Q and M.
 Q sits 44m to the right of R.
 The sum of the distance between the
persons R and M is the same as the sum
of the distance between S and U, when
counted from the right of R and left of U.
Directions (11-15):  T sits 264m away from U.

Click Here For Bundle PDF Course | support@guidely.in Page 10 of 11


SBI Clerk & RRB PO Mains PDF Course 2023
Reasoning Ability Day - 19 (Eng)

17) Answer: E
Person X took a bike to pick up Ram at &<^ -
>12:25 PM and it takes 120 minutes to reach
Ram’s house, which means person X reached
Ram’s house at 2:25 PM (?<^)
As Ram has to reach his location from the
Again, we have
medical shop 5 minutes before X’s reaching
 N sits 44m away from T.
time, then the reaching time would be ?@^ (2:20
 Both P and O are not immediate
PM)
neighbours.
After applying the above conditions case-1 gets
18) Answer: B
eliminated because both P and O are immediate
Mary went to his father’s office at #$^ (8:30 PM)
neighbours, hence case-2 shows the final
She spends 2 hrs for writing, 1 hr for reading, 1
arrangement.
hr for test, 30 minutes for snacks time and then 1
hr for travelling.
8:30- (2+1+1+1+30 minutes) =8:30-5:30 ->3:00
PM (>&^)

19) Answer: A
A+B=1/8 ->1/16+1/X=1/8
16) Answer: D
1/x=1/16-1/8=> 16 hours
Distance=360km
B starts at 4:15 AM and then finished at 8:15 PM
Speed=50
(#>^)
Time=Distance/speed=360/50=7.2 hours
7 hours 12 minutes
20) Answer: C
Start time = $@^ =6:20 PM
The bus starts at 11:30 PM
Then Alex reached his native at ->1:32 AM
Reach the destination = (11:30+3.50+1) =4:20
(approximately 1:30 AM)
!$ϖ ->1:30 AM AM ->@@ϖ

Click Here For Bundle PDF Course | support@guidely.in Page 11 of 11


SBI Clerk & RRB PO Mains PDF Course 2023
Quantitative Aptitude Day - 19 (Eng)

Quantitative Aptitude

Directions (1- 3): Study the following information the given selling price, find the difference
carefully and answer the questions given below. between the new and old profits of the
A fruit seller buys different quantity of apples in shopkeeper on Tuesday?
three days [Monday, Tuesday, and Wednesday] a) Rs.43
in the ratio of 5:3:4 and buys different quantity of b) Rs.40
oranges in the ratio of 4:3:1 respectively. Cost c) Rs.48
price per kg of apple on Monday, Tuesday, and d) Rs.45
Wednesday is Rs.40, Rs.60 and Rs. 75 e) Rs.42
respectively. Cost price per kg of orange on
Monday is Rs.10 less than the price of per kg of 3) Find the overall profit percentage of
apple on Monday. Ratio of cost price per kg of shopkeepers on Wednesday?
orange on Monday to that on Tuesday is 3:4 and a) 21.30%
the cost price per kg of Orange on Wednesday is b) 22.35%
Rs.50. Selling price Per kg of orange on Monday, c) 28.37%
Tuesday and Wednesday are Rs.50, Rs.50, and d) 29.31%
Rs.70, respectively and the selling price of per kg e) None of these
of apple on Monday, Tuesday and Wednesday is
Rs.60, 80 and Rs. 90 respectively. On Tuesday Directions (4 -6): Study the following information
the shopkeeper buys 15 kg apples and 12 kg carefully and answer the questions given below.
oranges. Train A covered 180 km in 2.5 hours and Train B
1) Find the difference in the profit percentage of covered the same distance in 2 hours. Train A
shopkeepers earned by selling all oranges and and train B start their journey from P and Q
by selling all apples on Monday? respectively towards Q and P respectively at the
a) 12.35% same time. The distance between P and Q is
b) 18.35% 324 km. They Meet after t hours. Train C starts
c) 16.66% the journey from P after t hours of train A starting
d) 17.34% his journey and takes 6 hours to reach Q. Train A
e) 19.35% cross a 160 m bridge in 20 sec and train C cross
a 40m bridge in 24 sec. Ratio of the length of
2) If the shopkeeper sold all oranges on Tuesday train A to that of train B is 12:13.
at Rs.10 more than the given selling price and
sold all apples on the same day at Rs.5 less than

Click Here For Bundle PDF Course | support@guidely.in Page 1 of 8


SBI Clerk & RRB PO Mains PDF Course 2023
Quantitative Aptitude Day – 19 (Eng)

4) Find the difference between the time taken by downstream in two hours in river R. Boat B
train A and train B to cross train C when all trains covers 56 km downstream in river Q in 2 hours.
are running in the same direction? Boat C covers 90 km in 3 hours in still water.
a) 57 sec Ratio of the speed of the stream of river R to that
b) 52 sec of Q is 3:4. Boat A covers 32 km upstream in 4
c) 54 sec hours in river P.
d) 59 sec 7) Boat B covers (D + 4) km downstream in river
e) 60 sec Q in T1 hours and boat C covers (3D + 12) km
downstream in river R in T2 hours. Find the
5) Find the ratio of time taken by train A to cross value T1/T2?
a 200 m long platform and time taken by train C a) 2/5
to cross a 210 m long bridge? b) 7/9
a) 51:58 c) 3/7
b) 37:55 d) 4/5
c) 31:59 e) 2/3
d) 33:53
e) 31:58 8) Boat A covers D km downstream in 3 hours in
river R. Find the time taken by boat B to cover
6) Length of train D is 40 m more than the length the 3D distance downstream in river P?
of train C. Speed of train D is 10 m/sec more a) 6.75 hours
than train A. Find the time taken by train D to b) 8.75 hours
cross a platform whose length is same as length c) 7.75 hours
of train B? d) 11.75 hours
a) 25.33 sec e) 10.35 hours
b) 20.66 sec
c) 27.35 sec 9) Boat C covers 209 km distance in downstream
d) 24.32 sec in t hours in river Q. Find the time taken by boat
e) None of these A to cover the same distance upstream in river
R?
Directions (07 - 09): Study the following a) (5.5t + 7.33)hours
information carefully and answer the questions b) (5t + 7.33)hours
given below. c) (6t - 7.33)hours
Ratio of the speed of boat A to that of boat B in d) (8t + 7.33)hours
still water is 3:5. Boat C covers 72 km e) None of these

Click Here For Bundle PDF Course | support@guidely.in Page 2 of 8


SBI Clerk & RRB PO Mains PDF Course 2023
Quantitative Aptitude Day – 19 (Eng)

Directions (10 - 12): Study the following 12) A invests his share of profit from business II
information carefully and answer the questions at 20% rate of interest for five years. Find the
given below. simple interest?
Business I: A, B, and C invest Rs. (D + 12000), a) Rs. 5262
Rs. (2D + 10000), and Rs. (3D + 2000) b) Rs. 3456
respectively. A, B and C invest their amounts for c) Rs. 2410
(t + 2), (t + 8), and (n + 2) months respectively. d) Rs. 4586
The ratio of their profit share after one year is e) None of these
12:27:14. B invest Rs.2000 more than A. Total
Profit is Rs.4134. Directions (13 - 16): Study the following
Business II: A invests 20% more than he invests information carefully and answer the questions
in business I and B invest 25% less than he given below.
invests in business I. C invest Rs.3000 more A person X has Rs.40000. He invests the
than his investment in business I. They invest the amount at 20% rate of simple interest for 3
same time as they invest in a business I. Total years. Total amount he gets is distributed equally
profit is Rs.12396. among his two sons and two daughters. One son
10) Find the difference in the share of profit A invests his amount at (R + 5) % rate of simple
and C in two businesses together? interest and gets Rs.20000 as interest after 5
a) Rs.445 years. Another son invests his money at 10%
b) Rs.742 rate of compound interest for 3 years and gets
c) Rs.780 Rs. I1 as interest. One daughter invests her
d) Rs.845 money at R% compound interest for 2 years and
e) Rs.950 earns Rs. I2 as interest and another daughter
invests her money at (R + 10) % rate of simple
11) Share of profit of B in two businesses is interest for 3 years and gets I3 as interest.
approximately what percent of the total 13) Find the difference between the interest
investment of B in two businesses together? earned by two daughters together and two sons
a) 19% together?
b) 11% a) Rs. 3250
c) 18% b) Rs. 3582
d) 22% c) Rs. 3856
e) 15% d) Rs. 4520
e) Rs. 3450

Click Here For Bundle PDF Course | support@guidely.in Page 3 of 8


SBI Clerk & RRB PO Mains PDF Course 2023
Quantitative Aptitude Day – 19 (Eng)

14) If the person X invested at (R + 10) % simple 17) A started the work and after (2t – 11) days A
interest for the same time, then find share of the left the work and B joined and worked for (t - 2)
amount of each of his kids? days. Then B left the work and D and C joined
a) Rs. 17500 and finished the work. Find number of days
b) Rs. 18500 taken by C and D to complete the remaining
c) Rs. 19600 work?
d) Rs. 19000 a) [10t + 21]/22days
e) Rs. 14500 b) [8t + 21]/22days
c) [12t + 21]/22days
15) If one of his son invests his money at (2R - d) [9t + 21]/22days
10) % rate of compound interest for 2 years then e) [12t + 31]/22days
finds his new interest amount?
a) Rs. 10240 18) A and E started the work and after 6 days
b) Rs. 11040 they left and remaining work is completed by D
c) Rs. 15620 and P in 6 days. Then find in how many days P
d) Rs. 11620 and B together can complete the work?
e) None of these a) [12t + 3]/22days
b) [11t + 3]/22days
16) Find the value 4[I1] + 3[I2] – [I3] =? c) [13t + 3]/22days
a) 27904 d) [13t + 3]/11days
b) 25262 e) [12t + 3]/11days
c) 28668
d) 25826 19) Find the difference between the time taken
e) None of these by B and C together to complete the work and
the time taken by A, D and E together to
Directions (17 - 20): Study the following complete the work?
information carefully and answer the questions a) (t/3 - 1.5)
given below. b) (t/2 - 0.5)
A can complete the work alone in (t + 15) days c) (t/2 + 0.5)
and B can complete work alone in (t + 11) days. d) (t/2 + 2.5)
D and E together complete work in 12 days. D e) None of these
can complete the work alone in 4t days. C can
complete the work in (t + 21) days. B is 20%
more efficient than A.

Click Here For Bundle PDF Course | support@guidely.in Page 4 of 8


SBI Clerk & RRB PO Mains PDF Course 2023
Quantitative Aptitude Day – 19 (Eng)

20) Total wage of the work is Rs.1800. A, C and b) Rs.14200/47


E together complete the work. Find the c) Rs.14535/47
difference in wages of C and E? d) Rs.14520/37
a) Rs.14400/47 e) None of these
Click Here to Get the Detailed Video Solution for the above given Questions
Or Scan the QR Code to Get the Detailed Video Solutions

Answer Key with Explanation

Directions (1- 3): = [(75 – 60) * 15 + (60 – 40) * 12] - [(80 – 60) *
15 + (50 – 40) * 12] = Rs.45

3. Answer: B
Total cost price = 75 * 20 + 50 * 4 = 1700
Total selling price = 90 * 20 + 70 * 4 = 2080
So, profit percentage = [2080 - 1700] * 100/1700
= 22.35%
1. Answer: C Directions (4 -6):
Total profit percentage by selling Orange = [50 - Speed of train A = 180/2.5 = 72 km/hr = 20
30] * 100/30 = 66.66% m/sec
Total profit percentage by selling Apple = [60 - Speed of train B = 180/2 = 90 km/hr = 25 m/sec
40] * 100/40 = 50% So, 72t + 90t = 324, t = 2
Required difference = 66.66% - 50% = 16.66 % So, speed of train C = 324/6 = 54 km/hr =
15m/sec.
2. Answer: D Length of train A = l
Difference of profit amount = Difference in profit So, l + 160 = 20 * 20
amount of both cases l = 400 – 160 = 240 m

Click Here For Bundle PDF Course | support@guidely.in Page 5 of 8


SBI Clerk & RRB PO Mains PDF Course 2023
Quantitative Aptitude Day - 19 (Eng)

Length of train C = M
So, M + 40 = 24 * 15 9. Answer: B
M = 360 – 40 = 320 m So, t = 209/38 = 5.5hours
Length of train B = 240 * 13/12 = 260 m Required time = 209/ (12 - 6) = 34.83 hours= 5t
4. Answer: C + 7.33
Time taken by train A = [240 + 320]/5 = 112 sec
Time taken by train B = [320 + 260]/10 = 58 sec Directions (10 - 12):
So, the difference between the time = 112 – 58 = 2D + 10000 – D – 12000 = 2000
54 sec Or, D = 4000
So, investment of A, B, and C in business I =
5. Answer: D Rs.16000, Rs.18000, and Rs.14000
Required ratio = [440/20]: [530/15] = 33:53 respectively.
6. Answer: B So, 16000 * (t + 2)/[18000 * (t + 8)] = 12/27
Required time = [260 + 40 + 320]/ [20 + 10] = From this we get t = 4
20.66 sec Now 16000 * 6/[14000 * (n + 2)] = 12/14
From this we get n = 6
Directions (07 - 09): A’s investment in business II = 16000 * 120/100
Speed of boat C = 90/3 = 30 km/hr = Rs.19200
Speed of the stream of river R = 72/2 – 30 = 6 B’s investment in business II = 18000 * 75/100 =
km/hr. Rs.13500
Speed of the stream of river Q = 6 * 4/3 = 8 C’s investment in business III = 14000 + 3000 =
km/hr Rs.17000
Speed of boat B = 56/2 – 8 = 20 km/hr So, their investment ratio = 19200 * 6:13500 *
Speed of boat A = 20 * 3/5 = 12 km/hr 12:17000 * 8
Speed of stream of river P = 12 - [32/4] = 4 = 288:405:340
km/hr 10. Answer: C
7. Answer: C Required difference = [12396 * 340/1033 + 4134
So, T1/T2 = [(D + 4)/28]: [(3D + 12)/36] = 12/28 * 14/53] - [12396 * 288/1033 + 4134 * 12/53] =
= 3/7 Rs.780

8. Answer: A 11. Answer: D


D = [12 + 6] * 3 = 54km Total share of B =12396 * 405/1033 + 4134 *
Required time = [54 * 3/24] = 6.75 hours 27/53 = Rs.6966

Click Here For Bundle PDF Course | support@guidely.in Page 6 of 8


SBI Clerk & RRB PO Mains PDF Course 2023
Quantitative Aptitude Day - 19 (Eng)

Required percentage = [6966/ (13500 + 18000)] Directions (17 - 20):


* 100 = 22.11% = 22% The ratio of efficiency of A and B = 5:6.
So, (t + 15)/ (t + 11) = 6/5
12. Answer: B Or, 5t + 75 = 6t + 66, t = 9
Simple interest = [12396 * 288/1033] * 20 * 5/100 D completes the work in 4 * 9 = 36 days, and C
= Rs. 3456 completes the work in t + 21 = 30 days.
One day’s work of E = 1/12 - 1/36 = 2/36 = 1/18
Directions (13 -16): So, E completes the work in 18 days.
Share of each child = [40000 * 20 * 3/100 + Total work = LCM of 24, 20, 30, 36, 18 =
40000]/4 = Rs.16000 360units
So, 16000 * (R + 5) * 5/100 = Rs.20000 The efficiency of A, B, C, D, and E = 15, 18, 12,
Or, (R + 5) = 20000 * 100/5 * 16000 = 25, so, 10, and 20units/day respectively.
R% = 20% 17. Answer: C
I1 = 16000 * (1 + 10/100)3 – 16000 = Rs.5296 Before joining of C and D, the total work
I2 = 16000 * 44/100 = Rs.7040 completed = [15 * (18 - 11) + 18 * 7] = 231units
I3 = 16000 * 30 * 3/100 = Rs.14400 So, the left work = 360 – 231 = 129units
13. Answer: C So, required days = [129/22] days = [12t + 21]/22
So, required difference = [20000 + 5296] -
[14400 + 7040] = Rs. 3856 18. Answer: D
In 6 days, A and E complete work = 6 * [15 + 20]
14. Answer: D = 210units
Share of each kid = [40000 * (20 + 10) * 3/100 + So, the left work = 360 – 210 = 150units
40000]/4 = Rs. 19000 D work in 6 days = 6 * 10 = 60units
P work in 6 days = 150 – 60 = 90units
15. Answer: B P’s efficiency = 90/6 = 15
2R – 10 = 2 * 20 – 10 = 30% So, P and B complete the work in 360/ (15 + 18)
Total interest = 16000 * [1 + 30/100]2 – 16000 = = [13t + 3]/11days
Rs.11040
19. Answer: B
16. Answer: A Required difference = [360/ (18 + 12)] – [360/ (15
4I1 + 3I2 - I3 = 4 * 5296 + 3 * 7040 – 14400 = + 10 + 20)] = 4 = (t/2 - 0.5)
27904
20. Answer: A

Click Here For Bundle PDF Course | support@guidely.in Page 7 of 8


SBI Clerk & RRB PO Mains PDF Course 2023
Quantitative Aptitude Day - 19 (Eng)

Time taken to complete the work = 360/ (15 + 12 So, difference = (360/47) * [20 - 12] * 5 =
+ 20) = 360/47 days Rs.14400/47
The wage of one unit of work = 1800/360 = Rs.5

Click Here For Bundle PDF Course | support@guidely.in Page 8 of 8


SBI Clerk & RRB PO Mains PDF Course 2023
ENGLISH Day - 19

English Language

Directions (1-7): Read the following passage and (inflation-adjusted) income decreases. The
answer the questions given below. standard of living is measured by real income.
Giving people the resources and ability to learn When actual incomes rise, the standard of living
about how things are going in our economy is an rises with them, and vice versa.
extremely important thing. One issue that is In actuality, prices fluctuate at various rates.
especially prevalent in today’s economy is Some, such as the prices of traded commodities,
inflation and how it affects the overall well-being fluctuate on a daily basis; others, such as
of the people in our country and around the contract-based pay, require longer to adjust. In
globe. This economic issue affects every an inflationary environment, unevenly growing
member of our society, and it is especially prices lower some customers’ purchasing power,
important that our citizens keep themselves well- and this erosion of real income is the single most
educated on this topic. In a sound economy, significant cost of inflation.
costs will in general increase – this is known as Inflation can also affect the purchasing power of
inflation. While you probably won’t care for that fixed-interest rate receivers and payers over
as a buyer, price growth rising moderately is an time. Take, for example, retirees who are
indication of a solid, developing economy. guaranteed a 5% annual rise in their pension.
The U.S. Federal Reserve currently considers a When inflation exceeds 5%, a retiree’s
2% inflation rate to act as the best growth rate for purchasing power decreases. A borrower paying
the economy, which is about its present level. Be a 5% fixed-rate mortgage, on the other hand,
that as it may, a few financial analysts, including would gain from 5% inflation because the real
those at the Federal Reserve, stress the interest rate (fixed rate of mortgage minus
economy is debilitating, which would make inflation rate) would be zero; servicing this debt
inflation dip under its objective, which is would be even easier if inflation were greater, as
something that needs to be avoided at all costs. long as the borrower’s income kept up with
The most recent information, which was released inflation. Of course, the lender’s real income
to the public on June 12, implied that this might drops as a result. When nominal interest rates
be occurring. aren’t adjusted for inflation, some people gain
Households are worse off if their nominal and others lose purchasing power.
income, which they receive in current money, A moderate measure of inflation is commonly
does not rise at the same rate as prices because viewed as an indication of a solid economy, in
they can afford to buy fewer things. To put it light of the fact that as the economy develops,
another way, their purchasing power or real the demand for goods and services grows. This

Click Here For Bundle PDF Course | support@guidely.in Page 1 of 15


SBI Clerk & RRB PO Mains PDF Course 2023
ENGLISH Day - 19

growth of demand pushes costs somewhat when the income of every person remains the
higher as providers attempt to make a greater same.
number of what buyers and organizations need d. Only options a and b
to purchase. When inflation becomes excessively e. Only option b
low, or vice versa, a horrendous cycle can wreak
havoc upon our economy. High inflation has a 2. Which of the following topics best suit the
wide scope of negative ramifications for context of the passage?
economies. At the point when work a. Overcoming the negative effects inflation using
compensation is unable to stay in line with the the right economic strategies.
inflation of retail costs, the power of purchasing b. Moderate inflation – An indication of a
of the pay checks that workers receive quickly declining economy
diminishes. This causes a huge problem for c. The varying effects of inflation
households that receive low income, because d. Exploring household economic issues
any increase in the price of any goods or e. A guide to improving the purchasing power of
services can have very serious negative effects. a common man
Laborers’ requests for growth in their wages can
prompt an expansion in costs of labor, bringing 3. According to the passage, why do
about lower benefits for organizations. These organizations suffer when the inflation rate of a
impacts of expansion can make a high level of country increases?
vulnerability in an economy, prompting a. When inflation increases, the cost of raw
diminished venture from those who aim to start materials increases which eventually results in
their own businesses. increased expenditures for the organizations.
1. Which of the following statement/statements b. When inflation increases, the government
is/are true according to the passage given tends to increase the taxes imposed on
above? production and distribution of products and this
a. When the rate of inflation falls below the could deplete the capital funds of the
recommended growth rate, the economy of a organization.
country can begin to lose strength and vitality. c. When inflation increases, customers tend to
b. Households can suffer when their income prevent buying products due to a surge in the
does not change in sync with the moving inflation cost of goods and this could lead to losses for
rates. organizations that produce commodities for the
c. The purchasing power of every person public.
increases when the inflation rates increase even d. When inflation increases, the laborers tend to
request wage compensations to make up for the

Click Here For Bundle PDF Course | support@guidely.in Page 2 of 15


SBI Clerk & RRB PO Mains PDF Course 2023
ENGLISH Day - 19

increase in the price of basic commodities and d. cause


this could bring in increased expenses for the e. effect
organization.
e. None of the above reasons is right. Directions (8-12): Given below are a set of
sentences in which only one of them is
4. According to the passage, what grammatically and contextually correct. Identity
section/sections of the society lose/loses the sentence that is error-free.
purchasing power after an increase in the rates 8.
of inflation? a. Both Rita and Sita are not considering taking
a. Borrowers who pay fixed rate mortgage the IAS exams this month since they were held
b. People with contract-based income options up at work for the last few months and eventually
c. Individuals who receive retirement benefits had no time for the exam preparations.
d. Only option c b. Arjun has been planning to marry whoever his
e. Options b and c mother chooses since he dislikes going against
his mother's words.
5. What is the antonym of the word ‘debilitating’? c. If Reena takes up the upcoming exams
a. invigorating seriously, she could pass the exams with flying
b. exhausting colors.
c. draining d. Doctors advise us to drink at least 8 glassfuls
d. enfeebling of water every day to keep ourselves hydrated
e. None of the above during the summers.
e. Deepthi was confused about how some of her
6. What is synonym of the word ‘horrendous’? money were missing, but later she realized that
a. calming she had lent 5000 rupees to her sister this
b. pleasant month.
c. non-threatening
d. formidable 9.
e. None of the above a. Meena offered to pay for the watch she had
chosen using two five hundred rupees notes, but
7. What is the right antonym for the word the shopkeeper wanted her to transfer the
‘ramifications’? amount through mobile banking apps since he
a. outcome had no change.
b. consequence
c. implication

Click Here For Bundle PDF Course | support@guidely.in Page 3 of 15


SBI Clerk & RRB PO Mains PDF Course 2023
ENGLISH Day - 19

b. My friend is okay with me borrowing her car for a. Tanya was living in Chennai before she
a day or two since I am yet to purchase my received an offer to study and work in one of the
vehicle. most prestigious colleges in the United Kingdom.
c. Rita exclaimed that she loved watching movies b. Eagerly waiting to get a much-needed break
more than me, but I had been an ardent movie from her everyday routine, Roopa was
buff since childhood. heartbroken heavily when her car broke down
d. Neither the students nor the teacher is happy minutes before she started to drive to the mall.
about how the school canceled the science c. She called me several times in the last four
exhibition at the last minute since the class weeks until I had agreed to set an appointment.
collectively put in a lot of hard work to create the d. Every student was assigned a part in the play
models. that was to be staged on the school's annual day
e. The new building constructed at the heart of to ensure that all students got equal opportunities
the city is three times taller than the LIC building to showcase their acting talents.
at Chennai's Mount Road. e. All sentences are incorrect

10. 12.
a. Teju had no doubt whether she would pass the a. He could have passed the English literature
upcoming exams, but I doubted that I would clear test and secured a job offer in the United States
mine. if his father had been an English professor.
b. Renu said that she performed yoga every day b. Many a student from the Science department
to maintain her body fitness and improve her is contesting in the elections conducted to select
mental health. the members of the Student Union.
c. Meera had been planning to meet up with c. Not only the newly recruited interns speak
Rahul since the time she had asked him to pair French, but they are also fluent in English.
up with her for the upcoming project. d. While addressing the gathering, the minister
d. I wish I can meet Reena today for old time's stated that he believed that the youth of his state
sake, but I am uncertain about my schedule after would become powerful and effective with the
today's classes. required training and exposure.
e. The road to the airport was blocked due to the e. None of the sentences are incorrect.
arrival of the President for the inauguration of the
new world-class multi-specialty hospital in Directions (13-17): The following sentence has
Chennai. one part that is highlighted in bold. This
highlighted part may or may not be
11. grammatically and contextually correct. Choose

Click Here For Bundle PDF Course | support@guidely.in Page 4 of 15


SBI Clerk & RRB PO Mains PDF Course 2023
ENGLISH Day - 19

the best alternatives among the four options e. No changes required


given below such that it rightly replaces the
highlighted part grammatically and contextually. 16. Karan does not deserve to take a claim on
If the part given in bold is grammatically and the profits owned by his family business since he
contextually right, then choose the option ‘No hardly turned up at the office and engaged in
replacement required’. work in the last few months.
13. The police department has clamped down of a. deserve to make a claim to the profits
piracy to prevent the illegal streaming of newly b. deserve to stake a claim to the profits
released films on various online platforms. c. deserve to stake a claim on the profits
a. have clamped down of piracy to prevent the d. deserve to make a claim on the profits
b. has clamped down on piracy to prevent the e. No changes required
c. has clamped down against piracy to prevent
the 17. Tanya was wet behind the ears since she
d. has clamped down on piracy for the prevention had just graduated but was excited about
e. No change required venturing into new job roles in the company of
her choice.
14. He, I, and you are collectively at fault for not a. waiting behind in years since she did
submitting the project within the specified b. wet behind in years since she had
deadline since we lost track of what we should c. waiting behind in years since she had
do midway through the project. d. wet beyond in ears since she had
a. I, he, and you are collectively at fault e. No changes required
b. He, you, and I are collectively at fault
c. I, you, and he are collectively at fault Directions (18-22): The following set of questions
d. He, you, and I are collectively in fault are not arranged in the right order. These
e. No corrections required sentences when arranged in the right order gives
right to meaningful passage. Identify the right
15. Rahul felt ashamed when he got his working pattern of arrangement and answer the
papers after being found guilty of leaking the questions that follow.
company's information to the competitors in A. Such kind of activism positively affect the
return for a ransom. society, as they attract the public’s attention to a
a. He got his walking papers after being topical issue.
b. He had gotten his walking papers after being B. Women account for one of the groups most
c. He got his working papers after being affected by the disparities in the legal system.
d. He got his working papers after been

Click Here For Bundle PDF Course | support@guidely.in Page 5 of 15


SBI Clerk & RRB PO Mains PDF Course 2023
ENGLISH Day - 19

C. Subsequently, various standpoints are d. reproduces


expressed through different means of e. None of the above
communication, calling for an equal approach to
justice for both sexes. 20. Which sentence comes second after
D. Therefore, this sphere has been an area of arranging the set of sentences correctly to form a
intense interest for feminist researchers and meaningful passage?
activists. a. A
F. Accordingly, it can be concluded that the b. B
country is currently experiencing the c. C
phenomenon of mass female incarceration, d. D
which _________ the unfair side of the legal e. E
system.
G. Reports suggest that the incarceration rate of 21. Which sentence comes fourth after arranging
women in a particular country has grown at a the set of sentences correctly to form a
quicker pace than a similar number of men. meaningful passage?
18. Which of the following words is the best a. A
antonym for the word highlighted in the passage b. B
– disparities? c. C
a. inequality d. D
b. distinction e. E
c. indifference
d. imparity 22. Which sentence comes sixth after arranging
e. None of the above the set of sentences correctly to form a
meaningful passage?
19. What word best suits the blank in the given a. A
passage? b. C
a. matches c. D
b. reflects d. E
c. resonates e. F

Click Here For Bundle PDF Course | support@guidely.in Page 6 of 15


SBI Clerk & RRB PO Mains PDF Course 2023
ENGLISH Day - 19

Click Here to Get the Detailed Video Solution for the above given Questions
Or Scan the QR Code to Get the Detailed Video Solutions

Answer Key with Explanation

1. Answer: E some people (not every person) increases when


The context of option A is clearly stated in the inflation rates increase. Thus, option c is
second paragraph of that passage. But the incorrect.
financial analysts state that the inflation rate
could fall below the recommended growth rate 2. Answer: C
when economy tends to become weak and lose Option a is incorrect. Because the passage does
vitality. But option ‘a’ conveys describes the said not focus on disclosing or proposing economic
course of actions in the other way. Option a says strategies to overcome the negative effects of
that the economy of the country becomes weak inflation.
when the inflation rate dips below the Option b is incorrect. Because according to the
recommended growth rate which does not match passage, moderate inflation is described as an
the message conveyed through the passage. indication of a developing economy and not a
Thus, option a is incorrect. falling economy.
Option b talks about how households can suffer Option c is correct. Because the passage briefly
when their income does not change with the describes about how inflation can positively and
moving inflation rates. This statement is clearly negatively affect the people of a country.
stated in the third paragraph of the passage. Option d is incorrect. Because the passage does
Option c is incorrect. Since according to the fifth not solely focus on household economic issues
paragraph of the passage, people can gain and since topic is just one among the many
lose their purchasing power when the inflation discussed in the passage. For instance, the last
rates rise. But option c normalizes this statement paragraph of the passage also talks about the
to every person. The purchasing power of only issues organizations face as a result of laborers

Click Here For Bundle PDF Course | support@guidely.in Page 7 of 15


SBI Clerk & RRB PO Mains PDF Course 2023
ENGLISH Day - 19

requesting wage compensations to tackle the get increased only after periodic intervals. Thus,
effects of inflation. the retirees would face financial challenges until
Option e is incorrect. Because the passage does the pension amounts gets renewed. Thus, option
not focus on improving the purchasing power of c is correct.
a common man but only explains how the Option b speaks about individuals with contract-
purchasing power of a common man gets affects based income options. Retirees are people with
when inflation rates change drastically. Also, the contract-based income options since they
passage does not explicitly state alternatives to receive contracted income options that increase
improve the purchasing power of a common only once in every year. Thus, option b is also
man. correct.
Thus, options b and c are correct.
3. Answer: D
According to the last paragraph of the passage, 5. Answer: A
it is clear that when the price of the commodities The word debilitating is an adjective which
increases, laborers tend to request wage means to impair the strength and vitality.
compensations and this could expand the costs Invigorating (adjective) – to impart strength and
of labor of the organization. Thus, option d is vitality
correct. Also, the passage does not explicitly talk Exhausting (adjective) – to have a debilitating
about how organizations get affected as a result effect
of increase in taxes, increase in the price of raw Draining (adjective) – to have a debilitating effect
materials, and less sales. Thus, options a, b, and Enfeebling (adjective) – to weaken
c are incorrect. The options b, c, and d are synonymous with
each other. The right answer is option a.
4. Answer: E
In the fifth paragraph of the passage, it is clearly 6. Answer: D
stated the borrowers who pay fixed rate The word ‘horrendous’ (adjective) means to
mortgage gain purchasing power when there is cause fear, dread, or terror.
an increase in the rate of inflation since in these Calming (adjective) – to be in a state of peace.
cases, their actual interest rates become zero. Pleasant (adjective) – calming and peaceful
Thus, option a is incorrect. Non-threatening (adjective) – not making feeling
In the same paragraph, it is stated that retirees nervous
lose purchasing power after an increase in the Formidable (adjective) – inspiring fear
rate of inflation because their pension amounts Thus, the right answer is option d.

Click Here For Bundle PDF Course | support@guidely.in Page 8 of 15


SBI Clerk & RRB PO Mains PDF Course 2023
ENGLISH Day - 19

indicates who his mother chooses). The pronoun


7. Answer: D ‘whomever’ is always used as an objective
The word ‘ramifications’ (noun) means the result pronoun. It is used as an object of a
of an event. verb/preposition in a sentence. Thus, whoever
The words outcome, consequence, implication, should be replaced with whomever in sentence
and effect are all nouns and they denote the B. The right sentence is:
result of an event. Thus, they are synonymous to Arjun has been planning to marry whomever his
the word ‘ramifications’. mother chooses since he dislikes going against
The word ‘cause’ (noun) indicates something his mother's words.
that encourages an event to occur. In simple The sentence C is a conditional sentence. The
contexts, the cause of an event is something that conditional part of the sentence is written in
happens before it and the outcome of an event simple present sentence. Thus, the main part of
happens after it. Thus, the word ‘cause’ is the the sentence should also be written in simple
right antonym for ramifications. present tense to follow uniformity. Thus, the use
of the word ‘could’ in the main part of the
8. Answer: D sentence is wrong since it is a simple past model
The sentence A uses the word ‘Both’ and ‘Not’ verb. The right choice of model verb (simple
together. The words ‘both’ and ‘not’ should not present) for this sentence is may/can. The right
be used together. In this case, it is advised to sentence is:
use ‘neither – nor’ with a singular or plural verb If Reena takes up the upcoming exams
according to the subject-verb agreement. The seriously, she can/may pass the exams with
right sentence is: flying colors.
Neither Rita nor Sita is considering taking the The sentence D is grammatically and
IAS exams this month since they were held up at contextually right.
work for the last few months and eventually had The sentence E uses the phrase ‘some of her
no time for the exam preparations. money’. The word money always takes singular
The sentence B has the word ‘whoever’. The form and thus, the verb that is used to describe
pronoun ‘whoever’ is always used as a money also takes singular form. Thus, the right
subjective pronoun. It is replaced as the subject sentence is:
of the verb in a sentence. But in this case, the Deepthi was confused about how some of her
pronoun is position to describe the object of the money was missing, but later she realized that
verb in the sentence, (i.e) the pronoun should be she had lent 5000 rupees to her sister this
positioned as an object of the verb (verb - month.

Click Here For Bundle PDF Course | support@guidely.in Page 9 of 15


SBI Clerk & RRB PO Mains PDF Course 2023
ENGLISH Day - 19

Thus, the word ‘me’ is incorrect. The word ‘I’ is


9. Answer: D the right first person subjective pronoun. The
The sentence A includes the phrase ‘two five right sentence is:
hundred rupees notes’ which is grammatically Rita exclaimed that she loved watching movies
incorrect. If the phrase is framed in the form – more than I, but I had been an ardent movie buff
‘number + unit + noun’ the unit mentioned here since childhood.
should take singular form. Here, the number is The sentence D is correct. Note that when there
‘five hundred’, unit is ‘rupees’ and noun is ‘notes. are two subjects separated by neither or nor,
The unit should take singular form. Thus, the then the verb should be inserted according to the
right sentence is: nearest subject like in the case of this sentence.
Meena offered to pay for the watch she had The sentence E speaks about comparisons.
chosen using two five hundred rupee notes, but When the word ‘times’ is used in the sentence, it
the shopkeeper wanted her to transfer the should not be followed by adjectives/nouns of
amount through mobile banking apps since he the comparative degree. Thus, ‘three times taller
had no change. than’ is grammatically incorrect. The comparison
The sentence B consists of a phrase ‘me is made using the phrase ‘as + positive degree
borrowing’ which is grammatically incorrect. If a adjective/verb + as’ to make it grammatically
pronoun is used before a gerund, the pronoun correct. The right sentence is:
should mandatorily take possessive form. Thus, The new building constructed at the heart of the
the word ‘me’ is an objective pronoun and hence city is three times as tall as the LIC building at
it is incorrect. The right word should be ‘my’ – Chennai's Mount Road.
possessive pronoun. The right sentence is:
My friend is okay with my borrowing her car for a 10. Answer: E
day or two since I am yet to purchase my The sentence A talks about exams. Sentences
vehicle. with the word ‘doubt/doubtful’ (unsure of
The sentence C compares the liking of two something) should be followed by the connectors
individuals. The comparison should be made ‘if/whether’ and the sentences with the word ‘no
between two similar entities, (i.e) here the doubt/not doubtful’ should be used with the
comparison is made between ‘she’ which a third connector ‘that’. Thus, the right sentence is:
person subjective pronoun and the speaker. Teju had no doubt that she would pass the
Thus, the speaker, begin the first person, should upcoming exams, but I doubted whether I would
be represented by a subjected pronoun as well. clear mine.
But ‘me’ is a first person objective pronoun.

Click Here For Bundle PDF Course | support@guidely.in Page 10 of 15


SBI Clerk & RRB PO Mains PDF Course 2023
ENGLISH Day - 19

The sentence B speaks about the daily habit of in simple present form. The right use of the verb
Renu. The tense of a verb when describing a should be ‘could’. The right sentence is:
habitual event of a person does not change even I wish I could meet Reena today for old time's
in the case of reported speech. The tense of the sake, but I am uncertain about my schedule after
verb should be in simple present tense in indirect today's classes.
speech when describing about a habitual event The sentence E is grammatically and
of a person. Thus, ‘she performed yoga’ is wrong contextually correct.
and it must be changed to ‘she performs yoga’.
The right sentence is: 11. Answer: D
Renu said that she performs yoga every day to The sentence A speaks about two events. If
maintain her body fitness and improve her there are two events wherein one of the two
mental health. events was in progress in the past until the other
The sentence C consists of the word ‘since’. event occurred. In this case, the event that
Sentences with ‘since’ should be framed in such happened first should be written in past perfect
a way that the part of the sentence before the continuous tense. Thus, ‘Tanya was living in
word ‘since’ should be written perfect/perfect Chennai’ (lived-simple past) is incorrect. The
continuous tense, while the part of the sentence right use of verb should be in past perfect
after the word ‘since’ should be written in simple continuous tense ‘has been living’. The right
tense. In this sentence, the part of the sentence sentence is:
before ‘since’ is correctly framed with ‘had been Tanya had been living in Chennai before she
planning’ – past perfect continuous tense. But received an offer to study and work in one of the
the sentence after the word ‘since’ is not written most prestigious colleges in the United Kingdom.
in simple past. The correct form of the verb here In sentence B, the adverb ‘heavily’ is placed
should be ‘she asked’ and not ‘she had asked’. after the verb ‘heartbroken’. The adverb is
The right sentence is: normally placed before the verb to bring in
Meera had been planning to meet up with Rahul greater emphasis of the verb. Thus, the right
since the time she asked him to pair up with her sentence is:
for the upcoming project. Eagerly waiting to get a much-needed break
The sentence D includes the word ‘wish’. The from her everyday routine, Roopa was heavily
verb after the word ‘wish’ should always be heartbroken when her car broke down minutes
included in simple past form. Thus, ‘I wish I can’ before she started to drive to the mall.
is grammatically incorrect since the verb ‘can’ is In the sentence C, there are two actions. When
there are two actions taking place, the one that

Click Here For Bundle PDF Course | support@guidely.in Page 11 of 15


SBI Clerk & RRB PO Mains PDF Course 2023
ENGLISH Day - 19

had occurred first should be written in past from the science department is’ and not ‘Many a
perfect tense and the action that follows that students from the science department are’. The
should be written in simple past tense. Thus, the right sentence is:
use of verb ‘called’ is incorrect (action that Many a student from the Science department is
happened first) and it should be changed to past contesting in the elections conducted to select
perfect tense – ‘had called’. Also, for the action the members of the Student Union.
that follow, the use of the verb ‘had agreed’ is The sentence D uses the phrases ‘not only – but
incorrect and it should be replaced by ‘agreed’ also’. This set of phrases makes use of the rule
(simple past). Thus, the right sentence is: of inversion. In this case, the sentence
She had called me several times in the last four arrangement should follow this pattern : ‘not only
weeks until I agreed to set an appointment. + auxiliary verb +subject’. The right use of
The sentence D is grammatically correct. phrase would be ‘Not only do the newly’. The
right sentence is:
12. Answer: D Not only do the newly recruited interns speak
The sentence A talks about how the speaker French, but they are also fluent in English.
would have passed the English test if his father The sentence D is grammatically correct.
was an English professor. But profession of his
father is something that could not be changed. It 13. Answer: B
is impossible for his father to become an English The sentence includes a phrasal verb that
professor. Thus, the latter part of the sentence should necessarily take the form ‘clamped down
talks about something unreal or impossible. on’. So the right use of preposition is ‘on’ and not
Thus, the verb used here should be ‘were’ and ‘of’ or ‘against’. Thus, the options ‘a’ and ‘c’ are
not ‘had been’. Thus, the right sentence is: incorrect. In option d, the phrase ‘for the
He could have passed the English literature test prevention’ grammatically incorrect because the
and secured a job offer in the United States if his sentence after the highlighted part does not have
father were an English professor. the preposition ‘of’ to rightly completely the
The sentence B starts with the word ‘many’ sentence. Thus, the right answer is option ‘b’.
which should be immediately followed by a plural The right sentence is:
noun in general cases. If the word ‘many’ is The police department has clamped down on
followed by ‘a’ then the noun that follows ‘Many piracy to prevent the illegal streaming of newly
a’ should be singular and the verb which follows released films on various online platforms.
the subject of the sentence should be plural.
Thus, the right phrase should be ‘Many a student 14. Answer: C

Click Here For Bundle PDF Course | support@guidely.in Page 12 of 15


SBI Clerk & RRB PO Mains PDF Course 2023
ENGLISH Day - 19

When the sentence speaks about the The sentence speaks about how a person
acceptance of guilt or grief, the order of the beings to claim the profits that he do not deserve
pronouns would be ‘first person, second person, in the first place since he had not worked for it.
and third person’. Thus, options a, b, and d are But the right use of preposition after claim in this
incorrect. The right order of pronouns is ‘I, you, context is ‘on’ and not ‘to’. The word claim
and he’. The right sentence is: follows the preposition ‘on’ when it speaks about
I, you, and he are collectively at fault for not something that the subject is not deserving of or
submitting the project within the specified have not rights on. Also, the right word that
deadline since we lost track of what we should should be used to denote the act of claiming
do midway through the project. properties or wealth is ‘stake’. Thus, the right
sentence is:
15. Answer: A Karan does not deserve to stake a claim on the
The sentence talks about how Rahul was fired profits owned by his family business since he
from his job after being found to be at fault for hardly turned up at the office and engaged in
selling company’s information. The sentence work in the last few months.
conveys this better with the use of the idiom ‘Got
your walking papers’ which means to ‘get fired 17. Answer: E
from the job’. Thus, the right use of word is The given sentence is grammatically and
‘walking’ and not ‘working’. Thus, options a and contextually right. The phrase or idiom ‘wet
d are incorrect. Also, the verb ‘had gotten’ is behind her ears’ means ‘young and lacking in
grammatically incorrect. Since the sentence is experience’.
written is simple past sentence but ‘had gotten’
is past participle. The use of ‘had gotten’ is 18. Answer: C
incorrect since these leads to non-uniformity Disparity is a noun which denotes inequality or
while considering about the tense of the difference in some respect.
sentence. Thus, option b is incorrect. Inequality (noun) – lack of equality
The right sentence is: Distinction (noun) – discrimination between
Rahul felt ashamed when he got his walking entities
papers after being found guilty of leaking the Imparity (noun) – lack of equality
company's information to the competitors in Thus, options a, b, and d are synonymous to the
return for a ransom. given word.
Indifference (noun) – the state of being
16. Answer: C unbiased, impartial, equal

Click Here For Bundle PDF Course | support@guidely.in Page 13 of 15


SBI Clerk & RRB PO Mains PDF Course 2023
ENGLISH Day - 19

Thus, option c is the right answer. feminist researchers. Thus, sentence D should
come second in the arrangement. Since
19. Answer: B researchers have been brought into the picture
Matches (verb) – to coincide in characteristics they are bound to deliver viewpoints and
Reflects (verb) – to give evidence for a certain opinions on this issue. Thus, sentence C should
behavior or pattern come next since it contains the word
Resonates (verb) – to be similar to what ‘standpoints’ (noun) which means ‘viewpoints’.
someone believes of thinks Sentence C talks about the need for an equal
Reproduces (verb) – to replicate something approach to justice for both genders. An equal
Here, the sentence speaks about how a country approach to justice for both genders can bring
is experiencing mass female incarceration. The about a much required social equality. Thus, the
best choice of verb for this blank is ‘reflects’ sentence A should come next because it
since the mass female incarceration shows includes the phrase ‘such kinds of activism’ in
evidence for the unfairness in the legal system. the beginning wherein the word activism (noun)
Thus, the right sentence is: means a policy of actively campaigning to
Accordingly, it can be concluded that the country achieve a political or social goal. Now, there are
is currently experiencing the phenomenon of two sentences left, one of which speaks about,
mass female incarceration, which reflects the the incarceration rate of women has grown at a
unfair side of the legal system. quicker pace when compared to a similar
number of men and the other speaks about how
20. Answer: D something reflects the unfair side of the legal
The passage talks about the unfairness system. Also, the sentence F refers to the mass
observed against women. The sentence B incarceration of women that is introduced into
comes first because it introduces the topic to the the passage using the statistics of a report that is
readers. The sentence D talks about a certain mentioned in sentence G. Thus, sentence F
sphere sparks the interest of feminist follows sentence G.
researchers. The word sphere (noun) denotes a Thus, the right pattern of arrangement is:
particular aspect of an activity. The sentence A B-D-C-A-G-F
which talks about disparities in the legal system Thus, the second sentence after rearrangement
that work against women. Feminist researchers is D. The right answer is option d.
believe women should equal rights. Thus, the
disparities in the legal system that work against 21. Answer: A
women is that sphere that sparks the interest of

Click Here For Bundle PDF Course | support@guidely.in Page 14 of 15


SBI Clerk & RRB PO Mains PDF Course 2023
ENGLISH Day - 19

The right pattern of arrangement is B-D-C-A-G- 22. Answer: E


F. Refer to the explanation given for question The right pattern of arrangement is B-D-C-A-G-
number 20 for a detailed solution for the F. Refer to the explanation given for question
rearrangement. Thus, the right answer is option number 20 for a detailed solution for the
a. rearrangement. Thus, the right answer is option
e.

Click Here For Bundle PDF Course | support@guidely.in Page 15 of 15


SBI Clerk & RRB PO Mains PDF Course 2023
Reasoning Ability Day - 20 (Eng)

Reasoning Ability
Directions (1-5): Study the following information work that he/she has to do is already doing by
carefully and answer the given questions. the person who is before him/her in the
Seven persons- A, B, C, D, E, F and G are alphabetical order), then that person works in
studying nursing course. They are allocated to do Cardiac ward.
work in different departments in the hospital for 1. How many weeks it took for the person who
practice in a week – ER ward, ICU ward, General completed the work at the second last person?
ward, OP ward, Surgical ward, Oncology ward a) 21 weeks
and Paediatric ward in alphabetical order. The b) 18 weeks
work allocated in OP ward will be completed c) 19 weeks
consecutively for three weeks and the work d) 20 weeks
allocated in ICU ward will be completed e) 17 weeks
consecutively for two weeks whereas the rest are
completed in one week. Each work will be 2. Who among the following person complete the
assigned and monitored by the individual chief work before B?
nurse. a) G
In the first week A will work in ER ward and B will b) C
work in General ward and in the second week A c) D
will work in General ward and B will work in ICU d) Both options a and c
ward and so on as in alphabetical order. The e) Both options b and c
work will be repeated in alphabetical order only.
Note: If a chief nurse is unavailable, then the 3. Which of the following pair completed the
next department in the order will be assigned. work in the same week?
i.e., if a person needs to work in OP ward in a) A and C
alphabetical order but that ward is already b) E and C
assigned to someone else, then that person c)A and D
should be assigned to Paediatric ward. d) E and B
The work allocated to Surgical ward will be e) Both options b and c
followed by the work allocated to ER ward and
the cycle is repeated. Each person works in each 4. Paediatric ward is related to B in __ and
department only once. If for a person, no chief Surgical ward is related to E in __
nurse is available for a particular week (either a) Week 1 and Week 9
because he/she has already done all the works b) Week 6 and Week 7
or he/she cannot do the available works or the c) Week 8 and Week 12

Click Here For Bundle PDF Course | support@guidely.in Page 1 of 9


SBI Clerk & RRB PO Mains PDF Course 2023
Reasoning Ability Day - 20 (Eng)

d) Week 3 and Week 5 e) None of these


e) Week 11 and Week 4
7. The distance between boat B and boat S is
5. Which of the following is not true as per the same as the distance between boat ___ and
final arrangement? boat___
a) The person who works in ICU in weeks 9 and a) B, G
10 is E b) K, T
b) G works in the General ward immediately after c) K, The boat which docks immediate right of S
ER ward d) G, The boat which docks immediate left of B
c) F works in the Cardiac ward for less than 7 e) J, G
weeks
d) B completed the work in week 15 8. How much distance is between the boat which
e) E and C completed the OP ward work before docks second to the right of S and the boat
week 8 which docks second from the right end?
a) 228m
Directions (6-10) Study the following information b) 114m
carefully and answer the given questions. c) 162m
A certain number of boats are docking in a row d) 240m
facing north. The distances between the adjacent e) 300m
boats are a consecutive multiple of 6 from the left
end. The distance between none of the boats is 9. Four of the following five are alike in a certain
more than 70m. Boat B docks 54m to the right of way based on the given arrangement and thus
G. Only three boats are docking between boat G form a group. Which one of the following does
and boat S. The number of boats docking not belong to the group?
between boat S and boat K is one less than the a) Boat G and Boat S
number of boats docking to the right of boat K. b) Boat J and Boat which docks immediate left of
Boat T docks 180m away from K. As many boats Boat K
dock to the left of boat G as to the right of boat J, c) Boat K and Boat S
which docks adjacent to boat T. d) Boat T and Boat K
6. How many boats are docked in the row? e) Boat G and Boat B
a) Fifteen
b) Thirteen 10. Which of the following statements is/are not
c) Twelve true as per the given arrangement?
d) Fourteen a) Boat S is 74m away from Boat G

Click Here For Bundle PDF Course | support@guidely.in Page 2 of 9


SBI Clerk & RRB PO Mains PDF Course 2023
Reasoning Ability Day - 20 (Eng)

b) Boat K docks exactly between boat T and boat d) Both A and B


S (not considering distance) e) Both A and C
c) Boat B docks fifth to the left of K.

Directions (11-15): Study the following information carefully and answer the given questions.
A shelf in a bakery is divided into 25 small shelves viz. 5 Rows and 5 Columns. Different types of cakes
are placed on the shelves. Different quantities of cakes are there on the shelves i.e. Vanilla-15 kg,
Strawberry-13 kg, Pista- 12 kg, Chocolate- 11 kg and Red velvet- 18 kg. You have to fill the shelf in which
the same cake should not be repeated in a particular row and a particular column. Some of the cakes are
filled in the diagram given below. One of the Red velvet cakes is not placed in Row-1 and Column-1.

11. Which of the following cake is placed in Row- 13. Four of the following five are alike in a certain
4 of Column-3? way based on the given arrangement and thus
a) Red velvet form a group. Which one of the following does
b) The cake which is also placed in Row-2 of not belong to the group?
Column-4 a) Row-2, Column-3
c) The cake which has 11 kg b) Row-4, Column-1
d) The cake which has 15 kg c) Row-1, Column-2
e) None of these d) Row-3, Column-4
e) Row-5, Column-4
12. What will be the sum of the quantities of
cakes on all shelves in Column-4 and the first 14. If Pista cake is related to Row-1, Chocolate
and last shelves in Column-5? cake is related to Row-2 in a certain way, then
a) 87 kg which of the following cakes are related to Row-5
b) 96 kg and Row-3 respectively?
c) 94 kg a) Red velvet cake and Pista cake
d) 84 kg b) Vanilla cake and Pista cake
e) 95 kg c) Strawberry cake and Pista cake

Click Here For Bundle PDF Course | support@guidely.in Page 3 of 9


SBI Clerk & RRB PO Mains PDF Course 2023
Reasoning Ability Day - 20 (Eng)

d) Chocolate cake and Vanilla cake Condition 5: If an even number is followed by a


e) None of the above non-prime odd number, then multiply the first
digits of each number’s square value.
15. Which of the following statements is/are true Condition 6: If an odd number is followed by an
with respect to the final arrangement? even number which is not a multiple of 4, then
a) The square value of the quantity of cake which the larger number is divided by the smaller
is placed in Row-3 of Column-4 is 144 number and the quotient is taken as a resultant.
b) Strawberry cake is placed immediately below 16. What is the square root of the sum of the
Row-2 of Column-2 resultant of both rows?
c) Vanilla and Pista cakes are placed next to 15 23 57
each other in Row-2. 19 22 46
d) Both A and C a) 7
e) None of these b) 8
c) 4
Directions (16-20): Study the following d) 6
information carefully and answer the given e) None of these
questions.
There are two rows in each question. To find out 17. What is the product of the square value of
the results of a particular row, the below the resultant of both rows?
conditions are to be followed. The operations of 26 16 33
numbers progress from left to right. 27 12 62
Condition 1: If an even number is followed by a a) 100
non-prime even number, then both numbers are b) 169
to be added. c) 144
Condition 2: If an even number is followed by a d) 121
prime number, then both numbers are multiplied. e) 131
Condition 3: If an odd number is followed by an
even number which is a multiple of 4, then that 18. If X is the resultant of Row 1, then what will
even number and the square value of that odd be the product of the resultants of both Rows?
number are to be added. 37 13 28
Condition 4: If an odd number is followed by 51 22 X
another odd number, then subtract both a) 2808
numbers. (Ignore the negative sign) b) 2818
c) 2718

Click Here For Bundle PDF Course | support@guidely.in Page 4 of 9


SBI Clerk & RRB PO Mains PDF Course 2023
Reasoning Ability Day - 20 (Eng)

d) 2708 e) 375
e) None of these
20. What is the sum of the resultant of both
19. What is the difference between the resultants rows?
of both rows? 25 56 79
42 11 24 12 03 07
23 72 54 a) 754
a) 485 b) 845
b) 465 c) 814
c) 425 d) 744
d) 475 e) 854
Click Here to Get the Detailed Video Solution for the above given Questions
Or Scan the QR Code to Get the Detailed Video Solutions

Answer Key with Explanation

Directions (1-5):
1. Answer: B
2. Answer: E
 The work allocated in OP ward will be
3. Answer: E
completed consecutively for three weeks
4. Answer: D
and the work allocated in ICU ward will be
5. Answer: C
completed consecutively for two weeks
We have,
whereas the rest are completed in one
 In the first week A will work in ER ward
week.
and B will work in General ward and so on
 The work allocated to Surgical ward will
as in alphabetical order. The work will be
be followed by the work allocated to ER
repeated in alphabetical order only.
ward and the cycle is repeated. Each person

Click Here For Bundle PDF Course | support@guidely.in Page 5 of 9


SBI Clerk & RRB PO Mains PDF Course 2023
Reasoning Ability Day - 20 (Eng)

works in each department only once. If for a  Boat B docks 54m to the right of G.
person, no chief nurse is available for a  Only three boats are docking between
particular week (either because he/she has boat G and boat S.
already done all the works or he/she cannot
do the available works or the work that
he/she has to do is already doing by the
person who is before him/her in the
alphabetical order), then that person works
in Cardiac ward.

Again we have,
 The number of boats docking between
boat S and boat K is one less than the
number of boats docking to the right of
boat K.
Directions (6-10):  Boat T docks 180m away from K.
6. Answer: C Applying the above conditions, Case 2
7. Answer: D and Case 3 are eliminated because there
8. Answer: C is no possibility to place boat T.
9. Answer: A(Only two boats dock between the
given pair of boats except option a)
10. Answer: E
Final Arrangement:

We have,

Click Here For Bundle PDF Course | support@guidely.in Page 6 of 9


SBI Clerk & RRB PO Mains PDF Course 2023
Reasoning Ability Day - 20 (Eng)

We have,
Same cake should not be repeated in a
particular row and a particular column.(5
different cakes should be filled in all rows and
columns)
 First we fill Column-5. In Column-5 there are
two empty shelves. Chocolate cake is placed in
Again we have,
Row-1 of Column-4.
 As many boats dock to the left of boat G
 Applying the above Rules, Vanilla cake is
as to the right of boat J, which docks
placed in Row-1 of Column-5 and Chocolate
adjacent to boat T.
cake is placed in Row-5 of Column-5.
Applying the above conditions, Case 1 is
Thus, Column-5 is filled.
eliminated because there is no possibility
 One of the Red velvet cakes is not placed in
to place J. Hence, Case 1a gives a final
Row-1 and Column-1. So, the Red velvet cake
arrangement.
is placed in Row-1 of Column-3.
 Remaining place of Row-1 was filled by Pista
Cake.

Directions (11-15):
11. Answer: C  Next, we fill Column-3. To fill Row-2 of

12. Answer: E Column-3, all cakes are there in Row-2 and

13. Answer: D(same cake placed on the given Column-3 together except the Strawberry cake.

shelves except option d) So we placed the Strawberry cakes in Row-2 of

14. Answer: D Column-3.

15. Answer: A  Vanilla cake is placed in Row-5 of Column-2.

Final arrangement: So we can place the Vanilla cake in Row-3 of

Click Here For Bundle PDF Course | support@guidely.in Page 7 of 9


SBI Clerk & RRB PO Mains PDF Course 2023
Reasoning Ability Day - 20 (Eng)

Column-3 and the Pista cake in Row-5 of


Column-3.

 Then, we can fill Row-4 and Row-5. We have


all three cakes except Strawberry cake and

 Then, we fill Row-2. We have all three cakes Vanilla cake. Vanilla cake is placed Row-2 and

placed in Row-2 except Chocolate cake and Column-1. So we can place Strawberry cake in

Red velvet cake. Chocolate cake is placed in Row-4 of Column-1 and Vanilla cake in Row-4 of

Row-1 of Column-4. So we can place Chocolate Column-4.

cake in Row-2 of Column-2 and Red velvet  Remaining cakes are placed in Row-5

cake in Row-2 of Column-4. accordingly.

 Now, we can fill Coloumn-2. We have all three


cakes placed in Column-2 except Pista cake and
Red velvet cake. Red velvet cake is placed in
Row-4 of Column-5. So we can place Red velvet
cake in Row-3 of Column-2 and Pista cake in
Row-4 of Column-
2 Directions (16-20):
16. Answer: B
In Row 1,
15 23 57, by condition 4, 23–15 = 8
8 57, by condition 5, 572= 3249 and 82= 64
3*6 = 18 is the resultant of row 1.
 Then, we can fill Row-3. We have all three
In Row 2,
cakes placed in Row-3 except Pista cake and
19 22 46, by condition 6, 22/19 = 1
Chocolate cake. Pista cake is placed in Row-1 of
1 46, by condition 6, 46/1 = 46 is the resultant of
Column-1. So we can place Pista cake in Row-3
row 2.
of Column-4 and Chocolate cake in Row-3 of
Sum: 18+46 = 64
Column-1.
√64 = 8

17. Answer: D
In Row 1,

Click Here For Bundle PDF Course | support@guidely.in Page 8 of 9


SBI Clerk & RRB PO Mains PDF Course 2023
Reasoning Ability Day - 20 (Eng)

26 16 33, by condition 1, 26+16 = 42 19. Answer: D


42 33, by condition 5, 422= 1764 and 332= 1089 In Row 1,
1*1 = 1 is the resultant of row 1. 42 11 24, by condition 2, 42*11 = 462
In Row 2, 462 24, by condition 1, 462+24 = 486 is the
27 12 62, by condition 3, 729+12 = 741 resultant of row 1.
741 62, by condition 6, 741/62 = 11 is the In Row 2,
resultant of row 2. 23 72 54, by condition 3, 529+72 = 601
12 * 112 = 121 601 54, by condition 6, 601/54 = 11 is the
resultant of row 2.
18. Answer: A Difference: 486-11 = 475
In Row 1,
37 13 28, by condition 4, 37-13 = 24 20. Answer: E
24 28, by condition 1, 24+28 = 52 is the resultant In Row 1,
of row 1. 25 56 79, by condition 3, 625+56 = 681
X = 52 681 79, by condition 4, 681-79 = 602 is the
51 22 X resultant of row 1.
51 22 52, by condition 6, 51/22 = 2 In Row 2,
2 52, by condition 1, 2+52 = 54 is the resultant of 12 03 07, by condition 2, 12*3 = 36
row 2. 36 07, by condition 2, 36*7 = 252is the resultant
Product: 52*54 = 2808 of row 2.
Sum: 602+252 = 854

Click Here For Bundle PDF Course | support@guidely.in Page 9 of 9


SBI Clerk & RRB PO Mains PDF Course 2023
Quantitative Aptitude Day - 20 (Eng)

Quantitative Aptitude

Directions (1-5): Study the following information carefully and answer the questions given below.
The given table shows the number of minutes of workouts by five persons [A, B, C, D, and E] on five days
of a week and the total number of minutes of workouts for five days for each person also given.

1) On Monday,the ratio of time spent by A onthe more than the total workout time of D and E on
plank and jumping jack was 5:4 out of his total Thursday and Friday together.
time working out. The next day, he reduced his Find which of the following value is satisfy the
time on the plank by 20%, and the rest of the blank?
time, he did jumping jacks. The ratio of time a) 85,3
spent doing jumping jacks on Wednesday and b) 88, 8
Tuesday is 1:2. Find the time spent by A to do c) 81, 6
plank on Wednesday? [A only did these two d) 89, 9
excise in these three days.] e) 89, 0
a) 4
b) 3 3)
c) 6 Quantity I: The Total workout time of all the
d) 2 people together on Monday is what percent of
e) 9 the total workout time of all the people together
on Tuesday?
2) Total workout time of B and C on Monday and Quantity II: The total workout time of all the
Tuesday together is _______ which is _______ people together on Thursday is what percent of

Click Here For Bundle PDF Course | support@guidely.in Page 1 of 10


SBI Clerk & RRB PO Mains PDF Course 2023
Quantitative Aptitude Day - 20 (Eng)

the total workout time on Wednesday of all the a) 7z minutes


people together? b) 5x+m+2 minutes
a) Quantity: I Quantity: II c) 8y minutes
b) Quantity: I > Quantity: II d) 13m+2 minutes
c) Quantity: I < Quantity: II e) None of these
d) Quantity: I ≤ Quantity: II
e) Quantity I = Quantity II 5) Find the value of 4.5x+8.5y+6.3z+9.3m=?
a) 126.6
4) Average time for the workout of C on Friday b) 124.32
and Saturday is 17 min and the average time for c) 124.25
the workout of D on Friday and Saturday is 24 d) 125.2
min. If the average time of workout of C, D, and e) None of these
E on Saturday is 23 min then find the workout
time of E on Saturday?

Directions (6 - 9): Study the following information carefully and answer the questions given below.
The given Radar graphs show the average number of employees in HR and IT, IT and accounts, and
Accounts and HR departments of five different companies [A, B, C, D, and E].

6)The Ratio of male and female employees in employee accounts departments of company A
the HR departments of company A is 3:2, and is 4:5. Find the number of males in all
the ratio of male and female employees in the IT department is what percent of the number of
department of company A is 3:4. Ratio of the female employees in all department company A?
number of female employees in the IT to male a) 125%

Click Here For Bundle PDF Course | support@guidely.in Page 2 of 10


SBI Clerk & RRB PO Mains PDF Course 2023
Quantitative Aptitude Day - 20 (Eng)

b) 124% 9) If the average of number employees in the IT


c) 110% department of companies E and F is 35. The
d) 159% number of employees in the HR department of
e) 124% company F is 20% less than thatof in IT
department of company F. If the total number of
7) 20% and 30% of the number of employees in employees of company F is 20% more than the
the HR and IT department respectively of same in company E then find the number of
Company E use private transport and the rest employees in the accounts department of
use public transport. If the average public company F?
transport use employee is 16 then find the total a) 54
number of employees who use private transport b) 52
of company E? c) 58
a) 42 d) 51
b) 37 m e) None of these
c) 55
d) 46 10) A completes the work in (2x+6) days, and B
e) 67 completes the work in (3x+9) days. and C
complete the work in (5x+5) days. The ratio of
8) Average number of employees in the efficiency of A and C is 2:1. Total wage of the
marketing department and accounts department work is Rs. 360x. IF A and B complete the whole
of company B is 20 and the average number of work then find the difference between the total
employees in the IT and marketing department of wage of A and B?
company C is 26. If the total number of a) 72x
employees in the marketing department of b) 74x
companies B,C, and D is 52 then find the ratio of c) 76x
the number of employees in marketing and d) 79x
accounts department of department of D? e) None of these
a) 2:3
b) 2:5 Directions (11-14): Study the following
c) 5:4 information carefully and answer the questions
d) 3:2 given below.
e) None of these Four batsmen [A, B, C, and D] hit different
numbers of 4's and 6's in a tournament. The ratio
of the number of 6's hit by A and B is 2:3.

Click Here For Bundle PDF Course | support@guidely.in Page 3 of 10


SBI Clerk & RRB PO Mains PDF Course 2023
Quantitative Aptitude Day - 20 (Eng)

Number of 4's hit by D is 20% more than the 2's and none 3 then finds the total runs scored by
number of 6's hit by him. The ratio of the number E?
of 6's hit by D and C is 5:4. B scored a total of a) 945
560 runs in boundary [4's and 6's] Number of 6's b) 923
hit by C is 16.67% less than the number of 4's hit c) 965
by the same person. The number of 6's hit by B d) 982
is the same as the number of 4's hit by C. The e) None of these
Ratio of the number of 4's hitby A and B is 1:2. A
scored a total of 820 runs out of these 40% runs 14) Find the difference between the total runs
in boundary [ by 4's and 6's]. score by B and D together in boundary and the
11) C scored 75% of runs from the boundary and total runs score by A and C together by
D scored 60% of runs from the boundary. If C boundary?
scores 25% runs by 2's and D scores 20% runs a) 360
by 2’s then find the difference between the b) 320
number of 2’s taken by C and D? c) 340
a) 12 d) 380
b) 14 e) None of these
c) 18
d) 16 15) Cost price of item A is Rs. (x+80) and the
e) 20 cost price of item B is Rs. (2x+60). Item A is
marked up by 40% and then sold at a 10%
12) Ratio of the number of 4's hit by C and F is discount and item B is marked up by 60% and
12:11. Ratio of the number of 6's and 4's hit by F then sold at a 25% discount. The difference in
is 1:2. IF F scores 40% runs in boundary and selling price of items B and A is Rs.108. Find the
ratio of 1's, 2's, and 3's is 81:45:20. Find the total sum of the selling price of both items, if it is sold
runs scored by F by 1's? at (x/6)% profit?
a) 182 a) 7x
b) 324 b) 8x
c) 243 c) 9x
d) 162 d) 11x
e) 172 e) None of these
13) E scored 112 runs more in boundary than A.
The ratio of 4's and 6's hit by E is 5:4. Ratio of 16) Ratio of milk and water in container A is 5:3
4's and 1's of E is 2:17 and he takes twenty-nine and the ratio of milk and water in container B is

Click Here For Bundle PDF Course | support@guidely.in Page 4 of 10


SBI Clerk & RRB PO Mains PDF Course 2023
Quantitative Aptitude Day - 20 (Eng)

3:4. Sum of water and milk in containers A and B b) only II


together is 78 liters and 86 liters respectively. c) only I and II
Find the ratio of milk and water in container C if d) only I and III
40% of the mixture of container A and 25% e) only III
mixture of container B is mixed in container C?
a) 21:23 Directions (18-20): Study the following
b) 24:29 information carefully and answer the questions
c) 29:24 given below. There are three rows, calculate the
d) 22:27 roots of the equation and compare them with
e) 26:29 other columns. Check which one matches.
18)
17) In the below image, there is a thief at point A,
and there are police at point E. At point C, there
is an exit point. Distances AB, AD, and BD are
30 km, 29 km, and 40 km, respectively. C is the
midpoint of BD. The distances of EF, EB, ED,
a) iii)-c)
and FB are 50 km,60 km, 40 km, and 30 km,
b) i)-b)
respectively. The speed of police is 25 km/h, and
c) i)-c) & ii) -c)
the speed of thieves is 15 km/h. They start
d) ii)-b)
moving at the same time. Find which of the
e) None of these
following ways the police cannot catch the thief:
I. Police flow EFBC path and Thief flow ABC
19)
path.
II. Police flow EBC path and Thief flow ADC
path.
III. Police flow EDC path and Thief flow ABC
path.

a) ii)-c)
b) i)-b)
c) iii)-b) & ii) -b)
d) iii)-c)
e) None of these

a) only I

Click Here For Bundle PDF Course | support@guidely.in Page 5 of 10


SBI Clerk & RRB PO Mains PDF Course 2023
Quantitative Aptitude Day - 20 (Eng)

20) 21)
Quantity I:
Quantity II:
a) Quantity: I Quantity: II
b) Quantity: I > Quantity: II
a) ii)-b) c) Quantity: I < Quantity: II
b) i)-b) d) Quantity: I ≤ Quantity: II
c) i)-a) & ii) -b) e) Quantity I = Quantity II
d) iii)-a)
e) None of these
Click Here to Get the Detailed Video Solution for the above given Questions
Or Scan the QR Code to Get the Detailed Video Solutions

Answer Key with Explanation

Directions (1-5): Or, 9m=106-9*5-3*6-4*4=27


From the table we can say, So, m=3
9x+9z+3y+2=101 ---i
8x+12y=112, or, 2x+3y=28 --ii
14z+9x=102-1=101-----iii
From ii we have value of 3y and if put the value
of 3y in I then we get ‘
9x+9z+28-2x+2=101
Or, 7x+9z=71 ---iv
So, if we solve iii and iv then we get x=5 and z=4 1. Answer: C
So, y=[28-2*5]/3=6 on Monday, plank time = 27*5/9=15, jumping
Now, 9m+9x+3y+4z=106 jack time = 27-15=12

Click Here For Bundle PDF Course | support@guidely.in Page 6 of 10


SBI Clerk & RRB PO Mains PDF Course 2023
Quantitative Aptitude Day - 20 (Eng)

on, Tuesday plank time = 15*80/100=12 5. Answer: A


jumping jack time = 30-12=18 4.5x+8.5y+6.3z+9.3m
on, Wednesday, jumping jack time = 18*1/2=9 =4.5*5+8.5*6+6.3*4+9.3*3=126.6
plank time = 15-9=6
Directions (6 - 9):
2. Answer: D For company A,
B and C on Monday and Tuesday together work The total number of employees in the HR and IT
for department is = 30*2=60
=20+20+24+25=89 minutes The total number of employees in the HR and
D and E on Thursday and Friday together work Accounts department is = 35*2=70
for The total number of employees in the IT and
=24+28+16+12=80 minutes accounts department is = 40*2=80
So, the difference is 89-80=9 So, the number of employees in the Accounts
So, option D is true department is
=[60+70+80]/2-60=45
3. Answer: B So, the number of employees in the IT
Quantity I department is
Required percentage = =[60+70+80]/2-70=35
[(27+20+24+21+28)/(30+20+25+24+21)]*100 So, the number of employees in the HR
=100% department is
Quantity II =[60+70+80]/2-80=25
Required percentage = Similarly, we can calculate the value of other
[(18+25+18+24+16)/(15+20+30+24+25)]*100 companies also.
=88.59%
Quantity I > Quantity II
4. Answer: B
The Workout time of C on Saturday is =17*2-
15=19
The Workout time of D on Saturday is =24*2-
6. Answer: C
28=20
The number of males in the HR department is =
The Workout time of E on Saturday is = 23*3-20-
25*3/5=15
19=30=5x+m+2
The number of females in the HR department is
= 25-15=10

Click Here For Bundle PDF Course | support@guidely.in Page 7 of 10


SBI Clerk & RRB PO Mains PDF Course 2023
Quantitative Aptitude Day - 20 (Eng)

The number of males in the IT department is = =90*120/100=108


35*3/7=15 So, the number of employees in the accounts
The number of females in the IT department is = department of the company is
35*4/7=20 = 108-24-30=54
The number of males in the Accounts
department is = 20*5/4=25 10. Answer: A
So, required percentage = So, we can say, =[2x+6]/[5x+5]= ½
[(15+15+25)/(10+20+20)]*100=110% Or, 4x+12=5x+5
7. Answer: D Or, x=7, ṇ
Number of employees of accounts department So, A, B, and C can complete the work in
use private employees is = [30-[16*3-20*80/100- [2*7+6]=20, [3*7+9]=30, and [5*7+5]= 40 days
40*70/100]=[30-4]=26 respectively.
So, the total number of employees who use Total work is = 120
private transport of Company E is = So, the efficiency of A and B is 6 and 4
26+20*20/100+40*30/100=26+4+12=42 respectively.
They complete the work in [120/(6+4)]=12 days.
8. Answer: B Total wage is = 360*7=2520
The number of employees in the marketing So, difference of wage is =
department of company B is = 20*2-22=18 [2520/120]*24=Rs.504=72x
The number of employees in the marketing
department of company C is = 26*2-32=20 Directions (11-14):
The number of employees in the marketing Let the number of 4's hit by A and B be x and 2x.
department of company D is The number of 6's hit by A and B is 2y and 3y.
= 52-18-20=14 So, [2x]*4+[3y]*6=560, 4x+9y=280 ------ (i)
So, the required ratio = 14:35=2:5 For, A, 4*x+[2y]*6=820*40/100=328, x+3y=82 ---
 (ii)
9. Answer: A If we solve (i) and (ii) then x=34 and y=16
The number of employees in the IT department the number of 4's hit by A and B is 34 and 68
of company F is 35*2-40=30 respectively.
The number of employees in the HR department the Number of 6's hit by A and B is 32 and 48
of company F is = 30*80/100=24 respectively.
Total number of employees in company F is = The number of 4's hit by C is 48.
[20+30+40]*120/100

Click Here For Bundle PDF Course | support@guidely.in Page 8 of 10


SBI Clerk & RRB PO Mains PDF Course 2023
Quantitative Aptitude Day - 20 (Eng)

The number of 6's hit by C is 48*[100- Total runs scored by E is = 425+440+29*2=923


16.67]/100=40
The number of 6's hit by D is =40*5/4=50 14. Answer: A
The number of 4's hit by D is = 50*120/100=60 Required difference
11. Answer: C = [68*4+48*6+60*4+50*6]-
Total runs scored by C = [34*4+32*6+48*4+40*6]=340
[48*4+40*6]*100/75=576
So, total runs scored by C in 2's = 15. Answer: E
576*25/100=144 So, [2x+60]*[160/100]*[75/100] –
So, the number of 2's hit by C=144/2=72 [x+80]*[140/100]*[90/100]=108
Total runs scored by D = Or, 1.2*2x+60*1.2-1.26x-80*1.26=108
[60*4+50*6]*100/60=900 Or, 1.14x=108+80*1.26-60*1.2=136.8
So, total runs scored by D in 2's = Or, x=136.8/1.14=120
900*20/100=180 So, the cost price of item A is = 120+80=200
So, the number of 2's hit by D=180/2=90 The cost price of item B is = 120*2+60=300
So, difference = 90-72=18 So, the profit percent is = 120/6=20
So, the total selling price when the item sold at
12. Answer: D 20% profit
The number of 4's hit by F is 48*11/12=44 =200*120/100+300*120/100=240+360=600=5x
The number of 6's hit by F is = 44/2=22
Total runs scored by F without boundary is = 16. Answer: C
[44*4+22*6]*60/40=462 letmilkand water in container A be 5x ,3x
So, number of runs score by 1’s = let milkand water in container B be 3y ,4y
[462*81/(81*1+45*2+20*3)]=162 So, 5x+3y=86 and 3x+4y=78
If we solve the above two equations, we get x
13. Answer: B =10 and y=12
Runs score by A in the boundary is So,milk in A is 5*10=50 and water is 3*10=30.
=34*4+32*6=328 Milk in B is 3*12=36 and water is 4*12=48
Runs score by E in the boundary is = So, milk in C is =
328+112=440 50*40/100+36*25/100=20+9=29
The number of 4's and 6's hit by E is 5z and 4z. Water in C is = 30*40/100+48*25/100=12+12=24
So, 20z+24z=440, z=10 So, the required ratio = 29:24
The number of 1's taken by E is = 50*17/2=425

Click Here For Bundle PDF Course | support@guidely.in Page 9 of 10


SBI Clerk & RRB PO Mains PDF Course 2023
Quantitative Aptitude Day - 20 (Eng)

17. Answer: A
I. Police flow EFBC path and Thief flow ABC
path.
So, total distance covered by police =
50+30+20=100
Time taken = 100/25=4
Total distance covered by thief = 30+20=50 19. Answer: C
Time taken = 50/15=3.3
So, cannot catch
II. Police flow EBC path and Thief flow ADC
path.
Time taken by police = [60+20]/25=3.2
Time taken by thief = [29+20]/15=3.26
So, police can catch the thief
20. Answer: D
III. Police flow EDC path and Thief flow ABC
path.
Time taken by police = [40+20]/25=2.4
Total distance covered by thief = 30+20=50
Time taken = 50/15=3.3
So, police can catch the thief
21. Answer: C
Quantity I
18. Answer: C
Quantity II
Quantity I < Quantity II

Click Here For Bundle PDF Course | support@guidely.in Page 10 of 10


SBI Clerk & RRB PO Mains PDF Course 2023
ENGLISH Day - 20

English Language

Directions (1-5): In each of these questions, a Earth’s rotational pole tends to wander in a
paragraph is given that has a blank in it. Out of circular pattern several metres wide every year
the given options, only one sentence fits in with due to the weather, season changes, the molten
the context of the paragraph. Select that as your core and even hurricanes. Scientists have been
answer. able to track this motion by looking at
1) A 26.7% boom in services exports last year astronomical phenomena such as centres of
had helped narrow the steep goods trade and bright galaxies or quasars by comparing their
current account deficits amid surging global changing positions in the sky as the Earth’s axis
prices of commodities such as oil and fertilisers shifted.
whose imports are inelastic for India. The trend a) Unlike a globe which has a fixed axis and
reversal in that pace of growth began this March rotates stably the Earth’s axis wobbles.
and has accelerated to a critical point with a b) Polar motion is the main concept behind
mere 0.7% rise in global services receipts in earth’s rotation.
May. ___________________. c) Crust here refers to the outermost shell of the
a) However the rise is negligible and India has to earth surface.
follow the US and China for recovery. d) The earth’s axis is also stable like the fixed
b) These studies show that India has to boost its axis of the globe.
service sector. e) None of the above
c) But India need not worry as service sector
contribution is very little towards the overall 3)The sacred grove of the Manikavu Sree
growth. Mahadeva temple in Wayanad under the
d) The global slowdown that had clearly hit Malabar Devaswom Board has set a model in
consumer demand for products, now appears to transforming barren land into a ‘forest’ with the
be infecting the appetite for services too. addition of rare endemic and threatened (RET)
e) None of the above plant species of the Western Ghats.The model
has been set up by the temple authorities with
2) The Earth’s rotational pole is the point along the support of M.S. Swaminathan Research
which the planet rotates. This point, also called Foundation (MSSRF), the Social Forestry and
the axis of the Earth, moves in a process called Soil Conservation departments and the
polar motion which is when the Earth’s pole Meenangadi grama panchayat.More than 29
varies relative to the crust. ____________.It is acres of land are now under the canopy of nearly
more like a spinning top gone off-kilter where the

Click Here For Bundle PDF Course | support@guidely.in Page 1 of 12


SBI Clerk & RRB PO Mains PDF Course 2023
ENGLISH Day - 20

4,000 plants that belong to more than 50 b) Google has depicted a Doodle dedicated to
species.____ _____. Sohonie, celebrating her 112th anniversary.
a) Wayanad is one of the districts in God’s own c) Biochemistry deals with the chemical
country Kerala. processes that are related to all the living
b) M.S. Swaminathan is the father of the green organisms.
revolution in India. d) Gender bias is when a person faces
c) The temple is known for its biodiversity as it advantages or disadvantages based on their
lies on the lap of lush green woods and water gender.
flows perennially on the deity from a stream e) None of the above
through a stone channel which is a major
attraction of the temple. 5) Earlier on May 19, RBI decided to withdraw
d) All these grama panchayats are covered the ₹2,000 denomination currency notes from
under the model. circulation, but said it will continue to remain as
e) None of the above legal tender. However, RBI has advised banks to
stop issuing ₹2,000 denomination banknotes
4) Google on June 18 paid tribute to an Indian with immediate effect. People can exchange or
biochemist and scientist Kamala Sohonie, who deposit their ₹2,000 notes in bank branches and
did pioneering work in her field of biochemistry regional branches of
and helped forge a path for future Indian women RBI.___________.September 30 has been
to overcome gender bias and pursue their decided as the last date for the purpose of
dreams.______ ____.Sohonie was born in Indore completing the exercise in a time-bound manner
on this day in 1911. Her father and uncle were and to provide adequate time to the public.
chemists and alumni of the erstwhile Tata a) Regional branches of RBI are limited in almost
Institute of Sciences (which later became the all the cities.
Indian Institute of Science) in Bengaluru.Inspired b) ₹2,000 can still be used because it remains as
by her family, Sohonie studied chemistry and a legal tender.
physics at Bombay University and graduated at c) A non-account holder also can exchange
the top of her class in 1933. She became the first ₹2000 banknotes up to a limit of ₹20,000 at a
woman to be inducted into the IISc but faced time at any bank branch
issues because its director Sir C.V. Raman d) People are not completely aware of how these
doubted the capabilities of women in science. decisions of RBI are going to affect their daily
a) Women face many problems while finding a life.
good career and one of them is gender e) None of the above
discrimination.

Click Here For Bundle PDF Course | support@guidely.in Page 2 of 12


SBI Clerk & RRB PO Mains PDF Course 2023
ENGLISH Day - 20

Directions (6-10): In each of the following 8) HALLUCINATE


questions, a highlighted word is given followed I. Sherin began hallucinating during the exam
by three sentences. Choose the sentence(s) that and eventually got fits.
has/have the wrong/inappropriate usage of the II. It is well known that the persons who use
highlighted word. drugs often hallucinate things.
6) ACCENT III. Doctor said that she is hallucinating pain as
I. Reena has a very good english accent the test results were all good.
however she always feels shy to hold a mike and a) Only I
talk on stage. b) Only II
II. The little boy completed his accent to the c) Both I and II
mountain Everest and made a Guinness world d) Both I and III
record as expected. e) All correct
III. The actress speaks many different accents
and this has been one of her favourite hobbies 9) DAMPEN
since her childhood. I. The excitement and enthusiasm of our road trip
a) I and II was dampened because of the authority’s
b) Only II strange behaviour.
c) II and III II. Do not try to dampen your feelings completely
d) Only III instead feel them and realise.
e) All correct III. All the clothes were dampen due to the rain
before they could be sun dried.
7) THROB a) Only I
I. My brother has sinusitis and whenever he gets b) Only II
a headache it is always a throbbing pain. c) Both I and II
II. When I saw my final results my heart literally d) Only III
throbbed with severe pain. e) All correct
III. The car’s music system is amazing and any
music lover would like it, as the bass throbs. 10) TEDIOUS
a) Only I I. It was a tedious meeting at the office today.
b) Only II II. Her plans started to become very tedious and
c) Only III tiring as the date approached.
d) Both I and III III. It was tedious a long journey from Mysore to
e) All correct the capital.
a) Only II

Click Here For Bundle PDF Course | support@guidely.in Page 3 of 12


SBI Clerk & RRB PO Mains PDF Course 2023
ENGLISH Day - 20

b) Only III
COLUMN 1 COLUMN 2
c) Both I and II
d) Both I and III A. whether D. affect
e) All correct
B. is E. arrive

Directions (11-15): In each question below, a C. of F. impinge


sentence followed by a table with two columns
a) A-E
has been given. The sentence has got two
b) B-D
blanks that would take words from the columns
c) A-D
respectively. Choose the correct combination of
d) A-F
words that complete the given sentences
e) C-E
meaningfully.
11) Engineering goods that _______ over a
13) Sea accidents are common in the Philippine
quarter of India’s goods export basket contracted
archipelago because of frequent storms, badly
for the 11th month in a row, while the
________vessels, overcrowding and spotty
employment-intensive textiles sector shrank for
enforcement of safety regulations, _______in
the seventh ________month.
remote provinces.
COLUMN 1 COLUMN 2
COLUMN 1 COLUMN 2

A. grow D. straight
A. painted D. around

B. make-up E. cross
B. built E. with

C. got F. time
C. maintained F. especially
a) A-D
a) A-F
b) B-E
b) C-F
c) B-D
c) B-E
d) C-D
d) B-D
e) C-F
e) C-E
12) A uniform civil code for the entire country is
14) While DDoS attacks are mainly a nuisance,
indeed a lofty goal, but the question
making websites __________without penetrating
________introducing one for all aspects of
them and security experts say they can
personal law would _______on the freedom of
_______the work of millions if they successfully
religion has been part of the debate.
interrupt the services of a software service giant

Click Here For Bundle PDF Course | support@guidely.in Page 4 of 12


SBI Clerk & RRB PO Mains PDF Course 2023
ENGLISH Day - 20

like Microsoft on which so much global highlighted as they might’ve been placed at the
commerce depends. wrong positions. One of these words might also
be incorrect and need a replacement. Read the
COLUMN 1 COLUMN 2
same carefully and mark the appropriate option
A. unreachable D. allow as the answer.
16) In a account (A) to CEOs of all general
B. operate E. disrupt
insurance companies, and standalone health
C. shuting F. enhance insurance companies, the Insurance Regulatory
and Development Authority of India (IRDAI) said
a) A-D
it needs to be ensured (B) that all claims are
b) B-D
surveyed immediately and claim on circular (C)
c) A-F
payments are disbursed at the earliest and in any
d) C-F
case not within (D) the stipulated timeline.
e) A-E
a) A-C, B-assured
b) A-C, C-circulation
15) In 2017, the Supreme Court of India
c) B-D, B-ensure
recognised privacy as a fundamental right,
d) A-C, D-exceeding
____________the need to protect personal
e) No swapping needed, D-exceeding
information but the country is still ___________to
frame a personal data protection policy.
17) Studies validated (A) by researchers at the
COLUMN 1 COLUMN 2
University of Virginia School of Medicine, U.S.,

A. highlighted D. struggled and Uppsala University, Sweden, together have


showned (B) that Loss of chromosomes(LoY) in
B. highlighting E. struggles humans occurs with age and is associated with
several debilitating medical conditions, a finding
C. highlight F. struggling
(C) that has been conducted(D)in mice with LoY,
a) B-F
resulting in weak heart muscles , stretched or
b) A-F
thickened heart tissue and heart failure.
c) C-E
a) A-D, B-shown
d) B-E
b) B-D, B-shown
e) A-D
c) B-C, no replacement
d) A-C, D-validation
Directions (16-20): The following questions given
e) No swapping needed, A-conduct
below have four words that have been

Click Here For Bundle PDF Course | support@guidely.in Page 5 of 12


SBI Clerk & RRB PO Mains PDF Course 2023
ENGLISH Day - 20

18) The decision to confer (A) the award on Gita a) A-D, no replacement needed
Press was taken by a jury headed by Prime b) C-D, A-been
Minister Narendra Modi after due c) A-C, D-there
(B)deliberations on Sunday in recognition of the d) B-C, C-enquire
publishing house’s “outstanding contribution e) No correction needed
towards (C) social, economic and political
transformation throw (D) non-violent and other 20) Issues such as developing a comprehensive
Gandhian methods”, the Culture Ministry said in approach towards (A)national security, pushing
a statement. domestic defence production and those
a) A-D, A-confers pertaining to serving and retired armed forces
b) B-C, A-confers personal (B) will be discussed (C) at a
c) C-D, D-threw brainstorming (D)session beginning here on
d) A-C, C-toward Monday, the Defence Ministry said in a
e) No swapping needed, D-through statement.
a) A-B, A-toward
19) After a recent amendment of the Registration b) No swapping needed, B- personnel
Act, District Registrars have been (A) c) C-D, C- discussing
empowered under (B) Section 77(A) to inquire d) B-C, D- brainstorm
(C)into complaints of fraudulent or bogus e) No correction required
registrations and order their (D) cancellation.
Click Here to Get the Detailed Video Solution for the above given Questions
Or Scan the QR Code to Get the Detailed Video Solutions

Answer Key with Explanation

1) Answer: D

Click Here For Bundle PDF Course | support@guidely.in Page 6 of 12


SBI Clerk & RRB PO Mains PDF Course 2023
ENGLISH Day - 20

The paragraph talks about the boom in services wobbles during rotation. The following sentence
exports last year and the trend reversal this year. also gives more information on the wobbling
The rise in global services is very little this year. mechanism but comparing the axis to a spinning
This context must be followed with sentence ‘d’ top.
which gives a reason for the slowdown in the Other given sentences do not fit in the blank of
service sector. The global shutdown due to the paragraph because :
pandemic has affected the consumer demands Option b - Polar motion is the main concept
as well as the services. So, the best option to behind earth’s rotation - this sentence cannot be
choose is ‘d’ followed with ‘it is more like a spinning top’. What
Other sentences do not fit in the paragraph is compared to a spinning top ? The comparison
because: is with the axis of the earth and not the motion.
Option a - the sentence is inappropriate to the Option c - The detail about crust is an unwanted
context because the paragraph is revolving information here and hence this sentence does
around the exports and demands of India and it not fit the blank, because the before and after
does not contain any information regarding the sentences deals only with the axis and the
US and China, so this option is wrong as it is rotation.
involving US and China. Option d- It is wrong because if the axis is stable
Option b - ‘These studies’ there is no such it is not going to spin or wobble, it makes the
information on any study in the paragraph so whole paragraph meaningless, hence this is not
‘these’ is meaningless as it does not have the answer.
anything to denote.
Option c - is wrong because we all know that all 3) Answer: C
the sectors are important for the ultimate growth The main topic here is Manikavu Sree
of the country, the given sentence makes no Mahadeva temple in Wayanad that has setup a
sense and thus we find it inappropriate. model to transform barren land to forest
area.And the passage continues to provide
2) Answer: A information on the model and the authorities who
Clearly the correct answer is ‘option a’ because helped to set it up.The third sentence talks about
before the given blank the paragraph has the area and the species it has.So, the fourth or
information regarding the axis of the earth and the final sentence would be a description of the
how the earth rotates. ‘Option a’ talks about how area in and around the temple. Also among the
the earth’s axis is different from the globe’s axis given options, only option ‘c’ would best suit the
which is fixed and stable and earth's axis context.

Click Here For Bundle PDF Course | support@guidely.in Page 7 of 12


SBI Clerk & RRB PO Mains PDF Course 2023
ENGLISH Day - 20

Other sentences do not fit the blank because : Option d - Definition of gender bias is
Option a - this sentence provides information on unnecessary in the passage.
Wayanad which is not even the desired topic or
context. 5) Answer: C
Option b - The passage tells about the research The sentences that precede and succeed the
foundation with M.S.Swaminathan’s name but blank are talking about the exchange of 2000
the main content here is the temple and its rupees notes before the deadline.So,the
model. sentence that fits the blank should also revolve
Option d - only one grama panchayat has been around the same idea.The only sentence related
mentioned in the paragraph, ‘these’ do not fit the to the exchange is Option c. Hence, this would
given context. be the right pick.
4) Answer: B Other options do not fit the blank because :
The paragraph’s introductory sentence is about Option a - Does not talk about exchange instead
Google's tribute to the Indian biochemist and about the RBI branches.
scientist Kamala Sohonie. Out of the four given Option b - It is still a legal tender but RBI ordered
options ‘b’ would be the relevant sentence to the banks not to issue the notes further - this has
context of the passage as it gives more been discussed in the first few sentences and
information on the tribute that Google gave the the topic has switched to exchange of notes
Biochemist celebrating her 112th birth before the given deadline, this sentence will not
anniversary. be a continuity.
Other given options do not fit the blank because Option d - completely irrelevant to the context of
: the given paragraph.
Option a - The main idea of the topic is Kamala
Sohonie, her studies, her achievements and 6) Answer: B
Google’s honour for her. The given sentence ‘a’ The task is to find the wrong usage of the word
seems inappropriate as it is based on gender ‘accent’. Only sentence II is the right option as
discrimination which is not the main idea of the the other two given sentences have used the
passage. word correctly and appear to be correct and
Option c - Kamala Sohonie was a biochemist but meaningful sentences.
the passage is not about the stream Error is sentence II - Accent must be replaced
biochemistry and hence this sentence is not with ascent in this sentence.Accent - way of
relevant to the passage. pronouncing words, ascent - rising or moving
upwards.

Click Here For Bundle PDF Course | support@guidely.in Page 8 of 12


SBI Clerk & RRB PO Mains PDF Course 2023
ENGLISH Day - 20

The correct sentence : The little boy completed Sentence III has the inappropriate usage of the
his ascent to the mountain Everest and made a given word and needs correction.
Guinness world record as expected. Meaning of the word tedious -
boring/tiring/lasting for long
7) Answer: E The correct sentence would be : It was a
All the above given sentences have the right tediously long journey from Mysore to the capital
usage of the word throb. They are correct and (or) It was a tedious long journey from Mysore
meaningful as well so we go for option e as the to the capital.
answer.
Throb - strong regular movements, to beat 11) Answer: C
strongly B-D is the right option that fits both the blanks
with respective words. Make-up and straight are
8) Answer: E the correct words.
All the three sentences I, II and III are correct Make-up - contribute to
and have used the given word appropriately. Seventh straight month - continuously for the 7th
Hallucinate - to imagine things which are not real month
and true Other words don’t fit because :
First blank
9) Answer: D Grow over a quarter of India’s goods export -
Only sentence III is wrong as it has used a quarter of India’s goods export means ‘¼ th of’ -
wrong word instead of dampen. engineering good grow over a 1/4th of India’s
Dampen - to make something dull, uninteresting, goods export has no logical meaning in it
wet Got over - means finished/empty - hence does
Dump - to put down something not fit the blank
Sentence III is wrong because dampen is a Second blank
wrong form of usage in the given sentence. Seventh cross month - no meaning
Instead replace dampen with dampened(past Seventh time month - redundancy (it should
tense) either be seventh time or seventh month)
The corrected sentence would be : . All the The sentence : Engineering goods that make up
clothes were dampened due to the rain before over a quarter of India’s goods export basket
they could be sun dried. contracted for the 11th month in a row, while the
employment-intensive textiles sector shrank for
10) Answer: B the seventh straight month.

Click Here For Bundle PDF Course | support@guidely.in Page 9 of 12


SBI Clerk & RRB PO Mains PDF Course 2023
ENGLISH Day - 20

With in - within is a right word, there is no space


12) Answer: D in between and it does not fit the blank too
A-F is the right pair of words that the sentence The sentence : Sea accidents are common in
needs. the Philippine archipelago because of frequent
First blank - needs a questioning word, ‘whether’ storms, badly maintained vessels, overcrowding
is the right fit and spotty enforcement of safety regulations,
Second blank - Impinge is the correct word especially in remote provinces.
which adds meaning to the sentence.
Impinge - to create a bad effect 14) Answer: E
Other words do not fit the blank because : Unreachable and disrupt are the correct picks for
The propositions ‘is’ and ‘of’ are the right fits. the given blanks.
Affect on freedom is wrong - instead it must be The sentence is talking about a network attack
‘affect freedom’ which obviously creates a negative impact on
Arrive on freedom - has no meaning. the public and the users. And according to the
The sentence : A uniform civil code for the entire given sentence and the blanks the requirement
country is indeed a lofty goal, but the question is also two words with negative meaning.Out of
whether introducing one for all aspects of the six words given unreachable and disrupt are
personal law would impinge on the freedom of the only two words that complete the given
religion has been part of the debate. sentence in a meaningful way.
Operate is just opposite to what the blank needs.
13) Answer: B Shuting does not fit the blank because of the
Words ‘C’ and ‘F’ are the apt words for the two word form.
blanks given. Allow and enhance have positive meaning
Other words do not fit the blank because : The sentence : While DDoS attacks are mainly a
Badly painted vessel - gives no proper meaning nuisance, making websites unreachable without
to the sentence, the paint of the vessel has penetrating them and security experts say they
nothing to do with the storm. can disrupt the work of millions if they
Badly built vessel - makes sense but the options successfully interrupt the services of a software
which involve the word ‘built’ have a wrong word service giant like Microsoft on which so much
along with it(around, with) that does not not fit global commerce depends.
the second blank.
Around in - contradictory 15) Answer: A

Click Here For Bundle PDF Course | support@guidely.in Page 10 of 12


SBI Clerk & RRB PO Mains PDF Course 2023
ENGLISH Day - 20

The given options are the same words but in disbursed at the earliest and in any case not
different forms. Highlighting and exceeding the stipulated timeline.
struggling(gerunds) make the sentence correct
and meaningful. 17) Answer: A
Highlight does not fit the blank. A and D needs swapping and the word showned
Highlighted fits only if there is a conjunction ‘and’ is wrong and must be replaced with shown.
‘Struggled’ and ‘struggles’ both are wrong forms Validated and conducted must be swapped to
of usage. make the sentence meaningful and correct.
The sentence : In 2017, the Supreme Court of No word such a ‘showned’ as shown itself
India recognised privacy as a fundamental right, represents a past tense.
highlighting the need to protect personal Other given options are not right as the
information but the country is still struggling to suggested swaps and replacement of words are
frame a personal data protection policy. not correct.
The correct sentence : Studies conducted by
16) Answer: D researchers at the University of Virginia School
The sentence needs swapping of the two words of Medicine, U.S., and Uppsala University,
A and C and also replacement of the word ‘D’ Sweden, together have shown that Loss of
In a circular to CEOs - circular is an official chromosomes(LoY) in humans occurs with age
information that will be circulated. and is associated with several debilitating
Claim on account payments, circular payments medical conditions, a finding that has been
are not meaningful and hence require a swap. validated in mice with LoY, resulting in weak
In any case not within the stipulated time - it is heart muscles , stretched or thickened heart
wrong according to the context so replace it with tissue and heart failure.
the word ‘exceeding’.
Other given options contain no proper swapping 18) Answer: E
and replacement choices. All the words are already in the right places and
The correct sentence : In a circular to CEOs of do not require swapping but the word ‘throw’ is
all general insurance coThe correct sentence inappropriate to the context of the sentence, it
:mpanies, and standalone health insurance must be replaced with ‘through’.
companies, the Insurance Regulatory and The correct sentence : The decision to confer
Development Authority of India (IRDAI) said it the award on Gita Press was taken by a jury
needs to be ensured that all claims are surveyed headed by Prime Minister Narendra Modi after
immediately and claim on account payments are due deliberations on Sunday in recognition of the

Click Here For Bundle PDF Course | support@guidely.in Page 11 of 12


SBI Clerk & RRB PO Mains PDF Course 2023
ENGLISH Day - 20

publishing house’s “outstanding contribution 20) Answer: B


towards social, economic and political All the given words are in the right position
transformation through non-violent and other already but the word ‘C’ alone needs a
Gandhian methods”, the Culture Ministry said in replacement.
a statement. Replaces personal with personnel
Personal - private, a belonging
19) Answer: E Personnel - an officer especially in the army
The given sentence is grammatically and All the given options do not suggest any
contextually correct and meaningful and the swapping but we need to watch out for the
highlighted words are already in the right place. correct replacement that is needed. ‘Towards,
Hence, there is no need to swap or replace any discussed and brainstorming’ do not require any
words. replacement. ‘Armed forces personal’ is wrong
The other given options are wrong/false because so replace it with ‘Armed forces personnel’.
the sentence is already in the correct form. The The correct sentence : Issues such as
individual words given in bold do not require any developing a comprehensive approach towards
replacement either. national security, pushing domestic defence
The correct sentence : After a recent production and those pertaining to serving and
amendment of the Registration Act, District retired armed forces personnel will be discussed
Registrars have been empowered under Section at a brainstorming session beginning here on
77(A) to inquire into complaints of fraudulent or Monday, the Defence Ministry said in a
bogus registrations and order their cancellation. statement.

Click Here For Bundle PDF Course | support@guidely.in Page 12 of 12


SBI Clerk & RRB PO Mains PDF Course 2023
Reasoning Ability Day - 21 (Eng)

Reasoning Ability
Directions (1-5): Study the following information 2) How V is related to the one who sits
carefully and answer the given questions. immediate left of S?
Eight persons – P, Q, R, S, T, U, V and W from a a) Nephew
three generations family are sitting around a b) Brother-in-law
concentric square table in such a way that four c) Son
persons are sitting in the middle of the sides of d) Daughter
the inner table facing away from the centre e) None of these
whereas four persons are sitting in the middle of
the sides of the outer table facing towards the 3) _____ sits immediate right of ______, who is
centre. the _____ of Q respectively.
Note: If the persons are facing each other, then a) U, P, Granddaughter
both are sitting at different tables whereas if the b) V, W, Wife
persons are sitting opposite to each other, then c) R, T, Sister-in-law
both are sitting at the same table. Only two d) Both A and B
married couples are there in the family, either e) None of these
both or none of the parents are alive.
P is facing the one who sits immediate right of 4) _____ male persons sit along with _____ at the
U’s sister-in-law. One person sits between U’s same table.
sister-in-law and V, who is the unmarried son of a) One, W
W. V is an immediate neighbour of his mother, b) Two, T
who is facing away from the centre. T is the c) Three, U
brother-in-law of Q and vice-versa. Q is the d) Two, S
father of S and faces W’s only brother. S is the e) None of these
only sister of P’s father. One person sits between
T and U’s daughter. R is not facing V’s sister. 5) Which of the following statement(s) is/are not
1) What is the position of R with respect to his true as per the given arrangement?
wife? I. T is the uncle of P
a) Immediate left II. Two persons sit between Q and T’s sister
b) Second to the right III. R is facing his brother
c) Facing each other a) Only III
d) Immediate right b) Only I and II
e) None of these c) Only I and III
d) All I, II, and III

Click Here For Bundle PDF Course | support@guidely.in Page 1 of 10


SBI Clerk & RRB PO Mains PDF Course 2023
Reasoning Ability Day - 21 (Eng)

e) Only II the one whose jersey number is 18 in week 1


and week 2 respectively?
Directions (6-10): Study the following information a) R and P
carefully and answer the given questions. b) P and S
Seven persons – P, Q, R, S, T, U and V watched c) S and V
the cricket match for two weeks starting from d) S and R
Sunday to Saturday. Each person wears a jersey e) None of these
which has different number between 5 - 20.
For week one: 7) The number of persons watched the match
At most two persons watched the cricket match before V in week 2 is the same as _______ in
after S, whose jersey number is a perfect square. week 1.
The number of persons watched the match after a) The number of persons watched the match
S is one more than the number of persons between R and S
watched the match before T. T’s jersey number b) The number of persons watched the match
is twice the jersey number of R. R’s jersey before Q
number is a prime number. V watched the match c) The number of persons watched the match
three days before the one whose jersey number after U
is 7. As many persons watched the match d) Both A and C
between V and T as between V and the one e) Both B and C
whose jersey number is 12. Q’s jersey number is
19. U’s jersey number is three less than the 8) _____ persons watched the match between
jersey number of P, who watched the match two _____ and ____ in week 2.
persons after Q. V’s jersey number is twice the a) Three, Q and the one whose jersey number is
jersey number of the one who watched the match 12
immediately before U. b) One, R, Q
For Week Two: c) Two, P, the one whose jersey number is 18
All the persons whose jersey numbers are odd d) Both A and C
went to watch the match in ascending order of e) Both A and B
their jersey number on alternate days starting
from Sunday. The remaining persons went to 9) What is the sum of the jersey number of S and
watch the match in reverse alphabetical order Q?
from Monday one after another. a) 21
6) Who among the following person watched the b) 32
match immediately after and immediately before c) 28

Click Here For Bundle PDF Course | support@guidely.in Page 2 of 10


SBI Clerk & RRB PO Mains PDF Course 2023
Reasoning Ability Day - 21 (Eng)

d) 25 persons sitting between F and D is the same as


e) None of these the number of persons sitting to the left of R. A
sits opposite to the one who sits immediate right
10) Which of the following statements is/are not of Q. E and R are facing in the opposite direction.
true as per the given arrangement? As many persons sit between Q and R as
I. T watched the match on Monday in week 1 between A and B, who sits immediate left of E.
II. Only two persons watched the match between Neither T nor V sits adjacent to Q. C sits
S and U in week 2 opposite to S and sits to the left of A. P neither
III. R watched the immediately after V in week 2 sits adjacent to T nor faces the same direction as
a) Only III A. Both V and C are facing the same direction as
b) Only II and III P, who faces the opposite direction as G.
c) Only I and III 11) Four of the following five are alike in a certain
d) All I, II, and III way based on the given arrangement and thus
e) Only I form a group. Which one of the following does
not belong to the group?
Directions (11-15): Study the following a) The one who sits immediate left of Q
information carefully and answer the given b) C
questions. c) The one who sits second to the right of E
Fourteen persons are sitting at two parallel rows d) V
containing seven persons each in such a way e) The one who sits immediate right of B
that there is an equal distance between adjacent
persons. Row 2 is north of Row 1. In row 2 P, Q, 12) _____ persons are sitting to the right of
R, S, T, U, and V are seated and some are _____.
facing north whereas some are facing south. In a) Three, G
row 1 A, B, C, D, E, F, and G are seated and b) Two, Q
some are facing north whereas some are facing c) Four, E
south. Not more than two adjacent persons are d) Both A and C
facing in the same direction. Each person in row e) Both B and C
1 sits exactly opposite to one person in row 2.
U sits third to the right of T and vice versa. T sits 13) _____ sits opposite to _____ who sits _____ to
second from one of the extreme ends. The one the right of E respectively.
who sits opposite to T sits fourth to the left of F. a) A, P, Fourth
Neither G nor D sits adjacent to F. G sits third to b) Q, F, Third
the right of D, who is facing south. The number of c) P, D, Immediate

Click Here For Bundle PDF Course | support@guidely.in Page 3 of 10


SBI Clerk & RRB PO Mains PDF Course 2023
Reasoning Ability Day - 21 (Eng)

d) S, C, Second
e) Both A and C

14) What is the position of P with respect to the


one who sits opposite to D?
a) Second to the right
b) Fourth to the left
c) Immediate left
d) Third to the right
16) What comes in place of ____(A)___.
e) None of these
a) X8U
b) Z9C
15) Which of the following statements is/are not
c) Z8V
true as per the given arrangement?
d) X9V
I. F sits immediate right of C
e) None of these
II. Only two persons sit between V and the one
who sits opposite to F
17) What comes in place of ____(D)___
III. R sits immediate left of T
a) L7V
a) Only III
b) J10U
b) Only II and III
c) L5T
c) Only I and III
d) J5V
d) Only II
e) Either B or C
e) All I, II, and III

18) What comes in place of ____(C)___.


Directions (16-20): Study the following
a) V4U
information carefully and answer the given
b) D9K
questions.
c) B6L
Some letters and numbers are given in square
d) C4F
boxes. A number and letter rearrangement
e) None of these
machine rearranges each letter step by step to
process the final output.
19) What comes in place of ____(B)___.
a) J7T
b) G7J
c) F9K

Click Here For Bundle PDF Course | support@guidely.in Page 4 of 10


SBI Clerk & RRB PO Mains PDF Course 2023
Reasoning Ability Day - 21 (Eng)

d) F8J a) 59
e) None of these b) 61
c) 64
20) What is the sum of all the numbers in step d) 48
III? e) None of these
Click Here to Get the Detailed Video Solution for the above given Questions
Or Scan the QR Code to Get the Detailed Video Solutions

Answer Key with Explanation

Direction (1-5):  V is an immediate neighbour of his


1. Answer: B mother, who is facing away from the
2. Answer: C centre.
3. Answer: D That means, in case (1) V sits immediate
4. Answer: D left of his mother, in case (2) V sits
5. Answer: B immediate right of his mother.
Based on the above given information we have:

We have:
 P is facing the one who sits immediate Again, we have:
right of U’s sister-in-law.  T is the brother-in-law of Q and vice-
 One person sits between U’s sister-in-law versa.
and V, who is the unmarried son of W.

Click Here For Bundle PDF Course | support@guidely.in Page 5 of 10


SBI Clerk & RRB PO Mains PDF Course 2023
Reasoning Ability Day - 21 (Eng)

 Q is the father of S and faces W’s only


brother.
 S is the only sister of P’s father.
Since, only two married couples are there
in the family, also either both or none of
the parents are alive, thus Q must be Case (1) is not valid as one person sits between
grandfather of P. P and T, who is facing Q.
That means, V must be the son of Q, now For Blood Relation:
as V is unmarried, thus V is not the father
of P.
As only either members are there in the
family, U must be sister-in-law of S.
Based on the above given information we have:

Directions (6-10):
6. Answer: D
7. Answer: D
8. Answer: C
Again, we have:
9. Answer: C
 One person sits between T and U’s
10. Answer: B
daughter.
Since, only eight persons are there in the
family, thus P must only daughter of U.
That means, in case (2) T sits facing Q,
case (1) is not valid.
 R is not facing V’s sister.
That means, U is facing S, who is the
sister of V.
Based on the above given information we have
the final arrangement:

We have:

Click Here For Bundle PDF Course | support@guidely.in Page 6 of 10


SBI Clerk & RRB PO Mains PDF Course 2023
Reasoning Ability Day - 21 (Eng)

 At most two persons watched the cricket  As many persons watched the match
match after S, whose jersey number is a between V and T as between V and the
perfect square. one whose jersey number is 12.
 The number of persons watched the That means, in case (1) V watched the
match after S is one more than the match on Monday, in case (2) V watched
number of persons watched the match the match on Wednesday, in case (3) V
before T. watched the match on Tuesday.
 T’s jersey number is twice the jersey Based on the above given information we have:
number of R.
 R’s jersey number is a prime number.
Since, only possible prime number jersey
are 7, 11, 13, 17, 19.
But jersey number of any person can’t be
more than 19, thus only possible jersey
number of R is 7 and jersey number of T
is 14.
That means, in case (1) S watched the
match on Friday, in case (2) S watched
Again, we have:
the match on Thursday.
 U’s jersey number is three less than the
Based on the above given information we have:
jersey number of P, who watched the
match two persons after Q.
 Q’s jersey number is 19.
That means, in case (2) Q watched the
match on Sunday, case (1) & case (1a)
are not valid.
 V’s jersey number is twice the jersey
number of the one who watched the
match immediately before U.
That means, S’s jersey number is 9.
Based on the above given information we have:
Again, we have:
 V watched the match three days before
the one whose jersey number is 7.

Click Here For Bundle PDF Course | support@guidely.in Page 7 of 10


SBI Clerk & RRB PO Mains PDF Course 2023
Reasoning Ability Day - 21 (Eng)

11. Answer: C( all the given persons are sitting


at the end of the row except option C)
12. Answer: D
13. Answer: B
14. Answer: D
15. Answer: E

Case (1) & case (2a) are not valid as no day


available to place Q.
For Week two: We have:
All the persons whose jersey numbers are odd  U sits third to the right of T and vice
went to watch the match in ascending order of versa.
their jersey number on alternate days starting  T sits second from one of the extreme
from Sunday. The remaining persons went to ends.
watch the match in reverse alphabetical order  The one who sits opposite to T sits fourth
from Monday one after another. to the left of F.
Based on the above given information we have: Based on the above given information we have:

Again, we have:
 Neither G nor D sits adjacent to F.
 G sits third to the right of D, who is facing
south.
 The number of persons sitting between F
and D is the same as the number of
persons sitting to the left of R.
Directions (11-15):

Click Here For Bundle PDF Course | support@guidely.in Page 8 of 10


SBI Clerk & RRB PO Mains PDF Course 2023
Reasoning Ability Day - 21 (Eng)

That means, in case (1) R sits immediate That means, in case (2) P sits immediate
left of U, in case (2) R sits second to the left of R.
right of U.  Both V and C are facing the same
Based on the above given information we have: direction as P, who faces the opposite
direction as G.
Based on the above given information we have:

Again, we have:
 A sits opposite to the one who sits
immediate right of Q.
 As many persons sit between Q and R as Directions (16-20):
between A and B, who sits immediate left 16. Answer: A
of E. 17. Answer: C
 E and R are facing in the opposite 18. Answer: D
direction. 19. Answer: B
 Neither T nor V sits adjacent to Q. 20. Answer: A
That means, in case (2) B sits immediate We have:
right of D, case (1) is not valid. For step I:
Based on the above given information we have: Letters are changed to their corresponding
reverse letter as per the alphabetical series,
whereas if the number is odd then, add ‘1’ else if
the number is even subtract ‘1’ from it.
For step II:
Letters in the diagonally opposite boxes are
Case (1) is not valid as E and R not facing the
interchanged and those letters are changed to
same direction.
the next letter as per the English alphabetical
Again, we have:
series, whereas the numbers in the opposite
 C sits opposite to S and sits to the left of
boxes are interchanged, except in corner boxes.
A.
For step III:
 P neither sits adjacent to T nor faces the
Letters at the corner boxes are changed to the
same direction as A.
opposite letter as per the English alphabetical

Click Here For Bundle PDF Course | support@guidely.in Page 9 of 10


SBI Clerk & RRB PO Mains PDF Course 2023
Reasoning Ability Day - 21 (Eng)

series, and numbers from diagonal boxes are


interchanged.
Also, Letters from the middle boxes are changed
to the third successive letter as per the English
alphabetical series, whereas ‘2’ is added to the
number.
Based on the above given step by step rule we
have the final arrangement as follows:

Click Here For Bundle PDF Course | support@guidely.in Page 10 of 10


SBI Clerk & RRB PO Mains PDF Course 2023
Quantitative Aptitude Day - 21 (Eng)

Quantitative Aptitude

Direction (1-3): Study the following data carefully and answer the questions:
Pie chart given below shows the degree distribution of total rainfall in 4 different states A, B, C and D in
the year of 2021.

Note: x2 + y2 = 2500
1)If the average rainfall in states A, B and C is 60 rainfall in state E is what per cent of the
cm and 60% of total rainfall in state A, 40% of maximum possible rainfall in state E?
total rainfall in state B and 75% of total rainfall in a) 48.25%
state C is in the 1st half of 2021, then find the b) 36.25%
average rainfall in states A, B and C in the 2nd c)60.25%
half of 2021? d) 40.25%
a) 27 cm e) 56.25%
b) 29 cm
c) 25 cm 3) In state B, if the rainfall in 1st2nd and 3rdquarter
d) 26 cm of 2021 is 24%,16% and 40% ofthe total rainfall
e) 28 cm respectively, and the rainfall in 4th quarter is 30
cm, then find the difference between the rainfall
2)If the difference between the rainfall in state B in states A and C?
and that in state D is 37.5 cm, and the difference a) 83.33 cm
between the rainfall in state D and that in state E b) 92.5 cm
is 21 cm, then find that the minimum possible c)87.5 cm

Click Here For Bundle PDF Course | support@guidely.in Page 1 of 16


SBI Clerk & RRB PO Mains PDF Course 2023
Quantitative Aptitude Day - 21 (Eng)

d) 80 cm average number of UPI transactions made by B


e) 93.75 cm in Jan, Feb, Mar and Apr is 39, the number of
UPI transaction made by A in Apr is M% of that
Direction (4-7): Study the following data carefully made by A in Feb and the number of UPI
and answer the questions: transactions made by B in Apr is N% of that
The data given below is related to the number of made by B in Jan, then find that which of the
UPI transactions made by A and B in three following is/are true?
different months Jan, Feb and Mar. a) Both M and N are multiple of 5, but not of 10.
Number of UPI transactions made by A and B in b) Ratio of M to N is 9: 8.
Jan is (x + 8) and (3y + 5) respectively. The ratio c) Value of (5N – 4M) is divisible by 15.
of number of UPI transactions made by A in Jan d) Both (a) and (b)
to that made by B in Feb is 2: 3 and the ratio of e) Both (b) and (c)
thenumber of UPI transactions made by A in Feb
to that made by B in Jan is 4: 5. Number of UPI 6) If the average amount per UPI transaction
transactions made by A and B in marchis ‘3x’ paid by B in Jan is ₹ 132, the average amount
and ‘2y’ respectively. Number of UPI transactions per UPI transaction paid by B in Feb is ₹ 150
made by A in Mar is 8 more than that made by A and the average amount per UPI transaction
in Feb and the number of UPI transactions made paid by B in Mar is ₹ 160, then find the average
by B in Mar is 6 less than that made by B in Feb. amount per month paid by B through UPI?
4) If the average amount per UPI transaction a) ₹ 5400
paid by A in Feb is ₹ 175, the average amount b) ₹ 5600
per UPI transaction paid by B in Jan is ₹ 126 c) None of these
and the ratio of total amount paid by A in Feb to d) ₹ 5700
that paid by B in Jan through UPI transaction isP: e) ₹ 5500
Q, then find that the value of (8P – 5Q) is
divisible by which of the following? 7) If the number of UPI transactions made by C
a) 4 in Jan is 50% of that made by A and B together
b) 3 in Jan, the number of UPI transactions made by
c) 7 C in Feb is 25% of that made by A and B
d) Both (b) and (c) together in Feb and the number of UPI
e) Both (a) and (c) transactions made by C in Mar is ___% of that
made by A and B together in Mar, then the
5)If the average number of UPI transactions average number of UPI transactions made by C
made by A in Jan, Feb, Mar and Apr is 37, the in Jan, Feb and Mar will be 29.

Click Here For Bundle PDF Course | support@guidely.in Page 2 of 16


SBI Clerk & RRB PO Mains PDF Course 2023
Quantitative Aptitude Day - 21 (Eng)

Find the value to be filled in the blank? b) 73


a) 32.2% c) 67
b) 45.3% d) 71
c) 36.5% e) 69
d) 48.1%
e) 39.7% 10) Two series I and II are given with one wrong
term in each series and each series follows a
Direction (8-10): Study the following data different pattern:
carefully and answer the questions: I: 13.5, 18, 27, 45, 72, 152
8) Two series I and II are given below with II: 3, 15, 90, 360, 1080, 2160
missing term P in series I and missing term Q in If P and Q are the numbers that will come in
series II.Each series follows a different pattern: place of wrong terms in series I and II
I: 39, P, 184, 233, 269, 294 respectively, then find that which of the following
II: 816, Q, 475, 411, 384, 376 is/are true?
Find out which of the following is/are true? a) Ratio of P to Q is 9: 2.
A: 4P > Q b) Sum of P and Q is divisible by 11.
B: Both P and Q are divisible by 40. c) Difference between P and Q is divisible by 5.
C: Value of is 49. d) Both (a) and (b)
a) Only A e) Both (a) and (c)
b) None is true
c) Both A and B Direction (11-14): Study the following data
d) Both B and C carefully and answer the questions:

e) Only B The data given below is related to the domestic


use (in units) and commercial use (in units) of

9) Three series I, II and III are given below with electricity in a building in five different months

one missing term in each series and each series Jan, Feb, Mar, Apr and May.
follows a different pattern: Thetable given below shows the following data:
I: 8, 12, 24, ?, 180, 630
II: 6, 9, 15, 27, ?, 99
III: 80, ?, 76, 92, 60, 124
Find the average of the missing term of each
series?
a) 65

Click Here For Bundle PDF Course | support@guidely.in Page 3 of 16


SBI Clerk & RRB PO Mains PDF Course 2023
Quantitative Aptitude Day - 21 (Eng)

13) If the average domestic use of electricity in


Jan, Feb, Mar, Apr, May and June is 600 units,
the average commercial use of electricity in Jan,
Feb, Mar, Apr, May and June is units and
the ratio of domestic use to commercial use of
electricity in June is P: Q, then find the value of
?

Note: Ratio of commercial use of electricity in a) 50

Jan to that in Mar is 42: 31. b) 45

11) If the cost of domestic use of electricity is c) 55

₹ 7.5 per unit and the cost of commercial use of d) 60

electricity is ₹ 12.5 per unit, then find that the e) 40

total electricity bill paid in Feb is what per cent


more/less than that paid in Apr? 14) If the cost of domestic use of electricity is ₹ 8

a) 5.30% per unit and the cost of commercial use of

b) 1.30% electricity is ₹ 15 per unit, then find the

c) 3.30% difference between the 2nd maximum electricity

d) 0.30% bill paid in a month and the minimum electricity

e) 7.30% bill paid in a month?


a) ₹ 3045

12) If a series is formed, whose 1st term is equal b) ₹ 3530

to the commercial use of electricity in Jan, 2nd c) ₹ 2460

term is (1st term - 112), 3rd term is (2nd term - 72), d) ₹ 3250

4th term is (3rd term - 52), 5th term is (4th term - 32) e) ₹ 2925

and 6th term is (5th term - 22), then find that the
6th term of the series is equal to which of the Direction (15-17): Study the following data

following? carefully and answer the questions:

a) The commercial use of electricity in May + 5 Two persons A and B started a business, in

b) The domestic use of electricity in Mar + 222 which A and B invested ₹ 5xand ₹ (5x +15000)

c) Both (a) and (b) respectively. After ‘t’ months, B left the business

d) The commercial use of electricity in Mar + 77 and C joined the business by investing 60%

e) All (a), (b) and (d) more than A. At the end of the year, A’s profit

Click Here For Bundle PDF Course | support@guidely.in Page 4 of 16


SBI Clerk & RRB PO Mains PDF Course 2023
Quantitative Aptitude Day - 21 (Eng)

share and B’s profit share are respectively 50% a) ₹ 132500


and more than C’s profit share. b) ₹ 130500
15) If the value of ‘t’ was more than its c) ₹ 135500
original value and the value of ‘x’ was less d) ₹ 131500

than its original value, then what would have e) ₹ 134500

been the ratio between the profit shares of A, B


and C respectively at the end of the year? Direction (18-20): Each of the following questions

a) 160: 165: 64 consists of three statements. You have to decide

b) 480: 495: 194 whether the data provided in the statements are

c) 160: 165: 63 sufficient to answer the question.

d) 480: 495: 191 18) Four inlet pipes A, B, C and D are connected

e) None of these to a tank. Pipes A and B together can fill the tank
in 4.5 minutes and pipes B and C together can fill

16) If the average of profit shares of A and B is the tank in 4.8 minutes. In what time, pipe B

₹ 11250 more than the average of profit shares alone can fill the tank?

of B and C, then find that which of the following Statement I: Ratio of the efficiency of pipe C to

can be the profit share of A? that of pipe D is 1: 2.

P: ₹ (9x – 13500) Statement II: Pipes A and D together can fill the

Q: ₹ (10000t – 3500) tank in 7.2 minutes.

R: ₹ (xt + 4500) Statement III: Pipes A and C together can fill the

a) Only P tank in minutes.

b) Only P and Q a) Statement I alone is sufficient to answer the


c) All P, Q and R question.
d) Only P and R b) Only statements I and II together are sufficient
e) Only R to answer the question.
c) Statement III alone is sufficient to answer the
17)If a person D had joined initially inthe question.
businesswith A by investing ₹ 6000 less than C d) All the statements together are not sufficient to
and leftafter (t – 2) months from the starting of answer the question.
the business and the difference between the e) Both (b) and (c)
profit shares of A and D were ₹ 31500, then 19) Two mixtures A and B contain different
what would have been the total profit shares of A quantities of milk and water. When both the
and D? mixtures are mixed together, the cost of new

Click Here For Bundle PDF Course | support@guidely.in Page 5 of 16


SBI Clerk & RRB PO Mains PDF Course 2023
Quantitative Aptitude Day - 21 (Eng)

mixture becomes ₹ 35 per L. If the cost of pure ₹ 4200, then find the monthly salary of the
milk is ₹ 50 per L, then find the cost of mixture person?
A? Statement I: If the investment of the person in
Statement I:Thecost of mixture B is per L the scheme were ₹ 1000 more, then the interest
and in mixture B, the quantity of milk is 5 L more received from the scheme would have been
than that of water. ₹ 4500.
Statement II: Thequantity of milk in mixture B is Statement II: The person spends of his
16 L less than that in mixture A. monthly expenditure on his child, of his
Statement III: When 8 L milk is added in mixture monthly expenditure on his wife and ₹ 4000 on
B, the cost of mixture B becomes equal the cost himself.
of mixture A. Statement III: If the rate of interest in the scheme
a) Only statements I and IIItogether are sufficient were (R + 2) %, then the interest received from
to answer the question. the scheme after 2 years would have been
b) Only statements I and II together are sufficient ₹ 4760.
to answer the question. a) Statement I alone is sufficient to answer the
c) All the statements together are necessary to question.
answer the question. b) Statements II alone is sufficient to answer the
d) All the statements together are not sufficient to question.
answer the question. c) Either statement I alone or statement II alone
e) Both (a) and (b) is sufficient to answer the question, but
statement III alone is not sufficient to answer the
20) A person saves 40% his monthly salary and question.
invested it in a scheme, which gives simple d) Either of the three statements alone is
interest at R% rate of interest. If the interest sufficient to answer the question.
received from the scheme after 2 years is e) All the statements together are not sufficient to
answer the question.

Click Here For Bundle PDF Course | support@guidely.in Page 6 of 16


SBI Clerk & RRB PO Mains PDF Course 2023
Quantitative Aptitude Day - 21 (Eng)

Click Here to Get the Detailed Video Solution for the above given Questions
Or Scan the QR Code to Get the Detailed Video Solutions

Answer Key with Explanation

Direction (1-3):
Sum of degree values of all the sectors of a pie
chart = 360°
1.5 (x + y) + (3y + 15) + x + 3x = 360
1.5 x + 1.5 y + 3y + 15 + x + 3x = 360
5.5x + 4.5y = 345
11x + 9y = 690

x2 + y2 = 2500 …… (2) 1) Answer: B


From (1) and (2): Since, the total rainfall in states A, B and C
together = 3 * 60 = 180 cm
So, the total rainfall in state A =
= 70 cm
The total rainfall in state B = =
90 cm
And the total rainfall in state C =
= 20 cm
Now, the rainfall in state A in the 2nd half of 2021
= 40% of 70 = 28 cm
The rainfall in state B in the 2nd half of 2021 =
y = 40, x = 30
60% of 90 = 54 cm

Click Here For Bundle PDF Course | support@guidely.in Page 7 of 16


SBI Clerk & RRB PO Mains PDF Course 2023
Quantitative Aptitude Day - 21 (Eng)

And the rainfall in state C in the 2nd half of 2021 Number of UPI transactions made by A in Feb =
= 25% of 20 = 5 cm
Required average = = 29 cm Number of UPI transactions made by B in Feb =

2) Answer: E Number of UPI transactions made by A in Mar =


Since, the difference between the rainfall in 3x
states B and D is 37.5 cm Number of UPI transactions made by B in Mar =
So, the rainfall in state D = = 75 cm 2y
Since, the difference between the rainfall in Since, the number of UPI transactions made by
states D and E is 21 cm. A in Mar is 8 more than that made by A in Feb.
So, minimum possible rainfall in state E = 75 – So,
21 = 54 cm
And the maximum possible rainfall in state E = 15x – 12y – 20 = 40
75 + 21 = 96 cm 5x – 4y = 20 -----------------(1)
Required percentage = = 56.25% Since, the number of UPI transactions made by
B in Mar is 6 less than that made by B in Feb.
3) Answer: A
Since, the total rainfall in state B in 1st three 3x + 24 – 4y = 12
quarters of 2021 = 24 + 16 + 40 = 80% 4y – 3x = 12 -----------------(2)
And the rainfall in state B in 4th quarter of 2021 = From equations (1) and (2):
30 cm 5x – 20 = 3x + 12
So, the total rainfall in state B in 2021 = x = 16
= 150 cm From equation (1):
And the difference between the total rainfall in 80 – 4y = 20
states A and C in 2021: y = 15

Direction (4-7):
Number of UPI transactions made by A in Jan =
(x + 8)
4) Answer: C
Number of UPI transactions made by B in Jan =
Average amount per UPI transaction paid by A in
(3y + 5)
Feb = ₹ 175

Click Here For Bundle PDF Course | support@guidely.in Page 8 of 16


SBI Clerk & RRB PO Mains PDF Course 2023
Quantitative Aptitude Day - 21 (Eng)

Number of UPI transactions made by A in Feb = Ratio of M to N = 90: 80 = 9: 8


40 So, option (b) is correct.
So, the total amount paid by A in Feb through From option (c):
UPI = 175 * 40 = ₹ 7000 Value of (5N – 4M) = 400 – 360 = 40
Average amount per UPI transaction paid by B in Since, the value of (5N – 4M) is not divisible by
Jan = ₹ 126 15.
Number of UPI transactions made by B in Jan = So, option (c) is not correct.
50 Hence, only option (b) is correct.
So, the total amount paid by B in Jan through
UPI = 126 * 50 = ₹ 6300 6) Answer: B
Ratio of P: Q = 7000: 6300 = 10: 9 Total amount paid by B in Jan through UPI = 50
The value of (8P – 5Q) = 80 – 45 = 35 * 132 = ₹ 6600
So, according to the given options, the value of Total amount paid by B in Feb through UPI = 36
(8P – 5Q) is divisible by 7 only. * 150 = ₹ 5400
Total amount paid by B in Mar through UPI = 30
5) Answer: B * 160 = ₹ 4800
Since, the average number of UPI transactions Required average = = ₹ 5600
made by A in Jan, Feb, Mar and Apr is 37.
So, the number of UPI transactions made by A in 7) Answer: E
Apr: Total number of UPI transactions made by C in
(4 * 37) – (24 + 40 + 48) = 36 Jan, Feb and Mar together:
And, M = = 90% 3 * 29 = 87
Since, the average number of UPI transactions The number of UPI transactions made by C in
made by B in Jan, Feb, Mar and Apr is 39. Jan = 50% of (24 + 50) = 37
So, the number of UPI transactions made by B in The number of UPI transactions made by C in
Apr: Feb = 25% of (40 + 36) = 19
(4 * 39) – (50 + 36 + 30) = 40 So, the number of UPI transactions made by C
And, N = = 80% in Mar = 87 – (37 + 19) = 31
From option (a): Required percentage = = 39.7%
Since, both M and N are multiple of 5 and also (approx.)
multiple of 10.
So, option (a) is not correct. 8) Answer: D
From option (b): Logic in series I:

Click Here For Bundle PDF Course | support@guidely.in Page 9 of 16


SBI Clerk & RRB PO Mains PDF Course 2023
Quantitative Aptitude Day - 21 (Eng)

39 + 92 = 120 So, the missing term in series I = 60


120 + 82 = 184 Logic in series II:
184 + 72 = 233 6+3=9
233 + 62 = 269 9 + 6 = 15
269 + 52 = 294 15 + 12 = 27
So, the missing term of series I = P = 120 27 + 24 = 51
Logic in series II: 51 + 48 = 99
816 - 63 = 600 So, the missing term in series II = 51
600 - 53 = 475 Logic in series III:
475 - 43 = 411 80 + 4 = 84
411 - 33 = 384 84 – 8 = 76
384 - 23 = 376 76 + 16 = 92
So, the missing term of series II = Q = 600 92 – 32 = 60
From A: 60 + 64 = 124
4P > Q So, the missing term in series III = 84
(4 * 120) > 600 Required average = = 65
Since, 480 < 600
So, A is not true. 10) Answer: D
From B: Logic in series I:
Since, both 120 and 600 are divisible by 40. 13.5 + 4.5 = 18
So, B is true. 18 + 9 = 27
From C: 27 + 18 = 45
Value of = = 49 45 + 36 = 81(72)
So, C is true. 81 + 72 = 153
Hence, both B and C are true. So, the wrong term in series I = 72
And the number that will come in place of wrong
9) Answer: A term = P = 81
Logic in series I: Logic in series II:
8 * 1.5 = 12 3 * 6 = 18(15)
12 * 2 = 24 18 * 5 = 90
24 * 2.5 = 60 90 * 4 = 360
60 * 3 = 180 360 * 3 = 1080
180 * 3.5 = 630 1080 * 2 = 2160

Click Here For Bundle PDF Course | support@guidely.in Page 10 of 16


SBI Clerk & RRB PO Mains PDF Course 2023
Quantitative Aptitude Day - 21 (Eng)

So, the wrong term in series II = 15 Since, the ratio of commercial use to domestic
And the number that will come in place of wrong use of electricity in Feb:
term = Q = 18 1.75: 1 = 7: 4
From option (a): And the average of commercial use and
Ratio of P to Q = P: Q = 81: 18 = 9: 2 domestic use of electricity in Feb:
So, option (a) is correct. (6 * 130) – 10 = 770 units
From option (b): So, the commercial use of electricity in Feb =
Sum of P and Q = 81 + 18 = 99 = 980 units
Since, (P + Q) is divisible by 11. And the domestic use of electricity in Feb = (2 *
So, option (b) is correct. 770) – 980 = 560 units
From option (c): Since, the ratio of commercial use to domestic
Difference between P and Q = 81 – 18 = 63 use of electricity in Apr:
Since, difference of P and Q is not divisible by 5. 1.6: 1 = 8: 5
So, option (c) is not correct. And the average of commercial use and
Hence, both options (a) and (b) are correct. domestic use of electricity in Apr:
6 * 130 = 780 units
Direction (11-14): So, the commercial use of electricity in Apr =
Since, the ratio of commercial use to domestic = 960 units
use of electricity in Mar = 1.25: 1 = 5: 4 And the domestic use of electricity in Apr = (2 *
So, the commercial use of electricity in Mar = 780) – 960 = 600 units
= 775 units Since, the ratio of commercial use to domestic
And the domestic use of electricity in Mar = (2 * use of electricity in May:
697.5) – 775 = 620 units 1.8: 1 = 9: 5
Since, the ratio of commercial use of electricity in And the average of commercial use and
Jan to that in Mar is 42: 31. domestic use of electricity in May:
So, the commercial use of electricity in Jan = (5 * 130) + 1 = 651 units
= 1050 units So, the commercial use of electricity in May =
And the domestic use of electricity in Jan = = 837 units
= 750 And the domestic use of electricity in May = (2 *
Since, the average of commercial use and 651) – 837 = 465 units
domestic use of electricity in Jan:

x = 130

Click Here For Bundle PDF Course | support@guidely.in Page 11 of 16


SBI Clerk & RRB PO Mains PDF Course 2023
Quantitative Aptitude Day - 21 (Eng)

The domestic use of electricity in Mar + 222 =


620 + 222 = 842 = The 6th term of the series
So, option (b) is correct.
From option (d):
The commercial use of electricity in Mar + 77 =
775 + 77 = 852 ≠ The 6th term of the series
So, option (d) is not correct.
11) Answer: D Hence, only options (a) and (b) are correct.
Since, the cost of domestic use of electricity =
₹ 7.5 per unit 13) Answer: A
And the cost of commercial use of electricity = Since, the average of domestic use of electricity
₹ 12.5 per unit in Jan, Feb, Mar, Apr, May and June is 600
So, the total electricity bill paid in Feb = (7.5 * units.
560) + (12.5 * 980) = ₹ 16450 So, the domestic use of electricity in June:
And the total electricity bill paid in Apr = (7.5 * (6 * 600) – (750 + 560 + 620 + 600 + 465) = 605
600) + (12.5 * 960) = ₹ 16500 units
Required percentage = = 0.30% Since, the average of commercial use of
electricity in Jan, Feb, Mar, Apr, May and June is
12) Answer: C units.
Since, the commercial use of electricity in Jan = So, the commercial use of electricity in June:
1050 units
So, the 1st term of the series = 1050 So, P: Q = 605: 770 = 11: 14
The 2nd term of the series = 1050 - 112 = 929 And the value of = = 50
The 3rd term of the series = 929 - 72 = 880
The 4th term of the series = 880 - 52 = 855
14) Answer: E
The 5th term of the series = 855 - 32 = 846
Total electricity bill paid in Jan = (750 * 8) +
And the 6th term of the series = 846 - 22 = 842
(1050 * 15) = ₹ 21750
From option (a):
Total electricity bill paid in Feb = (560 * 8) + (980
The commercial use of electricity in May + 5 =
* 15) = ₹ 19180
837 + 5 = 842 = The 6th term of the series
Total electricity bill paid in Mar = (620 * 8) + (775
So, option (a) is correct.
* 15) = ₹ 16585
From option (b):
Total electricity bill paid in Apr = (600 * 8) + (960
* 15) = ₹ 19200

Click Here For Bundle PDF Course | support@guidely.in Page 12 of 16


SBI Clerk & RRB PO Mains PDF Course 2023
Quantitative Aptitude Day - 21 (Eng)

Total electricity bill paid in May = (465 * 8) + (837 So, A’s investment = 5 * 9000 = ₹ 45000
* 15) = ₹ 16275 B’s investment = 45000 + 15000 = ₹ 60000
So, the 2nd maximum electricity bill was paid in And C’s investment = 160% of 45000 = ₹ 72000
Apr, which is = ₹ 19200 15) Answer: A
And the minimum electricity bill was paid in May, The original value of ‘t’ = 7 months
which is = ₹ 16275 So, the new value of ‘t’ = of 7 = 9 months
Required difference = 19200 – 16275 = ₹ 2925 The original value of ‘x’ = ₹ 9000
So, the new value of ‘x’ = of 9000 =
Direction (15-17): ₹ 8000
A’s investment = ₹ 5x At the end of the year, the new ratio between the
B’s investment = ₹ (5x + 15000) profit shares of A, B and C respectively:
C’s investment = 160% of 5x = ₹ 8x [5 * 8000 * 12]: [(5 * 8000 + 15000) * 9]: [8 *
At the end of the year, ratio between profit 8000 * (12 – 9)]
shares of A, B and C respectively: 480000: 495000: 192000
[5x * 12]: [(5x + 15000) * t]: [8x * (12 – t)] 160: 165: 64
Now, let the profit share of C = ₹ 6y
So, the profit share of A = 150% of 6y = ₹ 9y 16) Answer: D
And the profit share of B = of 6y = ₹ 7y Since, the profit shares of A, B and C are
Since, the ratio of A’s profit share to B’s profit respectively ₹ 9y, ₹ 7y and ₹ 6y respectively.
share = 9: 7 So,
So,

16y – 13y = 22500


420x = 45xt + 135000t y = 7500
28x = 3xt + 9000t ------------(1) And the profit share of A = 9 * 7500 = ₹ 67500 =
Since, the ratio of A’ profit share to C’s profit ₹ (9x – 13500) = ₹ (xt + 4500)
share = 9: 6 = 3: 2
So, 17) Answer: B
Since, C’s investment = ₹ 72000
So, D’s investment = 72000 – 6000 = ₹ 66000
40x = 8x * (12 – t)
And the ratio of A’s profit share to D’s profit
t=7
share:
From equation (1):
[45000 * 12]: [66000 * (7 – 2)] = 540000: 330000
28x = 21x + 63000
= 18: 11
x = 9000

Click Here For Bundle PDF Course | support@guidely.in Page 13 of 16


SBI Clerk & RRB PO Mains PDF Course 2023
Quantitative Aptitude Day - 21 (Eng)

Since, the difference between the profit shares Since, pipes A and D together can fill the tank in
of A and D is ₹ 31500. 7.2 minutes.
So, the total profit shares of A and D = So,
= ₹ 130500

From equation (3):


18) Answer: E
Since, pipes A and B together can fill the tank in
t = 12
4.5 minutes.
Since, the time, in which pipe C alone can fill the
So,
tank = 2 * 12 = 24 minutes
So, from equation (2):
Since, pipes B and C together can fill the tank in
4.8 minutes.
B=6
So,
And the time, in which pipe B alone can fill the
tank = 6 minutes
By equation (1) – equation (2): Statements I and II together are sufficient to
answer the question.
From statement I alone: From statement III alone:
Since, ratio of the efficiency of pipe C to that of Since, pipes A and C together can fill the tank in
pipe D is 1: 2. minutes.
Let the time, in which pipe C alone and pipe D So,
alone can fill the tank are ‘2t’ minutes and ‘t’
minutes respectively. From equations (3) and (4):
Since, there is no relation between pipes A and A = 18 and C = 24
D and also between pipes B and D. From equation (2):
So, the time, in which pipe B alone can fill the B=6
tank can’t be determined. The time, in which pipe B alone can fill the tank =
Statement I alone is not sufficient to answer the 6 minutes
question. Statement III alone is sufficient to answer the
From statements I and II together: question.
Let the time, in which pipe C alone and pipe D Hence, either statements I and II together or
alone can fill the tank are ‘2t’ minutes and ‘t’ statement III alone is sufficient to answer the
minutes respectively. question.

Click Here For Bundle PDF Course | support@guidely.in Page 14 of 16


SBI Clerk & RRB PO Mains PDF Course 2023
Quantitative Aptitude Day - 21 (Eng)

Since, the quantity of milk in mixture B is 16 L


19) Answer: B less than that in mixture A.
Since, after mixing mixtures A and B together, So, the quantity of milk in mixture A = 20 + 16 =
the cost of the new mixture is ₹ 35 per L. 36 L
And the cost of pure milk is ₹ 50 per L. The quantity of water in mixture A =
So, the ratio of milk to water in the new mixture = =9L
35: (50 – 35) = 7: 3 And the cost of mixture A = = ₹ 40 per L
Let the total quantities of milk and water in the Statement I and II together are sufficient to
new mixture are ‘7x’ L and ‘3x’ L respectively. answer the question.
From statements I and III together: Hence, only statements I and II together are
Let the quantity of water in mixture B is ‘y’ L. sufficient to answer the question.
And the quantity of milk in mixture B = (y + 5) L
So, 20) Answer: D
Let the monthly salary of the person = ₹ 5x

y = 15 So, the monthly savings of the person = 40% of

The quantity of water in mixture B = 15 L 5x = ₹ 2x

The quantity of milk in mixture B = 15 + 5 = 20 L And,

When 8 L milk is added in mixture B, the cost of


mixture B becomes equal to the cost of mixture From statement I alone:
A. If the investment of the person in the scheme
Since, we can’t find the quantities of milk and were ₹ 1000 more, then the interest received
water in mixture A, so the cost of mixture A can’t from the scheme would have been ₹ 4500.
be determined. So,
Statement I and III together are not sufficient to
answer the question. By equation (1) ÷ equation (2):
From statements I and II together:
Let the quantity of water in mixture B is ‘y’ L.
x = 7000
And the quantity of milk in mixture B = (y + 5) L
So, the monthly salary of the person = 5 * 7000
So,
= ₹ 35000
Statement I alone is sufficient to answer the
y = 15 question.
The quantity of water in mixture B = 15 L From statement II alone:
The quantity of milk in mixture B = 15 + 5 = 20 L

Click Here For Bundle PDF Course | support@guidely.in Page 15 of 16


SBI Clerk & RRB PO Mains PDF Course 2023
Quantitative Aptitude Day - 21 (Eng)

The person spends of his monthly So,


expenditure on his child, of his monthly
expenditure on his wife and ₹ 4000 on himself. By equation (1) ÷ equation (3):
Since, the monthly expenditure of the person =
5x – 2x = ₹ 3x
So,
R = 15
From equation (3):
x = 7000
x = 7000
So, the monthly salary of the person = 5 * 7000
So, the monthly salary of the person = 5 * 7000
= ₹ 35000
= ₹ 35000
Statement II alone is sufficient to answer the
Statement III alone is sufficient to answer the
question.
question.
From statement III alone:
Hence, either of the three statements alone is
If the rate of interest in the scheme were (R + 2)
sufficient to answer the question.
%, then the interest received from the scheme
after 2 years would have been ₹ 4760.

Click Here For Bundle PDF Course | support@guidely.in Page 16 of 16


SBI Clerk & RRB PO Mains PDF Course 2023
ENGLISH Day - 21

English Language

Directions (1-5): In the following questions two b) C-F


columns are given containing three c) B-D
Sentences/phrases each. In the first column, d) A-E
sentences/phrases are A, B and C and in the e) None of these
second column the sentences/phrases are D, E
and F. A sentence/phrase from the first column 2)
may or may not connect with another COLUMN 1 COLUMN 2
sentence/phrase from the second column to
make a grammatically and contextually correct A. Each denomination of D. that would be an
sentence. Each question has five options, four of currency notes has a advantage to the
which display the sequence(s) in which the hotels and
sentences/phrases can be joined to form a restaurants that were
grammatically and contextually correct sentence. hit hard during the
If none of the options given forms a correct pandemic.
sentence after combination, mark option (E), i.e.
B. U.S. Coast Guard E. got a surprise gift
“None of these” as your answer.
officials said the search for every customer.
1)
covered 10,000 square
COLUMN 1 COLUMN 2 miles

A. Prime Minister D. who spoke with C. A powerful American F. but turned up no


Narendra Modi’s visit to great wisdom as if he Senator has said he sign of the lost sub
Washington this week was a sage. plans to introduce a Bill known as the Titan.
will to make India part of the
NATO Plus grouping
B. One night, the E. reinforce shared
queen had a dream of democratic “values” a) A-E
a beautiful golden between India and the b) B-F
goose United States. c) C-F
d) C-D
C. As he walked F. he would win the
e) None of these
towards the elephant, upcoming election.
he felt
3)
a) A-F

Click Here For Bundle PDF Course | support@guidely.in Page 1 of 10


SBI Clerk & RRB PO Mains PDF Course 2023
ENGLISH Day - 21

COLUMN 1 COLUMN 2 that employment and Russia forces


opportunities in the together.
A. The encounter had D. as the world unites
country has
taken place when a to find peace and
search operation was prosperity. C. The previous F. and reservation for
carried out Congress government government school
had initiated steps to students in medical
B. Following E. names of certain
provide free education admission.
allegations of suspects had
to students from all
corruption against the allegedly been
sections of society
officials, the NCB dropped from the first
a) A-E
initiated an inquiry and information note and
b) A-F
found that those of some others
c) C-F
included.
d) B-D
C. Yoga Day gets F. by the school e) None of these
special focus in teachers of the high
Mysuru, which is school in the village. 5)
regarded as the yoga
COLUMN 1 COLUMN 2
capital

a) B-E A. It is for all the D. students across the

b) A-F stakeholders including globe pursuing

c) C-D political parties to different courses.

d) A-E ensure

e) None of these
B. Online banking has E. that the exercise of
emerged so well in the electing panchayat
4) last decade with functionaries does not
COLUMN 1 COLUMN 2 descend into
lawlessness and
A. India’s D. climbed up and
chaos.
manufacturing growth down the highest rock
fell to its lowest in more in the tourist area. C. He found that the F. it is not new for the
world was full of sorrow public to hear excuses
B. The survey shows E. than the Ukraine
and unhappiness also from the political

Click Here For Bundle PDF Course | support@guidely.in Page 2 of 10


SBI Clerk & RRB PO Mains PDF Course 2023
ENGLISH Day - 21

as marriage halls and banquet halls and that it


parties.
also does not allow ______ liquor to guests
a) B-E
during household celebrations, functions and
b) B-D
parties.
c) A-E
a) possessive, serve
d) C-F
b) possession, serving
e) None of these
c) drinking, serving
d) possessing, serving
Directions (6-10): Each of the following questions
e) drink, offering
has two blanks, each blank indicating that a
word/phrase has been omitted. Choose the set
8) The present government has ______(A) a blind
of words/phrases that best fits the given blanks
eye towards implementation of policies that
according to the context of the sentence. Few
_______(B) the students to avail free education
alphabets are given to help you locate the blanks
and proper reservation.
and fill them.
I asked the cab driver to _______ (B) the mobile
6) Officials said discussions ____(A) on about the
data to view the route to my destination even
site of an upcoming Memorial wall for fallen
after the proper navigations, he still made a
United Nations peacekeepers, but it was
mistake and ______ (A) right instead of left at the
________(B) if that is on Mr. Modi’s agenda at the
junction.
U.N. Last week.
a) returned, enables
She felt that his motives _____(A) always
b) turned, enables
________(B) and that was the reason to call off
c) turned, enable
the marriage.
d) turnoff, took
a) was, unique
e) turnon, went
b) was, clear
c) were, unclear
9) The Bench ________the State that the Central
d) were, clear
forces were only meant to ensure the conduct of
e) is, unclear
fair elections and it also said the Central forces
and the State would work together on the
7) The Tamil Nadu government has clarified to
deployment of forces if and when the situation
the Madras High Court that an amendment made
_____.
to the Tamil Nadu Liquor (License and Permit)
a) confirmation, come
Rules, 1981 on March 18, does not permit
b) assure, appears
___________of liquor in commercial places such
c) tell, rise

Click Here For Bundle PDF Course | support@guidely.in Page 3 of 10


SBI Clerk & RRB PO Mains PDF Course 2023
ENGLISH Day - 21

d) reassured, raised a) A
e) reassured, arose b) B
c) C
10) As per insights, streaming platforms are the d) D
most ________(A) for music consumption, with e) All correct
62% of respondents choosing them and nearly
half of the participants opting for personal 12)
wireless headphones for an __________listening (A) The lack of no funds is the major reason for
experience. the Non-governmental organisation to stay
When my grandpa was alive he always inactive for a long while.
_________(A) milk to tea. (B) Migraines may cause risk to the nervous
a) preferred, immersive system and it may also end in a critical situation.
b) choice, great (C) She had been sick for the past one week and
c) prefer, excellence still is not responding to any medications.
d) preference, better (D) I found him pitiful after he lost both of his
e) used, good parents in the accident.
a) A
Directions (11-15): In the questions given below, b) B
four statements are given labelled as (A),(B),(C) c) C
and (D), with one statement being grammatically d) D
and contextually incorrect.You are required to e) All correct
choose that sentence which is erroneous. If all
the sentences are correct then choose ‘option e’ 13)
as your answer. (A) The MSME sectors are hugely supported and
11) backed up by the current government.
(A) All the records at the corporation office will be (B) The actor greeted all his audience individually
computerised by the end of the year 2025. and spoke to each of them at the audio launch
(B) After the picnic was over the teacher counted event.
the number of students and found that one of (C) The cash prize was award to the most
them is missing. successful employee of the year by the CEO of
(C) He came home yesterday night around 11PM the company.
and fell asleep instantly. (D) Both of them were mature enough to handle
(D) His aim is to provide cheap and best hair all the tough situations that they came across.
growth oil to the public. a) A

Click Here For Bundle PDF Course | support@guidely.in Page 4 of 10


SBI Clerk & RRB PO Mains PDF Course 2023
ENGLISH Day - 21

b) B (D) It is evident that the service sector has


c) C underwent enormous changes in the last twenty
d) D years.
e) All correct a) A
b) B
14) c) C
(A) There was no trace of sadness on his radiant d) D
face, instead it was calm and shone with peace. e) All correct
(B) In today’s world parents feel that their
children do not spend time with them and are Directions (16-20): In each of the questions given
more involved in other works especially gadgets. below a phrase in the sentence has been
(C) Our association decided to organise a party highlighted. It may or may not need
to celebrate the 15th anniversary of association improvement. Read the question carefully and
formation. choose an option that would best replace the
(D) Most of the people which have been victims highlighted parts to make a grammatically correct
of many forms of abuse are afraid of the sentence. In case, the sentence is correct in its
consequences of a police complaint. current form and there is no improvement
a) A needed, mark (E) as your answer.
b) B 16) The Management has decided to go along
c) C with what we had discussed and planned during
d) D the last conference call.
e) All correct a) with whatever we discussed and planned
b) by the discussed and planned
15) c) with what being discussed and planned
(A) The doctor advised me to use UV protection d) by what is discussed and planned
spectacles while using computers and laptops as e) No improvement needed
it will help to retain my original power and also
protects me from having any eye issues. 17) The blood bank was no long able to function
(B) Ramosh was a rich old man who lived all after the false allegation that was made on it.
alone in a big house because he was stranded a) was no longer able to function
by his own children. b) was longer able to function
(C) Japan witnessed a substantial increase in c) was long able to function
corporate profits for the last four years. d) was not functioning longer
e) No improvement needed

Click Here For Bundle PDF Course | support@guidely.in Page 5 of 10


SBI Clerk & RRB PO Mains PDF Course 2023
ENGLISH Day - 21

18) My mentor always advised me not to jump on b) I would place


to conclusions rather to take time and analyse c) I would being placed in a
the situation which will help to make a clear cut d) I would be placed in a
decision. e) No improvement needed
a) jump to conclusions
b) not to jump to conclusions 20) Teja was very upset as she has to move to
c) not to jump by conclusions the boarding school in NewYork and she is afraid
d) not to jump for conclusion that she would get home sick after reaching the
e) No improvement needed states.
a) she getting home sick after
19) By the end of this year, I would been placed b) she could get sick of home after
in a multinational company with a good salary but c) she would be getting home sick after
I would still try to crack the competitive exam. d) she suffer from home sick after
a) I would have been placed in a e) No improvement needed

Click Here to Get the Detailed Video Solution for the above given Questions
Or Scan the QR Code to Get the Detailed Video Solutions

Answer Key with Explanation

1) Answer: D The other statements or phrases do not combine


Statements A and E combine to form a correct with each other to form a proper sentence.
and meaningful sentence. B-D aren’t a correct pair because the pronoun
The sentence formed is, “Prime Minister used for a goose or any animal without a name
Narendra Modi’s visit to Washington this week is ‘that/which’. Since, the pronoun used here is
will reinforce shared democratic values between ‘who’ it cannot be combined to form a correct
India and the United States”. sentence.

Click Here For Bundle PDF Course | support@guidely.in Page 6 of 10


SBI Clerk & RRB PO Mains PDF Course 2023
ENGLISH Day - 21

2) Answer: B The sentence formed is, “The previous Congress


B and F are the only two statements that government had initiated steps to provide free
combine to form a single sentence with a proper education to students from all sections of society
meaning and context. and reservation for government school students
The sentence formed is, “U.S. Coast Guard in medical admission”.
officials said the search covered 10,000 square
miles but turned up no sign of the lost sub known 5) Answer: C
as the Titan”. A and E form a correct and meaningful
Other statements and phrases if combined do sentence.
not form a meaningful and correct sentence. Other options contain a pair of statements which
do not combine to form a proper sentence and
3) Answer: A hence wrong.
Statements B and E form a complete sentence The sentence formed is, “It is for all the
with proper meaning and grammar rule. stakeholders including political parties to ensure
Other statements do not combine to form any that the exercise of electing panchayat
sentence in a similar manner. functionaries does not descend into lawlessness
A-F sentence may be grammatically correct but and chaos”.
it is not right contextually because search
operations are carried over by the police or other 6) Answer: C
forces but definitely not by the teachers. First blank - discussions is a plural word and
The sentence formed is, “Following allegations thus must be followed by ‘were’(is and was is
of corruption against the officials, the NCB used for singular words)
initiated an inquiry and found that names of Similarly motives(plural) - hence were must be
certain suspects had allegedly been dropped used
from the first information note and those of some By confirming the word for the first blank we can
others included”. eliminate the options a,b and d.
Second blank - if the motives were clear, the
4) Answer: C marriage would not have been called off.
C-F is the correct pair which combines to form a Unclear - to hide something/to give no clarity
meaningful and correct sentence. which caused confusion.
The rest of the options are not correct as the pair So, the correct option is c (were and unclear)
of statements in all of those options do not The sentences are : Officials said discussions
combine to form a meaningful sentence. were on about the site of an upcoming Memorial

Click Here For Bundle PDF Course | support@guidely.in Page 7 of 10


SBI Clerk & RRB PO Mains PDF Course 2023
ENGLISH Day - 21

wall for fallen United Nations peacekeepers, but Second blank - Policies that enable and driver to
it was unclear if that is on Mr. Modi’s agenda at enable are the correct forms.
the U.N. Last week. Rule: A plural noun will be followed by a singular
She felt that his motives were always unclear verb and a singular noun will be followed by a
and that was the reason to call off the marriage. plural verb.
Sheela(SN) goes(PV) for a walk everyday.
7) Answer: B They(PN) go(SV) for a walk everyday.
First blank - possessive(not wanting to share) - The sentences : The present government has
not a right pick turned a blind eye towards implementation of
Drinking of liquor,Drink of liquor,Possessing of policies that enable the students to avail free
liquor - all of these three phrases are wrong education and proper reservation.
because the preposition ‘of’ does not go along I asked the cab driver to enable the mobile data
with them and makes the context of the to view the route to my destination even after the
sentence wrong. proper navigations, he still made a mistake and
Only option left out is option b turned right instead of left at the junction.
Possession and serving fits the blanks
respectively and makes the sentence correct 9) Answer: E
and meaningful, First blank - the blank needs a word(past tense)
The sentence : The Tamil Nadu government has so eliminate all the three options a,b and c.
clarified to the Madras High Court that an Second blank - Arose is the right word when
amendment made to the Tamil Nadu Liquor compared to raised, as it adds a proper meaning
(License and Permit) Rules, 1981 on March 18, to the sentence.
does not permit possession of liquor in The sentence : The Bench reassured the State
commercial places such as marriage halls and that the Central forces were only meant to
banquet halls and that it also does not allow ensure the conduct of fair elections and it also
serving liquor to guests during household said the Central forces and the State would work
celebrations, functions and parties. together on the deployment of forces if and when
the situation arose.
8) Answer: C
First blank - turned a blind eye(to miss 10) Answer: A
something/to ignore/not focusing on something) To eliminate the choices given we may focus on
Other words like returned, turnoff, turnon do not the second blank which needs a word that starts
fit the blank with a vowel or sound of a vowel when

Click Here For Bundle PDF Course | support@guidely.in Page 8 of 10


SBI Clerk & RRB PO Mains PDF Course 2023
ENGLISH Day - 21

pronounced. (because of ‘an’) - by this we The corrected sentence : The lack of funds is the
eliminate b,d and e major reason for the Non-governmental
First blank - Preferred is the correct word organisation to stay inactive for a long while.
because we need a word in past tense to fill the
given blank. 13) Answer: C
The sentences : As per insights, streaming The error is in sentence C, replace ‘award’ to
platforms are the most preferred for music ‘awarded’.Awarded is the right verb that should
consumption, with 62% of respondents choosing be used here.
them and nearly half of the participants opting for All the other given sentences are grammatically
personal wireless headphones for an immersive correct and contain no error.
listening experience. The corrected sentence : The cash prize was
When my grandpa was alive he always preferred awarded to the most successful employee of the
milk to tea. year by the CEO of the company.
11) Answer: B
The error is in sentence B, replace ‘is’ with 14) Answer: D
‘was’.The sentence is in past tense and thus we D is erroneous in this given question. Replace
must use ‘was’ instead of is(present tense) ‘which’ with ‘who’. ‘Who’ is the correct pronoun in
The other sentences given are grammatically this context.
correct and error free. The other sentences are correct and contain no
The corrected sentence : After the picnic was error in them.
over the teacher counted the number of students The corrected sentence : Most of the people who
and found that one of them was missing. have been victims of many forms of abuse are
afraid of the consequences of a police complaint.
12) Answer: A
Erroneous sentence is ‘A’, lack of no funds is 15) Answer: D
wrong instead it must be ‘lack of funds’ because The error is in sentence D, replace underwent
lack of already means not having. Lack of no will with undergone to make the sentence
cause redundancy and hence it is wrong and grammatically correct and meaningful.
needs correction. The auxiliary verb ‘has’ is a clue that we must
The other sentences B,C and D are correct and used undergone instead of underwent.
error free. Eg: She underwent three surgeries
already.(without has)

Click Here For Bundle PDF Course | support@guidely.in Page 9 of 10


SBI Clerk & RRB PO Mains PDF Course 2023
ENGLISH Day - 21

She has undergone three surgeries already.(with Not to jump to conclusion - not concluding
has) something instantly/quickly
The other sentences are correct and contain no Jump on, jump for and jump by are not the right
error in them. ways to put the sentence and these might alter
The corrected sentence : It is evident that the the meaning of the given sentence.
service sector has undergone enormous The sentence: My mentor always advised me
changes in the last twenty years. not to jump to conclusions rather to take time
and analyse the situation which will help to make
16) Answer: E a clear cut decision.
No improvement needed for this sentence as it is
already in its correct form. 19) Answer: D
Had decided(present perfect) should be followed The sentence is talking about something that is
by an action in the past - we had discussed and going to happen in the future, so ‘i would be
planned(is the right way to put the sentence) - placed in a’ is the right form - this has both
the sentence is already in this form so we can certainty and uncertainty of happening.
leave it unaltered. I would place - placing something
The sentence : The Management has decided to I would have been - used to talk about
go along with what we had discussed and something that did not occur/happen
planned during the last conference call. I would being - is wrong because ‘being’ and
‘been’ are two different words.
17) Answer: A The sentence : By the end of this year, I would
No longer - not anymore. be placed in a multinational company with a
Was no longer able to function is the correct good salary but I would still try to crack the
phrase. competitive exam.
The other options given do not hold a proper
meaning and hence cannot be used to replace 20) Answer: E
the highlighted part. The given sentence needs no replacement
The sentence : The blood bank was no longer hence option e is the correct answer.
able to function after the false allegation that was The sentence: Teja was very upset as she has
made on it. to move to the boarding school in NewYork and
she is afraid that she would get home sick after
18) Answer: B reaching the states.

Click Here For Bundle PDF Course | support@guidely.in Page 10 of 10


SBI Clerk & RRB PO Mains PDF Course 2023
Reasoning Ability Day - 22 (Eng)

Reasoning Ability
Directions (1-5): Study the following information d) Rs.1650
carefully and answer the given questions e) Rs.1400
Nine persons - A, B, C, D, E, F, G, H and I
travelled to Mumbai by three different type of 2. Who among the following person travelled by
vehicles viz., Flight, Train and Bus. Each of them Bus?
paid different amount of money for their travel. I. B
Note: At least two persons not more than four II. C
persons travelled by the same type of vehicle. If III. G
P and the one who paid 500 rupees travelled in a) Only I
the same vehicle, then P didn’t pay 500 rupees. b) Only II
H and the one who paid 350 rupees travelled by c) Only I and III
the same type of vehicle. H and A, who paid 250 d) Only I and II
rupees travelled by different type of vehicles. e) All I, II and III
Only one person travelled by the same type of
vehicle as A, who did not travel by bus. C paid 3. C travelled by which of the following vehicle?
250rs more than A and travelled neither by bus a) Flight
nor by the same type of vehicle as A. B and D b) Train
travelled by the same type of vehicle but not by c) The vehicle in which F travelled
the same type of vehicle as C. B paid 100rs more d) both a and c
than C and D paid 100rs less than C. F and I, e) both b and c
who paid 50rs more than D, travelled by the
same type of vehicle. I and B did not travel by the 4.Which of the following statement is true as per
same type of vehicle. The one who paid 600 the given arrangement?
rupees and G travelled by the same type of a) G and A travelled by the same type of vehicle
vehicle. G paid the average amount of money b) B and A travelled by flight
paid by B and C. The total amount paid by the c) Four persons travelled by Bus
people who travelled by train is 1600 rupees. E d) F travelled by train
paid 350rs more than G. e) None is true
1.What is the total amount of money paid by both
E and B? 5. What is the total amount of money paid by the
a) Rs.1600 persons who travelled by Bus?
b) Rs.1500 a) Rs.1900
c) Rs.1450 b) Rs.2000

Click Here For Bundle PDF Course | support@guidely.in Page 1 of 10


SBI Clerk & RRB PO Mains PDF Course 2023
Reasoning Ability Day - 22 (Eng)

c) Rs.1500 a) The box which is kept two boxes below box Q


d) Rs.1700 b) Box U
e) Rs.1550 c) The box which is kept immediately below box
R
Directions (6-10): Study the following information d) Box W
carefully and answer the given questions e) Box S
A certain number of boxes are placed one above
the other in a stack. Each box has different 7.Which of the following box is kept seventh from
items. Information about only a few boxes is the top of the stack?
known. a) Box D
Note: b) Box R
1. The consecutive alphabetically named boxes c) The box which has shoes
and complementary paired boxes are not kept d) Box W
adjacent to each other. e) The box which has handbag
2. Not more than 15 boxes are kept in the stack.
Only two boxes are kept between box S and box 8. Which of the following boxes are not kept
W. Only four boxes are kept between box W and above box S?
the box which has Tie. Box T is kept three boxes I. Box V
below the box which has Tie. The number of II. The box which has watch
boxes kept between box S and the box which III. Box T
has Tie is two less than the number of boxes a) Only I
kept between box W and box R, which has Belt. b) Only II
Only two boxes are kept between box R and the c) Only I and II
box which has handbag. Only one box is kept d) Only II and III
between the box which has handbag and box D, e) All I, II and III
which has Ring. Only seven boxes are kept
between boxes D and U. The box which has
shoe is kept three boxes above box U. The 9. How many boxes are kept in the stack?
number of boxes kept between box T and the a) 13
box which has shoes is one more than the b) 9
number of boxes kept between box U and box Q, c) 12
which has Watch. Only two boxes are kept d) 15
between box Q and box V. e) 14
6. Which of the following box has Tie?

Click Here For Bundle PDF Course | support@guidely.in Page 2 of 10


SBI Clerk & RRB PO Mains PDF Course 2023
Reasoning Ability Day - 22 (Eng)

10. Which of the following statement is/are not 11. Who among the following person sits 112.5m
true as per the given arrangement? to the left of J?
a) Box R is kept at the top of the stack a) The one who sits 45m to the right of C
b) Box W and the box with handbag are kept b) I
adjacent to each other c) The one who sits to the immediate right of B
c) Box U has Tie d) G
d) Box T is kept third from the bottom of the stack e) The one who sits 67.5m to the left of E
e) All the statements are true
12. Four of the following five are alike in a certain
Directions (11-15): Study the following way based on the given arrangement and thus
information carefully and answer the given form a group. Which one of the following does
questions not belong to the group?
Ten persons - A, B, C, D, E, F, G, H, I and J are a) F
sitting around a Pentagonal shaped table in such b) E
a way that five persons sit in the middle of the c) D
sides facing away from the centre and five d) H
persons sit at the corners of the table facing e) J
towards the centre. The perimeter of this
pentagonal table is 225m. The distance between 13. Who among the following person sits exactly
adjacent persons is equal. between H and E, when counted to the left of E?
Note: The perimeter of the Pentagon = 5*side a) A
length of the pentagon b) The one who sits opposite to E
H sits 90m away from G. H sits 67.5m away from c) V
B, who is not an immediate neighbour of G. I sits d) The one who sits 67.5m to the left of J
45m to the left of B. I and D are immediate e) I
neighbours. E sits 22.5m away from the one who
sits opposite to D. The distance between B and 14. Which among the following statement(s)
G is 22.5m more than the distance between D is/are true as per the given arrangement?
and F, when measured from the left of both B I. I and G are immediate neighbours
and D. The distance between E and H, when II. D sits third to the left of I
measured from the right of E is twice the III. C sits second to the right of A
distance between F and A, when measured from IV. B sits opposite to G
the left of F. J sits 90m away from A. C does not a) Only I and II
face the centre of the table. b) Only II and IV

Click Here For Bundle PDF Course | support@guidely.in Page 3 of 10


SBI Clerk & RRB PO Mains PDF Course 2023
Reasoning Ability Day - 22 (Eng)

c) Only I and IV A%B(6m); C£B(12m); F@G(4m); D#E(8m);


d) Only I and III GΩH(7m); E®F(5m); H#I(10m); DΩC(5m);
e) Only III and IV J®I(3m)
16.What is the direction of point J with respect to
point F?
15. Who among the following person sits 90m a) Northeast
away from F? b) Northwest
I. E c) Southwest
II. H d) Southeast
III. G e)Cannot be determined
a) Only I
b) Only II 17. Four of the following five pairs of points are
c) Only I and III alike in a certain way based on the directions in
d) Only I and II the given arrangement and thus form a group.
e) All I, II and III Which one of the following does not belong to
the group?
Directions (16-20): Study the following a) GA
information carefully and answer the given b) DB
questions c) AC
A@B(13m) means A is 13m north of B d) GE
A#B(11m) means A is 11m south of B e) EB
A$B(22m) means A is 22m east of B
A%B(9m) means A is 9m west of B 18. What is the shortest distance and direction of
A®B means B moves in west direction to reach A point B with respect to point D?
AΩB means B moves in east direction to reach A a) 3m @%
A©B means B moves in north direction to reach b) 4m @$
A c) 15m #$
A£B means B moves in south direction to reach d) 13m @%
A e) 16m#%
For example:
A£B (12m) means point B movies in south 19. If point Y is to the west of point H and to the
direction for 12m to reach point A north of point C, then what is the shortest
distance between point A and point Y?
a) 10m

Click Here For Bundle PDF Course | support@guidely.in Page 4 of 10


SBI Clerk & RRB PO Mains PDF Course 2023
Reasoning Ability Day - 22 (Eng)

b) 16m a) ABE
c) 12m b) EFH
d) 13m c) DEI
e) 8m d) HGF
e) JBC
20. Which of the following three points make a
straight line?
Click Here to Get the Detailed Video Solution for the above given Questions
Or Scan the QR Code to Get the Detailed Video Solutions

Answer Key with Explanation

Directions (1-5):  H and A, who paid 250 rupees travelled


1. Answer: B by different type of vehicles.
2. Answer: C  Only one person travelled by the same
3. Answer: E type of vehicle as A, who did not travel by
4. Answer: D bus.
5. Answer: E  C paid 250rs more than A and travelled
Final arrangement: neither by bus nor by the same type of
vehicle as A.
From the above conditions, there are four
possibilities

We have,
 H and the one who paid 350 rupees
travelled by the same type of vehicle.

Click Here For Bundle PDF Course | support@guidely.in Page 5 of 10


SBI Clerk & RRB PO Mains PDF Course 2023
Reasoning Ability Day - 22 (Eng)

The amount of money G paid is contradicted to


the statement in case 3 and 4. Hence case 3
and case 4 are eliminated.

Again we have,
 B and D travelled by the same type of
vehicle but not by the same type of
vehicle as C.
 B paid 100rs more than C and D paid
100rs less than C.
 F and I, who paid 50rs more than D Again we have,

travelled by the same type of vehicle.  The total amount paid by the people who

 I and B did not travel by the same type of travelled by train is 1600 rupees.

vehicle.  E paid 350rs more than G.


The amount of money E paid is incorrect in case
1. Hence it is eliminated. Thus, case 2 gives the
final arrangement.

Again we have, Directions (6-10):


 The one who paid 600 rupees and G 6. Answer: B
travelled by the same type of vehicle. 7. Answer: C
 G paid the average amount of money paid 8. Answer: E
by B and C. 9. Answer: D

Click Here For Bundle PDF Course | support@guidely.in Page 6 of 10


SBI Clerk & RRB PO Mains PDF Course 2023
Reasoning Ability Day - 22 (Eng)

10. Answer: E  Box T is kept three boxes below the box


Final arrangement: which has Tie.
 The number of boxes kept between box S
and the box has Tie is two less than the
number of boxes kept between box W
and box R, which has Belt.
 Only two boxes are kept between box R
and the box which has handbag.
Box T and box S are kept adjacent to each other
in case 3. Hence it is eliminated.

We have,
 Only two boxes are kept between box S
and box W.
 Only four boxes are kept between box W
and the box which has Tie.
Again we have,
From the above conditions, we have four
 Only one box is kept between the box
possibilities,
which has handbag and box D, which has
Ring.
 Only seven boxes are kept between box
D and U.
 The box which has shoe is kept three
boxes above box U.
Box T and box U are kept adjacent to each other
in case 2. Hence it is eliminated.

Again we have,

Click Here For Bundle PDF Course | support@guidely.in Page 7 of 10


SBI Clerk & RRB PO Mains PDF Course 2023
Reasoning Ability Day - 22 (Eng)

Again we have,
 The number of boxes kept between box T
and the box which has shoes is one more
than the number of boxes kept between
Directions (11-15)
box U and box Q, which has Watch.
11. Answer: C
 Only two boxes are kept between box Q
12. Answer: E(All the persons are facing the
and box V.
centre of the table except option e)
Box Q cannot be placed in case 1. Hence it is
13. Answer: B
eliminated. Thus, case 4 gives the final
14. Answer: D
arrangement.
15. Answer: C
Final arrangement:

Click Here For Bundle PDF Course | support@guidely.in Page 8 of 10


SBI Clerk & RRB PO Mains PDF Course 2023
Reasoning Ability Day - 22 (Eng)

when measured from the left of both B


and D.
We cannot place F in cases 2 and 1a. Hence
cases 2 and 1a are eliminated.

We have,
 H sits 90m away from G.
 H sits 67.5m away from B, who is not an
immediate neighbour of G.
 I sits 45m to the left of B
Again we have,
From the above conditions, there are four
 The distance between E and H, when
possibilities
measured from the right of E is twice the
distance between F and A, when
measured from the left of F.
 J sits 90m away from A.
 C does not face the centre of the table.
C faces the centre of the table in case 2a. Hence
it is eliminated. Thus, case 1 gives the final
arrangement.

Again we have,
 I and D are immediate neighbours.
 E sits 22.5m away from the one who sits
opposite to D.
Directions (16-20)
 The distance between B and G is 22.5m
16. Answer: B
more than the distance between D and F,
17. Answer: C (The first point is to the southeast
of the second point except option c)

Click Here For Bundle PDF Course | support@guidely.in Page 9 of 10


SBI Clerk & RRB PO Mains PDF Course 2023
Reasoning Ability Day - 22 (Eng)

18. Answer: D
19. Answer: A
20. Answer: E
Final arrangement:

Click Here For Bundle PDF Course | support@guidely.in Page 10 of 10


SBI Clerk & RRB PO Mains PDF Course 2023
Quantitative Aptitude Day - 22 (Eng)

Quantitative Aptitude

1) Ratio of curved surface area to total surface Find the original discount given by the
area of a right circular cylinder is 3: 4 and the shopkeeper?
difference between its height and radius is 8 cm. a) ₹ 2160
If the height of a right circular cone is 3 cm more b) ₹ 1840
than that of the cylinder and the radius of the c) ₹ 2000
cone is 2 cm more than that of the cylinder, then d) ₹ 2250
find the ratio of the volume of the cylinder to that e) ₹ 2080
of the cone?
a) 37: 36 4) Two boats A and B are sailing in a river in
b) 16: 15 downstream direction and the speed of the
c)19: 18 stream is 2 m/s. Boat B is 102 m ahead of boat A
d) 25: 24 and ratio of speed in still water of boat A to boat
e) 31: 30 B is 6: 5. If boat A crosses boat B in 34 seconds,
then find the total downstream distance covered
2) Compound interest received on ₹ ‘x’ after 2 by boat A?
years at R% rate of interest compounded a) 640 m
annually, is ₹ 1680 more than that received on b) 510 m
₹ (x – 5000) after 1 year at R% rate of interest c) 680 m
compounded half-yearly. If the compound d) 560 m
interest received on ₹ (x – 3000) at 15% rate e) 600 m
after 2 years is ₹ 3870, then find the value of R?
a) 8% 5) When A and B work with their original
b) 12% efficiencies, they together can complete the work
c)10% in ‘16t’ hours. When A and B work with 80% and
d) 6% 160% of their respective efficiencies, they
e) 15% together can complete the work in ‘15t’ hours. If
A alone can complete the work in 12 hours and
3) A shopkeeper marked up an article by ₹ 4000 C alone can complete the work in 6 hours less
above the cost price and sold it at 20% discount than that of B, then in what time A and C
on its marked price. If he had given 25% together will complete the work?
discount on its marked price, his profit would a) 8.4 hours
have been ₹ 520 less than the original profit. b) 6.0 hours

Click Here For Bundle PDF Course | support@guidely.in Page 1 of 14


SBI Clerk & RRB PO Mains PDF Course 2023
Quantitative Aptitude Day - 22 (Eng)

c)7.2 hours e) 9.6 hours


d) 4.8 hours

Direction (6-10): Study the following data carefully and answer the questions:
Data given below is related to number of boys and girls in a school, who play 5 different sports
Basketball, Football, Tennis, Cricket and Badminton.
The pie chart given below shows the percentage distribution of the number of boys, who play the given 5
sports.

Pie chart given below shows the percentage distribution of the number of girls, who play the given 5
sports.

Click Here For Bundle PDF Course | support@guidely.in Page 2 of 14


SBI Clerk & RRB PO Mains PDF Course 2023
Quantitative Aptitude Day - 22 (Eng)

Note:
1: Difference between the number of boys, who play basketball and those, who play football, is 80.
2: 36% of the total number of students, who play football, are girls.
6) Total number of students, who play basketball, 8) Which of the following is/are true?
is approximately what per cent of the total A: Difference between number of boys and girls,
number of students, who play badminton? who play cricket, is 185.
a) 55.6% B: Total number of girls in the school, is 400.
b) 65.6% C: Ratio of boys to girls, who play badminton, is
c) 59.6% 8: 11.
d) 63.6% a) All are true
e) 57.6% b) Only A and B
c)Only B and C
7) Find the ratio of total number of boys, who d) None is true
play football, tennis and cricket to the total e) Only A and C
number of girls, who play basketball, football and
tennis? 9) What per cent of total number of students who
a) 25: 18 play badminton in the school?
b) 20: 9 a) 24.4%
c)60: 29 b) 16.4%
d) 3: 1 c)22.4%
e) 15: 8 d) 18.4%

Click Here For Bundle PDF Course | support@guidely.in Page 3 of 14


SBI Clerk & RRB PO Mains PDF Course 2023
Quantitative Aptitude Day - 22 (Eng)

e) 20.4% a) 259
b) 229
10) Find the average of the total number of c) 239
students, who play basketball, those who play d) 219
football and those who play cricket? e) 249

Direction (11-15): Study the following data carefully and answer the questions:
Table given below shows the ratio of male to female employees in five different companies. Average
salary of each male and female employee is different in every company. Table also shows the ratio of
total salary of male to female employees.

Note:
1. The total employees in companies P and Q are 100 each while total male employees in companies R
and S are 80 and 75 respectively. Male employees in company T are 10 more than female employees.
2. Difference between the average salaries of male and female employees of companies P, Q, R, S, and
T is ₹ 500, ₹ 1600, ₹ 700, ₹ 400, and ₹ 1000 respectively.
11) What is the average salary of all the c) I for both
employees (male and female together) of d) I and III
company Q and S respectively? e) II for both
I: ₹ 2800
II: ₹ 3250 12) If 20 more employees joined the company T
III: ₹ 3850 whose average salary is ₹ 4200, then find the
a) I and II average salary of all the employees of company
b) II and III T.

Click Here For Bundle PDF Course | support@guidely.in Page 4 of 14


SBI Clerk & RRB PO Mains PDF Course 2023
Quantitative Aptitude Day - 22 (Eng)

a) ₹ 4400 e) Only I and II


b) ₹ 4200
c) ₹ 4500 14) Find the ratio of the difference between the
d) ₹ 4100 total salary of male and female employees of
e) ₹ 4600 company P to the average of total salary of male
and female employees of company R.
13) If ‘x’ male employees and ‘x + 10’ female a) 6: 11
employees joined the company Q such that ratio b) 5: 13
of male to female employees becomes 17: 9, c) 4: 9
then which of the following can be determined. d) 7: 16
I: Average salary of male employees in the e) 5: 16
company after the joining of new employees.
II: Total number of employees in the company 15) If the average of the total salary of female
after the joining of new employees. employees of all the five companies together is
III: Ratio of newly joined male to female ₹ 355P, then the value of ‘P’ lies between:
employees in the company. a) 350 and 380
a) Only II b) 380 and 410
b) Only I c) 320 and 350
c) Only II and III d) 410 and 440
d) Only I and III e) 440 and 470

Direction (16-20): Study the following data carefully and answer the questions:
Some questions are assigned to a freelancer for creation in five different months, out of which he created
some questions while the remaining are yet to be created. Out of the total created questions, some are
easy, and the remaining are hard difficulty levels.
The line graph given below shows the difference between the number of created questions and the
number of questions that are yet to be created. It also shows the difference between the number of
easy-level and hard level questions created.

Click Here For Bundle PDF Course | support@guidely.in Page 5 of 14


SBI Clerk & RRB PO Mains PDF Course 2023
Quantitative Aptitude Day - 22 (Eng)

Table given below shows the number of questions created as percent of total assigned questions to him.

Note:
1. x2 + y2 = 21250
2. In each month he created the majority of the assigned questions to him and also, he created easy-level
questions more than hard-level questions.
3. The observation took place somewhere in the mid of a month and questions that are yet to be created
in that month will be created later in that month only.

Click Here For Bundle PDF Course | support@guidely.in Page 6 of 14


SBI Clerk & RRB PO Mains PDF Course 2023
Quantitative Aptitude Day - 22 (Eng)

16)If the freelancer is paid ₹ 40 per question for a) 300


each easy level and the average of total amount b) 350
received by him for creating easy level questions c)200
in all five months together is ‘80P’, then which of d) 250
the following can be a factor of ‘P’? e) 400
a) 15
b) 17 19) If the ratio of the total amount received by the
c)19 freelancer for creating easy level to hard level
d) 21 questions in Apr is 8: 5, then find the ratio of pay
e) 23 for each easy level to each hard level question to
the freelancer in that month.
17)Out of total questions yet to be created in a) 8: 15
Jan, number of easy and hard level questions b) 7: 17
are ____ and ____ respectively. If he is paid ₹ 30 c)4: 9
for easy level questions and ₹ 50 for hard level d) 5: 9
questions and will be paid ₹ 5600 for all the e) 6: 11
questions which are yet to be created in that
month, then which of the following value will fill 20)Total questions created by the freelancer in
the blanks in the same order? all five months together when the observation is
a)90 and 60 made is what percent of the total number of
b)95 and 55 questions assigned to him in all the five months
c)105 and 45 together?
d) 80 and 70 a) 60%
e) 100 and 50 b) 65%
c)70%
18) What is the difference between the total d) 75%
number of easy-level questions created and the e) 80%
total number of hard-level questions by the
freelancer in Jan, Mar, and May together?

Click Here For Bundle PDF Course | support@guidely.in Page 7 of 14


SBI Clerk & RRB PO Mains PDF Course 2023
Quantitative Aptitude Day - 22 (Eng)

Click Here to Get the Detailed Video Solution for the above given Questions
Or Scan the QR Code to Get the Detailed Video Solutions

Answer Key with Explanation

1) Answer: B
Let the height and the radius of the cylinder are
529x – 1587000 – 400x + 1200000 = 1548000
‘h’ cm and ‘r’ cm respectively.
x = 15000
So,
Since, the CI received on ₹ ’x’ after 2 years at
R% rate of interest compounded annually, is
4h =3h + 3r ₹ 1680 more than that received on ₹ (x – 5000)
h: r = 3: 1 after 1 year at R% rate of interest compounded
The height of the cylinder = = 12 cm half-yearly.
The radius of the cylinder = 12 – 8 = 4 cm So,
The height of the cone = 12 + 3 = 15 cm
And the radius of the cone = 4 + 2 = 6 cm
R = 8%
Now, the volume of the cylinder = Πr2h = 192Π
3) Answer: E
cm3
Let the CP of the article = ₹ ‘x’
And the volume of the cone = (1/3)Πr2h = 180Π
So, the MP of the article = ₹ (x + 4000)
cm3
The SP of the article after 20% discount on its
Required ratio = 192Π: 180Π = 16: 15
marked price:
80% of (x + 4000) =
2) Answer: A
And the SP of the article after 25% discount on
Since, the CI received on ₹ (x – 3000) at 15%
its marked price:
rate after 2 years is ₹ 3870.
75% of (x + 4000) =
So,
So,

Click Here For Bundle PDF Course | support@guidely.in Page 8 of 14


SBI Clerk & RRB PO Mains PDF Course 2023
Quantitative Aptitude Day - 22 (Eng)

Let the time, in which B alone can complete the

16x + 64000 – 15x – 60000 = 10400 work with his original efficiency = ‘8b’ hours

x = 6400 So,

Now, the MP of the article = 6400 + 4000 =


₹ 10400 And,
And the original discount given by the
shopkeeper = 20% of 10400 = ₹ 2080 By equation (1) * (1/5) – equation (2) * (1/8):

4) Answer: C
Let the speeds of boats A and B in still water are
t = 0.5
‘6x’ m/s and ‘5x’ m/s respectively.
From equation (1):
So, the downstream speed of boat A = (6x + 2)
m/s
And the downstream speed of boat B = (5x + 2) 8b = 24
m/s Now, the time, in which B alone can complete
Let the downstream distance covered by boat B the work = 24 hours
= ‘y’ m The time, in which C alone can complete the
So, the downstream distance covered by boat A work = 24 – 6 = 18 hours
= (102 + y) m Part of work complete by A and C together in 1
Now, hour =
So, the time, in which A and C together will
complete the work = 7.2 hours
And,

Direction (6-10):
From equations (1) and (2):
From the 1st pie chart:
102 + 170x + 68 = 204x + 68
2X + Y = 100 – (10 + 25 + 15)
x=3
2X + Y = 50 -----------(1)
From equation (2):
From the 2nd pie chart:

y = 578 m
3X + 18 + Y = 78
So, the downstream distance covered by boat A
3X + Y = 60 -----------(2)
= 102 + 578 = 680 m
From equations (1) and (2):
X = 10, Y = 30
5) Answer: C

Click Here For Bundle PDF Course | support@guidely.in Page 9 of 14


SBI Clerk & RRB PO Mains PDF Course 2023
Quantitative Aptitude Day - 22 (Eng)

Since, the difference between the number of Total number of boys, who play football, tennis,
boys, who play basketball and those, who play and cricket = 160 + 200 + 240 = 600
football, is 80. Total number of girls, who play basketball,
So, the number of boys, who play basketball = football, and tennis = 72 + 90 + 108 = 270
= 80 Required ratio = 600: 270 = 20: 9
The number of boys, who play football =
= 160 8) Answer: B
The number of boys, who play tennis = = From A:

200 Difference between number of boys and girls,

The number of boys, who play cricket = who play cricket = 240 – 45 = 195

= 240 So, A is not true.

And the number of boys, who play badminton = From B:

= 120 Total number of girls in the school = 72 + 90 +


108 + 45 + 135 = 450
Since, 36% of the total number of students, who
So, B is not true.
play football, are girls.
From C:
So, the number of girls, who play football =
Ratio of boys to girls, who play badminton = 120:
= 90
135 = 8: 9
The number of girls, who play basketball =
So, C is not true.
= 72
Hence, none is true.
The number of girls, who play tennis = =
108
9) Answer: E
The number of girls, who play cricket = =
Total number of students in the school = (80 +
45
160 + 200 + 240 + 120) + (72 + 90 + 108 + 45 +
And the number of girls, who play badminton =
135) = 1250
= 135
Total number of students, who play badminton =
6) Answer: C
120 + 135 = 255
Total number of students, who play basketball =
Required percentage = = 20.4%
80 + 72 = 152
Total number of students, who play badminton =
10) Answer: B
120 + 135 = 255
Total number of students, who play basketball =
Required average = = 59.6%
80 + 72 = 152

7) Answer: B

Click Here For Bundle PDF Course | support@guidely.in Page 10 of 14


SBI Clerk & RRB PO Mains PDF Course 2023
Quantitative Aptitude Day - 22 (Eng)

Total number of students, who play football = Let total salary of male and female employees in
160 + 90 = 250 company S is 50s and 27s respectively.
Total number of students, who play cricket = 240 Let total salary of male and female employees in
+ 45 = 285 company T is t and t respectively.
Required average = = 229 Table given below shows the number of
employees (male, female, and total) in each
Direction (11-15): company and average salary of male and female
Total employees in company P = 100 employees.
Male employees in company P =
Female employees in company P = 100 – 60 =
40
Total employees in company Q = 100
Male employees in company Q =
Female employees in company Q = 100 – 75 =
25
Male employees in company R = 80
According to the question:
Female employees in company R =
Total employees in company R = 80 + 60 = 140
p = 10000
Male employees in company S = 75
Female employees in company S =
q = 20000
Total employees in company S = 75 + 45 = 120
Difference between male and female employees
in company T = 10 r = 56000

Male employees in company T =


Female employees in company T = 50 – 10 = 40 s = 6000
Total employees in company T = 50 + 40 = 90
Let total salary of male and female employees in t = 200000
company P is 15p and 8p respectively.
Let total salary of male and female employees in
company Q is 9q and 5q respectively.
Let total salary of male and female employees in
company R is 5r and 3r respectively.

Click Here For Bundle PDF Course | support@guidely.in Page 11 of 14


SBI Clerk & RRB PO Mains PDF Course 2023
Quantitative Aptitude Day - 22 (Eng)

675 + 9x = 595 + 17x


8x = 80
x = 10
I: We cannot determine the average salary of
newly joined male employees as we are not
given the total salary of male employees.
II: Total number of employees in the company
after the joining of new employees = 100 + x + (x
11) Answer: D + 10) = 130
Total salary of all the employees of company Q = III: Ratio of newly joined male to female
180000 + 100000 = ₹ 280000 employees = x: (x + 10) = 10: 20 = 1: 2
Total employees in company Q = 100 Hence, only II and III can be determined.
Required average salary =
Total salary of all the employees of company S = 14) Answer: E
300000 + 162000 = ₹ 462000 Difference between total salary of male and
Total employees in company S = 120 female employees of company P = 150000 –
Required average salary = 80000 = ₹ 70000
Average of total salary of male and female
12) Answer: A employees of company R =
Total employees in company T = 90
Total salary of all 90 employees = 200000 + Required ratio = 70000: 224000
200000 = ₹ 400000 = 5: 16
New number of employees = 90 + 20 = 110
New total salary of all 110 employees = 400000 15) Answer: B
+ 20 * 4200 = ₹ 484000 Total salary of female employees of all the five
Required average = companies together = 80000 + 100000 + 168000
= ₹ 4400 + 162000 + 200000 = ₹ 710000
Average =
13) Answer: C 355P = 142000
Initial number of male and female employees in P = 400
company Q is 75 and 25 respectively. Hence, value of P lies between 380 and 410.
According to the question:
Direction (16-20):

Click Here For Bundle PDF Course | support@guidely.in Page 12 of 14


SBI Clerk & RRB PO Mains PDF Course 2023
Quantitative Aptitude Day - 22 (Eng)

In the Apr, legends of the both the lines are 0.4x = 140
equal. Which means: x = 350
x + 75 = y + 125 Total created questions = x = 350
x – y = 50 Total yet to be created questions = x – 200 = 150
x = y + 50 ………. (1) Total assigned questions = 350 + 150 = 500
We are given: Total easy level created questions =
x2 + y2 = 21250
From equation (1): Total hard level created questions =
(y + 50)2 + y2 = 21250 Similarly, we can calculate for other months as
y2 + 2500 + 100y + y2 – 21250 = 0 well:
y2 + 50y – 9375 = 0
y = 75 and -125
We cannot take negative value of ‘y’.
Hence, y = 75 and x = 125
Find the table below that shows the simplified
data for the line graph given above:
16) Answer: E
Total easy level questions created by the
freelancer in all the five months together = 250 +
200 + 250 + 300 + 150 = ₹ 1150
Total amount received by him = 1150 * 40 =
₹ 46000
Average =
80P = 9200
P = 115
Since, he created majority of the assigned Factor of 115 = 1, 5, 23, and 115
questions which means created questions are Hence, 23 will be a factor of P.
more than yet to be created questions.
Let total number of created and yet to be created 17) Answer: B
questions in the Jan is ‘x’ and ‘x – 200’ Total questions which are yet to be created in
respectively. Jan = 150
According to the question: Let number of easy level and hard level
70% of [x + (x – 200)] = x questions which are yet to be created in Jan are
1.4x – 140 = x ‘x’ and ‘150 – x’ respectively.

Click Here For Bundle PDF Course | support@guidely.in Page 13 of 14


SBI Clerk & RRB PO Mains PDF Course 2023
Quantitative Aptitude Day - 22 (Eng)

According to the question: 19) Answer: A


30x + 50(150 – x) = 5600 Let total amount received by the freelancer for
7500 – 20x = 5600 creating easy level and hard level questions in
20x = 1900 the month of Apr is 8x and 5x respectively.
x = 95 Total easy level questions created in Apr = 300
Number of easy level questions which are yet to Total hard level questions created in Apr = 100
be created in Jan = x = 95 Pay for each easy level questions in Apr =
Number of hard level questions which are yet to Pay for each hard level questions in Apr =
be created in Jan = 150 – x = 55
Hence, values that can fill the blanks are 95 and Required ratio =
55 respectively. = 8: 15

18) Answer: B 20) Answer: D


Total number of easy level questions created by Total questions created by the freelancer in all
the freelancer in Jan, Mar, and May together = the five months together when the observation is
250 + 250 + 150 = 650 made = 350 + 300 + 400 + 400 + 200 = 1650
Total number of hard level questions created by Total number of questions assigned to the
the freelancer in Jan, Mar, and May together = freelancer in all the five months together = 500 +
100 + 150 + 50 = 300 400 + 450 + 600 + 250 = 2200
Required difference = 650 – 300 Required percent =
= 350 = 75%

Click Here For Bundle PDF Course | support@guidely.in Page 14 of 14


SBI Clerk & RRB PO Mains PDF Course 2023
ENGLISH Day - 22

English Language

Directions (1-7): Read the given passage and machinery, seeds and recipe.It is proposed to
answer the following questions based on the establish an India Centre for Lab-grown Diamond
passage. Some words are highlighted to help (InCent-LGD) at IIT Madras with the estimated
you locate while answering the questions. cost of ₹ 242.96 crore over 5 years.Lab-grown
Prime Minister Narendra Modi, who is in the U.S. diamonds are produced through two
for a three-day State visit, has gifted an eco- technologies — High-Pressure High
friendly lab-grown 7.5-carat diamond, placed in Temperature (HPHT) and Chemical Vapour
Kashmir's exquisite Papier mâché box, to the Deposition (CVD).India is one of the leading
U.S. First Lady Jill Biden. U.S. President Joe producers of these diamonds using CVD
Biden and First Lady Jill Biden hosted the Prime technology.As per industry estimates, India's
Minister for an intimate dinner at the White share in its global trade in the financial year
House, during which they discussed a range of 2021-22 was 25.8%.However, India has to
issues, exchanged gifts and enjoyed a musical depend on other countries for the supply of
tribute to the regions of India.Bharat Ka Heera! critical machinery components and seeds —
(India's diamond) PM Narendra Modi gifts this which are the raw materials for producing
beautiful eco-friendly lab-grown diamond, placed synthetic diamonds.The aim of this (InCent-LGD)
in Kashmir's exquisite Papier mâché box to the project is to provide, in mission mode, technical
U.S First Lady.The diamond is eco-friendly as assistance to the industries, and entrepreneurs in
resources like solar and wind power were used in the country, in promoting indigenous
its making. The green diamond is sculpted with manufacturing of both CVD and HPHT systems
precision and care using cutting-edge along with the recipes for expanding the LGD
technology. It emits only 0.028 grams of carbon business at the upstream end.Besides the
per carat and is certified by the Gemological Lab, jewellery industry, lab-grown diamonds are used
IGI (International Gemological Institute).India is in computer chips, satellites, and 5G networks as
promoting manufacturing of lab-grown diamond they can be used in extreme environments due
(LGD) in the country and for that the government to their potential to operate at higher speeds
announced steps in the last Union Budget.The while using less power than silicon-based
government had announced elimination of chips.LGD has a vast application in the field of
customs duty on LGD seeds from 5% earlier. It defence, optics, jewellery, thermal and medical
also approved a five-year research grant to the industry.The lab-grown diamond jewellery market
Indian Institutes of Technology, Madras to is expected to rapidly rise to $5 billion by 2025
encourage the indigenous production of LGD and exceed $15 billion by 2035.The exports of

Click Here For Bundle PDF Course | support@guidely.in Page 1 of 12


SBI Clerk & RRB PO Mains PDF Course 2023
ENGLISH Day - 22

cut and polished (worked) lab-grown diamonds Kangra Tea, Thanjavur Paintings, Allahabad
stood at $1.4 billion during April-December 2022- Surkha, Farrukhabad Prints, Lucknow Zardozi
23. It was $1.35 billion in 2021-22. and Kashmir Walnut Wood Carving.
On the other hand, Kashmir's Papier mâché 1) The main difficulty India is facing with the
involves meticulous preparation of paper pulp production of lab grown diamonds is…
and naqqashi, where skilled artisans paint a) that the producers need to pay high custom
elaborate designs. It has a GI (Geographical duty on the lab grown diamond seeds.
Indication) tag.A GI is primarily a sign of b) that China is the leading producer of lab grown
authenticity of an agricultural, natural or a diamonds using the CVD technology and this has
manufactured product (handicrafts and industrial suppressed the Indian producers.
goods) originating from a definite geographical c) the dependency on other countries for
territory.Typically, such a name conveys an machineries and seeds required for production.
assurance of quality and distinctiveness, which is d) that both High-Pressure High Temperature
essentially attributable to the place of its (HPHT) and Chemical Vapour Deposition (CVD)
origin.There is a proper process of registration of methods are a long process and require days
GI products, which includes filing of application, and proper factory setup which is lacking in India.
preliminary scrutiny and examination, show- e) that there exists a monopoly in the production
cause notice, publication in the geographical of LGD and small producers are affected in large
indications journal, opposition to registration, and numbers.
registration.Any association of persons,
producers, organisation or authority established 2) Which of the following is not an application of
by or under the law can apply. The applicant the Lab grown diamonds ?
must represent the interest of the producers.It is a) Lab grown diamonds are used in the making
a legal right under which the GI holder can of precious jewelleries and ornaments.
prohibit others from using the same name.Once b) They are used in the fields of defence and
a product gets this tag, any person or company medicine.
cannot sell a similar item under that name. This c) LGD are used in generation networks as they
tag is valid for a period of 10 years following can operate at high speeds with less power.
which it can be renewed.The other benefits of GI d) They are used in chips and satellites and have
registration include legal protection to that item, great significance.
prevention against unauthorised use by others, e) These diamonds are used in agricultural
and promoting exports.The famous goods which studies as well.
carry this tag include Basmati rice, Darjeeling
Tea, Chanderi Fabric, Mysore Silk, Kullu Shawl,

Click Here For Bundle PDF Course | support@guidely.in Page 2 of 12


SBI Clerk & RRB PO Mains PDF Course 2023
ENGLISH Day - 22

3) Which of the following is inferred from the e) Both II and III


given passage regarding the lab grown
diamonds that are produced in India ? 5) Pick from the below options the odd-one out
I. The lab-grown diamond jewellery market is with regards to the advantages of GI tags.
expected to rapidly rise to $3 billion by 2025 and a) The GI tagged products are always priced
exceed $10 billion by 2035. high compared to other products which are more
II. The diamonds only 0.028 grams of carbon per or less similar.
carat. b) GI tag is a legal right that denies others from
III. India is one of the leading producers of LDG using the same name or selling a similar item
using Chemical vapour deposition method. under that name.
IV. The country has proposed to establish an c) The tag is renewable after a period of ten
India Centre for Lab-grown Diamond (InCent- years, this assures the producer to claim the GI
LGD) at IIT Madras over the next five years. tag forever.
a) Both I and II d) The GI products are an assurance of quality
b) Both II and III and distinctiveness and this increases the
c) I, II and IV demand of the products.
d) II, III and IV e) GI tags prevent unauthorised usage and also
e) I, II, III and IV promote exports.

4) Which of the following statements is definitely 6) Which of the following statements is true
false according to the given passage ? regarding the Geographical identification tags ?
I. The GI tags once issued are valid for 10 years I. Registration of GI products includes filing of
and after that they have to be renewed. application, preliminary scrutiny, examination,
II. Afghanistan is the first country to produce publication in the journal and registration.
Papier mâché and because of this reason there II. GI tags are issued only for handmade
is a conflict between Afghanistan and Pakistan in products.
the procurement of GI tag for Papier mâché . III. Registration of GI products includes filing of
III. The passage’s introduction was mainly based application, preliminary scrutiny, examination,
on the Indian Prime Minister’s two day visit to the publication in the journal and registration.
US. a) Only I
a) Only I b) Only II
b) Only II c) Both I and II
c) Only III d) Both II and III
d) Both I and II e) Both I and III

Click Here For Bundle PDF Course | support@guidely.in Page 3 of 12


SBI Clerk & RRB PO Mains PDF Course 2023
ENGLISH Day - 22

7) Which of the following is the EXACT A) The encounter took place around 3AM in the
OPPOSITE of the word ‘elaborate’ as given in morning near the forest areas.
the passage. B) The clients usually encounter many problems
a) garnished while using the application and thus customer
b) detailed service 24/7 is a must.
c) regular C) Sima had a brief encounter with the popular
d) refined actor which made her day.
e) thorough D) Please convey my wishes to him on his new
job if you encountering him.
Directions (8-12) : In each of the following a) A
questions, a highlighted word is given followed b) B
by four sentences. Choose the sentence(s) that c) C
has/have the wrong/inappropriate usage of the d) D
highlighted word. e) All are correct
8) CIRCUMSTANCE
A) The circumstances around her made her go 10) FRUSTRATE
for the divorce otherwise they were a nice A) The exam results came only after a period of
couple. six months and it was very frustrating for me to
B) Ravi gave up his UPSC dream and worked for stay positive throughout that time period.
a daily wage only because of his family B) Ram’s pet cat frustrated me every time I visit
circumstances. his home.
C) Adapting to a new environment and C) The celebrities look frustrated most of the time
circumstances is the best characteristic of a because people keep clicking pictures of them
good human being. wherever they go and whatever they do.
D) She said, based on the current circumstance I D) The journalist always tries to frustrate his
can only postpone the meeting but not cancel it. guests by asking them inappropriate and filthy
a) A questions.
b) B a) A
c) C b) B
d) D c) C
e) All correct d) D
e) All are correct
9) ENCOUNTER
11) COMPANION

Click Here For Bundle PDF Course | support@guidely.in Page 4 of 12


SBI Clerk & RRB PO Mains PDF Course 2023
ENGLISH Day - 22

A) In his recent press conference he said that his Directions (13-17) : Rearrange the following five
father is his greatest companion of all time. sentences (A), (B), (C), (D) and (E) in the proper
B) Companionship is such a beautiful feeling of sequence to form a coherent paragraph and then
having a person around all the time as a support answer the questions given below.
system. (A) During a session at the fourth and final EWG
C) Chaddu’s mother is proud of her daughter as (Education Working Group) meeting of the G20,
she is companioning and kind. Mr. Bhargava noted that several surveys indicate
D) Cats and dogs can live under the same roof, "a severe learning crisis concerning these most
in fact they are good companions. basic skills".
a) A (B) FLN is the ability to read and write simple text
b) B and perform basic operations with numbers.
c) C (C) G20 countries should commit to ensuring
d) D FLN (Foundational Literacy and Numeracy) for
e) All are correct all children entering class 3 by 2033, renowned
Indian-American mathematician Manjul Bhargava
12) MORAL said on June 19.
A) The team lead is always moral right but (D) “G20 countries should commit to FLN for all
sometimes his rude tone would make us upset. children entering grade 3 by 2033 and I’m
B) The teacher used to tell us a story everyday hopeful that the member countries can make this
and the best part is the deep morals of those commitment during the upcoming G20 Education
stories that are actually life lessons. Ministers' meeting”, he said.
C) Arnav never failed to stay hand in hand with (E) He also said that there is a need for a
Karan and we all know that he is a huge moral of developmentally appropriate curriculum that is
support to Karan. responsive and relevant to learners’
D) Few people refuse to eat beef and pork on developmental needs and interests at different
moral grounds but for few others it is a staple stages of their development.
food. 13) Which of the following sentences is the one
a) A which precedes sentence ‘D’ after the
b) B rearrangement ?
c) C a) A
d) D b) B
e) All are correct c) C
d) D
e) E

Click Here For Bundle PDF Course | support@guidely.in Page 5 of 12


SBI Clerk & RRB PO Mains PDF Course 2023
ENGLISH Day - 22

14) Which of the following sentences is the and (3) which can be used in the beginning (to
FOURTH sentence after rearrangement ? start the sentence) to form a single sentence
a) A from the two statements given while implying the
b) B same meaning as expressed in the statement
c) C sentences.
d) D 18)
e) E I. Manipur government telling the High Court in
Imphal that it would be difficult to restore internet
15) Which of the following sentences is the access while blocking social media websites,
LAST sentence after rearrangement ? II. a Bench of Justices Ahanthem Bimol Singh
a) A and A. Guneshwar Sharma has now sought a
b) B reply from internet service providers regarding
c) C the matter.
d) D 1. With the Manipur government…
e) E 2. After the Manipur government…
3. In Spite of the Manipur government…
16) Which of the following sentences is the a) Only 1
FIRST sentence after rearrangement ? b) Only 2
a) A c) Only 3
b) B d) Both 1 and 2
c) C e) Both 1 and 3
d) D
e) E 19)
I. the court issued an interim direction on June 16
17) Which of the following sentences is the to restore limited access to the internet
SECOND sentence after rearrangement ? II. in designated spaces under the control of the
a) A State government.
b) B 1. Till that the court…
c) C 2. After the court…
d) D 3. In the meantime the court…
e) E a) Only 1
b) Only 3
Directions (18-20) : Select the phrase/connector c) Both 1 and 2
out of three phrases/connectors given as (1), (2) d) Both 1 and 3

Click Here For Bundle PDF Course | support@guidely.in Page 6 of 12


SBI Clerk & RRB PO Mains PDF Course 2023
ENGLISH Day - 22

e) Both 2 and 3 2. Even Though she suffered…


3. Despite the fact that she suffered…
20) a) Only 1
I. she suffered from a severe fever and cough, b) Only 2
II. she somehow managed to clear the exam with c) Only 3
good grades required to choose her stream. d) Both 2 and 3
1. Besides, she suffered… e) All 1,2 and 3
Click Here to Get the Detailed Video Solution for the above given Questions
Or Scan the QR Code to Get the Detailed Video Solutions

Answer Key with Explanation

1) Answer: C the Lab grown diamonds as mentioned in the


India depends on other countries for the supply passage.
of machineries and seeds that are required to The applications of lab grown diamonds are:
produce the lab grown diamonds. - This is the Computer chips, Satellites, 5G networks,
major difficulty India is facing with respect to the Defence, Optics, Jewellery, Thermal and
production of LGD. medical industry.
The other given statements are not true 3) Answer: D
according to the information provided in the The statements II,III and IV are correct according
paragraph. to the given passage.
Statement I is wrong because the factual data
2) Answer: E given in the statement is false, the correct data is
The usage of LGD in agricultural studies is not “The lab-grown diamond jewellery market is
mentioned in the above passage. But the other expected to rapidly rise to $5 billion by 2025 and
given statements are the actual application of exceed $15 billion by 2035”.

Click Here For Bundle PDF Course | support@guidely.in Page 7 of 12


SBI Clerk & RRB PO Mains PDF Course 2023
ENGLISH Day - 22

4) Answer: E 9) Answer: D
Both the statements II and III are definitely false Encountering must be replaced with encounter.
according to the information given in the above A,B and C are already correct and meaningful
passage. The error is in sentence D.
Statement II - Pakistan is the producer of Papier Encounter(noun and verb) - to meet
mâché and it owns the GI tag as well. unexpectedly/to experience something/killing of
Statement III - It was a three day visit by the PM a person by policemen or other forces
to the US. The corrected sentence : Please convey my
Only statement I is true and valid in the context wishes to him on his new job if you encounter
of the passage, hence we go for ‘option e’. him.

5) Answer: A 10) Answer: B


All the four statements b,c,d and e are the The error is in sentence B.
advantages of GI tag that have been mentioned Replace ‘frustrated’ with ‘frustrates’
in the passage given above. Frustrate - to annoy/not being calm/impatient
‘Statement a’ is the odd one out here as no Two ways of putting it :
such information is given in the passage. Hence The cat frustrates me every time I visit
option a is the right answer. The cat frustrated me every time I visited.
The corrected sentence : Ram’s pet cat
6) Answer: E frustrates me every time I visit his home.
Both the statements I and III are true regarding
the GI tags as given in the passage. 11) Answer: C
Statement II is false/invalid according to the The error is in sentence C, replace
passage. companioning with companionable.
(companioning is a wrong form of word that is
7) Answer: C used and hence replacement is required)
Regular is the antonym of the word elaborate. Companion - a person/animal with whom one
spends a lot of time together
8) Answer: E The correct sentence is : Chaddu’s mother is
All the given four sentences are grammatically proud of her daughter as she is companionable
correct and meaningful, the usage of the given and kind.
word ‘circumstance’ is also perfect and correct.
12) Answer: A

Click Here For Bundle PDF Course | support@guidely.in Page 8 of 12


SBI Clerk & RRB PO Mains PDF Course 2023
ENGLISH Day - 22

Replace moral with morally in sentence A. would be ‘E’ - the sentence has left the actual
He is moral right - wrong topic and has an overall suggestion made by the
He is morally right - correct Mathematician and also it is concluding in
The corrected sentence : The team lead is nature.
always morally right but sometimes his rude tone
would make us upset. 14) Answer: D
The fourth sentence is ‘D’.
13) Answer: A The correct sequence after rearranging the given
The correct sequence after rearranging the given sentences is CBADE.
sentences is CBADE. Out of the given five sentences only ‘C’ has the
introductory part as the other sentences are a
Sentence that precedes ‘D’ in the above continuation and not a starter sentence.Hence,
sequence is ‘A’, so ‘option a’ is the right answer. ‘C’ is the first/introductory sentence.In sentence
Out of the given five sentences only ‘C’ has the C - G20 happened and the mathematician
introductory part as the other sentences are a Manjul Bhargava stressed on the implementation
continuation and not a starter sentence.Hence, of FLN.Now, it is obvious that the next sentence
‘C’ is the first/introductory sentence.In sentence would be ‘B’ because it has the information on
C - G20 happened and the mathematician what FLN actually is. ‘B’ should come
Manjul Bhargava stressed on the implementation immediately after ‘C’ because it cannot fit
of FLN.Now, it is obvious that the next sentence anywhere else.Third sentence would be ‘A’
would be ‘B’ because it has the information on because after the definition of FLN the
what FLN actually is. ‘B’ should come paragraph traces back to the meeting insights -
immediately after ‘C’ because it cannot fit the sentence also gives lead details like it is the
anywhere else.Third sentence would be ‘A’ 4th and final meeting(such sentences with solid
because after the definition of FLN the informations comes at the first half of the
paragraph traces back to the meeting insights - paragraph because the other sentences will be a
the sentence also gives lead details like it is the continuation with comparatively less information
4th and final meeting(such sentences with solid than this).Fourth sentence is ‘D’ which reiterates
informations comes at the first half of the the first sentence. The last and final sentence
paragraph because the other sentences will be a would be ‘E’ - the sentence has left the actual
continuation with comparatively less information topic and has an overall suggestion made by the
than this).Fourth sentence is ‘D’ which reiterates Mathematician and also it is concluding in
the first sentence. The last and final sentence nature.

Click Here For Bundle PDF Course | support@guidely.in Page 9 of 12


SBI Clerk & RRB PO Mains PDF Course 2023
ENGLISH Day - 22

The correct sequence after rearranging the given


15) Answer: E sentences is CBADE.
The last or the fifth sentence is ‘E’. Out of the given five sentences only ‘C’ has the
The correct sequence after rearranging the given introductory part as the other sentences are a
sentences is CBADE. continuation and not a starter sentence.Hence,
Out of the given five sentences only ‘C’ has the ‘C’ is the first/introductory sentence.In sentence
introductory part as the other sentences are a C - G20 happened and the mathematician
continuation and not a starter sentence.Hence, Manjul Bhargava stressed on the implementation
‘C’ is the first/introductory sentence.In sentence of FLN.Now, it is obvious that the next sentence
C - G20 happened and the mathematician would be ‘B’ because it has the information on
Manjul Bhargava stressed on the implementation what FLN actually is. ‘B’ should come
of FLN.Now, it is obvious that the next sentence immediately after ‘C’ because it cannot fit
would be ‘B’ because it has the information on anywhere else.Third sentence would be ‘A’
what FLN actually is. ‘B’ should come because after the definition of FLN the
immediately after ‘C’ because it cannot fit paragraph traces back to the meeting insights -
anywhere else.Third sentence would be ‘A’ the sentence also gives lead details like it is the
because after the definition of FLN the 4th and final meeting(such sentences with solid
paragraph traces back to the meeting insights - informations comes at the first half of the
the sentence also gives lead details like it is the paragraph because the other sentences will be a
4th and final meeting(such sentences with solid continuation with comparatively less information
informations comes at the first half of the than this).Fourth sentence is ‘D’ which reiterates
paragraph because the other sentences will be a the first sentence. The last and final sentence
continuation with comparatively less information would be ‘E’ - the sentence has left the actual
than this).Fourth sentence is ‘D’ which reiterates topic and has an overall suggestion made by the
the first sentence. The last and final sentence Mathematician and also it is concluding in
would be ‘E’ - the sentence has left the actual nature.
topic and has an overall suggestion made by the
Mathematician and also it is concluding in 17) Answer: B
nature. The second sentence is ‘B’.
The correct sequence after rearranging the given
16) Answer: C sentences is CBADE.
The first sentence is ‘C’. Out of the given five sentences only ‘C’ has the
introductory part as the other sentences are a

Click Here For Bundle PDF Course | support@guidely.in Page 10 of 12


SBI Clerk & RRB PO Mains PDF Course 2023
ENGLISH Day - 22

continuation and not a starter sentence.Hence, With the Manipur government telling the High
‘C’ is the first/introductory sentence.In sentence Court in Imphal that it would be difficult to restore
C - G20 happened and the mathematician internet access while blocking social media
Manjul Bhargava stressed on the implementation websites,a Bench of Justices Ahanthem Bimol
of FLN.Now, it is obvious that the next sentence Singh and A. Guneshwar Sharma has now
would be ‘B’ because it has the information on sought a reply from internet service providers
what FLN actually is. ‘B’ should come regarding the matter.
immediately after ‘C’ because it cannot fit
anywhere else.Third sentence would be ‘A’ 19) Answer: B
because after the definition of FLN the
paragraph traces back to the meeting insights - Only 3 is correct and a valid starter to the given
the sentence also gives lead details like it is the statements.
4th and final meeting(such sentences with solid 1st is wrong - any sentence will not start with till
informations comes at the first half of the that because he needs to have a proper reason
paragraph because the other sentences will be a preceding it.
continuation with comparatively less information 2nd is incorrect because ‘after..’ must have a
than this).Fourth sentence is ‘D’ which reiterates follow up action which is missing in statement II
the first sentence. The last and final sentence here. And hence 3 is a correct starter that can be
would be ‘E’ - the sentence has left the actual used without altering the meaning of the
topic and has an overall suggestion made by the sentence.
Mathematician and also it is concluding in In the meantime the court issued an interim
nature. direction on June 16 to restore limited access to
the internet in designated spaces under the
18) Answer: A control of the State government.
Only 1st starter is correct and valid.
2nd is wrong because ‘after the manipur 20) Answer: E
government telling’ is incorrect. All the given three starters are correct and can
Instead it should be ‘after the manipur be used to combine the sentence without
government told’ altering the meaning of the sentence.
3rd is wrong because ‘in spite’ is followed by an Besides, she suffered from a severe fever and
opposite action. But statement II is not holding cough, she somehow managed to clear the
any action opposite to statement I. exam with good grades required to choose her
stream.

Click Here For Bundle PDF Course | support@guidely.in Page 11 of 12


SBI Clerk & RRB PO Mains PDF Course 2023
ENGLISH Day - 22

Even Though she suffered she suffered from a Despite the fact that she suffered from a severe
severe fever and cough, she somehow managed fever and cough, she somehow managed to
to clear the exam with good grades required to clear the exam with good grades required to
choose her stream. choose her stream.

Click Here For Bundle PDF Course | support@guidely.in Page 12 of 12


SBI Clerk & RRB PO Mains PDF Course 2023
Reasoning Ability Day - 23 (Eng)

Reasoning Ability
Directions (1-5): Study the following information d) Only III
carefully and answer the given questions. e) All I, II and III
Twelve persons - A, C, D, E, F, G, I, J, K, M, O
and P are attending the meeting on three 2. How many persons attend the meeting
different dates 6th, 13th and 24th of four different between A and I?
months viz., January, April, July and October of a) As many persons attend between P and D
the same year. b) Two
Note: If the place value of the person’s name is c) As many persons attend between G and J
an even number as per the English alphabetical d) Three
series, then the person must attend the meeting e) None
on an odd numbered date of a month.
D attends the meeting three months before J. 3. Which of the following statements is/are true
Only four persons attend the meeting between J according to the given arrangement?
and G. As many persons attend the meeting I. F attends the meeting in the month having 30
before C as after K. F attends the meeting two days
persons after C. Both K and F are not attending II. K attends the meeting before O
the meeting in the same month. A attends the III. C attends the meeting immediately after P
meeting after P on an even numbered date. Only a) Only III
one person attends the meeting between A and b) Only I and III
O. The number of persons attending the meeting c) Only I
between O and M is three less than the number d) Only II and III
of persons attending the meeting between M and e) All I, II and III
I. Both E and I attend the meeting on the same
date in different months. 4. K attends the meeting on which of the
1. If T attends the meeting two months after P, following date and month?
then T attends the meeting on which of the a) 13th January
following date and month? b) 24th July
I. 6th April c) 6th October
II. 24th June d) 6th July
III. 13th March e) 24th April
a) Only I and II
b) Only II 5. Which among the following pairs of persons
c) Only II and III attend the meeting on the even numbered date?

Click Here For Bundle PDF Course | support@guidely.in Page 1 of 10


SBI Clerk & RRB PO Mains PDF Course 2023
Reasoning Ability Day - 23 (Eng)

a) E and F c) Step 1
b) D and G d) Step 3
c) J and A e) Both a and d
d) O and M
e) P and I 7. If all the digits are arranged in descending
order from left to right in step 2, then what is the
Directions (6-10): Study the following information difference between the digits which are fifth from
carefully and answer the given questions. the right end and seventh from the left end?
74912845319751365731498275 a) 4
2649 b) 6
Note: 0 is neither an even digit nor an odd digit c) 5
Step 1: One is added to every fourth digit from d) 3
the left end and two is subtracted from the digits e) 2
in the prime numbered positions from the left end
in the given series. 8. If one is subtracted from all the odd digits and
Step 2: After completing step 1, the odd numbers one is added to all the even digits in the last
which are either immediately followed by an odd step, then which of the following digits appear
digit or immediately preceded by an even digit more than thrice?
but not both are to be written in descending order a) 0
after the last digit from the left in the series. b) 3
Step 3: After completing step 2, the digits which c) 2
are continuously repeated for more than two d) Both b and c
times are dropped from the series. e) All a, b and c
Step 4: After completing step 3, the first 12 digits
from the left end are arranged in ascending order 9. In which of the following step the gap between
from the left end followed by the remaining digits the first “3” from the right end and first “3” from
are arranged in descending order. the left end is Minimum?
Step 4 is the last and final step and the outcome I. Step 1
in this step is the final series. II. Step 3
6. In which of the following step the product of III. Step 4
the digits which is fourth from the left end and a) Only II
third from the right end is Minimum? b) Only I and II
a) Step 2 c) Only III
b) Step 4 d) Only II and III

Click Here For Bundle PDF Course | support@guidely.in Page 2 of 10


SBI Clerk & RRB PO Mains PDF Course 2023
Reasoning Ability Day - 23 (Eng)

e) All I, II and III H and senior to E. DOR are working in the same
designation. As many designations higher than J
10. In step 1, how many such 2’s are each of as lower than T. F is senior to N but junior to A. A
which is immediately preceded as well as is not working in the senior most designation. U
immediately followed by an odd digit? is senior to M but Junior to B. M is not working
a) Four with R. The number of designations higher than
b) Three B is one less than the number of designations
c) Two lower than K. I is senior to C and G.
d) One 11. Who among the following persons are
e) Five designated as Assistant Manager (AM)?
I. R
Directions (11-15): Study the following II. K
information carefully and answer the given III. I
questions. a) Only III
21 persons from A to U are working in a bank in b) Only I and II
different designations viz., Chairman, General c) Only II and III
Manager (GM), Assistant General Manager d) Only II
(AGM), Manager, Assistant Manager (AM) and e) Only I and III
Clerk. The designations are given in decreasing
order such that Chairman is the senior most and 12. L is working on which of the following
Clerk is the junior most designations. Only one designation as per the given arrangement?
person works in the senior most designation and a) Manager
two persons work in the second senior most b) The designation in which E works
designation and so on. The number of persons c) General Manager (GM)
working in each designation is one less than the d) The designation in which C works
number of persons working in their immediately e) None of these
lower designation.
Note 1: If X works with Y, then X and Y are 13. Who among the following pair of persons are
working in the same designation. working in the same designation as per the given
Note 2: Consecutive alphabetically named arrangement?
persons are not working in the same designation. I. U and R
The one who is designated as AGM is senior to II. F and D
Q. Q works with S and is not designated as AM. III. E and I
T works with E. H works only with J. P is junior to a) Only II and III

Click Here For Bundle PDF Course | support@guidely.in Page 3 of 10


SBI Clerk & RRB PO Mains PDF Course 2023
Reasoning Ability Day - 23 (Eng)

b) Only III Mumbai, Pune, Chennai, Lucknow, Kolkata and


c) Only I and III Ranchi.
d) Only I and II Note: None of the city names with an equal
e) Only II number of vowel counts are opposite to each
other.
14. If H is related to R and P is related to K in a S and the one who went for an interview to
certain way, then who among the following Ranchi are sitting second to the left of each
person is related to N? other. At least one person sits between the one
a) B who went to Ranchi and the one who went to
b) T Lucknow, when counted from both sides. S does
c) D not go to Lucknow. T faces the one who went to
d) J Pune. Q sits fourth to the right of T. The number
e) E of persons sitting between Q and the one who
went to Mumbai is two less than the number of
15. What is the position of the one who works persons sitting between P and the one who went
with O with respect to the one who works with to Chennai, when counted from the right of both
Q? P and Q. Both the immediate neighbours of the
a) Three designations higher one who went to Mumbai are facing away from
b) Two designations higher the centre. P faces the opposite direction of T. U
c) Three designations lower neither went to Lucknow nor sits at the corner of
d) Two designations lower the table. R is not facing the same direction as U.
e) None of these 16. Who among the following person sits second
to the left of the one who went to Mumbai?
Directions (16-20): Study the following I. The one who went to Kolkata
information carefully and answer the given II. Q
questions. III. The one who went to Chennai
Six persons - P, Q, R, S, T and U are sitting a) Only II
around an equilateral triangular table in such a b) Only I and III
way that three of them are sitting at the corners c) Only III
and the remaining are sitting in the middle of the d) Only II and III
side. Some of them are facing the centre while e) Only I
some of them are facing away from the centre.
They went for an interview to different cities viz., 17. If all the persons are facing opposite to their
original direction, then who among the following

Click Here For Bundle PDF Course | support@guidely.in Page 4 of 10


SBI Clerk & RRB PO Mains PDF Course 2023
Reasoning Ability Day - 23 (Eng)

pair of persons are sitting second to the right of 19. How many persons sit between the one who
each other? went to Pune and U, when counted from the right
a) R and S of U?
b) T and Q a) Four
c) P and U b) None
d) R and U c) Two
e) Both b and d d) Three
e) One
18. Four of the following five are alike in a certain
way based on the given arrangement and thus 20. Who among the following person faces the
form a group. Which one of the following does one who went to Mumbai?
not belongs to the group? a) T
a) The one who went to Lucknow b) The one who went to Chennai
b) Q c) R
c) The one who went to Kolkata d) The one who went to Pune
d) S e) U
e) The one who went to Mumbai
Click Here to Get the Detailed Video Solution for the above given Questions
Or Scan the QR Code to Get the Detailed Video Solutions

Answer Key with Explanation

Directions (1-5): 5. Answer: D


1. Answer: D Final arrangement:
2. Answer: C
3. Answer: B
4. Answer: C

Click Here For Bundle PDF Course | support@guidely.in Page 5 of 10


SBI Clerk & RRB PO Mains PDF Course 2023
Reasoning Ability Day – 23 (Eng)

Again, we have
We have,  A attends the meeting after P on an even
 D attends the meeting three months numbered date of a month.
before J.  Only one person attends the meeting
 Only four persons attend the meeting between A and O.
between J and G.  The number of persons attending the
After applying the above conditions, there are meeting between O and M is three less
three possibilities. than the number of persons attending the
meeting between M and I.
 Both E and I attend the meeting on the
same date in different months.
After applying the above conditions, there is
another possibility. But we apply only one person
attends the meeting between A and O and both
E and I attend the meeting on the same date in
different months, we can cancel case 1, case 2
and case 3a, hence case 3 shows the final
Again, we have arrangement.
 F attends the meeting two persons after
C.
 As many persons attend the meeting
before C as after K.
 Both K and F not attend the meeting in
the same month.

Click Here For Bundle PDF Course | support@guidely.in Page 6 of 10


SBI Clerk & RRB PO Mains PDF Course 2023
Reasoning Ability Day – 23 (Eng)

Step 2: 9 9 8 8 7 7 7 7 7 7 7 6 6 5 4 3 3 3 3 3 2 2
2 2 2 2 1 1 10
Difference = 7 – 2 = 5

8. Answer: E
Step 4: 0 1 2 2 2 3 3 6 7 7 8 8 9 9 6 5 4 3 3 3 2 2
211
Step 4: 0 0 3 3 3 2 2 7 6 6 9 9 8 8 7 4 52 2 2 3 3
300

9. Answer: C

Directions (6-10): Step 1: 7 2 7 2 0 8 2 6 3 1 7 8 3 1 3 7 3 7 1 2 4 9

74912845319751365731498275 6 3 7 5 2 7 2 9-> 14 digits

2649 Step 2: 7 2 2 0 8 2 6 3 7 8 3 1 2 4 6 3 5 2 2 9 9 7

Step 1: 7 2 7 2 0 8 2 6 3 1 7 8 3 1 3 7 3 7 1 2 4 9 7 7 7 7 3 3 1 1->19 digits

63752729 Step 3: 7 2 2 0 8 2 6 3 7 8 3 1 246352299

Step 2: 7 2 2 0 8 2 6 3 7 8 3 1 2 4 6 3 5 2 2 9 9 7 3 3 1 1-> 14 digits

77773311 Step 4: 0 1 2 2 2 3 3 6 7 7 8 8 996543332

Step 3: 7 2 2 0 8 2 6 3 7 8 3 1 2 4 6 3 5 2 2 9 9 3 2 2 1 1 -> 13 digits

311
Step 4: 0 1 2 2 2 3 3 6 7 7 8 8 9 9 6 5 4 3 3 3 2 2 10. Answer: B

211 Step 1: 7 2 7 2 0 8 2 6 3 1 7 8 3 1 3 7 3 7 1 2 4 9

6. Answer: E 63752729

Step 1: 2 * 7 = 14
Step 2: 0* 3 = 0 Directions (11-15):

Step 3: 0*3 = 0 11. Answer: C

Step 4: 2*2 = 4 12. Answer: D


13. Answer: A

7. Answer: C 14. Answer: C

Step 2: 7 2 2 0 8 2 6 3 7 8 3 1 2 4 6 3 5 2 2 9 9 7 15. Answer: B

77773311 Final arrangement:

Descending order:

Click Here For Bundle PDF Course | support@guidely.in Page 7 of 10


SBI Clerk & RRB PO Mains PDF Course 2023
Reasoning Ability Day – 23 (Eng)

We have, Again, we have


 The one who is designated as AGM is  The number of designations higher than B
senior to Q. is one less than the number of
 Q works with S and is not designated as designations lower than K.
AM.  I is senior to C and G.
 T works with E. After applying the above conditions, case 1 and
 H works only with J. case 2 get eliminated because consecutive
 P is junior to H and senior to E. alphabetically named persons are not working in
 DOR are working in the same the same designation, hence case 2a shows the
designation. final arrangement.
 As many designations higher than J as
lower than T.
After applying the above conditions, there are
three possibilities.

Directions (16-20):
16. Answer: D
17. Answer: B
Again, we have
18. Answer: C (Except option c, all are facing
 F is senior to N but junior to A.
away from the centre)
 A is not working in the senior most
19. Answer: D
designation.
20. Answer: E
 U is senior to M but Junior to B.
Final arrangement:
 M is not working with R.
After applying the above conditions, there is
another one possibility, but case 3 gets
eliminated because F and E are not working in
the same designation.

Click Here For Bundle PDF Course | support@guidely.in Page 8 of 10


SBI Clerk & RRB PO Mains PDF Course 2023
Reasoning Ability Day – 23 (Eng)

 The number of persons sitting between Q


and the one who went to Mumbai is two
less than the number of persons sitting
between P and the one who went to
Chennai, when counted from the right of
both P and Q.
 Both the immediate neighbours of the one
who went to Mumbai are facing away
We have, from the centre.
 S and the one who went for an interview  P faces the opposite direction of T.
to Ranchi are sitting second to the left of
each other.
 At least one person sits between the one
who went to Ranchi and the one who
went to Lucknow when counted from both
sides.
 S does not go to Lucknow.
After applying the above conditions, there are
four possibilities.

Again, we have
 U neither went to Lucknow nor sits at the
corner of the table.
 R is not facing the same direction as U.
After applying the above conditions, case 1,
case 2 and case 4 get eliminated because U
neither went to Lucknow nor sits at the corner of
the table, hence case 3 shows the final
Again, we have
arrangement.
 T faces the one who went to Pune.
 Q sits fourth to the right of T.

Click Here For Bundle PDF Course | support@guidely.in Page 9 of 10


SBI Clerk & RRB PO Mains PDF Course 2023
Reasoning Ability Day – 23 (Eng)

Click Here For Bundle PDF Course | support@guidely.in Page 10 of 10


SBI Clerk & RRB PO Mains PDF Course 2023
Quantitative Aptitude Day - 23 (Eng)

Quantitative Aptitude

Directions (01 - 05): Study the following information carefully and answer the questions given below.
The given pie chart shows the percentage distribution of the number of selling target (kg) of five fruits
[Apple, orange, grapes, Papaya, Mango] by a fruit seller, producer company.

Note- Red, orange, black, green, and yellow colors represent fruit apple, orange, grapes, papaya, and
mango respectively. The percentage of actual sales of fruit apples, Orange, grapes, papaya, and mango
out of target sales is 80%, 70%, 60%, 80%, and 90% respectively. A total of 180 kg of grapes were sold
by the company.
1) Cost of per kg apple sold by the company is fulfilled for fruit strawberries then find the total
Rs.80, Per kg orange sold by the company is target sales for strawberries and Papaya
Rs.60 and Per kg Mango sold by the company is together?
Rs.40. Find the total revenue earned by the a) 652kg
company by selling these three fruits? b) 852kg
a) Rs.28400 c) 752kg
b) Rs.25800 d) 725kg
c) Rs.29700 e) 951kg
d) Rs.27600
e) Rs.22300 3) Find the ratio between the total number of
sales of the target not achieved for fruit apples
2) Ratio of the actual sale of fruit strawberries and grapes together and for fruit oranges and
and papaya is 14:13. If 70% of the target is papaya together?

Click Here For Bundle PDF Course | support@guidely.in Page 1 of 9


SBI Clerk & RRB PO Mains PDF Course 2023
Quantitative Aptitude Day - 23 (Eng)

a) 2:3 d) 135kg
b) 3:2 e) None of these
c) 3:5
d) 3:7 5) Find the difference between central angle
e) None of these formed by actual sales of oranges and actual
sales of mango if the total actual sales of fruit are
4) Find the difference between the actual sales represented in the pie chart?
of apple and orange fruit together and the actual a) 45.480
sales of grapes and papaya fruit together? b) 62.480
a) 155kg c) 55.480
b) 120kg d) 35.480
c) 165kg e) None of these

Directions (06 - 10): Study the following information carefully and answer the questions given below.
The given bar graphs show the number of test matches played, no of test matches drawn and no of
matches lost by the five teams is given.

Note – B+C=80, Total number matches played= Number of matches won+ Number of matches drawn +
Number of matches lost.
Win% = [Total Number of the match won/total Number of matches played] *100

Click Here For Bundle PDF Course | support@guidely.in Page 2 of 9


SBI Clerk & RRB PO Mains PDF Course 2023
Quantitative Aptitude Day - 23 (Eng)

6) Find the difference between the winning a) 11:7


percentage of team A and the winning b) 10:7
percentage of team E? c) 19:12
a) 9.35% d) 18:11
b) 2.58% e) None of these
c) 3.75%
d) 1.25% 9) Find the value 3A+4B-B% of 320+3C-C% of
e) 6.35% 420+325=?
a) 409
7) Team F wins 10 matches more than team C. b) 502
Ratio of total matches played by team C and c) 650
team F is 5:6. Find the winning percentage of d) 582
team F? e) None of these
a) 30%
b) 38% 10) In the next 50 matches team D lost 15
c) 37% matches and draw 9 matches. Find the increase
d) 33.33% or decrease in winning percentage for team D
e) 39% after and before 50 matches?
a) 0.58%
8) Find the ratio between the total number of b) 0.68%
matches won by teams A, B, and E together and c) 0.21%
the total number of matches drawn by teams C, d) 0.32%
D, and E together? e) None of these

Directions (11 - 15): Study the following information carefully and answer the questions given below.
The given table shows the expenditure on rent, grocery, transport, bill and savings of five people is given.

Click Here For Bundle PDF Course | support@guidely.in Page 3 of 9


SBI Clerk & RRB PO Mains PDF Course 2023
Quantitative Aptitude Day - 23 (Eng)

Note – Total income of a person is the sum of expenditure in rent, grocery, transport, bill, and saving
together.
11) Expenditure of D in rent, grocery transport, b) Rs.4520
and bill are 10%,20%,10%, and 20% more than c) Rs.6520
that of B. If the total income of D is Rs. 32500 d) Rs.6540
then find the saving of D? e) Rs.6320
a) Rs.5420
b) Rs.6820 13) Find which one is true?
c) Rs.8720 I. Total expenditure in the grocery of all three
d) Rs.8520 together is Rs.17200.
e) Rs.9520 II. Total expenditure in rent of all three together is
Rs.13900.
12) Find the difference between the average III. Total expenditure in transport of all three
income of A and B together and the total saving together is Rs.12800.
of A, B, and C together? a) Only III is true
a) Rs.5200 b) None is true

Click Here For Bundle PDF Course | support@guidely.in Page 4 of 9


SBI Clerk & RRB PO Mains PDF Course 2023
Quantitative Aptitude Day - 23 (Eng)

c) Only II is true 40% more than the number of boys in the same
d) Only II and III are true class and number of Number of girls in class E is
e) None of these 20% more than the number of boys in class E.
The total number of boys in classes D and C is
14) Find the ratio of total expenditure in transport 57 and the number of boys in all schools
and rent of B and total expenditure in grocery together is 139.
and bill of C? 16) Total number of students in classes A and B
a) 94:101 together is what percent of the total number of
b) 97:112 students in classes C and D together?
c) 98:105 a) 45.35%
d) 91:101 b) 68.36%
e) None of these c) 84.28%
d) 78.33%
15) If the income of A and B is increased by 15% e) 55.32%
and 20% respectively. then find the new average
income of A and B is approximately what percent 17) Number of students in class F is 15 more
of the income of C? than the number of students in class C. The ratio
a) 130% of boys and girls in class F is 3:2. Find the
b) 140% number of girls in class F?
c) 160% a) 32
d) 110% b) 30
e) None of these c) 48
d) 38
Directions (16 - 18): Study the following e) 42
information carefully and answer the questions
given below. 18) Find the difference between the total number
The total number of boys in classes A and B of girls in all classes together and the total
together is 52 and the number of girls in class number of boys in all classes together?
classes B and C together is 84. The total number a) 51
of boys in classes B and C together is 64 and the b) 46
total number of girls in A and B together is 66. c) 52
The ratio of the number of boys and girls in d) 56
classes A, B, and C is 2:3,8:9, and 2:3 e) None of these
respectively. The number of girls in class D is

Click Here For Bundle PDF Course | support@guidely.in Page 5 of 9


SBI Clerk & RRB PO Mains PDF Course 2023
Quantitative Aptitude Day - 23 (Eng)

Directions (19 - 20): Study the following b) 42L


information carefully and answer the questions c) 25L
given below. d) 38L
The ratio of milk and water in container A is 4:3 e) None of these
and the ratio of milk and water in container B is
5:3. Container C contains 40 L pure milk and 20) At first 20L of milk is mixed in container D
container D contains 30 L water. The amount of then 75% mixture from container A is mixed in
milk in container A is 50% more than the amount container D. After that 1.25 L water is added to
of milk in container C and the ratio of the amount the mixture. Find the ratio of milk and water at
of water in containers B and D is 3:5. the end of container D?
19) If 20% mixture from container A and 33.33% a) 1:1
mixture from container B is mixed in container C, b) 2:1
find the difference between the amount of milk c)3:2
and water in container C at the end? d) 1:2
a) 47L e) None of these
Click Here to Get the Detailed Video Solution for the above given Questions
Or Scan the QR Code to Get the Detailed Video Solutions

Answer Key with Explanation

Directions (01 - 05):


So, actual sales of grapes is 60%, so, 60%=180,
total target sales is 100%=[180/60]*100=300
So, 24%=300, 100%=[300/24]*100=1250

1) Answer: C
Required revenue =
180*80+105*60+225*40=Rs.29700

Click Here For Bundle PDF Course | support@guidely.in Page 6 of 9


SBI Clerk & RRB PO Mains PDF Course 2023
Quantitative Aptitude Day - 23 (Eng)

B+C=80, B=30
2) Answer: D For team A,
Total actual sales of fruit strawberry are The Number of matches played = 4*C=4*50=200
260*14/13=280kg The Number of matches drawn = 2*40=80
So, target sales for strawberries are The number of matches lost=40
[280/70]*100=400kg The number of matches won=200-80-40=80
Total target sales of strawberry and papaya are So, the winning percentage of team A is
400+325=725kg [80/200]*100=40%
Similarly, we can calculate the value of the other
3) Answer: B also.
Required ratio
=[(225-180)+(300-180)] : [(150-105)+(325-
260)]=165:110=3:2

4) Answer: A
Required difference =[180+260]- 6) Answer: C
[180+105]=155kg Required difference = [43.75-40]%=3.75%

5) Answer: A 7) Answer: D
Actual sales of orange is = The total number of the match won by team F is
[105/(180+105+180+260+225)]*360=39.780 30+10=40
Actual sales of Mango is = The total number of matches played by team F is
[225/(180+105+180+260+225)]*360=85.260 =100*6/5=120
So, the difference is = 85.26-39.78=45.480 So, winning percentage = [40/120]*100=33.33%

Directions (06 - 10): 8) Answer: B


From the graph, we see that the number of Required ratio = [80+50+70]
matches lost by team A and the number of :[50+40+50]=200:140=10:7
matches drawn by the team B are in the same
line. So, we can say A=40. 9) Answer: A
In the same way number of matches drawn by 3A+4B-B% of 320+3C-C% of 420+325
teams, C and E are the same, i.e., 50. =3*40+4*30-30*320/100+3*50-
So, C=50 50*420/100+325=409

Click Here For Bundle PDF Course | support@guidely.in Page 7 of 9


SBI Clerk & RRB PO Mains PDF Course 2023
Quantitative Aptitude Day - 23 (Eng)

Required difference=27900-21360=Rs. 6540


10) Answer: A
After the next 50 matches the total number of 13) Answer: B
winning the match is 60+[50-15-9]=86 I. Total expenditure in the grocery of all three
New winning percentage = together is Rs.17300.
[86/170]*100=50.58% II. Total expenditure in rent of all three together
Increase by 50.58-50=0.58% is Rs.14900.
III. Total expenditure in transport of all three
Directions (11 - 15): together is Rs.11700.
So, None is true.

14) Answer: A
Required ratio = [4000+5400] : [5600+4500]=
9400:10100=94:101

15) Answer: A
The new average income of A and B is
[28000*115/100+27800*120/100]/2=32780
Required percentage =
[32780/25360]*100=129.26%=130%

Directions (16 - 18):


Let, the number of boys and girls in class A be
2x and 3x.
11) Answer: C
Let, the number of boys and girls in class A be
The total expenditure of D is
8y and 9y.
=5400*110/100+5200*120/100+4000*110/100+6
Let, the number of boys and girls in class A be
000*120/100=Rs.23780
2z and 3z.
Total saving of D is =32500-23780= Rs.8720
2x+8y=52,-i 3x+9y=66,-ii 9y+3z=84 -iii
By solving i and ii we get, x=10 and y=4
12) Answer: D
So, 9*4+3z=84, z=[84-36]/3=16
The average income of A and B is
So, the number of boys and girls in class A is
[28000+27800]/2=55800/2=27900
2*10=20 and 3*10=30
Total saving is = 8400+7200+5760=21360

Click Here For Bundle PDF Course | support@guidely.in Page 8 of 9


SBI Clerk & RRB PO Mains PDF Course 2023
Quantitative Aptitude Day - 23 (Eng)

The number of boys and girls in class B is Directions (19 - 20):


8*4=32 and 9*4=36 Milk in container C is 40 L.
The number of boys and girls in class C is Milk in container A is 40*150/100=60L
2*16=32 and 3*16=48 Water in container A is 60*3/4=45L
The Number of boys in D is 57-32=25 Water in container D is 30 L.
The Number of boys in E is 139-20-32-32-25=30 Water in container B is 30*3/5=18L
The Number of girls in D is 25*140/100=35 Milk in container B is 18*5/3=30L
The Number of girls in E is 30*120/100=36 19) Answer: A
16) Answer: C Milk in container C is
Required percentage 40+60*20/100+30*33.33/100=40+12+10=62L
=[(50+68)/(80+60)]*100=84.28% Water in container C is
45*20/100+18*33.33/100=9+6=15L
17) Answer: D Required difference = 62-15=47L
The number of students in class F is
32+48+15=95 20) Answer: A
The number of girls in class F is 95*2/5=38 The final amount of milk in D is
20+60*75/100=65L
18) Answer: B The final amount of water in D is
Required difference 30+45*75/100+1.25=65L
=[30+36+48+35+36]- 139=46 The required ratio is 65:65=1:1

Click Here For Bundle PDF Course | support@guidely.in Page 9 of 9


SBI Clerk & RRB PO Mains PDF Course 2023
ENGLISH Day - 23

English Language

Directions (01-05): In the following questions, 1. Choose the word that is most appropriate for
you have a passage where some of the words the blank labelled 1.
have been omitted. Read the passage carefully a) Whopping
and choose the correct answer to each blank out b) Enduring
of the five alternatives. c) Wavering
Despite the warm welcome given millions of d) Staggering
Ukraine refugees in the European Union since e) None of these.
the Russian invasion, EU officials said Tuesday
there are some fears of _________ (1)_________ 2. Choose the word that is most appropriate for
support caused by a slowing economy that the blank labelled 2.
especially affects poor families and the a) Emanating
__________ (2)________ influence of Russian b) Wailing
propaganda. EU Home Affairs Commissioner c) Ailing
Ylva Johansson stressed that support for the four d) Creeping
million Ukraine refugees being cared for in the e) None of these.
bloc is ________ (3)_________, but a special
report said “what we might call ‘solidarity fatigue’ 3. Choose the word that is most appropriate for
is beginning to set in in some member states.” the blank labelled 3.
“The cost of living crisis has hit low and medium a) Confluence
income families in host societies and created a b) Steadfast
context in which Russian propaganda could be c) Subsidized
more successful,” special EU adviser on Ukraine d) Faltering
Lodewijk Asscher said in the report. Russian e) None of these.
President Vladimir Putin “has a huge interest in
influencing public opinion on ________ 4. Choose the word that is most appropriate for
(4)________ persons from Ukraine. The ________ the blank labelled 4.
(5)________ on migration refugees can be a) Displaced
weaponised to sow division within the EU,” the b) Engulfed
Dutch politician said, adding that some of the 27 c) Fabled
EU member states have already reported more d) Oppressed
disinformation campaigns targeting the issue. e) None of these.

Click Here For Bundle PDF Course | support@guidely.in Page 1 of 8


SBI Clerk & RRB PO Mains PDF Course 2023
ENGLISH Day - 23

5. Choose the word that is most appropriate for to the civil servants being ‘torn between
the blank labelled 5. conflicting loyalties’ (E).
a) Prospect a) Both A and B.
b) Consensus b) Both B and C.
c) Outlook c) Both C and D.
d) Discourse d) Both A and C.
e) None of these. e) Both D and E.

Directions (06-10): In each of the following 8. The formal equality of the two Houses seems
question, a sentence has been divided into five to have been done away with (A)/ and the
parts. Among them, only two parts are Leader of the Lok Sabha in his oration could
erroneous, you are required to choose the have suggested (but did not), measures to (B)/
grammatically incorrect part as your answer. increase the working days to 90100 days as in
6. The Amended Act marks a paradigm shift in the past, initiate the practice of having a (C)/
this regard because it (A)/ allows CCI to enter Prime Minister’s Question Hour each week in
settlements or commitments with potential both Houses, and proposed the more (D)/
wrongdoers (B)/, those ensure quick and timely effective measuring for the functioning of the
market correction through a process (C)/ of Committee system to enhance their
discussion. This allows the CCI to collect effectiveness and public confidence (E)/.
penalties (D)/ and swiftly correct market a) Both A and B.
conditions without protracted litigation (E)/. b) Both B and C.
a) Both A and B. c) Both C and D.
b) Both B and C. d) Both A and C.
c) Both C and D. e) Both D and E.
d) Both A and C.
e) Both D and E. 9. States are critical actors in India’s energy
transition as there is a multitier (A) governance of
7. One consequence of this trend, reflective of energy production and usage. An effective (B)/
the unease generates by it (A)/, is a statement in transition will require bridging the ambitions and
the shape of a letter written to the President of implementing (C)/ gaps between the Centre and
India (B)/ recently by a group of former civil the State. Simultaneously, national ambitions
servants expressing concerns over (C)/ attempts (D)/ need to factor the varying incentive
by the government to change the character of structures, processes, and institutional capacities
the civil service and its functioning (D)/, leading at the State level (E)/.

Click Here For Bundle PDF Course | support@guidely.in Page 2 of 8


SBI Clerk & RRB PO Mains PDF Course 2023
ENGLISH Day - 23

a) Both A and B. b) CDAB.


b) Both B and C. c) ADCB.
c) Both C and D. d) CBAD.
d) Both A and C. e) No rearrangement required.
e) Both D and E.
12. The researchers (A) reveals the presence of
10. India prospered in its recent exchanges different magnetic elements, and by dominating
against Australia, be it at home or (A)/ Down (B) them back to specific sources of pollution, the
Under, in what is prove to be an exhilarating technique (C) could tell which sources were
rivalry, but summit (B)/ clashes are loaded with tracing (D) in different places.
extra pressure and both units will step onto the a) BACD.
turf as equals (C)/. It is a sobering reality that b) CDAB.
India last won an ICC silverware during the 2013 c) ADCB.
ICC (D)/ Champions Trophy in England and this d) CBAD.
is a historical vacuum that Rohit and company e) No rearrangement required.
will feel a need to address (E)/.
a) Both B and D. 13. Discharge of untreated (A) domestic and
b) Both B and C. industrial effluents into the rivers help (B) the
c) Both C and D. natural inter-tidal flow along the coast and the
d) Both A and C. mixing (C) of fresh water and saline water which
e) Both D and E. impede (D) in gradual formation of the Mangrove
forest.
Directions (11-15): In the following questions, a a) BACD.
sentence is given and four words are highlighted b) CDAB.
in each. The highlighted words are may or may c) ADCB.
not in their position. You are required to d) CBAD.
rearrange the highlighted parts to make a e) No rearrangement required.
meaningful sentence.
11. It’s a bit (A) of a trade-off so if we could 14. The company (A) posted its first quarterly
slightly press the accelerator (B) this year, and if revenue drop (B) in almost four years after
growth (C) is kept up, then that eases the pandemic-driven sales (C) on its China factories
consolidation in the years to come because the curtailed restrictions (D) of the latest iPhone
consolidation (D) required is 0.7% per year. during the holiday season.
a) BACD. a) BACD.

Click Here For Bundle PDF Course | support@guidely.in Page 3 of 8


SBI Clerk & RRB PO Mains PDF Course 2023
ENGLISH Day - 23

b) CDAB. d) acts like bio-shield against extreme climate


c) ADCB. events.
d) ABDC. e) No replacement required.
e) No rearrangement required.
17. We are looking other customers also, where
15. The shares (A) that were allocated to be sold we developed these whole ecosystem in
to retail investors and employees of the company hydrogen.
(B), however, were witnessed (C) amid the a) we will developing the whole ecosystems
extreme volatility undersubscribed (D) in the around hydrogen.
stocks of the Adani Group. b) we can develop these whole ecosystems
a) ABDC. around hydrogen.
b) CDAB. c) we will deploy the whole ecosystems of
c) ADCB. hydrogen.
d) CBAD. d) we can deploy the complete system of
e) No rearrangement required. hydrogen.
e) No replacement required.
Directions (16-20): In the following questions, a
sentence is given with a highlighted phrase. The 18. The increased precipitation will alter the
highlighted phrase is either grammatically or inflows to the dam in North and Central India
contextually incorrect. You are required to than south India and also hydropower
choose the correct phrase to replace the generation.
grammatically incorrect one. If the highlighted a) will alter the inflow to the dams more in.
phrase is grammatically correct, then choose b) will altar the inflow from the dams most in.
option E, no replacement required as your c) will alter the inflow to the dam mostly in.
answer. d) will altar the inflow from the dams more on.
16. They are important refugees of coastal e) No replacement required.
biodiversity and also acting as bio-shield for
extreme climatic event. 19. This has seen the metal being increasingly
a) act like bio-shield on extreme climate events. deployed at the forefront of global efforts to move
b) acts as bio-shield over extreme climatic to sustainable technologies with a lower carbon
events. footprint.
c) act as bio-shields against extreme climate a) global efforts for moving into sustained
events. technology for a.

Click Here For Bundle PDF Course | support@guidely.in Page 4 of 8


SBI Clerk & RRB PO Mains PDF Course 2023
ENGLISH Day - 23

b) global efforts in moving sustainable doing business and provided to swift market
technology with a. correction.
c) global effort for moving into sustained a) the ease of doing businesses and provides
technology for a. swiftly.
d) global effort to moving sustained technologies b) the ease of doing business and providing for
with a. shift.
e) No replacement required. c) the eases of doing businesses and providing
swift.
20. It is imperative that it keeps in mind the d) the ease of doing business and provides to
government’s objectives of enhancing ease in swift.
e) No replacement required.
Click Here to Get the Detailed Video Solution for the above given Questions
Or Scan the QR Code to Get the Detailed Video Solutions

Answer Key with Explanation

1. Answer: C requires an adjective which is negative to


Wavering – becoming weaker. describe the word “support”. So, the word
Whopping – very large. “wavering” should be used. Therefore, option C
Enduring – lasting over a period of time; durable. is the correct answer.
Staggering – deeply shocking or astonishing.
The first statement is about due to Russian 2. Answer: D
invasion, Ukraine people went as refugees to Creeping – happening, developing or moving
European Union. Even EU welcomed them slowly.
warmly, some fears are revolving there because Emanating – to express a quality or feeling.
of the slowdown of economy and the influence of Wailing – crying with pain and anger.
Russian propaganda. Here, the first blank Ailing – in poor health.

Click Here For Bundle PDF Course | support@guidely.in Page 5 of 8


SBI Clerk & RRB PO Mains PDF Course 2023
ENGLISH Day - 23

Here, the fear is due to the slowdown of Discourse – spoken or written communication
economy and the influence of Russian between people.
propaganda. Here, the word “influence” should Prospect – the possibility of likelihood.
be explained here, so the word “creeping” should Consensus – a general agreement.
be used. Therefore, option D is the correct The statement says that migrated refugees
answer. matter is getting to sow the division in the EU, for
that the word “discourse” should be used.
3. Answer: B Therefore, option D is the correct answer.
Steadfast – steady; fixed in a place.
Confluence – an act or process of merging. 6. Answer: B
Subsidized – supported financially. The error lies in part B and C. In part B, the word
Faltering – losing strength or momentum. “CCI” should be preceded by an article, because
Here, the word “but” is used which is used to CCI is a special entity, to describe some
connect one positive phrase and one negative specified entity or a group or a person, we need
phrase. According to this logic, the phrase after to use “the” before it. In part C, the word “those”
“but” is in negative tone, so the blank requires a is used, but the statement is about a single entity
word which can be expressed in a positive way. (singular), so, “those” should be changed to
So, the word “steadfast” should be used in the “that”. Therefore, option B is the correct answer.
blank. Therefore, option B is the correct answer.
7. Answer: D
4. Answer: A The error lies in part A and C, in part A, the word
Displaced – force someone to leave their home. “generates” is present which is incorrect,
Engulfed – to surround and cover something. because the word “by” indicates that the
Fabled – famous. sentence is in passive form, so “generated”
Oppressed – exploited or systematically harmed should be used. In part C, the word “concerns” is
by others. incorrect, according to the passage, the only
Here, the blank requires a word that should concern for all is to change to character of civil
describe the persons who came from Ukraine as service. So, the word “concern” should be used.
refugees. The word “displaced” can be used in Therefore, option D is the correct answer.
the given blank. Therefore, option A is the
correct answer. 8. Answer: E
The error lies in part D and E. In part D, the word
5. Answer: D “more” is used, it is an adjective to say that the

Click Here For Bundle PDF Course | support@guidely.in Page 6 of 8


SBI Clerk & RRB PO Mains PDF Course 2023
ENGLISH Day - 23

measure should be more effective. So, the word Here, the words are arranged in their appropriate
“the” cannot be used, it can be used only with position hence need not to be rearranged.
the word “most” in superlative forms. In part D, Therefore, option E is the correct answer.
the word “their” should be changed to “its”, as
the subject (committee system) is singular. 12. Answer: B
Therefore, option E is the correct answer. Here, the words at A and C should be
interchanged, because the word “reveals”
9. Answer: C indicates that the noun preceding it should be
The error lies in part C and D. In part C, the word singular. Following this logic, we can say the
“implementing” is used, it is incorrect. Rule of word “technique” should be placed at A. So,
parallelism should be applied here due to the options A and C get eliminated. The researchers
word “and”. “Ambitions” is a noun so the word say different materials are present in different
“implementation” should be used. In part D, the places. To know which material is present in
usage of “state” is incorrect, because the which place, we need to trace it, not to dominate
statement does not refer a single state, it it. Therefore, option B is the correct answer.
completely refers the whole states. Therefore,
option C is the correct answer. 13. Answer: C
Generally, if untreated water (contaminated
10. Answer: A water) is mixed into rivers, it will provide a
The error lies in part B and D, in part B the word negative impact to it. It will not help the river, so
“is” is present which indicates that the word according to this, we can say “impede” (stop or
should be in v+ing form, so the word “prove” hinder) should be placed at B, and the mixing of
should be changed to “proving”. In part D, the saline and fresh water provides a way to form
word “sober” is used which is incorrect, to mangrove ecosystem. Therefore, “help” should
describe a noun an adjective should be used. be placed at D, and option C is the correct
Even though the word “sober” is an adjective, we answer.
cannot use “a” before it. So, the word should be
changed to sobering. Therefore, option A is the 14. Answer: D
correct answer. Here, the word “restrictions”, “company” and
“sales” are nouns. The phrase “posted its”
11. Answer: E denotes the subject relating to it should be a
singular one. According to this logic, we cannot
use “restrictions” and “sales” as both are present

Click Here For Bundle PDF Course | support@guidely.in Page 7 of 8


SBI Clerk & RRB PO Mains PDF Course 2023
ENGLISH Day - 23

in plural form. So, we can conclude that the word In option A, the phrase “will developing” is
“company” is placed appropriately. Following present, which is grammatically incorrect. So,
this, options A and B get eliminated. Also, the option B is the correct answer.
word “revenue” can be said as “increase in
revenue” or “decline in revenue”. So, the word 18. Answer: A
“drop” should not be changed. Therefore, option Alter – change.
D is the correct answer. Altar – a table or platform for the presentation of
religious offerings.
15. Answer: A Here, the sentence talks about the inflow to the
The statement explains the company planned to dams in North and Central India, not particularly
sell the shares to the employees of the one dam. Also, the word “than” is present which
company, but due to the extreme volatility (rapid indicates this is a comparison statement, so
change), there were not enough people to buy it. “more” should be used. Following this, options B
The words at C and D should be changed. and C get eliminated. Option D contains the
Therefore, option A is the correct answer. word “altar” which is contextually incorrect here.
Therefore, option A is the correct answer.
16. Answer: C
Here, the first half of the sentence is present in 19. Answer: E
simple present tense, so the following should The highlighted phrase is contextually and
use only the simple present tense because of grammatically correct, hence need not to be
the word “and” which is used to connect two replaced. Therefore, option E is the correct
same phrases. According to this, the word “act” answer.
should be used as here the subject is plural
“they”. The word “bio-shields” represents “they”, 20. Answer: B
so it cannot be singular. Therefore, option C is Here, the government objective is to enhance
the correct answer. the business and to provide correction. If
“enhancing” is used, the word “providing” should
17. Answer: B be used because of “and”. Also, the word “ease”
The word “deploy” cannot be used, as the it cannot be taken as “eases”, so option C gets
ecosystem cannot be deployed, it can only be eliminated. Therefore, option B is the correct
developed. So, options C and D get eliminated. answer.

Click Here For Bundle PDF Course | support@guidely.in Page 8 of 8


SBI Clerk & RRB PO Mains PDF Course 2023
Reasoning Ability Day - 24 (Eng)

Reasoning Ability
Directions (1-5): Study the following information b) E and R; R and P
carefully and answer the given questions. c) R and D; P and B
Eight persons - P, B, R, D, E, K, G and M joined d) P and G; M and D
the school as a teacher in different Months from e) None of these
January to September of the same year. Only
one person joined the school in each month. 2) Which among the following pair of persons
They are teaching different Subjects viz. joined in the month which has an odd number of
Mathematics, English, Tamil, Physics, Chemistry, days?
Biology, History and Accounts. There is only one I. EG
vacant month in which nobody joined the School. II. BM
Only three persons joined the school before K, III. RK
who taught Tamil. R joined three months after IV. BG
the one who taught Tamil. The one who taught a) Only I, II and III
Physics joined the school in the month which has b) Only I, II and IV
the least number of days. Only two persons c) Only II and IV
joined between the one who taught Physics and d) Only II, III and IV
G. M taught Chemistry but neither joined in the e) Only IV
month which has an even number of days nor
joined before May. As many months after M as 3) If all the subject names are arranged in the
before D. Only one person joined between the dictionary order from January to September in
one who taught Biology and B, who did not join the occupied place, then which among the
after the one who taught physics. E joined one of following combination is false with respect to the
the adjacent months of K. The number of new arrangement?
persons joined between B and G is one more a) E – March – Chemistry
than the number of persons joined between G b) R – July – Mathematics
and P. Either E or R taught History. B taught c) G – May – History
neither Mathematics nor Accounts. The one who d) M – August – Physics
taught Accounts did not join in the month which e) All are true
has an even number of days.
1) The number of persons joined between __ and 4) __ joined three months before G and __
__ is two less than the number of persons joined joined three persons after K respectively.
between __ and __ respectively. a) The one who taught Physics and R
a) D and G; M and E b) M and the one who taught Biology

Click Here For Bundle PDF Course | support@guidely.in Page 1 of 10


SBI Clerk & RRB PO Mains PDF Course 2023
Reasoning Ability Day - 24 (Eng)

c) D and the one who taught Chemistry immediately before or immediately after U but
d) E and the one who taught Mathematics doesn’t like Kheer. The number of years between
e) None of these the years in which R and T were promoted is
same as the number of years between the years
5) If M and G did not join due to some reason in which T and U were promoted. Only two
and their subject was additionally taught by K persons were promoted between the one who
and D respectively, then which among the two likes Peda and the one who likes Barfi.
subjects were taught by D with respect to the 6) Who among the following person was
new arrangement? promoted in the prime numbered year?
a) Physics and Accounts a) The one who likes Kheer
b) Physics and Chemistry b) The one who likes Halwa
c) Tamil and Chemistry c) The one who was promoted immediately
d) Tamil and Accounts before T
e) Chemistry and History d) The one who was promoted three persons
before R
Directions (6-10): Study the following information e) None of these
carefully and answer the given questions.
SBI promoted their seven employees viz.- P, Q, 7) Who among the following person was
R, S, T, U, and V during eight different years viz.- promoted in the year, which is exactly divisible
1993, 1996, 1998, 2001, 2005, 2009, 2012, and by 3?
2015. Only one person was promoted in each a) The one who likes Rabri
year and no person was promoted in one of the b) The one who was promoted immediately
given years. Each person likes different sweets before P
viz.- Laddu, Peda, Kheer, Rabri, Kulfi, Barfi, and c) The one who likes Barfi
Halwa. d) Both a and c
P was promoted in 2015 but neither likes Barfi e) Both a and b
nor Kheer. Only three persons were promoted
between P and the one who likes Rabri. Q, who 8) As many years gap between U and the one
likes Laddu, was promoted one of the leap years. who likes Barfi as between __ and __.
The one who likes Halwa was promoted three a) S and the one who likes Laddu
years before Q. T likes Kulfi and was promoted in b) T and the one who likes Kheer
the year whose sum of the digits is a multiple of c) P and the one who likes Kulfi
seven. R was promoted after T but not promoted d) U and the one who likes Halwa
in an odd numbered year. V was promoted either e) V and Q

Click Here For Bundle PDF Course | support@guidely.in Page 2 of 10


SBI Clerk & RRB PO Mains PDF Course 2023
Reasoning Ability Day - 24 (Eng)

9) How many persons were promoted between factor nor a common multiple. For example: If P,
the one who likes Kheer and U? Q and R are sitting together, then the sum of the
a) As many persons promoted between Q and T marks scored by PQ and QR is neither a
b) As many persons promoted between R and V common factor nor a common multiple.
c) Two A sits second to the right of the one who scored
d) Both a and b 24 marks, both are not facing the same direction.
e) Both a and c Two persons sit between A and C, who scored
31 marks. C is facing the centre. E sits third to
10) If all the persons and sweet names are the left of B, where neither of them sits adjacent
arranged in the dictionary order from 1993 to to A. B scored 8 marks less than A. D sits
2015 in the occupied position, then which of the second to the right of B, where both of them are
following persons were promoted after the facing the same direction. C neither sits adjacent
vacant year? to H nor D. H sits second to the right of the one
I.V - Rabri who scored 32 marks. H faces the same
II. R - Kheer direction as A. The persons sitting adjacent to
III. U - Peda the one who scored 32 marks are facing opposite
a) Only II directions. The one who scored 43 marks sits
b) Only I and II third to the left of the one who scored 76 marks.
c) Only I and III G scored less marks than B but doesn’t face
d) Only II and III away from the centre.
e) Only III 11) What is the position of F with respect to the
one who scored 24 marks?
Directions (11-15): Study the following a) Second to the left
information carefully and answer the given b) Third to the left
questions. c) Immediate left
Eight persons A, B, C, D, E, F, G and H are d) Fourth to the right
sitting around a circular table in such a way that e) None of these
some are facing the centre whereas others are
facing away from the centre. Each person scored 12) Who among the following person sits second
different marks in maths subjects viz. 24, 25, 31, to the right of G?
32, 43, 54, 62 and 76. Not more than three a) The one who scored 62 marks
persons sit together facing the same direction. b) The one who sits immediate right of H
Note: The sum of the marks scored by pair of c) A
persons sitting together is neither a common d) The one who scored 76 marks

Click Here For Bundle PDF Course | support@guidely.in Page 3 of 10


SBI Clerk & RRB PO Mains PDF Course 2023
Reasoning Ability Day - 24 (Eng)

e) None of these “A@B” means “A is the daughter of B”.


“A&B” means “B is the father of A”.
13) If the one who scored 24 is related to H, “A%B” means “A is the sister of B”.
similarly the one who scored 54 is related to G, “A#B” means “B is the son of A”.
then who among the following person is related “A$B” means “A is the mother of B”.
to D? “A*B” means “B is the wife of A”.
a) The one who scored 31 marks “A!B” means “A is the brother of B”.
b) The one who scored 62 marks 16) If P@J*U, then how J is related to M and
c) The one who sits second to the left of F how many female members are there in this
d) The one who scored 43 marks family?
e) None of these K%G$L, U&M, U@T%L@V
a) Son-in-law and Five
14) How many persons are sitting between H b) Father-in-law and Six
and the one who scored 25 marks when counted c) Brother-in-law and Five
to the right of H? d) Son-in-law and Six
a) Two e) Father-in-law and Four
b) Three
c) Four 17) What comes in the blank space respectively
d) One such that P is the daughter-in-law of M?
e) No one P_G@T_H!K%Q; H*M_T!L
a)$, %, and #
15) Which of the following statement is/are true b) $, &, and #
with respect to the final arrangement? c) %, % and $
a) C sits immediate left of D d) $, % and &
b) The one who scored 54 marks sits third to the e) #, $ and #
right H
c) F scored 24 marks 18) Which of the following correctly represents B
d) H sits immediate right of B is the brother-in-law of F?
e) E sits opposite to the one who scored 25 a) M#B*L%F@Q
marks b) F@M*Q$L%B
c) A&B*M@Q$F
Directions (16-18): Study the following d) Both a and b
information carefully and answer the given e) Both a and c
questions.

Click Here For Bundle PDF Course | support@guidely.in Page 4 of 10


SBI Clerk & RRB PO Mains PDF Course 2023
Reasoning Ability Day - 24 (Eng)

Directions (19-20): Study the following 19) If all the unknown genders are female, then
information carefully and answer the given which of the following statement(s) is/are true?
questions. a) J+Q
‘A @ B’ means ‘B is the father of A’ b) D$∞M@R
‘A # B’ means ‘A is the mother of B’ c) Z@∞J+K
‘A $ B’ means ‘B is the sister of A’ d) Both a and c
‘A % B’ means ‘A is the brother of B’ e) Both b and c
‘A & B’ means ‘B is the wife of A’
‘A * B’ means ‘A is the husband of B’ 20) If M%∞D#∞F, then how is F related to P?
‘A + B’ means ‘B is the daughter of A’ a) Niece’s husband
‘A ^ B’ means ‘B is the son of A’ b) Nephew’s son-in-law
‘A +∞ B’ means ‘A is the daughter-in-law of B’ c) Nephew’s daughter-in-law
“∞” means in-law d) Sister’s grand-daughter
O#P$K@J; N%M@R*K; Z#Q@P; B@N*D e) Either b or c
Click Here to Get the Detailed Video Solution for the above given Questions
Or Scan the QR Code to Get the Detailed Video Solutions

Answer Key with Explanation

Directions (1-5):
1) Answer: D
2) Answer: B
3) Answer: E
4) Answer: C
5) Answer: A
Final arrangement

Click Here For Bundle PDF Course | support@guidely.in Page 5 of 10


SBI Clerk & RRB PO Mains PDF Course 2023
Reasoning Ability Day - 24 (Eng)

We have,
Again we have,
 Only three persons joined the school
 Only one person joined between the one
before K, who taught Tamil.
who taught Biology and B, who did not
 R joined three months after the one who
join after the one who taught physics.
taught Tamil.
 E joined one of the adjacent months of K.
 The one who taught Physics joined the
 The number of persons joined between B
school in the month which has the least
and G is one more than the number of
number of days.
persons joined between G and P.
 Only two persons joined between the one
 Either E or R taught History. B taught
who taught Physics and G.
neither Mathematics nor Accounts.
 M taught Chemistry but neither joined in
 The one who taught Accounts did not join
the month which has an even number of
in the month which has an even number
days nor joined before May.
of days.
 As many months after M as before D.
From the above condition case-2 gets eliminated
From the above conditions, there are three
because we cannot place E, vacant place and
possibilities
Case-1(a) also eliminated because we cannot
place the one who taught Accounts.
Hence Case-1 shows the final answer.

Click Here For Bundle PDF Course | support@guidely.in Page 6 of 10


SBI Clerk & RRB PO Mains PDF Course 2023
Reasoning Ability Day - 24 (Eng)

We have,
 P was promoted in 2015 but neither likes
Barfi nor Kheer.
 Only three persons were promoted
between P and the one who likes Rabri.
 Q, who likes Laddu, was promoted one of
the leap years.
 The one who likes Halva was promoted
three years before Q.
 T likes Kulfi and was promoted in the year
whose sum of the digits is a multiple of
seven.
From the above conditions, there are four
possibilities

Directions (6-10):
6) Answer: B
7) Answer: D
8) Answer: A
9) Answer: E
10) Answer: C
Final arrangement

Again we have,
 R was promoted after T but in an odd
numbered year.
 V was promoted either immediately
before or immediately after U but doesn’t
like Kheer..
 The number of years between the years
in which R and T were promoted is same

Click Here For Bundle PDF Course | support@guidely.in Page 7 of 10


SBI Clerk & RRB PO Mains PDF Course 2023
Reasoning Ability Day - 24 (Eng)

as the number of years between the years


in which T and U were promoted.
 Only two persons were promoted
between the one who likes Peda and the
one who likes Barfi.
From the above conditions, Case 1 gets
eliminated because we cannot place V and U,
Case 1(a) and Case 2(a) get eliminated because
we cannot place R We have,
Hence, Case-2 shows the final answer.  A sits second to the right of the one who
scored 24 marks, both are not facing the
same direction.
 Two persons sit between A and C, who
scored 31 marks.
 C is facing the centre.

From the above condition, there are four


possibilities

Directions (11-15):
11) Answer: A
12) Answer: D
Again we have,
13) Answer: B
 E sits third to the left of B, where neither
14) Answer: C
of them sits adjacent to A.
15) Answer: E
 B scored 8 marks less than A. (The
Final arrangement
possible outcome of B’s score is either 24
or 54 and A’s score is either 32 or 62)

Click Here For Bundle PDF Course | support@guidely.in Page 8 of 10


SBI Clerk & RRB PO Mains PDF Course 2023
Reasoning Ability Day - 24 (Eng)

 D sits second to the right of B, where both who score 76 marks sits between B and C.
of them are facing the same direction. Similarly for Case-2
 C neither sits adjacent to H nor D. So Case-1 gets eliminated, hence Case-2 shows
 H sits second to the right of the one who the final arrangement
scored 32 marks.
 H faces the same direction as A.
 The persons sitting adjacent to the one
who scored 32 marks are facing opposite
directions.
Here Case-1a gets eliminated because as if D’s
Directions (16-18):
score is 32 then A+D and D+ the one who sits
16) Answer: D
immediate left of D have common factors,
similarly for Case-2a

17) Answer: B

Again we have,
 The one who scored 43 marks sits third to
the left of the one who scored 76 marks.
 G scored less marks than B but doesn’t
face away from the centre.
In Case-1 H doesn’t score 76 marks, as if score 18) Answer: E

76, then AH and HE have common factors and if


the person sits between D and B score 76
marks, then D and the one who score 76 marks
and B and the one who score 76 marks have
common factors. So, the only possible is the one
Directions (19-20):

Click Here For Bundle PDF Course | support@guidely.in Page 9 of 10


SBI Clerk & RRB PO Mains PDF Course 2023
Reasoning Ability Day - 24 (Eng)

19) Answer: E
20) Answer: E

Click Here For Bundle PDF Course | support@guidely.in Page 10 of 10


SBI Clerk & RRB PO Mains PDF Course 2023
Quantitative Aptitude Day - 23 (Eng)

Quantitative Aptitude

Directions (01 - 05): Study the following information carefully and answer the questions given below.
The given pie chart shows the percentage distribution of the number of selling target (kg) of five fruits
[Apple, orange, grapes, Papaya, Mango] by a fruit seller, producer company.

Note- Red, orange, black, green, and yellow colors represent fruit apple, orange, grapes, papaya, and
mango respectively. The percentage of actual sales of fruit apples, Orange, grapes, papaya, and mango
out of target sales is 80%, 70%, 60%, 80%, and 90% respectively. A total of 180 kg of grapes were sold
by the company.
1) Cost of per kg apple sold by the company is fulfilled for fruit strawberries then find the total
Rs.80, Per kg orange sold by the company is target sales for strawberries and Papaya
Rs.60 and Per kg Mango sold by the company is together?
Rs.40. Find the total revenue earned by the a) 652kg
company by selling these three fruits? b) 852kg
a) Rs.28400 c) 752kg
b) Rs.25800 d) 725kg
c) Rs.29700 e) 951kg
d) Rs.27600
e) Rs.22300 3) Find the ratio between the total number of
sales of the target not achieved for fruit apples
2) Ratio of the actual sale of fruit strawberries and grapes together and for fruit oranges and
and papaya is 14:13. If 70% of the target is papaya together?

Click Here For Bundle PDF Course | support@guidely.in Page 1 of 9


SBI Clerk & RRB PO Mains PDF Course 2023
Quantitative Aptitude Day - 23 (Eng)

a) 2:3 d) 135kg
b) 3:2 e) None of these
c) 3:5
d) 3:7 5) Find the difference between central angle
e) None of these formed by actual sales of oranges and actual
sales of mango if the total actual sales of fruit are
4) Find the difference between the actual sales represented in the pie chart?
of apple and orange fruit together and the actual a) 45.480
sales of grapes and papaya fruit together? b) 62.480
a) 155kg c) 55.480
b) 120kg d) 35.480
c) 165kg e) None of these

Directions (06 - 10): Study the following information carefully and answer the questions given below.
The given bar graphs show the number of test matches played, no of test matches drawn and no of
matches lost by the five teams is given.

Note – B+C=80, Total number matches played= Number of matches won+ Number of matches drawn +
Number of matches lost.
Win% = [Total Number of the match won/total Number of matches played] *100

Click Here For Bundle PDF Course | support@guidely.in Page 2 of 9


SBI Clerk & RRB PO Mains PDF Course 2023
Quantitative Aptitude Day - 23 (Eng)

6) Find the difference between the winning a) 11:7


percentage of team A and the winning b) 10:7
percentage of team E? c) 19:12
a) 9.35% d) 18:11
b) 2.58% e) None of these
c) 3.75%
d) 1.25% 9) Find the value 3A+4B-B% of 320+3C-C% of
e) 6.35% 420+325=?
a) 409
7) Team F wins 10 matches more than team C. b) 502
Ratio of total matches played by team C and c) 650
team F is 5:6. Find the winning percentage of d) 582
team F? e) None of these
a) 30%
b) 38% 10) In the next 50 matches team D lost 15
c) 37% matches and draw 9 matches. Find the increase
d) 33.33% or decrease in winning percentage for team D
e) 39% after and before 50 matches?
a) 0.58%
8) Find the ratio between the total number of b) 0.68%
matches won by teams A, B, and E together and c) 0.21%
the total number of matches drawn by teams C, d) 0.32%
D, and E together? e) None of these

Directions (11 - 15): Study the following information carefully and answer the questions given below.
The given table shows the expenditure on rent, grocery, transport, bill and savings of five people is given.

Click Here For Bundle PDF Course | support@guidely.in Page 3 of 9


SBI Clerk & RRB PO Mains PDF Course 2023
Quantitative Aptitude Day - 23 (Eng)

Note – Total income of a person is the sum of expenditure in rent, grocery, transport, bill, and saving
together.
11) Expenditure of D in rent, grocery transport, b) Rs.4520
and bill are 10%,20%,10%, and 20% more than c) Rs.6520
that of B. If the total income of D is Rs. 32500 d) Rs.6540
then find the saving of D? e) Rs.6320
a) Rs.5420
b) Rs.6820 13) Find which one is true?
c) Rs.8720 I. Total expenditure in the grocery of all three
d) Rs.8520 together is Rs.17200.
e) Rs.9520 II. Total expenditure in rent of all three together is
Rs.13900.
12) Find the difference between the average III. Total expenditure in transport of all three
income of A and B together and the total saving together is Rs.12800.
of A, B, and C together? a) Only III is true
a) Rs.5200 b) None is true

Click Here For Bundle PDF Course | support@guidely.in Page 4 of 9


SBI Clerk & RRB PO Mains PDF Course 2023
Quantitative Aptitude Day - 23 (Eng)

c) Only II is true 40% more than the number of boys in the same
d) Only II and III are true class and number of Number of girls in class E is
e) None of these 20% more than the number of boys in class E.
The total number of boys in classes D and C is
14) Find the ratio of total expenditure in transport 57 and the number of boys in all schools
and rent of B and total expenditure in grocery together is 139.
and bill of C? 16) Total number of students in classes A and B
a) 94:101 together is what percent of the total number of
b) 97:112 students in classes C and D together?
c) 98:105 a) 45.35%
d) 91:101 b) 68.36%
e) None of these c) 84.28%
d) 78.33%
15) If the income of A and B is increased by 15% e) 55.32%
and 20% respectively. then find the new average
income of A and B is approximately what percent 17) Number of students in class F is 15 more
of the income of C? than the number of students in class C. The ratio
a) 130% of boys and girls in class F is 3:2. Find the
b) 140% number of girls in class F?
c) 160% a) 32
d) 110% b) 30
e) None of these c) 48
d) 38
Directions (16 - 18): Study the following e) 42
information carefully and answer the questions
given below. 18) Find the difference between the total number
The total number of boys in classes A and B of girls in all classes together and the total
together is 52 and the number of girls in class number of boys in all classes together?
classes B and C together is 84. The total number a) 51
of boys in classes B and C together is 64 and the b) 46
total number of girls in A and B together is 66. c) 52
The ratio of the number of boys and girls in d) 56
classes A, B, and C is 2:3,8:9, and 2:3 e) None of these
respectively. The number of girls in class D is

Click Here For Bundle PDF Course | support@guidely.in Page 5 of 9


SBI Clerk & RRB PO Mains PDF Course 2023
Quantitative Aptitude Day - 23 (Eng)

Directions (19 - 20): Study the following b) 42L


information carefully and answer the questions c) 25L
given below. d) 38L
The ratio of milk and water in container A is 4:3 e) None of these
and the ratio of milk and water in container B is
5:3. Container C contains 40 L pure milk and 20) At first 20L of milk is mixed in container D
container D contains 30 L water. The amount of then 75% mixture from container A is mixed in
milk in container A is 50% more than the amount container D. After that 1.25 L water is added to
of milk in container C and the ratio of the amount the mixture. Find the ratio of milk and water at
of water in containers B and D is 3:5. the end of container D?
19) If 20% mixture from container A and 33.33% a) 1:1
mixture from container B is mixed in container C, b) 2:1
find the difference between the amount of milk c)3:2
and water in container C at the end? d) 1:2
a) 47L e) None of these
Click Here to Get the Detailed Video Solution for the above given Questions
Or Scan the QR Code to Get the Detailed Video Solutions

Answer Key with Explanation

Directions (01 - 05):


So, actual sales of grapes is 60%, so, 60%=180,
total target sales is 100%=[180/60]*100=300
So, 24%=300, 100%=[300/24]*100=1250

1) Answer: C
Required revenue =
180*80+105*60+225*40=Rs.29700

Click Here For Bundle PDF Course | support@guidely.in Page 6 of 9


SBI Clerk & RRB PO Mains PDF Course 2023
Quantitative Aptitude Day - 23 (Eng)

B+C=80, B=30
2) Answer: D For team A,
Total actual sales of fruit strawberry are The Number of matches played = 4*C=4*50=200
260*14/13=280kg The Number of matches drawn = 2*40=80
So, target sales for strawberries are The number of matches lost=40
[280/70]*100=400kg The number of matches won=200-80-40=80
Total target sales of strawberry and papaya are So, the winning percentage of team A is
400+325=725kg [80/200]*100=40%
Similarly, we can calculate the value of the other
3) Answer: B also.
Required ratio
=[(225-180)+(300-180)] : [(150-105)+(325-
260)]=165:110=3:2

4) Answer: A
Required difference =[180+260]- 6) Answer: C
[180+105]=155kg Required difference = [43.75-40]%=3.75%

5) Answer: A 7) Answer: D
Actual sales of orange is = The total number of the match won by team F is
[105/(180+105+180+260+225)]*360=39.780 30+10=40
Actual sales of Mango is = The total number of matches played by team F is
[225/(180+105+180+260+225)]*360=85.260 =100*6/5=120
So, the difference is = 85.26-39.78=45.480 So, winning percentage = [40/120]*100=33.33%

Directions (06 - 10): 8) Answer: B


From the graph, we see that the number of Required ratio = [80+50+70]
matches lost by team A and the number of :[50+40+50]=200:140=10:7
matches drawn by the team B are in the same
line. So, we can say A=40. 9) Answer: A
In the same way number of matches drawn by 3A+4B-B% of 320+3C-C% of 420+325
teams, C and E are the same, i.e., 50. =3*40+4*30-30*320/100+3*50-
So, C=50 50*420/100+325=409

Click Here For Bundle PDF Course | support@guidely.in Page 7 of 9


SBI Clerk & RRB PO Mains PDF Course 2023
Quantitative Aptitude Day - 23 (Eng)

Required difference=27900-21360=Rs. 6540


10) Answer: A
After the next 50 matches the total number of 13) Answer: B
winning the match is 60+[50-15-9]=86 I. Total expenditure in the grocery of all three
New winning percentage = together is Rs.17300.
[86/170]*100=50.58% II. Total expenditure in rent of all three together
Increase by 50.58-50=0.58% is Rs.14900.
III. Total expenditure in transport of all three
Directions (11 - 15): together is Rs.11700.
So, None is true.

14) Answer: A
Required ratio = [4000+5400] : [5600+4500]=
9400:10100=94:101

15) Answer: A
The new average income of A and B is
[28000*115/100+27800*120/100]/2=32780
Required percentage =
[32780/25360]*100=129.26%=130%

Directions (16 - 18):


Let, the number of boys and girls in class A be
2x and 3x.
11) Answer: C
Let, the number of boys and girls in class A be
The total expenditure of D is
8y and 9y.
=5400*110/100+5200*120/100+4000*110/100+6
Let, the number of boys and girls in class A be
000*120/100=Rs.23780
2z and 3z.
Total saving of D is =32500-23780= Rs.8720
2x+8y=52,-i 3x+9y=66,-ii 9y+3z=84 -iii
By solving i and ii we get, x=10 and y=4
12) Answer: D
So, 9*4+3z=84, z=[84-36]/3=16
The average income of A and B is
So, the number of boys and girls in class A is
[28000+27800]/2=55800/2=27900
2*10=20 and 3*10=30
Total saving is = 8400+7200+5760=21360

Click Here For Bundle PDF Course | support@guidely.in Page 8 of 9


SBI Clerk & RRB PO Mains PDF Course 2023
Quantitative Aptitude Day - 23 (Eng)

The number of boys and girls in class B is Directions (19 - 20):


8*4=32 and 9*4=36 Milk in container C is 40 L.
The number of boys and girls in class C is Milk in container A is 40*150/100=60L
2*16=32 and 3*16=48 Water in container A is 60*3/4=45L
The Number of boys in D is 57-32=25 Water in container D is 30 L.
The Number of boys in E is 139-20-32-32-25=30 Water in container B is 30*3/5=18L
The Number of girls in D is 25*140/100=35 Milk in container B is 18*5/3=30L
The Number of girls in E is 30*120/100=36 19) Answer: A
16) Answer: C Milk in container C is
Required percentage 40+60*20/100+30*33.33/100=40+12+10=62L
=[(50+68)/(80+60)]*100=84.28% Water in container C is
45*20/100+18*33.33/100=9+6=15L
17) Answer: D Required difference = 62-15=47L
The number of students in class F is
32+48+15=95 20) Answer: A
The number of girls in class F is 95*2/5=38 The final amount of milk in D is
20+60*75/100=65L
18) Answer: B The final amount of water in D is
Required difference 30+45*75/100+1.25=65L
=[30+36+48+35+36]- 139=46 The required ratio is 65:65=1:1

Click Here For Bundle PDF Course | support@guidely.in Page 9 of 9


SBI Clerk & RRB PO Mains PDF Course 2023
ENGLISH Day - 23

English Language

Directions (01-05): In the following questions, 1. Choose the word that is most appropriate for
you have a passage where some of the words the blank labelled 1.
have been omitted. Read the passage carefully a) Whopping
and choose the correct answer to each blank out b) Enduring
of the five alternatives. c) Wavering
Despite the warm welcome given millions of d) Staggering
Ukraine refugees in the European Union since e) None of these.
the Russian invasion, EU officials said Tuesday
there are some fears of _________ (1)_________ 2. Choose the word that is most appropriate for
support caused by a slowing economy that the blank labelled 2.
especially affects poor families and the a) Emanating
__________ (2)________ influence of Russian b) Wailing
propaganda. EU Home Affairs Commissioner c) Ailing
Ylva Johansson stressed that support for the four d) Creeping
million Ukraine refugees being cared for in the e) None of these.
bloc is ________ (3)_________, but a special
report said “what we might call ‘solidarity fatigue’ 3. Choose the word that is most appropriate for
is beginning to set in in some member states.” the blank labelled 3.
“The cost of living crisis has hit low and medium a) Confluence
income families in host societies and created a b) Steadfast
context in which Russian propaganda could be c) Subsidized
more successful,” special EU adviser on Ukraine d) Faltering
Lodewijk Asscher said in the report. Russian e) None of these.
President Vladimir Putin “has a huge interest in
influencing public opinion on ________ 4. Choose the word that is most appropriate for
(4)________ persons from Ukraine. The ________ the blank labelled 4.
(5)________ on migration refugees can be a) Displaced
weaponised to sow division within the EU,” the b) Engulfed
Dutch politician said, adding that some of the 27 c) Fabled
EU member states have already reported more d) Oppressed
disinformation campaigns targeting the issue. e) None of these.

Click Here For Bundle PDF Course | support@guidely.in Page 1 of 8


SBI Clerk & RRB PO Mains PDF Course 2023
ENGLISH Day - 23

5. Choose the word that is most appropriate for to the civil servants being ‘torn between
the blank labelled 5. conflicting loyalties’ (E).
a) Prospect a) Both A and B.
b) Consensus b) Both B and C.
c) Outlook c) Both C and D.
d) Discourse d) Both A and C.
e) None of these. e) Both D and E.

Directions (06-10): In each of the following 8. The formal equality of the two Houses seems
question, a sentence has been divided into five to have been done away with (A)/ and the
parts. Among them, only two parts are Leader of the Lok Sabha in his oration could
erroneous, you are required to choose the have suggested (but did not), measures to (B)/
grammatically incorrect part as your answer. increase the working days to 90100 days as in
6. The Amended Act marks a paradigm shift in the past, initiate the practice of having a (C)/
this regard because it (A)/ allows CCI to enter Prime Minister’s Question Hour each week in
settlements or commitments with potential both Houses, and proposed the more (D)/
wrongdoers (B)/, those ensure quick and timely effective measuring for the functioning of the
market correction through a process (C)/ of Committee system to enhance their
discussion. This allows the CCI to collect effectiveness and public confidence (E)/.
penalties (D)/ and swiftly correct market a) Both A and B.
conditions without protracted litigation (E)/. b) Both B and C.
a) Both A and B. c) Both C and D.
b) Both B and C. d) Both A and C.
c) Both C and D. e) Both D and E.
d) Both A and C.
e) Both D and E. 9. States are critical actors in India’s energy
transition as there is a multitier (A) governance of
7. One consequence of this trend, reflective of energy production and usage. An effective (B)/
the unease generates by it (A)/, is a statement in transition will require bridging the ambitions and
the shape of a letter written to the President of implementing (C)/ gaps between the Centre and
India (B)/ recently by a group of former civil the State. Simultaneously, national ambitions
servants expressing concerns over (C)/ attempts (D)/ need to factor the varying incentive
by the government to change the character of structures, processes, and institutional capacities
the civil service and its functioning (D)/, leading at the State level (E)/.

Click Here For Bundle PDF Course | support@guidely.in Page 2 of 8


SBI Clerk & RRB PO Mains PDF Course 2023
ENGLISH Day - 23

a) Both A and B. b) CDAB.


b) Both B and C. c) ADCB.
c) Both C and D. d) CBAD.
d) Both A and C. e) No rearrangement required.
e) Both D and E.
12. The researchers (A) reveals the presence of
10. India prospered in its recent exchanges different magnetic elements, and by dominating
against Australia, be it at home or (A)/ Down (B) them back to specific sources of pollution, the
Under, in what is prove to be an exhilarating technique (C) could tell which sources were
rivalry, but summit (B)/ clashes are loaded with tracing (D) in different places.
extra pressure and both units will step onto the a) BACD.
turf as equals (C)/. It is a sobering reality that b) CDAB.
India last won an ICC silverware during the 2013 c) ADCB.
ICC (D)/ Champions Trophy in England and this d) CBAD.
is a historical vacuum that Rohit and company e) No rearrangement required.
will feel a need to address (E)/.
a) Both B and D. 13. Discharge of untreated (A) domestic and
b) Both B and C. industrial effluents into the rivers help (B) the
c) Both C and D. natural inter-tidal flow along the coast and the
d) Both A and C. mixing (C) of fresh water and saline water which
e) Both D and E. impede (D) in gradual formation of the Mangrove
forest.
Directions (11-15): In the following questions, a a) BACD.
sentence is given and four words are highlighted b) CDAB.
in each. The highlighted words are may or may c) ADCB.
not in their position. You are required to d) CBAD.
rearrange the highlighted parts to make a e) No rearrangement required.
meaningful sentence.
11. It’s a bit (A) of a trade-off so if we could 14. The company (A) posted its first quarterly
slightly press the accelerator (B) this year, and if revenue drop (B) in almost four years after
growth (C) is kept up, then that eases the pandemic-driven sales (C) on its China factories
consolidation in the years to come because the curtailed restrictions (D) of the latest iPhone
consolidation (D) required is 0.7% per year. during the holiday season.
a) BACD. a) BACD.

Click Here For Bundle PDF Course | support@guidely.in Page 3 of 8


SBI Clerk & RRB PO Mains PDF Course 2023
ENGLISH Day - 23

b) CDAB. d) acts like bio-shield against extreme climate


c) ADCB. events.
d) ABDC. e) No replacement required.
e) No rearrangement required.
17. We are looking other customers also, where
15. The shares (A) that were allocated to be sold we developed these whole ecosystem in
to retail investors and employees of the company hydrogen.
(B), however, were witnessed (C) amid the a) we will developing the whole ecosystems
extreme volatility undersubscribed (D) in the around hydrogen.
stocks of the Adani Group. b) we can develop these whole ecosystems
a) ABDC. around hydrogen.
b) CDAB. c) we will deploy the whole ecosystems of
c) ADCB. hydrogen.
d) CBAD. d) we can deploy the complete system of
e) No rearrangement required. hydrogen.
e) No replacement required.
Directions (16-20): In the following questions, a
sentence is given with a highlighted phrase. The 18. The increased precipitation will alter the
highlighted phrase is either grammatically or inflows to the dam in North and Central India
contextually incorrect. You are required to than south India and also hydropower
choose the correct phrase to replace the generation.
grammatically incorrect one. If the highlighted a) will alter the inflow to the dams more in.
phrase is grammatically correct, then choose b) will altar the inflow from the dams most in.
option E, no replacement required as your c) will alter the inflow to the dam mostly in.
answer. d) will altar the inflow from the dams more on.
16. They are important refugees of coastal e) No replacement required.
biodiversity and also acting as bio-shield for
extreme climatic event. 19. This has seen the metal being increasingly
a) act like bio-shield on extreme climate events. deployed at the forefront of global efforts to move
b) acts as bio-shield over extreme climatic to sustainable technologies with a lower carbon
events. footprint.
c) act as bio-shields against extreme climate a) global efforts for moving into sustained
events. technology for a.

Click Here For Bundle PDF Course | support@guidely.in Page 4 of 8


SBI Clerk & RRB PO Mains PDF Course 2023
ENGLISH Day - 23

b) global efforts in moving sustainable doing business and provided to swift market
technology with a. correction.
c) global effort for moving into sustained a) the ease of doing businesses and provides
technology for a. swiftly.
d) global effort to moving sustained technologies b) the ease of doing business and providing for
with a. shift.
e) No replacement required. c) the eases of doing businesses and providing
swift.
20. It is imperative that it keeps in mind the d) the ease of doing business and provides to
government’s objectives of enhancing ease in swift.
e) No replacement required.
Click Here to Get the Detailed Video Solution for the above given Questions
Or Scan the QR Code to Get the Detailed Video Solutions

Answer Key with Explanation

1. Answer: C requires an adjective which is negative to


Wavering – becoming weaker. describe the word “support”. So, the word
Whopping – very large. “wavering” should be used. Therefore, option C
Enduring – lasting over a period of time; durable. is the correct answer.
Staggering – deeply shocking or astonishing.
The first statement is about due to Russian 2. Answer: D
invasion, Ukraine people went as refugees to Creeping – happening, developing or moving
European Union. Even EU welcomed them slowly.
warmly, some fears are revolving there because Emanating – to express a quality or feeling.
of the slowdown of economy and the influence of Wailing – crying with pain and anger.
Russian propaganda. Here, the first blank Ailing – in poor health.

Click Here For Bundle PDF Course | support@guidely.in Page 5 of 8


SBI Clerk & RRB PO Mains PDF Course 2023
ENGLISH Day - 23

Here, the fear is due to the slowdown of Discourse – spoken or written communication
economy and the influence of Russian between people.
propaganda. Here, the word “influence” should Prospect – the possibility of likelihood.
be explained here, so the word “creeping” should Consensus – a general agreement.
be used. Therefore, option D is the correct The statement says that migrated refugees
answer. matter is getting to sow the division in the EU, for
that the word “discourse” should be used.
3. Answer: B Therefore, option D is the correct answer.
Steadfast – steady; fixed in a place.
Confluence – an act or process of merging. 6. Answer: B
Subsidized – supported financially. The error lies in part B and C. In part B, the word
Faltering – losing strength or momentum. “CCI” should be preceded by an article, because
Here, the word “but” is used which is used to CCI is a special entity, to describe some
connect one positive phrase and one negative specified entity or a group or a person, we need
phrase. According to this logic, the phrase after to use “the” before it. In part C, the word “those”
“but” is in negative tone, so the blank requires a is used, but the statement is about a single entity
word which can be expressed in a positive way. (singular), so, “those” should be changed to
So, the word “steadfast” should be used in the “that”. Therefore, option B is the correct answer.
blank. Therefore, option B is the correct answer.
7. Answer: D
4. Answer: A The error lies in part A and C, in part A, the word
Displaced – force someone to leave their home. “generates” is present which is incorrect,
Engulfed – to surround and cover something. because the word “by” indicates that the
Fabled – famous. sentence is in passive form, so “generated”
Oppressed – exploited or systematically harmed should be used. In part C, the word “concerns” is
by others. incorrect, according to the passage, the only
Here, the blank requires a word that should concern for all is to change to character of civil
describe the persons who came from Ukraine as service. So, the word “concern” should be used.
refugees. The word “displaced” can be used in Therefore, option D is the correct answer.
the given blank. Therefore, option A is the
correct answer. 8. Answer: E
The error lies in part D and E. In part D, the word
5. Answer: D “more” is used, it is an adjective to say that the

Click Here For Bundle PDF Course | support@guidely.in Page 6 of 8


SBI Clerk & RRB PO Mains PDF Course 2023
ENGLISH Day - 23

measure should be more effective. So, the word Here, the words are arranged in their appropriate
“the” cannot be used, it can be used only with position hence need not to be rearranged.
the word “most” in superlative forms. In part D, Therefore, option E is the correct answer.
the word “their” should be changed to “its”, as
the subject (committee system) is singular. 12. Answer: B
Therefore, option E is the correct answer. Here, the words at A and C should be
interchanged, because the word “reveals”
9. Answer: C indicates that the noun preceding it should be
The error lies in part C and D. In part C, the word singular. Following this logic, we can say the
“implementing” is used, it is incorrect. Rule of word “technique” should be placed at A. So,
parallelism should be applied here due to the options A and C get eliminated. The researchers
word “and”. “Ambitions” is a noun so the word say different materials are present in different
“implementation” should be used. In part D, the places. To know which material is present in
usage of “state” is incorrect, because the which place, we need to trace it, not to dominate
statement does not refer a single state, it it. Therefore, option B is the correct answer.
completely refers the whole states. Therefore,
option C is the correct answer. 13. Answer: C
Generally, if untreated water (contaminated
10. Answer: A water) is mixed into rivers, it will provide a
The error lies in part B and D, in part B the word negative impact to it. It will not help the river, so
“is” is present which indicates that the word according to this, we can say “impede” (stop or
should be in v+ing form, so the word “prove” hinder) should be placed at B, and the mixing of
should be changed to “proving”. In part D, the saline and fresh water provides a way to form
word “sober” is used which is incorrect, to mangrove ecosystem. Therefore, “help” should
describe a noun an adjective should be used. be placed at D, and option C is the correct
Even though the word “sober” is an adjective, we answer.
cannot use “a” before it. So, the word should be
changed to sobering. Therefore, option A is the 14. Answer: D
correct answer. Here, the word “restrictions”, “company” and
“sales” are nouns. The phrase “posted its”
11. Answer: E denotes the subject relating to it should be a
singular one. According to this logic, we cannot
use “restrictions” and “sales” as both are present

Click Here For Bundle PDF Course | support@guidely.in Page 7 of 8


SBI Clerk & RRB PO Mains PDF Course 2023
ENGLISH Day - 23

in plural form. So, we can conclude that the word In option A, the phrase “will developing” is
“company” is placed appropriately. Following present, which is grammatically incorrect. So,
this, options A and B get eliminated. Also, the option B is the correct answer.
word “revenue” can be said as “increase in
revenue” or “decline in revenue”. So, the word 18. Answer: A
“drop” should not be changed. Therefore, option Alter – change.
D is the correct answer. Altar – a table or platform for the presentation of
religious offerings.
15. Answer: A Here, the sentence talks about the inflow to the
The statement explains the company planned to dams in North and Central India, not particularly
sell the shares to the employees of the one dam. Also, the word “than” is present which
company, but due to the extreme volatility (rapid indicates this is a comparison statement, so
change), there were not enough people to buy it. “more” should be used. Following this, options B
The words at C and D should be changed. and C get eliminated. Option D contains the
Therefore, option A is the correct answer. word “altar” which is contextually incorrect here.
Therefore, option A is the correct answer.
16. Answer: C
Here, the first half of the sentence is present in 19. Answer: E
simple present tense, so the following should The highlighted phrase is contextually and
use only the simple present tense because of grammatically correct, hence need not to be
the word “and” which is used to connect two replaced. Therefore, option E is the correct
same phrases. According to this, the word “act” answer.
should be used as here the subject is plural
“they”. The word “bio-shields” represents “they”, 20. Answer: B
so it cannot be singular. Therefore, option C is Here, the government objective is to enhance
the correct answer. the business and to provide correction. If
“enhancing” is used, the word “providing” should
17. Answer: B be used because of “and”. Also, the word “ease”
The word “deploy” cannot be used, as the it cannot be taken as “eases”, so option C gets
ecosystem cannot be deployed, it can only be eliminated. Therefore, option B is the correct
developed. So, options C and D get eliminated. answer.

Click Here For Bundle PDF Course | support@guidely.in Page 8 of 8


SBI Clerk & RRB PO Mains PDF Course 2023
Reasoning Ability Day - 25 (Eng)

Reasoning Ability
Directions (1-5): A string of letters are given as 2) How many letters are there in the English
input. Some conditions are given in four different alphabetical series between the sixth letters from
steps. Study the following information carefully both ends in step III?
and answer the given questions. a) Eight
Input: D G A F L I W P E Y S U H C Z O M J X B b) Fifteen
RKNTVQ c) Twelve
Step I: The first half of the letters is written in d) Three
reverse order after the second half of the letters. e) None of these
Step II: All the letters are changed to their
corresponding reverse letter as per the 3) How many consonants from the second half of
alphabetical series. the English alphabetical series are in step IV,
Step III: If the letters which come in the second which are immediately followed by a vowel?
half of the alphabetical series are immediately a) Four
followed by a vowel, then the letters are changed b) Two
to the second previous letter as per the c) Three
alphabetical series, else if the letters which come d) One
in the first half of the alphabetical series are e) None
immediately preceded by a vowel, then the
letters are changed to the third succeeding letter 4) Which of the following element is eight from
as per the alphabetical series. the right end in step IV?
Step IV: All the letters that appeared more than a) B
once are dropped from the series and the b) F
remaining letters are written in reverse order. c) P
Step IV is the final output of the given input. d) E
1) Which of the following letter is fifth to the left of e) None of these
the second vowel from the right end in step II?
a) I 5) _____ is fourth to the ____ of the letter which is
b) M _____ from the left end of step IV.
c) B a) Z, left, sixth
d) K b) F, right, Second
e) None of these c)G, Right, Seventh
d) Both a and c
e) Both a and b

Click Here For Bundle PDF Course | support@guidely.in Page 1 of 10


SBI Clerk & RRB PO Mains PDF Course 2023
Reasoning Ability Day - 25 (Eng)

Directions (6-10): Study the following information a) Only III


carefully and answer the given questions. b) All I, II, and III
A school conducts eight matches of four different c) Both II and III
sports on the sports day. The matches were held d) Both I and III
between 8 AM to 6 PM. Two matches for cricket, e) None of these
two matches for volleyball, three matches for
football, and one match for badminton. Each 8) Which of the following match was held
match was held for one hour. There are two immediately before Tea break?
breaks between the matches viz.- 1 PM – 2 PM a) Football
lunch break and tea break at 4 PM-5 PM. b) Cricket
Note: No two consecutive matches were held for c) Badminton
the same sport. d) None
One of the Cricket matches was held after the e) Volleyball
lunch break. Two Football matches were held at
a gap of 120 minutes. None of the football 9) Football is not played at which of the following
matches were held immediately after and time?
immediately before lunch break. Badminton a) 5 PM-6 PM
match was held immediately after one of the b) 11 AM-12 AM
cricket matches but not held after the lunch c) 2 PM-3 PM
break. Only one match was held between both d) 8 AM-9 AM
Volleyball matches. e) None of these
6) Badminton match was held at which of the
following time? 10) How many matches were held before the first
a) 12 PM-1 PM cricket match?
b) 9 AM-10 AM a) Three
c) 3 PM-4 PM b) One
d) 10 AM-11 AM c) Two
e) None of these d)Four
e) None
7) Which of the following match was held
between Lunch and Tea break? Directions (11-15): Study the following
I. Cricket information carefully and answer the questions
II. Football given below.
III. Volleyball

Click Here For Bundle PDF Course | support@guidely.in Page 2 of 10


SBI Clerk & RRB PO Mains PDF Course 2023
Reasoning Ability Day - 25 (Eng)

Certain number of persons from the same family b) Fifteen


of three generations are sitting in a row facing c) Twelve
north. Information about only a few persons are d) Seventeen
known. Not more than three known persons are e) Ten
sitting together. Some of the person’s relations
may or may not be known. 13) How K is related to the one who sits
B, who is the only daughter-in-law of K, sits five immediate right of Y?
places away from R. R sits second to the right of a) Mother-in-law
K. As many persons sit between B and K as b) Grandmother
between K and G, who is the only daughter of K. c) Aunt
G sits third from one of the extreme ends. L, who d) Nephew
is the only son of G, sits third to the right of the e) None of these
one who sits immediate left of G.L sits fourth to
the left of M, who is married to B. The one who 14) Four of the following five are alike in a certain
sits third to the right of M is the father of G. Only way based on the given arrangementand thus
one person sits between M and W, who is the form a group. Which one of the followingdoes not
father of L. Only six persons sit between W and belong to the group?
Q, who is the only daughter of M. Q and Y sit a) PW
together. Only four persons sit between Y and Z, b) UM
who sits at the extreme end of the row. As many c) QB
persons sit between Z and K as between Q and d) WK
U. U sits four places away from P, who is the e) MR
sibling of W’s spouse’s niece.
11) ______ is the _______ of the one who sits 15) If Y is the sister-in-law of R, who has no
fourth to the left of _____. siblings, then how Y is related to the one who
a)W, Uncle, R sits third to the right of U?
b) P, Son, Q a) Maternal Aunt
c) G, Mother, M b) Brother
d) Both A and C c) Sister
e) Both A and B d) Daughter
e) None of these
12) How many persons are sitting to the right of
B’s sister-in-law?
a) Eight

Click Here For Bundle PDF Course | support@guidely.in Page 3 of 10


SBI Clerk & RRB PO Mains PDF Course 2023
Reasoning Ability Day - 25 (Eng)

Directions (16-20):Read the given passage 17) Which of the following can be a probable
carefully and answer the questions based on the ‘Course Of Action’ to pacify the anxious people?
same respectively a) All the protestors should be jailed so that more
In scenes reminiscent of the Arab Spring, (A) people don’t join the protests
citizens stormed the residences of Sri Lanka’s b) The protestors should be promised with free
president and prime minister in Colombo, (B) food and basic facilities for all of them
forcing both leaders to announce that they would c) Present heads of the country should resign
quit their respective offices. President Gotabaya with immediate effect
Rajapaksa, the main target of the protests that d) A new government should take charge and try
erupted in April over food and fuel shortages and to restore the trust of the people
have continued since, promised to resign by e) The protestors should be given employment
Wednesday. The Speaker of Sri Lanka’s which would ensure consistent money flow
parliament, Yapa Abeywardena, is expected to
take over as acting president and prime minister, 18) Which option can be marked as an
and facilitate, possibly, the setting up of a ‘Inference’ as per the contents of the given
national unity government. Gotabaya’s paragraph?
resignation could bring down the temperature a) The people of Sri Lanka have lost trust in their
and help the authorities to persuade the leadership
protestors to return home. Though the marches b) Gotabaya’s presence in the government has
and sit-ins in Galle Face and elsewhere have sparked ire
been largely peaceful, the recent violent events c) People of the island country have become
suggest that the people, battling acute shortage anxious
of essentials, including milk, baby food, petrol, d) Both (a) and (b)
are on the edge. Only a new leadership can win e) None of the given options
back public trust in government and steer the
island nation’s economy out of choppy waters. 19) Which of the given statements can most
16) Which of the given options provides the likely be taken as an ‘assumption’?
correct relationship between sentences A and B? a) The speaker takes over the roles of the
a) A is the cause and B is its effect estranged leaders till the government is formed
b) A is the effect and B is one of the causes b) An island country can only be ruled by a unity
c) Both A and B are causes government
d) Both A and B are effects of independent c) Violent protests were the only way left before
causes the people to show their agony
e) A is the effect and B is its cause

Click Here For Bundle PDF Course | support@guidely.in Page 4 of 10


SBI Clerk & RRB PO Mains PDF Course 2023
Reasoning Ability Day - 25 (Eng)

d) The last resort to save the economy was to b) A new political leadership is needed to steer
oust their leaders out of power Sri Lanka out of present impasse
e) All statements can be assumed c) The violent protests are a result of the
negligence of the government
20) Which of the following can be ‘concluded’ d) The food scarcity is a proof that the country
from the given paragraph? was being mismanaged
a) It is only the citizens who can bring the country e) Graft had taken the centre stage and the
out of the severe trouble economy was in doldrums while the government
enjoyed
Click Here to Get the Detailed Video Solution for the above given Questions
Or Scan the QR Code to Get the Detailed Video Solutions

Answer Key with Explanation

Direction (1-5): Step I: C Z O M J X B R K N T V Q H U S Y E P


1) Answer: C WILFAGD
2) Answer: C For Step II: All the letters are changed to their
3) Answer: C corresponding reverse letter as per the
4) Answer: D alphabetical series.
5) Answer: D Step II: X A L N Q C Y I P M G E J S FH B V K D
We have: ROUZTW
Input: D G A F L I W P E Y S U H C Z O M J X B For Step III: If the letters which come in the
RKNTVQ second half of the alphabetical series are
For Step I: The first half of the letters are written immediately followed by a vowel, then the letters
in reverse order after the second half of the are changed to the second previous letter as per
letters. the alphabetical series, else if the letters which
come in the first half of alphabetical series are

Click Here For Bundle PDF Course | support@guidely.in Page 5 of 10


SBI Clerk & RRB PO Mains PDF Course 2023
Reasoning Ability Day - 25 (Eng)

immediately preceded by a vowel, then the We have:


letters are changed to the third succeeding letter  One of the Cricket matches was held after
as per the alphabetical series. the lunch break.
Step III: V A O N Q C W I P M G E M S F HB V  Two Football matches were held at a gap
KDPMUZTW of 120 minutes.
For Step IV: All the letters that appeared more  None of the football matches were held
than once are removed and the remaining letters immediately after and immediately before
are written in reverse order. lunch break.
Step IV: T Z U D K B H F S E G I C Q N O A That means, in case (1) Cricket match
was held immediately after the lunch
Direction (6-10): break, in case (2) Cricket match was held
6) Answer: D two matches after the lunch break, in
7) Answer: D case (3) Cricket match was held four
8) Answer: E matches after the lunch break.
9) Answer: C Based on the above given information we have:
10) Answer: B

Again, we have:
 Badminton match was held immediately
after one of the cricket matches but not
held after the lunch break.
That means, badminton match must be held at
10 AM.

Click Here For Bundle PDF Course | support@guidely.in Page 6 of 10


SBI Clerk & RRB PO Mains PDF Course 2023
Reasoning Ability Day - 25 (Eng)

 Only one match was held between both 11) Answer: C


Volleyball matches. 12) Answer: B
That means, case (2) is not valid. 13) Answer: B
Based on the above given information we have: 14) Answer: A (Only two persons sit between the
given pair of persons, except option A)
15) Answer: A

We have:
 B, who is the only daughter-in-law of K,
sits five places away from R.
 R sits second to the right of K.
Again, we have: That means, in case (1) B sits fifth to the
Since, three football matches were held, and left of R, in case (2) B sits fifth to the right
none of the football match was held immediately of R.
before or after lunch break, thus case (2) and  As many persons sit between B and K as
case (3) is not valid. between K and G, who is the only
Based on the above given information we have: daughter of K.
 G sits third from one of the extreme ends.
Based on the above given information we have:

Linear Arrangement:

Again, we have:

Direction (11-15):

Click Here For Bundle PDF Course | support@guidely.in Page 7 of 10


SBI Clerk & RRB PO Mains PDF Course 2023
Reasoning Ability Day - 25 (Eng)

 L, who is the only son of G, sits third to That means, in case (2) Y sits immediate
the right of the one who sits immediate right of R, in case (2a) Y sits third to the
left of G. right of R.
 L sits fourth to the left of M, who is Based on the above given information we have:
married to B.
That means in case (2) M sits immediate
For Blood relation:
left of K, case (1) is not valid.
Based on the above given information we have:

Again, we have:
 Only four persons sit between Y and Z,
who sits at the extreme end of the row.
 As many persons sit between Z and K as
For linear arrangement: between Q and U.
That means, in case (2) U sits immediate
right of L, in case (2a) U sits immediate
left of L.

Case (1) is not valid as no place for M.  U sits four places away from P, who is the

Again, we have: sibling of W’s spouse’s niece.

 The one who sits third to the right of M is That means, in case (2) P sits immediate

the father of G. left of G, case (2a) is not valid.

 Only one person sits between M and W, Based on the above given information we have:

who is the father of L.


Since not more than three known persons
are sitting together, thus W doesn’t sit
adjacent to R.
 Only six persons sit between W and Q,
who is the only daughter of M.
 Q and Y sit together. Linear arrangement:

Click Here For Bundle PDF Course | support@guidely.in Page 8 of 10


SBI Clerk & RRB PO Mains PDF Course 2023
Reasoning Ability Day - 25 (Eng)

Option (b) is not a solution as it would put a lot of


pressure on the country and increase the
problem indirectly.

Case(2a) is not valid as not more than three It has been given in the paragraph that the

known persons are sitting together. people have become anxious as they have no
access to even the basic necessities so as per

16) Answer: E the content above, it has been said that a new

The first sentence ie; A talks about the crowd leadership should quickly take charge. Option (c)

that barged into the residences of the President could be correct had the issue been stopping the

and the PM. If we wonder, what made them do protests. But the issue asked is mentioned at the

the same? We get our answer for this is B which end of the paragraph. (e) becomes superfluous

says that the citizens wanted that these two looking at the situation given in the paragraph.

leaders should leave their offices. So, we can So, the correct answer is option (d).

say that; because of B  A took place…


We can clearly understand that, A is the effect 18) Answer: D

and B is the cause. We have two options, (b) An Inference is something that can be deduced

and (e) to confuse from. on the basis of some known fact. It is not

If we go with (b) which says B is one of the something that is directly given in the paragraph.

causes, the other causes than this one should This becomes the reason that we cancel the

also be mentioned which is not the case. B is the third option as it has been discussed directly at

only immediate cause that has been quoted. multiple places in the paragraph.

So, this makes option (e) the best choice for the Towards the end of the paragraph, it has been

answer. given that a new government can restore the


trust of the people to inspire them to stop their

17) Answer: D violent protests. From this, we can infer that the

A Course of Action should always be inclined first option should be correct.

towards lessening the problem and never an It has been given that Gotabaya’s resignation

extreme step. The option that one chooses would bring down the anger. From this we can

should either solve the issue given in the context infer that his presence in the government has

or at least lessen the intensity of the same. sparked a rage in the country.

The first option is an extreme step which should Thus, of the three options, we can clearly see

be avoided as it won’t make any difference. that the first two can be inferred and the third

Click Here For Bundle PDF Course | support@guidely.in Page 9 of 10


SBI Clerk & RRB PO Mains PDF Course 2023
Reasoning Ability Day - 25 (Eng)

can be left out. So, the correct answer is option So, the best would be to mark option (a) as the
(d). answer.

19) Answer: A 20) Answer: B


An assumption should be chosen in the reverse A conclusion is logically deduced based on the
order. We must start with the options and try to contents of the given passage. The whole
find that if the particular option were assumed, paragraph revolves around one single idea that
would that lead us to any conclusion given in the the issues that have led to protests can be
paragraph or the sentences given. Like for addressed only with the change in the
example; government. The people have lost trust in their
If we take the first option to be an assumption, governance as they have been left at the mercy
we can very well conclude the point given in the of no one.
paragraph that the speaker Yapa would be The first option is superfluous and can be ruled
taking over the charge till the government is out. Option (c) is correct as given in the
formed. So, this is correct. paragraph but it cannot be taken as the
Option (b) cannot be taken as an assumption as conclusion. Same goes for option (d).
nothing about the same or related to the same We have no clue about graft and corruption
has been clearly mentioned. It has nowhere which makes it wrong for us to mark option (e).
been mentioned that the violent protests were So, it can be eliminated.
the only way out. Even the statement in (d) Thus, the correct answer is option (b).
doesn’t count for an assumption as it has directly
been discussed as an issue in the paragraph.

Click Here For Bundle PDF Course | support@guidely.in Page 10 of 10


SBI Clerk & RRB PO Mains PDF Course 2023
Quantitative Aptitude Day - 25 (Eng)

Quantitative Aptitude

Direction (1-5): Study the following data carefully and answer the questions:
Five students A, B, C, D and E participated in an exam consisting of three subjects Hindi, English and
Math. Maximum marks for each subject is different.
Table given below shows the following data.

Note:
1: Some data in the table is missing, which needs to be calculated in the questions.
2: Students D got 60% marks in Hindi and also 60% marks in English.
3: Students E got 81(1/3)% marks in Math.
1) If the average marks obtained by A, B and D e) Only Q and R
in Hindi, is 270, then which of the following is/are
true? 2) If A obtained total P% of maximum marks in
P: Ratio of marks obtained by B to D in English, the exam(including all the 3 subjects together)
is 4: 5 and C obtained 36 marks more in English than
Q: Difference between marks obtained B and E what A obtained in the same subject in such a
in Math, is 16. way that, C obtained P% of maximum marks in
R: Marks obtained by B in Math, is 32.5% of that English. Which of the following cannot be one of
obtained by him in Hindi. the factors of ‘P + 3’?
a) Only P a) 7
b) All are true b) 21
c) Only P and R c)9
d) Only Q d) 3

Click Here For Bundle PDF Course | support@guidely.in Page 1 of 13


SBI Clerk & RRB PO Mains PDF Course 2023
Quantitative Aptitude Day - 25 (Eng)

e) 11 e) 28%
Direction (6-10): Study the following data
3) If E obtained M% marks in English, N% marks carefully and answer the questions:
in Math and ratio of M to N is 54: 61, then find There are different number of shops in cities P
that the marks obtained by E in English is what and Q, which sells products A and B. In both the
per cent more than that obtained by D in cities, some shops sell product A only, some
English? shops sell product B only and remaining sell both
a) [3(N – M) – 1]% the products A and B.
b) In city P, ratio of number of shops, which sell
c)[4(N – M) + 2]% both the products A and B to the number of
d) [2(N – M) + 1]% shops, which sell product B only, is 5: 3 and the
e) [(N – M) + 3]% number of shops, which sell product A only, is 14
more than the number of shops, which sell
4) Average marks obtained by C in Hindi and product B only. In city Q, ratio of number of
English, is ‘P’ and total marks obtained by him in shops, which sell product A only to the number of
English and Math, is ‘P +184’. If ratio of marks shops, which sell product B only, is 2: 3 and the
obtained by A to C in Math is 29: 30, then what number of shops, which sell both the products A
must be added to ‘P’ to make it a perfect square? and B, is 8 less than the number of shops, which
a) 39 sell product A only. Number of shops in city Q,
b) 19 which sell product B only, is 24 more than that in
c) 23 city P, which sell product B only. Ratio of the
d) 25 number of shops in city P, which sell product A
e) 59 only to the number of shops in city Q, which sell
both the products A and B, is 8: 5.
5)If total marks obtained by D in Hindi and Math 6)Out of number of shops in city P that sells
together is % of the total maximum marks of product A only, 25% of the shops sells duplicate
Hindi and Math together, then find that the marks products and out of number of shops in city Q
obtained by D in Math is what per cent of that that sells product B only, % of the shops sells
obtained by him in English? duplicate products. Find the ratio of both of
a) 36% them?
b) 24% a) 4: 3
c) 30% b) 3: 2
d) 32% c) 11: 10

Click Here For Bundle PDF Course | support@guidely.in Page 2 of 13


SBI Clerk & RRB PO Mains PDF Course 2023
Quantitative Aptitude Day - 25 (Eng)

d) 10: 9 e) 60%
e) 4: 7 9) Total number of shops in cities P and Q
together that sell product B, sold product B of
7)The number of shops in city R, which sell two variants 1 and 2 in the ratio of 6: 5
product A, is 8 less than the number of shops in respectively. If cost of variant 1 and variant 2 of
city P, which sell product B. The number of product B is ₹ 150 and ₹ 200 respectively, then
shops in city R, which sell product B, is 12 more find the difference between total cost of variant 1
the number of shops in city Q, which sell product and variant 2 of product B sold by shops in both
A. If the total number of shops in city R, which the cities together?
sell product A and B, is 80, then find the number a) ₹ 800
of shops in city R, which sell both the products A b) ₹ 1000
and B? c)₹ 900
a) 25 d) ₹ 1200
b) 20 e) ₹ 1500
c)30
d) 10 10)Find the approximate average of total number
e) 15 of shops in city P that sells only one product,
total number of shops in city Q that sellsboth the
8) If M% of total number of shops in city P, sells products, and total number of shops in both the
product A only and N% of total number of shops cities together that sell product A only?
in city Q, sells product B only, and value of N is a) 43.33
more than the value of M, then find that b) 46.66
value of A is what percent of value of M? c) 33.33
a) 30% d) 36.66
b) 20% e) 53.33
c)40%
d) 5%

Direction (11-14): Study the following data carefully and answer the questions:
A person covered a certain distance by his car in 5 days (Monday to Friday). On each day, he covered
different distances at different speeds.
Pie chart given below shows the degree distribution of the distances covered in each of the days from
Mon to Fri.

Click Here For Bundle PDF Course | support@guidely.in Page 3 of 13


SBI Clerk & RRB PO Mains PDF Course 2023
Quantitative Aptitude Day - 25 (Eng)

Note:
1: Ratio of distance covered on Tue to that covered on Thu is 4: 5.
2: Ratio of distance covered on Wed to that covered on Fri is 5: 7.
3: The person drove his car at 75 km/h for 1.6 hours on Tue.
11)If the person drove his car for 1 hour 48 C: Ratio of the distance covered on Thu to that
minutes on Mon and for 1 hour 15 minutes on covered on Fri is 5: 4.
Wed, then find that the speed of the car on Wed a) Only A and B
was what per cent of more or less than that on b) Only C
Mon? c)Only B and C
a) 30% d) Only B
b) 75% e) Only A and C
c) 50%
d) 60% 13)Time taken by person to cover part of journey
e) 40% on Mon is ‘A’ min with 100 km/h and time taken
by another person to cover 95 km with speed 50
12) Which of the following is/are not true? km/h is ‘B’ min. Below given is a series of
A: Average of distances covered on Mon, Wed numbers that follows a certain pattern. Find the
and Fri is 110 km. difference of 6th and 8th term of that series.
B: The person covered 20% of the total distance A, X, 72, Y, B
on Tue. a) 78

Click Here For Bundle PDF Course | support@guidely.in Page 4 of 13


SBI Clerk & RRB PO Mains PDF Course 2023
Quantitative Aptitude Day - 25 (Eng)

b) 66 selling item A only is M%, which of the following


c)90 cannot be the possible value(s) of ‘M’?
d) 84 I: 25
e) 72 II: 27.5
III: 45
14)If the person drove his car at 60 km/h on Thu a) Only I
and the time, for which he drove his car on Thu, b) Only I and III
is 30 minutes more than that, for which he drove c) Only II
his car on Fri, then find that the speed of the car d) Only II and III
on Fri was what per cent of that on Tue? e) Only III
a) 87.5%
b) 83.33% 16)If item A sold at a profit of 11 (P + 5), then
c)92.5% profit percent earned after selling all the items
d) 90% together is P% and profit percent earned after
e) 93.33% selling item B is P ± ____ rupees. Which of the
following value will fill the blank?
Direction (15-18): Study the following data a) 5
carefully and answer the questions: b) 7
A shopkeeper has three articles A, B and C of c)2
different costs. He marked up each article at d) 6
different prices and sold each article by giving e) 8
different discounts on their marked prices.
Cost price of article A is ₹ 100 less than that of 17)If article B was marked up by M% and article
article C and ratio of cost price of article B to that C was marked up by N%, then the value of
of article C is 3: 2. Article A is marked up by lies between which of the following?
50%, article B is marked up by ₹ 300 and article P: 3 and 13
C is marked up by ₹ 120. Article B is sold at 20% Q: 6 and 15
discount on its marked price, article C is sold at R: 4 and 10
25% discount on its marked price and the selling a) Only P and Q
price of article B is ₹ 375 more than that of b) Only Q and R
article C. c)Only R
15)If profit or loss percent gained/incurred after d) Only P
selling all the three items A, B, and C together is e) Only P and R
10% and profit/loss percent gained/incurred after

Click Here For Bundle PDF Course | support@guidely.in Page 5 of 13


SBI Clerk & RRB PO Mains PDF Course 2023
Quantitative Aptitude Day - 25 (Eng)

18) If item A is further marked 15% above the a) (P – M, P – N), (Q – M, Q – O), (R – N, R – O)


previous marked price b) (P – M, P – O), (Q – M, Q – N), (R – N, R – O)
and sold at ₹ P after giving two successive c) (P – N, P – O), (Q – M, Q – N), (R – M, R – O)
discounts of ₹ 69 and 20% respectively. What d) (P – M, P – O), (Q – M, Q – O), (R – N, R – O)
will be the discount percent given on item A if the e) None of the combination is correct.
marked price of it will be the same as earlier and
selling price will be ₹ P only? 20) Solve each of the given equations of column
a) 13.5% A and find that which of the combination is not
b) 17.2% correct?
c)14.8%
d) 17.6%
e) 15.4%

Direction (19-20): In each question, there are two


a) P - N
columns A and B in the table. Quadratic
b) Q - O
equations are given in column A and their roots
c) R - O
are given in column B.
d) P - M
19) Solve each of the given equations of column
e) R - M
A and match with their roots in column B.

Click Here to Get the Detailed Video Solution for the above given Questions
Or Scan the QR Code to Get the Detailed Video Solutions

Answer Key with Explanation

Click Here For Bundle PDF Course | support@guidely.in Page 6 of 13


SBI Clerk & RRB PO Mains PDF Course 2023
Quantitative Aptitude Day - 25 (Eng)

Direction (1-5): So, Q is not true.


Since, the marks obtained by D in Hindi = 240 From R:
And D got 60% marks in Hindi. Marks obtained by B in Math = 2 * 212 – 320 =
So, the maximum marks for Hindi = = 104
400 Marks obtained by B in Hindi = 810 – 250 – 240
Since, the marks obtained by D in English = = 320
125% of 240 = 300 Required percentage = = 32.5%
And D got 60% marks in English also. So, R is true.
So, the maximum marks for English = Hence, only P and R are true.
= 500
Since, the marks obtained by E in Math = 2 * 211 2) Answer: E
– 300 = 122 Total maximum marks for the exam = 400 + 500
And E got 81(1/3)% marks in Math. + 150 = 1050
So, the maximum marks for Math = = Total marks obtained by A in the exam = P% of
150 1050 = 10.5P
1) Answer: C Marks obtained by A in Hindi = 250
Total marks obtained by A, B and D in Hindi = 3 Marks obtained by A in Math = 2 * 183 – 250 =
* 270 = 810 116
Marks obtained by B in Hindi = 810 – 250 – 240 Marks obtained by A in English = 10.5P – 250 –
= 320 116 = (10.5P – 366)
From P: Marks obtained by C in English = (10.5P – 366)
Marks obtained by B in English = 75% of 320 = + 36 = (10.5P – 330)
240 According to the question:
Marks obtained by D in English = 125% of 240 = P% of 500 = 10.5P – 330
300 5P = 10.5P – 330
Required ratio = 240: 300 = 4: 5 5.5P = 330
So, P is true. P = 60
From Q: P + 3 = 60 + 3 = 63 = 1 * 3 * 3 * 7
Marks obtained by B in Math = 2 * 212 – 320 = Factors of ‘P + 3’ = 1, 3, 7, 9, 21
104 Hence, 11 will not be a factor of ‘P + 3’.
Marks obtained by E in Math = 2 * 211 – 300 =
122 3) Answer: A
Required difference = 122 – 104 = 18

Click Here For Bundle PDF Course | support@guidely.in Page 7 of 13


SBI Clerk & RRB PO Mains PDF Course 2023
Quantitative Aptitude Day - 25 (Eng)

Marks obtained by E in Math = 2 * 211 – 300 = Total maximum marks of Hindi and Math
122 together = 400 + 150 = 550
So, N = Total marks obtained by D in Hindi and Math
And M = = 72% together = of 550 = 336
Marks obtained by E in English = 72% of 500 = Marks obtained by D in Hindi = 240
360 Marks obtained by D in English = 125% of 240 =
Marks obtained by D in English = 125% of 240 = 300
300 Marks obtained by D in math = 336 – 240 = 96
Required percentage = = 20% = Required percentage = = 32%
[3(N – M) – 1]%
Direction (6-10):
4) Answer: D Let the number of shops in city P, which sell both
Average marks obtained by C in Hindi and the products A and B = 5x
English = P So, the number of shops in city P, which sell
Marks obtained by C in English = (2P – 200) product B only = 3x
Total marks obtained by C in English and Math = And the number of shops in city P, which sell
(P +184) product A only = (3x + 14)
Marks obtained by C in Math =(P +184) – (2P – Let the number of shops in city Q, which sell
200) = (384 – P) product A only = 2y
Marks obtained by A in Math = 2 * 183 – 250 = So, the number of shops in city Q, which sell
116 product B only = 3y
According to the question: And the number of shops in city Q, which sell
29: 30 = 116: (384 – P) both the products A and B = (2y – 8)
120 = 384 – P Since, the number of shops in city Q, which sell
P = 264 product B only, is 24 more than that in city P,
√P = √264 = 16.2480… which sell product B only.
Nearest perfect square of an interest to 264 = So,
172 = 289 3y – 3x = 24
Hence, 25 must be added to P to make it a y – x = 8 ------------(1)
perfect square. Since, the ratio of the number of shops in city P,
which sell product A only to the number of shops
5) Answer: D in city Q, which sell both the products A and B, is
8: 5.

Click Here For Bundle PDF Course | support@guidely.in Page 8 of 13


SBI Clerk & RRB PO Mains PDF Course 2023
Quantitative Aptitude Day - 25 (Eng)

So, Number of shops in city Q that sells product B


only = 42
Number of shops in city Q that sells duplicate
15x + 70 = 16y – 64
product B= % of 42 = 14
16y – 15x = 134 ------------(2)
From equations (1) and (2): Required ratio = 8: 14

16(x + 8) – 15x = 134 = 4: 7

16x + 128 – 15x = 134


x=6 7) Answer: B

From equation (1): The number of shops in city P, which sell

y = 14 product B = 30 + 18 = 48

The number of shops in city P, which sell both So, the number of shops in city R, which sell

the products A and B = 30 product A = 48 – 8 = 40

The number of shops in city P, which sell The number of shops in city Q, which sell

product B only = 18 product A = 28 + 20 = 48

The number of shops in city P, which sell So, the number of shops in city R, which sell

product A only = 32 product B = 48 + 12 = 60

The number of shops in city Q, which sell Since, the total number of shops in city R, which

product A only = 28 sell products A and B = 80

The number of shops in city Q, which sell So, the number of shops in city R, which sell

product B only = 42 both the products A and B = (40 + 60) – 80 = 20

The number of shops in city Q, which sell both


the products A and B = 20 8) Answer: C
Total number of shops in city P, which sell
products A and B = 32 + 30 + 18 = 80
Number of shops in city P, which sell product A
only = 32
So, M = = 40%
Total number of shops in city Q, which sell
product A and B = 28 + 20 + 42 = 90
6) Answer: E
Number of shops in city Q, which sell product B
Number of shops in city P that sells product A
only = 42
only = 32
So, N =
Number of shops in city P that sells duplicate
According to the question:
product A = 25% of 32 = 8

Click Here For Bundle PDF Course | support@guidely.in Page 9 of 13


SBI Clerk & RRB PO Mains PDF Course 2023
Quantitative Aptitude Day - 25 (Eng)

Direction (11-14):
Since, ratio of distance covered on Tue to that
After comparing: covered on Thu is 4: 5.
302 + 3A = 350 So,
3A = 48
A = 16
5X + 60 = 4Y
Required percent =
4Y – 5X = 60 ------------(1)
= 40% Since, ratio of distance covered on Wed to that
covered on Fri is 5: 7.
9) Answer: B So,
Total number of shops in cities P and Q together
that sell product B = (30 + 18) + (20 + 42) = 110
7X = 5Y – 30
Total number of shops in cities P and Q together
5Y – 7X = 30 ------------(2)
that sell variant 1 of product B =
By equation (1) * 7 – equation (2) * 5:
Total number of shops in cities P and Q together
28Y – 35X – 25Y + 35X = 420 – 150
that sell variant 2 of product B =
Y = 90
Total cost of variant 1 of product B sold by shops
From equation (1):
in both the cities together = 60 * 150 = ₹ 9000
360 – 5X = 60
Total cost of variant 2 of product B sold by shops
X = 60
in both the cities together = 50 * 200 = ₹ 10000
Degree distribution of the distance covered on
Required difference = 10000 – 9000
Mon = 60 – 6 = 54°
= ₹ 1000
Degree distribution of the distance covered on
Tue = 60 + 12 = 72°
10) Answer: A
Degree distribution of the distance covered on
Total number of shops in city P that sells only
Wed = 60°
one product = 32 + 18 = 50
Degree distribution of the distance covered on
Total number of shops in city Q that sells both
Thu = 90°
the products = 20
Degree distribution of the distance covered on
Total number of shops in both the cities together
Fri = 90 – 6 = 84°
that sell product A only = 32 + 28 = 60
Since, the distance covered by the person on
Required average =
Tue = 75 * 1.6 = 120 km
= 43.33

Click Here For Bundle PDF Course | support@guidely.in Page 10 of 13


SBI Clerk & RRB PO Mains PDF Course 2023
Quantitative Aptitude Day - 25 (Eng)

So, the distance covered by the person on Mon So, C is not true.
= = 90 km Hence, only C is not true.
The distance covered by the person on Wed =
= 100 km 13) Answer: A
The distance covered by the person on Thu = Speed of person on Mon = 100 km/h
= 150 km Time taken by him to cover 90 km with speed of

And the distance covered by the person on Fri = 100 km/h = A min

= 140 km
11) Answer: D A = 54
The time, for which the person drove his car on Time taken by another person to cover 95 km
Mon = 1 hour 48 minutes = 1.8 hours with speed of 50 km/h = B min
So, the speed of the car on Mon = = 50 km/h
The time, for which the person drove his car on B = 114
Wed = 1 hour 15 minutes = 1.25 hours Number series: A, X, 72, Y, B
So, the speed of the car on Wed = = 80 km/h 54, 60, 72, 90, 114
Required percentage = = 60% Logic in the series:
54 + 6 = 60
12) Answer: B 60 + 12 = 72
From A: 72 + 18 = 90
Average of distances covered on Mon, Wed and 90 + 24 = 114
Fri: 114 + 30 = 144
144 + 36 = 180

So, A is true. 180 + 42 = 222

From B: 6th term of series = 144

Total distance covered by the person = 90 + 120 8th term of series = 222

+ 100 + 150 + 140 = 600 km Required difference = 222 – 144

The distance covered by the person on Tue = = 78

120 km
Required percentage = = 20% 14) Answer: E

So, B is true. Since, the person drove his car at 60 km/h on

From C: Thu.

Ratio of distance covered on Thu to that covered So, the time, for which he drove his car on Thu =

on Fri = 150: 140 = 15: 14 = 2.5 hours = 2 hours 30 minutes

Click Here For Bundle PDF Course | support@guidely.in Page 11 of 13


SBI Clerk & RRB PO Mains PDF Course 2023
Quantitative Aptitude Day - 25 (Eng)

The time, for which he drove his car on Fri = 2 Profit percent earned on item A =
hours
And the speed of the car on Fri = = 70 km/h Selling price of all the 3 items together when
The speed of the car on Tue = 75 km/h there is 10% loss = 90% of 1650 = ₹ 1485
Required percentage = = 93.33% Selling price of item A = 1485 – (840 + 465) =
₹ 180
Direction (15-18): Loss percent incurred on item A =
Let the CPs of articles B and C are ₹ ‘3x’ and
₹ ‘2x’ respectively. So, possible values of ‘M’ are 27.5 and 45 only.
So, the CP of article A = ₹ (2x – 100) Hence, statement II cannot be the possible
The MP of article A = 150% of (2x – 100) = ₹ (3x value.
– 150)
The MP of article B = ₹ (3x + 300) 16) Answer: E
And the MP of article C = ₹ (2x + 120) Cost price of item A = ₹ 400
Since, article B is sold at 20% discount on its Selling price of item A = 400 + 11 (P + 5) = (11P
marked price, article C is sold at 25% discount + 455)
on its marked price and the selling price of article Cost price of all the items together = 400 + 750 +
B is ₹ 375 more than that of article C. 500 = ₹ 1650
So, Selling price of all the items together = (11P +
455) + 840 + 465 = (11P + 1760)
According to the question:
240x + 24000 – 150x – 9000 = 37500
(100 + P)% of 1650 = 11P + 1760
x = 250
1650 + 16.5P = 11P + 1760
5.5P = 110
P = 20
Profit percent earned after selling item B =

15) Answer: B P ± ___ = 12


Cost price of all the 3 items together = 400 + 750 20 ± ___ = 12
+ 500 = ₹ 1650 Hence, value that can fill the blank is 8.
Selling price of all the 3 items together when
there is 10% profit = 110% of 1650 = ₹ 1815 17) Answer: A
Selling price of item A = 1815 – (840 + 465) = Since, M = = 40%
₹ 510

Click Here For Bundle PDF Course | support@guidely.in Page 12 of 13


SBI Clerk & RRB PO Mains PDF Course 2023
Quantitative Aptitude Day - 25 (Eng)

And, N = = 24% y(y – 15) – 14(y – 15) = 0


So, the value of = = 12 y = 15, 14
The value of lies between 3 and 13 and From equation R:

also between 6 and 15. z2 – 23z + 120 = 0


z2 – 15z – 8z + 120 = 0

18) Answer: B z(z – 15) – 8(z – 15) = 0

Marked price of item A = ₹ 600 z = 15, 8

New marked price of item A = 115% of 600 = So, the correct combinations are:

₹ 690 (P – M, P – O), (Q – M, Q – N), (R – N, R – O)

Selling price of item A = P


80% of (690 – 69) = P 20) Answer: C

80% of 621 = P From equation P:

P = 496.8 x2 – 14x – 95 = 0

Required discount percent when marked price x2 – 19x + 5x – 95 = 0

will be same as earlier and selling price will be x(x – 19) + 5(x – 19) = 0

₹ P= x = 19, -5

= 17.2% From equation Q:


y2 – 14y – 120 = 0

19) Answer: B y2 – 20y + 6y – 120 = 0

From equation P: y(y – 20) + 6(y – 20) = 0


y = 20, -6
From equation R:
x2 – 22x + 112 = 0
z2 + 12z + 35 = 0
x2 – 14x – 8x + 112 = 0
z2 + 7z + 5z + 35 = 0
x(x – 14) – 8(x – 14) = 0
z(z + 7) + 5(z + 7) = 0
x = 14, 8
z = -7, -5
From equation Q:
So, the correct combinations are:
(P – M, P – N), (Q – O), (R – M)
y2 – 29y + 210 = 0
y2 – 15y – 14y + 210 = 0

Click Here For Bundle PDF Course | support@guidely.in Page 13 of 13


SBI Clerk & RRB PO Mains PDF Course 2023
ENGLISH Day - 25

English Language

Directions (1-5): The following paragraph given clarity on the amount pension members and
below has five blanks in which words from pensioners will receive, should their applications
options given in each question would be filled in be accepted. Their _______(C) is understandable
order to complete the paragraph meaningfully. as they — particularly those still in service —
You must read the same carefully and choose give their consent to transfer a substantial portion
the correct answer for each question. of their PF savings to the Pension Fund. Though
The move by the Employees’ Provident Fund pensioners will have to make payments
Organisation (EPFO) to extend the deadline, for separately to be considered eligible for higher
a third time, for employees and pensioners to pension, they would also be keen to know how
exercise the joint option for higher pension is of much pension they would get. It is against this
limited relief, as several issues backdrop that the EPFO has sent demand letters
remain__________(A). As the entire process is an to about 1,000 pensioners and employees for
outcome of the Supreme Court judgement of collecting arrears. As the calculator on the
November 2022 on the validity of amendments EPFO’s website only indicates the amount to be
made in 2014 to the Employees’ Pension transferred, it is time that the EPFO gave an
Scheme (EPS), the matter ________(B) to two indicative figure of pension, at least to applicants
categories of applicants — those who retired who have been issued the letters. In the case of
from service prior to September 1, 2014, and the pre-2014 retirees, it appears that the PF
those who left service after the date and those authorities have not yet officially commenced
who are still in service. Nearly 16 lakh communication on the status of their
applications have been received so far. Yet applications, even though, under the rules, most
another extension cannot be ruled out. After all, it of these may not stand the test of________(D).
is too early to indicate that higher pension Employers have the _________(E)task of
disbursal will begin this financial year, even if producing physical records for every applicant.
applicants meet all the terms and conditions. The As not all establishments will have these records,
Union Labour and Employment Ministry, which the sensible option for the EPFO would be to
oversees the EPFO, told the standing share its database with employers for the limited
parliamentary committee concerned early this purpose of establishing applicant authenticity.
year that the implications of the Court’s verdict The situation is more complicated for
on the EPS had not been factored in, while establishments that are no longer in existence —
preparing the Budget estimates for 2023-24.The there appears to be no way out for their
most important issue of concern is the lack of employees and pensioners to apply for higher

Click Here For Bundle PDF Course | support@guidely.in Page 1 of 11


SBI Clerk & RRB PO Mains PDF Course 2023
ENGLISH Day - 25

pension. Given that the spirit of the judgement is 4) Which of the following words fits in ‘blank D’
to provide a better social security net, the Union given in the passage ?
Ministry and the EPFO should be proactive in a) attention
simplifying the process and ensuring that every b) study
deserving person gets the benefit. c) scrutiny
1) Which of the following words fits in ‘blank A’ d) inspect
given in the passage and the same word should e) None of these
fill in the blank given in the sentence below ?
The system issue still remains ________(A) and 5) Which of the following words fits in ‘blank E’
that’s why I was not able to complete my task on given in the passage and the antonym of the
time. word fills the blank in the given sentence below ?
a) addressed She does all the household chores in an ___way.
b) solved a) simple
c) unresolved b) herculean
d) fixed c) easiest
e) None of these d) effortless
e) None of these
2) Which of the following words fits in ‘blank B’
given in the passage ? Directions (6-10) : In each question a sentence is
a) concerning given followed by a blank. You are provided with
b) disconnects three fragments I, II and III. You have to identify
c) parts which statement/statements can carry forward
d) pertains the given sentence in the most logical way so as
e) None of these to make the sentence coherent and contextually
correct.
3) Which of the following words fits in ‘blank C’ 6) CNG typically costs less than gasoline or
given in the passage ? diesel and is often more economically viable for
a) anxiety fleet operators or individuals with CNG-powered
b) happiness vehicles, as _____.
c) joy I. CNG stations are present in every nook and
d) anxious corner.
e) None of these II. it can result in significant fuel cost savings
over time.
III. gasoline and diesel are dangerous.

Click Here For Bundle PDF Course | support@guidely.in Page 2 of 11


SBI Clerk & RRB PO Mains PDF Course 2023
ENGLISH Day - 25

a) Only I d) Only III


b) Only II e) Both I and III
c) Only III
d) Both I and II 9) In order to resolve all the client issues that
e) Both II and III followed the code production this week all the
testing and development leads_______.
7) Data science is a multidisciplinary field that I. agreed to work at night and extend their
combines statistical analysis, machine learning, support during the weekends too.
data visualisation, and domain II. came up with a solution to automate the
expertise____________and unstructured data. handling of frequently reported issues so that the
I. to combine structured team can concertare on new and difficult ones.
II. to store various data in the form of structured III. went on a two week long vacation to escape
III. to extract meaningful insights and knowledge from the workload.
from structured a) Only I
a) Only I b) Only II
b) Only II c) Both I and II
c) Only III d) Both II and III
d) Both I and II e) All I, II and III
e) Both I and III
10) ___________the visit was the MoU on India
8) ___________the three service chiefs who importing 10,000 MW of electricity from Nepal in
continue to operate in their respective domains, 10 years and exporting Nepali electricity to
but the role and function of the CDS is still in the Bangladesh via the Indian territory.
making.
I. Even Though they belong to three different I. Of the various agreements signed, the one that
forces there must be a coordination among was projected as the highlight of
II. The huge arms and ammunition of the country II. Many important decisions were made during
must be shared by the Nepal’s PM visit to India but the most
III. The Chief of Defence Staff, who functions as significant thing about
Principal Military Adviser to the Defence Minister, III. Nepal’s Prime Minister shared on his official
is expected to work in tandem with page the importance of
a) Only I a) Only I
b) Only II b) Only III
c) Both I and II c) Both I and II

Click Here For Bundle PDF Course | support@guidely.in Page 3 of 11


SBI Clerk & RRB PO Mains PDF Course 2023
ENGLISH Day - 25

d) Both I and III 11) Which of the following is the introductory


e) Both II and III sentence after the rearrangement ?
a) A
Directions (11-15) : Rearrange the following b) B
sentences in the proper sequence to form a c) C
meaningful paragraph and then answer the d) D
questions given below. Sentence E is the fifth e) F
sentence of the coherent paragraph and is fixed.
Rearrange the remaining sentences (A), (B), (C), 12) Fill in the blanks with correct alphabets in
(D) and (F). accordance with the rearranged sequence.
Note : E is the fifth sentence after the CF AB BD F_
rearrangement and is fixed. CD AE F_
(A) Red sanders is a native and endemic to a) A, E
India and can only be found in the Southern parts b) B, D
of the Eastern Ghats. c) C, F
(B) There is a huge demand for this timber in the d) D, A
domestic and international market, especially in e) E, B
East Asian Countries.
(C) The Andhra Pradesh forest department 13) Which of the following immediately follows
represented by Principal Chief Conservator of sentence ‘E’ in the coherent paragraph formed ?
Forest & Head of Forest Force have submitted a) A
an application seeking geographical indication b) B
(GI) tag for the famous Andhra Pradesh Red c) C
Sanders. d) D
(D) In addition, the timber is also exploited for the e) F
extraction of Santalin (a red pigment used as dye
and colourant in food), medicine and cosmetics. 14) Which of the following is the ODD ONE OUT.
(E) Red Sanders timber is used for carving a) CAB
furniture, poles, and house posts and the rare b) BEA
“wavy” grain variant is highly valued in Japan for c) FBD
its acoustic properties and is used to make d) AFE
musical instruments. e) EDA
(F) It is a small tree that grows to 5-8 metres in
height with a trunk 50-150 cm diameter.

Click Here For Bundle PDF Course | support@guidely.in Page 4 of 11


SBI Clerk & RRB PO Mains PDF Course 2023
ENGLISH Day - 25

15) Which of the following is an adjoining pair 18) The L-G of Jammu and Kashmir has
after the rearrangement ? informed the legal luminaries who attended the
a) AB meeting that the administration had scrapped
b) BE “obsolete land rules” in J&K.
c) CD a) bloggers, keep
d) DF b) influencers, dispose
e) EA c) infamous, preserve
d) unpopular, accept
Directions (16-20) : In each of the questions e) None of these
below a statement is given in which two words
have been highlighted. Below each question five 19) Despite desperate attempts by interested
options are given each containing a pair of parties to paint the expert committee’s report as
words. Choose the pair which contains the a ‘clean chit’, all the actions taken by SEBI
synonyms of the words respectively. indicate an admission of guilt and a belated
16) Nearly a decade after the Big Three’s hijack attempt to increase transparency regarding
bid, the ICC has proposed a similar surplus financial flows.
revenue-sharing model. a) satisfied, early
a) control, excess b) cheerful, premature
b) give, deficit c) hopeless, delayed
c) defence, equal d) composed, overdue
d) free, less e) None of these
e) None of these
20) A local court in Imphal on Friday took
17) Any non-compliance of the applicable cognizance of charges filed against two Kuki-
regulation shall be dealt with strictly and may Zomi leaders, accusing them of promoting enmity
also invite stringent enforcement action,” read between two groups.
the advisory issued by the DGCA. a) ignorance, love
a) disagree, lenient b) knowledgeable, hostility
b) denial, easy c) negligence, friendly
c) rebellion, flexible d) heedlessness, grudge
d) abide, rigorous e) None of these
e) None of these

Click Here For Bundle PDF Course | support@guidely.in Page 5 of 11


SBI Clerk & RRB PO Mains PDF Course 2023
ENGLISH Day - 25

Click Here to Get the Detailed Video Solution for the above given Questions
Or Scan the QR Code to Get the Detailed Video Solutions

Answer Key with Explanation

1) Answer: C Concerning can be eliminated for it being a -ing


The issues remain unresolved and thus the word which is not suitable to the sentence.
EPFO board has extended the deadline. Parts and disconnects have different meanings
Addressed, solved and fixed are the opposite in that vary from the context of the sentence and
meaning to the word what the blank needs. hence cannot fit the blank.
Resolve - to fix something The sentence:
Unresolve - to not fix something As the entire process is an outcome of the
The sentences : Supreme Court judgement of November 2022 on
The move by the Employees’ Provident Fund the validity of amendments made in 2014 to the
Organisation (EPFO) to extend the deadline, for Employees’ Pension Scheme (EPS), the matter
a third time, for employees and pensioners to pertains to two categories of applicants — those
exercise the joint option for higher pension is of who retired from service prior to September 1,
limited relief, as several issues remain 2014, and those who left service after the date
unresolved. and those who are still in service.
The system issue still remains unresolved and
that is why I was not able to complete my task 3) Answer: A
on time. Clearly there is confusion and lack of clarity
regarding the pension numbers - this is the
2) Answer: D context of the previous sentence to the sentence
Pertains is the correct word that fits in the given which holds the blank. And hence the following
blank ‘B’ and makes the sentence meaningful. sentence must also follow the same tone and
Pertains - applies to

Click Here For Bundle PDF Course | support@guidely.in Page 6 of 11


SBI Clerk & RRB PO Mains PDF Course 2023
ENGLISH Day - 25

context, from the given options only ‘anxiety’ is Herculean - task which needs great efforts and
the right word that can fit the blank properly. strength.
Anxiety - worry/fear The sentences :
Joy and happiness are opposite in meaning to Employers have the herculean task of producing
anxiety physical records for every applicant.
The sentence : She does all the household chores in an
Their anxiety is understandable as they — effortless way.
particularly those still in service — give their 6) Answer: B
consent to transfer a substantial portion of their The context here is the cost of gasoline, diesel
PF savings to the Pension Fund. and CNG, CNG is more affordable than the other
two. So, the right fragment to complete the
4) Answer: C sentence would be II.
Among the given words ‘Scrutiny’ is the correct I - talks about CNG station which is inappropriate
word. to the context and similarly II talks about the
Scrutiny - watching something closely/examining effects of usage which also seems inappropriate.
Inspect, attention are words with similar meaning The sentence:
but scrutiny is a better word that fits the blank. CNG typically costs less than gasoline or diesel
Study is inappropriate to the context. and is often more economically viable for fleet
The authorities haven’t communicated to the operators or individuals with CNG-powered
2014 retirees however this has not been a part vehicles, as it can result in significant fuel cost
of the test of scrutiny. savings over time.
The sentence: In the case of the pre-2014
retirees, it appears that the PF authorities have 7) Answer: C
not yet officially commenced communication on Statement III is the correct pick to complete the
the status of their applications, even though, sentence in a meaningful way.
under the rules, most of these may not stand the To combine structured and unstructured data -
test of scrutiny. data when organised in different forms is always
easy to use and there is no need to combine the
5) Answer: B data back and make it difficult
Producing physical record of every applicants is Data science does not deal with storage so
really hard and tiring so the right word would be statement II is also wrong.
‘herculean’ The sentence:

Click Here For Bundle PDF Course | support@guidely.in Page 7 of 11


SBI Clerk & RRB PO Mains PDF Course 2023
ENGLISH Day - 25

Data science is a multidisciplinary field that The sentences :


combines statistical analysis, machine learning, In order to resolve all the client issues that
data visualisation, and domain expertise to followed the code production this week all the
extract meaningful insights and knowledge from testing and development leads agreed to work at
structured and unstructured data. night and extend their support during the
weekends too.
8) Answer: D In order to resolve all the client issues that
followed the code production this week all the
III is the correct statement that can be used to testing and development leads came up with a
complete the given sentence. solution to automate the handling of frequently
The second half of the sentence talks about a reported issues so that the team can concertare
single officer(the CDS), so I and II can be on new and difficult ones.
eliminated because they talk about arms,
ammunition and coordination and about a 10) Answer: C
person/officer. Both the statements I and II are correct and fit
Only III describes the role of CDS-Chief Defence the blank in a perfect way adding meaning to the
Officer and hence it is the correct fragment that sentence.
fits the given blank. III is inappropriate and incomplete making the
The sentence : sentence incorrect.
The Chief of Defence Staff, who functions as The sentences:
Principal Military Adviser to the Defence Of the various agreements signed, the one that
Minister, is expected to work in tandem with the was projected as the highlight of the visit was the
three service chiefs who continue to operate in MoU on India importing 10,000 MW of electricity
their respective domains, but the role and from Nepal in 10 years and exporting Nepali
function of the CDS is still in the making. electricity to Bangladesh via the Indian territory.
Many important decisions were made during the
9) Answer: C Nepal’s PM visit to India but the most significant
Both the statements I and II fit the blank to make thing about the visit was the MoU on India
the sentence correct and meaningful. importing 10,000 MW of electricity from Nepal in
Statement III is inappropriate to the context of 10 years and exporting Nepali electricity to
the sentence because to solve the production Bangladesh via the Indian territory.
issues the employees usually work and do not
go on a vacation. 11) Answer: C

Click Here For Bundle PDF Course | support@guidely.in Page 8 of 11


SBI Clerk & RRB PO Mains PDF Course 2023
ENGLISH Day - 25

C is the first/introductory sentence after the sanders’. Now what is red sanders ? and the
rearrangement. answer is sentence ‘A’ and it is obvious that it
The rearranged sequence is CAFBED. will be the second sentence in the
Sentence ‘C’ is the perfect introduction to this rearrangement. Sentence ‘F’ tells more about
passage as it contains the information ‘AP the specifications of the red sanders, so place it
government has applied for a GI tag for its Red as the third sentence. Now, look at the fixed
sanders’. Now what is red sanders ? and the sentence ‘E’ which talks about the
answer is sentence ‘A’ and it is obvious that it usage/applications of red sanders and ‘D’ has
will be the second sentence in the the phrase ‘in addition’ and continues to discuss
rearrangement. Sentence ‘F’ tells more about red sander’s usage. So, ED is a pair. The left out
the specifications of the red sanders, so place it sentence is ‘B’ which sits right after CAF and
as the third sentence. Now, look at the fixed before ED. The final arrangement is CAFBED.
sentence ‘E’ which talks about the
usage/applications of red sanders and ‘D’ has 13) Answer: D
the phrase ‘in addition’ and continues to discuss ED is a pair, they revolve around the same
red sander’s usage. So, ED is a pair. The left out context.
sentence is ‘B’ which sits right after CAF and The rearranged sequence is CAFBED.
before ED. The final arrangement is CAFBED. Sentence ‘C’ is the perfect introduction to this
passage as it contains the information ‘AP
12) Answer: E government has applied for a GI tag for its Red
CF AB BD - F is two places away from A, B is sanders’. Now what is red sanders ? and the
two places away from A similarly D is two places answer is sentence ‘A’ and it is obvious that it
away from B. will be the second sentence in the
According to the pattern followed, E is two rearrangement. Sentence ‘F’ tells more about
places away from F. So the first blank should be the specifications of the red sanders, so place it
filled with ‘E’ as the third sentence. Now, look at the fixed
Second - CD first and last, AE- second and fifth, sentence ‘E’ which talks about the
FB- third and fourth usage/applications of red sanders and ‘D’ has
‘B’ fills the second blank. the phrase ‘in addition’ and continues to discuss
The rearranged sequence is CAFBED. red sander’s usage. So, ED is a pair. The left out
Sentence ‘C’ is the perfect introduction to this sentence is ‘B’ which sits right after CAF and
passage as it contains the information ‘AP before ED. The final arrangement is CAFBED.
government has applied for a GI tag for its Red

Click Here For Bundle PDF Course | support@guidely.in Page 9 of 11


SBI Clerk & RRB PO Mains PDF Course 2023
ENGLISH Day - 25

14) Answer: B rearrangement. Sentence ‘F’ tells more about


CA is a pair in the arrangement and B is two the specifications of the red sanders, so place it
places away from A. FB - pair and D is two as the third sentence. Now, look at the fixed
places away from B. A Similar pattern is followed sentence ‘E’ which talks about the
in AFE and EDA. usage/applications of red sanders and ‘D’ has
But in BEA the pattern is missed - rather BEC the phrase ‘in addition’ and continues to discuss
would be correct. red sander’s usage. So, ED is a pair. The left out
The rearranged sequence is CAFBED. sentence is ‘B’ which sits right after CAF and
Sentence ‘C’ is the perfect introduction to this before ED. The final arrangement is CAFBED.
passage as it contains the information ‘AP
government has applied for a GI tag for its Red 16) Answer: A
sanders’. Now what is red sanders ? and the The words in option a are the correct synonyms
answer is sentence ‘A’ and it is obvious that it of the words hijack and surplus respectively.
will be the second sentence in the Hijack - to take control of a plane
rearrangement. Sentence ‘F’ tells more about Surplus - something that is in excess amount
the specifications of the red sanders, so place it Control and excess is the right pair of synonyms
as the third sentence. Now, look at the fixed Other given pairs are either antonyms or words
sentence ‘E’ which talks about the with different meanings.
usage/applications of red sanders and ‘D’ has
the phrase ‘in addition’ and continues to discuss 17) Answer: D
red sander’s usage. So, ED is a pair. The left out The words in option d are the correct synonyms
sentence is ‘B’ which sits right after CAF and of the words compliance and stringent
before ED. The final arrangement is CAFBED. respectively.
Compliance - in accordance with rules and
15) Answer: B regulation/to follow laws or rules
BE is a pair in the rearranged sequence. Stringent - very strict
The rearranged sequence is CAFBED. Abide and rigorous is the right pair of synonyms
Sentence ‘C’ is the perfect introduction to this for the words compliance and stringent.
passage as it contains the information ‘AP The other given options are not the synonyms or
government has applied for a GI tag for its Red meanings of the words highlighted.
sanders’. Now what is red sanders ? and the Rebellion - fighting
answer is sentence ‘A’ and it is obvious that it Lenient - not strict
will be the second sentence in the

Click Here For Bundle PDF Course | support@guidely.in Page 10 of 11


SBI Clerk & RRB PO Mains PDF Course 2023
ENGLISH Day - 25

18) Answer: B Our requirement is to choose synonyms for both


The words in option b are the correct synonyms the highlighted words.
of the words luminaries and scrapped Composed - calm
respectively. Overdue - late in arriving
Luminary - a person who inspires or influence
others 20) Answer: B
Scrap - to get rid of something The words in option b are the correct synonyms
Influencers and dispose are the correct of the words cognisance and enmity
synonyms and other options given either have respectively.
different meaning or opposite to the given words. Cognizance - detailed knowledge about
something
19) Answer: C Enmity - hatred
The words in option c are the correct synonyms The other options either contain words which are
of the words desperate and belated respectively. antonyms or pairs of ‘synonym and antonym’.
Desperate - out of control, no hope Our requirement is to choose synonyms for both
Belated - late the highlighted words.
Hopeless and delayed is the best pair of words Hostility - strong feeling against
that can be picked. something/somebody
The other options either contain words which are Grudge - unfriendly
antonyms or pairs of ‘synonym and antonym’. Heedlessness - not considering something

Click Here For Bundle PDF Course | support@guidely.in Page 11 of 11


SBI Clerk & RRB PO Mains PDF Course 2023
Reasoning Ability Day - 26 (Eng)

Reasoning Ability
Directions (1-5): Study the following information 1. Who among the following person lives on the
carefully and answer the given questions. sixth floor?
Nine people – A, C, F, H, N, O, Q, T and W are a) The one whose floor height is 30ft
living on different floors of nine-storey building b) T
where the lowermost floor is numbered one and c) The one who lives immediately below Q
the floor immediately above it is numbered two d) N
and so on. The height of each floor is different. e) The one whose floor height is 48ft
Note I: The persons whose name starts with the
letter which comes before M as per the 2. What is the sum of the height of the floors of
alphabetical series are living on an even W, Q and O?
numbered floor and the persons whose name a) 89ft
starts with the letter which comes after M as per b) 97ft
the alphabetical series are living on an odd c) 131ft
numbered floor. d) 93ft
Note II: The height of each floor is the multiple of e) 124ft
their respective floor number and the height of
the floor is not less than 10 ft but not more than 3. Who among the following persons lives below
55 ft. H?
Only two persons live between A and the one I. O
whose floor height is 30ft. O live three floors II. N
above A. The height of O’s floor is 6ft less than III. W
the height of T’s floor. The height of the T’s floor IV. Q
is less than 25ft. The number of floors above T a) Only I, II and IV
is two more than the number of floors below N. b) Only III and IV
Only three persons live between C and the one c) Only II and III
whose floor height is 52ft. The second floor’s d) Only II
height is not 52 ft. The sum of the height of the e) All I, II, III and IV
floors of C and N is 53ft. The height of Q’s floor is
7ft less than the height of the fourth floor. The 4. Which of the following statement(s) is/are true
difference between the height of the floors of F as per the given arrangement?
and W is 24ft. More than one person lives a) The height of the fourth floor is 36ft
between H and W. The height of H’s floor is 6ft b) The height of the ninth floor is 45ft
less than the height of F’s floor. c) N lives on the seventh floor

Click Here For Bundle PDF Course | support@guidely.in Page 1 of 11


SBI Clerk & RRB PO Mains PDF Course 2023
Reasoning Ability Day - 26 (Eng)

d) C neither lives on the eighth floor nor lives on between Saru and Dora. Less than four persons
the second floor check the mail between Saru and Rohit. Only
e) All are true one person checks the mail between Rithvik and
Karthick. Joshi doesn’t check the mail on
5. If C and A interchange their positions, then in Sunday.
the same way C and H interchange their 6. Who among the following person checks the
positions, then who among the following person mail on second week?
lives between C and H and what is the height of a) Saru
that floor? b) Gabriel
a) W, 30ft c) George
b) H, 54ft d) Both b and c
c) C, 35ft e) Both a and b
d) T, 20ft
e) Both b and c 7. Joshi checks the mail on which of the following
day?
Directions (6-10): Study the following information a) Tuesday
carefully and answer the given questions. b) Thursday
Nine persons –Joshi, Saru, Gabriel, Dora, c) Monday
Karthick, Rohit, Carly, Rithvik and George check d) Friday
the mail on nine consecutive days of two weeks e) Cannot be determined
starting from Monday.
Note: 8. The person who checks the mail on __ checks
I. The person whose name has an even number the mail three days before__.
of letters must check the mail in the day which a) Saturday, Saru
has an odd number of vowel letters. b) Tuesday, Rithvik
II. The person whose name has an odd number c) Sunday, Joshi
of letters must check the mail in the day which d) Wednesday, Gabriel
has an even number of vowel letters. e) Saturday, George
Gabriel checks the mail on one of the days after
Thursday. Only two persons check the mail 9. What is the ratio of the number of persons
between Gabriel and Rohit. George checks the check the mail before Rithvik to the number of
mail two days after Carly. The number of days persons check the mail after Saru respectively?
before George is two more than the number of a) 1:2
days after Saru. Only two persons check the mail b) 1:1

Click Here For Bundle PDF Course | support@guidely.in Page 2 of 11


SBI Clerk & RRB PO Mains PDF Course 2023
Reasoning Ability Day - 26 (Eng)

c) 2:1 players won the award between Z and V. V did


d) 1:3 not win the award in August.
e) 3:1 Arrangement 2:
Each player belongs to different states viz.,
10. If the persons who check the mail from Karnataka, Tamil Nadu and Punjab. At least two
Wednesday to Sunday received the number of players but not more than three players belong to
mails, which is a consecutive multiple of 9 the same state.
(starting from 9), then what is the average The one who won the award in July in
number of the mails received by Rithvik and arrangement 1 belongs to Tamil Nadu. V does
Dora? not belong to Karnataka. Only S and the one who
a) 32.5 won the award four months before S in
b) 27 arrangement 1 belong to the same state.T and U
c) 36 belong to the same state. The players who won
d) 22.5 the award immediately before and immediately
e) Can’t be determined after X in arrangement 1 do not belong to the
same state as X.
Directions (11-15): Study the following Arrangement 3:
information carefully and answer the given All the players live on four different floors of a
questions. four storey hotel where the lowermost floor is
Arrangement 1: numbered one and the floor immediately above it
Eight Indian cricket players – S, T, U, V, W, X, Y is numbered two and so on. Each floor has two
and Z won the Player of the Month award in flats viz., Flat-A and Flat-B, where Flat A is to the
different months of the same year viz., January, west of Flat B. Only one player lives in each flat
March, April, July, August, September, October and only two players live on each floor.
and December. Only two floors are between W and the one who
W won the player of the month award in the belongs to the same state as Z, where both are
month having only 30 days. S won the award living in different type of flats. W lives to the east
three months after W. Only three players won the of the one who won the award immediately after
awards between S and Y. As many players won X in arrangement 1. T lives immediately above
the award before Y as after Z. At least two the flat of Y. Only one floor is between Y and the
players won the award between U and T. U won one who won the award three months before U in
the award in the month having an even number arrangement 1, where both are living in different
of days. The number of players won the award type of flats. X and V are not living in the same
between T and X is one more than the number of type of flat.

Click Here For Bundle PDF Course | support@guidely.in Page 3 of 11


SBI Clerk & RRB PO Mains PDF Course 2023
Reasoning Ability Day - 26 (Eng)

11. Who among the following player lives b) Y


immediately below the flat of X? c) Z
a) Y d) W
b) V e) X
c) S
d) T 15. Who among the following player belongs to
e) Z the same state as T as per arrangement 2?
a) S
12. As per the arrangement 2, who among the b) Y
following player belong to Tamil Nadu? c) V
I. V d) U
II. W e) Both c and d
III. X
a) Only III Directions (16-20): Study the following
b) Only I and II information carefully and answer the given
c) Only I questions.
d) Only I and III Eighteen persons from A to R are sitting in three
e) Only II and III parallel rows. Six persons sit in each row in such
a way that there is an equal distance between
13. How many players won the award between U the adjacent persons. Row 1 is in north of row 2
and Z as per arrangement 1? while row 2 is in north of row 3. The persons in
a) No one row 1 are facing south and the persons in row 3
b) As many players won the award between V are facing north. In row 2, the first three persons
and Y from the left end are facing north while the
c) Three remaining persons are facing south.
d) As many players won the award between W Note: If X is facing Y, then X and Y must be on
and S the adjacent rows and are facing each other.
e) Four K and E are sitting immediate right of each other.
C faces H and sits immediate left of K. P sits
14. If V is related to Z and T is related to Y in a third to the left of H. I sits second to the right of J
certain way in arrangement 3, then who among and faces D. Only three persons sit between D
the following players is related to S in and F. O sits immediate right of N and faces F. L
arrangement 3? sits immediate right of R. Neither A nor R sits at
a) U the extreme end. Only one person sits between

Click Here For Bundle PDF Course | support@guidely.in Page 4 of 11


SBI Clerk & RRB PO Mains PDF Course 2023
Reasoning Ability Day - 26 (Eng)

G and A. The number of persons sitting between a) Second to the left


J and G is two less than the number of persons b) Third to the right
sitting between B and M. Q, who faces M, is c) Third to the left
facing south direction. d) Immediate right
16. Who among the following person sits exactly e) Second to the right
between Q and the one who faces H?
a) A 19.Four of the following five are alike in a certain
b) The one who sits second to the right of Q way based on the given arrangement and thus
c) K form a group. Which one of the following does
d) The one who faces A not belong to the group?
e) Both b and d a) G
b) Q
17. Who among the following pair of persons are c) I
facing each other? d) E
I. PF e) C
II. LK
III. NE 20. In which of the following option, first person
a) Only II sits exactly between the second and the third
b) Only II and III person?
c) Only I and III a) KDQ
d) Only III b) RBO
e) All I, II and III c) JHP
d) EQC
18. What is the position of B with respect to N as e) NOM
per the given arrangement?

Click Here For Bundle PDF Course | support@guidely.in Page 5 of 11


SBI Clerk & RRB PO Mains PDF Course 2023
Reasoning Ability Day - 26 (Eng)

Click Here to Get the Detailed Video Solution for the above given Questions
Or Scan the QR Code to Get the Detailed Video Solutions

Answer Key with Explanation

Directions (1-5): From the above conditions, there are three


1) Answer: E possibilities
2) Answer: A
3) Answer: C
4) Answer: B
5) Answer: D
Final arrangement

Again, we have
 The height of O’s floor is 6ft less than the
height of T’s floor.
 The height of the T’s floor is less than
25ft.
 The number of floors above T is two more
than the number of floors below N.
 Only three persons live between C and
We have, the one whose floor height is 52ft.
 Only two persons live between A and the  The second floor’s height is not 52 ft.
one whose floor height is 30ft.  The sum of the height of the floors of C
 O lives three floors above A. and N is 53ft.

Click Here For Bundle PDF Course | support@guidely.in Page 6 of 11


SBI Clerk & RRB PO Mains PDF Course 2023
Reasoning Ability Day - 26 (Eng)

After applying the above conditions case-1 gets 6) Answer: D


eliminated because the height of the floor on 7) Answer: C
which T lives is more than 25ft. 8) Answer: E
9) Answer: A
10) Answer: B(18+36=54/2=27)
Final arrangement

Again, we have
 The height of Q’s floor is 7ft less than the
height of the fourth floor.
 The difference between the height of the
floors of F and W is 24ft.
 More than one person lives between H We have,

and W.  Gabriel checks the mail on one of the

 The height of floor H’s floor is 6ft less than days after Thursday.

the height of F’s floor.  Only two persons check the mail between

After applying the above conditions case-3 gets Gabriel and Rohit.

eliminated because no person lives between H From the above conditions, there are three

and W, hence case-2 shows the final possibilities

arrangement.

Again, we have
 George checks the mail two days after
Carly.
Directions (6-10):

Click Here For Bundle PDF Course | support@guidely.in Page 7 of 11


SBI Clerk & RRB PO Mains PDF Course 2023
Reasoning Ability Day - 26 (Eng)

 The number of days before George is two Directions (11-15):


more than the number of days after Saru. 11. Answer: D
 Only two persons check the mail between 12. Answer: E
Saru and Dora. 13. Answer: D
 Less than four persons check the mail 14. Answer: C
between Saru and Rohit. 15. Answer: E
After applying the above conditions one Final arrangement:
possibility is added and then case-2 gets Arrangement 1:
eliminated because less than four persons
should check the mail between Saru and Rohit.

Again, we have
 Only one person checks the mail between Arrangement 2:
Rithvik and Karthick.
 Joshi doesn’t check the mail on Sunday.
After applying the above conditions case-1 and
case-3 get eliminated because Joshi check the Arrangement 3:

mail on Sunday, hence case-3a shows the final


arrangement.

Arrangement 1:
We have,
 W won the player of the month award in
the month having only 30 days.

Click Here For Bundle PDF Course | support@guidely.in Page 8 of 11


SBI Clerk & RRB PO Mains PDF Course 2023
Reasoning Ability Day - 26 (Eng)

 S won the award three months after W.


 Only three players won the awards
between S and Y.
 As many players won the award before Y
as after Z.
After applying the above conditions, there are
two possibilities.

Arrangement 2:
We have,
 The one who won the award in July in
arrangement 1 belongs to Tamil Nadu.
 V does not belong to Karnataka.
 Only S and the one who won the award
four months before S in arrangement 1
Again, we have belong to the same state.
 At least two players won the award After applying the above conditions, there are
between U and T. three possibilities.
 U won the award in the month having
an even number of days.
 The number of players won the award
between T and X is one more than the
number of players won the award between
Z and V.
Again, we have
 V did not win the award in August.
 T and U belong to the same state.
After applying the above conditions, Case 1 gets
 The players who won the award
eliminated, because V won the award in the
immediately before and immediately after
month of August, hence case 2 shows the final
X in arrangement 1 do not belong to the
arrangement.
same state as X.
After applying the above conditions, case 2 and
3 get eliminated because not more than three
players belong to the same state. Hence case 1
shows the final arrangement.

Click Here For Bundle PDF Course | support@guidely.in Page 9 of 11


SBI Clerk & RRB PO Mains PDF Course 2023
Reasoning Ability Day - 26 (Eng)

Arrangement 3:
We have,
 Only two floors are between W and the
one who belongs to the same state as Z, Directions (16-20):
where both are living in different type of 16. Answer: B
flats. 17. Answer: D
 W lives to the east of the one who won 18. Answer: C
the award immediately after X in 19. Answer: E(In the given option, all the
arrangement 1. persons are facing south, except option e)
After applying the above conditions, there are 20. Answer: D
two possibilities. Final arrangement:

Again, we have
 T lives immediately above the flat of Y.
 Only one floor is between Y and the one
We have,
who won the award three months before
 K and E are sitting immediate right of
U in arrangement 1, where both are living
each other.
in different type of flats.
 C faces H and sits immediate left of K.
 X and V are not living in the same type of
 P sits third to the left of H.
flat.
After applying the above conditions, there are
After applying the above conditions, case 2 gets
two possibilities.
eliminated because there is no possibility to
place X. Hence case 1 shows the final
arrangement.

Click Here For Bundle PDF Course | support@guidely.in Page 10 of 11


SBI Clerk & RRB PO Mains PDF Course 2023
Reasoning Ability Day - 26 (Eng)

Again, we have
 Only one person sits between G and A.
 The number of persons sitting between J
and G is two less than the number of
persons sitting between B and M.
 Q, who faces M, is facing south direction.
Again, we have
After applying the above conditions, case 2 gets
 I sit second to the right of J and faces D.
eliminated because Q should face south, hence
 Only three persons sit between D and F.
case 1 shows the final arrangement.
 O sits immediate right of N and faces F.
 L sits immediate right of R.
 Neither A nor R sits at the extreme end.

Click Here For Bundle PDF Course | support@guidely.in Page 11 of 11


SBI Clerk & RRB PO Mains PDF Course 2023
Quantitative Aptitude Day - 26 (Eng)

Quantitative Aptitude

Direction (1 – 5): Study the given information carefully and answer the given questions.
There are five students – Rajat, Anshul, Kanak, Sachin and Bhanu. All are attending examination X,which
consist of two papers (Paper 1 and Paper 2). Paper 1 for 100 marks and Paper 2 for 200 marks. Both
papers have different weightage in the final score. Paper 1 has 40% weightage and Paper 2 has 60%
weightage in the total weightage mark. The table given below shows the marks and weightage obtained
by each student in two papers and the total weightage Mark.

Note:
I. Total weightage mark= Weightage of obtained marks in Paper 1 + Weightage of obtained marks in
Paper 2
1. Naina’s final score is 20% more than the final b. 18.24%
score of Kanak. Naina obtained 180 marks on c. 10.66%
Paper 2. Find her obtained marks in Paper 1. d. 22.36%
a. 75 e. None of these
b. 80
c. 60 3. For the final score of the student, govt of India
d. 90 (GoI) announces Rs 500 for each percent above
e. None of these 60% to below 71%, Rs. 1000 for each percent
above 70% to below 76%, and Rs 2000 for each
2. If Anshul obtained 20 more marks in Paper 1 percent above 75%. Find the amount Sachin got
and the marks obtained by Anshul in Paper 2 from GoI.
remain the same. Find the percentage increase a. Rs.15000
in the final score of Anshul. b. Rs.12000
a. 12.36% c. Rs.18000

Click Here For Bundle PDF Course | support@guidely.in Page 1 of 14


SBI Clerk & RRB PO Mains PDF Course 2023
Quantitative Aptitude Day - 26 (Eng)

d. Rs.21000 e. None of these


e. None of these
5. The sum of the total weightage marks of
4. If the marks obtained by Kanak in Paper 2are Sachin and Bhanu together is what percent more
increased by 20% and the total weightage mark than the sum of the total weightage marks of
of Kanak is 78, then find the percentage change Anshul and Kanak together.
in the marks obtained by Kanak in Paper 1. a. 15%
a. 30% b. 6%
b. 15% c. 12%
c. 25% d. 9%
d. 20% e. None of these

Direction (6 – 10): Study the given information carefully and answer the given questions.
There are five shopkeepers – A, B, C, D, and E who sells cold drink of three types – Pepsi, Thumbs Up,
and Coca-Cola. The bar graph given below shows the sum of Pepsi bottles and Thumbs Up bottles sold
by every shopkeeper. Also shows the difference between Thumbs Up bottles sold and Coca-Cola bottles
sold by each shopkeeper. The table given below shows the ratio of the Pepsi bottles sold to Coca-Cola
bottles sold by each shopkeeper. The number of Thumbs Up bottles sold by each shopkeeper is always
greater than the number of Coca-Cola bottles sold by each shopkeeper.

Click Here For Bundle PDF Course | support@guidely.in Page 2 of 14


SBI Clerk & RRB PO Mains PDF Course 2023
Quantitative Aptitude Day - 26 (Eng)

6. If the sum of the total number of Coca-Cola container such that the ratio of Pepsi, Thumbs
bottles sold by A and C together is increased by Up, and Soda in the container is 12 : 9 : 13
d% then it will become 5 less than the sum of the respectively. Find the quantity of mixture in
total number of Thumbs Up bottles sold by D and vessel B is what percent of the quantity of
E together. Find the value of d. mixture in vessel A.
a. 12 a. 70%
b. 22 b. 50%
c. 15 c. 60%
d. 18 d. 75%
e. None of these e. None of these

7. There are 3 different quantity of Pepsi bottles 9. Shopkeeper B sold 52% of Pepsi, 58% of
sold by B such as 600 ml, 1 liter, and 2 liter and Thumbs Up, and 65% of Coca-Cola bottles he
its ratio is 5 : 13 : 8 respectively. Find the total has. Find the total number of all cold drink bottles
quantity of Pepsi sold by B in liters? together that B has.
a. 520 liter a. 4300
b. 480 liter b. 3700
c. 560 liter c. 6100
d. 640 liter d. 3500
e. None of these e. None of these

8. Tanu makes a mixture of Pepsi and Soda in 10. If all the bottles of cold drinks sold by E are 1
vessel A in the ratio 3 : 2 and another mixture of liter in quantity. The selling price of a 1-liter bottle
Thumbs Up and Soda in the ratio 9 : 5 in vessel of Pepsi ,Thumbs Up and Coca-Cola is Rs 55,
B. She mixes the mixture of both the vessel in a

Click Here For Bundle PDF Course | support@guidely.in Page 3 of 14


SBI Clerk & RRB PO Mains PDF Course 2023
Quantitative Aptitude Day - 26 (Eng)

Rs 62 and Rs 60 respectively, Find the average c. Rs. 60.12


selling price of 1 bottle of cold drink sold by E. d. Rs. 54.68
a. Rs. 58.22 e. None of these
b. Rs. 64.12

Direction (11 – 15): Study the given information carefully and answer the given questions.
There are five villages- A, B, C, D, and E in which elections are held. The line graph given below shows
the difference between the winning candidate and runner-up candidate of elections in each village.

11. In village C, 80% of the population cast their of voters in village D, if 15% of casted votes are
votes, and 15% of the casted votes were invalid, If all the people in that village are eligible
declared invalid. The winning candidate gets and all are casted their votes?
48% of the total casted votes. Find the a. 18300
population of village C. b. 17200
a. 32567 c. 15400
b. 33400 d. 16800
c. 36700 e. None of these
d. 39500
e. None of these 13. The winning candidate of village B gets
15900 more votes than the winning candidate of
12.In village D, the winning candidate gets 52% village E. The ratio of the number of votes got by
of total casted votes. Then Find the total number the winning candidate of village E to the runner-

Click Here For Bundle PDF Course | support@guidely.in Page 4 of 14


SBI Clerk & RRB PO Mains PDF Course 2023
Quantitative Aptitude Day - 26 (Eng)

up candidate of village B is 2:3. Find the number d. 2731


of votes got by the runner-up candidate of village e. None of these
E.
a. 20142 Direction (16 – 18): Study the given information
b. 24012 carefully and answer the given questions.
c. 28412 In a township there are four blocks – A, B, C, and
d. 26114 D. The dimension of all room/house in a block is
e. None of these same but different from other blocks.
The length of the rooms of block A is equal to the
14. In village F, there are three candidates – P, height of rooms of block C which is 25% more
Q, and R in the election. The ratio of votes P, Q, than the height of the rooms of block D. Ratio of
and R get in the election is 6 : 4 : b. Q gets 1892 the length of the rooms of blocks B, C, and D is 5
votes more than R. Find the value of b. : 6 :4. Height of rooms of block A is 50% more
(Note: P is the winner of the election in village F than the breadth of rooms of block A. Breadth of
and Q is the runner-up of the election in village rooms of block B is 3 meters more than the
F.) breadth of rooms of block A and 3 meters less
a. 8 than the breadth of rooms of block C. Height of
b. 4 rooms of block B is 9 meter and is same as the
c. 7 breadth of rooms of block D and is 3/4th to the
d. 3 height of the rooms of block A. Length of rooms
e. None of these of block D is equal to the height of the rooms of
block A. Length of the rooms of block B is 50%
15. In village A, 70% of casted voters are literate more than the length of the rooms of block A.
and the rest are illiterate. The winning candidate 16. If the height of the rooms of block C is
gets 60% votes out of the total votes casted, of increased by d% to fit a solid spherical ball of
which 65% votes are literate voters. Find the volume 4312/3 m3. Find the new volume of 15
number of illiterate voters who cast their votes to rooms of block C.
runner-up candidate. a. 58462 m3
(Note: There are only two candidates in the b. 52920 m3
election in village A and all the eligible voters c. 51290 m3
cast their vote and all votes are valid.) d. 56460 m3
a. 2557 e. None of these
b. 2134
c. 2331

Click Here For Bundle PDF Course | support@guidely.in Page 5 of 14


SBI Clerk & RRB PO Mains PDF Course 2023
Quantitative Aptitude Day - 26 (Eng)

17. A painter charges Rs. 12/m2 for painting a oil of shopkeeper A is Rs. 60 more than the
house. Find the total amount painter gets for selling price of the hair oil of shopkeeper B. Find
painting the inner walls and ceiling of 42 rooms the selling price of the hair oil at which
of the houses of block A. shopkeeper A sold it.
a. Rs.258048 a. Rs. 212
b. Rs.248468 b. Rs. 204
c. Rs.254064 c. Rs. 180
d. Rs.263348 d. Rs. 192
e. None of these e. None of these

18. There are small wooden cubes of side 0.5 m 20. Station P is 900 km away from Station Q.
which are fitted in the 13 rooms of block B such Train A with a speed of M km/h and Train B with
that all 13 rooms are fully packed. Find the total a speed of N km/h start at the same time from
number of small wooden cubes which are fitted Station P and Station Q respectively move
in 13 rooms of block B. towards each other. After 4 hours from their
a. 136440 starting journey when they are 252 km away
b. 145440 from each other, a technical glitch occurs in the
c. 145880 engine of Train A by which its speed becomes
d. 154440 4/5th of its original speed. After 1 more hour, both
e. 158480 trains are 18 km away from each other. Find the
speed of Train B.
19. The cost price of hair oil is Rs. a. a. 60 km/h
Shopkeeper A marked it 50% above the cost b. 80 km/h
price and sell it after giving a discount of 15% on c. 72 km/h
the marked price. Shopkeeper B sold the article d. 90 km/h
at a 12.5% profit. If the marked price of the hair e. None of these

Click Here For Bundle PDF Course | support@guidely.in Page 6 of 14


SBI Clerk & RRB PO Mains PDF Course 2023
Quantitative Aptitude Day - 26 (Eng)

Click Here to Get the Detailed Video Solution for the above given Questions
Or Scan the QR Code to Get the Detailed Video Solutions

Answer Key with Explanation

Direction (1 – 5): Marks obtained in Paper 1 by Marks obtained in Paper 1 by Kanak = 75


Rajat = 70 Weightage of obtained marks in Paper 1 by
Weightage of obtained marks in Paper 1 by Kanak = 75/100 *40
Rajat = 70/100 * 40 = 30
= 28 Marks obtained in Paper 2 by Kanak = 150
Weightage of obtained marks in Paper 2 by Weightage of marks obtained in Paper 2 by
Rajat = 48 Kanak = 150/200*60 = 45
Marks obtained in Paper 2 by Rajat = 48/60 * Final score of Kanak = 30 + 45
200 = 75
= 160 Weightage of marks obtained in Paper 2 by
Final score of Rajat = 28 + 48 = 76 Sachin = 55.5
Weightage of obtained marks in Paper 1 by Marks obtained in Paper 2 by Sachin = 55.5/60
Anshul = 24 *200 = 185
Marks obtained in Paper 1 by Anshul = 24/40 Weightage of marks obtained in Paper 1 by
*100 Sachin = 77.5 – 55.5
= 60 = 22
Weightage of marks obtained in Paper 2 by Marks obtained in Paper 1 by Sachin =
Anshul = 75 – 24= 51 22/40*100
Marks obtained in Paper 2 by Anshul = 51/60 = 55
*200 Final score of Sachin = 22 + 55.5 = 77.5
= 170 Weightage of marks obtained in Paper 1 by
The final score of Anshul = 75 Bhanu = 32

Click Here For Bundle PDF Course | support@guidely.in Page 7 of 14


SBI Clerk & RRB PO Mains PDF Course 2023
Quantitative Aptitude Day - 26 (Eng)

Marks obtained in Paper 1 by Bhanu = Percentage increase in final score = ((83 –


32/40*100 = 80 75)/75) * 100
Marks obtained in Paper 2 by Bhanu = 165 = (8/75) * 100
Weightage of marks obtained in Paper 2 by = 10.66%
Bhanu = 165/200*60
= 49.5 3. Answer: A
Final score of Bhanu = 32 + 49.5 = 81.5 We know that sachin’s Total weightage Mark is
77.5
So,
Amount get by Sachin for 61% to 70 % = 500 *
10
= 5000
Amount get by Sachin for 71% to 75% = 1000 *
5
1. Answer: D = 5000
Naina’s final score = 75 *120/100 Amount got by Sachin for 76% to 77.5% = 2000
= 90 * 2.5
Marks obtained in Paper 2 by Naina = 180 = 5000
Weightage of marks obtained in Paper 2 by Total amount gets by Sachin = 5000 + 5000 +
Naina = 180/200*60 5000
= 54 = Rs.15000
Weightage of marks obtained in Paper 1 by
Naina = 90 – 54 4. Answer: D
= 36 New marks obtained in Paper 2 by Kanak = 150
Marks obtained in Paper 1 by Naina = 36/40*100 * (120/100)
= 90 = 180
2. Answer: C New weightage of obtained marks in Paper 2 by
New marks obtained in Paper 1 by Anshul = 60 Kanak = 180 * (3/10)
+ 20= 80 54
Weightage of marks obtained in Paper 1 by New weightage of obtained marks in Paper 1 by
Anshul = 80/100*40 = 32 Kanak = 78 – 54
Old final score of Anshul = 75 = 24
New final score of Anshul = 32 + 51= 83 Marks obtained in Paper 1 by Kanak = 24 * (5/2)

Click Here For Bundle PDF Course | support@guidely.in Page 8 of 14


SBI Clerk & RRB PO Mains PDF Course 2023
Quantitative Aptitude Day - 26 (Eng)

= 60 Number of Coca-Cola bottles sold by A = 3 * 144


Percentage change in marks obtained in Paper 1 = 432
by Kanak = ((75 – 60)/75) * 100 Number of Thumbs Up bottles sold by A = 1340
= (15/75) * 100 – 720 = 620
= 20% Let the number of Pepsi bottles sold by B be 2b
and the number of Coca-Cola bottles sold by B
5. Answer: B be 3b.
Total weightage marks Sachin = 77.5 We get, (Pepsi + Thumbs Up) – (Thumbs Up –
Total weightage marks of Bhanu = 81.5 Coca-Cola)
The sum of the total weightage marks of Sachin = Pepsi + Coca Cola
and Bhanu together = 77.5 + 81.5 So, 2b + 3b = 1390 – 90
= 159 5b = 1300
Total weightage marks of Anshul = 75 b = 260
Total weightage marks of Kanak = 75 Number of Pepsi bottles sold by B = 2 * 260 =
The sum of the total weightage marks of Anshul 520
and Kanak together = 75 + 75 Number of Coca-Cola bottles sold by B = 3 * 260
= 150 =780
Reqd. percentage = ((159 – 150)/150) * 100 Number of Thumbs Up bottles sold by B = 1390
= (9/150) * 100 – 520 = 870
= 6% Let the number of Pepsi bottles sold by C be 3c
and the number of Coca-Cola bottles sold by C
Direction (6 – 10): Let the number of Pepsi be 4c.
bottles sold by A be 5a and the number of Coca- We get, (Pepsi + Thumbs Up) – (Thumbs Up –
Cola bottles sold by A be 3a. Coca-Cola)
We get, (Pepsi + Thumbs Up) – (Thumbs Up – = Pepsi + Coca Cola
Coca-Cola) 3c + 4c = 1591 - 72
= Pepsi + Coca Cola 7c = 1519
So, 5a + 3a = 1340 – 188 c = 217
= 8a = 1152 Number of Pepsi bottles sold by C = 3 * 217 =
a = 144 651
Number of Pepsi bottles sold by A = 5 * 144 = Number of Coca-Cola bottles sold by C = 4 * 217
720 = 868

Click Here For Bundle PDF Course | support@guidely.in Page 9 of 14


SBI Clerk & RRB PO Mains PDF Course 2023
Quantitative Aptitude Day - 26 (Eng)

Number of Thumbs Up bottles sold by C = 1591


– 651 = 940
Let the number of Pepsi bottles sold by D be 5d
and the number of Coca-Cola bottles sold by D
be 2d.
We get, (Pepsi + Thumbs Up) – (Thumbs Up – 6. Answer: C
Coca-Cola) The sum of the number of Coca-Cola bottles
= Pepsi + Coca Cola sold by A and C together = 432 + 868
5d + 2d = 1475 – 124 = 1300
7d = 1351 The sum of the number of Thumbs Up bottles
d = 193 sold by D and E together = 510 + 990
Number of Pepsi bottles sold by D = 5 * 193 = = 1500
965 Then, 1300 * ((100 + d)/100) + 5 = 1500
Number of Coca-Cola bottles sold by D = 2 * 193 1300 * ((100 + d)/100) = 1495
= 386 ((100 + d)/100) = 1495/1300
Number of Thumbs Up bottles sold by D = 1475 100 + d = 115
– 965 = 510 d = 15
Let the number of Pepsi bottles sold by E be 2e
and the number of Coca-Cola bottles sold by E 7) Answer: D
be 5e. The number of 600 ml Pepsi bottles sold by B =
We get, (Pepsi + Thumbs Up) – (Thumbs Up – 520 * (5/26
Coca-Cola) = 100
= Pepsi + Coca Cola The number of 1 liter Pepsi bottles sold by B =
2e + 5e = 1332 – 135 520 * (13/26)
7e = 1197 = 260
e = 171 The number of 2-liter Pepsi bottles sold by B =
Number of Pepsi bottles sold by E = 2* 171 = 520 * (8/26)
342 = 160
Number of Coca-Cola bottles sold by E = 5 * 171 Total quantity of Pepsi sold by B = 100 * 0.6 +
= 855 260 * 1 + 160 * 2
Number of Thumbs Up bottles sold by E = 1332 = 60 + 260 + 320
– 342 = 990 = 640 liter

8) Answer: A

Click Here For Bundle PDF Course | support@guidely.in Page 10 of 14


SBI Clerk & RRB PO Mains PDF Course 2023
Quantitative Aptitude Day - 26 (Eng)

Let the quantity of Soda in vessel A be 2x and Total amount E gets after selling Thumbs Up
the quantity of Soda in vessel B be 5y. bottles = 62 * 990
Then, the ratio of Pepsi, Thumbs Up, and Soda = 61380
in the container = 12 : 9 : 13 Total amount E gets after selling Coca-Cola
3x : 9y : 2x + 5y = 12 : 9 : 13 bottles = 60 * 855
From here we can see, 3x/9y = 12/9 = 51300
x/y = 4 Total amount E gets after selling all cold drinks =
Or, x = 4y 131490
Quantity of mixture in vessel A = 3x + 2x = 5x = Total cold drinks sold by E = 342 + 990 + 855
20y = 2187
Quantity of mixture in vessel B = 9y + 5y = 14 y The average selling price of 1 bottle of cold drink
Reqd. percentage = (14y/20y) * 100 sold by E = 131490/2187
= 70% = 60.12

9. Answer: B 11) Answer: D


Total number of Pepsi bottles B has = 520 * Let total votes casted be 100%
(100/52) Total valid casted votes = 100% – 15% = 85%
= 1000 Total casted votes runner up candidate get =
Total number of Thumbs Up bottles B has = 870 85% – 48% = 37%
* (100/58) The difference of votes between the winning
= 1500 candidate and runner-up candidate gets = 3476
Total number of Coca Cola bottles B has = 780 * 48% - 37% = 3476
(100/65) 11% = 3476
= 1200 1% = 316
Total number of all cold drinks bottles B has = 100% = 31600
1000 + 1500 + 1200 Total number of casted votes = 31600
= 3700 Population of village C = 31600 * (100/80)
= 39500
10. Answer: C
Total amount E gets after selling Pepsi bottles = 12) Answer: B
55 * 342 Total valid votes = 100% – 15%
= 18810 = 85%

Click Here For Bundle PDF Course | support@guidely.in Page 11 of 14


SBI Clerk & RRB PO Mains PDF Course 2023
Quantitative Aptitude Day - 26 (Eng)

Valid votes runner-up candidate gets = 85% – The difference between the number of votes
52% winner candidate gets and the number of votes
= 33% runner up candidate gets = 3784
The difference between the number of votes 6x – 4x = 3784
winner candidate gets and the number of votes 2x = 3784
runner up candidate gets = 3268 x = 1892
52% - 33% = 3268 The number of votes P gets = 6 * 1892
19% = 3268 = 11352
1% = 172 The number of vote Q get = 4 * 1892
100% = 17200 = 7568
The total number of voters in village D is 17200. The number of vote R get = 7568 – 1892
= 5676
13) Answer: A The ratio of the number of votes get by P, Q, and
Let the number of votes the winner candidate of R = 11352 : 7568 : 5676
village E gets be x. =6:4:3
Number of votes got by winner candidate of The value of b is 3.
village B = x + 15900
Number of votes got by runner-up candidate of 15) Answer: C
village B = x + 15900 – 4392 Let total votes cast be 100%
= x + 11508 Percentage of votes winner candidate get = 60%
The ratio of the number of votes got by winner Percentage of votes runner-up candidate gets =
candidate of village E to the number of votes got 40%
by runner up candidate of village b = 2 : 3 The difference between the number of votes
x/(x + 11508) = 2 /3 winner candidate gets and the number of votes
3x = 2x + 23016 runner up candidate gets = 5180
x = 23016 60% - 40% = 5180
The number of votes got by runner up candidate 20% = 5180
of village E = 23016 – 2874 1% = 259
= 20142 100% = 25900
Total number of casted votes = 25900
14) Answer: D Number of literate voters = 25900 * (70/100)
Let the number of votes P gets be 6x and the = 18130
number of votes Q gets be 4x respectively. Number of illiterate voters = 25900-18130=7770

Click Here For Bundle PDF Course | support@guidely.in Page 12 of 14


SBI Clerk & RRB PO Mains PDF Course 2023
Quantitative Aptitude Day - 26 (Eng)

Number of votes winner candidate get = 25900 *


(60/100)= 15540
Number of literate voters voting to winner
candidate = 15540*(65/100) =10101
Number of illiterate voters who votes to winner
candidate = 15540 – 10101= 5439
16) Answer: B
Number of illiterate voters voting for runner-up
The volume of the spherical ball = 4312/3 m2
candidate = 7770 – 5439
(4/3) ∏r3 = 4312/3
= 2331
(4/3) (22/7) r3 = 4312/3
r3 = 49 * 7
Direction (16 – 18): Height of rooms of block B =
r = 7 meter
9 meter
The diameter of the sphere is the minimum
The breadth of rooms of block D = 9 meter
dimension of the length of the room.
Height of rooms of block A = 9 * (4/3)
Diameter = 2 * 7
= 12 meter
= 14 m
Breadth of rooms of block A = 12 * (2/3)
The new height of the room = 14 m
= 8 meter
Volume of 15 rooms of block C = 18 * 14 * 14 *
Length of rooms of block D = 12 meter
15
Length of the rooms of block B = 9 * (5/3)
= 52920 m3
= 15 meter
The breadth of rooms of block B = 8 + 3
17) Answer: A
= 11 meter
Surface area of four walls of the room block A =
Length of rooms of block C = 15 * (6/5)
2 (10 * 12) + 2 (8 * 12)
= 18 meter
= 240 + 192
The breadth of rooms of block C = 11 + 3
= 432 m2
= 14 meter
The surface area of the ceiling of the room of
Length of the rooms of block A = 15 * (2/3)
block A = 10 * 8
= 10 meter
= 80 m2
Height of the rooms of block C = 10 meter
Total surface area to be painted of the room of
Height of the rooms of block D = 10 * (4/5)
block A = 512 m2
= 8 meter
Total cost to paint a room of block A = 512 * 12
6144

Click Here For Bundle PDF Course | support@guidely.in Page 13 of 14


SBI Clerk & RRB PO Mains PDF Course 2023
Quantitative Aptitude Day - 26 (Eng)

Total cost to paint 42 rooms of block A = 6144 * 1.5a – 1.125a = 60


42 0.375a = 60
= Rs.258048 a = 160
The selling price of hair oil at which shopkeeper
18) Answer: D A sells = 1.275a
Volume of a room of block B = 15 * 11 * 9 = 1.275 * 160
= 1485 m3 = 204
Volume of a small wooden cube = 0.5 * 0.5 * 0.5
= 0.125 m3 20. Answer: C
Number of small wooden cubes fitted in a room Speed of Train A = M km/h
of block B = 1485/0.125 Speed of Train B = N km/h
= 11880 According to the question,
Number of small wooden cubes fitted in 13 (M + N) * 4 = 900 – 252
rooms of block B = 11880 * 13 (M + N) * 4 = 648
= 154440 Or, (M + N) = 162 …. (i)
And, ((4/5) M + N) * 1 = 144
19) Answer: B Or, 4M+5N = 720 …. (ii)
The cost price of Hair oil = a Subtracting eqn. ii from eqn. i
The marked price of hair oil marked by Multiplying eqn (i) with 4 both side, in order
shopkeeper A = 1.5a eliminate M.
Selling price of hair oil at which shopkeeper A 4M + 4N = 648
sell = 1.5a ((100 – 15)/100) (-)4M+(-)5N = (-)720
= 1.275a ------------------------
The selling price of hair oil at which shopkeeper -N = -72
B sells = a (112.5/100) N = 72 km/h
= 1.125a The speed of Train B is 72 km/h.
According to the question,

Click Here For Bundle PDF Course | support@guidely.in Page 14 of 14


SBI Clerk & RRB PO Mains PDF Course 2023
ENGLISH Day - 26

English Language

Directions (1-5) : Rearrange the following five 1) Which of the following is the FOURTH
sentences (A), (B), (C), (D) and (E) in the proper sentence after the rearrangement ?
sequence to form a coherent paragraph and then a) A
answer the questions given below. b) B
(A) Prior to this announcement, earlier in 2020, c) C
MEiTY notified rules stating that the Central d) D
government may allow Aadhaar authentication e) E
by requesting entities in the interest of good
governance, prevention of leakage of public 2) Which of the following is the FIRST sentence
funds and to promote ease of living. after the rearrangement ?
(B) The Registrar General of India (RGI) a) A
appointed under the Registration of Birth and b) B
Death Act, 1969 can now perform the yes or no c) C
Aadhaar authentication during registration of d) D
births and deaths in the country. e) E
(C) The State governments and Union Territory
administrations shall adhere to the guidelines 3) Which of the following sentences immediately
with respect to the use of Aadhaar authentication follows ‘sentence B’ in the rearranged sequence
as laid down by MEiTY. ?
(D) A gazette notification published in June said a) A
the Ministry of Electronics and Information b) B
Technology (MEiTY) has allowed the RGI office c) C
to use the Aadhaar database for authenticating d) D
the identity details provided during registration of e) E
births and deaths.
(E) The authentication may be on a voluntary 4) Which of the following is the SECOND
basis, for verification of Aadhaar number being sentence after the rearrangement ?
collected along with other details, as the case a) A
may be, for the purpose of establishing the b) B
identity of child, parent and the informant in case c) C
of deaths during registration of births or deaths”. d) D
e) E

Click Here For Bundle PDF Course | support@guidely.in Page 1 of 11


SBI Clerk & RRB PO Mains PDF Course 2023
ENGLISH Day - 26

5) Which of the following is the LAST sentence B. Rental homes are E. are not from a good
after the rearrangement ?
properties that are background and hence
a) A
leased out to tenants the women find it difficult
b) B
for a specified period to rent a house.
c) C of time, typically on a
d) D
e) E C. It is always good to F. and consulting with
prepare a rental local real estate agents

Directions (6-10) : In the following questions two application, including or property management

columns are given containing three your personal companies can provide

Sentences/phrases each. In the first column, information, you with more details.

sentences/phrases are A, B and C and in the employment details,

second column the sentences/phrases are D, E references,

and F. A sentence/phrase from the first column a) A-E


may or may not connect with another b) B-D
sentence/phrase from the second column to c) A-E and B-D
make a grammatically and contextually correct d) A-D and C-F
sentence. Each question has five options, four of e) None of these
which display the sequence(s) in which the
sentences/phrases can be joined to form a 7)
grammatically and contextually correct sentence. COLUMN 1 COLUMN 2
If none of the options given forms a correct
A. Battery technology D. charging
sentence after combination, mark option (e), i.e.
continues to evolve, infrastructure, and
“None of these” as your answer.
improving energy incentives can vary
6) COLUMN 1 COLUMN 2
density and reduced depending on your
A. Many assume that D. this will help you costs along country or region.
women who move out evaluate if the rental
B. The adoption of E. reducing air pollution
of their homes, or live prices are reasonable
electric vehicles is and greenhouse gas
without a family and negotiate with
increasing rapidly as emissions, thus helping
member landlords if needed.
technology advances, combat climate change.
costs decrease, and
environmental

Click Here For Bundle PDF Course | support@guidely.in Page 2 of 11


SBI Clerk & RRB PO Mains PDF Course 2023
ENGLISH Day - 26

c) C-E
C. Electric vehicles F. and have gained
d) A-D and C-F
(EVs) are vehicles significant popularity in
e) None of these
powered by electric recent years due to their
motors that run on environmental benefits
electricity and technological 9)

advancements. COLUMN 1 COLUMN 2

A. Intellectual property D. intellectual property


a) B-E
b) C-F rights are designed to of all the entities is very
protect and provide crucial and difficult at
c) A-D
d) C-F and B-D exclusive ownership the same time.

e) None of these and control over these


creations,
encouraging
8)
innovation,
COLUMN 1 COLUMN 2

A. The need of the D. ensure that banks do B. Seeking legal E. property rights are
hour is to analyse the not end up advice and registering protected and enforced
terms and conditions accumulating NPAs. intellectual property through laws and
of the agreement to with the relevant regulations.
authorities can be
B. An asset becomes E. and regulators may
classified as an NPA also introduce schemes C. In the field of F. rely on maintaining
when the borrower or institutions to assist scientific research the confidentiality rather
fails to make in the recovery process. need to protect the than obtaining legal
payments of interest protection through
and principal for a registration.

C. A high level of F. may vary depending a) A-E


b) A-D
NPAs in the banking on the regulations and
c) B-F
system can have policies of the country
d) C-D
adverse effects on or financial institution.
e) None of these
the

a) A-D
10)
b) B-F

Click Here For Bundle PDF Course | support@guidely.in Page 3 of 11


SBI Clerk & RRB PO Mains PDF Course 2023
ENGLISH Day - 26

COLUMN 1 COLUMN 2 11) According to Canada-based CBC News, the


federal government will be developing an
A. Engaging in D. and walking
immigration stream for some of the world's most
regular physical meditation incorporate
talented people that will be able to come to
activity and adopting gentle movements and
healthy lifestyle mindfulness to cultivate Canada to work for tech companies.

habits are essential a meditative state. a) who will be able to come to Canada to work

for b) that will be able to coming to Canada for work


c) who is able to come to Canada to working
B. Health and fitness E. and choose activities d) that will be come to Canada to work
are closely related that you enjoy to make e) No improvement needed
concepts that fitness a sustainable
encompass physical, and enjoyable part of 12) At last three people was killed and more than
mental, and your lifestyle. 40 wounded in a Russian rocket strike that hit a
restaurant in Kramatorsk, eastern Ukraine, the
C. Health is not F. they go hand in hand
authorities said.
merely the absence and are vital for leading
a) At last three people were killed and more than
of disease or infirmity a fulfilling and balanced
40 wounded
but a state of life.
b) At least three people were killed and more
complete
than 40 wounded
a) A-F
c) Almost three peoples was killed and more than
b) B-D
40 was wound
c) B-F
d) At the least of three people killed and 40
d) C-E
wounded
e) None of these
e) No improvement needed

Directions (11-15) : In each of the questions


13) The company undertaken the project by
given below a phrase in the sentence has been
helping of a team of experts, and significant
highlighted. It may or may not need replacement.
achievements were made on a large scale.
Read the question carefully and choose an
a) The company undertaken the project by the
option that would best replace the highlighted
help
parts to make the sentences grammatically and
b) The company undertook the project helping by
contextually correct. In case, the given sentence
c) The company undertaken the project with the
is correct in its current form and there is no
helping
improvement needed, mark (e) as your answer.

Click Here For Bundle PDF Course | support@guidely.in Page 4 of 11


SBI Clerk & RRB PO Mains PDF Course 2023
ENGLISH Day - 26

d) The company undertook the project with the Directions (16-20): Read each question to find
help out whether there is any grammatical error in it.
e) No improvement needed The error, if any, will be in one part of the
sentence. Choose the option with that part as
14) In April, China froze the visas of two of the your answer. If there is no error, mark(e) as your
four Indian correspondents, in what it described answer.
as countermeasures after India in March asked a 16) Families who were caught together/A at the
Chinese journalist to leave, leaving only two US-Mexico border and separated/B are now
Chinese reporters in the country. being offered an opportunity/C to reunite and
a) China freezed the visas of two of the four stay together/D.No error/E
Indian correspondents a) A
b) China freeze the visas of two of the four Indian b) B
correspondents c) C
c) China frozen the visas of two of the four Indian d) D
correspondents e) E
d) China freezing the visas of two of the four
Indian correspondents 17) He was right pretty much /A when he said
e) No improvement required almost all the people/B fail to change their
behaviour and continue to/C blame the system
15) ISRO was established in 1969 with the aim of for all their problems/D.No error/E
developing space technology and conducting a) A
space research activities in India and has been b) B
played a crucial role in advancing India's space c) C
capabilities and achieve significant milestones. d) D
a) has playing a crucial role in advance India's e) E
space capabilities and achieve
b) have played a crucial role in advancing India's 18) The human eardrum is able to receive/A and
space capabilities and achieve transmit signals across a radio/B frequency
c) have played a crucial role in advancing India's range more greater than/C what we can hear
space capabilities and achieving with the human ear/D.No error/E
d) has played a crucial role in advancing India’s a) A
space capabilities and achieving b) B
e) No improvement needed c) C
d) D

Click Here For Bundle PDF Course | support@guidely.in Page 5 of 11


SBI Clerk & RRB PO Mains PDF Course 2023
ENGLISH Day - 26

e) E
20) During the pandemic the/A companies were
19) Students should always stay motivated and providing more and more benefits to their/B
stop seeking/A motivation from outside instead employees and the employees fell for the/C
one must/B always ask themselves one question furniture and gift hampers not knowing what was
time and again,/C “Did I take a forward step coming for them/D.No error/E
today that leads to my success ?”/D.No error/E a) A
a) A b) B
b) B c) C
c) C d) D
d) D e) E
e) E
Click Here to Get the Detailed Video Solution for the above given Questions
Or Scan the QR Code to Get the Detailed Video Solutions

Answer Key with Explanation

1) Answer: C contains specific information on the


‘C’ is the fourth sentence after rearrangement. authentication mechanism. A paragraph always
The correct sequence is DBECA. with generic information and moves to specific
The first or the introductory sentence is ‘D’ content. With this we conclude that ‘D’ is our first
because sentences A,C and E do not contain statement of the paragraph. Immediate to ‘D’
any introduction, instead they are a continuation follows ‘B’ which holds the full form of RGI which
of one another. So the options to pick the is mentioned in the first sentence. After ‘B’
introductory sentence are two - B and D. If comes ‘E’ which talks about the purpose of this
noticed well, ‘D’ is more of a gist of the whole authentication and the need. Fourth sentence
paragraph that is to be formed whereas ‘B’ would be ‘C’ as it talks about who should adhere

Click Here For Bundle PDF Course | support@guidely.in Page 6 of 11


SBI Clerk & RRB PO Mains PDF Course 2023
ENGLISH Day - 26

to the announcement that has been made by the The sentence which immediately follows ‘B’ is
ministry. Lastly, it is ‘A’ which goes back to 2020 ‘E’.
when the ministry had an idea to allow Aadhar The first or the introductory sentence is ‘D’
authentication to make life easier. because sentences A,C and E do not contain
any introduction, instead they are a continuation
2) Answer: D of one another. So the options to pick the
‘D’ is the first sentence after the rearrangement. introductory sentence are two - B and D. If
The correct sequence is DBECA. noticed well, ‘D’ is more of a gist of the whole
The first or the introductory sentence is ‘D’ paragraph that is to be formed whereas ‘B’
because sentences A,C and E do not contain contains specific information on the
any introduction, instead they are a continuation authentication mechanism. A paragraph always
of one another. So the options to pick the with generic information and moves to specific
introductory sentence are two - B and D. If content. With this we conclude that ‘D’ is our first
noticed well, ‘D’ is more of a gist of the whole statement of the paragraph. Immediate to ‘D’
paragraph that is to be formed whereas ‘B’ follows ‘B’ which holds the full form of RGI which
contains specific information on the is mentioned in the first sentence. After ‘B’
authentication mechanism. A paragraph always comes ‘E’ which talks about the purpose of this
with generic information and moves to specific authentication and the need. Fourth sentence
content. With this we conclude that ‘D’ is our first would be ‘C’ as it talks about who should adhere
statement of the paragraph. Immediate to ‘D’ to the announcement that has been made by the
follows ‘B’ which holds the full form of RGI which ministry. Lastly, it is ‘A’ which goes back to 2020
is mentioned in the first sentence. After ‘B’ when the ministry had an idea to allow Aadhar
comes ‘E’ which talks about the purpose of this authentication to make life easier.
authentication and the need. Fourth sentence
would be ‘C’ as it talks about who should adhere 4) Answer: B
to the announcement that has been made by the ‘B’ is the second sentence after rearrangement.
ministry. Lastly, it is ‘A’ which goes back to 2020 The correct sequence is DBECA.
when the ministry had an idea to allow Aadhar The first or the introductory sentence is ‘D’
authentication to make life easier. because sentences A,C and E do not contain
any introduction, instead they are a continuation
3) Answer: E of one another. So the options to pick the
The correct sequence is DBECA. introductory sentence are two - B and D. If
noticed well, ‘D’ is more of a gist of the whole

Click Here For Bundle PDF Course | support@guidely.in Page 7 of 11


SBI Clerk & RRB PO Mains PDF Course 2023
ENGLISH Day - 26

paragraph that is to be formed whereas ‘B’ authentication and the need. Fourth sentence
contains specific information on the would be ‘C’ as it talks about who should adhere
authentication mechanism. A paragraph always to the announcement that has been made by the
with generic information and moves to specific ministry. Lastly, it is ‘A’ which goes back to 2020
content. With this we conclude that ‘D’ is our first when the ministry had an idea to allow Aadhar
statement of the paragraph. Immediate to ‘D’ authentication to make life easier.
follows ‘B’ which holds the full form of RGI which
is mentioned in the first sentence. After ‘B’ 6) Answer: A
comes ‘E’ which talks about the purpose of this The fragments/statements A and E combine to
authentication and the need. Fourth sentence form a meaningful sentence.
would be ‘C’ as it talks about who should adhere The other combinations are not correct as they
to the announcement that has been made by the do not form a meaningful and contextual
ministry. Lastly, it is ‘A’ which goes back to 2020 sentence.They conjunctions do not fit in right
when the ministry had an idea to allow Aadhar place to generate a complete sentence.
authentication to make life easier. The sentence : Many assume that women who
5) Answer: A move out of their homes, or live without a family
The last sentence after the rearrangement is ‘A’. member are not from a good background and
The correct sequence is DBECA. hence the women find it difficult to rent a house.
The first or the introductory sentence is ‘D’
because sentences A,C and E do not contain 7) Answer: B
any introduction, instead they are a continuation Statements C and F combine to form a
of one another. So the options to pick the contextually correct and meaningful sentence.
introductory sentence are two - B and D. If The combinations/pairs given in the other
noticed well, ‘D’ is more of a gist of the whole options do not form any correct sentence.
paragraph that is to be formed whereas ‘B’ The sentence : Electric vehicles (EVs) are
contains specific information on the vehicles powered by electric motors that run on
authentication mechanism. A paragraph always electricity and have gained significant popularity
with generic information and moves to specific in recent years due to their environmental
content. With this we conclude that ‘D’ is our first benefits and technological advancements.
statement of the paragraph. Immediate to ‘D’
follows ‘B’ which holds the full form of RGI which 8) Answer: A
is mentioned in the first sentence. After ‘B’ A-D both these statements combine to form a
comes ‘E’ which talks about the purpose of this sentence which is correct and meaningful.

Click Here For Bundle PDF Course | support@guidely.in Page 8 of 11


SBI Clerk & RRB PO Mains PDF Course 2023
ENGLISH Day - 26

Other given combinations do not combine to Replace ‘that’ with ‘who’ because the sentence
form any meaningful sentence and hence we go is talking about people who will be allowed to
for option a. enter Canada.
The sentence : The need of the hour is to Since options b and d have ‘that’ again - we can
analyse the terms and conditions of the eliminate those two options.
agreement to ensure that banks do not end up Option a is the only phrase which is
accumulating NPAs. grammatically correct.
The sentence: According to Canada-based CBC
9) Answer: D News, the federal government will be developing
The statements C and D together form a an immigration stream for some of the world's
meaningful sentence. most talented people who will be able to come to
The other options are wrong as the combinations Canada to work for tech companies.
of statements cannot be combined to form any
meaningful sentence. 12) Answer: B
The sentence : In the field of scientific research Replace ‘at last’ by a ‘at least’ and ‘was’ with
the need to protect the intellectual property of all ‘were’.
the entities is very crucial and difficult at the At last - finally
same time. At least - a minimum count
Since we are talking about three people use
10) Answer: C ‘were’ instead of ‘was’.
B and F combine to form a meaningful sentence. Option a is wrong because it has ‘at last’
Other combinations do not combine to form a Option c - peoples is wrong
correct sentence with meaning. The Option d -’At the least of’ is a wrong phrase to
connectors/conjunctions do not fall in place. use
The sentence : The sentence: At least three people were killed
Health and fitness are closely related concepts and more than 40 wounded in a Russian rocket
that encompass physical, mental, and they go strike that hit a restaurant in Kramatorsk, eastern
hand in hand and are vital for leading a fulfilling Ukraine, the authorities said.
and balanced life.
13) Answer: D
11) Answer: A Replace ‘undertaken’ with ‘undertook’ and
change the phrase ‘by helping’ to ‘with the help’.

Click Here For Bundle PDF Course | support@guidely.in Page 9 of 11


SBI Clerk & RRB PO Mains PDF Course 2023
ENGLISH Day - 26

The project was undertaken by the company (or) For eg : I was playing and eating at the same
The company undertook the project. time(we don’t say I was playing and eat at the
The sentence: The company undertook the same time)
project by helping a team of experts, and Options b and c are eliminated directly because
significant achievements were made on a large of the presence of ‘have’. ISRO has introduced
scale. something(it is a single body)
Option a - ‘has playing’ is a wrong phrase.
14) Answer: E The sentence: ISRO was established in 1969
No improvement needed, the sentence given is with the aim of developing space technology and
grammatically and contextually correct and conducting space research activities in India and
meaningful. has played a crucial role in advancing India's
Here in the context froze visas - prohibit them space capabilities and achieving significant
from using visa and passport to travel to milestones.
countries.
Different forms : 16) Answer: A
Freeze(present tense) - Please freeze the french The error is in part A of the sentence.
fries. Replace ‘who’ with ‘which’(even though family
Froze(past tense) - They froze to death. consist of member/people it is an entity and not
Freezes(simple present) - The water freezes a member by itself so it should be represented
easily. by ‘which’)
Frozen(past participle) - The frozen tree looked The corrected sentence: Families which were
amazing. caught together at the US-Mexico border and
Freezing(adjective) - It is freezing cold outside. separated are now being offered an opportunity
to reunite and stay together.
15) Answer: D
Replace ‘has been played’ to ‘has played’ and 17) Answer: A
‘achieve’ to achieving. The error is in part A of the sentence.
Has been playing - the action is still continuing He was right - is the main idea.
Has played - the action is over according to the Pretty much - nearly right(to agree that he was
sentence right)
Advancing(gerund)... and achieving(gerund) - He was pretty much right - is the correct phrase.
this is the rule. The corrected sentence : He was pretty much
right when he said almost all the people fail to

Click Here For Bundle PDF Course | support@guidely.in Page 10 of 11


SBI Clerk & RRB PO Mains PDF Course 2023
ENGLISH Day - 26

change their behaviour and continue to blame 19) Answer: E


the system for all their problems. The given sentence is error free as it is
grammatically correct and meaningful.
18) Answer: C
Part C is erroneous. 20) Answer: B
More greater(redundancy in the degree of Error is in part B of the sentence.
comparison) - it is wrong Replace ‘their’ with ‘its’. Company is an
The corrected sentences: entity(not a living thing) and usage of ‘their’ is
Replace ‘more’ with ‘far’ - The human eardrum is inappropriate.
able to receive and transmit signals across a The corrected sentence: During the pandemic
radio frequency range far greater than what we the companies were providing more and more
can hear with the human ear. benefits to their employees and the employees
Simply remove more - The human eardrum is fell for the furniture and gift hampers not knowing
able to receive and transmit signals across a what was coming for them.
radio frequency range greater than what we can
hear with the human ear.

Click Here For Bundle PDF Course | support@guidely.in Page 11 of 11


SBI Clerk & RRB PO Mains PDF Course 2023
Reasoning Ability Day - 27 (Eng)

Reasoning Ability
Directions (1-5): Study the following information
carefully and answer the given questions.
Nine different mobile shops – A, B, D, G, I, J, M,
N and O run on different floors of a nine storey
building where the lowermost floor is numbered
one and the floor immediately above it is
a) I-A, II-D, III-B, IV-C
numbered two and so on. No two shops were run
b) I-B, II-C, III-A, IV-D
on the same floor.
c) I-B, II-A, III-C, IV-D
Note 1: The shop names which start with a vowel
d) I-A, II-B, III-D, IV-C
were not run on an odd-numbered floor of the
e) I-B, II-D, III-A, IV-C
building.
Note 2: The consecutive alphabetically named
3. Which among the following shop runs on the
shops were not run on the adjacent floors of the
prime numbered floor as per the given
building.
arrangement?
G runs below fourth floor. Only three shops run
I. The shop which runs two floors below I.
between G and D. I runs either three floors below
II. The shop which runs immediately above A.
or three floors above D. The number of floors
III. The shop which runs on the adjacent floor of
above I is two less than the number of floors
N.
below M. N neither runs adjacent floor of D nor
a) Only III
runs below third floor. At most two shops run
b) Only I and II
between N and O. A runs below J. The number
c) Only I and III
of floors between J and A is three less than the
d) Only II
number of floors between D and B.
e) Only II and III
1. N runs __ floors below D and __ floors above
O respectively?
4. The number of floors between B and N is
a) One; Three
same as the number of floors between __ and __
b) Three; Three
a) M and I
c) Three; Four
b) A and O
d) Two; Three
c) Both a and b
e) One; Two
d) D and I
e) Both a and d
2. Match the list 1 with list 2

Click Here For Bundle PDF Course | support@guidely.in Page 1 of 10


SBI Clerk & RRB PO Mains PDF Course 2023
Reasoning Ability Day - 27 (Eng)

5. The difference between the floor numbers of G c) Third to the left; Daughter-in-law
and A is equal to the difference between the floor d) Fourth to the right; Daughter-in-law
numbers of __ and __. e) Third to the left; Mother
a) M and N
b) D and G 7. The number of persons sitting between __ and
c) B and N __ is two less than the number of persons sitting
d) J and I between __ and __, when counted from the left of
e) None of these the first persons in the given pairs respectively.
a) H and Husband of N; Son-in-law of P and N
Directions (6-10): Study the following information b) Mother-in-law of A and W; N and Son of H
carefully and answer the given questions. c) Husband of H and A; K and P
Eight persons from the same family – A, C, F, H, d) Both a and b
K, N, P, and W are sitting around a circular table e) Both a and c
facing the centre with equal distance between
adjacent persons. Four married couples are 8.__ sits second to the left of the mother of W,
there in the family. who sits ___ to the left of the Mother of C?
Note: Neither the couples nor the persons of the a) A; Second
same gender are sitting adjacent to each other. b) K; Fifth
K sits third to the right of W’s father. C is the only c) P; Fourth
child of K’s brother. Only one person sits d) A; Third
between K and C’s grandmother(either from left e) K; Fourth
or right). A is the brother-in-law of W and sits
adjacent to C’s grandmother. N sits immediate 9. Which of the following pair of persons are
left of the one who sits opposite to A, who has no married couples?
siblings. The number of persons sitting between
H and C’s father is one more than the number of
persons sitting between C’s father and C, when
counted from the right of H and left of C. H is the
parent of C and daughter-in-law of F. P sits
second to the left of K’s brother.
6. What is the position of W with respect to F and
How F is related to H’s sister-in-law?
a) Third to the right; Mother-in-law
b) Second to the left; Mother

Click Here For Bundle PDF Course | support@guidely.in Page 2 of 10


SBI Clerk & RRB PO Mains PDF Course 2023
Reasoning Ability Day - 27 (Eng)

Rickshaw A travels for 6m towards the south


direction from the rickshaw stand to reach
stoppage 1. Then turns towards the left and
travels for 8m to reach stoppage 2. Then, from
stoppage 2 again turns towards the left and
travels for 4m to reach stoppage 3, from there it
turns towards the right and travels for 9m to
reach stoppage 4.
Rickshaw B travels for 4m towards the east
direction from the rickshaw stand to reach
stoppage 5, from there it takes a left turn and
a) I-C, II-D, III-B, IV-C travels for 13m to reach stoppage 6. Then turns
b) I-C, II-A, III-A, IV-D towards the right and travels for 6m to reach
c) I-B, II-A, III-C, IV-D stoppage 7. Stoppage 8 is 7m south of stoppage
d) I-D, II-B, III-D, IV-C 7.
e) I-C, II-A, III-D, IV-B Rickshaw C travels for 9m towards the north
direction from the rickshaw stand to reach
10. In which among the following pair of persons stoppage 9. Stoppage 10 is east of stoppage 9
the first person sits second to the left of the and south of stoppage 7. Stoppage 11 is
second person? northeast of stoppage 10 and 7m east of
a) The one who faces C, A stoppage 7.
b) F, The one who sits immediate right of N These stoppages are assigned different names
c) The one who is the sister-in-law of K, N according to the below given conditions:
d) The one who is the mother of W, P 1). If the distance between the consecutive
e) None of these stoppages reached by the rickshaw is a multiple
of three, then “P” is named to the second
Directions (11-15): Study the following stoppage reached by the rickshaw (Consider
information carefully and answer the given rickshaw stand as one stoppage).
questions. 2). If the distance between the stoppage and the
There are three rickshaws i.e. rickshaw A, rickshaw stand is an even number (calculate as
rickshaw B, and rickshaw C starts travelling from total distance), then stoppage is named as “Q”,
a common rickshaw stand. Each rickshaw travel else the stoppage is named as “K”.
to some stoppages. 3). If the distance between the consecutive
stoppages reached by rickshaw is an odd prime

Click Here For Bundle PDF Course | support@guidely.in Page 3 of 10


SBI Clerk & RRB PO Mains PDF Course 2023
Reasoning Ability Day - 27 (Eng)

number, then “R” is named to the first stoppage a) 36m


reached by rickshaw. b) 25m
4). If the stoppage is in northeast or south c) 49m
direction from the rickshaw stand, then the d) 16m
stoppage is named as “M”. e) 64m
5). If the stoppage is in southeast or north
direction from the rickshaw stand, then the 14. Find the shortest distance between the
stoppage is named as “N”. stoppages S10 and S11.
6). If the distance between the consecutive a) √65m
stoppages is a root value, then only either b) √63m
conditions 4 or 5 follows. c) 2√31m
11. The total distance between which of the d) 2√27m
following two stoppages is greater than 15 with e) 2√11m
respect to the final arrangement? (If the rickshaw
stand is named as Y) 15. Which of the following pairs of stoppages,
a) Stoppages 1 and 4 the first stoppage is in northeast of the second
b) Stoppages 6 and Y stoppage with respect to the final arrangement?
c) Stoppages 5 and 11 a)Stoppages 7 and 5
d) Stoppages Y and 10 b) Stoppages 11 and 9
e) All of the above c) Stoppages 6 and 10
d) Both a and b
12. What is the shortest distance between the e) Both a and c
starting point to its final destination of rickshaw
B? Directions (16-20): Each of the questions below
a) √135 m consists of statements below it. You have to
b) 6√34 m decide whether the data provided in the
c) 2√34 m statements are sufficient to answer the question
d) 6√37 m and mark the appropriate option as answer.
e) 4√37 m 16. Seven vegetables viz. A, B, C, D, E, F, and G
were cooked on different days from Monday to
13. Find the square value of the difference Sunday of the same week. G was cooked after
between the sum of the total distance between Thursday. Only two vegetables were cooked
S5 and S8 and the sum of the total distance between G and C. Which of the following
between S5 and S4.

Click Here For Bundle PDF Course | support@guidely.in Page 4 of 10


SBI Clerk & RRB PO Mains PDF Course 2023
Reasoning Ability Day - 27 (Eng)

vegetables was cooked on the second day of the d) One person sits between B and G. A sits
week? adjacent to G. C sits third to the right of F.
a) B was cooked immediately after C. The e) H sits opposite to B. Two persons between H
number of vegetables cooked before B is one and F. G sits second to the right of F.
less than the number of vegetables cooked after
E. 18. Eight persons – P, Q, R, S, T, U, V and W
b) D was cooked two days before F. Only one live on eight different floors of an eight storey
vegetable was cooked between C and D. Only building where the bottommost floor is numbered
two vegetables were cooked between F and A. B 1 and the topmost floor is numbered 8. R lives on
was cooked neither before E nor immediately one of the even numbered floors. Only three
after E. floors between R and T. Who lives two floors
c) As many vegetables cooked before C as after above T?
E. F was cooked three days before E. A was a) As many floors above T as below P. Only two
cooked adjacent to either E or F. floors between P and W. Q lives four floors
d) A was cooked two days after C. Only three above W. S lives adjacent floor of neither T nor
vegetables were cooked between A and E. W.
Atmost one vegetable was cooked between E b) Q lives two floors below T. As many floors
and F. above Q as below W. Only one floor between W
e) D was cooked adjacent to neither C nor G. and P.
Only three vegetables were cooked between D c) The number of floors between R and T is one
and E. F was cooked five days after A. more than the number of floors between R and
W. Q lives two floors above W. Not less than
17. Eight persons - A, B, C, D, E, F, G and H are three floors between W and S.
sitting around a circular table facing the centre. E d) Only two floors between R and W. Q lives four
sits third to the left of D. B sits second to the left floors below W. As many floors above W as
of E. Who sits second to the left of F? below P. U lives adjacent to P.
a) Two persons sit between B and F. C sits e) Q lives three floors below R. The number of
adjacent to B. More than two persons sit floors between R and Q is one less than Q and
between C and G, when counted from the right of W. P lives immediately above W. Only two floors
C. between P and S.
b) As many persons sit between D and B as E
and H. G sits second to the right of A. 19. Six persons – E, G, J, K, T and W have
c) I sits adjacent to B. G sits two places away different number of Toys. Atmost one person has
from H. less number of toys than W. Who among the

Click Here For Bundle PDF Course | support@guidely.in Page 5 of 10


SBI Clerk & RRB PO Mains PDF Course 2023
Reasoning Ability Day - 27 (Eng)

following person has the second highest number 20. There are six members in the family. Two
of Toys? married members and each married couple have
a) J has more toys than W but less than T. K has at least one child. How Q is related to R?
more toys than J and T but less than E. G has a) R is the mother of Q. U is the son of R’s only
more toys than T but less than E. son.
b) T has less toys than J but more than K. G has b) U is the son of Q’s brother. M is the only son
more toys than K but does not have the second of E, who is the father-in-law of T.
highest number of toys. c) T is the only daughter-in-law of E. M is the
c) T has more toys than J but less than K. G has father of U, who is the nephew of Q.
more toys than K. d) U is the daughter of Q’s brother. M is the only
d) Either a or b son of E, who is married to R.
e) None of these e) None of these

Click Here to Get the Detailed Video Solution for the above given Questions
Or Scan the QR Code to Get the Detailed Video Solutions

Answer Key with Explanation

Directions (1-5):
1. Answer: D
2. Answer: E
3. Answer: B
4. Answer: C
5. Answer: A
Final arrangement

Click Here For Bundle PDF Course | support@guidely.in Page 6 of 10


SBI Clerk & RRB PO Mains PDF Course 2023
Reasoning Ability Day - 27 (Eng)

We have,
 G runs below fourth floor.
 Only three shops run between G and D.
 I runs either three floors below or three
floors above D.
From the above conditions, there are three
possibilities

Again we have,
 A runs below J.
 The number of floors between J and A is
three less than the number of floors
between D and B.
From the above condition Case-2 gets
eliminated because the number of floors
Again we have, between J and A is three less than the number
 The number of floors above I is two less of floors between D and B. Hence Case-1 shows
than the number of floors below M. the final answer.
 N neither runs adjacent floor of D nor runs
below third floor.
 At most two shops run between N and O.
From the above condition Case-3 gets
eliminated because the number of floors above I
is two less than the number of floors below M.

Directions (6-10):
6. Answer: E

Click Here For Bundle PDF Course | support@guidely.in Page 7 of 10


SBI Clerk & RRB PO Mains PDF Course 2023
Reasoning Ability Day - 27 (Eng)

7. Answer: D From the above conditions, there are three


8. Answer: B possibilities
9. Answer: E
10. Answer: C
Final arrangement

Again we have,
 The number of persons sitting between H
and C’s father is one more than the
number of persons sitting between C’s
father and C, when counted from the right
of H and left of C.
 H is the parent of C and daughter-in-law
We have,
of F.
 K sits third to the right of W’s father.
 P sits second to the left of K’s brother.
 C is the only child of K’s brother.
Applying the above conditions, Case-2 gets
 Only one person sits between K and C’s
eliminated because C is not the father of W.
grandmother(either from left or right).
Case-1(a) gets eliminated because we cannot
 A is the brother-in-law of W and sits
place P. Hence, Case-1 shows the final answer.
adjacent to C’s grandmother.
 N sits to the immediate left of one who
sits opposite to A, who has no siblings.

Click Here For Bundle PDF Course | support@guidely.in Page 8 of 10


SBI Clerk & RRB PO Mains PDF Course 2023
Reasoning Ability Day - 27 (Eng)

Directions (16-20):
16. Answer: B

Directions (11-15):
11. Answer: E
12. Answer: C
13. Answer: B 17. Answer: D
14. Answer: A
15. Answer: D
Final arrangement

Click Here For Bundle PDF Course | support@guidely.in Page 9 of 10


SBI Clerk & RRB PO Mains PDF Course 2023
Reasoning Ability Day - 27 (Eng)

19. Answer: E

20. Answer: E
Clearly, none of the statements gives the relation
18. Answer: A
between Q and R.
Hence, option E is the correct choice.

Click Here For Bundle PDF Course | support@guidely.in Page 10 of 10


SBI Clerk & RRB PO Mains PDF Course 2023
Quantitative Aptitude Day - 27 (Eng)

Quantitative Aptitude

Direction (1-5): Study the following data carefully and answer the questions:
A series of five T-20 matches is played between two teams A and B.
Pie chart given below shows the percentage distribution of total runs scored by both the teams together
in each match.

Pie chart given below shows the degree distribution of total runs scored by team A in each match.

Note:
1: Ratio of the runs scored by team B in 1st match to the runs scored by team B in 2nd match is 1: 2.

Click Here For Bundle PDF Course | support@guidely.in Page 1 of 18


SBI Clerk & RRB PO Mains PDF Course 2023
Quantitative Aptitude Day - 27 (Eng)

2: Runs scored by team B in 4th match are 20 more than that scored by team B in 5th match.
1) In the 4th match, of total runs were c)99: 56
scored by hitting boundaries and the remaining d) 37: 25
were scored by running between the wickets. If e) 3: 2
team A hit 21 fours and 6 sixes, then find the
runs scored by team B by running between the 4) If team A hit 15 fours and 5 sixes in 2nd match
wickets? and hit 9 fours and 4 sixes in 5th match, then find
a) 80 the average of runs scored by team A in 2nd
b)100 match by running between the wickets and that
c) 90 scored in the 5th match by running between the
d) 60 wickets?
e) 120 a) 60
b) 70
2) If the total runs scored by team B in each c)40
match are described in a circle and the degree d) 50
distribution of runs scored by team B in 3rd and e) 80
5th matches are M° and N° respectively, then find
the value of ? 5) In 2nd match, if ___% of total runs were scored
a) 5 by hitting boundaries and ___% of total runs
b) 2 scored by team A were scored by hitting
c) 3 boundaries and then 80% of total runs scored by
d) 6 team B were scored by hitting boundaries.
e) 4 Find that which of the following can be filled in
the blanks in the same order?
3) If team B hit 2 sixes and 7 fours in 1st match, 3 P: 50%, 20%
sixes and 5 fours in 3rd match and 10 sixes and Q: 70%, 60%
15 fours in 4th match, then find the ratio of total R: , 50%
runs scored by team B by hitting boundaries in a) Only P and R
1st, 3rd and 4th matches together to that scored by b) Only R
running between the wickets in 1st, 3rd and 4th c) Only Q and R
matches together? d) Only P and Q
a) 169: 141 e) Only P
b) 1: 1

Click Here For Bundle PDF Course | support@guidely.in Page 2 of 18


SBI Clerk & RRB PO Mains PDF Course 2023
Quantitative Aptitude Day - 27 (Eng)

Direction (6-9): Study the following data carefully and answer the questions:
The Pie chart given below shows the percentage and degree distribution of monthly expenditure of a
person across different categories. (Rent, Groceries, Transportation, Utilities, Entertainment and
Miscellaneous).

Note:
1: Monthly expenditure on rent, utilities and miscellaneous together is 45% of the total monthly
expenditure.
2: Difference between monthly expenditure on rent and that on utilities, is ₹ 4800.
6) Which of the following is/are true? 7) If the total monthly expenditure of the person
A: Monthly expenditure on groceries is (5X – is of his monthly salary, his monthly
2Y)% more than that on utilities. expenditures on rent and groceries are
B: Average monthly expenditure on respectively P% and Q% of his monthly salary,
transportation, utilities and entertainment is then find that his monthly expenditure on
₹ 3000. transportation is what percent of his monthly
C: The difference between monthly expenditures salary?
on rent and miscellaneous is ₹ 6000. a)
a) Only A and B b)
b) All are true c)
c) Only C d)
d) Only A and C
e)
e) Only B

Click Here For Bundle PDF Course | support@guidely.in Page 3 of 18


SBI Clerk & RRB PO Mains PDF Course 2023
Quantitative Aptitude Day - 27 (Eng)

8) If the ratio of total monthly expenditure on c) 16: 11


rent, groceries and transportation together to the d) 503: 308
total monthly expenditure on utilities, e) 64: 25
entertainment and miscellaneous together is M:
N, then find the value of (9M + 4N)M - N? 11) Two series (I) and (II) are given below with
a) 625 the wrong term P in series (I) and the missing
b) 1024 term Q in series (II).
c)676 I: 12, 16, 24, 48, 72, 136
d) 729 II: 15, 23, ?, 71, 135, 263
e) 961 Find that which of the following is/are true?
A: P and Q are co-prime with each other.
9) If the total monthly expenditure on B: Value of (4Q – 3P) is a multiple of 7.
transportation and entertainment together is C: Value of (3P + 2Q) is a multiple of 11.
14.4% of his monthly salary and he invested a) All are true
of his monthly savings in a scheme at b) Only A and B
15% rate of compound interest, and then find the c) None is true
compound interest received from the scheme d) Only B
after 2 years? e) Only C
a) ₹ 6450
b) ₹ 3870 12) Quantity I: Average age of A, B, and C are
c) ₹ 7740 22 years, and the average age of B, C, and D is
d) ₹ 5160 20 years. If A’s age is neither more than 32 years
e) ₹ 5805 nor less than 28 years, then find D’s age?
Quantity II: P and Q together can complete a
10) A series is given below with the wrong term work in 6.75 hours, while P and R together can
‘P’ in it. complete the same work in 6 hours. If the ratio of
698, 673, 624, 503, 320, 45 Q’s efficiency to R’s efficiency is 2: 3, then at
If a new series is formed, whose 1st term is P and what time Q alone can complete the work?
the new series follows the same pattern as in the a) Quantity I > Quantity II
given series, then find the ratio of the 1st term to b) Quantity I < Quantity II
4th term of the new series? c) Quantity I ≥ Quantity II
a) 698: 503 d) Quantity I ≤ Quantity II
b) 673: 478 e) Quantity I = Quantity II or relation can’t be
established

Click Here For Bundle PDF Course | support@guidely.in Page 4 of 18


SBI Clerk & RRB PO Mains PDF Course 2023
Quantitative Aptitude Day - 27 (Eng)

13) Two trains A and B are running in opposite e) Quantity I = Quantity II or relation can’t be
directions. The speed of train A is 32.4 km/h established
more than that of train B and they cross each
other in 10 seconds. Train A crosses a 60 m long 15) Quantity I: A bag contains 5 similar balls and
tunnel in 10 seconds and Train B crosses the number of ways in which ‘x’ balls can be drawn
same tunnel in 18 seconds. from the bag is 10. Find the value of ‘y’ if y = x2 +
Quantity I: Find the difference between the 2x + 3 (x > 0).
lengths of both trains. Quantity II: Two persons A and B together can
Quantity II: At what time, train A will cross a 540 finish a work in 11.25 days while the ratio of their
m long bridge? efficiency is 3: 5 (It is unknown who is more
a) Quantity I > Quantity II efficient). Find the time taken (in days) by A
b) Quantity I < Quantity II alone to finish the work.
c) Quantity I ≥ Quantity II a) Quantity I > Quantity II
d) Quantity I ≤ Quantity II b) Quantity I < Quantity II
e) Quantity I = Quantity II or relation can’t be c) Quantity I ≥ Quantity II
established d) Quantity I ≤ Quantity II
e) Quantity I = Quantity II or relation can’t be
14) Quantity I: Boat A goes 540 m downstream established
in 45 seconds and boat B goes 240 m upstream
in 48 seconds. If the ratio of the speed of boat A 16) The cost price of an item is ₹ 700 which is
in still water to that of boat B in still water is 19: marked ‘x’% above the cost price. The item is
15, then find the time taken by boat A to go 224 sold after giving a discount of ₹ 245 such that
m upstream. difference between cost price and selling price
Quantity II: In a mixture of milk and water, the becomes ₹ 35.
quantity of milk is 6 L more than that of water Quantity I: Find the value of ‘x’.
and ratio of the cost of pure milk to the cost of Quantity II: If the item is sold for ₹ y more, then
the mixture is 5: 3. If 5 L quantity of the mixture is find the profit percent earned after selling that
replaced with the same quantity of milk, then item. Assume initially the item was sold in loss
what will be the new cost of the mixture?. and y ≥ 210.
a) Quantity I > Quantity II a) Quantity I > Quantity II
b) Quantity I < Quantity II b) Quantity I < Quantity II
c) Quantity I ≥ Quantity II c) Quantity I ≥ Quantity II
d) Quantity I ≤ Quantity II d) Quantity I ≤ Quantity II

Click Here For Bundle PDF Course | support@guidely.in Page 5 of 18


SBI Clerk & RRB PO Mains PDF Course 2023
Quantitative Aptitude Day - 27 (Eng)

e) Quantity I = Quantity II or relation can’t be established

Direction (17-20): Study the following data carefully and answer the questions:
Table given below shows the ratio of the boys to girls in five difference colleges A, B, C, D, and E. It also
shows the average age of all the students (Boys + Girls) in each college and also the difference between
the average age of boys and girls in each college.

Note:
1. ‘+’ sign indicates that average age of boys is more than the average age of girls while ‘–’ sign indicates
that the average age of girls is more than the average age of boys.
2. Average age of girls in college B and C is same and average age of all the boys of college A is 28
years.
3. Average age of all the boys of college D is same as the average age of all the girls of college E.
17)If the ratio of boys in college A to college E is 18) If 15 new boys and 5 new girls joined
10: 3 and the average age of all the students at college B, then the ratio of boys to girls in that
colleges A and E together is P/10, then how college becomes 1: 1. If the average age of all
many factors of P are possible? the newly joined boys and girls is ‘a + c – 1’
a) 5 years ‘b – 2’ years respectively, then find the
b) 2 average age of all the students (including newly
c) 4 joined) of college B.
d) 3 a) 19 years
e) 6 b) 18 years
c) 21 years

Click Here For Bundle PDF Course | support@guidely.in Page 6 of 18


SBI Clerk & RRB PO Mains PDF Course 2023
Quantitative Aptitude Day - 27 (Eng)

d) 22 years 20) The difference between the sum of the ages


e) 20 years of all the boys and the sum of the ages of all the
girls of college A is 245 years and the total
19) If 20 more students joined college D such number of students in college A is ‘d’. If the
that their average age is 27 years, then average numerical value of all the variables a, b, c, and d
age of all the students at that college increased are arranged in a series in ascending order, then
by 1.9 years. Find the ratio of boys to girls in the find that 5thterm of the series is what percent of
college after the joining of new students if the the 8th term of that series?
newly joined boys are multiple of both 3 and 4. a)
a) 31: 19 b)
b) 7: 3 c)
c) 17: 8 d)
d) 33: 17 e)
e) 18: 7
Click Here to Get the Detailed Video Solution for the above given Questions
Or Scan the QR Code to Get the Detailed Video Solutions

Answer Key with Explanation

Direction (1-5): Since, the ratio of the runs scored by team B in


Let the total runs scored by both the teams 1st match to the runs scored by team B in 2nd
together in 1st, 2nd, 3rd, 4th and 5th matches are match is 1: 2.
‘15x’, ‘25x’, ‘10x’, ‘30x’ and ‘20x’ respectively. So,
Also let the runs scored by team A in 1st, 2nd, 3rd,
4th and 5th matches are ‘63y’, ‘90y’, ‘27y’, ‘120y’
30x – 126y = 25x – 90y
and ‘60y’ respectively.
5x = 36y --------------(1)

Click Here For Bundle PDF Course | support@guidely.in Page 7 of 18


SBI Clerk & RRB PO Mains PDF Course 2023
Quantitative Aptitude Day - 27 (Eng)

Since, the runs scored by team B in 4th match Total runs scored by team B in all the 5 matches
are 20 more than that scored by team B in 5th together:
match. 75 + 150 + 75 + 160 + 140 = 600
So, Since, the runs scored by team B in 3rd match =
(30x – 120y) – (20x – 60y) = 20 75
10x – 60y = 20 -------------- (2) So, M = = 45°
From equations (1) and (2): Since, the runs scored by team B in 5th match =
2 * 36y – 60y = 20 140
y = 5/3 So, N = = 84°
From equation (1): And the value of = =7–5=2
x = 12

3) Answer: C
Runs scored by team B by hitting boundaries in
1st match = 2 * 6 + 7 * 4 = 40
So, runs scored by team B by running between
the wickets in 1st match = 75 – 40 = 35
Runs scored by team B by hitting boundaries in
3rd match = 3 * 6 + 5 * 4 = 38
So runs scored by team B by running between

1) Answer: A the wickets in 3rd match = 75 – 38 = 37

In the 4th match: Runs scored by team B by hitting boundaries in

Total runs scored = 360 4th match = 10 * 6 + 15 * 4 = 120

Runs scored by hitting boundaries = of So, runs scored by team B by running between
the wickets in 4th match = 160 – 120 = 40
360 = 200
Required ratio = (40 + 38 + 120): (35 + 37 + 40)
Runs scored by team A by hitting boundaries =
= 198: 112 = 99: 56
(21 * 4) + (6 * 6) = 120
Runs scored by team B by hitting boundaries =
4) Answer: D
200 – 120 = 80
Since, total runs scored by team A in 2nd match
Total runs scored by team B = 160
= 150
So, the runs scored by team B by running
So, the runs scored by team A in 2nd match by
between the wickets = 160 – 80 = 80
running between the wickets:
150 – [(15 * 4) + (5 * 6)] = 60
2) Answer: B

Click Here For Bundle PDF Course | support@guidely.in Page 8 of 18


SBI Clerk & RRB PO Mains PDF Course 2023
Quantitative Aptitude Day - 27 (Eng)

Since, total runs scored by team A in 5th match = From R:


100 If of total runs scored in 2nd match were
So, the runs scored by team A in 5th match by scored by hitting boundaries.
running between the wickets: So, total runs scored by hitting boundaries in 2nd
100 – [(9 * 4) + (4 * 6)] = 40 match = of 300 = 200
Required average = = 50 Runs scored by team A by hitting boundaries in
2nd match = 200 – 120 = 80
5) Answer: D Which is = 53.33% of total runs scored
Total runs scored by team B in 2nd match = 150 by team A in 2nd match.
So, the runs scored by team B by hitting So, values in R can’t be filled in the blanks in the
boundaries in 2nd match = 80% of 150 = 120 same order.
From P: Direction (6-9):
If 50% of total runs scored in 2nd match were Since, the monthly expenditure on rent, utilities
scored by hitting boundaries. and miscellaneous together is 45% of the total
So, total runs scored by hitting boundaries in 2nd monthly expenditure.
match = 50% of 300 = 150 So,
Runs scored by team A by hitting boundaries in 3X + X + (X – 18) = 45% of 360
2nd match = 150 – 120 = 30 X = 36
Which is = 20% of total runs scored by And,
team A in 2nd match. (2Y + 5) + 2Y + Y = 100 – 45
So, values in P can filled in the blanks in the Y = 10
same order. Percentage distribution of monthly expenditure
From Q: on rent = = 30%
If 70% of total runs scored in 2nd match were Percentage distribution of monthly expenditure
scored by hitting boundaries. on utilities = = 10%
So, total runs scored by hitting boundaries in 2nd Percentage distribution of monthly expenditure
match = 70% of 300 = 210 on miscellaneous = = 5%
Runs scored by team A by hitting boundaries in Since, difference between monthly expenditure
2nd match = 210 – 120 = 90 on rent and that on utilities, is ₹ 4800.
Which is = 60% of total runs scored by So, the monthly expenditure on rent =
team A in 2nd match. = ₹ 7200
So, values in Q can filled in the blanks in the The monthly expenditure on groceries =
same order. = ₹ 6000

Click Here For Bundle PDF Course | support@guidely.in Page 9 of 18


SBI Clerk & RRB PO Mains PDF Course 2023
Quantitative Aptitude Day - 27 (Eng)

The monthly expenditure on transportation = Since, monthly expenditure on rent = ₹ 7200


= ₹ 4800 So, P = = 20%
The monthly expenditure on utilities = 7200 – Since, monthly expenditure on groceries =
4800 = ₹ 2400 ₹ 6000
The monthly expenditure on entertainment = So, Q =
= ₹ 2400 Since, monthly expenditure on transportation =
And the monthly expenditure on miscellaneous = ₹ 4800
= ₹ 1200 So, required percentage =
6) Answer: C
From A:
Monthly expenditure on groceries is 8) Answer: E
= 150% more than that on Total monthly expenditure on rent, groceries and
utilities. transportation together = 7200 + 6000 + 4800 =
Since, 150% = (5X – 3Y) % ₹ 18000
So, A is not true. Total monthly expenditure on utilities,
From B: entertainment and miscellaneous together =
Average monthly expenditure on transportation, 2400 + 2400 + 1200 = ₹ 6000
utilities and entertainment: Required ratio = M: N = 18000: 6000 = 3: 1
So, the value of (9M + 4N)M– N = (27 + 4)2 = 961

So, B is not true.


9) Answer: E
From C:
Total monthly expenditure on transportation on
Difference between monthly expenditures on
entertainment together = 4800 + 2400 = ₹ 7200
rent and miscellaneous:
So, the monthly salary of the person =
7200 – 1200 = ₹ 6000
= ₹ 50000
So, C is true.
Hence, only C is true. Since, total monthly expenditure on the person:
7200 + 6000 + 4800 + 2400 + 2400 + 1200 =

7) Answer: B ₹ 24000

Total monthly expenditure of the person: So, the monthly savings of the person = 50000 –

7200 + 6000 + 4800 + 2400 + 2400 + 1200 = 24000 = ₹ 26000

₹ 24000 Now, the amount invested in the scheme =

Total monthly salary of the person = of 26000 = ₹ 18000

= ₹ 36000

Click Here For Bundle PDF Course | support@guidely.in Page 10 of 18


SBI Clerk & RRB PO Mains PDF Course 2023
Quantitative Aptitude Day - 27 (Eng)

And the CI received from the scheme after 2 From A:


years: Factors of P = 48 = 2 * 2 * 2 * 2 * 3
Factors of Q = 39 = 3 * 13
Since, P and Q have a common factor.
So, A is not true.
10) Answer: E
From B:
Logic in the given series:
Value of (4Q – 3P) = (4 * 39) – (3 * 48) = 156 –
698 - 52 = 673
144 = 12
673 - 72 = 624
Since, 12 is not a multiple of 7.
624 - 112 = 503
So, B is not true.
503 - 132 = 334(320)
From C:
334 - 172 = 45
Value of (3P + 2Q) = (3 * 48) + (2 * 39) = 144 +
So, the wrong term in the series = P = 320
78 = 222
Now, 1st term of the new series = 320
Since, 222 is not a multiple of 11.
2nd term of the new series = 320 – 52 = 295
So, C is not true,
3rd term of the new series = 295 - 72 = 246
Hence, none is true.
4th term of the new series = 246 - 112 = 125
Required ratio = 320: 125 = 64: 25
12) Answer: B
Quantity I:
11) Answer: C
Since, A + B + C = 3 * 22 = 66 ----------(1)
Logic in series (I):
And, B + C + D = 3 * 20 = 60 ---------- (2)
12 + 4 = 16
By equation (1) – equation (2):
16 + 8 = 24
A–D=6
24 + 16 = 40(48)
Since, A’s is age neither more than 32 years nor
40 + 32 = 72
less than 28 years.
72 + 64 = 136
If A’s age is 32 years.
So, the wrong term in series (I) = P = 48
So, D’s age = 32 – 6 = 26 years
Logic in series (II):
If A’s age is 28 years.
15 + 8 = 23
So, D’s age = 28 – 6 = 22 years
23 + 16 = 39
So, the possible ages of D are 22 years, 23
39 + 32 = 71
years, 24 years, 25 years and 26 years.
71 + 64 = 135
Quantity II:
135 + 128 = 263
So, the missing term in series (II) = Q = 39

Click Here For Bundle PDF Course | support@guidely.in Page 11 of 18


SBI Clerk & RRB PO Mains PDF Course 2023
Quantitative Aptitude Day - 27 (Eng)

Since, ratio of Q’s efficiency to R’s efficiency is


2: 3. a + 60 = 10x + 90 ------------- (2)
Let the time, in which Q alone can R alone can Since, train B crosses the 60 m long tunnel in 18
complete the work are ‘3t’ hours and ‘2t’ hours seconds.
respectively. So,
So,

And,
From equations (1) and (3):
18a + 18b = 20b + 1200 + 1620
From equations (1) and (2):
9a – b = 1410 -------------- (4)
From equations (2) and (3):
18a + 1080 = 10b + 600 + 1620
9a – 5b = 570 --------------- (5)
By equation (4) – equation (5):
t=9
9a – b – 9a + 5b = 1410 – 570
So, the time, in which Q alone can complete the
b = 210
work = 3 * 9 = 27 hours
From equation (4):
Hence, Quantity I < Quantity II
a = 180
From equation (3):
13) Answer: E
x = 15
Let the speed of train B = ‘x’ m/s
Quantity I:
So, the speed of train A = x + 32.4 * (5/18) = (x +
Difference between the lengths of both the trains
9) m/s
= 210 – 180 = 30 m
Also let the lengths of trains A and B are ‘a’ m
Quantity II:
and ‘b’ m respectively.
Length of train A = 180 m
Since, both the trains cross each other in 10
Speed of train A = 15 + 9 = 24 m/s
seconds.
So, the time, in which train A will cross the 540
So,
m long bridge:

a + b = 20x + 90 ------------- (1)


Hence, Quantity I = Quantity II
Since, train A crosses the 60 m long tunnel in 10
seconds.
14) Answer: E
So,

Click Here For Bundle PDF Course | support@guidely.in Page 12 of 18


SBI Clerk & RRB PO Mains PDF Course 2023
Quantitative Aptitude Day - 27 (Eng)

Quantity I: Hence, relation can’t be established.


Let the speeds of boats A and B in still water are
‘19x’ m/s and ‘15x’ m/s respectively. 15) Answer: D
Also let the speed of the stream is ‘y’ m/s. Quantity I:
So, Total balls in the bag = 5
Total number of balls which are withdrawn from

19x + y = 12 ---------- (1) the bag = ‘x’

And Number of ways in which ‘x’ balls can be drawn


from the bag = 5Cx = 10

15x – y = 5 ---------- (2)


From equations (1) and (2):
x = 0.5, y = 2.5 x! (5 – x)! = 12
Since, the upstream speed of boat A = 19 * 0.5 – Maximum value ‘x’ can take is 5.
2.5 = 7 m/s When x = 1, then x! (5 – x)! = 1! (5 – 1)! = 24
So, the time, in which boat A can cover 224 m When x = 2, then x! (5 – x)! = 2! (5 – 2)! = 12
upstream = = 32 seconds When x = 3, then x! (5 – x)! = 3! (5 – 3)! = 12
Quantity II: When x = 4, then x! (5 – x)! = 4! (5 – 4)! = 24
Let the initial quantity of water = ‘x’ L When x = 5, then x! (5 – x)! = 5! (5 – 5)! = 1
So, the initial quantity of milk = (x + 6) L Hence, possible values of ‘x’ = 2 and 3.
Let the cost of pure milk is ₹ ’5y’ per L and the Now,
initial cost of the mixture is ₹ ‘3y’ per L. y = x2 + 2x + 3 = 11 (when x = 2) and 18 (when x
So, = 3)
Hence, possible values of y are 11 and 18.

5x + 30 = 6x + 18 Quantity II:

x = 12 Case 1: When A is more efficient than B.

The initial quantity of water = 12 L Ratio of efficiency of time taken by A to B is 5: 3.

The initial quantity of milk = 12 + 6 = 18 L Let time taken A and B alone to finish the work is

After replacing 5 L quantity of the mixture with ‘3x’ days and ‘5x’ days respectively.

the same quantity of milk: According to the question:

The new quantity of milk = 18 – 3 + 5 = 20 L


The new quantity of water = 12 – 2 = 10 L
Since, we don’t know the cost of the pure milk.
x=6
So, the cost of new mixture can’t be determined.

Click Here For Bundle PDF Course | support@guidely.in Page 13 of 18


SBI Clerk & RRB PO Mains PDF Course 2023
Quantitative Aptitude Day - 27 (Eng)

Time taken by A alone to finish the work = 3x = New selling price of the item (when initially there
18 days is loss) = (455 + 7x) = ₹ 665
Case 2: When A is less efficient than B. Minimum selling price of the item, when it is sold
Ratio of efficiency of time taken by A to B is 3: 5. for ₹ y more = 665 + 210 = ₹ 875
Let time taken A and B alone to finish the work is Minimum profit percent =
‘5x’ days and ‘3x’ days respectively. Which means profit percent will be more than
According to the question: 25%.
Hence, relationship between Quantity I and
Quantity II cannot be determined.

x=6
Direction (17-20):
Time taken by A alone to finish the work = 5x =
Let average age of girls in colleges B and C is ‘x’
30 days
years each.
Hence, Quantity I ≤ Quantity II
Average age of boys in college B = (x + 24)
years
16) Answer: E
Average age of boys in college C = (x + 20)
Cost price of the item = ₹ 700
years
Marked price of the item = (100 + x) % of 700 =
College B:
(700 + 7x)
By the rule of allegation:
Selling price of the item = (700 + 7x) – 245 =
(455 + 7x)
Quantity I:
Case 1: When there is profit on the item (CP <
SP).
(455 + 7x) – 700 = 35
a(-x – 4) = a(x – 20) + (2x – 40)
7x = 280
40 – 2x = a[(x – 20) – (-x - 4)]
x = 40
40 – 2x = a(2x – 16)
Case 2: When there is loss on the item (CP >
SP).
700 – (455 + 7x) = 35 College C:
7x = 210 By the rule of allegation:
x = 30
Possible values of ‘x’ = 30 and 40
Quantity II:

Click Here For Bundle PDF Course | support@guidely.in Page 14 of 18


SBI Clerk & RRB PO Mains PDF Course 2023
Quantitative Aptitude Day - 27 (Eng)

Average age of all the boys of college A = 28


years
Average age of all the girls of college A = 28 – (b
– 3) = (31 – b) years
College A:
a(4 – x) + 2(4 – x) = a(x – 24) –2(x – 24)
a(4 – x) + 8 – 2x = a(x – 24) – 2x + 48
8 – 2x + 2x – 48 = a(x – 24 – 4 + x)
40 = a(28 – 2x)

18 – 3b = -12
From (1) and (2):
3b = 30
b = 10
20 – x) (14 – x) = 20 (x – 8) Average age of all the girls of college A = (31 –
280 – 34x + x2 = 20x – 160 b) = 21 years
x2 – 54x + 440 = 0 Difference between average age of all the boys
After solving: and girls of college D = 2b = 20 years
x = 44 and 10 Difference between average age of all the boys
When x = 44, a = -2/3 [Invalid] and girls of college E = (b + 5) = 15 years
When x = 10, a = 5 Let average age of all the boys of college D and
Average age of all the boys of college B = (x + all the girls of college E is ‘y’ each.
24) = 34 years Average age of all the girls of college D = (y +
Average age of all the girls of college B = 20) years
Average age of all the girls of college C x = 10 Average age of all the boys of college E = (y +
years 15) years
Average age of all the boys of college C = (x + College D:
20) = 30 years
Ratio of boys to girls in college A = (a – 1): (a –
2) = 4: 3
Ratio of boys to girls in college B = a: (a + 2) = 5:
7
Ratio of boys to girls in college C = (a + 2): (a –
2) = 7: 3
c(y + 2.5) – 8(y + 2.5) = c(17.5-y)

Click Here For Bundle PDF Course | support@guidely.in Page 15 of 18


SBI Clerk & RRB PO Mains PDF Course 2023
Quantitative Aptitude Day - 27 (Eng)

c(y + 2.5) – c(17.5 – y) = 8(y + 2.5) Average age of all the girls of college D = (y +
c(y+2.5-17.5+y) = 8(y + 2.5) 20) = 30 years
c(2y-15) =8(y+2.5) Average age of all the boys of college E = (y +
15) = 25 years
Average age of all the girls of college E = y = 10
College E:
years

(y-16)(c+10) = c(1-y)
c(y-16) + 10(y-16) = c(1-y)
c(y-16)- c(1-y) = -10(y-16) 17) Answer: B

c(y-16-1+y) = -10(y-16) Let number of boys and girls in college A are 4m

c(2y-17) = -10(y-16) and 3m respectively.

c= -10(y-16)/(2y-17) Number of boys in college E =


Number of girls in college E =
Total students in colleges A and E together =
From (3) and (4):
(4m + 3m) + ( ) = 10m
Sum of ages of all the students of college A =
7m * 25 = 175m
2 (2y + 5) (17 – 2y) = 5 (y – 16) (2y – 15) Sum of ages of all the students of college E =
18y2 – 283y + 1030 = 0 3m * 16 = 48m
y = 103/18 and 10 Sum of ages of all the students of colleges A and
When y = 103/18, then c = -37/2 [Invalid] E together = 175m + 48m = 223m
When y = 10, then c = 20 Average of ages of all the students of colleges A
Ratio of boys to girls in college D = = 10: and E together =

6 = 5: 3 P = 223
Ratio of boys to girls in college E = c: (c + 10) = Factors of 223 = 1, 223
20: 30 = 2: 3 Hence, total factors of P are 2.
Average age of all the boys of college D = 10
years 18) Answer: E

Click Here For Bundle PDF Course | support@guidely.in Page 16 of 18


SBI Clerk & RRB PO Mains PDF Course 2023
Quantitative Aptitude Day - 27 (Eng)

Let number of boys and girls in college B is 5m of newly joined boys in college D must be either
and 7m respectively. 12 or 24 or 36 or ……..
According to the question: Total newly joined students are 20 which mean
total newly joined boys must be 12.

5m + 15 = 7m + 5 Newly joined girls in college D = 20 – 12 = 8

2m = 10 Required ratio = (50 + 12): (30 + 8)

m=5 = 62: 38

Total students (including newly joined) in college = 31: 19

D = 5m + 7m + (15 + 5) = 60 + 20 = 80
Average age of all the newly joined boys = a + c 20) Answer: E

– 1 = 5 + 20 – 1 = 24 years Let number of boys and girls in college A is 4m

Average age of all the newly joined girls = b – 2 and 3m respectively.

= 10 – 2 = 8 years Sum of ages of all the boys of college A = 4m *

Sum of ages of all the students (including newly 28 = 112m

joined) of college D = 60 * 20 + 15 * 24 + 5 * 8 = Sum of ages of all the girls of college A = 3m *

1200 + 360 + 40 = 1600 years 21 = 63m

Required average age = According to the question:


112m – 63m = 245
= 20 years
m=5
Total students in college A = d = 4m + 3m = 7m
19) Answer: A
d = 35
Let number of boys and girls in college D is 5m
Values of a, b, c, and d are 5, 10, 20, and 35
and 3m respectively.
respectively.
According to the question:
Series: 5, 10, 20, 35
(5m + 3m) * 17.5 + 20 * 27 = (5m + 3m + 20) *
Pattern in the series:
(17.5 + 1.9)
5 + 5 = 10
140m + 540 = 155.2m + 388
10 + 10 = 20
15.2m = 152
20 + 15 = 35
m = 10
35 + 20 = 55
Initial number of boys and girls in college D is 50
55 + 25 = 80
and 30 respectively.
80 + 30 = 110
Since number of newly joined boys in college D
110 + 35 = 145
is multiple of both 3 and 4. Which means number
5th term of the series = 55

Click Here For Bundle PDF Course | support@guidely.in Page 17 of 18


SBI Clerk & RRB PO Mains PDF Course 2023
Quantitative Aptitude Day - 27 (Eng)

8th term of the series = 145


Required percent =

Click Here For Bundle PDF Course | support@guidely.in Page 18 of 18


SBI Clerk & RRB PO Mains PDF Course 2023
ENGLISH Day - 27

English Language

Directions (1-5): In each of the questions given (I) (II)


below a sentence is given with two blanks in A. extensive D. disorganize
each. Corresponding to each question two
columns are given with three words in each B. extended E. formulation

column. You are required to choose the correct


C. small F. formulate
combination of words from the two columns that
a) A-E
will perfectly fit into the blanks to make the
b) A-F
sentence contextually correct and meaningful.
c) B-D
1) The Karnataka government’s __________(I) to
d) C-D
convert the promised Anna Bhagya scheme to a
e) C-F
direct benefit transfer temporarily has brought
into focus the limits of a State government’s
3) GST Compensation cess levies have been
policy intervention on a _____(II) matter such as
extended _____(I)at least March 2026, ______(II)
food security.
of the initial five-year tenure, due to the transitory
(I) (II)
shock of COVID-19 lockdowns on revenues.
A. confusion D. crucial
(I) (II)

B. decision E. important A. from D. despite

C. deciding F. unimportant B. still E. instead

a) A-D
C. till F. in spite
b) B-E
a) A-D
c) C-D
b) A-F
d) B-D
c) B-E
e) C-F
d) C-E
e) C-F
2)Observations of our _______(I) fieldwork and
discussions with migrant workers of eight districts
4) The committee went on to note that the
of Kerala, interactions with policy-makers, trade
immediate next focus of administrations should
unionists, have laid bare the key aspects to
be to ______(I) zero deaths due to hazardous
_________(II) a rights-based and inclusive policy.
cleaning of sewers and septic tanks, for which an

Click Here For Bundle PDF Course | support@guidely.in Page 1 of 11


SBI Clerk & RRB PO Mains PDF Course 2023
ENGLISH Day - 27

integrated scheme ‘the NAMASTE scheme’ has which of the sentences have incorrectly used the
been ________(II). highlighted word, and mark that as your answer.
(I) (II) 6)
I. The artwork's vibrant colours and intricate
A. ensure D. created
details make it incredibly appealing to viewers.
B. insure E. launched II. The movie preview gave us a tantalizing
glimpse into the thrilling action and captivating
C. prohibit F. start
storyline of the upcoming film.
a) A-E
III. The government decided to banned the use
b) A-F
of single-use plastic bags in order to reduce
c) B-D
environmental pollution.
d) B-F
a) Only I
e) C-E
b) Only II
c) Only III
5) The flyover was _____(I) over 15 years ago,
d) Both I and III
which will be remembered when the scaffolding
e) All are correct
collapsed and killed a techie, and
_______(II)several others in September 2007.
7)
(I) (II)
I. Despite facing numerous obstacles, she
A. build D. saved remained persistent and yielded impressive
results in her academic pursuits.
B. constructing E. damaged
II. John sought tutor to improve his
C. built F. injured understanding of calculus and boost his grades

a) A-D in the subject.

b) A-F III. As children grow older, they often outgrow

c) B-E their clothes and need to buy new ones .

d) C-E a) Only I

e) C-F b) Only II
c) Only III

Directions (6-9) : In the following given d) Both II and III

questions, three sentences with an highlighted e) All are correct

word have been given. You are required to check


8)

Click Here For Bundle PDF Course | support@guidely.in Page 2 of 11


SBI Clerk & RRB PO Mains PDF Course 2023
ENGLISH Day - 27

I. Not wearing a helmet while riding a bicycle can from the three statements given while implying
pose a significant hazard to your safety. the same meaning as expressed in the
II. The studious student spent hours each day in statement sentences.
the library, diligently studying and preparing for 10)
exams. 1. she always disagreed with his decision no
III. The notorious kid in the neighbourhood was matter if it was right or wrong,
known for his mischievous pranks and constant 2. she did not do it intentionally
troublemaking. 3. it was purely a sign of affection and care.
a) Only I I) since, because
b) Only II II) although, as
c) Only III III) while, incase
d) Both I and II IV) owing to, hence
e) All are correct a) I
b) II
9) c) III
I. The renowned chef is repute for his culinary d) IV
expertise and is often sought after by food e) None of these
enthusiasts from around the world.
II. The elaborate questions asked by the NFHS 11)
will provide valuable specifics on the lives of the 1. of the Development having experienced
disabled. android developers,
III. Lately, local contractors with political 2. the team couldn’t reach production on time
connections have thrived thanks to the vast 3. the technical glitches related to the server and
resources that the State spends. the cloud tool.
a) Only I I) despite, as
b) Only II II) in spite, therefore
c) Both I and II III) in spite, due to
d) Both II and III IV) instead , henceforth
e) All are correct a) I
b) II
Directions (10-13) : Select the right option of c) III
phrases/connectors out of given four (I), (II), (III) d) IV
and (IV) which can be used in the beginning (to e) None of these
join the statements) to form a single sentence

Click Here For Bundle PDF Course | support@guidely.in Page 3 of 11


SBI Clerk & RRB PO Mains PDF Course 2023
ENGLISH Day - 27

12) India’s tryst with the Goods and Services Tax


1. she being a single mother has to concentrate (GST), launched at a special midnight Parliament
on her career and session with unusual fanfare, completed six
2. she needs to take care of her children years this month. Marking the occasion, Finance
3. they are too young to manage things on their Minister Nirmala Sitharaman emphasised that
own. the GST has moved the country towards
I) on the other hand, as a_______(A)market from a situation where each
II) although, even though State mandated different indirect tax structures
III) since, therefore and procedures, while inter-State borders were
IV) However, although marked by bottlenecked check posts that added
a) I to logistics costs and subtracted from Indian
b) II goods’ competitiveness.Introduced soon after the
c) III informal(1) shock, the GST was viewed(2) as
d) IV another disruptor for the demonetisation(3)
e) None of these economy and its initial technical, structural and
procedural challenges(4) took a while to sort out
13) (B). That all businesses with annual turnover of
1. The lead is not at all satisfied with Revant’s ₹ 5 crore will have to generate e-invoices
work starting this August, and that there has been no
2. I will talk to him about this first thing after the ostensible pushback from smaller businesses
meeting over this, indicates that firms have gradually
3. this will affect his appraisal. embraced the change. The Revenue
I) and, by comparison Department’s crackdown on fake invoicing and
II) despite, nevertheless other techniques deployed by tax evaders may
III) hence, though ______(C)the few outliers to fall in line
IV) since, yet too.Tightening compliance and the post-
a) I pandemic rebound in economic activity have
b) II helped improve revenues from the GST, which
c) III Ms. Sitharaman had suggested were
d) IV underwhelming as of late 2021 when the Council
e) None of these set up a ministerial group to rationalise the
unwieldy multiple rate structure and enhance tax
Directions (14-19) : Read the following passage inflows. This June, GST revenues crossed ₹ 1.6
carefully and answer the questions that follow. lakh crore, only the fourth such occasion in its 72

Click Here For Bundle PDF Course | support@guidely.in Page 4 of 11


SBI Clerk & RRB PO Mains PDF Course 2023
ENGLISH Day - 27

months’ existence, lifting the average collections 15) Swap the words (1), (2), (3) and (4) in
in the first quarter of this year to nearly ₹ 1.7 lakh sentence (B) to make the sentence
crore — a healthy 12% over last year’s kitty. The grammatically and contextually correct and
recent revenue buoyancy, even if it may face a meaningful, ONLY IF NECESSARY.
blip if consumption growth falters among(D)a a) 1-3
slowing global economy, bodes well for States b) 2-4
that were worried about their fiscal capacity after c) 1-4
five years of assured revenues through GST d) 2-3
compensation expired last July. For taxpayers e) No changes required
and consumers, however, much remains to be
done till the GST can be considered a Good, 16) Which of the following words would fit in the
Simple Tax. GST Compensation cess levies blank (C) of the passage? And the same word
have been extended till at least March 2026, should fit in the blank in the sentence given
instead of the initial five-year tenure, due to the below.
transitory shock of COVID-19 lockdowns on The teacher encouraged creativity in the
revenues. Dispute resolution remains a pain classroom and did not ________the students to
point for industry, with GST appellate tribunals follow a specific artistic style.
still not set up (E). There is no road map in sight a) pull
on the rate rationalisation exercise or the b) ask
inclusion of excluded items such as electricity, c) allowed
petroleum and real estate, without which the d) compel
efficiency gains from the GST stand e) let
(F)constricted. The GST Council needs to meet
more often and turn its to-do list into a must-do 17) Which of the following words would replace
list expeditiously. the word (D) in the passage ?
14) Which of the following words would fit in the a) around
blank (A) of the passage ? b) between
a) separated c) amid
b) unified d) along
c) disunited e) No replacement required
d) combination
e) None of these 18) Which among the following convey(s) an
exact meaning as the Statement (E) given in
italics ?

Click Here For Bundle PDF Course | support@guidely.in Page 5 of 11


SBI Clerk & RRB PO Mains PDF Course 2023
ENGLISH Day - 27

I. The resolution of disputes remain unsolved and c) Only III


because of this the GST appellate tribunals d) Both I and III
cannot be set up. e) Both II and III
II. The industries find it really hard and difficult to
solve the disputes due to lack of the GST 19) Which of the following words would replace
appellate tribunals. the word (F) in the passage ?
III. Resolution of disputes remains difficult for the a) remind
industries as the GST appellate tribunals have b) remain
not yet been set up. c) stay
a) Only I d) look
b) Only II e) No replacement required
Click Here to Get the Detailed Video Solution for the above given Questions
Or Scan the QR Code to Get the Detailed Video Solutions

Answer Key with Explanation

1) Answer: D because it is inappropriate adds no context to


Decision and crucial are the correct words that fit the given sentence)
the given two blanks in an appropriate way Second blank - intervention on a ______
making the sentence grammatically and (eliminate unimportant and important because
contextually correct and meaningful. the preposition used here is ‘a’ - to use words
Decision(noun) - to decide that starts with a vowel or sound of a vowel ‘an’
Crucial - very important must be used) So the left out option is ‘crucial’
The first blank needs a noun - confusion and which turns to be the right word to fit the given
decision are the nouns from the given options. blank.
The decision has led to focus on the limits of the The sentence: The Karnataka government’s
state government.(confusion cannot fit the blank decision to convert the promised Anna Bhagya

Click Here For Bundle PDF Course | support@guidely.in Page 6 of 11


SBI Clerk & RRB PO Mains PDF Course 2023
ENGLISH Day - 27

scheme to a direct benefit transfer temporarily Till and despite are the correct words that fit the
has brought into focus the limits of a State given two blanks making the sentence
government’s policy intervention on a crucial meaningful.
matter such as food security. First blank - extended to/till a date(correct), the
deadline is in future so ‘from’ is wrong and ‘still’
2) Answer: B has a completely different meaning ‘continuing’.
Extensive and formulate are the correct words Second blank - Instead of the 5 year tenure the
that fit the blanks given. deadline has been extended, in spite of and
Extensive - large in area/amount despite of must be followed by a reason or an
Formulate - to organise/to prepare excuse.
First blank - observations of our extensive The sentence: GST Compensation cess levies
fieldwork(covering a wide area). have been extended till at least March 2026,
Small - will be inappropriate instead of the initial five-year tenure, due to the
Extended - to extend something (extensive is transitory shock of COVID-19 lockdowns on
more suitable to the context of the given revenues.
sentence) 4) Answer: A
Second blank - the fieldwork and discussion Ensure and launched are the correct words to fill
have helped to lay the key aspects to create a the blanks in the given sentence.
new policy. Ensure - to make sure
The word which has a similar meaning to ‘create’ First blank - ensure zero deaths(is the right pick),
is ‘formulate’.(formulate is the right word which insure - to buy/sell insurance, prohibit - to stop
fits the blank in a perfect way - both entry.
grammatically and contextually correct) Second blank - the scheme has been launched
The sentence: Observations of our extensive to the public. ‘Start and created’ are
fieldwork and discussions with migrant workers inappropriate and cannot be a best fit.
of eight districts of Kerala, interactions with The sentence : The committee went on to note
policy-makers, trade unionists, have laid bare that the immediate next focus of administrations
the key aspects to formulate a rights-based and should be to ensure zero deaths due to
inclusive policy. hazardous cleaning of sewers and septic tanks,
for which an integrated scheme ‘the NAMASTE
3) Answer: D scheme’ has been launched.

5) Answer: E

Click Here For Bundle PDF Course | support@guidely.in Page 7 of 11


SBI Clerk & RRB PO Mains PDF Course 2023
ENGLISH Day - 27

Built and injured are the correct words that fist Replace tutor with tutoring to make the sentence
both the blanks in the sentence. sound correct.
First blank - the flyover was built 15 years ago - John sought tutoring to improve his
past tense. (built - past tense), other two words understanding of calculus and boost his grades
are not in past tense and hence can be in the subject.
eliminated. Yield - to produce
Second blank - saved is opposite in meaning to Outgrow - to become too old big for something
the context of the sentence, ‘damage’ is usually Tutoring - academic support
related to non-living things. Injured is the correct The other two sentences I and III have used the
word that can be used to fill in the given blank. highlighted word properly and hence are correct.
The sentence : The flyover was built over 15
years ago, which will be remembered when the 8) Answer: E
scaffolding collapsed and killed a techie, and All the given sentences I, II and III have used the
injured several others in September 2007. highlighted word correctly.

6) Answer: C Hazard - danger/risk


The word ‘banned’ is inappropriate. Replace it Studious - studying most of the time
with ‘ban’ to make the sentence correct and error Notorious - naughty
free. All the words are used correctly in accordance
Appealing - attractive with their meaning.
Glimpse - a quick view
Ban - not allow something 9) Answer: A
The government decided to ban the use of Replace ‘repute’ with ‘reputed’ to make the word
single-use plastic bags in order to reduce usage correct and appropriate.
environmental pollution. The renowned chef is reputed for his culinary
The sentences I and II are correct and expertise and is often sought after by food
meaningful - have used the highlighted word in a enthusiasts from around the world.
proper way. Repute - to have an opinion
Elaborate - in detail
7) Answer: B Thrive - to grow vigorously
Tutor is inappropriately used in the given The sentences II and III have used the
sentence II. highlighted word in an appropriate way.
He sought a tutor/he sought tutoring.

Click Here For Bundle PDF Course | support@guidely.in Page 8 of 11


SBI Clerk & RRB PO Mains PDF Course 2023
ENGLISH Day - 27

10) Answer: B On the other hand - doing two thing


Although she always disagreed with his decision simultaneously
no matter if it was right or wrong, it was not She is a single mother and has to deal with both
intentional as it was purely a sign of affection her career and children as the children are too
and care. young to take care of themselves. - meaning of
Meaning - Even though she disagrees with him the statements given
most of the time, it was not intentional and just a She being a single mother has to concentrate on
pure sign of affection. her career and on the other hand she needs to
The other connectors do not help to join the take care of her children as they are too young
three statements to form a meaningful and to manage things on their own.
correct sentence. The other given connectors do not fall in place to
combine the statements in a correct way.Hence
11) Answer: C we go for option a.

In spite and due to are the right connectors to


combine the given statements and form a single 13) Answer: E
meaningful sentence. None of the given set of connectors can be used
Even though the team had good developers it to connect/combine the given statements to form
could not move to production because of some a meaningful sentence.
technical glitch. - this is the meaning conveyed. The lead is not satisfied with Revant’s work, so I
In spite of the Development having experienced will talk to him regarding this after the meeting
android developers,the team couldn’t reach because this will actually affect his appraisal. -
production on time due to the technical glitches meaning of the statements
related to the server and the cloud tool. The correct pair of connectors that’s needed is -
The other connectors are either inappropriate or and, because
do not connect the statements in a proper way The lead is not at all satisfied with Revant’s work
so as to convey a meaning. and I will talk to him about this first thing after the
meeting because this will affect his appraisal.
12) Answer: A
Option a - I has the correct pair of connectors to 14) Answer: B
combine the statements without altering the Unified is the correct word that fits the blank (A)
original meaning. to make the sentence contextually correct.

Click Here For Bundle PDF Course | support@guidely.in Page 9 of 11


SBI Clerk & RRB PO Mains PDF Course 2023
ENGLISH Day - 27

Finance Minister Nirmala Sitharaman evaders may compel the few outliers to fall in
emphasised that the GST has moved the line too.
country towards a unified market from a situation The teacher encouraged creativity in the
where each State mandated different indirect tax classroom but did not compel the students to
structures and procedures. follow a specific artistic style.
Each state had different taxes, procedures and
structure which is now streamlined and made as 17) Answer: C
a single market. Amid is the correct word to replace among.
Single - unified Amid - in the middle of
Separated and disunited have different Between,along and around does not fit the given
meaning(opposite to united/unified) blank
Combination - inappropriate and not a good pick Something is happening in the midst of/in the
middle of an economic slowdown - so amid is
15) Answer: A the right word.
Swap the words ‘informal’ and ‘demonetisation’ The recent revenue buoyancy, even if it may
to make the sentence correct and meaningful. face a blip if consumption growth falters amid a
Introduced soon after the demonetisation shock, slowing global economy, bodes well for States
the GST was viewed as another disruptor for the that were worried about their fiscal capacity after
informal economy and its initial technical, five years of assured revenues through GST
structural and procedural challenges took a while compensation expired last July.
to sort out.
Words 3 and 4 are already placed right and 18) Answer: E
hence need no replacement. Both statements II and III convey the exact
meaning of the highlighted sentence in italics.
16) Answer: D So, we choose option e as our answer to this
Compel is the right word that can be used to fill question.
both the given blanks. Sentence I is wrong, unsolved disputes is not the
Compel - to force somebody reason for not setting up GST appellate
The other given words are either inappropriate or tribunals..
fit only a single blank out of the two.
The Revenue Department’s crackdown on fake 19) Answer: B
invoicing and other techniques deployed by tax Remain is the correct word to replace the word
stand.

Click Here For Bundle PDF Course | support@guidely.in Page 10 of 11


SBI Clerk & RRB PO Mains PDF Course 2023
ENGLISH Day - 27

Remain - to stay(in this context) There is no road map in sight on the rate
Remind - to help someone remember something rationalisation exercise or the inclusion of
Remain is the correct word that makes the excluded items such as electricity, petroleum
sentence meaningful. and real estate, without which the efficiency
gains from the GST remain constricted.

Click Here For Bundle PDF Course | support@guidely.in Page 11 of 11


SBI Clerk & RRB PO Mains PDF Course 2023
Reasoning Ability Day - 28 (Eng)

Reasoning Ability
Directions (1-5): Study the following information a) SP
carefully and answer the given questions. b) ZM
12 persons – A, D, E, F, I, M, O, P, S, U, W and c) PI
Z are sitting in a two rectangular table, such that d) WO
one rectangular table is inscribed in another e) AI
rectangular table. Two persons are sitting on
longer sides of the table and one person sitting 2. Who among the following person sits third to
on the smaller sides of the table. Persons in the the right of P?
inner tables are facing away from the center and a) The one who sits opposite to Z
person sitting in the outer table faces towards the b) S
center. c) The one who faces W
Note: d) D
I. If it is given that A and B face each other, then e) The one who sits immediate left of the one
both are sitting at different table and if it is given who faces I
that A and B sit opposite to each other, then both
are sitting at the same table. 3. Who among the following person sits
II. The person whose name starts with a vowel immediate right of the one who sits opposite to I?
neither faces each other nor sits opposite to a) Z
each other. b) S
S, who does not sit adjacent to U, sits second to c) F
the left of the one who faces E. E, who sits on d) O
the shorter side of the table, sits opposite to the e) I
one who sits second to the right of M. M faces
the one who sits third to the right of F. The 4. What is the position of A with respect to the
number of persons sitting between F and S is one who faces S?
one less than the number of persons sitting a) Fourth to the left
between U and D, when counted from the right of b) Second to the left
D and F. The one who sits immediate right of D c) Second to the right
faces O. One person sits between A and the one d) Both a and c
who faces Z. W, who sits at the outer table, b) Both b and c
neither sits adjacent to A nor sits opposite to A.
1. Who among the following pair of persons sit at 5. If the persons facing each other interchange
the same table? their positions, then which of the following

Click Here For Bundle PDF Course | support@guidely.in Page 1 of 11


SBI Clerk & RRB PO Mains PDF Course 2023
Reasoning Ability Day - 28 (Eng)

statement is/are true with respect to the final a) Only I


arrangement? b) Only III
a) W sits immediate left of the one who faces S. c) Both II and III
b) Only one person sits between P and U, when d) Only II
counted from the left of P e) Both I and II
c) E and O sit opposite to each other
d) Both a and b 8. If in the given words, the odd positioned letters
e) Both b and c from the left end are replaced by the second
succeeding letter and the remaining letters are
6. If all the letters in the given words are replaced replaced by the third preceding letter as per the
by the fourth succeeding letter, then all the English alphabetical series, then all the letters
letters are arranged in reverse alphabetical order are arranged in alphabetical order from the right
from the left end and then the place value (as per end. Then which of the following word has the
alphabetical series) of the even positioned letters maximum number of letters between the letters
of each word (from the right end) is added. Then which is third from both the ends in the given
which of the following resultant gives the second words (as per the English alphabetical series)?
highest value? (Consider the previous letter of A as Z)
I. COMPLICATED I. MEDITATION
II. GERMINATION II. JOURNALIST
III. AGRICULTURE III. HYDROMETER
a) Only III a) Only I
b) Both II and III b) Only II
c) Only II c) Both I and III
d) Only I d) Only III
e) Both I and II e) Both I and II

7. If one is added to the even digit of each 9. If all the digits are multiplied within the number
number and two is subtracted from the odd digit and then add all the resultant values within the
of each number, then all the digits are added row, then the final resultant of which of the
within the number. Then the resultant of which of following I, II, III will be multiple of 7?
the following number is the second lowest? I. 9431, 5873, 5236
I. 3985478256 II. 4961, 8275, 3916
II. 5293678524 III. 6198, 4281, 7543
III. 2964736528 a) Only I

Click Here For Bundle PDF Course | support@guidely.in Page 2 of 11


SBI Clerk & RRB PO Mains PDF Course 2023
Reasoning Ability Day - 28 (Eng)

b) Only III closed their bank accounts in the month which


c) Both I and III has an even number of days.
d) Both II and III Sarah closed the bank account four months
e) Only II before the one who closed immediately after
Ellis. The number of months after the month in
10. If the middle letter of each word in each row which Ellis closed the account is two less than
is taken, then which of the following row forms a the number of months before the month in which
meaningful word (Using each letter only once)? the one who closed immediately before or
I. Fruit, Retry, Score, Cubes, Acnes immediately after Mateo. There is a gap of four
II. Dream, Sings, Below, Elite, Catch months between the months in which Isla and
III. Scary, Dance, Flows, Actor, Blind Olivia closed the account. The number of months
a) Only I between the months in which Akira and Dante
b) Only II closed their account is one less than the number
c) Only III of months between the months in which Dante
d) Both I and II and the one who closed immediately after Isla.
e) Both II and III The number of months between the month in
which Gael closed the account and the one who
Directions (11-15): Study the following closed three months after Alden is an even
information carefully and answer the given number.
questions. 11. Alden closed the account in which of the
Nine persons – Alden, Isla, Dante, Olivia, Gael, following month?
Mateo, Ellis, Akira and Sarah closed their bank a) April
accounts in nine different months- January, b) June
March, April, June, July, August, September, c) September
November and December of the same year. d) November
Note: e) Can’t be determined
I. If a person's name either starts and ends with a
vowel or starts and ends with a consonant, then 12. If all the persons open the new account two
they closed their bank accounts in the month months after they closed their account, then in
which has an odd number of days. which of the following month does Gael, Dante
II. If a person's name either starts with a vowel and Sarah open their new account?
and ends with a consonant or starts with a a) March, August, November
consonant and ends with a vowel, then they b) May, June, October
c) May, July, December

Click Here For Bundle PDF Course | support@guidely.in Page 3 of 11


SBI Clerk & RRB PO Mains PDF Course 2023
Reasoning Ability Day - 28 (Eng)

d) June, October, March d) Rs.41000


e) None of these e) Rs.43000

13. If Gael closed immediate next month of Akira Directions (16-20): Study the following
and Ellis closed immediate next month of Alden, information carefully and answer the given
then how many months are between the month questions.
in which Gael closed the account and the one In a building there are certain number of floors
who closed two months after Ellis? and certain number of persons living in the floor,
a) Five where the ground floor is numbered as one and
b) Seven the floor immediately above it is numbered as
c) Ten two and so on.
d) Nine Note-I: Each floor has two type of flats viz., Flat-
e) Eight A and Flat-B, where Flat A is to the west of Flat
B.
14. Which of the following statement is/are true Note-II: Flat B of floor 2 is immediately above
with respect to the final arrangement? Flat B of floor 1 and immediately below Flat B of
a) Dante and Mateo closed their bank account in floor 3 and so on. Similarly, Flat A of floor 2 is
Consecutive months immediately above Flat A of floor 1 and
b) Only three persons closed their bank account immediately below Flat A of floor 3 and so on.
between Akira and the one who closed Note-III: The area of each flat on each floor is
immediately before Sarah same.
c) Only two months are between the month in B lives six floors above I, where both of them
which Mateo and Gael closed their bank account. neither lives in the same type of flat as J nor lives
d) Both a and b on same floor as J. Only one floor is between I
e) Both b and c and C, who lives two floors above the flat of J.
Only five floors are between J and D, where both
15. If the bank balance of Akira and Gael is are living in different type of flat. D lives
Rs.12000 and Rs.13000 respectively such that immediately below K. No one lives to the east of
the bank balance of each person is increased by G, who lives five floors above the flat of K. The
Rs.1000 from Akira to Olivia, then what is the number of floors between G and B is two less
bank balance of Ellis, Isla and Mateo? than the number of floors above G. The number
a) Rs.50000 of floors above K is one more than the number of
b) Rs.40000 floors below K. F lives eight floors below the flat
c) Rs.35000 of A, where both of them live on a prime

Click Here For Bundle PDF Course | support@guidely.in Page 4 of 11


SBI Clerk & RRB PO Mains PDF Course 2023
Reasoning Ability Day - 28 (Eng)

numbered floor. L lives four floors above F, b) Three


where both are living in different type of flat. c) Five
16. If Z lives exactly between A and L, Y lives d) Four
exactly between L and F, then how many known e) More than six
persons are living in flat A?
a) Four 19. As many floors between B and __ as
b) Five between C and ___ respectively
c) Six a) A, D
d) Seven b) I, K
e) Eight c) J, G
d) L, G
17. Who among the following person lives three e) K, J
floors above R, who lives five floors above I?
a) B 20. If the rent of the persons living on even
b) A numbered floor is Rs.9000 and odd numbered
c) The one who lives three floors above L floor is Rs.11000, then what is the total rent paid
d) The one who lives six floors above the flat of D by L, K and Q, who lives immediately above G?
e) Both b and d a) Rs.33000
b) Rs.27000
18. The number of floors below I is three less c) Rs.31000
than the number of floors above P, then how d) Rs.29000
many known persons live below the flat of P? e) Can’t be determined
a) Six
Click Here to Get the Detailed Video Solution for the above given Questions
Or Scan the QR Code to Get the Detailed Video Solutions

Answer Key with Explanation

Click Here For Bundle PDF Course | support@guidely.in Page 5 of 11


SBI Clerk & RRB PO Mains PDF Course 2023
Reasoning Ability Day - 28 (Eng)

Directions (1-5)
1. Answer: D
2. Answer: E
3. Answer: C
4. Answer: D
5. Answer: B
Final arrangement:

We have,
 S, who does not sit adjacent to U, sits
second to the left of the one who faces E.
Again we have,
 E, who sits on the shorter side of the
 The number of persons sitting between F
table, sits opposite to the one who sits
and S is one less than the number of
second to the right of M.
persons sitting between U and D, when
 M faces the one who sits third to the right
counted from the right of D and F.
of F.
 The one who sits immediate right of D
From the above conditions, we have two
faces O.
possibilities:

Click Here For Bundle PDF Course | support@guidely.in Page 6 of 11


SBI Clerk & RRB PO Mains PDF Course 2023
Reasoning Ability Day - 28 (Eng)

Again we have,
 One person sits between A and the one
6. Answer: C
who faces Z.
I. COMPLICATED -> GSQTPMGEXIH -
 W, who sits at the outer table, neither sits
> XTSQPMIHGGE -> 65
adjacent to A nor sits opposite to A.
II. GERMINATION -> KIVQMREXMSR -
After applying above conditions, case 1 gets
> XVSRRQMMKIE -> 79
eliminated, because W and A sit adjacent to
III. AGRICULTURE -> EKVMGYPXYVI ->
each other. Thus, case 2 gives the final
YYXVVPMKIGE -> 81
arrangement.

7. Answer: B
I. 3985478256 -> 1793559337 -> 52
II. 5293678524 -> 3371759335 -> 46
III. 2964736528 -> 3775517339 ->50

8. Answer: A

Click Here For Bundle PDF Course | support@guidely.in Page 7 of 11


SBI Clerk & RRB PO Mains PDF Course 2023
Reasoning Ability Day - 28 (Eng)

I. MEDITATION -> OBFFVXVFQK -


> XVVQOKFFFB ->15
II. JOURNALIST -> LLWOPXNFUQ -
> XWUQPONLLF -> 8
III. HYDROMETER -> JVFOQJGQGO ->
VQQOOJJGGF -> 9

9. Answer: E
I. 9431, 5873, 5236 -> 108 + 840 + 180 ->
1128
II. 4961, 8275, 3916 -> 216 + 560 + 162 ->
938 We have,
III. 6198, 4281, 7543 -> 432 + 64 +  Sarah closed the bank account four
420 -> 916 months before the one who closed
immediately after Ellis.
10. Answer: B  The number of months after the month in
I. Fruit, Retry, Score, Cubes, Acnes -> which Ellis closed the account is two less
UTOBN than the number of months before the
II. Dream, Sings, Below, Elite, Catch -> month in which the one who closed
ENLIT (Intel, Inlet, Elint) immediately before or immediately after
III. Scary, Dance, Flows, Actor, Blind -> Mateo.
ANOTI From above conditions, we have three
possibilities:
Directions (11-15)
11. Answer: C
12. Answer: B
13. Answer: D
14. Answer: E
15. Answer: A
Final arrangement:

Again we have,

Click Here For Bundle PDF Course | support@guidely.in Page 8 of 11


SBI Clerk & RRB PO Mains PDF Course 2023
Reasoning Ability Day - 28 (Eng)

 There is a gap of four months between


the month in which Isla and Olivia closed
the account.
 The number of months between the
months in which Akira and Dante closed
their account is one less than the number
of months between the months in which
Dante and the one who closed
immediately after Isla.

Directions (16-20):
16. Answer: C
17. Answer: E
18. Answer: B
19. Answer: C
Again we have, 20. Answer: D
 The number of months between the Final arrangement:
month in which Gael closed the account
and the one who closed three months
after Alden is an even number.
While applying above condition, case 1 and 2
gets eliminated, because odd number of months
is between the number of months in which Gael
and the one who close three months after Alden
closes the account. Thus, case 3 gives the final
arrangement.

Click Here For Bundle PDF Course | support@guidely.in Page 9 of 11


SBI Clerk & RRB PO Mains PDF Course 2023
Reasoning Ability Day - 28 (Eng)

We have,
 B lives six floors above I, where both of
them neither lives in the same type of flat
as J nor lives on same floor as J.
 Only one floor is between I and C, who
lives two floors above the flat of J.
From the above conditions, we have two
possibilities:

Again we have,
Again we have,  The number of floors between G and B is
 Only five floors are between J and D, two less than the number of floors above
where both are living in different type of G.
flat.  The number of floors above K is one more
 D lives immediately below K. than the number of floors below K.
 No one lives to the east of G, who lives  F lives eight floors below the flat of A,
five floors above the flat of K. where both of them live on a prime
numbered floor.
 L lives four floors above F, where both are
living in different type of flat.
From the above conditions, case 1 and case 2a
get eliminated because in case 1 there is no
possibility to place A and F and in case 2a the
number of floors above K is not one more than

Click Here For Bundle PDF Course | support@guidely.in Page 10 of 11


SBI Clerk & RRB PO Mains PDF Course 2023
Reasoning Ability Day - 28 (Eng)

the number of floors below K, hence case-2


shows the final arrangement.

Click Here For Bundle PDF Course | support@guidely.in Page 11 of 11


SBI Clerk & RRB PO Mains PDF Course 2023
Quantitative Aptitude Day - 28 (Eng)

Quantitative Aptitude

Directions (01 - 05): Study the following information carefully and answer the questions given below.
The line graphs shows the entry fees (Rs) of adults and children in five different tourist spots. The table
shows the total number of visitors [adult + child] and the total revenue earned by each spot from child
visitors.

1) Find the difference between the total revenue d) Rs.4800


earned by spots A and B from adult visitors and e) Rs.5200
the total revenue earned by spots C and D from
adult visitors? 2) Number of adult visitors in spot F is 30% more
a) Rs.4400 than the same as C and the number of child
b) Rs.4500 visitors in spot F is 25 more than the same as D.
c) Rs.4600 If entry fees are for adults and child in spot F is

Click Here For Bundle PDF Course | support@guidely.in Page 1 of 10


SBI Clerk & RRB PO Mains PDF Course 2023
Quantitative Aptitude Day - 28 (Eng)

Rs.30 and Rs.15 respectively in spot F then find 4) I= The total number of adult visitors in spots A,
the total revenue earned by spot F? B, and C together
a) Rs.9250 M=Total number of child visitors in all spots
b) Rs.9750 together
c) Rs.9955 Find the relation between I and M?
d) Rs.9405 a) I-M=125
e) Rs.9455 b) I+M=920
c) I-M=140
3) Ratio of male and female adult visitors in d) I+2M=1240
spots D and E is 3:2 and 3:5 respectively. Find e) None of these
the ratio of the total number of male visitors in
spots D and E together and female visitors in 5) Find the percentage of child visitors in spots A
spots D and E together? and E together out of total visitors in spots A and
a) 2:5 E together?
b) 3:4 a) 31.5%
c) 1:5 b) 45.2%
d) 4:5 c) 83.5%
e) None of these d) 73.8%
e) None of these

Directions (06 - 10): Study the following information carefully and answer the questions given below.
The given Radar graph shows the number of 4-wheeler and the number of 2-wheeler vehicles parked in
the parking of five different malls. Each mall has a total of 300 people. In each 4-wheeler, four people
come to the mall and in each 2-wheeler, two persons come to the mall and the rest of the people come to
the mall using public transport.

Click Here For Bundle PDF Course | support@guidely.in Page 2 of 10


SBI Clerk & RRB PO Mains PDF Course 2023
Quantitative Aptitude Day - 28 (Eng)

6) In mall A, average age of people who come in c) 262


4-wheeler is 28 years, the average age of people d) 246
who come in 2-wheeler is 22 years and the e) 282
average age of people who come in public
transport is 25 years. Find the average age of all 8) Ratio of the number of males to females who
the people who come to mall A? come in mall D in 4-wheeler, 2-wheeler, and
a) 24.5 years public transport is 3:2,2:3, and 5:3 respectively.
b) 28.5 years Find the difference between the total number of
c) 26.2 years males and females who come to mall D?
d) 22.5 years a) 65
e) 25.5 years b) 60
c) 72
7) Number of people come by 4-wheelers to mall d) 86
F is 20% more than the same in mall B and the e) None of these
number of people come by 2-wheeler to mall F
is 20% more than the same in mall C. Find the 9) Ratio of the number of people coming by bus
total number of people come by public transport and train and the total number of people coming
into mall C and F if totally same people come to by public transport in malls A and C is 3:2 and
mall F? 2:3. Find the total number of people coming by
a) 225 train in malls A and C together if people using
b) 242 public transport is bus and train only?

Click Here For Bundle PDF Course | support@guidely.in Page 3 of 10


SBI Clerk & RRB PO Mains PDF Course 2023
Quantitative Aptitude Day - 28 (Eng)

a) 130 scored in the T20. The ratio of runs scored by


b) 120 batters A and B in ODI is 5:3.
c) 140 11) Total runs of batter D in three formats is
d) 150 2400. The ratio of runs scored in T20 of batters A
e) None of these and D is 9:8 and the ratio of runs scored in ODI
of batters B and D is 5:4. Find the runs scored of
10) Find the ratio between the total number of batter D in the test?
people who come by 4-wheeler together in malls a) 1420
A, C, and E and the total number of people who b) 1520
come by public transport together in malls A, C, c) 1240
and E? d) 1590
a) 14:11 e) 1260
b) 12:19
c) 17:11 12) Runs scored of batter A is increased by
d) 18:13 25%,20%, and 25% in T20, test, and ODI
e) None of these respectively in the year 2022 than in the year
2021. Find the total runs of batter A in the year
Directions (11 - 15): Study the following 2021?
information carefully and answer the questions a) 2420
given below. b) 2520
Three batters scored different runs in three c) 2460
formats - T20, test, and ODI in the year 2022. d) 2320
The total runs scored in T20 by three batters is e) 2210
2700. The ratio of runs scored by the batter A in
T20 and the test is 3:4. Runs scored in ODI by 13) Find the difference between the total runs
batter A is 16.66% less than the runs scored by scored by batter C and the total runs scored by
the same batter in T20. Ratio of runs scored by batter B?
batter B in T20 and Test is 3:4. Runs scored by a) 2400
batter B in T20 is 33.33% more than the run b) 2050
scored by the same batter in ODI. The run c) 2200
scored by C in the Test is 16.66% more than the d) 2520
runs scored by C in the T20. C scores double run e) None of these
in T20 than B in the same format. The runs
scored by C in ODI is 8.33% more than he

Click Here For Bundle PDF Course | support@guidely.in Page 4 of 10


SBI Clerk & RRB PO Mains PDF Course 2023
Quantitative Aptitude Day - 28 (Eng)

14) If Batter B played thirty T20 matches and 15) In a test match, there are two innings and
forty five ODI matches then find the sum of each batter bats in both innings. Batter C played
average runs scored by batter B in T20 and ODI a total of 10 test matches. Find the average runs
together? scored by batter C in each inning?
a) 30 a) 70
b) 12 b) 75
c) 22 c) 80
d) 25 d) 90
e) None of these e) None of these

Directions (16 - 20): Study the following information carefully and answer the questions given below.
The given tables show the cumulative percentage distribution of the total number of books
[math+English] sold in five shops and the percentage of math books sold in five shops out of the total
books sold in each shop. The number of math books sold in shop C is 144.

Click Here For Bundle PDF Course | support@guidely.in Page 5 of 10


SBI Clerk & RRB PO Mains PDF Course 2023
Quantitative Aptitude Day - 28 (Eng)

16) Price of each math book sold in shop D is b) 18


Rs.320 and the price of each English book sold c) 11
in shop D is Rs.280. Find the total revenue d) 16
earned by shop D by selling all books? e) None of these
a) Rs.54260
b) Rs.55420 19) I= Difference in the number of math and
c) Rs.52560 English books sold in shop C
d) Rs.57560 J= Difference in the number of math and English
e) Rs.54200 books sold in shop E
Find which of the following relation is true?
17) Ratio of the number of math books sold in a) 2I+J=264
shops B and F is 4:5 and the ratio of the number b) I+J=258
of English books sold in shops C and F is 4:7. c) J-I=110
Find the total number of books sold in shop F? d) J+5I=452
a) 220 e) None of these
b) 240
c) 260 20) M= Total number of books sold in shop B
d) 258 N= Total number of books English books sold in
e) 248 all shops together
Find the value M+N?
18) Find the difference between the number of a) 618
math books sold in shops A and B together and b) 628
the number of English books sold in shops D and c) 668
E together? d) 658
a) 12 e) None of these

Click Here For Bundle PDF Course | support@guidely.in Page 6 of 10


SBI Clerk & RRB PO Mains PDF Course 2023
Quantitative Aptitude Day - 28 (Eng)

Click Here to Get the Detailed Video Solution for the above given Questions
Or Scan the QR Code to Get the Detailed Video Solutions

Answer Key with Explanation

Directions (01 - 05): = [220*130/100]


The Number of child visits A is 500/10=50 *30+[30+25]*15=8580+825=Rs.9405
The Number of adult visits A is 230-50=180
So, revenue earned from adult visitors in A is 3) Answer: B
180*20=Rs.3600 The total number of adult male visitors in D and
So, similarly, we can calculate the value of the E are
others also. = 100*(3/5)+320*(3/8) =180
The total number of adult female visitors in D
and E are
=100*(2/5)+320*(5/8)=240
Required ratio = 180:240=3:4

4) Answer: E
I=180+120+220=520
M=50+60+70+30+180=390
None of the given options are true
1) Answer: C
Required difference =[8800+3000]-[3600+3600]= 5) Answer: A
Rs.4600 Total child visitors in A and E together are =
[50+180] =230
2) Answer: D Total visitors in A and E together are =
The total revenue earned by spot F is [230+500]=730

Click Here For Bundle PDF Course | support@guidely.in Page 7 of 10


SBI Clerk & RRB PO Mains PDF Course 2023
Quantitative Aptitude Day - 28 (Eng)

Required percentage = [230/730]*100=31.5% Required difference= 180-120=60

Directions (06 - 10): 9) Answer: A


The number of people coming in 4-wheeler to A The total number of people coming on the train
is 40*4=160 is [100*2/5+150*3/5]=40+90=130
The Number of people coming in 2-wheeler to A
is 20*2=40 10) Answer: A
The number of people coming by public Required ratio is =
transport is 300-160-40=100 [160+100+160]:[100+150+80]=420:330=14:11
Similarly, we can calculate others' values also.
Directions (11 - 15):
Let, runs scored by A in T20 be 6x and runs
scored by A in Test 8x.
Runs score by A in ODI is 6x*(100-
16.66)/100=5x
The runs scored by B in ODI are 3x
Runs scored by B in T20 is 3x*133.33/100=4x
6) Answer: C So, 6x+4x+4x*2=2700
The total age of all the people in mall A is Or, 18x=2700, x=2700/18=150
=160*28+40*22+100*25=7860 Runs scored by A in T20 is 6*150=900 and the
Average age is = [7860/300] =26.2 years runs scored by A in Test 8*150=1200.
Runs scored by A in ODI is 5*150=750
7) Answer: D Runs scored by B in ODI is 3*150=450
The number of people come by public transport Runs scored by B in T20 is 4*150=600
in F is Runs scored by B in the Test is 600*4/3=800
=300-120*(120/100) + 50*(120/100) =300-144- Runs scored by C in T20 is 600*2 = 1200
60=96 Runs scored by C in the Test is
Required value is =96+150=246 1200*116.66/100=1400
Runs scored by C in ODI is
8) Answer: B 1200*(108.33)/100=1300
The total number of male visitors in D is 11) Answer: C
=120*3/5+20*2/5+160*5/8=72+8+100=180 Runs scored by batter D in T20 is = 900*8/9=800
The total number of female visitors in D is
=120*2/5+20*3/5+160*3/8=48+12+60=120

Click Here For Bundle PDF Course | support@guidely.in Page 8 of 10


SBI Clerk & RRB PO Mains PDF Course 2023
Quantitative Aptitude Day - 28 (Eng)

The runs scored by batter D in ODI is 18%=12%,50%-30%=20%,65%-50%=15%, and


=450*4/5=360 100%-65%=35% respectively.
Runs scored by batter D in the test is =2400- The number of Math books sold in shop C is 144
800-360=1240 Number of total books sold in shop C is [144/60]
*100=240
12) Answer: D Total number of books sold in all shops together
The total runs scored by batters in 2021 is is = [240/20] *100=1200
= [900*100/125]+[1200*100/120]+[750*100/125]
= 720+1000+600=2320

13) Answer: B
The total runs to score by batter C is
=1200+1400+1300=3900
The total runs to score by batter B is
=600+800+450=1850 16) Answer: C
The total revenue earned by the shop D is
Required difference=3900-1850=2050
=54*320 +126*280=17280+35280=Rs.52560

14) Answer: A
Average runs score by batter B is = 17) Answer: D
The total number of books sold in F is =
[600/30+450/45]=30
72*5/4+96*7/4=90+168=258

15) Answer: A
The total number of innings played by batter C is 18) Answer: B
Required difference = [126+126]-[162+72]=18
= 10*2=20
So, the required average is=1400/20=70
19) Answer: A
I= Difference in number of math and English
Directions (16 - 20):
books sold in shop C=144-96=48
The cumulative percentage distribution of the
J= Difference in number of math and English
number of books sold in five shops A, B, C, D,
books sold in shop E=294-126=168
and E is 18%,30%,50%,65%, and 100%.
By substituting I and J values in option,
Percentage distribution of the number of books
sold in five shops A, B, C, D, and E is 18%,30%- 2I+J=264

20) Answer: A

Click Here For Bundle PDF Course | support@guidely.in Page 9 of 10


SBI Clerk & RRB PO Mains PDF Course 2023
Quantitative Aptitude Day - 28 (Eng)

M= Total number of books sold in shop B=144 =54+72+96+126+126=474


N= Total number of books English books sold in So, M+N=474+144=618
all shops together

Click Here For Bundle PDF Course | support@guidely.in Page 10 of 10


SBI Clerk & RRB PO Mains PDF Course 2023
ENGLISH Day - 28

English Language

Directions (1-5): In the following questions three during the interested in and the
columns are given containing three
Second World northeastern
Sentences/phrases each. In the first column,
War as parts of India.
sentences/phrases are A, B and C, in the second
column the sentences/phrases are D, E and F C. During his F. he also I. we know it’s

and in the third column the sentences/phrases visit to highlighted the an Indian

are G,H and I are given. A sentence/phrase from Kolkata, Mr. strategic matter and

one column may or may not connect with Garcetti met significance of we pray for it.

another sentence/phrase from the other two the

columns to make a grammatically and a) A-F-G


contextually correct sentence. Each question has b) A-E-H
five options, four of which display the c) B-D-I
sequence(s) in which the sentences/phrases can d) C-F-G
be joined to form a grammatically and e) None of these
contextually correct sentence. If none of the
options given forms a correct sentence after 2) Match the column.
combination, mark option (E), i.e. “None of
COLUMN 1 COLUMN 2 COLUMN 3
these” as your answer.
1) Match the column. A. The Prime D. Road Rail G. and a 17
Minister will Line completed km long new
COLUMN 1 COLUMN 2 COLUMN 3
also distribute at a cost of ₹750 railway line
A. Speaking at D. and parts of G. that the loans under crore connecting
a press northeastern U.S. opened Keoti-
conference in India served as one of its first Antagarh.
Kolkata, Mr. a gathering consulates in
B. Mr. Modi E. PM SVANidhi H. will
Garcetti point the world in
will also and hand over receive their
appreciated 1784.
dedicate to the keys to PMAY
the
nation the beneficiaries of benefits.
B. The ties E. multi-religious H. bringing doubling of the PM Awaas
with the U.S. nature of more 103 km long Yojana rural
were Kolkata and said investment to Raipur-Khariar houses,
strengthened that the U.S. is the eastern

Click Here For Bundle PDF Course | support@guidely.in Page 1 of 14


SBI Clerk & RRB PO Mains PDF Course 2023
ENGLISH Day - 28

C. The Prime F. his I. more than key aspect of algorithmic manufacturing,


Minister will constituency ₹550 crore AI, where decision- logistics, and
also lay the Varanasi where under the Jal algorithms are making, healthcare.
foundation he will Jeevan trained accountability
stone of inaugurate and Mission. for AI systems,
lay foundation
a) A-F-G
stone for
b) B-D-H
a) B-D-G c) A-E-I
b) B-E-H d) C-F-G
c) A-F-I e) None of these
d) C-E-H
e) None of these 4) Match the column.

COLUMN 1 COLUMN 2 COLUMN 3


3) Match the column.
A. Philosophy D. individuals G. and engage
COLUMN 1 COLUMN 2 COLUMN 3
is a discipline develop the in reasoned
A. Robotics D. have G. also that explores ability to debate.
combines AI become requires fundamental evaluate
and physical increasingly careful questions arguments,
machines to popular, consideration question
develop making use of of its impact assumptions,
natural on society.
B. Philosophy E. invites H. seeking to
language
has influenced individuals to uncover
processing to
various fields engage in deep deeper
understand
of study, reflection, meanings and
B. Bias and E. intelligent H. adapt and including pursue insights.
fairness in AI robots that can improve their science, knowledge, and
algorithms are perform performance seek
topics of complex tasks over time.
C. Moreover, F. that has I. Complexities
in
philosophy been a of human
C. Machine F. with I. industries fosters cornerstone of behaviour and
learning is a transparency in like analytical and human thought social

Click Here For Bundle PDF Course | support@guidely.in Page 2 of 14


SBI Clerk & RRB PO Mains PDF Course 2023
ENGLISH Day - 28

Directions (6-10): In each of the questions given


critical thinking structures.
below two phrases in the sentence have been
skills, helping
highlighted. It may or may not need
a) A-E-G
improvement. Read the question carefully and
b) B-D-I
choose an option that would best replace the
c) C-D-G
highlighted parts to make a grammatically correct
d) C-F-H
sentence. In case, the sentence is correct in its
e) None of these
current form and there is no improvement
needed, mark (E) as your answer.
5) Match the column.
6) Analysts and industry pundits forecast that the
COLUMN 1 COLUMN 2 COLUMN 3 notebook market, which has been growing faster
from the desktop market for the past three years,
A. In terms of D. often G. importance
is expected to overtake the desktop market.
governance, recognized as on the well-
a) analyst and industry pundits have forecast
Finland is a one of the being of
b) which has been growing faster since the
parliamentary world's students.
desktop market
republic with happiest
c) which has been growing faster than the
B. The country E. and strong H. making desktop market
is known for its social policies their mark d) analysts and industry’s pundit forecast
love of that prioritize both locally e) No improvement needed
saunas, healthcare, and globally.
education, and 7) Risk aggregation would mean aggregating the
individual risk measures to deciding the most
C. The Finnish F. which hold a I. seen as
appropriate asset class that would contain the
education significant places of
risk to the desired level dictated by the risk
system is place in Finnish relaxation and
appetite.
renowned culture and are socializing.
a) will contain the risk to the desired level
worldwide for
b) will be containing the risk to the desired level
its
c) on deciding the most appropriate asset class
a) A-F-G d) to decide the most appropriate asset class
b) A-E-H e) No improvement needed
c) C-F-G
d) B-F-I 8) The recent introduction of the Goods and
e) None of these Services Tax, which is the most significant

Click Here For Bundle PDF Course | support@guidely.in Page 3 of 14


SBI Clerk & RRB PO Mains PDF Course 2023
ENGLISH Day - 28

overhaul of the taxation system in India ever, d) on how to bring about greater and more
also aim to achieve a unified market through the consistent
nation for the first time. e) No improvement needed
a) that is the most significant overhaul of the
taxation system Directions (11-14): In the questions given below,
b) aims to achieve a unified marked across the four statements are given labelled as (A),(B),(C)
nation and (D), with one statement being grammatically
c) aims to achieve a unified market by the nation and contextually incorrect.You are required to
d) which is more significant overhaul of the choose that sentence which is erroneous. If all
taxation system the sentences are correct then choose ‘option e’
e) No improvement needed as your answer.
11) Pick the erroneous sentence.
9) According to a recent survey, almost 57% of A) Gold worth ₹3 crores were seized from the
Indian respondents using the internet prefer to apartment by the police.
bank online and use other financial services due B) There is a lot many opportunities for banking
to hassle free access and time saving features of aspirants in the upcoming year.
online banking. C) They were no longer able to teach the needy
a) using the internet prefer to banking online students after the natural disaster.
b) use the internet prefer banking online D) Ranveer firmly believed that he could make it
c) due to hassle free accessing and time saving to the finals.
features a) A
d) which are hassle free access and time saving b) B
feature c) C
e) No improvement needed d) D
e) All are correct
10) In India, innovation is emerging as one of the
most important rubrics in the discourse of how to 12) Pick the erroneous sentence.
bring about greater and more consistent A) The team lead decided to resign as all his
economic and social development. ideas were rejected by the senior architect.
a) is emerging as one of the mostly important B) My mom and I have decided to go on a long
b) is emerging as one of the most importance holiday to Maldives this August.
c) in how to bring about great and more C) The temple and the area around it have
consistent become a good tourist spot.

Click Here For Bundle PDF Course | support@guidely.in Page 4 of 14


SBI Clerk & RRB PO Mains PDF Course 2023
ENGLISH Day - 28

D) The weather outside was extremely pleasant D) During our stay we were been cautioned
and hence we decided to enjoy a morning bike about thieves and were told to lock the doors
ride. always.
a) A a) A
b) B b) B
c) C c) C
d) D d) D
e) All are correct e) All are correct

13) Pick the erroneous sentence. Directions (15-21): Read the given passage
A) The duo studied hard for the exam but also carefully and answer the following questions.
still didn't get good grades. According to a recent estimate (Kerala Planning
B) Smita is more excited about the vacation than Board Report 2021), Kerala is an employment
anyone else in the gang. hub for 34 lakh inter-State migrant workers.
C) Before the injury my uncle used to take a walk Higher wages, regularity of work, and better
in the park every evening. social and cultural milieu compared to many
D) Diverse societies and workplaces have been other States are the key drivers influencing the
shown to have positive impacts on economic workers to flow towards Kerala. Kerala has
growth. implemented a range of welfare, health, and
a) A literacy schemes for migrant workers. These
b) B policy initiatives sustain Kerala as a migrant-
c) C friendly state. Yet, systematic micro-level
d) D enquiries point to various shortcomings.
e) All are correct Preliminary observations of the Science and
Engineering Research Board (SERB 2022-25)
14) Pick the erroneous sentence. study titled “Effect of Social Institution and
A) Both Jothi and Indra realised their mistakes Technological Interventions on Access to
and were ashamed about what they had said to Healthcare Among Interstate Migrant Labourers
their parents. in Kerala” conducted by the Mahatma Gandhi
B) I would neither join a BPO service nor a food University reveal that the benefits entitled in
delivery service. these policies do not reach a majority of migrant
C) Two of the eleven members in the team were workers. This article details the path towards an
severely injured and were rushed to the hospital. inclusive culture of policy-making, in the context
of the discussions held at the International

Click Here For Bundle PDF Course | support@guidely.in Page 5 of 14


SBI Clerk & RRB PO Mains PDF Course 2023
ENGLISH Day - 28

Labour Conclave on May 25, 2023, at abuse, as well as verbal and symbolic violences,
Thiruvananthapuram, Kerala. Migrant labour are frequently reported too. Such situations
governance is facing organising trouble, broadly beckon the need for the protection of the rights
conceived as unity trouble. Unity trouble refers to and dignity of the workers. The health and well-
the absence of a disaggregated assessment of being of workers are constantly at risk due to
the work and life situations of the migrant labour exposure to hazardous substances and also by
population. It is widely argued that the official diseases and frequent accidents in their
name ‘guest worker’ is an ambiguous symbolic workplaces.
expression, uncritically reproduced by the media Another menace is the phenomenon of the chain
and even the research community. Critics argue of exploitation — a network of wage exploitation
that this expression fails to account for the activities involving influential members of the
nuanced aspects of rights denials. They also migrant labour communities aiding and abetting
argue that current policies lack a comprehensive wage exploitation and theft, which is similar to
vision and strategy compared to the Interstate the infamous colonial indenture labour practices.
Migrant Workmen Act, 1979. Strikes and protests Migrant workers have been stripped of their
by migrant labourers over the past three decades citizenship rights and basic human rights, directly
in Kerala point to the need for inclusive policy- and otherwise, rendering them continually
making. It is crucial that policymakers address marginalised within society. This predicament
the core rights issues raised by the Uzhavoor aligns with the profound words of Hannah
strike (2012), which aimed to secure better Arendt, who said that every individual deserves
wages and eliminate social inequality; the the inherent dignity of humanity. In contemporary
Athirampuzha strike (2023), which demanded capitalist society, without social justice, a
equal pay for equal work; the Etumanoor strike systematic disregard for rights emerges, creating
(2023), targeting contractors who denied the disposable populations who are subject to
rights of workers; the anti-exploitation strike exploitation, exclusion, or even elimination.
(2016) against the forced collection of union fee Kerala can enhance its reputation/A as a space
in Kochi Metro; and the anti-wage theft strike of inclusivity and sustenance/B by address of
(2022) against contractors who absconded the/C social justice concerns of migrant
without paying workers. Regrettably, the workers/D (P). Observations of our extensive
government mechanism failed to address the fieldwork and discussions with migrant workers
rights denials highlighted by these strikes. In of eight districts of Kerala, interactions with
addition to wage-related issues, these workers policy-makers, trade unionists and NGO activists,
are faced with unhygienic living and working have laid bare the key aspects to formulate a
conditions and a lack of social security. Physical rights-based and inclusive policy.A set of

Click Here For Bundle PDF Course | support@guidely.in Page 6 of 14


SBI Clerk & RRB PO Mains PDF Course 2023
ENGLISH Day - 28

recommendations to address the crucial task of I. There is no attention paid to the health and
upholding human rights need to be embraced (X) wellbeing of migrant workers.
in this context. They are, firstly, a shift from the II. Migrant workers often face unique health
usual numerical data collection approach to a challenges and vulnerabilities due to factors such
thematic method to develop a panchayat-wise as limited access to healthcare, language
functional data bank of migrant workers. Second, barriers, precarious living conditions, and social
it is necessary to ensure that government isolation.
decisions regarding the health, employment, III. The benefits of the policies that are
cultural life, and social situation of migrant implemented for the welfare, health, and literacy
workers are firmly grounded in rights-based of migrant workers do not reach the workers
principles. Third, mechanisms must be properly.
established in collaboration with the original a) Only I
State of labourers to enhance access to justice b) Only II
for migrant workers. Fourth, there must be c) Only III
promotion of awareness among migrants about d) Both I and II
their rights and the available legal remedies. e) Both I and III
Fifth, set up initiatives similar to global efforts to
promote cultural exchange, community 16) As per the information given in the passage,
engagement, fostering mutual understanding etc. what do you understand by the phrase ‘unity
to implement inclusive policies for social trouble’ ?
integration of migrant workers into the local a) Unity trouble collectively represents all the
community. Sixth, multi-stakeholder policy and difficulties faced by the migrant workers.
monitoring dialogues for ensuring migrant b) Unity trouble refers to the threats posed by a
workers’ rights and welfare must be fostered, and unity of the state where the workers migrate to, it
lastly, priority must be accorded to establishing a is more or less a inter-state dispute.
comprehensive (Y) vision for sustainable c) The migrant workers join together to form a
workers’ human rights in tune with global unity to strike and protest against the
instruments like Global Compact on Migration government to avail their basic rights.
(GCM) for the migrant populations in Kerala. d) Unity trouble means to leave out the migrant
15) Which of the following given statements workers and do not pay attention to their work
is/are true according to the study “Effect of Social and life.
Institution and Technological Interventions on e) None of these
Access to Healthcare Among Interstate Migrant
Labourers in Kerala” as per the given passage ?

Click Here For Bundle PDF Course | support@guidely.in Page 7 of 14


SBI Clerk & RRB PO Mains PDF Course 2023
ENGLISH Day - 28

17) Which of the following does not uphold 19) Which of the following statements, if true,
human rights ? You are required to pick the odd could be an 8th recommendation to keep human
one out. rights in place according to the context of the
a) Migrant workers are denied the basic right to sentence?
vote as they reside in a different state from their a) New policies related to education, nutrition
home state, this must be addressed by and health of the migrant workers and their
introducing a modern voting system or proxy children must be established.
voting. b) Rent-free housing can be made available to
b) There must be cooperation between the the migrant workers as they find it really hard to
workers from the same state and migrants who survive with a family without a proper shelter.
have moved to the other state for work. c) It must be ensured that the migrant workers
c) Rights based principles, rules and laws should take up an annual health checkup.
be firmly established by the government for d) The workers must be provided with mid day
migrant workers to enjoy human rights properly. meals everyday to attain minimum and basic
d) Social integration of migrant workers must be nutrition.
given importance by implementing inclusive e) None of these
policies.
e) None of these 20) Choose the part which is erroneous in
sentence (P) as your answer.
18) As per the passage, which of the following a) A
can be said to be false? b) B
a) All the current policies related to the migrant c) C
workers are not comprehensive as they exclude d) D
many migrant workers based on various e) No error
standards.
b) Uzhavoor strike and Athirampuzha strike were 21) Choose the pair which has the synonym of
focused on wages and inequality. the word ‘X’ and antonym of the word ‘Y’ as your
c) The migrant workers are more in number in answer.
Kerala because of the high wage and better a) reject, inclusive
social and cultural norms. b) encircle, limited
d) The important issues the migrant workers face c) exclude, exclude
includes physical abuse, lack of security and d) abandon, narrow
unhygienic living. e) welcome, broad
e) None of these

Click Here For Bundle PDF Course | support@guidely.in Page 8 of 14


SBI Clerk & RRB PO Mains PDF Course 2023
ENGLISH Day - 28

Click Here to Get the Detailed Video Solution for the above given Questions
Or Scan the QR Code to Get the Detailed Video Solutions

Answer Key with Explanation

1) Answer: B The sentence: Mr. Modi will also dedicate to the


The fragments/phrases A, E and H combine to nation the doubling of 103 km long Raipur-
form a proper sentence which is contextually Khariar Road Rail Line completed at a cost of
correct and meaningful. ₹750 crore and a 17 km long new railway line
Look at the options and eliminate the wrong connecting Keoti-Antagarh.
combination to arrive at the best possible
answer. 3) Answer: C
The other combinations do not form a proper The fragments A-E-I combine to form a proper
sentence and hence are discarded. sentence which is grammatically and
The sentence: Speaking at a press conference contextually correct.
in Kolkata, Mr. Garcetti appreciated the multi- The combinations given in the rest of the options
religious nature of Kolkata and said that the U.S. do not form a proper sentence and hence are
is interested in bringing more investment to the discarded.
eastern and the northeastern parts of India. The sentence: Robotics combines AI and
physical machines to develop intelligent robots
2) Answer: A that can perform complex tasks in industries like
The combination B-D-G is correct out of the manufacturing, logistics, and healthcare.
given four options.
The other combinations do not form a proper 4) Answer: C
sentence and hence are discarded(eliminate the The statements C, D and G combine to form a
incorrect options/combinations) complete sentence with meaning.

Click Here For Bundle PDF Course | support@guidely.in Page 9 of 14


SBI Clerk & RRB PO Mains PDF Course 2023
ENGLISH Day - 28

The combinations given in the rest of the options Option b - growing faster since the desktop
do not form a proper sentence and hence are market(faster is a comparative word, hence
discarded(use elimination method to eliminate ‘than’ must be used)
the incorrect options and arrive at the right one) Option d - industry’s pundit is wrong, instead it
The sentence: Moreover, philosophy fosters must be industry pundits(pundits - an expert)
analytical and critical thinking skills, helping Analysts and industry pundits forecast that the
individuals develop the ability to evaluate notebook market, which has been growing faster
arguments, question assumptions, and engage than the desktop market for the past three years,
in reasoned debate. is expected to overtake the desktop market.

5) Answer: D 7) Answer: D
B-F-I is the right combination that forms a The first highlighted phrase is wrong and hence
complete sentence with proper meaning and needs replacement.
context. Replace ‘to deciding’ with ‘to decide’ to make the
Eliminate the other wrong combinations - as they sentence correct and meaningful.
do not form a meaningful and correct sentence. The second highlighted phrase is already correct
The sentence: The country is known for its love in its current form and no change is required.
of saunas, which hold a significant place in The phrases/statements given in other options
Finnish culture and are seen as places of are not correct and hence can be eliminated.
relaxation and socializing. Option a - will contain is wrong, the first half of
the sentence has would and the second half
6) Answer: C should also follow the same tense
The second highlighted part in the sentence Option b - will be containing is wrong - tense is
needs improvement. wrong
There is a comparison between the notebook Option c - on deciding is incorrect, instead it
market and desktop market. must be to decide
The notebook market has been growing faster Risk aggregation would mean aggregating the
than the desktop market(not from the) individual risk measures to decide the most
The other replacement options given are appropriate asset class that would contain the
unnecessary/wrong and hence can be ignored. risk to the desired level dictated by the risk
Option a -analyst(singular), should be analysts appetite.
and ‘have forecast’ is wrong
8) Answer: B

Click Here For Bundle PDF Course | support@guidely.in Page 10 of 14


SBI Clerk & RRB PO Mains PDF Course 2023
ENGLISH Day - 28

Option b holds the correct replacement phrase to Option a - banking online is wrong, replace
replace the second highlighted phrase in the banking with bank(verb in this context)
sentence above. Option b - use is incorrect, instead it must be
First highlighted phrase is already correct and using(the customers using internet find online
needs no replacement. banking easier)
Second phrase - replace ‘aim’ with ‘aims’ and Option c - hassle free accessing must be
‘through’ with across(through the nation - nation replaced with hassle free access(the access is
as a medium, across the nation - the whole hassle free)
nation) Option d - which are is incorrect instead it must
Option ‘b’ is the only correct phrase that can be be due to
used to replace the wrong part, other options are
incorrect. 10) Answer: D
Option a - that is incorrect, the correct pronoun Only the second highlighted phrase needs
that must be used is ‘which’ replacement, the first is correct and error free.
Option c - by the nation (by is a wrong In the second phrase replace ‘of’ with ‘on’. On is
preposition here, instead it must across the the correct preposition that must be used to
nation) make the sentence grammatically correct.
Option d - which is more(presence of the word Other given options are inappropriate/incorrect.
‘more’ signifies that there is a comparison, but if Option a - mostly important is wrong, instead it is
we look at the given sentence there is no most important
comparison that has been made, so the phrase Option b - importance(noun) must be replaced
given in option d is absolutely wrong) with important(adjective)
The recent introduction of the Goods and Option c - in is a wrong preposition, it must be
Services Tax, which is the most significant ‘on’
overhaul of the taxation system in India ever, In India, innovation is emerging as one of the
also aims to achieve a unified market across the most important rubrics in the discourse on how
nation for the first time. to bring about greater and more consistent
economic and social development.
9) Answer: E
The given sentence is correct and error free. 11) Answer: B
Hence, it does not need any The erroneous sentence is B.
improvement/replacement. There are a lot of opportunities for banking
aspirants in the upcoming year.

Click Here For Bundle PDF Course | support@guidely.in Page 11 of 14


SBI Clerk & RRB PO Mains PDF Course 2023
ENGLISH Day - 28

A lot - means many, so ‘is’ is incorrect instead it in Kerala” done by Science and Engineering
must be ‘are’ Research Board.
The sentences A, C and D are correct and error The other two statements may or may not be
free. true regarding the migrant workers but only III is
definitely true as we have the information given
12) Answer: E in the passage.
All the four sentences A, B, C and D are correct Hence we go for option c - only III.
and error free. Hence, we choose option e as
our answer. 16) Answer: D
Unity trouble as referred to in the passage is
13) Answer: A “Unity trouble refers to the absence of a
Error is in sentence A. disaggregated assessment of the work and life
In sentence A, ‘but also’ causes complication, situations of the migrant labour population”. Only
remove ‘also’ to make the sentence correct. sentence ‘d’ conveys a similar meaning.
The duo studied hard for the exam but still didn't Hence, we go for option d as our answer.
get good grades.
17) Answer: A
14) Answer: D In the whole passage given there is no
Sentence D is erroneous. discussion about the voting rights of the migrant
Remove ‘been’ in the sentence as it is totally workers, their health and other social rights are
unnecessary and after removing it, the meaning only discussed as a matter of concern.
of the sentence is unaffected. The statements b, c and d are among the seven
The sentences A, B and C are correct in their recommendations given in the passage to
given form and error free. uphold human rights of the migrant workers.
During our stay we were cautioned about thieves Second recommendation - It is necessary to
and were told to lock the doors always. ensure that government decisions regarding the
health, employment, cultural life, and social
15) Answer: C situation of migrant workers are firmly grounded
According to the given passage, only statement in rights-based principles.
III is true based on the information given Third recommendation - Mechanisms must be
regarding the study “Effect of Social Institution established in collaboration with the original
and Technological Interventions on Access to State of labourers to enhance access to justice
Healthcare Among Interstate Migrant Labourers for migrant workers.

Click Here For Bundle PDF Course | support@guidely.in Page 12 of 14


SBI Clerk & RRB PO Mains PDF Course 2023
ENGLISH Day - 28

Fifth recommendation - To set up initiatives The mid day meals schemes and other free food
similar to global efforts to promote cultural schemes are entitled to the people of that
exchange, community engagement, fostering particular state(not everyone can avail it). It is
mutual understanding etc. to implement inclusive difficult for any state to entire;y change their
policies for social integration of migrant workers scheme beneficiaries as this will affect the
into the local community. people who already belong to the state and
depend on the free food schemes.
18) Answer: E
None of the given statements is false. All of them 20) Answer: C
are true in accordance with the passage. Error is in part C of the given sentence (P).
The question is to find and choose the statement Replace ‘address of’ with ‘addressing’.
which is false as per the passage, but all a, b, c Addressing - to give attention to something
and d are true and genuine information given in (according to this context)
the passage. Hence, we go for option e, none of The correct sentence: Kerala can enhance its
these as our answer. reputation as a space of inclusivity and
sustenance by addressing the social justice
19) Answer: A concerns of migrant workers.
The passage is talking about the basic and
essential rights of the migrant workers, so 21) Answer: B
statement ‘a’ seems more related to the ideology ‘Encircle’ is a synonym of the word ‘embrace’
and it is a practical recommendation too. and ‘limited’ is an antonym of the word
Statement ‘a’ can be a good recommendation as ‘comprehensive’
it is more relevant to the context of the passage. Embrace - to include/to put arms around
Rent free housing sounds inappropriate because Comprehensive - including everything/not
it involves a huge budget and it is not a practical leaving out anything
idea to provide a home for every worker who Reject, exclude, abandon are the antonyms of
migrates from state to state. the word embrace (but we need synonym) -
Since, the migrants move from place to place eliminate options a,c,d
and do not have a permanent address in the Similarly Inclusive, broad are the synonyms of
state of work, it is difficult to track their annual the word comprehensive(we need antonyms) -
checkups - it is impossible at least at this point of eliminate options a and e
time.

Click Here For Bundle PDF Course | support@guidely.in Page 13 of 14


SBI Clerk & RRB PO Mains PDF Course 2023
ENGLISH Day - 28

The only left out option is ‘b’ which has the is our answer.
correct pair of a synonym and antonym, hence b

Click Here For Bundle PDF Course | support@guidely.in Page 14 of 14


SBI Clerk & RRB PO Mains PDF Course 2023
Reasoning Ability Day - 29 (Eng)

Reasoning Ability
Directions (1-5): Study the following information likes a number which is 12 less than the number
carefully and answer the given questions. liked by U. Off day is neither before the one who
Eight persons – P, Q, R, S, T, U, V, and W are likes 11 nor between the one who likes 9 and Q.
attending the seminar in five different months 1) How many persons are attending the seminar
viz.- January, April, July, September, and between the one who likes 11 and S?
November of the same year on two different a) Four
dates 9 and 12. Only one person attends the b) As many persons attend the seminar before
seminar on each date. There was no seminar on the one who likes 12
any two dates. None of the two adjacent dates c) As many persons attend the seminar before T
are vacant. Each person likes different numbers d) Three
viz.- 9, 11, 12, 14, 15, 18, 21, and 25. e) Either B or C
Note: Off day is only between any two persons
seminar date. 2) Who among the following person attends the
The one who likes 18 attends the seminars in a seminar on September 12?
month having 31 days. Three persons attends a) None
the seminar between the one who likes 18 and V, b) The one who likes 9
who attends the seminar before the one who c) P
likes 18. W attends the seminar three persons d) The one who attends the seminar two persons
before the one who likes 25. The one who likes after U
25 attends the seminar on 9 of any month but not e) The one who likes 12
in September. The sum of the numbers liked by
W and T is equal to the number liked by P. W 3) Who among the following person likes 11?
neither attends the seminar in the same month a) The one who attends the seminar just after R
with V nor attends after the one who likes 21. T b) U
attends the seminar just after the one who likes c) The one who attends the seminar on July 12
12, both are not attending in the same month. d) V
The number of persons who attends the seminar e) None of these
before T is one less than the number of persons
who attends the seminar after Q. T and the one 4) Four of the following five are alike in a certain
who likes 14 are not attending the seminar in the way based on the given arrangement and thus
same month. Q likes a number which is 3 more form a group. Which one of the following does
than the number liked by V. The one who likes not belong to the group?
11 attends the seminar three persons before S. S a) The one who likes 25

Click Here For Bundle PDF Course | support@guidely.in Page 1 of 11


SBI Clerk & RRB PO Mains PDF Course 2023
Reasoning Ability Day - 29 (Eng)

b) Q C’s father lives northeast of C’s brother, who


c) The one who attends the seminar two persons lives on an even numbered floor. D, who is the
after V mother of C, lives immediately below the flat of
d) The one who likes 15 C’s father. D’s only daughter-in-law, who has
e) The one who attends the seminar immediately only mother, lives two floors below the flat of A,
after Q who is the maternal grandfather of B. A is the
spouse of D, who has only two children. As many
5) Which of the following statement is not true? floors below G as above E, who is the sister-in-
a) The one who likes 12 attends the seminar on law of B’s mother. G is not living in the same type
January 12 of flat as F. Mother-in-law of H lives on the same
b) R attends the seminar three persons before floor as A’s son-in-law. F’s son lives to the north
the one who likes 18 of the daughter of A. F’s spouse is not living in
c) P likes 25 the same type of flat as F.
d) U attends the seminar after S 6) A’s son-in-law lives on which of the following
e) All the above statements are true floor and flat?
a) Floor 1, Flat X
Directions (6-10): Study the following information b) Floor 1, Flat Y
carefully and answer the given questions. c) Floor 4, Flat X
Eight persons – A, B, C, D, E, F, G and H are d) Floor 4, Flat Y
from the same family has three generations living e) Floor 3, Flat Y
on different floors of a four storey building where
the lowermost floor is numbered one and the 7) G’s flat is in which direction with respect to E’s
floor immediately above it is numbered two and mother-in-law of flat?
so on. a) Northeast
Note-I: Each floor has two type of Flats viz., flat X b) Northwest
and flat Y, where flat X is to the west of flat Y. c) North
Note-II: Flat Y of floor 2 is immediately above flat d) South
Y of floor 1 and immediately below flat Y of floor e) Southeast
3 and so on. Similarly, flat X of floor 2 is
immediately above flat X of floor 1 and 8) Which of the following combination is true?
immediately below flat X of floor 3 and so on. a) D - Floor 2, Flat X
Note-III: Only two persons live on each floor and b) The one who is the spouse of H - Floor 2, Flat
only one person lives in each flat. Y
Note-IV: Area of each flat on each floor is same

Click Here For Bundle PDF Course | support@guidely.in Page 2 of 11


SBI Clerk & RRB PO Mains PDF Course 2023
Reasoning Ability Day - 29 (Eng)

c) The one who is the spouse of F - Floor 1, Flat “Bring your own shine” is coded as “#∞@
X %Ω$ %!$ %∞&”
d) The one who is the mother-in-law of H- Floor “Good things are coming” is coded as “%!&
3, Flat Y %∞µ #Ω@ #Ωµ”
e) The one who is the sister-in-law of E - Floor 4, “Let your soul shine” is coded as “%!$ #∞@
Flat X #∞& #Ω@”
“Lights up from inside” is coded as “%∞© #∞µ
9) Which of the following person is the brother-in- %!µ %Ω@”
law of F? (Note: All the given codes are three symbol
I. The one who is the son-in-law of G codes only)
II. The one who is the brother of C 11) Which of the following word represents the
III. The one who is the father of B code “%∞$ #!@ %∞© #!&” in the given coded
a) Only I and II language?
b) Only I and III a) You only live once
c) Only II and III b) Wake up and live
d) All I, II and III c) Time is well spent
e) Only II d) Life is too short
e) None of these
10) Which of the following person lives northwest
of the daughter of G? 12) Which of the following code is represents the
I. The one who is the mother of H word “Do what you love” in the given coded
II. The one who is the grandchild of A language?
III. The one who is the son-in-law of D a) “#Ω& %!$ %∞@ #∞@”
a) All I, II and III b) “#Ω@ #!& #∞@ %!©”
b) Only II and III c) “%∞@ #!@ #!© #!&”
c) Only III d) “%∞& #!@ #∞$ #Ω&”
d) Only I and II e) None of these
e) Only II
13) If “Just one more chapter” is coded as “#!@
Directions (11-15): Study the following %!& %∞£ #∞@”, then what will be the code for
information carefully and answer the given “Collect” in the given coded language?
questions. a) #∞£
In a certain code language, b) %∞£
c) #Ω£

Click Here For Bundle PDF Course | support@guidely.in Page 3 of 11


SBI Clerk & RRB PO Mains PDF Course 2023
Reasoning Ability Day - 29 (Eng)

d) %Ω£ A @ B means the rate of car A is Rs. 15000 less


e) #∞µ than the rate of car B.
A % B means the average rate of cars A and B is
14) What is the code for the words “Lights Rs. 35000.
coming” in the given coded language? A $ B means the sum of the rate of cars A and B
a) #Ω$ %Ω& is Rs. 75000.
b) #Ωµ #∞µ Only four bikes are parked between V and L. At
c) %∞@ #∞© most two bikes are parked to the left of L, which
d) #Ω& #∞µ rate is Rs.45000. P parked third to the right of L,
e) #∞µ %Ω© where P%L. As many bikes parked to the left of
V as to the right of T, where V#T. The number of
15) Match the given lists. bikes parked between F and E is two more than
the number of bikes parked between C and N,
where C$T. The rate of C is Rs. 38000. Neither
C nor F is parked adjacent to V. F@E. The rate
of E is more than C but less than L. The rate of F
is less than P. V@N, which is parked to the left
of V.
a) 1-A, 2-B, 3-C, 4-D 16) Which of the following bike rate is more than
b) 1-B, 2-A, 3-D, 4-C P but less than L?
c) 1-D, 2-C, 3-A, 4-B a) T
d) 1-C, 2-D, 3-B, 4-A b) The bike which is parked at the right end
e) 1-D, 2-A, 3-B, 4-C c) The bike which is parked second to the right of
N
Directions (16-20): Study the following d) Both a and b
information carefully and answer the given e) All a, b and c
questions.
Eight bikes C, V, F, L, T, P, E and N are parked 17) Which of the following bikes rate is the
in a linear row and all are facing north. The rate second highest and second lowest respectively?
of each car is different in the multiples of 1000s. a) P and N
The rate is mentioned in the coded format as b) V and P
follows. c) T and N
A # B means the rate of car A is twice the rate of d) E and F
car B. e) N and L

Click Here For Bundle PDF Course | support@guidely.in Page 4 of 11


SBI Clerk & RRB PO Mains PDF Course 2023
Reasoning Ability Day - 29 (Eng)

18) The average rate of cars __ and __ is Rs. 20) Match the column according to the final
63000. arrangement.
a) N, C
b) V, L
c) T, N
d) E, V
e) L, F

a) 1-C, 2-A, 3-D, 4-B


19) As many bikes parked to the left of P as
b) 1-B, 2-A, 3-C, 4-D
__________.
c) 1-A, 2-B, 3-C, 4-D
a) Between F and N
d) 1-D, 2-B, 3-A, 4-C
b) To the right of C
e) 1-C, 2- D, 3-B, 4-A
c) To the right of E
d) To the right of N
e) Between V and C
Click Here to Get the Detailed Video Solution for the above given Questions
Or Scan the QR Code to Get the Detailed Video Solutions

Answer Key with Explanation

Directions (1-5):
1) Answer: C
2) Answer: A
3) Answer: A
4) Answer: B
5) Answer: E

Click Here For Bundle PDF Course | support@guidely.in Page 5 of 11


SBI Clerk & RRB PO Mains PDF Course 2023
Reasoning Ability Day - 29 (Eng)

Since, only such possible combinations


are (9, 12, 21), and (11, 14, 25).
 W neither attends the seminar in the
same month with V nor attends after the
one who likes 21.
That means case (2) is not valid.
Based on the above given information we have:

We have:
 The one who likes 18 attends the
seminars in a month having 31 days.
 Three persons attends the seminar
between the one who likes 18 and V, who
attends the seminar before the one who
likes 18.
That means, in case (1) the one who likes
18 attends seminar on 9 July, and V
attends the seminar on 9 January, in case
(2) the one who likes 18 attended seminar
on 12 July, and V attends the seminar on
12 January, in case (3) the one who likes
18 attends seminar on 12 July, and V
attends the seminar on 9 January.
 W attends the seminar three persons
before the one who likes 25.
 The one who likes 25 attends the seminar
on 9 of any month but not in September. Case (2) is not valid as W attended seminar
 The sum of the numbers liked by W and T three persons before the one who likes 25.
is equal to the number liked by P. Again, we have:

Click Here For Bundle PDF Course | support@guidely.in Page 6 of 11


SBI Clerk & RRB PO Mains PDF Course 2023
Reasoning Ability Day - 29 (Eng)

 T attends the seminar just after the one Since, only such possible combination is
who likes 12, both are not attending in the (9, 21).
same month. Thus, S likes 9, and U likes 21.
 The number of persons who attends the As, the sum of the numbers liked by W
seminar before T is one less than the and T is equal to the number liked by P.
number of persons who attends the That means, P likes 25.
seminar after Q.  Off day is neither before the one who likes
 T and the one who likes 14 are not 11 nor between the one who likes 9 and
attending the seminar in the same month. Q.
 Q likes a number which is 3 more than the That means, in case (3) S attends the
number liked by V. seminar on September 9, case (1) is not
Since, only such possible combination is valid.
(9, 12), (12, 15), and (15, 18). Based on the above given information we have:
That means, in case (1) & case (3) T
attended the seminar on April 9.
Based on the above given information we have:

Case (1) is not valid as the off day is neither


before the one who likes 11 nor between the one
who likes 9 and Q.
Again, we have:
 The one who likes 11 attends the seminar Directions (6-10):
three persons before S. 6) Answer: D
 S likes a number which is 12 less than the 7) Answer: B
number liked by U. 8) Answer: C
9) Answer: A

Click Here For Bundle PDF Course | support@guidely.in Page 7 of 11


SBI Clerk & RRB PO Mains PDF Course 2023
Reasoning Ability Day - 29 (Eng)

10) Answer: E  A is the spouse of D, who has only two


Final arrangement: children.
 As many floors below G as above E, who
is the sister-in-law of B’s mother. G is not
living in the same flat as F.
Applying the above conditions, there is one extra
possibility.

We have,
 C’s father lives northeast of C’s brother,
who lives on an even numbered floor.
 D, who is the mother of C, lives
immediately below the flat of C’s father.
Applying the above conditions, there are two
possibilities.
Again we have,
 Mother-in-law of H lives on the same floor
as A’s son-in-law.
 F’s son lives to the north of the daughter
of A.
 F’s spouse is not living in the same flat as
F.
Applying the above conditions, Case 1 and
Case 2 get eliminated because there is
wrong position for F in Case 2 and wrong
Again we have, position for C and F in Case 1. Hence, Case
 D’s only daughter-in-law, who has only 3 gives a final arrangement.
mother, lives two floors below the flat of
A, who is the maternal grandfather of B.

Click Here For Bundle PDF Course | support@guidely.in Page 8 of 11


SBI Clerk & RRB PO Mains PDF Course 2023
Reasoning Ability Day - 29 (Eng)

If the last letter of the word is in the second half


of the English alphabetical series then the
second code of the word is “∞”
If the last letter of the word is a vowel then the
second code of the word is “!”
The logic for the third symbol for each word is
based on the number of letters in each word,
If the word has two letters then the third code of
the word is “©”
If the word has three letters then the third code
of the word is “&”
If the word has four letters then the third code of
the word is “@”
Directions (11-15): If the word has five letters then the third code of
11) Answer: D the word is “$”
12) Answer: C If the word has six letters then the third code of
13) Answer: A the word is “µ"
14) Answer: B
15) Answer: E Directions (16-20):
The logic of the coded language is, 16) Answer: E
The logic for the first symbol for each word is 17) Answer: B
based on the second letter of each word, 18) Answer: C
If the second letter is a vowel then the first code 19) Answer: D
of the word is “#” 20) Answer: A
If the second letter is a consonant then the first Final arrangement:
code of the word is “%”
The logic for the second symbol for each word is
based on the last letter of each word,
If the last letter of the word is in the first half of
the English alphabetical series then the second We have,

code of the word is “Ω”  Only four bikes are parked between V and
L.

Click Here For Bundle PDF Course | support@guidely.in Page 9 of 11


SBI Clerk & RRB PO Mains PDF Course 2023
Reasoning Ability Day - 29 (Eng)

 At most two bikes are parked to the left of  The number of bikes parked between F
L, which rate is Rs.45000. and E is two more than the number of
 P parked third to the right of L, where bikes parked between C and N, where
P%L. C$T.
Applying the above conditions, there are  The rate of C is Rs. 38000.
three possibilities.  Neither C nor F is parked adjacent to V.
Applying the above conditions, Case 3 gets
eliminated because there is no possibility to
place C and N. There is one extra possibility.

Again we have,
 As many bikes parked to the left of V as
to the right of T, where V#T.
Applying the above condition, Case 2 gets Again we have,
eliminated because there is no possibility to  F@E.
place T.  The rate of E is more than C but less than
L.
 The rate of F is less than P.
 V@N, which is parked to the left of V.
Applying the above conditions, Case 1 gets
eliminated because there is N is parked to
the right of V. Hence, Case 1a gives a final
arrangement.

Again we have,

Click Here For Bundle PDF Course | support@guidely.in Page 10 of 11


SBI Clerk & RRB PO Mains PDF Course 2023
Reasoning Ability Day - 29 (Eng)

Click Here For Bundle PDF Course | support@guidely.in Page 11 of 11


SBI Clerk & RRB PO Mains PDF Course 2023
Quantitative Aptitude Day – 29 (Eng)

Quantitative Aptitude

Directions (1-5): Each of the following questions that of bike A and car P crosses bike A in 15
consists of three statements. You have to decide seconds.
whether the data provided in the statements are Statement II: Bike A is running 450 m ahead of
sufficient to answer the question. car Q, the speed of car Q is 2 m/s more than that
1) The average of the present ages of A, B and of bike B and car Q crosses bike A in 75
C is 30 years and the average of the present seconds.
ages of B, C and D is 25 years. Find the average Statement III: The speed of bike A is 70% of that
of the present ages of B and D? of car Q and car Q can cross a 140 m long tunnel
Statement I: The ratio of A’s age after 2 years to in 7 seconds.
D’s age after 2 years will be 9: 4. a) Statement I alone is sufficient to answer the
Statement II: The ratio of A’s age before 5 years question.
to C’s age after 5 years is 2: 3. b) Statement II alone is sufficient to answer the
Statement III: The difference between B’s question.
present age and C’s present age is 15 years. c) Statement III alone is sufficient to answer the
a) Statement I alone is sufficient to answer the question.
question. d) Only statements II and III together are
b) Only statements I and II together are sufficient sufficient to answer the question.
to answer the question. e) Either of three statements is sufficient to
c) Statement III alone is sufficient to answer the answer the question.
question.
d) All the statements together are not sufficient to 3) A shopkeeper has two articles A and B and
answer the question. the cost of article B is ₹200 more than that of
e) Both (b) and (c) article A. Articles A and B are marked up by 25%
and 20% respectively, and article B is sold at 5%
2) Two bikes A and B are running in the same discount on its marked price. Find the profit
direction such that bike B is chasing bike A. The amount received on article B?
distance between bikes A and B is 144 m and Statement I: If article A is sold at 20% profit, then
bike B crosses bike A in 36 seconds. Find the the selling price of article B will be ₹192 more
speed of bike B? than that of article A.
Statement I: Bike A is running 165 m ahead of Statement II: If article A is sold at 2% discount on
car P, the speed of car P is 11 m/s more than its marked price, the profit received on article A
will be ₹135.

Click Here For Bundle PDF Course | support@guidely.in Page 1 of 18


SBI Clerk & RRB PO Mains PDF Course 2023
Quantitative Aptitude Day - 29 (Eng)

Statement III: The ratio of the marked price of b) Statement II alone is sufficient to answer the
article A to that of article B is 25: 32. question.
a) Statement I alone is sufficient to answer the c) Statement III alone is sufficient to answer the
question. question.
b) Statement II alone is sufficient to answer the d) Either statement I alone or statement II alone
question. is sufficient to answer the question.
c) Statement III alone is sufficient to answer the e) Either of the three statements is sufficient to
question. answer the question.
d) Only statements II and III together are
sufficient to answer the question. 5) Five workers A, B, C, D and E are hired to
e) Either of three statements is sufficient to complete a work and they are paid in the ratio of
answer the question. their efficiencies. A and B together can complete
the work in 8 hours, while C and D together can
4) The monthly savings of a person is ₹4000 less complete the work in 12 hours. In what time B
than his monthly expenditure and he spends and C together can complete the work?
of his monthly expenditure on Statement I: The ratio of D’s wages to E’s wages
entertainment. Find his monthly salary? is 5: 3 and the ratio of C’s wages to E’s wages is
Statement I: If the person invests his monthly 5: 6.
savings in a scheme, then difference between Statement II: A’s and B’s wages are respectively
the compound interest and simple interest after 2 ₹360 and ₹1440 more than C’s wages.
years at 15% rate will be ₹315. Statement III: The total wages received by A and
Statement II: His monthly expenditure on child’s B together is ₹3240.
education is ₹2000 more than that on a) Statement I alone is sufficient to answer the
entertainment and his other monthly question.
expenditures are ₹2000 less than that on b) All the statements together are not sufficient to
entertainment. The ratio of monthly expenditure answer the question.
on child’s education to other monthly c) Only Statement I and II together are sufficient
expenditures is 2: 1. to answer the question.
Statement III: Monthly savings of the person is d) All the statements together are necessary to
₹10000 more than his other monthly expenditure. answer the question.
a) Statement I alone is sufficient to answer the e) Statement III alone is sufficient to answer the
question. question.

Click Here For Bundle PDF Course | support@guidely.in Page 2 of 18


SBI Clerk & RRB PO Mains PDF Course 2023
Quantitative Aptitude Day - 29 (Eng)

Directions (6-9): Study the following data c) Only Y and Z


carefully and answer the questions: d) Only X and Y
The data given below is related to the investment e) Only Y
(in crore ₹) and earnings (in crore ₹) of 5
difference companies A, B, C, D and E in the 7)If the total investment of companies B and D
year of 2020. together is 65% more than that of company F
Table given below shows the following data: and the total earning of companies B and C
together is 143.75% more than that of company
F, then find that the total profit of companies C
and D together is what per cent more than that of
company F?
a) 132%
b) 110%
c) 121%
d) 88%
e) 143%

8)If the investment of company A were ___% of


Note:
its original investment and the earning of
1: Profit = Earning – Investment
company A were ___% of its original earning,
2: Difference between the investments of
then the profit of company A would be 20% more
companies B and D is ₹9 crore.
than its original profit.
3: Profit of company A is of its investment.
Find which of the following can be filled in the
4: Earning of company B is ₹2 crore less than
blanks in the same order?
that of company A and also ₹3 crore less than
M: 92, 99
that of company C.
N: , 112
6)If the profits of companies B, C and E are
O: 82, 90
respectively P%, Q% and R% of that of company
a) All M, N and O
A, then find which of the following is are true?
b) Only M and N
X: All of P, Q and R are multiple of 3.
c) Only N
Y: 6 * (P – R) = Q
d) Only M
Z:
e) Only N and O
a) Only X
b) None is true

Click Here For Bundle PDF Course | support@guidely.in Page 3 of 18


SBI Clerk & RRB PO Mains PDF Course 2023
Quantitative Aptitude Day - 29 (Eng)

9)If the investment and earning of company C in a) ₹6.625


2021 are respectively 25% and 30% more than b) ₹4.625
that in 2020 and the investment and earning of c) ₹2.625
company E in 2021 are respectively 50% and d) ₹5.625
25% more than that in 2020, then find the e) ₹3.625
average profit of companies C and E in 2021?

Directions (10-13): Study the following data carefully and answer the questions:
A survey is conducted in a company to find out what kind of movies the employees like to watch among
action, comedy, horror, documentary and adventure movies.
The pie chart given below shows the percentage distribution of the number of employees, who like the
given kinds of movies.

Note:
1: Ratio of the number of employees, who like action movies, to those, who like comedy movies, is 3: 2.
2: Ratio of the number of employees, who like horror movies, to those, who like documentary movies, is
2: 1.
3: Difference between the number of employees, who like action movies and those, who like horror
movies, is 18.
10) If the ratio of male employees to female who like comedy movies, is 5: 7 and the ratio of
employees, who like action movies, is 5: 4, the total male employees, who like action and
ratio of male employees to female employees, comedy movies together, to the total female

Click Here For Bundle PDF Course | support@guidely.in Page 4 of 18


SBI Clerk & RRB PO Mains PDF Course 2023
Quantitative Aptitude Day - 29 (Eng)

employees, who like action and comedy movies B: 90, 110


together, is P: Q, then find the value of ? C: 70, 120
a) 3.9 a) Both B and C
b) 5.2 b) Only A
c) 6.5 c) None can be filled
d) 7.8 d) Only C
e) 2.6 e) Only A and C

11) If 40% of total employees in the company, 13)If 25% of the employees, who like action
are female employees, out of which, (2x + 5) movies, 50% of the employees, who like comedy
female employees like action movies, (2z + 7) movies, 30% of the employees, who like horror
female employees like comedy movies, ‘9y’ movies, 20% of the employees, who like
female employees like horror movies and (x – 2) documentary movies and 40% of the employees,
female employees like documentary movies, who like adventure movies, left the company,
then find that what percent of total female then find the number of employees left in the
employees like adventure movies? company?
a) (6y + 1)% a) 288
b) (x + z)% b) 270
c) (x + y + z + 1)% c) 279
d) (2z - 5)% d) 261
e) 2z% e) 297

12)If the number of employees in another Directions (14-17): Study the following data
company, who like horror movies, those, who like carefully and answer the questions:
documentary movies and those, who like A survey is conducted in two villages A and B to
adventure movies, are respectively ___%, ___% find the number of unfit (physically + mentally)
and 60% of those, in the given company, then persons. In both the villages, out of total unfit
the average number of employees in another persons, some are only physically unfit, some
company, who like horror movies, documentary are only mentally unfit and remaining are both
movies and adventure movies, is 72. physically and mentally unfit.
Find out which of the following can’t be filled in In village A, the ratio of the number of persons,
the blanks in the same order? who are only physically unfit, to those, who are
A: 80, 140 only mentally unfit, is 9: 5 and the number of
persons, who are both physically and mentally

Click Here For Bundle PDF Course | support@guidely.in Page 5 of 18


SBI Clerk & RRB PO Mains PDF Course 2023
Quantitative Aptitude Day - 29 (Eng)

unfit, is 5 less than those, who are only mentally c) 45.5%


unfit. The number of persons in village B, who d) 41.1%
are only physically unfit, is 12 more than those in e) 57.2%
company A. In village B,the number of persons,
who are only mentally unfit, is 80% of those, who 16) If the number of persons in village A, who are
are both physically and mentally unfit.The both physically and mentally unfit, is M% of the
number of persons in village A, who are only total number of unfit persons in village A and the
mentally unfit, is equal to those in village B, who number of persons in village B, who are both
are both physically and mentally unfit.The physically and mentally unfit, is N% of total
number of persons in village A, who are both number of unfit persons in village B, then find the
physically and mentally unfit, is 1 less than those ratio of M to N?
in village B, who are only mentally unfit. a) 42: 71
14)If 16% of the total number of persons in b) 70: 71
village A are only mentally unfit and 10% of the c) 63: 71
total number of persons in village B are only d) 56: 71
mentally unfit, the find that approximately what e) 49: 71
percent of the total number of persons in both
villages together, are only physically unfit? 17) The total number of persons in both villages
a) 32.5% together, who are both physically and mentally
b) 25.5% unfit, is what percent of the total number of
c) 29.5% persons in both villages together, who are only
d) 35.5% physically unfit and who are only mentally unfit?
e) 21.5% a) 33.33%
b) 21.17%
15)If the ratio of male to female in village A, who c) 35.17%
are both physically and mentally unfit, is 2: 3 and d) 29.17%
the ratio of male to female in village B, who are e) 25.17%
both physically and mentally unfit, is 3: 2, then
find that approximately what percent of total Directions (18-20): Study the following data
number of persons in both villages together, who carefully and answer the questions:
are both physically and mentally unfit, are The data given below is related to the runs
female? conceded by two bowlers A and B in three
a) 48.6% different years 2020, 2021 and 2022.
b) 53.3%

Click Here For Bundle PDF Course | support@guidely.in Page 6 of 18


SBI Clerk & RRB PO Mains PDF Course 2023
Quantitative Aptitude Day - 29 (Eng)

The ratio of runs conceded by A in 2020 to that Q: The ratio of runs conceded by A in 2020 to
conceded by B in 2020 is 6: 7, the ratio of runs that conceded by B in 2022 is 5: 6.
conceded by A in 2021 to that conceded by B in R: Runs conceded by B in 2020 is of the
2021 is 8: 7 and the ratio of runs conceded by A total runs conceded by B in 2021 and 2022
in 2022 to that conceded by B in 2022 is 3: 4. together.
The runs conceded by B in 2021 is 80% of that a) Only P
conceded by B in 2020. The runs conceded by A b) None is true
in 2022 is 75% of that conceded by A in 2021. c) Only P and R
The average of runs conceded by B in 2020 and d) Only P and Q
2022 is 1675. e) Only R
18) If the ratio between runs conceded by A in
2020 in test matches, ODI matches and T-20 20) If the runs conceded by A in 2019, are 25%
matches, is 12: 7: 6 and the ratio between runs of total runs conceded by A in 2020, 2021 and
conceded by A in 2021 in test matches, ODI 2022 together and the runs conceded by B in
matches and T-20 matches, is, 9: 6: 5, then find 2019, are 24% of the total runs conceded by B in
the ratio of runs conceded by A in T-20 matches 2020, 2021 and 2022 together, then find the
in 2020 to that in 2021? difference between total runs conceded by A and
a) 18: 19 the total runs conceded by B in all the 4 years
b) 9: 10 together?
c) 20: 21 a) 535
d) 3: 4 b) 505
e) 6: 7 c) 515
d) 545
19) Which of the following is/are not true? e) 525
P: The average runs conceded by B in 2020 and
2021, is 1525.

Click Here For Bundle PDF Course | support@guidely.in Page 7 of 18


SBI Clerk & RRB PO Mains PDF Course 2023
Quantitative Aptitude Day - 29 (Eng)

Click Here to Get the Detailed Video Solution for the above given Questions
Or Scan the QR Code to Get the Detailed Video Solutions

Answer Key with Explanation

1) Answer: B Statement I alone is not sufficient to answer the


Since, average of present ages of A, B and C is question.
30 years. From statements I and II together:
So, 4A – 9D = 10 ---------------(4)
A + B + C = 3 * 30 From equations (3) and (4):
A + B + C = 90 -------------(1) A = 25, D = 10
Since, average of present ages of B, C and D is Since, ratio of A’s age before 5 years to C’s age
25 years. after 5 years is 2: 3.
So, So,
B + C + D = 3 * 25
B + C + D = 75 -------------(2)
C = 25
By equation (1) – (2):
From equation (1):
A – D = 15 -------------(3)
B = 40
From statement I alone:
So, average of present ages of B and D = =
Since, ratio of A’s age after 2 years to D’s age
25 years
after 2 years will be 9: 4.
Statements I and II together are sufficient to
So,
answer the question.
From statement III alone:
4A + 8 = 9D + 18 The difference between B’s present age and C’s
4A – 9D = 10 present age is 15 years, but we don’t know who
We can’t find the present ages of B and D. is elder between B and C.

Click Here For Bundle PDF Course | support@guidely.in Page 8 of 18


SBI Clerk & RRB PO Mains PDF Course 2023
Quantitative Aptitude Day - 29 (Eng)

Statement III alone is not sufficient to answer the From statement III alone:
question. Since, the speed of car Q = = 20 m/s
Hence, only statements I and II together are So, the speed of bike A = a = 70% of 20 = 14
sufficient to answer the question. m/s
From equation (1):
2) Answer: C b = 18
Let the speeds of bikes A and B are ‘a’ m/s and The speed of bike B = 18 m/s
‘b’ m/s respectively. Statement III alone is sufficient to answer the
So, question.
Hence, statement III alone is sufficient to answer

b – a = 4 -----------------(1) the question.

From statement I alone:


Let the speed of car P = (a + 11) m/s 3) Answer: E

Since, bike A is running 165 m ahead of car P Let the CPs of articles A and B are ₹100x and

and car P crosses bike A in 15 seconds. ₹(100x + 200) respectively.

So, So, the MP of article A = 125% of 100x = ₹125x


The MP of article B = 120% of (100x + 200) =
₹(120x + 240)
Since, we can’t find the values of ‘a’ and ‘b’, so
The SP of article B = 95% of (120x + 240) =
the speed of bike B can’t be determined.
₹(114x + 228)
So, statement I alone is not sufficient to answer
And the profit amount received on article B =
the question.
(114x + 228) – (100x + 200) = ₹(14x + 28)
From statement II alone:
From statement I alone:
Let the speed of car Q = (b + 2) m/s
When article A is sold at 20% profit:
Since, bike A is running 450 m ahead of car Q
The SP of article A = 120% of 100x = ₹120x
and car Q crosses bike A in 75 seconds.
So,
So,
(114x + 228) – 120x = 192
x=6
b–a=4 And the profit amount received on article B = (14
Since, we can’t find the values of ‘a’ and ‘b’, so * 6 + 28) = ₹112
the speed of bike B can’t be determined. Statement I alone is sufficient to answer the
Statement II alone is not sufficient to answer the question.
question. From statement II alone:

Click Here For Bundle PDF Course | support@guidely.in Page 9 of 18


SBI Clerk & RRB PO Mains PDF Course 2023
Quantitative Aptitude Day - 29 (Eng)

When article A is sold at 2% discount on its MP:


The SP of article A = 98% of 125x = ₹122.5x
3x – 4000 = 14000
So,
x = 6000
122.5x – 100x = 135
And the monthly salary of the person = (6 * 6000
x=6
– 4000) = ₹32000
And the profit amount received on article B = (14
Statement I alone is sufficient to answer the
* 6 + 28) = ₹112
question.
Statement II alone is sufficient to answer the
From statement II alone:
question.
Since, the monthly expenditure on entertainment
From statement III alone:
= ₹x
Since, the ratio of MP of article A to that of article
So, the monthly expenditure on child’s education
B is 25: 32.
= ₹(x + 2000)
So,
And the other monthly expenditures = ₹(x –
2000)
160x = 120x + 240 So,
x=6
And the profit amount received on article B = (14
x + 2000 = 2x – 4000
* 6 + 28) = ₹112
x = 6000
Statement II alone is sufficient to answer the
And the monthly salary of the person = (6 * 6000
question.
– 4000) = ₹32000
Hence, either of the three statements alone is
Statement II alone is sufficient to answer the
sufficient to answer the question.
question.
From statement III alone:
4) Answer: D
Since, the monthly savings = ₹(3x – 4000)
Let the monthly expenditure = ₹3x
So, the other monthly expenditures = (3x – 4000
So, the monthly savings = ₹(3x – 4000)
– 10000) = ₹(3x – 14000)
And the monthly expenditure on entertainment =
There is no other information to calculate the
of 3x = ₹x
value of ‘x’.
From statement I alone:
Statement III alone is not sufficient to answer the
Since, the difference between CI and SI received
question.
on monthly savings after 2 years at 15% rate is
Hence, either statement I alone or statement II
₹315
alone is sufficient to answer the question.
So,

Click Here For Bundle PDF Course | support@guidely.in Page 10 of 18


SBI Clerk & RRB PO Mains PDF Course 2023
Quantitative Aptitude Day - 29 (Eng)

So, A’s and B’s wages are ₹(x + 360) and ₹(x +
5) Answer: D 1440) respectively.
Since, A and B together can complete the work Since, there is no relation between the
in 8 hours. efficiencies of A and C or B and C, so we can’t
So, determine the time, in which B and C together
can complete the work.
Statements I and II together are not sufficient to
Since, C and D together can complete the work
answer the question.
in 12 hours.
From all the statements together:
So,
Since, A’s and B’s wages are ₹(x + 360) and ₹(x
+ 1440) respectively.
From statement I alone: And the total wages received by A and B
Let the time, in which E alone can complete the together is ₹3240.
work = ‘5t’ hours So,
So, the time, in which D alone can complete the (x + 360) + (x + 1440) = 3240
work = ‘3t’ hours x = 720
And the time, in which C alone can complete the Ratio of A’s efficiency to B’s efficiency = (720 +
work = ‘6t’ hours 360): (720 + 1440) = 1: 2
From equation (2): Let the time, in which A alone and B alone can
complete the work, are ‘2a’ hours and ‘a’ hours
t=6 respectively.
The time, in which C alone can complete the From equation (1):
work = 6 * 6 = 36 minutes
Since, we can’t find the time, in which B alone a = 12
can complete the work, so we can’t determine Now, the time, in which B alone can complete
the time, in which B and C together can the work = 12 hours
complete the work. Since, the time, in which C alone can complete
Statement I alone is not sufficient to answer the the work = 36 minutes
question. So, the time, in which B and C together can
From statements I and II together: complete the work = 4 hours
The time, in which C alone can complete the Hence, all the three statements together are
work = 6 * 6 = 36 minutes necessary to answer the question.
Let C’s wages is ₹x.

Click Here For Bundle PDF Course | support@guidely.in Page 11 of 18


SBI Clerk & RRB PO Mains PDF Course 2023
Quantitative Aptitude Day - 29 (Eng)

Directions (6-9):
Since, the difference between the investments of
companies B and D is₹9 crore.
So, the investment of company A = =
₹15 crore
The investment of company B = 80% of 15 = ₹12
crore
The investment of company C = 112% of 15 =
₹16.8 crore
The investment of company D = 140% of 15 =
6) Answer: E
₹21 crore
The profit of company A = ₹5 crore
And the investment of company E = 98% of 15 =
The profit of company B = ₹6 crore
₹14.7 crore
So, P = = 120%
Since, the profit of company A is of its
The profit of company C = ₹4.2 crore
investment.
So, Q = = 84%
So, the earning of company A = of 15 =
The profit of company E = ₹5.3 crore
₹20 crore
So, R = = 106%
Since, the earning of company B is ₹2 crore less
From X:
than that of company A.
Since, P and Q are multiple of 3, but R is not the
So, the earning of company B:
multiple of 3.
5x% of 20 = 20 – 2
So, X is not true.
x = 18
From Y:
Since, the earning of company B is ₹3 crore less
6 * (P – R) = Q
than that of company C.
6 * (120 – 106) = 84
So, the earning of company C:
84 = 84
(y + 5)% of 20 = 18 + 3
So, Y is true.
y = 100
From Z:

So, Z is not true.

Click Here For Bundle PDF Course | support@guidely.in Page 12 of 18


SBI Clerk & RRB PO Mains PDF Course 2023
Quantitative Aptitude Day - 29 (Eng)

Hence, only Y is true. The new profit of company A = 22.4 – 16 = ₹6.4


crore
7) Answer: B Which is = 28% more than its original
Since, the total investment of companies B and profit.
D together = 12 + 21 = ₹33 crore So, the value of N can’t be filled in the blanks.
So, the investment of company F = = From O:
₹20 crore The new investment of company A = 82% of 15
Since, the total earning of companies B and C = ₹12.3 crore
together = 18 + 21 = ₹39 crore The new earning of company A = 90% of 20 =
So, the earning of company F = = ₹18 crore
₹16 crore The new profit of company A = 18 – 12.3 = ₹5.7
Since, the total profit of companies C and D crore
together = 4.2 + 4.2 = ₹8.4 crore Which is = 14% more than its original
And the profit of company F = 20 – 16 = ₹4 crore profit.
So, required percentage = = 110% So, the value of O can’t be filled in the blanks.
Hence, only the values of M can be filled in the
8) Answer: D blanks in the same order.
From M:
The new investment of company A = 92% of 15 9) Answer: B
= ₹13.8 crore The investment of company C in 2021 = 125% of
The new earning of company A = 99% of 20 = 16.8 = ₹21 crore
₹19.8 crore The earning of company C in 2021 = 130% of 21
The new profit of company A = 19.8 – 13.8 = ₹6 = ₹27.3 crore
crore So, the profit of company C in 2021 = 27.3 – 21
Which is = 20% more than its original = ₹6.3 crore

profit. The investment of company E in 2021 = 150% of

So, the value of M can be filled in the blanks. 14.7 = ₹22.05 crore

From N: The earning of company E in 2021 = 125% of 20

The new investment of company A = of = ₹25 crore

15 = ₹16 crore So, the profit of company E in 2021 = 25 – 22.05

The new earning of company A = 112% of 20 = = ₹2.95 crore

₹22.4 crore Required average = = ₹4.625

Click Here For Bundle PDF Course | support@guidely.in Page 13 of 18


SBI Clerk & RRB PO Mains PDF Course 2023
Quantitative Aptitude Day - 29 (Eng)

Directions (10-13): The percentage distribution of the number of


Since, the ratio of the number of employees, employees, who like documentary movies = 20 –
who like action movies, to those, who like 10 = 10%
comedy movies, is 3: 2. And the percentage distribution of the number of
So, movies, who like adventure movies = 3 * 10 =
30%
Since, the difference between the number of
2x + 2y = 12y
employees, who like action movies and those,
x = 5y --------------(1)
who like horror movies, is 18.
Since, the ratio of the number of employees,
So, the number of employees, who like action
who like horror movies, to those, who like
movies = = 108
documentary movies, is 2: 1.
The number of employees, who like comedy
movies = = 72
x = 2x – 2z
The number of employees, who like horror
x = 2z --------------(2)
movies = = 90
From the pie chart:
The number of employees, who like
(x + y) + 4y + x + (x – z) + 3z = 100
documentary movies = = 45
3x + 5y + 2z = 100 --------------(3)
And the number of employees, who like
From equations (1), (2) and (3):
adventure movies = = 135
3x + x + x = 100
10) Answer: C
x = 20
Since, the total number of employees, who like
From equation (1):
action movies = 108
y=4
So, the number of male employees, who like
From equation (2):
action movies = = 60
z = 10
Now, the percentage distribution of the number And the number of female employees, who like

of employees, who like action movies = (20 + 4) action movies = 108 – 60 = 48

= 24% Since, the total number of employees, who like

The percentage distribution of the number of comedy movies = 72

employees, who like comedy movies = 4 * 4 = So, the number of male employees, who like

16% comedy movies = = 30

The percentage distribution of the number of And the number of female employees, who like

employees, who like horror movies = 20% comedy movies = 72 – 30 = 42


So, P: Q = (60 + 30): (48 + 42) = 1: 1

Click Here For Bundle PDF Course | support@guidely.in Page 14 of 18


SBI Clerk & RRB PO Mains PDF Course 2023
Quantitative Aptitude Day - 29 (Eng)

And the value of = = 6.5 The number of employees in another company,


who like horror movies = 90% of 90 = 81
11) Answer: B The number of employees in another company,
Total number of employees in the company: who like documentary movies = 110% of 45 =
108 + 72 + 90 + 45 + 135 = 450 49.5
So, the number of female employees in the Since, the number of persons can’t be in
company = 40% of 450 = 180 decimals.
Since, the number of female employees, who So, the values of B can’t be filled in the blanks in
like action movies = (2x + 5) = (2 * 20 + 5) = 45 the same order.
The number of female employees, who like From C:
comedy movies = (2z + 7) = (2 * 10 + 7) = 27 The number of employees in another company,
The number of female employees, who like who like horror movies = 70% of 90 = 63
horror movies = 9y = 9 * 4 = 36 The number of employees in another company,
The number of female employees, who like who like documentary movies = 120% of 45 = 54
documentary movies = (x – 2) = 20 – 2 = 18 The number of employees in another company,
So, the number of female employees, who like who like adventure movies = 60% of 135 = 81
adventure movies = 180 – (45 + 27 + 36 + 18) = Required average = = 66
54 So, the values of C can’t be filled in the blanks in
Required percentage = = 30% = (x + the same order.
z)% Hence, the values of B and C can’t be filled in
the blanks in the same order.
12) Answer: A
From A: 13) Answer: E
The number of employees in another company, The number of employees left in the company,
who like horror movies = 80% of 90 = 72 who like action movies = 75% of 108 = 81
The number of employees in another company, The number of employees left in the company,
who like documentary movies = 140% of 45 = 63 who like comedy movies = 50% of 72 = 36
The number of employees in another company, The number of employees left in the company,
who like adventure movies = 60% of 135 = 81 who like horror movies = 70% of 90 = 63
Required average = = 72 The number of employees left in the company,

So, the values of A can be filled in the blanks in who like documentary movies = 80% of 45 = 36

the same order. The number of employees left in the company,

From B: who like adventure movies = 60% of 135 = 81

Click Here For Bundle PDF Course | support@guidely.in Page 15 of 18


SBI Clerk & RRB PO Mains PDF Course 2023
Quantitative Aptitude Day - 29 (Eng)

So, the total number of employees left in the Number of persons in village A, who are only
company: physically unfit = 9 * 4 = 36
81 + 36 + 63 + 36 + 81 = 297 Number of persons in village A, who are only
mentally unfit = 5 * 4 = 20
Directions (14-17): Number of persons in village A, who are both
Let the number of persons in village A, who are physically and mentally unfit = 20 – 5 = 15
only physically unfit and those, who are only Number of persons in village B, who are only
mentally unfit, are 9x and 5x respectively. physically unfit = 36 + 12 = 48
So, the number of persons in village A, who are Number of persons in village B, who are only
both physically and mentally unfit = (5x – 5) mentally unfit = 4 * 4 = 16
And the number of person in village B, who are Number of persons in village B, who are both
only physically unfit = (9x + 12) physically and mentally unfit = 5 * 4 = 20
Also let the number of persons in village B, who
are both physically and mentally unfit = 5y
So, the number of person in village B, who are
only mentally unfit = 80% of 5y = 4y
Since, the number of persons in village A, who
14) Answer: C
are only mentally unfit, is equal to those in
The number of persons in village A, who are only
village B, who are both physically and mentally
mentally unfit = 20
unfit.
So, the total number of persons in village A =
So,
= 125
5x = 5y
x = y ----------(1) The number of persons in village B, who are only

Since, the number of persons in village A, who mentally unfit = 16

are both physically and mentally unfit, is 1 less So, the total number of persons in village B =
= 160
than those in village B, who are only mentally
unfit. The total number of persons in both the villages

4y – (5x – 5) = 1 together = 125 + 160 = 285

5x – 4y = 4 ----------(2) The total number of persons in both the villages

From equations (1) and (2): together, who are only physically unfit = 36 + 48

5x – 4x = 4 = 84

x = 4, y = 4 Required percentage = = 29.5%


(approx.)

Click Here For Bundle PDF Course | support@guidely.in Page 16 of 18


SBI Clerk & RRB PO Mains PDF Course 2023
Quantitative Aptitude Day - 29 (Eng)

15) Answer: A Directions (18-20):


Number of males in village A, who are both Let the runs conceded by A and B in 2020 are 6x
physically and mentally unfit = =6 and 7x respectively, the runs conceded by A and
Number of females in village A, who are both B in 2021 are 8y and 7y respectively and the
physically and mentally unfit =15 – 6 = 9 runs conceded by A and B in 2022 are 3z and 4z
Number of males in village B, who are both respectively.
physically and mentally unfit = = 12 Since, the runs conceded by B in 2021 is 80% of

Number of females in village B, who are both that conceded by B in 2020.

physically and mentally unfit = 20 – 12 = 8 So,

Required percentage = = 48.6%

5y = 4x -------------(1)
16) Answer: C And the runs conceded by B in 2020 = =
The number of persons in village A, who are 8.75y
both physically and mentally unfit = 15 Since, the runs conceded by A in 2022 is 75% of
The total number of unfit persons in village A = that conceded by A in 2021.
36 + 15 + 20 = 71 So,
So, M =
The number of persons in village B, who are
z = 2y -------------(2)
both physically and mentally unfit = 20
And the runs conceded by B in 2022 = 4 * 2y =
The total number of unfit persons in village B =
8y
48 + 20 + 16 = 84
Since, the average of runs conceded by B in
So, N =
2020 and 2022 is 1675.
Required ratio = M: N = = 63: 71
So,

17) Answer: D
16.75y = 2 * 1675
Total number of persons in both the villages
y = 200
together, who are both physically and mentally
From equation (1):
unfit = 15 + 20 = 35
x = 250
Total number of persons in both the villages
From equation (2):
together, who are only physically unfit and who
z = 400
are only mentally unfit = 36 + 20 + 48 + 16 = 120
Required percentage = = 29.17%

Click Here For Bundle PDF Course | support@guidely.in Page 17 of 18


SBI Clerk & RRB PO Mains PDF Course 2023
Quantitative Aptitude Day - 29 (Eng)

1500: 1600 = 15: 16


So, Q is not true.
From R:
Runs conceded by B in 2020 = 1750
Total runs conceded by B in 2021 and 2022
together = 1400 + 1600 = 3000
Required percentage =

18) Answer: B So, R is true.

Total runs conceded by A in 2020 = 1500 Hence, only P and Q are not true.

So, the runs conceded by A in T-20 matches in


2020 = = 360 20) Answer: C
Total runs conceded by A in 2020, 2021 and
Total runs conceded by A in 2021 = 1600
2022 together:
So, the runs conceded by A in 2021 =
1500 + 1600 + 1200 = 4300
= 400
So, the runs conceded by A in 2019 = 25% of
Required ratio = 360: 400 = 9: 10
4300 = 1075
Total runs conceded by B in 2020, 2021 and
19) Answer: D
2022 together:
From P:
1750 + 1400 + 1600 = 4750
Average runs conceded by B in 2020 and 2021:
So, the runs conceded by B in 2019 = 24% of
4750 = 1140
So, P is not true. Required difference = (4750 + 1140) – (4300 +
From Q: 1075) = 515
Ratio of runs conceded by A in 2020 to that
conceded by B in 2022:

Click Here For Bundle PDF Course | support@guidely.in Page 18 of 18


SBI Clerk & RRB PO Mains PDF Course 2023
ENGLISH Day - 29

English Language

Directions (1-5): Swap the words given in bold (if e) No changes required
necessary) to make the given sentences
meaningful and correct. 3) Swap the bold words given in the sentences, if
1) Swap the bold words given in the sentences I needed.
and II, if needed. I. The police nabbed(A) a notorious(B) criminal
I. The ISRO took reported(A) actions after(B) who had been terrorising builders and
conducting an in-depth analysis of the fault and exhorted(C) money from them for the past two
carried out a successful(C) launch of the SSLV in years.
February. II. The coach extorted(D) the team to give their
II. Thirteen landslides and nine flash floods have best effort(E) and never give up, inspiring them
been corrective (D) in the State in the past(E) 36 to push through challenges and strive(F) for
hours, according(F) to the State emergency victory.
operation centre. a) A-D
a) A-C b) A-F
b) A-D c) C-D
c) B-E d) C-E
d) C-F e) No changes required
e) No changes required
4) Swap the bold words given in the sentences, if
2) Swap the bold words given in the sentences, if needed.
needed. I. Every modern society depends(A) on the
I. The Minister said that the State was facing an opportunities(B) in the skills and ethics of a
acute(A) financial crunch and feared(B) that the variety(C) of institutions such as schools and
government might be judged(C) to take loans to colleges.
pay its employees. II. The ability(D) of a woman to do well does not
II. Rita was afraid(D) to speak up during the rest(E) on whether it is a man’s world or not,
meeting due to the fear(E) of being compelled(F) because everyone has their own trust(F).
by the young audience. a) A-E
a) A-D b) B-F
b) B-D c) C-D
c) B-E d) C-F
d) C-F e) No changes required

Click Here For Bundle PDF Course | support@guidely.in Page 1 of 11


SBI Clerk & RRB PO Mains PDF Course 2023
ENGLISH Day - 29

5) Swap the bold words given in the sentences, if c) The council is not ready to accept the
needed. amendment to exempt the centre from paying a
I. He was a good mathematician so he could not fine
fathom(A) why other people were stressed(B) d) The Act empowers the Centre to require that
about such an important(C) and easy subject. any forest land diverted for non-forestry purposes
II. Janani lost an interesting(D) document and be duly compensated
rather than confessing(E) her mistake she e) None of these
blamed(F) her friend for losing it.
a) A-F 7) _______________. But defence forces are not
b) B-F bound by international climate agreements to
c) C-D report or cut their carbon emissions, and the data
d) C-E that is published by some militaries is unreliable
e) No changes required or incomplete at best.That's because military
emissions abroad, from flying jets to sailing ships
Directions (6-10) : In each of these questions, a to training exercises, were left out of the 1997
paragraph is given that has a blank in it. Out of Kyoto Protocol on reducing greenhouse gases
the given options, only one sentence fits in with and exempted again from the 2015 Paris
the context of the paragraph. Select that as your accords, on the grounds that data about energy
answer. use by armies could undermine national security.
6) A Parliamentary committee, set up to examine a) Among the world's biggest consumers of fuel,
controversial proposed amendments to the militaries account for 5.5% of global greenhouse
Forest (Conservation) Act, 1980, has endorsed gas emissions, according to a 2022 estimate by
the amendment Bill in its entirety. The Bill seeks international experts
to amend the pivotal 1980 law which was b) Aircraft are a significant source of greenhouse
enacted to ensure that India’s forest land is not gas emissions, primarily carbon dioxide (CO2),
wantonly usurped for non-forestry purposes. ___ which is a major contributor to climate change.
. It also extends its remit to land which is not c) When it comes to taking stock of global
officially classified as ‘forest’ in State or Central emissions, there's an elephant in the room.
government records. d) Airlines and air traffic control systems can
a) Amending a bill is making changes or implement more efficient flight procedures, such
improvements to an existing bill as continuous descent and climb, to minimize
b) The parties are opposing the new amendment fuel consumption and emissions during takeoff
that has been made and landing.
e) None of these

Click Here For Bundle PDF Course | support@guidely.in Page 2 of 11


SBI Clerk & RRB PO Mains PDF Course 2023
ENGLISH Day - 29

8) According to Isro officials, the Chandrayaan-3 view, modify, and distribute the code for a
will reach the lunar orbit almost a month after its particular software project, fostering a culture of
launch, and its lander, Vikram, and rover, openness and innovation.In open source
Pragyaan, are likely to land on the Moon on development, the source code of a software
August 23. Notably, the landing site of the latest program is made freely available to the public,
mission is more or less the same as the typically under a licence that grants users the
Chandrayaan-2: near the south pole of the moon right to study, modify, and distribute the code.
at 70 degrees latitude. If everything goes well, ______________.
the Chandrayaan-3 will become the world’s first a) This transparency also fosters trust and
mission to soft-land near the lunar south accountability within the community.
pole.___________. The furthest that any b) This iterative process of improvement and
spacecraft has gone from the equator was innovation drives the evolution of open source
Surveyor 7, launched by NASA, which made a software.
moon landing way back on January 10, 1968. c) This collaborative approach fosters a spirit of
a) The terrain and temperature are more cooperation and mutual support.
hospitable and conducive for a long and d)This collaborative model encourages a diverse
sustained operation of instruments. community of developers to contribute their
b) But several Orbiter missions have provided knowledge and expertise to improve the
evidence that these regions could be very software.
interesting to explore. e) None of these
c) All the previous spacecraft to have landed on
the Moon have landed in the equatorial region, a 10) Blue Flag beaches are internationally
few degrees latitude north or south of the lunar recognized symbols of cleanliness, safety, and
equator environmental sustainability. The Blue Flag
d) It is a follow-up to the 2019 Chandrayaan-2 program, run by the Foundation for
mission, which partially failed after its lander and Environmental Education (FEE), awards this
rover couldn’t execute a soft-landing on the prestigious eco-label to beaches and marinas
Moon. that meet stringent criteria regarding water
e) None of these quality, environmental management, safety, and
education.___________. The primary focus is on
9) Open source refers to a software development water quality, which requires the beach to
approach that promotes open collaboration, consistently meet strict standards for bathing
transparency, and the sharing of source code water.
with the wider community. It allows anyone to

Click Here For Bundle PDF Course | support@guidely.in Page 3 of 11


SBI Clerk & RRB PO Mains PDF Course 2023
ENGLISH Day - 29

a) To be designated as a Blue Flag beach, a 12) The project ran into rough weather, among
coastal area must meet a range of criteria that other issues over one of the specifications
ensure its ecological integrity and promote mentioned, that the submarine on offer should
responsible tourism. have a proven module with an ________of two
b) They aim to raise awareness among visitors weeks.
about the importance of environmental protection 1. importance
and sustainable practices. 2. endurance
c) They must provide adequate lifeguard 3. tolerance
services, appropriate signage. 4. acceptance
d) Educational programs and activities are often a) 1 and 2
organized to engage beachgoers, encouraging b) 1 and 3
them to become responsible stewards of the c) 2 and 4
coastal environment. d) 3 and 4
e) None of these e) None of these

Directions (11-15) : In the statements given 13) Heavy rains that triggered landslides have
below, a blank has been given. This needs to be left hundreds _______ as several roads have
filled up using a correct combination of words been damaged due to landslides, adversely
from the options mentioned below. The impacting vehicular movement in the state.
statement so formed must be contextually and 1. rescued
grammatically sound. 2. saved
11) The Food Corporation of India mainly aims to 3. helpless
support the grain producers by ________wheat 4. stranded
and rice at minimum support prices. a) 1 and 2
1. acquiring b) 1 and 4
2. procuring c) 2 and 4
3. eliminating d) 3 and 4
4. endsoring e) None of these
a) 1 and 2
b) 1 and 4 14) In June 2022, the member-countries of the
c) 2 and 4 World Trade Organization managed to hammer
d) 3 and 4 out a face-saving deal and India played a vital
e) None of these role at the Geneva ministerial conference,

Click Here For Bundle PDF Course | support@guidely.in Page 4 of 11


SBI Clerk & RRB PO Mains PDF Course 2023
ENGLISH Day - 29

thereby keeping faith in trade multilateralism (B) The Houthis asked that the blockade(1) of
alive. Sanaa airport and Hodeidah port be eased and
1. essential the ceasefire(2) to pay the salaries from the
2. crucial country’s oil revenues.
3. vital (C) The Houthis are also seeking “compensation”
4. exuberant from the Saudis for war damage; the latter are
a) 1 and 2 willing to consider contributing to reconstruction,
b) 2 and 3 but baulk at the idea of “compensation”.
c) 2 and 4 (D) Challenges coming in the way of the peace
d) 3 and 4 process/A include the Houthi insists that the
e) None of these kingdom pay the salaries of all/B government
officials, including armed forces personnel/C, for
15) Article 21 protects the right to life and liberty, the last few years from Yemen’s oil revenues/D.
which also encompasses the right to education No error/E
and the right to exercise one’s freedom to access (E) Some details of the talks soon became
the Internet. public: building on the year-long kingdom(3), the
1. encompasses two sides agreed to a six-month truce(4), to be
2. utilizes followed by talks over three months to agree on a
3. denies two-year “transition” period.
4. distinguishes 16) Which of the following is the correct
a) 1 and 2 sequence that makes a coherent passage ?
b) 1 and 4 a) ABDEC
c) 2 and 3 b) AEBDC
d) 2 and 4 c) AECDB
e) None of these d) DECAB
e) DBAEC
Directions (16-20) : Rearrange the following five
sentences (A), (B), (C), (D) and (E) in the proper 17) Swap the words (1), (2), (3) and (4) to make
sequence to form a coherent paragraph and then the sentences correct and meaningful.
answer the questions given below. a) 1-3
(A) In April this year, just as the Saudi-led war in b) 1-4
Yemen completed eight years, a diplomatic c) 2-3
delegation from the kingdom reached Sanaa d) 2-4
airport for talks with its Houthi enemies. e) No changes required

Click Here For Bundle PDF Course | support@guidely.in Page 5 of 11


SBI Clerk & RRB PO Mains PDF Course 2023
ENGLISH Day - 29

b) B
18) Choose the odd one out based on the c) C
pattern followed. d) D
a) AC e) E
b) BE
c) EA 20) Which of the following is the antonym of the
d) CD word ‘baulk’ highlighted in sentence C ?
e) DE a) resist
b) refuse
19) Find the erroneous part in sentence D and c) hesitate
choose that as your answer. d) welcome
a) A e) flinch
Click Here to Get the Detailed Video Solution for the above given Questions
Or Scan the QR Code to Get the Detailed Video Solutions

Answer Key with Explanation

1. Answer: B Similarly in sentence II - The landslides and


Swap the words ‘reported’ and corrective to floods have been reported(a meaningful
make both the sentences I and II contextually sentence).
correct and meaningful. Words B, C , E and F are in the right place and
I - Phrase ‘ISRO took reported’ is wrong need no replacement.
because two consecutive past tense words are The sentences :
present. Rather the sentence needs an adjective The ISRO took corrective actions after
in the place of (A). conducting an in-depth analysis of the fault and
Q-what actions ?, A-corrective actions. carried out a successful launch of the SSLV in
February.

Click Here For Bundle PDF Course | support@guidely.in Page 6 of 11


SBI Clerk & RRB PO Mains PDF Course 2023
ENGLISH Day - 29

Thirteen landslides and nine flash floods have Notorious - well known for something bad
been reported in the State in the past 36 hours, Exhorted - encourage
according to the State emergency operation Extorted - to threaten someone and get
centre. something from them
Strive - work very hard
2. Answer: D I - The criminal extorted money (forced and got
Swap the words C and F (judged and compelled) money)
to make the given two sentences correct and II - The coach exhorted the team (encouraged
meaningful. the team)
I - The government might be compelled(forced) The other words A, B, E and F are in correct
to take loans - ‘judged’ does not add meaning to positions and add meaning to the respective
this sentence and hence inappropriate sentence, hence no swapping is needed.
II - Rita was afraid due to the fear of being The sentences:
judged by the audience.(judged - to come to The police nabbed a notorious criminal who had
some conclusion/to have an opinion) been terrorising builders and extorted money
Only after swapping these two words do the from them for the past two years.
sentences sound right. The coach exhorted the team to give their best
Words A, B, D and E are correctly placed in the effort and never give up, inspiring them to push
right positions and need no replacement. through challenges and strive for victory.
The sentences :
The Minister said that the State was facing an 4. Answer: B
acute financial crunch and feared that the The words ‘opportunities’ and ‘trust’ must be
government might be compelled to take loans to replaced to make sentence I and II correct and
pay its employees. meaningful.
Rita was afraid to speak up during the meeting The words A, C, D and E are in the right place,
due to the fear of being judged by the young only B and F are misplaced and incorrect. Swap
audience. words B and F to make the given sentences
correct.
3. Answer: C The sentences:
Exhorted and extorted are two different words Every modern society depends on the trust in
with distinct meanings and they often cause the skills and ethics of a variety of institutions
confusion. such as schools and colleges.
Nab - to catch/arrest

Click Here For Bundle PDF Course | support@guidely.in Page 7 of 11


SBI Clerk & RRB PO Mains PDF Course 2023
ENGLISH Day - 29

The ability of a woman to do well does not rest Sentence ‘a’ perfectly fits the blank and it is
on whether it is a man’s world or not, because relevant to the context of the passage. Militaries
everyone has their own opportunities. account for 5.55 of global greenhouse gas
emission but they are not bound by any climate
5. Answer: C agreements.
Swap the words important and interesting to B - inappropriate because is it generic in nature,
make the I and II correct. it talks about aircraft and emission associate with
Janani lost an important document whereas it but does not have military or armed force
Maths is an interesting subject. reference in the sentence
Fathom - to try to understand something C - There will be no connection between this
Confess - to accept/to admit sentence and the second sentence in the given
The sentences : passage.
He was a good mathematician so he could not D - explains a method for consumption of energy
fathom why other people were stressed about - not related to the context of the passage
such an interesting and easy subject. Hence we go for option ‘a’ as our answer.
Janani lost an important document and rather
than confessing her mistake she blamed her 8. Answer: C
friend for losing it. Statement ‘c’ completes the passage in the best
possible way because the sentences before and
6. Answer: D after the blank it talking about the distance the
Even though all the given four sentences revolve spacecrafts should land or have landed before
around the ideology of bill amendment, the most from the lunar equator, so this particular
appropriate sentence is ‘d’ because look at the sentence which also discusses about the
sentence which succeeds the blank - it talks spacecrafting landing and the distance from the
about the act in detail and it has got a phrase ‘it lunar equator will be the right fit.
also extends’ which means the previous Other sentences do not act as an intermediate
sentence should also talk the act in detail. sentence connecting both the sentences that are
Only ‘d’ talks about the act and its before and after the blank.
empowerment. The other sentences do not fit Hence, we go for option c.
the blank as they break the flow of the passage.
So, we go for option d as our answer. 9. Answer: D
The sentence that completes the given passage
7. Answer: A is sentence ‘d’. All the options given seem to be

Click Here For Bundle PDF Course | support@guidely.in Page 8 of 11


SBI Clerk & RRB PO Mains PDF Course 2023
ENGLISH Day - 29

a concluding sentence but if we carefully look at The Food Corporation of India mainly aims to
the context of the passage it talks about the support the grain producers by acquiring wheat
open source model, the concluding sentence and rice at minimum support prices.
would also speak about the open source model The Food Corporation of India mainly aims to
rather than the transparency, iterative process or support the grain producers by procuring wheat
the approach. and rice at minimum support prices.
Hence, d is the right option.
12. Answer: C
10. Answer: A Endurance and acceptance both fit the given
Sentence ‘a’ completes the given passage in the blank.
correct way. Endurance - to tolerate something
Foundation for Environment Education is Acceptance can also fill the given blank as it
providing these blue flags to beaches - this gives a similar meaning to the sentence
sentence is continued by sentence ‘a’ by Tolerance does not fit the blank because the
explaining the criterias to obtain the eco label. blank is preceded by ‘an’
The other sentences are related to the topic in Importance is incorrect as it does not make the
one or the other way but they do not fit the given sentence meaningful
blank in the passage to complete it. The sentences:
The project ran into rough weather, among other
11. Answer: A issues over one of the specifications mentioned,
correct answer is option a - acquiring and that the submarine on offer should have a
procuring(both the words have a similar proven module with an endurance of two weeks.
meaning) The project ran into rough weather, among other
Acquire - to obtain or buy something issues over one of the specifications mentioned,
Procure - to obtain something that the submarine on offer should have a
Endorsing - to support publicly(this does not proven module with an acceptance of two
make sense to the context of the given weeks.
sentence)
Eliminating - is incorrect as it does not add 13. Answer: D
meaning to the sentence Both the words ‘helpless’ and ‘stranded’ have a
The sentences: similar meaning and fit the blank correctly.
Stranded - being left somewhere unpleasant

Click Here For Bundle PDF Course | support@guidely.in Page 9 of 11


SBI Clerk & RRB PO Mains PDF Course 2023
ENGLISH Day - 29

Rescued and saved are the antonyms of the thereby keeping faith in trade multilateralism
word stranded - and they do not fit the blank as alive.
they convey an incorrect meaning.
The sentences: 15. Answer: E
Heavy rains that triggered landslides have left Encompasses is the only word that fits the given
hundreds stranded as several roads have been blank in the proper way, the other words are
damaged due to landslides, adversely impacting inappropriate/incorrect. Hence, we go for option
vehicular movement in the state. e as our answer.
Heavy rains that triggered landslides have left The sentence:
hundreds helpless as several roads have been Article 21 protects the right to life and liberty,
damaged due to landslides, adversely impacting which also encompasses the right to education
vehicular movement in the state. and the right to exercise one’s freedom to
access the Internet.
14. Answer: B
All the four words are similar in meaning and the 16. Answer: B
synonyms of the word ‘important’, but looking at AEBDC is the right sequence that forms a
the preposition used ‘a’ we can eliminate the coherent passage.
words 1 and 4 as they start with a vowel. The correct sequence is AEBDC.
So the left out words are crucial and vital - both The passage starts with sentence ‘A’ being the
fit the given blank in the best possible way and introductory sentences, all the other four
are correct. sentences are a continuation and does not have
The sentences: any introduction. Sentence ‘E’ is the second
In June 2022, the member-countries of the sentence because it has more details about the
World Trade Organization managed to hammer talks which have been mentioned in sentence A.
out a face-saving deal and India played a vital So, AE is a pair and E is a consecutive sentence
role at the Geneva ministerial conference, to A. Third sentence would be ‘B’ as it talks
thereby keeping faith in trade multilateralism about the demands the Houthis seek from the
alive. Saudis. This should be followed by sentence ‘D’
In June 2022, the member-countries of the as it exaggerates ‘paying the salaries from the
World Trade Organization managed to hammer country’s oil revenues’ as a big problem or
out a face-saving deal and India played a crucial challenge. Finally the passage ends with
role at the Geneva ministerial conference, sentence ‘C’ which talks about the Houthis’s
additional demand from Saudis.

Click Here For Bundle PDF Course | support@guidely.in Page 10 of 11


SBI Clerk & RRB PO Mains PDF Course 2023
ENGLISH Day - 29

17. Answer: C AC - In the correct sequence C is four places


Swap the words 2 and 3 to make both the away A(starting from c towards the right). Similar
sentences B and E correct. pattern is followed in BE, EA and CD
The word ceasefire is inappropriate in sentence DE - odd one out as E is three places away from
B, similarly the word kingdom is D (starting from D towards right)
incorrect/inappropriate in sentence E. Swap
them to make the sentences meaningful and 19. Answer: B
correct. The error is in part B of the given sentence.
1 and 4 are in correct positions and hence do not Replace ‘insists’ with ‘insisting’ to make the
need any replacement. given sentence correct.
The Houthis asked that the blockade of Sanaa The sentence is in present tense so ‘insisting’ is
airport and Hodeidah port be eased and the the correct word.
kingdom to pay the salaries from the country’s oil The correct sentence : Challenges coming in the
revenues. way of the peace process include the Houthi
insisting that the kingdom pay the salaries of all
Some details of the talks soon became public: government officials, including armed forces
building on the year-long ceasefire, the two sides personnel, for the last few years from Yemen’s
agreed to a six-month truce, to be followed by oil revenues.
talks over three months to agree on a two-year
“transition” period. 20. Answer: D
Welcome is the word which is opposite in
18. Answer: E meaning.
DE is the odd one out here. Baulk - not agreeing to do something
The pattern followed is : Resist, refuse, hesitate and flinch are the
The correct sequence is AEBDC. synonyms of the word baulk.
Flinch - to avoid doing something

Click Here For Bundle PDF Course | support@guidely.in Page 11 of 11


SBI Clerk & RRB PO Mains PDF Course 2023
Reasoning Ability Day - 30 (Eng)

Reasoning Ability
Directions (1-5): Study the following information d) B
carefully and answer the given questions. e) Z
Twelve persons A, B, C, D, E, F, U, V, W, X, Y
and Z are sitting around a hexagonal shaped 2. What is the distance between X and the one
table in such a way that six persons sit in the who sits immediate right of E?
middle of the sides facing away from the centre a) 27.5m
and six persons sit at the corners of the table b) 275m
facing the centre. The perimeter of this c) 55m
hexagonal table is 330m. The distance between d) Either a or b
adjacent persons is equal. e) Either b or c
Note: The perimeter of the hexagon = 6*side
length of the hexagon 3. Who among the following person does not
D sits 82.5m to the left of V and sits adjacent to face towards the centre of the table?
B. F and D are immediate neighbours. A sits 55m a) X
to the right of the one who sits opposite to B. The b) The one who sits immediate left of A
distance between V and A is the same as the c) The one who sits third to the left of V
distance between F and Z, who does not sit d) Y
opposite to V. The distance between C and the e) The one who sits opposite to F
one who sits opposite to Z is 27.5m. The
distance between C and X is 110m. W sits 55m 4. If all the persons who are sitting opposite to
to the right of X. The number of persons sitting each other are interchanged in their positions,
between W and D is two more than the number then who among the following sits second to the
of persons sitting between X and Y, when right of V?
counted from the right of both W and Y. U and Y a) C
are not immediate neighbours. E neither sits at b) F
the corner of the table nor sits fifth to the right of c) E
U. d) Z
1. Who among the following person sits third to e) X
the right of the one, who sits 55m to the right of
W? 5. In which of the following option, the second
a) The one who sits opposite to U person sits immediate left of the first person?
b) D I. BD
c) The one who sits immediate left of C II. UZ

Click Here For Bundle PDF Course | support@guidely.in Page 1 of 12


SBI Clerk & RRB PO Mains PDF Course 2023
Reasoning Ability Day - 30 (Eng)

III. AU toys made by P, who made twice the number of


a) Only I toys made by U, then what is the sum of the
b) Only II number of toys made by T and U?
c) Both I and II a) 55
d) Both II and III b) 64
e) All I, II and III c) 90
d) 65
Directions (6-8): Study the following information e) Cannot be determined
carefully and answer the given questions.
Nine persons- P, Q, R, S, T, U, V, W and X made 8. If the sum of the number of toys made by S
different number of toys in a day. V made more and X is five more than four times the number of
toys than both Q and W. X made more toys than toys made by W, then what is the total number of
P. U made less toys than W but did not make the toys made by W and V?
least number of toys. T did not make more toys a) 110
than P. The sum of the number of toys made by b) 90
S and P is twice the number of toys made by X. c) 120
S made 30 toys more than P. V made 70 toys, d) 100
which is 25 less than the number of toys made e) Cannot be determined
by X. The number of persons made less toys
than S is two more than the number of persons Directions (9-10): Study the following information
made more toys than W. V made just more toys carefully and answer the given questions
than T. R didn’t make more toys than Q. Eight persons A, B, C, D, E, F, G and H solved
6. If the average of the toys made by P, T, Q and different number of questions. B solved less
V is 65, then what is the ratio of the total number questions than A and H, who doesn’t solve the
of toys made by T and Q to the number of toys highest number of questions. C solved less
made by X? questions than G. F solved 40 questions more
a) 12:17 than A. The sum of the number of questions
b) 33:13 solved by F and A is 760. E solved more
c) 21:25 questions than B but less questions than H. G
d) 22:19 solved less questions than B. D solved more
e) 27:8 questions than F. The number of persons solved
less questions than B is one less than the
7. If the difference between the number of toys number of persons solved more questions than
made by V and T is one fourth of the number of H.

Click Here For Bundle PDF Course | support@guidely.in Page 2 of 12


SBI Clerk & RRB PO Mains PDF Course 2023
Reasoning Ability Day - 30 (Eng)

9. How many questions may be solved by D? III. If an odd number (but not a prime number) is
a) 450 followed by an even number, then the difference
b) 420 between both numbers is obtained. /F>C
c) 380 IV. If an odd number (but not a perfect square) is
d) Either a or b followed by another odd number, then the unit
e) Either a or c digit of the two numbers are to be multiplied.
/D>B
10. If the sum of the number of questions solved V. If a prime number is followed by an even
by F and H is 720 and the difference between number, then the unit digit of the two numbers
the number of questions solved by A and E is 60, are to be added. /G>A
then what is the sum of the number of questions VI. If a perfect square number is followed by an
solved by H and E? odd number, then divide the first number by the
a) 640 second number. /E>C
b) 620 11. What is the sum of the resultant of the two
c) 580 rows?
d) 520 24 32 7
e) None of these 225 5 28
D <L< E ≤ F < R; A ≥ B > Q = R; D> M > C ≤ T
Directions (11-15): In each of the following a) 425
questions, two rows of numbers are given. The b) 432
resultant number in each row is to be worked out c) 409
separately based on the following Conditions and d) 450
the questions below the rows of numbers are to e) Cannot be determined
be answered. The operation of numbers
progresses from left to right. The given 12.What is the difference between the resultant
conditions are applicable only when the of the two rows?
inequality conclusions are definitely true. 27 35 74
Conditions: 49 7 21
I. If an even number is followed by an odd J>G = T ≥ D = F; Q< B = E ≤ S<F; C ≤ N ≤ Q
number, then both numbers are to be multiplied. a) 32
/A>D b) 44
II. If an even number is followed by another even c) 56
number, then both numbers are to be added. d) 67
/B>E e) Cannot be determined

Click Here For Bundle PDF Course | support@guidely.in Page 3 of 12


SBI Clerk & RRB PO Mains PDF Course 2023
Reasoning Ability Day - 30 (Eng)

13. Find the value of X+Y? 21 persons are working in a company in different
X = 17 28 54 designations such as Director, CEO, President,
Y = 12 5 26 General Manager (GM), Supervisor, and Clerk
F ≤ M < G ≤ X; F > B ≥ A = N > C; E < D ≤ C where Director is the senior most designation
a) 150 and Clerk is the junior most designation.
b) 125 Note I: The number of persons working in each
c) 120 designation is one less than the number of
d) 115 persons working in their immediate junior
e) Cannot be determined designation.
Note II: If “A” works with “B”, then A and B are
14. If the sum of the resultants of the two rows is working in the same designation.
440. Then find the value of X. D is senior to the General Manager. Only two
23 48 35 designations are between D and F. Only one
12 34 X designation is between F and B. As many
B > G ≥ Y ≥ M; E< T ≤ A = D < M; U ≥ I >E designations higher than B as lower than I. M
a) 98 works with I. Q is working in the designation
b) 123 which is immediately lower to M. A and K are
c) 76 working with Q. The difference between the
d) 86 number of persons working with A and E is two.
e) Cannot be determined C is senior to J but junior to E. N and R are junior
to C, who is not working as General Manager. L
15. Find the value of X*Y? and O are working with S. P and S is not working
81 9=X in the same designation. H is senior to P but not
33 38 = Y senior to G, who is not working in the senior most
M ≤ F < R ≤ E; M > B ≥ H = T < N; J < C ≤ T designation. T is junior to U. T works as neither
a) 81 General Manager nor supervisor.
b) 75 All the persons except the persons who are
c) 105 senior to the president are transferred to three
d) 45 departments viz. Production, Marketing and
e) Cannot be determined Sales departments based on the below
conditions:
Directions (16-20): Study the following I. Among the persons who are junior to CEO, the
information carefully and answer the given persons whose name starts with the letter which
questions is adjacent to the vowels (as per the English

Click Here For Bundle PDF Course | support@guidely.in Page 4 of 12


SBI Clerk & RRB PO Mains PDF Course 2023
Reasoning Ability Day - 30 (Eng)

alphabetical series) are transferred to the form a group. Which one of the following does
production department and the persons whose not belong to the group?
names start with a vowel are transferred to a) NQ
Marketing department. b) LM
II. Among the persons who are junior to the c) DA
General manager, the persons whose name d) CH
starts with a consonant that comes after N (as e) SN
per English alphabetical series) is transferred to
the sales department and the persons whose 18. Who among the following is the head of sales
name starts with a consonant that comes before department?
N (as per English alphabetical series) is a) The one who is the Director
transferred to the Marketing department. b) A
III. Among the remaining persons, the person c) The one who works with G
whose name starts with the letter, which place d) J
value is a prime number as per the English e) N
alphabetical series are transferred to the sales
department and the remaining persons are 19. What is the difference between the number
transferred to the marketing department. of persons working in Production and Sales
The persons who are senior to the President are department?
appointed as the head of the three department a) 1
viz. Production, Marketing and Sales b) 2
departments as per the English alphabetical c) 3
order respectively. d) 4
16. The number of persons working with ____ is e) 5
two less than the number of persons working
with _____respectively. 20. The number of designations higher than
a) M, Q ______ is one more than the number of
b) P, N designations lower than _____.
c) G, K a) E, A
d) U, E b) D, M
e) C, A c) L, N
d) A, H
17. Four of the following five are alike in a certain e) I, O
way based on the given arrangement and thus

Click Here For Bundle PDF Course | support@guidely.in Page 5 of 12


SBI Clerk & RRB PO Mains PDF Course 2023
Reasoning Ability Day - 30 (Eng)

Click Here to Get the Detailed Video Solution for the above given Questions
Or Scan the QR Code to Get the Detailed Video Solutions

Answer Key with Explanation

Directions (1-5):
1. Answer: C
2. Answer: E
3. Answer: E
4. Answer: D
5. Answer: A
Final arrangement:

Again, we have
 A sits 55m to the right of the one who sits
opposite to B.
 The distance between V and A is the
same as the distance between F and Z,
who does not sit opposite to V.
We have,  The distance between C and the one who
 D sits 82.5m to the left of V and sits sits opposite to Z is 27.5m.
adjacent to B.
 F and D are immediate neighbours.
From the above conditions, there are four
possibilities,

Click Here For Bundle PDF Course | support@guidely.in Page 6 of 12


SBI Clerk & RRB PO Mains PDF Course 2023
Reasoning Ability Day - 30 (Eng)

Directions (6-8):
Again, we have
Final arrangement:
 The distance between C and X is 110m.
 W sits 55m to the right of X.
We have,
 The number of persons sits between W
 V made more toys than both Q and W.
and D is two more than the number of
 U made less toys than W but does not
persons sits between X and Y, when
make the least number of toys.
counted from the right of both W and Y.
V>Q/W
 U and Y are not immediate neighbours.
W>U>__
 E neither sits at the corner of the table nor
 X made more toys than P.
sits fifth to the right of U.
 T did not make more toys than P.
After applying the above conditions case-1,
X>P>T
case-2 and case-2a get eliminated because E sit
 S made 30 toys more than P.
at the corner of the table in case-2 and case-2a
 The sum of the number of toys made by S
and then E sits fifth to the right of U in case-1,
and P is twice the number of toys made
hence case-1a shows the final arrangement.
by X.
S = P + 30
S+P=2X
 V made 70 toys, which is 25 less than the
number of toys made by X.
 V made just more toys than T.

Click Here For Bundle PDF Course | support@guidely.in Page 7 of 12


SBI Clerk & RRB PO Mains PDF Course 2023
Reasoning Ability Day - 30 (Eng)

 The number of persons made less toys Then the total number of toys made by W and V
than S is two more than the number of is 50+70=120
persons made more toys than W.
 R didn’t make more toys than Q. Directions (9-10):
V=70; X=95 Final arrangement:
S+P=2X
S+P=2(95); S+P=190; x+x-30=190; 2x=220; We have,
S=110 and P=80  B solved less questions than A and H,
S (110)>X (95)>P (80)>V (70)>T>Q>W>U>R who doesn’t solve the highest number of
6. Answer: D questions.
S (110)>X (95)>P (80)>V (70)>T>Q>W>U>R  C solved less questions than G.
The average of the toys made by P, T, Q and V A, H>B
is 65 80+Q+T+70=65*4=260; Q+T=110 G>C
Then the ratio of the total number of toys made  F solved 40 questions more than A.
by T and Q to the number of toys made by X is -  The sum of the number of questions
>110:95->22:19 solved by F and A is 760.
F (400)>A (360)
7. Answer: C  G solved less questions than B.
S (110)>X (95)>P (80)>V (70)>T>Q>W>U>R B>G
The difference between the number of toys D solved more questions than F.
made by V and T is one fourth of the number of D>F
toys made by P then 70-T=1/4*80 ->T=50  The number of persons solved less
P made twice that of the number of toys made questions than B is one less than the
by U ->P=80 then U=40 number of persons solved more questions
Then the sum of the number of toys made by T than H.
and U is 50+40=90  E solved more questions than B but less
question than H.
8. Answer: C After combining above all statements, we get
S (110)>X (95)>P (80)>V (70)>T>Q>W>U>R I. D>F (400)>A (360)>H>E>B>G>C
The sum of the number of toys made by S and X II. D>F>H>A>B>G>C
is five more than the four times the number of III. D>H>F>A>B>G>C
toys made by W -> 110+95=5+4(W) - 9. Answer: D
>205=5+4W; W=50

Click Here For Bundle PDF Course | support@guidely.in Page 8 of 12


SBI Clerk & RRB PO Mains PDF Course 2023
Reasoning Ability Day - 30 (Eng)

10. Answer: B
D>F (400)>A (360)>H>E>B>G>C 13. Answer: B
The sum of the number of questions solved by F F ≤ M < G ≤ X; F > B ≥ A = N > C; E < D ≤ C
and H is 720; 400+H=720; H=320 7+8=15 (Condition V)
The difference between the number of questions G>A (G>M≥F>B≥A) -> True
solved by A and E is 60; 360-E=60; E=300 54-15=39 (Condition III)
Then the sum of the number of questions solved F>C (F>B≥A=N>C) -> True
by H and E 320+300=620 12*5=60 (Condition I)
A>D (A=N>C≥D) -> True
Directions (11-15): 60+26=86 (Condition II)
11. Answer: C B>E (B≥A=N>C≥D>E) -> True
D <L< E ≤ F < R; A ≥ B > Q = R; D> M > C ≤ T 39+86 = 125
24+32=56 (Condition II)
B>E (B>Q=R>F≥E) -> True 14. Answer: E
56*7=392 (Condition I) -> True B > G ≥ Y ≥ M; E< T ≤ A = D < M; U ≥ I >E
A>D (A≥B>Q=R>F≥E>L>D) 8+3=11 (Condition V)
225/5=25 (Condition VI) -> True G>A (G ≥ Y ≥ M>D=A) -> True
E>C (E>L>D>M>C) 1*5=5 (Condition IV)
45-28=17 (Condition III) D>B (B>G≥Y≥M>D) -> False
F>C (F≥E>L>D>M>C) -> True We cannot find the resultant for the first row
392+17=409 because the conclusion (D>B) is false. So the
answer cannot be determined.
12. Answer: A
J>G = T ≥ D = F; Q< B = E ≤ S<F; C ≤ N ≤ Q 15. Answer: D
7*5=35 (Condition IV) M ≤ F < R ≤ E; M > B ≥ H = T < N; J < C ≤ T
D>B (D=F>S≥E=B) -> True 81/9 = 9 (Condition VI)
74-35=39 (Condition III) E>C (E≥R>F≥M>B≥H=T≥C) -> True
F>C (C≤N≤Q<B=E≤S<F) -> True 38-33=5 (Condition III)
49/7=7 (Condition VI) F>C (F≥M>B≥H=T≥C) -> True
E>C (C≤N≤Q<B=E) -> True 5*9 = 45
7*1=7 (Condition IV)
D>B (D=F>S≥E=B) -> True Directions (16-20):
39-7=32 16. Answer: C

Click Here For Bundle PDF Course | support@guidely.in Page 9 of 12


SBI Clerk & RRB PO Mains PDF Course 2023
Reasoning Ability Day - 30 (Eng)

17. Answer: D (All the pair of persons are  M works with I.


working in different designations except option d)  Q is working in the designation which is
18. Answer: A immediately lower to M.
19. Answer: C  A and K are working with Q.
20. Answer: E After applying the above conditions case-4 gets
Final arrangement: eliminated because two persons are working as
Director.

Again, we have
 The difference between the number of
persons working with A and E is two.
 C is senior to J but junior to E.
We have,  N and R are junior to C, who is not
 D is senior to the General Manager. working as the General Manager.
 Only two designations are between D and  L and O are working with S.
F. 
 Only one designation is between F and B.
From the above conditions, we have four
possibilities,

Again, we have Again, we have


 As many designations higher than B as  S and P are not working in the same
lower than I. designation.

Click Here For Bundle PDF Course | support@guidely.in Page 10 of 12


SBI Clerk & RRB PO Mains PDF Course 2023
Reasoning Ability Day - 30 (Eng)

 H is senior to P but not senior to G, who is I, A, O are transferred to the Marketing


not working in the senior most department
designation. II. Among the persons who are junior to the
 T is junior to U. General manager, the persons whose name
 T is neither the General manager nor the starts with a consonant that comes after N (as
supervisor. per English alphabetical series) is transferred to
After applying the above conditions case-2 and the sales department and the persons whose
case-3 get eliminated because T works as name starts with a consonant that comes before
General Manager in case-2 and Supervisor in N (as per English alphabetical series) is
case-3. Thus, case -1 gives the final transferred to the Marketing department.
arrangement. From the above conditions, S and R are
transferred to the sales department and C and L
are transferred to the Marketing department.
III. Among the remaining persons, the person
whose name starts with the letter, which place
value is a prime number as per the English
alphabetical series are transferred to the sales
department and the remaining persons are
transferred to the marketing department.
From the above conditions, M, Q and K are
All the persons except the persons who are
transferred to the sales department and nobody
senior to president are transferred to three
is transferred to the marketing department.
department viz. Production, Marketing and Sales
The persons who are senior to the President are
departments based on the below conditions:
appointed as the head of the three departments
I. Among the persons who are junior to CEO, the
viz. Production, Marketing and Sales
persons whose name starts with the letter which
departments as per the English alphabetical
is adjacent to the vowels (as per English
order respectively.
alphabetical series) are transferred to the
From the above conditions, E, G and U are
production department and the persons whose
appointed as the head of Production, Marketing
names start with a vowel are transferred to
and Sales departments respectively.
Marketing department.
From the above conditions, D, B, H, F, J, N, P, T
are transferred to the production department and

Click Here For Bundle PDF Course | support@guidely.in Page 11 of 12


SBI Clerk & RRB PO Mains PDF Course 2023
Reasoning Ability Day - 30 (Eng)

Click Here For Bundle PDF Course | support@guidely.in Page 12 of 12


SBI Clerk & RRB PO Mains PDF Course 2023
Quantitative Aptitude Day – 30 (Eng)

Quantitative Aptitude

Directions (1 – 5): Study the given information carefully and answer the given questions.
There are 5 different electronic stores A, B, C, D, and E, which sold fans of two different brands – Havells
and Osaka. The first pie chart shows the degree of distribution of the total number of fans by five stores.
The second pie chart shows the percentage distribution of the total number of Havells fans sold by five
stores.

Note:
i. 7, 9, m, 17, 24, 35
ii. n2 + 3n – 108 = 0, n is a positive number.

Click Here For Bundle PDF Course | support@guidely.in Page 1 of 17


SBI Clerk & RRB PO Mains PDF Course 2023
Quantitative Aptitude Day - 30 (Eng)

1. If the sale of Havells fans by store A is 1600 per unit. Find the total amount earned by
increased by a% and the sale of Osaka fans by store D by selling all his fans at 25% profit.
store A is increased by b% then the total sales of a. 200000
the fan by store A become 2370. Find the correct b. 2400000
relation c. 250000
(Note: a is half of b) d. 2000000
i. (5/4) a = 2b – 11 * 5 e. None of these
ii. 11a + 20 = 8b – 60
iii. 1.75 b = 2a + 30 4. Store B sells 60% of the fans he has. If the
a. Only i is correct ratio of Havells fans to Osaka fans that store B
b. Only ii and iii are correct has is 5 : 3. Find the unsold Havells fans of store
c. Only i and iii are correct B is what percent of the unsold Osaka fans of
d. All are correct store B.
e. None of these a. 77.77%
b. 55.55%
2. Havells fan consumes 2 units of electricity per c. 72.5%
hour and Osaka fan consumes 3 units of d. 66.66%
electricity per hour. A hospital building has 24 e. None of these
Havells fans and 32 Osaka fans. Find the
electricity units consumed by all fans of the 5. The difference between the Havells fans sold
hospital in a week. by store A and the Osaka fans sold by store A is
(Note: All fans remain On on all days and nights.) what percent of the difference between the
a. 24192 units Havells fans sold by store B and the Osaka fans
b. 25162 units sold by store D.
c. 28292 units a. 41.44%
d. 22142 units b. 21.42%
e. None of these c. 25%
d. 20%
3. The cost price of a Havells fan is Rs. 1200 per e. None of these
unit, and the cost price of an Osaka fan is Rs.

Directions (6 – 10): Study the given information carefully and answer the given questions.
There are five Ice – cream parlours – A, B, C, D, and E which sold ice creams of three flavours –
Chocolate, Vanilla, and Strawberry. The line graph given below shows the average number of three types

Click Here For Bundle PDF Course | support@guidely.in Page 2 of 17


SBI Clerk & RRB PO Mains PDF Course 2023
Quantitative Aptitude Day - 30 (Eng)

of ice creams sold by five ice cream parlours. The table given below shows the ratio of Chocolate flavour
ice creams sold to the Vanilla flavour ice creams sold and the difference between the Strawberry flavour
and Vanilla flavour ice creams sold by five ice cream parlour.

Note: The number of Strawberry ice creams sold by each parlour is more than the number of Vanilla ice
creams sold by each parlour.
6. To make a mixed cup of ice cream an ice Vanilla ice cream in the cup is what percent of
cream vendor takes Chocolate, Vanilla, and the total ice cream in the ice cream cup.
Strawberry ice creams in the ratio k : 3 : 5. The a. 25%
total quantity of ice cream in the cup is 200 ml. b. 33.33%
The quantity of Chocolate ice cream in the ice c. 30 %
cream cup is 33.33% less than the quantity of d. 16.66%
Vanilla ice cream in the ice cream cup. Find the e. None of these

Click Here For Bundle PDF Course | support@guidely.in Page 3 of 17


SBI Clerk & RRB PO Mains PDF Course 2023
Quantitative Aptitude Day - 30 (Eng)

7. If all ice creams sold by Carein 500 ml packs. 9. The difference between the Strawberry ice
The selling price of a Chocolate ice cream pack cream packs sold by A and B is what percentage
is Rs.200,a Vanilla ice cream pack is Rs.180, of the sum of the Chocolate ice cream packs
and a Strawberry ice cream pack is Rs.140. Find sold by B and the Vanilla ice cream packs sold
the average selling price of a 500 ml pack of ice by E together.
cream sold by C. a. 12%
a. 163.42 b. 8%
b. 169.74 c. 16%
c. 181.23 d. 9%
d. 159.23 e. None of these
e. None of these
10. The ratio of the Chocolate ice cream pack,
8. If 70% of the total ice cream packs that B has Vanilla ice cream pack, and Strawberry ice
are sold and the remaining packs are damaged. cream pack sold by ice cream parlour F is 5 : 4 :
The ratio of the Chocolate ice cream pack, 7. The chocolate ice cream packs sold by parlour
Vanilla ice cream pack, and Strawberry ice F are20% more than the chocolate ice cream
cream pack that B has is 5 : 3 : 4. Find what packs sold by parlour A. Find the difference
percent of the Chocolate ice cream pack that B between the number of Strawberry ice cream
has damaged. packs sold by parlour F and the Strawberry ice
a. 40% cream pack sold by parlour A.
b. 75% a. 210
c. 15% b. 200
d. 60% c. 180
e. None of these d. 175
e. None of these

Directions (11 – 13): Study the given information carefully and answer the given questions.
A train has four types of coaches – General, Sleeper, 2nd AC, and 1st AC. Different numbers of
passengers are travelling in these coaches on different days of the week. The table given below shows
the sum of passengers travelling in General coaches and 1st AC coaches, the difference between the
number of passengers travelling in General coaches and Sleeper coaches, the difference between the
number of passengers travelling in Sleeper coaches and 2nd AC coaches, and the difference between the
number of passengers travel in 2nd AC coaches and 1st AC coaches.

Click Here For Bundle PDF Course | support@guidely.in Page 4 of 17


SBI Clerk & RRB PO Mains PDF Course 2023
Quantitative Aptitude Day - 30 (Eng)

Note:-
The number of passengers ina general coach is always greater than the number of passengers in sleeper
a coach.
The number of passengers in a sleeper coach is always greater than the number of passengers in the 2 nd
AC coach.
The number of passengers in the 2nd AC coach is always greater than thenumber of passengers in the 1st
AC coach
11. The ratio of per person ticket price for 12. The total number of seats in a General coach
General coach, Sleeper coach, 2nd AC coach, is 90, in a Sleeper coach is 70, in a 2nd AC coach
and 1st AC coach is 2 : 5 : 9 : 13 . Themoney is 50, and in a 1st Ac coach is 20. Find the total
collected on Wednesday from 2nd AC coach number of coaches on the train on Friday.
passengers is Rs. 99974 more than the money a. 43
collected on Wednesday from General coach b. 37
passengers. Find the total money collected on c. 35
Wednesday from all passengers. d. 29
a. 424814 e. None of these
b. 432664
c. 442165 13. The table shows the data for only
d. 444148 passengers who bought tickets. If 80% of
e. None of these passengers on Tuesday in General Coach

Click Here For Bundle PDF Course | support@guidely.in Page 5 of 17


SBI Clerk & RRB PO Mains PDF Course 2023
Quantitative Aptitude Day - 30 (Eng)

bought tickets, then find the total number of The cost price of the Boat smartwatch is Rs.2200
passengers in General Coach on Tuesday. and it is sold at a profit of Rs.200. Cost price of
a. 580 the One Plus smartwatch is Rs.1000 more than
b. 630 the selling price of the Noise smartwatch. The
c. 540 marked price of the Noise smarty watch is
d. 520 Rs.1200 less than the marked price of the One
e. None of these Plus smartwatch.
14. If the shopkeeper doubles the discount on
Directions (14 – 17): Study the given information the marked price of the One Plus smartwatch.
carefully and answer the given questions. Find the loss percentage that occurred by selling
In a shop of watches, there are five brands of the One Plus smartwatch.
smartwatches – Boat, Noise, One Plus, Wave, a. 15%
and Fire Boltt. Cost prices, selling prices, and b. 10%
marked prices are different for each brand. c. 8%
The marked price of the Wave d. 12%
smartwatch is 33.33% more than the cost price e. None of these
of the Noise smartwatch. The marked price of the
Wave smartwatch is 100% more than the cost 15. If the marked price of a Fire Boltt smartwatch
price of the Wave smartwatch. One Plus is increased by Rs.350 and the discount
smartwatch is sold at adiscount of 12.5% of the percentage remains the same, then find the
marked price, which is 20% more than the cost percentage increase in the amount of profit by
price of the One Plus smartwatch. The cost price selling the Fire Boltt smartwatch.
of the Fire Boltt smartwatch is 50% more than a. 62.5%
the selling price of the Wave smartwatch. Wave b. 40%
smartwatch is sold at a profit of 25%. The cost c. 35%
price of the Fire Boltt smartwatch is equal to the d. 50%
marked price of the Boat smartwatch. The e. None of these
marked price of the Fire Boltt smartwatch is 50%
more than the selling price of the Noise 16. If Noise launches its new smartwatch, the
smartwatch. Noise smartwatch is sold at a profit cost price of which is 25% more than the old
of 16(2/3) %. The selling price of the Fire Boltt smartwatch. The ratio of the cost price of Noise’s
smartwatch is 20% more than the marked price new smartwatch to the marked price of Noise’s
of the Boat smartwatch. The boat smartwatch is new smartwatch is 3 : 4. Find the new discount
sold at a discount of 20% of the marked price.

Click Here For Bundle PDF Course | support@guidely.in Page 6 of 17


SBI Clerk & RRB PO Mains PDF Course 2023
Quantitative Aptitude Day - 30 (Eng)

percentage by which the profit remains the same a. Only ii and iii are correct
as the profit percent of the old Noise smartwatch. b. Only i and iii are correct
a. 11.11% c. Only i and ii are correct
b. 12.5% d. All are correct
c. 9% e. None of these
d. 12%
e. None of these 19. A can complete a task in ‘P’ days. B and C
together can complete the same task in 28 days,
17. Basant has 8 Wave smart watches, which he and A and C together can complete the task in
bought at selling price. How above he should 32 days. The efficiency of A is 20% less than the
mark the price of a watch on which, allowing a efficiency of B. Find the value of P.
discount of 15% he gets Rs.4400 as profit by a. 56
selling all the watches? b. 64
a. 3000 c. 52
b. 3300 d. 55
c. 2700 e. None of these
d. 3500
e. None of these 20. The total surface area of a cylinder of radius
‘k’ meters and height 42 meters is 4928 m2. Find
18. The cost price of a radio is Rs.800. A the volume of the cylinder.
shopkeeper marked it ____ % above the cost a. 25374 m3
price and offers a discount of ____ % on the b. 25864 m3
marked price to get a profit of Rs.160. c. 25872 m3
i. 50%, 20% d. 25776 m3
ii. 40%, 14.28% e. None of these
iii. 30%, 5%

Click Here For Bundle PDF Course | support@guidely.in Page 7 of 17


SBI Clerk & RRB PO Mains PDF Course 2023
Quantitative Aptitude Day - 30 (Eng)

Click Here to Get the Detailed Video Solution for the above given Questions
Or Scan the QR Code to Get the Detailed Video Solutions

Answer Key with Explanation

Directions (1 – 5): Total number of fans sold by store C = 8000 *


7, 9, m, 17, 24, 35 (45/360)
7+2=9 = 1000
9 + 3 = 12 Total number of fans sold by store D = 8000 *
12 + 5 = 17 (54/360)
17 + 7 = 24 = 1200
24 + 11 = 35 Total number of fans sold by store E = 8000 *
Value of m = 12 (72/360)
And, n2 + 3n – 108 = 0 = 1600
n2 + (12 – 9)n – 108 = 0 Number of Havells fans sold by store A = 5000 *
n2 + 12n – 9n – 108 = 0 (15/100)
n (n + 12) – 9 (n + 12) = 0 = 750
(n + 12) (n – 9) = 0 Number of Havells fans sold by store B = 5000 *
n = -12, 9 (36/100)
Value of n = 9 = 1800
Total number of fans sold by store A = 8000 * Number of Havells fans sold by store C = 5000 *
(81/360) (12/100)
= 1800 = 600
Total number of fans sold by store B = 8000 * Number of Havells fans sold by store D = 5000 *
(108/360) (16/100)
= 2400 = 800

Click Here For Bundle PDF Course | support@guidely.in Page 8 of 17


SBI Clerk & RRB PO Mains PDF Course 2023
Quantitative Aptitude Day - 30 (Eng)

Number of Havells fans sold by store E = 5000 * ii is incorrect.


(21/100) From iii,
= 1050 1.75 * 40 = 2 * 20 + 30
70 = 40 + 30
70 = 70
iii is correct.
Only i and iii are correct.

2. Answer: A
Units consume by Havells fan in 1 hour = 2
Units consume by Osaka fans in 1 hour = 3
1. Answer: C Number of Havells fans in the hospital = 24
New sales of Havells fans by store A = 750 * Number of Osaka fans in the hospital = 32
((100 + a)/100) Electricity units consumes by all fans in a week =
New sales of Osaka fans by store A = 1050 * 7 * 24 * (24 * 2 + 32 * 3)
((100 + b)/100) = 7 * 24 * 144
750 * ((100 + a)/100) + 1050 * ((100 + b)/100) = = 24192 units
2370
75000 + 750a + 105000 + 1050b = 237000 3. Answer: D
750a + 1050b = 57000 The cost price of a Havells fan = 1200
750a + 1050 * 2a = 57000 Selling price of a Havells fan = 1200 * (5/4)
2850a = 57000 = 1500
a = 20 The cost price of an Osaka fan = 1600
b = 2 * 20 = 40 Selling price of an Osaka fan = 1600 * (5/4)
From i, = 2000
(5/4) * 20 = 40 * 2 – 11 * 5 Amount get by store D by selling Havells fans =
25 = 80 – 55 800 * 1500
25 = 25 = 1200000
i is correct. Amount get by store D by selling Osaka fans =
From ii, 400 * 2000
11 * 20 + 20 = 8 * 40 – 60 = 800000
220 + 20 = 320 – 60 Total amount get by store D by selling all fans =
240 = 260 1200000 + 800000

Click Here For Bundle PDF Course | support@guidely.in Page 9 of 17


SBI Clerk & RRB PO Mains PDF Course 2023
Quantitative Aptitude Day - 30 (Eng)

= 2000000
Directions (6 – 10):
4. Answer: A Total number of ice creams sold by ice cream
Total number of fans sold by store B = 2400 parlour A = 190 * 3
Total number of fans store B has = 2400 * = 570
(100/60) Total number of ice creams sold by ice cream
= 4000 parlour B = 210 * 3
Number of Havells fans store B has = 4000 * = 630
(5/8) Total number of ice creams sold by ice cream
= 2500 parlour C = 390 * 3
The number of Osaka fans store B has = 4000 – = 1170
2500 Total number of ice creams sold by ice cream
= 1500 parlour D = 180 * 3
Number of unsold Havells fans of store B = 2500 = 540
– 1800 Total number of ice creams sold by ice cream
= 700 parlour E = 420 * 3
Number of unsold Osaka fans of store B = 1500 = 1260
– 600 Let the number of Chocolate ice cream sold by
= 900 ice cream parlour A be 5a and the Vanilla ice
Required percentage = (700/900) * 100 cream sold by ice cream parlour A be 2a.
= 77.77% Then, the Number of Strawberry ice cream sold
by ice cream parlour A = 2a + 120
5. Answer: B 5a + 2a + 2a + 120 = 570
Difference between the number of Havells fans 9a = 450
and Osaka fans sold by store A = 1050 – 750 a = 50
= 300 Number of Chocolate ice cream sold by ice
Difference between the number of Havells fans cream parlour A = 5 * 50
sold by store B and the number of Osaka fans = 250
sold by store D = 1800 – 400 Number of Vanilla ice cream sold by ice cream
= 1400 parlour A = 2 * 50
Reqd. percentage = (300/1400) * 100 = 100
= 300/14 Number of Strawberry ice cream sold by ice
= 21.42% cream parlour A = 100 + 120

Click Here For Bundle PDF Course | support@guidely.in Page 10 of 17


SBI Clerk & RRB PO Mains PDF Course 2023
Quantitative Aptitude Day - 30 (Eng)

= 220 Number of Strawberry ice cream sold by ice


Similarly, Let the number of Chocolate ice cream cream parlour C = 420 + 30
sold by ice cream parlour B be 3b and the = 450
Vanilla ice cream sold by ice cream parlour B be Let the number of Chocolate ice cream sold by
4b. ice cream parlour D be 3d and the Vanilla ice
Then, the Number of Strawberry ice cream sold cream sold by ice cream parlour D be 2d.
by ice cream parlour B = 4b + 80 Then, the Number of Strawberry ice cream sold
3b + 4b + 4b + 80 = 630 by ice cream parlour D = 2d + 120
11b = 550 3d + 2d + 2d + 120 = 540
b = 50 7d = 420
Number of Chocolate ice cream sold by ice d = 60
cream parlour B = 3 * 50 Number of Chocolate ice cream sold by ice
= 150 cream parlour D = 3 * 60
Number of Vanilla ice cream sold by ice cream = 180
parlour B = 4 * 50 Number of Vanilla ice cream sold by ice cream
= 200 parlour D = 2 * 60
Number of Strawberry ice cream sold by ice = 120
cream parlour B = 200 + 80 Number of Strawberry ice cream sold by ice
= 280 cream parlour D = 120 + 120
Let the number of Chocolate ice cream sold by = 240
ice cream parlour C be 5c and the Vanilla ice Let the number of Chocolate ice cream sold by
cream sold by ice cream parlour C be 7c. ice cream parlour E be 6e and the Vanilla ice
Then, the Number of Strawberry ice cream sold cream sold by ice cream parlour E be 5e.
by ice cream parlour C = 7c + 30 Then, the Number of Strawberry ice cream sold
5c + 7c + 7c + 30 = 1170 by ice cream parlour E = 5e + 140
19c = 1140 6e + 5e + 5e + 140 = 1260
c = 60 16e = 1120
Number of Chocolate ice cream sold by ice e= 70
cream parlour C = 5 * 60 Number of Chocolate ice cream sold by ice
= 300 cream parlour E = 6 * 70
Number of Vanilla ice cream sold by ice cream = 420
parlour C = 7 * 60 Number of Vanilla ice cream sold by ice cream
= 420 parlour E = 5 * 70

Click Here For Bundle PDF Course | support@guidely.in Page 11 of 17


SBI Clerk & RRB PO Mains PDF Course 2023
Quantitative Aptitude Day - 30 (Eng)

= 350 The selling price of a Strawberry ice cream pack


Number of Strawberry ice cream sold by ice = 140
cream parlour E = 350 + 140 Total amount C get by selling Chocolate ice
= 490 cream = 200 * 300
= 60000
Total amount C get by selling Vanilla ice cream =
180 * 420
= 75600
Total amount C get by selling Strawberry ice

6. Answer: C cream = 140 * 450

Let the quantity of Chocolate ice cream be kx, = 63000

the quantity of Vanilla ice cream be 3x, and the Total amount C get by selling all ice creams =

quantity of Strawberry ice cream be 5x. 60000 + 75600

According to the question, + 63000

kx/3x = 2/3 = 198600

k=2 The average selling price of an ice cream sold

Quantity of Chocolate ice cream in the cup = 200 by C = 198600/1170

* (2/10) =169.74

= 40 ml
Quantity of Vanilla ice cream in the cup = 200 * 8. Answer: D

(3/10) Total number of ice cream packs sold by B = 150

= 60 ml + 200 + 280

Quantity of Strawberry ice cream in the cup = = 630

200 * (5/10) Total number of ice cream packs B has = 630 *

= 100 ml (100/70)

Required percentage = (60/200) * 100 = 900

= 30% Chocolate ice cream B has = 900 * (5/12)


= 75 * 5

7. Answer: B = 375

The selling price of a Chocolate ice cream pack Vanilla ice cream B has = 900 * (3/12)

= 200 = 75 * 3

The selling price of a Vanilla ice cream pack = = 225

180 Strawberry ice cream B has = 900 * (4/12)

Click Here For Bundle PDF Course | support@guidely.in Page 12 of 17


SBI Clerk & RRB PO Mains PDF Course 2023
Quantitative Aptitude Day - 30 (Eng)

= 75 * 4 Directions (11 – 13):


= 300 (Passengers in 1st AC coach on Monday +
Required percentage = ((375 – 150)/375) * 100 Passengers in General coach on Monday) –
= (225/375) * 100 (Passengers in General coach on Monday –
= 60% Passengers in Sleeper coach on Monday) = 728
– 64
9. Answer: A Passengers in 1st AC coach on Monday +
Difference between the number of Strawberry Passengers in Sleeper coach on Monday = 664
ice cream packs sold by A and B = 280 – 220 Then, (Passengers in 1st AC coach on Monday +
= 60 Passengers in Sleeper coach on Monday) –
The sum of the number of Chocolate ice cream (Passengers in Sleeper coach on Monday –
packs sold by B and the number of Vanilla ice Passengers in 2nd AC coach on Monday) = 664 –
cream packs sold by E together = 150 + 350 42
= 500 Passengers in 1st AC coach on Monday +
Required percentage = (60/500) * 100 Passengers in 2nd AC coach on Monday = 622
= 12% Passengers in 2nd AC coach on Monday –
Passengers in 1st AC coach on Monday = 62
10. Answer: B By adding both the above statements,
Let the number of Chocolate ice cream sold by F 2 * Passengers in 2nd AC coach on Monday =
be 5x, the number of Vanilla ice cream sold be 622 + 62
4x, and the number of Strawberry ice cream sold Passengers in 2nd AC coach on Monday = 342
be 7x. Passengers in 1st AC coach on Monday = 342 –
According to the question, 62 = 280
Chocolate ice cream packs sold by F = 250 * Passengers in General coach on Monday = 728
(120/100) – 280 = 448
= 300 Passengers in Sleeper coach on Monday = 448
Strawberry ice cream packs sold by F = 300 * – 64 = 384
(7/5) Similarly,
= 420 Passengers in 2nd AC coach on Tuesday = 387
Strawberry ice cream packs sold by A = 220 Passengers in 1st AC coach on Tuesday = 387 –
Reqd. Difference = 420 – 220 71 = 316
= 200 Passengers in General coach on Tuesday = 780
– 316 = 464

Click Here For Bundle PDF Course | support@guidely.in Page 13 of 17


SBI Clerk & RRB PO Mains PDF Course 2023
Quantitative Aptitude Day - 30 (Eng)

Passengers in Sleeper coach on Tuesday = 464


–42= 422
Passengers in 2nd AC coach on Wednesday =
414
Passengers in 1st AC coach on Wednesday =
414 – 36 = 378 11. Answer: D
Passengers in General coach on Wednesday = Let the ticket price for the General coach be 2x
890 – 378 = 512 and for the 2nd AC, coach be 9x.
Passengers in Sleeper coach on Wednesday = Then, (9x * 414) – (2x * 512) = 99974
512 – 44 = 468 3726x – 1024x = 99974
Passengers in 2nd AC coach on Thursday = 435 2702x = 99974
Passengers in 1st AC coach on Thursday = 435 x = 37
– 52 = 383 Money collected from General coach
Passengers in General coach on Thursday = passengers = 512 * 37 * 2
915 – 383 = 532 = 37888
Passengers in Sleeper coach on Thursday = 532 Money collected from Sleeper coach passengers
– 42 = 490 = 5 * 37 * 468
Passengers in 2nd AC coach on Friday = 383 = 86580
Passengers in 1st AC coach on Friday = 383 – Money collected from 2nd AC coach passengers
66 = 317 = 9 * 37 * 414
Passengers in General coach on Friday = 813 – = 137862
317 = 496 Money collected from 1st AC coach passengers
Passengers in Sleeper coach on Friday = 496 – = 13 * 37 * 378
60= 436 = 181818
Passengers in 2nd AC coach on Saturday = 357 Total money collected on Wednesday from all
Passengers in 1st AC coach on Saturday = 357 – passengers = 37888 + 86580 + 137862 +
74 = 283 181818
Passengers in General coach on Saturday = 771 = 444148
– 283 = 488
Passengers in Sleeper coach on Saturday = 488 12. Answer: B
– 80 = 408 Number of General coaches in the train = 496/90
= 5.51
Or, 6 (Number of coaches is a whole number)

Click Here For Bundle PDF Course | support@guidely.in Page 14 of 17


SBI Clerk & RRB PO Mains PDF Course 2023
Quantitative Aptitude Day - 30 (Eng)

Number of Sleeper coaches in the train = 436/70 Selling price of Wave smart watch = 2a * (5/4) =
= 6.22 2.5a
Or, 7 Cost price of Fire Boltt smart watch = 2.5a * (3/2)
Number of 2nd AC coaches in the train = 383/50 = 3.75a
= 7.66 The marked price of the Boat smart watch =
Or, 8 3.75a
Number of 1st AC coaches in the train = 317/20 Selling price of Noise smart watch = 3a * (7/6) =
= 15.85 3.5a
Or, 16 Marked price of Fire Boltt smart watch = 3.5a *
Total number of coaches in the train on Friday = (3/2) = 5.25a
6 + 7 + 8 + 16 Selling price of Fire Boltt smart watch = 3.75a *
= 37 (6/5) = 4.5a
Selling price of Boat smart watch = 3.75a * (4/5)
13. Answer: A = 3a
The number of passengers who bought tickets in The cost price of the Boat smart watch = 2200
General coach on Tuesday = 464 The selling price of the Boat smart watch = 2200
Total number of passengers travelling in General + 200
coach on Tuesday = 464 * (5/4) = 2400
= 116 * 5 3a = 2400
= 580 a = 800
The selling price of the Noise smart watch = 3.5
Directions (14 – 17): * 800
Let the cost price of the Noise smartwatch be 3a. = 2800
Then, the Marked price of the Wave smart watch The cost price of the One Plus smartwatch =
= 4a 2800 + 1000
The cost price of the Wave smart watch = 4a/2 = = 3800
2a (40b/6) = 3800
Let the marked price of the One Plus smartwatch b = 570
be 8b. The marked price of the One Plus smartwatch =
Then, the Selling price of the One Plus 8 * 570
smartwatch = 7b = 4560
Cost price of One plus smart watch = 8b * (5/6) The marked price of Noise smart watch = 4560 –
1200

Click Here For Bundle PDF Course | support@guidely.in Page 15 of 17


SBI Clerk & RRB PO Mains PDF Course 2023
Quantitative Aptitude Day - 30 (Eng)

= 3360 Actual profit by selling Fire Boltt smart watch =


By putting the value of a and b in all variables, 3600 – 3000
= 600
New profit by selling Fire Boltt smart watch =
3900 – 3000
= 900
Percentage increase in the amount of profit =
((900 – 600)/600) * 100
= 50%
14. Answer: B
The marked price of One Plus smartwatch = 16. Answer: B
4560 The cost price of Noise’s new smartwatch =
The selling price of the One Plus smartwatch = 2400 * (5/4)
3990 = 3000
The cost price of the One Plus smartwatch = The marked price of Noise’s new smartwatch =
3800 3000 * (4/3)
Discount percentage = ((4560 – 3990)/4560) * = 4000
100 The selling price of Noise’s new smartwatch =
= 12.5% 3000 * (7/6)
New discount percentage = 2 * 12.5 = 25% = 3500
The new selling price of the One Plus New discount percentage = ((4000 –
smartwatch = 4560 * (3/4) 3500)/4000) * 100
= 3420 = 12.5%
Loss percentage = ((3800 – 3420)/3800) * 100
= 10% 17. Answer: A
Price at which Basant bought a Wave smart
15. Answer: D watch = 2000
The new marked price of the Fire Boltt smart Let Basant mark the price of the Wave
watch = 4200 + 350 smartwatch be x.
= 4550 Profit of Basant per smartwatch = 4400/8
The new selling price of the Fire Boltt smart = 550
watch = 4550 * (6/7) Basant sold a Wave smartwatch at = 2000 + 550
= 3900 = 2550

Click Here For Bundle PDF Course | support@guidely.in Page 16 of 17


SBI Clerk & RRB PO Mains PDF Course 2023
Quantitative Aptitude Day - 30 (Eng)

Then, x * ((100 – 15)/100) = 2550 Only i and ii are correct.


x= 2550 * (20/17) 19. Answer: A
x = 150 * 20 L.C. M of 28 and 32 = 224
x = 3000 Total work = 224 unit
Efficiency of A = 224/P
18. Answer: C Efficiency of B and C together = 224/28 = 8
The cost price of the radio = 800 Efficiency of A and C together = 224/32 = 7
From (i) Let the efficiency of B be 5x and the efficiency of
Marked price of radio = 800 * (150/100) A be 4x.
= 1200 Efficiency of C = 8 – 5x or 7 – 4x
Selling price of the radio = 1200 * (80/100) 8- 5x = 7 – 4x
= 960 x=1
Profit = 960 – 800 Efficiency of A = 4 * 1
= 160 =4
i is correct. A alone can complete the work in = 224/4 days
From (ii) = 56 days
Marked price of radio = 800 * (140/100) The value of P is 56.
= 1120
Selling price of the radio = 1120 * (6/7) 20. Answer: C
= 960 Surface area of a cylinder = 2∏r2 + 2∏rh
Profit = 960 – 800 2∏ (k2 + k 42) = 4928
= 160 k2 + 42k = 784
ii is correct. k2 + (56 – 14) k – 784 = 0
From (iii) k2 + 56k – 14k – 784 = 0
Marked price of radio = 800 * (130/100) k ( k + 56) – 14 (k + 56) = 0
= 1040 (k + 56) (k – 14) = 0
Selling price of radio = 1040 * (95/100) k = -56, 14
= 988 The radius of the cylinder is 14 meters.
Profit = 988 – 800 The volume of the cylinder = ∏r2h
= 188 = ∏ * 14 * 14 * 42
iii is incorrect. = 25872 m3

Click Here For Bundle PDF Course | support@guidely.in Page 17 of 17


SBI Clerk & RRB PO Mains PDF Course 2023
ENGLISH Day - 30

English Language

Directions (1-5) : In each of the following c) applied, sends


questions given below, there is a sentence in d) deployed, drives
which two words are highlighted. They may or e) No changes required
may not be grammatically and contextually
correct. Choose the best alternatives among the 4) When burnt, CNG produces fewer greenhouse
four given options to replace those words. If no gas emissions, particulate matter, and other
replacement is required choose option (e) as pollutants that addition to air pollution and
your answer. climate change.
1) The Article had gave special rights and a) combusted, contributed
advantages to the people of Jammu and Kashmir b) fired, add
since 1954 in accordance with the Instrument of c) blazed, governed
Accession. d) ignited, increased
a) give, opportunities e) No changes required
b) accorded, privileges
c) grant, freedom 5) Some drones also have automatic capabilities
d) awarded, rights that allow them to perform pre-programmed
e) No changes required tasks, follow flight paths, or use advanced
algorithms to navigation and make decisions
2) Many private banks have an international based on sensor inputs.
presence, allowing clients to assess services and a) different, decide
investment opportunities in many jurisdictions. b) advancing , fly
a) access, multiple c) free, sail
b) availing, few d) autonomous, navigate
c) use, numerous e) No changes required
d) enable, most
e) No changes required Directions (6-10): Seven sentences are given
below out of which one sentence is either
3) Blood pressure is the force exerted by the irrelevant or inappropriate (odd one out).
blood against the walls of the arteries as the Rearrange the remaining six sentences in a
heart pumps it throughout the body. proper sequence so as to form a coherent
a) released, pushes paragraph and then answer the questions given
b) on, pulls below.

Click Here For Bundle PDF Course | support@guidely.in Page 1 of 12


SBI Clerk & RRB PO Mains PDF Course 2023
ENGLISH Day - 30

(A) Tightening compliance and the post- a) FG


pandemic rebound in economic activity have b) FA
helped improve revenues from the GST. c) GF
(B) Marking the occasion, Finance Minister d) CA
Nirmala Sitharaman emphasised that the GST e) EB
has moved the country towards a unified market
from a situation where each State mandated 7) Fill in the blanks based on the sequence of
different indirect tax structures and procedures. rearrangement. The following represents the first
(C) GST Compensation cess levies have been five sentences.
extended till at least March 2026, instead of the F_D_ _
initial five-year tenure, due to the transitory shock a) AEB
of COVID-19 lockdowns on revenues. b) EBG
(D) Introduced soon after the demonetisation c) BEG
shock/A, the GST was viewed as another d) EAG
disruptor/B for the informal economy and its e) ABG
initial technical, structural/C and procedural
challenges took a while to sorting out/D. No 8) Which is the odd one out sentence among the
error/E given options ?
(E) starting this August, and that there has a) A
been(P) now that all businesses with annual b) B
turnover of ₹ 5 crore(Q) indicates that firms have c) C
gradually embraced the change(R) no ostensible d) E
pushback from smaller businesses over this,(S) e) G
will have to generate e-invoices(T)
(F) India’s tryst with the Goods and Services Tax 9) Choose the erroneous part in sentence D as
(GST), launched at a special midnight Parliament your answer.
session with unusual fanfare, completed six a) A
years this month. b) B
(G) The Revenue Department’s crackdown on c) C
fake invoicing and other techniques deployed by d) D
tax evaders may compel the few outliers to fall in e) No error
line too.
6) Which of the following options represents the 10) Sentence E is divided into five parts namely
First and the last sentence after rearrangement ? P, Q, R, S and T which may or may not be in

Click Here For Bundle PDF Course | support@guidely.in Page 2 of 12


SBI Clerk & RRB PO Mains PDF Course 2023
ENGLISH Day - 30

correct order, rearrange them if necessary to the ideal population size of a family, a
form a proper meaningful sentence. Choose the community, a country and even the world ought
correct sequence of rearrangement as your to be. It is important to understand that there are
answer. no ideal numbers or figures.
a) PQRST Population stability comes when reproductive
b) RPQST and sexual health decisions are free of
c) TRPQS discrimination, coercion(B) and violence, that
d) STRPQ reproductive and sexual health services are
e) QTPSR affordable, acceptable, accessible and of high
quality, and that women and couples are
Directions (11-17): Read the given passage and supported to have the number of children they
answer the following questions based on the want, when they want them. On World
passage. Some words are highlighted to help Population Day (July 11), India deserves to be
you locate while answering the questions. commended for its family planning initiatives,
The theme of this year’s World Population Day, where despite the many challenges, the aim is to
i.e., ‘Unleashing the power of gender equality: provide an increasingly comprehensive package
Uplifting the voices of women and girls to unlock of reproductive health services to every potential
our world’s infinite possibilities’, could not be beneficiary — with a focus on the provision of
more apt for India. When we unlock the full modern short and long-acting reversible
potential of women and girls, encouraging and contraceptives, permanent methods, information,
nurturing their desires for their families and counselling, and services, including emergency
themselves, we galvanise(A) half the leadership, contraception.India’s commitment towards the
ideas, innovation, and creativity available to Family Planning 2030 partnership includes
societies. In India, the world’s most populous expanding its contraceptive basket. The inclusion
nation, the template for women-led development, of new contraceptive options advances women’s
be it in science, technology, agriculture, rights and autonomy, leading to a spike in
education or health care, must also include modern contraceptive prevalence. Access to
reproductive autonomy at its core.For far too long timely, quality and affordable family planning
the world has been obsessed with population services is crucial because unspaced
numbers and targets. Instead of ensuring pregnancies may have a detrimental influence on
reproductive autonomy for each woman, we are the new-born’s health as well as major effects on
obsessed with total fertility rates; instead of maternal mortality, morbidity, and health-care
ensuring that family planning services reach all expenditure. The Indian government’s health,
those who want it, we are obsessed with what population and development programmes have

Click Here For Bundle PDF Course | support@guidely.in Page 3 of 12


SBI Clerk & RRB PO Mains PDF Course 2023
ENGLISH Day - 30

shown steady progress over the years. Life importantly, the focus on gender equality helps
expectancy at birth has significantly increased in shift the focus away from the notion of
the country over the years. Compared to the ‘population stabilisation’ to ‘population dynamics’
1990s, Indians are currently living a decade based on reproductive choices people make.
longer. In terms of maternal health, India has India has a significant opportunity to advance
made impressive strides. The current rate of gender equality and grow its economy. In fact,
maternal mortality is 97 (per 100,000 live births in raising the women’s labour force participation by
a year), down from 254 in 2004. Another triumph 10 percentage points might account for more
of these programmes is gender empowerment. than 70% of the potential GDP growth
Since the beginning of 2000, India has cut the opportunity ($770 billion in additional GDP by
number of child marriages by half. Teen 2025). The path to such a bright future is clear.
pregnancies, too, have dramatically decreased. Focusing on gender equality-centred growth,
Access to vital services, including health, rights, and choices promises to help all achieve
education, and nutrition, has also improved. their aspirations. Gender equality can be ensured
However, this progress has a fine print too. Many by making investments in a woman’s life at every
women continue to lack physical autonomy. stage, from childbirth to adolescence to maturity.
According to the most recent National Family Engaging with women, girls and other
Health Survey (NFHS-5), just 10% of women in marginalised people and formulating legislation
India are independently able to make decisions and policies that empower them to asset their
about their own health, and 11% of women rights and to taking life changing personal
believe that marital violence is acceptable if a decisions(X) are the first steps in this
woman refuses to have sex with her husband. direction.World Population Day this year is an
Nearly half of all pregnancies in India are opportunity to reaffirm our commitment to putting
unplanned, as they are globally. Advancing individual rights, particularly women’s rights and
gender equality is not just about women but also well-being, at the centre of the population and
about populations as a whole. In ageing societies development discourse(C). Gender-just
that worry about labour productivity, achieving approaches and solutions are the fundamental
gender parity in the workforce is the most building blocks of a more prosperous India, and
effective way to improve output and income indeed the world.
growth. And in countries experiencing rapid 11) According to the given passage, which of the
population growth, women’s empowerment following contributes to the growth of the
through education and family planning can bring economy ?
enormous benefits by way of human capital and
inclusive economic development. More

Click Here For Bundle PDF Course | support@guidely.in Page 4 of 12


SBI Clerk & RRB PO Mains PDF Course 2023
ENGLISH Day - 30

I. Empowering women through education and 13) Which of the following best conveys the
creating awareness to them may help in meaning of the sentence “Life expectancy at
achieving good economic growth. birth has significantly increased in the country
II. By achieving equal contribution of women and over the years” in the passage ?
men in the workforce which is considered as the a) Expected life period of the infant during birth
most effective way to improve the economy. has increased over the years.
III. Focusing more on gender equality and equal b) Infant mortality rate has increased over the
rights may also contribute to the growth of the years.
nation in terms of economy. c) Expected life period of the mother during birth
a) Only I has increased over the years.
b) Only II d) Both a and c
c) Both I and III e) None of these
d) Both II and III
e) All I, II and III 14) Which of the following questions cannot be
answered after reading the given passage ?
12) Which of the following given statements a) What is the main motive or agenda behind
is/are true and correct as per the context of the celebrating world population day every year ?
passage ? b) What should India as a nation must focus on
I. Every year India celebrates Population day on other than population numbers and targets ?
July 11th and this year the theme is Unleashing c) In what way men are superior to women when
the power of gender equality. it comes to work ?
II. Most of the women in India find it difficult to d) What are the steps to achieve inclusive
make independent decisions because they are economic development ?
dependent on others and accept everything e) None of these
including violence against them.
III. For every 100,000 childbirths 97 women die 15) Which of the following best replaces the
but this is an improved figure as India has highlighted phrase (X) in the above passage to
focused on population, health and development make the sentence grammatically and
programmes for the past years. contextually correct and meaningful ?
a) Only I a) take life changing decisions on the assets and
b) Only II rights
c) Only III b) assert there rights and take personal changes
d) Both I and III that will change life
e) Both II and III

Click Here For Bundle PDF Course | support@guidely.in Page 5 of 12


SBI Clerk & RRB PO Mains PDF Course 2023
ENGLISH Day - 30

c) asset their right and takes life changing COLUMN 1 COLUMN 2


personnel decisions (A) galvanise (D) dialogue
d) assert their rights and take life changing
personal decisions (B) coercion (E) energise

e) to take their assets through rights and making


(C) discourse (F) oppression
life changing decisions
a) A-E, B-D, C-F
b) A-D, B-E, C-F
16) Three statements are given below, you are
c) A-E, B-F, C-D
required to mark the statements as TRUE OR
d) A-F, B-D, C-E
FALSE in accordance with the information given
e) None of the above
in the passage.
I. Child marriages In India have been reduced by
Directions (18-22) : Read each question to find
half in numbers with the base year as 2000.
out whether they are erroneous , errors if any,
II. Increasing the men’s labour force participation
will be in two parts of the sentence. Choose the
by 25% percentage points might account for
option that represents the incorrect parts as your
more than 70% of the potential GDP growth
answer. If there is no error, mark(e) as your
opportunity
answer.
III. India’s Family Planning 2030 partnership
18) NASA’s Hubble Space Telescope has
includes new contraceptive options.
discovered/A the furthest individual star/B ever
a) True, False, False
seen, an enormous big blue stellar/C body
b) True, false, True
nicknamed Icarus located over halfway across
c) False, False, True
the universe/D. No error/E
d) False, True, False
a) AB
e) True, True, False
b) BC
c) BD
17) Three words have been highlighted in the
d) CD
given passage namely (A), (B) and (C). Two
e) No error
columns are given below with column 1 having
the highlighted words and column 2 having the
19) The report sites research that shows/A that
synonyms of the words. You are required to
gender gaps cause an average income loss/B of
match the words with their appropriate
15 percent in the OECD economies,/C 40
synonyms.
percent of which is since entrepreneurship
gaps/D. No error/E

Click Here For Bundle PDF Course | support@guidely.in Page 6 of 12


SBI Clerk & RRB PO Mains PDF Course 2023
ENGLISH Day - 30

a) AC companies dealing in crypto currencies/D.No


b) AD error/E
c) BC a) AC
d) CD b) BC
e) No error c) BD
d) CD
20) Manipuri ponies which are central/A to the e) No error
game are facing a very precarious/B situation of
possible extinction/C sooner rather than 22) In most countries, a combustible mixture/A of
later/D.No error/E authoritarianism, unemployment and youth/B has
a) AD given rise to disaffection of strongmen/C rulers
b) BC which has in turn spill over into uprising/D.No
c) BD error/E
d) CD a) AB
e) No error b) AC
c) BD
21) The central bank issues a circular/A three d) CD
months ago, banning all financial institutions/B e) No error
under its control by providing service to/C
Click Here to Get the Detailed Video Solution for the above given Questions
Or Scan the QR Code to Get the Detailed Video Solutions

Answer Key with Explanation

1. Answer: B Replace ‘gave’ with ‘accorded’ and ‘advantages’


with ‘privileges’ to make the sentence correct
and meaningful.

Click Here For Bundle PDF Course | support@guidely.in Page 7 of 12


SBI Clerk & RRB PO Mains PDF Course 2023
ENGLISH Day - 30

‘Had gave’ is wrong - hence it needs Many private banks have an international
replacement, similarly the word ‘privileges’ is presence, allowing clients to access services
more suitable to the context of the sentence than and investment opportunities in multiple
‘advantages’. jurisdictions.
Accorded - to give something to somebody
Privileges - a right or advantage 3. Answer: E
Option a - give is wrong, opportunities sound No replacement required, the highlighted words
inappropriate are correct and are used appropriately in the
Option c - grant wrong form of usage(not a past sentence.
tense) Exerted - to make a big effort
Option d - rights is wrong because it is
redundant(special rights and rights - wrong) 4. Answer: A
Hence, Option b is the best answer. Replace ‘burnt’ with ‘combusted’ and ‘addition’
The sentence after replacement : with ‘contributed’ to make the given sentence
The Article had accorded special rights and meaningful and correct.
privileges to the people of Jammu and Kashmir Combusted - consumed/destroyed by fire
since 1954 in accordance with the Instrument of CNG - compressed natural gas can be
Accession. consumed(combusted), other options like fired,
blazed and ignited seem inappropriate and
2. Answer: A hence we discard them.
Replace assess with access(because assess is Option a - combusted and contributed are the
to judge or test), similarly replace many with right words that can be used to replace the
multiple. highlighted word in the sentence.
Access - to use The sentence after replacement :
Multiple - more in number When combusted, CNG produces fewer
Option b - to availing is incorrect, few is greenhouse gas emissions, particulate matter,
inappropriate because the sentence talk and other pollutants that contribute to air
Option c - numerous in incorrect to the context of pollution and climate change.
the sentence
Option d - clients don't enable services(client will 5. Answer: D
access the service), most is also inappropriate Replace ‘automatic’ with ‘autonomous’ and
Hence, option a is the right answer. ‘navigation’ with ‘navigate’ to make the sentence
The sentence after replacement : correct.

Click Here For Bundle PDF Course | support@guidely.in Page 8 of 12


SBI Clerk & RRB PO Mains PDF Course 2023
ENGLISH Day - 30

Automatic and navigation are either incorrect or first because A seems to be a concluding
inappropriate in the sentence. sentence because it has taken a generic form.
Autonomous - one’s own So, the sequence of the coherent passage
Navigate - to use map and travel formed is FBDEGA.
Option a - different is not relevant, to decide and C is the odd one out because it is irrelevant to
make decisions(makes no sense) the passage as it talks about the compensation
Option b - advancing wrong form of word, cess which is not the main idea of the passage.
navigate is a better word than fly in this context Although it comes under the vast topic ‘GST’,
Option c - free and sail(both are inappropriate in having a look at the other sentences, sentence C
the sentence) seems to be deviating and hence is the odd one
The sentence after replacement : out.
Some drones also have autonomous capabilities 6. Answer: B
that allow them to perform pre-programmed The first sentence after rearrangement is ‘F’ and
tasks, follow flight paths, or use advanced the last sentence is ‘A’.
algorithms to navigate and make decisions
based on sensor inputs. 7. Answer: C
Directions (6-10): BEG is the correct answer because B is the
The correct sequence of rearrangement is second sentence, E is the fourth sentence and G
FBDEGA (C is the odd one out) is the fifth sentence.
Sentence ‘F’ is the first sentence as it gives
proper introduction to the passage with 8. Answer: C
information that it’s been 6 months since the Sentence C is the odd one out sentence.
launch of GST. The other sentences do not
seem to be a starter of the passage. The second 9. Answer: D
sentence would be ‘B’ because ‘marking the The error is in part d of the given sentence D.
occasion the Finance Minister has said…’ this Replace the word ‘sorting’ with ‘sort’ to make the
phrase makes it clear that sentence B succeeds sentence correct.
sentence F. The third sentence is ‘D’ as it talks ‘To sorting’(wrong) instead it must be ‘to sort’.
about the initial phase of GST introduction, how Introduced soon after the demonetisation shock,
it was a shock then and its challenges. The the GST was viewed as another disruptor for the
fourth sentence is ‘E’ which explains that the informal economy and its initial technical,
firms/companies have adapted to change(GST). structural and procedural challenges took a while
Between the sentences G and A, G will come to sort out.

Click Here For Bundle PDF Course | support@guidely.in Page 9 of 12


SBI Clerk & RRB PO Mains PDF Course 2023
ENGLISH Day - 30

II - Attaining gender parity meaning equal


10. Answer: E workforce of men and women will lead to good
QTPSR is the correct sequence of economic growth, it is also proven that women
rearrangement. workforce are more productive in nature.
P - cannot be the first fragment because ‘starting III - Women are deprived of basic human rights
this august and’ cannot be the first part of the and equality which acts as a hindrance for their
sentence growth, by providing equal rights to all, everyone
Q- Good starter starting with ‘now’, it can be a can contribute to the nation and for its growth.
continuation to the previous sentence in the So, all the three statements hold true and hence
coherent passage. This is followed by T, now the option (e) is the right answer.
firms have to generate invoices.Now comes ‘P’
as the third fragment. Following P comes S 12. Answer: E
‘there has been no ostensible pushback’(joining Both the statements II and III are true according
P and S). Lastly it is R to end the sentence in a to the information given in the passage.
correct way. Statement I is wrong because July 11th is World
Now that all businesses with annual turnover of population day and it is not specific to India.
₹ 5 crore will have to generate e-invoices II and III are information that are inferred from
starting this August, and that there has been no the given passage and are true and valid.
ostensible pushback from smaller businesses Hence, option (e) is the correct answer for this
over this, indicates that firms have gradually question.
embraced the change.
13. Answer: A
11. Answer: E Life expectancy at birth means the average
According to the given passage, all the three number of years a newborn infant is expected to
statements I , II and III contribute to the live based on the prevailing patterns of mortality
economic growth of a nation or country. rate.
I - Women empowerment plays a major role in So, only option (a) conveys the exact meaning of
the economic development of a nation, all the highlighted phrase as given in the passage.
women must be given good education and
awareness about every difficulty they face and 14. Answer: C
how they can face and voice out their All the questions a,b and d can be answered
opinions.According to the passage, empowered with the information given in the passage above
women can contribute really well to the society. as the passage contains relevant and

Click Here For Bundle PDF Course | support@guidely.in Page 10 of 12


SBI Clerk & RRB PO Mains PDF Course 2023
ENGLISH Day - 30

appropriate information regarding all these topics Option c is the correct answer.
of discussion. Synonym of galvanise is energise
Question in option (c) cannot be answered Synonyms of coercion is oppression
because there is no such discussion made in the Synonym of discourse is dialogue
passage and the data/information is inadequate Galvanise - to stimulate or excite
to answer the question. Coercion - the use of force to persuade
Hence, option(c) is the correct answer for this someone to do something
question. Discourse - long serious discussion

15. Answer: D 18. Answer: B


Only phrase/fragment d is correct and can Part B and C has errors.
possibly replace the highlighted part (X) to make B - Replace ‘furthest’ with ‘farthest’(furthest -
the respective sentence correct and meaningful. higher degree, farthest - actual physical
Assert - to be clear/confident/bold distance)
Options a,c and e are wrong because they talk C - enormous big is incorrect as both the words
about assets which is not even a part of the have the same meaning
discussion in the given passage. The correct sentence : NASA’s Hubble Space
Option b is wrong because ‘there’ is incorrect Telescope has discovered the farthest individual
and the statement is inappropriate and incorrect star ever seen, an enormous blue stellar body
by itself. nicknamed Icarus located over halfway across
the universe.
16. Answer: B
Statement I is true, II is false and III is true 19. Answer: B
according to the given passage. The errors are in part A and D of the given
II is false because the passage has got the sentence.
information only regarding the women,i.e, by Replaces ‘sites’ with ‘cites’ in part A (site - a
increasing the women’s labour force participation piece of land, cite - to mention)
by 10% percentage points might account for Change ‘since’ to ‘due to’ in part D (since in
more than 70% of the potential GDP growth incorrect here)
opportunity. The correct sentence : The report cites research
Hence, the correct answer is Option (b). that shows that gender gaps cause an average
income loss of 15 percent in the OECD
17. Answer: C

Click Here For Bundle PDF Course | support@guidely.in Page 11 of 12


SBI Clerk & RRB PO Mains PDF Course 2023
ENGLISH Day - 30

economies, 40 percent of which is due to service to companies dealing in crypto


entrepreneurship gaps. currencies.

20. Answer: E 22. Answer: D


The given sentence is correct and error free, Errors are in part C and D of the given sentence.
hence we go for option (e) as our answer. C - ‘disaffection of’ is incorrect instead it must be
‘disaffection with’, one gets disappointed with
21. Answer: A something/someone not ‘of someone/something’
The erroneous parts are A and C. (change the preposition used to make the part
The circular had been issued three months ago - correct)
so replace ‘issues’ with ‘issued’. D - has in turn spilled over(replace spill with
Banning institutions from providing services is spilled to make it correct, use past tense)
the correct way to put the phrase(banning by is The correct sentence : In most countries, a
wrong, it should be banning from) combustible mixture of authoritarianism,
The correct sentence : The central bank issued unemployment and youth has given rise to
a circular three months ago, banning all financial disaffection with strongmen rulers which has in
institutions under its control from providing turn spilled over into uprising.

Click Here For Bundle PDF Course | support@guidely.in Page 12 of 12


SBI Clerk & RRB PO Mains PDF Course 2023
Reasoning Ability Day - 31 (Eng)

Reasoning Ability
Directions (1-5): Study the following information a) D
carefully and answer the given questions. b) The box which has 13 red balls
Six boxes – B, D, F, G, M, and T are placed in a c) The box which has 26 balls
linear row facing north. Each box has different d) The box which has 6 black balls
number of balls viz.-13, 15, 17, 21, 24, and 26. e) Both B and C
There are two different types of balls viz. Red
and Black are kept inside each box. No two 2) Which of the following box has 8 black balls?
boxes have the same number of same coloured a) D
balls. b) The box which is placed second from the left
Note: The total number of balls kept in adjacent end
boxes is neither have common multiple nor c) The box which has 15 balls
common factors. No box has less than 4 balls of d) F
each type. e) None of these
B is placed third to the left of the box which has
24 balls. The number of balls in box M is thrice 3) The box which has ______ black balls is
the black balls in box G, which is placed to the placed _____ to the right of T.
left of box B. Only one box is placed between the a) 11, third
box which has 21 balls and box G. Box M has an b) 8, Immediate
even number of balls. The box which has 9 black c) 9, Second
balls is kept third to the right of the box which has d) 6, Fourth
15 red balls. Only one box is placed between the e) None of these
box which has 9 black balls and box T, which has
half the number of balls with box B. The number 4) What is the sum of black balls in D and red
of boxes placed to the right of T is one more than balls in box B?
the number of boxes placed to the left of the box a) 16
which has 11 red balls. The number of black b) 20
balls in box B is same as the number of red balls c) 17
in box F. Box F and the box which has 4 black d) 22
balls are placed together. The difference e) None of these
between the red balls in box G and black balls in
box M is equal to the black balls in box T. 5) Which of the following statements is/are not
1) Which of the following box is placed true as per the given arrangement?
immediate right of box T? a) D has 9 black balls

Click Here For Bundle PDF Course | support@guidely.in Page 1 of 10


SBI Clerk & RRB PO Mains PDF Course 2023
Reasoning Ability Day - 31 (Eng)

b) The box which is placed second from the left ordered. All the persons whose name is a vowel
end has 15 black balls received their order in reverse alphabetical order
c) Box F has 21 balls on alternate days starting from Monday. Q didn’t
d) G has 7 black balls receive on Saturday. D didn’t receive on the
e) All the above statements are true adjacent days of O.
6) Which of the following item is ordered by E?
Directions (6-10): Study the following information a) The item which is ordered on Wednesday
carefully and answer the given questions. b) Wallet
Seven persons – A, D, E, O, Q, U and W ordered c) The item which is ordered three days after D
different items viz.- Mobile, Shoes, Jeans, Wallet, d) T-Shirt
Cap, Laptop, and T-shirt from an online store e) None of these
during a week starting from Monday to Sunday.
Week 1: 7) _____ item ordered on Tuesday and ______
O ordered on one of the days after Friday but item received on Saturday.
neither ordered Laptop nor ordered Mobile. The a) Wallet and Mobile
number of persons ordered after O is one less b) Mobile and T-shirt
than the number of persons ordered before the c) Cap and Shoes
one who ordered Jeans. W ordered three days d) Mobile and Wallet
before the one who ordered Wallet. The one who e) None of these
ordered Wallet neither ordered on Thursday nor
ordered on Saturday. Q ordered immediately 8) _____ item received _____ days after the order
before the one who ordered Wallet. W ordered received by O?
neither Jeans nor Laptop. D ordered Shoes but a) Wallet, Four
didn’t order on Sunday. As many persons b) Laptop, Two
ordered between Q and the one who ordered c) Shoes, Three
Laptop as between Q and D. Only two persons d) Both A and C
ordered between U and the one who ordered e) Both A and B
Mobile. A ordered immediately before the one
who ordered T-shirt. The one who ordered Cap 9) Which of the following item ordered and
didn’t order on Saturday. received on Friday?
Week 2: a) T-Shirt and Jeans
All persons received their order in the next week b) Laptop and Cap
starting from Monday to Sunday. No person c) Cap and Wallet
received their order on the same day they d) Wallet and Mobile

Click Here For Bundle PDF Course | support@guidely.in Page 2 of 10


SBI Clerk & RRB PO Mains PDF Course 2023
Reasoning Ability Day - 31 (Eng)

e) None of these

10) Which of the following statements is/are not


true as per the given arrangement?
a) Wallet received on Sunday
b) O ordered immediately after Q
c) E received Jeans on Friday a) Only ARY
d) O ordered T-Shirt b) Only CQX
e) All the above statements are true c) Both CPZ and BRY
d) Both AQZ and CPZ
Direction (11-12): Below questions consists of e) None of these
some statements in three columns. Study the
following information carefully and answer the Directions (13-15): Below questions consists of
question. some statements in two columns. Study the
11) Which of the following combination gives “L > following information carefully and answer the
G” is definitely true and “M < K” is definitely question.
false? 13) Which of the following combination gives “G
> L” is definitely false?

a) Both CPZ and BRY


b) Only ARY
c) Only CQX
d) Both AQZ and CPZ
e) None of these a) Only BQ
b) Only CQ

12) Which of the following combination gives “U c) Both CP and BR

< F” is definitely true and “G ≥ D” is false? d) Both AR and CP


e) None of these

Click Here For Bundle PDF Course | support@guidely.in Page 3 of 10


SBI Clerk & RRB PO Mains PDF Course 2023
Reasoning Ability Day - 31 (Eng)

14) Which of the following combination gives “T > Directions (16-20): Study the following
U” is definitely true? information carefully and answer the given
questions.
Two buses viz. Bus 1 and Bus 2 start from the
same depot for two different routes. The
stoppages of each route assigned some names.
Bus1 starts towards the west from the depot and
travels for 8m to reach stoppage S1. From
stoppage S1 Bus1 takes a right turn and travels
for 5m to reach stoppage S2. stoppage S3 is 7m

a) Only AR towards the west of stoppage S2. From stoppage

b) Only CQ S3 Bus1 travels for 13m towards the north to

c) Both CP and BR reach stoppage S4. From stoppage S4 Bus1

d) Both AR and CP takes two consecutive right turns of 15m and 6m

e) None of these to reach stoppage S5 and stoppage S6


respectively. From stoppage S6 Bus 1 takes a

15) Which of the following combination gives “R left turn and travels for 9m to reach stoppage S7

> M” is definitely false? which is 12m towards north of stoppage S8.


Bus 2 travels for 6m from the depot towards the
south to reach stoppage S9, which is 10m east of
stoppage S10. From stoppage S10 Bus 2 takes a
left turn and travels for 13m to reach stoppage
S11. Now again Bus2 takes a left turn and
travels for 19m to reach stoppage S12, which is
11m south of stoppage S13.
Bus depot management decides to provide some
a) Only AR additional names to each stoppage based on
b) Only CQ certain conditions:
c) Both CP and BR I. If the stoppage is to the northwest or southeast
d) Both AR and CP of the depot then assign name X.
e) None of these II. If the stoppage is to the southwest or
northeast of the depot then assign name Y.
III. If the distance between two consecutive
stoppages reached by the bus is a prime

Click Here For Bundle PDF Course | support@guidely.in Page 4 of 10


SBI Clerk & RRB PO Mains PDF Course 2023
Reasoning Ability Day - 31 (Eng)

number, then assign the name W to the second


stoppage reached by the bus. 18) _______ stoppages are in the northeast of
IV. If the distance between two consecutive stoppage S2?
stoppages reached by the bus is a multiple of a) Four
three but not a multiple of four, then assign the b) Two
name Z to the first stoppage reached by the bus. c) Three
V. If the distance between two consecutive d) One
stoppages reached by the bus is a multiple of e) None
four, then assign the name T to the first
stoppage reached by the bus. 19) Which of the following stoppages are
16) What is the shortest distance between the allocated name as Y?
final position of both buses? a) S10, S11, and S7
a) 5m b) S7, S9, and S11
b) 12m c) S5, S12, and S11
c) 9m d) S10, S11, and S9
d) 8m e) None of these
e) None of these
20) What is the position of S5 with respect to the
17) Which of the following stoppages are depot?
allocated as Z? a) West
a) S12, S4, and S6 b) Northeast
b) S11, S5, and S8 c) North
c) S12, S6, and S3 d) Northwest
d) S4, S5 and S6 e) None of these
e) None of these

Click Here For Bundle PDF Course | support@guidely.in Page 5 of 10


SBI Clerk & RRB PO Mains PDF Course 2023
Reasoning Ability Day - 31 (Eng)

Click Here to Get the Detailed Video Solution for the above given Questions
Or Scan the QR Code to Get the Detailed Video Solutions

Answer Key with Explanation

Directions (1-5): case(3) box B is placed third from the left


1) Answer: B end.
2) Answer: C Based on the above given information we have:
3) Answer: C
4) Answer: B
5) Answer: D

We have:
 B is placed third to the left of the box
which has 24 balls.
 The number of balls in box M is thrice the
black balls in box G, which is placed to Again, we have:
the left of box B.  The box which has 9 black balls is kept
 Box M has an even number of balls. third to the right of the box which has 15
 Only one box is placed between the box red balls.
which has 21 balls and box G.  Only one box is placed between the box
Since, such possible combinations are (8, which contains 9 black balls and box T,
24) which has half the number of balls with
Thus, box M has 24 balls. box B.
That means, in case (1) Box B is placed  The number of boxes placed to the right
second from the left end, in case (2)& of T is one more than the number of

Click Here For Bundle PDF Course | support@guidely.in Page 6 of 10


SBI Clerk & RRB PO Mains PDF Course 2023
Reasoning Ability Day - 31 (Eng)

boxes placed to the left of the box which Since, no box contains less than 4 balls of
has 11 red balls. any colour, thus black balls in box M is
Since, only possible combination of box B 11.
and T is (13, 26). Based on the above given information we have:
That means, in case (1) Box T contains
13 balls and placed immediate left of box
M, case (2) & case (3) are not valid.
Based on the above given information we have:
Directions (6-10):
6) Answer: A
7) Answer: C
8) Answer: D
9) Answer: E
10) Answer: B

Case (2) is not valid as only one box is placed


between the box which contains 9 black balls
and box T, case (3) is not valid as the number of
balls in box B is 26.
Again, we have:
 The number of black balls in box B is
same as the number of red balls in box F.
 Box F and the box which has 4 black balls
are placed together.
We have:
Since, the total balls kept within each
 O ordered on one of the days after Friday
boxes adjacent nether have common
but neither ordered Laptop nor ordered
multiple nor common factors, thus box D
Mobile.
can’t contain 15 balls.
 The number of persons ordered after O is
That means, box G contains 15 balls.
one less than the number of persons
 The difference between the red balls in
ordered before the one who ordered
box G is and black balls in box M is equal
Jeans.
to the black balls in box T.

Click Here For Bundle PDF Course | support@guidely.in Page 7 of 10


SBI Clerk & RRB PO Mains PDF Course 2023
Reasoning Ability Day - 31 (Eng)

That means, in case (1) O ordered on  Only two persons ordered between U and
Saturday, in case (2) O ordered on the one who ordered Mobile.
Sunday. That means, in case (1) W ordered
 W ordered three days before the one who mobile, in case (1a) Q ordered mobile.
ordered Wallet. Based on the above given information we have:
 The one who ordered Wallet neither
ordered on Thursday nor ordered on
Saturday.
 Q ordered immediately before the one
who ordered Wallet.
 W ordered neither Jeans nor Laptop.
That means, in case (1) W ordered on
Tuesday, in case (2) W ordered on
Thursday.
Based on the given information we have:

Again, we have:
 As many persons ordered between Q and
the one who ordered Laptop as between
Case (2) is not valid as no day available to place
Q and D.
D.
 D ordered Shoes but didn’t order on
Again, we have:
Sunday.
 A ordered immediately before the one
That means, in case (1) D ordered on
who ordered T-shirt.
Monday, case (2) is not valid.

Click Here For Bundle PDF Course | support@guidely.in Page 8 of 10


SBI Clerk & RRB PO Mains PDF Course 2023
Reasoning Ability Day - 31 (Eng)

 The one who ordered Cap didn’t order on


Saturday.
That means, in case (1a) A ordered
wallet, case (1) is not valid.
Based on the above given information we have:

11) Answer: C
From CQX:
H≥L=C>D=G≤M>U≥B=T>K
L > G (L = C > D = G)  definitely true
Case (1) is not valid cap is not ordered on
M < K (M > U ≥ B = T > K)  definitely false
Saturday.
All persons received their order in the next week
12) Answer: A
starting from Monday to Sunday. No person
From ARY:
received their order on the same day they
F≥W>T=G≥U=L≥H>S≥D=K
ordered. All the persons whose name is a vowel
U < F (F ≥ W > T = G ≥ U)  definitely true
received their order in reverse alphabetical order
G ≥ D (G ≥ U = L ≥ H > S ≥ D)  false
on alternate days starting from Monday. Q didn’t
receive on Saturday. D didn’t receive on the
13) Answer: D
adjacent day of O.
From AR:
Based on the above given information we have:
G > L (G < U ≤ S = W < K ≤ L)  definitely false
From CP:
G > L ( G ≤ M < D ≤ S ≤ L)  definitely false
14) Answer: C
From CP:
T > U (T ≥ L > M ≥ H = F > J ≥ U)  definitely
true

Click Here For Bundle PDF Course | support@guidely.in Page 9 of 10


SBI Clerk & RRB PO Mains PDF Course 2023
Reasoning Ability Day - 31 (Eng)

From BR: I. If the stoppage is to the northwest or southeast


T > U ( T ≥ F > H ≥ J > M > U)  definitely true of the depot then assign name X.
II. If the stoppage is to the southwest or
15) Answer: B northeast of the depot then assign name Y.
From CQ: III. If the distance between two consecutive
R > M (R ≤ T < P ≤ D ≤ M)  definitely false stoppages reached by the bus is a prime
number, then assign the name W to the second
Directions (16-20): stoppage reached by the bus.
16) Answer: D IV. If the distance between two consecutive
17) Answer: D stoppages reached by the bus is a multiple of
18) Answer: C three but not a multiple of four, then assign the
19) Answer: A name Z to the first stoppage reached by the bus.
20) Answer: C V. If the distance between two consecutive
We have: stoppages reached by the bus is a multiple of
four, then assign the name T to the first
stoppage reached by the bus.
Based on the above given information we have:

Bus depot management decides to provide


some additional names to each stoppage based
on certain conditions:

Click Here For Bundle PDF Course | support@guidely.in Page 10 of 10


SBI Clerk & RRB PO Mains PDF Course 2023
Quantitative Aptitude Day – 31 (Eng)

Quantitative Aptitude

Directions (01 - 04): Study the following information carefully and answer the questions given below.
The given pie chart shows the percentage distribution of the number of items [charger + headphone]
sold in five different shops [A, B, C, D and E]. The given table shows the percentage of chargers sold out
of the number of headphones sold, The Ratio of the number of wireless chargers and wire chargers sold,
and the ratio of the number of wireless chargers and wireless headphones sold in each shop. The Total
number of items sold in all shops together is 6000.

1)The number of wire headphones sold in number of wire headphones sold in shop C is n.
shop A is z more than the number of wireless find which of the following is true?
chargers sold in shop D. Number of wireless I. n=[z+m]
headphones sold in shop B is m more than the II. 3n=[2z+3m]
number of wireless chargers sold in shop E. The III.5n=2*[z+7m]+23

Click Here For Bundle PDF Course | support@guidely.in Page 1 of 13


SBI Clerk & RRB PO Mains PDF Course 2023
Quantitative Aptitude Day - 31 (Eng)

a) All true in Shop E. If the total of 456 headphones and


b) Only II and III true 657 chargers sold in shop F then find the
c) All false difference between the total no. of wired
d) Only II false headphone and the total number of wired
e) Only III false chargers sold in shop F?
a) 323
2) The average price of 40% of wire headphones b) 372
sold in shop C is Rs.455 and the average price c) 347
of the rest of the wire headphones is Rs.855. d) 368
The average price of 60% of wireless e) None of these
headphones is Rs.1050 and the average price of
the rest wireless headphones is Rs. 1250. Find 4) Find which of the following is true?
the total revenue earned by shop C by selling all I. Average number of wireless headphones sold
the headphones? in shops C and D is 449.
a) Rs.422350 II. Average number of wireless chargers sold in
b) Rs.402350 shops A and D is 315.
c) Rs.432350 III. Total number of wire headphones sold in all
d) Rs.442350 shops together is 1859.
e) Rs.412350 a) All true
b) All false
3) The no. of wireless headphones sold in Shop c) Only I false
F, is 20% more than the same in Shop E, and d) Only I true
the no. of wireless chargers sold in Shop F, is e) None of these
10% less than the no. of wireless chargers sold

Directions (05 - 08): Study the following information carefully and answer the questions given below.
The given line graph shows the ratio of boys and girls in class X and the ratio of girls and boys in class
XII in five different schools [A, B, C, D, and E].

Click Here For Bundle PDF Course | support@guidely.in Page 2 of 13


SBI Clerk & RRB PO Mains PDF Course 2023
Quantitative Aptitude Day - 31 (Eng)

Note:
I. The number of girls in class XII of schools A, B, C, D and E is 56, 30, 60, 48 and 63 respectively.
II. The number of students [boys + girls] in class X of schools A and B is 94 and 38 more than the number
of students in class XIIof the same school.
III. The number of students in class X of schools C, D and E is 16,39, and 56 less than the number of
students in class XII of the same school.
5) The ratio of the total no. of students of class X e) None of these
take admission to class XI in stream science,
arts and commerce of school A is 4:5:1 and the 6) The ratio of the number of boys and girls in
ratio of same of school B is 3:2:3. Find the total class X of school F is 3:2 and the ratio of the
no. of students of class X take admission in number of boys and girls in class XII of school F
science stream of school A and B together is is 4:5. If the total number of boys in school F is 3
what per cent students of class X take admission more than the number of boys of school B and
in arts stream of school A and B together? number of girls in school F is 6 less than the
a) 92.17% number of girls in school D. Find the total
b) 92.5% number of students in school F?
c)93.80% a) 124
d) 95.64% b) 125

Click Here For Bundle PDF Course | support@guidely.in Page 3 of 13


SBI Clerk & RRB PO Mains PDF Course 2023
Quantitative Aptitude Day - 31 (Eng)

c) 119 Directions (09-12): Study the following


d) 128 information carefully and answer the questions
e) 138 given below.
In four different cities [A, B, C, and D], different
7) numbers of government and private buses are
Quantity I: Find the difference between the running. The number of government buses
average number of boys in class XII in schools A running in city A is three less than the number of
and B together and the average number of girls government buses running in city C. The ratio of
in class X in schools B and D together? private and government buses running in city D
Quantity II: Find the difference between the is 2:3. The number of private buses running in
average number of boys in class X of schools A city A is five less than the number of government
and E together and the average number of girls buses running in city B. The number of private
in class XII of schools A and D together? buses running in city B is 40% more than the
a) Quantity: I Quantity: II number of government buses running in the
b) Quantity: I > Quantity: II same city. The ratio of private and government
c) Quantity: I < Quantity: II buses running in city C is 2:3. The number of
d) Quantity: I ≤ Quantity: II private buses running in city D is three less than
e) Quantity I = Quantity II the number of private buses running in city B.
The number of government buses running in city
8) Out of the total no. of students in class XII of D is 60% more than the number of government
school C, 40% of students take admitted to buses running in city A. The total number of
government colleges and the rest of the students government buses running in cities A and B
take admission to private colleges. The ratio of together is 55.
the no. of students take admission to private 9) The ratio of ac and non-ac buses out of the
colleges of schools C and D is 3:2. Find the total number of government buses running in city
percentage of students of class XII who take A is 3:2. The total number ac buses running in
admission to government college from school D city A is 24. Find the total number of private non-
out of the total no. of students in class XIIof ac buses is what per cent of the total number of
school D? government non-ac buses running in city A?
a) 52.35% a) 154.22%
b) 54.54% b) 159.33%
c) 44.44% c) 116.66%
d) 65.25% d) 120.33%
e) None of these e) None of these

Click Here For Bundle PDF Course | support@guidely.in Page 4 of 13


SBI Clerk & RRB PO Mains PDF Course 2023
Quantitative Aptitude Day - 31 (Eng)

10) The no. of autos running in city B is 20% a) 14546


more than the no. of government buses running b) 14594
in the city, and the no. of taxis running in city B is c) 14654
20% more than the no. of private buses running d) 14824
in city B. If the total no. of autos running in the e) 14464
city B and D is 56, then find the difference
between the no. of autos running in D and the Directions (13 - 16): Study the following
number of taxis running in B? information carefully and answer the questions
a) 17 given below.
b) 19 Difference schemes provide either Simple
c) 16 interest or compound interest.
d) 13 Scheme A – Provide at 25% interest rate. If
e) None of these Rs.15000 invest for two years then the amount of
interest is Rs.8438.
11) The ratio of the no. of government buses Scheme B – Provide at 15% interest rate. If
running in cities A and F is 6:7, and the ratio of Rs.12320 invest for three years then the amount
the no. of private buses running in cities B and F of interest is Rs.5544.
is 7:8. The total no. of buses running in city G is Scheme C – Provide simple interest. If Rs.15000
20 more than the total no. of buses running in F. invest for two years then the amount of interest is
Find the difference between the no. of Rs.5400.
government buses running in cities G and B if Scheme D – Provide compound interest. If
the ratio of private and government bus running Rs.8000 invest for two years then the amount of
in G is 3:2? interest is Rs.3520.
a) 14 Scheme E – Provide at 10% interest rate. If
b) 19 Rs.13500 invest for two years then the amount of
c) 13 interest is Rs.2835.
d) 16 Scheme F – Provide at 16% interest rate. If
e) 18 Rs.12562 invest for three years then the amount
of interest is Rs.6029.76.
12) If x is equal to the total number of Scheme G – Provide at 30% interest rate. If
government buses running in all given cities Rs.20000 invest for two years then the amount of
together and y is equal to the total number of interest is Rs.13800.
private buses running in all given cities together. 13) A person has Rs.28000. some of the amount
Find the value of x*y? he invests in scheme A and some of the amount

Click Here For Bundle PDF Course | support@guidely.in Page 5 of 13


SBI Clerk & RRB PO Mains PDF Course 2023
Quantitative Aptitude Day - 31 (Eng)

he invests in scheme B for 2 years. After 2 years, scheme E and Rs.18600 was invested in scheme
the interest amount is Rs.12600. Find the F for 2 years?
difference between the amounts invested in Quantity II: Find the difference between the
scheme A and scheme B? interest amount when Rs.10400 invests in
a) Rs.4500 scheme G and Rs.11600 invests in scheme B for
b) Rs.4000 2 years?
c) Rs.6400 a) Quantity: I Quantity: II
d) Rs.8400 b) Quantity: I > Quantity: II
e) None of these c) Quantity: I < Quantity: II
d) Quantity: I ≤ Quantity: II
14) Rs.12400 was invested in scheme C for t e) Quantity I = Quantity II
years and earned Rs.13392 as interest. IF Rs.
20400 was invested in scheme D for (t/2) years 16) Rs. _______ invest at scheme ______ for 3
find the interest amount? years and interest earned after 3 years is
a) Rs.14851 Rs.9913 (approx.) and Rs.15200 invest at
b) Rs.12546 scheme ____ for 2 years and after 2 years
c) Rs.14581 interest amount is Rs.4864. find which of the
d) Rs.15632 following value satisfy the blank?
e) None of these a) 10400, F, A
b) 10900, A, D
15) c) 10400, A, F
Quantity I: Find the difference between the d) 10400, A, G
interest amount when Rs.16400 was invested in e) 11400, B, D

Directions (17-20): Study the following information carefully and answer the questions given below.
The below funnel DI shows the recruitment process of an organisation and the posting of selected
students. In the Recruitment process, data was given about the number of students who applied, the
percentage of students who appeared, the number of students who qualified in round I, the percentage of
students who appeared in round II, and the number of students who were rejected in round II. In the post
about selected students, data is given about the percentage of students selected in different cities [A, B,
C, D, and E].

Click Here For Bundle PDF Course | support@guidely.in Page 6 of 13


SBI Clerk & RRB PO Mains PDF Course 2023
Quantitative Aptitude Day - 31 (Eng)

Note- The total number of students selected is only five cities. All the percentages on the posting of
selected parts are calculated based on the total number of students selected in the second round. The
percentage of students who appeared is calculated on the number of students apply. The percentage of
students who appeared in round II is calculated on the number of students qualify round I. 10,000
students did not appear in the round I exam.
17) The ratio of male and female students posted e) 214.33%
in city A is 3:1 and the ratio of male and female
students posted in city B is 3:2. Find the total 18) Find which of the following is true?
number of males posted in city A and B together I. Difference between the number of students
is what percentage of total number of females who did not appear in Round I and the number of
posted in city A and B together? students who did not appear in Round II is 7000.
a) 214.52% II. Difference between the number of students
b) 215.37% rejected in the first round and the number of
c) 259.32% students rejected in secondround is 16400.
d) 218.18%

Click Here For Bundle PDF Course | support@guidely.in Page 7 of 13


SBI Clerk & RRB PO Mains PDF Course 2023
Quantitative Aptitude Day - 31 (Eng)

III.Total number of students posted in cities D b) 1750


and E is 1450. c) 1680
III. d) 1950
a) Only I true e) None of these
b) Only II true
c) Only III true 20) The number of students rejected in round II
d) All false is ______% of the number of students posted in
e) None of these City E and the number of students posted in City
D is _____% of the number of students rejected
19) Out of the total number of students posted in in Round I.
city C, 60% are living in the company's hostel Find which of the following is satisfy the blank?
and the rest are living in the room rent. Out of the a) 625%, 4.32%
total number of students posted in city D, 25% b) 600, 4.32%
are living in company's hostel and the rest are c) 625%, 5.32%
living in the room rent. Find the total number of d) 550%, 5%
students living in room rent? e) None of these
a) 1800
Click Here to Get the Detailed Video Solution for the above given Questions
Or Scan the QR Code to Get the Detailed Video Solutions

Answer Key with Explanation

Directions (01 - 04): The Number of chargers sold is 60x.


The Number of items sold in shop A is So, 160x=1320
6000*22/100=1320 Or, x=8.25
Let the number of headphones sold in A be So, the number of headphones sold is
100x, =8.25*100=825

Click Here For Bundle PDF Course | support@guidely.in Page 8 of 13


SBI Clerk & RRB PO Mains PDF Course 2023
Quantitative Aptitude Day – 31 (Eng)

The Number of chargers sold is =60*8.25=495 =92*455+138*855+150*1050+100*1250


The Number of wire chargers sold is = =Rs.442350
495*3/5=297
The Number of wireless chargers sold is =495- 3) Answer: B
297=198 Number of wireless headphones sold in shop F
The Number of wireless headphones sold is is 210*120/100=252
=198*3/2=297 Number of wireless chargers sold n shop F is =
The Number of wire headphones sold is = 825- 90*90/100=81
297=528 So, the required difference = [657-81] – [456-
Similarly, we can calculate others' values also. 252] =372
4) Answer: A
I. Average number of wireless headphones sold
in shops C and D is =[250+648]/2=449.
II. Average number of wireless chargers sold in
shops A and D is =[198+432]/2=315.
III. The Total number of wire headphones sold in
all shops together is
=[528+384+230+552+165]=1859.
1) Answer: C
So, z=528-432=96, m=216-90=126, n=230 Directions (05 - 08):
I. n=[z+m] The number of boys in class XII of school A is =
[z+m]=126+96=222, so, not true. 56*10/14=40
II. 3n=[2z+3m] The total number of students in class XII of
2z+3m=2*96+3*126=570, 3n=690, so, it is not school A is 56+40=96
true The Number of students in class X of school A is
III. 5n=2*[z+7m]+23 = 94+96=190
5n=1150 , 2*[z+7m]+23=2*[96+7*126]+23=1956 The Number of boys in class X of school A is =
So, it is not true. 190*9/19=90
The Number of girls in class X of school A is =
2) Answer: D 190-90=100
The total revenue earned by shops C is Similarly, we calculate school B also.
=[230*40/100]*455+[230*60/100]*855+[250*60/1 The Number of boys in class XII of school C is
00]*1050+[250*40/100]*1250 =60*10/15=40

Click Here For Bundle PDF Course | support@guidely.in Page 9 of 13


SBI Clerk & RRB PO Mains PDF Course 2023
Quantitative Aptitude Day – 31 (Eng)

The Total number of students in class XII of So, 3x+4y=60+3=63 and 2x+5y=62-6=56
school C is =60+40=100 By solving the above equation, we get x =13 and
Total number of students in class X of school C y=6
is=100-16=84 So, the total number of students in school F is =
The Number of boys in class X of school C is = 5*13+9*6=65+54=119
84*14/24=49
The Number of girls in class X of school C is = 7) Answer: C
84-49=35 Quantity I
Similarly, we can calculate the values of schools So, difference is = [40+12]/2-[32+14]/2=26-23=4
D, E also. Quantity II
So, difference = [90+42]/2- [56+48]/2=66-52=14
Quantity I < Quantity II

8) Answer: B
The Number of students of class XII take
admission to private colleges from C is =
[40+60]*60/100=60
The Number of students of class XII take
admission to private colleges from D is

5) Answer: A 60*2/3=40

The total number of students take admission in The Number of students of class XII takes

the science stream of schools A and B together admission to government college from school D

is = [100+90] *4/10+ [48+32]*3/8=76 + 30 = 106 is =[40+48]-40=48

The total number of students take admission in So, required percentage = [48/88]*100=54.54%

the arts stream of school B is = [48+32]*2/8 +


[100+90] *5/10 =20 + 95 = 115 Directions (09-12):

Required percentage = [106/115] *100=92.17% Let's say the number of government and private
buses running in C is 3x and 2x, respectively.

6) Answer: C Let's, The Number of government and private

Let the number of boys and girls in class X of buses running in D is between 3y and 2y.

school F be 3x and 2x. The number of government buses running in A is

The number of boys and girls in class XII of 3x-3.

school F is 4y and 5y.

Click Here For Bundle PDF Course | support@guidely.in Page 10 of 13


SBI Clerk & RRB PO Mains PDF Course 2023
Quantitative Aptitude Day – 31 (Eng)

The number of government buses running in A is Number of government non-ac buses running in
3y*100/160=15y/8. city A is 30-18=12
So, 3x-3=15y/8 The number of private ac buses running in city A
Or, 8x-8=5y is = 24-18=6
Or, 8x-5y=8 The Number of private non-ac buses running in
The number of private buses running in B is = city A is = 20-6=14
[2y+3]. So, required percentage = [14/12]*100=116.66%
The number of government buses running B is =
[2y+3] *100/140 = [2y+3] *5/7. 10) Answer: C
So, [2y+3]*5/7+15y/8=55 The number of auto running in D is 56-
Or, 10y+15+105y/8=55*7 25*120/100=56-30=26
Or, 80y+120+105y=55*7*8=3080 The number of taxis running in B is =
Or, 185y=2960, y=2960/185=16 35*120/100=42
So, 8x=8+80=88 So, the required difference is = 42-26=16
So, x=88/8=11
The Number of government and private buses 11) Answer: C
running in C is 3* 11 = 33 and 2* 11 = 22. The number of buses running in city F is
Let, The Number of government and private =30*7/6+ 35*8/7=35+40=75
buses running in D is 3* 16 = 48 and 2* 16 = 32. The number of buses running in city G is =
The number of government buses running in A is 75+20=95
3x-3 = 3*11-3 = 30. So, the difference is 95*2/5-25=38-25=13
The number of government buses running in A is
3y*100/160=15y/8. 12) Answer: D
=15*16/8=30 So, x=30+25+33+48=136
The number of private buses running in B is = So, y=20+35+22+32=109
[2y+3] = 32 + 3 = 35. So, 136*109=14824
The number of government buses running B is =
[2y+3]. *5/7=35*5/7=25 Directions (13 - 16):
The number of private buses running in A is 25- So, in scheme A, 15000*25*2/100=7500
5=20. 15000*(1+25/100)2-15000=8438
9) Answer: C So, scheme A provides compound interest.
The number of government ac buses running in In scheme B, 12320*15*3/100=5544
city A is 30*3/5=18 So, scheme B provides simple interest.

Click Here For Bundle PDF Course | support@guidely.in Page 11 of 13


SBI Clerk & RRB PO Mains PDF Course 2023
Quantitative Aptitude Day – 31 (Eng)

Let scheme C provides r% interest So, t=13392*100/ (18*12400) =6


So, r=5400*100/ (15000*2) =18% Scheme D provide compound interest and the
Scheme C provides at 18% rate of interest. interest rate is 20%
Let Scheme D provides at n% rate of interest. Time is = 6/2=3 years
So, 8000*(1+n/100)2=11520 So, interest amount is =20400*[1+20/100]3-
So, (1+n/100)2=[11520/8000]=1.44 20400= Rs.14851 (approx)
Or, [1+n/100) ^2= [1.2] ^2
So, n=0.2*100=20% 15) Answer: C
Scheme E 13500*10*2/100=2700 Quantity I
So, scheme E provides compound interest. Interest in scheme E is = 16400*[1+10/100]2-
Scheme F, 12562*16*3/100=6029.76 16400 =19844-16400=3444
So, scheme F provides simple interest. Interest in scheme F is = 18600*16*2/100 =5952
Scheme G, 20000*30*2/100=12000 So, difference = 5952-3444=2508
20000*(1+30/100)2-20000=13800 Quantity II
So, scheme G provides compound interest. The difference of interest is
13) Answer: B = [10400*[1+30/100]2-10400]-11600*15*2/100
Let the person invest Rs. x at scheme A and Rs. = 7176-3480=3696
(28000-x) at scheme B. Quantity I < Quantity II
So, interest earn from scheme A is
x*(1+25/100)2-x=25x/16 – x =9x/16 16) Answer: C
Interest earns from scheme B is = [28000- Let's check options C,
x]*15*2/100 Rs.10400 invest in scheme A,
= [28000-x]*3/10 So, interest = 10400*[1+25/100]3-10400=9913
So, 9x/16 + [28000-x]*3/10=12600 15200 invest in scheme F, so,
Or, 90x+28000*48-48x=126000*16 15200*16*2/100=4864
Or, 42x=672000, x=672000/42=16000 So, option C is satisfied.
So, required difference is = [16000-(28000-
16000)] =4000 Directions (17-20):
The number of students who appeared in the
14) Answer: A exam is = 50000-10000=40000
Scheme C provides simple interest and an So, R%= [40000/50000]*100=80%
interest rate 18% So, the number of students applying is 50000,
So, 12400*t*18/100=13392

Click Here For Bundle PDF Course | support@guidely.in Page 12 of 13


SBI Clerk & RRB PO Mains PDF Course 2023
Quantitative Aptitude Day – 31 (Eng)

The number of students who qualify for round I is Required percentage =


20000 [6912/3168]*100=218.18%
The Percentage of students who appeared in
round II is = R+10=90% 18) Answer: B
The Number of students who appeared in round I. Difference between the number of students
II exams is =20000*90/100=18000 who did not appear in round I and the number of
The Number of students selected in the exam is students who did not appear in round II is
= 18000-3600=14400 =10000-2000=8000.
So, 14400 students posted in different cities. II. Difference between the number of students
The Number of students posted in city A is = rejected in the first round and the number of
[14400*80*100/2] =5760 students rejected in round II is =20000-
So, M %+[( M/6)-1] %=( 100-40-6-20) % 3600=16400.
Or, 7M=34*6+6=210, M=210/7=30 III.Total number of students posted in cities D
So, The Number of students posted in city B is = and E is 864+576=1440.
14400*30/100=4320 So, only II true
The Number of students posted in city C is =
14400*20/100=2880 19) Answer: A
The Number of students posted in city D is = The total number of people living room rent is
14400*6/100=864 = 2880*40/100+864*75/100=1800
The Number of students posted in city E is =
14400-864-2880-4320-5760=576 20) Answer: A
17) Answer: D The number of students rejected in round II is
The Total number of males posted in cities A 3600
and B together is So, percentage is = [3600/576]*100=625%
= [5760*3/4] + [4320*3/5] =4320+2592=6912 The number of students rejected in the round I is
The Total number of females posted in cities A 20000
and B together is So, percentage is = [864/20000]*100=4.32%
= [5760+4320]-6912=3168

Click Here For Bundle PDF Course | support@guidely.in Page 13 of 13


SBI Clerk & RRB PO Mains PDF Course 2023
ENGLISH Day - 31

English Language

Directions (1-5): The following paragraph has continue to be flooded. Delhi’s officials have
five words/phrases which are in bold; they may attributed this to the flooding of the Yamuna in
or may not be correct and might need upstream States, particularly at Yamunanagar in
replacement. For each of the questions that Haryana, with the bargains (C) in Delhi unable to
follows the paragraph, choose a word/phrase to effectively regulate and redirect the river’s flow.
replace the respective bold word/phrase (or) This however elides the role of Delhi’s
choose option (e) if no replacement is required. infrastructural development that through the
The devastating floods across North India have years has given short shrift to restricting
renewed attention on the dynamic between construction on the Yamuna’s floodplains, failed
climate change, urbanisation and the to prioritise the desilting (D) of drains ahead of
infrastructural lacunae that bedevil India’s large the monsoon, and scrimped on steps to avoid the
cities. India is now right in the middle of the large-scale concretisation of the city. Though
monsoon and it is only to be expected, given the there is the case that even upstream of Delhi,
topography of the hill States, that lengthening (A) riverbed mining has meant that huge amounts of
rains will cause landslides and pose extreme silt from Haryana block the natural flow of the
threats to life and property. Himachal Pradesh, river, blame games and pointing to “record rains”
Punjab, Haryana and Delhi are among the States are unhelpful. The increased probability of spells
that have reported record rainfall and at least 60 of extreme rain, given warming trends in the
deaths have been confirmed though the actual Arctic as well as the Arabian Sea, means that
toll may be higher. However, it is the inundation there will be several more instances of flooding in
of Delhi, a city that is not usually associated with the future. While urban flooding is far more
rains, that has brought to national focus the frequent in Bengaluru, Chennai and Mumbai,
disasters that lie in store. In the terminology of Delhi should no longer consider itself immune
the India Meteorological Department, Delhi given that its population and infrastructural needs
received ‘excess’ and ‘large excess’ rain on five are only going to expand. Much like the National
out of eight days, from July 3-10. On July 9, it Capital Territory evolved a joint management
recorded 221.4 mm of rain, more than the 209.7 strategy to handling (E) air pollution, on realising
mm that is the average for all of July. While this that clean air is interdependent on action by all
contributed to the flooding, rainfall in the last few cities, these States must set aside their
days has alone (B) reduced. And yet, large parts differences and evolve a joint strategy on
of the city which include iconic landmarks such countering future floods.
as the Red Fort and the Supreme Court,

Click Here For Bundle PDF Course | support@guidely.in Page 1 of 11


SBI Clerk & RRB PO Mains PDF Course 2023
ENGLISH Day - 31

1) Which of the following words will best replace e) No replacement required


the word (A) to make the respective sentence
correct and meaningful ? 5) Which of the following words will best replace
a) expanded the word (E) to make the respective sentence
b) wide correct and meaningful ?
c) enlarged a) avoid
d) extended b) tackle
e) No replacement required c) evade
d) messing
2) Which of the following words will best replace e) No replacement required
the word (B) to make the respective sentence
correct and meaningful ? Directions (6-10) : In each of the questions given
a) dramatically below a sentence is given with three blanks in
b) somewhat each. Corresponding to each question three
c) mildly columns are given with three words in each
d) lightly column. You are required to choose the correct
e) No replacement required combination of words from the three columns
that will perfectly fit into the blanks to make the
3) Which of the following words will best replace sentence contextually correct and meaningful.
the word (C) to make the respective sentence 6) _________ (1) 25 years of the India-France
correct and meaningful? strategic partnership was at the top of the
a) dam _______ (2) during Prime Minister Narendra
b) garage Modi’s two-day visit to France and his meetings
c) barrages with French President Emmanuel Macron last
d) barriers week, which ended with a ________ (3) of
e) No replacement required agreements and a slew of defence deals.

COLUMN 1 COLUMN 2 COLUMN 3


4) Which of the following words will best replace
the word (D) to make the respective sentence A. celebrating D. aim G. few

correct and meaningful?


B. condemning E. agenda H. number
a) collecting
b) enhancing C. observing F. meeting I. many
c) storing a) A-F-I
d) depositing

Click Here For Bundle PDF Course | support@guidely.in Page 2 of 11


SBI Clerk & RRB PO Mains PDF Course 2023
ENGLISH Day - 31

b) A-E-H
C. striated F. prospering I. capturing
c) B-F-G
a) B-E-G
d) C-D-G
b) A-D-H
e) C-F-I
c) A-F-I
d) B–D-I
7) The latest Consumer Price Index data
e) C-F-H
showing a _________ (1) in retail inflation proves
exactly why the RBI’s monetary authorities have
9) India and the U.S. on July 17 __________ (1)
_______ (2) the need to keep the policy approach
to strengthening bilateral ties and to _______ (2)
firmly _____ (3) towards ensuring price stability.
pursue the G20 agenda, which includes taking
COLUMN 1 COLUMN 2 COLUMN 3 coordinated action to deal with climate change

A. falling D. reiterated G. stable and look _____ (3) to alternate mechanisms


to fund energy transition.
B. resurgence E. stopped H. unbent
COLUMN 1 COLUMN 2 COLUMN 3
C. improvement F. weaken I. tilted
A. committing D. passively G. backward
a) A-E-I
b) B-E-H B. commits E. inactively H. forward

c) B-D-I C. committed F. actively I. away


d) C-D-G
a) A-D-I
e) C-F-I
b) A-F-G
c) B-E-H
8) Immigration has ________ (1) housing
d) B-F-I
capacity in the Netherlands at a time when the
e) C-F-H
continent has been ________ (2) with a cost of
living crisis, and worsened by Russia’s _________
10) Fishing activities off the coast of Kerala have
(3) of Ukraine and western sanctions on
been ________ (1) till July 20 as squally weather,
Moscow.
with wind speed reaching 40-45 kmph ______ (2)
COLUMN 1 COLUMN 2 COLUMN 3 to 55 kmph, is likely to ______ (3) along and off

A. stained D. flourishing G. invasion the Kerala coast.

COLUMN 1 COLUMN 2 COLUMN 3


B. strained E. struggling H. withdrawal
A. stop D. calm G. surrender

Click Here For Bundle PDF Course | support@guidely.in Page 3 of 11


SBI Clerk & RRB PO Mains PDF Course 2023
ENGLISH Day - 31

CCB, ____________to crack one of the biggest-


B. promoted E. gusting H. cease
ever seizures so far.
C. suspended F. trickle I. prevail I. who stayed in and around the city
a) A-D-H II. which stayed within the city
b) B-E-G III. whom stays in the city
c) B-F-I a) Only I
d) C-E-I b) Only II
e) C-D-H c) Only II
d) Both I and II
Directions (11-15): In each question a sentence e) Both II and III
is given with a blank. You are provided with three
fragments I, II and III. You have to identify which 13) If you encounter any issues with your order,
statement/statements can carry forward the such as receiving a damaged or incorrect item,
given sentence in the most logical way so as to _________.
make the sentence coherent and contextually I. you will reached out to the customer support
correct. for redressal
11) __________, she has to give bank account II. you can reach out to the online store's
details with a pass book of that account during customer support for assistance
registration. III. customer support executive would contact
I. If she wishes to receive money in another you automatically
account a) Only I
II. If she wishes to receive money to another b) Only II
account c) Only III
III. If she wishes to do transactions with a d) Both I and II
different account e) Both II and III
a) Only I
b) Only II 14) Balancing social activities with personal
c) Only III needs and maintaining positive relationships
d) Both I and II ________.
e) Both I and III I. can enhance the quality of social life
II. can improve the social life of an individual
12) It was a three week-long covert operation by III. can lead us to a happy and lively life
officials of the Women Protection Wing of the a) Only I
b) Only II

Click Here For Bundle PDF Course | support@guidely.in Page 4 of 11


SBI Clerk & RRB PO Mains PDF Course 2023
ENGLISH Day - 31

c) Both I and II massive repair and revamp (C) exercise is afoot


d) Both I and III (D).
e) All I, II and III a) A
b) B
15) Effective communication is the process of c) C
conveying information, ideas and thoughts d) D
clearly and accurately to others ____________. e) All correct
I. inorder of proving yourself and showcase your
talents 17) The National MPI measures simultaneous
II. to make them feel low and inferior deprievations (A) across three equally weighted
III. in a way that is understood and leads to the dimensions (B) of health, education, and
desired outcome standard of living that are represented by 12
a) Only I sustainable (C) development goal (SDG) aligned
b) Only II (D) indicators.
c) Only III a) A
d) Both II and III b) B
e) None of the above c) C
d) D
Directions (16-20): In each question below, four e) All correct
words printed in bold are given. These are
numbered (A), (B), (C) and (D). One these words 18) Russia has halted an unprecedented (A)
printed in bold might either be wrongly spelt or wartime deal that allows grain (B) to flow from
inappropriate to the context of the sentence. Find Ukraine to countries in Africa, the Middle East
out the word that is inappropriate or wrongly and Asia where hunger (C) is a growing threaten
spelt, if any. The alphabet of the word is your (D) and high food prices have pushed more
answer. If the words printed in bold are correctly people into poverty.
spelt and appropriate to the context of the a) A
sentence then mark (e), i.e. 'All Correct', as your b) B
answer. c) C
16) Crumbling seats, broken armrests, and d) D
dysfunctional (A) seat-back entertainment e) All correct
screens on Air India’s aircraft often make the
airline a subject of passenger ire (B) but a 19) Insights (A) into the evolutionery (B)
processes of organisms also open big windows

Click Here For Bundle PDF Course | support@guidely.in Page 5 of 11


SBI Clerk & RRB PO Mains PDF Course 2023
ENGLISH Day - 31

into understanding how antimicrobial resistance 20) The first comet observed orbiting (A) a
(C) emerges, how pathogens evade immune planet rather than the sun, it had already been
systems, and possibly, new opportunities (D) to torned (B) into more than 20 pieces and was
prevent them. travelling (C) around Jupiter in a two-year orbit at
a) A the time of discovery (D).
b) B a) A
c) C b) B
d) D c) C
e) All correct d) D
e) All correct
Click Here to Get the Detailed Video Solution for the above given Questions
Or Scan the QR Code to Get the Detailed Video Solutions

Answer Key with Explanation

1) Answer: D Wide, expanded and enlarged are not the right


Extended is the correct word that would replace adjectives in this context and hence discarded.
‘lengthening’ which is incorrect in the context of India is now right in the middle of the monsoon
the sentence. and it is only to be expected, given the
Rains are extended for more days, not topography of the hill States, that extended rains
expanded, wide, enlarged and lengthening. will cause landslides and pose extreme threats
Extended is the correct word among the given to life and property.
options that best replaces the word (A) in the
passage, so we go for option d as our answer. 2) Answer: A
Lengthening - is wrong and definitely needs Alone is an incorrect word in the passage and
replacement needs replacement.

Click Here For Bundle PDF Course | support@guidely.in Page 6 of 11


SBI Clerk & RRB PO Mains PDF Course 2023
ENGLISH Day - 31

Looking at the meaning of the adjacent Desilting - the removal of silt from a body of
sentences to the sentence which has the bold water.
word - convey, the rainfall has reduced in a As the word ‘desilting’ gives proper meaning to
noticeable/considerable way (because the the sentence, we go for option e - no
sentence following starts with ‘yet’ which means replacement required.
something noticeable has happened)
Among the given options dramatically is the right 5) Answer: B
pick to replace the given word ‘alone’. Handling is not the correct word in the sentence
Somewhat, mildly and lightly are just the and needs to be replaced.
opposite to dramatically and hence are not the From the given options ‘tackle’ is the right pick to
right pick. replace the given bold word making the
While this contributed to the flooding, rainfall in sentence correct and meaningful.
the last few days has dramatically reduced. Mess, avoid and evade are the opposites of
tackle and are irrelevant to the sentence, hence
3) Answer: C discarded.
Bargains is totally irrelevant to the context of the Much like the National Capital Territory evolved
passage and hence needs replacement. a joint management strategy to tackle air
Out of the given options ‘barrages’ is the right pollution, on realising that clean air is
word that can replace ‘bargain’ to add meaning interdependent on action by all cities, these
to the sentence. States must set aside their differences and
Barrage(noun) in this context is a dam. evolve a joint strategy on countering future
Garage and barriers are incorrect and do not fit floods.
the passage, ‘dam’ is singular and hence not the
right fit. 6) Answer: B
Delhi’s officials have attributed this to the Three words that are required to fill the blanks
flooding of the Yamuna in upstream States, are celebrating, agenda and number.
particularly at Yamunanagar in Haryana, with the First blank - condemning is a wrong word in this
barrages in Delhi unable to effectively regulate context because the sentence is in positive tone
and redirect the river’s flow. whereas condemning has a negative/bold/harsh
meaning(to say someone is wrong) - eliminate
4) Answer : E option c
The given word desilting is appropriate and
hence no change is required.

Click Here For Bundle PDF Course | support@guidely.in Page 7 of 11


SBI Clerk & RRB PO Mains PDF Course 2023
ENGLISH Day - 31

Second blank - aim and meeting are not the best Third blank - ‘tilted towards’ is the correct
picks among the given options in column 2 - phrase(unbent and stable does not fit the blank)
eliminate option a,d and e The latest Consumer Price Index data showing a
Hence, option b is the correct answer for this resurgence in retail inflation proves exactly why
question. the RBI’s monetary authorities have reiterated
Third blank - few and many are wrong, H) the need to keep the policy approach firmly tilted
number is the right fit for the third blank. towards ensuring price stability.
Celebrating 25 years of the India-France
strategic partnership was at the top of the 8) Answer: A
agenda during Prime Minister Narendra Modi’s Strained, struggling and invasion are the right
two-day visit to France and his meetings with words that fit the given blanks.
French President Emmanuel Macron last week, First blank - stained and striated have meanings
which ended with a number of agreements and a which are different from the context of the given
slew of defence deals. sentence, hence does not fit the first blank -
eliminate b,c and e.
7) Answer: C Stained - dyed or discoloured
Resurgence, reiterated and tilted are the three Strained - worried because of having too much
words that fill the given three blanks to deal with
appropriately. Striated - marked with long, thin parallel streaks.
First blank - Resurgence(the return and growth Too much immigration has strained the housing
of an activity that had stopped) will fit the blank capacity.
in a perfect way, falling and improvement are not Second blank - the sentence talks about a living
the appropriate picks for the first blank as they crisis hence flourishing and prospering aren’t the
do not add meaning to the given sentence - correct words, word(E) struggling is the right pick
eliminate option a, d and e - eliminate option d
Second blank - out of the three given words, Option a is the correct answer for this question
reiterated is the correct word that fits the given Third blank - the situation is worsened by - this
blank(weaken is wrong both in meaning and means the blank has to be filled with a word
form of usage, stopped is incorrect in this which creates a negative impact and worsens
context) - eliminate option b the condition of Netherland - invasion is the
Reiterated - to repeat/ to make sure correct word.
Option c is the correct answer for this particular Immigration has strained housing capacity in the
question. Netherlands at a time when the continent has

Click Here For Bundle PDF Course | support@guidely.in Page 8 of 11


SBI Clerk & RRB PO Mains PDF Course 2023
ENGLISH Day - 31

been struggling with a cost of living crisis, and opposite to gust of wind) - eliminate options c
worsened by Russia’s invasion of Ukraine and and e
western sanctions on Moscow. Gusting - a sudden brief rush of wind
Trickle - to go somewhere slowly and gradually
9) Answer: E Third blank - prevail si the appropriate word to fill
The three words required to fill the given blanks the third blank(because the situation will prevail
are committed, actively and forward. and hence the precautionary measures)
First blank - both commits and committing are Prevail - to exist
wrong and do not fit the first blank of the Surrender and cease means to end/to stop
sentence - eliminate options a, b, c and d. Fishing activities off the coast of Kerala have
Option e is the answer for this question. been suspended till July 20 as squally weather,
Second blank - actively is the right fit because it with wind speed reaching 40-45 kmph gusting to
is the only word which is relevant to the context 55 kmph, is likely to prevail along and off the
of the sentence (passively and inactively have Kerala coast.
the opposite meaning)
Third blank - away and backward are irrelevant 11) Answer: E
and do not fit the third blank in an appropriate Both the statements I and III are grammatically
way. correct and make the given sentence meaningful
India and the U.S. on July 17 committed to by filling the given blank.
strengthening bilateral ties and to actively pursue Sentence II is incorrect because the preposition
the G20 agenda, which includes taking ‘to’ is wrong and incorrect. The correct phrases -
coordinated action to deal with climate change Receive money from (or) receive money in.
and look forward to alternate mechanisms to The sentences:
fund energy transition. If she wishes to receive money in another
account, she has to give bank account details
10) Answer: D with a pass book of that account during
The three words required to fill the given blanks registration.
are suspended, gusting and prevail. If she wishes to do transactions with a different
First blank - stop is an incorrect word and hence account, she has to give bank account details
it can be discarded - eliminate option a with a pass book of that account during
Second blank - gusting is the right word(trickle registration.
and calm have similar meanings and just the
12) Answer: A

Click Here For Bundle PDF Course | support@guidely.in Page 9 of 11


SBI Clerk & RRB PO Mains PDF Course 2023
ENGLISH Day - 31

Who is the correct pronoun to be used in the Balancing social activities with personal needs
given sentence. Which and whom are incorrect and maintaining positive relationships can
and hence the respectives options are improve the social life of an individual.
eliminated. Balancing social activities with personal needs
The sentence: and maintaining positive relationships can lead
It was a three week-long covert operation by us to a happy and lively life.
officials of the Women Protection Wing of the
CCB, who stayed in and around the city to crack 15) Answer: C
one of the biggest-ever seizures so far. Only statement III fits the blank perfectly to
complete the sentence in the correct way
13) Answer: B possible.
Only statement II is grammatically correct and Statement III makes the sentence meaningful
meaningful too. too.
Statement I - grammar error(you will reached out Statement I and II are either grammatically
is wrong) incorrect or irrelevant.
Statement II - grammar error and irrelevant to The sentence:
the context Effective communication is the process of
The sentence: conveying information, ideas and thoughts
If you encounter any issues with your order, clearly and accurately to others in a way that is
such as receiving a damaged or incorrect item, understood and leads to the desired outcome.
you can reach out to the online store's customer
support for assistance. 16) Answer: A
The word (A) is misspelt and the right spelling is
14) Answer: E dysfunctional.
All the given statements can be used to Dysfunctional - not working normally/not
complete the given sentence to make it following the usual patterns
meaningful and correct. Ire - anger
The sentences: Revamp - to change something in order to make
Balancing social activities with personal needs it more attractive or more modern
and maintaining positive relationships can Afoot - being prepared
enhance the quality of social life. Crumbling seats, broken armrests, and
dysfunctional seat-back entertainment screens
on Air India’s aircraft often make the airline a

Click Here For Bundle PDF Course | support@guidely.in Page 10 of 11


SBI Clerk & RRB PO Mains PDF Course 2023
ENGLISH Day - 31

subject of passenger ire but a massive repair countries in Africa, the Middle East and Asia
and revamp exercise is afoot. where hunger is a growing threat and high food
prices have pushed more people into poverty.
17) Answer: A
It must be ‘deprivations’ instead of 19) Answer: B
‘depreivations’. The word (A) is wrongly spelt The word (B) is misspelt, it must be evolutionary
Deprivations - the state of being kept from instead of evolutionery.
possessing Insights - an accurate and deep understanding
Sustainable - involving the use of natural Evolutionary - relating to the gradual
products and energy in a way that does not harm development of something
the environment Insights into the evolutionary processes of
The National MPI measures simultaneous organisms also open big windows into
deprivations across three equally weighted understanding how antimicrobial resistance
dimensions of health, education, and standard of emerges, how pathogens evade immune
living that are represented by 12 sustainable systems, and possibly, new opportunities to
development goal (SDG) aligned indicators. prevent them.

18) Answer: D 20) Answer: B


The word (D) is inappropriate, replace threaten The word (B) is incorrect because ‘torn’ is
with threat to make the sentence correct and already a past participle of tear, torned is wrong.
meaningful. The first comet observed orbiting a planet rather
Unprecedented - never having happened or than the sun, it had already been torn into more
existed before than 20 pieces and was travelling around Jupiter
Threat - a warning in a two-year orbit at the time of discovery.
Russia has halted an unprecedented wartime
deal that allows grain to flow from Ukraine to

Click Here For Bundle PDF Course | support@guidely.in Page 11 of 11


SBI Clerk & RRB PO Mains PDF Course 2023
Reasoning Ability Day - 32 (Eng)

Reasoning Ability
Directions (1-5): Study the following information 1. Which of the following box is kept on Shelf 3 of
carefully and answer the questions given below. Stack 2?
Eight boxes D, F, J, L, N, Q, T and W are kept on a) The box which is kept immediately below box
four shelves. Each Box is different in shapes viz. J
Cone, Square, Rectangle, Circle, Oval, Sphere, b) Box N
Cube and Octagon but not necessarily in the c) The box which is square in shape
same order. d) Box L
Note: e) The box which is rectangle in shape
I. Each shelf has two stacks viz., Stack 1 and
Stack 2. Stack 1 is exactly to the west of Stack 2. 2. On which of the following shelf and stack does
Only one box is kept on each stack of each shelf. the cube shape box is kept?
II. Stack 1 of shelf 2 is immediately above Stack a) Shelf 4, Stack 1
1 of Shelf 1 whereas Stack 2 of Shelf 2 is b) Shelf 2, Stack 1
immediately above Stack 2 of Shelf 1 and so on. c) Shelf 2, Stack 2
III. The shelves are numbered 1 to 4 from the d) Shelf 3, Stack 1
bottom to top respectively. e) Shelf 4, Stack 2
Box W is kept on an even numbered Shelf. Oval
shape box is kept immediately below box W in 3. Which of the following box is kept two shelves
different stacks. The number of shelves above above Oval shape box?
the Oval shape box is one more than the number a) Box T
of shelves below box D. Square shape box is b) The sphere shaped box
kept immediate northeast of box D. Box J is kept c) The Cone shaped box
two shelves below the Cone shape box. Sphere d) Box Q
shape box is kept southwest of box J. Cone e) The Octagon shaped box
shape box is kept northwest of box F, which is
not oval shaped box. Circle shape box is kept 4. Four of the following five are alike in a certain
immediately below box F in different stacks. Box way and thus form a group as per the given
Q is kept southeast of box L, which is neither in arrangement. Which of the following does not
circle shape nor in Oval shape. Either box F or belong to that group?
box Q is rectangle in shape. Box T is kept a) Box T
immediately above the Rectangle shape box. b) The box which is in Circle shape
Octagon shape box is kept immediate northwest c) Box F
of the Cube shape box. d) The box which is in Oval shape

Click Here For Bundle PDF Course | support@guidely.in Page 1 of 9


SBI Clerk & RRB PO Mains PDF Course 2023
Reasoning Ability Day - 32 (Eng)

e) Box J the right of the one who is facing R. W is the only


daughter of U’s only son. X faces the one who
5. Which of the following statement(s) is/are true sits immediate left of S, who is the brother of W.
with respect to the final arrangement? Z, who is the son-in-law of U, faces the one who
I. Boxes D and J are kept in different stacks sits fourth to the left of S. S is the husband of T.
II. Box J is in Square shape Y is the niece of O, who is the brother-in-law of
III. Box N is kept immediately below box L in the Q. V sits immediate right of X, who is the brother-
same stack in-law of Z. Neither O nor P faces V. Both R and
a) Only I V are siblings, where R is the son of Y and sits
b) Only II and III adjacent to his mother. O neither faces away
c) Only III from the centre nor the married person.
d) Only I and III 6) How X is related to the one who sits second to
e) All I, II and III the right of U?
a) Niece
Directions (6-10): Study the following information b) Son
carefully and answer the given question. c) Nephew
Twelve persons – O, P, Q, R, S, T, U, V, W, X, Y d) Father
and Z of three generations family are sitting e) None of these.
around two concentric circular tables facing each
other in such a way that six persons are sitting at 7) Who among the following person sits third to
each table. The person sitting at the outer the left of S’s wife?
circular table is facing the centre and the person a) The one who sits third to the left of Q
sitting at the inner circular table is facing away b) U
from the centre. c) The one who faces the one who sits
Note: I-Not any single parents have a child. immediate right of X.
II- Not any couple faces to each other. d) P
III- If it is given that A and B face each other, e) O
then both are sitting at different tables and if it is
given that A and B sit opposite to each other, 8) If V is the sister of R, then how V is related to
then both are sitting at the same table. Q?
Q is the mother-in-law of P and sits third to the a) Niece
left of R. Only two persons sit between the one b) Granddaughter
who faces Q and S, who sits immediate left of T. c) Daughter
U is the father of P’s husband and sits second to d) Aunt

Click Here For Bundle PDF Course | support@guidely.in Page 2 of 9


SBI Clerk & RRB PO Mains PDF Course 2023
Reasoning Ability Day - 32 (Eng)

e) Sister Step III: 19 164 10 81 181 25


Step IV: 57 328 20 243 543 75
9) The number of persons sitting between Q’s Step V: 74 19 02 15 38 74
husband and O is same as the number of Step V is the last step of the input.
persons sitting between_______ Input: 3276 5891 8423 9376 6512 2497
a) Z and his brother-in-law 11) How many even digits are there in the final
b) R and his sister output?
c) U and S a) Three
d) Z and R b) Five
e) None of these c) Four
d) Six
10) Which of the following statement(s) is/are not e) More than six
true as per the given arrangement?
I. Q is the grandmother of Y’s son. 12) What is the sum of the numbers which is
II. Y is not an immediate neighbour of her third from the left end in step II and fifth from the
husband. right end in step III?
III. Only two persons sit between T and X’s a) 1246
daughter b) 1162
a) Only I and III c) 1076
b) Only II d) 1026
c) Only III e) None of these
d) Only II and III
e) Only I 13) What is the 3rd step of the number "8693”?
a) 216
Directions (11-15): Study the following b) 116
information carefully and answer the given c) 146
questions. Number arrangement machines when d) 236
given an input line of numbers rearrange them e) 106
following a particular rule in each step. The
following is an illustration of input and 14) What is the sum of all the prime digits in the
rearrangement. second step?
Input: 4679 3285 9731 6830 4583 5321 a) 31
Step I: 9764 8532 9731 8630 8543 5321 b) 28
Step II: 1313 1008 1010 0809 1109 0605 c) 46

Click Here For Bundle PDF Course | support@guidely.in Page 3 of 9


SBI Clerk & RRB PO Mains PDF Course 2023
Reasoning Ability Day - 32 (Eng)

d) 29 Emirates, which doesn’t take off before B. The


e) None of these flight which belongs to Qatar Airways takes off
immediately after Air Asia. F takes off two flights
15) What is the difference between the number after C, which doesn’t belong to Indigo.
which is second from the left end and third from 16) Which of the following flight takes off
the right end in step IV? immediately after E?
a) 0 a) The flight which takes off two flights before B.
b) 10 b) C
c) 15 c) A
d) 13 d) The one which belongs to Qatar Airways.
e) 09 e) None of these

Directions (16-20): Study the following 17) Which of the following flight belongs to Air
information carefully and answer the given Asia?
questions. a) B
Eight flights - A, B, C, D, E, F, G and H take off in b) The one which takes off immediately before D.
a sequence one after another. They belong to c) E
eight different airlines- Air India, Indigo, Spice-jet, d) The one which takes off two flights after A.
Air Asia, Go-first, Vistara, Emirates and Qatar e) H
Airways.
Note: The flight whose name starts with a vowel 18) How many flights take off between the one
neither takes off consecutively nor belongs to the which belongs to Vistara and Flight A?
airlines whose name starts with a vowel. a) Two
B takes off three flights before the one which b) As many flights take off between G and the
belongs to Go-first. Only two flights take off one which belongs to Spice- jet.
between the one which belongs to Air India and c)As many flights take off between C and the one
D, which takes off immediately before B.E takes which belongs to Indigo.
off four flights after G, neither of them belongs to d) Three
Air India and Go-first. G doesn’t take off e) None
immediately before D. The number of flights
takes off before E is one less than the number of 19) Which of the following statement is/are not
flights takes off after the flight which belongs to true as per the given arrangement?
Vistara, which is not D. Only two flights take off I. Flight B takes off three flights before the one
between A and the one which belongs to which belongs to Go-first.

Click Here For Bundle PDF Course | support@guidely.in Page 4 of 9


SBI Clerk & RRB PO Mains PDF Course 2023
Reasoning Ability Day - 32 (Eng)

II. Two flights take off between one which 20) If Flight B is related to the one which belongs
belongs to Air Asia and Spice-jet. to Air Asia and Flight H is related to the one
III. B doesn’t belong to Emirates which belongs to Emirates in a certain way, then
a) Only I and III Flight F is related to which of the following flight?
b) Only III a) The one which belongs to Go-first
c) Only I b) The one which belongs to Vistara.
d) Only II and III c) The one which belongs to Indigo.
e) Only II d) The one which belongs to Air India
e) The one which belongs to Spice Jet
Click Here to Get the Detailed Video Solution for the above given Questions
Or Scan the QR Code to Get the Detailed Video Solutions

Answer Key with Explanation

Directions (1-5): We have,


1. Answer: E  Box W is kept on an even numbered
2. Answer: C Shelf.
3. Answer: E  Oval shape box is kept immediately below
4. Answer: B (All boxes are kept in stack 2 the box W in different stacks.
except option b) From the above conditions, there are four
5. Answer: A possibilities
Final arrangement

Click Here For Bundle PDF Course | support@guidely.in Page 5 of 9


SBI Clerk & RRB PO Mains PDF Course 2023
Reasoning Ability Day - 32 (Eng)

 Box Q is kept southeast of box L, which is


neither in circle shape nor in Oval shape.
 Either box F or box Q is rectangle in
shape.
 Box T is kept immediately above the
Rectangle shape box.
Again we have,  Octagon shape box is kept immediate
 The number of shelves above the Oval northwest of the Cube shape box.
shape box is one more than the number After applying above conditionsCase2 gets
of shelves below box D. eliminated, Hence Case3 shows the final
 Square shape box is kept immediate arrangement
northeast of box D.
 Box J is kept two shelves below the Cone
shape box.
 Sphere shape box is kept southwest of
box J.
 Cone shape box is kept northwest of box
F, which is not oval shaped box.
 Circle shape box is kept immediately Direction (6-10):
below box F in different stacks. 6. Answer: D
After applying above conditionsCase1 and 7. Answer: C
Case4 get eliminated 8. Answer: B
9. Answer: A
10. Answer: B
Final Arrangement

Again we have,

Click Here For Bundle PDF Course | support@guidely.in Page 6 of 9


SBI Clerk & RRB PO Mains PDF Course 2023
Reasoning Ability Day - 32 (Eng)

Q-

P= (+/-)
Again we have,
 U is the father of P’s husband and sits
second to the right of the one who is facing
R.
 W is the only daughter of U’s only son.
 X faces the one who sits immediate left of
S, who is the brother of W.
Blood relation:
 Z, who is the son-in-law of U, faces the
one who sits fourth to the left of S.
 S is the husband of T.
 Y is the niece of O, who is the brother-in-
law of Q.
 V sits immediate right of X, who is the
brother-in-law of Z.

We have,
 Q is the mother-in-law of P and sits third to
the left of R.
 Only two persons sit between the one who
faces Q and S, who sits immediate left of T.
From the above conditions, we have two
possibilities:
Blood relation:
Q-=U+ O+

P-=X+ Y-=Z+

W- S+=T-
Again we have,
B Blood relations:  Neither O nor P faces V.

Click Here For Bundle PDF Course | support@guidely.in Page 7 of 9


SBI Clerk & RRB PO Mains PDF Course 2023
Reasoning Ability Day - 32 (Eng)

 Both R and V are siblings, where R is the Step II: 0909 1013 1007 1213 0707 1111
son of Y and sits adjacent to his mother. Step III: 81 19 149 19 49 11
 O neither faces away from the centre nor Step IV: 243 57 447 57 147 33
married person. Step V: 15 74 57 74 54 18
From the above conditions, case 1 gets Step I: All the digits are arranged in descending
eliminated because O faces away from the order within the numbers.
centre. Hence case 2 shows the final Step II: Sum of the first and fourth digits is
arrangement. written first and then sum of the second and third
digits is written.
Step III: Square of the first and last digits of the
numbers and remove the second and third digits.
Step IV: All the even numbers are multiplied by 2
and all the odd numbers are multiplied by 3.
Step V: Add all the even digits and the square of
all the odd digits within the numbers.
Blood relation

Directions (16-20):
16) Answer: E
17) Answer: B
18) Answer: C
19) Answer: B
20) Answer: D
Final arrangement

Directions (11-15):
11) Answer: C
12) Answer: D
13) Answer: B
14) Answer: D
15) Answer: A
Final arrangement
Input: 3276 5891 8432 9376 6512 2497
Step I: 7632 9851 8432 9763 6521 9742

Click Here For Bundle PDF Course | support@guidely.in Page 8 of 9


SBI Clerk & RRB PO Mains PDF Course 2023
Reasoning Ability Day - 32 (Eng)

after the flight which belongs to Vistara,


which is not D.
 Only two flights take off between A and the
one which belongs to Emirates, which
doesn’t take off before B.

We have,
 B takes off three flights before the one which
belongs to Go-first.
 Only two flights take off between the one
Again we have,
which belongs to Air India and D, which takes off
 The flight which belongs to Qatar Airways
immediately before B.
takes off immediately after Air Asia.
From the above conditions, we have two
 F takes off two flights after C, which doesn’t
possibilities:
belongs to Indigo
From the above condition case 2 gets eliminated
because C belongs to Indigo. Hence case 1
shows the final arrangement.

Again we have,
 E takes off four flights after G, neither of them
belongs to Air India and Go-first. G doesn’t
take off immediately before D.
 The number of flights takes off before E is
one less than the number of flights takes off

Click Here For Bundle PDF Course | support@guidely.in Page 9 of 9


SBI Clerk & RRB PO Mains PDF Course 2023
Quantitative Aptitude Day - 32 (Eng)

Quantitative Aptitude

1) In a mixture of milk and water, the quantity of efficiencies of A, B and C are respectively 20%,
milk is 20 L more than that of water and the cost 30% and 50% of the efficiency of D, then find the
of pure milk is ₹ 48 per L. If the cost of pure milk value which can be filled in the blank?
were 12.5% less, then the cost of the mixture a) Can’t be determined
was ₹ 4.5 per L less than the original cost of the b) 5 days
mixture. When 8 L quantity of this mixture is c)3 days
replaced with the same quantity of milk, then d) 8 days
what will be the cost of the new mixture? e) 6 days
a) ₹ 36.4 per L
b) ₹ 40.4 per L 4) A boat can cover ___ m downstream in
c) ₹ 38.4 per L 25seconds and ___ m upstream in 35 seconds.
d) ₹ 42.4 per L The time taken by the boat to go 540 m
e) ₹ 34.4 per L downstream is 20% more than that taken by the
boat to go 300 m upstream. If the speed of the
2) Ratio of the difference between compound boat in still water were 20% more, then its
interest and simple interest on ₹ X at R% rate downstream speed was 21 m/s.
after 2 years to the difference between Quantity I: Find the value which will be filled in
compound interest and simple interest on ₹ X at the 1st blank?
R% rate after 3 years is 25: 76. If the difference Quantity II: Find the value which will be filled in
between compound interest and simple interest the 2nd blank?
on ₹ (X – 7000) at (R + 1)% rate after 2 years is a) Quantity I > Quantity II
₹ 45, then find the simple interest received on b) Quantity I = Quantity II or the relation can’t be
at (R + 4)% rate after 2 years? established
a) ₹ 1250 c)Quantity I ≤ Quantity II
b) ₹ 2500 d) Quantity I > Quantity II
c)₹ 1750 e) Quantity I ≥ Quantity II
d) ₹ 1500
e) ₹ 2000 5) Average of present ages of A, B and C is 23
years and A’s present age is ___% of D's present
3) Workers A, B and C together can complete a age. After 6 years, the ratio of A’s age to B’s age
work in 9 days and workers B, C and D together will be 2: 3 and C’s present age is 70% of B’s
can complete the same work in ___ days. If the

Click Here For Bundle PDF Course | support@guidely.in Page 1 of 16


SBI Clerk & RRB PO Mains PDF Course 2023
Quantitative Aptitude Day - 32 (Eng)

present age. The difference between the seconds and (t + 2) seconds respectively. The
present ages of B and D is 10 years. speed of the car is 50% more than that of the
Quantity I: Find the value which will be filled in bike and also 4 m/s more than that of the bus.
the blank? Quantity I: If the speed of the bus were ___% of
Quantity II:A’s present age is what per cent of its original speed, it would have crossed the
B’s present age? bridge in 16 seconds.
a) Quantity I > Quantity II Quantity II:If the speed of the bike were ___% of
b) Quantity I = Quantity II or the relation can’t be its original speed, it would have crossed the
established bridge in 20 seconds.
c)Quantity I ≤ Quantity II a) Quantity I > Quantity II
d) Quantity I > Quantity II b) Quantity I ≤ Quantity II
e) Quantity I ≥ Quantity II c)Quantity I > Quantity II
d) Quantity I = Quantity II or the relation can’t be
6)Time taken by a car, a bike and a bus to cross established
a 240 m long bridge is ‘t’ seconds, (t + 5) e) Quantity I ≥ Quantity II

Direction (7-10): Study the following data carefully and answer the questions:
There are three tanks P, Q and R of different capacities and each tank has an inlet pipe and an outlet
pipe connected to it.
Inlet pipe A and outlet pipe M are connected to tank P.
Inlet pipe B and outlet pipe N are connected to tank Q.
Inlet pipe C and outlet pipe O are connected to tank R.
The table given below shows the following data:

Note:
1) The ratio of the efficiency of pipe M to that of pipe N is 3: 2 and the ratio of the efficiency of the pipe M
to that of pipe O is 4: 3.

Click Here For Bundle PDF Course | support@guidely.in Page 2 of 16


SBI Clerk & RRB PO Mains PDF Course 2023
Quantitative Aptitude Day - 32 (Eng)

7) The flowing rate of water from pipe M is 4.5 L 9) If the ratio between capacities of tanks P, Q
per minute, which is 90% of the flowing rate of and R is 7: 6: 8 respectively, the difference
water from pipe N. If the flowing rate of water between the capacities of tanks P and Q is 30 L
from pipe A is ‘x’ L per minute and that from pipe and only pipes A, B and C are connected to tank
B is ‘y’ L per minute, then find the value of ? R, then in what time tank R will be filled?
a) 5.4 a) 8 minutes
b) 6.8 b) 12 minutes
c)5.6 c)10 minutes
d) 7.5 d) 6 minutes
e) 4.9 e) 15 minutes

8) If pipes A and M work with 125% and of 10) Which of the following is/are true?

their respective efficiencies and pipes B and N X: The average time, in which pipe A alone, pipe

work with 125% and 93.75% of their respective B alone and pipe C alone can fill tanks P, Q and

efficiencies, then find that the time, in which R respectively, is 22 minutes.

pipes A and M together can fill tank P with their Y: If the capacity of tank R is 210 L, then the

new efficiencies, will be what per cent of the flowing rate of water from pipe O is 5.75 L.

time, in which pipes B and N together can fill Z: If the difference between the flowing rates of

tank Q with their new efficiencies? water from pipes B and N is 3 L per minute, then

a) 33.33% the capacity of tank Q is 300 L.

b) 40% a) Both X and Y

c)30% b) Both X and Z

d) 37.5% c) None is true

e) 44.44% d) Both Y and Z


e) All are true

Direction (11-14): Study the following data carefully and answer the questions:
The data given below is related to the total number of persons (male + female) who visited a museum
on 4 different days Mon, Tue, Wed and Thu.
The pie chart given below shows the percentage distribution of the total number of persons who visited
the museum on each day.

Click Here For Bundle PDF Course | support@guidely.in Page 3 of 16


SBI Clerk & RRB PO Mains PDF Course 2023
Quantitative Aptitude Day - 32 (Eng)

The Pie chart given below shows the percentage distribution of the number of males, who visited the
museum on each day.

Note:
1) The number of females, who visited the museum on Tue, is 110, which is 175% more than the
females , who visited the museum on Mon.
11) The Cost of ticket for adults is ₹ 50 per adult child. If on Mon, 45% of the total persons were
and the cost of the ticket for children is ₹ 20 per adults and on Wed, 65% of the total persons

Click Here For Bundle PDF Course | support@guidely.in Page 4 of 16


SBI Clerk & RRB PO Mains PDF Course 2023
Quantitative Aptitude Day - 32 (Eng)

were adults, then find the difference between the a) Only B


total amount collected by the museum by selling b) Only B and C
adult’s tickets on Mon and Wed together and that c) Only A and B
collected by selling children’s tickets on Mon and d) Only A and C
Wed together? e) Only C
a) ₹ 9100
b) ₹ 9900 14) If 40% of total males, who visited the
c) ₹ 9500 museum on Mon, were children, 20% of total
d) ₹ 9000 males, who visited the museum on Tue, were
e) ₹ 9300 children, 30% of total males, who visited the
museum on Wed, were children and 45% of total
12)If 74% of the total males, who visited the males, who visited the museum on Thu, were
museum on Tue, were Indian and the remaining children, then find the average of the number of
were foreigners and 90% of the total females, male children, who visited the museum on each
who visited the museum on Tue, were Indian and of the given days?
the remaining were foreigners, then find what a) 40
per cent of the total number of persons, who b) 55
visited the museum on Tue, were foreigners? c) 45
a) 12.5% d) 50
b) 25% e) 60
c) 20%
d) 16.67% Direction (15-18): Study the following data
e) 15% carefully and answer the questions:
The data given below is related to the number of
13) If the number of females, who visited the persons, who took membership in three different
museum on each day, is described in a circle sports (Tennis, Swimming, and Golf) in a sports
and the degree distribution of the number of club. Some took membership in only one sport,
females, who visited the museum on Wed and some took membership in only two sports and
Thu are M° and N° respectively, then find that the remaining took membership in all three
which of the following is/are true? sports.
A: Both M and N are multiple of 24. The number of people, who took membership in
B: The Difference of M and N is a multiple of 12. swimming only, is 80% of those, who took
C:The Value of is 2.5. membership in Tennis only and also 2 more than
those, who took membership in Golf only. The

Click Here For Bundle PDF Course | support@guidely.in Page 5 of 16


SBI Clerk & RRB PO Mains PDF Course 2023
Quantitative Aptitude Day - 32 (Eng)

ratio of the number of persons, who took a) ₹ 2290


membership in both Tennis and Swimming but b) ₹ 2490
not Golf to those, who took membership in both c) ₹ 2590
Tennis and Golf but not Swimming, is 2: 3 and d) ₹ 2190
the number of persons, who took membership in e) ₹ 2390
both Swimming and Golf but not Tennis, is 3
more than those, who took membership of both 17) If the total number of persons, who took
Tennis and Golf but not Swimming. The number membership of only one sports, is M% of the
of persons, who took membership of both total number of persons in the sports club and
Swimming and Golf but not Tennis, is 60% of the total number of persons, who took
those, who took membership of Golf only. The membership of only two sports, is N% of the total
number of persons, who took membership in all number of persons in the sports club, then find
the three sports, is 5, which is 12.5% of those, thatthe value of (2M – 3N) is divisible by which of
who took membership in Tennis only. the following?
15) If the ratio of males to females, who took a) 10
membership of Tennis only, is 11: 9, the ratio of b) 5
males to females, who took membership of c) 15
Swimming only, is 5: 3 and the ratio of males to d) Both (a) and (b)
female, who took membership of Golf only, is 1: e) Both (b) and (c)
2, then find the ratio of total males to total
females, who took membership of only one 18) If 56% of the total number of persons in the
sport? sports club are male, out of which, took
a) 52: 51 membership of at-least two sports, then find that
b) 4: 3 what per cent of total females took membership
c) 104: 103 of only one sport?
d) 13: 12 a) 63.63%
e) 26: 25 b) 57.5%
c) 66.67%
16)If the cost of a Swimming membership is d) 53.33%
₹ 150 per person and the cost of a Golf e) 60%
membership is ₹ 180 per person, then find the
difference between the amount collected by the Direction (19-20): Study the following data
sports club by giving Swimming membership and carefully and answer the questions:
that collected by giving Golf membership?

Click Here For Bundle PDF Course | support@guidely.in Page 6 of 16


SBI Clerk & RRB PO Mains PDF Course 2023
Quantitative Aptitude Day - 32 (Eng)

I: e) Only A and B
II:
III: 20) Find the roots of the equation: px2 + qx + (r –

IV: b 3) = 0, where:

19) If the sum of bigger roots of all the four p = Difference between smaller roots of

equations together is ‘P’, then which of the equations I and III.

following regarding P is TRUE? q = Sum of smaller roots of equations II and IV.

A: ‘P’ is a prime number. r = Smaller root of equation III.

B: ‘P + 2’ is a prime number. a) 4 and 2

C: ‘P – 2’ is a prime number. b) -4 and -2

a) Only A c) -4 and 2

b) Only A and C d) 4 and -2

c) Only C e) None of these

d) Only B and C
Click Here to Get the Detailed Video Solution for the above given Questions
Or Scan the QR Code to Get the Detailed Video Solutions

Answer Key with Explanation

1) Answer: C The new cost of the mixture will also be


Let the initial quantity of water = ‘x’ L 12.5%less than the original cost of the mixture.
So, the initial quantity of milk = (x + 20) L So, the original cost of the mixture = y =
Let the initial cost of the mixture = ₹ ‘y’ per L = ₹ 36 per L
So, From equation (1):

If the cost of pure milk were 12.5% less: 4x + 80 = 6x + 60

Click Here For Bundle PDF Course | support@guidely.in Page 7 of 16


SBI Clerk & RRB PO Mains PDF Course 2023
Quantitative Aptitude Day - 32 (Eng)

x = 10 X = 25000
Initial quantity of water = 10 L Now, the SI received on at (R + 4)% rate
Initial quantity of milk = (10 + 20) = 30 L after 2 years:
Ratio of milk to water in the mixture = 30: 10 = 3:
1
When 8 L quantity of the mixture is replaced with
3) Answer: B
the same quantity of milk:
Let the time, in which D alone can complete the
The new quantity of milk in the mixture =
work is ‘3t’ days.
= 32 L
So, the time, in which A alone can complete the
And the new quantity of water in the mixture = 40
work = = ‘15t’ days
– 32 = 8 L
So, the new cost of the mixture = = ₹ 38.4 The time, in which B alone can complete the
work = = ‘10t’ days
per L
And the time, in which C alone can complete the

2) Answer: E work = = ‘6t’ days

Since, the difference between CI and SI after 2 Since, A, B and C together can complete the

years: work in 9 days.


So,

And the difference between CI and SI after 3


years:
t=3
Now, the time, in which B alone can complete
So,
the work = 10 * 3 = 30 days
The time, in which C alone can complete the
work = 6 * 3 = 18 days
And the time, in which D alone can complete the

R = 4% work = 3 * 3 = 9 days

Since, the difference between compound interest Since, the part of work completed by B, C and D

and simple interest on ₹ (X – 7000) at (R + 1)% together in 1 day:

rate after 2 years is ₹ 45.


So, So, the time, in which B, C and D together can
complete the work = 5 days

Click Here For Bundle PDF Course | support@guidely.in Page 8 of 16


SBI Clerk & RRB PO Mains PDF Course 2023
Quantitative Aptitude Day - 32 (Eng)

4) Answer: A 5) Answer: B
Let the speed of the boat in still water is ‘5x’ m/s Since, the average of present ages of A, B and
and the speed of the stream is ‘y’ m/s. C is 23 years.
Since, the time taken by the boat to go 540 m So,
downstream is 20% more than that taken by the A + B + C = 69 -------------(1)
boat to go 300 m upstream. Since, the ratio of A’s age to B’s age after 6
So, years will be 2: 3.
So,

3A + 18 = 2B + 12
15x – 3y = 10x + 2y
2B – 3A = 6 -------------(2)
x = y ------------(1)
By equation (1) * 3 + equation (2):
If the speed of the boat in still water were 20%
3A + 3B + 3C + 2B – 3A = 207 + 6
more:
5B + 3C = 213 -------------(3)
The new speed of boat in still water = 120% of
Since, C’s present age is 70% of B’s present
5x = ‘6x’ m/s
age.
And
So,
6x + y = 21
From equation (1):
6x + x = 21 From equation (1):

x = 3, y = 3
The speed of the boat in still water = 5 * 3 = 15 B = 30
m/s From equation (2):
The speed of the stream = 3 m/s A = 18
Quantity I: From equation (1):
The downstream distance covered by the boat in C = 21
25 seconds: Since, the difference between B’s present age
(15 + 3) * 25 = 450 m and D’s present age is 10 years.
Quantity II: So, D’s present age will be either 30 + 10 = 40
The upstream distance covered by the boat in 35 years or 30 – 10 = 20 years.
seconds: Quantity I:
(15 – 3) * 35 = 420 m A’s present age = 18 years
Hence Quantity I > Quantity II If D’s present age is 40 years:

Click Here For Bundle PDF Course | support@guidely.in Page 9 of 16


SBI Clerk & RRB PO Mains PDF Course 2023
Quantitative Aptitude Day - 32 (Eng)

So, the value, which will be filled in the blank = The new time, in which the bus will cross the
= 45% bridge = 16 seconds
If D’s present age is 20 years: So, the new speed of the bus = = 15 m/s
So, the value, which will be filled in the blank = Since, the original speed of the bus = 20 m/s
= 90% So, the value which will be filled in the blank =
Quantity II: = 75%
A’s present age = 18 years Quantity II:
B’s present age = 30 years The new time, in which the bike will cross the
Required percentage = = 60% bridge = 20 seconds
Hence, the relation can’t be established. So, the new speed of the bike = = 12 m/s
Since, the original speed of the bike = 16 m/s
6) Answer: D So, the value which will be filled in the blank =
Let the speed of the bike = ‘2x’ m/s = 75%
So, the speed of the car = 150 of ‘2x’ = ‘3x’ m/s Hence, Quantity I = Quantity II
And the speed of the Bus = (3x – 4) m/s
Since, the time taken by the car to cross the Direction (7-10):
bridge is ‘t’ seconds. Let the time, in which pipe M alone can empty
So, tank P is ‘6m’ minutes.
So, the time, in which pipe N alone can empty

Since, the time taken by the bike to cross the tank Q = = ‘9m’ minutes

bridge is (t + 5) seconds. And the time, in which pipe O alone can empty
So, tank R = = ‘8m’ minutes
Since, the time, in which pipes A and M together

From equations (1) and (2): can fill tank P is 30 minutes.


So,

2t + 10 = 3t
Since, the time, in which pipes B and N together
t = 10
can fill tank Q, is 90 minutes.
From equation (1):
So,
x=8
Speed of the bike = 2 * 8 = 16 m/s
Speed of the car = 3 * 8 = 24 m/s From equations (1) and (2):
Speed of the bus = 24 – 4 = 20 m/s
Quantity I:

Click Here For Bundle PDF Course | support@guidely.in Page 10 of 16


SBI Clerk & RRB PO Mains PDF Course 2023
Quantitative Aptitude Day - 32 (Eng)

So, the value of = = 6.8

8) Answer: D
m=5
The time, in which pipe A alone can fill tank P
From equation (1):
with 125% of its original efficiency = =
12 minutes
t = 15
The time, in which pipe M alone can empty tank
P with of its original efficiency = =
36 minutes
Now, the part of tank P filled by pipes A and M
together in 1 minute:

So, the time, in which pipes A and M together


will fill tank P = 18 minutes
7) Answer: B
The time, in which pipe B alone can fill tank Q
Since, the flowing rate of water from pipe M is
with 125% of its original efficiency = =
4.5 L per minute.
24 minutes
And the time, in which pipe M alone can empty
The time, in which pipe N alone can empty tank
tank P = 30 minutes
Q with 93.75% of its original efficiency =
So, the capacity of tank P = 4.5 * 30 = 135 L
= 48 minutes
Since, the time, in which pipe A alone can fill
Now, the part of tank Q filled by pipes B and N
tank P = 15 minutes
together in 1 minute:
So, the flowing rate of water from pipe A = x =
= 9 L per minute
Since, the flowing rate of water from pipe M = So, the time, in which pipes B and N together will

= 5 L per minute fill tank Q = 48 minutes


Required percentage = = 37.5%
And the time, in which pipe N alone can empty
tank Q = 45 minutes
So, the capacity of tank Q = 5 * 45 = 225 L 9) Answer: A

Since, the time, in which pipe B alone can fill Since, the difference between the capacities of

tank Q = 30 minutes tanks P and Q is 30 L.

So, the flowing rate of water from pipe B = y = So, the capacity of tank P = = 210 L

= 7.5 L per minute The capacity of tank Q = = 180 L

Click Here For Bundle PDF Course | support@guidely.in Page 11 of 16


SBI Clerk & RRB PO Mains PDF Course 2023
Quantitative Aptitude Day - 32 (Eng)

And the capacity of tank R = = 240 L


Now, the flowing rate of water from pipe A = Direction (11-14):
= 14 L per minute From the 1st pie chart:
The flowing rate of water from pipe B = =6L X + (Y + 20) = 100 – 30 – 15

per minute X + Y = 35 -------------(1)

And the flowing rate of water from pipe C = = From the 2nd pie chart:

10 L per minute
So, the time, in which pipes A, B and C together X + 4Y = 80 -------------(2)
will fill tank R: From equations (1) and (2):
35 – Y = 80 – 4Y
Y = 15
From equation (1):
10) Answer: C
X = 20
From X:
Average of time, in which pipe A alone, pipe B
alone and pipe C alone can fill tanks P, Q and R
respectively:

So, X is not true.


From Y:
If the capacity of tank R is 210 L:
Let the total number of persons, who visited the
And the time, in which pipe O alone can empty
museum on Mon, Tue, Wed and Thu are 30t,
tank R = 40 minutes
20t, 35t and 15t respectively.
So, the flowing rate of water from pipe O = =
Also, let the number of males, who visited the
5.25 L per minute
museum on Mon, Tue, Wed and Thu are 40m,
So, Y is not true.
10m, 30m and 20m respectively.
From Z:
Since, the number of females, who visited the
If the difference between the flowing rates of
museum on Tue, is 110.
water from pipes B and N is 3 L per minute:
So,
And pipes B and N together can fill tank Q in 90
20t – 10m = 110
minutes.
2t – m = 11 ----------------(3)
So, the capacity of tank Q = 3 * 90 = 270 L
So, Z is not true.
Hence, none is true.

Click Here For Bundle PDF Course | support@guidely.in Page 12 of 16


SBI Clerk & RRB PO Mains PDF Course 2023
Quantitative Aptitude Day - 32 (Eng)

Since, the number of females, who visited the So, the number of male foreigners, who visited
museum on Tue, is 175% more than the the museum on Tue = 26% of 50 = 13
females, who visited the museum on Mon. Total females, who visited the museum on Tue =
So, 110
So, the number of female foreigners, who visited

3t – 4m = 4 ---------------- (4) the museum on Tue = 10% of 110 = 11

From equations (3) and (4): Total number of foreigners, who visited the

t = 8, m = 5 museum on Tue = 13 + 11 = 24
Total number of persons, who visited the
museum on Tue = 160
Required percentage = = 15%

13) Answer: A
Total number of females, who visited the
11) Answer: B museum in all the 4 days together = 40 + 110 +
Total number of persons, who visited the 130 + 20 = 300
museum on Mon = 240 So, M = = 156°
So, the number of adults in the museum on Mon And, N = = 24°
= 45% of 240 = 108 From A:
And the number of children in the museum on Factors of M = 156 = 12 * 13
Mon = 240 – 108 = 132 Factors of N = 24 = 12 * 2
Total number of persons, who visited the Since, both M and N are multiple of 12, but not
museum on Wed = 280 24.
So, the number of adults in the museum on Wed So, A is not true.
= 65% of 280 = 182 From B:
And the number of children in the museum on Difference of M and N = 156 – 24 = 132
Wed = 280 – 182 = 98 Since, 132 is a multiple of 12.
Required difference = (108 + 182) * 50 – (132 + So, B is true.
98) * 20 = ₹ 9900 From C:
Value of = = 1.5
12) Answer: E So, C is not true.
Total males, who visited the museum on Tue = Hence, only B is true.
50

Click Here For Bundle PDF Course | support@guidely.in Page 13 of 16


SBI Clerk & RRB PO Mains PDF Course 2023
Quantitative Aptitude Day - 32 (Eng)

14) Answer: C
The number of male children, who visited the
museum on Mon = 40% of 200 = 80
The number of male children, who visited the
museum on Tue = 20% of 50 = 10
The number of male children, who visited the
museum on Wed = 30% of 150 = 45
The number of male children, who visited the 15) Answer: E
museum on Thu = 45% of 100 = 45 Number of males, who took membership in
Required average = = 45 Tennis only = = 22
Number of females, who took membership in
Direction (15-18): Tennis only = 40 – 22 = 18
Number of persons, who took membership in all Number of males, who took membership in
the three sports = 5 Swimming only = = 20
Number of persons, who took membership in
Number of females, who took membership in
Tennis only = = 40
Swimming only = 32 – 20 = 12
Number of persons, who took membership in Number of males, who took membership in Golf
Swimming only = 80% of 40 = 32 only = = 10
Number of persons, who took membership in
Number of females, who took membership in
Golf only = 32 – 2 = 30
Golf only = 30 – 10 = 20
Number of persons, who took membership in
Required ratio = (22 + 20 + 10): (18 + 12 + 20) =
both Swimming and Golf, but not Tennis = 60%
52: 50 = 26: 25
of 30 = 18
Number of persons, who took membership in
16) Answer: B
both Tennis and Golf, but not Swimming = 18 – 3
Total number of persons, who took Swimming
= 15
membership:
Number of persons, who took membership in
32 + 10 + 18 + 5 = 65
both Tennis and Swimming, but not Golf =
So, the total amount collected by the sports club
= 10
by giving Swimming membership = 65 * 150 =
₹ 9750
Total number of persons, who took Golf
membership:
30 + 15 + 18 + 5 = 68

Click Here For Bundle PDF Course | support@guidely.in Page 14 of 16


SBI Clerk & RRB PO Mains PDF Course 2023
Quantitative Aptitude Day - 32 (Eng)

So, the total amount collected by the sports club So, the number of females, who took
by giving Golf membership = 68 * 180 = ₹ 12240 membership in only one sport = 102 – 60 = 42
Required difference = 12240 – 9750 = ₹ 2490 Required percentage = = 63.63%

17) Answer: D Direction (19-20):


Total number of persons in the sports club: I:
40 + 32 + 30 + 10 + 18 + 15 + 5 = 150
Total number of persons, who took membership
a2 + 5a – 66=0
of only one sport:
a2 + 11a – 6a – 66 0
40 + 32 + 30 = 102
a (a + 11) – 6 (a + 11) = 0
So, M = = 68%
(a + 11) (a – 6) = 0
Total number of persons, who took membership
a = -11 and 6
of only two sports:
II:
10 + 18 + 15 = 43
So, N =
So, the value of (2M – 3N) = (136 – 86) = 50 2b2 – 3b – 9 = 0
[Which is divisible by both 10 and 5 but not by 2b2 – 6b + 3b – 9 = 0
15]. 2b (b – 3) + 3 (b – 3) = 0
(b – 3) (2b + 3) = 0
18) Answer: A b = 3 and -3/2
Total number of members in the sports club: III:
40 + 32 + 30 + 10 + 18 + 15 + 5 = 150
So, the total number of males in the sports club 3c2 – 4c = 221 + 2c2
= 56% of 150 = 84 c2 – 4c – 221 = 0
And the total number of females in the sports c2 – 17c + 13c – 221 = 0
club = 150 – 84 = 66 c (c – 17) + 13 (c – 17) = 0
Total number of males, who took membership in (c – 17) (c + 13) = 0
at-least two sports = of 84 = 24 c = 17 and -13
So, the number of males, who took membership IV:
in only one sport = 84 – 24 = 60 3d2 – d2 = d + 15
Total number of persons, who took membership 2d2 – d – 15 = 0
in only one sport = 40 + 32 + 30 = 102 2d2 – 6d + 5d – 15 = 0
2d (d – 3) + 5 (d – 3) = 0

Click Here For Bundle PDF Course | support@guidely.in Page 15 of 16


SBI Clerk & RRB PO Mains PDF Course 2023
Quantitative Aptitude Day - 32 (Eng)

(d – 3) (2d + 5) = 0 p = Difference between smaller roots of


d = 3 and -5/2 equations I and III
p = -11 – (-13)
p=2
q = Sum of smaller roots of equations II and IV
q = (-3/2) + (-5/2) = (-3 – 5)/2 =
q = -4
r = Smaller root of equation III
r = -13
Now equation:
19) Answer: E
px2 + qx + (r – 3) = 0
Bigger root of equation I = 6
2x2 + (-4)x + (-13 – 3) = 0
Bigger root of equation II = 3
2x2– 4x – 16 = 0
Bigger root of equation III = 17
x2– 2x – 8 = 0
Bigger root of equation IV = 3
x2– 4x + 2x – 8 = 0
Required sum = P = 6 + 3 + 17 + 3 = 29
x (x – 4) + 2 (x – 4) = 0
P + 2 = 31 and P – 2 = 27
(x – 4) (x + 2) = 0
We can clearly see that, P and ‘P + 2’ are prime
x = 4 and -2
numbers while ‘P – 2’ is not a prime number.

20) Answer: D

Click Here For Bundle PDF Course | support@guidely.in Page 16 of 16


SBI Clerk & RRB PO Mains PDF Course 2023
ENGLISH Day - 32

English Language

Directions (1-7): Read the given passage ways individuals can make a difference is by
carefully and answer the following questions reducing their carbon emissions. This can be
based on the passage. Some words are achieved through small steps such as using
highlighted to help you locate while answering energy-efficient appliances, insulating homes
the questions. properly, and reducing energy consumption.
In an era where climate change has become one Another impactful action is choosing sustainable
of the most pressing global challenges, it’s easy transportation options. By opting for public
to feel overwhelmed and disempowered. transport, cycling, or carpooling, individuals can
However, it’s crucial to remember that even the reduce their carbon footprint significantly.
smallest actions can have a significant impact on Furthermore, supporting renewable energy
mitigating climate change. The power of small sources and advocating for clean energy policies
steps lies in their ability to create a ripple effect, can help shift our societies towards a sustainable
inspiring others to follow suit and collectively energy future.
create a meaningful change. Whether it’s Transportation is a major contributor to
reducing our carbon footprint, advocating for greenhouse gas emissions. Opting for public
renewable energy, or making conscious choices transportation whenever possible not only
in our daily lives, every individual has the power reduces emissions but also alleviates traffic
to contribute to the larger goal of protecting our congestion. Additionally, individuals can consider
planet. Together, let’s discover the immense alternatives such as cycling or walking for shorter
power we hold as individuals and harness it to distances. These small changes in transportation
create a better and more sustainable future for habits can have a cumulative positive impact on
generations to come. Climate change is a global the environment. Another way to make a
issue that requires collective action, but difference is by carpooling or ridesharing.
individual actions can play a crucial role in driving Sharing rides with others going in the same
change. Each person’s choices and behaviours, direction not only reduces emissions but also
when multiplied across millions of individuals, saves money and fosters a sense of community.
can have a significant impact on reducing For those who own cars, choosing electric or
greenhouse gas emissions and mitigating climate hybrid vehicles can reduce carbon emissions
change. By understanding the power of our associated with transportation. Conserving
individual actions, we can take responsibility for energy is another effective way individuals can
our carbon footprint and contribute to a more mitigate climate change. Simple steps such as
sustainable future. One of the most effective turning off lights when not in use and using

Click Here For Bundle PDF Course | support@guidely.in Page 1 of 12


SBI Clerk & RRB PO Mains PDF Course 2023
ENGLISH Day - 32

energy-efficient appliances can help. Individuals one of the most effective ways to lower carbon
can also consider using natural light whenever emissions associated with food production.
possible and adjusting thermostats to conserve Plant-based diets require fewer resources and
energy. At the workplace, encouraging energy- produce fewer greenhouse gas emissions.
saving practices such as turning off computers However, even small changes, such as
and other devices overnight can make a incorporating more plant-based meals into one’s
substantial difference. Renewable energy diet or choosing locally sourced, organic
sources, such as solar or wind power, are also a produce, can contribute to a more sustainable
viable option for individuals looking to reduce food system. Waste production and improper
their carbon footprint. Installing solar panels on disposal contribute to greenhouse gas emissions
rooftops or supporting community solar initiatives and environmental degradation. By reducing
can help shift towards a more sustainable energy waste and promoting recycling, individuals can
future. These initiatives allow individuals to minimise their environmental impact. Reducing
collectively pool resources and invest in waste can be achieved through small steps such
renewable energy projects in their communities. as avoiding single-use plastics, composting
Additionally, individuals can advocate for policies organic waste, and repurposing or repairing
that promote renewable energy and support items instead of discarding them. Additionally,
organisations working towards clean energy individuals can support businesses and
solutions. One way to support renewable energy organisations that prioritise sustainability and
is by choosing a utility provider that offers offer recycling programs. By properly sorting and
renewable energy options. Many companies now recycling waste, individuals can reduce the
provide the choice of purchasing renewable amount of waste sent to landfills. Promoting
energy credits, which ensure that a portion of the recycling and waste reduction within
electricity consumed comes from renewable communities and workplaces is another way
sources. By making this choice, individuals can individuals can make a difference. By organising
directly support the growth and development of recycling initiatives, educating others about the
renewable energy infrastructure. importance of waste reduction, and supporting
The food we consume also has an environmental local recycling programs, individuals can create a
impact. Animal agriculture, particularly the culture of sustainability. Advocacy and
production of meat and dairy, contributes community involvement are powerful tools for
significantly to greenhouse gas emissions. By driving change. By raising awareness, engaging
making conscious choices in our diet, individuals in dialogue, and advocating for sustainable
can reduce their carbon footprint. Reducing meat practices, individuals can inspire others to take
consumption or adopting a plant-based diet is action. One way to advocate for change is by

Click Here For Bundle PDF Course | support@guidely.in Page 2 of 12


SBI Clerk & RRB PO Mains PDF Course 2023
ENGLISH Day - 32

contacting elected officials and voicing concerns 2) Which of the following statements is false
about climate change and the importance of according to the given passage ?
sustainability. By participating in public hearings, a) Supporting renewable energy sources and
signing petitions, and supporting organisations clean energy policies can help shift societies
that work towards climate change mitigation, towards a sustainable energy future.
individuals can influence policy decisions and b) Individuals can influence policy decisions and
promote sustainable practices at a larger scale. promote sustainable practices at a larger scale
By engaging with local sustainability initiatives, by participating in public hearings, signing
participating in community gardens, or supporting petitions, and supporting organisations working
local environmental organisations, individuals towards climate change mitigation
can contribute to a sense of collective c) Advocacy, community involvement, and
responsibility and foster sustainable practices engaging in dialogue can inspire others and drive
within their communities. In conclusion, the sustainable change
power of small steps should not be d) Reducing meat consumption or adopting a
underestimated when it comes to climate change plant-based diet is one of the most effective ways
mitigation. to lower carbon emissions
1) According to the given passage, how can e) All are true
individuals conserve energy at home and in the
workplace? 3) What are the benefits of supporting renewable
I. By turning off lights when not in use, using energy sources as per the given passage?
energy-efficient appliances and using natural I. Decreases the reliance on fossil fuels and
light whenever possible. promotes clean energy.
II. Not adjusting thermostat settings when leaving II. Contributes to the growth and development of
the work premises. renewable energy infrastructure.
III. Encouraging energy-saving practices at home III. Renewable energy sources have very low
and workplace and supporting renewable energy upfront costs.
sources. a) Only I
a) Only I b) Only II
b) Only II c) Only III
c) Both I and II d) Both I and II
d) Both I and III e) All I, II and III
e) All are correct
4) The word ‘mitigating’ is used in the passage,
below are three sentences which have used the

Click Here For Bundle PDF Course | support@guidely.in Page 3 of 12


SBI Clerk & RRB PO Mains PDF Course 2023
ENGLISH Day - 32

word MITIGATE, you are required to choose d) II, III and IV


which of the following sentence(s) has/have used e) All I, II, III and IV
it properly.
I. The ministry implemented various measures to 6) What is the tone of the given passage?
mitigate the impact of the economic crisis on a) Laudatory
small businesses. b) Informative
II. The company implemented some mitigate c) Nostalgic
strategies to reduce the risk of data breaches. d) Sarcastic
III. The development project included a e) Ethical
comprehensive plan for environmental mitigation
to minimise the impact on local wildlife habitats. 7) Choose a suitable title for the above passage
a) Only I that represents the whole ideology.
b) Only III a) Conserving energy for a better world
c) Both I and III b) Renewable resources and its benefits
d) Both II and III c) Empowering individuals and taking action for a
e) All I, II and III sustainable future
d) Clean and green energy
5) How do you think the given passage highlights e) Challenges in reducing carbon emissions
the importance of individual actions in alleviating
climate change? Directions (8-12): In each of the questions given
I. The passage suggests various actions that below a phrase in the sentence has been
individuals can perform to mitigate climate highlighted. It may or may not need replacement.
changes and other concerns. Read the question carefully and choose an
II. It stresses that every individual has the power option that would best replace the highlighted
to contribute to the larger goal of protecting the part to make the sentence grammatically and
planet. contextually correct. In case, the given sentence
III. The passage intensifies new development is correct in its current form and there is no
startups by individuals. improvement needed, mark (e) as your answer.
IV. The passage suggests that individual actions 8) Without knowing complete details of the
can create a ripple effect, inspiring others to project, it is most difficult to making informed
follow the same. decisions or provide accurate recommendations.
a) Both I and II a) it is difficult to make informed decisions or
b) Both II and IV providing accurate
c) I, II and IV

Click Here For Bundle PDF Course | support@guidely.in Page 4 of 12


SBI Clerk & RRB PO Mains PDF Course 2023
ENGLISH Day - 32

b) it is many difficult to make informed decisions 11) The children at the park were gleefully
or provide accurate running and laughing, their carefree spirits filling
c) it is difficult to make informed decisions or the air with joy as they played on the swings and
provide accurate raced each other across the grassy field.
d) it is difficulty to make informed decisions or a) their carefree spirits fill the air with joy as they
providing accurate play off
e) No changes required b) their carefree spirit filled the air to joy as they
played
9) It is hilarious to see the cat attempted a daring c) their carefree spirit fills the air with joy of
leap and end up stuck mid-air, displaying a playing
priceless expression of surprise. d) their carefree spirits filling the air with joyful
a) It is hilarious to seeing the cat attempting a play
daring leap e) No changes required
b) It is hilarious to seeing the cat attempt a daring
leap 12) The exam been rescheduled for next week,
c) It is hilarious of seeing the cat attempt the giving students additional time to prepare and
daring leap ensuring a fair and equitable testing environment
d) It is hilarious to see the cat attempt a daring for all.
leap a) The exam is rescheduled next week, giving
e) No changes required students
b) The exam has been rescheduled to next
10) The team has cancelled their high week, giving students
anticipation trip to the championship due to c) The exam had rescheduled for the next week,
unforeseen circumstances, leaving everyone giving students
disappointed and disheartened. d) The exam is rescheduling to next week to give
a) The team had cancelled their highly students
anticipated trip e) No changes required
b) The team has cancelled their high anticipated
trip Directions (13-17): In each of the following
c) The team was cancelled there high anticipate questions, a highlighted word is given followed
trip by three sentences. Choose the sentence(s) that
d) The team has cancelled their trip which were has/have the wrong/inappropriate usage of the
highly anticipate highlighted word.
e) No changes required 13) ENDURE

Click Here For Bundle PDF Course | support@guidely.in Page 5 of 12


SBI Clerk & RRB PO Mains PDF Course 2023
ENGLISH Day - 32

I. In many ways, the event was about the value pressed suit, projecting a polished image to the
of design and the ability of Indian architects to public.
build endure structures on time. II. Grooming private forests might look good in
II. Despite facing numerous challenges and theory but expecting them to be a permanent
setbacks, the dedicated athlete persevered and carbon stock is wishful thinking.
demonstrated incredible endurance throughout III. The groom looked dashing in his tailored suit
the gruelling race. as he eagerly awaited his bride at the altar.
III. They'd never allow their children to have the a) Only I
kind of life or experiences they had to endure. b) Only II
a) Only II c) Both I and II
b) Only III d) Both I and III
c) Only I e) All correct
d) Both I and III
e) All correct 16) CONSORT
I. The violinist played in perfect consort with the
14) STRIKE rest of the orchestra.
I. Despite his best efforts, the batter couldn't II. A healthy thought of self consorts well with a
connect with the ball and ended up striking out desire and an endeavour to help others.
three times in a row. III. During the last gig the musicians consorting
II. Malla Srinivasa Rao defended the strike with one another to produce a harmonious
saying that it would be observed till the last week symphony.
of July to stabilise the prices in the market. a) Only I
III. During the intense carrom match, she b) Only II
executed a perfect carrom strike, effortlessly c) Only III
pocketing multiple coins in a single shot. d) Both II and III
a) Only I e) All correct
b) Only III
c) Both I and II 17) DIGNIFY
d) Both II and III I. Yesterday the speaker’s thoughtful words and
e) All correct ideas dignifies the occasion.
II. The painter uses vibrant colours and intricate
15) GROOM details which dignifies the whole art.
I. The politician always ensured he was well-
groom, with perfectly styled hair and a neatly

Click Here For Bundle PDF Course | support@guidely.in Page 6 of 12


SBI Clerk & RRB PO Mains PDF Course 2023
ENGLISH Day - 32

III. The team lead always tries to dignifying the b) BC


work of her team members by encouraging and c) BD
acknowledging their efforts and ideas. d) CD
a) Both I and II e) E
b) Both I and III
c) Both II and III 20) Last week, more than 500 scientists and
d) All correct academics/A wrote to the Indian institute of
e) All wrong Science (IISc) criticising its/B administration for
prohibiting a discussion/C on the Unlawful
Directions (18-22): Read each question carefully Activities Prevention Act/D. All correct/E
to find out which parts have errors and parts that a) AC
are error free. Choose the option which has the b) BC
correct/error free parts as your answer. Choose c) BD
option (e) as your answer if all the four parts are d) CD
correct. e) E
18) The Supreme Court’s verdict upholding
statutory amendments/A made in 2021 allowed 21) The Reserve Bank of India and the Central
multiple extensions of service/B to heads of Bank of the UAE/A agrees on July 15 to establish
investigative agencies was a setback/C to the a framework/B for enable to the use of the two/C
cause of protecting their institutional countries’ local currencies for cross-border
independence/D. All correct/E transactions/D. All correct/E
a) AB a) AD
b) BC b) BC
c) AD c) AC
d) CD d) BD
e) E e) E

19) With the exception from three vegetables/A, 22) The existed Forest Conservation Act, 1980/A
including lady’s finger and lemon, in the 19- in the Environment Ministry’s reckoning were
member basket/B, all the others including the insufficient/B for these ends, as it did not
mostly wide used/C potatoes and onions incentivise/C private agro-forestry and tree
registered sharp sequential inflation/D. All plantation activities/D. All correct/E
correct/E a) AB
a) AC b) BC

Click Here For Bundle PDF Course | support@guidely.in Page 7 of 12


SBI Clerk & RRB PO Mains PDF Course 2023
ENGLISH Day - 32

c) CD e) E
d) BD
Click Here to Get the Detailed Video Solution for the above given Questions
Or Scan the QR Code to Get the Detailed Video Solutions

Answer Key with Explanation

1) Answer: D Statement III is not mentioned in the passage


Only statements I and III are the ways for and is wrong too because renewable energy is
individuals to conserve energy at home and not cost efficient and the initial amount/cost to
workplace. use renewable energy is high.
Statement II is wrong because people must
adjust thermostat settings before leaving work - 4) Answer: C
this is a way of conserving energy as mentioned Both the statements I and III have used the word
in the passage. ‘mitigate’ appropriately.
Sentence II alone is wrong/incorrect because
2) Answer: E ‘mitigate’ is a wrong usage, it must be replaced
All the given statements are true according to with mitigating to make the sentence
the information given in the passage. grammatically correct and meaningful.
Hence option (e) All are true is the correct
answer for this question. 5) Answer: C
Statements I, II and IV are derived from the
3) Answer: D given passage and hence are true whereas
Only statements I and II are the benefits of using statement III is completely irrelevant and no such
renewable energy sources as per the given information is given in the above passage.
information in the above passage. Hence we go for option (c) as our answer to this
question.

Click Here For Bundle PDF Course | support@guidely.in Page 8 of 12


SBI Clerk & RRB PO Mains PDF Course 2023
ENGLISH Day - 32

Option d - it is difficulty is wrong(difficulty - noun,


6) Answer: B difficult - adjective)
The tone of the passage is possibly very Without knowing complete details of the project,
informative as it contains more information on it is difficult to make informed decisions or
the power the society holds to make a difference provide accurate recommendations.
in addressing climate change.
Laudatory - applauding 9) Answer: D
Nostalgic - speaking about the past The highlighted phrase definitely needs
Sarcastic - harsh and cutting remarks or replacement because ‘attempted’ is a past tense
criticisms word and it does not fit the sentence.
Ethical - dealing with principles of morality and Option a - seeing the cat attempting is wrong
honesty Option b - to seeing is wrong instead it must be
7) Answer: C to see
The title - Empowering individuals and taking Option c - of is a wrong preposition here
actions for a sustainable future is more relevant Option d has the right phrase that can be used to
to the whole context of the given passage. replace the highlighted phrase in the given
The other options are also related to the idea of sentence.
the passage but are specific to a part of the It is hilarious to see the cat attempt a daring leap
passage given whereas option c represents the and end up stuck mid-air, displaying a priceless
whole concept and is more inclusive. expression of surprise.

8) Answer: C 10) Answer: A


The correct phrase is “it is difficult to make In the highlighted phrase “high anticipation” is
informed decisions or provide accurate”, replace wrong grammatically and affects the meaning of
the highlighted phrase with phrase in option c to the sentence and hence it needs replacement.
make the sentence correct Option b - high anticipated is wrong(must be
To making is wrong in the given sentence - highly)
hence replacement required Option c - was, there, high, anticipate - all four
Option a - providing is wrong instead it must be words are wrong
provide Option d - were and anticipate is wrong
Option b - many difficult is wrong(many Option a has the correct replacement phrase.
represent the number, whereas more and most The team had cancelled their highly anticipated
represents degree or amount) trip to the championship due to unforeseen

Click Here For Bundle PDF Course | support@guidely.in Page 9 of 12


SBI Clerk & RRB PO Mains PDF Course 2023
ENGLISH Day - 32

circumstances, leaving everyone disappointed In many ways, the event was about the value of
and disheartened. design and the ability of Indian architects to build
enduring structures on time.
11) Answer: E
The given sentence is both grammatically and 14) Answer: E
contextually correct. Hence, no All the given three statements I, II and III are
replacement/change required. correct and have used the word ‘strike’
appropriately.
12) Answer: B In statement I, strike out (phrasal verb) - be
The highlighted phrase is wrong and needs dismissed by means of three strikes
replacement. In statement II, strike (noun) is referred to as a
Option a - is rescheduled next week - is period of time when people refuse to go to work
incomplete and wrong and protest
Option c - had and for is wrong (rescheduled In statement III, carrom strike where strike is
to(time), rescheduled for(reason) referred to hit somebody/something
Option d - is rescheduling is completely wrong
and hence this phrase is inappropriate 15) Answer: A
Option b has the correct phrase that can replace Well-groom is wrong instead it must be well-
the highlighted part of the sentence. groomed.
The exam has been rescheduled to next week, Well-groomed - clean, tidy, and smart.
giving students additional time to prepare and The politician always ensured he was well-
ensuring a fair and equitable testing environment groomed, with perfectly styled hair and a neatly
for all. pressed suit, projecting a polished image to the
public.
13) Answer: C
Statement I has inappropriate usage of the word 16) Answer: C
‘endure’. Statement III has used the word in an
Replace ‘endure(verb)’ with ‘enduring(adjective)’ inappropriate way.
to make the statement I correct. Statements I and II are correct and meaningful.
In statement II, the noun endurance is used. In statement III, replace ‘consorting’ with
In statement III, the verb endure is used. ‘consorted’ as the sentence is in past tense.
Endure - to continue/to suffer something painful Consort - associate with someone or something
or uncomfortable,

Click Here For Bundle PDF Course | support@guidely.in Page 10 of 12


SBI Clerk & RRB PO Mains PDF Course 2023
ENGLISH Day - 32

During the last gig the musicians consorted with agencies is a setback to the cause of protecting
one another to produce a harmonious their institutional independence.
symphony.
19) Answer: C
17) Answer: B The erroneous parts are A and C and the correct
Both the statements I and III have used the parts are B and D.
given word inappropriately and hence wrong. A - Change ‘from’ to ‘of’ , exception of is the
Statement I - replace ‘diginifies’ to ‘dignified’ to correct easy to use.
make the sentence correct and meaningful. C - It should be ‘most widely used’ instead of
Statement III - replace ‘dignifying’ to ‘dignify’ to ‘mostly wide used’.
make the sentence correct and meaningful The corrected sentence:
Statement II is correct and meaningful and has With the exception of three vegetables, including
also used the highlighted word in an appropriate lady’s finger and lemon, in the 19-member
way. basket, all the others including the most widely
Yesterday the speaker’s thoughtful words and used potatoes and onions registered sharp
ideas dignified the occasion. sequential inflation.
The team lead always tries to dignify the work of
her team members by encouraging and 20) Answer: E
acknowledging their efforts and ideas. The given is correct and has no errors. Hence
option(e) is the correct answer for this question.
18) Answer: C
The parts A and D are correct and contain no 21) Answer: A
error. B and C are erroneous and need The parts A and D are correct and error free.
correction to make the sentence grammatically And parts B and C are erroneous.
and contextually correct and meaningful. B - replace ‘agrees’ with ‘agree’ to make the
In part b - allowed is wrong instead in must be sentence correct
either ‘allowing’ or ‘to allow’ C - enable to is incorrect, instead it must be
In part c - replace ‘was’ with ‘is’ because the enabling.
sentence is in present form. The corrected sentence :
The corrected sentence : The Reserve Bank of India and the Central Bank
The Supreme Court’s verdict upholding statutory of the UAE agreed on July 15 to establish a
amendments made in 2021 to allow multiple framework for enabling the use of the two
extensions of service to heads of investigative

Click Here For Bundle PDF Course | support@guidely.in Page 11 of 12


SBI Clerk & RRB PO Mains PDF Course 2023
ENGLISH Day - 32

countries’ local currencies for cross-border B - replace ‘were’ with ‘was’ as the sentence
transactions. talks about a single act.
The corrected sentence:
22) Answer: C The existing Forest Conservation Act, 1980 in
Parts C and D are correct and have no errors the Environment Ministry’s reckoning was
whereas parts A and B are incorrect. insufficient for these ends, as it did not
A - replace ‘existed’ with ‘existing’ as it is still in incentivise private agro-forestry and tree
place plantation activities.

Click Here For Bundle PDF Course | support@guidely.in Page 12 of 12


SBI Clerk & RRB PO Mains PDF Course 2023
Reasoning Ability Day - 33 (Eng)

Reasoning Ability
Directions (1-5): Study the following information lowermost nor on the topmost floor. Neither W
carefully and answer the questions given below. nor T stands on the adjacent floor of Z whereas
Ten persons viz. Q, R, S, T, U, V, W, X, Y and Z R stands on the floor immediately above S. W
are standing on two different staircases viz. stands below the one who has Fossil but above
Staircase 1 and Staircase 2 of a five storey the one who has Aquatan. The one who has
building such that the lowermost floor is Peperone neither stands above the one who has
numbered as 1 and the floor immediately above Aquatan nor stands on staircase 2.
it is numbered as 2 and so on. Staircase 1 is to 1. X has which of the following bag?
the west of Staircase 2. Each person has a) Aquatan
different brand bags viz. Aquatan, Baggit, b) Kipling
Caprese, Esbeda, Fossil, Hidesign, Kipling, c) Esbeda
Lavie, Peperone and Viari. All the information is d) Peperone
not necessarily in the same order. e) None of these
The one who has Esbeda stands on staircase 2
of an odd numbered floor and stands on an 2. On which of the following floor and staircase
adjacent floor of Q, where both do not stand on does T stand?
the same staircase. Equal number of persons a) Floor2, staircase 2
stand above and below U. V stands two floors b) Floor 1, staircase 2
above U in different staircases. Neither U nor V c) Floor 2, staircase 1
has Esbeda whereas the one who has Hidesign d) Floor 1, staircase 1
stands three floors below V on the same e) None of these
staircase. The number of floors below the one
who has Hidesign is one less than the number of 3. How many floors are there above the one who
floors above the one who has Viari on different has Fossil?
staircases. X stands on staircase 1 of the a) One
lowermost floor and three floors below the one b) Two
who has Lavie but not on the same staircase. c) Three
The one who has Caprese stands on one of the d) Four
floors above the one who has Lavie but not on e) None
the same staircase. Only one floor is between
the one who has Caprese and Y, who stands on 4. Who among the following person has Lavie?
the floor immediately above the one who has a) The one who stands on staircase 1 of floor 4
Kipling. Z has Baggit but neither stands on the b) R

Click Here For Bundle PDF Course | support@guidely.in Page 1 of 10


SBI Clerk & RRB PO Mains PDF Course 2023
Reasoning Ability Day - 33 (Eng)

c) The one who stands on staircase 2 of floor 4 joins in the month having 31 days. W is a male.
d) T R has the son-in-law. Only one person joins
e) None of these between X and Y, who is the husband of V but
not the brother of W. Z neither has any
5. If Z is related to Kipling and Y is related to grandsons nor any siblings whereas W has only
Aquatan in a certain way, then who among the one sibling, who is a female. Y and S join in the
following person is related to Viari? same month but not in the same month as W.
a) R 6. Who among the following person joins in June
b) V 2019?
c) The one who has Fossil a) The one who is the sister of W
d) The one who has Baggit b) X
e) None of these c) The one who is the daughter of Y
d) V
Directions (6-10): Study the following information
e) None of these
carefully and answer the questions given below.
Nine persons R, S, T, U, V, W, X, Y and Z from
7. How is R related to T?
the same family with three generations are
a) Sister
joining for a yoga class in June, August and
b) Brother
November of 2019, 2020 and 2021. No single
c) Father
parent has a child and the number of males in
d) Mother
the family is more than the number of females.
e) Cannot be determined
Only two married couples are there in the family.
T is the son of Z, who joins the class in the leap
8. Who among the following person is the
year and immediately after T. The only daughter
brother-in-law of W?
of Z joins in the same month as Z but neither the
a) The one who joins in November 2020
first nor the last. The number of persons joining
b) W
before the daughter of Z is one more than the
c) The one who joins in August 2021
number of persons joining after the mother of V,
d) U
who doesn’t join before T. V who is the sibling of
e) None of these
T. The brother of Y neither joins in the same
month and year as T nor in the same year as the
9. In which of the following month and year does
daughter of Z. V is the sister-in-law of U, who is a
the father of X join the class?
male and joins in the same year as R. The sister
a) June, 2021
of S, who is neither married nor the child of Z,
b) June, 2020

Click Here For Bundle PDF Course | support@guidely.in Page 2 of 10


SBI Clerk & RRB PO Mains PDF Course 2023
Reasoning Ability Day - 33 (Eng)

c) November, 2020 12. What is the shortest distance between T’s


d) August, 2019 sister and L?
e) None of these a) 12m
b) 8√2m
10. How many persons join between the brother c) 10m
of Y and the son of R? d) 8m
a) One e) None of these
b) Two
c) Three 13. How L is related to T?
d) More than three a) Mother
e) None b) Maternal Aunty
c) Grandfather
Directions (11-15): Study the following d) Uncle
information carefully and answer the below e) None of these
questions.
Seven persons from the same family with three 14. If P is married to M, then how T is related to
generations are sitting on the ground at some P?
distance. Two married couples are in the family. a) Mother-in-law
T is 16m west of G’s only daughter. S is the only b) Brother
son of M’s brother and sits 12m north of T. W is c) Sister-in-law
the only daughter-in-law of G. Z is the daughter d) Brother-in-law
of T, who is the son of L. G’s wife sits 12m north e) Sister
of W. Z sits 6m north of M. W is 20m east of M’s
father. The gender of M and Z is same. W’s 15. Which of the following statement is not true?
daughter sits 8m west of L. a) S sits northwest of G’s wife
11. What is the position of M with respect to W’s b) T is the mother of Z
father-in-law? c) Z sits northeast of T
a) Southwest d) M is the aunty of S
b) Northeast e) None of these

c) North
Directions (16-20): In the following questions, the
d) Southeast
symbol $,@,%,# and & are used with the
e) None of these
following meanings as illustrated below:

Click Here For Bundle PDF Course | support@guidely.in Page 3 of 10


SBI Clerk & RRB PO Mains PDF Course 2023
Reasoning Ability Day - 33 (Eng)

A@B means A is neither greater than nor equal e) D@F


to B
A&B means A is not smaller than B. 18. Statements:
A$B means A is neither smaller than nor equal to U@V#W&X%Y; J&W@G#H%I; N%M#Y$K@L
B. Conclusions:
A#B means A is not greater than B a) V&M
A%B means A is neither greater than nor smaller b) J#K
than B c) G$U
In each of the following questions, the d) L$W
relationship between different elements is shown e) X%L
in the statements followed by some conclusions.
Find a conclusion which logically follows. 19. Statements:
16. Statements: Q&I@N%O&P; I$J%K&L#M; S%R@L$T#U
T@S&Q%K@V; J#K$L%M&R; H@G&L&D%E Conclusions:
Conclusions: a) M#U
a) K#H b) O@J
b) S$J c) K$N
c) Q#G d) K@Q
d) R@V e) N@T
e) S#M
20. Statements:
17. Statements: A&B%C#D@E; H&B$F%G#I; M@L%K&C#J
I#H%B@F&G; A#B$C%D#E; M$E#J%K&L Conclusions:
Conclusions: a) A#K
a) J&B b) J$E
b) I@D c) L&G
c) D&K d) I@A
d) B&M e) A$G

Click Here For Bundle PDF Course | support@guidely.in Page 4 of 10


SBI Clerk & RRB PO Mains PDF Course 2023
Reasoning Ability Day - 33 (Eng)

Click Here to Get the Detailed Video Solution for the above given Questions
Or Scan the QR Code to Get the Detailed Video Solutions

Answer Key with Explanation

Directions (1-5):  Neither U nor V has Esbeda whereas the


1. Answer: D one who has Hidesign stands three floors
2. Answer: B below V on the same staircase.
3. Answer: E
4. Answer: C
5. Answer: A

 The one who has Esbeda stands on


staircase 2 of an odd numbered floor and
is an adjacent floor of Q, where both do
not stand on the same staircase.
 Equal number of persons stand above
and below U.
 V stands two floors above U in different
staircases.

Click Here For Bundle PDF Course | support@guidely.in Page 5 of 10


SBI Clerk & RRB PO Mains PDF Course 2023
Reasoning Ability Day - 33 (Eng)

 The number of floors below the one who  Only one floor is between the one who
has Hidesign is one less than the number has Caprese and Y, who stands on the
of floors above the one who has Viari on floor immediately above the one who has
different staircases. Kipling.
 X stands on staircase 1 of the lowermost  Z has Baggit but neither stands on the
floor and three floors below the one who lowermost nor on the topmost floor.
has Lavie but not on the same staircase.  Neither W nor T stands on the adjacent
 The one who has Caprese stands on one floor of Z whereas R stands on the floor
of the floors above the one who has Lavie immediately above S.
but not on the same staircase. Hence, cases 1 and 2 get eliminated.

Click Here For Bundle PDF Course | support@guidely.in Page 6 of 10


SBI Clerk & RRB PO Mains PDF Course 2023
Reasoning Ability Day - 33 (Eng)

8. Answer: A
9. Answer: B
10. Answer: A

 W stands below the one who has Fossil


but above the one who has Aquatan.
 The one who has Peperone neither
stands above the one who has Aquatan
nor stands on staircase 2.
Hence, cases 3 and 3a get eliminated.

 T is the son of Z, who joins the class in


the leap year and immediately after T.
 The only daughter of Z joins in the same
month as Z but neither the first nor the
last.
 The number of persons joining before the
daughter of Z is one more than the
number of persons joining after the
mother of V, who doesn’t join before T.
 V who is the sibling of T.
 The brother of Y neither joins in the same
month and year as T nor in the same year
as the daughter of Z.

Directions (6-10):
6. Answer: C
7. Answer: D

Click Here For Bundle PDF Course | support@guidely.in Page 7 of 10


SBI Clerk & RRB PO Mains PDF Course 2023
Reasoning Ability Day - 33 (Eng)

 V is the sister-in-law of U, who is a male


and joins in the same year as R.
 The sister of S, who is neither married nor
the child of Z, joins in the month having
31 days.
 W is a male.R has the son-in-law.
 Only one person joins between X and Y,
who is the husband of V but not the
brother of W. Directions (11-15):
 Z neither has any grandsons nor any 11. Answer: B
siblings whereas W has only one sibling, 12. Answer: C
who is a female. 13. Answer: A
 Y and S join in the same month but not in 14. Answer: D
the same month as W 15. Answer: B
 No single parent has a child and the We have:
number of males in the family is more  S is the only son of M’s brother and sits
than the number of females. Only two 12m north of T.
married couples are there in the family.  T is 16m west of G’s only daughter.
 W is the only daughter-in-law of G.
 G’s wife sits 12m north of W.
Based on the above given information we have:

Click Here For Bundle PDF Course | support@guidely.in Page 8 of 10


SBI Clerk & RRB PO Mains PDF Course 2023
Reasoning Ability Day - 33 (Eng)

For Direction:

16. Answer: D
a) K≤H -> K>L≤G>H ->is False
b) S>J -> J≤K=Q≤S ->is False
c) Q≤G ->G≥L<K=Q ->is False
Again, we have:
d) R<V->V>K>L=M≥R ->is True
 Z is the daughter of T, who is the son of L.
 Z sits 6m north of M. e) S≤M -> S≥Q=K>L=M ->is False

Since, only three generations are in the


17. Answer: E
family, thus L must be married to G.
 W is 20m east of M’s father. a) J≥B -> B>C=D≤E≤J ->is False

 The gender of M and Z is same. b) I<D -> I≤H=B>C=D ->is False

 W’s daughter sits 8m west of L. c) D≥K -> D≤E≤J=K ->is False


d) B≥M -> B>C=D≤E<M ->is False
That means, Z is 8m west of L.
e) D<F-> D=C<B<F ->is True
Based on the above given information we have:

18. Answer: C
a) V≥M -> V≤W≥X=Y≥M ->is False
b) J≤K -> J≥W≥X=Y>K ->is False
c) G>U-> U<V≤W<G ->is True
d) L>W ->W≥X=Y>K<L ->is False
e) X=L ->X=Y>K<L ->is False

Direction:
19. Answer: D
a) M≤U -> M≥L>T≤U ->is False
b) O<J -> O=N>I>J ->is False
c) K>N -> K=J<I<N ->is False
d) K<Q -> Q≥I>J=K ->is True

Click Here For Bundle PDF Course | support@guidely.in Page 9 of 10


SBI Clerk & RRB PO Mains PDF Course 2023
Reasoning Ability Day - 33 (Eng)

e) N<T -> N>I>J=K≥L>T ->is False b) J>E -> J≥C≤D<E ->is False
c) L≥G -> G=F<B=C≤K=L ->is False
20. Answer: E d) I<A -> I≥G=F<B≤A ->is False
a) A≤K ->K≥C=B≤A ->is False e) A>G->G=F<B≤A ->is True

Click Here For Bundle PDF Course | support@guidely.in Page 10 of 10


SBI Clerk & RRB PO Mains PDF Course 2023
Quantitative Aptitude Day – 33 (Eng)

Quantitative Aptitude

Directions (01 - 05): Study the following information carefully and answer the questions given below.
The given Radar graph shows the total number of students [boys +girls] preparing for three exams from
city A.

Note- The ratio of the number of boys and girls who prepare CAT exam is 2:3. Number of girls who
prepare CAT exam is 12.5% more than the number of girls who prepare MAT exam. The ratio of the
number of girls prepare MAT and XAT exam is 4:5.
1) In city A, number of boys who prepare for the 2) Number of boys preparing for the CAT exam
CMAT exam is 40% more than that for the XAT from city B is 40 more than the number of boys
exam and the number of girls who prepare for preparing for CAT exam from city A.If the total of
the CMAT exam is 20% less than that for XAT 290 students prepare for the CAT exam from city
exam. Find the total number of students B, then find the total number of girls from cities A
preparing for the CMAT exam from city A? and B together preparing for the CAT exam?
a) 520 a) 320
b) 560 b) 340
c) 510 c) 380
d) 570 d) 310
e) 540 e) 360

Click Here For Bundle PDF Course | support@guidely.in Page 1 of 11


SBI Clerk & RRB PO Mains PDF Course 2023
Quantitative Aptitude Day - 33 (Eng)

3) Ratio of the number students from science, together and the number of girls who prepare for
arts, and commerce background preparing for the MAT and XAT exams together?
MAT exam is 5:3:2, and the ratio of number a) 0
students from science, arts, and commerce b) 50
backgrounds preparing for XAT exam is 5:1:3. c) 10
Find the difference between the total students d) 20
who are from science background prepare for e) None of these
these two exams and total number students from
commerce background prepare for these two 5) Find the approximate difference between the
examsif only these three types of students percentage of boys and percentage of girls
prepare for these exams? preparing for allexams out of the total number of
a) 260 students preparing for all three exams?
b) 220 a) 6%
c) 260 b) 5%
d) 240 c) 2%
e) None of these d) 9%
e) None of these
4) Find the difference between the number of
boys who prepare for CAT and MAT exams

Directions (06 - 10): Study the following information carefully and answer the questions given below.
The given two pie charts show the percentage distribution of the number of employees in a company
using different companies' Laptop [win11+win10] and the percentage distribution of the number of
employees using the Win11 version.

Click Here For Bundle PDF Course | support@guidely.in Page 2 of 11


SBI Clerk & RRB PO Mains PDF Course 2023
Quantitative Aptitude Day - 33 (Eng)

Note:
I. Ratio of x:y=2:5.
II.Total of 75 employees use the win10 version of company D Laptop and a total of 100 employees use
the win11 version of C company Laptop.
6) Which of the given pie chart is correct if we
represent the percentage distribution of the
number of employees who use the Win10
version?

Click Here For Bundle PDF Course | support@guidely.in Page 3 of 11


SBI Clerk & RRB PO Mains PDF Course 2023
Quantitative Aptitude Day - 33 (Eng)

a) 7) Ratio of number of male and female


employees who use the Win11 version of A
company laptop is 3:2 and the ratio of male and
female employees who use the Win10 version of
A company is 2:1. Find the total number of
females employees who use company A laptop?
a) 62
b) 52
c) 88
b)
d) 70
e) 71

8) Find the difference between the totalnumber


of employees who use the win11 version of B
and C company laptops and the number of
employees who use the win10 version of D and
c)
E company laptops?
a) 82
b) 80
c) 85
d) 90
e) None of these

9) Number of employees who use F company


d) laptops is 15 more than the number of
employees who use E company laptops. 40% of
employees use win10 of laptop company F then
find the number of employees who use win11 of
laptop company F?
a) 90
b) 96
c) 84
e) None of these d) 60
e) None of these

Click Here For Bundle PDF Course | support@guidely.in Page 4 of 11


SBI Clerk & RRB PO Mains PDF Course 2023
Quantitative Aptitude Day - 33 (Eng)

10) Find the ratio of the number of employees the work and C and E together to complete the
who use the win10 laptops of companies A and work?
E together and the number of employees who a) 2 days
use the win11 laptop of companies B and D b) 1 day
together? c) 0 days
a) 203:309 d) 3 days
b) 223:398 e) 5 days
c) 227:390
d) 223:391 13) Find the numerical value of [t+y]?
e) None of these a) 20
b) 25
Directions (11 - 13): Study the following c) 22
information carefully and answer the questions d) 23
given below. e) None of these
A can complete a work in (t+25) days and B can
complete the work in (y+20) days. A is 25% more Directions (14 - 16): Study the following
efficient than D. Ratio of efficiency of C and E is information carefully and answer the questions
3:1. D can complete the work in 50 days. When given below.
A and B start the work and after 12 days B left A survey conducts in a city about the number of
the work then A can finish the rest of the work in people who like to travel to the Hill, Forest, and
12 days. If C and B work on alternative days Sea. The ratio of the number of people who like
starts with C, then the total work will be only the Hill and who like only to the Sea is 5:6.
completed in 24 days. Number of people who like both the hill and
11) A and E start the work and after [y+5] days, forest but not the sea is 1/6 of the number of
they left the work and C finish the work in z days. people who like only the sea. The number of
Find the value of z? people who likes all three is 10. The number of
a) 4.5 people who like only forest is six times the
b) 8.5 number of people who like both hill and sea but
c) 7.5 not forest. The ratio of the number of people who
d) 9.5 like both hill and forest but not sea and the
e) 2.5 number of people who like both sea and forest
but not hill is 2:1. The total number of people who
12) Find the difference between the total number likes forest is 250. 200 people likes only Hill.
of days taken by B and C together to complete

Click Here For Bundle PDF Course | support@guidely.in Page 5 of 11


SBI Clerk & RRB PO Mains PDF Course 2023
Quantitative Aptitude Day - 33 (Eng)

14) Ratio of the number of males and females out then 20 liters water is added then the
who like only hills, forests, and the sea is 3:2,4:5 difference between milk and water is _____liters.
and 5:3. Find the difference between the total I.120,20,5
number of males and females who like only hills, II.80,30,6
forests and seas? III.160,50,20
a) 68 Find which of the following options is correct?
b) 75 a) Only I is false
c) 80 b) Only III is true
d) 90 c) Only II is true
e) 85 d) Only I and III are true
e) Only I is true
15) I= Number of people who like only Hill and
Sea but not Forest 18) Cost price of item X is Rs.160. The cost price
M=Number of people who like only Sea and of Item Y is Rs. _____ more than that of item X.
Forest, not Hill Item X is sold at ____% profit and Item Y is sold
Find the correct relation of I and M? at 20% profit. The overall profit by selling two
a) I+M=80 items is Rs.80.
b) I+2M=120 Find which of the options fill in the blank to make
c) 40%of I+20%of M=25 the statement true?
d) 30% of I+40% of M=17 a) 60,30
e) None of these b) 40, 25
c) 60,120
16) Find the difference between the total number d) More than one option is true
of people who like the sea and the total number e) None of these
of people who like Hill?
a) 25 19) Ratio of the speed of the boat and the speed
b) 20 of stream is 3:1. Boat covers ___ Km in
c) 30 downstream in 3 hours and the boat covers
d) 40 ____km upstream in 2 hours?
e) None of these I. 48,16
II.60,20
17) A mixture contains ___ liters mixture of milk III.102,34
and water in a ratio of 5:3. __% mixture is taken Find which of the following is true?
a) All are true

Click Here For Bundle PDF Course | support@guidely.in Page 6 of 11


SBI Clerk & RRB PO Mains PDF Course 2023
Quantitative Aptitude Day - 33 (Eng)

b) All are false years. The ratio of age of C and D is 5:4. Find
c) Only I is true which of the following option fills the blank to
d) Only II and III are true make the statement true?
e) None of these a) 5,45
b) 3,54
20) A is ____ years older than B. B is 5 years c) 4,36
older than C. Sum of the age of C and D is ____ d) 7,27
years and the sum of the age of A and B is 54 e) More than one option is true
Click Here to Get the Detailed Video Solution for the above given Questions
Or Scan the QR Code to Get the Detailed Video Solutions

Answer Key with Explanation

Directions (01 - 05): The number of students prepare for CMAT exam
The Number of boys preparing CAT exam is is
300*2/5=120 =250*140/100+200*80/100=350+160=510
The Number of girls preparing CAT exam is
300*3/5=180 2) Answer: D
The Number of girls preparing MAT exam is The number of girls preparing for the CAT exam
180*100/112.5=160 from cities A and B together is = [290-(120+40)]
The Number of boys preparing MAT exam is +180=130+180=310
=400-160=240
The Number of girls preparing XAT exam is = 3) Answer: B
160*5/4=200 The total number of students from science
The Number of boys preparing XAT exam is backgrounds prepare forthese two exams is =
=450-200=250 400*5/10+450*5/9=200+250=450
1) Answer: C

Click Here For Bundle PDF Course | support@guidely.in Page 7 of 11


SBI Clerk & RRB PO Mains PDF Course 2023
Quantitative Aptitude Day – 33 (Eng)

The total number of students from commerce So, 20%=100, 100%=500.


backgrounds prepare forthese two exams is The number of employees using the win11
=400*2/10+450*3/9=80+150=230 version of the laptop of company D is
Required difference=450-230=220 500*30/100=150
The total number of employees using laptops of
4) Answer: A company D is = 150+75=225
Required difference = [160+200]-[120+240] So, 25%=225, 100%=900
=360-360=0 A total number of 900 employees use laptops of
companies A, B, C, D, and E respectively.
5) Answer: A A total number of 400 employees use the win10
The percentage of boys prepare for exam is version of the laptop of companies A, B, C, D,
= and E respectively.
[(120+240+250)/(300+400+450)]*100=53.04%=5
3%
The percentage of girls prepare for exam is
=[(180+160+200)/(300+400+450)]*100=46.95=4
7%
Required difference=53-47=6%

Directions (06 - 10):


So, from the second pie chart,
6x+14x+10x+15x+5x=100, x=2
So, y=2*5/2=5
Percentage distribution of the number of
employees who use laptops of companies A, B, 6) Answer: C
C, D, and E is 22%,18%,20%,25%, and 15% The percentage of employees who use the
respectively. win10 version laptop of company A is
Percentage distribution of the number of [138/400]*100=34.5%
employees who use company win11 version Similarly, we can calculate the other company
laptops of companies A, B, C, D, and E is also.
12%,28%,20%,30%, and 10% respectively. So, the pie chart's correct representation is
The number of employees who use the win11 option
version of the laptop of company C is 100.

Click Here For Bundle PDF Course | support@guidely.in Page 8 of 11


SBI Clerk & RRB PO Mains PDF Course 2023
Quantitative Aptitude Day – 33 (Eng)

C If C and B work on alternative days start with C


then the total work will be completed in 24 days.
So, B in 12 days completes 12/30=2/5 work.
C in 12 days complete 3/5 work.
C can complete work in 12*5/3=20 days.
E complete work in 20*3=60 days.
Total work=LCM of 40,30,20,50, and 60=600
units
The efficiency of A, B, C, D, and E is
15,20,30,12, and 10 units/day respectively.
11) Answer: C
7) Answer: D
A and E did [15+10]*15=375 units of work.
Number of females use company A laptop is
C did (600-375)=225 unit of work in 225/30=7.5
=60*2/5+138*1/3=24+46=70
days
So, z=7.5
8) Answer: B
Required difference= [140+100]-[85+75]=80
12) Answer: D
Required difference= [600/40]-[600/50]=15-12=3
9) Answer: A
days
The number of employees who use win11 on the
laptop of company F is
13) Answer: B
= [135+15]*60/100=90
Value of t+y=15+10=25

10) Answer: E
Directions (14 - 16):
Required ratio= [138+85] : [140+150]=223: 290
The Number of people who like only the Sea is
200*6/5=240
Directions (11 - 13):
The Number of people who like only hills and
The ratio efficiency of A and D is 5:4.
forests but not the sea is 240/6=40
So, (t+25)/50=4/5, t=40-25=15
The Number of people who like only sea and
So, A can complete the work in 40 days.
forest but not hill is 40/2=20
In 24 days, A complete 24/40=3/5 work.
The Number of people who like only forest
So, [1-(3/5)]=2/5 work completed B in 12 days, B
is=250-40-20-10=180
completes the whole work in 12*5/2=30 days.
So, y=30-20=10

Click Here For Bundle PDF Course | support@guidely.in Page 9 of 11


SBI Clerk & RRB PO Mains PDF Course 2023
Quantitative Aptitude Day – 33 (Eng)

The Number of people who like only hills and And the amount of water is 80*3/8=30 liters
sea, but not the forest is 180/6=30 So, Difference of milk and water is
[30*70/100+20]-[50*70/100]
=41-35=6 liters
So, Only II is true.

18) Answer: B
Let's check option B,
The cost price of item Y is 160+40=Rs.200
So, total selling price is =
160*125/100+200*120/100=200+240=Rs.440
So, the overall profit is 440-160-200=Rs.80
So, option B is true.
14) Answer: C
The total number of males like only Hill, Forest
19) Answer: A
and Sea is
For the question we can say, the speed of the
=200*3/5+180*4/9+240*5/8=120+80+150=350
boat and the stream is 3a and a.
Total female like only hill, forest and sea is =
So, the ratio of downstream speed and upstream
[80+100+90]=270
speed is [4a/2a]=2/1
so, 350-270 =80
In option I, the downstream speed is
48/3=16km/hr
15) Answer: D
Upstream speed is 16/2=8km/hr
I=30, M=20
So, the ratio is 16:8=2:1
So, 30% of I+40% of
So, it is true.
M=30*30/100+20*40/100=17
Similarly, we can check others' value also.
And all are true.
16) Answer: B
Required difference=[240+30+10+20]-
20) Answer: C
[200+40+10+30]
Let's check option C,
=300-280=20
The Sum of the ages of C and D is 36 years.
Age of C is 36*5/9=20 years
17) Answer: C
Age of B is 20+5=25 years, age of A is 25+4=29
Let’s check option II,
years
The amount of milk is 80*5/8=50 liters

Click Here For Bundle PDF Course | support@guidely.in Page 10 of 11


SBI Clerk & RRB PO Mains PDF Course 2023
Quantitative Aptitude Day – 33 (Eng)

So, the sum of the ages of B and A is =54 years.

Click Here For Bundle PDF Course | support@guidely.in Page 11 of 11


SBI Clerk & RRB PO Mains PDF Course 2023
ENGLISH Day - 33

English Language

Directions (1-5) : Rearrange the following five d) D


sentences (A), (B), (C), (D) and (E) in the proper e) E
sequence to form a coherent paragraph and then
answer the questions given below. 2) Choose from the below options the
(A) The alliance operates on the principle of SYNONYM of the word PROMOTE as given in
collective defence/A, stating that an attack the sentence B.
against one member is/B considered an attack a) Stimulate
against to all, leading to/C joint action to deter b) demote
and respond to potential threats/D. c) discourage
(B) Formed in 1949, NATO is a political and d) dishonour
military alliance that aims to promote collective e) desist
defence, preserve peace, and safeguard the
freedom of its member nations. 3) Fill in the blank given in sentence C and
(C) NATO, the North Atlantic Treaty complete it.
Organization, stands as a cornerstone of a) commemoration
international security and __________ . b) contemplation
(D) a pivotal role in promoting stability and c) connotation
peace in /P Europe and beyond, undertaking d) cooperation
missions /Q throughout its history, NATO has e) confrontation
played /R management, and disaster relief
operations /S such as peacekeeping, crisis /T 4) Rearrange the parts of the sentence in D to
(E) Comprising 30 member countries from North form a meaningful sentence.
America and Europe, NATO fosters close a) TQSPR
diplomatic ties and military cooperation among its b) QPTRS
members to address a wide range of security c) STRQP
challenges. d) PQRST
1) Find which part of sentence ‘A’ has an error in e) RPQTS
it and mark that as your answer. Mark option (e)
if no error is found. 5) Which of the following is the correct sequence
a) A of rearrangement that forms a meaningful
b) B coherent paragraph ?
c) C

Click Here For Bundle PDF Course | support@guidely.in Page 1 of 11


SBI Clerk & RRB PO Mains PDF Course 2023
ENGLISH Day - 33

a) CDEAB
where immortality.
b) CBEAD
c) BCEAD C. Over time, the art F. against the darkness,

d) AECDB of creating fireworks igniting a sense of

e) EDBCA spread to different childlike wonder in our


parts of the world, hearts.

Directions (6-10): In the following questions two each culture

columns are given containing three a) A-F


Sentences/phrases each. In the first column, b) A-E
sentences/phrases are A, B and C and in the c) B-E
second column the sentences/phrases are D, E d) A-E and C-D
and F. A sentence/phrase from the first column e) None of these
may or may not connect with another
sentence/phrase from the second column to 7)
make a grammatically and contextually correct COLUMN 1 COLUMN 2
sentence. Each question has five options, four of
A. While conjunctivitis D. dirty hands, using
which display the sequence(s) in which the
is generally not a contaminated eye
sentences/phrases can be joined to form a
severe condition, it makeup, or swimming in
grammatically and contextually correct sentence.
can be highly water with harmful
If none of the options given forms a correct
contagious, bacteria.
sentence after combination, mark option (e), i.e.
necessitating
“None of these” as your answer.
6) B. These infections E. self-medicate eye
COLUMN 1 COLUMN 2 can lead to more infections, as improper
severe symptoms like treatment can
A. From brilliant D. create entertainment
severe pain, blurred exacerbate the condition
cascades of sparkles and began incorporating
vision, sensitivity or delay proper care.
to thunderous them into religious
explosions that ceremonies and festive C. Eye infections can F. to light, and even
reverberate in the events. be caused by various vision loss if left
factors, including untreated.
B. The history of E. they were initially
poor hygiene,
fireworks dates back invented by alchemists
touching the eyes
to ancient China, searching for the elixir of
with

Click Here For Bundle PDF Course | support@guidely.in Page 2 of 11


SBI Clerk & RRB PO Mains PDF Course 2023
ENGLISH Day - 33

9)
a) A-D COLUMN 1 COLUMN 2
b) B-F
A. In an age D. simple pleasure of
c) A-F and B-D
dominated by traditional games, the
d) B-F and C-D screens and great outdoors offers an
e) None of these technology, the expansive canvas for
allure of outdoor boundless fun and
8)
games remains as adventure.
COLUMN 1 COLUMN 2
strong as ever,
A. By being D. technology's benefits
B. From soccer and E. and the thrill of
positive role and ensuring healthy real-
basketball to cricket scoring a point can
models and world interactions is a
and baseball, team create unforgettable
displaying qualities challenge parents face in
sports have a unique memories with friends
such as kindness, the digital age.
way of fostering and family.
B. It is also crucial E. resilience, and honesty, camaraderie,
for parents to parents impart invaluable teaching
remember that no life lessons that extend far
C. Whether it's the F. bring people together,
one is infallible, beyond the classroom.
excitement of team encouraging physical
and making
sports, the thrill of activity and imaginative
mistakes
running freely on play in equal measure.
C. The constant F. ample time to children open fields, or the
juggling act can sometimes leave a) A-F
between work, parents feeling
b) C-D
household overwhelmed and c) B-D and C-F
responsibilities, exhausted.
d) B-E and C-D
and giving e) None of these
a) A-E and C-F
b) A-D and C-E 10)
c) B-F and C-D COLUMN 1 COLUMN 2
d) B-E and C-F A. Adulteration, a D. affects human health
e) None of these
deceptive practice but also tarnishes the

Click Here For Bundle PDF Course | support@guidely.in Page 3 of 11


SBI Clerk & RRB PO Mains PDF Course 2023
ENGLISH Day - 33

11) The storekeeper was instructed to _______


that involves adding reputation of businesses
the newly arrived merchandise on the shelves,
inferior or harmful and industries.
ensuring a tidy _______ that would attract
substances to food,
customers and make their shopping experience
beverages, or other
products, poses a more enjoyable.
a) count, place
B. The history of E. adulteration has b) arrange, display
adulteration dates expanded to a wide range c) disturb, present
back centuries, of products, including d) keeping, look
where unscrupulous food items, e) organized, exhibit
individuals sought to pharmaceuticals,
profit at the expense cosmetics, and even 12) Typewriting classes were in high demand
fuels. during the pre-computer era, as they offered
valuable skills for ______office professionals and
C. Contaminated F. serious threat to public
typists, ensuring fast and ______ document
food and beverages health and undermines
creation.
can lead to a host of the integrity of industries
a) demotivated, inexact
health issues, and businesses.
b) beginning, furious
ranging from mild
c) aspiring, accurate
discomfort and
d) disinterested, improper
a) A-F
e) energetic, false
b) B-E
c) B-D and C-F
13) Free meals in the government schools aimed
d) B-E and C-F
to _______ the issue of malnutrition among
e) None of these
underprivileged children, ensuring they received
_______ nutrition for their overall well-being.
Directions (11-15) : Each of the following
a) address, adequate
questions has two blanks, each blank indicating
b) fixed, good
that a word/phrase has been omitted. Choose
c) solve, insufficient
the set of words/phrases that best fits the given
d) complicate, satisfaction
blanks according to the context of the sentence.
e) preclude, fair
Few alphabets are given to help you locate the
blanks and fill them.
14) She worked really hard to manage both her
_______job and her responsibilities as a single

Click Here For Bundle PDF Course | support@guidely.in Page 4 of 11


SBI Clerk & RRB PO Mains PDF Course 2023
ENGLISH Day - 33

parent, proving her determination and dedication e) No changes required


to ______the best for her child.
a) demanding, provide 17) The medicines at the store were neatly
b) easy, take arranged (A) on the shelves, with a wide
c) simple, deliver assortment (B) of prescription drugs, over-the-
d) new, lend counter remedies, and health supplements
e) teaching, refuse catering (C) to various healthcare needs (D).
a) A-B, B-assorted
15) The launch of the restaurant was a grand b) A-D, D-equipments
______, with celebrities, food critics, and locals c) B-C, A-arranging
alike attending the event to _____the delectable d) C-D, C-caters
offerings and celebrate the culinary journey. e) No changes required
a) event, tasted
b) launch, enjoy 18) Exams are arriving (A) up, and there's a mix
c) opening, dislike of nervousness and determination among the
d) affair, savour students as they diligently (B) prepare and
e) party, oppose achieve (C) to perform at their best and revise
(D) their academic goals.
Directions (16-20) : The following questions a) A-C, D-revised
given below have four words that have been b) B-C, C-achieving
highlighted as they might’ve been placed at the c) B-D, A-arrived
wrong positions. One of these words might also d) C-D, A-coming
be incorrect and need a replacement. Read the e) No changes required
same carefully and mark the appropriate option
as the answer. 19) Ballpoint pens are better choice (A) for
16) My friend is celebration (A) married this everyday (B) writing tasks due to their smooth ink
month, and the excitement in the air is palpable flow, reliability, and convenience, making them a
(B) as we eagerly anticipate getting (C) their love popular suit (C) among students and
and witnessing (D) the beginning of a beautiful professionals alike (D).
new chapter in their lives. a) A-C, C-suited
a) A-B, D-encountering b) A-D, B-every
b) B-D, C-readily c) B-C, C-suited
c) A-C, A- celebrating d) C-D,B-every
d) C-D, A-celebrating e) No changes required

Click Here For Bundle PDF Course | support@guidely.in Page 5 of 11


SBI Clerk & RRB PO Mains PDF Course 2023
ENGLISH Day - 33

20) The OMR sheet given (A) in the examination a) A-D, A-gave
was carefully designed with printed bubbles b) B-C, C-corresponding
allowing (B) to each question, correspond (C) c) B-D, B- allow
candidates to mark their answers precisely and d) C-D, B-allowed
facilitating (D) efficient evaluation. e) No changes required

Click Here to Get the Detailed Video Solution for the above given Questions
Or Scan the QR Code to Get the Detailed Video Solutions

Answer Key with Explanation

1) Answer: C The word stimulate is similar in meaning to the


Part C of the sentence has an error in it. word ‘promote’
Replace ‘against to all’ with ‘against all’ to make The other words wither have different meaning
the given sentence grammatically correct and or they are the opposite of ‘promote’
meaningful.
The corrected sentence : The alliance operates Demote - to move somebody to a lower position
on the principle of collective defence, stating that or level
an attack against one member is considered an Dishonour - lack or loss of honour
attack against all, leading to joint action to deter Desist - stop doing something; cease or abstain
and respond to potential threats.
3) Answer: D
2) Answer: A Cooperation is the right word that fits the given
The meaning of promote - to encourage blank correctly.
something Commemoration - something that is done to
Stimulate - to make something active or more remember officially and give respect to a great
active person or event

Click Here For Bundle PDF Course | support@guidely.in Page 6 of 11


SBI Clerk & RRB PO Mains PDF Course 2023
ENGLISH Day - 33

Contemplation - deeply or seriously thoughtful. members and the diplomatic ties which should
Connotation - an idea expressed by a word in come after the introduction and before the actual
addition to its main meaning work done by NATO. As, sentence ‘E’ spoke
Confrontation - a fight or an argument about the security challenges ‘A’ is the correct
The sentence: NATO, the North Atlantic Treaty sentence that follows ‘E’. The final sentence or
Organization, stands as a cornerstone of the last sentence is ‘D’ which seems to be a
international security and cooperation. concluding sentence representing the actions of
NATO throughout history.
4) Answer: E
The correct sequence of rearrangement from the 6) Answer: C
given option is option (e) RPQTS. B-E forms a complete sentence which is both
R is the starter sentence and the remaining are a meaningful and correct.
continuation of one another, P follows R joining The other options have a combination of
the sentence. Next comes Q which is a statements/fragments which do not combine to
continuation of P. Among S and T, S is the form a complete sentence.
concluding sentence because it has the last The sentence formed : The history of fireworks
fragment “and…”. So, TS forms the last two dates back to ancient China, where they were
sentences. initially invented by alchemists searching for the
The sentence : Throughout its history, NATO elixir of immortality.
has played a pivotal role in promoting stability
and peace in Europe and beyond, undertaking 7) Answer: D
missions such as peacekeeping, crisis Both B-F and C-D are the correct combinations
management, and disaster relief operations. which combine to form a complete sentence
which is meaningful and correct.
5) Answer: B The other options are incorrect as those
CBEAD is the right sequence of rearrangement combinations of fragments/statements do not
that forms a meaningful paragraph. form a proper sentence.
C is the first sentence, as it has the full form of The sentences formed :
NATO and also the basic/simple definition or B-F : These infections can lead to more severe
purpose of NATO. ‘B’ follows ‘C’ as it talks about symptoms like severe pain, blurred vision,
the formation of NATO and gives more details on sensitivity to light, and even vision loss if left
its purpose. Sentence ‘E’ follows CB and comes untreated.
before both A and D because it talks about the C-D : Eye infections can be caused by various

Click Here For Bundle PDF Course | support@guidely.in Page 7 of 11


SBI Clerk & RRB PO Mains PDF Course 2023
ENGLISH Day - 33

factors, including poor hygiene, touching the expansive canvas for boundless fun and
eyes with dirty hands, using contaminated eye adventure.
makeup, or swimming in water with harmful
bacteria. 10) Answer: A
A-F is the correct combination that forms a
8) Answer: A complete sentence.
A-E forms a complete meaningful sentence The other given options are wrong as they don’t
similarly the combination C-F forms a complete contain statements that form a meaningful
sentence with meaning. Fragments B and D do sentence.
not combine with the other statements to The sentence formed : Adulteration, a deceptive
generate a sentence and hence are discarded. practice that involves adding inferior or harmful
The sentences formed : substances to food, beverages, or other
A-E : By being positive role models and products, poses a serious threat to public health
displaying qualities such as kindness, resilience, and undermines the integrity of industries and
and honesty, parents impart invaluable life businesses.
lessons that extend far beyond the classroom. 11) Answer: B
C-F : The constant juggling act between work, Arrange and display are the right words that fit
household responsibilities, and giving ample both the given blanks respectively to complete
time to children can sometimes leave parents the meaning of the sentence.
feeling overwhelmed and exhausted. First blank - this blank needs a verb in the first
form - count, arrange and disturb are the verbs
9) Answer: B in the first form - so eliminate options d and e.
The fragment C and D combine to form a Disturb completely inappropriate to the given
meaningful and complete sentence. context - eliminate option c
Even though the other statements also revolve Count on the shelves is wrong - so eliminate
around the same topic they do not combine with option a
each other to form a sentence that is meaningful The left out and the correct option is ‘b’.
and correct. The sentence : The storekeeper was instructed
The sentence formed : Whether it's the to arrange the newly arrived merchandise on the
excitement of team sports, the thrill of running shelves, ensuring a tidy display that would
freely on open fields, or the simple pleasure of attract customers and make their shopping
traditional games, the great outdoors offers an experience more enjoyable.

Click Here For Bundle PDF Course | support@guidely.in Page 8 of 11


SBI Clerk & RRB PO Mains PDF Course 2023
ENGLISH Day - 33

12) Answer: C the right word that fits the second blank in
Aspiring and accurate are the two words that fit accordance with the context of the sentence.
the given blanks respectively. Option a is the correct answer.
First blank - aspiring and energetic are the only The sentence : Free meals in the government
words that fit the context of the sentence, the schools aimed to address the issue of
other words create a negative impact on the malnutrition among underprivileged children,
sentence, so discard options a, b and d. ensuring they received adequate nutrition for
Second blank - again ‘false’ creates a negative their overall well-being.
impact but the sentence is in a positive tone
where professionals learnt typewriting to develop 14) Answer: A
their skills. So, to match the context ‘accurate’ is The correct pair is “demanding and provide”.
the correct word. First blank - All the given words fit the first blank
The sentence : Typewriting classes were in high but ‘easy and simple’ are opposite in context of
demand during the pre-computer era, as they the given sentence and hence options b and c
offered valuable skills for aspiring office are discarded.
professionals and typists, ensuring fast and Among a,d and e - the option which has the
accurate document creation. correct word to fill the second blank is option(a) -
‘provider the best’
13) Answer: A ‘Lend the best’ and ‘refuse the best’ are
Address and adequate are the correct words and incorrect and inappropriate and hence not the
hence option (a) is the right answer for this right pick
question. The sentence : She worked really hard to
The first blank needs a verb in the first form - manage both her demanding job and her
discard option b responsibilities as a single parent, proving her
The government scheme is in place to help the determination and dedication to provide the best
people and children so ‘complicate and preclude’ for her child.
are incorrect words that can be used - eliminate
options d and e 15) Answer: D
Preclude - to prevent something from happening Affair and savour are the right words.
or somebody from doing something; to make Savour - to taste or smell with pleasure
something impossible First blank - launch and opening do not fit the
Between options a and c - look at the second first blank because they are redundant and
words adequate and insufficient - adequate is

Click Here For Bundle PDF Course | support@guidely.in Page 9 of 11


SBI Clerk & RRB PO Mains PDF Course 2023
ENGLISH Day - 33

conveying the same meaning repeatedly - so Out of the given options ‘coming’ is the right
discard options b and c word to put in place (A) and replace words C and
Out of options a,d and e - ‘to tasted’ is wrong, D to make the sentence correct.
similarly ‘to oppose’ is inappropriate to the The sentence: Exams are coming up, and
context of the sentence, hence ‘savour’ is the there's a mix of nervousness and determination
correct word. among the students as they diligently prepare
The sentence : The launch of the restaurant was and revise to perform at their best and achieve
a grand affair, with celebrities, food critics, and their academic goals.
locals alike attending the event to savour the
delectable offerings and celebrate the culinary 19) Answer: A
journey. ‘Are better choice’ is wrong instead it must be
‘are a better choice' so definitely word (A) is
16) Answer: C misplaced, swap it with word C to make the
First of all the word ‘celebration’ is incorrect, sentence correct.
instead it must be celebrating. And the positions The word ‘suit’ is still inappropriate, change it to
of the words A and C must be interchanged to ‘suited’ to make the words and the respective
make the given sentence all correct and sentence correct and meaningful.
meaningful. The sentence: Ballpoint pens are better suited
The sentence: My friend is getting married this for everyday writing tasks due to their smooth ink
month, and the excitement in the air is palpable flow, reliability, and convenience, making them a
as we eagerly anticipate celebrating their love popular choice among students and
and witnessing the beginning of a beautiful new professionals alike.
chapter in their lives.
20) Answer: B
17) Answer: E Swap the words B and C to make the sentence
The given four words are correctly placed and correct and meaningful. The word ‘correspond’ is
are appropriate too, hence there is no need to incorrect, instead it must be ‘corresponding’ to
swap or replace them. make the sentence more accurate and
meaningful.
18) Answer: D The sentence: The OMR sheet given in the
‘Arriving up’ is completely wrong and hence examination was carefully designed with printed
needs replacement of the word itself. bubbles corresponding to each question,

Click Here For Bundle PDF Course | support@guidely.in Page 10 of 11


SBI Clerk & RRB PO Mains PDF Course 2023
ENGLISH Day - 33

allowing candidates to mark their answers precisely and facilitating efficient evaluation.

Click Here For Bundle PDF Course | support@guidely.in Page 11 of 11


SBI Clerk & RRB PO Mains PDF Course 2023
Reasoning Ability Day - 34 (Eng)

Reasoning Ability

Directions (1-5): Study the following information e) All a, b, and c


carefully and answer the given questions.
Eight persons – G, K, L, N, P, Q, S, and Y were 2. Who among the following persons was born
born in eight different months in the year 2011. in the month which has an odd number of days
Not more than two persons were born in the but not on an even-numbered date?
same month. No two persons were born on the I. The one who was born immediately before G
same date of the same month. II. The one who was born two persons after Q
Y was born on 12th of the month which has an III. The one who was born three persons before
even number of days but after August. N was Y
born on 4th of the month and three months before a) Only I and II
Y. K was born 22 days before N but one of the b) Only II
prime-numbered dates. Only two persons were c) All I, II, and III
born between K and Q but both were born on the d) Only I
same numbered date. G was born in the same e) Only I and III
month as Q but not in the month which has an
even number of days. Three months gap 3. __ was born two persons before __, who was
between the born months of G and P, who was born three persons after__ respectively with
born on 18th of July. L was born in the month respect to the final arrangement
which has the least number of days but on the a) S, P, and G
same numbered date as P. The number of days b) K, N, and S
between the dates on which L and G were born c) P, Y, and L
is 25. S was born on the same numbered date as d) Both a and b
K but was born in the month which has an odd e) Both a and c
number of days. At least two months gap
between the months in which S and N were born. 4. How many following combination(s) is/are true
1. The number of persons born before G is __ with respect to the final arrangement?
the number of persons born after __ I. Q – 13th – March
a) Equal; P II. S – 13th – May
b) One less; the one who was born immediately III. K – 18th – July
after S IV. G – 16th – March
c) Two less; the one who was born in May a) None
d) Both a and b b) Three

Click Here For Bundle PDF Course | support@guidely.in Page 1 of 10


SBI Clerk & RRB PO Mains PDF Course 2023
Reasoning Ability Day - 34 (Eng)

c) One two less than the number of persons sitting


d) Two between R and G. The one who faces R sits
e) Four adjacent to T. Not more than two persons sit
between P and the one who faces F. A sits
5. How many persons were born between G and neither adjacent to L nor faces south. F and A
N? are facing different directions.
a) Three 6. Which of the following statements is/are true
b) One as per the given arrangement
c) Two I. G sits second to the left of the one who faces D
d) None II. W sits third to the right of the one who faces T
e) More than three III. I is facing G
IV. Q faces the one who sits immediate right of X
Directions (6-10): Study the following information a) Only I, II, and III
carefully and answer the given questions. b) Only I, III, and IV
Fourteen persons – A, B, D, F, G, I, L, P, Q, R, T, c) Only II and IV
V, W, and X are sitting in two parallel rows in d) Only II, III, and IV
such a way that equal distance between adjacent e) All I, II, III, and IV
persons. In row 1 seven persons are seated and
all of them are facing north while in row 2 seven 7. L sits ___ of X and __ from the left end of the
persons are seated and all of them are facing row.
south. Each person in row 1 faces another a) Third to the right; Fourth
person in row 2. b) Third to the left; Third
Note: The person whose name starts with a c) Second to the left; Fifth
vowel does not face the person whose name d) Third to the left; Fifth
comes after Q in the English alphabetical order. e) Second to the right; Third
D sits third from one of the extreme ends. Only
two persons sit between D and the one who 8. The number of persons sitting between B and
faces X. I sits either second to the right or third to the one who faces F is ___ than the number of
the left of D. As many persons sit to the right of I persons sitting between __ and the one who
as to the left of the one who faces W. Only three faces W with respect to the final arrangement.
persons sit between W and L. B faces the one a) One less; A
who sits third to the right of L. Only one person b) One more; the one who faces L
sits between B and the one who faces V. The c) Two less; the one who faces V
number of persons sitting between V and X is d) Equal; T

Click Here For Bundle PDF Course | support@guidely.in Page 2 of 10


SBI Clerk & RRB PO Mains PDF Course 2023
Reasoning Ability Day - 34 (Eng)

e) Two more; the one who faces G II. Some (b) is not baba
III. All (c)can never be Baba
9. Which among the following pairs of persons a) Baba; Baal; Babe
the first person sits third to the left of the second b) Baal; Babe; Babe
person and the second person sits second to the c) Baba; Babe; Baal
right of the third person with respect to the final d) Both a and c
arrangement? e) Both a and b
a) L, X, and V
b) F, R, and W 12. Statements
c) P, A, and T Some (a) are (b)
d) Both a and c Only a few (c) are Cade
e) Both a and b All Cade is (d)
Conclusions
10. How many persons are sitting between P and I. All Cabs can never be Cade
the one who sits immediate right of A? II. No Cade can be Cack
a) As many persons sit between Q and I III. Some Cads are Cade
b) Two a) Cabs; Cack; Cabs; Cads
c) As many persons between G and the one who b) Cabs; Cads; Cads;Cack
faces P c) Cack; Cabs; Cads; Cack
d) One d) Cade; Cack; Cabs; Cads
e) Four e) Cads; Cabs; Cack; Cads

Directions (11-15): Each of the questions below 13. Statements


consists of some statements followed by some Only a few Games are Good
conclusions. You have to decide which of the No Good is Mars
option correctly fill the blanks for (a), (b), and (c) All Good is Best
respectively, such that the conclusion logically Conclusions
follows. I. (a) Games can never be Mars
11. Statements II. (b) Best is not Mars
Only a few Baal is Baas III. (c) Games being Best is a possibility
No Baba is Baas a) Some; All; Some
Only a few Babe is (a) b) Some; Some; No
Conclusions c) All; Some; All
I. All Baas can be Babe d) Some; All; No

Click Here For Bundle PDF Course | support@guidely.in Page 3 of 10


SBI Clerk & RRB PO Mains PDF Course 2023
Reasoning Ability Day - 34 (Eng)

e) All; No; Some 16. In the given words, all the vowels are
changed to the next letter and all the consonants
14. Statements are changed to the second previous letter as per
Only a few Tides are Ride the English alphabetical series, then how many
Only (a) is Race words have repeated letters in it and what are
No Hide is (b) the letters?
Conclusions a) 2; B and J
I. A few (c) is not Hide b) 3; P, B and J
II. All Ride is being a Tide is a possibility c) 3; P and J
III. (d) Tide can never be Hide d) 1; P
a)Ride; Tide; Race; No e) 3; U, J and B
b) Tide; Ride; Ride; Some
c) Hide; Ride; Tide;All 17. If the first and last letters are interchanged,
d) Both a and c and the second and third letters are interchanged
e) Both b and c in each word, and the words are arranged in
dictionary order from right to left, then which of
15. Statements the following word remains unchanged in their
Only a few Dresses are Fiber position?
No Fault is Fiber a) CROWN
Some Beaches are not (a) b) TULIP
Conclusions c) CRAZE
I. Some (b) may be Fiber d) WORLD
II. Some Dresses are not (c) e) None
III. Some Beach being Fault is a possibility
a) Fiber; Fault; Fiber 18. If in the given words all the consonants are
b) Fault; Beach; Fault changed to the previous letter as per the English
c) Dresses; Beach; Beach alphabetical series, then how many words have
d) Fault; Dresses; Fiber more than two vowels?
e) None of these a) Two
b) One
Directions (16-20): Study the following c) Three
information and answer the given questions: d) Four
WORLD CRAZE CROWN TULIP e) None

Click Here For Bundle PDF Course | support@guidely.in Page 4 of 10


SBI Clerk & RRB PO Mains PDF Course 2023
Reasoning Ability Day - 34 (Eng)

19. If in the given series, the words are arranged 20. If all the letters in each word are changed to
in dictionary order from the left end, then what is the succeeding letter as per the English
the place value(as per the alphabetical series) of alphabetical series and the letters in each word
the last letter of the third word from the right are arranged in alphabetical order from the left
end? end, then how many words start with a
a) 14 vowel?(consider the succeeding letter of Z is A)
b) 13 a) None
c) 15 b) One
d) 19 c) Three
e) 23 d) Two
e) Four
Click Here to Get the Detailed Video Solution for the above given Questions
Or Scan the QR Code to Get the Detailed Video Solutions

Answer Key with Explanation

Directions (1-5):
1. Answer: E
2. Answer: C
3. Answer: D
4. Answer: B
5. Answer: A
Final arrangement

We have,

Click Here For Bundle PDF Course | support@guidely.in Page 5 of 10


SBI Clerk & RRB PO Mains PDF Course 2023
Reasoning Ability Day – 34 (Eng)

 Y was born on 12th of the month which  L was born in the month which has the
has an even number of days but after least number of days but on the same
August. numbered date as P.
 N was born three months before Y and 4th  The number of days between the dates
of the month. on which L and G were born is 25.
 K was born 22 days before N but one of From the above condition Case–1(a) & Case–
the prime-numbered dates. 2(a) are eliminated because of this statement, G
 Only two persons were born between K was born in the same month as Q but not in the
and Q but both were born on the same month, which has an even number of days.
numbered date.
From the above conditions, there are four
possibilities

Again we have,
 G was born in the same month as Q but
Again we have,
not in the month which has an even
 S was born on the same numbered date
number of days.
as K but was born in the month which has
 Three months gap between the born
an odd number of days.
months of G and P, who was born on 18th
 At least two months gap between the
of July.
months in which S and N were born.

Click Here For Bundle PDF Course | support@guidely.in Page 6 of 10


SBI Clerk & RRB PO Mains PDF Course 2023
Reasoning Ability Day – 34 (Eng)

From the above condition case-1 gets eliminated  I sits either second to the right or third to
because of this statement, At least two months the left of D.
gap between the months in which S and N were  As many persons sit to the right of I as to
born. Hence case 2 shows the final arrangement the left of the one who faces W.
From the above conditions, there are four
possibilities

Directions (6-10):
6. Answer: E
7. Answer: B
8. Answer: A
9. Answer: D
10. Answer: C
Final arrangement

We have, Again we have,


 D sits third from one of the extreme ends.  Only three persons sit between W and L.
 Only two persons sit between D and the  B faces the one who sits third to the right
one who faces X. of L.

Click Here For Bundle PDF Course | support@guidely.in Page 7 of 10


SBI Clerk & RRB PO Mains PDF Course 2023
Reasoning Ability Day – 34 (Eng)

 Only one person sits between B and the From the above condition Case 2 and Case 3
one who faces V. get eliminated because the one who faces R sits
 The number of persons sitting between V adjacent to T. Case 1 gets eliminated because A
and X is two less than the number of does not face the south. Hence, Case 4 shows
persons sitting between R and G. the final answer.

Again we have,
 The one who faces R sits adjacent to T.
 Not more than two persons sit between P
and the one who faces F. 11. Answer: D

 A sits neither adjacent to L nor faces


south.
 F and A are facing different directions.

Click Here For Bundle PDF Course | support@guidely.in Page 8 of 10


SBI Clerk & RRB PO Mains PDF Course 2023
Reasoning Ability Day – 34 (Eng)

c)

12. Answer: A

15. Answer: B

13. Answer: C

16. Answer: C
WORLD CRAZE CROWN TULIP
UPPJB APBXF APPUL RVJJN

14. Answer: E 17. Answer: E


b) WORLD CRAZE CROWN TULIP
DROLW EARZC NORWC PLUIT
PLUIT NORWC EARZC DROLW

Click Here For Bundle PDF Course | support@guidely.in Page 9 of 10


SBI Clerk & RRB PO Mains PDF Course 2023
Reasoning Ability Day – 34 (Eng)

18. Answer: B N=14


WORLD CRAZE CROWN TULIP
VOQKC BQAYE BQOVM SUKIO 20. Answer: D
WORLD CRAZE CROWN TULIP
19. Answer: A XPSME DSBAF DSPXO UVMJQ
WORLD CRAZE CROWN TULIP EMPSX ABDFS DOPSX JMQUV
CRAZE CROWN TULIP WORLD

Click Here For Bundle PDF Course | support@guidely.in Page 10 of 10


SBI Clerk & RRB PO Mains PDF Course 2023
Quantitative Aptitude Day - 34 (Eng)

Quantitative Aptitude

Direction (1-5): Study the following data carefully and answer the questions:
The data given below is related to the number of units sold by a shopkeeper of 5 different articles A, B, C,
D and E in a particular month.
The line graph given below shows the selling price per unit of articles B, C, D and E as per cent
more/less than that of article A.

The table given below shows the number of sold units of each of the given articles.

Note:
1: The difference between the selling price per unit of article B and that of article C is ₹ 225.

Click Here For Bundle PDF Course | support@guidely.in Page 1 of 17


SBI Clerk & RRB PO Mains PDF Course 2023
Quantitative Aptitude Day - 32 (Eng)

2: The ratio of the total amount received by the shopkeeper by selling the given units of article A to that
received by selling the given units of article C is 20: 27.
3: The total amount received by the shopkeeper by selling the given units of article A is ₹ 600 more than
that received by selling the given units of article B.
1) If the cost price per unit of article A is ₹ 600, Q: The average of the number of sold units of
the cost price per unit of article B is ₹ 800 and articles C, D and E are 9.
the cost price per unit of article C is ₹ 500, then R: If the cost price per unit of article D is ₹ 700,
find the average of the total profit received by then the profit per cent received by selling the
selling the given units of article A, that received given units of article D is 20%.
by selling the given units of article B and that a) All are true
received by selling the given units of article C? b) Only A and C
a) ₹ 1500 c) Only B
b) ₹ 1100 d) Only B and C
c) ₹ 1300 e) Only A
d) ₹ 1400
e) ₹ 1200 4) If the total profit received by selling the given
units of article C is ₹ 600, the total profit received
2) If the shopkeeper received 12.5% profit by by selling the given units of article E is ₹ 1200,
selling the given units of article B and received each unit of article C was marked up by 20% and
profit by selling the given units of article D, each unit of article E was marked up by 25%,
then find that the cost price per unit of article D is then find the ratio of discount per cent given on
what per cent more/less than that of article B? article C to that given on article E?
a) 12.5% a) 5: 2
b) 16.67% b) 10: 7
c) 5% c) 2: 1
d) 10% d) 4: 3
e) 9.09% e) 3: 1

3) Which of the following is/are true? 5) If the average number of sold units of articles
P: The ratio of the total amount received by D, E and F is 11 and the average selling price
selling the given units of article C to that received per unit of articles B, C and F is ₹ 675, then find
by selling the given units of article E is 9: 8. the average total amount received by selling the
given units of article A and that received by
selling the given units of article F?

Click Here For Bundle PDF Course | support@guidely.in Page 2 of 17


SBI Clerk & RRB PO Mains PDF Course 2023
Quantitative Aptitude Day - 32 (Eng)

a) ₹ 7150 and studying, is N% of the total number of


b) ₹ 7450 students in school B, find that the value of (3M –
c) ₹ 7250 1.5N) is divisible by which of the following?
d) ₹ 7350 a) 10
e) ₹ 7050 b) 15
c) 5
Direction (6-8): Study the following data carefully d) Both (b) and (c)
and answer the questions: e) Both (a) and (c)
The data given below is related to the number of
students in schools A and B who like playing and 7) The ratio of boys to girls in school A is 3: 2
studying. In both schools, some students like and 35% of the total girls in school A like only
only playing, some students like only studying playing. If the ratio of boys to girls in school B is
and others like both playing and studying. 3: 2 and 25% of total girls in school B like only
In school A, the ratio of the number of students, playing, then find out what percent of the total
who like only playing, to those, who like only girls in both the schools together like only
studying, is 6: 5. In school B, the ratio of the playing?
number of students, who like only playing, to a) 34.625%
those, who like only studying, is 3: 5. The b) 28.625%
number of students in school A, who like both c) 30.625%
playing and studying, is 50 more than those in d) 32.625%
school B and also 60 less than the number of e) 36.625%
students in school A, who like only playing. In
school B, number of students, who like only 8) Find the ratio of the total number of students
playing, is 35 more than those, who like both in both the schools together, who like playing to
playing and studying. The number of students in the total number of students in both the schools
school B, who like only studying, is 25 more than together, who like only studying?
those in school A. a) 19: 13
6) If the number of students in school A, who like b) 47: 33
both playing and studying, is M% of the total c) 5: 3
number of students in school A and the number d) 23: 17
of students in school B, who like both playing e) 7: 5

Direction (9-12): Study the following data carefully and answer the questions:

Click Here For Bundle PDF Course | support@guidely.in Page 3 of 17


SBI Clerk & RRB PO Mains PDF Course 2023
Quantitative Aptitude Day - 32 (Eng)

A company releases 5 different types of posts A, B, C, D and E and different number of candidates
applied for these posts.
The table given below shows the following data:

Note: The average number of non-selected candidates for each post is 30.
9) If the ratio of male to female, selected for post Q: The Value of (5M – N) is divisible by 4.
A, is 3: 2, the ratio of male to female, selected for R: The Ratio of M to N is 1: 3.
post B, is 5: 6 and the ratio of male to female a) Only P and R
selected for post C, is p: q, then the average of b) Only P and Q
the number of females selected for posts A, B c) All are true
and C will be 14. d) Only Q and R
Find that the value of (3p + 5q) is divisible by e) None is true
which of the following?
a) 3 11) If the average salary of the candidates
b) 5 selected for the post B, is ₹ 30000, that of the
c) 7 candidates selected for the post C, is ₹ 25000
d) Both (a) and (b) and that of the candidates selected for the post
e) Both (a) and (c) D, is ₹ 40000, then find the average salary of
each candidate selected for the posts B, C and
10) If the average of the number of candidates D?
selected for the posts C, D and E is M and the a) ₹ 23166.67
average of the number of candidates not b) ₹ 31166.67
selected for the posts A, B and C is N, then find c) ₹ 25166.67
out which of the following is/are true? d) ₹ 29166.67
P: Both M and N are multiples of 7. e) ₹ 27166.67

Click Here For Bundle PDF Course | support@guidely.in Page 4 of 17


SBI Clerk & RRB PO Mains PDF Course 2023
Quantitative Aptitude Day - 32 (Eng)

12) If the ratio of male to female selected for the of the total number of candidates selected for all
post A, is 2: 1, the ratio of male to female the posts together, are female?
selected for the post B, is 20: 13, the ratio of a) 36.67%
male to female selected for the post C, 2: 1, the b) 31.11%
ratio of male to female selected for the post D, is c) 33.33%
5: 4 and the ratio of male to female selected for d) 39.09%
the post E, is 2: 1, then find out what percentage e) 35.5%

Direction (13-15): Study the following data carefully and answer the questions:
Four street vendors A, B, C and D have different quantities (in kg) of mangoes at the start of a particular
day and each of them sold different quantities (in kg) of mangoes by the end of the day.
The pie chart given below shows the percentage distribution of the quantities of mangoes with each
street vendor at the start of the day.

The bar graph given below shows the quantity of mangoes sold by the end of the day as a percent of the
total quantity of mangoes with each street vendor.

Click Here For Bundle PDF Course | support@guidely.in Page 5 of 17


SBI Clerk & RRB PO Mains PDF Course 2023
Quantitative Aptitude Day - 32 (Eng)

Note: The Difference between the quantity of unsold mangoes in C and that of D is 20 kg.
13) A and B bought mangoes for a total of c) ₹ 745
₹ 2240 and ₹ 1200 respectively. If A and B sold d) ₹ 715
mangoes at 12.5% profit and 13% profit e) ₹ 755
respectively, then find the ratio of the total
amount collected by A to that collected by B by 15) Two columns X and Y are given below. Find
the end of the day? the correct combination of column X to column Y:
a) 189: 113
b) 189: 127
c) 315: 113
d) 315: 127
e) None of these

14) If A, B, C and D sold the remaining quantities


a) M-T, N-Q, O-S, P-R
of mangoes on the next day at ₹ 40 per kg, ₹ 50
b) M-S, N-R, O-T, P-Q
per kg, ₹ 45 per kg and ₹ 55 per kg, then find
c) M-Q, N-S, O-T, P-R
the average of amount collected by each of them
d) M-S, N-Q, O-R, P-T
by selling the remaining quantities of mangoes?
e) M-S, N-Q, O-T, P-R
a) ₹ 735
b) ₹ 725

Click Here For Bundle PDF Course | support@guidely.in Page 6 of 17


SBI Clerk & RRB PO Mains PDF Course 2023
Quantitative Aptitude Day - 32 (Eng)

Direction (16-20): Study the following data carefully and answer the questions:
The graph given below shows the average number of runs scored by 5 batsmen in their first 40 innings
and in their first 50 innings.
The data in the horizontal axis shows the average runs scored in the first 40 innings while the data in the
vertical axis shows the average runs scored in the first 50 innings.
Note: The first 50 innings also included the first 40 innings. Also, a batsman can play more innings than
50 in his career.

Note:
1. The Total runs scored by B in his last 10 innings are 160.
2. Total runs scored by A in his last 10 innings x2 + y2 + 80.
3. The Values of ‘x’ and ‘y’ are integers.
16) If the average runs scored by batsman B the total runs scored by B in his last
after ‘P’ innings becomes ‘x + y + 6’ and after ‘P innings. Assume P > 50.
+ 2’ innings it becomes ‘4x – y + 3’. If he scores a a) 420
total of 122 runs in his last 2 innings, then find b) 450
c) 480

Click Here For Bundle PDF Course | support@guidely.in Page 7 of 17


SBI Clerk & RRB PO Mains PDF Course 2023
Quantitative Aptitude Day - 32 (Eng)

d) 440 19) Match the following.


e) 400

17) The table given below shows the distribution


of the number of innings played by batsman D
against five different opponents P, Q, R, S, and T
out of his first 40 innings. It also shows the
average number of runs per innings scored
a) P – A, Q – B, R – C
against each opponent.
b) P – C, Q – A, R – B
c) P – C, Q – B, R – A
d) P – B, Q – A, R – C
e) P – A, Q – C, R – B

20) Which of the following statement(s) can be


determined?
Find the value of √(x2 + y2 + a2). I: Out of his last 10 innings, if batsman B scored
a) 25 an average of 16 runs when batting first, the
b) 15√5 average number of runs scored by him while
c) 20√2 batting second.
d) 10√10 II: Average runs scored by batsman D in his first
e) 20 20 innings.
III: If batsman D played ‘n’ more innings in his
18) In his last 10 innings, batsman C scored runs career after 50 innings and he scored an
by hitting 1’s, 2’s, 3’s, 4’s, and 6’s only. If he runs average of 20 runs in his last ‘n’ innings. The
1’s ‘m + 25’ times, 2’s ‘m’ times, 3’s ‘m – 7’ times, average number of runs scored by him in his
4’s ‘m – 4’ times, and 6’s ‘m – 6’ times. Find the career.
total number of factors of ‘m + 25’ (Include ‘1’ as a) Only I and III
a factor). b) Only III
a) 6 c) Only II and III
b) 2 d) All I, II, and III
c) 5 e) Only II
d) 3
e) 4

Click Here For Bundle PDF Course | support@guidely.in Page 8 of 17


SBI Clerk & RRB PO Mains PDF Course 2023
Quantitative Aptitude Day - 34 (Eng)

Click Here to Get the Detailed Video Solution for the above given Questions
Or Scan the QR Code to Get the Detailed Video Solutions

Answer Key with Explanation

Direction (1-5): Since, the total amount received by the


Since, the difference between the selling price shopkeeper by selling the given units of article A
per unit of article B and that of article C is ₹ 225. is ₹ 600 more than that received by selling the
So, the selling price per unit of article A = given units of article B.
= ₹ 750 So,
The selling price per unit of article B = 120% of [750 * x] – [900 * (x – 2)] = 600
750 = ₹ 900 750x – 900x + 1800 = 600
The selling price per unit of article C = 90% of x=8
750 = ₹ 675 From equation (1):
The selling price per unit of article D = 112% of y=4
750 = ₹ 840
And the selling price per unit of article E = 96%
of 750 = ₹ 720
Since, the ratio of total amount received by the
shopkeeper by selling the given units of article A
to that received by selling the given units of
article C, is 20: 27.
1) Answer: C
So,
Since, the CP per unit of article A = ₹ 600
And the SP per unit of article A = ₹ 750
3x = 2x + 2y So, the total profit received by selling the given
x = 2y ----------- (1) units of article A:

Click Here For Bundle PDF Course | support@guidely.in Page 9 of 17


SBI Clerk & RRB PO Mains PDF Course 2023
Quantitative Aptitude Day - 34 (Eng)

(750 – 600) * 8 = ₹ 1200 Average of the number of sold units of articles C,


Since, the CP per unit of article B = ₹ 800 D and E:
And the SP per unit of article B = ₹ 900
So, the total profit received by selling the given So, Q is true.
units of article B: From R:
(900 – 800) * 6 = ₹ 600 Since, the CP per unit of article D = ₹ 700
Since, the CP per unit of article C = ₹ 500 And the SP per unit of article D = = ₹ 840
And the SP per unit of article C = ₹ 675
Required percentage = = 20%
So, the total profit received by selling the given
So, R is true.
units of article C:
Hence, all are true.
(675 – 500) * 12 = ₹ 2100
Required average = = ₹ 1300
4) Answer: A
2) Answer: D
Total SP of the sold units of article C = ₹ 8100
Since, the total SP of 6 units of article B =
Total CP of the sold units of article C = 8100 –
₹ 5400
600 = ₹ 7500
And the total CP of 6 units of article B =
Total MP of the sold units of article C = 120% of
= ₹ 4800
7500 = ₹ 9000
So, the CP per unit of article B = = ₹ 800 Discount per cent on article C = =
Since, the total SP of 5 units of article D = 10%
₹ 4200 Total SP of the sold units of article E = ₹ 7200
And the total CP of 5 units of article D = Total CP of the sold units of article E = 7200 –
= ₹ 3600 1200 = ₹ 6000
So, the CP per unit of article D = = ₹ 720 Total MP of the sold units of article E = 125% of
Required percentage = = 10% 6000 = ₹ 7500
Discount per cent on article E = =
3) Answer: A 4%
From P: Required ratio = 10: 4 = 5: 2
Ratio of total amount received by selling the
given units of article C to that received by selling 5) Answer: E
the given units of article E = 8100: 7200 = 9: 8 Since, the average number of sold units of
So, P is true. articles D, E and F is 11
From Q: So, the number of sold units of article F = (3 *
11) – (5 + 10) = 18

Click Here For Bundle PDF Course | support@guidely.in Page 10 of 17


SBI Clerk & RRB PO Mains PDF Course 2023
Quantitative Aptitude Day - 34 (Eng)

Since, the average SP per unit of articles B, C Since, the number of students in school B, who
and F is ₹ 675. like only studying, is 25 more than those in
So, the SP per units of article F = (3 * 675) – school A.
(900 + 675) = ₹ 450 5y – 5x = 25
Since, the total amount received by selling the y – x = 5 -------------- (3)
given units of article F: From equations (1) and (2):
450 * 18 = ₹ 8100 6x – 110 = 3y – 35
And the total amount received by selling the 6x – 3y = 75
given units of article A = ₹ 6000 2x – y = 25 -------------- (4)
Required average = = ₹ 7050 From equations (3) and (4):
5 + x = 2x – 25
Direction (6-8): x = 30
Let the number of students in school A, who like From equation (3):
only playing and those, who like only studying, y = 35
are ‘6x’ and ‘5x’ respectively. From equation (1):
Let the number of students in school B, who like z = 70
only playing and those, who like only studying, Number of students in school A, who like only
are ‘3y’ and ‘5y’ respectively. playing = 6 * 30 = 180
Let the number of students in school B, who like Number of students in school A, who like only
both playing and studying, is ‘z’. studying = 5 * 30 = 150
So, the number of students in school A, who like Number of students in school A, who like both
both playing and studying = (z + 50) playing and studying = 70 + 50 = 120
Since, in school A, the number of students, who Number of students in school B, who like only
like both playing and studying, is 60 less than playing = 3 * 35 = 105
those, who like only playing. Number of students in school B, who like only
So, studying = 5 * 35 = 175
6x – z – 50 = 60 Number of students in school B, who like both
6x – z = 110 -------------- (1) playing and studying = 70
Since, in school B, number of students, who like
only playing, is 35 more than those, who like
both playing and studying.
3y – z = 35 -------------- (2)

Click Here For Bundle PDF Course | support@guidely.in Page 11 of 17


SBI Clerk & RRB PO Mains PDF Course 2023
Quantitative Aptitude Day - 34 (Eng)

6) Answer: E Total number of students in both the schools


The number of students in school A, who like together, who like only studying:
both playing and studying = 120 150 + 175 = 325
The total number of students in school A = 180 + Required ratio = 475: 325 = 19: 13
120 + 150 = 450
So, M = Direction (9-12):
The number of students in school B, who like Let the number of candidates applied for posts
both playing and studying = 70 A, B, C, D and E are 20x, 30x, 35x, 10x and 5x
The total number of students in school B = 105 + respectively.
70 + 175 = 350 So, the number of non-selected candidates for
So, N = = 20% post A =
Now, the value of (3M – 1.5N) = (80 – 30) = 50 The number of non-selected candidates for post
And the value of (3M – 1.5N) is divisible by both B=
5 and 10. The number of non-selected candidates for post
C=
7) Answer: C The number of non-selected candidates for post
Total number of students in school A = 180 + D=
120 + 150 = 450 And the number of non-selected candidates for
The number of girls in school A = = 180 post E =
And the number of girls in school A, who like Since, the average of the number of non-
only playing = 35% of 180 = 63 selected candidates for each post is 30.
Total number of students in school B = 105 + 70 So,
+ 175 = 350
The number of girls in school B = = 140
And the number of girls in school B, who like
only playing = 25% of 140 = 35 250x = 600
Required percentage = = 30.625% x = 2.4

8) Answer: A
Total number of students in both the schools
together, who like playing:
(180 + 120) + (105 + 70) = 475

Click Here For Bundle PDF Course | support@guidely.in Page 12 of 17


SBI Clerk & RRB PO Mains PDF Course 2023
Quantitative Aptitude Day - 34 (Eng)

Average of the number of candidates not


selected for the posts A, B and C:

From P:
Both M and N are multiple of 7.
So, P is true.
9) Answer: E From Q:
Since, the total number of candidates selected Value of (5M – N) = 70 – 42 = 28
for post A = 15 Since, the value of (5M – N) is divisible by 4.
So, the number of females selected for post A = So, Q is true.
=6 From R:

Since, the total number of candidates selected Ratio of M to N = 14: 42 = 1: 3

for post B = 33 So, R is true.

So, the number of females selected for post B = Hence, all are true.

= 18
Since, the total number of females selected for 11) Answer: D

posts A, B and C = 3 * 14 = 42 Total salary of the candidates selected for the

So, the number of females selected for post C = post B = 33 * 30000 = ₹ 990000

42 – (6 + 18) = 18 Total salary of the candidates selected for the

Since, the total number of candidates selected post C = 30 * 25000 = ₹ 750000

for post C = 30 Total salary of the candidates selected for the

So, the number of males selected for post C = post D = 9 * 40000 = ₹ 360000

30 – 18 = 12 Required average = =

And, p: q = 12: 18 = 2: 3 ₹ 29166.67

Now, the value of (3p + 5q) = 6 + 15 = 21


So, the value of (3p + 5q) is divisible by both 3 12) Answer: A

and 7, but not divisible by 5. Number of females selected for the post A =
=5

10) Answer: C Number of females selected for the post B =


Average of the number of candidates selected = 13
for the posts C, D and E: Number of females selected for the post C =
= 10

Click Here For Bundle PDF Course | support@guidely.in Page 13 of 17


SBI Clerk & RRB PO Mains PDF Course 2023
Quantitative Aptitude Day - 34 (Eng)

Number of females selected for the post D = Since, the price of mangoes, at which A bought
=4 them = = ₹ 40 per kg
Number of females selected for the post E = So, the price of mangoes, at which A sold them
=1 = 112.5% of 40 = ₹ 45 per kg
Total number of females selected for all the And the total amount collected by A by the end
posts together: of the day = 42 * 45 = ₹ 1890
5 + 13 + 10 + 4 + 1 = 33 Since, the price of mangoes, at which B bought
Total number of candidates selected for all the them = = ₹ 50 per kg
posts together: So, the price of mangoes, at which B sold them
15 + 33 + 30 + 9 + 3 = 90 = 113% of 50 = ₹ 56.5 per kg
Required percentage = = 36.67% And the total amount collected by B by the end
of the day = 12 * 56.5 = ₹ 678
Direction (13-15): Required ratio = 315: 113
Let the total quantity of mangoes with A, B, C
and D are 35x kg, 15x kg, 20x kg and 30x kg 14) Answer: B
respectively. The amount collected by A by selling the
So, the quantity of unsold mangoes of C = 20% remaining quantities of mangoes:
of 20x = 4x kg 14 * 40 = ₹ 560
And the quantity of unsold mangoes of D = 55% The amount collected by B by selling the
of 30x= 16.5x kg remaining quantities of mangoes:
Since, the difference between the quantity of 12 * 50 = ₹ 600
unsold mangoes of C and that of D is 20 kg. The amount collected by C by selling the
So, remaining quantities of mangoes:
16.5x – 4x = 20 6.4 * 45 = ₹ 288
x = 1.6 The amount collected by D by selling the
remaining quantities of mangoes:
26.4 * 55 = ₹ 1452
Required average = = ₹ 725

15) Answer: E
Average of the quantities of mangoes with B and
C at the start of the day:

13) Answer: C

Click Here For Bundle PDF Course | support@guidely.in Page 14 of 17


SBI Clerk & RRB PO Mains PDF Course 2023
Quantitative Aptitude Day - 34 (Eng)

Average of the sold quantities of mangoes of C 50 * (y + 16) – 40 * (x + 16) = x2 + y2 + 80


and D: 50y – 40x + 160 = x2 + y2 + 80
From equation (1) and (2):

Difference between the sold and unsold 320 + 160 = x2 + [(4x + 32)/5]2 + 80

quantities of mangoes of C: 400 = x2 + [(4x + 32)/5]2

25.6 – 6.4 = 19.2 kg 41x2 + 256x – 8976 = 0

Difference between sold quantities of mangoes 41x2 – 492x + 748x – 8976 = 0

of A and B: 41x (x – 12) + 748 (x – 12) = 0

42 – 12 = 30 kg (x – 12) (41x + 748) = 0

So, the correct combination: x = 12 and -748/41

M-S, N-Q, O-T, P-R Value of ‘x’ = 12


y = (4x + 32)/5 = 16

Direction (16-20): Now the table will look like:

The table given below shows the average runs


scored by five batsmen after 40 innings and after
50 innings.

16) Answer: B
Total runs scored in ‘P + 2’ innings = (4x – y + 3)
* (P + 2) = 35 (P + 2)
Total runs scored by B in his last 10 innings = Total runs scored in ‘P’ innings = (x + y + 6) * P
160 = 34P
50 * (y + 16) – 40 * (x + 24) = 160 Since value of P is more than 50. That means
50y + 800 – 40x – 960 = 160 the last 2 innings of the batsman will be the 2
50y – 40x = 320 ………. (1) innings that he played after the Pth match?
5y – 4x = 32 According to the question:
5y = 4x + 32 35 (P + 2) – 34P = 122
y = (4x + 32)/5 ……. (2) 35P + 70 – 34P = 122
Total runs scored by A in his last 10 innings = x2 P = 52
+ y2 + 80

Click Here For Bundle PDF Course | support@guidely.in Page 15 of 17


SBI Clerk & RRB PO Mains PDF Course 2023
Quantitative Aptitude Day - 34 (Eng)

Which means according to this question 1 * (m + 25) + 2 * m + 3 * (m – 7) + 4 * (m – 4) +


batsman played total of P + 2 = 54 innings. 6 * (m – 6) = 1200 – 1120
Hence, his last = 14 m + 25 + 2m + 3m – 21 + 4m – 16 + 6m – 36 =
innings will be from 41st to 54th innings. 80
Total runs scored till 54th innings =54 * (4x – y + 16m = 128
3) = 54 * 35 = 1890 m=8
Total runs scored till 40th innings = 40 * 36 = Now,
1440 m + 25 = 8 + 25 = 33 = 1 * 3 * 11
Total runs scored in his last 14 innings = 1890 – Total factors of 33 = 1, 3, 11, 33
1440 = 450 Hence, total factors of 33 = 4
Hence, total runs scored in his last 14 innings =
450 19) Answer: D
Total runs scored by A in his last 10 innings = 32
17) Answer: A * 50 – 28 * 40 = 1600 – 1120 = 480
Total runs scored by D in first 40 innings = 16 * Average = 480/10 = 48 [Which is ‘1’ less than
40 the perfect square of an integer.]
5 * (a + 10) + 8 * (a – 5) + 13 * a + 4 * (a + 5) + Total runs scored by B in his last 10 innings = 32
10 * (a + 1) = 640 * 50 – 36 * 40 = 1600 – 1440 = 160
5a + 50 + 8a – 40 + 13a + 4a + 20 + 10a + 10 = Average = 160/10 = 16 [Which is a perfect
640 square.]
40a + 40 = 640 Total runs scored by C in his last 10 innings = 40
40a = 600 * 50 – 48 * 40 = 2000 – 1920 = 80
a = 15 Average = 80/10 = 8 [Which is a perfect cube.]
Now, Hence, correct combination is P – B, Q – A, R –
√(x2 + y2 + a2) C.
= √ (122 + 162 + 152)
= √ (144 + 256 + 225) 20) Answer: B
= √625 Statement I:
= 25 It is not given that, in how many innings the
batsman B batted first and batted last. So, we
18) Answer: E cannot determine the average runs scored by
Total runs scored by C in his last 10 innings = 24 him while batting second out of his last 10
* 50 – 28 * 40 innings.

Click Here For Bundle PDF Course | support@guidely.in Page 16 of 17


SBI Clerk & RRB PO Mains PDF Course 2023
Quantitative Aptitude Day - 34 (Eng)

So, statement I cannot be determined. Total runs scored by D in his last ‘n’ innings = n *
Statement II: 20 = 20n
We are given information only about the average Total runs scored by D in his career = 1000 +
runs scored by D in first 40 innings, but we are 20n
not given any information about his first 20 Total innings played by D in his career = 50 + n
innings. So, we cannot determine the average Average runs scored by D in his career = (1000
runs scored by D in his first 20 innings. + 20n)/ (50 + n) = [20 (50 + n)]/ (50 + n) = 20
So, statement II cannot be determined. Hence, average runs scored by D in his career =
Statement III: 20
Total runs scored by D in his first 50 innings = 50 So, statement III can be determined.
* 20 = 1000 Hence, only statement III can be determined

Click Here For Bundle PDF Course | support@guidely.in Page 17 of 17


SBI Clerk & RRB PO Mains PDF Course 2023
ENGLISH Day - 34

English Language

Directions (1-5) : In the questions given below, (A) The roses in the garden bloomed with
four statements are given labelled as (A), (B), (C) radiant colours, filling the air with their
and (D), with one statement being grammatically enchanting fragrance, creating a picturesque
and contextually incorrect. You are required to sight that brought joy to anyone who beheld
choose that sentence which is erroneous. If all them.
the sentences are correct then choose ‘option e’ (B) I would prefer to like to have a latte,
as your answer. especially on chilly mornings, as its creamy
1) SPOT THE ERRONEOUS SENTENCE texture and rich espresso blend offer a
(A) The army personnel marched in disciplined comforting start to my day.
formation, his resolute expressions reflecting (C) My parents wished me the best of luck as I
their unwavering commitment to defending their embarked on a new journey, encouraging me to
nation. pursue my dreams and reminding me that they'll
(B) The family returned from the US with always be there to support me.
suitcases filled with cherished memories, (D) His younger brother is an architect, and his
souvenirs from their unforgettable journey. innovative designs and attention to detail have
(C) The colour of the building was a striking earned him accolades in the field of architecture.
blend of vibrant red and elegant gold, standing a) A
out majestically against the surrounding b) B
landscape. c) C
(D) The project was an ambitious undertaking d) D
that required meticulous planning, dedicated e) All correct
teamwork, and a creative approach to overcome
the challenges and achieve its remarkable 3) SPOT THE ERRONEOUS SENTENCE
success. (A) Doctors are treated as true lifesavers, as
a) A their expertise, compassion, and dedication play
b) B a crucial role in healing the sick and easing the
c) C suffering of those in need.
d) D (B) My tooth is aching, and I already planning to
e) All correct schedule a dental appointment to get it examined
and treated before the pain worsens.
2) SPOT THE ERRONEOUS SENTENCE

Click Here For Bundle PDF Course | support@guidely.in Page 1 of 12


SBI Clerk & RRB PO Mains PDF Course 2023
ENGLISH Day - 34

(C) She sells reusable straws, promoting 5) SPOT THE ERRONEOUS SENTENCE
environmental consciousness and reducing (A) Mobile phones are very addictive, with their
plastic waste, one step at a time. constant notifications, social media apps, and
(D) The train was late due to unforeseen gaming options captivating our attention and
technical issues, causing inconvenience to the making it hard to put them down.
passengers who had planned their journey (B) Painkillers are essential for managing
meticulously. discomfort and alleviating pain, providing relief to
a) A individuals dealing with various health conditions
b) B or recovering from injuries.
c) C (C) The country was immersed in a celebration of
d) D unity and patriotism as people from all walks of
e) All correct life came together to mark the national day.
(D) We bought a new car, and its sleek design,
4) SPOT THE ERRONEOUS SENTENCE advanced features, and smooth performance has
(A) The driver was made to pay a fine for already made it a delightful addition to our family.
violating traffic rules, emphasizing the a) A
importance of following regulations to ensure b) B
road safety. c) C
(B) The party went well, with laughter, music, and d) D
good company making it an unforgettable e) All correct
evening filled with joyous memories.
(C) The results of the board exams were eagerly Directions (6-10) : In each of these questions, a
awaited by students and parents alike, as they paragraph is given that has a blank in it. Out of
determined the next steps in their academic and the given options, only one sentence fits in with
career paths. the context of the paragraph. Select that as your
(D) Fashion technology is something that blends answer.
creativity and innovation, allowing designers to 6) Deforestation, an alarming ecological crisis,
push boundaries and redefine the way we refers to the widespread clearance and removal
perceive and interact with clothing. of forests, leading to their transformation into
a) A non-forest areas. This insidious practice has
b) B been occurring for centuries, but its intensity has
c) C escalated dramatically in recent times, causing
d) D severe environmental repercussions on a global
e) All correct scale._ ________________.They play a vital role

Click Here For Bundle PDF Course | support@guidely.in Page 2 of 12


SBI Clerk & RRB PO Mains PDF Course 2023
ENGLISH Day - 34

in balancing Earth's ecosystems, regulating the b) Cuisine refers to a specific style or method of
climate, and supporting a myriad of plant and cooking, as well as the characteristic dishes and
animal species. food traditions associated with a particular
a) Forests are lush and biodiverse ecosystems, region, culture, or country.
covering vast stretches of land with a dense c) The Indian Subcontinent, also known as South
canopy of trees. Asia, is a region located in the southern part of
b) Forests are often referred to as the lungs of Asia.
our planet, and with good reason. d) India's geographical diversity has bestowed
c) There are several types of forests around the upon it a remarkable array of spices, each
world, each characterized by their unique originating from different regions and climates.
ecological features and geographical locations. e) In the case of spices, many regions around
d) Uncontrolled wildfires can spread rapidly, the world have been granted GI tags to protect
destroying vast areas of forest and wildlife the uniqueness and distinctiveness of their spice
habitats. produce.
e) Forest officials, also known as forestry officials
or forest rangers, are government employees 8) ____________. This compassionate gesture
responsible for the management and protection has the power to transform the lives of those
of forests. suffering from organ failure or chronic illnesses,
offering them a second chance at life. The
7) The allure of Indian cuisine lies not only in its donation of vital organs such as the heart, lungs,
vibrant colours and enticing aromas but also in liver, kidneys, and pancreas, as well as tissues
the exquisite blend of spices that dance upon the like corneas and skin, can significantly improve
taste buds. For millennia, Indian spices have the quality of life for recipients and even be the
been an integral part of the country's culinary difference between life and death.
landscape, elevating dishes to unparalleled a) Organ failure is a medical emergency that
heights and reflecting the rich cultural tapestry of requires immediate intervention and critical care.
the subcontinent. _________From the tropical b) Blood banks are specialized facilities that
coastline of Kerala to the arid deserts of collect, process, store, and distribute blood and
Rajasthan, every corner of the country harbours blood products for medical use.
its unique spice treasures. c) The human body is a complex organism made
a) Spices add a burst of flavour and aroma to up of several different organs, each with specific
culinary creations, making dishes from various functions and roles to maintain overall health and
cuisines around the world truly delightful. homeostasis.

Click Here For Bundle PDF Course | support@guidely.in Page 3 of 12


SBI Clerk & RRB PO Mains PDF Course 2023
ENGLISH Day - 34

d) Food donation is a significant step in reducing prepare students for a career in the healthcare
food wastage and addressing hunger and industry.
malnutrition in various parts of the world. e) It assesses cognitive abilities, situational
e) Organ donation is a noble act that involves judgement, and other relevant qualities for
the selfless giving of one's organs or tissues to healthcare professionals.
save the lives of others in need.
10) Dark chocolates, with their alluring richness
9) Medical entrance exams are a critical and distinct bittersweet flavour, are a delectable
milestone for aspiring healthcare professionals, treat beloved by many. Made from cocoa beans
marking the beginning of their journey into the with a higher cocoa solids content and less
world of medicine._______________.Students added sugar, dark chocolates stand apart from
who dream of becoming doctors, dentists, or their milk chocolate counterparts in terms of both
other medical specialists must prepare diligently taste and nutritional benefits. The intense cocoa
and demonstrate exceptional skills in subjects flavour, accompanied by subtle hints of fruitiness
like biology, chemistry, physics, and aptitude or earthiness, delights the senses with each
tests. The stakes are high, as the results velvety bite. _________________.
determine whether they can pursue their passion a) While imported chocolates can be a delightful
for medicine and contribute to the well-being of treat for many, it's essential to consume them in
society. moderation, as they are still high in sugar and
a) These highly competitive and rigorous calories.
examinations serve as gateways to some of the b) "Velvet" refers to a type of luxurious fabric
most prestigious medical colleges and known for its soft and smooth texture.
universities c) Beyond their delightful taste, dark chocolates
b) These centers are established to provide a boast a host of health benefits.
controlled and secure environment for d) Chocolates are enjoyed by people of all ages
conducting exams, ensuring fairness and and are a beloved indulgence around the world.
integrity in the assessment process. e) Dark chocolate contains compounds like
c) Generic medicines are identical or theobromine and phenylethylamine, which can
bioequivalent versions of brand-name stimulate the release of endorphins and
medications. serotonin in the brain.
d) These colleges offer comprehensive programs
in medicine, including theoretical knowledge, Directions (11-15) : A sentence is given below
practical training, and clinical experience, to with two words highlighted in bold. Select the
option that gives the correct set of words as a

Click Here For Bundle PDF Course | support@guidely.in Page 4 of 12


SBI Clerk & RRB PO Mains PDF Course 2023
ENGLISH Day - 34

replacement to the bold words. In case the but also demanding responsible usage to
sentence is correct, select ‘No improvement produce a healthy balance.
required’. a) separable, strike
11) Rohan was so talented that his skills in b) indivisible, making
playing multiple musical instruments left c) inseparable, strike
everyone in awe, and he effortlessly charmed the d) integral, producing
audience with his performances. e) No improvement required
a) shock, impressed
b) amusement, disappointed 15) Latha's innocence was known in her wide-
c) happy, cheered eyed curiosity and pure heart, radiating a charm
d) tears, attracted that instantly bared her to everyone she met.
e) No improvement required a) evident, endeared
b) well-known, impressed
12) The orchestra played a symphony that c) clear, displease
tagged at the heartstrings, filling the concert hall d) apparent, belittle
with a boring and harmonious melody. e) No improvement required
a) pushed, repellent
b) tugged, captivating Directions (16-20) : In each of the questions
c) idled, irksome given below a phrase in the sentence has been
d) dragging, pleased highlighted. It may or may not need replacement.
e) No improvement required Read the question carefully and choose an
option that would best replace the highlighted
13) The show had a huge fan base, as its forcing parts to make the sentences grammatically and
storyline, relatable characters, and unexpected contextually correct. In case, the given sentence
plot twists kept viewers unwanted awaiting each is correct in its current form and there is no
new episode. improvement needed, mark (e) as your answer.
a) innovation, impatiently 16) His voice was smooth as silk, with a soothing
b) unique, long tone that captivated listeners and had a calmed
c) dull, readily effect of anyone who heard him speak or sing.
d) compelling, eagerly a) have a calming effect on anyone who heard
e) No improvement required him speak or sing
14) Gadgets are becoming an intact part of our b) had a calming effects about anyone who
daily lives, providing convenience and efficiency heard him speak or sing

Click Here For Bundle PDF Course | support@guidely.in Page 5 of 12


SBI Clerk & RRB PO Mains PDF Course 2023
ENGLISH Day - 34

c) had a calming effect on anyone who heard him d) and serene lakes whom attracted tourism from
speak or sing all over the country
d) had a calming effect on anyone whom heard e) No improvement required
him speak or sing
e) No improvement required 19) The dispute was mainly centered around the
ownership of the land, leading to heated
17) To grow long hair, one needs patients and a arguments and legal battles between the parties
consists hair care routine, including regular trims, involved.
proper nourishment, and gentle handling to avoid a) the disputes were mainly centric around the
breakage. ownership of the land
a) one need patience and a consistent hair care b) their dispute was mainly centering around the
routine ownership of the land
b) one needs patiently and consistently hair care c) there dispute are mainly centered about the
routine ownership of the land
c) one needs patient and a consistent hair care d) the dispute were mainly centered around the
routine ownership on the land
d) one needs patience and a consistent hair care e) No improvement required
routine
e) No improvement required 20) Admissions are open for the new academic
year, invite eager students to enrolling in various
18) The state had a diverse landscape, boasting courses and embark on a journey of knowledge
lush green valleys, majestic mountains, and and growth.
serene lakes who attracted tourists from all a) inviting eager students to enrolling in various
around over the country. courses and embark on a
a) and serene lakes that attracted tourists from b) inviting eager students to enrolled in various
all over the country courses and embark on a
b) and serene lake those attracted tourists from c) invites eager students to enrol in various
all over the country courses and embarked on a
c) and serene lakes that attracting tourists from d) inviting eager students to enrol in various
all around the country courses and embark on a
e) No improvement required

Click Here For Bundle PDF Course | support@guidely.in Page 6 of 12


SBI Clerk & RRB PO Mains PDF Course 2023
ENGLISH Day - 34

Click Here to Get the Detailed Video Solution for the above given Questions
Or Scan the QR Code to Get the Detailed Video Solutions

Answer Key with Explanation

1) Answer: A person so a simple phrase ‘ I like to’ will convey


Sentence ‘A’ has an error and hence it is the meaning.
erroneous. Remove ‘would prefer to’ from the given
Personnel - the people who work for a large sentence to make it correct and meaningful.
organization or one of the armed forces The corrected sentence : I like to have a latte,
Replace ‘his’ with ‘their’ to make the sentence especially on chilly mornings, as its creamy
correct and meaningful. texture and rich espresso blend offer a
His represents an individual and it is comforting start to my day.
inappropriate in the given sentence as the 3) Answer: B
sentence talks about the army personnel and the Sentence ‘B’ is incorrect and erroneous.
correct pronoun to be used is ‘their’. ‘I already planning’ - is incorrect because
The corrected sentence : The army personnel ‘already’ is in the past and it is not correct to say
marched in disciplined formation, their resolute ‘already planning’. Simply ‘I plan’ will make the
expressions reflecting their unwavering sentence correct and error free.
commitment to defending their nation. The corrected sentence: My tooth is aching, and
I plan to schedule a dental appointment to get it
2) Answer: B examined and treated before the pain worsens.
The sentence ‘B’ is grammatically incorrect.
‘Prefer to like to’ is wrong because it is 4) Answer: E
redundant, ‘would’ is also unnecessary because All the given four sentences are grammatically
it is a general statement denoting the liking of a correct and meaningful, hence we go for option
(e) as our answer.

Click Here For Bundle PDF Course | support@guidely.in Page 7 of 12


SBI Clerk & RRB PO Mains PDF Course 2023
ENGLISH Day - 34

5) Answer: D sentence of ‘d’ also seems to be a continuation


The sentence ‘D’ is incorrect and hence option and hence (d) would fit the blank aptly.
(d) is the right answer for this question. Option a - it is so generic and hence does not fit
‘Has’ is inappropriate here because “sleek the passage, the passage is specific to India
design, advanced features, and smooth Option b - talks about cuisines which is not
performance” must be represented by ‘have’ and relevant to the passage, the passage deals with
not ‘has’ ‘spices’
Have for plural and has for singular Option c - details about Indian subcontinent is
The corrected sentence : We bought a new car, irrelevant
and the sleek design, advanced features, and Option e - talking about GI tags in the middle of
smooth performance have already made it a the passage is inappropriate and unnecessary
delightful addition to our family. Hence d is the correct sentence that completes
the passage with the same context and
6) Answer: B meaning.
The statement ‘b’ fits the paragraph correctly as
it matches the context. 8) Answer: E
The paragraph talks about deforestation and the Sentence in ‘e’ is the correct one that fits the
importance of forest. So, option (b) would best fit blank in the passage to complete it. Organ
the given blank in the paragraph. donation is a noble act and the second sentence
Option a - this statement is simply the definition continues to use ‘this compassionate act’
of forest Option a - the passage is about organ donation
Option c - talks about the types of forest not organ failure, also this sentence does not
Option d - is about the disaster caused by add meaning to the passage and its context
wildfire and inappropriate to the context Option b - is irrelevant as it talks about blood
Option e - information on ‘forest officials’ is blanks
inappropriate to this context Option c - is about human body which is not
Hence b is the correct sentence to fill the blank related to the given passage
which suits the context of the passage. Option d - is about food donation and not
relevant to the context, hence discarded
7) Answer: D Hence, e is the right sentence that completes the
Sentence given in option (d) is the right pick passage in the appropriate manner with the right
because the sentence that follows the blank also context.
talks about India and its countries. And the

Click Here For Bundle PDF Course | support@guidely.in Page 8 of 12


SBI Clerk & RRB PO Mains PDF Course 2023
ENGLISH Day - 34

9) Answer: A Hence, option c is the best answer and this


The context of the passage is ‘medical entrance sentence perfectly fits in the given blank.
exams’ and sentence in option a is the right fit
for the given blank. 11) Answer: E
Option b - sentence is about the exam centers The two words highlighted in the given sentence
which is not appropriate are already correct and appropriate to the
Option c - the passage is about medical context of the sentence and hence needs no
entrance and this sentence is totally irrelevant replacement.
speaking about generic medicines
Option d - is about medical institutions/colleges 12) Answer: B
and does not sit right in the given blank ‘Tagged’ and ‘boring’ are inappropriate and add
Option e - something that is related to healthcare no meaning to the sentence and definitely need
service but not right for the given passgae replacement.
Hence, a is the correct sentence to fill the given Tugged and captivating are the right words to
blank in the passage which suits the context. replace ‘tagged’ and ‘boring’ respectively to
make the sentence correct and meaningful.
10) Answer: C Tugged - to pull something hard and quickly for
Statement in option c is a perfect concluding several times
sentence to the given passage on the topic dark Captivating - holding your attention by being
chocolates. extremely interesting, exciting, pleasant, or
Option a - the passage does not discuss about attractive
the imports of chocolates and hence this Repellent - causing a strong feeling of disgust
sentence is inappropriate Idled - spend time doing nothing
Option b - definition of velvet which is not really Irksome - annoying and irritating
necessary as the main topic here is dark The sentence: The orchestra played a
chocolate symphony that tugged at the heartstrings, filling
Option d - very generic information and not the concert hall with a captivating and
specific to the idea of the passage harmonious melody.
Option e - is talking about dark chocolate but if
we look carefully sentence c is more meaningful 13) Answer: D
whereas sentence e suddenly from nowhere ‘Compelling and eagerly’ are the correct words
talks about the chemical components present in to replace the words ‘forcing and unwanted’
dark chocolate. respectively.

Click Here For Bundle PDF Course | support@guidely.in Page 9 of 12


SBI Clerk & RRB PO Mains PDF Course 2023
ENGLISH Day - 34

Compelling - that forces or persuades you to do convenience and efficiency but also demanding
or to believe something responsible usage to strike a healthy balance.
Innovation is a wrong word form here and does
not fit in the sentence 15) Answer: A
Unique fits the first blank, but the word ‘long’ is Known and bared are wrong and need
appropriate in the context and does not fit the replacement.
second blank Bared - uncover and expose it to view
Dull is inappropriate to the context of the Evident - obvious
sentence Endeared - to make somebody/yourself liked by
somebody
The sentence: The show had a huge fan base, Option b is wrong because ‘impressed her to’ is
as its compelling storyline, relatable characters, wrong
and unexpected plot twists kept viewers eagerly Option c is incorrect because ‘displease’ is
awaiting each new episode. inappropriate to the context
Option d - apparent(readily seen or understood)
14) Answer: C is correct and can replace the first word but
The words ‘intact’ and ‘produce’ are incorrect ‘belittle’ is wrong because of its meaning ‘to
and need replacement. The correct options out make somebody/something seem unimportant’
of the given 5 is option c (inseparable and strike) The sentence : Latha's innocence was evident in
Intact - complete and not damaged her wide-eyed curiosity and pure heart, radiating
Inseparable - cannot separate a charm that instantly endeared her to everyone
To strike a balance - to achieve a state in which she met.
different things occur in equal or proper amounts
or have an equal or proper amount of 16) Answer: C
importance. Calmed is wrong instead it must be calming
Indivisible, inseparable and integral fits the first because a gerund is the right form, ‘effect of’ is
blank correctly but their pairs (making, strike and wrong instead it must be ‘effect on’.
produce) need to fit the second blank correctly to The statement is option c is the right phrase to
be the correct answer for the given question. replace the highlighted incorrect part.
Out of making, strike and produce - strike is the Option a - have is wrong instead it must be ‘had’
correct word. Option b - ‘effects about anyone’ is wrong
The sentence : Gadgets are becoming an Option d - whom is an incorrect pronoun, instead
inseparable part of our daily lives, providing it must be ‘who’

Click Here For Bundle PDF Course | support@guidely.in Page 10 of 12


SBI Clerk & RRB PO Mains PDF Course 2023
ENGLISH Day - 34

The sentence : His voice was smooth as silk,


with a soothing tone that captivated listeners and 18) Answer: A
had a calming effect on anyone who heard him The highlighted part is incorrect and definitely
speak or sing. needs replacement because ‘who’ and ‘around
over’ in the sentence is making it incorrect.
17) Answer: D The statement is option a is the best
The highlighted phrase in the sentence is replacement among the given option.
completely wrong and hence need Option b - lake and those are incorrect, lake is
improvement/replacement singular and those denotes plural
The phrase is option d can be used to replace Option c - ‘that attracting tourists’ is a wrong
the incorrect part in the sentence and make the phrase
sentence correct and meaningful. Option d- ‘whom attracted tourism’ is a wrong
Consistent - always having the same opinions, phrase
standard, behaviour/not changing The sentence: The state had a diverse
Patient(adjective) - able to stay calm and not get landscape, boasting lush green valleys, majestic
angry, especially when there is a difficulty or you mountains, and serene lakes that attracted
have to wait a long time tourists from all over the country.
Patient (singular noun)
Patients(plural noun) - people who are receiving 19) Answer: E
medical treatment The highlighted part of the sentence is correct
patience (noun)- the quality of being able to stay and hence need not be replaced.
calm and not get angry, especially when there is
a difficulty or you have to wait a long time 20) Answer: D
Option a - need is wrong, singular noun must be Invite and enrolling makes the sentence
followed by plural verb incorrect and hence the highlighted phrase
Option b - both patiently and consistently are needs replacement.
inappropriate Option d has the correct statement to replace the
Option c- patient is wrong and inappropriate wrong part.
Option a - to enrolling is incorrect instead it must
The sentence: To grow long hair, one needs be to enrol
patience and a consistent hair care routine, Option b - to enrolled is incorrect instead it must
including regular trims, proper nourishment, and be to enrol
gentle handling to avoid breakage.

Click Here For Bundle PDF Course | support@guidely.in Page 11 of 12


SBI Clerk & RRB PO Mains PDF Course 2023
ENGLISH Day - 34

Option c - invites is wrong and ‘embarked’ is also The sentence: Admissions are open for the new
wrong(enrol and embark - must follow the same academic year, inviting eager students to enrol
word form) in various courses and embark on a journey of
knowledge and growth.

Click Here For Bundle PDF Course | support@guidely.in Page 12 of 12


SBI Clerk & RRB PO Mains PDF Course 2023
Reasoning Ability Day - 35 (Eng)

Reasoning Ability
Directions (1-5): Study the following information 1) Experience of who among the following
carefully and answer the below questions. person is 26 years?
Nine persons namely – A, B, C, D, E, F, G, H, a) E
and I live in a nine floored building marked 1 to 9 b) The one who lives three floors above F
from bottom to top. Each person has different c) The one who lives just below D
working experience viz.- 7, 9, 12, 13, 14, 17, 21, d) Either A or C
24, and 26. All the information is not necessarily e) Either A or B
in the same order.
Note: The experience of the persons living on the 2) The number of persons living between D and
adjacent floor is neither a common factor nor a the one whose experience is 7 years is the same
common multiple. as ____?
B lives on an even number floor. C, who lives on a) The number of persons living between A and
any floor below floor number 4, lives three floors the one whose experience is 12 years.
away from B and doesn’t have experience of 12 b) Two
years. H lives on any floor above the one whose c) The number of persons living below C.
experience is 12 years but not adjacent to B’s d) One
floor. Neither B nor C lives on the adjacent floor e) Both B and C
of I, who lives four floors away from the one
whose experience is 7 years. G lives three floors 3) What is the sum of the experience of the
above the one whose experience is 21 years. persons living on floor numbers 6, 4, and 1?
The number of persons living between B and I is a) 43
the same as the number of persons living below b) 45
A, whose experience is 17 years. The number of c) 40
persons living between G and the one whose d) 49
experience is 26 years is one more than the e) None of these
number of persons living between the one whose
experience is 12 years and H. Experience of I is 4) If all the persons from the top of the building
neither 26 nor 14 years. The one whose live in ascending order of their experience then,
experience is 14 years lives two floors away from the floor of how many persons remains
D. The number of persons living between D and unchanged?
B is the same as between D and the one whose a) One
experience is 9 years. H doesn’t live on the b) Two
topmost floor. F has less experience than E. c) None

Click Here For Bundle PDF Course | support@guidely.in Page 1 of 10


SBI Clerk & RRB PO Mains PDF Course 2023
Reasoning Ability Day - 35 (Eng)

d) More than three one less than the number of shelves between
e) Three box T and box J. Box Q is kept on the top-most
shelf. No box is kept to the east of box Q. Box M
5) How many persons are living between G and is kept two shelves below box N. Only fifteen
D? shelves are between Box M and Box S. Box S is
a) One kept north of box M, which is kept at the bottom.
b) Two 6) How many shelves are there in the stack?
c) None a) 18
d) More than three b) 22
e) Three c) 21
d) 19
Directions (6-10): Study the following information e) 20
carefully and answer the questions given below.
A certain number of boxes are kept in two 7) Four of the following five are alike in a certain
different stacks viz. Stack I and Stack II on way as per the given arrangement and hence
different shelves. Stack I is to the west of stack II. form a group. Find the one that does not belong
Information about some of them is given. The to that group.
lowermost shelf is numbered as 1, the shelf a) The box which is kept north-west of box T
immediately above it is numbered as 2, and so b) Box I
on. c) Box M
Box K is kept four boxes above the box which is d) Box T
kept two boxes below box L. Box K is kept north e) The box which is kept immediately below Box
of box P, which is kept south-west of box L. Only P in the same stack
one shelf is between Box L and Box P. Box N is
kept on the adjacent shelf of box P but not in the 8) Which of the following statement(s) is/are true
same stack. Only two shelves are between Box with respect to the final arrangement?
K and Box J. Box J is kept in the same stack as I. Both Box L and Box N are kept on the even-
Box N but not kept on the adjacent shelf of box L. numbered shelves.
Box S is kept four boxes above Box I, which is II. Box I is kept on an odd numbered shelf.
kept three boxes below box T. Box I is kept III. Only five shelves are between box J and box
north-west of box J and south-west of box T. N.
Only one shelf is between box I and box J, which a) Only I and III
is not kept on the same stack as box S. The b) Only III
number of shelves between box Q and box T is c) Only II and III

Click Here For Bundle PDF Course | support@guidely.in Page 2 of 10


SBI Clerk & RRB PO Mains PDF Course 2023
Reasoning Ability Day - 35 (Eng)

d) Only II IV. If both the numbers are odd, then add the
e) None of these numbers.
V. If more than one condition is applied, then
9) How many shelves are between box I and box only one condition is applied as per the order.
Q? VI. If no conditions are applied, then the sum of
a) Three the squares of the unit digits is taken.
b) Six 11)What is the sum of the digits of the resultant
c) Nine of all three rows?
d) Eight 12 25 8
e) None of these 33 48 23
13 56 97
10) Which of the following statement(s) is/are not a) 13
true with respect to the final arrangement? b) 14
a) Box T is kept north of box N c) 18
b) Box I is kept south-west of box K d) 19
c) Box M is kept three places below box P e) None of these
d) Only seven shelves are below box K
e) All the above statements are true 12) What is the product of the digital sum of the
resultant of all three rows?
Direction (11-15): Study the following information 11 34 53
carefully and answer the questions given below. 32 21 76
Below questions consist of numbers in three 9 17 46
rows. Results from each row can be obtained by a) 232
applying certain rules given below. Study the b) 234
following information carefully and answer the c) 123
below questions. d) 165
I.If an odd number is followed by a composite e) None of these
number, then the sum of the unit digits is taken.
II. If an even number is followed by a perfect 13) If the sum of the result of all the three rows is
square number, then the product of the digits of 31, then what is the value of X?
the even number is taken. 46 10 49
III. If an even number is followed by a perfect 23 47 27
cube, then multiply the unit digit of the even 33 X 23
number by the cube root of the perfect cube. a) 14

Click Here For Bundle PDF Course | support@guidely.in Page 3 of 10


SBI Clerk & RRB PO Mains PDF Course 2023
Reasoning Ability Day - 35 (Eng)

b) 16 P12%Q means P is 12m to the west of Q.


c) 17 P34#$Q means P is 34m to the southeast of Q.
d) 19 Two buses – bus 1 and bus 2, are travelling from
e) 11 point D. The paths travelled by the two buses are
given below using the above symbols along with
14) What is the sum of the resultant of all three distance.
rows? Bus 1: D8%A, E4@J, B3#R, J9$V, A13#V,
20 36 53 E17$B
19 18 43 Bus 2: N7#M, D4@L, M2$C, L1%U, O17@U,
25 38 40 N8%O
a) 63 On the basis of the above statements answer the
b) 72 questions given below.
c) 75 16) What is the shortest distance between M and
d) 68 A?
e) None of these a) 22m
b) 25m
15) What is half of the sum of the resultants of all c) 35m
three rows? d) 28m
44 67 35 e) 32m
28 64 8
31 46 79 17) What is the direction of E with respect to D?
a) 58 a) #$
b) 44 b) @%
c) 54 c) #%
d) 53 d) @$
e) None of these e) None of these

Directions (16-20) Study the following 18)If Bus 3 is 16m north of point D, then which of
information carefully and answer the below the following statement is/are true?
questions I. R is north-east of bus 3
P@Qmeans P is to the north of Q. II. E is south-west of bus 3
P#Q means P is to the south of Q. III. O is south of bus 3
P$Q means P is to the east of Q. a) Only I
P%Q means P is to the west of Q. b) All are true

Click Here For Bundle PDF Course | support@guidely.in Page 4 of 10


SBI Clerk & RRB PO Mains PDF Course 2023
Reasoning Ability Day - 35 (Eng)

c) Only III 20) If a person wants to reach point B from point


d) Only II A, then which of the following can be the
e) None is true minimum path?
I. Go to point D, then turn right and reach point B
19) What is the distance between the ending II. Go to point V, then go in the north-west
points of Bus 1 and bus 2? direction to reach point B
a) √15m III. Go to point O, then turns left to reach point N,
b) 8m then go in north-east direction to reach point B
c) √50m a) Both II and III
d) 9m b) Only I
e) None of these c) Only II
d) Both I and III
e) Only III
Click Here to Get the Detailed Video Solution for the above given Questions
Or Scan the QR Code to Get the Detailed Video Solutions

Answer Key with Explanation

Directions (1-5):
1) Answer: A
2) Answer: E
3) Answer: C
4) Answer: A
5) Answer: A

Click Here For Bundle PDF Course | support@guidely.in Page 5 of 10


SBI Clerk & RRB PO Mains PDF Course 2023
Reasoning Ability Day - 35 (Eng)

We have: Case (1) is not valid as the number of persons


 B lives on an even number floor. living between B and I is the same as the
 C, who lives on any floor below floor number of persons living below A, whose
number 4, lives three floors away from B experience is 17 years.
and doesn’t have experience of 12 years. Again, we have:
That means, in case (2) B lives on floor  G lives three floors above the one whose
number 6, in case (1) B lives on floor experience is 21 years.
number 4. That means, in case (2) G lives on floor
 Neither B nor C lives on the adjacent floor number 9, in case (2a) G lives on floor
of I, who lives four floors away from the number 4.
 one whose experience is 7 years.  The number of persons living between G
 The number of persons living between B and the one whose experience is 26
and I is the same as the number of years is one more than the number of
persons living below A, whose experience persons living between the one whose
is 17 years. experience is 12 years and H.
That means, in case (2) I lives on floor  The experience of I is neither 26 nor 14
number 8, case (1) is not valid. years.
Based on the above given information we have:  H lives on any floor above the one whose
experience is 12 years but not adjacent to
B’s floor.

Click Here For Bundle PDF Course | support@guidely.in Page 6 of 10


SBI Clerk & RRB PO Mains PDF Course 2023
Reasoning Ability Day - 35 (Eng)

 H doesn’t live on the topmost floor. This, the experience of D is 13 years.


That means, in case (2) the one whose Based on the above given information we have:
experience is 12 years lives on floor
number 1, case (2a) is not valid.
Based on the above given information we have:

Directions (6-10):
6) Answer: E
7) Answer: D
Case (2a) is not valid as H lives on any floor 8) Answer: D
above the one whose experience is 12 years but 9) Answer: B
not adjacent to B’s floor and H doesn’t live on 10) Answer: B
the topmost floor. Final arrangement:
Again, we have:
 The one whose experience is 14 years
lives two floors away from D.
 F has less experience than E.
 The number of persons living between D
and B is the same as between D and the
one whose experience is 9 years.
Since, persons whose experiences are in
common multiple are not living on the
adjacent floor.

Click Here For Bundle PDF Course | support@guidely.in Page 7 of 10


SBI Clerk & RRB PO Mains PDF Course 2023
Reasoning Ability Day - 35 (Eng)

From the above condition, there are two


possibilities.

Again we have,
 Only two shelves are between Box K and
Box J.
 Box J is kept in the same stack as Box N
but not kept on the adjacent shelf of box
L.
 Box S is kept four boxes above Box I,
which is kept three boxes below box T.
 Box I is kept north-west of box J and
south-west of box T.
 Only one shelf is between box I and box
J, which is not kept on the same stack as
box S.
 The number of shelves between box Q
We have, and box T is one less than the number of
 Box K is kept four boxes above the box shelves between box T and box J.
which is kept two boxes below box L.  Box Q is kept on the top-most shelf.
 Box K is kept north of box P, which is kept  No box is kept to the east of box Q.
south-west of box L.
 Only one shelf is between Box L and Box
P.
 Box N is kept on the adjacent shelf of box
P but not in the same stack.

Click Here For Bundle PDF Course | support@guidely.in Page 8 of 10


SBI Clerk & RRB PO Mains PDF Course 2023
Reasoning Ability Day - 35 (Eng)

Again we have,
 Box M is kept two shelves below box N.
 Only fifteen shelves are between Box M
and Box S. Direction (11-15):

 Box S is kept north of box M, which is 11) Answer: C

kept at the bottom. 12 25 8

From the above condition, case 2 gets From condition II: (1*2) = 2

eliminated. Case1 shows the final arrangement. From condition III: (2*2) = 4
33 48 23
From condition I: (3+8) = 11
From condition IV: (11+23) = 34
13 56 97
From condition I: (3+6) = 9
From condition IV: (9+97) = 106
Required number = 4+3+4+1+0+6= 18

Click Here For Bundle PDF Course | support@guidely.in Page 9 of 10


SBI Clerk & RRB PO Mains PDF Course 2023
Reasoning Ability Day - 35 (Eng)

12) Answer: B From condition I: (8+5) = 13


11 34 53 From condition I: (3+0) = 3
From condition I: (1+4) = 5 Required number = 9+60+3 = 72
From condition IV: (5+53) =58
32 21 76 15) Answer: C
From condition VI: (4+1) = 5 44 67 35
From condition I: (5+6) = 11 From condition VI: (16+49) = 65
9 17 46 From condition I: (5+5) = 10
From condition IV: (9+17) = 26 28 64 8
From condition VI: (36+36) = 72 From condition II: (2*8) = 16
Required number = 13*2*9 = 234 From condition III: (6*2) = 12
31 46 79
13) Answer: D From condition I: (1+6) = 7
46 10 49 From condition IV: (7+79) = 86
From condition VI: (36+0) = 36 Required number = (86+12+10)/2 = 54
From condition II: (3*6) = 18
23 47 27 Directions (16-20) :
From condition IV: (23+47) = 70 16) Answer: B
From condition III: (0*3) = 0 17) Answer: D
Thus, result of 3rd row= 31-18= 13 18) Answer: E
33 19 23 19) Answer: D
From condition IV: (33+19)= 52 20) Answer: C
From condition VI: (4+9) = 13

14) Answer: B
20 36 53
From condition II: (2*0) = 0
From condition VI: (0+9) = 9
19 18 43
From condition I: (9+8) = 17
From condition IV: (17+43) = 60
25 38 40

Click Here For Bundle PDF Course | support@guidely.in Page 10 of 10


SBI Clerk & RRB PO Mains PDF Course 2023
Quantitative Aptitude Day – 35 (Eng)

Quantitative Aptitude

Directions (1-5): Study the following data X: The ratio of the maximum speed of bike E to
carefully and answer the questions: that of car T is 23: 36.
The data given below is related to the maximum Y: The distance covered by bike C at 50% of its
speeds (in km/h) of 5 different bikes A, B, C, D maximum speed in 1.2 hours is 10 km less than
and E and the maximum speeds (in km/h) of 5 that covered by bike D at 50% of its maximum
different cars P, Q, R, S and T. speed in 1.5 hours.
The table given below shows the maximum Z: The average of the maximum speeds of cars
speeds of bikes B, C, D and E as percent more R, S and T is 210 km/h.
or less than that of those of bike A and the a) Only X
maximum speeds of cars Q, R, S and T as b) Only Y and Z
percent more or less than that of car P. c) Only Y
d) Only A and Z
e) None is true.

2) A person has to cover a distance of 345 km by


cars P, Q and S. First he travelled by car P at
48% of its maximum speed for 1.25 hours, then
he travelled by car Q at 45% of its maximum
speed for 1 hour. If he has to cover the
remaining distance in 1.5 hours, then find out at
what per cent of the maximum speed of car S
should he travel?
a) 45%
Note: b) 35%
1: Maximum speed of car Q is 100 km/h more c) 50%
than that of bike B. d) 40%
2: The ratio of the distance covered by bike A at e) 30%
64% of its maximum speed in 2.5 hours to that
covered by car P at 60% of its maximum speed 3) If the ratio of distance covered by bike A at
in 1.5 hours, is 10: 9. 72% of its original speed in 1.8 hours to that
1) Which of the following is/are not true? covered by car R at 60% of its maximum speed

Click Here For Bundle PDF Course | support@guidely.in Page 1 of 14


SBI Clerk & RRB PO Mains PDF Course 2023
Quantitative Aptitude Day - 35 (Eng)

in 1.5 hours is M: N, then find that the value of e) Only X and Y


(5M – 2N) is divisible by which of the following?
a) 8 Directions (6-8): Study the following data
b) 7 carefully and answer the questions:
c) 5 A shopkeeper has three articles A, B and C of
d) Both (a) and (b) different costs. The ratio of the cost price of
e) Both (a) and (c) article A to that of article B is 2: 5 and the cost
price of article C is ₹1500 less than that of article
4) The bike E is running ‘d’ km ahead of car P. If B. Article A is marked up by 75%, article B is
bike E is running at 80% of its maximum speed, marked up by ₹900 and the marked price of
car P is running at 64% of its maximum speed article C is ₹1350 more than that of article A. If
and car P crosses bike E in 2.5 minutes, then article B is sold at 8% discount on its marked
find the time taken by bike B to cover ‘d’ km at price and article C is sold at 4% discount on its
75% of its maximum speed? marked price, then the selling price of article B
a) 60 seconds will be ₹648 more than that of article C.
b) 75 seconds 6) If article A is sold at a 20% discount on its
c) 45 seconds marked price, article B is sold at 10% discount
d) 50 seconds on its marked price and article C is sold at a 28%
e) 80 seconds discount on its marked price, then find the
average of profit received on articles A, B and C?
5) The distance covered by bike B at ___% of its a) ₹410
maximum speed in 1.6 hours is 14 km more than b) ₹440
that covered by car T at ___% of its maximum c) ₹420
speed in 1.2 hours. d) ₹450
Find out which of the following can’t be filled in e) ₹430
the blanks in the same order?
X: 60, 40 7) If article B is marked up by M% and article C
Y: 75, 60 is marked up by N%, then which of the following
Z: , 50 is/are true?
a) Only X P: The sum of M and N is divisible by a prime
b) Only Y and Z number greater than 5.
c) All X, Y and Z Q: Both M and N are divisible by 5 but not by 10.
d) Only Z R: The value of (N2 – M2) is divisible by 11.
a) Only P and R

Click Here For Bundle PDF Course | support@guidely.in Page 2 of 14


SBI Clerk & RRB PO Mains PDF Course 2023
Quantitative Aptitude Day - 35 (Eng)

b) All P, Q and R B, then what will be the cost of the new mixture
c) Only P (in terms of M and N)?
d) Only Q a)
e) Only R b)
c)
8) The cost price of article D is equal to the d)
average of the cost prices of articles A, B and C
e) None of these
and article D is marked up by 140%. If article D
is sold at ₹1085 discount on its marked price,
10) If the ratio of milk in mixture C to that in
then find the ratio of the marked up per cent of
mixture D is 3: 5 and the ratio of water in mixture
article D to the profit per cent received on article
B to that in mixture D is 8: 5, then find which of
D?
the following is/are true?
a) 4: 1
P: The ratio of milk in mixture A to that in mixture
b) 5: 1
D is 4: 5.
c) 10: 1
Q: The average quantity of water in mixtures A,
d) 8: 1
B, C and D is 9.5 L.
e) 2: 1
R: The cost of mixture D is ₹30 per L.
a) All are true
Directions (9-11): Study the following data
b) Only P
carefully and answer the questions:
c) Only P and Q
Three mixtures A, B and C contain different
d) Only P and R
quantities (in L) of milk and water and the cost of
e) None is true.
pure milk is ₹45 per L.
The ratio of milk to water in mixture A is 4: 1 and
11) When 50% quantity of mixture A, 75%
the ratio of milk to water in mixture B is 3: 2.
quantity of mixture B and 25% quantity of mixture
When both the mixtures are mixed together, the
C are mixed together, then what will be the cost
cost of the new mixture becomes ₹30 per L. The
of the new mixture?
quantity of milk in mixture A is 8 L less than that
a) ₹28 per L
in mixture B and also 4 L more than that in
b) ₹24 per L
mixture C. The cost of mixture C is ₹27 per L.
c) ₹30 per L
9) The cost of mixture A is ₹M per L and the cost
d) ₹29 per L
of mixture B is ₹N per L. If 15 L quantity of
e) ₹32 per L
mixture A is mixed with 10 L quantity of mixture

Click Here For Bundle PDF Course | support@guidely.in Page 3 of 14


SBI Clerk & RRB PO Mains PDF Course 2023
Quantitative Aptitude Day - 35 (Eng)

Directions (12-16): Study the following data carefully and answer the questions:
The data given below is related to the number of satellites launched through 6 different rockets A, B, C,
D, E and F in a country and each rocket carried a different weight (in kg).
The bar graph given below shows the total weight carried by each of the rockets B, C, D, E and F as per
cent more/less than that carried by rocket A.

The table given below shows the number of satellites carried by each rocket.

Note:
1: The average weight of each satellite carried by rocket A is 640 kg more than the average weight of
each satellite carried by rocket B.
2:

Click Here For Bundle PDF Course | support@guidely.in Page 4 of 14


SBI Clerk & RRB PO Mains PDF Course 2023
Quantitative Aptitude Day - 35 (Eng)

12) If the average weight of each satellite carried R: The difference between the average weight of
by rocket C is M% more/less than the average each satellite carried by rocket B and the
weight of each satellite carried by rocket A and average weight of each satellite carried by rocket
the average weight of each satellite carried by C is 380 kg.
rocket E is N% more/less than the average a) Only P
weight of each satellite carried by rocket D, then b) Only Q and R
find the ratio of M to N? c) Only Q
a) 10: 167 d) Only P and R
b) 5: 67 e) Only R
c) 40: 467
d) 20: 267 15) If the total weight carried by rocket G is 25%
e) None of these more than that carried by rocket C and the
average weight of each satellite carried by rocket
13) If the weight of rocket D without satellites is E is 76.8% of the average weight of each
12.5% more than the total weight of satellites satellite carried by rocket G, then find that the
carried by rocket D and the weight of rocket F number of satellites carried by rocket G is what
without satellites is 5% more than the total percent more/less than that carried by rocket A?
weight of satellites carried by rocket F, then find a) 16.67%
that the weight of rocket F without satellites is b) 20%
what percent more or less than that of rocket D c) 15%
without satellites? d) 12.5%
a) 5.78% e) 10%
b) 12.21%
c) 10% 16) If the total weight carried by rocket A were
d) 2.24% 4000 kg less and the new total weight carried by
e) 19.81% E is 20% more than the new total weight carried
by A. If the number of satellites carried by rocket
14) Which of the following is/are true? E were the same as initially, then the average
P: The ratio of the average weight of each weight of each satellite carried by rocket E would
satellite carried by rocket E to the average weight be what per cent less than the original average
of each satellite carried by rocket F is 48: 83. weight of each satellite carried by rocket E?
Q: The average of the total weight carried by a) 7.25%
rockets C, D and E is 60800 kg. b) 2.25%
c) 9.25%

Click Here For Bundle PDF Course | support@guidely.in Page 5 of 14


SBI Clerk & RRB PO Mains PDF Course 2023
Quantitative Aptitude Day - 35 (Eng)

d) 8.25% 19) Around a rectangular park, there is a path


e) 6.25% running outside it of uniform width given in the
figure below.
17) A boat goes ‘D’ km upstream and ‘D + 10’ km
downstream and takes a total ‘17t’ min. If the
boat covers total ‘D + 10’ km upstream and ‘D’
km in downstream in ‘19t’ min. If the ratio of the
upstream speed to the downstream speed of the
boat is 1: 2, then which of the following values
can be determined?
I: Upstream speed of the boat.
II: Time taken by the same boat to go ‘D – 10’ km
upstream.
If the cost of flooring the shaded portion at the
III: Value of ‘D’.
rate ₹5/m2 is ₹2540 and the length and the width
a) Only statements I and II
of the park (without the path) are20 m and 15 m
b) Only statements I and III
respectively and the radius of the semi-circular
c) Only statement III
figure is 7 m, then find the width of the path.
d) None of the statements follows
a) 2.25 m
e) Only statement II
b) 3 m
c) 2 m
18) The time taken by persons A, B, and C alone
d) 2.5 m
to finish a work is (24 +32 – 1) days, (52 + 11)
e) 3.5 m
days, (52 – 32 + 2) days respectively. If A and B
started the work and after ‘6x’ days A is replaced
20) The ratio of milk to water in two mixtures A
by C and now B and C together can finish that
and B is 7: x and x: 13 respectively. These two
work in ‘x’ days. Find the value of ‘x2 + 9’.
mixtures are mixed in the ratio of 5: 8 and when
a) 52 – 32
the final mixture is sold at the cost of pure milk,
b) 32 +5
the profit percent earned would be 160%. The
c) 12 +9
ratio of milk to water in a mixture that is formed
d) 22 +7
after mixing mixtures A and B in the same
e) 42 – 3
quantity.
a) 61: 99
b) 71: 89

Click Here For Bundle PDF Course | support@guidely.in Page 6 of 14


SBI Clerk & RRB PO Mains PDF Course 2023
Quantitative Aptitude Day - 35 (Eng)

c) 81: 79 e) 91: 69
d) 51: 109
Click Here to Get the Detailed Video Solution for the above given Questions
Or Scan the QR Code to Get the Detailed Video Solutions

Answer Key with Explanation

Directions (1-5):
Let the maximum speeds of bike A and car P are
176x – 96x = 100
100x km/h and 100y km/h respectively.
x = 1.25
So, the maximum speeds of bikes B, C, D and E
From equation (2):
are 96x km/h, 120x km/h, 112x km/h and 92x
y=2
km/h.
And the maximum speeds of cars Q, R, S and T
are 110y km/h, 85y km/h, 120y km/h and 95y
km/h.
Since, the maximum speed of car Q is 100 km/h
more than that of bike B.
So,
110y – 96x = 100 -------------(1)
Since, the ratio of the distance covered by bike A
at 64% of its maximum speed in 2.5 hours to that
covered by car P at 60% of its maximum speed 1) Answer: E

in 1.5 hours, is 10: 9. From X:

So, Ratio of the maximum speed of bike E to that of


car T = 115: 190 = 23: 38
So, X is not true.

Click Here For Bundle PDF Course | support@guidely.in Page 7 of 14


SBI Clerk & RRB PO Mains PDF Course 2023
Quantitative Aptitude Day – 35 (Eng)

From Y: The distance covered by car R at 60% of its


Distance covered by bike C at 50% of its maximum speed in 1.5 hours:
maximum speed in 1.2 hours: 60% of 170 * 1.5 = 153 km
50% of 150 * 1.2 = 90 km Required ratio = M: N = 162: 153 = 18: 17
Distance covered by bike D at 50% of its Now, the value of (5M – 2N) = 90 – 34 = 56
maximum speed in 1.5 hours: And the value of (5M – 2N) is divisible by both 8
50% of 140 * 1.5 = 105 km and 7.
Required difference = 105 – 90 = 15 km
So, Y is not true. 4) Answer: A
From Z: The speed, at which bike E is running = 80% of
Average of maximum speeds of cars R, S and T: 115 = 92 km/h
The speed, at which car P is running = 64% of
200 = 128 km/h
So, Z is not true.
So,
Hence, none is true.

2) Answer: B d = 1500 m = 1.5 km


Distance covered by the person by car P at 48% Now, the time taken by bike B to cover 1.5 km at
of its maximum speed in 1.25 hours = 48% of 75% of its maximum speed:
200 * 1.25 = 120 km
Distance covered by the persons by car Q at
45% of its maximum speed in 1 hour = 45% of
220 * 1 = 99 km 5) Answer: E
Remaining distance = 126 km From X:
The speed of the car S while covering the 60% of 120 * 1.6 – 40% of 190 * 1.2 = 115.2 –
remaining distance = = 84 km/h 91.2 ≠ 14 km

The maximum speed of car S = 240 km/h So, the values in X can’t be filled in the blanks in
Required percentage = = 35% the same order.
From Y:

3) Answer: D 75% of 120 * 1.6 – 60% of 190 * 1.2 = 144 –

The distance covered by bike A at 72% of its 136.8 ≠ 14 km

maximum speed in 1.8 hours: So, the values in Y can’t be filled in the blanks in

72% of 125 * 1.8 = 162 km the same order.


From Z:

Click Here For Bundle PDF Course | support@guidely.in Page 8 of 14


SBI Clerk & RRB PO Mains PDF Course 2023
Quantitative Aptitude Day – 35 (Eng)

66(2/3)% of 120 * 1.6 – 50% of 190 * 1.2 = 128 – The profit received on article B = 4860 – 4500 =
114 = 14 km ₹360
So, the values in Z can be filled in the blanks in The SP of article C = 72% of 4500 = ₹3240
the same order. The profit received on article C = 3240 – 3000 =
Hence, the values in Z only can be filled in the ₹240
blanks in the same order. Required average = = ₹440

Directions (6-8): 7) Answer: C


Let the CPs of articles A and B be ₹2x and ₹5x The CP of article B = ₹4500
respectively. The MP of article B = ₹5400
So, the CP of article C = ₹(5x – 1500) So, M = = 20%
The MP of article A = 175% of 2x = ₹3.5x The CP of article C = ₹3000
The MP of article B = ₹(5x + 900) The MP of article C = ₹4500
And the MP of article C = ₹(3.5x + 1350) So, N = = 50%
If article B is sold at 8% discount on its marked From P:
price and article C is sold at 4% discount on its Sum of M and N = 20 + 50 = 70
marked price, then the selling price of article B Since, the sum of M and N is divisible by a prime
will be ₹648 more than that of article C. number 7, which is greater than 5.
So, So, P is true.
From Q:
460x + 82800 – 336x – 129600 = 64800 Since, both M and N are divisible by 5 and also
x = 900 divisible by 10.
The CP of article A = 2 * 900 = ₹1800 So, Q is not true.
The CP of article B = 5 * 900 = ₹4500 From R:
The CP of article C = 4500 – 1500 = ₹3000 Value of (N2 – M2) = 2500 – 400 = 2100
The MP of article A = 175% of 1800 = ₹3150 Since, 2100 is not divisible by 11.
The MP of article B = 4500 + 900 = ₹5400 So, R is not true.
The MP of article C = 3150 + 1350 = ₹4500 Hence, only P is true.
6) Answer: B
The SP of article A = 80% of 3150 = ₹2520 8) Answer: D
The profit received on article A = 2520 – 1800 = The CP of article D = = ₹3100
₹720 The MP of article D = 140% of 3100 = ₹4340
The SP of article B = 90% of 5400 = ₹4860 And the SP of article D = 4340 – 1085 = ₹3255

Click Here For Bundle PDF Course | support@guidely.in Page 9 of 14


SBI Clerk & RRB PO Mains PDF Course 2023
Quantitative Aptitude Day – 35 (Eng)

So, the profit per cent on article D =


= 5% 9) Answer: B
Required ratio = 40: 5 = 8: 1 Cost of mixture A = M = = ₹36 per L
Cost of mixture B = N = = ₹27 per L
Directions (9-11): When 15 L quantity of mixture A is mixed with 10
Let the quantities of milk and water in mixture A L quantity of mixture B:
are ‘4a’ L and ‘a’ L respectively. Quantity of milk in 15 L quantity of mixture A =
Also let the quantities of milk and water in = 12 L
mixture B are ‘3b’ L and ‘2b’ L respectively. Quantity of water in 15 L quantity of mixture A =
So, 15 – 12 = 3 L
Quantity of milk in 10 L quantity of mixture B =
=6L
12a + 9b = 10a + 10b
2a = b ------------(1) Quantity of water in 10 L quantity of mixture B =

Since, the quantity of milk in mixture A is 8 L less 10 – 6 = 4 L

than that in mixture B. Cost of new mixture = = ₹32.4 per L =

So,
3b – 4a = 8 ------------(2)
From equations (1) and (2): 10) Answer: A
3b – 2b = 8 Quantity of milk in mixture D = = 20 L
b=8 Quantity of water in mixture D = = 10 L
From equation (1): From P:
a=4 Ratio of milk in mixture A to that in mixture D =
Quantity of milk in mixture A = 4 * 4 = 16 L 16: 20 = 4: 5
Quantity of water in mixture A = 4 L So, P is true.
Quantity of milk in mixture B = 3 * 8 = 24 L From Q:
Quantity of water in mixture B = 2 * 8 = 16 L Average quantity of water in mixtures A, B, C
Quantity of milk in mixture C = 16 – 4 = 12 L and D:
Let the quantity of water in mixture C = ‘c’ L
And,
So, Q is true.
From R:
c=8 Cost of mixture D = = ₹30 per L
So, quantity of water in mixture C = 8 L So, R is true.

Click Here For Bundle PDF Course | support@guidely.in Page 10 of 14


SBI Clerk & RRB PO Mains PDF Course 2023
Quantitative Aptitude Day – 35 (Eng)

Hence, all are true.

11) Answer: D
Total quantity of milk in the new mixture:
50% of 16 + 75% of 24 + 25% of 12 = 8 + 18 + 3
= 29 L
Total quantity of water in the new mixture:
50% of 4 + 75% of 16 + 25% of 8 = 2 + 12 + 2 =
16 L
So, the cost of the new mixture:

12) Answer: B
Directions (12-16):
The average weight of each satellite carried by
Let the total weight carried by rocket A is 100x
rocket A = 4000 kg
kg.
The average weight of each satellite carried by
So, the total weights carried by rockets B, C, D,
rocket C = 3840 kg
E and F are 105x kg, 90x kg, 75x kg, 120x kg
The average weight of each satellite carried by
and 85x kg respectively.
rocket D = 2000 kg
The average weight of each satellite carried by
The average weight of each satellite carried by
rocket A = kg
rocket E = 3072 kg
And the average weight of each satellite carried So, M = = 4%
by rocket B = kg
And, N = = 53.6%
Since, the average weight of each satellite
Required ratio = M: N = 4: 53.6 = 40: 536 = 5: 67
carried by rocket A is 640 kg more than the
average weight of each satellite carried by rocket
13) Answer: A
B.
Total weight carried by rocket D = 48000 kg
So,
So, the weight of rocket D without satellites =
112.5% of 48000 = 54000 kg
1000x – 840x = 160 * 640 Total weight carried by rocket F = 54400 kg
x = 640 So, the weight of rocket F without satellites =
105% of 54400 = 57120 kg

Click Here For Bundle PDF Course | support@guidely.in Page 11 of 14


SBI Clerk & RRB PO Mains PDF Course 2023
Quantitative Aptitude Day – 35 (Eng)

Required percentage = = And the number of satellites carried by rocket G


5.78% = = 18
Since, the number of satellites carried by rocket
14) Answer: C A = 16
From P: So, required percentage = = 12.5%
Ratio of the average weight of each satellite
carried by rocket E to the average weight of 16) Answer: E
each satellite carried by rocket F = 3072: 5440 = The original total weight carried by rocket A =
48: 85 64000 kg
So, P is not true. So, the new total weight carried by rocket A
From Q: would be = 64000 – 4000 = 60000 kg
Average of total weight carried by rockets C, D And the new total weight carried by rocket E
and E: would be = 120% of 60000 = 72000 kg
If the number of satellites carried by rocket E

So, Q is true. were the same as initially:

From R: So, the average weight of each satellite carried

Difference between the average weight of each by rocket E would be:

satellite carried by rocket B and the average


weight of each satellite carried by rocket C: Since, original average weight of each satellite
3840 – 3360 = 480 kg carried by rocket E = 3072 kg
So, R is not true. So, required percentage = =
Hence, only Q is true. 6.25%

15) Answer: D 17) Answer: C


Since, the total weight carried by rocket C = Let the upstream speed and downstream speed
57600 kg of the boat is ‘x’ and ‘2x’ respectively.
So, the total weight carried by rocket G = 125% According to the question:
of 57600 = 72000 kg
Since, the average weight of each satellite
carried by rocket E = 3072 kg
So, the average weight of each satellite carried
by rocket G = = 4000 kg

Click Here For Bundle PDF Course | support@guidely.in Page 12 of 14


SBI Clerk & RRB PO Mains PDF Course 2023
Quantitative Aptitude Day – 35 (Eng)

36x = 72
x=2
From (1) and (2):
Value of ‘x2 + 9’ = 22 + 9 = 13 = 42 – 3

19) Answer: D
57D + 190 = 51D + 340 Let the width of the path = ‘x’ m
6D = 150 Length of path with path = (20 + 2x) m
D = 25 Width of path with path = (15 + 2x) m
By putting the value of ‘D’ in equation (1) and Area of the shaded portion =
(2):

We can clearly see than we cannot find the


value of ‘x’ and ‘t’.
So, we cannot calculate the upstream and
downstream speed of the boat.
So, we also cannot calculate the time taken by
same boat to go ‘D – 10’ km in upstream.
Hence, only statement III can be determined.

Hence, the width of the path = 5/2 m = 2.5 m


18) Answer: E
Time taken by A alone to finish the work = 24 +
20) Answer: B
32 – 1 = 16 + 9 – 1 = 24 days
Part of milk in mixture A =
Time taken by B alone to finish the work = 52 +
Part of milk in mixture B =
11 = 25 + 11 = 36 days
Time taken by C alone to finish the work = 52 – Since the profit percent earned when the final

32+ 2 = 25 – 9 + 2 = 18 days mixture is sold at the cost of pure milk is 160%.

Total days for which A, B, and C work are ‘6x’ Which means quantity of water in the final

days, ‘7x’ days, and ‘x’ days respectively. mixture will be 160% of quantity of pure milk.

According to the question: Ratio of milk to water in the final mixture = 100:
160 = 5: 8
Part of milk in final mixture =

Click Here For Bundle PDF Course | support@guidely.in Page 13 of 14


SBI Clerk & RRB PO Mains PDF Course 2023
Quantitative Aptitude Day – 35 (Eng)

By the rule of alligation: Let the part of milk in the final mixture when
mixtures A and B are mixed in equal quantity = P
By the rule of alligation:

5 8

1 1

Required ratio of milk to water = 71: (160 – 71) =


71: 89

Part of milk in mixture A =


Part of milk in mixture B =

Click Here For Bundle PDF Course | support@guidely.in Page 14 of 14


SBI Clerk & RRB PO Mains PDF Course 2023
ENGLISH Day - 35

English Language

Directions (01-07): Read the following passage market. Moreover, the onslaught of
and answer the questions that follow based on globalisationstripped away traditional skills and
the information provided in the passage. knowledge, forcing artisans to surrender to
India’s artisanal crafts embody deep-rooted and modern production techniques. In response, the
diverse traditions, nurtured by countless skilled Centre, state governments and non-
craftspeople across the nation. Their tireless governmental organisations (NGOs) made
hands weave a tapestry of exquisite treasures, attempts to address the hardships faced by
spanning textiles and jewellery to pottery and Indian artisans, with mixed results. The NGOs
woodwork. emerged as champions of artisans’ rights and
Yet, despite their undeniable talent and welfare, particularly those in rural areas. Through
contribution, artisans face challenges such as various programmes, these organisations
meagre wages, limited access to markets, and imparted training and education to artisans,
the surge of mass-produced goods, plunging empowering them with enhanced skills and a
them into a struggle for growth and sustainability. fighting chance in the marketplace.
These tribulations have unleashed a silent Parallelly, government organisations launched
epidemic, causing the gradual erosion of artisan initiatives. The AmbedkarHastshilpVikasYojana,
communities and the fading echoes of traditional in collaboration with
crafts. It is a battle fought on many fronts, where DastkarSashaktikaranYojana, and the Prime
the forces of globalisation have both nurtured Minister’s
and wounded the spirit of these artisans. VishwakarmaKaushalSammanYojana(PM
The liberalisation, privatisation and globalisation VIKAS) stand as a testament to these efforts. By
that swept India in 1991 brought winds of change mobilising artisans into self-help groups and
with both blessings and burdens for the artisans. societies, these schemes aim to unleash the
Globalisation ushered in a new era of power of bulk production and leverage
possibilities, as the demand for handicrafts and economies of scale in procuring raw materials.
authentic Indian products soared in international Through design and technology upgrades,
markets. Moreover, the exchange of ideas and workshops and training programmes,artisans are
techniques, facilitated by globalisation’s equipped with commercialmarket intelligence and
interconnectedness, breathed life into innovation, new techniques. However, artisans continue to
birthing new artistic creations and designs. confront an array of challenges. The spectre of
Yet, foreign lands unleashed a deluge of low- low wages, limited market access, and difficulty
cost, mass-produced goods, saturating the in securing loans haunts their every step. A fresh

Click Here For Bundle PDF Course | support@guidely.in Page 1 of 11


SBI Clerk & RRB PO Mains PDF Course 2023
ENGLISH Day - 35

impetus is required to dismantle the systemic II.It has led to make the products with modern-
barriers that stifle growth and development. This day technologies.
renewed focus must encompass increased III.Demand of Indian products in the International
access to credit, technical support and market Markets.
connections. IV.Modernised skills and Knowledge is highly
In the long run, fostering an environment that infused across the sector.
nurtures innovation and entrepreneurship within a) Both I and II.
the artisanal sector becomes paramount. b) Both II and III.
Investing in the skills and talents of Indian c) Both III and IV.
artisans transcends mere preservation -- it paves d)All I, II, III, IV.
the way for sustainable and inclusive economic e) Both I and III.
growth, intertwining tradition with immense
opportunity. 4. What are the initiatives has been brought up
1. Choose the word that is most SIMILAR to the by the government to address the hardships
given highlighted word “UNLEASHED” in the faced by Indian artisans?
passage. I. Monthly stipends were provided to the artisans
a) Bridle on behalf of NGOs.
b) Emancipate II.AmbedkarHastshilpVikasYojana and
c) Dissipate VishwakarmaKaushalSammanYojana(PM
d) Banish VIKAS) has been launched.
e) None of these. III.Organisations imparted training and education
to artisans.
2. Choose the word that is most OPPOSITE to a) Only I.
the given highlighted word “FOSTERING” in the b) Only II.
passage. c) Only III.
a) Deflate d) Both II and III.
b) Articulate e) All of these.
c) Vicious
d) Congenial 5. According to the given passage, which of the
e) None of these. following statement/statements are TRUE?
I.The artisans around the world are still facing
3. Which of the following option can convey the numerous challenges.
benefits that are acquired by the globalisation? II.Globalisation that swept the world in 1991.
I.Globalisation leads to produce innovative ideas.

Click Here For Bundle PDF Course | support@guidely.in Page 2 of 11


SBI Clerk & RRB PO Mains PDF Course 2023
ENGLISH Day - 35

III.Preservation will pave the way for sustainable c)Both I and III.
and inclusive economic growth. d) Both I and II.
a) Only I. e) Both II and III.
b) Only II.
c) Only III. Directions (08-12): In the following questions,
d) Both II and III. two columns (column I and column II) are given.
e) Both I and III. In column I, sentences A, B and C are present,
while column II has sentences D, E and F. The
6. Which of the following options listed below are table is followed by three connectors which are
considered as unpropitious events faced by the used to connect the sentences with column I and
artisans? II. You are required to choose the option that can
I.They are receiving meagre wages, even though make the most meaningful and contextually
their contribution is immense. correct sentence with the help of the following
II.They are accessing only the limited markets. options.
III.Plunging of goods cost in the arts sector. 8.
a) Only I. COLUMN I COLUMN II
b) Only II. A.These programs can D.also play a vital role
c) Only III. help them develop a in reducing the loss of
d) Both I and III. deeper understanding valuable professionals
e) All I, II and III. of their own mental from the workforce.
B.Behind the white E.physical health
7. From the given passage, which of the coats and needs, fostering a
following can be inferred as the preventive stethoscopes, doctors proactive approach
measures to tackle the challenges? face a range of towards well-being.
I.A new drive should be initiated to break the occupational stressors
existing barriers. C.These services not F.put their physical
II.Government and NGOs should consider this only provide an avenue and mental health at
and provide subsidies to the artisans and for doctors to disclose risk.
recommend the sector to stall the souring price their struggles
of goods. I.But
III.It has to focus in assisting the artisans for the II.And
technical and credit support. III.That
a) Only I. a) Only C-I-D.
b) Only II.

Click Here For Bundle PDF Course | support@guidely.in Page 3 of 11


SBI Clerk & RRB PO Mains PDF Course 2023
ENGLISH Day - 35

b) Only A-II-E. 10.


c) Only B-III-F. COLUMN I COLUMN II
d) None of these.
e) All A, B, C. A.Instruments on the D.Vikram crash landed
lander will measure in a spray of lunar
9. temperatures, search dust, leaving it, and
COLUMN I COLUMN II for moonquakes, the small rover inside,
inoperable.
A.“The government and the D.borders B.The Chandrayaan E.study the effects of
SGPC are having a face­off Pakistan, this missions may only be solar winds. The
controversy the first small steps in lander will also deploy
poses yet India's dreams for the a rover to conduct
another moon. geological studies.
challenge. C.The more audacious F.if it wants to be a
B.No doubt, this has helped E.it appears that Chandrayaan-2 in 2019 top-tier player in lunar
the Akali Dal retain power in both are not was to deploy a lander exploration.
Punjab, doing enough to called Vikram.
resolve the I.However
issue. II.And
C.For the AAP, which has F.there have III.That
formed the government for the been occasions a) Only C-II-D.
first time in the State of Sikh cases b) Only A-III-E.
suffering at the c) Only B-I-F.
altar of political d) None of these.
exigency. e) All A, B, C.
I.But
II.And 11.
III.That COLUMN I COLUMN II
a) Only C-III-D.
A.The decision not to D.average
b) Only A-I-E.
raise the urea MRP production/import cost is
c) Only B-II-F.
till 2024-25 has, in the region of Rs
d) None of these.
ironically, been 30,000 per tonne.
e) All A, B, C.
announced

Click Here For Bundle PDF Course | support@guidely.in Page 4 of 11


SBI Clerk & RRB PO Mains PDF Course 2023
ENGLISH Day - 35

B.The Modi E.a PM-PRANAM C.Leaders in innovation F.electricity and the


government has scheme that aims to and research like China internet have reached
proposed the promote alternative and the US spend almost all villages.
introduction of fertilisers and balanced massive sums on R&D.
sulphur-coated urea, plant nutrition. I.But
C.That works out to F.it claims, would be II.However
roughly $45 billion for more economical and III.While
subsidising a single efficient than the plain- a) Only C-II-D.
nutrient product, vanilla fertiliser. b) Only A-III-E.
I.Which c) Only B-I-F.
II.If d) None of these.
III.Along with e) All A, B, C.
a) Only C-II-D.
b) Only A-III-E. Directions (13-15): In the following questions,
c) Only B-I-F. two statements are given. You are required to
d) Both B and C. join the sentences with the starters or connectors
e) All A, B, C. given below each question without altering the
original meaning of the sentence.
12. 13.
COLUMN I COLUMN II A.The film did not meet any eligibility criteria
proposed by the organiser.
A.Most of our D.India's private and
B.It was premiered at the Cannes Festival held in
innovations are public funding on R&D
Pune.
secondary is less than 0.7 per
I.Despite
improvements of cent of its GDP, China
II.Respite
existing technologies. spends over 2.5 per
III.Notwithstanding
Catching up is not bad. cent of its GDP on the
a) Only I.
same.
b) Only II.
B.An optical fibre cable E.a country of India's
c) Only III.
network of more than 35 potential should
d) Both I and III.
lakh kilometres has develop institutions of
e) All I, II, III.
been laid out, world class research
as well.
14.

Click Here For Bundle PDF Course | support@guidely.in Page 5 of 11


SBI Clerk & RRB PO Mains PDF Course 2023
ENGLISH Day - 35

A.RBI has recently announced the withdrawal of government has invested heavily in building
2000 notes in circulation. robust architecture.
B.People should deposit all the notes in the a) Infrastructure for ramping up the country’s
banks. innovation potential.
I.Subsequently b) Infrastructure for ramp up the countries
II.Accordingly innovative potential.
III.As a result c) Infrastructure across ramping up the countries
a) Only I. innovative potential.
b) Only II. d) Infrastructure for ramp up the countries
c) Only III. innovation potential.
d) Both I and III. e) No improvement required.
e) All I, II, III.
17.The government later recognize that a major
15. pull is needed to nudge India's scientific and
A.Being the best employee in the company. technological community into the research and
B.She always wanted to be the top entrepreneur innovation mode.
in the world. a) The governments now recognizing a major
I.Besides push.
II.According to b) The government now recognize a major push.
III.Similarly c) The government now recognizes that a major
a) Only I. push.
b) Only II. d) The government is now recognized as a major
c) Only III. push.
d) Both I and III. e) No improvement required.
e) All I, II, III.
18. Within the demanding misconception of
Directions (16-20): In the following questions, a medicine, doctors find themselves exposed to
sentence is given with a phrase highlighted in numerous occupational hazards that impact the
each. The highlighted phrase is grammatically significance in well-being.
incorrect, you are required to improve the a)That significantly impacted the well-being.
highlighted phrase with the correct option. b)But will impacts the significance of well-being.
16. Realizing the importance of digital c) Which impacts the significance in well-being.
infrastructuresin ramp up the countries d) That significantly impact their well-being.
innovation potential, in the last nine years, the e) No improvement required.

Click Here For Bundle PDF Course | support@guidely.in Page 6 of 11


SBI Clerk & RRB PO Mains PDF Course 2023
ENGLISH Day - 35

19. This would help in generating prudence to 20. The introduction and use of alternatives to
anticipate the forebodings about a possible banned items led to the creation of new
threatforeither many state nor non-state actors. employmentopportunities in sustainable green
a) From either many state or non-state actor. ventures.
b) From either any state or non-state actors. a) For ban items lead to the creation of new.
c) From neither any state nor non-state actor. b) In banning item lead to the creation in new.
d) From neither any of the state or non-state c) For banning item lead to creation of the new.
actors. d) To Banned items lead to creation of the new.
e) No improvement required. e) No improvement required.
Click Here to Get the Detailed Video Solution for the above given Questions
Or Scan the QR Code to Get the Detailed Video Solutions

Answer Key with Explanation

1. Answer: B Deflate – having suddenly lost confidence.


Unleash – set free; release. Articulate –having or showing the ability to speak
Emancipate – set free from restrictions. fluently.
Bridle – hold back from or control in doing Vicious –deliberately cruel.
something. Congenial – pleasant or friendly.
Dissipate – disappear. The word “deflate” is the antonym of the given
Banish – send someone away. highlighted word “fostering”. Therefore, option A
The word “emancipate” is the synonym of the is the correct answer.
given highlighted word “unleash”. Therefore,
option B is the correct answer. 3. Answer: E
Only statements I and III are correct according to
2. Answer: A the given passage.
Fostering – encourage the development.

Click Here For Bundle PDF Course | support@guidely.in Page 7 of 11


SBI Clerk & RRB PO Mains PDF Course 2023
ENGLISH Day - 35

Statements II and IV are incorrect, both are inferred as the preventive measures of the
discussed as the negative impact of artisan sector and this is found in the last
globalization. Traditional techniques are getting passage. Therefore, option C is the correct
eroded, because of the modern day answer.
technologies.
Therefore, option E is the correct answer. 8. Answer: E
Here, all the given options are grammatically and
4. Answer: B contextually correct.
Only statement II is correct. Statement I is not The correct sentence is,
discussed anywhere in the passage, while C-D: These services not only provide an avenue
statement III is an initiative brought up by NGOs for doctors to disclose their struggles but also
not by the government organisations. Therefore, play a vital role in reducing the loss of valuable
option B is the correct answer. professionals from the workforce.
5. Answer: E A-E: These programs can help them develop a
Statement I and III are correct and true to the deeper understanding of their own mental and
passage. physical health needs, fostering a proactive
Statement II is incorrect, according to the given approach towards well-being.
context, globalisation had swept INDIA in 1991, B-F: Behind the white coats and stethoscopes,
not the world. The passage discusses India only, doctors face a range of occupational stressors
not the whole world. that put their physical and mental health at risk.
Therefore, option E is the correct answer. Therefore, option E is the correct answer.

6. Answer: D 9. Answer: A
Statement II is incorrect, the consequences Option B is incorrect, statement A says both are
faced by the artisans are they have only having face-off and the following statement E
LIMITED ACCESS to the market, it doesn't says they are not taking steps to resolve the
mean they are accessing the limited market. The issue. Both statements have continuation. So,
phrase “limited market” is incorrect. Therefore, we cannot use "but" as a connector here.
option D is the correct answer. Option C is incorrect, statement B says they
have power, the following statement says people
7. Answer: C are suffering, so this should be connected using
Here, statement II is incorrect and not given in the connector "but".
the passage. Only statement I and II can be The correct sentence is,

Click Here For Bundle PDF Course | support@guidely.in Page 8 of 11


SBI Clerk & RRB PO Mains PDF Course 2023
ENGLISH Day - 35

A-E: The government and the SGPC are having study the effects of solar winds. The lander will
a face-off and it appears that both are not doing also deploy a rover to conduct geological
enough to resolve the issue. studies.
C-D: For the AAP, which has formed the Therefore, option C is the correct answer.
government for the first time in the State that
borders Pakistan, this controversy poses yet 11. Answer: D
another challenge. Option A is incorrect, here the usage of “if” is
B-F: No doubt, this has helped the Akali Dal incorrect.
retain power in Punjab, but there have been The correct sentence is,
occasions of Sikh cases suffering at the altar of That works out to roughly $45 billion for
political exigency. subsidising a single nutrient product, whose
Therefore, option A is the correct answer. average production/import cost is in the region of
Rs 30,000 per tonne.
10. Answer: C The decision not to raise the urea MRP till 2024-
Option A is incorrect, both are contradictory 25 has, ironically, been announced along with a
statements so we should use however here. PM-PRANAM scheme that aims to promote
Option B is incorrect, here the connector "that" is alternative fertilisers and balanced plant
representing moonquakes, it will not help to nutrition.
study the effects of solar winds, so "and" should The Modi government has proposed the
be used. And is representing the instruments in introduction of sulphur-coated urea, which, it
this statement. claims, would be more economical and efficient
The correct sentence is, than the plain-vanilla fertiliser.
B-F: The Chandrayaan missions may only be the Therefore, option D is the correct answer.
first small steps in India's dreams for the moon.
However, if it wants to be a top-tier player in 12. Answer: D
lunar exploration. While – during the time you are doing something.
C-D: The more audacious Chandrayaan-2 in Option B is incorrect, the first statement says our
2019 was to deploy a lander called Vikram. improvements are from existing one, the second
However,Vikram crash landed in a spray of lunar statement says that will not be enough for world
dust, leaving it, and the small rover inside, class research. So, “but” should be used as a
inoperable. connector.
A-E: Instruments on the lander will measure Option A is incorrect, both statements should be
temperatures, search for moonquakes, and connected using “while”.

Click Here For Bundle PDF Course | support@guidely.in Page 9 of 11


SBI Clerk & RRB PO Mains PDF Course 2023
ENGLISH Day - 35

Option C is incorrect,both statements should be Accordingly – consequently.


connected using “and”. Here, statement A and B can be joined using
The correct sentence is, either accordingly or subsequently or as a result.
A-E: Most of our innovations are secondary RBI has announced the withdrawal of 2000
improvements of existing technologies. Catching notes so, people deposited the notes in the
up is not bad. But a country of India's potential banks. Therefore, option E is the correct answer.
should develop institutions of world class
research as well. 15. Answer: A
C-D: Leaders in innovation and research like Besides – in addition to.
China and the US spend massive sums on R&D. The statements can be joined using the starter
While India's private and public funding on R&D BESIDES, which means in addition to. She is the
is less than 0.7 per cent of its GDP, China best employee in the company, in addition to
spends over 2.5 per cent of its GDP on the that she always wanted to be the top
same. entrepreneur in the world.
B-F: An optical fibre cable network of more than Therefore, option A is the correct answer.
35 lakh kilometres has been laid out,and
electricity and the internet have reached almost 16. Answer: A
all villages. Here, the latter part of the sentence discusses
Therefore, option D is the correct answer. "the government" which is a singular noun, so
we cannot use countries. Also, the words
13. Answer: D "innovative" and "potential" both are adjectives, it
Despite – without being affected by. is incorrect, so innovative should be replaced
Respite – a short period of rest or relief. with innovation.
Notwithstanding – in spite of; although. Therefore, option A is the correct answer.
Statement Asays that the film didn't meet any
eligibility conditions, but it was premiered. This 17. Answer: C
can be termed as "even though". For that, The sentence particularly talks about India's
despite and notwithstanding can be used. government so usage of plural noun is
Therefore, option D is the correct answer. redundant here. Option A is incorrect, as it does
not have any B form verb for present continuous
14. Answer: E tense. Option B is incorrect, recognise should be
Subsequently – after a particular thing has replaced with recognizes. Option D is
happened. contextually incorrect, why the government

Click Here For Bundle PDF Course | support@guidely.in Page 10 of 11


SBI Clerk & RRB PO Mains PDF Course 2023
ENGLISH Day - 35

should be recognized as a major push, this Here, only two subjects are discussed, actors
question would be unanswered. Therefore, and non-state actors. Option A is incorrect, many
option C is the correct answer. should be followed by plural nouns. Option C is
incorrect, as the discussion is the possibility of
18. Answer: D threat which can damage, here neither is used,
Option B and C are incorrect, here "hazards" is a this makes the option incorrect. Option D is
plural noun, so the usage of "impacts" is incorrect, neither should take norafter it, not or.
grammatically incorrect. Option A is incorrect, Therefore, option B is the correct answer.
"significantly impacted" is grammatically
incorrect as the sentence is in present tense. 20. Answer: E
Therefore, option D is the correct answer. Here, the highlighted phrase is grammatically
and contextually correct, thus need no
19. Answer: B improvement. Hence, option E is the correct
answer.

Click Here For Bundle PDF Course | support@guidely.in Page 11 of 11


SBI Clerk & RRB PO Mains PDF Course 2023
Reasoning Ability Day - 36 (Eng)

Reasoning Ability

Directions (1-5): Study the following information c) The one who is immediately senior to Minu
carefully and answer the below questions. d) Pihu
Seven medical persons – Jenu, Sinu, Minu, Kittu, e) None of these
Pihu, Rinku, and Katty are working in an
organization with different positions viz.- HOD, 2) How many persons are senior to the one who
Chief Surgeon, Surgeon, Senior Doctor, Doctor, likes Orange?
Nurse, and Ward Assistant. The designation of a) Three
the persons is given in decreasing order such b) As many persons junior to Jenu
that HOD is the seniormost whereas ward c) No one
assistant is the juniormost. Each person likes d) Four
different fruits viz.- Mango, Apple, Orange, e) None of these
Papaya, Guava, Grapes, and Kiwi. All the
information is not necessary in the same order. 3) Who among the following person is Chief
Only two persons are junior to the one who likes Surgeon?
Kiwi. Three persons are designated between the a) Rinku
one who likes Kiwi and Pihu. Sinu is three b) Pihu
positions senior to the one who likes Mango. c) The one who likes Grapes
Kittu is not designated as Surgeon. Katty is d) The one who likes Apple
senior to Minu and the one who likes Papaya but e) None of these
not senior to Kittu. Minu likes neither Mango nor
papaya. Sinu likes neither Papaya nor Grapes. 4) Four of the following five are alike in a certain
The number of persons designated between way as per the given arrangement and thus form
Jenu and the one who likes Apple is one less a group. Find the one that doesn’t belong to that
than the number of persons senior to the one group?
who likes Guava. Rinku doesn’t like Mango. a) Doctor – Rinku
Atleast one person is designated between the b) Kittu – Grapes
one who likes Guava and the one who likes c) HOD – Orange
Grapes. . The one who likes Grapes is not d) Nurse – Jenu
designated as HOD. e) Katty – Guava
1) Who among the following person likes Apple?
a) The one who is designated as Surgeon 5) Which of the following statement is not true?
b) Jenu a) Minu is a Doctor

Click Here For Bundle PDF Course | support@guidely.in Page 1 of 10


SBI Clerk & RRB PO Mains PDF Course 2023
Reasoning Ability Day - 36 (Eng)

b) Nurse doesn’t like Orange C. The UGC had to shelve the plan due to the
c) The one who likes Kiwi is a Ward Assistant Covid-19 pandemic
d) Pihu is the HOD a) A is the Cause and C is the Effect
e) All the above statements are true b) B and C are Causes of independent Effects
c) B is the Cause and C is its Effect
Directions (6-10): Read the given passage d) A and C are Effects of independent Causes
carefully and answer the questions based on the e) C is a complete Effect and Cause statement in
same respectively itself
From the 2022-23 academic session, a common
entrance test is likely to be implemented across 7) Which of the given options can be an
central universities in India for admissions to assumption as per the given passage?
undergraduate and postgraduate courses, a) The unrealistic cutoffs for admission to
marking a departure from the current premier institutions like Delhi University have
predominant pattern of screening based on class underlined the need for a CET
12 marks. b) The students have faced difficulties in getting
On November 26, the University Grants proper coaching for getting good marks in
Commission (UGC) wrote to the vice-chancellors Boards
of the 45 central universities that “after detailed c) At present, the CUCET papers consist of two
deliberations, it was resolved that the Common segments that includes an aptitude test followed
Entrance Test for UG and PG may be conducted by domain knowledge test
for Central Universities from the academic d) There are criticisms that are being faced by
session 2022-23 through National Testing the new pattern of exams that is under
Agency. The committee’s report gave the discussion
proposal the go-ahead, but the UGC had to e) None of the above
shelve the plan due to the Covid-19 pandemic.
The latest push came on November 22, when the 8) Find the best conclusion as per the contents
UGC held a meeting with the vice-chancellors of of the passage discussed above?
45 central universities, following which the letter a) The students will find it difficult to get
was sent. admissions in colleges due to the COVID
6) Statements: pandemic
A. There are contemplations that it will create b) The Common Entrance Test will be the level
socio-economic disparities among students playing field which will help students get their
B. The committee’s report gave the proposal the deserving colleges
go-ahead

Click Here For Bundle PDF Course | support@guidely.in Page 2 of 10


SBI Clerk & RRB PO Mains PDF Course 2023
Reasoning Ability Day - 36 (Eng)

c) The year 2022-23 will be very crucial for the a) Only I


upcoming educational reforms b) Only II
d) The coaching classes will benefit from the c) Both I and II
reforms by the committee recommendations d) Either I or II
e) Both (a) and (c) e) Neither I nor II

9) Choose the option that weakens the premise Directions (11-15): Read the following
on which the CET is being proposed information carefully and answer the questions
a) The students come from different educational given below.
and social backgrounds so it would be difficult for Eight person namely-G, H, I, J, K, L, M, and N
them to compete are sitting in the linear facing north. Distance
b) The proposed changes will give rise to the between adjacent persons is a consecutive
unhealthy competition and largely benefit the integral multiple of ‘9m’ from the left end. The
coaching culture distance between adjacent persons is neither
c) The students will understand the need of less than 10m nor more than ‘80m’
logical and analytical skills and improve G sits 45m left of J. The distance between M and
themselves to compete for the seats G is twice the distance between H and K. G does
d) The level playing field will eliminate the barrier not sit to the left of M. The distance between J
that prevent certain students to get a college of and N is thrice of the distance between H and K.
their choice The number of persons sits to the left of H is the
e) Both (a) and (b) same as to the right of I. The one who sits
second to the right of N is sitting immediate right
10) Statement: The current pattern of screening of L.
based on class 12 marks is predominant which After the above arrangement, K walks 20m
creates a problem for some students to get an towards the north then took a left turn and walks
admission in their desired universities 8m to reach K’. H walks 20m towards the south
Course of Action: then took the left turn and walks 10m to reach H’.
I. The admission criteria should be made more G walks 10m towards south then took left turn
transparent and evaluation should be done and walks 40m to reach G’. J walks 15m towards
based on various parameters instead of just the the north then took the right and walks 20m to
marks reach J’. L walks 30m towards the east then took
II. A common exam or selection process should the right turn and walks 25m to reach L’.
be introduced that can help bridge the gap and
make the process look achievable

Click Here For Bundle PDF Course | support@guidely.in Page 3 of 10


SBI Clerk & RRB PO Mains PDF Course 2023
Reasoning Ability Day - 36 (Eng)

11) Who among the following person sits exactly 15) What is the direction of G’ with respect to L’
between G and the one who sits immediate right in the final arrangement?
of L before the movement? a) East
a) M b) North-west
b) The one who sits second to the right of M c) North-east
c) K d) South-east
d) The one who sits immediate left of L e) South-west
e) None of the above
Directions (16-20): Study the following
12) What is the direction and distance of K’ with information carefully and answer the questions
respect to H’ in the final arrangement? given below:
a) 20m, south An alphanumeric machine when given an input
b) 30m, north rearranges them by following a particular rule in
c) 35m, south each step. The following is an illustration of the
d) 40m, south input and steps rearrangement:
e) 40m, north Input: corona 54 pandemic 38 isolated 62
quarantine 82 containment 94
13) What is the distance between J and L before Step –I: 94 54 pandemic 38 isolated 62
the movement? quarantine 82 containment corona
a) 189 m Step-II: 94 pandemic 54 isolated 62 quarantine
b)117 m 82 containment corona 38
c) 99 m Step-III: 94 pandemic 82 54 62 quarantine
d) 162 m containment corona 38 isolated
e) None of the above Step-IV: 94 pandemic 82 quarantine 62
containment corona 38 isolated 54
14) What is the position of N from the extreme Step-V: 94 pandemic 82 quarantine 62 corona
right end? 38 isolated 54 containment
a) Fourth Step 5 is the last and final step of the
b) Third rearrangement. With the same rules followed in
c) Second the above arrangement, determine the steps for
d) Sixth the following input.
e) Fifth Input: Errors 93 Fancy 55 Mango 65 Next 62
Rose 81

Click Here For Bundle PDF Course | support@guidely.in Page 4 of 10


SBI Clerk & RRB PO Mains PDF Course 2023
Reasoning Ability Day - 36 (Eng)

16) Which of the following element is second to c) Mango


the left of “Next” in step III of the given input? d) Next
a) 55 e) Errors
b) Fancy
c) Mango 19) What is the sum of the numbers between the
d) 81 elements “Mango” and “Rose” in step III of the
e) None of the above given input?
a) 213
17) In which of the following step, the elements b) 198
“Next Fancy 62 Rose 65” obtained in the same c) 207
order? d) 239
a) Step I e) 202
b) Step II
c) Step III 20) How many numbers are there between the
d) Step IV elements “Next” and “Rose” in step IV of the
e) There is no such step given input?
a) Two
18) Which of the following element is sixth from b) Three
the left in step III of the given input? c) Four
a) 62 d) Five
b) 55 e) One
Click Here to Get the Detailed Video Solution for the above given Questions
Or Scan the QR Code to Get the Detailed Video Solutions

Answer Key with Explanation

Directions (1-5): 1) Answer: A

Click Here For Bundle PDF Course | support@guidely.in Page 5 of 10


SBI Clerk & RRB PO Mains PDF Course 2023
Reasoning Ability Day – 36 (Eng)

2) Answer: C
3) Answer: C
4) Answer: A
5) Answer: C

Again, we have:
 Kittu is not designated as Surgeon.
 Katty is senior to Minu and the one who
likes Papaya but not senior to Kittu.
We have:  Minu likes neither Mango nor papaya.
 Only two persons are junior to the one  Sinu likes neither Papaya nor Grapes.
who likes Kiwi. That means, in case (1) & case (2) Minu
 Three persons are designated between likes Kiwi.
the one who likes Kiwi and Pihu. Based on the above given information we have:
 Sinu is three positions senior to the one
who likes Mango.
That means, in case (1) Sinu is a
Surgeon, in case (2) Sinu is a senior
doctor.
Based on the above given information we have:

Again, we have:

Click Here For Bundle PDF Course | support@guidely.in Page 6 of 10


SBI Clerk & RRB PO Mains PDF Course 2023
Reasoning Ability Day – 36 (Eng)

 The number of persons designated


between Jenu and the one who likes
Apple is one less than the number of
persons senior to the one who likes
Guava.
 Rinku doesn’t like Mango.
That means, in case (1) Sinu likes Guava,
in case (1a) Sinu likes Apple, case (2) is
not valid.
Based on the above given information we have:

Case (1a) is not valid as the one who likes


Guava is designated adjacent designation of
either the one who likes Apple or Grapes.

6) Answer: E
If we look at the given sentences and read the
passage, we find that anything directly related to
A has not been discussed. So, it doesn’t fall well
Again, we have: in the Cause-Effect relationship. Also, B looks
 Atleast one person is designated between like an information and is not related to A or C as
the one who likes Guava and the one who a Cause or an Effect. So, options other than (e)
likes Grapes. can all be ruled out.
 The one who likes Grapes is not Hence, (e) is the correct answer.
designated as HOD.
That means, in case (1a) the one who is 7) Answer: A
HOD likes Orange, and Kittu likes Of all the given statements, statement (a) can be
Grapes, case (1) is not valid. counted as an assumption as it provides a base
Based on the above given information we have: for the whole idea about why a common
examination is being planned at all.
Therefore, the best option is (a).

8) Answer: B

Click Here For Bundle PDF Course | support@guidely.in Page 7 of 10


SBI Clerk & RRB PO Mains PDF Course 2023
Reasoning Ability Day – 36 (Eng)

Statement (b) can serve as the best conclusion


as the other options are too specific and can be
eliminated for being able to present a clear
closure or result to the passage.

9) Answer: E
The passage discusses the reasons and the
updates about the proposed changes in the
admissions to the elite colleges by the UGC. By
far all the positive aspects have only been
discussed which help us understand that the We have,

proposed changes are beneficial.  G sits 45m left of J.

If we look at the options, statements (a) and (b)  The distance between M and G is twice

present a critical picture by pointing out the the distance between H and K.G does not

problems that might be created due to the sit left of M.

changes. So, these would surely weaken the From the above condition, there are two

author’s contention and hence should be the possibilities.

best options.

10) Answer: C
The given problem can be lessened or solved by
applying any of the two given COAs. So, both I
and II will be the correct answer.
Hence, option (c) is correct.

Directions (11-15):
11) Answer: D
12) Answer: E
13) Answer: B
14) Answer: B Again we have,
15) Answer: B  The distance between J and N is thrice of
Final Arrangement the distance between H and K.

Click Here For Bundle PDF Course | support@guidely.in Page 8 of 10


SBI Clerk & RRB PO Mains PDF Course 2023
Reasoning Ability Day – 36 (Eng)

 The number of persons sits to the left of H


is the same as to the right of I.
 The one who sits second to the right of N
is sitting immediate right of L.
From the above condition, case2a get
eliminated. case2 shows the final arrangement.

Directions (16-20):
16) Answer: A
17) Answer: D
18) Answer: A
Again we have,
19) Answer: B
 K walks 20m towards the north then took
55 + 62 + 81 = 198
a left turn and walks 8m to reach K’.
20) Answer: E
 H walks 20m towards the south then took
For words: The words are arranged in
the left turn and walks 10m to reach H’.
alphabetical order based on the last letter. For
 G walks 10m towards south then took left
each step, one word is shifted to the left and
turn and walks 40m to reach G’.
right alternatively, starting with the word at the
 J walks 15m towards the north then took
right end in step I.
the right turn and walks 20m to reach J’.
For Number: The numbers are arranged in
 L walks 30m towards the east then took
descending order based on its digital sum. For
the right turn and walks 25m to reach L’.
each step, one number is shifted to the left and
Based on the above-given information we have
right alternatively, starting with the number at the
the final arrangement as follows.
left end in step I.
Input: Errors 93 Fancy 55 Mango 65 Next 62
Rose 81
Step- I: 93 Errors Fancy 55 Mango 65 Next 62
81 Rose

Click Here For Bundle PDF Course | support@guidely.in Page 9 of 10


SBI Clerk & RRB PO Mains PDF Course 2023
Reasoning Ability Day – 36 (Eng)

Step-II: 93 Mango Errors Fancy 55 Next 62 81 Step- IV: 93 Mango 55 Next Fancy 62 Rose 65
Rose 65 Errors 81
Step–III: 93 Mango 55 Fancy Next 62 81 Rose Step-V: 93 Mango 55 Next 62 Rose 65 Errors 81
65 Errors fancy

Click Here For Bundle PDF Course | support@guidely.in Page 10 of 10


SBI Clerk & RRB PO Mains PDF Course 2023
Quantitative Aptitude Day – 36 (Eng)

Quantitative Aptitude

Directions (1-3): Study the following information


carefully and answer the questions.
Five Shopkeepers (A, B, C, D and E) ordered different number of packets of chocolates (Eclairs +
Kopico). The given below pie chart shows the percentage distribution of number of packets of chocolates
ordered by given five shopkeepers out of total packets of chocolates ordered by all five shopkeepers
together.
Note: a) Total number of packets of chocolates ordered by given five shopkeepers is 7500.

The given below radar graph shows the percentage distribution of the number of packets of eclairs
chocolates ordered by given five shopkeepers.

Click Here For Bundle PDF Course | support@guidely.in Page 1 of 16


SBI Clerk & RRB PO Mains PDF Course 2023
Quantitative Aptitude Day - 36 (Eng)

1) Total number of packets of chocolates ordered B.34:19


by shopkeeper C is how much percent of number C.18:37
of packets of eclairs chocolates ordered by D.16:15
shopkeeper A and B together? E.32:13
A.66.67%
B.83.66% 3) If each packet of kopico chocolates costs Rs.
C.86.33% 80 then find the difference between total cost of
D.83.33% number of packets of kopico chocolates ordered
E.66.33% by shopkeeper A and C?
A.Rs. 42000
2) Find the ratio of number of packets of kopico B.Rs. 24000
chocolates ordered by shopkeeper C and E C.Rs. 48000
together to the number of packets of eclairs D.Rs. 32000
chocolates ordered by shopkeeper C? E.Rs. 30000
A.37:18

Directions (4-6): Study the following information carefully and answer the questions given below.
The given bar graph shows the number of boys and girls who are passed in the examination in three
different schools.

Click Here For Bundle PDF Course | support@guidely.in Page 2 of 16


SBI Clerk & RRB PO Mains PDF Course 2023
Quantitative Aptitude Day - 36 (Eng)

Note: The total number of students who are passed in the exam in A is 75% of the number of students
who are passed in the exam in B and the number of students who are passed the exam in C is (s – 60)%.
Total number of students who are passed in the exam in all the schools together is ‘n’. The number of
students who are passed in the exam in all the schools together is s% of the total number of students in
all the schools together which is n + 4800. Total number of students who are passed in the exam in A is
30% of the total number of students who are passed in the exam in all the schools together.
4) If the total number of students in all the B.32832
schools together is 15600, then find the C.32932
difference between the number of boys who D.32632
are passed the exam in A and B? E.32532
A.242
B.248 6) If the difference between the total number of
C.252 students who are passed the exam in A and B is
D.256 800, then find the total number of students who
E.262 are passed the exam in all the schools together?
A.9600
5) If the number of students who are failed in the B.8400
exam in all the schools together is 22800, then C.8000
find the number of boys who are passed in the D.8800
exam in C? E.None of these
A.32732

Click Here For Bundle PDF Course | support@guidely.in Page 3 of 16


SBI Clerk & RRB PO Mains PDF Course 2023
Quantitative Aptitude Day - 36 (Eng)

Directions (7-10): Study the following data carefully and answer the questions:
There are 7 workers A, B, C, D, E, F and G and efficiency of each worker is different.
TheTable given below gives the following information about the efficiencies of each worker.

Note:
1: Time taken by C and E together to complete the work is 11 hours 15 minutes.
2: Time taken by B and D together to complete the work is 20 hours.
7) If A and C started the work together and A left 9) A and G are given to complete the work. A
the work after ‘t’ hours and the remaining work is started the work alone and worked for 6 hours
completed by C and D together in 7 hours 12 and remaining work is completed by G alone. If A
minutes, then find the value of ‘t’. is paid Rs.150 per hour and G is paid Rs.250 per
A.9 hours hour, then find the total amount received by A
B.15 hours and G together for completing the work.
C.6 hours A.Rs.2100
D.12 hours B.Rs.1500
E.3 hours C.Rs.2700
D.Rs.2400
8) Find the ratio of time taken by B and C E.Rs.1800
together to complete the work to that taken by D
and E together to complete the work. 10) If E works with 90% of its original efficiency
A.14: 11 and F works with 80% of its original efficiency,
B.12: 17 then in what time E and F together will complete
C.36: 31 the work?
D.15: 19 A.15 hours
E.30: 23 B.12 hours
C.20 hours

Click Here For Bundle PDF Course | support@guidely.in Page 4 of 16


SBI Clerk & RRB PO Mains PDF Course 2023
Quantitative Aptitude Day - 36 (Eng)

D.10 hours E.Quantity I = Quantity II


E.18 hours
13) Question consists of two quantities, I and II.
Directions (11-13): Question consists of two You have to read the data carefully and answer
quantities, I and II. You have to read the data accordingly.
carefully and answer accordingly. Quantity I: Find the speed of train in m/sec, if it
Quantity I: Quantity of milk in a mixture is 150% crosses a platform in 8.4 seconds and a pole in
with respect to the quantity of honey in it. 405ml 4.4 seconds andLength of platform is 80 metres?
is taken out from the mixture and 45ml is honey Quantity II: A vendor marks up a pen at100%
is added to it, which makes the quantity of milk above CP and sold it at 90% of its MP and the
and honey in the mixture as 11:9. Find the profit is k%. Find the value of 1/4 x k?
quantity of milk initially? A.Quantity I ≥ Quantity II
Quantity II: A busis running from A to B. If its B.Quantity I = Quantity II
average speed isdecreased by (a - 10)%, there is C.Quantity I < Quantity II
an increase in the travelling time by (2a + 10)%. D.Quantity I ≤ Quantity II
Find the value of 8a? E.Quantity I > Quantity II
A.Quantity I ≥ Quantity II
B.Quantity I ≤Quantity II Directions (14-16): Read the following
C.Quantity I > Quantity II information carefully and answer the questions
D.Quantity I = Quantity II based on it.
E.Quantity I < Quantity II In cities, A, B and C, apples and oranges are
sold by the vendors. Some people buy both and
12) Quantity I: Two circles one with 20cm radius some don’t buy either any of the fruits. Also,
and another circle is of diameter 19cm; find the some people buy any one fruit.
difference in their areas? (∏ = 22/7) In city A, people who buy both apples and
Quantity II: Radius of a circle measures 40%of oranges are 40% of the people who buy only
the side of a square. Square has a 3m path oranges, and they differ by 90.People, who don’t
around it and area of circle is 1452m², find the buy any fruit, are 480 less than the people stayed
value of thrice the area of path around the in city A. People who don’t buy any fruit are 120.
square? (∏ = 3) In city B, people who buy only oranges are half
A.Quantity I ≥ Quantity II the people who buy only apples and they sum up
B.Quantity I ≤ Quantity II to 240. People who don’t buy any fruit are 62.5%
C.Quantity I < Quantity II of those who buy only apples. Total people
D.Quantity I > Quantity II staying in city B, are 2/3rd of the total people

Click Here For Bundle PDF Course | support@guidely.in Page 5 of 16


SBI Clerk & RRB PO Mains PDF Course 2023
Quantitative Aptitude Day - 36 (Eng)

staying in city A. Total women staying in city B, 15) Total people who buy both the fruits and
are half the number of women staying in city A. those who buy only apples in city A together are
Women staying in city B, are 60% of the number how much more or less than the people in city B,
of men staying in the same city. who buy both the fruits and those who buy only
In city C, 40% people buy only apples. People oranges together?
who don’t buy any fruit are 1/3rdof the people A.190
who buy both the fruits. People who buy only B.380
oranges are 160, which is half the number of C.170
people who buy only apples. Average men in D.160
cities A and C, together are 360. E.None of these
14) Total people who don’t buy any fruit in city B
are what percentage more/ less than the people 16) In city C, 35% people who buy only apples
who buy only oranges in city C? are men. Find the number of men who buy only
A.20% apples is what percent of people who buy only
B.34% oranges?
C.32.5% A.45%
D.37.5% B.50%
E.None of these C.60%
D.65%
E.None of these

Directions (17 – 20): Read the following information carefully and answer the questions based on it.
“KhanKhudi” is e – commerce websites which delivers several general items on online order. They have
two-day delivery promise otherwise the order considered as DAMAGE. An order booked on particular day
may be delivered the next or a day after. The line chart given below shows the cumulative number of
orders that were booked up to and including that day, number of orders delivered on that day, and
cumulative number of orders that were damage up to and including that day. It is known that number of
orders that were booked on the 15th, 16th and 17th day that took two days to deliver were 8, 12, and 16
respectively.

Click Here For Bundle PDF Course | support@guidely.in Page 6 of 16


SBI Clerk & RRB PO Mains PDF Course 2023
Quantitative Aptitude Day - 36 (Eng)

17) Find the number of orders that were booked after one day)/ (Number of orders booked on a
on 19th but delivered on 21st? particular day delivered after two days)] x 2.5
a) 12 Find which of the following day have highest
b) 6 balance factor?
c) 16 a) 19th
d) 8 b) 20th
e) None of these c) 17th
d) 18th
18) Find total number of orders booked on 17th? e) 21st
a) 64
b) 62 20) If number of orders damaged on 24th were 4,
c) 60 find number of orders that were delivered on 24th
d) 68 but booked on 22rd?
e) Can’t be determined a) 4
b) 2
19) Balance factor can be defined as = [(number c) 6
of orders booked on a particular day delivered d) 3
e) Can’t be determined

Click Here For Bundle PDF Course | support@guidely.in Page 7 of 16


SBI Clerk & RRB PO Mains PDF Course 2023
Quantitative Aptitude Day – 36 (Eng)

Click Here to Get the Detailed Video Solution for the above given Questions
Or Scan the QR Code to Get the Detailed Video Solutions

Answer Key with Explanation

Directions (1-3): Number of kopico chocolates ordered = 1800 –


According to the pie chart, 540 = 1260.
(a + 8 + 2a – 12 + 1.5a – 8 + 2a + 0.25a + 4) = Similarly, we find the values for other
100 shopkeepers also,
6.75a = 108
a = 16%

1) Answer: D
According to question,
Total number of packets of chocolates orders by
shopkeeper C = 1200
From radar graph,
Number of packets of eclairs chocolates of
For shopkeeper A,
shopkeeper A and B = 540+900 =1440
Total number of packets of chocolates ordered =
Required percentage = 1200/1440 × 100 =
1800.
83.33%
Number of packets of eclairs chocolate ordered
= 30/100 × 1800 = 540.
2) Answer: A

Click Here For Bundle PDF Course | support@guidely.in Page 8 of 16


SBI Clerk & RRB PO Mains PDF Course 2023
Quantitative Aptitude Day – 36 (Eng)

Number of packets of kopico chocolates ordered Total number of students who are passed in the
by shopkeeper C and E together = 660+450 = exam in A = 30%
1110 Number of students who are passed in the exam
Required ratio = 1110:540 = 37:18 in B = 30 * 100/75 = 40%
Number of students who are passed in the exam
3) Answer: C in C = 100 – 30 – 40 = 30%
Difference of Number of packets of kopico S – 60 = 30
chocolates ordered by shopkeeper A and C = S = 90%
1260 – 660 = 600 Number of students who are passed in the exam
Total cost = 600 ×80 = 48000 in all the schools together = 90/10 * 22800
= 205200
4) Answer: C Number of students who are passed the exam in
n + 4800 = 15600 C = 205200 * 30/100 = 61560
n = 10800 61560 = 7z + 8z
Total number of students who are passed in Z = 4104
school A = 30/100 * 10800 = 3240 Number of boys who are passed the exam in C =
Number of students who are passed in B = 4104 * 8 = 32832
100/75 * 3240 = 4320
x + 720 + x = 3240 6) Answer: C
2x = 2520 Total number of students who are passed the
x = 1260 exam in A = 30%
Number of boys who are passed the exam in A = Number of students who are passed the exam in
1260 + 720 = 1980 B = 30 * 100/75 = 40%
Number of girls who are passed the exam in A = 40% – 30% = 800
1260 10% = 800
2y + 3y = 4320 100% = 8000
y = 864
Number of boys who are passed the exam in B = Directions (7-10):
864 * 2 = 1728 Since, ratio of efficiency of A to C = A: C = 2: 1
Difference = 1980 – 1728 = 252 Ratio of efficiency of C to D = C: D = 3: 2
Ratio of efficiency of D to G = D: G = 2: 9
5) Answer: B So, ratio between efficiencies of A, C, D and G:
A: C: D: G = 12: 6: 4: 18 = 6: 3: 2: 9

Click Here For Bundle PDF Course | support@guidely.in Page 9 of 16


SBI Clerk & RRB PO Mains PDF Course 2023
Quantitative Aptitude Day – 36 (Eng)

And ratio between time taken by A alone, C 5y = xy


alone, D alone and G alone respectively to x = 5, y = 6
complete the work:
(18/6): (18/3): (18/2): (18/9) = 3: 6: 9: 2
Since, ratio of efficiency of B to E = B: E = 1: 2
Ratio of efficiency of E to F = E: F = 4: 3
So, ratio between efficiencies of B, E and F:
B: E: F = 4: 8: 6 = 2: 4: 3
And ratio between time taken by B alone, E
alone and F alone respectively to complete a
work:
(12/2): (12/4): (12/3) = 6: 3: 4
Let the time taken by A alone, C alone, D alone
and G alone to complete the work be ‘3x’ hours,
‘6x’ hours, ‘9x’ hours and ‘2x’ hours respectively. 7) Answer: C

Also, let the time taken by B alone, E alone and Time taken by A alone to complete the work = 15

F alone to complete the work be ‘6y’ hours, ‘3y’ hours

hours and ‘4y’ hours respectively. Time taken by C alone to complete the work =

Since, time taken by C and E together to 30 hours

complete the work = 11 hours 15 minutes = Time taken by D alone to complete the work =

(45/4) hours 45 hours

So, Let total work = LCM of 15, 30, 45 = 90 units

(1/6x) + (1/3y) = 4/45 Work done by A alone in 1 hour = 90/15 = 6

(y + 2x)/6xy = 4/45 units

15y + 30x = 8xy ------------------- (1) Work done by C alone in 1 hour = 90/30 = 3

Since, time taken by B and D together to hours

complete a work = 20 hours Work done by D alone in 1 hour = 90/45 = 2

So, hours

(1/9x) + (1/6y) = 1/20 Work done by C and D together in 7 hours 12

(2y + 3x)/18xy = 1/20 minutes:

20y + 30x = 9xy ------------------- (2) (3 + 2) * 7.2 = 36 units

By equation (2) – equation (1): And work done by A and C together in ‘t’ hours =

20y + 30x – 15y – 30x = 9xy – 8xy 90 – 36 = 54 units


So,

Click Here For Bundle PDF Course | support@guidely.in Page 10 of 16


SBI Clerk & RRB PO Mains PDF Course 2023
Quantitative Aptitude Day – 36 (Eng)

(6 + 3) * t = 54 Work done by A alone in 6 hours = 2 * 6 = 12


t = 6 hours units
So, time taken by G alone to complete the
8) Answer: A remaining work:
Time taken by B alone to complete the work = 36 (30 – 12)/3 = 6 hours
hours And the total amount received by A and G
Time taken by C alone to complete the work = together for completing the work:
30 hours (6 * 150) + (6 * 250) = 900 + 1500 = Rs.2400
Work done by B and C together in 1 hour =
(1/36) + (1/30) = 11/180 10) Answer: B
So, time taken by B and C together to complete Time taken by E alone to complete the work with
the work = (180/11) hours its original efficiency = 18 hours
Time taken by D alone to complete the work = Time taken by E alone to complete the work with
45 hours its new efficiency:
Time taken by E alone to complete the work = 18 18 * (100/90) = 20 hours
hours Time taken by F alone to complete the work with
Work done by D and E together in 1 hour = its original efficiency = 24 hours
(1/45) + (1/18) = 7/90 Time taken by F alone to complete the work with
So, time taken by D and E together to complete its new efficiency:
the work = (90/7) hours 24 * (100/80) = 30 hours
Required ratio = (180/11): (90/7) = 14: 11 Work done by E and F together in 1 hour with
their new efficiencies:
9) Answer: D (1/20) + (1/30) = (3 + 2)/60 = 1/12
Time taken by A alone to complete the work = 15 So, time taken by E and F together to complete
hours the work with their new efficiencies = 12 hours
Time taken by G alone to complete the work =
10 hours 11) Answer: E
Let total work = LCM of 15 and 10 = 30 units According to question,
Work done by A alone in 1 hour = 30/15 = 2 Quantity I,
units Let the initial quantity of milk: quantity of honey =
Work done by G alone in 1 hour = 30/10 = 3 3a:2a
units 405ml is taken out
Quantity of milk in 405ml = 243ml

Click Here For Bundle PDF Course | support@guidely.in Page 11 of 16


SBI Clerk & RRB PO Mains PDF Course 2023
Quantitative Aptitude Day – 36 (Eng)

Quantity of honey in 405ml = 162ml Radius of a circle = 40% x side of a square


45ml is honey is added Radius of a circle: side of a square = 2x:5x
Final quantity of milk = 3a – 243 ml Area of circle = 3 x (2x)²
Final quantity of honey = 2a – 162 + 45 ml √(1452/12) = x
Quantity of milk: quantity of honey = 11:9 Thus, x = 11m
(3a – 243) / (2a – 117) = 11/9 Side of the square = 55m
a = 180 Square has a 3m path around it
Quantity of milk initially = 3a = 540ml Area of path = 61² – 55² = 696m²
Quantity II, Thrice the area of path around the square =
Let the initial speed of bus = ‘S’ kmph 2088 m²
Let the initial time taken by the bus = ‘T’ hours Thus, Quantity-I < Quantity-II
A bus runs from A to B Hence, answer is option C
When the average speed decreased by (a -
10)% 13) Answer: B
Travel time increased by (2a + 10)% According to question,
Now, in both the cases distance covered is Quantity I,
constant Let the speed of train = S m/sec.
So, S x T = S x [1 - (a - 10)%] x T x [1 + (2a + Let the length of train = D m
10)%] Train crosses a platform in 8.4 seconds
a² - 55a – 1050 = 0 Length of platform = 80 metres
So, a = 70 or -15(negative values are neglected) Total distance covered while crossing the
Value of 8a = 8 x 70 = 560 platform = (D + 80)m
Thus, Quantity-I < Quantity-II S = (D + 80)/8.4
Hence, answer is option E Train crosses a pole in 4.4 seconds
S = D/4.4
12) Answer: C So, S = 20m/sec.
According to question, Speed of train = 20m/sec.
Quantity I, Quantity II,
Two circles one with 20 cm radius and another A vendor marks up a pen at100% above CP
circle is of diameter 19 cm. Vendor sold it at 90% x MP
Difference in their areas = 22/7 x [20² – 9.5²] = SP: MP = 9x: 10x
973.5cm² CP = 5x
Quantity II, Profit = k%

Click Here For Bundle PDF Course | support@guidely.in Page 12 of 16


SBI Clerk & RRB PO Mains PDF Course 2023
Quantitative Aptitude Day – 36 (Eng)

Profit = 4x/5x x 100 = 80% So, people who buy only apples = 160
Value of 1/4 x k = 1/4 x 80 = 20 People who buy only oranges = 80
Thus, Quantity-I = Quantity-II Thus,
Hence, answer is option B People who don’t buy any fruit = 100
Now, total people staying in city B = 2/3 x 600 =
Directions (14-16): 400
In city A, Thus, people who buy both the frits in city B =
People who buy both apples and oranges: 400 – (100 + 160 + 80) = 60
people who buy only oranges = 2x: 5x Total women who stay in city B: total women
As, 3x = 90 who stay in city A = 1:2
So, x = 30 Total women who stay in city B: total men who
People who buy both apples and oranges = 60 stay in city B = 3:5
People who buy only oranges = 150 Total men who stay in city B = 5/8 x 400 = 250
Total people stayed – people who don’t buy any Total women who stay in city B = 3/8 x 400 =
fruit = 480 150
People who don’t buy any fruit = 120 Total women who stay in city A = 2 x 150 = 300
Thus, total people stayed = 600 Total men who stay in city A = 600 – 300 = 300
So, people who buy only apples = 600 – (150 +
120 + 60) = 270

In city C,
Total people who buy only apples = 40%
People who don’t buy any fruit: people who buy
In city B,
both the fruits = 1c: 3c
People who buy only oranges: people who buy
People who buy only oranges = 160
only apples = 1a: 2a
People who buy only apples = 160 × 2 =320
Thus, 3a = 240
Now, 40% = 320
So, a = 80
So, 100% = 800
People who don’t buy any fruit:people who buy
Thus,
only apples = 5: 8

Click Here For Bundle PDF Course | support@guidely.in Page 13 of 16


SBI Clerk & RRB PO Mains PDF Course 2023
Quantitative Aptitude Day – 36 (Eng)

Total people who stay in city C = 800 Total = 140


Also, 4c + 320 + 160 = 800 Required difference = 330 – 140 = 190
So, c = 80 Hence, answer is option A
Thus, people who buy both the fruits = 3 x 80 =
240 16) Answer: E
People who don’t buy any fruit = 80 In city C,
(Men in city A + men in city C)/2 = 360 People who buy only apples = 320
Thus, Men who buy only apples = 35% x 320 = 112
300 + men in city C = 720 According to question,
So, Required percentage = (Men who buy only
Men in city C = 420 apples / people who buy only oranges) x 100 =
Thus, women in city C = 800 – 420 = 380 112/160 x 100 = 70%
Hence, answer is option E

Directions (17 – 20):


Till 17th, number of orders booked = 438
Till 18th, number of orders booked = 498
So, number of orders booked on 18th = 498 –
438 = 60
14) Answer: D Number of orders booked on 19th = 554 – 498 =
According to question, 56
People who don’t buy any fruit in city B = 100 Number of orders booked on 20th = 604 – 554 =
People who buy only oranges in city C = 160 50
Required percentage = 60/160 x 100 = 37.5% Number of orders booked on 21th = 654 – 604 =
Hence, answer is option D 50
Number of orders booked on 22th = 664 – 654 =
15) Answer: A 10
According to question, Number of orders booked on 23th = 674 – 664 =
People who buy both the fruits in city A = 60 10
People who buy only apples in city A = 270 Till, 17th number of damaged order = 182
Total = 330 Till, 18th number of damaged order = 184
People in city B, who buy both the fruits = 60 So, number of damaged on 18th = 184 – 182 = 2
People who buy only oranges in city B = 80 That means these two orders booked on 16th.

Click Here For Bundle PDF Course | support@guidely.in Page 14 of 16


SBI Clerk & RRB PO Mains PDF Course 2023
Quantitative Aptitude Day – 36 (Eng)

Similarly, we can find the damage for particular 60 – 30 – 24 = 6 = orders booked on 18th but
day on which the order booked. delivered on 20th
Now, we have the following scenario. Similarly, we can find the all-related data.

On 17th, number of orders delivered = 22 17) Answer: C


Out of these 22 orders that were delivered on According to question,
17th, 8 orders are those that were booked on 15th Number of orders that were booked on 19th but
(delivered after two days). delivered on 21st = 16
So, number of orders that were booked on Hence answer is option C
16thand delivered on 17th = 22 – 8 = 14
Number of orders delivered on 18th = 54 18) Answer: B
Out of 54 orders delivered on 18th, 12 orders Number of orders booked on 17th = 62
those were booked on 16th were delivered on Hence answer is option B
18th.
So, number of orders those booked on 19) Answer: D
17thdelivered on 18th = 54 – 12 = 42 Balance factor on 17th = 42/16 x 2.5 = 6.5625
So, on 17th total number of orders booked = Balance factor on 18th = 30/6 x 2.5 = 12.5
delivered after one day + delivered after two Balance factor on 19th = 16/16 x 2.5 = 2.5
days + orders that will damage after two days Balance factor on 20th = 26/20 x 2.5 = 3.25
(on 19th) but booked on 17th = 42 + 16 + 4 = 62 Balance factor on 21st = 6/26 x 2.5 = 0.6
Number of orders delivered on 19th = 46 (approx.)
Out of 46 orders, 16 orders were booked on 17th, Hence answer is option D
so orders booked on 18th those delivered on 19th
= 46 – 16 = 30 20) Answer: A
Total orders booked on 18th = delivered on 19th + Number of orders booked on 22rd = orders
delivered on 20th + damage on 20th delivered on 23th + orders delivered on 24th +
Damage on 24th

Click Here For Bundle PDF Course | support@guidely.in Page 15 of 16


SBI Clerk & RRB PO Mains PDF Course 2023
Quantitative Aptitude Day – 36 (Eng)

10 = 2 + orders delivered on 24th + 4 Hence answer is option A


So, required orders = 10 – 2 – 4 = 4

Click Here For Bundle PDF Course | support@guidely.in Page 16 of 16


SBI Clerk & RRB PO Mains PDF Course 2023
ENGLISH Day - 36

English Language

Directions (01-05): In the following question, a 1. Choose the word that is most appropriate for
passage has been given with few blanks the blank labelled 1.
indicating that some words have been omitted. a) Unleashed
You are required to fill those blanks with the b) Scourged
appropriate words from the given options. c) Frittered
RIOTING and looting raged in cities around d) Outraged
France for a fourth night despite a huge police e) None of these.
deployment and 1,311 arrests, as family and
friends prepared on Saturday to bury the 17- 2. Choose the word that is most appropriate for
year-old whose killing by police _______ (1) the the blank labelled 2.
unrest and forced President Emmanuel Macron a) Running out
to cancel an important trip abroad. France’s b) Fanned out
Interior Ministry announced the new figure for c) Bailed out
arrests around the country, where 45,000 police d) Baffled out
officers _______ (2) in a so-far unsuccessful bid e) None of these.
to quell days of violence that was triggered after
the teen’s death on Tuesday. 3. Choose the word that is most appropriate for
Despite an appeal to parents by Macron to keep the blank labelled 3.
their kids at home, street clashes between young a) Ameliorated
protesters and police raged on. About 2,500 fires b) Exasperated
were set and stores were _____ (3), according to c) Abused
authorities. d) Ransacked
The violence in France was taking a toll on e) None of these.
Macron’s international commitments. German
President Frank-Walter Steinmeir’s office said 4. Choose the word that is most appropriate for
that Macron phoned on Saturday to request a the blank labelled 4.
________ (4) of what would have been the first a) Postponement
state visit by a French president to Germany in b) Embellishment
23 years. The trip, ______ (5) to officially start on c) Impediment
Monday, would have seen Macron travel to d) Contentment
Berlin and two other German cities. e) None of these.

Click Here For Bundle PDF Course | support@guidely.in Page 1 of 8


SBI Clerk & RRB PO Mains PDF Course 2023
ENGLISH Day - 36

5. Choose the word that is most appropriate for tested a nuclear weapon. China quickly backed
the blank labelled 5. UN Security Resolution no. 1718, which directly
a) Harnessed ______ sanctions on North Korea. China
b) Snapped _________ North Korea to reduce actions that
c) Supposed might create tensions in the Korean Peninsula.
d) Reposed a) Demur, Overlooked, Enforced, Wipe out.
e) None of these. b) Diligence, Disbursed, Enticed, Reckon on.
c) Meagre, Restrained, Emanated, Break on.
Directions (06-10): In the following questions, a d) Leverage, Surfaced, Imposed, Called on.
passage has been given with multiple blanks. e) None of these.
You are required to choose the appropriate word
for each blank from the given options. 8. All three sites have very extensive remains of
6. The project will ________ with a national mining and __________ activity that occurred
conference where the findings, experiences, and through three millennia that is 3000 years of
best practices from the project will be shared history that lies in mines. The major minerals
with a wider audience. Additionally, a ________ ______ at these mines were mixed lead/zinc
teaching guide will be developed, ________ the sulphides. Zinc is currently the __________ metal
innovative methods used in the project. This extracted together with minor amounts of lead
teaching guide will be made freely available to and zinc. Till date, the three mines namely
educators across the country, ________ its Zawar, Dariba and Agucha are categorised as
accessibility and promoting widespread adoption ‘heritage mines’ by Hindustan Zinc Ltd, now part
of disability inclusive teaching practices. of the Vedanta Group, and continue to be _____.
a) Obstinate, Perspective, Faltering, Pertaining. a) Melting, Rioted, Entire, Vested.
b) Culminate, Comprehensive, Encompassing, b) Grueling, Sundered, Main, Embarked.
Ensuring. c) Renouncing, Elongated, Subsided, Asserted.
c) Annihilate, Emphasize, Striving, Replenishing. d) Smelting, Exploited, Principal, Mined.
d) Stumble, Divergence, Recurring, Denouncing. e) None of these.
e) None of these.
9. The policeman involved in shooting the boy
7. Today, China has become the biggest trade during a routine traffic stop has been charged
partner of North Korea and has the strongest with voluntary homicide and placed in provisional
_________ of Xi since his coming to power in ________; he claimed he opened fire because he
2012. But then, the thaw in the bilateral feared the boy would run him over with his car.
relationship _________ in 2006 when Pyongyang But the action taken by authorities has failed to

Click Here For Bundle PDF Course | support@guidely.in Page 2 of 8


SBI Clerk & RRB PO Mains PDF Course 2023
ENGLISH Day - 36

calm __________, forcing authorities to deploy another. Out of which, only one sentence will not
45,000 extra policemen to prevent _____. But be relevant to the given context. You are
despite nearly 500 arrests on a single night, the required to choose the odd sentence and mark
violence continues and, as is often the case the same as your answer.
when mob fury erupts, has resulted in mass 11. A. Nibhav Company wanted to keep its name
looting of stores and properties. ________, in the top list in the Lifts Manufacturing
among the places looted was a shop selling Company.
rifles. B. Nibav Home Lifts has expanded operations in
a) Detention, Tempers, Rioting, Ominously. Chennai with the opening of its fourth
b) Resentment, Elates, Stifling, Periodically. manufacturing unit.
c) Dereliction, Abstains, Venting, Occasionally. C. It also announced plans to hire 4,500
d) Concoction, Dejects, Looting, Meticulously. employees across India and overseas by
e) None of these. December.
D. It is our fourth manufacturing facility and will
10. The solution lies in Kim addressing his produce two types of home lifts.
domestic issues, especially the _________ E. None of these.
economy of his country. Instead of ________
nuclear arsenals to the world, he must focus on 12.
his own people and their endless plight. Whether A. Those in need of medical care should be
he projects his daughter or sister, the family rule transferred to Army or other suitable hospitals,
continues in this poor Asian nation. Its people will the court said.
not get any ______ from the atrocities of a B. The Bench urged the government to ensure
madman and his family. No one knows the future that displaced people are able to go back.
of millions of people in North Korea. It is ______ C. This order was given after the death of seven
when they will be free from the Kim family. pilgrimages in the religious place in North.
a) Decadent, Bolstering, Respite, Uncertain. D. It said the government should take all
b) Plummet, Demarcating, Even though, necessary steps for rehabilitation of displaced
Corporeal. persons and for protection of religious places of
c) Incessant, Mining, Although, Inferential. worship.
d) Contentment, Effacing, Despite, Steering. E. None of these.
e) None of these. 13.
A. One person sustained minor injuries after a
Directions (11-13): In the following questions, a low - intensity explosion took place in Punjab's
set of sentences is given and is related to one

Click Here For Bundle PDF Course | support@guidely.in Page 3 of 8


SBI Clerk & RRB PO Mains PDF Course 2023
ENGLISH Day - 36

Amritsar on Monday, following which security 14. Which of the following is the FIRST
was beefed up in the city. statement after rearrangement?
B. The blast occurred around 6.15 am on a a) A
heritage street near the Golden temple, close to b) B
the area where an explosion had taken place on c) C
Saturday. d) D
C. The explosion occurred around 6.15 am and e) E
our technical team is looking into the incident.
D. The bomb blast was planned by the terrorists 15. Which of the following is the FIFTH
to threaten the people in Gujarat. statement after rearrangement?
E. None of these. a) A
b) B
Directions (14-18): In the following questions, a c) C
set of sentences is given and are jumbled. You d) D
are required to rearrange the given sentences in e) E
a proper manner to form a meaningful
paragraph. 16. Which of the following is the FOURTH
A. "We should not downplay the importance of statement after rearrangement?
loneliness on physical and emotional health. a) A
B. "The quality of social contact appears to be b) B
more important for heart health in people with c) C
diabetes than the number of engagements," said d) D
by the Professor. e) E
C. The results of this study were published in the
European Heart Journal, a publication of the 17. Which of the following is the SECOND
ESC. statement after rearrangement?
D. I would encourage patients with diabetes who a) A
feel lonely to join a group or class and try to b) B
make friends with people who have shared c) C
interests." d) D
E. Researchers have found that among people e) E
with diabetes, loneliness poses a larger risk for
heart disease than diet, exercise, smoking, and 18. Which of the following is the THIRD
depression. statement after rearrangement?

Click Here For Bundle PDF Course | support@guidely.in Page 4 of 8


SBI Clerk & RRB PO Mains PDF Course 2023
ENGLISH Day - 36

a) A d) D
b) B e) E
c) C
Click Here to Get the Detailed Video Solution for the above given Questions
Or Scan the QR Code to Get the Detailed Video Solutions

Answer Key with Explanation

1. Answer: A Baffled out – completely unable to understand.


Unleashed – release; set free. The second statement says that people are
Scourged – cause great suffering to. raging due to the death of the person. To avoid
Frittered – wasting time. any misconduct or wrongdoing police personnel
Outraged – cause someone to feel angry. have been placed in the city. For that, we should
The passage is about the conflicts and outrages use "fanned out" in the blank. Therefore, option
in France due to the killing of a 17 year old boy B is the correct answer.
by the police. The first statement says police 3. Answer: D
have killed the boy which even triggered the Ransacked – damage completely.
unrest (a situation where people are angry). This Ameliorated – make something better.
forced the country's president to cancel his trip. Exasperated – intensely irritated.
From this, we can understand "unleash" will be The third statement says that fires were set. If
used for the blank. there are fires, stores will get damaged. For that,
Therefore, option A is the correct answer. the word "ransacked" should be used. Therefore,
option D is the correct answer.
2. Answer: B
Fanned out – spread to. 4. Answer: A
Running out – finish; come to an end. Embellishment – decorative detail.
Bailed out – help them out of a difficult situation. Impediment – hindrance or obstruction.

Click Here For Bundle PDF Course | support@guidely.in Page 5 of 8


SBI Clerk & RRB PO Mains PDF Course 2023
ENGLISH Day - 36

Contentment – state of happiness and


satisfaction. 7. Answer: D
In this situation the first state visit by the Here, the word "strongest" is an adjective, so a
President should be cancelled. For that, the noun should follow it. According to this logic,
word "postponement" should be used. option C gets eliminated, because "meagre" is
Therefore, option A is the correct answer. an adjective. In option B, "disbursed" is used
which is inappropriate for the term "bilateral
5. Answer: C relationship", so it gets eliminated. In option A,
Supposed – believed to be true. "wipe out" is used which is grammatically
Reposed – to rest or lie. incorrect as the sentence is in past tense.
Harnessed – control or make use of. Therefore, option D is the correct answer.
The fifth statement says due to the incident the
plan is cancelled. But, originally it had to happen 8. Answer: D
on Monday. For that, the word "supposed" Smelting – extraction of metal from its ores.
should be used in the blank. Therefore, option C Exploited – make full use of.
is the correct answer. Here, the word "and" is used, which is used to
connect two similar phrases, so "mining" should
6. Answer: B take "smelting" after it. From the mines, to
Culminate – reach a climax point; put an end. describe the major minerals "exploited" should
Encompassing – including several things. be used. If a minor amount of something is
The project should be either started or ended. In present in the material, then the question will be
the given option, we neither have the word like what is the major material. For that, the word
"start" nor any word similar to it. So, we can fix "principal" should be used. Therefore, option D is
that the blank will have the word "end", for that, the correct answer.
we can use "culminate". In addition to the
project, a guide is developed. That should be 9. Answer: A
understandable, and should include methods or Detention – act of holding back.
ideas. For that, "comprehensive" and Ominously – in a way suggesting that something
"encompassing" should be used. The fourth bad is going to happen.
blank will have "ensuring" as the project wants to The statement says that the police have killed
ensure that it is freely available to all the the boy in a traffic routine, but the police have
educators. been charged under homicide. This will lead to
Therefore, option B is the correct answer. the suspension of the police. For that, the word

Click Here For Bundle PDF Course | support@guidely.in Page 6 of 8


SBI Clerk & RRB PO Mains PDF Course 2023
ENGLISH Day - 36

"detention" should be used. 45000 personnel hospitals, it does not mean that it is to prevent
were deployed to prevent the conflict. For that, death. Therefore, option C is the correct answer.
"rioting" should be used.
Therefore, option A is the correct answer. 13. Answer: D
Here, statement D is the odd one out, as it is
10. Answer: A describing about the reasons for the bomb blast.
Decadent – characterized by or reflecting a state The remaining statements discusses only about
of moral or cultural decline. where and when the bomb blasted.
Bolstering – support or strengthen. Therefore, option D is the correct answer.
Respite – characterized by or reflecting a state of
moral or cultural decline; postpone. 14. Answer: E
The word "the" will take noun after it, so options The correct sequence is ECBAD.
B and C get eliminated as it is verb and adjective Statement C has the phrase "this study" which
respectively. In the blank 3, except "respite" we indicates that some introduction to the study is
cannot use any other words. given in the previous statements. From this, we
Therefore, option A is the correct answer. can infer statement E will come before C, also E
is the independent statement and gives an
11. Answer: A introduction to the topic which is being
Here, statement A is the odd one out. discussed. The next statement should say the
Statements B, C and D talk about the company result of the study. Statement B says that
is opening its fourth unit, it wants to hire diabetes people need to be engaged. We should
employees and the types of lifts it is producing. A take this as a serious concern. This is further
is different from the rest, and it is not related to expressed in statement D. So, BAD is the
the fourth unit, it expresses the place that it correct sequence.
wants to be in the sector. The correct sentences are,
Therefore, option A is the correct answer. E. Researchers have found that among people
with diabetes, loneliness poses a larger risk for
12. Answer: C heart disease than diet, exercise, smoking, and
Here, sentence C is the odd one as it is talking depression.
about the death of pilgrimages in North. But C. The results of this study were published in the
none of the remaining statements has mentioned European Heart Journal, a publication of the
either the place or death. The Bench has simply ESC.
said the persons should be taken to Army

Click Here For Bundle PDF Course | support@guidely.in Page 7 of 8


SBI Clerk & RRB PO Mains PDF Course 2023
ENGLISH Day - 36

B. "The quality of social contact appears to be Therefore, option D is the correct answer.
more important for heart health in people with
diabetes than the number of engagements," said 16. Answer: A
by the Professor. The correct sequence is ECBAD.
A. "We should not downplay the importance of Therefore, option A is the correct answer.
loneliness on physical and emotional health.
D. I would encourage patients with diabetes who 17. Answer: C
feel lonely to join a group or class and try to The correct sequence is ECBAD.
make friends with people who have shared Therefore, option C is the correct answer.
interests."
Therefore, option E is the correct answer. 18. Answer: B
The correct sequence is ECBAD.
15. Answer: D Therefore, option B is the correct answer.
The correct sequence is ECBAD.

Click Here For Bundle PDF Course | support@guidely.in Page 8 of 8


SBI Clerk & RRB PO Mains PDF Course 2023
Reasoning Ability Day - 37 (Eng)

Reasoning Ability
Directions (1-5): Study the following information e) F and B
carefully and answer the given questions.
A certain number of flowers were kept around the 3. If the flowers facing the centre is in Red colour
circular ring for making a garland. and the flowers facing away from the centre is in
Note: The flowers kept adjacent to each other Pink colour, then how many known flowers are
were not facing in the same direction. Atleast 20 Red and Pink respectively?
flowers were kept around the ring. a) 4, 3
R was kept third to the left of V. Only four flowers b) 3, 4
were kept between R and D, which was not Kept c) 2, 5
two places away from V. As many flowers kept d) 6, 1
between D and V as between G and M, when e) 4, 4
counted from the right of D and left of G. G was
kept three places away from V. The number of 4. Which of the following flower was kept second
flowers kept between R and G is one more than to the right of the flower which was kept fifth to
the number of flowers kept between B and F, the left of the flower which was kept third to the
when counted from the right of both G and F. F right of F?
was kept immediate left of D. Only one flower a) The one which was kept third to the right of V
was kept between F and M when counted from b) D
the right of F. B faces towards the centre. c) The one which was kept second to the right of
1. If each flower costs Rs. 10, then what will be Rs
the total cost of flowers for making a garland? d) B
a) Rs.210 e) The one which was kept fifth to the left of M
b) Rs.200
c) Rs.220 5. Which of the following statements is/are true
d) Rs.190 as per the given arrangement?
e) Rs.230 I. D faces towards the centre and three places
away from M
2. As many flowers kept between G and V as II. V and M face opposite directions
between ________. III. As many flowers kept between G and R as
a) R and M between D and M
b) M and G a) Only I
c) F and M b) Only II and III
d) B and M c) Only II

Click Here For Bundle PDF Course | support@guidely.in Page 1 of 10


SBI Clerk & RRB PO Mains PDF Course 2023
Reasoning Ability Day - 37 (Eng)

d) All I, II and III e) Can’t be determined


e) Only I and II
8. As many persons stand between J and __ as
Directions (6-10): Study the following information between B and __.
carefully and answer the given questions. a) F, I
Ten persons -A, B, C, D, E, F, G, H, I and J are b) C, D
standing in an assembly one after another. The c) A, H
person who stands at first is marked as position d) E, J
number one and the person who stands at last is e) G, F
marked as position number eleven, where one of
the positions is left vacant. 9. If C and G interchange their position, similarly
B stands six positions after G. J stands two J and F interchange their position, then how
positions before G. Only three positions are many persons stand between F and G?
between J and C. F stands two persons after C, a) One
where no position is left vacant between them. b) Two
As many persons stand before F as after D. E, c) Three
who does not stand adjacent position to C, d) More than three
stands four positions after D. As many persons e) No one
stand between E and J as between G and I. A
stands three persons before I. 10. What is the position of A with respect to G?
6. Who among the following person stands five a) Three persons before
persons after H? b) Four persons after
a) E c) Five positions after
b) The one who stands immediately after F d) Immediately before
c) J e) None of these
d) The one who stands two persons before I
e) F Directions (11-15): Study the following
information carefully and answer the given
7. Who among the following person stands questions.
exactly between H and E? In a certain code language,
a) C The consonants that appear in the first half of the
b) D English alphabetical series are coded from 0 to 9
c) F in descending order, such that B=9, C=8 and so
d) A on. Similarly, the consonants that appear in the

Click Here For Bundle PDF Course | support@guidely.in Page 2 of 10


SBI Clerk & RRB PO Mains PDF Course 2023
Reasoning Ability Day - 37 (Eng)

second half of the English alphabetical series are take the odd numbered conditions in ascending
coded from 0 to 9 in ascending order. Z is coded order into consideration.
as 1. The vowels A, E, I, O and U are codes as 11. Find the code for the word “DEMOCRATIC”?
@, $, %, # and & respectively. The codes are a) 8$0#83@5!7
written from left to right. b) 8$0#83@5$7
Below conditions are followed while coding: c) 8!0#83@5%8
Condition 1:If the first and the last letters of the d) 8!0#83@5$7
word are vowels, then both the vowels are to be e) 7!0#83@5$8
coded as the code for the third letter from the left
end of the word and that third letter of the word is 12. Find the word for the code “9!011$8%#8”?
coded as the code for its fourth succeeding letter a) Leadership
in the English alphabetical series. (Note: After Z b) References
starts with A again) c) Complexity
Condition 2:If the first and the last letters of the d) Crystalize
word are consonants, then both are interchanges e) Detergents
their code with each other.
Condition 3:If the first letter is a vowel and the 13. What is the code for the phrase “Motivation
last letter is a consonant, then that vowel is makes passion”?
coded as the code for the third succeeding vowel a) “%@5!6@750% #!61# @!4002#”
and that consonant is coded as the code for the b)“0!5%6@5%#0 4!6@0 0!40%@1”
third previous consonant letter in the English c) “0!6@5%5#5% 4!70$ 0!400#%”
alphabetical series. d) “$!6#5%1@0$ @!71@ 0!2@0#%”
Condition 4:If the second letter of the word is a e) None of these
vowel, then the second letter from the right end
of the word is coded as the code for the second 14. What is the code for the phrase “Embrace
letter of the word and the second letter of the elegance”?
word is coded as ‘!’. a)“9060@89 $1%5@08$”
Condition 5: If the exact middle letter of the word b)“9060@89 3135@083”
is a consonant, then that letter is coded as the c) “9063@89 12$6@091”
code for its fifth previous letter in the English d) “9100@99 $135@08$”
alphabetical series. e) “92@0#99 12$6$091”
Note: If more than one condition applies, then
first take the even numbered conditions and then 15. Match the words with respective codes?

Click Here For Bundle PDF Course | support@guidely.in Page 3 of 10


SBI Clerk & RRB PO Mains PDF Course 2023
Reasoning Ability Day - 37 (Eng)

iii) The unit which has an odd number of boxes


are kept in an even numbered position and vice
versa.
Toy H is kept in an odd numbered unit but above
unit number 4. The unit with 9 boxes is kept
immediately above the unit with toy H. Only three
a) 1-B, 2-C, 3-D, 4-A units are kept between the unit with 9 boxes and
b) 1-C, 2-B, 3-A, 4-D the unit with a height of 55cm. As many units
c) 1-D, 2-B, 3-A, 4-C above the unit with toy H as below the unit with
d) 1-A, 2-B, 3-C, 4-D toy E. The unit with Toy E has 5 boxes and the
e) 1-C, 2-D, 3-A, 4-B height of each box with toy E is 6 cm. The height
of the units which are kept immediately above
Directions (16-20): Study the following and immediately below the unit with Toy E is
information carefully and answer the given 28cm and 48cm respectively. Toy P is kept four
questions. units above the unit with toy G. The unit with 11
There are eight units placed one above the other boxes has toy G. The height of each box kept in
in a single stack in logistics warehouse. Each the unit with Toy Q is 6cm and is kept below unit
unit contains some boxes. The lowermost unit is number 5. The number of units between the units
numbered as 1 and the unit immediately above it with Toy Q and Toy G is one less than the
is numbered as 2 and so on. Eight different toys number of units between the unit with a height of
viz. E, F, G, H, P, Q, R and S are placed in each 27cm and the unit with toy F. The unit with 7
unit. Total height of each unit is between 25cm boxes has Toy F and the height of that unit is
and 85cm No unit has the same height and the one more than the unit with toy Q. Toy S is not
same number of boxes. The height of each box kept below unit number 5. The unit number of toy
in a unit is same. F is equal to the height of each box of the unit
Note: with toy S. The height of each box in the unit with
i) If the total height of the unit is 18cm and the toy R is 9 cm.
number of boxes in that unit is 3, then the height 16. In which of the following unit, the height of
of each box will be 6cm. each box is highest?
ii) The total height of each unit will be the multiple a) Unit 8
of the number of boxes in that unit. At least four b) The unit which has toy R
boxes but not more than eleven boxes are kept c) Unit 7
in each unit. d) The unit which has 11 boxes
e) The unit which has a height of 48cm

Click Here For Bundle PDF Course | support@guidely.in Page 4 of 10


SBI Clerk & RRB PO Mains PDF Course 2023
Reasoning Ability Day - 37 (Eng)

17. The height of the first four units from the them. If the machine accepts only those units
bottom is increased by two and the remaining which have a height more than 35cm but less
units is decreased by two, then what will be the than 70cm, then which of the following units are
total height of Units 1, 4, 7 and 8? rejected?
a) 211 cm I. Unit 4 and Unit 6
b) 223 cm II. Unit 3 and Unit 8
c) 213 cm III. Unit 7 and Unit 5
d) 191 cm a) Only I and III
e) 233 cm b) Only I
c) Only II and III
18. Which of the following toys are kept in the d) Only I and II
units whose height is highest and the lowest e) Only III
respectively?
a) H and S 20. In how many units, both the height of the
b) G and P units and the boxes in it is an odd number?
c) S and P a) One
d) E and H b) Two
e) S and R c) More than four
d) Four
19. For a proper arrangement of units in a e) Three
warehouse, they use a machine to organize
Click Here to Get the Detailed Video Solution for the above given Questions
Or Scan the QR Code to Get the Detailed Video Solutions

Answer Key with Explanation

Directions (1-5): 1. Answer: C

Click Here For Bundle PDF Course | support@guidely.in Page 5 of 10


SBI Clerk & RRB PO Mains PDF Course 2023
Reasoning Ability Day - 37 (Eng)

2. Answer: D
3. Answer: B
4. Answer: A
5. Answer: E
Final arrangement:

Again we have,
 The number of flowers kept between R
and G is one more than the number of
flowers kept between B and F when
counted from the right of both G and F.
 F was kept immediate left of D.
 Only one flower was kept between F and
We have,
M, when counted from the right of F.
 R was kept third to the left of V.
 B faces towards the centre.
 Four flowers were Kept between R and D,
Applying the above conditions, Case 2 gets
which was not Kept two places away from
eliminated because B faces away from the
V.
centre. Hence, Case 1 gives a final
Applying the above conditions, there are two
arrangement.
possibilities.

Again we have,
 As many flowers kept between D and V Directions (6-10):
as between G and M, when counted from 6. Answer: D
the right of D and left of G. 7. Answer: C
 G was kept three places away from V. 8. Answer: B
9. Answer: B
10. Answer: C
Final arrangement:

Click Here For Bundle PDF Course | support@guidely.in Page 6 of 10


SBI Clerk & RRB PO Mains PDF Course 2023
Reasoning Ability Day - 37 (Eng)

Again we have,
 F stands two persons after C, where no
position is left vacant between them.
We have,  As many persons stand before F as after
 B stands six positions after G. D.
 J stands two positions before G.  E, who does not stands adjacent position
 Only three positions are between J and C. to C, stands four positions after D.
From the above conditions, we have three While applying the above conditions, case 3 gets
possibilities: eliminated, because can’t place E.

Click Here For Bundle PDF Course | support@guidely.in Page 7 of 10


SBI Clerk & RRB PO Mains PDF Course 2023
Reasoning Ability Day - 37 (Eng)

Word: Democratic
Condition 2: 8emocrati7
Condition 4: 8!mocrat$7
Final code: 8!0#83@5$7
Hence, the final answer is option d
12. Answer: C
Complexity
Conditions 2 and 4 are applied
Condition 2: 9omplexit8
Condition 4: 9!mplexi#8
Again we have, Final code: 9!011$8%#8
 As many persons stand between E and J Hence, the answer id option c.
as between G and I.
 A stands three persons before I. 13. Answer: B
While applying the above conditions, case 1.a Conditions 2, 4 and 5 are applied
and 2 get eliminated, because can’t place A.
Thus, case 1 gives the final arrangement.

Final code: 0!5%6@5%#0 4!6@0 0!40%@1


Hence, the code for the sentence “Motivation
makes passion” is option b.

14. Answer: A
Conditions 1 and 5 are applied
9060@89$135@08$

Directions (11-15):
Hence, the code for the sentence “Embrace
11. Answer: D
elegance” is 9060@89 $1%5@08$ option a.
Conditions 2 and 4 are applied

Click Here For Bundle PDF Course | support@guidely.in Page 8 of 10


SBI Clerk & RRB PO Mains PDF Course 2023
Reasoning Ability Day - 37 (Eng)

15. Answer: E  The unit with 9 boxes is kept immediately


ARTICLE-condition 1-5R8ICL5-538%815 above the unit with toy H.
PEOPLE-condition 4- P!OP$E-1!#1$$  Only three units are kept between the unit
KIND-condition 2-7IN2-condition 4-7%02 with 9 boxes and the unit with a height of
ANCHOR-condition 3-#NCHO0--%084#0 55cm.
Hence, the answer is option e.  As many units above the unit with toy H
as below the unit with toy E.
 The unit with Toy E has 5 boxes and the
height of each box with toy E is 6 cm.
Applying the above conditions, there are two
possibilities.

Directions (16-20):
16. Answer: B
17. Answer: C
18. Answer: C
19. Answer: A
20. Answer: E
Final arrangement:

Again we have,
 The height of the units which are kept
immediately above and immediately
below the unit with Toy E is 28cm and
48cm respectively.
 Toy P is kept four units above the unit
with toy G.
 The unit with 11 boxes has toy G.

We have,  The height of each box kept in the unit

 Toy H is kept in an odd numbered unit but with Toy Q is 6cm and is kept below unit

above unit number 4. number 5.

Click Here For Bundle PDF Course | support@guidely.in Page 9 of 10


SBI Clerk & RRB PO Mains PDF Course 2023
Reasoning Ability Day - 37 (Eng)

Applying the above conditions, there is one  The unit number of toy F is equal to the
extra possibility. height of each box of the unit with toy S.
 The height of each box in the unit with toy
R is 9 cm.
Applying the above conditions, Case 2 gets
eliminated because the height of the unit with
box H is 27, so the number of boxes should be
either 9 or 3, which is not possible and Case 1a
gets eliminated there because the height of each
box of the unit in which toy R is kept is not 9 cm.
Hence, Case 1 gives a final arrangement.

Again we have,
 The number of units between the units
with Toy Q and Toy G is one less than the
number of units between the unit with a
height of 27cm and the unit with toy F.
 The unit with 7 boxes has Toy F and the
height of the unit is one more than the unit
with toy Q
 Toy S is not kept below unit number 5.

Click Here For Bundle PDF Course | support@guidely.in Page 10 of 10


SBI Clerk & RRB PO Mains PDF Course 2023
Quantitative Aptitude Day – 37 (Eng)

Quantitative Aptitude

Directions (01 - 05): Study the following information carefully and answer the questions given below.
The given table shows the number of matches played by five all-rounders, the total runs scored, the
Number of the match in which the batsman is out, the total runs conceded in bowling and the total
number of wickets taken, batting average and bowling average of five all-rounders.

Note: Batting average = [total runs scored/ number of times player was out]
Bowling average=[total runs conceded/ Number of wickets taken]
1) Ratio of the number of runs scored by players a) I+2M=68
B and D is 2:1. Find the difference between the b) 2I+M=58
batting average of player B and batting average c) I+M+25=85
of player D? d) I+M=34
a) 1 e) 3I+2M=98
b) 5
c) 0 3) B scores 800 runs more than A. Find the
d) 2 difference between the batting average of A and
e) 3 B?
a) 4.65
2) b) 5.72
I= Difference in the number of times players C c) 8.52
and E were out. d) 8.65
M= Difference between the number of wickets e) None of these
taken by C and E.
Find the relation between I and M?

Click Here For Bundle PDF Course | support@guidely.in Page 1 of 10


SBI Clerk & RRB PO Mains PDF Course 2023
Quantitative Aptitude Day - 37 (Eng)

4) Ratio of runs conceded by C and D is 2:1. 5) Ratio of runs scored by A and F is 2:3. F
Find t he average of wickets taken by all five played 50 matches and in 10 matches he
players? remains not out. F conceded 1400 runs and took
a) 83.2 60 wickets. Find the batting and bowling average
b) 84.5 of F respectively?
c) 88.3 a) 45,21.33
d) 89.3 b) 48.23.35
e) None of these c) 41,25.33
d) 44,23.33
e) None of these

Directions (06 - 10): Study the following information carefully and answer the questions given below.
The given bar graphs shows the total duration of season I and season II of four web series. Each web
series has five episodes in each season. The given table shows the duration of episode 5 [E5], duration
of episode1[E1],the total duration of episode 2 [E2] and episode 4 [E4] together, and the total duration of
episode 1 [E1] ,episode 2[E2] and episode 3 [E3] together(in hours).

Click Here For Bundle PDF Course | support@guidely.in Page 2 of 10


SBI Clerk & RRB PO Mains PDF Course 2023
Quantitative Aptitude Day - 37 (Eng)

SEASON I

SEASON II

6) Find the difference between the total duration total duration of E3 of web series C and D in
of E4 in season I of all web series and the total Season II?
duration E4 in season II of all web series? a) 81:85
a) 22 hours b) 82:77
b) 26 hours c) 96:51
c) 28 hours d) 88:89
d) 29 hours e) None of these
e) 30 hours
9) Find the difference between the total duration
7) Find the sum of the total duration of E3 of the of the last three episodes of web series C in
season I of all web series? season I and the total duration of the last three
a) 142 hours episodes of web series D in season II?
b) 158 hours a) 1hours
c) 162 hours b) 2hours
d) 149 hours c) 3hours
e) 168 hours d) 4hours
e) None of these
8) Find the ratio of the sum of the total duration
of E2 of web series A and B in season I and the

Click Here For Bundle PDF Course | support@guidely.in Page 3 of 10


SBI Clerk & RRB PO Mains PDF Course 2023
Quantitative Aptitude Day - 37 (Eng)

10) Find the sum of the total duration of episode


3 of season II of all web series together?
a) 162hours
b) 125hours
c) 142hours
d) 182hours
e) None of these

Directions (11-15): Study the following


information carefully and answer the questions
given below. There are three rows, calculate the a) ii)-b)
roots of the equation and compare them with b) i)-b)
other column. Check which one matches. c) i)-a) & ii) - c)
11) d) ii)-c)
e) None of these

13)

a) ii)-a)
b) i)-c)
c) i)-b) & iii) -a)
d) ii)-c)
e) None of these a) ii)-c)
b) i)-b)
12) c) iii)-c)
d) i)-c)
e) None of these

Click Here For Bundle PDF Course | support@guidely.in Page 4 of 10


SBI Clerk & RRB PO Mains PDF Course 2023
Quantitative Aptitude Day - 37 (Eng)

14) Directions (16-20): Study the following


information carefully and answer the questions
given below. Each of the following question given
below is followed by some statements. You must
read those statements and find which of the
following statement(s) is/are sufficient to answer
the question.
16) A, B and C start a business. Find the
investment of B?
Statement I: Total profit after one year is
Rs.3400. The share of profit of C is Rs.1200.
a) i)-a)
Statement II: Investment of A is Rs.18000. Share
b) i)-b)
profit of B is Rs.200 less than A.
c) ii) -b)
a) The data in statement I alone is sufficient to
d) iii)-c)
e) None of these answer the question, while the data in statement
II alone is not sufficient to answer the question
b) The data in statement II alone is sufficient to
15)
answer the question, while the data in statement
I alone is not sufficient to answer the question
c) The data either in statement I alone or in
statement II alone is sufficient to answer the
question
d) The data given in both statements I and II
together are not sufficient to answer the question
e) The data given in both statements I and II
together are necessary to answer the question.

a) iii)-a) 17) Find in how many days A, B, and C complete


b) i)-b) the work?
c) i)-a) & ii) -b) Statement I: When A and B work on an
d) iii)-c) alternative day starting with A, the total work is
e) None of these completed in 12 days. A is double efficient as C.
Statement II: B can complete the work in 30
days. The ratio of efficiency of B and C is 4:3. A

Click Here For Bundle PDF Course | support@guidely.in Page 5 of 10


SBI Clerk & RRB PO Mains PDF Course 2023
Quantitative Aptitude Day - 37 (Eng)

and C together can complete the work in 40/3 e) The data given in both statements I and II
days. together are necessary to answer the question.
a) The data in statement I alone is sufficient to
answer the question, while the data in statement 19) Find the speed of the car?
II alone is not sufficient to answer the question Statement I: Ram covers the distance from A to
b) The data in statement II alone is sufficient to B on the bike in 5 hours while Shyam covers the
answer the question, while the data in statement same distance by a car in 4 hours.
I alone is not sufficient to answer the question Statement II: The ratio of the speed of the bus
c) The data either in statement I alone or in and bike is 2:3.The speed of a bus is 40 km/hr
statement II alone is sufficient to answer the which covers 40% distance of A to B in 3 hours.
question a) The data in statement I alone is sufficient to
d) The data given in both statements I and II answer the question, while the data in statement
together are not sufficient to answer the question II alone is not sufficient to answer the question
e) The data given in both statements I and II b) The data in statement II alone is sufficient to
together are necessary to answer the question. answer the question, while the data in statement
I alone is not sufficient to answer the question
18) Find the cost price of the watch? c) The data either in statement I alone or in
Statement I: The shopkeeper marked up the statement II alone is sufficient to answer the
watch by 50% above the cost price and sold it at question
20% discount and makes 20% as profit. d) The data given in both statements I and II
Statement II: If the shopkeeper gives a 10% together are not sufficient to answer the question
discount, then there is a profit of 35%. The cost e) The data given in both statements I and II
price of the watch is more than Rs.300. together are necessary to answer the question.
a) The data in statement I alone is sufficient to
answer the question, while the data in statement 20) Find the sum of the ages of A and B?
II alone is not sufficient to answer the question Statement I: The ratio of the age of B and D is
b) The data in statement II alone is sufficient to 5:3 and the Ratio of the age of A and C is 3:2.
answer the question, while the data in statement Sum of the age of C and D is 22 years.
I alone is not sufficient to answer the question Statement II: The difference in age between A
c) The data either in statement I alone or in and B is 5 years. The age of B after 7 years is 27
statement II alone is sufficient to answer the years.
question a) The data in statement I alone is sufficient to
d) The data given in both statements I and II answer the question, while the data in statement
together are not sufficient to answer the question II alone is not sufficient to answer the question

Click Here For Bundle PDF Course | support@guidely.in Page 6 of 10


SBI Clerk & RRB PO Mains PDF Course 2023
Quantitative Aptitude Day - 37 (Eng)

b) The data in statement II alone is sufficient to d) The data given in both statements I and II
answer the question, while the data in statement together are not sufficient to answer the question
I alone is not sufficient to answer the question e) The data given in both statements I and II
c) The data either in statement I alone or in together are necessary to answer the question.
statement II alone is sufficient to answer the
question
Click Here to Get the Detailed Video Solution for the above given Questions
Or Scan the QR Code to Get the Detailed Video Solutions

Answer Key with Explanation

1) Answer: C Batting average of A is =1200/35=34.28


Runs scored by player D is = 25*40=1000 Batting average of B is =[1200+800]/50=40
Runs scored by player B is =1000*2/1=2000 So, the difference is 40-34.28=5.72
Batting average of player B is=2000/50=40
So, the difference is 40-40=0 4) Answer: A
Runs conceded by D is =2400/2=1200
2) Answer: D Wicket taken by D is =1200/30=40
The number of times C was out is =2800/40=70 Total wicket taken by five players is
The number of times E was out is =1800/30=60 =[70+90+(2400/20)+(40)+(2400/25)]=416
I=70-60=10 So, the average is 416/5=83.2
Wicket taken by C is = 2400/20=120
Wicket took by E is=2400/25=96 5) Answer: E
So, M=120-96=24 Run score by F is =1200*3/2=1800
So, I+M=10+24=34 Batting average of F is 1800/(50-
10)=1800/40=45
3) Answer: B The bowling average of F is 1400/60=23.33

Click Here For Bundle PDF Course | support@guidely.in Page 7 of 10


SBI Clerk & RRB PO Mains PDF Course 2023
Quantitative Aptitude Day – 37 (Eng)

Directions (06 - 10): Required difference =[45+30+40]-


For the season I in web series A, [30+42+42]=1hours
Duration of E1+E2+E3+E4=217-40=177hours
Duration of E4=177-132=45hours 10) Answer: A
Duration of E2=85-45=40hours Required sum =[45+40+32+45]=162hours
Duration of E1+E3=132-40=92hours
Duration of E3=92-50=42hours 11) Answer: C
Similarly, we can calculate all others values of all X2-6X-16=0
other web series. Or, X2-8X+2x-16=0
Or, (X-8)(X+2)=0, X=8,-2
X2+13X+40=0
Or, X2+8X+5X+40=0
Or, (X+8)(X+5)=0, X=-8,-5
X2-8X+15=0
Or, X2-5X-3x+15=0
SEASON II Or, (X-5)(X-3)=0
Or, X=5,3
So, options C is true

12) Answer: C
X2-29X+210=0
Or, X2-14X-15X+210=0
6) Answer: C Or, (X-14)(X-15)=0
Required difference =[45+44+42+45]- Or, X=14,15
[40+40+38+30]=28hours X2-8X+15=0
Or, X2-5X-3X+15=0
7) Answer: D Or, (X-5)(X-3)=0
Total sum = [42+35+30+42] =149hours Or, X=5,3
X2-11X+30=0
8) Answer: B Or, X2-6X-5X+30=0
Required ratio =[40+42]:[32+45]=82:77 Or, (X-6)(X-5)=0, X=6,5
So, options C is true
9) Answer: A

Click Here For Bundle PDF Course | support@guidely.in Page 8 of 10


SBI Clerk & RRB PO Mains PDF Course 2023
Quantitative Aptitude Day – 37 (Eng)

13) Answer: A Or, (X+29)(X+6)=0, X=-29,-6


X2-10X+16=0 So, options B is true
Or, X2-8X-2x+16=0
Or, (X-8)(X-2)=0, X=8,2 16) Answer: E
X2-16X+63=0 From I, the total share of profit A and B is 3400-
X2-9X-7X+63=0 1200=Rs.2200
Or, (X-9)(X-7)=0, X=9,7 From II, the difference in the share of profit of A
X2-17X+60=0 and B is Rs.200
Or, X2-12X-5X+60=0 So, from I and II, the share of profit of A and B is
Or, (X-12)(X-5)=0, X=12,5 [2200+200]/2=1200 and [2200-200]/2=Rs.1000
So, options A is true respectively.
So, the ratio of the share of profit is 6:5:6.
14) Answer: C So, B’s investment is 18000*5/6=Rs.15000
X2+12X-85=0
Or, X2+17X-5X+85=0 17) Answer: B
Or, (X+17)(X-5)=0, X=-17,5 From statement I, we can't calculate the answer
X2-20X+64=0 because no direct data is given,
Or, X2-16X-4X+64=0 From statement II, the number of days taken By
Or, (X-16)(X-4)=0, X=16,4 C is 30*4/3=40days
X2+9X-52=0 A’s one day’s work is =3/40-1/40=2/40=1/20
Or, X2+13X-4x-52=0 So, A, B, and C can alone complete the work in
Or, (X+13)(X-4)=0, X=-13,4 20,30 and 40 days.
So, options C is true So, we can easily calculate the answer.
Only II is necessary.
15) Answer: B
X2+14X+49=0 18) Answer: D
Or, X2+7X+7X+49=0 From statement, I, the Marked up percentage is
Or, (X+7)(X+7)=0, X=-7,-7 50% above the cost price
X2-22X+85=0 From statement II, the Cost price of the watch is
Or, X2-17X-5X+85=0 more than Rs.300.
Or, (X-17)(X-5)=0, X=17,5 So, from the given data we cannot calculate the
X2+35X+174=0 cost price of the watch.
Or, X2+29X+6X+174=0

Click Here For Bundle PDF Course | support@guidely.in Page 9 of 10


SBI Clerk & RRB PO Mains PDF Course 2023
Quantitative Aptitude Day – 37 (Eng)

19) Answer: E From statement II,


From each statement alone we cannot get the Age of B is 27-7=20 years,
answer, but from II, we got the distance of A to B Age of A is either 25 years or 15 years.
i.e., [40*3]*100/40=300 From I, we can calculate the sum of the age of A
So, speed of car is =300/4 km/hr and B.
So, both statements are required. But we use both statements' data we can see, if
the age of A is 15 then the ratio given in I is
20) Answer: E satisfied, so both statements are necessary.

Click Here For Bundle PDF Course | support@guidely.in Page 10 of 10


SBI Clerk & RRB PO Mains PDF Course 2023
ENGLISH Day - 37

English Language

Directions (1-7): Read the following passage and both current and future generations. By contrast,
answer the questions that follow. with appropriate prescience and planning,
Population ageing is a defining global trend of Governments can manage the challenges from
our time. People are living longer, and more are population ageing while enhancing opportunities
older than ever before. Spectacular for all people to thrive and ensuring that no one
improvements in health and survival and is left behind. Population ageing needs to be
diminution in fecundity have driven this widely understood as more than just a set of
momentous shift, which has begun or is discrete concerns mainly for one group of people
expected to begin soon in all countries and who have advanced beyond a given age. Ageing
areas. This change brings both challenges and touches all parts of economies and societies,
opportunities as countries strive to achieve the from health care and education to employment
Sustainable Development Goals (SDGs). In and taxation. Each stage of life can contribute to
2022, the world marked the twentieth anniversary or detract from well-being at older ages.
of the adoption of the Madrid International Plan Population ageing signals our extraordinary
of Action on Ageing. To commemorate this collective success in improving living conditions
landmark, the World Social Report 2023 explores for billions of people around the world. Better
the economic and social entailments of the sanitation and medical therapies, greater access
ageing of the human population. It builds on the to education and family planning, and strides
Plan of Action’s framework for national policies to towards gender equality and women’s
create equitable, compendious societies for empowerment have all contributed to, and in
people of all ages, providing recommendations to some cases benefitted from, the steady move
put the rights and well-being of older persons at from high to low levels of fertility and mortality.
the center, across the life course. Population These advances have ushered in an era where
ageing is an ineluctable result of the rapid population growth is slowly coming to an
demographic transition towards longer lives and end, accompanied by a gradual but permanent
smaller families. While the shift towards older shift towards older ages. Over several decades,
populations is largely irredeemable, collective both the number and population share of older
actions and policy decisions shape its path and persons have risen globally, while the number
consequences. Postponing critical measures that and share of children and youth have begun to
allow societies to benefit from and adapt to shrink. By 2050, the number of persons aged 65
population ageing would impose high social, years or older is expected to double, surpassing
economic, fiscal and health-related costs, for 1.6 billion. Currently, population ageing is

Click Here For Bundle PDF Course | support@guidely.in Page 1 of 14


SBI Clerk & RRB PO Mains PDF Course 2023
ENGLISH Day - 37

furthest along in Europe and Northern America, occupational hazards that are more common
Australia and New Zealand, and most of Eastern among men and people with limited income and
and South-Eastern Asia. In most countries of education. In 2020, the World Health
those regions, the proportion of older persons – Organization and the United Nations designated
by convention, those aged 65 years or older – 20212030 as the Decade of Healthy Ageing. Its
exceeds 10 per cent and in some cases 20 per purpose is to promote strategies, grounded in
cent of the total population. Most parts of sub- solid evidence, that support well-being among
Saharan Africa and Oceania (excluding Australia older people. It advocates for developing and
and New Zealand) are still in an early stage of maintaining functional abilities, recognizing that
this transition, while most countries in Central these depend on each individual’s immanent
and Southern Asia, Western Asia and Northern capacity, the surrounding environment and
Africa, and Latin America and the Caribbean are interactions between the two.
at an intermediate stage. Declining mortality 1. What is the tone of the passage?
throughout the life course has driven the a. subjective
increase of life expectancy at birth in most b. satire
countries and globally. Greater longevity has c. jocular
accompanied a narrowing of the age range in d. critical
which most deaths occur. In the past, death was e. None of these
common at all ages. Many children died from
infectious diseases, for example, and women 2. Which of the following is/are reason/reasons
frequently perished in childbirth. In most for people to live longer than before according to
countries today, “premature death” before age 60 the passage?
or 70 is relatively rare. Greater global life a. Reduction in the ability of the women to bear
expectancy reflects underlying improvements in and produce offspring.
health. In countries with available data, the b. Striking advancements in the fields of health
number of years lived in good health has and survival.
climbed, accounting for most of the increase in c. Decreased prevalence of death-inducing and
years lived overall. Statistical averages hide grievous diseases amongst mankind.
broad disparities in life expectancy, however, d. Only a and b
including by sex and socioeconomic status. In e. Options a, b, and c
almost all societies, women live longer than men
on average, and the rich longer than the poor. 3. Which of the following statement/statements
These differences stem partly from poor nutrition is/are TRUE according to the passage?
and exposures to environmental and

Click Here For Bundle PDF Course | support@guidely.in Page 2 of 14


SBI Clerk & RRB PO Mains PDF Course 2023
ENGLISH Day - 37

a. The World Social Report 2023 digs into the 6. Which of the following is/are the
economic and social implications of the ageing of synonym/synonyms of the word ‘prescience’?
human population to commemorate the a. blemishes
anniversary of the Madrid International Plan of b. excrescencies
Action on Ageing which was adopted in 2022. c. dereliction
b. Population ageing is an avoidable result of the d. foresight
demographic transition towards longer lives and e. blotches
smaller families.
c. The shift towards older populations is largely 7. Which of the following is/are the
irreversible. synonym/synonyms of the word ‘immanent’?
d. Only a and c a. inexorable
e. Options a, b, and c b. inadvertent
c. innate
4. Which of the following statement/statements d. inept
is/are FALSE according to the passage? e. None of the above
a. The advances in the living conditions for
billions of people around the world have paved Directions (8-12): Each of the following questions
the way to an era where rapid growth in has a sentence that is divided into 4 parts. One
population is slowly coming to a halt. of these parts may/may not contain a
b. Longer life-span has resulted in the shrinking grammatical error in it. Identify the part that is
of the age range between which most deaths are grammatically incorrect. If all the parts of the
recorded to occur. sentence are grammatically right, then mark
c. Countries like Australia and New Zealand are option ‘e’.
in the early stage of population ageing. 8. Kiran had been trying to (A)/ lead a happy and
d. Options a and c peaceful life since (B)/ he had less problems
e. None of the options a, b, and c is false when (C)/ compared to the others around him
(D)/ No error (E).
5. Which of the following is/are the a. A
antonym/antonyms for the word ‘compendious’? b. B
a. embracive c. C
b. panoramic d. D
c. comprehensive e. E
d. exhaustive
e. None of the above

Click Here For Bundle PDF Course | support@guidely.in Page 3 of 14


SBI Clerk & RRB PO Mains PDF Course 2023
ENGLISH Day - 37

9. Tanya knew very little of the abilities and (A)/ interior designing, and many more (D)/ No error
competencies of the client with (B)/ who she had (E).
promised to venture into (C)/ new business deals a. A
for the next few months (D)/ No error (E). b. B
a. A c. C
b. B d. D
c. C e. E
d. D
e. E Directions (13-17): In the following questions two
columns are given containing three
10. Vimal is not cognizant of no ways (A)/ to get Sentences/phrases each. A sentence/phrase
the job done quickly (B)/, but he is not willing to from the first column may or may not connect
(C)/ outsource professionals for the same (D)/ No with another sentence/phrase from the second
error (E). column to make a grammatically and
a. A contextually correct sentence. Each question has
b. B five options, four of which display the
c. C sequence(s) in which the sentences/phrases can
d. D be joined to form a grammatically and
e. E contextually correct sentence. If none of the
options given forms
11. Meena worked in her warehouse for (A)/ a correct sentence after combination, mark
hours yesterday, and thus, she (B)/ was unable option (e), i.e. “None of the above” as your
to make it to her appointment (C)/ with a new answer.
client on time (D)/ No error (E). 13.
a. A Column I Column II
b. B A. Markets regulator D. after considering
c. C SEBI barred IIFL some other factors that
d. D Securities, one of the are mentioned
e. E leading domestic elaborately in the order
broking houses, released in June this
12. The team will attend a meeting from noon year.
(A)/ to early evening tomorrow to discuss the B. SEBI had E. from taking on new
pros (B)/ and cons of investing a portion of considered cancellation clients for the next two
company shares (C)/ in ventures like real estate,

Click Here For Bundle PDF Course | support@guidely.in Page 4 of 14


SBI Clerk & RRB PO Mains PDF Course 2023
ENGLISH Day - 37

of the certificate of the years for violating the Swadesh tourism globally while giving
IIFL securities as a regulatory forms. that acknowledges special attention to the
broken but refrained guidelines for tourism industry of their
from doing that sustainable tourism countries.
C. IIFL Securities has F. using funds of one a. A-D, B-E, C-F
been penalized for client for meeting the b. A-F, B-E, C-D
indulging in unfair obligations of another. c. A-E, B-D, C-F
trading practices and d. A-F, B-D, C-E
a. A-D, B-E, C-F e. None of the above
b. A-F, B-E, C-D
c. A-E, B-D, C-F 15.
d. A-F, B-D, C-E Column I Column II
e. None of the above A. Describing Prime D. and talks about how
Minister Modi’s visit to defense industrial
14. the U.S. this June as a ecosystems could
Column I Column II milestone in India-U.S. cooperate much
A. Union tourism D. and focuses on relations, foreign better.
minister G Kishan shifting the contribution secretary said a
Reddy proposed that of its sectors towards roadmap for defense
the G20 members green, inclusive, and industrial cooperation
should take concrete circular economic B. The defense E. before departing for
steps growth. roadmap essentially Washington D.C.
B. While addressing E. should focus on focusses on aspects of where he held bilateral
the fourth G20 tourism green initiatives, defense co-production talks with the U.S.
working group meeting, inclusive strategies, and co-development president and
G Kishan Reddy and circular economic addressed a joint
proposed that the growth to promote sitting of the U.S.
tourism industry sustainability and congress.
renewable energy C. The prime minister F. will be among the
management. met some prominent key outcomes of what
C. The Indian tourism F. to drive a collective personalities in New will be Modi’s first
industry has enforced a change in how plastic York and participate in state visit to the
national strategy called pollution can be tackled a capacity-building country and his

Click Here For Bundle PDF Course | support@guidely.in Page 5 of 14


SBI Clerk & RRB PO Mains PDF Course 2023
ENGLISH Day - 37

event seventh as PM. c. A-E, B-D, C-F


a. A-D, B-E, C-F d. A-D, B-F, C-E
b. A-F, B-E, C-D e. None of the above
c. A-E, B-D, C-F
d. A-F, B-D, C-E 17.
e. None of the above Column I Column II
A. Victims of extensive D. but the construction
16. burns and acid attack of the same was
Column I Column II patients with deep, delayed due to the
A. The state D. in Srinagar, severe scars now sudden outbreak of the
investigation agency of Anantnag, Pulwama, have a fresh ray of pandemic three years
Jammu and Kashmir and Kupwara districts hope ago.
police carried out in a case pertaining to B. The proposal to set E. in the form of a
searches at several the misuse of social up a skin bank at the dedicated skin bank at
locations media platforms for Safdarjung hospital at center-run Safdarjung
unlawful and New Delhi was hospital in the nation’s
secessionist activities. initiated in 2012 after capital New Delhi.
B. The identified E. in Anantnag have gaining the approval
entities are suspected been found targeting of the health ministry
of conniving with government servants C. The initiative is the F. which are majorly
foreign associates to by hindering their first-of-its-kind in located in Maharashtra,
further their nefarious ability to perform their Delhi-NCR and north Odisha, Madhya
agenda and specifically lawful duties. India, while presently Pradesh, and the
the entities India retains 16 skin southern states like
C. The searches led to F. would be banks Tamil Nadu and
the seizure of meticulously analyzed Karnataka.
substantial digital and to build a strong case a. A-D, B-E, C-F
physical evidence, against the accused b. A-F, B-E, C-D
including mobile entities. c. A-E, B-D, C-F
phones and sim cards, d. A-D, B-F, C-E
which e. None of the above
a. A-D, B-E, C-F
b. A-F, B-E, C-D

Click Here For Bundle PDF Course | support@guidely.in Page 6 of 14


SBI Clerk & RRB PO Mains PDF Course 2023
ENGLISH Day - 37

Directions (18-22): Given below is a word that is C. Free healthcare and education were of little
followed by 3 sentences. Each of the three comport amongst other obstacles when food
sentences includes the word. Identify the rationing and poverty were all around.
sentence/sentences that best express/expresses a. Only A and B
the meaning of the word. Choose option e when b. Only B and C
the word is not suitable for any of the sentences. c. Only C
18. HOBBLE d. All statements A, B, and C
A. Poverty not only demeans our society but its e. None of the sentences A, B, and C is right
cost also hobbles the growth of our economy to a
significant extent. 20. DEPLORE
B. Sita picked up her crutches and hobbled A. We, as humans, should deplore the
across the room since she had retained a serious development of certain inventions like nuclear
knee injury last week. weapons to guarantee peace and harmony for
C. Mules and horses typically required herding the future generations.
day and night and often had to be staked out on B. Early mornings and late evenings are some of
a rope or hobbled to keep them from straying or the best parts of the day to deplore the town and
wandering. get acquainted with its scenic beauty.
a. Only A C. The leader deplored his actions and wilfully
b. Only B accepted to have created a toxic environment
c. Only C and A through his incompetent and biased governance.
d. Options A, B, and C a. Only C
e. None of the sentences A, B, and C is right b. Only A and B
c. Only A and C
19. COMPORT d. Only B and C
A. The employee was fired from his position e. None of the sentences A, B, and C is right
yesterday since his actions did not comport with
the guidelines and code of conduct of the 21. RESCIND
company. A. Match tapes are routinely studied after the
B. The grieving relatives comported themselves final whistle to either get red cards rescinded on
with grace and dignity during the funeral appeal or charges brought for malicious tackles
proceeding and briefly expressed their that the officials miss.
condolences to the family of the deceased to let B. The memories of Hera’s early childhood
them mourn in peace and solitude. rescinded as she grew older and became mature
with time.

Click Here For Bundle PDF Course | support@guidely.in Page 7 of 14


SBI Clerk & RRB PO Mains PDF Course 2023
ENGLISH Day - 37

C. With the right combination of a proven B. Because our company just recently opened its
treatment plan and natural remedies, a doors, we are inchoate and are not offering all of
rescinding hairline can often grow back. our services yet.
a. Only C C. The designs of the newly launched cars
b. Only B and C successfully inchoate art and technology.
c. All statements A, B and C are correct a. Only C
d. Only A b. Options A and B
e. None of the statements A, B, C is right c. Options A and C
d. All statements A, B, and C
22. INCHOATE e. None of the statements A, B, and C is right
A. Since the power went out in the building, the
electrical service has been inchoate, leaving
many floors without lights.
Click Here to Get the Detailed Video Solution for the above given Questions
Or Scan the QR Code to Get the Detailed Video Solutions

Answer Key with Explanation

1. Answer: E Jocular - When the author is humorous, he/she


Subjective - A subjective tone uses words that tries to make the context of the passage funny
describe feelings, judgments, or opinions. The and amusing.
details are likely to include experiences, senses, Critical - A fault-finding attitude of the author is
feelings, and thoughts. frequently described by a critical tone, hence in a
Satire - uses sarcasm or irony to make fun of negative meaning.
something. The tone of the passage does not match with
any of the tones mentioned above. Hence,
option e is the right answer.

Click Here For Bundle PDF Course | support@guidely.in Page 8 of 14


SBI Clerk & RRB PO Mains PDF Course 2023
ENGLISH Day - 37

2. Answer: D with the statement mentioned in the passage.


In the first paragraph of the passage, there is a Thus, option b is incorrect.
line which states that ‘People are living longer, In the first paragraph of the passage, there is a
and more are older than ever before. line which states that ‘the shift towards older
Spectacular improvements in health and survival populations is largely irredeemable’, here
and diminution in fecundity have driven this irredeemable means irreversible. Thus, option c
momentous shift’. Here, diminution means a is correct.
decrease or reduction and fecundity means the Only option c is true.
ability to produce offspring. Thus, options a and
b are the reasons for the momentous shift. Thus, 4. Answer: C
option d is the right answer. Option a is true. There is a line in the second
paragraph of the passage that states that
3. Answer: C ‘Population ageing signals our extraordinary
In the first paragraph of the passage, there is a collective success in improving living conditions
line which states that ‘In 2022, the world marked for billions of people around the world. Better
the twentieth anniversary of the adoption of the sanitation and medical therapies, greater access
Madrid International Plan of Action on Ageing. to education and family planning, and strides
To commemorate this landmark, the World towards gender equality and women’s
Social Report 2023 explores the economic and empowerment have all contributed to, and in
social entailments of the ageing of the human some cases benefitted from, the steady move
population’. Here, explores means to inquire or from high to low levels of fertility and mortality.
dig into; and entailments means implications. It These advances have ushered in an era where
is also given than the Madrid international plan rapid population growth is slowly coming to an
celebrated its 20th anniversary in 2022. Thus, it end’. Here the advances refer to the ones
should have been adopted in 2002. Thus, option described in the second line of the text within the
a in inorrect. inverted commas. Here, urged means to steer or
In the first paragraph of the passage, there is a pave the way for something. Thus, option a is
line which states that ‘Population ageing is an true.
ineluctable result of the demographic transition Option b is true. There is a line in the second
towards longer lives and smaller families’. Here, paragraph of the passage that states that
ineluctable means impossible to avoid or evade. ‘Greater longevity has accompanied a narrowing
But the statement in option c does not match of the age range in which most deaths occur’.
Here longevity means the time period during

Click Here For Bundle PDF Course | support@guidely.in Page 9 of 14


SBI Clerk & RRB PO Mains PDF Course 2023
ENGLISH Day - 37

which something exists or lasts or lifespan. Only option d is synonymous to prescience.


Thus, option b is true. Thus, option d is the right answer.
Option c is false. There is a line in the second
paragraph of the passage which states that 7. Answer: C
‘Most parts of sub-Saharan Africa and Oceania Immanent (adjective) – as in inherent; being a
(excluding Australia and New Zealand) are still in part of the innermost nature of a person or thing
an early stage of this transition’. Here, the Innate (adjective) – as in inherent; being a part of
countries Australia and New Zealand are the innermost nature of a person or thing
excluded from the list of countries in the early Inexorable (adjective) – as in inevitable; as in
stage of this transition – population ageing. inevitable
Thus, option c is false. Inadvertent (adjective) – Happening by chance,
Thus, option c is the right answer. unexpectedly or unintentionally
Inept (adjective) - Generally incompetent and
5. Answer: E ineffectual
Compendious, embracive, panoramic, Only option c is synonymous to the word
comprehensive, and exhaustive are all immanent. Thus, option c is the right answer.
adjectives that unanimously mean to cover or
include everything or all important points. Thus, 8. Answer: C
none of these options is an antonym. The use of the quantifier is wrong in part C.
Thus, option e is the right answer. ‘Fewer’ means a quantifiable number and thus, it
is used to denote a definite number but, ‘Less’ is
6. Answer: D used in a non-quantifiable situation to denote
Prescience (noun) - T he special ability to see or that something which is abstract is less in size or
know about events before they actually occur amount. Thus, ‘less’ should be replaced by
Foresight (noun) - T he special ability to see or ‘fewer’ is part C to make the sentence
know about events before they actually occur grammatically right.
Blemishes (noun) - something that spoils the The right sentence is:
appearance or completeness of a thing; defects Kiran had been trying to lead a happy and
Blotches (noun) - something that spoils the peaceful life since he had fewer problems when
appearance or completeness of a thing; defects compared to the others around him.
Excrescencies (noun) - something that spoils the
appearance or completeness of a thing; defects 9. Answer: C
Dereliction (noun) - Wilful negligence

Click Here For Bundle PDF Course | support@guidely.in Page 10 of 14


SBI Clerk & RRB PO Mains PDF Course 2023
ENGLISH Day - 37

Part C is grammatically incorrect. ‘Who’ is a in the past but the use of tense is incorrect
subjective pronoun, whereas ‘whom’ is an (simple past tense is used). Thus, ‘worked’
objective pronoun. If the word, ‘he’ can be should be replaced by ‘was working’.
substituted into the sentence, use ‘who’. If the The right sentence is:
word ‘him’ can be substituted into the sentence, Meena was working in her warehouse for hours
use ‘whom’. Thus, the word ‘who’ should be yesterday, and thus, she was unable to make it
replaced by ‘whom’ to make the sentence to her appointment with a new client on time.
grammatically right.
The right sentence is; 12. Answer: A
Tanya knew very little of the abilities and Use the future continuous tense for future
competencies of the client with whom she had actions happening over a period of time,
promised to venture into new business deals for especially when a specific time is mentioned.
the next few months. Thus, in part A, the phrase ‘will attend’ (simple
future) should be replaced by ‘will be attending’
10. Answer: A (future continuous) to make the sentence
Option a is grammatically incorrect. Double grammatically correct.
negation is grammatically incorrect: any negative The right sentence is:
proposition must only contain one negative. The team will be attending a meeting from noon
Thus, one of the negatives should be removed to to early evening tomorrow to discuss the pros
make the sentence grammatically right. Thus, and cons of investing a portion of company
replace ‘no’ by ‘any’. shares in ventures like real estate, interior
The right sentence is: designing, and many more.
Vimal is not cognizant of any ways to get the job
done quickly, but he is not willing to outsource 13. Answer: C
professionals for the same. Among the given options, phrases A, B and C
connects well with phrases E, D, and F
11. Answer: A respectively to result in meaningful and
The Past Continuous tense is used to describe grammatically right sentences. Thus, option c is
actions that began in the past and often right.
continued for a short period of time after the All other options do not result in grammatically
action started. Thus, part A is incorrect here right and coherent sentences.
because the action referred to here is expressed The right sentences include:
to have taken place for a specific amount of time

Click Here For Bundle PDF Course | support@guidely.in Page 11 of 14


SBI Clerk & RRB PO Mains PDF Course 2023
ENGLISH Day - 37

Markets regulator SEBI barred IIFL Securities, initiatives, inclusive strategies, and circular
one of the leading domestic broking houses, economic growth to promote sustainability and
from taking on new clients for the next two years renewable energy management.
for violating regulatory forms. The Indian tourism industry has enforced a
SEBI had considered cancellation of the national strategy called the Swadesh tourism
certificate of the IIFL securities as a broken but that acknowledges guidelines for sustainable
refrained from doing that after considering some tourism and focuses on shifting the contribution
other factors that are mentioned elaborately in of its sectors towards green, inclusive, and
the order released in June this year. circular economic growth.
IIFL Securities has been penalized for indulging
in unfair trading practices and using funds of one 15. Answer: D
client for meeting the obligations of another. Among the given options, phrases A, B and C
connects well with phrases F, D, and E
14. Answer: B respectively to result in meaningful and
Among the given options, phrases A, B and C grammatically right sentences. Thus, option d is
connects well with phrases D, E, and F right.
respectively to result in meaningful and All other options do not result in grammatically
grammatically right sentences. Thus, option b is right and coherent sentences.
right. The right sentences include:
All other options do not result in grammatically Describing Prime Minister Modi’s visit to the U.S.
right and coherent sentences. this June as a milestone in India-U.S. relations,
The right sentences include: foreign secretary said a roadmap for defense
Union tourism minister G Kishan Reddy industrial cooperation will be among the key
proposed that the G20 members should take outcomes of what will be Modi’s first state visit to
concrete steps to drive a collective change in the country and his seventh as PM.
how plastic pollution can be tackled globally The defense roadmap essentially focusses on
while giving special attention to the tourism aspects of defense co-production and co-
industry of their countries. development and talks about how defense
While addressing the fourth G20 tourism working industrial ecosystems could cooperate much
group meeting, G Kishan Reddy proposed that better.
the tourism industry should focus on green The prime minister met some prominent
personalities in New York and participate in a
capacity-building event before departing for

Click Here For Bundle PDF Course | support@guidely.in Page 12 of 14


SBI Clerk & RRB PO Mains PDF Course 2023
ENGLISH Day - 37

Washington D.C. where he held bilateral talks Among the given options, phrases A, B and C
with the U.S. president and addressed a joint connects well with phrases E, D, and F
sitting of the U.S. congress. respectively to result in meaningful and
grammatically right sentences. Thus, option a is
16. Answer: A right.
Among the given options, phrases A, B and C All other options do not result in grammatically
connects well with phrases D, E, and F right and coherent sentences.
respectively to result in meaningful and The right sentences include:
grammatically right sentences. Thus, option a is Victims of extensive burns and acid attack
right. patients with deep, severe scars now have a
All other options do not result in grammatically fresh ray of hope in the form of a dedicated skin
right and coherent sentences. bank at center-run Safdarjung hospital in the
The right sentences include: nation’s capital New Delhi.
The state investigation agency of Jammu and The proposal to set up a skin bank at the
Kashmir police carried out searches at several Safdarjung hospital at New Delhi was initiated in
locations in Srinagar, Anantnag, Pulwama, and 2012 after gaining the approval of the health
Kupwara districts in a case pertaining to the ministry but the construction of the same was
misuse of social media platforms for unlawful delayed due to the sudden outbreak of the
and secessionist activities. pandemic three years ago.
The identified entities are suspected of conniving The initiative is the first-of-its-kind in Delhi-NCR
with foreign associates to further their nefarious and north India, while presently India retains 16
agenda and specifically the entities in Anantnag skin banks which are majorly located in
have been found targeting government servants Maharashtra, Odisha, Madhya Pradesh, and the
by hindering their ability to perform their lawful southern states like Tamil
duties. Nadu and Karnataka.
The searches led to the seizure of substantial
digital and physical evidence, including mobile 18. Answer: D
phones and sim cards, which would be i. Hobble (verb) - to slow the movement,
meticulously analyzed to build a strong case progress, or action of (someone or something)
against the accused entities. ii. Hobble (verb) - to walk with difficulty because
of injury or weakness; to limp
17. Answer: C

Click Here For Bundle PDF Course | support@guidely.in Page 13 of 14


SBI Clerk & RRB PO Mains PDF Course 2023
ENGLISH Day - 37

iii. Hobble (verb) - to fasten together the legs of Sentence B becomes grammatically right and
(a horse, mule, etc.) by short lengths of rope to meaningful when the word ‘deplore’ is replaced
prevent free motion by ‘explore’.
Sentences A, B, and C use the word ‘hobble’
rightly to denote meanings as described in i, ii, 21. Answer: D
and iii respectively. Thus, all sentences are right. Rescind (verb) – to cancel officially
Thus, option d is the right answer. Only statement A rightly uses the word ‘rescind’
by denoting the meaning ‘to cancel officially.
19. Answer: A Thus, only statement D is right.
i. Comport (verb) - to be fitting; Accord or agree Statement B can be made grammatically right
with and meaningful if the word ‘rescinded’ is
ii. Comport (verb) - to behave in a manner replaced by ‘receded’.
conformable to what is right, proper, or expected Statement C can be made grammatically right
Sentences A and B use the word rightly to and meaningful if the word ‘rescinding’ is
denote meaning mentioned in i and ii replaced by ‘receding’.
respectively. Thus, only A and B is right.
Sentence C becomes grammatically right and 22. Answer: B
meaningful when the word ‘comport’ is replaced i. Inchoate (adjective) – partly in existence
by ‘comfort’. ii. Inchoate (adjective) – imperfectly or recently
formed
20. Answer: C Statements A and B rightly use the word
i. Deplore (verb) – to express strong disapproval ‘inchoate’ by denoting meanings that are
of described in i and ii respectively. Thus, only
ii. Deplore (verb) - to regret strongly statements A and B are correct.
Sentences A and C rightly use the word ‘deplore’ Statement C can be made grammatically right
to denote meanings as described in i and ii. and meaningful if the word ‘inchoate’ is replaced
Thus, sentences A and C are correct. by ‘integrate’.

Click Here For Bundle PDF Course | support@guidely.in Page 14 of 14


SBI Clerk & RRB PO Mains PDF Course 2023
Reasoning Ability Day - 38 (Eng)

Reasoning Ability
Directions (1-5): Study the following information a) The letter which is placed immediately above
carefully and answer the below questions. N
The first sixteen consecutive even numbers are b) C
arranged in ascending order from the top to c) The letter which is placed two positions below
bottom. The word “GOAL” is arranged in the G
multiple of 8’s position from the top in d) I
alphabetical order. e) P
Only one letter is placed between A and P. The
number of letters placed above P is the same as 3) What is the position of “R” in the
the number of letters placed below K. Only one arrangement?
letter is placed between K and N. The number of a) 20
letters placed between G and K is the same as b) 14
the number of letters placed between P and B. B c) 10
is not placed two letters before K. Only one letter d) 12
is placed between B and H which is placed after e) 18
B. M is placed three positions above V which is
not placed adjacent to L. M and A are not placed 4) What is the sum of the position of H and N?
adjacent to each other. D is placed immediately a) 42
above R. The number of letters placed between b) 32
D and P is the same as the number of letters c) 30
placed between R and B. I is placed two d) 48
positions above T. C is one of the letters placed e) 60
in the arrangement.
1) How many letters are placed between R and 5) Which of the following statement is true?
N? a) B is placed three letters below R
a) Five b) More than two letters are placed between B
b) Nine and N
c) Eight c) T is placed at sixth position
d) Seven d) L is placed five letters after M
e) Six e) All the statements are true

2) The number of letters placed above T is same


as the number of letters placed below_____

Click Here For Bundle PDF Course | support@guidely.in Page 1 of 13


SBI Clerk & RRB PO Mains PDF Course 2023
Reasoning Ability Day - 38 (Eng)

Directions (6-10): Each of the questions below e) All the statements I, II and III together are not
consists of statements below it. You must decide sufficient to answer the question.
whether the data provided in the statements are
sufficient to answer the question and mark the 7) Eight persons M, N, O, P, Q, R, S, and T are
appropriate option as answer. living in an eight storey building. The lowermost
6) Eight persons are sitting around a circular floor is numbered as one and the floor
table and some of them facing the center while immediately above it is numbered as two and so
some of them facing away from the center of the on. R lives on which floor?
table. How many persons are facing away from I. As many persons live between Q and O is
the center? same as between P and R.
I. The one who sits second to the left of G sits II. No one lives below Q. S lives immediately
third to the left of B. A and B sit opposite to each below P and two floors above M.
other and don’t face each other. As many III. P lives one of the floors above O and does
persons sit between F and H is same as not live below R. T lives three floors below O.
between E and H, where these three persons a) Only statement I alone is sufficient to answer
face the same direction. the question
II. The person who sits immediate right of D and b) Only statement II alone is sufficient to answer
the person who sits second to the left of F are the question.
facing each other. Immediate neighbors of all c) All the statements I, II and III together are not
persons are facing the same direction. sufficient to answer the question
III. A is an immediate neighbor of both E and G. d) Both statements II and III together are
Two persons sit between B and G. B is an sufficient to answer the question
immediate neighbor of both H and F. Immediate e) All the statements I, II and III together are
neighbors of A are facing the same direction. sufficient to answer the question
a) Only statement I alone is sufficient to answer
the question 8) Seven persons- P, Q, R, S, T, U, and V attend
b) Only statement II alone is sufficient to answer the interview on seven different days of a week
the question. starting from Sunday to Saturday. Who attends
c) Both statements I and III together are the interview immediately before P?
sufficient to answer the question I. Four persons attend the interview between V
d) All the statements I, II and III together are and P. T attends the interview one of the days
necessary to answer the question. after V, who attends the interview one of the
days after Q.

Click Here For Bundle PDF Course | support@guidely.in Page 2 of 13


SBI Clerk & RRB PO Mains PDF Course 2023
Reasoning Ability Day - 38 (Eng)

II. U attends the interview immediately before S. III. All oranges are Apple, Few Kiwi is Apple,
Q attends the interview immediately after R. Few Mango is Kiwi, Only a few Grapes are
III. As many persons attend the interview Apples.
between T and Q is same as between V and R. a) Only statement I alone is sufficient to answer
a) Only statement I alone is sufficient to answer the question.
the question. b) Only statement II alone is sufficient to answer
b) Only statement II alone is sufficient to answer the question.
the question. c) Only statement III alone is sufficient to answer
c) Both statements I and II together are sufficient the question.
to answer the question. d) Either statement II or statement III is sufficient
d) All the statements I, II and III together are to answer the question.
sufficient to answer the question. e) All the statements I, II and III together are
e) All the statements I, II and III together are not sufficient to answer the question.
sufficient to answer the question.
10) Point F is in which direction with respect to
9) In the below question some conclusions are B?
given followed by some set of statements. You I. Person G walks for 9m from his home (Point
have to decide which set of statements logically A) towards the east direction to reach Point B
follow the given conclusions disregarding the and turns 2700 anti-clockwise direction and
commonly known facts. walks for 12m to reach Point C. Point H is west
Conclusions: of Point E and east of point B.
No Kiwi being Orange is a Possibility, All Grapes II. Point C is 15m west of Point D and southwest
can never be Orange, Some Mango can be of Point E. Point I is 18m south Point of E. Point I
Apple is to the southeast of Point A.
Statements: III. Point C, D and F are in the horizontal straight
I. All Kiwi are Mangoes, Some Oranges are line. Point F is south of Point E. Point F is east of
Mangoes, Only a few Apples are Mangoes, No Point D which is in the south of Point H.
Kiwi is Grape. a) Only statement I alone is sufficient to answer
II.Only a few Mangoes are Kiwi, Few Oranges the question.
are Mangoes, Only Kiwi is Apple, All Grapes are b) Both statements I and III together are
Orange. sufficient to answer the question.
c) All the statements I, II and III together are
sufficient to answer the question.

Click Here For Bundle PDF Course | support@guidely.in Page 3 of 13


SBI Clerk & RRB PO Mains PDF Course 2023
Reasoning Ability Day - 38 (Eng)

d) Either statement II alone or III alone is one who sits to the immediate right of H faces J
sufficient to answer the question. who sits neither opposite to L nor adjacent to I.
e) All the statements I, II and III together are not One person sits between F and G who sits
sufficient to answer the question. adjacent to K. As many persons sit between E
and F as between E and H. K doesn’t sit
Directions (11-15): Study the following adjacent to H.
information carefully and answer the questions They are playing cards in the following order.
given below. A- 7 of spade, B- King of heart, D- Queen of
Twelve persons A, B, C, D, E, F, G, H, I, J, K diamond, G- 10 of club, H- 6 of heart
and L are sitting around the concentric table as Condition 1:
shown in the figure. If the person gets the “number card of spade”,
then the person interchanges the position with
the one who sits third to the right of that person;
else the person interchanges the position with
the one who faces that person.
Condition 2:
If the person gets the “face card of club”, then
the person interchanges the position with the
one who sits second to the left of that person;
else the person interchanges the position with
the one who sits immediate right of that person.
Note: Condition 3:
I) Consecutive alphabetically named persons If the person gets the “number card of heart”,
didn’t sit adjacent to each other (before playing then the person interchanges the position with
cards). the one who sits fifth to the right of that person;
II) If it is given that A faces B, then both A and B else the person interchanges the position with
are in different tables and if it is given that A sits the one who sits opposite to the one who sits
opposite to B, then both A and B are in the same third to the right of that person.
table. Condition 4:
The one who faces A sits second to the left of I. If the person gets the “face card of diamond”,
B who doesn’t sit at the same table of I and then the person interchanges the position with
faces the one who sits opposite to L. C who the one who sits third to the left of that person;
doesn’t face B and sits third to the left of H. The else the person interchanges the position with

Click Here For Bundle PDF Course | support@guidely.in Page 4 of 13


SBI Clerk & RRB PO Mains PDF Course 2023
Reasoning Ability Day - 38 (Eng)

the one who sits second to the right of the one e) None is true
who faces that person.
11) Who among the following person sits third to 15) Four of the following five are alike in a
the left of J in the final arrangement? certain way based on the final arrangement and
a) The one who sits to the immediate right of D thus form a group. Which one of the following
b) B doesn’t belong to that group?
c) The one who faces E a) E-I
d) A b) F-J
e) None of these c) G-L
d) H-D
12) What is the position of G with respect to H in e) B-I
the final arrangement?
a) Third to the right Directions (16-20): Study the following
b) Second to the left information carefully and answer the below
c) Third to the left questions.
d) Second to the right Eight persons – P, Q, R, S, T, U, V, and W were
e) None of these promoted in four different months viz.- January,
April, August, and November of two different
13) How many persons are sitting between C years viz.- 2012 and 2015. The age of each
and K when counted from the right of C in the person is different viz.- 21, 23, 24, 25, 27, 28,
final arrangement? 32, and 36. All the information is not necessary
a) Six in the same order.
b) Five Note: The one whose age is an odd number was
c) Two promoted in 2012 whereas the one whose age is
d) One an even number was promoted in 2015.
e) None of these Only two persons were promoted between W
and the one whose age is 32 years. S and the
14) Which of the following statements is/are true one whose age is 23 years were promoted in the
according with the final arrangement? same month. Only one person was promoted
a) A sits third to the right of F between the one whose age is 23 years and W,
b) J faces G who was promoted in a month having an even
c) B sits to the immediate right of E number of days. Only four persons were
d) All are true promoted between Q and the one whose age is

Click Here For Bundle PDF Course | support@guidely.in Page 5 of 13


SBI Clerk & RRB PO Mains PDF Course 2023
Reasoning Ability Day - 38 (Eng)

36 years. T’s age is 9 years more than U’s age.


T was promoted two persons before the one 18) Who among the following person was
whose age is 28 years. V’s age is 3 years more promoted in August?
than the age of the one who was promoted in a) V
April 2012. Neither V nor R was promoted in the b) The one whose age is 27 years
same month with U. P’s age is more than R’s c) S
age and was promoted after R. P and the one d) The one whose age is 25 years
whose age is a perfect square number were not e) None of these
promoted in the same month.
16) Who among the following person promoted 19) Four of the following five are alike in a
immediately before U? certain way as per the given arrangement and
a) The one whose age is 25 years hence form a group. Find the one that doesn’t
b) S belong to that group.
c) The one whose age is 27 years a) The one whose age is 25 years
d) P b) P
e) None of these c) The one whose age is 27 years
d) U
17) Who among the following pair of persons e) The one whose age is 32 years
doesn’t promote in the same month?
I. RT 20) Which of the following information is not true
II. U and the one whose age is 28 years regarding V?
III. V and the one whose age is 21 years a) V was promoted two persons before P
a) Only II b) Age of V is 24 years
b) Both I and III c) V was promoted in April 2015
c) Both II and III d) V was promoted three persons after Q
d) Only I e) All the above statements are true
e) None of these
Click Here to Get the Detailed Video Solution for the above given Questions-Will Update Soon
Or Scan the QR Code to Get the Detailed Video Solutions

Answer Key with Explanation

Directions (1-5): 2) Answer: B


1) Answer: E 3) Answer: D

Click Here For Bundle PDF Course | support@guidely.in Page 6 of 13


SBI Clerk & RRB PO Mains PDF Course 2023
Reasoning Ability Day - 38 (Eng)

4) Answer: D
5) Answer: E
Final Arrangement

We have, Again we have,


 The word “GOAL” is arranged in multiples  The number of letters placed between G
of 8’s position from the top in alphabetical and K is the same as the number of
order. letters placed between P and B.
 Only one letter is placed between A and  B is not placed two letters before K
P.  Only one letter is placed between B and H
 The number of letters placed above P is which is placed after B.
the same as the number of letters placed  M is placed three positions above V which
below K. is not placed adjacent to each other
 Only one letter is placed between K and  M and A are not placed adjacent to each
N. other
From the above condition, there are three
possibilities.

Click Here For Bundle PDF Course | support@guidely.in Page 7 of 13


SBI Clerk & RRB PO Mains PDF Course 2023
Reasoning Ability Day - 38 (Eng)

Again we have,
6) Answer: D
 D is placed immediately above R.
We have,
 The number of letters placed between D
All the statements I, II and III together are
and P is the same as the number of
necessary to answer the question
letters placed between R and B.
 I is placed two positions above T.
 C is one of the letters placed in the
arrangement.
From the above condition, Case-2 and Case-2a
get eliminated. Case-1 shows the final
arrangement.

7) Answer: E
We have,
I. As many persons live between Q and O is
same as between P and R.
II. No one lives below Q. S lives immediately
below P and two floors above M.

Click Here For Bundle PDF Course | support@guidely.in Page 8 of 13


SBI Clerk & RRB PO Mains PDF Course 2023
Reasoning Ability Day - 38 (Eng)

III. P lives one of the floors above O and does


not live below R. T lives three floors below O.
After combining all the three statements above,
we have two possibilities.
Then Case2 gets eliminated. Hence Case1
shows the final arrangement.
R lives on the 3rd Floor.

9) Answer: C
We have,
Conclusion:
No Kiwi being Orange is a Possibility, All Grapes
can never be Orange, Some Mango can be
Apple
Statement:
III. All oranges are Apple, Few Kiwi is Apple,
8) Answer: C Few Mango is Kiwi, Only a few Grapes are
We have, Apples.
I. Four persons attend the interview between V All the conclusions follow by using Only
and P. T attends the interview one of the days statement III alone.
after V, who attends the interview one of the
days after Q.
II. U attends the interview immediately before S.
Q attends the interview immediately after R.
III. As many persons attend the interview
between T and Q is same as between V and R.
After combining statements I and II we have two
possibilities. 10) Answer: B
No one attends the interview immediately before We have,
P. After combining statements I and III, Point F is
in the southeast direction of Point B.

Click Here For Bundle PDF Course | support@guidely.in Page 9 of 13


SBI Clerk & RRB PO Mains PDF Course 2023
Reasoning Ability Day - 38 (Eng)

Directions (11-15):
11) Answer: C
12) Answer: B
13) Answer: D
14) Answer: A
15) Answer: E (All the pairs are facing each
other except for option e)
Final arrangement:
Again we have,
 C who doesn’t face B and sits third to the
left of H.
 The one who sits to the immediate right of
H faces J who sits neither opposite to L
nor adjacent to I.
So Case 2 and Case 1b get eliminated

We have,
 The one who faces A sits second to the
left of I.
 B, who doesn’t sit at the same table of I
and faces the one who sits opposite to L.
From the above conditions, there are four
possibilities

Click Here For Bundle PDF Course | support@guidely.in Page 10 of 13


SBI Clerk & RRB PO Mains PDF Course 2023
Reasoning Ability Day - 38 (Eng)

 The position of G and K


interchanges(from condition 2)

Again we have,
 One person sits between F and G who
sits adjacent to K.
 As many persons sit between E and F as
between E and H.
 K doesn’t sit adjacent to H. Directions (16-20):

So Case 1 gets eliminated, hence the final 16) Answer: C

arrangement is 17) Answer: A


18) Answer: B
19) Answer: D (All the persons were promoted in
the month of having 31 days except option D)
20) Answer: A

Final arrangement after applying the conditions


while playing cards
 The position of H and C
interchanges(from condition 3)
 The position of D and I interchanges(from
We have:
condition 4)
 Only two persons were promoted
 The position of A and F interchanges(from
between W and the one whose age is 32
condition 1)
years.
 The position of B and E
 S and the one whose age is 23 years
interchanges(from condition 3)
were promoted in the same month.

Click Here For Bundle PDF Course | support@guidely.in Page 11 of 13


SBI Clerk & RRB PO Mains PDF Course 2023
Reasoning Ability Day - 38 (Eng)

 Only one person was promoted between


the one whose age is 23 years and W,
who was promoted in a month having an
even number of days.
That means, in case (1) W was promoted
in April 2012, in case (2) W was promoted
in November 2012.
Based on the above given information we have:

Again, we have:
Case (2) & case (2a) are not valid as T was
 Only four persons were promoted
promoted two persons before the one whose
between Q and the one whose age is 36
age is 28 years.
years.
Again, we have:
That means, in case (1) & case (2) Q was
 V’s age is 3 years more than the age of
promoted in January 2012, in case (1a) &
the one who was promoted in April 2012.
case (2a) Q was promoted in August
Since, the one who was promoted in 2012
2012.
has odd ages, thus possible combinations
 T’s age is 9 years more than U’s age.
are (21, 24), (25, 28).
Since, only such possible age is (23, 32)
 Neither V nor R was promoted in the
& (27, 36).
same month with U.
 T was promoted two persons before the
 P’s age is more than R’s age and was
one whose age is 28 years.
promoted after R.
That means, in case (1) T was promoted
 P and the one whose age is a perfect
in April 2015, in case (1a) and case (1b) T
square number were not promoted in the
was promoted in January 2015, case (2)
same month.
& case (2a) are not valid.
That means, in case (1a) R was promoted
Based on the above given information we have:
in January 2012 and the age of R must be
25 years, case (1) and case (1b) are not
valid.

Click Here For Bundle PDF Course | support@guidely.in Page 12 of 13


SBI Clerk & RRB PO Mains PDF Course 2023
Reasoning Ability Day - 38 (Eng)

Based on the above given information we have: square number were not promoted in the same
month

Case (1) is not valid as V and U are not


promoted in the same month. Case (1b) is not
valid as P and the one whose age is a perfect

Click Here For Bundle PDF Course | support@guidely.in Page 13 of 13


SBI Clerk & RRB PO Mains PDF Course 2023
Quantitative Aptitude Day – 38 (Eng)

Quantitative Aptitude

Directions (1-4): Study the following data 3) Series that follows a certain logic given below
carefully and answer the questions: and contains one wrong term. There is another
1) Given below is a number series that follows series that follows the same logic as in first
certain logic. If ‘P’ is the nth term and ‘Q’ is ‘n + series and starts with the wrong term of first
1’th term, then find the correct relationship series. Find the 3rd term of second series?
between ‘P’ and ‘Q’. Series I: 2, 14, 112, 672, 3360, 13440, 40320
Series: 8, 25, 76, 229, 688 A.784
A.Q = 3P – 1 B.6272
B.Q = 2P + 3 C.5376
C.Q = 3P + 1 D.896
D.Q = 2P – 1 E.4704
E.Q = 2P – 3
4) Two series I and II with different logics are
2) There are two series I and II. Logic in both the given below. Series I contains one wrong
series are different. element termed as A and series II contains one
Series I: 3, 12, 28, 53, 89, ……. missing element termed as B. Which of the
Series II: 259, 270, 283, 300, 319. following is not TRUE regarding A and B?
Which of the following will be the first term which Series I: 18480, 1680, 240, 48, 18, 8
is common in both the series? Series II: 200, 79, ?, 5, -4, -8
A.504 A.HCF of A and B is 6.
B.342 B.LCM of A and B is 90.
C.383 C. Total number of factors of A are 6.
D.283 D. Total number of factors of B are 8.
E.371 E.A > B

Directions (5-8): Study the following data carefully and answer the questions:
There are 6 shopkeepers A, B, C, D, E and F. Each shopkeeper purchased two types of bread i,e;
Banana bread and Garlic bread and sold only some of the breads.
The Pie chart given below shows the percentage distribution of the number of banana bread sold by each
shopkeeper on a particular day.

Click Here For Bundle PDF Course | support@guidely.in Page 1 of 12


SBI Clerk & RRB PO Mains PDF Course 2023
Quantitative Aptitude Day - 38 (Eng)

The table given below is the average number of banana bread and that of garlic bread sold by
shopkeepers B, C, D, E and F.

Note:
1: Number of garlic bread sold by shopkeeper A is 40 more than that of banana bread sold by him.
2: Average number of banana bread and garlic bread sold by shopkeeper A is 90% of the number of
garlic bread sold by him.
5) Number of banana bread sold by C is 60% of total unsold bread (banana + garlic) of C are
the total banana bread purchased by him and the rotten, then find the number of unsold bread
number of garlic bread sold by C is 66(2/3) % of (banana + garlic) of C, which are not rotten?
total garlic bread purchased by him. If 20% of A.164

Click Here For Bundle PDF Course | support@guidely.in Page 2 of 12


SBI Clerk & RRB PO Mains PDF Course 2023
Quantitative Aptitude Day - 38 (Eng)

B.120 C.50
C.180 D.25
D.148 E.45
E.152
9) In container A, contains the mixture of milk
6) Number of banana bread sold by C and F and water. The ratio of milk and water in
together is what percent less than the number of container A is 8:5. 39L mixture is taken out and
garlic bread sold by C and F together? put it on container B. when 4L milk and 5L water
A.34% is added in container A,then the ratio of milk to
B.42% water in the container A becomes 3:2.Find the
C.40% difference between the total milk in container A
D.32% and B together and total water in container A and
E.36% B together?
A.29 L
7) If 66(2/3) % of total bread (banana + garlic) B.20 L
sold by A are sold to males, out of which 40% C.22 L
are banana bread and 50% of total bread D.31 L
(banana + garlic) sold by D are sold to males, out E. None of these
of which 40% are banana bread, then find the
number of garlic bread sold by A and D together 10) If A started the business with initial
to males. investment of Rs. 6000 and after X months B
A.204 joined the business with initial investment of Rs.
B.216 9000. 3 months after B joining the business A
C.184 withdraws 1/4th of his initial investment. At the
D.198 end of the year A’s share is Rs. 3850 out of total
E.210 profit Rs. 8800. Find the value of X?
A.7 months
8) Shopkeeper B sold 82% of the total number of B.4 months
bread (banana + garlic) purchased by him and C.6 months
sold 80% of banana bread purchased by him. D.3 months
Find the number of unsold garlic bread of E.5 months
shopkeeper B.
A.40 11) Ratio of the radius of the cylinder to the
B.30 length of the rectangle is 1:2 and the breadth of

Click Here For Bundle PDF Course | support@guidely.in Page 3 of 12


SBI Clerk & RRB PO Mains PDF Course 2023
Quantitative Aptitude Day - 38 (Eng)

the rectangle is 15% of the perimeter of the and speed ratio between Train B and Train A is
rectangle. If the height of the cylinder is half of 5:3. If the two Trains are travelling in opposite
the breadth of the rectangle and the curved direction they will meet after 10 seconds. Find
surface area of the cylinder is 528 cm2, then find the ratio between time taken by Train A and
the radius of the cylinder? Train B to cross the platform has length of
A.7 cm 220m?
B.14 cm A.7:8
C.3.5 cm B.9:7
D.21 cm C.9:5
E. Cannot be determined D.8:9
E.7:9
12) If the ratio between length of two Trains A
and B is 2:3. The speed of Train B is 180 kmph

Directions (13-16): Study the following information carefully and answer the questions given below.
The given table shows the average number of people who visit park A and B and the percentage of the
number of people who visit park B is more than that of A and the ratio of the number of people who visit
park A and C in two different seasons in five different states.

Click Here For Bundle PDF Course | support@guidely.in Page 4 of 12


SBI Clerk & RRB PO Mains PDF Course 2023
Quantitative Aptitude Day - 38 (Eng)

13) What is the ratio of the total number of 15) The difference between the total number of
people who visit park C in summer season in people who visit park B and C in winter season in
Tamilnadu and Bihar together to the total number Bihar is what percent of the difference between
of people who visit park B in winter season in the number of people who visit park A and C in
Tamilnadu and Kerala together? summer season in Assam?
A.21:16 A.37.78%
B.22:17 B.39.10%
C.7:5 C.41.67%
D.11:8 D.43.56%
E. None of these E.45.89%

14) What is the difference between the number 16) What is the difference between the number
of people who visit park B and C together in of people who visit park B and C together in
winter and summer season in Assam? winter season in Manipur and the number of
A.2460 people who visit park A and C in summer season
B.2465 in Kerala?
C.2470 A.9000
D.2475 B.9200
E.2480 C.9100
D.9300
E.9400

Directions (17-20): Study the following data carefully and answer the questions:
There are 5 car manufacturing companies P, Q, R, S and T. Each company manufactures three types of
cars i,e; TUV, SUV and KUV.
The bar graph given below shows the number of SUVs manufactured by a company as percent of that of
TUVs manufactured by the respective company and also shows the average number of TUVs, SUVs and
KUVs manufactured by a company in a month.

Click Here For Bundle PDF Course | support@guidely.in Page 5 of 12


SBI Clerk & RRB PO Mains PDF Course 2023
Quantitative Aptitude Day - 38 (Eng)

Note:
1: Ratio of the number of TUVs manufactured by P and R is 4: 5 respectively and the difference between
them is 37.5% of the number of TUVs manufactured by Q.
2: Ratio of the number of TUVs manufactured by R and S is 3: 2 respectively and the number of TUVs
manufactured by Q is 40.
3: Average number of TUVs manufactured by S and T is 65.
17) Cost of manufacturing one SUV in of the month and the ratio of the number of SUVs
companies P, R and S is Rs.5 Lakh, Rs.6 Lakh to that of KUVs manufactured by company Q in
and Rs.4 Lakh respectively, then find the the last 5 days is 5: 2, then find the ratio of
average total cost of manufacturing SUVs of number of SUVs to that of KUVs manufactured
these three companies in a month. by company Q in first 25 days. (Assume 30 days
A.Rs.30 million in the month)
B.Rs.36 million A.4: 7
C.Rs.24 million B.2: 3
D.Rs.40 million C.5: 9
E.Rs.32 million D.1: 2
E.3: 5
18) If 16(2/3) % of total KUVs manufactured by
company Q are manufactured in the last 5 days

Click Here For Bundle PDF Course | support@guidely.in Page 6 of 12


SBI Clerk & RRB PO Mains PDF Course 2023
Quantitative Aptitude Day - 38 (Eng)

19) Total number of KUVs manufactured by 20) Total number of TUVs produced by
companies R and S together is what percent companies P and T together is what percent of
more than that of TUVs manufactured by these that of KUVs produced by these two companies
two companies together? together?
A.83% A.87.33%
B.76% B.91.33%
C.80% C.93.33%
D.85% D.85.33%
E.78% E.89.33%
Click Here to Get the Detailed Video Solution for the above given Questions-Will Update Soon
Or Scan the QR Code to Get the Detailed Video Solutions

Answer Key with Explanation

1) Answer: C 259 + 11 = 270


Logic in the series is: 270 + 13 = 283
(8 * 3) + 1 = 25 283 + 17 = 300
(25 * 3) + 1 = 76 300 + 19 = 319
(76 * 3) + 1 = 229 319 + 23 = 342
(229 * 3) + 1 = 688 Hence, we can see that the first term which is
Hence, common in both the series is 283.
Q = 3P + 1
3) Answer: A
2) Answer: D Logic in the series is:
Logic in series I: 2 * 8 = 16
3 + 32 = 12 16 * 7 = 112
12 + 42 = 28 112 * 6 = 672
28 + 52 = 53 672 * 5 = 3360
53 + 62 = 89 3360 * 4 = 13440
89 + 72 = 138 13440 * 3 = 40320
138 + 82 = 202 Wrong term = 14
202 + 92 = 283 New series:
283 + 102 = 383 14 * 8 = 112
Logic in series II: 112 * 7 = 784

Click Here For Bundle PDF Course | support@guidely.in Page 7 of 12


SBI Clerk & RRB PO Mains PDF Course 2023
Quantitative Aptitude Day – 38 (Eng)

14, 112, 784 And average of number of banana bread and


Hence, 3rd term of second series = 784 garlic bread sold by shopkeeper A = (20x + 40) *
(90/100)
4) Answer: E Now,
Logic in series I: (20x + 20x + 40)/2 = (20x + 40) * (90/100)
18480 ÷ 11 = 1680 40x + 40 = (20x + 40) * (9/5)
1680 ÷ 7 = 240 200x + 200 = 180x + 360
240 ÷ 5 = 48 x=8
48 ÷ 3 = 16 (Not 18) Average of number of banana bread and garlic
16 ÷ 2 = 8 bread sold by shopkeeper A = (20 * 8 + 40) *
A = 18 = 21 * 32 (90/100) = 180
Logic in series II:
200 – 112 = 79
79 – 72 = 30
30 – 52 = 5
5 – 32 = -4
-4 – 22 = -8
B = 30 = 21 * 31 * 51
(a): HCF of 18 and 30 = 6
(b): LCM of 18 and 30 = 90
(c): Total number of factors of A = 6
(d): Total number of factors of B = 8 5) Answer: E
(e): A < B Number of banana bread sold by C = 120
Hence, option (e) is not TRUE. So, the total number of banana bread,
purchased by C:
Directions (5-8): 120 * (100/60) = 200
Let the number of banana bread sold by Number of garlic bread sold by C = 220
shopkeepers A, B, C, D, E and F are ‘20x’, ‘12x’, So, the total number of garlic bread, purchased
‘15x’, ‘10x’, ‘25x’ and ‘18x’ respectively. by C:
So, number of garlic bread sold by shopkeeper A 220 * (300/200) = 330
= 20x + 40 Total number of breads (banana + garlic),
purchased by C:
200 + 330 = 530

Click Here For Bundle PDF Course | support@guidely.in Page 8 of 12


SBI Clerk & RRB PO Mains PDF Course 2023
Quantitative Aptitude Day – 38 (Eng)

Total number of unsold bread (banana + garlic) Number of garlic breads sold by D to male =
of C: 60% of 120 = 72
530 – 120 – 220 = 190 Number of garlic breads sold by A and D
Since 20% of total unsold bread (banana + together to males = 144 + 72 = 216
garlic) of C are rotten.
So, the number of unsold bread (banana + 8) Answer: B
garlic) of C, which are not rotten: Total number of breads (banana + garlic) sold by
80% of 190 = 152 B = 96 + 150 = 246
So, total number of breads (banana + garlic)
6) Answer: A purchased by B:
Number of banana bread sold by C = 120 246 * (100/82) = 300
Number of banana bread sold by F = 144 Number of banana breads sold by B = 96
Total number of banana bread sold by C and F So, number of banana breads purchased by B =
together = 120 + 144 = 264 96 * (100/80) = 120
Number of garlic bread sold by C = 220 And number of garlic breads purchased by B =
Number of garlic bread sold by F = 180 300 – 120 = 180
Total number of garlic breads sold by C and F Number of garlic breads sold by B = 150
together = 220 + 180 = 400 So, number of unsold garlic breads of B = 180 –
Required percentage = [(400 – 264)/400] * 100 = 150 = 30
34%
9) Answer: A
7) Answer: B Let milk and water in container A is 8x and 5x
Total number of breads (banana + garlic) sold by respectively.
A = 160 + 200 = 360 In 39L milk and water is 39*(8/13)=24 and
Number of breads (banana + garlic) sold by A to 39*(5/13)=15L respectively.
males = 66(2/3) % of 360 = 240 So, we can say, (8x-24+4)/(5x-15+5) = 3/2
Number of garlic breads sold by A to male = Or, 16x-40=15x-30
60% of 240 = 144 Or, x=10 L
Total number of breads (banana + garlic) sold by So, in container A amount of milk is = 80-
D = 80 + 160 = 240 24+4=60 L
Number of breads (banana + garlic) sold by D to In container A amount of water is = 50-15+5=40
males = 50% of 240 = 120 L
Required difference = (60+24) – (40+15) = 29 L

Click Here For Bundle PDF Course | support@guidely.in Page 9 of 12


SBI Clerk & RRB PO Mains PDF Course 2023
Quantitative Aptitude Day – 38 (Eng)

10) Answer: D Time taken by Train A to cross the platform has


At the end of the year, length of 220m = (320 + 220) / 30 = 18s
A’s share =Rs. 3850 Time taken by Train B to cross the platform has
B’s share = Rs. 4950 length of 220m = 700 / 50 = 14s
Profit ratio of A to B = 7:9 Required ratio = 18:14 = 9:7
(6000*X + 6000*3 + 4500*(9-X)) / (9000*3 +
9000(9-X)) = 7/9 Directions (13-16):
(1500x + 58500) / (10800 – 9000x) = 7/9 Tamilnadu in winter season:
765x = 2295 Number of people visit park A and B = 12900 * 2
X=3 = 25800
A + A * 115/100 = 25800
11) Answer: B A = 12000
Ratio of the breadth and perimeter of the B = 25800 – 12000 = 13800
rectangle =15:100 = 3:20 C = 5/4 * 12000 = 15000
Breadth of the rectangle = 3x Similarly we can find the remaining values.
Perimeter of the rectangle = 20x
Radius of the cylinder = y
Length of the rectangle = 2y
2 * (3x + 2y) = 20x
2y = 7x
y = 7x/2
2 * 22/7 * 7x/2 * 3x/2 = 528 13) Answer: A
x=4 Required ratio = (22400 + 19600):(13800 +
Radius of the cylinder = 7 * 4/2 = 14 cm 18200)
= 42000:32000
12) Answer: B = 21:16
The speed of Train B = 180 kmph = 180 * 5 / 18
= 50 m/s 14) Answer: D
The speed of Train A = 50 * 3 / 5 = 30 m/s Required difference = (15000 + 16875) – (13800
Length of two trains = 10 * (50 + 30) = 800 + 15600)
Length of Train A = 800 * 2 / 5 = 320m = 2475
Length of Train B = 800 * 3 / 5 = 480m
15) Answer: C

Click Here For Bundle PDF Course | support@guidely.in Page 10 of 12


SBI Clerk & RRB PO Mains PDF Course 2023
Quantitative Aptitude Day – 38 (Eng)

Required % = (14000 – 12500)/(15600 – 12000)


* 100
=1500/3600*100
= 41.67%

16) Answer: B
Difference = (18000 + 28800) - (17600 + 20000)
= 9200

Directions (17-20):
17) Answer: A
Since, number of TUVs manufactured by Q = 40
Number of SUVs manufactured by company P in
So, difference between number of TUVs
a month = 72
manufactured by P and R = 37.5% of 40 = 15
So, total cost of manufacturing SUVs of
Since, the ratio of number of TUVs
company P in a month = 72 * 500000 = Rs.36
manufactured by P and R is 4: 5 respectively.
million
So, number of TUVs manufactured by P = 15 *
Number of SUVs manufactured by company R in
4/ (5 – 4) = 60
a month = 60
And number of TUVs manufactured by R = 60 *
So, total cost of manufacturing SUVs of
(5/4) = 75
company R = 60 * 600000 = Rs.36 million
Since, the ratio of number of TUVs
Number of SUVs manufactured by company S in
manufactured by R and S is 3: 2 respectively.
a month = 45
So, number of TUVs manufactured by S = 75 *
So, total cost of manufacturing SUVs of
(2/3) = 50
company S = 45 * 400000 = Rs.18 million
Since, the average number of TUVs
Required average = (36 + 36 + 18)/3 = Rs.30
manufactured by S and T is 65.
million
So, number of TUVs manufactured by T = (2 *
65) – 50 = 80
18) Answer: D
Total number of KUVs manufactured by
company Q in the month = 60
So, number of KUVs manufactured by company
Q in first 25 days of the month = 83(1/3) % of 60
= 50

Click Here For Bundle PDF Course | support@guidely.in Page 11 of 12


SBI Clerk & RRB PO Mains PDF Course 2023
Quantitative Aptitude Day – 38 (Eng)

Number of SUVs manufactured by company Q in Total number of TUVs manufactured by


last 5 days of the month = 16(2/3) % of 60 * (5/2) companies R and S together:
= 25 75 + 50 = 125
Total number of SUVs manufactured by Required percentage = [(220 – 125)/125] * 100 =
company Q in the month = 50 76%
So, number of SUVs manufactured by company
Q in the first 25 days of the month = 50 – 25 = 25 20) Answer: C
Required ratio = 25: 50 = 1: 2 Total number of TUVs produced by P and T
together = 60 + 80 = 140
19) Answer: B Total number of KUVs produced by P and T
Total number of KUVs manufactured by together = 48 + 102 = 150
companies R and S together: Required percentage = (140/150) * 100 =
105 + 115 = 220 93.33%

Click Here For Bundle PDF Course | support@guidely.in Page 12 of 12


SBI Clerk & RRB PO Mains PDF Course 2023
ENGLISH Day - 38

English Language

Directions (1-5): In each of the following C. communities like Muslims, Christians,


questions, a sentence is divided into five parts. Buddhists, Sikhs, Jains, and Parsis that account
One part of the sentence is fixed and highlighted to
in bold. The highlighted part of the sentence D. their people, who are largely backward in
appears at the position given and no change in many areas, specifically in education
position should be applied for the highlighted E. granted by the state government for the
part. You have to arrange the remaining parts of welfare and development of
the sentence to form a meaningful sentence. a. ABECD
1. b. ABDCE
A. A sessions court in Jabalpur has sentenced a c. ACBED
man d. AECBD
B. in the name of MP high court to at least 15 e. No rearrangement required
job-seeking youth
C. to 110 years of rigorous imprisonment under 3.
non-bailable conditions A. the Supreme Court referred the challenge to
D. after charging them Rs 5000 to Rs 30,000 the constitutional validity
each B. the center which substantially neglected its
E. and imposed a fine of Rs. 25000 for giving May 11 judgment
fake appointment letters C. of the recently registered May 19 Delhi
a. ABDEC services ordinance
b. ACEBD D. to a five-bench judge but officially refused to
c. ABCED stay the order of
d. ACDBE E. giving legislative and executive control over
e. No rearrangement required the bureaucracy to the Delhi government.
a. ADCBE
2. b. BCADE
A. Pointing to alleged discrimination by the c. ACDBE
previous government, d. BACDE
B. 16% of the total population of Karnataka e. No rearrangement required
expect a hike in the allocation of funds
4.

Click Here For Bundle PDF Course | support@guidely.in Page 1 of 12


SBI Clerk & RRB PO Mains PDF Course 2023
ENGLISH Day - 38

A. the state government as per his


recommendation is in the process of developing Directions (6-10): In the following passage, some
fruit hubs of the words have been left out, each of which is
B. adopting high density fruit plantation while indicated by a number. Find the suitable word
micro irrigation facility efforts from the options given against each number and
C. to increase fruit production according to the fill up the blanks with appropriate words to make
region-specific conditions by the paragraph meaningfully complete.
D. Chief Minister said that under the Himachal The empowerment and ____________ (6) of
Pradesh Shiva Project women and the improvement of their political,
E. are underway to increase the area under social, economic and health status are highly
horticulture in the state important to create a fair society. In addition, it is
a. BACDE essential for the achievement of sustainable
b. DACBE development. The full participation and
c. DECAB partnership of both women and men is required
d. AECBD in productive and reproductive life, including
e. No rearrangement required shared responsibilities for the care and nurturing
of children and maintenance of the household. In
5. all parts of the world, women are facing threats
A. Four teams of the National Disaster to their lives, health and well- being as a result of
Response Force are leading being overburdened with work and of their lack
B. by over 400 mm of rainfall flattened a remote of power and influence. In most regions of the
tribal hamlet named Irshalwadi world, women receive less formal education than
C. recorded multiple cases of missing residents men, and at the same time, women's own
and casualties knowledge, abilities and coping mechanisms
D. resulting in one of the biggest natural often go unrecognized. The power dynamics and
disasters in Maharashtra that gender inequality that ______________ (7)
E. the rescue operation in Pune after a massive women's attainment of healthy and fulfilling lives
landslide precipitated operate at many levels of society, from the most
a. ABEDC personal to the highly public. Achieving change
b. ADEBC requires policy and program actions that will
c. AEBDC improve women's access to secure livelihoods
d. ACBDE and economic resources, ________ (8) their
e. No rearrangement required extreme physical responsibilities with regard to

Click Here For Bundle PDF Course | support@guidely.in Page 2 of 12


SBI Clerk & RRB PO Mains PDF Course 2023
ENGLISH Day - 38

housework, remove legal impediments to their 7. Which of the following words should fill the
participation in public life, and raise social blank 7?
awareness through effective programs of a. impede
education and mass communication. All b. streamline
Countries should act to empower women and c. incubate
should take steps to eliminate inequalities d. prompt
between men and women as soon as possible e. coax
by establishing mechanisms for women's equal
participation and equitable representation at all 8. Which of the following words should fill the
levels of the political process and public life in blank 8?
each community and society and enabling a. intensify
women to articulate their concerns and needs. b. exacerbate
Governments should take measures to promote c. assuage
the fulfilment of girls’ and women’s potential d. proliferate
through education, skills development and the e. inflate
___________ (9) of illiteracy amongst girls, giving
paramount importance to the elimination of 9. Which of the following words should fill the
poverty and ill health. The society should blank 9?
____________ (10) gender-based discriminations a. approbation
and promote full and equal access of women to b. endorsement
literacy, education and training, and remove all c. countenance
obstacles to their access to credit and other d. commendation
productive resources and to their ability to buy, e. abrogation
hold and sell property and land equally with men.
6. Which of the following words should fill the 10. Which of the following words should fill the
blank 6? blank 10?
a. subjugation a. Acquiesce
b. autonomy b. enculturate
c. immurement c. assimilate
d. captivity d. oppugn
e. internment e. espouse

Click Here For Bundle PDF Course | support@guidely.in Page 3 of 12


SBI Clerk & RRB PO Mains PDF Course 2023
ENGLISH Day - 38

Direction (11-15): In each question a sentence is immediate actions to avoid similar offenses in
given with a blank. You have to identify which the future.
statement/statements mentioned in the options c. his lunch in the toilet as soon as he arrived
can carry forward the given sentence in the most home after work since he had been having a few
logical way so as to make the sentence coherent drinks and partying recklessly since the
and contextually correct. afternoon.
11. When the company head came to know that d. Only options a and b
the employees of the accounting department e. Options a, b, and c
were cutting corners ________
a. he fired them at the instant as a sign of 13. Linda has been playing tennis since her ninth
warning to the remaining workforce to grade, yet _______
accomplish their duties and responsibilities a. she was awarded the best female tennis
effectively. player for the year 2022 at the annual sports
b. he joined them promptly to make sure that the meet held in Patna last month.
decorations have been done perfectly before the b. she grew up watching players like Novak
arrival of the branch officer. Djokovik and Rafael Nadel play the sport
c. he congratulated them and announced diligently.
incentives appreciating their commitment c. soccer remains to be her favorite sport of all
towards making the company more successful time.
and productive. d. Options a and c
d. at the roads to reach the company, he advised e. None of the above
them to drive safely and follow traffic rules
diligently to avoid accidents and property 14. John’s mother assumed that he was studying
damage. meticulously in his room for his upcoming exams
e. None of the above from the morning, but on the contrary _________
12. The shopkeeper brought up _______ a. he had been up from early morning taking
a. his children in the most diligent ways and notes and learning the necessary topics to
encouraged them to do what their hearts say secure good marks in his exams.
and not go by the societal standards. b. he had been sleeping and wailing away his
b. the issue of the recent thefts in the time all morning just playing games on his
neighborhood at the meeting held by local police computer.
department and requested the police to take c. she was proud of seeing her son studying
seriously to secure good marks in his exams.

Click Here For Bundle PDF Course | support@guidely.in Page 4 of 12


SBI Clerk & RRB PO Mains PDF Course 2023
ENGLISH Day - 38

d. Options a and c 17. Citizens have the right to demur and protest
e. None of the above any law they feel is unconstitutional and
detrimental to the progress of the nation.
15. We, as a community, should extol ______ a. advocate, prohibited, unsupportive
a. the values of honesty and equality and b. expostulate, unlawful, deleterious
incorporate the same in our daily lives to create c. accede ,impermissible, adverse
a fair and corruption-free society for the future d. assent ,unjust, baneful
generations. e. None of the above
b. corrupt and inept politicians and be 18. For a man in imminent danger of losing his
increasingly cautious while electing our leaders job he appeared quite unruffled and nonchalant
in the future. about turning financially unstable with loss of
c. the lethargic attitudes of our leaders who income and livelihood in the near future.
intentionally refrain from meeting up with the a. looming, unperturbed, ardent
public in fear of receiving direct criticisms. b. approaching, tranquil, fervent
d. Options b and c c. impending, placid, insouciant
e. Option a and b d. proximate, detached, avid
e. None of the above
Directions (16-20): In each of the questions
below, a sentence is given with three words 19. An astute observation is thought-provoking,
highlighted in bold. From the given options, unlike an asinine observance that could never
identify the correct set of synonyms for the result in sagacious judgments.
highlighted words. a. gullible, lunatic, obtuse
16. The journey to the mountains was arduous b. ingenuous, absurd, imbecilic
because the weather had been spasmodic lately c. savvy, judicious, purblind
and turned the roads into muddy quagmires. d. shrewd, absurd, sapient
a. formidable, erratic, morasses e. None of the above
b. undemanding, methodical, swamps
c. doable, orchestrated, trails 20. Though the manager claimed to be contrite,
d. facile, ,systematic, bogs she was truly indurate and continued to
e. None of the above denigrate the people around her.
a. repentant, benignant, exalt
b. penitent, callous, disparage
c. pitiless, sympathetic, extol

Click Here For Bundle PDF Course | support@guidely.in Page 5 of 12


SBI Clerk & RRB PO Mains PDF Course 2023
ENGLISH Day - 38

d. unapologetic, amicable, glorify e. None of the above


Click Here to Get the Detailed Video Solution for the above given Questions-Will Update Soon
Or Scan the QR Code to Get the Detailed Video Solutions

Answer Key with Explanation

1. Answer: B talks about the funds that is mentioned in the


Part A is highlighted and fixed and thus, it comes end of part B. Next comes part D since it adds
first. Part A should be followed by part C since further information of the people who are
the part A ends with ‘sentenced a man’ and the mentioned in the end of part B. The right pattern
part C begins with ‘to imprisonment’ (sentenced of rearrangement is ACBED.
in always followed by the preposition ‘to’). Part E The right sentence is:
should come next since this part adds Pointing to alleged discrimination by the previous
information regarding the other penalization government, minority communities like Muslims,
announced for the man apart from imprisonment. Christians, Buddhists, Sikhs, Jains, and Parsis
Part E should be followed by part B since part B that account to almost 16% of the total
proceeds to explain about the offense committed population of Karnataka expect a hike in the
by the accused man. Part D comes finally. Thus, allocation of funds granted by the state
the right rearrangement is ACEBD. government for the welfare and development of
The right sentence is: their people, who are largely backward in many
A sessions court in Jabalpur has sentenced a areas, specifically in education.
man to 110 years of rigorous imprisonment
under non-bailable conditions and imposed a 3. Answer: C
fine of Rs. 25000 for giving fake appointment Part A comes first in the rearrangement since it
letters in the name of MP high court to at least introduces the subject of the sentence. Part B
15 job-seeking youth after charging them Rs does not come first because it contains a phrase
5000 to Rs 30,000 each. that says ‘its May 11 judgment’ which is
contextually false because when placing part B
2. Answer: C first, the pronoun refers to the center but the
Part A comes first since it is highlighted. Next center does not pass judgments in the first place
comes part C since it introduces the subject of (only courts can pass judgments) to eventually
the sentence. Next comes part B since it adds refuse it. Thus, part A comes first. Next comes
further information of the subject of the sentence part C since part A ends with ‘validity’ which is
mentioned in part B. Next comes part E since it often followed by the preposition ‘of’. Next

Click Here For Bundle PDF Course | support@guidely.in Page 6 of 12


SBI Clerk & RRB PO Mains PDF Course 2023
ENGLISH Day - 38

comes part D since it renders information of adopting high density fruit plantation while micro
where the challenge that is mentioned in part A irrigation facility efforts are underway to increase
is referred to (as described in part A again). the area under horticulture in the state.
Since part E is highlighted it comes last and
thus, part B comes fourth. The right pattern of 5. Answer: C
rearrangement is ACDBE. Since part A is highlighted it should come first.
The right sentence is: Part E should come first since it talks about what
The Supreme Court referred the challenge to the the four teams of the NDRF are leading (as
constitutional validity of the recently registered mentioned in part A). Then comes part B since
May 19 Delhi services ordinance to a five-bench part E ends with the word ‘precipitated’ which
judge but refused to stay the order of the center relates to rainfall that is mentioned in part B.
which substantially neglected its May 11 Then comes part D since it adds further
judgment giving legislative and executive control information of the landslide. Part C comes last
over the bureaucracy to the Delhi government. since it proceeds to describe the figures
associated with the disaster in specific. Thus, the
4. Answer: B right pattern of rearrangement is AEBDC.
Part D comes first because it introduces the The right sentence is:
context of the sentence to the readers. Part A Four teams of the National Disaster Response
does not come first because it mentions a Force are leading the rescue operation in Pune
pronoun ‘his’ which has no references to a noun after a massive landslide precipitated by over
mentioned before or in part A. Thus, part D 400 mm of rainfall flattened a remote tribal
comes first. Then comes part A since it talks hamlet named Irshalwadi resulting in one of the
more about the project. Since part C is biggest natural disasters in Maharashtra that
highlighted it comes third. Then comes part B recorded multiple cases of missing residents and
since it explains how to increase fruit production casualties.
that is mentioned in part C. Part E comes last.
The right pattern of rearrangement is DACBE. 6. Answer: B
The right sentence is: Subjugation (noun) – the act of forcing to submit
Chief Minister said that under the Himachal or subdue
Pradesh Shiva Project, the state government as Autonomy (noun) – Immunity from arbitrary
per his recommendation is in the process of exercise of authority: personal independence
developing fruit hubs to increase fruit production Immurement (noun) – The state of being
according to the region-specific conditions by imprisoned

Click Here For Bundle PDF Course | support@guidely.in Page 7 of 12


SBI Clerk & RRB PO Mains PDF Course 2023
ENGLISH Day - 38

Captivity (noun) – The state of being a slave Assuage (verb) - to make more bearable or less
Internment (noun) – The act of confining severe
someone in a prison Proliferate (verb) – cause to grow in number or
Options b, c, d, and e talk about something bad increase rapidly in size
but the blank should be filled with a positive Inflate (verb) – to make greater in size, amount,
noun since the sentence focuses on the or number
attributes that are necessary to create a fair The sentence with blank 8 speaks about
society. Thus, these options are incorrect. changes that could bring about betterment in the
Option a is right since it conveys something lives of women in varied aspects. In specific, the
positive. blank speaks about the extreme physical
responsibilities of women with regard to
7. Answer: A household work. This should be made less
Impede (verb) – Be a hindrance or obstacle to severe or alleviated to lessen their burden in
Streamline (verb) – Contour economically or household work. Options a, b, d, and e are
efficiently, direct incorrect since they talk about worsening,
Incubate (verb) – Grow under conditions that intensifying, and increasing these responsibilities
promote development of women but this does not facilitate the growth
Prompt (verb) - Give an incentive for action of women. Option c talks about alleviating
Coax (verb) - To persuade somebody gently. responsibilities and thus, option c is correct.
The sentence with blank 7 speaks about how the
influences of negative traits like power dynamics 9. Answer: E
and gender inequality in the lives and health of Approbation (noun) – official approval or
women. These negative traits only give rise to recognition
negative influences or actions. Thus, options b, Endorsement (noun) - Formal and explicit
c, d, and e that signify actions like promote, approval
persuade, contour, and growth do not best fit the Countenance (noun) - Formal and explicit
context of the sentence. Thus, option a which approval
denotes hindrance fits the blank rightly. Commendation (noun) - A message expressing
a favorable opinion
8. Answer: C Abrogation (noun) - an official or legal
Intensify (verb) – to make more intense cancellation
Exacerbate (verb) – to make worse The sentence with blank 9 talks about what the
government is expected to do while handling

Click Here For Bundle PDF Course | support@guidely.in Page 8 of 12


SBI Clerk & RRB PO Mains PDF Course 2023
ENGLISH Day - 38

illiteracy amongst girls. Illiteracy should be and hence part A should be used to complete
eliminated and removed to make the lives of the sentence. Part A gives rise to a coherent and
women better. Thus, the blank should include a meaningful sentence.
word that indicates cancellation or elimination. Option b talks about decorating the company,
Thus, option e is right. Options a, b, c, and d that option c talks about something positive, and
talk about approval and favoring are incorrect. option d talks about the ways in which
employees travel on roads. These options do not
match the context of the idiom. Thus, these
10. Answer: D options are incorrect.
Acquiesce (verb) - to give or express one's The right sentence is:
approval (as to a proposal) When the company head came to know that the
Enculturate (verb) – as in to accustom employees of the accounting department were
Assimilate (verb) - to make a part of a body or cutting corners, he fired them at the instant as a
system; integrate sign of warning to the remaining workforce to
oppugn (verb) - to strive to reduce or eliminate accomplish their duties and responsibilities
Espouse (verb) - choose and follow; as of effectively
theories, ideas, policies, strategies or plans
Options a, b, c, and e are incorrect since they 12. Answer: E
talk about getting accustomed to, approving, The part of the sentence given in the question
choosing to follow, integrating, approving and mentions a phrasal verb – brought up.
thus, these words cannot be filled in blank 10 i. Brought up – raise a child
because the context of the sentence deals with ii. Brought up – start talking about a subject to
the what the society should do when it comes to gain attention
gender-based discriminations. It is meaningfully iii. Brought up – vomit or puke
right to say that the society should reduce or Sentences A, B, C rightly fit the context of the
eliminate these discriminations. Thus, option d is sentence given in the question by denoting the
right. meanings expressed in i, ii, and iii. Thus, option
e is the right answer.
11. Answer: A The right sentences include:
The part of the sentence given in the question The shopkeeper brought up his children in the
consists of an idiom ‘cutting corners’ which most diligent ways and encouraged them to do
means to do something poorly in order to save what their hearts say and not go by the societal
time or money. This is a bad working attitude standards.

Click Here For Bundle PDF Course | support@guidely.in Page 9 of 12


SBI Clerk & RRB PO Mains PDF Course 2023
ENGLISH Day - 38

The shopkeeper brought up the issue of the mentioned in the question. Thus, option b is the
recent thefts in the neighborhood at the meeting right answer.
held by local police department and requested Options b and c are incorrect because they just
the police to take immediate actions to avoid add additional information to the detail stated in
similar offenses in the future. the question and do not convey something
The shopkeeper brought up his lunch in the toilet opposite.
as soon as he arrived home after work since he The right sentence is:
had been having a few drinks and partying John’s mother assumed that he was studying
recklessly since the afternoon. meticulously in his room for his upcoming exams
from the morning, but on the contrary he had
13. Answer: C been sleeping and wailing away his time all
The sentence is the question ends with ‘yet’ morning just playing games on his computer.
which is a connector and means ‘but’ or ‘inspite
of something’. Only option c matches the context 15. Answer: A
of the sentence in the question. Options a fits the The sentence has the word ‘extol’ – to praise
sentence when ‘yet’ is replaces with other enthusiastically.
connectors like ‘consequently, thus, therefore, Only statement A speaks about something
etc.’ Option b fits the sentence when ‘yet’ is positive and worthy of praising while statements
replaced by the connector ‘since’. Thus, only B and C talk about something that is bad and not
option c is right. praiseworthy. Thus, only option a is right and
The right sentence is: options b and c are incorrect.
Linda has been playing tennis since her ninth The right sentence is:
grade, yet soccer remains to be her favorite We, as a community, should extol the values of
sport of all time. honesty and equality and incorporate the same
in our daily lives to create a fair and corruption-
14. Answer: B free society for the future generations.
The sentence given in the question consists of a
connector ‘on the contrary’ which is used to
expresses something that is opposite to the 16. Answer: A
information conveyed prior. Thus, option b rightly Arduous (adjective) – requiring much time, effort,
fits the context of the sentence by conveying or careful attention
something that is opposite to the information Spasmodic (adjective) – Occurring in spells and
often abruptly; unpredictable

Click Here For Bundle PDF Course | support@guidely.in Page 10 of 12


SBI Clerk & RRB PO Mains PDF Course 2023
ENGLISH Day - 38

Quagmire (noun) - A soft wet area of low-lying Unruffled (adjective) – free from emotional or
land that sinks underfoot mental agitation
Options b, c, and d contain antonyms of the Nonchalant (Adjective) – having or showing a
word ‘arduous’. Undemanding - Requiring little if lack of interest, feeling or concern.
any patience, effort or skill; doable – easily Options a, b, and d contain antonyms for the
possible to do; facile - Performing adroitly and word ‘nonchalant’.
without effort. Thus, option a is right. Ardent – showing great depth of feeling and
Formidable - requiring considerable physical or enthusiasm; fervent - showing great depth of
mental effort, erratic - lacking a definite plan, feeling and enthusiasm; avid - showing urgent
purpose, or pattern; unpredictable; and desire or interest.
morasses - A soft wet area of low-lying land that Thus, option c is right.
sinks underfoot. Impending – being soon to appear or take place
Placid – free from emotional and mental agitation
17. Answer: B Insouciant – having or showing freedom from
Demur (verb) - to present an opposing opinion or worried or trouble; nonchalant
argument
Unconstitutional (adjective) – not in accordance 19. Answer: D
with the political constitution or with the Astute (adjective) – having or showing a practical
procedural rules cleverness or judgment
Detrimental (adjective) – tending to cause harm Asinine (adjective) – showing or marked by lack
Options a, c, d contain antonyms for the word of good sense of judgment
‘demur’. Thus, these options are incorrect. Sagacious (adjective) – having or showing deep
Advocate (verb) – publicly recommend; accede understanding and intelligent application of
(verb) – agree to demand, request, ideas; assent knowledge
(verb) – express approval or agreement. Option a, b, and c contain antonyms for the word
Thus, option b is right. sagacious. Obtuse - Lacking in insight or
Expostulate – to present an opposing opinion or discernment; dumb, imbecilic - Devoid of
argument; unlawful – contrary to or forbidden by good sense or judgment; purblind - Lacking in
law; deleterious – capable of causing harm. insight or discernment; obtuse
Thus, option d is the right answer.
18. Answer: C Shrewd – having or showing practical cleverness
Imminent (adjective) – giving signs of immediate or judgment; absurd – showing or marked by
occurrence lack of good sense or judgment; sapient - having

Click Here For Bundle PDF Course | support@guidely.in Page 11 of 12


SBI Clerk & RRB PO Mains PDF Course 2023
ENGLISH Day - 38

or showing deep understanding and intelligent Benignant having or marked by sympathy and
application of knowledge. consideration for others, sympathetic – having or
showing the capacity for sharing the feelings of
20. Answer: B one another; amicable – having or showing
Contrite (adjective) – feeling sorrow for a wrong kindly feeling or sincere interest.
that one has done, remorseful Thus, option b is right.
Indurate (adjective) – having or showing a lack of Penitent – feeling sorrow for a wrong that one
sympathy or tender feeling, considerate or has done; callous - having or showing a lack of
unkind sympathy or tender feeling, considerate or
Denigrate (verb) – to express scornfully ones low unkind; disparage – to express scornfully one’s
opinion of something or something; belittle low opinion of something or something; belittle.
Options a, c, and d are incorrect since they
contain antonyms of the word indurate.

Click Here For Bundle PDF Course | support@guidely.in Page 12 of 12


SBI Clerk & RRB PO Mains PDF Course 2023
Reasoning Ability Day - 39 (Eng)

Reasoning Ability
Directions (1-5): Study the following information c) Only I
carefully and answer the given questions. d) Only I and III
Ten persons - A, B, C, D, E, F, G, I, O, and U are e) Only II
living on different floors of a twelve storey
building where the lowermost floor is numbered 2. How many persons live between the two
one and the floor immediately above it is vacant floors as per the given arrangement?
numbered two and so on. No two persons live on a) Three
the same floor. b) As many persons live between D and F
Note 1: If the persons whose name is a vowel, c) Four
then they are living on the even numbered floors, d) Five
while others are living on the odd numbered e) As many persons live between C and A
floors.
Note 2: There are two vacant floors. None of the 3. Who among the following person lives on the
adjacent floors are vacant. Neither the topmost third floor?
nor the lowermost floor is vacant a) B
Not more than three floors are above C. At least b) The one who lives three persons below D
five persons live between C and G. Only four c) D
floors are between G and A. The number of d) The one who lives immediately above E
floors between A and B is three more than the e) G
number of floors between E and I. E lives above
B. D lives three floors below I. The number of 4. In which of the following the sum of the floor
persons living above D is two more than the number is Maximum?
number of persons living below F. Not more than I. I and O
one person lives between O and U, who lives II. C and F
below O. III. E and U
1. If Z occupies one of the vacant floors, then a) Only III
how many persons live above O as per the given b) Only I and II
arrangement? c) Only II
I. As many persons live below C d) Only I and III
II. Six e) Only I
III. Seven
a) Only III
b) Only I and II

Click Here For Bundle PDF Course | support@guidely.in Page 1 of 9


SBI Clerk & RRB PO Mains PDF Course 2023
Reasoning Ability Day - 39 (Eng)

5. If B is related to D and O is related to G in a


certain way, then who among the following
person is related to A?
a) F
b) C
c) E
d) U
e) None of these

Direction (6-10): Study the following statements a) Only AP follows


and then decide which of the given conclusions b) Both BR and CQ follow
logically follows from the given statements c) Only CQ follows
disregarding the commonly known facts. d) Only BQ follows
6. e) Both AR and CP follow

8.

a) Only AR follows
b) Both BP and CR follow a) Only BQ follows

c) Only AQ follows b) Both AR and CP follow

d) Both AQ and CR follow c) Both BR and AP follow

e) Only BP follows d) All AQ, BR and CP follow


e) Only BR follows

7.
9.

Click Here For Bundle PDF Course | support@guidely.in Page 2 of 9


SBI Clerk & RRB PO Mains PDF Course 2023
Reasoning Ability Day - 39 (Eng)

Directions (11-15): Study the following


information carefully and answer the given
questions.
“A % B (37)” means “A is 22m north of B”
“A # B (18)” means “A is 26m east of B”
“A @ B (25)” means “A is 13m south of B”
“A & B (48)” means “A is 25m west of B”
If “A %# B” means “A is northeast of B”
Statement:
U%V (33); Q#R (8); S@Q (36); P@R (24); W#V
(9); Q&V (44); T&U (50); Y@W (31).
a) Both BP and CQ follow
11. If X&T (33) then what is the direction and
b) Only BR follows
shortest distance of X with respect to R?
c) Both AQ and BR follow
a) South, 18m
d) Only CP follows
b) North, 18m
e) All CR, AQ and BP follow
c) South, 16m
d) North, 16m
10.
e) None of these

12. What is the direction of T with respect to S?


a) T%S
b) T@&S
c) T%&S
d) S@&T
e) S%#T

13. Four of the following five pairs of points are


alike in a certain way based on the directions in
a) Both CP and AQ follow the given arrangement and thus form a group.
b) Only CP follows Which one of the following does not belong to
c) Only AP follows the group?
d) Both BR and AQ follow a) QP
e) None follows b) VS
c) UV

Click Here For Bundle PDF Course | support@guidely.in Page 3 of 9


SBI Clerk & RRB PO Mains PDF Course 2023
Reasoning Ability Day - 39 (Eng)

d) TR post. At least one person and not more than two


e) WS persons work in each Clerk post.
Note: If it is given that the person who is a
14. If Z%&S and the distance between Q and Z Probationary officer or Clerk works with person X
is 10m less than the distance between Q and V or the one whose age is Y, then it indicates that
in the final arrangement, then what is the both persons work in the same bank but not
shortest distance between Z and P? necessarily at the same post. If it is given that X
a) 12m works with one whose age is “Y” years, then X’s
b) 11m age is not “Y” years.
c) 9m N, who works in UCO bank, neither works in the
d) 13m same bank nor in the same post as K. K works
e) 14m with the one whose age is 28 years but not in
PNB bank. The age of M is a perfect square and
15. If K@#T and K@U, then what is the direction works with P in the same post but works neither
of K with respect to V? in Indian bank nor in UCO bank. L is 31 years old
a) East and does not work in PNB bank. The difference
b) North between the ages of L and the one who works in
c) West PNB bank is 29 years. The sum of the ages of M
d) South and the one who works with M is 90 years. The
e) Either b or d one who works in Indian Bank is 15 years
younger than P. The age of N is 31 years elder
Directions (16-20): Study the following than O. O is not working in UCO bank. I is
information carefully and answer the given younger than J and the difference between the
questions. ages of them is 6 years.
Eight persons – I, J, K, L, M, N, O and P are 16. Which of the following persons work in
working in three different banks viz., Indian Bank, Punjab National Bank (PNB) bank?
UCO Bank, and Punjab National Bank (PNB) at I. The one whose age is 60 years
different posts – Probationary officer (PO) and II. I
Clerk. They are all of different ages. The age of III. The one whose age is 41 years
the person who is a Probationary officer is a a) Only III
prime number and the age of the person who is a b) Only I and II
Clerk is a composite number. Each person’s age c) Only II
is between 21 and 65 years. Not more than one d) Only I and III
person is working in each Probationary officer e) All I, II and III

Click Here For Bundle PDF Course | support@guidely.in Page 4 of 9


SBI Clerk & RRB PO Mains PDF Course 2023
Reasoning Ability Day - 39 (Eng)

17. How many persons are younger than the one I. I is four years younger than L
who works in UCO bank as PO? II. The age of P is 60 years
a) Five III. Only two persons work in UCO Bank
b) Four a) Only III
c) Six b) Only I and II
d) Three c) Only II and III
e) None d) Only I
e) All I, II and III
18. What is the sum of the ages of the persons
who work in Indian Bank? 20. Who among the following person is not
a) 104 years working as a Clerk?
b) 102 years a) P
c) 105 years b) J
d) 100 years c) K
e) 108 years d) L
e) Both b and d
19. Which of the following statement(s) is/are
true as per the given arrangement?
Click Here to Get the Detailed Video Solution for the above given Questions
Or Scan the QR Code to Get the Detailed Video Solutions

Answer Key with Explanation

Directions (1-5): 4. Answer: D


1. Answer: D 5. Answer: E
2. Answer: B Final arrangement:
3. Answer: B

Click Here For Bundle PDF Course | support@guidely.in Page 5 of 9


SBI Clerk & RRB PO Mains PDF Course 2023
Reasoning Ability Day - 39 (Eng)

 D lives three floors below I.


After applying the above conditions, case 1a
gets eliminated because more than three floors
between A and B.

 Not more than three floors are above C.


 At least five persons live between C and
G.
 Only four floors are between G and A. Again, we have
After applying the above conditions, there are  The number of persons living above D is
four possibilities. two more than the number of persons
living below F.
 Not more than one person lives between
O and U, who lives below O.
After applying the above condition, case 1 and
case 2 are eliminated because the number of
persons living above D is two more than the
number of persons living below F is not satisfied,
hence case 2a shows the final arrangement.

Again, we have
 The number of floors between A and B is
three more than the number of floors
between E and I.
 E lives above B.

Click Here For Bundle PDF Course | support@guidely.in Page 6 of 9


SBI Clerk & RRB PO Mains PDF Course 2023
Reasoning Ability Day - 39 (Eng)

9. Answer: E

10. Answer: B
6. Answer: D

Directions (11-15):
% - (37m-22m) – (-15m)
# - (18m-26m) – (+8m)
7. Answer: C
@ - (25m-13m) – (-12m)
& - (48m-25m) – (-23m)

8. Answer: E

Click Here For Bundle PDF Course | support@guidely.in Page 7 of 9


SBI Clerk & RRB PO Mains PDF Course 2023
Reasoning Ability Day - 39 (Eng)

We have,
 N, who works in UCO bank, neither works in
the same bank nor same post as K.
 K works with the one whose age is 28 years
but not in PNB bank.
 The age of M is a perfect square and works
with P in the same post but works neither in
11. Answer: B Indian Bank nor in UCO bank.
12. Answer: C After applying above conditions, there are two
13. Answer: C (First point is northeast of second possibilities.
point except option c)
14. Answer: D
If Z%&S,
The distance between Z and Q is 11m and Z is
in west of Q.
(PZ)2 = (ZR)2 + (RP)2
Again, we have
= (5)2 + (12)2
 L is 31 years old and does not work in PNB
= 25 + 144
bank.
(PZ)2 = 169
 The difference between the ages of L and the
PZ = 13m
one who works in PNB bank is 29 years.
15. Answer: E
 The sum of the ages of M and the one who
works with M is 90 years.
Directions (16-20):
After applying the above conditions, the age of
16. Answer: D
the one who works in PNB is 60 years because
17. Answer: C
the difference between the ages of L and the
18. Answer: A
one who works in PNB bank is 29 years and also
19. Answer: C
the only possible age of M is 49 years and the
20. Answer: E
one who works with M is 41 years because the
Final arrangement:
sum of the ages of M and the one who works

Click Here For Bundle PDF Course | support@guidely.in Page 8 of 9


SBI Clerk & RRB PO Mains PDF Course 2023
Reasoning Ability Day - 39 (Eng)

with M is 90 years and M’s age is a perfect elder than O, hence the possible age of N is 72
square. years but the age of each person is not more
than 65 years.
In case 1, the one who works in Indian bank is
45 years old because the one who works in
Indian Bank is 15 years younger than the age of
P and also the age of N and O is 59 years and
28 years respectively because the age of N is 31
Again, we have years elder than O and O is not working in UCO
 The one who works in Indian Bank is 15 bank.
years younger than P.
 The age of N is 31 years elder than O.
 O is not working in UCO bank.
 I is younger than J and the difference
between the ages of them is 6 years.
After applying the above conditions, case 2 gets
eliminated because the age of N is 31 years

Click Here For Bundle PDF Course | support@guidely.in Page 9 of 9


SBI Clerk & RRB PO Mains PDF Course 2023
Quantitative Aptitude Day – 39 (Eng)

Quantitative Aptitude

Direction (1-5): Study the following data carefully and answer the questions:
There are 5 families A, B, C, D and E and each family has a husband, a wife and a child.
The pie chart given below shows the percentage distribution of present ages of husbands of each family.

The line graph given below shows the difference between the husband’s present age and the wife’s
present age and also shows the difference between the wife’s present age and the child’s present age in
each family.

Click Here For Bundle PDF Course | support@guidely.in Page 1 of 13


SBI Clerk & RRB PO Mains PDF Course 2023
Quantitative Aptitude Day - 39 (Eng)

Note: The Average of the present age of children in each family is 15.6 years.
1) If the average age of family B is P years, the b) 27.25 years
average age of family C is Q years and the c) 22.25 years
average age of family D is R years, then find the d) 26.25 years
ratio between P, Q and R respectively? e) 24.25 years
a) 24: 18: 29
b) 60: 54: 73 4) If the present ages of wives of each family are
c) 240: 216: 273 described in a circle and the percentage
d) 4: 3: 5 distributions of present ages of wives of families
e) 120: 108: 137 A, C and D are M%, N% and O%, then find the
value of ?
2)If the ratio between the child’s age of family A a) 12
after ‘a’ years, the child’s age of family B after ‘b’ b) 18
years and the child’s age of family C after ‘c’ c)9
years is 2: 4: 3 respectively, then find that the d) 16
value of [abc – (a + b + c)] is divisible by which of e) 6
the following?
a) 47 5) If the husband’s present age in family B is
b) 19 35% more than that in family F, the wife’s
c)37 present age in family F is 87.5% of that in family
d) 21 E and the average age of family F is 70% of that
e) Can’t be determined of family B, then find the child’s present age in
family F is what per cent of that in family C?
3) When a new child was born in family A after ‘t’ a) 44.44%
years, the average age of family A becomes b) 53.33%
21.25 years. If a new child was born in family E c)36.36%
after (t + 1) years, then what will be the average d) 56.25%
age of family E at that time? e) 47.5%
a) 25.25 years

Direction (6-10): Study the following data carefully and answer the questions:
The data given below is related to the total earnings (in crore rupees) of 5 movies A, B, C, D and E in two
days: Mon and Tue.

Click Here For Bundle PDF Course | support@guidely.in Page 2 of 13


SBI Clerk & RRB PO Mains PDF Course 2023
Quantitative Aptitude Day - 39 (Eng)

Note:
1: On Tue, the earnings of movie D is ₹ 11.4 crore more than that of movie E and also ₹ 9.6 crore more
than that of movie A.
2: The earnings of movie C on Tue is ₹ 21.6 crore less than that on Mon.
6)If the average earnings of movie A on Mon, c) ₹ 30 crore
Tue and Wed is ₹ 34.8 crore and the average d) ₹ 33 crore
earnings of movie C on Mon, Tue and Wed is e) None of these
₹ 36.8 crore, then find that which of the following
can’t be the ratio of earnings of movie A on Wed 8) Which of the following is/are true.
to that of movie C on Wed? P: The Ratio of total earnings of movie A on
a) x: y Mon and Tue together to that of movie B on Mon
b) (z – 3): (x – 1) and Tue together is 129: 105.
c) x: (z – 2) Q: Average earnings of movies B, C and E on
d) (y + 1): (x – 1) Mon is ₹ 42 crore.
e) Both (b) and (d) R: Total earnings of movies C and E together on
Mon is 70% of that of movies C and E together
7)If the earnings of movie B on Wed is 12(x + y – on Mon.
z)% of that on Mon, the earnings of movie D on a) Only P
Wed is (xz)% of that on Mon and the earnings of b) Only P and Q
movie E on Wed is (yz)% of that on Mon, then c) R
find the average earnings of movies B, D and E d) Only P and R
on Wed? e) Only Q
a) ₹ 30.3 crore
b) ₹ 33.3 crore

Click Here For Bundle PDF Course | support@guidely.in Page 3 of 13


SBI Clerk & RRB PO Mains PDF Course 2023
Quantitative Aptitude Day - 39 (Eng)

9) If the earnings of movie F on Mon is 40% 10) If the average earnings of movie D from Wed
more than that of movie D on Mon, the earnings to Sun is ₹ 28.4 crore and the average earnings
of movie F on Tue is 25% more than that of of movie E from Wed to Sun is ₹ 20.28 crore,
movie D on Tue and the ratio of the earnings of then find the ratio of the total earnings of movie
movie F on Mon to that on Tue is M: N, then find D from Mon to Sun to that of movie E from Mon
that the sum of M and N is divisible by which of to Sun?
the following? a) 112: 105
a) 31 b) 4: 3
b) 17 c) 56: 51
c) 9 d) 28: 23
d) Both (a) and (b) e) 8: 5
e) Bothe (b) and (c)

Direction (11-15): Study the following data carefully and answer the questions:
The data given below is related to the costs of three articles P, Q and R at 5 different shops A, B, C, D
and E. Cost of each article at each shop is different.
Line graph given below shows the following data:

11) If the sum of the cost of article P at shop A of the cost of article R at shop A to the cost of
and the cost of article Q at shop C is ₹ 1110 and article R at shop C?
the cost of article P at shop A is ₹ 50 less than a) 66: 37
the cost of article P at shop C, then find the ratio b) 11: 7

Click Here For Bundle PDF Course | support@guidely.in Page 4 of 13


SBI Clerk & RRB PO Mains PDF Course 2023
Quantitative Aptitude Day - 39 (Eng)

c) 33: 14 than the cost of article R at shop F. At shop F, if


d) 55: 24 the cost of article Q is 40% of that of article P
e) 22: 17 and the cost of article R is 125% of that of article
Q, then find that the cost of article P at shop F is
12) If the ratio of the cost of article P at shop D to what per cent more/less than the cost of article P
the cost of article P at shop E is 3: 4 and the at shop D?
average of the cost of article R at shops D and E a) 21%
is ₹ 391.5, then find which of the following is/are b) 25%
true? c) 24%
X: The average of the costs of articles P, Q and d) 20%
R at shop D is ₹ 465. e) 22%
Y: The difference between the cost of article Q at
shops D and E is ₹ 180. 15) If the ratio between the cost of article P at
Z: The ratio of the cost of article R at shop D to shops B, C and D is 4: 8: 3 and the average of
the cost of article R at shop E is 15: 13. the cost of article Q at shops B, C and D is
a) Only X and Y ₹ 610, then find the average of the cost of article
b) Only Y R at shops B, C and D?
c) All are true a) Can’t be determined
d) Only Y and Z b) ₹ 504
e) Only X c) ₹ 324
d) ₹ 576
13) If the ratio between the cost of article P at e) ₹ 423
shops A, B and C is 15: 8: 40 and the average of
the cost of article Q at shops A, B and C is Direction (16-18): Study the following data
₹ 400, then find the average of the cost of article carefully and answer the questions:
R at shops A, B and C? The data given below is related to the number of
a) ₹ 356 views (in lakhs) on two YouTube videos A and B
b) ₹ 326 uploaded on Mon.
c) ₹ 346 Three equations I, II and III are given below:
d) ₹ 336 I: x + y + z = 37
e) ₹ 366 II: 4x + 5y = 140
III: 4y + 3z = 63
14) The cost of article P at shop D is ₹ 750 and The Number of views on video A on Mon is [25%
the cost of article R at shop D is ₹ 210 more of (y * z)] lakhs and also 25% more than the

Click Here For Bundle PDF Course | support@guidely.in Page 5 of 13


SBI Clerk & RRB PO Mains PDF Course 2023
Quantitative Aptitude Day - 39 (Eng)

number of views on video B on Mon. The 18) If On Mon, the number of views on video C is
Number of views on video A till Tue is [10% of (x (x – 5)% more than that on video B, on Tue, the
* y)] lakhs and also 50% more than the number number of views on video C is (y + 3)% more
of views on video B till Tue. Number of views on than that on video B and on Wed, the number of
video A till Wed is (x + y – z) lakhs and the views on video C is (z + 5)% more than that on
number of views on video B till Wed is (x + y – video B, then find the total number of views on
2z) lakhs. video C on Mon, Tue and Wed together?
16) If the ratio between the number of views on a) (y + z + 2) lakhs
video A on Mon, Tue and Wed is p: q: r b) (2y – z) lakhs
respectively and the ratio between the number of c) (x + y – 2z) lakhs
views on video B on Mon, Tue and Wed d) (x + z – 4) lakhs
respectively is a: b: c, then find the ratio of (p + q e) (x + z) lakhs
+ r) to (a + b + c)?
a) 9: 11 Direction (19-20): In each question, column A
b) 5: 8 and B are given. Three equations P, Q and R are
c) 8: 11 given in column A and their roots are given in
d) 7: 8 column B. Find the correct combination of the
e) None of these equations with their roots.
19)
17) The number of views on video A is increased
by from Wed to Thu and the number of
views on video B is increased by 50% from Wed
to Thu. If the number of views on video A is
increased by M% from Mon to Thu and the
number of views on video B is increased by N%
a) P-M, P-N, Q-M, R-N, R-O
from Mon to Thu, then find the ratio of M to N?
b) P-M, Q-M, Q-N, R-M, R-O
a) 14: 17
c) P-M, Q-M, Q-N, R-N, R-O
b) 70: 87
d) P-N, Q-M, Q-N, R-N, R-O
c) 4: 5
e) P-M, Q-M, Q-O, R-N, R-O
d) 35: 41
e) 2: 3
20)

Click Here For Bundle PDF Course | support@guidely.in Page 6 of 13


SBI Clerk & RRB PO Mains PDF Course 2023
Quantitative Aptitude Day - 39 (Eng)

a) P-M, Q-N, R-O


b) P-O, P-M, R-N
c) Q-M, Q-N, R-O
d) P-O, Q-N, R-M
e) P-M, P-N, R-O

Click Here to Get the Detailed Video Solution for the above given Questions
Or Scan the QR Code to Get the Detailed Video Solutions

Answer Key with Explanation

Direction (1-5):
Let the present ages of husbands in families A,
100x – 162 = 78
B, C, D and E are 15x years, 22.5x years, 20x
x = 2.4
years, 25x years and 17.5x years respectively.
So, the present age of child in family A = (15x –
3 – 26) = (15x – 29) years
The present age of child in family B = (22.5x – 6
– 30) = (22.5x – 36) years
The present age of child in family C = (20x – 4 –
28) = (20x – 32) years
The present age of child in family D = (25x – 5 –
33) = (25x – 38) years
1) Answer: E
And the present age of child in family E = (17.5x
The average age of family B = P = = 40
– 2 – 25) = (17.5x – 27) years
years
Since, the average of the present ages of child
The average age of family C = Q = = 36
of each family is 15.6 years.
years
So,

Click Here For Bundle PDF Course | support@guidely.in Page 7 of 13


SBI Clerk & RRB PO Mains PDF Course 2023
Quantitative Aptitude Day – 39 (Eng)

The average age of family D = R = 5) Answer: D


Since, the husband’s present age in family B =
Required ratio = P: Q: R = 120: 108: 137 54 years
So, the husband’s present age in family F =
2) Answer: E = 40 years
From the question: Since, the wife’s present age in family E = 40
years
So, the wife’s present age in family F = 87.5% of
And,
40 = 35 years
Since, the average age of family B = =
Since, there are 2 equations and 3 variables, so
40 years
the equations can’t be solved and the answer
So, the average age of family F = 70% of 40 =
can’t be determined.
28 years
And the child’s present age in family F = (3 * 28)
3) Answer: B
– 40 – 35 = 9 years
Since, whena new child was born in family A
Since, the child’s present age in family C = 16
after ‘t’ years, the average age of family A
years
becomes 21.25 years.
So, required percentage = = 56.25%
So,

Direction (6-10):
t=3 Earnings of movie D on Tue = 105% of 40 =
Now, the average age of family E at the time of a ₹ 42 crore
child born after (t + 1) years: So,
42 – 5(x + y)% of 36 = 11.4
5(x + y)% of 36 = 30.6
4) Answer: A x + y = 17 -----------------(1)
The sum of present ages of wives of each family And,
= 33 + 48 + 44 + 55 + 40 = 220 years 42 – 9y% of 45 = 9.6
So, M = = 15% 9y% of 45 = 32.4
N= = 20% y=8

And, O = = 25% From equation (1):

Now, the value of = = 12 x=9

Click Here For Bundle PDF Course | support@guidely.in Page 8 of 13


SBI Clerk & RRB PO Mains PDF Course 2023
Quantitative Aptitude Day – 39 (Eng)

Since, the earnings of movie C on Mon = (7 * 8 – From option (d):


2) = ₹ 54 crore (y + 1): (x – 1) = 9: 8
So, the earnings of movies C on Tue: So, option (d) can’t be the answer.
6z% of 54 = 54 – 21.6 Hence, the correct answer is option (b).
z = 10
7) Answer: C
The earnings of movie B on Wed = 12(x + y –
z)% of 30 = 12(9 + 8 – 10)% of 30 = ₹ 25.2 crore
The earnings of movie D on Wed = (xz)% of 40 =
90% of 40 = ₹ 36 crore
The earnings of movie E on Wed = (yz)% of 36 =
80% of 36 = ₹ 28.8 crore
Required average = = ₹ 30 crore

8) Answer: D
6) Answer: B
From P:
Since, the total earnings of movie A on Mon, Tue
Total earnings of movie A on Mon and Tue
and Wed together = 3 * 34.8 = ₹ 104.4 crore
together = 45 + 32.4 = ₹ 77.4 crore
So, the earnings of movie A on Wed = 104.4 –
Total earnings of movie B on Mon and Tue
45 – 32.4 = ₹ 27 crore
together = 30 + 33 = ₹ 63 crore
Since, the total earnings of movie C on Mon, Tue
Required ratio = 77.4: 63 = 774: 630 = 129: 105
and Wed together = 3 * 36.8 = ₹ 110.4 crore
So, P is true.
So, the earnings of movie C on Wed = 110.4 –
From Q:
54 – 32.4 = ₹ 24 crore
Average earnings of movies B, C and E on Mon:
Required ratio = 27: 24 = 9: 8
From option (a):
So, Q is not true.
x: y = 9: 8
From R:
So, option (a) can’t be the answer.
Total earnings of movies C and E together on
From option (b):
Mon = 54 + 36 = ₹ 90 crore
(z – 3): (x – 1) = 7: 8
Total earnings of movies C and E together on
So, option (b) can be answer.
Tue = 32.4 + 30.6 = ₹ 63 crore
From option (c):
Required percentage = = 70%
x: (z – 2) = 9: 8
So, R is true.
So, option (c) can’t be the answer.

Click Here For Bundle PDF Course | support@guidely.in Page 9 of 13


SBI Clerk & RRB PO Mains PDF Course 2023
Quantitative Aptitude Day – 39 (Eng)

Hence, only P and R are true. By equation (1) + equation (2) * 10:
20x + 9y + 20y – 20x = 222 + 10
9) Answer: A y = 8, x = 7.5
Since, the earnings of movie D on Mon = ₹ 40 The cost of article P at shop A = 100 * 7.5 =
crore ₹ 750
So, the earnings of movie F on Mon = 140% of The cost of article R at shop A = 110% of 80% of
40 = ₹ 56 crore 750 = ₹ 660
Since, the earnings of movie D on Tue = ₹ 42 The cost of article P at shop C = 100 * 8 = ₹ 800
crore The cost of article R at shop C = 80% of 45% of
So, the earnings of movie F on Tue = 125% of 800 = ₹ 288
42 = ₹ 52.5 crore Required ratio = 660: 288 = 55: 24
So, M: N = 56: 52.5 = 112: 105
And the sum of M and N = 112 + 105 = 217 12) Answer: A
The sum of M and N is divisible by 31 only. Let the cost of article P at shops D and E is
₹ 300x and ₹ 400x respectively.
10) Answer: B So, the cost of article R at shop D = 75% of
Total earnings of movie D from Mon to Sun: 120% of 300x = ₹ 270x
40 + 42 + (5 * 28.4) = ₹ 224 crore And the cost of article R at shop E = 105% of
Total earnings of movie E from Mon to Sun: 60% of 400x = ₹ 252x
36 + 30.6 + (5 * 20.28) = ₹ 168 crore So,
Required ratio = 224: 168 = 4: 3 270x + 252x = 2 * 391.5
x = 1.5
11) Answer: D From X:
Let the cost of article P at shop A = ₹ 100x Cost of article P at shop D = 300 * 1.5 = ₹ 450
Also let the cost of article P at shop C = ₹ 100y Cost of article Q at shop D = 120% of 450 =
And the cost of article Q at shop C = 45% of ₹ 540
100y = ₹ 45y Cost of article R at shop D = 75% of 540 = ₹ 405
So, Required average = = ₹ 465
100x + 45y = 1110 So, X is true.
20x + 9y = 222 --------------(1) From Y:
And, Cost of article Q at shop D = 120% of (300 * 1.5)
100y – 100x = 50 = ₹ 540
2y – 2x = 1 ---------------(2)

Click Here For Bundle PDF Course | support@guidely.in Page 10 of 13


SBI Clerk & RRB PO Mains PDF Course 2023
Quantitative Aptitude Day – 39 (Eng)

Cost of article Q at shop E = 60% of (400 * 1.5) 14) Answer: C


= ₹ 360 The cost of article P at shop D = ₹ 750
Required difference = 540 – 360 = ₹ 180 The cost of article R at shop D = 75% of 120% of
So, Y is true. 750 = ₹ 675
From Z: The cost of article R at shop F = 675 – 210 =
Cost of article R at shop D = 75% of 120% of ₹ 465
(300 * 1.5) = ₹ 405 The cost of article Q at shop F = =
Cost of article R at shop E = 105% of 60% of ₹ 372
(400 * 1.5) = ₹ 378 The cost of article P at shop F = =
Required ratio = 405: 378 = 15: 14 ₹ 930
So, Z is not true. Required percentage = = 24%
Hence, only X and Y are true.

15) Answer: B
13) Answer: E Let the cost of article P at shops B, C and D is
Let the cost of article P at shops A, B and C is ₹ 40x, ₹ 80x and ₹ 30x respectively.
₹ 15x, ₹ 8x and ₹ 40x respectively. So, the cost of article Q at shop B = 125% of 40x
So, the cost of article Q at shop A = 80% of 15x = ₹ 50x
= ₹ 12x The cost of article Q at shop C = 45% of 80x =
The cost of article Q at shop B = 125% of 8x = ₹ 36x
₹ 10x And the cost of article Q at shop D = 120% of
And the cost of article Q at shop C = 45% of 40x 30x = ₹ 36x
= ₹ 18x So,
So, 50x + 36x + 36x = 3 * 610
12x + 10x + 18x = 3 * 400 x = 15
x = 30 The cost of article R at shop B = 90% of (50 *
Cost of article R at shop A = 110% of (12 * 30) = 15) = ₹ 675
₹ 396 The cost of article R at shop C = 80% of (36 *
Cost of article R at shop B = 90% of (10 * 30) = 15) = ₹ 432
₹ 270 The cost of article R at shop D = 75% of (36 *
Cost of article R at shop C = 80% of (18 * 30) = 15) = ₹ 405
₹ 432 Required average = = ₹ 504
Required average = = ₹ 366

Direction (16-18):

Click Here For Bundle PDF Course | support@guidely.in Page 11 of 13


SBI Clerk & RRB PO Mains PDF Course 2023
Quantitative Aptitude Day – 39 (Eng)

By equation (I) * 4 – equation (II): The number of views on video A on Mon = 15


4x + 4y + 4z – 4x – 5y = 148 – 140 lakhs
4z – y = 8 ------------(IV) So, M = = 140%
By equation (III) + equation (IV) * 4: The number of views on video B till Thu = 150%
4y + 3x + 16z – 4y = 63 + 32 of 22 = 33 lakhs
z=5 The number of views on video B on Mon = 12
From equation (IV): lakhs
y = 12 So, n = = 175%
From equation (I): Required ratio = M: N = 140: 175 = 4: 5
x = 20
18) Answer: E
The number of views on video C on Mon = [100
+ (20 – 5)]% of 12 = 13.8 lakhs
The number of views on video C on Tue = [100 +
(12 + 3)]% of 4 = 4.6 lakhs
The number of views on video C on Wed = [100
+ (5 + 5)]% of 6 = 6.6 lakhs
Required sum = 13.8 + 4.6 + 6.6 = 25 lakhs = (x
+ z) lakhs

16) Answer: A 19) Answer: C


The ratio of between the number of views on From equation P:
video A on Mon, Tue and Wed respectively = p:
x2 + 23x – 420 = 0
q: r = 15: 9: 3 = 5: 3: 1 x2 – 12x + 35x – 420 = 0
The ratio of between the number of views on
x(x – 12) + 35(x – 12) = 0
video B on Mon, Tue and Wed respectively = a: x = -35, 12
b: c = 12: 4: 6 = 6: 2: 3 From equation Q:
The ratio of (p + q + r) to (a + b + c) y2 – 7y – 60 = 0
(p + q + r): (a + b + c) = (5 + 3 + 1): (6 + 2 + 3) = y2 – 12y + 5y – 60 = 0
9: 11 y(y – 12) + 5(y – 12) = 0
y = -5, 12
17) Answer: C
From equation R:
The number of views on video A till Thu =
z2 – 13z – 90 = 0
of 27 = 36 lakhs

Click Here For Bundle PDF Course | support@guidely.in Page 12 of 13


SBI Clerk & RRB PO Mains PDF Course 2023
Quantitative Aptitude Day – 39 (Eng)

z2 – 18z + 5z – 90 = 0 From equation Q:


z(z – 18) + 5(z – 18) = 0 4y2 – 40y + 99 = 0
z = -5, 18 4y2 – 18y – 22y + 99 = 0
The correct combination: P-M, Q-M, Q-N, R-N, 2y(2y – 9) – 11(2y – 9) = 0
R-O y = 11/2, 9/2
From equation R:
20) Answer: E 2z2 – 31z + 120 = 0
From equation P: 2z2 – 16z – 15z + 120 = 0
2x2 – 29x + 95 = 0 2z(z – 8) – 15(z – 8) = 0
2x2 – 19x – 10x + 95 = 0 z = 15/2, 8
x(2x – 19) – 5(2x – 19) = 0 The correct combination: P-M, P-N, R-O
x = 5, 19/2

Click Here For Bundle PDF Course | support@guidely.in Page 13 of 13


SBI Clerk & RRB PO Mains PDF Course 2023
ENGLISH Day - 39

English Language

Directions (1-5): The questions given below carry 2)


tables in which parts of sentences have been Column I Column II
given in two columns. There are connectors A. Currently, India D. the competence
given below the table that might join the given majority imports solar gap is addressed.
parts into complete and meaningful sentences. panels from China
Choose the best combinations as your answer B. The current Indian E. raises legitimate
from the given options. regulatory environment concerns for India's
1) is very congenial to national security
Column I Column II production solar panels
A. This figure pales in D. zero-tillage, drip domestically
comparison to biofertilizers and C. This brazen act of F. the domestic panel
countries like the UK biopesticides now violence highlight the manufacturers have
and USA, occupy the centre stage. growing boldness of limited updated
Khalistani groups capacity.
B. In India, there is E. growth in food grain I. As
little scope for production has to come II. For
bringing in more area largely through III. And
under cultivation; productivity A. B-(i)-D
enhancement. B. C-(iii)-E
C. A-(ii)-F
C. Resource- F. have 34 and 24 such D. None matches
conserving and sites, respectively. E. All (a), (b) and (c)
climate-smart
technologies 3)
I. Which… Column I Column II
II. Such as… A. Employee burnout has D. experience chronic
III. Therefore… become a pervasive stress at their
A. A-(i)-F, B-(iii)-E, C-(ii)-D issue for many workplace
B. B-(ii)-D, A-(iii)-E, C-(i)-F individuals
C. Only A-(iii)-F B. A vital aspect of E. valued for their
D. B-(ii)-E, A-(i)-D employee well-being is contributions.
E. None matches feeling recognised

Click Here For Bundle PDF Course | support@guidely.in Page 1 of 9


SBI Clerk & RRB PO Mains PDF Course 2023
ENGLISH Day - 39

C. Premier research F. many companies I. while


institutions and closely monitor the II. that
universities have long discoveries from III. and
been collaborating with these labs. A. A-(i)-E, B-(iii)-D
industries, B. B-(ii)-F, A-(i)-D, C-(iii)-E
I. Who C. Only B-(iii)-E
II. And D. A-(ii)-F, B-(i)-D, C-(iii)-E
III. In fact E. None matches
A. B-(ii)-F, A-(iii)-D
B. A-(i)-D, B-(ii)-E, C-(iii)-F 5)
C. C-(i)-E, B-(ii)-F, A-(iii)-D Column I Column II
D. B-(i)-D, C-(ii)-E A. Meditation helps to D. publications in foreign
E. None matches cultivate mindfulness journals for job selection.

4) B. Without a doubt, E. helps to bring the


Column I Column II Yoga has now mind into the present
A. The way AI is D. maintaining a become a global moment.
advancing it will fulfilling career. good,
develop various
applications C. We still provide F. promises unflinching
more value to people support to further good
B. The real estate E. teaching having overseas health and wellness
industry recognizes the methodologies to rise experience across the globe.
aspirations of digital to the occasion.
nomads and their I. Which
desire to explore the II. And
world III. Or
A. A-(ii)-E, B-(i)-F, C-(iii)-D
C. India still faces a F. will be able to do B. B-(i)-D, A-(iii)-E
shortage of quality acts the doing of which C. B-(iii)-E, A-(i)-F, C-(ii)-D
teachers seem humanly D. A-(i)-E, C-(iii)-F
impossible at present. E. None matches

Click Here For Bundle PDF Course | support@guidely.in Page 2 of 9


SBI Clerk & RRB PO Mains PDF Course 2023
ENGLISH Day - 39

Directions (6-10): The given sentences carry a E. No replacement needed


part or a phrase that has been highlighted as it
might need to be improved. Choose the option 10) I feel terrible- that food didn’t agree with my
that replaces the highlighted portion to form the stomach.
correct sentence as your answer. A. Adjust up
6) I was very taken aback with the performance- B. Bear on
it was superb. C. Cut through
A. impressed towards D. Hold off
B. taken with E. No replacement needed
C. burned down
D. acted up Directions (11-15): Given below are different
E. no replacement needed sentences which have been taken from a single
7) The marathon runner barely beat out his rival paragraph. These are not in their proper order
at the tape. and some statements carry activities. You must
A. Black out read the same and answer the questions that
B. Carried away follow.
C. Dig into G. Users order clothes online, (A)/ striked a pose
D. Get ahead of for the perfect (B)/ Instagram picture, and then
E. No replacement needed (C)/ return the outfits once the (D)/ 'likes' have
rolled in (E).
8) This shows that your interest is not in helping H. This transient ownership of fashion, solely for
the student, but in living out. online validation, not only highlights the
A. Keeping about performative aspect of social media but also
B. Kicking out signals deeper issues at play.
C. Capping off I. Also dubbed the 'snap and send back' culture,
D. Jawing away this practice is a wry reflection of our times.
E. No replacement needed J. It's an emblem of the pressures exited by
digital life, which might be subtly eroding our
9) Work just keeps on piping up and I really can’t mental health.
manage to get it all done. K. In the era of social media, where 'likes' have
A. Piling up become a currency of popularity and a measure
B. Breaking down of 'coolness,' a peculiar trend known as
C. Putting off 'wardrobing' has _________.
D. Uplifting

Click Here For Bundle PDF Course | support@guidely.in Page 3 of 9


SBI Clerk & RRB PO Mains PDF Course 2023
ENGLISH Day - 39

11) Choose the word that best replaces the word B. Perpetual
highlighted in sentence D above. C. Temporary
A. Creating D. Continuous
B. Exerted E. All of these
C. Imposed
D. Claimed Directions (16-20): Parts of sentences have been
E. No replacement needed given in each question below. One of these
might be erroneous and need to be improved
12) Which of the given parts in sentence A grammatically. You must choose the part
carries an error in the same? containing the same as your answer.
A. B 16)
B. D A. Six of top-10 most valued firms together
C. E added
D. A B. Rs 2,03,010.73 crore in market valuation last
E. C week,
C. with Reliance Industries and Tata Consultancy
13) Which of the given options provides the best Services
sequence for the sentences given above? D. emerging as the biggest gainers, amid an
A. AECBD E.optimistic trend in equities.
B. CEBDA
C. ECABD 17)
D. BDECA A. More than 1,000 participants including
E. No rearrangement needed B. policymakers, delegates, invitees, business
C. leaders and researchers are expected to
14) Which of the given words can best fill the D. attend various meetings and
blank given in sentence E? E. events along the four days.
A. Surfaced
B. Exposed 18)
C. Vanished A. Hailing India’s remarkable progress in
D. Discover building unicorns and startups,
E. None of these B. Union Minister Rajeev Chandrasekhar
prophecised that the next
15) Choose the synonym of the word ‘transient’ C. 4-5 years will witness substantial growth for
A. Constant such enterprises

Click Here For Bundle PDF Course | support@guidely.in Page 4 of 9


SBI Clerk & RRB PO Mains PDF Course 2023
ENGLISH Day - 39

D. and the startups in the country will increase by E. No Error


10 times 20)
E. No Error A. Foreign Portfolio Investors (FPIs) flow into the
Indian equity
19) B. market remained unabated as they invested
A. It is advisable to evaluate the differing charge, over Rs 30,600
B. bearing in mind that larger rooms may be C. crore in the first fortnight of this month, driven
more by the
C. expensive while also present the possibility D. country’s robust economic growth and strong
D. of finding cashback offers and discount corporate earnings.
packages. E. No Error
Click Here to Get the Detailed Video Solution for the above given Questions
Or Scan the QR Code to Get the Detailed Video Solutions

Answer Key with Explanation

1) Answer: A C-(ii)-D  Resource-conserving and climate-


The sentences after being joined can be found smart technologies such as zero-tillage, drip
as given below: biofertilizers and biopesticides now occupy the
A-(i)-F  This figure pales in comparison to centre stage.
countries like the UK and USA, which have 34 The above combinations can be found in option
and 24 such sites, respectively. (a).
B-(iii)-E  In India, there is little scope for
bringing in more area under cultivation; 2) Answer: D
therefore, growth in food grain production has to The sentences given below can be joined but
come largely through productivity enhancement. some of the given parts are grammatically

Click Here For Bundle PDF Course | support@guidely.in Page 5 of 9


SBI Clerk & RRB PO Mains PDF Course 2023
ENGLISH Day - 39

incorrect. The sentences after being corrected A-(ii)-F  The way AI is advancing it will develop
have been given below: various applications that will be able to do acts
A-(ii)-F  Currently, India majorly imports solar the doing of which seem humanly impossible at
panels from China, for the domestic panel present.
manufacturers have limited updated capacity. B-(i)-D  The real estate industry recognizes the
B-(i)-D  The current Indian regulatory aspirations of digital nomads and their desire to
environment is very congenial to produce solar explore the world while maintaining a fulfilling
panels domestically as the competence gap is career.
addressed. C-(iii)-E  India still faces a shortage of quality
C-(iii)-E  This brazen act of violence highlights teachers and teaching methodologies to rise to
the growing boldness of Khalistani groups and the occasion.
raises legitimate concerns for India's national The given combinations can be found in option
security. (d).
Since the given parts are grammatically
incorrect, the correct answer should be option 5) Answer: A
(d). The sentences after joining the given parts are:
3) Answer: B A-(ii)-E  Meditation helps to cultivate
The sentences after joining the given parts are: mindfulness and helps to bring the mind into the
A-(i)-D  Employee burnout has become a present moment.
pervasive issue for many individuals who B-(i)-F  Without a doubt, Yoga has now
experience chronic stress at their workplace. become a global good, which promises
B-(ii)-E  A vital aspect of employee well-being unflinching support to further good health and
is feeling recognised and valued for their wellness across the globe.
contributions. C-(iii)-D  We still provide more value to people
C-(iii)-F  Premier research institutions and having overseas experience or publications in
universities have long been collaborating with foreign journals for job selection.
industries, in fact, many companies closely The given combinations can be found in option
monitor the discoveries from these labs. (a).
The best would be to mark option (b) as the
answer. 6) Answer: B
The phrase taken aback means shock or
4) Answer: D surprise someone… This does not fit the
The sentences after joining the given parts are: sentence above as the tone of the sentence

Click Here For Bundle PDF Course | support@guidely.in Page 6 of 9


SBI Clerk & RRB PO Mains PDF Course 2023
ENGLISH Day - 39

provides a hint that the person is getting Kicking out  to continue to be in a particular
impressed with the performance. state or position
The first option is incorrect as it uses towards Capping off  to bring to a triumphant
incorrectly. The second option is correct as it conclusion
means to find something interesting and Jawing away  to talk for a long time
attractive. The third option is irrelevant and can This makes option (d) the best option.
be eliminated. The fourth phrase means behave
badly… 9) Answer: A
The best would be to mark option (b) as the The phrase pile up means to increase in
answer. quantity. This phrase can correctly replace the
highlighted portion.
7) Answer: E Piping up means to suddenly say something…
The given sentence is correct and needs no Breaking down means failing or stop working
improvement. The phrases given above can be Putting off means someone losing interest or
understood as: postponing something.
Beat out  come out better in a competition, The phrase that fits the context of the sentence
race, or conflict is (a).
Black out  to lose consciousness for a short
time 10) Answer: E
Carried away  to be so excited that one is no The given phrase is correct and needs no
longer in control of one's behaviour. change. The phrases given above can be
Dig into  use a lot of one's physical, mental, or understood as given below:
financial resources. Agree with means to settle in… (one’s stomach)
Get ahead of  to progress and be successful in as per the sentence.
something, especially a career. Adjust up  to change something slightly
The correct answer would be option (e). Bear on  be a burden on someone.
Cut through  go through or across
8) Answer: D something…
The phrase in the sentence does not fit in the Hold off  fail to occur
sentence contextually. The correct answer hence, is option (e).
Keeping about  to continue to be in a particular
state or position… 11) Answer: B

Click Here For Bundle PDF Course | support@guidely.in Page 7 of 9


SBI Clerk & RRB PO Mains PDF Course 2023
ENGLISH Day - 39

The sentence above talks about pressures being with ’it’ which means something should have
applied by the digital life. The first option is been mentioned before the same.
grammatically incorrect as we need a word in the The correct sequence can thus, be marked as
third form for the sentence. The word exerted option ECABD which has been given in option
means to use power or the ability to make (c).
something happen (in this case negative).
The word imposed means to make a law, rule, 14) Answer: A
opinion, etc. be accepted by using your power or The best word that should fill the blank in E as
authority. This does not fit contextually. The per the context is surfaced. This word is
word claimed is irrelevant as well. generally used in the content/articles that talk
Therefore, the best would be to mark option (b) about social media. It means things that have
as the answer. suddenly become visible or popular (on the
social media).
12) Answer: A The word exposed carries a negative tone and
The incorrect part is B as the word striked is does not fit the context as well so, can be
incorrect grammatically. It should match with the discarded. The word vanished means
other verbs being used in the sentence. All the disappeared which makes it irrelevant. Discover
other verbs are in the first form ie; order, means to find something that was not previously
return… The word strike will continue the parallel known.
construction in the sentence. As per the above meanings, we can find that the
The best would hence be to mark option (a) as best would be option (a).
the answer.
15) Answer: C
13) Answer: C The word transient has been used to describe
The paragraph should best start with sentence E fashion in the sentence which is always
as it introduces the ‘likes’ culture on the social temporary as it keeps changing with time…
media that is further being discussed in the other The words given in all options except the third
sentences. C then tells us that this particular one are antonyms to the given word.
culture is known as ‘snap and send back’ A This makes option (c) the most logical answer
continues the idea by explaining why such a choice.
name has been given. This tells us that the first
three sentences should be ECA. Now, D starts 16) Answer: A

Click Here For Bundle PDF Course | support@guidely.in Page 8 of 9


SBI Clerk & RRB PO Mains PDF Course 2023
ENGLISH Day - 39

The first part of the sentence carries an error as The word propheCy is a noun whereas
a superlative has been used in the same (most) propheSy is a verb. The sentence needs a verb
but (the) is missing which is mandatory before a as the minister foresees the growth of startups.
superlative. The correct phrase should be; ‘… six The correct answer is option (b).
of the top 10 most valued…’
This makes option (a) the most logical answer 19) Answer: C
choice. The sentence uses a gerund bearing which
makes it important for the other verb to be a
17) Answer: E gerund too. The word presenting should be
The fifth option contains an error as along has correct as it would follow parallel construction.
incorrectly been used. Across the four days The best would be to mark option (c) as the
should be the correct usage. answer.
Along means on or beside something long which
doesn’t fit the sentence. 20) Answer: E
So, the best would be to mark option (e) as the The given sentence is correct and needs no
answer. improvement.
This makes option (e) the correct answer choice.
18) Answer: B

Click Here For Bundle PDF Course | support@guidely.in Page 9 of 9


SBI Clerk & RRB PO Mains PDF Course 2023
Reasoning Ability Day - 40 (Eng)

Reasoning Ability
Directions (1-5): Study the following information A. Box Q
carefully and answer the below questions. B. The box which is placed immediately above
Six boxes – P, Q, R, S, T, and U are placed one box P
above the other in a footwear shop. Each box C. The box whose number is 23
contains different footwear brands viz.- Puma, D. Box R
Relaxo, Sparx, Lee Cooper, Reebok, and Bata. E. None of these
Each box has different numbers viz.- 8, 12, 15,
21, 23, and 35. All the information is not 2) What is the box number of the box which
necessary in the same order. contains Bata?
Note: The box number of adjacent boxes is A. 35
neither a common multiple nor a common factor. B. 8
Only two boxes are placed below the box which C. 12
contains Sparx. The box placed immediately D. 21
below the box which contains Sparx has box E. None of these
number 8. The box which contains Reebok has
box number 12. Only three boxes are placed 3) Which of the following combination is not true?
between the box which contains Reebok and box A. Box Q – 35
R. Two boxes are placed between the box which B. Lee Cooper - Box U
contains Lee Cooper and the box whose number C. Box P – Puma
is 21. The box which contains Lee Cooper is kept D. Box R – 21
immediately above box U. Box Q neither E. Box T – Reebok
contains Sparx nor Reebok. Box Q is placed two
places above box S, whose box number is 23. 4) Which of the following box is placed
The number of boxes placed between S and R is immediately above the box which contains Lee
the same as the number of boxes placed below Cooper?
the box whose number is 15. At least one box is A. Box Q
placed between box P and the box whose B. The box which contains Reebok
number is 35. Box P and the box which contains C. The box whose number is 8
Bata are placed adjacent to each other. Box D. Box U
number of the box which contains Relaxo is an E. None of these
odd number.
1) Which of the following box contains Lee 5) Which of the following statement is not true?
Cooper? A. Box number of the box at the top is 35

Click Here For Bundle PDF Course | support@guidely.in Page 1 of 10


SBI Clerk & RRB PO Mains PDF Course 2023
Reasoning Ability Day - 40 (Eng)

B. The box placed at the bottom contains Bata 7) What is the sum of the number of chocolates
C. The box which contains Sparx is placed of D and F’s sister?
immediately below box Q A. 113
D. Only two boxes are placed above Box S. B. 67
E. All the above statements are true C. 63
D. 50
Direction (6-10): Study the following information E. None of these.
carefully and answer the below questions.
Seven persons from a three generations family 8) How E is related to the one who has 98
viz.- A, B, C, D, E, F, and G are related to A in a chocolates?
certain relation. Initially, A has 325 chocolates he A. Sister
distributes among all the remaining members in B. Uncle
such a way that each person has a different C. Brother-in-law
number of chocolates. D. Daughter
A takes 18 chocolates and gives the remaining E. None of these
to E. E is the brother of F’s mother and takes 22
chocolates and gives the remaining to D’s 9) How B is related to the one who has half the
mother. B takes 1/3rd of the chocolate and gives number of chocolates as G?
the remaining 190 to G’s wife C. G is the father A. Sister
of A.G’s wife takes 14 of her initial chocolate and B. Grandfather
gives the remaining to A’s brother. The one who C. Aunty
has 95 chocolates is the daughter-in-law of G, D. Daughter
who takes 147 chocolates from B’s husband. The E. None of these
one who has 18 chocolates is the only daughter
of E’s sister. G gives 1/3rd of the initial chocolate 10) Four of the following five are alike in a certain
to F’s son. way as per the given arrangement and thus form
6) How the one who has 14 chocolates is related a group. Which one of the following doesn’t
to F? belong to that group?
A. Aunt A. G
B. Sister B. The one who has 29 chocolates
C. Grandfather C. D
D. Mother D. The one who has 14 chocolates
E. None of these E. E

Click Here For Bundle PDF Course | support@guidely.in Page 2 of 10


SBI Clerk & RRB PO Mains PDF Course 2023
Reasoning Ability Day - 40 (Eng)

Directions (11-15): Study the following A. As many persons are sitting between U and
information carefully and answer the below Q’s aunty
questions. B. One
Seven persons – P, Q, R, S, T, U, and V are from C. As many persons are sitting to the right of R
the same family of three generations and sitting D. Two
in a linear row facing the north. Two married E. None of these
couples are there in the family. Either both or
none of the parents are alive. 14) Who among the following is the daughter of
U is the sister-in-law of V, and vice-versa. P’s T?
father sits second from the left end. Only two A. The one who sits immediate left of R
persons are sitting between P’s father and U. B. The one who sits second to the right of S
One person sits between U and P, who is the C. The one who sits at the left end
father of S. S sits third to the left of T and is the D. Both A and C
only son of V’s brother. Only one person sits E. None of these
between S and Q, who is the granddaughter of T.
The gender of T and S is not the same. P’s wife 15) Which of the following statement is true?
and V’s mother are not sitting together. A. P is the father of the one who sits third to the
11) What is the position of S with respect to P’s right of R
sister? B. S is the sister-in-law of V
A. Immediate left C. Q sits immediate right of R’s daughter-in-law
B. Third to the right D. Four persons are sitting between S and V
C. Second to the left E. None of the statements are true
D. Fourth to the right
E. None of these Directions (16-20): Study the following
information carefully and answer the questions
12) How S is related to U? given below.
A. Daughter-in-law Nine persons viz. M, N, O, P, Q, R, S, T and U
B. Nephew joined in different banks viz Citi, CSB, DCB,
C. Son HDFC, ICICI, IDFC FIRST, PNB, RBL and YES
D. Brother-in-law in different years from 2014 to 2022. All the
E. None of these information is not necessarily in the same order.
The one who joined in DCB joined in an even
13) How many persons are sitting between P numbered year but not in the leap year. Only
and R’s wife? three persons joined between the one who joined

Click Here For Bundle PDF Course | support@guidely.in Page 3 of 10


SBI Clerk & RRB PO Mains PDF Course 2023
Reasoning Ability Day - 40 (Eng)

in DCB and U, who joined immediately after the E. None of these


one who joined in HDFC. More than three
persons joined between the one who joined in 18) How many persons joined between Q and
HDFC and Q, who doesn’t join the first. The the one who joined in RBL?
number of persons joined before Q is one more A. Four
than the number of persons joined after P. The B. Three
one who joined in ICICI joined in an odd C. More than five
numbered year where neither P nor Q joined in D. Five
ICICI. S joined two years before the one who E. None of these
joined in ICICI but neither joined in HDFC nor in
CSB. The number of persons joined between S 19) Four of the following five are alike in a certain
and the one who joined in DCB is the same as way as per the given arrangement and hence
the number of persons joined between Q and N. form a group. Which of the following one doesn’t
The number of persons joined after N is one less belong to that group?
than the number of persons joined before the A. T
one who joined in RBL. The one who joined in B. The one who joined in YES bank
CSB joined four years before R, who joined C. M
immediately after M. O joined two years before D. The one who joined in Citi bank
the one who joined in PNB whereas T joined E. P
three years after the one who joined in Citi but
doesn’t join in YES. 20) Which of the following statement(s) is/are
16) Who among the following person joined in true with respect to the final arrangement?
IDFC FIRST bank? I. Only three persons joined between S and the
A. The one who joined in 2015 one who joined in RBL bank.
B. R II. Either the one who joined in PNB or P joined
C. The one who joined in 2020 in an even numbered year.
D. M III. Neither O nor the one who joined in DCB
E. None of these joined at the first.
IV. Both M and the one who joined in ICICI joined
17) In which of the following bank does N join? in the adjacent years.
A. YES bank A. Both I and IV
B. CSB bank B. Both II and IV
C. DCB bank C. Both I and III
D. ICICI bank D. Both II and III

Click Here For Bundle PDF Course | support@guidely.in Page 4 of 10


SBI Clerk & RRB PO Mains PDF Course 2023
Reasoning Ability Day - 40 (Eng)

E. None of these
Click Here to Get the Detailed Video Solution for the above given Questions
Or Scan the QR Code to Get the Detailed Video Solutions

Answer Key with Explanation

Directions (1-5):  The box placed immediately below the


1) Answer: C box which contains Sparx has box
2) Answer: D number 8.
3) Answer: B  The box which contains Reebok has box
4) Answer: B number 12.
5) Answer: C  Only three boxes are placed between the
box which contains Reebok and box R.
That means, in case (1) box R is placed
at the bottom, in case (2) box R is placed
second from the bottom.
Based on the above given information we have:

We have:
 Only two boxes are placed below the box
which contains Sparx.
Again, we have:

Click Here For Bundle PDF Course | support@guidely.in Page 5 of 10


SBI Clerk & RRB PO Mains PDF Course 2023
Reasoning Ability Day - 40 (Eng)

 Two boxes are placed between the box Case (2) is not valid as the box number of S is
which contains Lee Cooper and the box 23.
whose number is 21. Again, we have:
 The box which contains Lee Cooper is  The number of boxes placed between S
kept immediately above box U. and R is the same as the number of
That means, in case (1) box U is placed boxes placed below the box whose
second from the top, in case (1a) box U is number is 15.
placed third from the bottom, and in case That means, in case (1a) box number of
(2) box U is placed third from the bottom. U is 15, and case (1) is not valid.
 Box Q neither contains Sparx nor  At least one box is placed between box P
Reebok. and the box whose number is 35.
 Box Q is placed two places above box S,  Box P and the box which contains Bata
whose box number is 23. are placed adjacent to each other.
That means, in case (1) & case (1a) box  Box number of the box which contains
Q is placed at the top, case (2) is not Relaxo is an odd number.
valid. That means, box P contains Puma.
Based on the above given information we have: Based on the above given information we have:

Case (1) is not valid as the number of boxes


placed between S and R is the same as the
number of boxes placed below box number 15.

Direction (6-10):
6) Answer: D
7) Answer: B
8) Answer: C

Click Here For Bundle PDF Course | support@guidely.in Page 6 of 10


SBI Clerk & RRB PO Mains PDF Course 2023
Reasoning Ability Day - 40 (Eng)

9) Answer: E
10) Answer: D

Again, we have:
 The one who has 95 chocolates is the
daughter-in-law of G, who takes 147
chocolates from B’s husband.
We have: Since, no two persons have the same number of
 A takes 18 chocolates and gives the chocolates, thus B must be the daughter-in-law
remaining to E. of G.
 E is the brother of F’s mother and takes  The one who has 18 chocolates is the
22 chocolates and gives the remaining to only daughter of E’s sister.
D’s mother.  G gives 1/3rd of the initial chocolate to F’s
Based on the above given information we have: son.
That means, G has 98 chocolates.
Based on the above given information we have:

Again, we have:
 B takes 1/3rd of the chocolate and gives
the remaining 190 to G’s wife C.
Since, 2/3rd of B’s chocolate is 190, thus B must
have initially 285 chocolates.
 G is the father of A.
 G’s wife takes 14 of her initial chocolate
and gives the remaining to A’s brother. Directions (11-15):

Thus, currently, A’s brother has 176 chocolates. 11) Answer: B

Based on the above given information we have: 12) Answer: C

Click Here For Bundle PDF Course | support@guidely.in Page 7 of 10


SBI Clerk & RRB PO Mains PDF Course 2023
Reasoning Ability Day - 40 (Eng)

13) Answer: A  S sits third to the left of T and is the only


14) Answer: D son of V’s brother.
15) Answer: C  Only one person sits between S and Q,
who is the granddaughter of T.
 The gender of T and S is not the same.
 P’s wife and V’s mother are not sitting
together.
That means, in case (1) T sits at the right
end, and Q must be the sister of S, case
(2) is not valid.
Based on the above given information we have:

We have:
 U is the sister-in-law of V, and vice-versa.
 P’s father sits second from the left end.
 Only two persons are sitting between P’s
father and U.
 One person sits between U and P, who is
Case (2) is not valid as T and U are not sitting
the father of S.
together.
That means, in case (1) P sits second to
For Blood Relation:
the left of U, in case (2) P sits second to
the right of U.
Based on the above given information we have:

Directions (16-20):
16) Answer: C
17) Answer: B
18) Answer: D
Again, we have:

Click Here For Bundle PDF Course | support@guidely.in Page 8 of 10


SBI Clerk & RRB PO Mains PDF Course 2023
Reasoning Ability Day - 40 (Eng)

19) Answer: A (All the persons joined in a non


leap year except option a)
20) Answer: B

 The one who joined in ICICI joined in an


odd numbered year where neither P nor
 The one who joined in DCB joined in an Q joined in ICICI.
even numbered year but not in the leap  S joined two years before the one who
year. joined in ICICI but neither joined in HDFC
 Only three persons joined between the nor in CSB.
one who joined in DCB and U, who joined  The number of persons joined between S
immediately after the one who joined in and the one who joined in DCB is the
HDFC. same as the number of persons joined
 More than three persons joined between between Q and N.
the one who joined in HDFC and Q, who  The number of persons joined after N is
doesn’t join the first. one less than the number of persons
 The number of persons joined before Q is joined before the one who joined in RBL.
one more than the number of persons Hence, case 3 gets eliminated.
joined after P.

Click Here For Bundle PDF Course | support@guidely.in Page 9 of 10


SBI Clerk & RRB PO Mains PDF Course 2023
Reasoning Ability Day - 40 (Eng)

years after the one who joined in Citi but


doesn’t join in YES.
Hence, cases 1 gets eliminated.

 The one who joined in CSB joined four


years before R, who joined immediately
after M.
 O joined two years before the one who
joined in PNB whereas T joined three

Click Here For Bundle PDF Course | support@guidely.in Page 10 of 10


SBI Clerk & RRB PO Mains PDF Course 2023
Quantitative Aptitude Day – 40 (Eng)

Quantitative Aptitude

Directions (1-4): Study the following information carefully and answer the questions given below.
There are five colleges [A, B, C, D and E]. There are three streams [science, Arts and commerce] in each
college. The given below table shows total number of students in the college, Percentage of students in
science stream, Number of students in Arts stream, Ratio of boys and girls in commerce stream, Number
of boys in science stream and Number of girls in Arts stream.

1) Number of girls in the science stream of A.66


college A is half of the number of students in the B.70
Arts stream of the same college. Number of boys C.75
in Arts stream of college A is 1/4th of the number D.80
of boys of commerce stream of the same E.50
college. Find the number of girls in the Arts
stream of college A. 3) Number of boys in science stream of college
A.80 C is 20% less than the number of girls in Arts
B.85 stream of college B. Number of girls in
C.70 commerce stream of college C is 25% of the
D.60 number of boys in science stream of college C.
E.65 Then find the percentage of girls in commerce
stream of college C out of total number of
2) Ratio of the number of students of Arts stream students in commerce stream of college C?
to commerce streams of college B is 15:13. A.25.32%
Number of girls in commerce stream of college B B.16.66%
is 78. If 33.33% of the total number of students of C.20%
science stream of college B is boys, then find the D.15%
number of girls in science stream of college B. E. None of these

Click Here For Bundle PDF Course | support@guidely.in Page 1 of 12


SBI Clerk & RRB PO Mains PDF Course 2023
Quantitative Aptitude Day - 40 (Eng)

4) Number of boys in commerce stream of A. The data in statement I alone are sufficient to
college D is 5 more than the number of boys in answer the question, while the data in statement
science stream. Number of Arts students in II alone are not sufficient to answer the question.
college D is 20% less than the total number of B. The data in statement II alone are sufficient to
commerce stream students in college D. Then answer the question, while the data in statement
find the number of students in the science I alone are not sufficient to answer the question.
stream of college D,if 60% of the total students C. The data either in statement I alone or in
are from Arts and commerce streams. statement II alone are sufficient to answer the
A.240 question.
B.252 D. The data given in both statements I and II
C.236 together are not sufficient to answer the
D.221 question.
E. None of these E. The data in both statements I and II together
are necessary to answer the question.
Directions (5-8): Following questions contain two
statements the statement I and statement II. You 6) There are three different mixtures of milk and
have to determine which statement/s is/are water P, Q and R. Find the ratio in which all three
necessary to answer the question and give an mixtures must be mixed together so that average
answer as, price of the resultant mixture is Rs. 62 per litres.
5) A certain number of students gave at least Statement I: Price of the mixture P and Q is Rs.
one of the two exams i.e., Physics and 40 per litres and Rs. 75 per litres, respectively.
Chemistry. Find total number of students who Statement II: Price of 15 litres of mixture R is Rs.
gave the exam if all the students who gave the 900.
exam passed it. A. The data in statement I alone are sufficient to
Statement I: 73(1/3)% of the students passed in answer the question, while the data in statement
Chemistry and 59(2/3)% of students passed in II alone are not sufficient to answer the question.
Physics and number of students who passed in B. The data in statement II alone are sufficient to
both the exam is 396. answer the question, while the data in statement
Statement II: Number of students who passed I alone are not sufficient to answer the question.
only in Physics and only in Chemistry are 76 less C. The data either in statement I alone or in
and 88 more, respectively, than the number of statement II alone are sufficient to answer the
students who passed in both the exams. Total question.
number of students who passed in Physics is
716.

Click Here For Bundle PDF Course | support@guidely.in Page 2 of 12


SBI Clerk & RRB PO Mains PDF Course 2023
Quantitative Aptitude Day - 40 (Eng)

D. The data given in both statements I and II Statement II: B and C together complete work in
together are not sufficient to answer the 20/3 days. A alone complete the work in 10 days.
question. D is 25% more efficient than A.
E. The data in both statements I and II together A. The data in statement I alone is sufficient to
are necessary to answer the question. answer the question, while the data in statement
II alone is not sufficient to answer the question
7) What is the difference between the length of B. The data in statement II alone is sufficient to
the platform and the train itself? answer the question, while the data in statement
Statement I: The train running at a speed of 54 I alone is not sufficient to answer the question
kmph can cross a bus going in the same C. The data either in statement I alone or in
direction at a speed of 36 kmph in 20 seconds. statement II alone is sufficient to answer the
Statement II: Time taken by the bus to cross the question
platform is 30 seconds more than the time taken D. The data given in both statements I and II
by the train to do the same. together are not sufficient to answer the question
A. The data in statement I alone are sufficient to E. The data given in both statements I and II
answer the question, while the data in statement together are necessary to answer the question.
II alone are not sufficient to answer the question.
B. The data in statement II alone are sufficient to Directions (9-12): Study the following information
answer the question, while the data in statement carefully and answers the questions based on it.
I alone are not sufficient to answer the question. In a Car Expo in the New Delhi, there are a total
C. The data either in statement I alone or in of 67,200 visitors who visited the car expo. The
statement II alone are sufficient to answer the ratio of number of males to number of females
question. who visited in the car expo is 9: 7 respectively.
D. The data given in both statements I and II All the visitors who visited the car expo, visits the
together are not sufficient to answer the six different car company pavilion namely Ford,
question. Maruti, Volkswagen, Toyota, Honda and Nissan.
E. The data in both statements I and II together Total number of visitors who visited Ford pavilion
are necessary to answer the question. is 16,128 which is (__P%__) of the total number
of visitors. Number of female visitors who visited
8) Find the number of days taken by A and C Maruti pavilion is (__Q%__) of the total number of
together to complete the work? female visitors who visited car expo and 10,983
Statement I: Efficiency of A is 20% more than the less than the total number of visitors who visited
efficiency of B. D completes the work in 8 days. Ford pavilion.
Ratio of efficiency of D and C is 15:8.

Click Here For Bundle PDF Course | support@guidely.in Page 3 of 12


SBI Clerk & RRB PO Mains PDF Course 2023
Quantitative Aptitude Day - 40 (Eng)

Number of male visitors who visited Honda C.49


pavilion is 15%of the total male visitors. Number D.18
of male visitors who visited Volkswagen pavilion E. None of these
is 1890 more than male visitors who visited
Honda pavilion and is(__R%__)of the total 10) Find the value of Q?
number of male visitors who visited car expo. A.17.5
Number of male visitors who visited Maruti B.10.25
pavilion is 1/3rd of the number of female visitors C.21
who visited Maruti pavilion. D.33.33
Number of female visitors who visited Toyota E. None of these
pavilion is 7³ more than the number of male
visitors who visited Maruti pavilion and is 11) Find the value of S and T respectively?
(__S%__) of the female visitors who visited A.17 and 20
Maruti pavilion. Number of female visitors who B.51 and 27
visited Nissan pavilion is 4410 which is (__T%__) C.32 and 22
of the total number of female visitors who visited D.40 and 15
car expo. E. None of these
Number of male visitors who visited Toyota
pavilion is 3 times of the number of female 12) Find the value of [(Q × S × T) – (P × R ×
visitors who visited same pavilion. Total number T)]/100?
of visitors who visited Honda pavilion is 22% of A.23
total visitors. The ratio of male to female visitors B.17 (5/7)
who visited Nissan pavilion is 9: 5 respectively. C.33
9) Find the value of P? D.16 (2/3)
A.37 E. None of these
B.24

Directions (13-16): Study the table given below carefully and answer the following questions.
The given pie chart shows the percentage distribution of students who paid fees for semester exams in 5
different years. The given table shows the percentage of students who paid the fees by online mode in
five different years.
Note: Fees are paid through online and offline modes.
Total number of students who paid fees = 6,00,000.

Click Here For Bundle PDF Course | support@guidely.in Page 4 of 12


SBI Clerk & RRB PO Mains PDF Course 2023
Quantitative Aptitude Day - 40 (Eng)

13) Number of students who paid their fees 14) Number of students who paid fees through
through offline in 2016 and 2017 together is how online in 2015 is equal to the average of students
much more/less than the number of students who paid through online in 2019 and 2020
who paid fees through online in the years 2018 together and the number of students who paid
and 2019 together? fees through online is 3⁄4th of the total number of
A.54750 students in 2015. Then find the total students in
B.57450 2015.
C.54450 A.43000
D.52389 B.44000
E.54455 C.40500
D.45500
E.44500

Click Here For Bundle PDF Course | support@guidely.in Page 5 of 12


SBI Clerk & RRB PO Mains PDF Course 2023
Quantitative Aptitude Day - 40 (Eng)

15) Students who paid fees through offline in 16) Find the average number of students who
2020 is what percent of the students who paid paid their fees through online in the years 2016,
fees through online in 2018? 2017 and 2019 together?
A.140% A.36000
B.145% B.34560
C.150% C.36850
D.160% D.36750
E.135% E.37750

Directions (17-20): Study the following information carefully and answer the questions given below.
There are five cities. In each, there are three types of cars are registered, i.e., Petrol, Diesel, and Electric.
Sum of the number of petrol and diesel cars and difference of the number of petrol and diesel cars
registered in five cities are given in the below graph.

Note: -
a) Number of petrol cars registered in cities A, C, and E is more than the number of diesel cars registered
in the same cities, in other cities the number of diesel cars is more than the number of petrol cars.
b) Percentage of electric cars out of the total cars registered in cities A, B, C, D, and E is 20%, 20%,
30%, 25%, and 20% respectively.
17) Find the difference between the 20% of A.50
petrol cars registered in cities D and E together B.55
and 10% of electric cars registered in cities A C.75
and C together. D.45

Click Here For Bundle PDF Course | support@guidely.in Page 6 of 12


SBI Clerk & RRB PO Mains PDF Course 2023
Quantitative Aptitude Day - 40 (Eng)

E.26 number of Tata company’s registered cars in city


A?
18) If the number of registered diesel and electric A.94.94%
cars of all cities is represented individually in a B.84.84%
pie chart then find the difference of central angle C.74.74%
formed by the number of diesel car register in C D.64.64%
and central angle formed by the number of E. None of these
electric car register in city B?
A.20̊ 20) Find which one is true.
B.85̊ I) 75% of the number of diesel cars registered in
C.65̊ city A is equal to the number of electric cars
D.15̊ registered in city B.
E.45̊ II) 20% of the total number of electric cars
registered in city D and city E together is equal to
19) Number of all registered petrol and diesel 20% less than the total number of electric cars
cars are from only three companies, i.e., Maruti, registered in city E.
Hyundai, and Tata in city A. No electric cars in A. Only I is true
city A are from these companies. Ratio of petrol B. Only II is true
cars and diesel cars from companies Maruti, C. Both are true
Hyundai and Tata are 4:3:5 and 2:1:2 in city A. D. None is True
Find the total number of Maruti company's E. Can’t be determined
registered cars is what percent of the total

Click Here to Get the Detailed Video Solution for the above given Questions
Or Scan the QR Code to Get the Detailed Video Solutions

Answer Key with Explanation

Click Here For Bundle PDF Course | support@guidely.in Page 7 of 12


SBI Clerk & RRB PO Mains PDF Course 2023
Quantitative Aptitude Day – 40 (Eng)

1) Answer: C 4) Answer: A
Total number of students in science stream of Number of boys in commerce stream of college
college A =150+(100/2)=200 D =120+5=125
Total number of students in commerce stream of Total number of students in commerce
college A = 500-100-200=200 department is =125*8/5=200
Number of boys in commerce stream of college Total number of student in science stream is =
A =200*3/5=120 {[200+160]/60} *40=240
Number of boys in Arts stream of college A
=120/4=30 5) Answer: C
Number of girls in Arts stream of college A =100- Statement I,
30=70 Let total number of students who gave exam be
2) Answer: D 100x
Let the number of Arts and commerce students Number of students who passed only in Physics
of college B be 150x and 130x. = 26(2/3)% of 100x = 80x/3
So, 130x*3/5=78 Number of students who passed only in
Or, 78x=78, x=1 chemistry = 40(1/3)% of 100x = 121x/3
The number of Arts and commerce students of So according to question,
college B is 150 and 130. 80x/3 + 121x/3 + 396 = 100x
So, number of student in science stream is = 300x – 80x + 121x = 2412
[280/70] *30= 120 99x = 1188
Number of boys in science stream is = 120* (1/3) x = 12
=40 Required number of students = 12 × 100 = 1200
So, number of girls in science stream of college Data in statement I alone is sufficient to answer
B is = 120-40=80 the question.
Statement II,
3) Answer: E Let number of students who passed in both
Number of boys in science stream of college C is exams be x
=100*80/100=80 Number of students who passed only in Physics
Number of girls in commerce stream of college C = x – 76
is =80*25/100=20 Number of students who passed only in
Total number of students in commerce stream is Chemistry = x + 88
= 300-150-100=50 x + x – 76 = 716
So, required percentage = [20/50]*100=40% 2x = 792

Click Here For Bundle PDF Course | support@guidely.in Page 8 of 12


SBI Clerk & RRB PO Mains PDF Course 2023
Quantitative Aptitude Day – 40 (Eng)

x = 396 Relative speed between the train and the bus =


Required number of students = x – 76 + x + 88 + (54 – 36) = 18 kmph = 18 × 5/18 = 5 m/s
x = 3x + 12 = 3 × 396 + 12 = 1200 If L is the length of the train, then,
Data in statement II alone is sufficient to answer L = 5 × 20 = 100 meter.
the question. Data in statement I alone is not sufficient to
Therefore, the data either in statement I alone or answer the question.
in statement II alone are sufficient to answer the Statement II,
question. Let the length of the platform be xmeter.
Time taken by the bus to cross the platform =
6) Answer: D {x/(36 × 5/18)} = x/10 sec
Statement I, Time taken by the train to cross the platform =
Price of the mixture P and Q is Rs. 40 per litres {(L + x)/(54 × 5/18)} = (L + x)/18 sec
and Rs. 75 per litres, respectively Now,
Data in statement I alone is not sufficient to x/10 – (L + x)/15 = 30
answer the question. Data in statement II alone is not sufficient to
Statement II: answer the question.
Price of one liter of mixture R = 900/15 = Rs. 60 Combining statement, I and II,
Data in statement II alone is not sufficient to L = 100
answer the question. x/10 - (L + x)/15 = 30
Combining statement, I and II we get x/10 – (100 + x)/15 = 30
Let ratio in which mixture P, Q and R are mixed (3x – 200 – 2x)/30 = 30
is a:b:c x – 200 = 900
62 = {(40 × a + 75 × b + 60 × c)/(a + b + c)} x = 1100meter
62a + 62b + 62c = 40a + 75b + 60c Required difference = 1100 – 100 = 1000 meter
22a + 2c = 13b So, data in statement I and II together are
Since, we cannot find the value of a, b and c. sufficient to answer the question.
So, the data given in both statements I and II
together are not sufficient to answer the 8) Answer: E
question. From statement I,
C completes the work in 8*15/8= 15 days.
7) Answer: E But we cannot calculate the number of days
Statement I, taken by A to complete the work.
From statement II,

Click Here For Bundle PDF Course | support@guidely.in Page 9 of 12


SBI Clerk & RRB PO Mains PDF Course 2023
Quantitative Aptitude Day – 40 (Eng)

A alone complete the work in 10 days. But we Number of male visitors who visited Volkswagen
cannot calculate the number of days taken by C pavilion = 5670 + 1890 = 7560
to complete the work. So, the percentage of male visitors who visited
By combining I and II, Volkswagen pavilion is,
A alone can complete the work in 10 days = 7560/37,800 × 100 =>20 (Value of R)
C alone can complete the work in 15 days Number of male visitors who visited Maruti
= 1/10 + 1/15 = 6 days pavilion = 5145 × 1/3 = 1715
From both statements we can get the number of Number of female visitors who visited Toyota
days taken by A and C together to complete pavilion is = 1715 + 7³ = 1715 + 343 => 2058
work. So, the percentage of female visitors who visited
Hence, the data given in both statements I and II Toyota pavilion is,
together are necessary to answer the question. = 2058/5145 × 100 =>40 (Value of S)
Number of female visitors who visited Nissan
Directions (9-12): pavilion = 4410
Total number of visitors who visited the car expo So, the percentage of female visitors who visited
is = 67,200 Nissan pavilion is,
Total number of male visitors who visited car = 4410/29,400 × 100 =>15 (Value of T)
expo = 67,200 × 9/16 => 37,800 Number of male visitors who visited Toyota
Total number of female visitors who visited car pavilion = 3 × 2058 =>6174
expo = 67,200 × 7/16 => 29,400 Total number of visitors who visited Honda
Total number of visitors who visited Ford pavilion pavilion = 67,200 × 22/100 => 14,784
= 16,128 Number of male visitors who visited Nissan
So, the percentage of visitors who visited Ford pavilion = 4410 × 9/5 =>7938
Pavilion is, Number of female visitors who visited Honda
= 16,128/67,200 × 100 =>24 (Value of P) pavilion = 14,784 – 5670 => 9114
Number of female visitors who visited Maruti Number of male visitors who visited Ford
pavilion = 16,128 – 10,983 => 5145 pavilion is,
So, the percentage of female visitors who visited = 37,800 – (5670 + 1715 + 7560 + 7938 + 6174)
Maruti pavilion is, = 37,800 – 29,057 =>8743
= 5145/29,400 × 100 =>17.5 (Value of Q) Number of female visitors who visited Ford
Number of male visitors who visited Honda pavilion = 16,128 – 8743 => 7385
pavilion = 37,800 × 15/100 = 5670 Number of female visitors who visited
Volkswagen pavilion is,

Click Here For Bundle PDF Course | support@guidely.in Page 10 of 12


SBI Clerk & RRB PO Mains PDF Course 2023
Quantitative Aptitude Day – 40 (Eng)

= 29,400 – (9114 + 5145 + 4410 + 7385 + 2058) = 168000


= 29,400 – 28,112 => 1288 Number of students who paid fees through
For question 11: online in 2018 & 2019 together = 600000 *
9) Answer: B (20/100 * 62.5/100 + 15/100 * 42.5/100)
From the above common explanation it is clear = 60 * (1250+637.50)
that, = 113250
The value of P is = 24 Required difference = 168000-113250=54750
Hence, the required answer is = 24.
14) Answer: D
10) Answer: A Number of students who paid through online in
From the above common explanation it is clear 2015 = 1/2 * 600000 * (15/100 * 42.5/100 +
that, 25/100 * 20/100)
The value of Q is = 17.5 = 30 * (637.5 + 500)
Hence, the required answer is = 17.5 = 34125
Therefore Total number of students in 2015 =
11) Answer: D 34125 * 4/3 = 45500
From the above common explanation it is clear
that, 15) Answer: D
The value of S and T is = 40 and 15 Required percentage = (600000 * 25/100 *
Hence, the required answer is = 40 and 15 80/100)/(600000 * 20/100 * 62.5/100) * 100 =
160%
12) Answer: C
Value of [(Q × S × T) – (P × R × T)]/100 16) Answer: D
= [(17.5 × 40 × 15) – (24 × 20 × 15)]/100 Required average = 1/3 * 600000 (16/100 *
= (10,500 – 7200)/100 37.5/100 + 24/100 * 25/100 + 15/100 * 42.5/100)
= 3300/100 => 33 = 36750

13) Answer: A Directions (17-20):


Number of students who paid fees through The total number of petrol cars and diesel cars
offline in 2016 & 2017 together registered in city A is 400 and the difference
= 600000 * (16/100 * 62.5/100 + 24/100 * between petrol and diesel cars registered in city
75/100) A is 200.
= 60 * (1000+1800)

Click Here For Bundle PDF Course | support@guidely.in Page 11 of 12


SBI Clerk & RRB PO Mains PDF Course 2023
Quantitative Aptitude Day – 40 (Eng)

We know the number of petrol cars register is Total number of electric cars registered = 100 +
more than number of diesel cars registered in 75 + 150 + 150 + 125 = 600
city A. so we can say the number of petrol cars Central angle of electric cars in B = [360/600] *
register = [400 + 200]/2= 300 75 = 45̊
Number of diesel cars registered = [400 - 200]/2 So, the required difference = 15̊
= 100
Percentage of electric cars registered in city A is 19) Answer: B
20%, so the number of electric cars registered in For City A:
city A = [400/80] * 20 = 100 Total number of Maruti cars registered in the city
Similarly, we can also calculate the value of = [300 * 4/12] + [100 * 2/5] = 100 + 40 = 140
other cities. Total number of Tata cars registered in the city =
[300 * 5/12] + [100 * 2/5] = 125 + 40 = 165
Required percentage = [140/165] *100 = 84.84%

20) Answer: A
75% of the number of diesel cars registered in
city A = 100 * 75/100 = 75
Number of electric cars registered in city B = 75
So, this statement is true.
17) Answer: C
20% of the total number of electric cars
Required difference = [(150 + 350) * 20/100] –
registered in city D and E together = [150 + 125]
[(100 + 150) * 10/100] = 75
* 20/100 = 55
20% less than the total number of electric cars
18) Answer: D
registered in city E = 125 * 80/100 = 100
Total number of diesel cars registered = 100 +
So, this statement is false.
200 + 150 + 300 + 150 = 900
Central angle of diesel cars in C = [360/900]
*150 = 60̊

Click Here For Bundle PDF Course | support@guidely.in Page 12 of 12


SBI Clerk & RRB PO Mains PDF Course 2023
ENGLISH Day - 40

English Language

Directions (1-5): The questions below carry a phrases in the start-up world on a regular basis.
paragraph from which a part has been deleted. Investors ___________ whether they want to
You must read the same carefully and choose an invest or not.
option that correctly fits the given paragraph to (a) can survive downturns in the economy
complete the same in the most appropriate (b) value and trust the user over rapid
manner. development
1) Monsoon is here and so is the season for (c) analyse the types of start-ups and then
numerous hair problems. From frizz to fall, the decide
change in weather and increase in humidity (d) grow quickly and draw substantial
brings with it a hair-related crisis of sorts. Hairfall investments
during monsoon may also be caused by (e) None of these
hormonal changes triggered by high
temperatures and sun exposure during summer. 3) Layoffs are on the rise and with it, people are
Dust, sweat and oil may ________ haircare also becoming victims of various kinds of job
products leading to residue build-up that leads to scams. Recently in Delhi, the police arrested
more hair fall. At times, increased humidity can impostors ________ a job in the Airports Authority
also dry the scalp leaving the follicles weak and of India. In another case, a Maharashtra woman
deprived of nutrients. lost Rs 15 lakh in a work-from-home job scam.
(a) avoid hair fall in during the rainy season And mind you, these scams are not just
(b) also accumulate on the scalp along with restricted to India.
(c) absorb excess water from your hair (a) created various fake job postings for
(d) maintain scalp health during monsoon Singapore
(e) None of these (b) know the times are tough and the job market
is brutal
2) It might sound strange today, but tech start- (c) talking to people from the same field
ups were called just "start-ups" initially. (d) who duped people on the pretext of providing
Sometimes venture investors referred to them as (e) None of these
"portfolio companies", while customers called
them "tech companies", but "start-up" remained 4) Twitter users have asserted that they are able
to be the basic term. In 2013, start-ups began to access the old TweetDeck along with the free
their Animorphs journey, and that continues. API access. This free API made__________.
Several animal names are used as terms and Days ago, Twitter introduced rate-limits and

Click Here For Bundle PDF Course | support@guidely.in Page 1 of 9


SBI Clerk & RRB PO Mains PDF Course 2023
ENGLISH Day - 40

restrictions on tweets, along with killing the the word that should replace the inappropriate
legacy APIs that facilitated the above-mentioned word as well.
feature. Access to third-party apps, meanwhile, 6) These channels offer (A) engage opportunities
was banned back in January. for brands to (B) unprecedented with their
(a) accessing third-party apps possible customers in real-time, (C) deliver personalised
(b) the decision to limit the number of tweets content, and (D) neglect meaningful interactions.
(c) to do anything to encourage those verticals (a) A-D; suppress
(d) that the new Tweet Deck would have been (b) A-B; foster
accessible (c) B-C; create
(e) None of these (d) B-D; multiple
(e) No swapping needed
5) In today’s fast-paced digital landscape,
technology has emerged as a driving force of 7) In today’s fast- (A) regulated digital landscape,
transformation across various industries. One technology has (B) transformation as a driving
sector that has experienced a seismic shift is the force of (C) emerged across various (D)
advertising industry, where technological industries.
advancements have _____ their target audience. (a) B-D; transformed
Technology is changing the advertising (b) A-C; growing
landscape, empowering brands to create (c) B-C; paced
impactful and personalised experiences for their (d) A-D; companies
customers. (e) No swapping needed
(a) spark curiosity, and leave a lasting
impression 8) State-owned Bank of India is (A) exploring the
(b) revolutionised advertising (B) possibility of share sale to investors over the
(c) has experienced a remarkable revolution next one year to meet the (C) minimum public
(d) reshaped the way brands connect with holding (D) requirement of 25 per cent.
(e) None of these (a) A-D; maximum
(b) B-C; discovering
Directions (6-10): The questions given below (c) A-B; target
carry highlighted words which might be placed (d) B-D; business
incorrectly. One of these words would be (e) No swapping needed
inappropriate in the context and need to be
replaced. You must choose an option that carries 9) Suzlon is (A) committed to partner with an
the pair of words that should be swapped and increasing number of Indian industries, (B)

Click Here For Bundle PDF Course | support@guidely.in Page 2 of 9


SBI Clerk & RRB PO Mains PDF Course 2023
ENGLISH Day - 40

driving them toward their net‐zero (C) sum while 12) What __________ nearly 4,000 years ago in
powering the nation with (D) targets energy. the court of the Kauravas is still happening
(a) C-D; sustainable because basic human nature has ________
(b) A-B; slated changed.
(c) B-C; pushing (a) Happened, hardly
(d) B-D; energy (b) Came about, not much
(e) No swapping needed (c) Materialised, bare
(d) Hit upon, slightly
10) Demonetisation (A) continue confidence in (e) None of these
the rupee, (B) especially in Bhutan and Nepal,
and both countries (C) shook to fear additional 13) While Assam is the most _________ example,
(D) relevant policy changes. several states in the Northeast have witnessed a
(a) B-C; refer significant ________ from across the border.
(b) B-D; specially (a) Serious, migrants
(c) A-C; sudden (b) Obvious, influx
(d) A-B; ensured (c) Evident, exodus
(e) No swapping needed (d) Apparent, arrival
(e) Both (c) and (d)
Directions (11-15): The questions given below
carry sentences in which words from the options 14) This fact is _________ since about 96% of
given below them can be filled to complete the China’s container trade with Europe went
same grammatically and meaningfully. You must ________ sea routes.
choose the most appropriate option for each (a) Importance, across
sentence as your answer. (b) Venerable, haywire
11) Our regulators have had a record of ______ (c) Crucial, via
and poor oversight over unlisted companies’ (d) Paramount, though
_________ startups. (e) None of these
(a) Negligible, of
(b) Meticulous, preferring 15) A _________ search operation was set in
(c) Careful, excluding motion and hours later the vessel was found
(d) Lax, including _________ with deceased individuals.
(e) Careless, counted as (a) Evacuation, mended
(b) Massive, imploded
(c) Tremendous, splintered

Click Here For Bundle PDF Course | support@guidely.in Page 3 of 9


SBI Clerk & RRB PO Mains PDF Course 2023
ENGLISH Day - 40

(d) Vast, burnt (a) Only II


(e) Immense, sunk (b) Both I and III
(c) Only III
Directions (16-20): The questions given below (d) Only I
carry tables in which a sentence has been given (e) Both II and III
in the first column. A part of the sentence has
been deleted. You are required to choose an 18)
option from the second column that can Column I Column II
complete the given sentence meaningfully and When you make I. what’s
grammatically. organisations safe for heartbreaking is the
16) women, derision
Column I Column II ________________ for II. you make them
Knowledge centres I. by hire and fire, everybody. safer
_____________________ of stick and carrot III. it is the job of the
productivity and II. do not operate IC to identify those
accountability. by tactics of the (a) Only III
market (b) Both I and II
III. call for rigorous (c) Only I
standards (d) Both I and III
(a) Only I (e) Only II
(b) Only II
(c) Only III 19)
(d) Both I and III Column I Column II
(e) None matches Flipkart customers I. will now be able to
_______________ get II. has entered a
17) personal loans from Axis partnership to
Column I Column II Bank. III. will also provide
Simply having the largest I. is not enough in customers
labour force an (a) Only I
___________________ II. has never been (b) Both II and III
environment of fast- enough in (c) Only II
changing production III. will not be (d) Both I and II
processes. considered an (e) None of these

Click Here For Bundle PDF Course | support@guidely.in Page 4 of 9


SBI Clerk & RRB PO Mains PDF Course 2023
ENGLISH Day - 40

20) insurance. main trigger


Column I Column II (a) Only I
For 56%, a I. familiar with or the (b) Only II
recommendation from brands (c) Only III
friends and II. consumers (d) Both I and II
________________ for indicated that (e) None
purchasing health III. family was the

Click Here to Get the Detailed Video Solution for the above given Questions
Or Scan the QR Code to Get the Detailed Video Solutions

Answer Key with Explanation

1) Answer: B The part of the sentence after the blank says;


The sentence with the blank talks about dust, ‘whether they want to invest or not’ which means
sweat etc that would harm the hair obviously so, the investors properly analyse before they
the first option that says ‘avoid hair fall’ will go invest…
against the context. The second option makes The first option mentions a new aspect and does
sense as it says these things might get collected not match the discussion so it can be ruled out.
on the scalp (leading to residue build-up)… The The second option looks incomplete. The third
third option is irrelevant as these things do not option completes the idea that we discussed
absorb water. The fourth option; like the first one above. Why would investors draw investments?
is also irrelevant. This makes the fourth option irrelevant.
This makes option (b) the most logical answer The best would be to mark option (c) as the
choice. answer.

2) Answer: C 3) Answer: D

Click Here For Bundle PDF Course | support@guidely.in Page 5 of 9


SBI Clerk & RRB PO Mains PDF Course 2023
ENGLISH Day - 40

The first and the second options do not continue for using the verb has (which would go wrong
the sentence grammatically so these can be immediately after have). The fourth option
ruled out. makes sense as; it ends in with and the
The third option says people from the same field sentence is continued with their target
which makes it irrelevant as nothing of the sort audience…
has been discussed. The best would be to mark option (d) as the
The fourth option continues well as imposters answer.
are people who pretend to be someone else so
they can dupe that is cheat or trick people. 6) Answer: B
This makes option (d) the best choice. The word engage does not fit well before
opportunities and we need an adjective in its
4) Answer: A place. What kind of opportunities?
The latter part of the paragraph tells us that the Unprecedented opportunities means those that
access to third-party apps was banned in have been seen for the first time and were been
January which now makes the first option presented with ever before…This tells us that
logically support the idea that the API would the words in A and B need to be swapped.
make accessing the third-party apps possible. Also, the word neglect has incorrectly been used
The second option talks about limiting the because the sentence intends to promote or
number of tweets which has again been talked nurture meaningful interactions and not neglect
about in the next line making it repetitive and them. So, the word foster would best replace the
redundant. (c) is unrelated and vague so can be same.
ruled out. Same logic eliminates the fourth option This makes option (b) the most appropriate
as well. answer choice.
The best would hence, be to mark option (a) as
the answer. 7) Answer: C
The word regulated in the given sentence neither
5) Answer: D fits in A nor anywhere else in the sentence. This
The first option is incorrect as have has been clarifies that this word needs to be replaced. The
used before the blank and spark begins the other words highlighted in the sentence cannot
option making it incorrect grammatically. Have be used after the word fast which makes it
should be followed by the third form of the verb. necessary for us to choose a suitable word from
The second option is incomplete and would not the options. Fast-paced makes a correct phrase
fit in the sentence. The third option goes wrong as it means moving or developing very quickly…

Click Here For Bundle PDF Course | support@guidely.in Page 6 of 9


SBI Clerk & RRB PO Mains PDF Course 2023
ENGLISH Day - 40

This is a hint that the words in B and C should The word relevant is positive but such policy
swapped as well for the sentence to be correct. changes would not create fear. So, this word is
Therefore, the best would be to mark option (c) contextually incorrect.
as the answer. The best would be to mark option (c) as the
answer.
8) Answer: E
The given sentence uses all the words correctly 11) Answer: D
and no swapping or rearrangement is needed. The word negligible means very small and
This makes option (e) the most logical answer therefore not important. This can be taken as
choice. correct but the second word of makes no sense
in the context. The word meticulous is same as
9) Answer: A careful but both cannot fill the first blank as they
The first two words seem correct as given. will go contextually incorrect. The choice for the
The word sum does not fit in the sentence at any second blank in the last option makes it
place so it should be changed. incorrect.
The word targets should come after net-zero Lax means not having high standards; not strict.
which means the words in C and D should be The words given in option (d) fill the sentence
interchanged for the sentence to make sense. correctly.
Sustainable means involving the use of natural
products and energy in a way that does not harm 12) Answer: A
the environment. This word fits best before the The first blank can take the word happened as
word energy. the sentence itself gives a hint by using the
So, the correct option should be (a). phrase ‘still happening’ in the later part of the
sentence.
10) Answer: C The phrase came about also means happened
The word continue doesn’t fit well after the but the choice for the second blank does not fit
singular subject Demonetisation so it needs to in.
be swapped for sure… The sentence talks about Materialised means to become real; to happen…
fear (of countries in relation to the currency) bare means uncovered. Both these words make
which would shake the confidence of these no sense in the given sentence.
countries. So, the word shook should come in Hit upon means discover or think of something,
place of A. This tells us that A and C must be especially by chance… This word cancels the
interchanged. fourth option as well.

Click Here For Bundle PDF Course | support@guidely.in Page 7 of 9


SBI Clerk & RRB PO Mains PDF Course 2023
ENGLISH Day - 40

The only option carrying both the correct words carries the word search after the first blank
is (a). which would make the given word redundant.
Mended means to repair something that is
13) Answer: B damaged or broken… imploded means
The first option can be ruled out as the word collapsed. This word describes the state of the
migrants does not fit in grammatically. vessel mentioned in the sentence. Tremendous
The word obvious means easily seen or is very large or great; very good but this carries a
understood; clear. Influx means large numbers positive tone making it unfit for the sentence
of people or things arriving suddenly… (this fits given above. Splintered means to split or rend
as it describes people coming from across the into long thin pieces.
border). Exodus is the opposite of influx as it Of all the given options, the pair of words that
means the mass departure of people. Apparent make the sentence grammatically and
means the same as obvious or evident but meaningfully correct is (b).
arrival is not fit here. It is best used with respect
to railway stations or airports etc. 16) Answer: C
The only pair of words that fits correctly can be The sentence does not take the first option
found in option (b). correctly as it is a misfit in the given context. If
we take the second option, the preposition ‘of’
14) Answer: C will make it an incorrect continuation.
The first blank can take the words in the third The third option is correct as it completes the
and the fourth options only and not the others. idea that knowledge centres make rigorous
The word venerable means highly respectable… standards of productivity necessary.
The word haywire means erratic; out of control… Therefore, the correct answer is option (c).
important could have been a better choice
grammatically for choosing the first option. The 17) Answer: D
fourth option gets eliminated for the usage of the The sentence starts with the word simply which
word though… means used to emphasize how easy or basic
The best combination of words that can be found something is. This means that having the largest
in the options hence, is (c). labour force is just a basic thing and much more
is needed.
15) Answer: B The first option completes the above meaning
Evacuation means the removal of persons or contextually. Also, it uses an which will be
things from an endangered area. The sentence

Click Here For Bundle PDF Course | support@guidely.in Page 8 of 9


SBI Clerk & RRB PO Mains PDF Course 2023
ENGLISH Day - 40

correct before environment. The absence of the This makes the fist option the best answer
article makes the second option incorrect. choice.
The third option is incorrect as it makes the
sentence contextually incorrect. 19) Answer: A
This makes option (d) the most logical answer The only option that fits the blank grammatically
choice. is the first one. The second option uses has
incorrectly for the plural subject customers which
18) Answer: E makes it a misfit. The third option does not fit in
The first option mentions the word derision which grammatically.
means the situation in which someone or This makes option (a) the best answer choice.
something is laughed at and considered stupid
or of no value… making organisations safe will 20) Answer: C
certainly not create any such situation. The third option continues the given sentence
The second option fits perfectly because it most appropriately by completing the phrase
continues the idea of organisations becoming ‘…friends and family…’
safe for women and everyone else. The other two options can be eliminated for
The third option brings in the IC which is a new being unrelated and grammatically incorrect.
perspective as per the sentence so it can be Thus, the correct answer is option (c).
cancelled.

Click Here For Bundle PDF Course | support@guidely.in Page 9 of 9


SBI Clerk & RRB PO Mains PDF Course 2023
Reasoning Ability Day - 41 (Eng)

Reasoning Ability
Directions (1-5): Study the following information c) 8
carefully and answer the given questions. d) 6
Step 1: Take the alphabets whose place value is e) None of these
an even number as per the English alphabetical
series and they are written in the reverse 2. If the last 15 elements are reversed, then how
alphabetical order from the left end. Then write many such consonants are there in the series
the vowels from the right end in alphabetical which is immediately preceded by a symbol and
order. not immediately followed by a vowel?
Step 2: After completing step 1, take the a) One
consecutive three digit odd numbers up to the b) Two
number 117 and add the digits within the number c) Three
till it becomes a single digit, then place the d) Four
resultant number one by one in the same order e) None
after every two letters from the left end.
Step 3: After completing step 2, the first 12 3. What is the sum of the numbers between the
elements from the left end are reversed and the third vowel from the right end and the fourth
following symbols “$, &, %, #, @, ©, *, ®, ∞”, are consonant from the left end?
placed (in the same order) in the series after a) 10
every three elements from the left end. b) 17
Step 4: After completing step 3, the letters which c) 12
are immediately preceded by a symbol and d) 14
immediately followed by a vowel are changed to e) 8
the immediate next letter as per the alphabetical
series. 4. In the given series, all the letters in the first
Step IV is the final step and answer the given sixteen elements are replaced by the
questions according to the final step. complementary paired letter in the English
1. If all the symbols are dropped from the series alphabetical series, then which among the
and all the numbers are arranged in ascending following letters are repeated more than once?
order from the right end then which of the a) E, D, M, U
following element will be 11th to the right of the 9th b) N, P, T, E
element from the left end? c) F, D, P, T
a) A d) Z, X, G, A
b) 9 e) I, C, A, F

Click Here For Bundle PDF Course | support@guidely.in Page 1 of 11


SBI Clerk & RRB PO Mains PDF Course 2023
Reasoning Ability Day - 41 (Eng)

5. In the given series, all the consonants which below input and answer the questions given
come before G and after T in the alphabetical below.
series are dropped, then how many symbols are
immediately preceded by a consonant and
immediately followed by a number? 6. What will be the resultant if the fourth digit

a) Two from the left end in step 1 is multiplied with the

b) One first digit from the right end in step 3 of the given

c) Four input?

d) Three a) 14

e) None b) 18
c) 16

Directions (6-10): Study the following information d) 24

carefully and answer the given questions. e) 20

The number arrangement machine, when given


an input, rearranges them by following a 7. What is the difference between the square

particular rule in each step. The following is the values of the two numbers in step 3 of the given

illustration of an input and rearrangement. input?


a) 200
b) 384
c) 250
d) 376
e) 380

8. What will be the output, if the second digit


from the left end in step 2 is doubled?
a) 6
b) 4
c) 10
d) 12
e) 8

9. If all the digits in step 1 are arranged in


Step 4 is the last and final step of the given input. ascending order from left to right, then how many
By following the same rules, find the steps for the digits remain unchanged in their position?

Click Here For Bundle PDF Course | support@guidely.in Page 2 of 11


SBI Clerk & RRB PO Mains PDF Course 2023
Reasoning Ability Day - 41 (Eng)

a) Three that A and B sit opposite to each other, then both


b) Two are sitting at the same table.
c) One F sits third to the right of B, who is facing the
d) Four centre of the table. One person sits between F
e) None and K. Two persons sit between J and the one
who faces K. H sits opposite to J. L sits second
10. What is the difference between the digits of to the right of the one who faces J. The number
the number in the penultimate step of the given of persons sitting between L and F is two less
input? than the number of persons sitting between J
a) Five and O, who does not face K. C is an immediate
b) Seven neighbour of both O and P. N sits second to the
c) Six left of the one who faces P. I sits to the
d) Eight immediate left of E. The number of persons
e) Four sitting between E and C is two more than the
number of persons sitting between B and A. M
Directions (11-15): Study the following does not face the centre of the table.
information carefully and answer the given 11. If all the persons sitting at the outer circular
questions table are rearranged in alphabetical order in a
Sixteen persons - A, B, C, D, E, F, G, H, I, J, K, clockwise direction starting from A, then how
L, M, N, O, and P are sitting around the two many persons remain unchanged except A?
concentric circular tables as one is inscribed in a) One
another one. Eight persons are sitting on the b) Two
dinner table facing away from the centre. Eight c) Three
persons are sitting on the outer table facing the d) Four
centre. Each person sitting on the outer table e) None
faces the person sitting on the dinner table and
vice versa. 12. If all the persons interchanged their positions
Note: with the person who is facing them, then who
i) The persons whose name’s place value is a among the following sits third to the right of L?
prime number as per alphabetical order are not a) The one who faces I
sitting adjacent to each other. b) N
ii) If it is given that A and B face each other, then c) The one who sits second to the left of D
both are sitting at different tables and if it is given d) A
e) G

Click Here For Bundle PDF Course | support@guidely.in Page 3 of 11


SBI Clerk & RRB PO Mains PDF Course 2023
Reasoning Ability Day - 41 (Eng)

13. Four of the following five are alike in a certain lowermost floor is numbered one and the floor
way based on the given arrangement and thus immediately above it is numbered two and so on.
form a group. Which one of the following does There are three lifts Lift I, Lift II, and Lift III from
not belong to the group? west to east in the basement of the building.
a) L Each delivery boy takes lifts to deliver the food.
b) D Not more than two persons delivered food on the
c) E same floor. Also, persons delivering on the same
d) B floor take different lifts.
e) A Note:
i) The person whose name’s place value is an
14. Which among the following pair of persons odd number as per the alphabetical series
are facing each other? delivered in an even numbered floor and the
I. KM person whose name’s place value is an even
II. AP number as per the alphabetical series delivered
III. GJ in an odd numbered floor.
a) Only I ii) No two persons delivered the food on the
b) Only II adjacent floors take the same lift.
c) Only I and II F delivered the food three floors above I, where
d) Only II and III neither of them uses the same lift nor uses lift II.
e) All I, II and III F did not deliver above floor number 6. Both N
and H delivered below I. J does not use lift II and
15. How many persons sit between P and the delivered two floors above H, where both of them
one who faces N when counted from the right of use the same lift. J and D delivered on the same
P? floor. The number of persons delivered the food
a) Two below D is three more than the number of
b) One persons delivered the food above C, who did not
c) Three use the same lift as N and J. Less than three
d) Four floors are between the floors on which C and G
e) None delivered the food. L delivered immediately
above the floor on which G delivered the food. G
Direction (16-20): Study the following information uses lift II. More than two floors are between the
carefully and answer the given questions. floors on which L and K delivered the food. The
Fourteen Zomato delivery boys – A to N number of floors between the floors on which K
delivered food in a ten storey building where the and I delivered the food is one less than the

Click Here For Bundle PDF Course | support@guidely.in Page 4 of 11


SBI Clerk & RRB PO Mains PDF Course 2023
Reasoning Ability Day - 41 (Eng)

number of floors between the floors on which F a) One


and M delivered the food. E delivered below A b) Two
but above B. E does not use lift III. c) Three
16. How many persons are using lift II? d) Four
a) Three e) Five
b) Four
c) Six 19. As many floors between B and N as between
d) Five _____ and _____.
e) Seven a) I and H
b) L and F
17. Who among the following person(s) delivered c) A and G
food to floor number 5? d) M and B
I. B e) C and G
II. D
III. F 20. Four of the following five are alike in a certain
a) Only I way based on the given arrangement and thus
b) Only II form a group. Which one of the following does
c) Only I and III not belong to the group?
d) Only II and III a) L
e) All I, II and III b) C
c) M
18. How many persons delivered the food above d) E
E’s Floor? e) I
Click Here to Get the Detailed Video Solution for the above given Questions
Or Scan the QR Code to Get the Detailed Video Solutions

Answer Key with Explanation

Click Here For Bundle PDF Course | support@guidely.in Page 5 of 11


SBI Clerk & RRB PO Mains PDF Course 2023
Reasoning Ability Day - 41 (Eng)

Directions (1-5): 8LN$6PR&4TV%2XZ#JH1@FD3


Step 1: Take the alphabets whose place value is ©CU5*PI7®FA9∞
an even number as per the English alphabetical 8LN6PR4TV2XZJH1FD3CU5PI7
series and they are written in the reverse FA9
alphabetical order from the left end. Then write LNPRTVXZJHFDCUPIFA987654
the vowels from the right end in alphabetical 321
order.
ZXVTRPNLJHFDBUOIEA 2. Answer: B
Step 2: After completing step 1, take the Given series:
consecutive three digit odd numbers up to the 8LN$6PR&4TV%2XZ#JH1@FD3
number 117 and add the digits within the number ©CU5*PI7®FA9∞
till it becomes a single digit, then place the 8LN$6PR&4TV%2XZ#JH1@F∞9A
resultant number one by one in the same order F®7IP*5UC©3D
after every two letters from the left end.
ZX2VT4RP6NL8JH1FD3BU5OI7 3. Answer: A
EA9 Given series:
Step 3: After completing step 2, the first 12 8LN$6PR&4TV%2XZ#JH1@FD3
elements from the left end are reversed and the ©CU5*PI7®FA9∞
following symbols “$, &, %, #, @, ©, *, ®, ∞”, are 4+2+1+3= 10
placed (in the same order) in the series after
every three elements from the left end. 4. Answer: E
8LN$6PR&4TV%2XZ#JH1@FD3 Given series:
©BU5*OI7®EA9∞ 8LN$6PR&4TV%2XZ#JH1@FD3
Step 4: After completing step 3, the letters which ©CU5*PI7®FA9∞
are immediately preceded by a symbol and 8OM$6KI&4GE%2CA#JH1@FD3
immediately followed by a vowel are changed to ©CU5*PI7®FA9∞
the immediate next letter as per the alphabetical
series. 5. Answer: D
8LN$6PR&4TV%2XZ#JH1@FD3 Given series:
©CU5*PI7®FA9∞ 8LN$6PR&4TV%2XZ#JH1@FD3
1. Answer: C ©CU5*PI7®FA9∞
Given series: 8LN$6PR&4T%2#JH1@3©5*PI7
®9∞

Click Here For Bundle PDF Course | support@guidely.in Page 6 of 11


SBI Clerk & RRB PO Mains PDF Course 2023
Reasoning Ability Day - 41 (Eng)

11. Answer: A
Directions (6-10): 12. Answer: D
6. Answer: D 13. Answer: C (All the persons face the centre of
7. Answer: B the table except option c)
8. Answer: E 14. Answer: E
9. Answer: E 15. Answer: B
10. Answer: A Final arrangement:
Step 1: The product of the first digit of the two
boxes and the product of the second digit of the
two boxes.
Step 2: The difference between the product of
the two digits of the first and second boxes and
the difference between the product of the two
digits of the second and third boxes.
Step 3: The sum of the squares of the two digits
within the box.
Step 4: The difference between the two numbers
of both boxes. We have,
Step 5: The sum of the two digits of the number.  F sits third to the right of B, who is facing
the centre of the table.
 One person sits between F and K.
 Two persons sit between J and the one
who faces K.
From the above conditions, we have two
possibilities,

Directions (11-15):

Click Here For Bundle PDF Course | support@guidely.in Page 7 of 11


SBI Clerk & RRB PO Mains PDF Course 2023
Reasoning Ability Day - 41 (Eng)

Again we have,
 H sits opposite to J.
 L sits second to the right of the one who Again we have,
faces J.  I sits to the immediate left of E.
 The number of persons sitting between L  The number of persons sitting between E
and F is two less than the number of and C is two more than the number of
persons sitting between J and O, who persons sitting between B and A.
does not face K.  M does not face the centre of the table.
 C is an immediate neighbour of both O We cannot place G in case 1. Hence it is
and P. eliminated. Thus, case 2 gives the final
 N sits second to the left of the one who arrangement.
faces P.

Click Here For Bundle PDF Course | support@guidely.in Page 8 of 11


SBI Clerk & RRB PO Mains PDF Course 2023
Reasoning Ability Day - 41 (Eng)

We have,
 F delivered the food three floors above I,
where neither of them uses the same lift
nor uses lift II.
 F did not deliver above floor number 6.
 Both N and H deliver below I.
 J does not use lift II and delivered two
floors above H, where both of them use

Directions (16-20): the same lift.

16. Answer: D After applying the above conditions, there are

17. Answer: C two possibilities.

18. Answer: E
19. Answer: B
20. Answer: A (All the persons use lift I except
option a)
Final arrangement:

Click Here For Bundle PDF Course | support@guidely.in Page 9 of 11


SBI Clerk & RRB PO Mains PDF Course 2023
Reasoning Ability Day - 41 (Eng)

 Less than three floors are between the


floors on which C and G delivered the
food.
 L delivered immediately above the floor
on which G delivered the food. G uses lift
II.

Again, we have
 J and D delivered on the same floor.
 The number of persons delivered the food
below D is three more than the number of
persons delivered the food above C, who
did not use the same lift as N and J.
Again, we have

Click Here For Bundle PDF Course | support@guidely.in Page 10 of 11


SBI Clerk & RRB PO Mains PDF Course 2023
Reasoning Ability Day - 41 (Eng)

 More than two floors are between the


floors on which L and K delivered the
food.
 The number of floors between the floors
on which K and I delivered the food is one
less than the number of floors between
the floors on which F and M delivered the
food.
 E delivers below A but above B.
 E does not use lift III.
After applying the above conditions, case 1 gets
eliminated because E does not use lift III, hence
case 2 shows the final arrangement.

Click Here For Bundle PDF Course | support@guidely.in Page 11 of 11


SBI Clerk & RRB PO Mains PDF Course 2023
Quantitative Aptitude Day – 41 (Eng)

Quantitative Aptitude

Direction (1-5): Study the following data carefully and answer the questions:
The data given below is related to the expenditure on food, expenditure of a person on travelling and
other expenditures of a person on 4 different days Mon, Tue, Wed and Thu of a certain month.
The table given below shows the following data:

Note:
1: Total expenditure on Mon is ₹ 760.
2: The ratio of the average expenditure on food and travelling on Mon to that on Wed is 3: 2.
3: The ratio of the average expenditure on food and travelling on Wed to that on Thu is 10: 11.
1) If the average expenditure on food on Mon, is of that on Wed and the other
Tue and Wed is ₹ P, the average expenditure on expenditure on Fri is 137.5% of that on Mon,
travelling on Mon, Tue and Wed is ₹ Q and the then find the difference between the total
average of other expenditures on Mon, Tue and expenditure on Thu and that on Fri?
Wed is ₹ R, then find the ratio between P, Q and a) ₹ (x – 150)
R respectively? b) ₹ (y – 80)
a) 9: 7: 6 c) ₹ (z – 120)
b) 27: 23: 18 d) Both (a) and (b)
c) 3: 2: 1 e) Both (a) and (c)
d) 18: 15: 11
e) 36: 31: 28 3) If the ratio of total expenditure on food on all 4
days together to the total other expenditures on
2) If the expenditure on food on Fri is 93.75% of all 4 days together is m: n, then find that which of
that on Tue, the expenditure on travelling on Fri the following is/are true?

Click Here For Bundle PDF Course | support@guidely.in Page 1 of 16


SBI Clerk & RRB PO Mains PDF Course 2023
Quantitative Aptitude Day - 41 (Eng)

X: Both ‘m’ and ‘n’ are multiples of 17. b) 10(a – b)%


Y: The Sum of ‘m’ and ‘n’ is divisible by 4. c)(a + b – 3.52)%
Z: The Difference of ‘m’ and ‘n’ is divisible by 3. d) 12(a – b)%
a) None is true e) None of these
b) Only Z
c) Only X and Z 5) If the total expenditure on food for the whole
d) Only Y month is ₹ 7575 and the total expenditure on
e) Only X travelling for the whole month is ₹ 6975, then
find the ratio of the total expenditure on food to
4) If the total expenditure on Mon and the total the total expenditure on travelling for the
expenditure on Tue are respectively a% and b% remaining days of the month?
of the total monthly salary of the person, and the a) 111: 92
monthly salary of the person is ₹ 25000, then b) 91: 81
find that the total expenditure on Wed is what c)101: 93
percent of the total monthly salary? d) 81: 73
a) (a + b – 3.82)% e) 121: 93

Direction (6-10): Study the following data carefully and answer the questions:
The data given below is related to the speeds (in m/s) and the lengths (in m) of 5 different trains A, B, C,
D and E.
The pie chart given below shows the percentage distribution of the speed of each train.

The pie chart given below shows the percentage distribution of the length of each train.

Click Here For Bundle PDF Course | support@guidely.in Page 2 of 16


SBI Clerk & RRB PO Mains PDF Course 2023
Quantitative Aptitude Day - 41 (Eng)

Note:
1: The time taken by train A to cross a 120 m long tunnel is 15 seconds more than that taken by train B to
cross the same tunnel.
2: The time taken by train D to cross a man standing on a platform, is 18 seconds.
6) Time taken by train A to cross the 120 m long cross the same tunnel, then find which of the
platform at ___% of its original speed is 7 following is/are true?
seconds more than that taken by train B to cross P: Train A can cross the tunnel in 41 seconds.
the same platform at ___% of its original speed. Q: The Length of the tunnel is a multiple of 3.
Find which of the following can be filled in the R: Average length of train E and that of the
blanks in the same order? tunnel is 210 m.
P: 120, a) Only P and R
Q: 80, 50 b) Only P and Q
R: 90, 75 c) Only Q and R
a) Only P d) All are true
b) Only P and R e) None is true
c) Only Q
d) Only Q and R 8) If the speed of train F is 62.5% of that of train
e) Only P and Q B and train B can cross train F in 50 seconds
while running in the same direction, then find that
7) If the time taken by train E to cross a tunnel is the length of train F is of that of which
seconds more than that taken by train C to train?
a) Train D

Click Here For Bundle PDF Course | support@guidely.in Page 3 of 16


SBI Clerk & RRB PO Mains PDF Course 2023
Quantitative Aptitude Day - 41 (Eng)

b) Train B e) None of these


c) Train A
d) Train C 10) Train B is running 120 m behind train D and
e) Train E train D is running 150 m behind train C. If train B
crosses train D in M seconds and train D crosses
9) Find the ratio of the time taken by train B to train C in N seconds, then find that the difference
cross train D while running in the same direction between M and N is divisible by which of the
to that taken by train B to cross train D while following?
running in the opposite direction? a) 7
a) 15: 1 b) 8
b) 21: 1 c) 4
c) 5: 1 d) Both (a) and (b)
d) 11: 1 e) Both (a) and (c)

Direction (11-15): Study the following data carefully and answer the questions:
The data given below is about the number of matches played, matches won, matches lost, and matches
in which no result came in five different years of team M.
The bar graph given below shows the number of matches won as a percent of the total number of
matches played. It also shows the number of tied matches as a percent of the total number of no result
matches.

Click Here For Bundle PDF Course | support@guidely.in Page 4 of 16


SBI Clerk & RRB PO Mains PDF Course 2023
Quantitative Aptitude Day - 41 (Eng)

The table given below shows the number of matches lost by the team and the ratio of the number of
matches won to number of drawn matches.

Note: 1.Total matches played = Matched won + Matches lost + No result matches
2. No result matches = Tied matches + Drawn matches
11) Out of the total matches played bye team M
in 2015, x% were played against team A. Out of
the total matches played against team A in 2015,
in ‘x – 12’ matches team A batted first and in the Out of the total runs scored by team M in 2005,
remaining matches team M batted first. If the 54% came in winning matches. Find the average
number of matches in which team M batted first runs/match scored by team M in 2005 in winning
is a prime number, then find the sum of all the matches.
possible matches in which team A batted first. a) 221.2
a) 68 b) 220.4
b) 76 c) 232.8
c) 84 d) 219.6
d) 72 e) None of these
e) 80
13) If In 2020, the average runs scored by team
12) In 2005, out of the total runs scored by team M in winning matches, losing matches, tied
M are contributed by four players only P, Q, R, matches, and drawn matches is 325, 275, 250,
and S and the table given below shows the and 300 respectively and the average runs
average runs per match scored by all the four scored per match in all the matches by team M in
players in all the matches.

Click Here For Bundle PDF Course | support@guidely.in Page 5 of 16


SBI Clerk & RRB PO Mains PDF Course 2023
Quantitative Aptitude Day - 41 (Eng)

2020 is X, then which of the following regarding y% of the number of matches lost by team M in
the value of ‘X’ is incorrect? all the five years together.
a) ‘X’ is a prime number. Find the sum of all three quantities I, II, and III.
b) ‘X + 2’ is a multiple of 59. a) 245
c) Closest perfect square to‘X’ is 289. b) 220
d) ‘X – 3’ is a multiple of 29. c) 235
e) One of the immediate neighbours of ‘X’ is a d) 265
prime number. e) 250

14) Out of the total matches played by team M in 16)Two persons A and B started a business with
2010, are played against team P and the the ratio of their investment as 3: 4. After 2
ratio of the number of matches played against months, C joins them with an initial investment of
team P in 2000, 2005, 2010, 2015, and 2020 is ₹ 60000 and after 4 more months, A added
3: 5: 4: 6: 2 respectively. If out of total matches ₹ 15000 more of his capital. If A receives
played against team P by team, ‘x’% are won ₹ 1800/month as salary since he is an active
and ‘x – 1’ are lost(Assume there are not any no partner in the business and the total profit
result matches between team P and M). Find the received from the business at the end of a year
HCF of ‘X’ and ‘X + 5’. is ₹ 144000 out of which share of B is ₹ 38400.
a) 5 Find the difference between the share of the total
b) 10 profit amount received by A (including salary)
c) 15 and the profit share of C.
d) 20 a) ₹ 8400
e) 25 b) ₹ 9600
c) ₹ 10800
15) Quantity I: The difference between number of d) ₹ 9000
tied matches and the number of drawn matches e) ₹ 7200
by team M in all the five years.
Quantity II: Value of ‘x’ if x% of the number of 17) An item costs ₹ 1500 which is marked 80%
lost matches by team M in all five years is equal above cost price and sold after giving the three
to the square of 10. successive discounts mentioned below:
Quantity III: Value of ‘y’ if the number of matches A: Discount of 10%.
won by team M in all the five years together is B: Discount of ₹ 100.
C: Discount of ₹ 180

Click Here For Bundle PDF Course | support@guidely.in Page 6 of 16


SBI Clerk & RRB PO Mains PDF Course 2023
Quantitative Aptitude Day - 41 (Eng)

If the profit percent earned after selling the item less than the perfect square of an integer. If the
is 44%, then find the correct order of successive average age of all the 3 persons is ‘M’, then
discounts given on the item. which of the following can be the possible
a) ABC value(s) of ‘M’?
b) BAC Note: Ages (in years) of all the 3 persons are 2-
c) CAB digit numbers.
d) ACB I: 39
e) None of these II: 42
III: 44
18) An amount of ₹ 11000 (A) is invested at a a) Only I
20% (B) annual rate of simple interest for 3 years b) Only II
(C) such that the interest amount received is c) Only III
₹ 7260. Values marked with the letters A, B, and d) Only I and III
C needs to be replace with the values given e) Only II and III
below.
A: ₹ 12100 20) Ratio of male to female employees in a
B: 11% company is 8: 7. For a particular event only
C: 6 years of male and (50% of 300 – 82– √36)
Which of the following value(s) need to be % of female employees were present. If the
replaced? difference between male and female absent
a) Only B employees for the event is ,
b) Only A then find the total number of employees (male
c) Only B and C together and female) present for the event.
d) Either (a) or (b) a) 640
e) Either (b) or (c) b) 720
c) 560
19) Average age of A and B (A > B) and the d) 600
difference between their ages is given by the e) 480
roots of the equation: x2 – 48x + 432 = 0. HCF of
the ages of B and C is 15 while the age of C is 4

Click Here For Bundle PDF Course | support@guidely.in Page 7 of 16


SBI Clerk & RRB PO Mains PDF Course 2023
Quantitative Aptitude Day – 41 (Eng)

Click Here to Get the Detailed Video Solution for the above given Questions
Or Scan the QR Code to Get the Detailed Video Solutions

Answer Key with Explanation

Direction (1-5):
Since, the total expenditure on Mon is ₹ 760.
So,

x = 250
Since, the ratio of the average expenditure on
food and travelling on Mon to that on Wed is 3:
2.
So,

y = 180
Since, the ratio of the average expenditure on
food and travelling on Wed to that on Thu is 10:
11.

z = 200

1) Answer: B
Average expenditure on food on Mon, Tue and
Wed:

Click Here For Bundle PDF Course | support@guidely.in Page 8 of 16


SBI Clerk & RRB PO Mains PDF Course 2023
Quantitative Aptitude Day – 41 (Eng)

Average expenditure on travelling on Mon, Tue So, X is not true.


and Wed: From Y:
Sum of ‘m’ and ‘n’ = 101 + 68 = 169

Average of other expenditures on Mon, Tue and Since, the sum of ‘m’ and ‘n’ is not divisible by 4.

Wed: So, Y is not true.


From Z:
Difference between ‘m’ and ‘n’ = 101 – 68 = 33
Required ratio = P: Q: R = 270: 230: 180 = 27:
Since, the difference between ‘m’ and ‘n’ is
23: 18
divisible by 3.
So, Z is true.
2) Answer: D
Hence, only Z is true.
The expenditure on food on Fri = 93.75% of 320
= ₹ 300
4) Answer: C
The expenditure on travelling on Fri = of
Total expenditure on Mon = 240 + 360 + 160 =
150 = ₹ 160
₹ 760
The other expenditures on Fri = 137.5% of 160 =
So, a = = 3.04%
₹ 220
Total expenditure on Tue = 320 + 180 + 200 =
The total expenditure on Fri = 300 + 160 + 220 =
₹ 700
₹ 680
So, b = = 2.8 %
Since, the total expenditure on Thu = 200 + 240
Total expenditure on Wed = 250 + 150 + 180 =
+ 140 = ₹ 580
₹ 580
So, the required difference = 680 – 580 = ₹ 100
So, the required percentage = =
= ₹ (x – 150) = ₹ (y – 80)
2.32% = (a + b – 3.52)%

3) Answer: B
5) Answer: C
Total expenditure on food on all 4 days
The total expenditure on food for the remaining
together:
days of the month:
240 + 320 + 250 + 200 = ₹ 1010
7575 – (240 + 320 + 250 + 200) = ₹ 6565
Total other expenditures on all 4 days together:
The total expenditure on travelling for the
160 + 200 + 180 + 140 = ₹ 680
remaining days of the month:
So, m: n = 101: 68
6975 – (360 + 180 + 150 + 240) = ₹ 6045
From X:
Required ratio = 6565: 6045 = 1313: 1209 = 101:
Since, ‘n’ is divisible by 17 but ‘m’ is not divisible
93
by 17.

Click Here For Bundle PDF Course | support@guidely.in Page 9 of 16


SBI Clerk & RRB PO Mains PDF Course 2023
Quantitative Aptitude Day – 41 (Eng)

And the speed of train B = of 24 = 20 m/s


Direction (6-10): So,
Let the speeds of trains A, B, C, D and E are
12.5x m/s, 30x m/s, 17.5x m/s, 25x m/s and 15x So, the values in P can be filled in the blanks in
m/s respectively.
the same order.
Also let the lengths of trains A, B, C, D and E are From Q:
15y m, 20y m, 12.5y m, 30y m and 22.5y m If the speed of train A = 80% of 10 = 8 m/s
respectively. And the speed of train B = 50% of 24 = 12 m/s
Since, the time taken by train A to cross a 120 m So,
long tunnel is 15 seconds more than that taken
by train B to cross the same tunnel.
So, the values in Q can’t be filled in the blanks in
So,
the same order.
From R:
Since, the time taken by train D to cross a man If the speed of train A = 90% of 10 = 9 m/s
standing at a platform, is 18 seconds. And the speed of train B = 75% of 24 = 18 m/s
So, So,

y = 15x So, the values in R can’t be filled in the blanks in


From equation (1): the same order.
Hence, the values in P only can be filled in the
2700x + 1440 – 1500x – 600 = 2250x blanks in the same order.
x = 0.8, y = 12
7) Answer: E
Let the length of the tunnel be ‘d’ m.
So,

1890 + 7d – 900 – 6d = 1120


d = 130
6) Answer: A From P:
From P: Time taken by train A to cross the tunnel:
If the speed of train A = 120% of 10 = 12 m/s

Click Here For Bundle PDF Course | support@guidely.in Page 10 of 16


SBI Clerk & RRB PO Mains PDF Course 2023
Quantitative Aptitude Day – 41 (Eng)

So, P is not true. The time taken by train B to cross train D while
From Q: running in the same direction:
Since, the length of the tunnel is 130 m, which is
not divisible by 3. The time taken by train B to cross train D while
So, Q is not true. running in the opposite direction:
From R:
Average of the length of train E and that of the
Required ratio = 150: = 11: 1
tunnel:

10) Answer: A
So, R is not true.
The time, in which train B will cross train D:
Hence, none is true.

The time, in which train D will cross train C:


8) Answer: A
Let the length of train F be ‘f’ m.
Since, the speed of train B = 24 m/s Difference between M and N = 180 – 110 = 70

And the speed of train F = 62.5% of 24 = 15 m/s The difference between M and N is divisible by

So, only 7.

Direction (11-15):
f = 210 m
In 2000:
The length of train F as % of that of train A =
Let the total matches played = 100a
= 116.67%
Number of matches won = 25% of 100a = 25a
The length of train F as % of that of train B =
Number of matches drawn = 25a * (1/10) = 2.5a
= 87.5%
Since, the number of tied matches is 80% of
The length of train F as % of that of train C =
number of no result matches. Which means
= 140%
number of drawn matches will be 20% of number
The length of train F as % of that of train D =
of no result matches.
= 58.33%
Number of no result matches = 2.5a * (100/20) =
The length of train F as % of that of train E = 12.5a
= 77.78% Number of matches tied = 12.5a – 2.5a = 10a
So, the length of train F is 58.33% of that of train Number of matches lost = 50
D. According to the question:
100a = 25a + 50 + 12.5a
9) Answer: D

Click Here For Bundle PDF Course | support@guidely.in Page 11 of 16


SBI Clerk & RRB PO Mains PDF Course 2023
Quantitative Aptitude Day – 41 (Eng)

62.5a = 50 250c/9 = 25
a = 0.8 c = 0.9
In 2005: In 2015:
Let total matches played = 100b Let total matches played = 100d
Number of matches won = 75% of 100b = 75b Number of matches won = 60% of 100d = 60d
Number of matches drawn = 75b * (2/15) = 10b Number of matches drawn = 60d * (1/45) = 4d/3
Since, the number of tied matches is 40% of Since, number of tied matches is 80% of number
number of no result matches. Which means of no result matches. Which means number of
number of drawn matches will be 60% of number drawn matches will be 20% of number of no
of no result matches. result matches.
Number of no result matches = 10b * (100/60) = Number of no result matches = 4d/3 * (100/20) =
50b/3 20d/3
Number of matches tied = 50b/3 – 10b = 20b/3 Number of matches tied = 20d/3 – 4d/3 = 16d/3
Number of matches lost = 5 Number of matches lost = 25
According to the question: According to the question:
100b = 75b + 5 + 50b/3 100d = 60d + 25 + 20d/3
25b/3 = 5 100d/3 = 25
b = 0.6 d = 0.75
In 2010: In 2020:
Let total matches played = 100c Let total matches played = 100e
Number of matches won = 50% of 100c = 50c Number of matches won = 40% of 100e = 40e
Number of matches drawn = 50c * (8/45) = 80c/9 Number of matches drawn = 40e * (3/20) = 6e
Since, number of tied matches is 60% of number Since, the number of tied matches is 70% of the
of no result matches. Which means number of number of no result matches. this means the
drawn matches will be 40% of number of no number of drawn matches will be 30% of number
result matches. of no result matches.
Number of no result matches = (80c/9) * Number of no result matches = 6e * (100/30) =
(100/40) = 200c/9 20e
Number of matches tied = 200c/9 – 80c/9 = Number of matches tied = 20e – 6e = 14e
40c/3 Number of matches lost = 20
Number of matches lost = 25 According to the question:
According to the question: 100e = 40e + 20 + 20e
100c = 50c + 25 + 200c/9 40e = 20

Click Here For Bundle PDF Course | support@guidely.in Page 12 of 16


SBI Clerk & RRB PO Mains PDF Course 2023
Quantitative Aptitude Day – 41 (Eng)

e = 0.5 Case 4: When 12 – 0.25x = 3, x = 36 => Number


of matches in which team A batted first = x – 12
= 24
Case 5: When 12 – 0.25x = 2, x = 40 => Number
of matches in which team A batted first = x – 12
= 28
Required sum = 8 + 16 + 24 + 28
= 76

12) Answer: D
Total matches played by team M in 2005 = 60
11) Answer: B Total runs scored by team M in 2005 in all the
Total matches played by the team M in 2015 = matches = Total runs scored by four players P,
75 Q, R, and S = 80 * 60 + 60 * 60 + 75 * 60 + 90 *
Total matches played by the team M in 2015 60 = 4800 + 3600 + 4500 + 5400 = 18300
against team A = x% of 75 = 0.75x Total runs scored by team M in 2005 in all the
Total matches played by the team M in 2015 winning matches = 54% of 18300 = 9882
against team A in which team A batted first = (x – Total matches won by team M in 2005 = 45
12) Average runs/match scored by team M in 2005
Total matches played by the team M in 2015 in winning matches = 9882/45
against team A in which team M batted first = = 219.6
0.75x – (x – 12) = (12 – 0.25x)
(12 – 0.25x) is a prime number, then possible 13) Answer: E
values for (12 – 0.25x) are 11, 7, 5, 3, and 2 Total matches won by team M in 2020 = 20
Case 1: When 12 – 0.25x = 11, x = 4 => Number Total matches lost by team M in 2020 = 20
of matches in which team A batted first = x – 12 Total matches tied by team M in 2020 = 7
= -8 [Invalid] Total matches drawn by team M in 2020 = 3
Case 2: When 12 – 0.25x = 7, x = 20 => Number Total runs scored by team M in 2020 in all the
of matches in which team A batted first = x – 12 matches played = 325 * 20 + 275 * 20 + 250 * 7
=8 + 300 * 3 = 14650
Case 3: When 12 – 0.25x = 5, x = 28 => Number Total matches played by team M in 2020 = 50
of matches in which team A batted first = x – 12 Average of runs scored by team M in 2020 in all
= 16 the matches played = 14650/50 = 293

Click Here For Bundle PDF Course | support@guidely.in Page 13 of 16


SBI Clerk & RRB PO Mains PDF Course 2023
Quantitative Aptitude Day – 41 (Eng)

293 can be writes as 1 * 293. 81 = 1.8x


‘X’ is a prime number. x = 45 = 1 * 32 * 5
‘X + 2’ is 295 which can be written as 1 * 5 * 59, X + 5 = 50 = 1 * 2 * 52
which means ‘X + 2’ is a multiple of 59. HCF of X and X + 5 = 1 * 5 = 5
Closest perfect square to 293 is 289 which is a
perfect square of 17. 15) Answer: C
‘X – 3’ is 290 which can be written as 1 * 2 * 5 * Quantity I:
29, which means ‘X –3’ is a multiple of 29. Number of tied matches by team M in all the five
‘X’ lies between 292 and 294 and both are not a years = 8 + 4 + 12 + 4 + 7 = 35
prime number. Number of drawn matches by team M in all the
Hence, option (e) is incorrect. five years = 2 + 6 + 8 + 1 + 3 = 20
Difference = 35 – 20 = 15
14) Answer: A Quantity II:
Total matches played by team M in 2010 = 90 Number of lost matches by team M in all the five
Total matches played by team M in 2010 against years = 50 + 5 + 25 + 25 + 20 = 125
team P = = 16 According to the question:
Total matches played by team M in 2000 against x% of 125 = 102 = 100
team P = 16 * (3/4) = 12 x = 10000/125
Total matches played by team M in 2005 against x = 80
team P = 16 * (5/4) = 20 Quantity III:
Total matches played by team M in 2015 against Number of won matches by team M in all the five
team P = 16 * (6/4) = 24 years = 20 + 45 + 45 + 45 + 20 = 175
Total matches played by team M in 2020 against Number of lost matches by team M in all the five
team P = 16 * (2/4) = 8 years = 125
Total matches played by team M against team P According to the question:
in all the five years together = 16 + 12 + 20 + 24
+ 8 = 80 y% = 140%
Total matches played by team M against team P y = 140
in all the five years together in which team P won Quantity I = 15
= x% of 80 = 0.8x Quantity II = 80
Total matches played by team M against team P Quantity III = 140
in all the five years together in which team P lost Required sum = 15 + 80 + 140 = 235
= 80 – 0.8x = x – 1

Click Here For Bundle PDF Course | support@guidely.in Page 14 of 16


SBI Clerk & RRB PO Mains PDF Course 2023
Quantitative Aptitude Day – 41 (Eng)

16) Answer: B Marked price = 180% of 1500 = ₹ 2700


Let initial capital of A and B is ‘3x’ and ‘4x’ To get a profit percent of 44%, the selling price
respectively. should be equal to 144% of 1500 = ₹ 2160
Profit weightage of A (without salary) = 3x * 6 + Selling price when order of discounts is ABC =
(3x + 15000) * 6 = (36x + 90000) 90% of 2700 – 100 – 180 = ₹ 2150
Profit weightage of B = 4x * 12 = 48x Selling price when order of discounts is BAC =
Profit weightage of C = 60000 * 10 = 600000 90% of (2700 – 100) – 180 = ₹ 2160
Ratio of profit share of A, B, and C = (36x + Selling price when order of discounts is CAB =
90000): 48x: 600000 = (3x + 7500): 4x: 50000 90% of (2700 – 180) – 100 = ₹ 2168
Total annual profit from the business = ₹ 144000 Selling price when order of discounts is ACB =
Total salary given to A = 1800 * 12 = ₹ 21600 90% of 2700 – 180 – 100 = ₹ 2150
Remaining amount = 144000 – 21600 = Hence, correct order is BAC.
₹ 122400
Profit share of B = ₹ 38400 18) Answer: E
When only value of A is replaced:
Interest received after 3 years = =
₹ 7260
When only value of B is replaced:
51x = 28x + 230000 Interest received after 3 years = =
23x = 230000 ₹ 3630
x = 10000 When values of B and C are replaced:
Ratio of profit share of A, B, and C = (3x + Interest received after 6 years = =
7500): 4x: 50000 = 37500: 40000: 50000 = 15: ₹ 7260
16: 20 Hence, either value of A or values of both B and
Total profit amount received by A (including C should be replaced.
salary) = = ₹ 57600
Total profit amount received by C = = 19) Answer: A
₹ 48000 x2 – 48x + 432 = 0
Required difference = 57600 – 48000 x2 – 36x – 12x + 432 = 0
= ₹ 9600 x (x – 36) – 12 (x – 36) = 0
(x – 36) (x – 12) = 0
17) Answer: B x = 36 and 12
Cost price = ₹ 1500 Average age of A and B = 36 years

Click Here For Bundle PDF Course | support@guidely.in Page 15 of 16


SBI Clerk & RRB PO Mains PDF Course 2023
Quantitative Aptitude Day – 41 (Eng)

Sum of ages of A and B = 36 * 2 = 72 years (50% of 300 – 82 – √36) = (150 – 64– 6) = 150 –
Difference between ages of A and B = 12 years 70 = 80
Age of A = (72 + 12)/2 = 42 years
Age of B = (72 – 12)/2 = 30 years Let male and female employees in the company
Since HCF of ages of B and C is 15, then let the are 8x and 7x respectively.
ages of B and C is 15a and 15b respectively. [‘a’ Present male employees for the event = 90% of
and ‘b’ are co-prime numbers.] 8x = 7.2x
Given: 15a = 30 => a = 2 Present female employees for the event = 80%
Possible values of b = 1, 3, and 5 [Since, age (in of 7x = 5.6x
years) of C will be a 2-digit number.] Absent male employees for the event = 8x – 7.2x
Possible ages of C = 15 years, 45 years, and 75 = 0.8x
years respectively. Absent female employees for the event = 7x –
Also, it is given that the age of C is 4 less than 5.6x = 1.4x
the perfect square of an integer. Hence, only According to the question:
possible value of age of C = 45 years 1.4x – 0.8x = 30
Average age of all the 3 persons = (42 + 30 + 0.6x = 30
45)/3 = 117/3 = 39 years x = 50
Hence, Only statement I is correct. Total number of employees (male and female)
20) Answer: A present for the event = 7.2x + 5.6x = 12.8x = 640

Click Here For Bundle PDF Course | support@guidely.in Page 16 of 16


SBI Clerk & RRB PO Mains PDF Course 2023
ENGLISH Day - 41

English Language

Directions (01-08): Read the following passage Indian diaspora, whose leading lights have
and answer the questions that follow based on already prepared a series of programmes that
the information provided in the passage. will make the visit a big event. All this is a
Prime Minister Narendra Modi’s ongoing visit to reflection of the growing status of India, which
the United States is by far the most prestigious. has already overtaken Britain as the fifth largest
Though he has visited the US six times since he economy.
took over as prime minister in 2014, it is his first It is not for no reason that India and the US
state visit. That he is the third Indian leader after describe their relationship as a strategic
Dr S Radhakrishnan and Dr Manmohan Singh to partnership. An important aspect of his visit is
be honoured in this manner is quite revealing in likely to be military cooperation. The two sides
itself. Internationally, he is the third after the have been strengthening their defence ties over
French and South Korean presidents to be the past few years, with both engaging in joint
accorded a state visit by the Joe Biden military exercises and defence technology
administration. He will be the first Indian to cooperation. There is also a possibility that India
address a joint session of the US Congress for will be looking for better technology than what
the second time, an honour he shares with a few Russia can offer when it comes to defence
world leaders like Winston Churchill. He will be purchases. The two countries are also expected
taking yoga to the world stage — literally — to discuss regional security issues, particularly in
when he will lead the yoga session on the context of the present situation in
International Yoga Day in the UN premises. Afghanistan where India has invested heavily in
Modi will not only be meeting top officials of the infrastructural development. One notable feature
Biden administration, including the President, is that Kashmir, which used to be an irritant in
who hosts a gala dinner reception in his honour, India-US bilateral relations, is no longer on the
but also chief executives of a large number of US radar. In any case, the US had stopped worrying
companies. Agreements worth billions of dollars about Pakistan when it discussed relations with
are expected to be signed during the visit. In a India.
first of its kind, the US will be signing an One undeniable aspect of the strategic
agreement with India under which jet engines partnership is the threat from China, which has
would be manufactured in India. That the US has been bent upon strengthening its sphere of
agreed to partner with India in this area is a influence in the region, much to the dismay of
measure of the importance it attaches to bilateral neighbouring countries, including India. It
relations. Modi’s visit has already impacted the continues to create border problems either

Click Here For Bundle PDF Course | support@guidely.in Page 1 of 11


SBI Clerk & RRB PO Mains PDF Course 2023
ENGLISH Day - 41

through incursions or staking claims on Indian I. It is to boost bilateral ties between the US and
territories, for instance, Arunachal Pradesh. It is India.
in this context that India is involved in the II. US invited India to participate in the strategic
Quadrilateral Security Dialogue (QSD), cooperation at the QUAD meeting.
commonly known as the Quad, a strategic III. As PM will lead the yoga session on
security dialogue among Australia, India, Japan International Yoga Day in the UN premises.
and the US that is maintained by talks among a) Only III.
member countries. Meanwhile, many US b) Both II and III.
companies, including iPhone manufacturer c) Both I and III.
Apple, have been shifting their manufacturing d) All I, II and III.
base from China to India, buoyed by the ease of e) None of these.
doing business in India.
However, it would be a negation of reality to 2. Which of the following statement/statements
claim that there are no irritants in bilateral can be stated as TRUE to the context of the
relations. The US never misses an opportunity to passage?
preach democracy to the rest of the world. Its I. Modi has planned to meet the president of
commission on international religious freedom Biden Administration.
has been submitting reports that do not show II. All this is a reflection to grow the status of
that religious freedom in India has been on an India which has overtaken the Britain as the fifth
even keel. There have been changes that do not largest economy.
show the country in a good light. What is III. The two sides have been strengthening their
happening in Manipur, where thousands of defence ties over the past decade for joint
houses and hundreds of churches were military exercises.
destroyed and tens of thousands of people were a) Both I and II.
pushed to the refugee camps, is a matter of b) Both II and III.
shame. True, it is an internal matter. While Uncle c) Both I and III.
Sam has no right to question India on this count, d) All I, II and III.
the need to address fears of democratic values e) None of these.
losing steam in India cannot be overemphasised.
This should not, however, detract from the 3. Which of the following statement/statements
importance of Modi’s visit. can be stated as FALSE to the context of the
1. According to the passage, why the recent visit passage?
of PM to US is considered as remarkable?

Click Here For Bundle PDF Course | support@guidely.in Page 2 of 11


SBI Clerk & RRB PO Mains PDF Course 2023
ENGLISH Day - 41

I. China has invested heavily in infrastructural I. China creates border problems causing
development so it claims Arunachal Pradesh and dismaying concern for all its neighbouring
creates border disputes. countries.
II. Kashmir is the irritant for India-US bilateral II. Strategic partnership between India and US
relations. was marred due to the claims of China in
III. US never misses an opportunity to preach Arunachal Pradesh.
democracy to the country. III. As China is helping Russia for the betterment
a) Only I. of the respective countries.
b) Only II. a) Only I.
c) Only III. b) Only II.
d) Both I and II. c) Only III.
e) Both I and III. d) Both I and II.
e) Both I and III.
4. According to the given passage, which of the
following agreement has been signed during 6. Find the suitable title for the passage.
PM’s visit? a) How PM’s US visit has helped India?
I. The agreement signed is related to b) PM’s US visit and its aftermath.
technological cooperation, as India is expecting c) PM’s US visit and what does it reflect.
better technology than the one which is currently d) India’s economy inclination after PM’s visit to
providing by Russia. US.
II. An agreement is signed for the procurement of e) None of these.
Jet engines for US that would be manufactured
by India for the very first time. 7. Choose the word that is most similar to the
III. An agreement has been signed between the given word “buoyed” in the passage.
countries to celebrate yoga as a global event on a) Thronged
the Yoga day. b) Anticipated
a) Only I. c) Bolstered
b) Only II. d) Augmented
c) Only III. e) None of these.
d) Both I and II.
e) Both I and III. 8. Choose the word that is most opposite to the
given word “detract” in the passage.
5. Why do US companies have shifted their a) Significant
production base from China to India? b) Boycotted

Click Here For Bundle PDF Course | support@guidely.in Page 3 of 11


SBI Clerk & RRB PO Mains PDF Course 2023
ENGLISH Day - 41

c) Stringent 11. Sides and a serious commitment to work


d) Imperative together (A)/ the task at hand is not easy and
e) None of these. requires champions on both (B)/ through the
defence innovation bridge, the two countries (C)/
Directions (9-11): In the following questions, a are making a serious attempt to de-centralise
sentence is divided into four parts and arranged innovation (D)/.
in a jumbled manner. You are required to a) ACDB
rearrange the sentences in a proper manner to b) CDBA
make the sentence meaningfully correct. c) DBCA
9. Seem to be running the drug syndicate and d) BADC
that’s (A)/ camped for few days in Imphal and e) No rearrangement required.
tried to explore a militaristic solution to the crisis
(B)/ the reason the central and state Directions (12-16): In the following questions,
governments have been unable to stem (C)/ two columns are given with three sentences
sadly, many from the political fraternity cutting each. You are required to match the column I
across party lines (D)/. with column II, if any of the given sentences
a) CABD provide contextual meaning mark that as your
b) BDAC answer.
c) ACBD 12.
d) DACB COLUMN I COLUMN II
e) No rearrangement required.

10. The options market is not as liquid as spot A. Leaders set aside D. the opposition is
their ideological prepared to challenge
USD-INR and option prices are not (A)/ readily
differences and focused the ruling party's
available to most corporates, banks often build
dominance.
(B)/ when considering options, it is important to
recognise that as (C)/ in unconscionable charges B. The unity E. in the shared goal

onto the spreads they charge for options (D)/. demonstrated in Patna of strengthening the

a) DBCA sends a clear message democratic

b) BADC to the electorate framework.

c) CADB C. The meeting of F. towards forging a

d) ADCB opposition parties in stronger alliance,

e) No rearrangement required. Patna marks a crucial driven by the objective


step of providing an

Click Here For Bundle PDF Course | support@guidely.in Page 4 of 11


SBI Clerk & RRB PO Mains PDF Course 2023
ENGLISH Day - 41

alternative vision for recovery procedures after creating dummy or


the country. shell companies for the
loan
a) Only A-E. B. To show multiple E. the same banks
b) Only B-D. entries, the loan became paralyzed and
c) Only C-F. amount was learn about these NPAs
d) Both A-E and C-F. transferred much later.
e) None of these.
C. However, when it F. other than keeping
comes to business large corporate bad
13.
owners or loans from being written
COLUMN I COLUMN II industrialists, off.
a) Only A-F.
A. Enzymes and D. as well as
b) Only B-D.
chemicals secreted by microbes present in
c) Only C-E.
the liver, pancreas, the gut are
d) All of these.
intestine, etc. responsible for quality
e) None of these.
of digestion.
B. Everyone wants to E. and quality food for
15.
consume a well- better immunity and
Column I Column II
balanced diet endurance.
C. As a result we are F. in the ingested a) The global d) adapt to changing
neither benefited by the food, or are the level community is actively climate patterns has
nutrients of immunity improved. addressing various gained significant
a) Only A-D. traction worldwide.
b) Only B-E. b) The urgency to e) pressing climate
c) Only C-F. transition to clean challenges to secure a
d) Both A-D and B-E. energy sources, reduce sustainable future.
e) All of these. greenhouse gas
emissions, and
14.
COLUMN I COLUMN II c) Several countries f) are investing in
have set ambitious large-scale renewable
A. The RBI should D. to one shell renewable energy projects, such as solar
strengthen its loan company to another targets and and wind farms.

Click Here For Bundle PDF Course | support@guidely.in Page 5 of 11


SBI Clerk & RRB PO Mains PDF Course 2023
ENGLISH Day - 41

a) Only A-F and B-D make the sentence contextually correct and
b) A-D, B-E, C-F meaningful.
c) A-D, B-F, C-E 17. The average level of confidence in every tech
d) A-E, B-D, C-F company had dropped ________________ by
e) None of these 2021. In fact, technology companies have
experienced the steepest drops over the past
16. five years when compared to all other
Column I Column II institutions.
a) Skeptically
a) The need of the hour d) infrastructure b) Sporadically
is to review our development does not c) Precipitously
business-as-usual come at the cost of the d) Culpably
development models natural environment. e) None of these.
and build and redesign
our 18. The focus is to educate or inform the public,
b) While urbanisation is e) our cities are better particularly in the provinces, because scammers
inevitable and able to handle extreme often victimize the gullible who are easy to
accelerates economic climate events and our deceive, or even those desperate for cash. They
development, it is economic growth is should _______________ in the minds of
crucial that sustainable. everyone the provision in the SIM registration
c) Our collective f) urban infrastructure law that it is illegal to sell one’s SIM card.
wisdom and to bolster our natural a) Reproach
technological prowess resources, instead of b) Imbibe
are sure to enable us in reclaiming them. c) Pursue
ensuring that d) Instill
a) A-E, B-D, C-F e) None of these.
b) Only B-F and A-E
c) A-F, B-D, C-E 19. More worryingly, one FIR noted that it was
d) Only B-D the police constable of the local station who
e) None of these broke the lock of the armoury and helped the
mob get access to firearms, highlighting again
Directions (17-20): In the following questions, a the disturbing ______________ between rioters
sentence is given with a blank. You are required and some elements of the police machinery that
to choose the appropriate word for the blank to

Click Here For Bundle PDF Course | support@guidely.in Page 6 of 11


SBI Clerk & RRB PO Mains PDF Course 2023
ENGLISH Day - 41

has been alleged by observers and civil society Chinese forces and systems. Growing linkages
groups. between our two defence ecosystems and cost-
a) Nexus efficient platforms will also accelerate India’s
b) Debris _________________ away from Russian platform
c) Slouch systems.
d) Consent a) Scattering
e) None of these. b) Weaning
c) Scolded
20. India not only has the geographical reach but d) Withering
also active experience in defending against e) None of these.

Click Here to Get the Detailed Video Solution for the above given Questions
Or Scan the QR Code to Get the Detailed Video Solutions

Answer Key with Explanation

1. Answer: E 2. Answer: E
According to the given passage, this visit is PM's Statement I is incorrect, Modi has planned to
first state visit. Only two PMs were visited since meet top officials of the Biden administration
independence, now Narendra Modi joins in the including president, it doesn't mean presidents
list. Also, he is the third after the French and are only considered as top officials, there would
South Korean presidents during the Biden be several delegates in the adminstration.
administration. He is the first Indian to address a Statement II is incorrect, according to the given
joint session of the US Congress in a row. This passage, the visit to the US and signed
makes the visit more remarkable, none of the agreements all describes that India's economy is
statements have been given in the options. growing, not to grow India’s status from these
Therefore, option E is the correct answer. things.

Click Here For Bundle PDF Course | support@guidely.in Page 7 of 11


SBI Clerk & RRB PO Mains PDF Course 2023
ENGLISH Day - 41

Statement III is incorrect, the two sides have would be better. That's the reason behind US
been strengthening their defence ties over the shifting the companies to India from China.
recent years, not the decade. Therefore, option A is the correct answer.
Therefore, all the statements are incorrect,
option E is the correct answer. 6. Answer: C
The passage is in a positive tone, the only
3. Answer: E negative thing that is present in the paragraph is
Statement I is incorrect, according to the given about China. According to the given passage,
passage, India has invested heavily in we can infer that US has signed an agreement
infrastructural development in Afghanistan, not with India, Yoga session has been conducted on
China, it contradicts the given passage. US premises and India has overtaken Britain it
Statement III is incorrect, according to the all indicates that India or India’s status is
passage, US never misses an opportunity to growing. Option A is incorrect, the passage did
preach (talk on a religious subject) democracy to not mention that PM’s US visit has helped India
the WORLD, not to the country. in any manner. Option B is incorrect, because of
Therefore, option E is the correct answer. the word “aftermath” (consequences). Option D
is contradicted to the given passage.
4. Answer: B Therefore, option C is the correct answer.
Statement I is incorrect, because the passage
itself says in a sceptical manner that India may 7. Answer: C
plan to achieve better technological value. Buoyed – to support something or make it more
Only statement II is correct, this can be inferred successful.
from the following lines of the given passage, "In Bolstered – support or strengthen.
a first of its kind, the US will be signing an Thronged – to be in large numbers.
agreement with India under which jet engines Augmented – make stronger or bigger.
would be manufactured in India". Therefore, we The word "bolstered" is the synonym of the given
can infer that only option B is the correct answer. highlighted word "buoyed" in the passage.
Therefore, option C is the correct answer.
5. Answer: A
Only statement I is correct, China is creating 8. Answer: A
many problems across the borders and claims Detract – diminish the importance.
the states that is causing worry to the Significant – Extremely great; important.
neighbouring countries. In this situation, nothing Boycotted – withdraw; refuse.

Click Here For Bundle PDF Course | support@guidely.in Page 8 of 11


SBI Clerk & RRB PO Mains PDF Course 2023
ENGLISH Day - 41

The word "significant" is the antonym of the liquid as spot USD-INR and option prices are not
given highlighted word "detract" in the passage. readily available to most corporates, banks often
Therefore, option A is the correct answer. build in unconscionable charges onto the
spreads they charge for options.
9. Answer: D Therefore, option C is the correct answer.
Here, the word "seem" is used, which is a plural
verb so a plural noun should precede it, so we 11. Answer: B
cannot use "stem" as it is a verb, and "crisis" Here, the word "both" should be followed by
cannot be used as it is a singular noun. So, we sides, so we can infer that B will be followed by
can infer that part A will come after D. If B is A. In part D, "are" is used so a plural noun
following A, the phrase "to the crisis the reason" should be used before it. We can infer that
would be contextually incorrect. So, part C and B "countries" should be placed before it. So, C
should be followed by A respectively. Therefore, should be followed by D. To form a contextually
option D is the correct answer. meaningful sentence, CDBA is the correct
The correct sentence after rearrangement is, sequence.
Sadly, many from the political fraternity cutting The correct sentence after rearrangement is,
across party lines seem to be running the drug Through the defence innovation bridge, the two
syndicate and that’s the reason the central and countries are making a serious attempt to de-
state governments have been unable to stem centralise innovation. The task at hand is not
camped for few days in Imphal and tried to easy and requires champions on both sides and
explore a militaristic solution to the crisis. a serious commitment to work together.
Therefore, option D is the correct answer. Therefore, option B is the correct answer.

10. Answer: C 12. Answer: C


The only part that will help to begin the sentence Option A is incorrect, the preposition “in” is
is C, and the word "as" will take either adjective incorrect here. Generally, focus should be used
or noun after it. According to this logic, we can with the preposition “on”.
infer only part A will come after C. The phrase Option B is incorrect, the phrase “electorate the”
"bank often build in unconscionable charges" is is incorrect here.
contextually incorrect. So, D will take B after it. The correct sentences are,
The correct sentence after rearrangement is, A-E: Leaders set aside their ideological
When considering options, it is important to differences and focused on the shared goal of
recognise that as the options market is not as strengthening the democratic framework.

Click Here For Bundle PDF Course | support@guidely.in Page 9 of 11


SBI Clerk & RRB PO Mains PDF Course 2023
ENGLISH Day - 41

C-F: The meeting of opposition parties in Patna A-F: The RBI should strengthen its loan recovery
marks a crucial step towards forging a stronger procedures rather than keeping large corporate
alliance, driven by the objective of providing an bad loans from being written off.
alternative vision for the country. B-D: To show multiple entries, the loan amount
B-D: The unity demonstrated in Patna sends a was transferred from one shell-company to
clear message to the electorate that the another after creating dummy or shell companies
opposition is prepared to challenge the ruling for the loan.
party's dominance. C-E: However, when it comes to business
Therefore, option C is the correct answer. owners or industrialists, the same banks become
paralyzed and learn about these NPAs much
13. Answer: D later.
Option C is incorrect, usually “neither” will take Therefore, option E is the correct answer.
“nor” after it, only “either” will take “or”. Thus
makes it incorrect. 15. Answer: D
The correct sentences are, 1. The global community is actively addressing
C-F: As a result we are neither benefited by the various pressing climate challenges to secure a
nutrients present in the ingested food, nor are sustainable future.
the level of immunity improved. 2. The urgency to transition to clean energy
B-E: Everyone wants to consume a well- sources, reduce greenhouse gas emissions, and
balanced diet and quality food for better adapt to changing climate patterns has gained
immunity and endurance. significant traction worldwide.
A-D: Enzymes and chemicals secreted by the 3. Several countries have set ambitious
liver, pancreas, intestine, etc. as well as renewable energy targets and are investing in
microbes present in the gut are responsible for large-scale renewable projects, such as solar
quality of digestion. and wind farms.
Therefore, option D is the correct answer.
16. Answer: C
14. Answer: E 1. The need of the hour is to review our
Here, all the sentences in column I can be business-as-usual development models and
matched with the column II, but it is build and redesign our urban infrastructure to
grammatically incorrect. bolster our natural resources, instead of
Rather than – instead of; reclaiming them.
The correct sentences are,

Click Here For Bundle PDF Course | support@guidely.in Page 10 of 11


SBI Clerk & RRB PO Mains PDF Course 2023
ENGLISH Day - 41

2. While urbanisation is inevitable and deceiving. So, they should keep in mind that
accelerates economic development, it is crucial selling SIM card of someone is illegal. For that,
that infrastructure development does not come the word "instill" should be used. Therefore,
at the cost of the natural environment. option D is the correct answer.
3. Our collective wisdom and technological
prowess are sure to enable us in ensuring that 19. Answer: A
our cities are better able to handle extreme Nexus – a connection or series of connections.
climate events and our economic growth is Debris – scattered pieces of rubbish.
sustainable. Slouch – stand, move in a drooping way.
The statement expresses that a constable is
17. Answer: C behind the access to firearms, it indicates that he
Precipitously – very steeply. has a connection with the thief. For that, the
Skeptically – with doubt or hesitation. word "nexus" should be used. Therefore, option
Sporadically – occasionally; irregular intervals. A should be used.
Culpably – deserves to be blamed.
The statement indicates that companies have 20. Answer: B
dropped over the past five years. The word Weaning away – make someone stop doing
before the blank is "dropped" a verb, to something.
emphasize this further an adverb should be Withering – intended to make someone feel
used. So "precipitously" should be used. humiliated.
Therefore, option C is the correct answer. Here, the statement says that the relationship
between two countries are growing in a good
18. Answer: D manner. This relationship will help to stop
Instill – gradually but firmly establish. depending on Russia. For that, the word
Reproach – express ones disapproval. “weaning away” should be used. Therefore,
Imbibe – absorb; assimilate. option B is the correct answer.
The statement expresses that the focus should
be on people who are easily vulnerable for

Click Here For Bundle PDF Course | support@guidely.in Page 11 of 11


SBI Clerk & RRB PO Mains PDF Course 2023
Reasoning Ability Day - 42 (Eng)

Reasoning Ability
Directions (1-5): Answer the questions based on are arranged in such a way to form a third
the information given below. meaningful word.
A nine-letter word (may or may not be 1) Which of the following letter is placed
meaningful) with no repeated letters is arranged immediate left of W in arrangement 2?
in three arrangements to form the series. The a) K
same letters are used in each of the b) The letter which is second to the right of F
arrangements. c) No letter
Arrangement 1: d) E
R is placed fourth to the left of W but none of the e) A
letters is placed at the end. Only two letters are
placed between R and I. The number of letters 2) What will be the first three letters meaningful
placed to the left of I is the same as to the right of word formed?
F.E is placed third to the left of T.T is placed a) RAT
adjacent to neither W nor R. As many letters b) ART
placed between F and T as between W and D.A c) TRA
is placed to the left of K.The number of letters d) TAR
placed between A and R is one more than the e) None of the above
number of letters placed between K and D.
Arrangement 2: 3) What is the position of I from the right end in
All the given letters from arrangement 1 are Arrangement 2?
arranged in alphabetical order from right to left. a) Fourth
Thus, forms the new arrangement of the series. b) Fifth
Arrangement 3: c) Second
The first letter and third letter from the left end in d) Third
arrangement 1 are taken along with the exact e) None of these
middle letter in arrangement 1 to form a first
meaningful word. R is placed to the right of A. 4) Which of the following combination of the
The exact middle letter of arrangement 1 is not letters are arranged adjacent to each other in
the first letter of any meaningful words. The arrangement 3?
fourth letter from the left end in arrangement 1 I. WK
and the third letter from the right end of both II. TF
arrangements 1 and 2 are taken to form a III. AT
second meaningful word. The remaining letters a) Only I

Click Here For Bundle PDF Course | support@guidely.in Page 1 of 10


SBI Clerk & RRB PO Mains PDF Course 2023
Reasoning Ability Day - 42 (Eng)

b) Only III immediately before S. R is not married to Q. At


c) Both I and II least two persons attend the event between S’s
d) All I, II, and III daughter and Q.
e) None of these 6) How V is related to P?
a) Mother-in-law
5) How many letters are there between T and K b) Son
in Arrangement 3? c) Brother
a) Four d) Niece
b) Same as between E and I in Arrangement 1 e) None of these
c) More than Five
d) Same as between R and F in Arrangement 2 7) Who among the following persons attend the
e) Five event between V and S’s son?
I. The one who attends the event on Friday
Directions (6-10) Study the following information II. The one who attends the event three persons
carefully and answer the below questions. after S’s husband
Seven persons from a three generations family – III. S
P, Q, R, S, T, U, and V attend the event during a a) Only II
week from Monday to Sunday. Two married b) Both I and III
couples are in the family. No single parent has a c) Both II and III
child. d) All I, II, and III
T’s daughter-in-law attends three persons after e) None
T. T neither attends on Monday nor Wednesday.
U is the only son of P’s brother. The number of 8) Who among the following person attends the
persons attending before U is one more than the event on Tuesday?
number of persons attending after V. Both U and a) R
V attend the event on neither Tuesday nor b) U’s grandfather
Thursday. S is the sister-in-law of P and vice- c) S
versa. Only two persons attend the events d) Q
between P and Q, who is the father of V. The e) None of these
number of persons attending the event between
U and Q is the same as the number of persons 9) Four of the following five are alike in a certain
attending the event between V and P. R, who is way as per the given arrangement and hence
the mother of P and attends the event three form a group. Find the one that doesn’t belong to
persons after the one who attends the event that group.

Click Here For Bundle PDF Course | support@guidely.in Page 2 of 10


SBI Clerk & RRB PO Mains PDF Course 2023
Reasoning Ability Day - 42 (Eng)

a) The one who attends the event on Monday Q, who sits to the right of P. Neither P nor S is
b) R facing F. D sits second to the right of B but none
c) The one who attends the event two persons of them neither sits adjacent chair of C nor at the
before R end. S sits third to the left of U. Only two persons
d) V sit between P and T. E sits adjacent to the one
e) The one who attends the event on Thursday who is facing T.
11) What is the position of D with respect to the
10) Which of the following information is not one who sits opposite to U?
true? a) Third to the left
a) P is the sister of the one who attends the b) Fourth to the right
event on Monday c) Immediate left
b) More than three persons attend the event d) Second to the right
before U’s sister e) None of these
c) R attends the event immediately after P’s
daughter 12) Four of the following five are alike in a certain
d) S attends the event on Friday way as per the given arrangement and thus form
e) All the above statements are true a group. Find the one that does not belong to
that group.
Directions (11-15) Study the following a) The one who sits immediate right of D
information carefully and answer the below b) C
questions. c) The one who sits third to the right of U
Fourteen chairs are placed in two parallel rows in d) S
such a way that in row 1 – persons A, B, C, D, E, e) The one who sits opposite to R
and F are sitting and all of them face south
whereas, in row2 – persons P, Q, R, S, T, and U 13) How many persons are sitting between F and
are sitting and all of them face north. Row 1 is E?
north of row 2. One chair in each row is vacant. a) Three
Neither the chairs at the end nor the chairs facing b) As many persons sitting to the left of U
each other are vacant. c) As many persons sitting between S and P
F sits third from the extreme right end. Only one d) One
person sits between F and C. The one who is e) Either B or C
facing C sits immediate right of Q. Only three
persons sit between P and R, and either of them 14) Who among the following person sits
sits at the end. Neither P nor R sits adjacent to adjacent chair of C?

Click Here For Bundle PDF Course | support@guidely.in Page 3 of 10


SBI Clerk & RRB PO Mains PDF Course 2023
Reasoning Ability Day - 42 (Eng)

a) Only B neither holds the first rank nor ranks before X. Y


b) Both A and E holds the rank after A but before E. Z holds the
c) Only E rank after B and is four ranks after C. Y is not
d) Both A and F secured the first rank.
e) None of these 16) In which direction is C standing with respect to
A?
15) Which of the following statement is not true? a) North-east
a) One person sits between B and E b) South-east
b) D is facing the one who sits immediate right of c) North-west
S d) South-west
c) T is facing C e) Cannot be determined
d) Two persons are sitting between Q and S
e) All the above statements are true 17) How many persons rank after E in the final
arrangement?
Directions (16-20): Study the following information a) 1
carefully and answer the questions given below. b) 2
Nine students viz., A, B, C, D, E, W, X, Y and Z c) 3
have cleared the class test and are ranked from 1 d) 4
to 9. They are standing in the ground at a certain e) None
distance. No two persons secured the same rank.
Note: The distance between any two students is 18) What is the approximate shortest distance
the twice of the lowest rank among them. For between D and A and who ranks according to that
example, if P is the first rank and Q is the second value?
rank, the distance between them is 4m. a) 5, Y
X is to the west of A and is the immediate previous b) 7, E
rank of D who is to the north of X. The number of c) 5, B
persons ranked between D and W is an odd prime d) 7, B
number where no one ranks after W who is to the e) 5, E
east of D. E ranks exactly between D and W and
is to the south of Y. C stands to the south of W 19) Find the odd one out according to their
and ranks before E. Z stands to the west of C and standing position in the ground?
ranks after Y. The direction of Y from E is same as a) D-Y
the direction of B from Z. B holds the immediate b) X-B
next rank to E and stands to the west of E. C c) A-E

Click Here For Bundle PDF Course | support@guidely.in Page 4 of 10


SBI Clerk & RRB PO Mains PDF Course 2023
Reasoning Ability Day - 42 (Eng)

d) Z-W a) A,B,C,E
e) C-D b) X,A,C,W
c) A,Y,E,B
20) Who among the following group of persons d) Z,W,A,B
hold the prime numbered ranks? e) X,D,Y,B

Click Here to Get the Detailed Video Solution for the above given Questions
Or Scan the QR Code to Get the Detailed Video Solutions

Answer Key with Explanation

Directions (1-5): From the above condition, there are four


1) Answer: C possibilities.
2) Answer: B
3) Answer: B
4) Answer: C
5) Answer: B
Final Arrangement: Again we have,
 The number of letters placed to the left of
I is the same as to the right of F.
 E is placed third to the left of T.
 T is placed adjacent to neither W nor R.

We have, From the above condition, Case-1 and case 3

 R is placed fourth to the left of W but none get eliminated.

of the letters is placed at the end.


 Only two letters are placed between R
and I.

Again we have,

Click Here For Bundle PDF Course | support@guidely.in Page 5 of 10


SBI Clerk & RRB PO Mains PDF Course 2023
Reasoning Ability Day - 42 (Eng)

 As many letters placed between F and T


as between W and D.
Again we have,
 A is placed to the left of K.
 The fourth letter from the left end in
 The number of letters placed between A
arrangement 1 and the third letter from
and R is one more than the number of
the right end of both arrangements 1 and
letters placed between K and D.
2 are taken to form a second meaningful
From the above condition, Case-4 get
word.
eliminated. Case-2 shows the final arrangement.

Again we have,
 The remaining letters are arranged in
such a way to form a third meaningful
word.
Arrangement 2:
Again we have,
 All the given letters from arrangement 1
Therefore the final arrangements of the given
are arranged in alphabetical order from
series are as follows.
right to left. Thus, forms the new
arrangement of the series.

Arrangement 3:
Again we have, Directions (6-10):

 The first letter and third letter from the left 6) Answer: D

end in arrangement 1 are taken along 7) Answer: D

with the exact middle letter in 8) Answer: B

arrangement 1 to form a first meaningful 9) Answer: A(All the persons are of the same

word. gender except option A)

 R is placed to the right of A. 10) Answer: C

 The exact middle letter of arrangement 1


is not the first letter of any meaningful
words.

Click Here For Bundle PDF Course | support@guidely.in Page 6 of 10


SBI Clerk & RRB PO Mains PDF Course 2023
Reasoning Ability Day - 42 (Eng)

We have:
 T’s daughter-in-law attends three persons
after T. Again, we have:
 T neither attends on Monday nor  S is the sister-in-law of P and vice-versa.
Wednesday. That means, S must be the mother of U.
That means, in case (1) T visited on  Only two persons attend the event
Tuesday, in case (2) T visited on between P and Q, who is the father of V.
Thursday.  The number of persons attending the
 U is the only son of P’s brother. event between U and Q is the same as
 The number of persons attending before the number of persons attending the
U is one more than the number of event between V and P.
persons attending after V.  R, who is the mother of P and attends the
 Both U and V attend the event on neither event three persons after the one who
Tuesday nor Thursday. attends the event immediately before S.
That means, in case (1) & case (2) U  R is not married to Q.
attends the event on Wednesday, in case Since, only two married couples are in the
(1a) & case (2a) U attended the event on family, and Q is not married to R, thus Q
Saturday. must be married to S, and T must be
Based on the above given information we have: married to R.
That means, in case (1) Q attends the
event on Monday, in case (1a) Q attends
the event on Thursday, case (2) & case
(2a) are not valid.
Now, since, U is the only son of P’s
brother, and Q is the father of V, thus, V
must be the sister of U.

Click Here For Bundle PDF Course | support@guidely.in Page 7 of 10


SBI Clerk & RRB PO Mains PDF Course 2023
Reasoning Ability Day - 42 (Eng)

 At least two persons attend the event


between S’s daughter and Q.
That means, case (1a) is not valid.
Based on the above given information we have:

We have:
 F sits third from the extreme right end.
 Only one person sits between F and C.

Case (2) & case (2a) are not valid as R attends  The one who is facing C sits immediate

the event two persons after S, case (1a) At least right of Q.

two persons attend the event between V and Q. That means, in case (1) C sits third from

For Blood Relation: the left end of row1, in case (2) C sits
second from the left end of row1.
Based on the above given information we have:

Direction (11-15):
11) Answer: D Again, we have:
12) Answer: B  Only three persons sit between P and R,
13) Answer: B and either of them sits at the end.
14) Answer: C  Neither P nor R sits adjacent to Q, who
15) Answer: E sits to the right of P.
 Neither P nor S is facing F.
That means, in case (1) P sits at the left
end of row2, in case (1a) & case (2) P sits
second from the left end of row 2.
Based on the above given information we have:

Click Here For Bundle PDF Course | support@guidely.in Page 8 of 10


SBI Clerk & RRB PO Mains PDF Course 2023
Reasoning Ability Day - 42 (Eng)

Case (1) & case (1a) are not valid as D sits


second to the right of B.
Again, we have:
 S sits third to the left of U.
Since, S doesn’t sit facing F.
That means, S sits at the left end of row2.
 Only two persons sit between P and T.
 E sits adjacent to the one who is facing T.
That means, E sits immediate left of C.
Based on the above given information we have:

Again, we have:
 D sits second to the right of B but none of
them neither sits adjacent chair of C nor
at the end.
That means, in case (2) D sits second
from the right end of row1, case (1) &
case (1a) are not valid. Directions (16-20):
Based on the above given information we have: 16) Answer: B
17) Answer: C
18) Answer: D
19) Answer: E
20) Answer: E
Following the statements given above, the rank,
distance and the directions of each student are
obtained as follows.

Click Here For Bundle PDF Course | support@guidely.in Page 9 of 10


SBI Clerk & RRB PO Mains PDF Course 2023
Reasoning Ability Day - 42 (Eng)

Click Here For Bundle PDF Course | support@guidely.in Page 10 of 10


SBI Clerk & RRB PO Mains PDF Course 2023
Quantitative Aptitude Day – 42 (Eng)

Quantitative Aptitude

Directions (01 - 05): Study the following information carefully and answer the questions given below.
The below graphs show the number of cars sold in five months and the ratio of the number of diesel and
petrol cars sold in those five months.

1) The number of diesel cars sold in June is 3) I= The difference between petrol and diesel
increased by 10% and the number of petrol cars cars sold in May.
sold in June is increased by 25% than sold to J= The difference between petrol and diesel cars
those same in May respectively. Find the total sold in March.
number of cars sold in June? K= The difference between petrol and diesel cars
a) 110 sold in February.
b) 120 Find the value of I+J-K=?
c) 108 a) 57
d) 130 b) 54
e) 160 c) 52
d) 56
2) M= Increase in percentage of petrol cars sold e) None of these
in March from February.
N= Increase in the percentage of diesel cars sold 4) Find the difference between the total number
in March from February. of petrol cars sold in March and April together
Find the value of the difference between the and the total number of diesel cars sold in
values of M and N? February and May together?
a) 14.24% a) 8
b) 16.35% b) 4
c) 18.32% c) 3
d) 12.58% d) 2
e) 19.68% e) None of these

Click Here For Bundle PDF Course | support@guidely.in Page 1 of 8


SBI Clerk & RRB PO Mains PDF Course 2023
Quantitative Aptitude Day - 42 (Eng)

5) If the number of petrol cars sold in five months b) 7.56 hours


represented in the pie chart, then find the c) 9.45 hours
approximate central angle formed by the number d) 8.35 hours
of petrol sold in March? e) 8.20 hours
a) 770
b) 870 7) The speed of car E in the daytime is 20%
c) 880 more than the speed of car A in the daytime and
d) 850 the speed of car E at nighttime is 25% more than
e) None of these the speed of car C in the nighttime. Find the
difference between distance traveled by car B
Directions (06 - 10): Study the following and car E when both travel 5 hours in the
information carefully and answer the questions daytime and 3 hours at nighttime?
given below. a) 75 km
The ratio of the speed of car A in daytime and b) 72 km
nighttime is 4:3. The speed of car B in daytime is c) 78 km
33.33% more than the speed of car B in the d) 70 km
nighttime. The speed of car C at nighttime is e) 68 km
20% less than the speed of car C in the daytime.
Car A covers 180 km in 3 hours in the daytime. 8) I= The distance covered by car A in 3 hours
Car A covers a certain distance in 4 hours at daytime,
nighttime is the same distance covered by car B J= The distance covered by car B in 4 hours a
in 3 hours at nighttime. The distance of car A daytime.
covers in 5 hours in the daytimeis the same Find the time taken by car C to cover [I+J]/2 km
distance covered by car C in the daytime in 6 distance at nighttime?
hours. a) 5.25 hours
6) The ratio of the speed of car B and Car D in b) 6.25 hours
the daytime is 5:6, and the ratio of the speed of c) 8.25 hours
car A and Car D in the nighttime is 5:6. Car D d) 4.25 hours
covers M km distance by traveling 4 hours at e) None of these
nighttime and 6 hours in the daytime then find in
how many hours car A should travel to cover M 9) Car B starts his journey from point x in the
km distance by daytime if car A already traveled daytime, after t1 hour he reached point y and
5 hours at nighttime? after another 2 hours, he reached point z. when
a) 8.25 hours car B return from z, the car covers distance from

Click Here For Bundle PDF Course | support@guidely.in Page 2 of 8


SBI Clerk & RRB PO Mains PDF Course 2023
Quantitative Aptitude Day - 42 (Eng)

z to y in day time but car B covers the y to x 10) Find the difference between the distance
distance at nighttime. Find the value of t1 if the travelled by car A and B in 4 hours day time and
total time to return is 6 hours? by car B and car C in 6 hours at nighttime?
a) 3 hours a) 40
b) 2.5 hours b) 42
c) 1.5 hours c) 45
d) 4 hours d) 35
e) None of these e) None of these

Directions (11 - 15): Study the following information carefully and answer the questions given below.
The line graphs show the number of students in class XI of five schools and the percentage of the
number of students in class XII of five schools out of the total students [ class XI+ Class XII].

Note – The ratio of the number of boys and girls in school A in class XI and Class XII is 3:2. The number
of boys in school B in class XI and class XII is 60%. The number of boys in school C in class XI and class
XII is 30%. The number of girls in school D in class XI and class XII is 40%. The number of boys and girls
in school E in class XI and Class XII is equal.
11) Find the difference between the total number e) 155
of girls in class XI of all the schools together and
the total number of boys in class XII of all the 12) The average age of boys in class XI is 16
schools together? years, and the average age of girls in class XI is
a) 188 17 years from school A, and the average age of
b) 154 boys in class XII is 19 years, and the average
c) 182 age of girls in class XII is 18 years from school A.
d) 166

Click Here For Bundle PDF Course | support@guidely.in Page 3 of 8


SBI Clerk & RRB PO Mains PDF Course 2023
Quantitative Aptitude Day - 42 (Eng)

Find the average age of all the students in school Find the value of (M*N) =?
A? a) 432
a) 18.35 b) 420
b) 18.24 c) 412
c) 17.24 d) 452
d) 17.72 e) None of these
e) 17.35
16) The cost price of an item is Rs.400. If the
13) Find the ratio of the number of boys in item is marked up by 60% above the cost price
schools B and C together in class XII and the then the item is sold at two successive discounts
number of girls in schools D and E together in of d% and (d+5)%. If the profit percentage is
class XI? 22.4% then find the selling price when the item is
a) 12:7 sold at a 2d% discount?
b) 18:7 a) Rs.554
c) 19:5 b) Rs.562
d) 11:13 c) Rs.582
e) None of these d) Rs.512
e) Rs.546
14)The number of students in class XI of school
F is 20% more than the same in class XI of 17) A boat covers D+15 km downstream in 4
school E and the number of students in class XII hours, and at the same time boat covers D-25
of school F is 25% more than the same in class km upstream. The ratio of the speed of the boat
XII of school E. Find the total number of students in still water to the speed of the stream is 3:1.
in school F? Find the time taken by the boat to cover (D+25)
a) 246 km downstream and (D+30) km upstream?
b) 225 a) 11 hours
c) 258 b) 19 hours
d) 268 c) 13 hours
e) None of these d) 14 hours
e) 22 hours
15) M= The difference between boys and girls in
school B in class XI. 18) The radius of the circle is 21 cm. The area of
N=The difference of boys and girls in school C in the circle and the area of the rectangle are the
class XII. same. The side of a square whose perimeter is

Click Here For Bundle PDF Course | support@guidely.in Page 4 of 8


SBI Clerk & RRB PO Mains PDF Course 2023
Quantitative Aptitude Day - 42 (Eng)

132 m is equal to the breadth of the rectangle. If b) t3+9t2-8t+8=256


the difference between the breadth and length of c) t3+2t2-4t+5=250
the rectangle is d then find the value of d2? d) t3+5t2-4t+5=235
a) 82 e) None of these
b) 81
c) 55 20) The ratio of milk and water in a mixture is
d) 65 8:3. 40% mixture is taken, and half the amount of
e) None of these water is added to the mixture. If the final amount
of water is 40 L then find the ratio of milk and
19) The ratio of the ages of A and B is 4:5. After water in the final quantity?
8 years age of A is 4 years less than the present a) 5:4
age of C who is 7 years older than B. The b) 2:1
difference between the ages of A and B is t then c)3:2
find which of the following relation is true? d) 6:5
a) t3+5t2-4t+8=258 e) None of these

Click Here to Get the Detailed Video Solution for the above given Questions
Or Scan the QR Code to Get the Detailed Video Solutions

Answer Key with Explanation

Directions (01 - 05): The Number of cars sold in February is 200-


The Number of cars sold in January is 120 120=80
The number of diesel cars sold in January is The Number of diesel cars sold in February is
120*3/5=72 200*5/8-72=53
The Number of petrol cars sold in January is The Number of petrol cars sold in February is
120*2/5=48 200*3/8-48=27

Click Here For Bundle PDF Course | support@guidely.in Page 5 of 8


SBI Clerk & RRB PO Mains PDF Course 2023
Quantitative Aptitude Day – 42 (Eng)

Similarly, we can calculate the value of the other


also. Directions (06 - 10):
The speed of car A in the daytime is 180/3=60
km/hr
The speed of car A at the nighttime is 60*3/4=45
km/hr
The Speed of car B at the nighttime is =
45*4/3=60km/hr
1) Answer: C
The speed of car B in daytime is =
The total number of cars sold in June is =
60*133.33/100=80 km/hr
30*110/100+60*125/100=33+75=108
The Speed of car C in the daytime is =
60*5/6=50km/hr
2) Answer: D
The Speed of car C at the nighttime is
M= [(45-27)/27] *100=66.66%
50*80/100=40 km/hr
N= [(95-53)/53] *100=79.24%
6) Answer: C
So, N-M=79.24-66.66=12.58%
The Speed of car D in the daytime is =80*6/5=96
km/hr
3) Answer: B
The Speed of car D at nighttime is 45*6/5=54
I=The difference between petrol and diesel cars
km/hr
sold in the month of May=60-30=30
So, M=54*4+96*6=792 km
J= The difference between petrol and diesel cars
So, required time =[792-45*5]/60=9.45 hours
sold in the month of March= 95-45=50
K= The difference between petrol and diesel
7) Answer: D
cars sold in the month of February=53-27=26
The speed of car E in daytime is =
So, I+J-K=30+50-26=54
60*120/100=72 km/hr
The speed of car E at Nighttime
4) Answer: A
is=40*125/100=50 km/hr
Required difference = [53+30]-[45+30]=83-75=8
The total distance traveled by car B is
80*5+60*3=580 km
5) Answer: A
The total distance traveled by Car E is
The total number of petrol cars sold in all months
72*5+50*3=510 km
together is
So, the required difference is 580-510=70 km
=48+27+45+30+60=210
Required central angle [360/210]*45=770

Click Here For Bundle PDF Course | support@guidely.in Page 6 of 8


SBI Clerk & RRB PO Mains PDF Course 2023
Quantitative Aptitude Day – 42 (Eng)

8) Answer: B
I=3*60=180 km
J=4*80=320 km
So, I+J=180+320=500 Km
So, required time =(500/2)/40=6.25 hours

9) Answer: A
So, distance of y to x is =60*[6-2]=240
So, t1=240/80=3 hours 11) Answer: C
Required difference = [72+126+18+72+60]-
10) Answer: A [32+36+42+16+40]=182
The distance traveled by cars A and B in
daytime is = [60*4+80*4]=240+320=560km 12) Answer: D
The distance traveled by cars B and C in The average age of students in school A is
nighttime is = [60*6+40*6]=360+240=600 km =[16*48+17*32+72*19+18*48]/200=3544/200=1
So, the difference is 600-560=40 7.72 years

Directions (11 - 15): 13) Answer: B


The number of students in class XII of school A Required ratio =[126+18]:[16+40]=144:56=18:7
is 80*60/40=120
The number of boys in class XI of school A is 14) Answer: A
80*3/5=48 The total number of students in class F is =
The number of girls in class XI of school A is 80*120/100+120*125/100
80*2/5=32 =96+150=246
The number of boys in class XII of school A is
120*3/5=72 15) Answer: A
The number of girls in class XII of school A is M= The difference between boys and girls in
120*2/5=48 school B in class XI=54-36=18
Similarly, we can calculate others' values also. N=The difference between boys and girls in
school C in class XII=42-18=24
the value of (M*N) = 18*24=432

16) Answer: D

Click Here For Bundle PDF Course | support@guidely.in Page 7 of 8


SBI Clerk & RRB PO Mains PDF Course 2023
Quantitative Aptitude Day – 42 (Eng)

According to the question, The area of the circle is = 22/7*21*21=1386


Marked price is =400*160/100=640 The side of the square is 132/4=33= breadth of
So, 640*[(100-d)/100]*[(100-d- the rectangle
5)/100]=400*122.4/100=489.6 The length of a rectangle is 1386/33=42
Or, (100-d)*(100-d-5)=7650 So, 42-33=9=d
Or, d2-195d+1850=0 So, d2=81
From this, we can calculate d=10
So, required selling price =640*80/100=512 19) Answer: D
According to the question,
17) Answer: D The difference in age of A and B is [8+4]-7=5=t
According to the question, So, t3+5t2-4t+5=125+5*25-4*5+5=250-20+5=235
Downstream speed is [D+15]/4
Upstream Speed is [D-25]/4 20) Answer: D
So, {[D+15]/4}/{[D-25]/4}=4/2=2/1 According to the question,
Or, 2D-50=D+15 Let, the amount of milk and water initially is 80a
Or, D=65 and 30a.
So, the downstream speed is 80/4=20 km/hr and so, [30a*60/100]+[110a*40/100]/2=40L
the upstream speed is 40/4=10 km/hr. or, 18a+22a=40
So, required time is = [90/20]+[95/10]=14 hours or, a=1
so, the amount of milk initially is 80*1=80 L
18) Answer: B final milk is 80*60/100=48 L
According to the question, so, the required ratio is 48:40=6:5

Click Here For Bundle PDF Course | support@guidely.in Page 8 of 8


SBI Clerk & RRB PO Mains PDF Course 2023
ENGLISH Day - 42

English Language

Directions (1-5) : Swap the words given in bold (if I. The GDP has seen a improve (A) upward
necessary) to make the given sentences trend, reflecting a steady growth in economic
meaningful and correct. activity and indicating (B) positive momentum for
1) the overall economy.
I. The furniture (A) was so soft and luxurious, it II. The country has to gradual (I) on exports by
felt like a gentle appeal (B) against my skin. implementing strategic trade policies, diversifying
II. The fabric (I) cushion is selected based on its (II) its export markets, and enhancing
durability, comfort, and aesthetic embrace (II) to competitiveness.
create the perfect balance of style and a) A-I
functionality. b) B-II
a) A-I c) Both a and b
b) B-II d) A-II
c) Both a and b e) No changes required
d) A-II
e) No changes required 4)
I. The baby was asleep, cradled (A) in the
2) exhaustion (B) of the crib, breathing softly and
I. Worshipping God is like a deeply (A) spiritual peacefully as dreams danced through its
journey, a connection to the divine that fills the innocent slumber.
heart with reverence and gratitude (B). II. The online session was so tiring, leaving me
II. The professor asked the students to be drained (I) and longing for a break from the
actively (I) engaged in the discussion, fostering screen-induced warmth (II).
(II) an environment of critical thinking and a) A-I
collaborative learning. b) B-II
a) A-I c) A-II
b) B-II d) Both a and b
c) Both a and b e) No changes required
d) B-I
e) No changes required 5)
I. The book has 15 chapters, each expressed (A)
3) into a different aspect of the protagonist's

Click Here For Bundle PDF Course | support@guidely.in Page 1 of 10


SBI Clerk & RRB PO Mains PDF Course 2023
ENGLISH Day - 42

transformative journey, creating a rich (B) and c) Bihar, a state in eastern India, holds a
multifaceted narrative. significant place in the country's history and
II.The author has delving (I) her profound heritage also known for its ancient roots
insights on human nature and the complexities of d) While many factors contribute to the dynamics
relationships through eloquent (II) prose that of the tigers present in a region
resonates deeply with readers' emotions and e) None of these
experiences.
a) A-I 7) The Digital Data Protection Bill, 2023, was
b) B-I passed in the Lok Sabha on Monday and will
c) B-II now have to be cleared by the Rajya Sabha. The
d) Both a and c fresh iteration, which has undergone a few
e) No changes required drafts, seems to have incorporated suggestions
made to its 2022 version. Although it is not clear
Directions (6-10) : In each of these questions, a what the submissions were as the consultation
paragraph is given that has a blank in it. Out of process was not brought to light by the
the given options, only one sentence fits in with government. __________.
the context of the paragraph. Select that as your a) The highlight of the Bill is the provision that
answer. personal data of an individual may be processed
6) Madhya Pradesh, for the second time in eight without the consent of the data principal
years, reported 785 tigers, or about a fifth of the b) Data protection and privacy are essential
national count. The State reported a 50% rise in rights in the digital age, ensuring that individuals
tigers since the last census, a figure bettered have control over their personal information
only by Bihar which has less than 10% of M.P’s c) The Lok Sabha is the lower house of the
tigers______________________. M.P. over the Parliament of India, consisting of elected
years has perfected the approach of actively representatives from various constituencies
moving both tigers, as well as their prey, within across the country
the State to balance predator and prey d) The highlight of the Bill is the provision that
population. personal data of an individual may be processed
a) Tigers, majestic and powerful predators, roam only after the consent of the data principal
the dense jungles with their distinctive orange e) None of these
coats and dark stripes
b) Madhya Pradesh, located in the heart of India, 8) For the second time in three months,
boasts a rich tapestry of history, culture, and Pakistan’s former Prime Minister Imran Khan has
natural beauty been arrested.This arrest was after a conviction

Click Here For Bundle PDF Course | support@guidely.in Page 2 of 10


SBI Clerk & RRB PO Mains PDF Course 2023
ENGLISH Day - 42

in a case involving the misappropriation of official c) The spacecraft entered lunar orbit on 5 August
gifts meant for the ‘Toshakhana’. He was last 2023
arrested on May 9 in the ‘Al Qadir’ trust case d) The primary issue was the five engines which
charge sheet.__________________. It means that were used to reduce the velocity.
he is now disqualified from Parliament for the e) None of these
next five years, and will not be able to stand for
election in polls due by November. 10) The Supreme Court on August 10
a) The latest conviction, which carries a sentence announced the launch of portal 'Suswagatam'.
of three years This would enable advocates, visitors, interns
b) Established in 1974, the Tosha­khana is a and others to register themselves online and get
department which stores precious gifts given to e-passes to enter the apex court. Suswagatam'
rulers is a web-based and mobile-friendly application
c) He transitioned into politics and became Prime that allows users to register themselves online
Minister after his party won the general elections and request for e-passes for various purposes
in 2018 such as attending court hearings, meeting
d) Conviction refers to a formal declaration by a advocates. ____________________
court of law that someone has been found guilty a) The portal issues electronic passes to the
of a criminal offence after a legal trial or people
proceeding b) The Supreme Court of India is the country's
e) None of these highest judicial court
c) The 'Suswagatam' portal was tested as a pilot
9) Indian Space Research Organisation (ISRO) project from July 25, 2023
has released two images taken by the d) Many such portals are available that help the
Chandrayaan-3 spacecraft._________________. functioning of courts easier
The photo was taken on July 14, when the e) None of these
Chandrayaan-3 mission was launched from the
Satish Dhawan Space Centre in Sriharikota.The Directions (11-15) : Rearrange the following five
second image is of the Moon captured by the sentences (A), (B), (C), (D) and (E) in the proper
Lander Horizontal Velocity Camera (LHVC) on sequence to form a coherent paragraph and then
board the spacecraft on August 6. answer the questions given below.
a) Chandrayaan-3 was launched on 14 July 2023 (A) And while monetary authorities are right in
from Satish Dhawan Space Centre laying the onus on the government for ensuring
b) The first image is of the Earth as viewed by timely supply side interventions to ‘limit the
the Lander Imager Camera severity and duration of such shocks’, a broader

Click Here For Bundle PDF Course | support@guidely.in Page 3 of 10


SBI Clerk & RRB PO Mains PDF Course 2023
ENGLISH Day - 42

unmooring of price stability will undermine c) C


macroeconomic stability and growth. d) D
(B) RBI Governor Shaktikanta Das e) No error
acknowledged that while/A the likely short-term
nature of these shocks allowed for policymakers 13) Fill the given blank in sentence D.
to/B look a couple past of high inflation prints, a) remained
frequently/C recurring food price shocks risk b) unchanged
destabilising inflation expectations/D. c) altered
(C) Barely two months after projecting d) changed
Consumer Price Index-based inflation to average e) unnoticed
5.1% over the current fiscal year ending in March
2024, the MPC has raised its forecast for the 14) Replace the bold word in sentence E to
annual average by 30 basis points to 5.4%. make it correct and meaningful.
(D) The latest decision by the Reserve Bank of a) expansion
India’s Monetary Policy Committee (MPC) to b) climb
leave its policy rate _______ is a calculated risk c) growth
taken by the rate setting panel, especially when d) spike
viewed in light of the upward revision of its e) addition
inflation forecast.
(E) Conceding that a improvement in tomato 15) Which of the following is the correct
prices had contributed to a shock that had forced sequence of rearrangement ?
a revision in the headline inflation projection for a) eadcb
the July-September quarter by a ‘substantial’ 100 b) ceadb
basis points to 6.2%. c) bceda
11) Meaning of the word ‘Onus’ in sentence A. d) abdec
a) something that is one's duty or responsibility. e) dceba
b) onus is when all people come together
c) an official or person in high position Directions (16-20) : In the statements given
d) praising someone for his/her act below, a blank has been given. This needs to be
e) one’s unwanted commitment filled up using a correct word from the options
mentioned below. The statement so formed must
12) Spot the erroneous part in sentence B. be contextually and grammatically sound.
a) A 16) The curbs were to kick in with immediate
b) B effect, rattling the entire supply chain and ______

Click Here For Bundle PDF Course | support@guidely.in Page 4 of 10


SBI Clerk & RRB PO Mains PDF Course 2023
ENGLISH Day - 42

fears of shortages and price surges, especially b) conquer


ahead of the festive season. c) construct
a) enhancing d) consensus
b) illuminating e) convince
c) igniting
d) deciding 19) All the claims of scientists having found a
e) running room-temperature superconductor so far have
failed to _______ independent scrutiny.
17) The government sought to suggest this was a) withstand
primarily driven by security concerns and b) standby
imported devices could be used for__________, c) stick with
just as mobile phones could have spyware. d) give up
a) survival e) stand up
b) surveillance
c) surpassing 20) A Supreme Court judgement in March said
d) surveying the panel should _______ the Prime Minister, the
e) surrendering Leader of the Opposition in Lok Sabha and the
Chief Justice of India.
18) The main goal of Ukraine and its western a) compromise
partners was to build ________ among major b) apprise
powers, especially in the Global South, on c) reprise
working towards a fair and durable peace. d) supervise
a) consult e) comprise

Click Here For Bundle PDF Course | support@guidely.in Page 5 of 10


SBI Clerk & RRB PO Mains PDF Course 2023
ENGLISH Day - 42

Click Here to Get the Detailed Video Solution for the above given Questions
Or Scan the QR Code to Get the Detailed Video Solutions

Answer Key with Explanation

1) Answer: C The furniture cushion is selected based on its


Both A-I and B-II need to be swapped to make durability, comfort, and aesthetic appeal to
the sentences I and II correct and meaningful. create the perfect balance of style and
Swap fabric and furniture, similarly swap appeal functionality.
and embrace.
2) Answer: E
Furniture is an object that cannot be worn, All the labelled words are correct and
whereas fabric refers to the material and can be appropriate. Hence, no changes required.
felt on the skin - hence replacing fabric and
furniture. 3) Answer: A
Only words A and I need swapping. Improve and
Embrace - to put your arms around somebody gradual and placed inappropriately and must be
as a sign of love, happiness interchanged to make the given two sentences
Appeal - to be attractive or interesting to correct and meaningful. The other two words
somebody indicating and diversifying are correct.
In the sentence I -’ felt like gentle embrace’ In sentence I - 'gradual upward trend’ would be
would be the correct phrase correct - gradual(slow)
In the sentence II - ‘aesthetic appeal’ would be In sentence II - ‘the country has to improve on
the correct phrase exports’ would be correct
The sentences: The sentences:
The fabric was so soft and luxurious, it felt like a
gentle embrace against my skin.

Click Here For Bundle PDF Course | support@guidely.in Page 6 of 10


SBI Clerk & RRB PO Mains PDF Course 2023
ENGLISH Day - 42

The GDP has seen a gradual upward trend, The sentences:


reflecting a steady growth in economic activity The book has 15 chapters, each delving into a
and indicating positive momentum for the overall different aspect of the protagonist's
economy. transformative journey, creating a rich and
The country has to improve on exports by multifaceted narrative.
implementing strategic trade policies, The author has expressed her profound insights
diversifying its export markets, and enhancing on human nature and the complexities of
competitiveness. relationships through eloquent prose that
resonates deeply with readers' emotions and
4) Answer: B experiences.
The words exhaustion and warmth must be
interchanged to make the sentences I and II 6) Answer: D
correct and meaningful. Sentence in option d fits the blank perfectly as
Exhaustion - a state of being incredibly tired the passage deals with the conservation of
Warmth - a fairly high temperature or the effect tigers, talking about tigers in general or about the
created by this, especially when it is pleasant involved states is not necessary here.
In sentence I - baby fell asleep by the warmth of D is more appropriate and suitable here.
the crib A - generic about tigers (not relevant)
In sentence II - the session was tiring and B - about Madhya Pradesh (not relevant)
exhausting C - about Bihar (not relevant)
The sentences: The only relevant sentence is D.
The baby was asleep, cradled in the warmth of
the crib, breathing softly and peacefully as 7) Answer: D
dreams danced through its innocent slumber. D is the correct sentence that is talking about the
The online session was so tiring, leaving me data protection bill and privacy.
drained and longing for a break from the screen- A is incorrect because it is well known that
induced exhaustion. usage of personal information without consent is
illegal.
5) Answer: A B is a general statement about data protection
Interchange ‘expressed’ and ‘delving’ to make and privacy
the sentence correct and meaningful. C conveys general information about the Lok
Delve - to search inside something sabha which is also irrelevant to the context of
the passage.

Click Here For Bundle PDF Course | support@guidely.in Page 7 of 10


SBI Clerk & RRB PO Mains PDF Course 2023
ENGLISH Day - 42

8) Answer: A B - generic sentence about supreme court


Among the given statements the sentence which D - does not fit the context
completes the given passage is ‘a’ as it talks
about the recent conviction, the sentence which 11) Answer: A
follows the blank also talks about the conviction Onus means - one’s duty or responsibility
and punishment.
B - Toshakhanna is somehow related to the 12) Answer: C
context of the passage but the blank cannot be The error is in part c of the given sentence.
filled with this sentence as there is no continuity. The phrase ‘look a couple past of’ is completely
C - the sentence is about Imran Khan but in the wrong instead it must be ‘look past a couple of’.
passage given it is irrelevant to talk about his Look past - To see potential beyond obvious
D - definition of conviction is not really needed in flaws; to consider something more than
this context something else.
The sentence : RBI Governor Shaktikanta Das
9) Answer: B acknowledged that while the likely short-term
B is the right statement/sentence to fill the given nature of these shocks allowed for policymakers
blank because the sentence following the blank to look past a couple of high inflation prints,
talks about a photo/image which must be a frequently recurring food price shocks risk
continuation. So, B is the correct sentence when destabilising inflation expectations.
compared to the other given options.
A - launch date of chandrayaan 3 is 13) Answer: B
comparatively less relevant than sentence b ‘Unchanged’ is the correct word that fits the
C - also irrelevant because the passage talks given blank.
about images that ISRO released ‘To leave something’ conveys a meaning - to
D - talks about some issues related to the leave something undisturbed/to leave it the
spacecraft launch same way it is
Changed and altered are wrong, unnoticed and
10) Answer: C remained are also incorrect and does not convey
C is the correct sentence that completes the a proper meaning
passage as it revolves around the main idea of The sentence:
the passage - Suswagatam portal. The latest decision by the Reserve Bank of
A - redundant information as the passage India’s Monetary Policy Committee (MPC) to
already spoke about the same leave its policy rate unchanged is a calculated

Click Here For Bundle PDF Course | support@guidely.in Page 8 of 10


SBI Clerk & RRB PO Mains PDF Course 2023
ENGLISH Day - 42

risk taken by the rate setting panel, especially 16) Answer: C


when viewed in light of the upward revision of its Out of the given options, igniting is the best word
inflation forecast. that fits the blank without changing the actual
meaning of the sentence.
14) Answer: D Igniting - to start burning or to make something
Improvement is an incorrect word and does not start burning
fit the given sentence. Enhancing - to improve something or to make
Among the given words ‘spike’ is the correct something look better (enhance fear would
word that fits the sentence without altering the sound wrong)
original meaning. Illuminating - helping to explain something or
Spike - A spike is a sudden and large move in make something clear (irrelevant)
the price of an asset Deciding and running are also inappropriate
The sentence : Conceding that a spike in tomato words and hence discarded
prices had contributed to a shock that had forced The sentence: The curbs were to kick in with
a revision in the headline inflation projection for immediate effect, rattling the entire supply chain
the July-September quarter by a ‘substantial’ and igniting fears of shortages and price surges,
100 basis points to 6.2%. especially ahead of the festive season.

15) Answer: E 17) Answer: B


The correct sequence of rearrangement is According to the given context ‘surveillance’ is
DCEBA. the correct word that fits the blank correctly.
D is the introductory sentence as it has got the As the context is related to security, surveillance
information on the latest decision of RBI. D is would be the best fit.
followed by C which conveys information The other given words have a different meaning
regarding the RBI’s forecast in the midst of and hence are not suitable.
which the RBI has left the policy rate unchanged. Survival - the act or fact of living
After DC, comes E which cites the reason for the Surveillance - the careful watching of somebody
inflation project - price of tomatoes. E is followed who may have done something wrong
by B which talks about the price shocks in detail Surpass - to do something better than
and what it leads to. Last sentence would be A, somebody/something else or better than
which adds more to the price shocks and expected
inflation projection.

Click Here For Bundle PDF Course | support@guidely.in Page 9 of 10


SBI Clerk & RRB PO Mains PDF Course 2023
ENGLISH Day - 42

Survey - the process of collecting, analysing and Withstand is the correct word that fits the blank
interpreting data from many individuals making the sentence complete and meaningful.
Surrender - to stop fighting and admit that you Withstand - to be strong enough not to break,
have lost/to give somebody/something to give up
somebody else Standby - a thing or person that can be used if
The sentence: needed
The government sought to suggest this was Stick with - to continue using or doing
primarily driven by security concerns and Give up - to cease doing
imported devices could be used for surveillance, Stand up - to be or become vertical
just as mobile phones could have spyware. The sentence:
All the claims of scientists having found a room-
18) Answer: D temperature superconductor so far have failed to
Consensus is the right word that fits the blank. withstand independent scrutiny.
Consensus - agreement among a group of
people (build agreement would be the correct 20) Answer: E
phrase as it conveys a proper meaning) Comprise is the correct word that fits the given
Conquer - to take control (irrelevant) blank.
Consult - to ask somebody for some information Other words are discarded because of their
or advice (does not fit because different meaning as they don’t fit the context of the
meaning) sentence.
Construct - to build (not relevant) Compromise - an agreement
Convince - to succeed in making someone Apprise - inform or tell
believe something (does not fit the blank Reprise - repeat
properly) Supervise - to watch somebody/something to
The sentence: make sure that work is being done properly
The main goal of Ukraine and its western Comprise - to consist of
partners was to build consensus among major The sentence:
powers, especially in the Global South, on A Supreme Court judgement in March said the
working towards a fair and durable peace. panel should comprise the Prime Minister, the
Leader of the Opposition in Lok Sabha and the
19) Answer: A Chief Justice of India.

Click Here For Bundle PDF Course | support@guidely.in Page 10 of 10


SBI Clerk & RRB PO Mains PDF Course 2023
Reasoning Ability Day - 43 (Eng)

Reasoning Ability
Directions (1-5) Study the following information b) O
carefully and answer the given questions. c) The car which is parked immediate right of E
Nine cars – A, D, E, H, K, O, S, U, and Z are d) Both a and c
parked around a triangular parking yard in such a e) Both a and b
way that two cars are parked on each side of the
parking yard and facing towards the centre 2) The car which is from ____ is parked ____ to
whereas one car is parked on each corner of the the right of A.
parking yard and facing away from the centre. a) West Bengal, Fourth
Each car is from different states viz.- Delhi, Uttar b) Punjab, Third
Pradesh, Punjab, Bihar, Gujarat, Haryana, West c) Assam, Immediate
Bengal, Assam, and Kerala. d) Bihar, Second
D, which is from Uttar Pradesh, is parked fourth e) None of these
to the left of S and vice-versa. S is parked
second to the right of the car which is from 3) Which of the following car is not parked at the
Assam. As many cars parked between the car corner?
which is from Assam and D as between E and I. The car which is parked immediate left of car
the car which is from Assam. The car which is which is from Kerala
from Bihar is parked fourth to the right of Z, II. H
which is parked adjacent to the car which is from III. The car which is from west Bengal
Assam. Neither car which is from Bihar nor a) Only I and III
Kerala is parked adjacent to car D. Car O, which b) Only III
is from Punjab, is parked on the same side along c) Only II
with H. As many cars parked between O and the d) Only I and II
car which is from Kerala as between H and the e) Only I
car which is from Gujarat. Both cars S and Z are
neither from Kerala nor West Bengal. The cars 4) How many cars are parked between O and
which is from Gujarat and Delhi are parked the car which is from Bihar when counted from
together. Only two cars are parked between U the right of O?
and the car which is from Haryana. A is not a) As many cars parked between car U and the
parked adjacent to U. car which is from Gujarat, when counted from the
1) Which of the following car parked second to left of U
the left of the car which is from Uttar Pradesh? b) Four
a) The car which is from Bihar

Click Here For Bundle PDF Course | support@guidely.in Page 1 of 11


SBI Clerk & RRB PO Mains PDF Course 2023
Reasoning Ability Day - 43 (Eng)

c) As many cars parked between S and car after J, who was born on 9th August. The
which is from Delhi, when counted from the right number of persons born between I and D is twice
of S the number of persons born between D and H,
d) Three who was born on a perfect square numbered
e) None date. H and C were born in the month having an
odd number of days. B was born on a prime
5) Which of the following statements is/are not number date but not immediately after D. D and
true as per the given arrangement? G were not born in the same month. A and G
a) The car which is from Kerala is parked third to were born on the same date. A was not born on
the right of H 3rd of any month. C was not born on an odd
b) Car O is parked immediate left of car S numbered date.
c) The car which is parked immediate left of K is 6) Who among the following persons was born
from Kerala on an even numbered date?
d) The car which is from Delhi is parked at the I. The one who was born two persons after B
corner II. J
e) All the statements are true III. The one who was born immediately before F
a) Only II
Directions (6-10) Study the following information b) Only III
carefully and answer the given questions. c) Only I and III
Ten persons were born on four different dates d) Only I and II
viz.- 3, 6, 9, and 12 of four different months viz.- e) Only I
January, April, June, and August. At least two
persons were born in the same month. No two 7) ____ was born on ___ and immediately after
persons were born on the same date of the same ____.
month. a) I, 12th August, J
Only three persons were born before E, who was b) E, 6th April, D
born on an even numbered date. Only two c) H, 9th January, C
persons were born between E and F, who was d) A, 12th June, F
born on 9th June. Only four persons were born e) None of these
between C and A, who was born immediately
before or immediately after F. The one who was 8) How many persons were born between A and
born immediately before C was born on 9th of the one who was born on April 6?
the month. The number of persons born before C I. As many persons born before H
is one more than the number of persons born II. Four

Click Here For Bundle PDF Course | support@guidely.in Page 2 of 11


SBI Clerk & RRB PO Mains PDF Course 2023
Reasoning Ability Day - 43 (Eng)

III. As many persons born after F c) Only I and III


a) Only I and III d) Only III
b) Only III e) Only II
c) Only II
d) Only I and II 12) If all the letters of the given words are
e) Only I changed to their complementary paired letters in
alphabetical series and then letters are
9) D was born in which of the following month rearranged in alphabetical order from the left
and date? end, then what is the word (meaningful/non-
a) April 9 meaningful) formed by using the fourth letter
b) June 12 from the right end of each word?
c) April 12 I. PIGMENT
d) Either a or c II. FLOWER
e) Either a or b III. RADEON
a) NPR
10) Which of the following statement is not true b) NLM
as per the given arrangement? c) MOL
a) F was born immediately after G d) VON
b) E was born on April 6 e) None of these
c) Only three persons born after A
d) H was born immediately after B 13) If a meaningful word can be formed from the
e) All the statements are true first, third, fourth, and fifth letters of each word
respectively (from the left end), then which of the
11) If the first half of the digits are written in following set of words has more than one
descending order from the left end whereas the meaningful word formed?
second half of digits are written in descending I. Graph, Extend, Champion, Stream
order from the right end, then in which of the II. Move, Grant, Stream, Counter
following numbers sum of the third digit from III. Count, Reason, Appear, Paternal
both ends gives a prime number? a) Only I
I. 37689256 b) Only II
II. 49634572 c) Only I and III
III. 65899743 d) Only II and III
a) Only II and III e) Only III
b) Only I

Click Here For Bundle PDF Course | support@guidely.in Page 3 of 11


SBI Clerk & RRB PO Mains PDF Course 2023
Reasoning Ability Day - 43 (Eng)

14) If in the number “372965847392” the first half and medium sized enterprises (SMEs) are set to
the of number is written in descending order from rise in the coming months of 2018 ‘ Rising
the left end followed by the second half of the interest rates, in addition to the muted operating
number is written in ascending order from left to environment for small businesses in India, will
right, now all the odd positioned odd digits from lead to an increase in delinquencies on LAP
the left end are dropped, then what is the product extended to SMEs’. The introduction of the
of the third digits from both ends? goods and services tax (GST) in July 2017 and
a) 36 the government's demonetization policy have
b) 35 placed stress on the SME sector, which rising
c) 30 interest rates will exacerbate.
d) 42 Which of the following can be logically inferred
e) None of these from the statement above?
A.In addition to rising delinquencies over the last
15)If all the letters of the following words are few years would force lenders to be more
changed to the third successive letter in the cautious while underwriting loans. This would
alphabetical series and all the letters are limit refinancing options, adversely affecting
arranged in alphabetical order from the right end existing borrowers.
then how many words have a consonant which is B.But losses will be limited because these loans
immediately followed by a vowel? are secured and have relatively low loan-to-value
I. ISOLATION ratios.
II.PANDEMIC C.Rising interest rates will limit refinancing
III. SOLITUDE options and increase repayment amounts,
a) Only II and III increasing risks for asset backed securities
b) Only II (ABS) backed by LAP to SMEs.
c) Only I and III D.The ABS (asset backed securities) backed by
d) Only III LAP extended to SMEs have non-amortizing
e) All I, II, and III cash reserves, substantial excess spread and
the possibility to extend the life of the loans and
16) Read the given statement and with that hence the life of the transactions.
information answer the following question E.None can be inferred.
carefully.
After GST and demonetisation, rising interest 17) Read the given statement and with that
rates are set to hurt SMEs. Interest rates for information answer the following question
loans against property (LAP) extended to small carefully.

Click Here For Bundle PDF Course | support@guidely.in Page 4 of 11


SBI Clerk & RRB PO Mains PDF Course 2023
Reasoning Ability Day - 43 (Eng)

The government has held internal preliminary 18. Statement:


discussions over a possible infusion of fresh “Lok Sabha passed the Motor Vehicles
capital in debt-laden IDBI by Life Insurance (Amendment) Bill in 2017. The amendments
Corporation of India (LIC) but no decision has were targeted towards bringing changes in the
been taken on selling its stake in the bank to the transport sector to encourage safer driving
insurer, said a senior official. LIC would use that practices among Indian motor vehicle drivers.
amount to capitalize the bank to maintain its The amendment’s draft was put forward in the
capital adequacy ratio. It would not be a stake lower house of the Parliament to impose rigid
sale by government. The government is fines for the traffic rules violation and to promote
expected to hold discussions with banking, e-governance.”
insurance and market regulators before taking a Assumptions:
final call. I). Now people would be more careful before
Which of the following can be logically deduced riding their two-wheelers without a helmet and
from the statement above? while driving their cars without wearing seatbelts
A.IDBI will need more capital infusion and LIC or without valid motor insurance.
can put that money towards further strengthening II). Contractors will be held responsible and
the bank. The government may hold informal blacklisted for the bad condition of roads.
consultations with Reserve Bank of India (RBI), III). The previous governments have not taken
Insurance Regulatory and Development Authority enough measures for road safety.
of India (IRDA) and market regulator Securities A.only I and III are implicit
and Exchange Board of India (SEBI). B.only I and II are implicit
B.The government had in 2015 announced its C.Only III is implicit
intention to transform IDBI Bank on the lines of D.Only I is implicit
Axis Bank. Issuance of new equity will raise more E.Only II and III are implicit
capital and also dilute the government’s stake in
the bank 19. Statement: The court asked the Petroleum
C.The insurer already has a 10.82% stake in and Safety Explosives Organization (PESO) to
IDBI Bank and would require regulatory review the clinical composition of fireworks,
clearance to increase this beyond 15%. particularly with respect to reducing aluminium
D.The government is considering a preferential content, and submit a report on this within two
capital issue by IDBI Bank to bring its holding weeks.
below 50% as part of plans to transform its Assumptions:
fortunes, along the lines of Axis Bank. I). The court put the onus on the Centre, state
E.All can be deduced. governments, schools and colleges to take up

Click Here For Bundle PDF Course | support@guidely.in Page 5 of 11


SBI Clerk & RRB PO Mains PDF Course 2023
Reasoning Ability Day - 43 (Eng)

public awareness campaigns informing the public elimination from the area. Since then naturalists
about the harmful effects of firecrackers. working in the region have discovered no hard
II). Firecrackers alone could not be held as the evidence of its survival, such as carcasses or
reason for pollution during Diwali and an entire tracks. In spite of alleged sightings of the animal,
industry should not be shut down as a the White rhino no longer exists.
consequence. They contended that other factors Which one of the following is the conclusion on
such as vehicular pollution, construction dust and which the forester’s statement depends?
crop stubble burning also contribute to a great A. Deforestation drove the last White rhinos to
measure to the pollution in Delhi. starvation by chasing them from their natural
A.Only I Implicit habitat.
B.Only II Implicit B. Every naturalist working in the White rhino’s
C.If either I or II Implicit natural habitat has looked systematically for
D.If neither I nor II is Implicit evidence of the rhino’s survival.
E.If both I and II are Implicit C.The White rhino did not move and adapt to a
different region in response to the loss of habitat.
20. Statement : Forester -- The recent claims that D. Those who have reported sightings of the
the White rhino is not extinct are false. The White White rhino are not experienced naturalists or
rhino’s natural habitat was taken over by foresters.
deforestation, resulting in the animal’s systematic E. none of the above
Click Here to Get the Detailed Video Solution for the above given Questions
Or Scan the QR Code to Get the Detailed Video Solutions

Answer Key with Explanation

Direction (1-5): 3. Answer: C


1. Answer: D 4. Answer: C
2. Answer: B 5. Answer: B

Click Here For Bundle PDF Course | support@guidely.in Page 6 of 11


SBI Clerk & RRB PO Mains PDF Course 2023
Reasoning Ability Day - 43 (Eng)

 Neither car which is from Bihar nor Kerala


is parked adjacent to car D.
That means, in case (1) & case (2) car
from Bihar parked immediate right of E.
 Car O, which is from Punjab, is parked on
the same side along with H.
That means, in case (1) O is parked
immediate left of D, in case (2) O is
parked immediate right of S.
We have:
 As many cars parked between O and the
 D, which is from Uttar Pradesh, is parked
car which is from Kerala as between H
fourth to the left of S and vice-versa.
and the car which is from Gujarat.
 S is parked second to the right of the car
 Both cars S and Z are neither from Kerala
which is from Assam.
nor West Bengal.
That means, in case (1) D is parked at the
 The cars which is from Gujarat and Delhi
corner, in case (2) D is parked at the side.
are parked together.
 As many cars parked between the car
That means, in case (1) & case (2) car E
which is from Assam and D as between E
is from Kerala, in case (1a) the car parked
and the car which is from Assam.
immediate right of S is from Kerala.
That means, in case (1) & case (2) E is
Based on the above given information we have:
parked immediate left of S.
Based on the above given information we have:

Again, we have:
 The car which is from Bihar parked fourth
to the right of Z, which is parked adjacent
to the car which is from Assam.

Click Here For Bundle PDF Course | support@guidely.in Page 7 of 11


SBI Clerk & RRB PO Mains PDF Course 2023
Reasoning Ability Day - 43 (Eng)

Again, we have: 10. Answer: C


 Only two cars are parked between U and
the car which is from Haryana.
That means, in case (1) car U belongs to
Delhi, in case (1a) car U belongs to
Assam, in case (2) car U is parked
immediate left of D.
 A is not parked adjacent to U.
in case (2) Car A parked is immediate
right of D, case (1a) is not valid and case
(1) is not valid.
Based on the above given information we have:

We have:
 Only three persons were born before E,
who was born on an even numbered date.
 Only two persons were born between E
and F, who was born on 9th June.
Since, three persons were born before E
and was born on an even numbered date,
thus E must born on either April 6 or April
12.
Based on the above given information we have:

Case (1) and (1a) is not valid as neither S nor Z


is from West Bengal.

Direction (6-10):
6. Answer: C
7. Answer: A
8. Answer: B
9. Answer: D

Click Here For Bundle PDF Course | support@guidely.in Page 8 of 11


SBI Clerk & RRB PO Mains PDF Course 2023
Reasoning Ability Day - 43 (Eng)

Again, we have:
 Only four persons were born between C
and A, who was born immediately before
or immediately after F.
 The one who was born immediately Again, we have:
before C was born on 9th of the month.  The number of persons born between I
 The number of persons born before C is and D is twice the number of persons
one more than the number of persons born between D and H, who was born on
born after J, who was born on 9th August. a perfect square numbered date.
That means, in case (1) & case (2) C was  H and C were born in the month having
born immediately before E. an odd number of days.
Based on the above given information we have: Since, only possible month for H is
January, thus H must born on 9 January.
 B was born on a prime number date but
not immediately after D.
Thus, B must born on 3 January.
 D and G were not born in the same
month.
That means, case (2) is not valid.
Based on the above given information we have:

Click Here For Bundle PDF Course | support@guidely.in Page 9 of 11


SBI Clerk & RRB PO Mains PDF Course 2023
Reasoning Ability Day - 43 (Eng)

Case (2) is not valid as after 12 April, only June


is possible and D and G were not born in the 11) Answer: B
same month. Explanation:
Again, we have: I. 37689256  87632569  (6 + 5) = 11
 A and G were born on the same date. II. 49634572  96432457  (4 + 4) = 8
 A was not born on 3rd of any month. III. 65899743  98653479  (6 + 4) = 10
 C was not born on an odd numbered
date. 12) Answer: B
Thus, A and G must born on 6th of the Explanation:
month. I. PIGMENT KRTNVMG  GKMNRTV
Based on the above given information we have: II. FLOWER  UOLDVI  DILOUV
III. RADEON IZWVLM  ILMVWZ

13) Answer: D
I. Graph, Extend, Champion, Stream  GTMA
(no word)

Click Here For Bundle PDF Course | support@guidely.in Page 10 of 11


SBI Clerk & RRB PO Mains PDF Course 2023
Reasoning Ability Day - 43 (Eng)

II. Move, Grant, Stream, Counter TEAM, Assumption I: It is implicit because the decision
MATE, MEAT to impose rigid fines for the traffic rules violation
III. Count, Reason, Appear, Paternal  CARE, would make people careful while coming on the
RACE, ACRE roads with their vehicles.
Assumption II: This may be included in the
14) Answer: B amendment bill, but it could not be an
372965847392 976532234789  7652234789 assumption.
Thus, required product = (5 x 7) = 35 Assumption III: It is not implicit, as nothing about
the previous governments or their measures is
15) Answer: C mentioned in the statement.
ISOLATION LVRODWLRQWVRRQOLLD
PANDEMIC SDQGHPLF SQPLHGFD 19) Answer: D
SOLITUDE VROLWXGH  XWVROLHG The given statement is saying that the court
asked the related Organization (PESO) to review
16) Answer: A the clinical composition of fireworks, particularly
The correct answer is option a, i.e. In addition to with respect to reducing aluminium content. In
rising delinquencies over the last few years this statement it is no where mentioned that the
would force lenders to be more cautious while court asked the schools colleges to take up
underwriting loans. This would limit refinancing public awareness campaigns informing the
options, adversely affecting existing borrowers. public about the harmful effects of firecrackers.
Also, the statement did not mention that the
17) Answer: A firecrackers are causing pollution during Diwali
The correct answer is option 1, i.e. IDBI will need and an entire industry. Therefore, both the
more capital infusion and LIC can put that statements are not implicit.
money towards further strengthening the bank.
The government may hold informal consultations 20. Answer: C
with Reserve Bank of India (RBI), Insurance If this point was not true and the White rhinos
Regulatory and Development Authority of India have shifted to another location, then the
(IRDA) and market regulator Securities and conclusion will not hold good. However, if this
Exchange Board of India (SEBI). point was actually true, it would clear any
18) Answer: D confusion and provide evidence for the foresters’
view.

Click Here For Bundle PDF Course | support@guidely.in Page 11 of 11


SBI Clerk & RRB PO Mains PDF Course 2023
Quantitative Aptitude Day – 43 (Eng)

Quantitative Aptitude

Direction (1-5): Study the following data carefully and answer the questions:
The pie chart given below shows the percentage distribution of amounts (in ₹) invested in 5 different
schemes A, B, C, D, and E.

The pie chart given below shows the percentage distribution of rate of interest in each of the given
schemes.

Note:
1: The SI received from scheme B after 2 years is ₹540 more than that received from scheme A after 3
years.

Click Here For Bundle PDF Course | support@guidely.in Page 1 of 14


SBI Clerk & RRB PO Mains PDF Course 2023
Quantitative Aptitude Day - 43 (Eng)

2: The difference between SI and CI received from scheme E after 2 years is ₹960.
1) If the ratio of the difference between SI and CI would have been approximately what per cent
received from scheme A after 2 years to the more than the original SI received from scheme
difference between SI and CI received from B after 1 year?
scheme D after 2 years is m: n, then find that a) 17.8%
which of the following is/are true? b) 22.5%
P: Value of (m + n) is divisible by 41. c)8.6%
Q: Value of (m – n) is divisible by 11. d) 13.4%
R: Both ‘m’ and ‘n’ are odd numbers. e) 20.1%
a) Only Q
b) Only P and Q 4) If the SI received from scheme C after 2 years
c) Only R is ₹M and the SI received from scheme E after 3
d) Only Q and R years is ₹N, then find the difference between SI
e) Only P and R and CI received on ₹(M + N) at 15% rate after 2
years?
2) A certain amount is invested in scheme F at a) ₹571.6
15% rate of interest. If the SI received from b) ₹561.6
scheme F after 3 years is ₹3120 less than the c)₹531.6
total SI received from schemes C and D together d) ₹551.6
after 2 years, then find that the amount invested e) ₹541.6
in scheme F is what per cent of the total amount
invested in schemes C and D together? 5) Quantity I: If the total amount, which is
a) 56.67% invested in schemes D and E together, were
b) 52.5% invested in scheme A, then what would have
c)53.33% been the SI received from scheme A after 1
d) 59.09% year?
e) 55.55% Quantity II: If the total amount, which is invested
in schemes A and C together, were invested in
3) If the amount invested in scheme B were scheme B, then what would have been the SI
₹1000 less and the numerical value of rate of received from scheme B after 1 year?
interest in scheme B were more than the a) Quantity I > Quantity II
original numerical value of rate of interest, then b) Quantity I < Quantity II
the SI received from scheme B after 1 year c)Quantity I ≥ Quantity II
d) Quantity I ≤ Quantity II

Click Here For Bundle PDF Course | support@guidely.in Page 2 of 14


SBI Clerk & RRB PO Mains PDF Course 2023
Quantitative Aptitude Day - 43 (Eng)

e) Quantity I = Quantity II or the relation can’t be b) 5


established c) 7
d) 6
Direction (6-9): Study the following data carefully e) 8
and answer the questions:
Data given below is related to the number of 7) In academy D, if the ratio of male to female
cricketers, the number of footballers and the cricketers is 5: 3, the ratio of male to female
number of swimmers in 5 sports academies A, B, footballers is 3: 5 and the ratio of male to female
C, D and E. swimmers is 1: 1, then find the average of the
Table given below shows the following data: number of male cricketers, the number of male
footballers and the number of male swimmers in
academy D?
a) 16
b) 10
c)14
d) 11
e) 12
Note:
1: Ratio of the number of swimmers in academy
8) If the sum of the numbers of cricketers,
A to those in academy C is 9: 5 and the ratio
footballers and swimmers in academy C is 44%
between the number of swimmers in academies
of the total number of players in academy C and
B, D and E is 4: 3: 2 respectively.
the total number of players in academy E is 96%
2: Number of swimmers in academy A is 2 more of those in academy C, then find that the sum of
than those in academy B and the number of
the numbers of cricketers, footballers and
swimmers in academy C is 2 less than those in
swimmers in academy E is what per cent of the
academy D.
total number of players in academy E?
6) If the average of the number of cricketers in
a) 62.5%
academies A, C and D is P, the average of the b) 75%
number of footballers in academies C, D and E is
c)66.67%
Q and the average of the number of swimmers in
d) 60%
academies A, B and E is R, then find that the
e) 50%
value of (P + Q – R) is divisible by which of the
following?
9) Which of the following is/are true?
a) 11

Click Here For Bundle PDF Course | support@guidely.in Page 3 of 14


SBI Clerk & RRB PO Mains PDF Course 2023
Quantitative Aptitude Day - 43 (Eng)

X: Average of the number of cricketers, of swimmers in academies B and D together is


footballers and swimmers in academy A is 33. 15: 7.
Y: The total number of swimmers in academies a) All are true
D and E together is 25% of the total number of b) Only X and Y
cricketers in academies D and E together. c)Only X and Z
Z: Ratio of the total number of footballers in d) Only X
academies B and D together to the total number e) None is true

Direction (10-13): Study the following data carefully and answer the questions:
There are four tanks (A, B, C and D) of different capacities. Each tank has a circular cross section
(circular whole) in the bottom and the areas (in cm2) of circular cross sections of each tank are different.
The pie chart given below shows the degree distribution of area of the circular whole of each tank.

Note:
1: Flowing speed (in cm/s) of water from circular whole of a tank is 10% of the volume (in L) of the tank
and all the tanks are fully filled with water.
2: Flowing speed of water from the circular whole of tank A is 15 cm/s and the volume of water flowing
from tank A in 1 second is 240 cm3. (1 cm3 = 0.001 L)
10) If the flowing speed of water from tank B is in tank A after 5 minutes to the volume of water
of that of the flowing speed of water from left in tank B after 5 minutes?
tank A, then find the ratio of volume of water left a) 39: 17
b) 39: 40

Click Here For Bundle PDF Course | support@guidely.in Page 4 of 14


SBI Clerk & RRB PO Mains PDF Course 2023
Quantitative Aptitude Day - 43 (Eng)

c) 26: 27 b) 4: 3
d) 6: 7 c)2: 3
e) 2: 3 d) 8: 7
e) 4: 5
11) If the volume of water flowed from tanks C
and D in 1 second is respectively 10 cm3 and Direction (14-17): Study the following data
400 cm3 more than the volume of water flowed carefully and answer the questions:
from tank A in 1 second, then find the average of Data given below is related to the total number of
flowing speed of water from tank C and that from students in 4 classes A, B, C and D and the
tank D? number of students from each class, who were
a) 20.5 cm/s present on a particular Day.
b) 24.5 cm/s Three equations I, II and III are given below:
c)21.5 cm/s I: 3x + 2y – z = 119
d) 23.5 cm/s II: 3y – 2z = 36
e) 22.5 cm/s III: 3x – y = 68
The total number of students in class A is 50%
12) If the volume of water flowed from tank B in 1 more than ‘x’, the total number of students in
second is 330 cm3 and the volume of water class B is 4 times of ‘z’, the total number of
flowed from tank C in 1 second is 200 cm3, then students in class C is 50% more than ‘y’ and the
find that difference between the volume of tank B total number of students in class D is .
and that of tank C? Number of students present in class A is half of
a) 40 L those in class B and the number of students
b) 75 L present in class B is 120% of (x + z). Number of
c)50 L students present in class C is 75% of those in
d) 80 L class D and the number of students present in
e) 60 L class D is 80% of (x + z).
14) If M% of the total number of students in class
13) If the volume of water flowed from tank D in 1 A were absent and N% of the total number of
second is 640 cm3, the volume of tank E is 50 L students in class D were absent, then find that
more than that of tank D and the volume of water which of the following is/are true?
flowed from tank E in 1 second is 600 cm3, then P: M and N are co-prime with each other.
find the ratio of the area of circular whole of tank Q: Value of (M – N)2 is divisible by 22.
D to that of tank E? R: Value of (M + N) is divisible by 16.
a) 2: 1

Click Here For Bundle PDF Course | support@guidely.in Page 5 of 14


SBI Clerk & RRB PO Mains PDF Course 2023
Quantitative Aptitude Day - 43 (Eng)

a) All are true average of total number of students on classes


b) Only Q and R A, B, C, D and E?
c) Only Q a) 41
d) Only R b) 44
e) None is true c)40
d) 43
15) If the ratio of the total number of boys to total e) 42
number of girls in class C is 6: 5 and the ratio of
the boys to girls present in class C is 5: 4, then Direction (18-20): In each question, two
find that what per cent of total number of girls in quantities I and II are given. Compare the two
class C were absent? quantities on their basis: (Only quantity is to be
a) 25% compared)
b) 16.67% 18) Quantity I: Ratio of time taken by train A to
c)20% cross train B, while running in opposite direction,
d) 12.5% to the time taken by train A to cross train B, while
e) 15% running in the same direction, is 1: 7. Find that
the speed of train B is what per cent of that of
16) The ratio of present boys to present girls in train A?
class B on the given day is 1: 2. If the ratio of Quantity II: x2 – 79x + 370 = 0
present boys in class B on the given day to those a) Quantity I > Quantity II
in class B on the next day is 2: 3 and the ratio of b) Quantity I < Quantity II
present girls in class B on the given day to those c)Quantity I ≥ Quantity II
in class B on the next day is 2: 1, then find that d) Quantity I ≤ Quantity II
what per cent of the total number of students in e) Quantity I = Quantity II or the relation can’t be
class B were present on the next day? established
a) 70%
b) 75% 19) A, B and C together can complete a work in
c)60% 6 hours, A and B together can complete the
d) 65% same work in 12 hours and B and C together can
e) 80% complete the same work in 9 hours.
17) If the average of the number of students Quantity I: In what time, A and C together can
present in classes A, B, C, D and E is 31 and the complete the work.
average of the number of students absent in Quantity II: If A works with 150% of its original
classes A, B, C, D and E is 11, then find the efficiency and B works with 112.5% of its original

Click Here For Bundle PDF Course | support@guidely.in Page 6 of 14


SBI Clerk & RRB PO Mains PDF Course 2023
Quantitative Aptitude Day - 43 (Eng)

efficiency, then in what time A and B together will Quantity I: If the cost of pure milk is ₹45 per L
complete the work? and 6 L milk and 4 L water is added to the
a) Quantity I > Quantity II mixture, then what will be the cost of the new
b) Quantity I < Quantity II mixture?
c) Quantity I ≥ Quantity II Quantity II: If the quantity of milk in mixture is 31
d) Quantity I ≤ Quantity II L, then what will be the cost of pure milk?
e) Quantity I = Quantity II or the relation can’t be a) Quantity I > Quantity II
established b) Quantity I < Quantity II
c) Quantity I ≥ Quantity II
20) In a mixture of milk and water, quantity of d) Quantity I ≤ Quantity II
milk is 22 L more than that of water and the cost e) Quantity I = Quantity II or the relation can’t be
of the mixture is ₹32.4 per L. established
Click Here to Get the Detailed Video Solution for the above given Questions
Or Scan the QR Code to Get the Detailed Video Solutions

Answer Key with Explanation

Direction (1-5): So,


Let the amounts invested in schemes A, B, C, D
and E are ₹12.5x, ₹30x, ₹27.5x, ₹10x and ₹20x
900xy – 843.75xy = 54000
respectively.
xy = 960 ---------------------(1)
Also let the rates of interest in schemes A, B, C,
Since, the difference between SI and CI received
D and E are 22.5y%, 15y%, 20y%, 17.5y% and
from scheme E after 2 years is ₹960.
25y% respectively.
So,
Since, the SI received from scheme B after 2
years is ₹540 more than that received from
scheme A after 3 years. xy2 = 768 ---------------------(2)

Click Here For Bundle PDF Course | support@guidely.in Page 7 of 14


SBI Clerk & RRB PO Mains PDF Course 2023
Quantitative Aptitude Day – 43 (Eng)

From equations (1) and (2):


y = 0.8, x = 1200
So, the SI received from scheme F after 3 years
= 13920 – 3120 = ₹10800
And the amount invested in scheme F =
= ₹24000
Since, the total amount invested in schemes C
and D together:
1. Answer: A 33000 + 12000 = ₹45000
Since, the difference between SI and CI received Required percentage = = 53.33%
from scheme A after 2 years:

3. Answer: D
And the difference between SI and CI received Since, the original amount invested in scheme B
from scheme D after 2 years: = ₹36000
So, the new amount invested in scheme B would
be = 36000 – 1000 = ₹35000
So, m: n = 486: 235.2 = 405: 196
Since, the rate of interest in scheme B = 12%
From P:
So, the new rate of interest in scheme B would
The value of (m + n) = 405 + 196 = 601
be = of 12 = 14%
Since, 601 is not divisible by 41.
Since, the original SI received from scheme B
So, P is not true.
after 1 year:
From Q:
The value of (m – n) = 405 – 196 = 209
Since, 209 is divisible by 11. And the new SI received from scheme B after 1

So, Q is true. year would be:

From R:
Since, ‘m’ is an odd number and ‘n’ is an even So, required percentage = =
number. 13.4% (approx.)
So, R is not true.
Hence, only Q is true. 4. Answer: B
Since, the SI received from scheme C after 2
2. Answer: C years:
Since, the total SI received from schemes C and
D together after 2 years:

Click Here For Bundle PDF Course | support@guidely.in Page 8 of 14


SBI Clerk & RRB PO Mains PDF Course 2023
Quantitative Aptitude Day – 43 (Eng)

And the SI received from scheme E after 3 Since, the number of swimmers in academy C is
years: 2 less than those in academy D.
So,
3y – 5x = 2 ------------(2)
So, the difference between SI and CI received
By equation (1) * 3 + equation (2) * 4:
on ₹(M + N) at 15% rate after 2 years:
27x – 12y + 12y – 20x = 6 + 8
x=2
From equation (1):
5. Answer: A y=4
Quantity I:
Since, the total amount invested in schemes D
and E together = 12000 + 24000 = ₹36000
So, the SI received from scheme A would have
been:

Quantity II:
Since, the total amount invested in schemes A
and C together = 15000 + 33000 = ₹48000 6. Answer: A
So, the SI received from scheme B would have The average of the number of cricketers in
been: academies A, C and D:

Hence, Quantity I > Quantity II The average of the number of footballers in


academies C, D and E:
Direction (6-9):
Let the number of swimmers in academies A and
The average of the number of swimmers in
C are 9x and 5x respectively.
academies A, B and E:
Also let the number of swimmers in academies
B, D and E are 4y, 3y and 2y respectively.
Since, the number of swimmers in academy A is The value of (P + Q – R) = 41 + 17 – 14 = 44

2 more than those in academy B. And the value of (P + Q – R) is divisible by 11

So, only.

9x – 4y = 2 ------------(1)
7. Answer: C

Click Here For Bundle PDF Course | support@guidely.in Page 9 of 14


SBI Clerk & RRB PO Mains PDF Course 2023
Quantitative Aptitude Day – 43 (Eng)

The number of male cricketers in academy D = Required percentage = = 25%


= 30 So, Y is true.
The number of male footballers in academy D = From Z:
=6 Ratio of the total number of footballers in
The number of male swimmers in academy D = academies B and D together to the total number
=6 of swimmers in academies B and D together:

Required average = = 14 (44 + 16): (16 + 12) = 60: 28 = 15: 7


So, Z is true.

8. Answer: E Hence, all are true.

Since, the sum of the numbers of cricketers,


footballers and swimmers in academy C = 30 + Direction (10-13):

15 + 10 = 55 Since, the flowing speed of water from tank A =

So, the total number of players in academy C = 15 cm/s

= 125 And the volume of water flowed from tank A in 1


second = 240 cm3
And the total number of players in academy E =
So, the area of circular whole of tank A = =
96% of 125 = 120
Since, the sum of the numbers of cricketers, 16 cm2

footballers and swimmers in academy E = 32 + The area of circular whole of tank B = =

20 + 8 = 60 22 cm2

So, the required percentage = = 50% The area of circular whole of tank C = =
10 cm2

9. Answer: A The area of circular whole of tank D =

From X: = 32 cm2
Average of the number of cricketers, footballers 10. Answer: A
and swimmers in academy A: Since, the flowing speed of water from tank A =
15 cm/s
So, the flowing speed of water from tank B =
So, X is true.
of 15 = 10 cm/s
From Y:
The volume of water flowed from tank A in 1
The total number of swimmers in academies D
second = 240 cm3 = 0.24 L = 240 ml
and E together = 12 + 8 = 20
The volume of water flowed from tank B in 1
The total number of cricketers in academies D
second = 22 * 10 = 220 cm3 = 0.22 L = 220 ml
and E together = 48 + 32 = 80

Click Here For Bundle PDF Course | support@guidely.in Page 10 of 14


SBI Clerk & RRB PO Mains PDF Course 2023
Quantitative Aptitude Day – 43 (Eng)

The volume of water in tank A = = 150 So, the flowing speed of water from tank B =
L = 15 cm/s
The volume of water in tank B = = 100 And the volume of tank B = = 150 L
L If the volume of water flowed from tank C in 1
The volume of water left in tank A after 5 second = 200 cm3
minutes: So, the flowing speed of water from tank C =
= 20 cm/s

The volume of water left in tank B after 5 And the volume of tank C = = 200 L

minutes: Required difference = 200 – 150 = 50 L

13. Answer: B
Required ratio = 78: 34 = 39: 17
Since, the volume of water flowed from tank D in
1 second = 640 cm3
11. Answer: E
So, the flowing speed of water from tank D =
Since, the volume of water flowed from tank A in
= 20 cm/s
1 second = 240 cm3
The volume of tank D = = 200 L
So, the volume of water flowed from tank C in 1
The volume of tank E = 200 + 50 = 250 L
second = 240 + 10 = 250 cm3
And the flowing speed of water from tank E =
And the volume of water flowed from tank D in 1
10% of 250 = 25 cm/s
second = 240 + 400 = 640 cm3
Since, the volume of water flowed from tank E in
Since, the area of circular whole of tank C = 10
1 second = 600 cm3
cm2
So, the area of circular whole of tank E = =
So, the flowing speed of water from tank C =
24 cm2
= 25 cm/s
Required ratio = 32: 24 = 4: 3
Since, the area of circular whole of tank D = 32
cm2
Direction (14-17):
So, the flowing speed of water from tank D =
By equation (II) + equation (III):
= 20 cm/s
3x + 2y – 2z = 104 -------------(IV)
Required average = = 22.5 cm/s
By equation (I) – equation (IV)
3x + 2y – z – 3x – 2y + 2z = 119 – 104
12. Answer: C
z = 15
If the volume of water flowed from tank B in 1
From equation (II):
second = 330 cm3
3y – 30 = 36

Click Here For Bundle PDF Course | support@guidely.in Page 11 of 14


SBI Clerk & RRB PO Mains PDF Course 2023
Quantitative Aptitude Day – 43 (Eng)

y = 22 From P:
From equation (III): Since, there is a common factor of M and N,
3x – 22 = 68 which is 4.
x = 30 So, P is not true.
The total number of students in class A = 150% From Q:
of 30 = 45 Value of (M – N)2 = (40 – 28)2 = 144
The total number of students in class B = 4 * 15 Since, 144 is not divisible by 22.
= 60 So, Q is not true.
The total number of students in class C = 150% From R:
of 22 = 33 Value of (M + N) = 40 + 28 = 68
The total number of students in class D = Since, 68 is not divisible by 16.
= 50 So, R is not true.
The number of present students in class B = Hence, none is true.
120% of (30 + 15) = 54
The number of present students in class A = = 15. Answer: C

27 Total number of students in class C = 33

The number of present students in class D = So, the total number of girls in class C =

80% of (x + z) = 36 = 15
The number of present students in class C = The number of students present in class C = 27
75% of 36 = 27 So, the number of girls present in class C =
= 12
And the number of girls absent in class C = 15 –
12 = 3
Required percentage = = 20%

16. Answer: B
14. Answer: E Since, the number of students present in class B
The total number of students in class A = 45 on the given day = 54
The number of absent students in class A = 18 So, the number of boys present in class B on the
So, M = = 40% given day = = 18
The total number of students in class D = 50 The number of girls present in class B on the
The number of absent students in class D = 14 given day = 54 – 18 = 36
So, N = = 28%

Click Here For Bundle PDF Course | support@guidely.in Page 12 of 14


SBI Clerk & RRB PO Mains PDF Course 2023
Quantitative Aptitude Day – 43 (Eng)

The number of boys present in class B on the time taken by train A to cross train B, while
next day = = 27 running in the same direction, is 1: 7.
And the number of girls present in class B on the So, the ratio of the relative speed, when both the
next day = = 18 trains are running in opposite directions, to the

Since, the total number of students in class B = relative speed, when both the trains are running

60 in the same direction = 7: 1

And the number of students present in class B And,

on the next day = 27 + 18 = 45


So, the required percentage = = 75%
x + y = 7x – 7y
6x = 8y
17. Answer: E x: y = 4: 3
Since, the average of the number of students Required percentage = = 75%
present in classes A, B, C, D and E = 31
From quantity II:
So, the number of students present in class E =
x2 – 79x + 370 = 0
(5 * 31) – (27 + 54 + 27 + 36) = 11
x2 – 5x – 74x + 370 = 0
Since, the average of the number of students
x(x – 5) – 74(x – 74) = 0
absent in classes A, B, C, D and E = 11
x = 5, 74
So, the number of students absent in class E =
Hence, Quantity I > Quantity II
(5 * 11) – (18 + 6 + 6 + 14) = 11
And the total number of students in class E = 11
19. Answer: B
+ 11 = 22
Required average = = 42

18. Answer: A
Let the speeds of trains A and B are ‘a’ and ‘b’ By equation (1) – equation (2):
respectively.
So, the relative speed, when both the trains are
C = 12
running in opposite directions = (x + y)
From equation (3):
And the relative speed, when both the trains are
running in the same direction = (x – y)
Since, the ratio of time taken by train A to cross B =36
train B, while running in opposite direction, to the From equation (2):

Click Here For Bundle PDF Course | support@guidely.in Page 13 of 14


SBI Clerk & RRB PO Mains PDF Course 2023
Quantitative Aptitude Day – 43 (Eng)

So, the quantity of milk in the mixture = (x + 22)


L
A = 18
Also let the cost of pure milk = ₹’y’ per L
Quantity I:
So,
Part of work complete by A and C together in 1
hour =
So, the time, in which A and C together will Quantity I:
complete the work = 7.2 hours Since, the cost of pure milk = y = ₹45 per L
Quantity II: From equation (1):
The time, in which A alone will complete the
work with 150% of its original efficiency =
45x + 990 = 64.8x + 712.8
= 12 hours
x = 14
The time, in which B alone will complete the
If 6 L milk and 4 L water is added to the mixture:
work with 112.5% of its original efficiency =
So, the cost of the new mixture:
= 32 hours
Part of work completed by A and B together with
their new efficiencies: Quantity II:
If the quantity of milk = 31 L
So, the quantity of water = 31 – 22 = 9 L
So, the time, in which A and B together will
From equation (1):
complete the work with their new efficiencies =
= 8.72 hours
Hence, Quantity I < Quantity II y = ₹41.8 per L (approx.)
Hence, Quantity I < Quantity II

20. Answer: B
Let the quantity of water in the mixture = ‘x’ L

Click Here For Bundle PDF Course | support@guidely.in Page 14 of 14


SBI Clerk & RRB PO Mains PDF Course 2023
ENGLISH Day - 43

English Language

Directions (1-7): Read the given passage and took Narasimha Rao and Manmohan Singh as
answer the following questions based on the Prime Ministers to understand and reform the
passage. A word is highlighted to help you locate Indian economic system, to reduce state
while answering the questions. participation and increase incentives for capital
The National Statistical Office (NSO) has and labour providers, thus achieving a higher
released the 2022-23 GDP fourth-quarter growth and faster growth of the economy. What is
rate figures. Measured against fourth-quarter alarming today is the serious and continuous
figures of the previous year, the data give a decline in GDP growth rates which began in
gloomier picture than what the media 2016. And that decline continues even now. The
publications of the Press Information Bureau Modi government has failed to structure
present. According to NSO data, in the first economic policy coherently and which has
COVID-19 pandemic quarter of 2020-21, i.e., prevailed during the period 2014-2023.
April 1 to June 30, 2020, GDP growth rate was The growth rate of GDP has been consistently in
minus 23.8% when compared to GDP of the decline since 2016. There are also the brazen
same period in 2019-20. Three conclusions announcements of rosy predictions being
based on NSO data since 2014-2015 are published annually in the media, with outrageous
important for a reality check. First, the growth claims made by the Prime Minister such as $5
rate of GDP, since 2015-16 had been declining trillion GDP by 2024 (announced in 2019),
annually, and has fallen in the fourth quarter to implying an annual doubling of GDP in five years,
what it was earlier, and sneeringly referred to by or, in other words, a 15% annual growth rate of
economists as “The Hindu Rate of Growth” — GDP. No policy structuring has been presented.
3.5% growth rate in GDP. Second, it is essential By “structuring”, this writer means a clear
to recognise that since 2014, Prime Minister implementation of what the economic objectives
Narendra Modi’s widely publicised “Vikas” in will be, and priorities that should be assigned to
reality achieved the so-called “Hindu rate of the various objectives. Thereafter, there ought to
growth” in GDP of what had been achieved in the be a strategy on what should be incentivised and
period 1950-77 — the socialism period. Third, what should be deleted or discontinued. For
during the tenures of P.V. Narasimha Rao and example, in today’s dark economic condition, it is
Manmohan Singh, GDP growth rates rose for the essential that personal income tax is abolished
first time to between 6% to 8% per year over a and Goods and Services Tax scrapped to
15-year period, i.e., 1991-96 and 2004-2014 incentivise investors and earners. Resources by
(with the usual cyclic ups and downs). That is, it the government should be mobilised through

Click Here For Bundle PDF Course | support@guidely.in Page 1 of 12


SBI Clerk & RRB PO Mains PDF Course 2023
ENGLISH Day - 43

indirect taxes and also by liberal printing of Deregulations should also not mean that we
currency notes and which is circulated by paying reject government intervention for safety nets,
wages to the employment generated in extensive affirmative action, market failure and creating a
public works. The annual interest paid on fixed- level-playing field. Democratic institutions have to
term savings in bank accounts should be 9% or be empowered to guard against public disorder
so to increase purchasing power of the middle arising from rapid de-regulation — as it
classes. Interest rates on loans to small and happened in Russia post-1991. Russia
medium industries should be no more than 6% of experienced chaos and misery. This dictatorship
the loans to increase production of these sectors, has returned for the Russians, and with it a
and thus employment. This writer is prepared to complete loss of human rights and democratic
have a public debate with any government official values.The trade-off between the public sector
and prove that “Modinomics” is an unstructured and de-regulation and the sale of loss-making
and gigantic flop. No macroeconomic goal that units, increasing employment, through affirmative
has been announced has been reached to date. action, and easy access to social security and a
India needs a new economic policy urgently. It safety net are essential to create a stake for the
needs to be a policy that is based on clear poor in the system. This creates a level-playing
objectives, priorities, have a strategy to achieve field in a competitive system, ensures
targets, and spell out an intelligent and transparency, accountability, and trusteeship
transparent resource mobilisation plan to finance (philanthropy), as well as corporate governance
policies. As far as the Finance Ministry is to legitimise profit-making smoothly which drives
concerned, we have only incoherent public the market system. Such steps reduce
announcements — a hotchpotch — with no monopolistic tendencies and help in the
accountability. The market system is not a free- formation of a democratic and harmonious
for-all or an ad hoc measure. It has a structure society.
with rules for transactions. Market system 1) How has been the GDP growth since 2016 as
capitalism works as the principal drivers are per the information given in the passage ?
incentive and capital (whose use for innovation a) the GDP growth rate has been consistently
raises factory productivity and the growth rate of declining since 2016
GDP). Even a totalitarian state such as China b) the GDP growth rate has been consistently
understood this. During the tenure of Deng increasing since 2016
Xiaoping as the supremo, it allowed the socialist c) the GDP growth rate has been consistent and
economic system to die and an economic same since 2016
market-based system came in. d) the GDP growth rate has been varying and
showing on and off pattern since 2016

Click Here For Bundle PDF Course | support@guidely.in Page 2 of 12


SBI Clerk & RRB PO Mains PDF Course 2023
ENGLISH Day - 43

e) None of these I. The 2022-23 GDP fourth-quarter growth rate


figures that the media of PIB published were
2) What is the main reason for the decline in appropriate and true.
GDP growth rates during the Modi tenure that II. Democratic institutions in Russia were not
has been mentioned in the passage ? empowered back in 1991 which resulted in
a) the government has failed to follow the same dictatorship.
policy of the other Prime Ministers III. A good economic policy must be framed on
b) the government has failed to put proper the basis of clear objectives and strategies along
officials in place in the finance ministry with transparency.
c) the government has failed to change its a) Only I
monetary policy based on the prevailing trends b) Only III
d) the government has failed to structure c) Both I and II
economic policy coherently d) Both II and III
e) None of these e) All I, II and III

3) According to the passage, which of the 5) What are some of the policy suggestions
following statements is/are true ? made by the author to address India’s dark
I. Social security and safety nets help to reduce economy ?
monoplayers in the economy. I. To completely abolish Goods and Services tax.
II. During the Narasimha Rao and Manmohan II. The rate for fixed deposits must be no more
Singh period, the GDP rates rose between 6% - than 6%.
8%. III. The interest rates on loans to MSME must be
III. Market capitalism paves way for productivity lower to empower them.
and improving GDP rate. a) Both I and II
a) Only II b) Both I and III
b) Both I and II c) Both II and III
c) Both I and III d) All I, II and III
d) Both II and III e) None of these
e) All I, II and III
6) SYNONYM of the word BRAZEN as
4) According to the passage, which of the mentioned in the passage.
following statements is/are false ? a) impudent
b) timid
c) shy

Click Here For Bundle PDF Course | support@guidely.in Page 3 of 12


SBI Clerk & RRB PO Mains PDF Course 2023
ENGLISH Day - 43

d) humble at Chushul as part of ongoing efforts to resolve


e) meek the standoff in eastern Ladakh, defence sources
confirmed.
7) What is the tone of the author ? (F) The above entails restoration of patrolling
a) apologetic rights till the traditional patrolling points, a
b) informative defence source said.
c) emotional 8) Which of the following sentences is the ODD
d) biased ONE OUT ?
e) skeptical a) A
b) B
Directions (8-12) : Six sentences are given below c) C
out of which one sentence is either irrelevant or d) D
inappropriate (odd one out). Rearrange the e) F
remaining five sentences in a proper sequence
so as to form a coherent paragraph and then 9) Which of the following is the first sentence
answer the questions given below. after the rearrangement ?
(A) The Indian stance has been consistent, i.e. a) A
restoration of status quo ante as of April 2020 b) B
and the focus would be on disengagement from c) C
Depsang and Demchok. d) E
(B) Like in the earlier round, the Indian side for e) F
the talks will be led by Lt. Gen. Rashim Bali, the
Leh-based 14 Corps Commander. 10) Which of the following is the last sentence
(C) The talks come less than a month before the after the rearrangement ?
G20 leaders summit for which expectations are a) A
running high that Chinese President Xi Jinping b) B
would attend in person. c) C
(D) Ahead of the G20 summit in New Delhi on d) D
September 9 and 10, Prime Minister Narendra e) E
Modi and Mr. Xi are scheduled to attend the
BRICS Summit in Johannesburg on August 22- 11) Which of the following is the fourth sentence
24. after the rearrangement ?
(E) India and China are set to hold the 19th a) A
round of Corps Commander talks on August 14 b) B

Click Here For Bundle PDF Course | support@guidely.in Page 4 of 12


SBI Clerk & RRB PO Mains PDF Course 2023
ENGLISH Day - 43

c) C a) facing, feature
d) E b) battling, commodity
e) F c) running, creature
d) leading, item
12) Which of the following is the second e) No changes required
sentence after the rearrangement ?
a) A 15) The estimates show that there are over 4,000
b) B hostels and paying guest building in Chennai and
c) C the industry is begged at over ₹ 2,000 crore.
d) D a) around, based
e) E b) hotels, bugged
c) accommodations, pegged
Directions (13-17) : In each of the following d) flats, counted
questions given below, there is a sentence in e) No changes required
which two words are highlighted. They may or
may not be grammatically and contextually 16) The weight, texture, and scent of paper
correct. Choose the best alternatives among the create a sensitive experience that digital screens
four given options to replace those words. If no cannot repeated.
replacement is required choose option (e) as a) sensible, replica
your answer. b) sensor, repeat
13) The State government appointed 24 Vice- c) sensation, replicated
Chancellors, reportedly without consulting the d) sensory, replicate
Governor, whose validity was challenged by a e) No changes required
writ petition.
a) consenting, period 17) As per the existing documents between India
b) asking, posting and China, operation of fighter aircraft and armed
c) telling, tenure helicopters is ban to a distance from the Line of
d) enquiring, identity Actual Control.
e) No changes required a) agreements, restricted
b) talks, allowing
14) Since 2006, the world has been batting a c) discussions, unlimited
shortage of helium, which despite being the d) problems, hinder
second most abundant element in the universe, e) No changes required
is a rare comorbidity on earth.

Click Here For Bundle PDF Course | support@guidely.in Page 5 of 12


SBI Clerk & RRB PO Mains PDF Course 2023
ENGLISH Day - 43

Directions (18-22) : Read each question to find opened a new chapter in terms/C of its nature,
out whether they are erroneous , errors if any, character and consequences of Indian polity/D.
will be in two parts of the sentence. Choose the a) AB
option that represents the incorrect parts as your b) AC
answer. If there is no error, mark(e) as your c) BC
answer. d) CD
18) Even children studying in middle school is/A e) No error
getting too so much engaged in social media/B,
playing games leading to complete decline/C in 21) Looking back to the broader arc of the U.S.’s
their focus on academic tasks/D. economic trajectory/A, it is true that Mr. Obama
a) AB represented the mainstream consensus/B of
b) CD global, market-driven development, albeit by
c) AD healthy/C policy interventions by governments to
d) BC correct market failures in some areas/D.
e) No error a) AB
b) AC
19) Rehabilitation, in turn, does not simply c) BD
mean/A picking up people from one part of the d) CD
town/B and dumping them in another, but e) No error
ensuring/C that there is no substantial disruption
to their precarious lives/D. 22) This has given way to political expediency
a) AB and capital/A, the electoral advantages of
b) AC teaching a lesson to/B the minorities, for the
c) AD rising crescendo of hate-politics/C, all by an eye
d) BD on the next general election/D.
e) No error a) AC
b) BC
20) Though Hindu-Muslim violence is not new in c) BD
India,/A the one that erupted in Nuh district and d) CD
Gurgaon in Haryana/B has recently perhaps e) No error

Click Here For Bundle PDF Course | support@guidely.in Page 6 of 12


SBI Clerk & RRB PO Mains PDF Course 2023
ENGLISH Day - 43

Click Here to Get the Detailed Video Solution for the above given Questions
Or Scan the QR Code to Get the Detailed Video Solutions

Answer Key with Explanation

1. Answer: A All the given statements I, II and III are true


The GDP growth rate has been consistently according to the information given in the
declining since 2015-2016 and it continues to passage.
decline following the same pattern even now.
Information given in the passage : What is 4. Answer: A
alarming today is the serious and continuous The statement given in option ‘a’ is definitely
decline in GDP growth rates which began in false because according to the given passage
2016. And that decline continues even now. “the NSO released data on the 2022-23 GDP
From this we can infer that the rate has been fourth-quarter growth rate which was gloomier
declining since 2016. than what the media publications of the Press
Hence, option a is the right answer. Information Bureau presented to the public”.
This means the condition of the growth rate of
2. Answer: D GDP was worse than what was presented by the
From the given information in the passage we media - this signifies that the data presented
can understand that the main reason for the fall were not appropriate and had some
in GDP since 2016 is that there is no proper discrepancies.
“structured economic policy”
Only the statement given in option d has been 5. Answer: B
mentioned in the passage as the reason for the Both I and III are the suggestions made by the
fall in GDP. author to improve the current existing dark
economy.

3. Answer: E

Click Here For Bundle PDF Course | support@guidely.in Page 7 of 12


SBI Clerk & RRB PO Mains PDF Course 2023
ENGLISH Day - 43

II is wrong because the author has suggested to around the topic - 19th round of talks between
maintain 9% or more as interest rates for fixed India and China.
term deposits. Sentence D is considered as out of topic when
compared to the other given sentences.
6. Answer: A
Impudent is a synonym of the given bold word. 9. Answer: D
Brazen - without embarrassment, especially in a E is the first sentence.
way which shocks people The correct sequence of rearrangement is
Impudent - very rude; lacking respect and not ECAFB.
polite If you watch carefully only sentence D and E
Timid - shy and nervous/frightened appear to be the starter sentence and the rest
Shy - nervous and uncomfortable seem to be a continuation of one and another.
Humble - not proud But D seems to be a little deviated from the
Meek - quiet actual topic and hence it is considered as the
Brazen is to do something bold without any odd one out. So, the first sentence of the
embarrassment, a similar word would be coherent passage formed is E. E gives an
Impudent as the other given words are just the introduction about the 19th round of talks
mere opposite of the given bold word. between the powers India and China. E is
followed by C which gives more details on the
7. Answer: E talks that have been planned between the
The tone of the author seems to be skeptical. nations. The countries are going to discuss
Skeptical - doubtful restoration is what we can infer from sentence A
Apologetic - expressing a feeling of guilt and it precedes B and F. Following A comes F
Biased - favouring one side which talks about the restoration plan and the
The author is concerned about the economic final/last sentence would be B which gives an
policies in place and is doubtful about the add on information regarding the lead of the talk
existing structure and calls out for a new form India.
economic policy.
10. Answer: B
8. Answer: D B is the last sentence.
D is the odd one out sentence as it talks about The correct sequence of rearrangement is
G20 and BRICS but the other sentences revolve ECAFB.

Click Here For Bundle PDF Course | support@guidely.in Page 8 of 12


SBI Clerk & RRB PO Mains PDF Course 2023
ENGLISH Day - 43

If you watch carefully only sentence D and E talks that have been planned between the
appear to be the starter sentence and the rest nations. The countries are going to discuss
seem to be a continuation of one and another. restoration is what we can infer from sentence A
But D seems to be a little deviated from the and it precedes B and F. Following A comes F
actual topic and hence it is considered as the which talks about the restoration plan and the
odd one out. So, the first sentence of the final/last sentence would be B which gives an
coherent passage formed is E. E gives an add on information regarding the lead of the talk
introduction about the 19th round of talks form India.
between the powers India and China. E is
followed by C which gives more details on the 12. Answer: C
talks that have been planned between the C is the second sentence.
nations. The countries are going to discuss The correct sequence of rearrangement is
restoration is what we can infer from sentence A ECAFB.
and it precedes B and F. Following A comes F If you watch carefully only sentence D and E
which talks about the restoration plan and the appear to be the starter sentence and the rest
final/last sentence would be B which gives an seem to be a continuation of one and another.
add on information regarding the lead of the talk But D seems to be a little deviated from the
form India. actual topic and hence it is considered as the
odd one out. So, the first sentence of the
11. Answer: E coherent passage formed is E. E gives an
F is the fourth sentence. introduction about the 19th round of talks
The correct sequence of rearrangement is between the powers India and China. E is
ECAFB. followed by C which gives more details on the
If you watch carefully only sentence D and E talks that have been planned between the
appear to be the starter sentence and the rest nations. The countries are going to discuss
seem to be a continuation of one and another. restoration is what we can infer from sentence A
But D seems to be a little deviated from the and it precedes B and F. Following A comes F
actual topic and hence it is considered as the which talks about the restoration plan and the
odd one out. So, the first sentence of the final/last sentence would be B which gives an
coherent passage formed is E. E gives an add on information regarding the lead of the talk
introduction about the 19th round of talks form India.
between the powers India and China. E is
followed by C which gives more details on the 13. Answer: E

Click Here For Bundle PDF Course | support@guidely.in Page 9 of 12


SBI Clerk & RRB PO Mains PDF Course 2023
ENGLISH Day - 43

Both the highlighted words are correct and


appropriate. Hence, no replacement is required. 15. Answer: C
Consult - to ask somebody for some information Both ‘buidling’ and ‘begged’ are incorrect and
or advice need replacement.
Validity - the extent to which/the time period Accommodation and pegged is the right pair of
Consenting - to agree words that can be used to replace the given
Tenure - period highlighted words to make the sentence correct
and meaningful.
14. Answer: B Accommodations - places that can
The given words ‘batting’ and ‘comorbidity’ are accommodate people
wrong and definitely need replacement to make Pegged - to fix or keep something at a certain
the sentence correct and meaningful. level
Batting - cricket batting Other options are wrong because :
Comorbidity - presence of multiple medical Option a - both around and based are wrong and
conditions or diseases hence discarded
Both the above words are inappropriate to the Option b - hotels and bugged do not fit the
context of the sentence. sentence and hence discarded
Option b has the correct set of words - battling Option d - flats and counted do not add meaning
and commodity to the sentence
Battling - fighting/struggling The sentence : The estimates show that there
Commodity - a product or material that can be are over 4,000 hostels and paying guest
bought and sold accommodations in Chennai and the industry is
Other options are wrong because : pegged at over ₹ 2,000 crore.
Option a (facing, feature) - helium is not a
feature but an element 16. Answer: D
Option c (running, creature) - helium is not a Both the given words ‘sensitive’ and ‘repeated’
creature but an element are incorrect and inappropriate in the context of
Option d (leading, item) - leading and item both the sentence. Hence, the words need
are inappropriate replacement.
The sentence : Since 2006, the world has been Sensitive - showing that you are conscious of
battling a shortage of helium, which despite and able to understand people’s feelings/
being the second most abundant element in the easily upset, offended or annoyed
universe, is a rare commodity on earth.

Click Here For Bundle PDF Course | support@guidely.in Page 10 of 12


SBI Clerk & RRB PO Mains PDF Course 2023
ENGLISH Day - 43

Replace ‘sensitive’ with ‘sensory’ and ‘repeated’ aircraft and armed helicopters is restricted to a
with ‘replicate’ distance from the Line of Actual Control.
Sensory - connected with your physical senses
(sensory experience is correct) 18. Answer: A
Replicate - to copy something exactly (digital Both the parts A and B are erroneous.
screens cannot replicate paper books) A - students is a plural word so use ‘are’ instead
Other options are wrong because : of ‘is’
Option a - Both the given words causes B - ‘getting too so much engaged’ is incorrect,
grammatical error instead it must be ‘getting too much engaged’
Option b - sensor and repeat are incorrect words Parts C and D are correct and error free.
and hence cannot be used to replace The correct sentence : Even children studying in
Option c - both words are incorrect and cannot middle school are getting too much engaged in
be used in the sentence social media, playing games leading to complete
The sentence : The weight, texture, and scent of decline in their focus on academic tasks.
paper create a sensory experience that digital
screens cannot replicate. 19. Answer: E
17. Answer: A The given sentence is correct and error free.
The words ‘documents’ and ‘ban’ are not correct Hence, option e - no error is the correct answer.
in the given sentence and we need to replace
them to make the sentence correct and 20. Answer: D
meaningful. C and D are erroneous parts in the given
‘Agreements and restricted’ are the right words sentence.
to replace the highlighted words. C - ‘has recently’ is wrong, instead it must be
Hinder - to make something difficult ‘recently has’. The violence which has occurred
Other options are wrong because : recently has some effects - is the meaning.
Option b - allowing does not fit the sentence D - consequence on Indian polity not ‘of’
Option c - distance and unlimited - both the A and B are correct and appropriate.
words are wrong and incorrect The correct sentence : Though Hindu-Muslim
Option d - problems and hinder - both the words violence is not new in India, the one that erupted
are different from the given context of the in Nuh district and Gurgaon in Haryana recently
sentence has perhaps opened a new chapter in terms of
The sentence : As per the existing agreements its nature, character and consequences on
between India and China, operation of fighter Indian polity.

Click Here For Bundle PDF Course | support@guidely.in Page 11 of 12


SBI Clerk & RRB PO Mains PDF Course 2023
ENGLISH Day - 43

21. Answer: B 22. Answer: D


A and C are erroneous parts. The prepositions ‘for’ in C and ‘by’ in D are
Looking back to is wrong, instead it must be wrong causing grammatical errors.
‘looking back at’ Replace ‘for’ with ‘to’ in part c
Albeit by healthy policy is wrong - change the Replace ‘by’ with ‘with’ in part d
preposition (replace ‘by’ with ‘with’) With an eye - to watch someone/something very
Albeit - although closely/carefully
Although with healthy policy interventions. The correct sentence : This has given way to
The correct sentence : Looking back at the political expediency and capital, the electoral
broader arc of the U.S.’s economic trajectory, it advantages of teaching a lesson to the
is true that Mr. Obama represented the minorities, to the rising crescendo of hate-
mainstream consensus of global, market-driven politics, all with an eye on the next general
development, albeit with healthy policy election.
interventions by governments to correct market
failures in some areas.

Click Here For Bundle PDF Course | support@guidely.in Page 12 of 12


SBI Clerk & RRB PO Mains PDF Course 2023
Reasoning Ability Day - 44 (Eng)

Reasoning Ability
Directions (1-5): In each question, three d) Only a few Bajaj is TVS; All TVS is KTM; All
conclusions have been given followed by 5 sets Bajaj is Suzuki
of possible statements. You have to take the e) All the statements are true
given conclusions to be true even if they seem to
be at variance with the commonly known facts 3) Conclusions:
and then decide that the conclusions logically No Samsung is Xiaomi.
follow for which of the given statements. Some Nokia is Apple.
1) Conclusions: Some Samsung is not Apple.
All Hero can be Honda. Statements:
Some Royal Enfield can be Yamaha. a) Only a few Apple is Samsung; Some
All Royal Enfield can never be Hero. Samsung is Nokia; Some Nokia is Xiaomi
Statements: b) Only a few Samsung is Apple; All Samsung is
a) Only a few Honda is Yamaha; All Hero is Nokia; No Nokia is Xiaomi
Royal Enfield; All Royal Enfield is Honda c) Only a few Nokia is Apple; All Apple is Xiaomi;
b) Only a few Royal Enfield is Yamaha; Some All Xiaomi is Samsung
Yamaha is Hero; No Yamaha is Honda d) Only a few Samsung is Apple; All Apple is
c) Only a few Royal Enfield is Hero; Some Hero Nokia; Some Nokia is Xiaomi
is Yamaha; All Yamaha is Honda e) All the statements are true
d) Only a few Honda is Hero; Some Hero is
Yamaha; All Yamaha is Royal Enfield 4) Conclusions:
e) All the statements are true Some Honor being One plus is a possibility.
Some One plus is not Oppo.
2) Conclusions: Some Vivo can be Honor.
Some Suzuki is KTM. Statements:
Some Suzuki is not TVS. a) Only a few One plus is Vivo; No Vivo is Oppo;
Some TVS is Suzuki. Some Oppo is Honor
Statements: b) Only a few Vivo is One plus; No One plus is
a) Only a few KTM is TVS; Some TVS is Suzuki; Oppo; All One plus is Honor
No Suzuki is Bajaj c) Only a few One plus is Vivo; Some Vivo is
b) Only a few Bajaj is KTM; All KTM is TVS; All Oppo; Some Oppo is Honor
TVS is Suzuki d) Only a few Vivo is Honor; Some Honor is
c) Only a few Suzuki is TVS; Some TVS is KTM; Oppo; No Oppo is One plus
No KTM is Bajaj e) None of the statements is true

Click Here For Bundle PDF Course | support@guidely.in Page 1 of 9


SBI Clerk & RRB PO Mains PDF Course 2023
Reasoning Ability Day - 44 (Eng)

5) Conclusions: sit adjacent to the vacant chair. W and X are not


Some Huawei is Karbonn. sitting together.
Some Huawei is not Intex. 6) What is the position of V with respect to R?
No Micromax is Intex. a) Second to the right
Statements: b) Third to the left
a) Only a few Micromax is Karbonn; Some c) Immediate left
Karbonn is Intex; Some Micromax is Huawei d) Fourth to the right
b) Only a few Huawei is Karbonn; Some Karbonn e) None of these
is Intex; All Micromax is Huawei
c) Only a few Micromax is Karbonn; No Karbonn 7) How many persons are sitting between P and
is Intex; All Micromax is Huawei X?
d) Only a few Intex is Karbonn; Some Karbonn is a) Three
Huawei; Some Huawei is Micromax b) As many persons are sitting to the right of T
e) None of the statements is true c) One
d) As many persons are sitting to the left of S
Directions (6-10): Study the following information e) None of these
carefully and answer the below questions.
Nine persons – P, Q, R, S, T, U, V, W, and X are 8) Who among the following person sits third to
sitting in a linear row facing the north. There are the right of R?
twelve chairs in the row some of the chairs are a) The one who sits third to the left of X
vacant. Neither the chairs at the end nor the b) U
adjacent chairs are vacant. Chairs are marked 1 c) The one who sits second to the left of T
to 12 from left to right. Only one chair in an odd d) S
position is vacant. e) None of these
R sits at the fifth chair from the left end. Three
persons are sitting between R and X. Neither X 9) Which of the following person sits adjacent to
nor Q sits at the end. T sits in an even number the vacant chair?
chair. P sits third to the left of T but none of them I. S, T, and V
sits at the end. Only two vacant chairs are placed II. P, R, and X
between P and Q. Both Q and S are sitting III. W, R, and X
adjacent seat to neither X nor T. One person sits a) Only II
between T and V, who sits to the right of Q. Two b) Both II and III
persons are sitting between S and T. S doesn’t c) Only I
d) Both I and II

Click Here For Bundle PDF Course | support@guidely.in Page 2 of 9


SBI Clerk & RRB PO Mains PDF Course 2023
Reasoning Ability Day - 44 (Eng)

e) None of these e) DU

10) Which of the following statement is not true? 13) Which of the following statement is/are not
a) S sits third to the left of R true?
b) Two persons are sitting between U and Q I. K is the daughter of U’s brother
c) X sits second to the right of Q II. G is the mother-in-law of S’s mother.
d) Four persons are sitting to the left of T III. L is the father-in-law of T’s brother
e) All the above statements are true a) Both II and III
b) Only III
Directions (11-13): Study the following c) Only I
information carefully and answer the below d) Both I and II
questions. e) None of these
A family of three generations consists of nine
members and two married couples. S is the only Directions (14-15): Answer the questions based
daughter of T’s brother. R is the only sister-in-law on the information given below.
of the one who is the daughter of G. D is the only Eleven persons viz. D, E, F, G, H, I, J, K, L, M
son-in-law of L, who is the maternal grandfather and N are from the same family with three
of K. D and U are siblings but of the opposite generations, but not necessarily in the same
gender. E is the father of T, who is the sister of order. The number of females is one more than
U. R does not have a sibling. the number of males in the family. M is the only
11) If V is married to S, then how R is related to brother-in-law of L who is married to H. K is the
V? mother of H whereas J is the sister of L but is
a) Sister-in-law unmarried. D is the wife of M and mother of F
b) Grandfather who is the cousin of N.H is the only daughter of
c) Uncle G who is the grandfather of N. I is the wife of E
d) Mother-in-law who has only two children and one
e) None of these granddaughter.
14) Who among the following are the children of
12) Four of the five are related to each other and I?
thus form a group. find the odd one out? a) L, D
a) SD b) H, M
b) TE c) J, L
c) RL d) M, L
d) UE e) None of these

Click Here For Bundle PDF Course | support@guidely.in Page 3 of 9


SBI Clerk & RRB PO Mains PDF Course 2023
Reasoning Ability Day - 44 (Eng)

15) How is H related to M? a) M


a) Brother b) The one who sits immediate right of Q
b) Sister c) N
c) Sister-in-law d) The one who sits second to the right of S
d) Cannot be determined e) None of these
e) None of these
17) Who among the following person has the
Directions (16-20): Study the following maximum number of pens?
information carefully and answer the questions a) The one who sits third to the right of P
given below. b) O
Eight persons - L, M, N, O, P, Q, R, and S are c) The one who sits immediate right of the one
sitting around a square table facing towards the who has 35 pens
centre, but not necessarily in the same order. d) L
Four persons sit on the corners of the table and e) The one who has 62 pens
four persons sit in the middle of the sides of the
table. Each person has different number of pens. 18) Which of the following statement is false with
The one who has 31 pens sits third to the left of respect to the final arrangement?
N. N sits in the middle of the sides of the table. a) Both S and Q sit opposite to each other
Two persons sit between N and S. The one who b) P has 27 pens
has 19 pens neither sits adjacent to S nor N. R c) N sits immediate left of Q
sits opposite to the one who sits second to the d) Both R and O are immediate neighbors
right of the one who has 19 pens. M sits second e) M has the fewest pens
to the right of R. Both L and M are sitting
opposite to each other. The one who has 44 19) In which of the following statement(s), the
pens sits opposite to the one who sits second to first person does not sit exactly between the
the left of L. Q sits second to the right of the one second and the third persons in the final
who sits immediate right of R. O neither sits arrangement?
adjacent to L nor N. S has twice the number of I. MOQ
pens with O. N has nine pens more than L. P has II. PLN
four pens less than O and one pen more than L. III.SLO
Q has twice the number of pens with the one a) Only II
who sits immediate left of S. b) Both I and III
16) Who among the following person sits c) Only III
immediate right of R? d) All I, II, and III

Click Here For Bundle PDF Course | support@guidely.in Page 4 of 9


SBI Clerk & RRB PO Mains PDF Course 2023
Reasoning Ability Day - 44 (Eng)

e) None of these a) The one who has 62 pens


b) The one who sits immediate left of L
20) Four of the following five are alike in a certain c) The one who sits second to the left of P
way as per the given arrangement and hence d) The one who has 31 pens
form a group. Find the one that doesn’t belong to e) M
that group.
Click Here to Get the Detailed Video Solution for the above given Questions
Or Scan the QR Code to Get the Detailed Video Solutions

Answer Key with Explanation

1) Answer: C 3) Answer: B

4) Answer: A

2) Answer: D

5) Answer: E
None of the statements is true

Direction (6-10):

Click Here For Bundle PDF Course | support@guidely.in Page 5 of 9


SBI Clerk & RRB PO Mains PDF Course 2023
Reasoning Ability Day - 44 (Eng)

6) Answer: D  Both Q and S are sitting adjacent seat to


7) Answer: B neither X nor T.
8) Answer: A  One person sits between T and V, who
9) Answer: A sits to the right of Q.
10) Answer: C Since, V sits to the right of Q.
That means, in case (3) P sits third from
the left end, and V sits second to the left
of X, in case (3a) P sits second to the
We have: right of R, and V sits immediate right of X,
 R sits at the fifth chair from the left end. case (1) & case (2) are not valid.
 Three persons are sitting between R and Based on the above given information we have:
X.
 Neither X nor Q sits at the end.
That means, in case (1) X sits fourth from
the right end, in case (2) X sits third from
the right end, in case (3) X sits second
from the right end.
Based on the above given information we have:

Case (1) & case (2) are not valid as P sits third
to the left of T, and two vacant chairs are
between Q and P.
Again, we have:
Again, we have:  Two persons are sitting between S and T.
 T sits in an even number chair.  S doesn’t sit adjacent to the vacant chair.
 P sits third to the left of T but none of Since, S and X are not sitting together.
them sits at the end. That means, in case (3) S sits second
 Only two vacant chairs are placed from the left end, case (3a) is not valid.
between P and Q. Based on the above given information we have:

Click Here For Bundle PDF Course | support@guidely.in Page 6 of 9


SBI Clerk & RRB PO Mains PDF Course 2023
Reasoning Ability Day - 44 (Eng)

 R is the only sister-in-law of the one who


is the daughter of G.
 S is the only daughter of T’s brother.
 D is the only son-in-law of L, who is the
maternal grandfather of K.
 R does not have a sibling.
Based on the above given information we have:

Case (3a) is not valid as two persons are sitting


between S and T.
Again, we have:
 W and X are not sitting together.
That means, W sits immediate left of S.
Based on the above given information we have:

Again, we have:
 E is the father of T, who is the sister of U.
 D and U are siblings but of the opposite
Directions (11-13):
gender.
11) Answer: D
Based on the above given information we have:
12) Answer: E
13) Answer: C

Directions (14-15):
We have:
14) Answer: C
15) Answer: B

Click Here For Bundle PDF Course | support@guidely.in Page 7 of 9


SBI Clerk & RRB PO Mains PDF Course 2023
Reasoning Ability Day - 44 (Eng)

Directions (16-20):
16) Answer: D
17) Answer: E
18) Answer: C
19) Answer: C
20) Answer: E (All the persons sit on the corners
of the table, except option e)
Final Arrangement
Again we have,
 R sits opposite to the one who sits second
to the right of the one who has 19 pens.
 M sits second to the right of R.
 Both L and M are sitting opposite to each
other.
 The one who has 44 pens sits opposite to

We have, the one who sits second to the left of L.

 The one who has 31 pens sits third to the From the above condition, case 2a gets

left of N. eliminated.

 N sits in the middle of the sides of the


table.
 Two persons sit between N and S.
 The one who has 19 pens neither sits
adjacent to S nor N.
From the above condition, there are three
possibilities.

Click Here For Bundle PDF Course | support@guidely.in Page 8 of 9


SBI Clerk & RRB PO Mains PDF Course 2023
Reasoning Ability Day - 44 (Eng)

 N has nine pens more than L.


 P has four pens less than O and one pen
more than L.
 Q has twice the number of pens with the
one who sits immediate left of S.
From the above condition, case 1 shows the final
arrangement.

Again we have,
 Q sits second to the right of the one who
sits immediate right of R.
 O neither sits adjacent to L nor N.
 S has twice the number of pens with O.
From the above condition, case 2 gets
eliminated.

Again we have,

Click Here For Bundle PDF Course | support@guidely.in Page 9 of 9


SBI Clerk & RRB PO Mains PDF Course 2023
Quantitative Aptitude Day – 44 (Eng)

Quantitative Aptitude

Directions (01 - 05): Study the following information carefully and answer the questions given below.
The given bar graphs show the total marked price of a plate of veg Biryani and non-veg biryani and the
price of non-veg Biryani more than the price of veg Biryani in five different shops [ A, B, C, D, and E].

Note:
I. Shops A and B provided a 40% discount on the total marked price or up to Rs. 80 whichever is
minimum and shops C and D provide a 50% discount on the total amount or up to Rs.100.Whichever is
minimum
II. Shop E provided a 20% discount on the total price.
1) The making costs of veg biryani and non-veg e) 640
biryani in shop A is Rs. 120 and Rs. 150,
respectively. One day, one person buys five 2) The marked price of one plate of a panner
plates of veg and non-veg biryani, and another item in B is 20% less than that of veg biryani in
day, five different people buy one plate of veg B, and the marked price of one plate of a chicken
and another five people buy one plate non-veg item in B is 10% less than that of non-veg biryani
biryani. Find the difference between the profit in B. The difference between the selling price of
amount on two days of shop A. one plate of panner item and one plate chicken
a) 250 item in B is x. Find the sum of the marked price
b) 690 of veg biryani in shops C and D?[ discount
c) 720 percent is the same as veg and non veg biriyani
d) 490 of B]

Click Here For Bundle PDF Course | support@guidely.in Page 1 of 12


SBI Clerk & RRB PO Mains PDF Course 2023
Quantitative Aptitude Day - 44 (Eng)

a) 9x-83.2 shop F is 4:5. The total selling price of two plates


b) 6x+23.2 of veg biryani in shops F and D together is
c) 5x-13.2 Rs.684. Find the difference between the selling
d) 7x-53.2 prices of two plates of non-veg biryani in shops E
e) 3x+53.2 and F. [ shop F provides 20% discount].
a) 32
3) b) 39
Quantity I: Find the selling price of 1 plate of veg c) 37
biryani and 2 plates of non-veg biryani from shop d) 39
C? e) None of these
Quantity II: Find the selling price of 2 plates of
veg biryani and 1 plate of non-veg biryani from 5) The difference between the total price of one
shop D? plate of veg biryani and non-veg biryani together
a) 2*Quantity: I < 1.5*Quantity: II in shops E and B is y. Find out what is the
b) 2*Quantity: I ≥ 1.5Quantity: II possible marked price for one plate of non-veg
c) 3*Quantity: II ≥ 4*Quantity: I biryani from shops A and D together?
d) 5.5*Quantity: I > 6.4*Quantity: II I.13y+30, II.14y-10, III.12.5y+50
e) Quantity I = Quantity II or relation can't be a) All three
established b) None
c) only I and II
4) The ratio of the marked price of one plate veg d) only I and III
biryani price and the non-veg biryani price of e) None of these

Directions (06 - 09): Study the following information carefully and answer the questions given below.
The given pie chart shows the percentage distribution of the number of females in five different villages
[A, B, C, D, and E] together in the year.

Click Here For Bundle PDF Course | support@guidely.in Page 2 of 12


SBI Clerk & RRB PO Mains PDF Course 2023
Quantitative Aptitude Day - 44 (Eng)

The given pie chart shows the percentage distribution of the number of males in five different villages [A,
B, C, D, and E] together in 2023.

Note:
I. In the year 2022, the number of females in each village is 33.33% less than the number of males in
each village. The total number of people in village E in 2022 is 500, and the number of males in E in the
year 2023 is increased by 60% of number of males in 2022.
II. In the year 2023 the number of females in A and E is 50% and 25% more than the number of males in
the same village in the same year respectively. The number of females in B is half of the number of
males in B and the number of females in C and D is 10% less than the number of males in the same
village in the same year respectively.

Click Here For Bundle PDF Course | support@guidely.in Page 3 of 12


SBI Clerk & RRB PO Mains PDF Course 2023
Quantitative Aptitude Day - 44 (Eng)

6) Find which of the following is true? d) 66%


I. The difference between males and females in e) None of these
2023 in village C is 66.
II. The number of females increases 9) The average height of males in village D in
by166.66%in village A in 2023 than 2022. 2023 is 167 cm and the average height of
III. The number of males in 2023 in village A is females in village D in 2023 is 162 cm. Find the
133.33% of the number of males in 2023 in average height of all the people in village D in
village E. 2023?
a) only II a) 164.63
b) only III b) 163.35
c) only II and III c) 165.34
d) only I and III d) 166.35
e) only I and II e) None of these

7) The sum of males and females in village B in 10) A can complete a work in ______ days. B is
2021 is 902, and the number of females is 198 twice efficient than A. B and C together
less than the number of males in village B in complete the work in ______ days. A and C
2021. Find the difference in the increasing together complete the work in ______ days.
percentage of males and females in 2022 than in I. 20, 6, 60/7
2021 in village B? II. 24, 4.8, 6
a) 4% III. 30, 120/13, 120/7
b) 3% Find the which of following satisfies the blank?
c) 2% a) only I and II
d) 5% b) only II and III
e) 1% c) only I and III
d) all three
8) Out of the total male in village C in the year e) None of these
2023, 60% are literate and the total number of
literate people in village C in the year 2023 is Directions (11 - 14): Study the following
1224. Find the percentage of female illiterate in information carefully and answer the questions
village C in 2023? given below.
a) 12% A survey is conducted among the students of a
b) 25% college. The number of students who like only
c) 18% love stories is 40% less than the number of

Click Here For Bundle PDF Course | support@guidely.in Page 4 of 12


SBI Clerk & RRB PO Mains PDF Course 2023
Quantitative Aptitude Day - 44 (Eng)

students who like only action movies. The and comedy but not love stories. Then find the
number of students who like both love stories number of students who like comedy movies?
and action movies but not comedy movies is a) 4r-30
twice the number of students who like all three b) 2r-45
types of movies. The ratio of the number of c) 7r-37
students who like both love stories and action but d) 3r+5
not comedy and the number of students who like e) 5r-20
both action and comedy but not love stories is
3:2. The number of students who like both love 13) The average age of students who like only
stories and comedy but not action is two more comedy is 22 years, and the average age of
than the number of students who like both students who like only love stories is 19 years.
comedy and action but not love stories. The Find the average age of the students who like
number of students who like only comedies is only comedy and love stories?
20% more than the number of students who like a) 20
only action movies. The total number of students b) 23
who like the Love Story movie is 71. The number c) 24
of students who like only comedy movies is five d) 21
times the number of students who like both e) None of these
action and comedy movies but not love stories.
11) The ratio of males and females who like only 14) m number of students who did not like any
love stories is 2:3 and the ratio of males and movies. Find the percentage of students who like
females who like only action movie is 3:2. Find any movie out of the total number of students?
the total number of females who like only love 255, 3m, 204,68,136
story is what percent of the total number of a) 91.90%
males who like only action movies? b) 84.35%
a) 55% c) 88.87%
b) 50% d) 82.5%
c) 60% e) None of these
d) 40%
e) 59% 15) The boat covers a 120 km distance
downstream in 5 hours when the speed of the
12) If r is the sum of the number of students who boat is increased by 25% and the boat covers a
like both love stories and action but not comedy 100 km distance upstream in 5 hours when the
and the number of students who like both action speed of the boat increases by 50%. The boat

Click Here For Bundle PDF Course | support@guidely.in Page 5 of 12


SBI Clerk & RRB PO Mains PDF Course 2023
Quantitative Aptitude Day - 44 (Eng)

covers a 48 km distance in still water t hours at b) 105


its original speed. Find the distance cover by c) 122
boat in 3t hours upstream? d) 127
a) 108 e) None of these

Directions (16 - 18): Study the following information carefully and answer the questions given below.
The line graph shows the total number of government schools [primary + high], the number of
government high schools more than the number of government primary schools, the sum of primary and
high private schools, and the difference between primary and high private schools in five different cities.

Note:
I. Only in cities D and E number of primary private schools more than the number of high private schools.
II. Total government school = total government high + total government primary school, Total private
school = total private high +total private primary school, Total schools in city = total government schools +
total private schools
16) The average number of students in the total number of students studying in the
government primary school of city A is 150 and government school of city A?
the average number of students in the a) 7450
government high school of city A is 280. Find the b) 7960
c) 7290

Click Here For Bundle PDF Course | support@guidely.in Page 6 of 12


SBI Clerk & RRB PO Mains PDF Course 2023
Quantitative Aptitude Day - 44 (Eng)

d) 7460 c) 97
e) 7850 d) 91
e) None of these
17) Find which of the following is true?
I. The difference between the total government 19) Find the sum of the missing number of
schools in city B and total private schools in city series?
B is 25. Series I – 52,316,? ,11404,68428
II. The total number of schools in city C is 90. Series II- 5,10,47,94,131,?
III. The difference between the total government a) 2162
schools in city D and total private schools in city b) 2265
D is 29. c) 2542
a) only III true d) 2584
b) only II and II true e) None of these
c) only I true
d) all false 20) Both series are in the same logic. Find the
e) all true value of a?
Series I – 78, 84, 102, 105,126, c
18) The ratio of the number of government Series II- c, d, a, b, e
schools in E and F is 11:12, and the ratio of the a) 154
number of private schools in E and F is 23:25. b) 158
Find the total number of schools in city F? c) 168
a) 92 d) 126
b) 98 e) None of these

Click Here For Bundle PDF Course | support@guidely.in Page 7 of 12


SBI Clerk & RRB PO Mains PDF Course 2023
Quantitative Aptitude Day – 44 (Eng)

Click Here to Get the Detailed Video Solution for the above given Questions
Or Scan the QR Code to Get the Detailed Video Solutions

Answer Key with Explanation

Directions (01 - 05): The marked price of the chicken item is =


260*90/100=234
Difference of selling price is = [234-80]-
[144*60/100]=67.6=x
So, the required sum is =220+200=420=7x-53.2

3) Answer: D
Quantity I
1) Answer: C Required price = [220-100]+[280-100]*2=480
The total selling price of 1st day = [200*5+250*5]- Quantity II
80=2170 [discount amount is Rs.80] Required price = [200-100]*2 + [300-100]*1=400
The total selling price of 2nd day = [200*60/100] 5.5*Quantity I > 6.4*Quantity II
*5 + [250-80]*5=1450 [discount amount for veg
biryani is 40% and for non-veg biryani is Rs.80] 4) Answer: A
So, the profit of 1st day =2170-5*120-150*5=820 The total selling price of 2 plates of veg biryani in
The profit of 2nd day = 1450-5*120-150*5=100 D is = 200*2-100=300
So, the difference is 820-100=720 So, the total selling price of 2 plates of veg
biryani in F is =684-300=384
2) Answer: D Marked price 1 plate veg biryani in F is =
The marked price of the panner item is = [384*100/80]/2=480/2=240
180*80/100=144 The marked price for one plate of non-veg
biryani is = 240*5/4=300

Click Here For Bundle PDF Course | support@guidely.in Page 8 of 12


SBI Clerk & RRB PO Mains PDF Course 2023
Quantitative Aptitude Day – 44 (Eng)

So, required difference = [300*2]*80/100- So, the total male in all villages together in 2023
[280*2]*80/100=480-448=32 is
= [480/12] *100=4000
5) Answer: A So, male in A in 2023 is = 4000*16/100=640
So, value y = [220+280] *80/100-[180+260-80] Female in A in 2023 is = 640*150/100=960
=400-360=40 Similarly, we can calculate the other villages
Required sum = 250+300=550=13y+30,14y- also.
10,12.5y+50 For 2023 –

Directions (06 - 09):


The number of females is 33.33% less than the
number of males in the same village in the year
2022.
So, we can say, the ratio of male and female in
each village in 2022 is 3:2.
So, Male in E in 2022 is = 500*3/5=300 and
6) Answer: C
female in E is = 500-300=200
I. The difference between males and females in
So, the total number of females in all villages
2023 in village C is =[960-864]=96.
together is 200*100/10=2000
II. The number of females increases by
Female in A in 2022 = 2000*18/100=360
[600/360] *100=166.66% in village A in 2023
Male in A in 2022 = 360*3/2=540
than in 2022.
Similarly, we can calculate the other village
III. The number of males in 2023 in village A is
values for the year 2022.
[640/480]*100=133.33% of the number of males
For 2022 –
in 2023 in village E.
7) Answer: D
Male in 2021 =[902+198]/2=550
Female in 2021= [902-198]/2=352
So, male increase percentage = [660-550]
*100/550=20%
Female increase percentage = [440-352]
*100/352 =25%
Male in E in 2023 is = 300*160/100=480 So, difference is = 25%-20%=5%

Click Here For Bundle PDF Course | support@guidely.in Page 9 of 12


SBI Clerk & RRB PO Mains PDF Course 2023
Quantitative Aptitude Day – 44 (Eng)

8) Answer: B Number of students who like only love stories


Female literate is = 1224-960*60/100=648 =50x*60/100=30x
So, female illiterate percentage = [864- So, 30x+18x+9x+[12x+2]=71
648]*100/864=25% Or, 69x=69, x=1
number of students who like both comedy and
9) Answer: A action but not love stories is 12*1=12.
So, average height is = So, the number of students who like both love
[880*167+792*162]/[1672] stories and action but not comedy
=[146960+128304]/1672=164.63 movie is =18*1=18.
The number of students who like all three is
10) Answer: A 9*1=9
For I, The number of students who like only comedy is
A takes 20 days, so, B takes 20/2=10 days. =60*1=60
B and C complete 6 days, The number of students who like only action is =
So, C complete =[1/{(1/6)-(1/10)}]=1/{(2/30)=15 50
So, total work = LCM of 20, 10, 15 =60 Number of students who like only love stories
The efficiency of A and C is 3 and 4 respectively. =30*1=30
So, A and C take = 60/7 days.
So similarly, we can check others' values also.

Directions (11 - 14):


Let the number of students who like both
comedy and action but not love stories be 12x.
So, the number of students who like both loves
stories and action movie but not comedy
movie is 12x*3/2=18x.
The number of students who like all three is
18x/2=9x
The number of students who like only comedy is 11) Answer: C
= 12x*5=60x The number of females who like only love stories
The number of students who like only action is = is = 30*3/5=18
60x*100/120=50x The number of males who like only action is
50*3/5=30

Click Here For Bundle PDF Course | support@guidely.in Page 10 of 12


SBI Clerk & RRB PO Mains PDF Course 2023
Quantitative Aptitude Day – 44 (Eng)

So, required percentage = [18/30]*100=60% So, a=24-20=4


speed of the boat in still water is 8*2=16 and the
12) Answer: D speed of the stream is 4.
So, r= 18+12=30 So, t=48/16=3
The total number of people who like comedy is = So, required distance = 3*3*[16-4]=108 km
14+12+9+60=95=3r+5
Directions (16 - 18):
13) Answer: D Let the number of government primary schools
So, required average age = in A is 5x, and the number of government high
[60*22+19*30]/90=21 school in A is 6x.
So, 11x=33, x=3
14) Answer: A The number of government primary schools in A
So, is 5*3=15, number of government high school in
A is 6*3=18
The number of private primary school in A is =
[50-10]/2=20
So, m=17 The number of private high schools in A is =
So, the total number of students = [50+10]/1=30
71+12+60+50+17=210 Similarly, we can calculate others' values also.
So, required percentage =
[71+12+60+50]*100/210=91.90%

15) Answer: A
Let the speed of the boat in still water is 8x and
the speed of the stream is a.
So, the speed of the boat, when it increases by
16) Answer: C
25%, is =8x*125/100=10x
So, the total number of students =
The speed of the boat, when it increases by
15*150+18*280=7290
50%, is = 8x*150/100=12x
So, 120/10x+a=5
17) Answer: D
Or, 10x+a=24,
I. The difference between total government
100/12x-a=5, or, 12x-a=20
schools in city B and total private schools in city
So, 22x=44, x=2
B =[ 60-36]=24.

Click Here For Bundle PDF Course | support@guidely.in Page 11 of 12


SBI Clerk & RRB PO Mains PDF Course 2023
Quantitative Aptitude Day – 44 (Eng)

II. The total number of students in city C is


12+18+22+28=80.
III. The difference between the total number of
government schools in city D and the total
number of private schools in city D = 28+35-24-
20=19.

So, the sum is 1900+262=2162


18) Answer: B
Total number of school in F is = [44*12/11]
20) Answer: A
+[46*25/23]
Series I – 78, 84, 102, 105,126, c
=48+50=98
84-78=6=[2*3][23],
102-84=18= [2*9][29],
19) Answer: A
105-102=3=[3*1][31],
126-105=21=[3*7][37]
so…….c=126+[4*1]=130
Series II- c=130 d=130+6=136, a=136+18=154

Click Here For Bundle PDF Course | support@guidely.in Page 12 of 12


SBI Clerk & RRB PO Mains PDF Course 2023
ENGLISH Day - 44

English Language

Directions (1-5): The following paragraph has these conditions, States cannot avail of a share
five words/phrases which are in bold, they may in the funds, an outlay of ₹ 12,926.10 crore
or may not be correct and might need between 2023-24 and 2025-26, earmarked to
replacement. For each of the questions that improve State-run higher education. This despite
follows the paragraph, choose a word/phrase to the fact that 40% of funding has to be born (C)
replace the respective bold word/phrase (or) by the respective State governments.
choose option (e) if no replacement is required. Over the past several years, a few States have
By mandating States and Union Territories to strongly opposed NEP 2020, both at the draft
implement the National Education Policy (NEP) and implementation stages. Some such as Tamil
2020 and adopt contentious academic criteria in Nadu have initiated measures to draft their own
order to avail funds under the Pradhan Mantri State Education Policy. There are demands to
Uchchatar Shiksha Abhiyan (PM-USHA), the restore ‘education’ to the State List from the
Ministry of Education (MoE) appears to have Concurrent List, where it was moved without
made the central scheme exclusivist. The debate during the Emergency. Against this
guidelines for the scheme — an upgrade (A) backdrop, it would appear that the conditions for
version of the Rashtriya Uchchatar Shiksha the PM-USHA have been told (D) to impose NEP
Abhiyan (RUSA 1 and 2) to ensure increased 2020 through the backdoor. The scheme also
access, equity and excellence in the State higher requires adoption of the multiple entry and exit
education system with central funding — were options in degree programmes and National
released in June. Only 22 States and Union Higher Education Qualifications Framework
Territories have joined the PM-USHA, which guidelines. This, when academic stakeholders
requires a memorandum of understanding to be and administrators have expressed serious
signed between the State and the Department of concerns about the multiple entry and exit
Higher Education (MoE). West Bengal, Tamil options possibly triggering more dropouts from
Nadu and Kerala are among 14 States and the higher education system. The emphasis on a
Union Territories which have refused to get on four-year undergraduate programme has also
board. Apart from embracing (B) the NEP 2020, evoked concerns as to whether socially and
the memorandum of understanding, includes, economically backward students could afford the
inter alia, commitment from States on adopting money and the time to pursue an extra year in
guidelines for the National Credit Framework and college to acquire a degree. Besides, even a
Choice Based Credit System for Four Year State supporting the NEP 2020, Meghalaya, has
Undergraduate Programme. Without agreeing to called for a revisit of the funding criteria and an

Click Here For Bundle PDF Course | support@guidely.in Page 1 of 11


SBI Clerk & RRB PO Mains PDF Course 2023
ENGLISH Day - 44

increase in the number of beneficiary units, since b) unfitted


its State university is in the initial stage of being c) tailored
operational. Under these environment (E), it d) unmodified
would be prudent for the Ministry of Education to e) No replacement required
negotiate better terms with the dissenting States,
respecting the essence of cooperative 5) Which of the following words replaces the
federalism. word (E) in the given passage ?
1) Which of the following words replaces the a) purpose
word (A) in the given passage? b) circumstances
a) next c) planning
b) update d) conditions
c) better e) No replacement required
d) improvised
e) No replacement required Directions (6-10) : In each of the following
questions, a highlighted word is given followed
2) Which of the following words replaces the by three sentences. Choose the sentence(s) that
word (B) in the given passage? has/have the wrong/inappropriate usage of the
a) including highlighted word.
b) inducing 6) OVERHAUL
c) eliminating I. The company decided to undergo a complete
d) governing overhaul of its outdated technology
e) No replacement required infrastructure.
II. The team performed well individually, but their
3) Which of the following words replaces the lack of coordination ultimately impacted their
word (C) in the given passage? overhaul performance in the competition.
a) borne III. After months of planning and discussions, the
b) taken outlet’s sales processes were successfully
c) dropped overhauled.
d) ignored a) Only I
e) No replacement required b) Only II
c) Both I and III
4) Which of the following words replaces the d) Both II and III
word (D) in the given passage ? e) All correct
a) misadjusted

Click Here For Bundle PDF Course | support@guidely.in Page 2 of 11


SBI Clerk & RRB PO Mains PDF Course 2023
ENGLISH Day - 44

7) PILOT I. I am currently reviewing the draft of the report


I. It was a pilot project aimed at testing the to ensure that all the data and analysis are
feasibility of renewable energy solutions for rural accurate and well-presented.
communities in the region. II. The review team comprised experts from
II. The airplane was controlled by an experienced various fields, all working together to assess the
pilot with decades of flying expertise, ensuring a project's strengths and weaknesses.
safe and smooth journey for all passengers on III. The manuscript was carefully reviewed by a
board. panel of scholars before being accepted for
III. She was piloting the new software publication in the prestigious academic journal.
implementation, guiding the team through the a) Only I
intricacies of the transition process. b) Only II
a) Only II c) Both a and b
b) Both I and III d) Both I and III
c) Both II and III e) All correct
d) Both I and II
e) All correct 10) TRANSPIRE
I. The meeting had been confidential, but details
8) CONDUCT of what transpiring during the discussions were
I. The researchers conducted a comprehensive leaked to the media.
study to analyze the effects of climate change on II. Certainly, I will keep you informed about what
marine ecosystems. transpires during the meeting and any
II. Our association is planning to conducting a developments that arise.
series of workshops aimed at fostering III. The agenda was set to transpire in a
networking among our members. structured manner, addressing each topic in a
III. She has a good conduct, consistently logical sequence.
demonstrating a high level of professionalism a) Only I
and ethical behaviour in all aspects of her work. b) Only II
a) Both I and III c) Only III
b) Only II d) Both I and III
c) Both I and II e) All correct
d) All wrong
e) All correct Directions (11-15) : In each of the questions
given below a phrase in the sentence has been
9) REVIEW highlighted. It may or may not need replacement.

Click Here For Bundle PDF Course | support@guidely.in Page 3 of 11


SBI Clerk & RRB PO Mains PDF Course 2023
ENGLISH Day - 44

Read the question carefully and choose an a) the Prime Minister urging fellow citizens not to
option that would best replace the highlighted lose the
part to make the sentence grammatically and b) the Prime Minister urge fellow citizens to lose
contextually correct. In case, the given sentence not the
is correct in its current form and there is no c) the Prime Minister urged fellow citizens not to
improvement needed, mark (e) as your answer. losing the
11) Lalbagh, a garden boasting a rich history d) the Prime Minister urging follow citizens not to
spanning over two centuries, was initial lose by
conceived and lay out during Hyder Ali’s period. e) No changes required
a) were initially conceived and laid out during
b) was initialized conceived and laid out during 14) My mom cooks very well, and her culinary
c) was initially conceived and laid out during creations is always met with praise and
d) were initially conceive and lay out during anticipation by everyone lucky enough to enjoy
e) No changes required them.
a) her culinary creation are always met with
12) The historic Maui coastal town of Lahaina praise
was almost destroyed by the fast-moving inferno b) her culinary creations are always met with
last week, by survivors saying there has been no praise
warning. c) her culinary creations are always met by
a) with survivors saying there had been no praise
warnings d) her culinary creations are always meet with
b) by survivors saying there had been no praise
warnings e) No changes required
c) with survivors saying their had been no
warnings 15) The city is famous for its rich historical
d) with survivors saying there had no been landmarks, vibrant cultural scene, and diverse
warnings culinary offerings whose attracted by visitors
e) No changes required from around the world.
a) when attract visitors from around the world
13) Addressing the nation on the 77th b) whom attract visitors from around the world
Independence Day, the Prime Minister urged c) them attract visitors from around the world
fellow citizens not to lose the opportunity to d) that attract visitors from around the world
realise the country’s potential. e) No changes required

Click Here For Bundle PDF Course | support@guidely.in Page 4 of 11


SBI Clerk & RRB PO Mains PDF Course 2023
ENGLISH Day - 44

Directions (16-20): In each of the questions given September to press for ______(2) of poll
below a sentence is given with three blanks in _______(3).
each. Corresponding to each question three COLUMN 1 COLUMN 2 COLUMN 3
columns are given with three words in each
A. strategy D. empowerment G. targets
column. You are required to choose the correct
combination of words from the three columns B. outcome E. achievement H. promises

that will perfectly fit into the blanks to make the


C. meeting F. fulfilment I. goals
sentence contextually correct and meaningful.
a) A-D-G
16) The Bharatiya Janata Party (BJP) and the
b) A-F-H
Centre are ________(1) in their defence of
c) B-E-G
Manipur Chief Minister N. Biren Singh. Mr. Shah
d) C-E-G
noted that the fresh influx of _______(2) in recent
e) C-F-I
years from Myanmar into Manipur and Mizoram
has _________(3) the age-old ethnic rivalry
18) Computer vision zones, image processing
between the Meities and the Kukis.
COLUMN 1 COLUMN 2 COLUMN 3 zones and metaverse learning zones will be
________(1) at the university level soon,
A. confused D.migrants G.aggravated
______(2) developing faculty application
programmes ____(3) the use of AI tools, the
B. E.refugees H. calmed
unambiguous Chief Minister said.
COLUMN 1 COLUMN 2 COLUMN 3
C. ambitious F. people I. improved
A. launch D. on G. by
a) A-E-I
B. constructed E. along H. to
b) B-D-H
c) B-E-G C. established F. besides I. with
d) C-D-I
a) A-D-G
e) C-F-G
b) A-F-H
c) B-E-H
17) With the slogan of ‘KCR Ko Hatao,
d) C-F-I
Telangana Ko Bachao, BJP Ko Jeetao’, the
e) C-E-I
Telangana BJP has come out with a three-
pronged ______(1) to take on the BRS
19) Shenanigans of this ____(1) do not bode well
government by organising a ‘Million March’
for an otherwise robust statistical system, which
protest programme in the first week of

Click Here For Bundle PDF Course | support@guidely.in Page 5 of 11


SBI Clerk & RRB PO Mains PDF Course 2023
ENGLISH Day - 44

has been ______(2) to strains at a time when 20) After facing _____(1) charges relating to hush
there has been an explosion of public data in a money paid to cover up an ______(2) and the
______(3) digital world of commerce and illegal retention of top-secret documents,
governance. ______(3) U.S. President Donald Trump has
COLUMN 1 COLUMN 2 COLUMN 3 been indicted for the third time.

A. way D. subject G. running COLUMN 1 COLUMN 2 COLUMN 3

A. less D. problem G. former


B. kind E. object H. evolving
B. huge E. affair H. previous
C. type F. constrained I. growing
C. prior F. issue I. before
a) A-D-I
b) B-D-I a) A-E-G
c) B-E-H b) A-D-H
d) B-F-G c) A-F-H
e) C-E-I d) B-D-I
e) C-E-G
Click Here to Get the Detailed Video Solution for the above given Questions
Or Scan the QR Code to Get the Detailed Video Solutions

Answer Key with Explanation

1) Answer: D Out of the given options, ‘improvised’ is the best


The given word A is incorrect and needs word that can replace ‘upgrade’ in the passage
replacement. (an improvised version)
An upgrade version - is completely wrong (an An next version - ‘an next’ is wrong (an comes
upgraded version would be a correct phrase before only the words that either start with a
instead) vowel or vowel sound with few exceptions)

Click Here For Bundle PDF Course | support@guidely.in Page 6 of 11


SBI Clerk & RRB PO Mains PDF Course 2023
ENGLISH Day - 44

An update version - is incorrect instead it must Condition of PM USHA is not a person but a
be ‘an updated version’ scheme as inferred from the given passage - it
An better version - ‘an better’ is wrong/incorrect cannot be told
The sentence : The guidelines for the scheme — Among the given words only ‘tailored’ is the right
an improvised version of the Rashtriya fit.
Uchchatar Shiksha Abhiyan (RUSA 1 and 2) to Tailored - to make or design something for a
ensure increased access, equity and excellence particular person or purpose
in the State higher education system with central The other given words are antonyms of the word
funding — were released in June. tailored and hence discarded.
The sentence : Against this backdrop, it would
2) Answer: E appear that the conditions for the PM-USHA
The highlighted word ‘embracing’ is correct and have been tailored to impose NEP 2020 through
appropriate. Hence, no replacement is required. the backdoor.
Embrace - to include
5) Answer: B
3) Answer: A ‘Environment’ is inappropriate and does not
Born is inappropriate and does not convey a convey a wholesome meaning.
proper meaning in the respective sentence. It Replace it with ‘circumstances’ to make the
has to be replaced to make the sentence correct sentence sound correct and meaningful.
and meaningful. ‘Under these circumstances’ is a correct phrase.
Borne - carried by the thing mentioned Circumstance - the facts and events that affect
In the respective sentence - borne by the states what happens in a particular situation
means ‘40% share is the state's responsibility’. Purpose, planning and conditions aren’t correct
Taken, dropped and ignored does not fit in the words that can replace the highlighted word in
sentence and hence are discarded. the passage and hence are discarded.
The sentence : This despite the fact that 40% of The sentence : Under these circumstances, it
funding has to be borne by the respective State would be prudent for the Ministry of Education to
governments. negotiate better terms with the dissenting States,
respecting the essence of cooperative
4) Answer: C federalism.
The given word ‘told’ is incorrect and has to be
replaced to make the respective sentence 6) Answer: B
correct and meaningful.

Click Here For Bundle PDF Course | support@guidely.in Page 7 of 11


SBI Clerk & RRB PO Mains PDF Course 2023
ENGLISH Day - 44

The sentences I and III have used the word All the given three sentences I, II and III have
‘overhaul’ in the correct way such that the word used the given word ‘review’ in the most
makes the respective sentence meaningful. appropriate way.
But in sentence II - overhaul is an inappropriate Review - a look back at something in order to
usage. check, remember, or be clear about
In sentence II, overall must be used instead of something/to examine
overhaul Hence Option e is the correct answer.
Overhaul - to look at something carefully and
change or repair it if necessary 10) Answer: A
Overall - to include everything II and III are correct and have used the given
word in an appropriate way.
7) Answer: E Only sentence I is wrong - ‘transpiring’ is
All the given three sentences I, II and III have incorrect instead it must be ‘transpired’ -
used the highlighted word correctly. sentence talks about an action that occurred in
the past
In sentence I - pilot project (trial project)
In sentence II - pilot (person who controls a Transpire - to become known; to be shown to be
vehicle, especially an aircraft or a boat) true/to happen/to develop
In sentence III - piloting (leading)
11) Answer: C
8) Answer: B The highlighted phrase is grammatically
The sentences I and III are correct and have incorrect and definitely needs
used the highlighted word in the most improvement/replacement.
appropriate way possible. Was initial conceived and lay out during - no
However sentence II is wrong because ‘to proper meaning is conveyed
conducting’ is grammatically incorrect, instead it Among the given options, only c has a proper
must be ‘to conduct’ phrase that can be used to replace the wrong
Conduct (verb) - to organize and do something, phrase in the sentence.
especially research Other options are wrong because :
Conduct (noun) - a person’s behaviour Option a - ‘were’ is incorrect (Lalbagh is a single
place/singular noun - must use ‘was’)
9) Answer: E Option b - ‘initialized conceived’ is incorrect

Click Here For Bundle PDF Course | support@guidely.in Page 8 of 11


SBI Clerk & RRB PO Mains PDF Course 2023
ENGLISH Day - 44

Option d - the whole phrase is grammatically Other options are wrong because:
wrong and hence discarded Option a - creation(singular), are(plural)
The sentence : Lalbagh, a garden boasting a Option c - by is a wrong preposition used
rich history spanning over two centuries, was Option d - meet is wrong, instead it must be ‘met’
initially conceived and laid out during Hyder Ali’s The sentence: My mom cooks very well, and her
period. culinary creations are always met with praise
and anticipation by everyone lucky enough to
12) Answer: A enjoy them.
The highlighted phrase is incorrect and must be
improved(by survivors is wrong) 15) Answer: D
Out of the given Options - option a has the In the highlighted phrase ‘whose’ is incorrect
correct phrase for replacement (whose - refers to ‘of whom’ , whom is incorrect
Other option are wrong because : in this context)
Option b - ‘by survivors saying’ is incorrect Option d - that attracted is correct and can be
Option c - their is wrong, instead it must be there used to replace the wrong phrase
Option d - had no been (grammatically incorrect) Other options are wrong because:

The sentence : The historic Maui coastal town of Option a - when is incorrect(refers timings)
Lahaina was almost destroyed by the fast- Option b - whom is incorrect(refers people)
moving inferno last week, with survivors saying Option c - them is incorrect(refers living beings)
there had been no warnings. The sentence : The city is famous for its rich
historical landmarks, vibrant cultural scene, and
13) Answer: E diverse culinary offerings that attract visitors
The given sentence is correct and meaningful. from around the world.
The highlighted phrase is error free and hence
needs no improvement or replacement. 16) Answer: C
B-E-G is the right combination of words that fill
14) Answer: B the given three blanks in the most appropriate
The highlighted phrase is wrong and hence way.
needs replacement (‘creations’ is plural and ‘is’ Unambiguous - clear and cannot be understood
is singular) wrongly
Option b - is correct and has the correct Refugees - people who leave their home area for
replacement phrase their own safety or survival

Click Here For Bundle PDF Course | support@guidely.in Page 9 of 11


SBI Clerk & RRB PO Mains PDF Course 2023
ENGLISH Day - 44

Aggravated - to make something worse or more pronged strategy to take on the BRS
serious government by organising a ‘Million March’
First blank - both confused and ambitious are protest programme in the first week of
irrelevant September to press for fulfilment of poll
Second blank - refugees is a better term that can promises.
be used in regards with the context
Third blank - the situation is negative (calmed 18) Answer: D
and improved positive words and hence C-F-I is the correct combination of words.
discarded) Established, besides and with are the correct
The sentence: The Bharatiya Janata Party words from the words given as options.
(BJP) and the Centre are unambiguous in their First blank - launch(wrong word form),
defence of Manipur Chief Minister N. Biren constructed (irrelevant word)
Singh. Mr. Shah noted that the fresh influx of Second blank - on and along are not relevant
refugees in recent years from Myanmar into and causes grammatical error
Manipur and Mizoram has aggravated the age- Third blank - both ‘by’ and ‘to’ are incorrect
old ethnic rivalry between the Meities and the prepositions
Kukis.
17) Answer: B The sentence : Computer vision zones, image
A-F-H is the correct combination of words. processing zones and metaverse learning zones
Strategies, fulfilment and promises are the will be established at the university level soon,
correct words that fill the given blanks. besides developing faculty application
Other given words alter the entire meaning of the programmes with the use of AI tools, the Chief
sentence and hence are discarded. Minister said.
First blank - strategy is the best word (plan)
Second blank - fulfilment is the right word that 19) Answer: B
fits blank 2 B-D-I - kind, subject and growing are the right
Third blank - ‘promises’ is a better fit than goals words.
and targets because the parties make promises First blank - way is incorrect, either it should be
to people before elections and try to achieve kind or type
them later. Second blank - both object and constrained are
The sentence : With the slogan of ‘KCR Ko irrelevant
Hatao, Telangana Ko Bachao, BJP Ko Jeetao’, Third blank - it must be either growing or
the Telangana BJP has come out with a three- evolving

Click Here For Bundle PDF Course | support@guidely.in Page 10 of 11


SBI Clerk & RRB PO Mains PDF Course 2023
ENGLISH Day - 44

The only combination which has all the right First blank - less and huge both are wrong
words is option b and hence(type and evolving (charges were made earlier to cover up the
are discarded) wrong doing is the meaning)
The sentence : Shenanigans of this kind do not Second blank - ‘an’ is present before the
bode well for an otherwise robust statistical blank(must be either affair or issue)
system, which has been subject to strains at a Third blank - the president of last term must be
time when there has been an explosion of public referred to as the ‘former president’ (previous
data in a growing digital world of commerce and president, before president are wrong)
governance.
The sentence : After facing prior charges relating
20) Answer: E to hush money paid to cover up an affair and the
C-E-G is the correct combination. The three illegal retention of top-secret documents, former
words are prior, affair and former. U.S. President Donald Trump has been indicted
for the third time.

Click Here For Bundle PDF Course | support@guidely.in Page 11 of 11


SBI Clerk & RRB PO Mains PDF Course 2023
Reasoning Ability Day - 45 (Eng)

Reasoning Ability
Directions (1-5): Study the following information I-B sits opposite to the one who is an immediate
carefully and answer the given question. neighbour of C
Ten persons viz., A, B, C, D, E, F, G, H, I and J II- H faces outside the table.
are sitting around a rectangular table in such a III-As many persons sit between I and D as
way that three persons sit on each longer side between C and F.
and faces inside the table, while two persons sit a) Only II
on each shorter side and faces outside the table. b) Only I and II
Note: If the person's name is a vowel letter, then c) Only I and III
he must face outside the table. d) Only III
B sits third to the left of A. Three persons sit e) None of these
between G and the one who sits opposite to B. I
is an immediate neighbour of the one who sits 3) What is the position of C with a respect to I?
opposite to A. The number of persons sitting a) Third to the right
between I and G is the same as the number of b) Fourth to the left
persons sitting between I and H. One person sits c) Second to the right
between C and D, who sits adjacent to H but d) Immediate left
neither of them faces outside the table. Both C e) None of these
and F are not facing G. F is the immediate
neighbour of the one who sits opposite to J. Both 4) If X sits immediate left of D then, who among
E and F are immediate neighbours of each other. the following person sits exactly between X and
1) The number of persons sitting E and H is G?
_____ than the number of persons sitting a) The one who sits immediate right of E
between B and D, when counted from the right of b) The one who sits second to the left of B
both E and B? c) The one who sits second to the right of J
a) Three more d) Both (a) and (c)
b) Two less e) None of these
c) One more
d) Three less 5) If G is related to F and E is related to C, in a
e) None of these certain way then J is related to who among the
following person?
2) Which of the following statement(s) is/are true a) The one who sits opposite to H
based on the final arrangement? b) I
c) The one who is an immediate left of D

Click Here For Bundle PDF Course | support@guidely.in Page 1 of 8


SBI Clerk & RRB PO Mains PDF Course 2023
Reasoning Ability Day - 45 (Eng)

d) B 7) Who among the following person sits to the


e) None of these immediate right of the one who is facing U?
a) The one who sits second to the left of S
Directions (6-10): Study the following information b) V
carefully and answer the given question. c) The one who sits immediate right of X
Nine persons-, P, Q, R, S, T, U, V, W and X are d) Vacant seat
sitting around a circular table, which has ten e) None of these
seats, where one seat is vacant. All are facing
towards the centre. 8) Which among the following pairs are sitting
Note: Vacant seat is not adjacent to vowel adjacent to the vacant seat?
named person. a) P and S
R sits third to the right of T, who doesn’t sit b) V and W
adjacent to the vacant seat. Only two persons sit c) X and Q
between S and the one who sits immediate left of d) S and W
T. W sits second to the left of the one who sits e) None of these
third to the right of R. The number of persons
sitting between W and S, who sits opposite to Q 9) If T is related to P and W is related to S in a
is the same as the number of persons sitting certain way. Then who among the following is
between X and Q. U sits fourth to the right of the related to V?
one who sits opposite to X, who is not sitting a) The one who is second to the right of X
adjacent to W. P sits neither adjacent to the b) Vacant seat
vacant seat nor adjacent to S. The number of c) R
persons sitting between U and P is one more d) The one who sits third to the left of Q
than the number of persons sitting between P e) None of the above
and the vacant seat, when counted from the right
of U and left of the vacant seat. 10) Which of the following statement is/are not
6) What is the position of P with respect to the true?
vacant seat? I. The one who sits second to the left R faces S
a) Second to the right II. Only two persons sit between X and Q
b) Immediate right III. U faces the immediate neighbour of W
c) Third to the left a) Both II and III
d) Second to the left b) Both I and III
e) None of these c) Only II
d) Both I and II

Click Here For Bundle PDF Course | support@guidely.in Page 2 of 8


SBI Clerk & RRB PO Mains PDF Course 2023
Reasoning Ability Day - 45 (Eng)

e) None of these a) 25m


b) √45m
Directions (11-15): Study the following c) √55m
information carefully and answer the given d) √80m
questions e) Either (b) or (c)
P#Q means P is south of Q
P+Q means P is east of Q 13) Four of the following are alike in a certain
P@Q means P is west of Q way. Which of the following does not belong to
P&Q means P is north of Q the group?
% means either 4m or 6m a) IL
^ means either 4m or 7m b) GH
P*QS means P is the middle point of Q and S c) KD
i.e., P #+ Q means P is south-east of Q d) OJ
P&^Q means P north of Q either 4m or 7m e) RK
The Distance between O and L is the same as I
and K but two meters less than the distance 14) What is the shortest distance and direction of
between J and L. O with respect to K?
The Distance between L and K is one meter less a) √185m, North-east
than the distance between I and R. The distance b) √180m, South-west
between O and L is 12m c) √181m, North-West
Statement: d) √186m, South-east
G#@H; G+^I; J&K; L%#K; G@K; H&%K; H*KJ; e) None of these
O@L; K*GB; D&^B; R#^I
11) What is the direction of H with respect to the 15) If X is 5m north of J, then the distance
one which is north of O? between X and K is one meter more than the
a) North West distance between which of the following?
b) South East a) G and B
c) North East b) K and G
d) North c) L and O
e) None of these d) K and J
e) Cannot be determined
12) If the distance between K and B is 8m and
point Z is 8m east of Point H, then what is the 16) Statement: The students who are bright
shortest distance between Z and K? since the childhood are trained so that they

Click Here For Bundle PDF Course | support@guidely.in Page 3 of 8


SBI Clerk & RRB PO Mains PDF Course 2023
Reasoning Ability Day - 45 (Eng)

choose to go out of the country for jobs after a) Deforestation drove the last White rhinos to
studies taking all our intelligent brains outside starvation by chasing them from their natural
Course of Action: habitat.
A. Students should be asked to sign a letter of b) Every naturalist working in the White rhino’s
consent with the college that they would be natural habitat has looked systematically for
taking up a job in their own country evidence of the rhino’s survival.
B. The students who are willing to go out should c) The White rhino did not move and adapt to a
be given more support as they take the country’s different region in response to the loss of habitat.
name to the world d) Those who have reported sightings of the
a) Only I White rhino are not experienced naturalists or
b) Either I or II foresters.
c) Both I and II e) none of the above
d) Only II
e) None of the above 19) Statement:
“Lok Sabha passed the Motor Vehicles
17) Which of the given options is correct? (Amendment) Bill in 2017. The amendments
a) C is the Effect and B is its Cause were targeted towards bringing changes in the
b) A is the Cause and B is its Effect transport sector to encourage safer driving
c) A and B are Effects and C is the common practices among Indian motor vehicle drivers.
Cause The amendment’s draft was put forward in the
d) A is the Effect and C is its Cause lower house of the Parliament to impose rigid
e) None of the above fines for the traffic rules violation and to promote
e-governance.”
18) Statement : Forester -- The recent claims Assumptions:
that the White rhino is not extinct are false. The I). Now people would be more careful before
White rhino’s natural habitat was taken over by riding their two-wheelers without a helmet and
deforestation, resulting in the animal’s systematic while driving their cars without wearing seatbelts
elimination from the area. Since then naturalists or without valid motor insurance.
working in the region have discovered no hard II). Contractors will be held responsible and
evidence of its survival, such as carcasses or blacklisted for the bad condition of roads.
tracks. In spite of alleged sightings of the animal, III). The previous governments have not taken
the White rhino no longer exists. enough measures for road safety.
Which one of the following is the conclusion on a) only I and III are implicit
which the forester’s statement depends? b) only I and II are implicit

Click Here For Bundle PDF Course | support@guidely.in Page 4 of 8


SBI Clerk & RRB PO Mains PDF Course 2023
Reasoning Ability Day - 45 (Eng)

c) Only III is implicit pollutants heavier and fall on ground and the
d) Only I is implicit invention should be implemented.
e) Only II and III are implicit II). Apart from controlling air pollution created by
the car in which it is installed, the device also has
20) Statement: IIT Kharagpur graduate develops potential to neutralize the pollution emitted from
unique device ‘PM 2.5’ to curb vehicular air- 10 cars in its vicinity and it should be
pollution. implemented.
Arguments: a) If only argument I is strong
I). The invention works on a combination of b) If only argument II is strong
electrical and wave energy which include PM 2.5 c) If either I or II is strong
pollutants which attracts large number of PM 2.5 d) If neither I or II is strong
pollutants together, and the device makes such e) If both I and II are strong
Click Here to Get the Detailed Video Solution for the above given Questions
Or Scan the QR Code to Get the Detailed Video Solutions

Answer Key with Explanation

Directions (1-5):
1. Answer: C
2. Answer: B
3. Answer: B
4. Answer: D
5. Answer: A
Final arrangement
We have,
 B sits third to the left of A.

Click Here For Bundle PDF Course | support@guidely.in Page 5 of 8


SBI Clerk & RRB PO Mains PDF Course 2023
Reasoning Ability Day - 45 (Eng)

 Three persons sit between G and the one who


sits opposite to B.
From the above conditions, we have three
possibilities

Again we have,
 F is the immediate neighbour of the one who
sits opposite to J.
 Both E and F are immediate neighbours of
Again we have,
each other.
 I is an immediate neighbour of the one who sits
From case 1 gets eliminated because both E
opposite to A.
and F are not immediate neighbour of each
 The number of persons sitting between I and
other. Hence case 1a shows final arrangement.
G is the same as the number of persons sitting
between I and H.
 One person sits between C and D, who sits
adjacent to H but neither of them faces outside
the table.
 Both C and F are not facing G.
From the above conditions case 2 gets
eliminated because either C or F is facing G.
Directions (6-10):
6. Answer: C
7. Answer: A
8. Answer: D
9. Answer: B

Click Here For Bundle PDF Course | support@guidely.in Page 6 of 8


SBI Clerk & RRB PO Mains PDF Course 2023
Reasoning Ability Day - 45 (Eng)

10. Answer: C
Final arrangement

Again we have,
 P sits neither adjacent to the vacant seat nor
adjacent to S.
 The number of persons sitting between U and
P is one more than the number of persons sitting

We have, between P and the vacant seat, when counted

 R sits third to the right of T, who doesn’t sit from the right of U and left of the vacant seat.

adjacent to the vacant seat. From the above conditions case 1 gets

 Only two persons sit between S and the one eliminated because there is any fixed place for

who sits immediate left of T vacant seat. Hence case 2 shows final
arrangement.

Again we have,
 W sits second to the left of the one who sits
Directions (11-15):
third to the right of R.
11. Answer: C
 The number of persons sitting between W and
12. Answer: D
S, who sits opposite to Q, is the same as the
13. Answer: A
number of persons sitting between X and Q.
14. Answer: B
 U sits fourth to the right of the one who sits
15. Answer: C
opposite to X, who is not sitting adjacent to W.
Final arrangement

Click Here For Bundle PDF Course | support@guidely.in Page 7 of 8


SBI Clerk & RRB PO Mains PDF Course 2023
Reasoning Ability Day - 45 (Eng)

18) Answer: C
If this point was not true and the White rhinos
have shifted to another location, then the
conclusion will not hold good. However, if this
point was actually true, it would clear any
confusion and provide evidence for the foresters’
view.

19) Answer: D
Assumption I: It is implicit because the decision
to impose rigid fines for the traffic rules violation

16) Answer: E would make people careful while coming on the

No student can be forced to work against his/her roads with their vehicles.

wish so, the first sentence is not correct. Assumption II: This may be included in the

The second sentence also strengthens the issue amendment bill, but it could not be an

further and not solves the same, so can be ruled assumption.

out. Assumption III: It is not implicit, as nothing about

Hence, option (e) is correct. the previous governments or their measures is

Statements: mentioned in the statement.

A. Indian team lost the crucial tournament


B. The team must train more players for the 20) Answer: E

crisis times The device attracts large number of PM 2.5

C. Two of its important players were injured pollutants together and makes such pollutants

during that time heavier and in turn they fall safely on the ground
instead of being air borne and polluting the air.

17) Answer: D The device also has potential to neutralize the

The answer is quite clear as we can see that pollution emitted from 10 cars in its vicinity. So,

because players were injured, the team lost. both arguments I and II are strong.

So, option (d) is the correct answer.

Click Here For Bundle PDF Course | support@guidely.in Page 8 of 8


SBI Clerk & RRB PO Mains PDF Course 2023
Quantitative Aptitude Day – 45 (Eng)

Quantitative Aptitude

Directions (1-5): Read the data carefully and A.84.50


answer the following questions. B.98.60
The table given below tableshows thetotal C.66.840
number of passengers(men, women, and D.94.610
children) travelled and the number of men’s and E. None of these
women’s travelled on different days of a week
starting from Monday. 3) Find the sum of number of men’s and
children’s travelled on Tuesday and number of
women’s and children’s travelled on Friday?
A.310
B.325
C.311
D.270
E. None of these

4) Find the total number of children’s travelled on


all the given days together is how much percent
Note: Difference between number of women
more or less than the number of women’s
travelled up to Thursday and the number of
travelled on all the given days together?
women travelled up to Tuesday is 465.
A.55.67% less
(Thursday-Tuesday)
B.63.63% less
1) Find the ratio between the number of men,
C.72.25% less
women and children travelled on Tuesday?
D.67.54% less
A.7:18:5
E. None of these
B.8:18:9
C.6:17:7
5) In Friday, sitting ticket and sleeper ticket fare
D.5:17:9
is Rs.210 and Rs.330 respectively. 1/3rd of men’s
E. None of these
travelled in sitting and the rest travelled in
sleeper. Both the number of Women’s and
2) Find the central angle for the number of
children’s travelled sitting and sleeper ticket are
children’s travelled on Thursday out of the total
in the ratio of 2:1. Find the total revenue
number of children’s travelled on all the given
collected on that day.
days together?

Click Here For Bundle PDF Course | support@guidely.in Page 1 of 11


SBI Clerk & RRB PO Mains PDF Course 2023
Quantitative Aptitude Day - 45 (Eng)

A.Rs.133560 D.Rs.123460
B.Rs.234560 E. None of these
C.Rs.153560

Directions (6-10): Study the following information and answer the given questions.
The pie chart shows the percentage distribution of the number of PG doctors, Junior Doctors and
Senior Doctors working in a hospital. The total number of doctors is 180.

The given table shows the number of patients treated by the different doctors on a particular day.

Click Here For Bundle PDF Course | support@guidely.in Page 2 of 11


SBI Clerk & RRB PO Mains PDF Course 2023
Quantitative Aptitude Day - 45 (Eng)

6) 2/3rd of the PG doctors treated male patients D.15.29 hours


and the rest of the PG doctors treated female E. none of these
patients. If each doctor treating male patients is
treating an equal number of male patients, 9) Find the ratio of the number of female patients
similarly each doctor treating female patients is treated by senior doctors to the average number
treating the equal number of female patients, of male patients treated by senior and PG
then find the difference between the number of doctors.
female patients treated and the number of male A.13: 12
patients treated by a PG doctor. B.14: 13
A.10 C.15: 14
B.15 D.16: 15
C.20 E. None of these
D.25
E. None of these 10) The average number of male patients treated
by all three categories of doctors is what
7) What is the total number of patients treated by percentage of the average number of female
a junior and a PG doctor, if all junior and PG patients treated by all three categories of
doctors treat the same number of patients? doctors?
A.113.33 A.75%
B.103.33 B.76.33%
C.106.66 C.77.38%
D.123.33 D. 78.49%
E.126.66 E. None of these

8) If 33.33% of the junior doctors were absent on 11) Arun invested Rs. (6P + 3Q) in a scheme
a particular day but the total number of patients offering simple interest at 12.5% per annum.
was the same and the remaining doctors treated After three years this money amounts to (3Q +
the patients for equal time. If they treated a male 9P). Compound interest earned on 8(P + Q) at
patient for 8 minutes and a female patient for 5 50% per annum for three years is _____.
minutes, then in how many hours did each junior I. 95P/2
doctor treat the patients on that day? II. 19(P + Q)
A.12.29 hours III. 95Q/2
B.13.29 hours A.I and II only
C.14.29 hours B.II and III only

Click Here For Bundle PDF Course | support@guidely.in Page 3 of 11


SBI Clerk & RRB PO Mains PDF Course 2023
Quantitative Aptitude Day - 45 (Eng)

C.III and I only 14) A box contains 3 red balls, ---- white balls
D.I, II, and III and 2 pink balls. Two balls are drawn at random
E. None of these and the probability of that balls are red is 3/28.
The probability of one pink ball is drawn at
12) If number of workers in a company is random is ---
increased by 2.5M% and working hours per day A.6, 1/11
decreased by (M – 10) % then production of B.7, 1/6
company is increased by 35% as compared to C.3, 1/4
initial production, where value of (4M + 12) is D.10, 2/15
_______. Working efficiency of each worker is E.9, 1/7
same.
I. 92 15) M started a business with initial investment of
II. 212 ______. After 3 months N and O joined the
III. 162 business with Rs. 35000 and Rs. 40000
A.I and II only respectively. O quits from the business and M
B.III and I only and N increase their investments by 20% and
C.II only _____ before 5 months of the end of the business
D.I, II, and III respectively. The profit ratio between P, Q and R
E. None of these becomes 156:147:64 at the end of one year.
A.Rs. 30000, 30%
13) Time taken by train A to cross a platform is 9 B.Rs. 42000, 25%
seconds and that taken by train B to cross the C.Rs. 35000, 30%
same platform is ___ seconds. Ratio of speeds of D.Rs. 32000, 20%
train A to train B is 3: 2 and length of train B is 50 E. None of these
m more than that of train A. Length of the
platform is 80% of length of train A and train A Directions (16-20): Study the following data
will cross train B in 7 seconds while running in carefully and answer the given questions:
opposite direction. A school has a certain number of students. Some
What value will be filled in the blank? students like only action movies, some students
A.12 seconds like only comedy movies, some students like only
B.20 seconds thriller movies, some students like both action
C.15 seconds and comedy movies but not thriller movies, some
D.18 seconds students like both comedy and thriller movies but
E.16 seconds not action movies, some students like both action

Click Here For Bundle PDF Course | support@guidely.in Page 4 of 11


SBI Clerk & RRB PO Mains PDF Course 2023
Quantitative Aptitude Day - 45 (Eng)

and thriller movies but not comedy movies and girls who like only two types of movies is what
the remaining students like all the three types of percent of total number of girls in the school.
movies. A.40%
Ratio of the number of students who like only B.25%
action movies to those who like only comedy C.50%
movies is 8:5. Number of students who like all D.20%
the three types of movies is 90% of those who E.30%
like both action and comedy movies but not
thriller movies. Number of students who like both 18) Find the average number of students who
action and thriller movies but not comedy movies like only action movies, those who like only
is 20 more than those who like both comedy and comedy movies and those who like all the 3
thriller movies but not action movies. Total types of movies.
number of students who like action movies is 65 A.60
more than those who like comedy movies. B.90
Number of students who like only thriller movies C.80
is 25 less than those who like only comedy D.50
movies and also equal to those who like both E.70
action and comedy movies but not thriller
movies. Total number of students who like thriller 19) The number of students who like at least 2
movies is 155. types of movies is what percent of the total
16) Find the ratio between the total number of number of students in the school?
students, who like only one type of movie to the A.38.75%
number of students who like all the 3 types of B.40%
movies. C.37.5%
A.91: 18 D.41.25%
B.49: 9 E.35%
C.15: 4
D.18: 5 20) If ratio of boys to girls, who don’t like action
E.73: 12 movies is 16:13, then find the difference between
the number of such boys and such girls?
17) If the ratio of boys to girls in the school is 5:3 A.25
and the ratio of boys to girls, who like only two B.15
types of movies is 13:9, then find the number of C.20
D.5

Click Here For Bundle PDF Course | support@guidely.in Page 5 of 11


SBI Clerk & RRB PO Mains PDF Course 2023
Quantitative Aptitude Day - 45 (Eng)

E.10
Click Here to Get the Detailed Video Solution for the above given Questions
Or Scan the QR Code to Get the Detailed Video Solutions

Answer Key with Explanation

Directions (1-5): Number of men travelled till Tuesday = 260


Difference between number of women travelled Number of women travelled till Tuesday =270
till Thursday and the number of women travelled Number of children travelled till Tuesday = 650-
till Tuesday = 465. 260 – 270 = 120
61.25(X+7) - 30(X+4) = 465 Up to Wednesday,
61.25X+428.75 – 30X – 120 = 465 Number of passengers travelled till Wednesday
31.25X = 156.25 = 1060
X =5 Number of men travelled till Wednesday = 371
Number of women travelled up to Tuesday = Number of women travelled till Wednesday =
30(5+4) = 270 477
Number of women travelled up to Thursday = Number of children travelled till Wednesday =
61.25(5+7) = 735 1060 - 371 – 477 = 212
On Monday, Up to Thursday,
Number of passengers travelled = 300 Number of passengers travelled till Thursday =
Number of men travelled = 180 1470
Number of women travelled = 90 Number of men travelled till Thursday = 441
Number of children travelled = 300 - 180 – 90 = Number of women travelled till Thursday = 735
30 Number of children travelled till Thursday = 1470
Up to Tuesday, – 441 - 735 = 294
Number of passengers travelled till Tuesday = Up to Friday,
650

Click Here For Bundle PDF Course | support@guidely.in Page 6 of 11


SBI Clerk & RRB PO Mains PDF Course 2023
Quantitative Aptitude Day – 45 (Eng)

Number of passengers travelled till Friday = Number of women’s and children’s travelled on
1950 Friday = 123+18 = 141
Number of men travelled till Friday = 780 Required sum = 170+141 = 311
Number of women travelled till Friday = 858 Hence, the answer is option C.
Number of children travelled till Friday = 1950 –
858 -780 = 312 4) Answer: B
Total number of children travelled on all days =
30+90+92+82+18 = 312
Total number of women travelled on all days =
90+180+207+258+123 = 858
Required percentage = (858 - 312)/858*100
=63.63%
Hence, the answer is option B.

5) Answer: A
1) Answer: B Number of men’s travelled on Friday = 339
Number of men’s travelled on Tuesday = 80 Number of women’s travelled on Friday = 123
Number of women’s travelled on Tuesday = 180 Number of children’s travelled on Friday = 18
Number of children’s travelled on Tuesday = 90 Amount collected bymen’s = 339/3*210 +
Required ratio = 80:180:90 = 8:18:9 339*2/3*330 = Rs.98310
Hence, the answer is option B. Amount collected bywomen’s = 123/3*2*210
+123/3*1*330 = Rs.30750
2) Answer: D Amount collected bychildren’s = 18/3*2*210 +
Number of children’stravelled onThursday = 82 18/3*1*330 =Rs. 4500
Total number of children on all the days = Total revenue collected= 98310 + 30750 + 4500
30+90+92+82+18 = 312 = Rs.133560
Required central angle= 82/312*360 = 94.61° Hence, the answer is option A.
Hence, the answer is option D.
Directions (6-10):
3) Answer: C Total number of PG doctors = 180 × 15/100 = 27
Number of men’s and children’s travelled on Total number of junior doctors = 180 × 55/100 =
Tuesday = 80 + 90 = 170 99

Click Here For Bundle PDF Course | support@guidely.in Page 7 of 11


SBI Clerk & RRB PO Mains PDF Course 2023
Quantitative Aptitude Day – 45 (Eng)

Total number of senior doctors = 180 × 30/100 = Percentage of absent junior doctors =
54 Number of present junior doctors = 99 × (100 –
33.33)/100 = 99 × 66.66/100 = 66
Total time taken to treat male patients = 3465 ×
8 = 27720 min
Total time taken to treat female patients = 5775
× 5 = 28875 min
Total time taken to treat patients = 27720 +
28875 = 56595 min = 56595/60 hr = 943.25 hr
Required time = 943.25/66 = 14.29 hours

9) Answer: B
The number of female patients treated by senior
doctors = 1260
6) Answer: B
The average number male patients treated by
2/3rd of PG doctor = 27 × 2/3 = 18
senior and PG doctors= (1890 + 450)/2 = 2340/2
Number of male patients treated by a PG doctor
= 1170
= 450/18 = 25
The ratio of the number of female patients
Number of female patients treated by a
treated by senior doctors and the average
PG doctor = 360/9 = 40
number of male patients treated by senior and
Difference between the number of male patients
PG doctors = 1260: 1170 = 14: 13
treated and the number of female patients
treated by a PG resident doctor = 40 – 25 = 15
10) Answer: D
Total number of male patients = 3465 + 1890 +
7) Answer: D
450 = 5805
Total number of junior doctors = 99
Average number male patients treated by all
Number of patients treated by a junior doctor =
three categories of doctors = 5805/3 = 1935
9240/99 =93.33
Total number of female patients = 5775 + 1260 +
Total number of PG doctors = 27
360 = 7395
Number of patients treated by a PG doctor =
Average number female patients treated by all
810/27 = 30
three categories of doctors = 7395/3 = 2465
Required sum = 93.33+30 = 123.33
Required percentage = 100 × 1935/2465 =
78.49%
8) Answer: C

Click Here For Bundle PDF Course | support@guidely.in Page 8 of 11


SBI Clerk & RRB PO Mains PDF Course 2023
Quantitative Aptitude Day – 45 (Eng)

Also let length of train A and train B are ‘5y’ m


11) Answer: B and (5y + 50) m respectively.
Simple interest earned = 3Q + 9P – 6P – 3Q = And length of the platform = 80% of ‘5y’ = ‘4y’ m
3P Since, time taken by train A to cross the platform
Now, is 9 seconds.
(6P + 3Q) x 3 x 1/8 = 3P So, (5y + 4y)/3x = 9
18P + 9Q = 24P y = 3x --------------(1)
6P = 9Q Since, train A will cross train B in 7 seconds
So, P = 3Q/2---------(1) while running in the opposite direction.
Compound interest earned on 8(P + Q) at 50% So, (5y + 5y + 50)/ (3x + 2x) = 7
per annum = 8(P + Q) x [(3/2)3 – 1] = 19 x (P + (2y + 10)/x = 7 --------------(2)
Q) From equations (1) and (2):
From equation (1), 6x + 10 = 7x
19 x (P + Q) = 95Q/2 = 95P/3 x = 10, y = 30
Only II and III follows Speed of train B = 2 * 10 = 20 m/s
Hence answer is option B Length of train B = 5 * 30 + 50 = 200 m
Length of the platform = 80% of 150 = 120 m
12) Answer: A So, the value, which will be filled in the blank =
Products of company = number of workers x (200 + 120)/20 = 16 seconds
working hours
Now, 14) Answer: C
2.5M – (M – 10) – [2.5M x (M – 10)/100] = 35 3/28 = 3C2/(5 + x)C2
[Successive % change] 3/28 = 3 * 2/(5 + x) * (4 + x)
M2 – 70M + 1000 = 0 56 = 20 + 5x + 4x + x2
(M – 50) (M – 20) = 0 X2 + 9x – 36 = 0
Value of M = 50 and 20 X2 + 12x – 3x – 36 = 0
Value of (4M + 12) = 92 and 212 X(X + 12) – 3(X + 12) = 0
Hence answer is option A (x – 3)(x + 12) = 0
X = 3,
13) Answer: E Total number of balls = 5 + 3 = 8
Let speed of train A and train B are ‘3x’ m/s and Required probability = 2C1/8C1 = ¼
‘2x’ m/s respectively.
15) Answer: A

Click Here For Bundle PDF Course | support@guidely.in Page 9 of 11


SBI Clerk & RRB PO Mains PDF Course 2023
Quantitative Aptitude Day – 45 (Eng)

According to question, And total number of students, who like comedy


Let X and r% be the initial investment of M and movies:
investment increases by N. = 5x + 10y + 9y + c
(X*7)+(120X/100*5) : = 5x + 19y + c
(35000*4)+(35000*(100+r)/100*5) : (40000*4) 8x + 19y + c + 20 – 5x – 19y – c = 65
13X : 315000 + 1750r : 160000 x = 15
13X/ 160000 = 156/64 Number of students, who like only action movies
X= 160000*156/(64*13) = 8 * 15 = 120
X = 30,000 Number of students, who like only comedy
Initial investment of M = Rs.30000 movies = 5 * 15 = 75
(315000+1750r)/160000 = 147/64 Since, number of students, who like only thriller
315000+1750r = 367500 movies is 25 less than those, who like only
1750r = 52500 comedy movies.
r=30% So, number of students, who like only thriller
Hence the option is A. movies = 75 – 25 = 50
Since, number of students, who like only thriller
Directions (16-20): movies is equal to those, who like both action
Let number of students, who like only actions and comedy movies but not thriller movies.
movies and those, who like only comedy movies, So, number of students, who like both action and
are ‘8x’ and ‘5x’ respectively. comedy movies but not thriller movies = 50
Let number of students, who like both action and And number of students, who like all the three
comedy movies but not thriller movies = 10y types of movies = 90% of 50 = 45
So, number of students, who like all the three Since, total number of students, who like thriller
types of movies = 90% of 10y = 9y movies is 155.
Let number of students, who like both comedy 50 + 45 + c + c + 20 = 155
and thriller movies but not action movies = c c = 20
So, number of students, who like both action and Number of students, who like both comedy and
thriller movies but not comedy movies = c + 20 thriller movies but not action movies = 20
Since, total number of students, who like action Number of students, who like both action and
movies: thriller movies but not comedy movies = 20 + 20
= 8x + 10y + 9y + c + 20 = 40
= 8x + 19y + c + 20 Total number of students in the school = 120 +
75 + 50 + 50 + 20 + 40 + 45 = 400

Click Here For Bundle PDF Course | support@guidely.in Page 10 of 11


SBI Clerk & RRB PO Mains PDF Course 2023
Quantitative Aptitude Day – 45 (Eng)

According to the question, Required average = (120 + 75 + 45)/3 = 80


16) Answer: B
Total number of students, who like only one type 19) Answer: A
of movie = 120 + 75 + 50 = 245 The number of students, who like at least 2
Total number of students, who like all the 3 types types of movies = Number of students, who like
of movies = 45 only two types of movies + Number of students,
Required ratio = 245:45 = 49:9 who like all the three types of movies
The number of students, who like minimum 2
17) Answer: E types of movies = 50 + 40 + 20 + 45 = 155
Total number of students in the school = 400 Total number of students in the school = 400
So, total number of girls in the school = 400 * Required percentage = (155/400) * 100 =
(3/8) = 150 38.75%
Total number of students, who like only two
types of movies = 50 + 40 + 20 = 110 20) Answer: B
So, number of girls, who like only two types of Number of students, who don’t like action
movies = 110 * (9/22) = 45 movies = Total number of students in the school
Required percentage = (45/150) * 100 = 30% – Number of students, who like action movies
Number of students, who don’t like action
18) Answer: C movies = 400 – (120 + 50 + 40 + 45) = 145
Number of students, who like only action movies Number of boys, who don’t like action movies =
= 120 145 * (16/29) = 80
Number of students, who like only comedy Number of girls, who don’t like action movies =
movies = 75 145 – 80 = 65
Number of students, who like all the 3 types of Required difference = 80 – 65 = 15
movies = 45

Click Here For Bundle PDF Course | support@guidely.in Page 11 of 11


SBI Clerk & RRB PO Mains PDF Course 2023
ENGLISH Day - 45

English Language

Directions (1-7): Read the given passage climbing down to 27.5 per cent. Thus, within ten
carefully and answer the following questions years, the number of poor people in India fell by
based on the passage. Some words are more than 271 million — a truly massive gain”.
highlighted to help you locate while answering This is high praise indeed. Is the conclusion of
the questions. global MPI a new revelation? No, as far as the
A recent report by NITI Aayog on 2015-16 estimates are concerned. The estimates
multidimensional poverty shows that the of poverty based on consumer expenditure and
percentage of the poor has gone down from 25% using the Tendulkar committee methodology
in 2015-16 to 15% in 2019-21 and around 135 show (over a seven-year period between 2004-
million people were lifted out of poverty during 05 and 2011-12) that the number of poor came
this period. The Global Multidimensional Poverty down by 137 million despite an increase in
Index report of 2023 of the United Nations population. According to the Rangarajan
Development Programme (UNDP) and the Committee methodology, the decline between
Oxford Poverty & Human Development Initiative 2009-10 and 2011-12 is 92 million, which is 46
(OPHI), which was released recently, also shows million per annum. For a decade, it will be larger
that the incidence of the multidimensional than that of global MPI. However, in absolute
poverty index declined from 27.5% in 2015-16 to terms, the poverty ratios based on the Tendulkar
16.2% in 2019-21.In this context, we briefly and Rangarajan Committee methodologies are
examine the issues, particularly on methodology lower than as estimated by global MPI.
relating to the multidimensional poverty index, The search for non-income dimensions of
and argue that consumption-based poverty poverty possibly stems from a view that in terms
estimates are still very relevant. Multidimensional of the capabilities approach to the concept and
poverty estimates are not substitutes for National measurement of poverty, some of these
Sample Survey (NSS) consumption-based ‘capabilities’ may not be tightly linked to the
poverty ratios. In the end, we also flag some privately purchased consumption basket in terms
concerns about consumption expenditure of which the poverty lines are currently drawn.
surveys and the need to correct them. The report Therefore, poverty based on income or
of the Global Multidimensional Poverty Index consumption is different from deprivations based
(MPI) 2018 says: “India has made momentous on education or health. As pointed out by the
progress in reducing multidimensional poverty. Expert Group to Review the Methodology for
The incidence of multidimensional poverty was Measuring Poverty (2014), there are reservations
almost halved between 2005/06 and 2015/16, on using multiple indicators as these

Click Here For Bundle PDF Course | support@guidely.in Page 1 of 12


SBI Clerk & RRB PO Mains PDF Course 2023
ENGLISH Day - 45

multidimensional indicators/measures raise more fruitful. However, he is critical of


several issues regarding their measurability, multidimensional indices. He says that
aggregation across indicators, and, crucially, of “collapsing many relevant but not necessarily
databases that provide the requisite information commensurate dimensions into a single index
at reasonably short intervals. These need to be defined as an arbitrarily weighted sum of
considered and evaluated carefully. For example, disparate indexes makes little sense. The Human
there is a problem with the child mortality Development Index pioneered by the United
indicator as it is for population groups and not for Nations Development Programme is an example
households. Aggregation is another problem. In of an arbitrarily weighted sum of non-
principle, they should be independent. Access to commensurate indexes. It certainly is not a
safe drinking water, for example, cannot be multidimensional conceptualisation in any
aggregated with indicators such as child meaningful sense but simply yet another arbitrary
mortality. Even in respect of independent unidimensional index”. In the minds of most
indicators, analytically appropriate rules of people, being rich or poor is associated with
aggregation require that all of them relate to the levels of income. The various non-income
same household. More generally, this indicators of poverty are in fact reflections of
requirement poses several data constraints. It inadequate income. Defining poverty in terms of
may be noted that we are not against income or in the absence of such data in terms of
multidimensional poverty or deprivations. One expenditure seems most appropriate, and it is
can analyse the progress of non-income this method which is followed in most countries.
indicators such as education, health, sanitation, We do not have official data on consumer
drinking water, and child mortality over time with expenditure after 2011-12 to make a comparison
income or consumption poverty. But, converting with trends in the multidimensional poverty index.
all of them into an index poses several problems. The survey data on consumption expenditures
Deaton and Drèze (2014) also indicate that “it is done in 2017-18 have not been released
important to supplement expenditure-based officially. In the absence of such data, there have
poverty estimates with other indicators of living been several studies on poverty using indirect
standards, relating for instance to nutrition, methods and using Centre for Monitoring Indian
health, education and the quality of the Economy (CMIE) and Periodic Labour Force
environment”. Survey (PLFS) data sources — and they have
On multidimensional issues, Srinivasan (2007) come up with differing conclusions. The
says viewing public services as another consumption expenditure survey is being
dimension besides consumption in a conducted in the current year. For purposes of
multidimensional conceptualisation of poverty is comparison, we need to follow one method.

Click Here For Bundle PDF Course | support@guidely.in Page 2 of 12


SBI Clerk & RRB PO Mains PDF Course 2023
ENGLISH Day - 45

Therefore, it is best to wait for the survey results the estimated target by global Multidimensional
to be published. Earlier surveys clearly indicate Poverty Index.
that the poverty ratio comes down strongly during III. Multidimensional index involves a tiring
a period of high growth. If you look at recent process as it is difficult to collect/analyse data on
years including the COVID-19 period, the growth health, education and other factors individually.
rate has come down. There is ground to believe a) Only II
that the rate of reduction in the poverty ratio must b) Both I and II
have slowed down. This is at best a guess. We c) Both II and III
need to wait for consumption expenditure survey d) Both I and III
data. An important issue is the differences in e) All are true
aggregate consumption estimates between
National Accounts Statistics (NAS) and NSS 2) According to the given passage, what is the
data. These two estimates of consumption (NSS need for consumption-based poverty estimates ?
and NAS) do not match in any country; India is I. It is an easy and cost efficient estimation
no exception. What is perplexing is that the method when compared to other indexes that
difference in India between the NSS and the aim to reduce poverty in the country.
NAS consumption is widening over time. From a II. Consumption based poverty indexes provide
difference of less than 10% in the late 1970s, it better understanding based on income or
has come to 53.1% in 2011-12, i.e., the Survey consumption data.
Estimate is only 46.9% of NAS estimates. The III. It has no limitations and concerns related to
difference is too big to be brushed aside. The methodology, aggregation, and data availability.
National Statistical Office must study the problem a) Only III
and come out with possible suggestions to b) Both I and III
improve the collection of data through both c) Both II and III
routes. d) Both I and II
1) Which of the following statements is/are true e) All I, II and III
according to the information given in the passage
? 3) What is the need for official consumption
I. Multidimensional poverty is not equivalent or expenditure survey data when there are many
proportionate to consumption-based poverty studies related to the poverty trends in the
indices. nation?
II. Even Though according to the Rangarajan I. Only official consumption expenditure survey
Committee methodology there was a decline data are accurate and reliable.
between the years 2009-2012, it still did not meet

Click Here For Bundle PDF Course | support@guidely.in Page 3 of 12


SBI Clerk & RRB PO Mains PDF Course 2023
ENGLISH Day - 45

II. Because different studies use different III. The National Statistical Office is reluctant to
methods to assess poverty and hence have address the issue of different estimates of NSS
variable results. and NAS.
III. According to the passage it is always best to a) Only I
follow one method/data for the purpose of b) Only III
comparison. c) Both I and II
a) Only I d) Both II and III
b) Both I and II e) All I, II and III
c) Both I and III
d) Both II and III 6) Choose from the below options the
e) All I, II and III SYNONYM of the word ‘REQUISITE’ as
mentioned in the passage.
4) One of the following five questions cannot be a) surplus
answered after reading the passage, pick that as b) needless
your answer. c) extra
a) What are the challenges when it comes to d) compulsory
child mortality indicators ? e) extraneous
b) What is the difference between NSS and NAS
estimates ? 7) Choose from the below options the
c) Why is aggregating diverse/various indicators ANTONYM of the word ‘COMMENSURATE’ as
into a single index difficult in MPI? mentioned in the passage.
d) Which is better - Tendulkar committee a) corresponding
methodology or Rangarajan committee b) equivalent
methodology ? c) comparable
e) What is the observed poverty trend between d) proportionate
the years 2015 and 2021 ? e) disproportionate

5) Which of the following is/are true regarding the Directions (8-12): In each question below, four
NSS and NAS as mentioned in the passage? words printed in bold are given. These are
I. National Sample Survey and National Accounts numbered (A), (B), (C) and (D). One these words
Statistics are alway dissimilar. printed in bold might either be wrongly spelt or
II. The difference is India is however less and is inappropriate to the context of the sentence. Find
still reducing over the years. out the word that is inappropriate or wrongly
spelt, if any. The alphabet of the word is your

Click Here For Bundle PDF Course | support@guidely.in Page 4 of 12


SBI Clerk & RRB PO Mains PDF Course 2023
ENGLISH Day - 45

answer. If the words printed in bold are correctly d) D


spelt and appropriate to the context of the e) All correct
sentence then mark (e), i.e. 'All Correct', as your
answer. 11) Then there are more direct measures (A) for
8) July saw record (A) rainfall in many parts of mass survillance (B) that dispense (C) with any
Himachal Pradesh, Punjab and Uttarakhand and camouflage (D).
the rare (B) cite (C) of the Yamuna nearly a) A
spilling (D) into the Red Fort in Delhi. b) B
a) A c) C
b) B d) D
c) C e) All correct
d) D
e) All correct 12) However, this expresses (A) children to heat
(B) stress and other harsh (C) weather conditions
9) The anouncement (A) by Prime Minister and puts them at risk of injury (D) and accidents.
Narendra Modi of a scheme to help the urban a) A
poor build houses in cities is a signal (B) that the b) B
government is to undertake (C) one more plan to c) C
address the problem of a shortage (D) in urban d) D
housing. e) All correct
a) A
b) B Directions (13-17): In each question a sentence
c) C is given with a blank. You are provided with three
d) D fragments I, II and III. You have to identify which
e) All correct statement/statements can carry forward the
given sentence in the most logical way so as to
10) At the Consulative (A) Committee (B) make the sentence coherent and contextually
meeting, Minister for Environment, Forest and correct.
Climate Change Bhupendra Yadav spoke of the 13) ________________________________________
importance of the conservation (C) of wetlands so that I can regain my strength and well-being.
(D) in the country. I. I am totally sick and I want to rest and recover
a) A II. I have decided to prioritize self-love and eat
b) B junk food
c) C

Click Here For Bundle PDF Course | support@guidely.in Page 5 of 12


SBI Clerk & RRB PO Mains PDF Course 2023
ENGLISH Day - 45

III. I am planning to follow my doctor's advice c) Both I and III


diligently d) Both II and III
a) Only I e) All I, II and III
b) Both I and II
c) Both I and III 16) She practiced every day and night to crack
d) Only III the challenging
e) All I, II and III ___________________________________.
I. dance routine for the upcoming performance
14) The recently married couple has started their II. huge rock to make the beautiful sculpture
own business, III. fish tank as she calls herself a zoophobic, one
___________________________________________ who hates pets
________. a) Only I
I. as both of them are superstitious and have b) Only II
many believes c) Only III
II. Embarking on a new entrepreneurial journey d) Both I and II
together e) Both I and III
III. Hoping to build a successful enterprise that
supports their financial needs 17) My aunt suffers from knee pain and is
a) Only II planning to
b) Only III _________________________________________.
c) Both I and II I. undergo physiotherapy to reduce the pain
d) Both II and III symptoms
e) All I, II and III II. undergo knee surgery as it was suggested by
her family doctor
15) There are many job openings for this year, III. undergo knee surgery as she is not well-off
but my aim a) Only III
________________________________________. b) Both I and II
I. is to secure a position that aligns with my c) Both I and III
career goals and skills d) Both II and III
II. is to fit in a challenging and passionate role e) All I, II and III
III. is to find a position in the banking sector
which is my long term goal Directions (18-22): Read each question carefully
a) Only I to find out which parts have errors and parts that
b) Both I and II are error free. Choose the option which has the

Click Here For Bundle PDF Course | support@guidely.in Page 6 of 12


SBI Clerk & RRB PO Mains PDF Course 2023
ENGLISH Day - 45

correct/error free parts as your answer. Choose question clerical interpretations/C that present
option (e) as your answer if all the four parts are Islam as an exclusivist and intolerant religion/D.
correct. a) AC
18) Amidst this unfolding discourse, whatever/A, b) AD
a prevailing challenge in the Indian context/B lie c) BC
in effectively channelling these debates/C to d) CD
generate substantive and pertinent e) All correct
contributions/D.
a) AB 21) The history of civilisation shows that the/A
b) AC conclusions drawn from the simple/B observation
c) BD that the sun cross the sky/C every day could be
d) CD even varied and contradictory/D.
e) All correct a) AB
b) BC
19) By announcing the candidates first, the/A c) BD
party leadership will be in a position to/B pacify d) CD
the disappointed aspirants with an assurance/C e) All correct
of some kind of future reward in the post-election
period/D. 22) Moreover, as a signatory of the Paris/A
a) AC Agreement, India has along lectured all/B about
b) BC the necessary to protect/C the environment,
c) BD forest and biodiversity/D.
d) CD a) AC
e) All correct b) AD
c) BC
20) One of the reason for this sort of d) CD
Islamophobia/A to rage unabated among the e) All correct
world is the failure/B of Muslim societies to

Click Here For Bundle PDF Course | support@guidely.in Page 7 of 12


SBI Clerk & RRB PO Mains PDF Course 2023
ENGLISH Day - 45

Click Here to Get the Detailed Video Solution for the above given Questions
Or Scan the QR Code to Get the Detailed Video Solutions

Answer Key with Explanation

1) Answer: B poverty index as mentioned in the given


The given statements I and II are true and valid passage.
as they can be inferred from the given passage. Statement I is absolutely wrong/incorrect
However statement III is wrong/incorrect. because no such information has been given in
III is wrong because - From the given passage the passage.
we can understand that it is not difficult to
analyse the various factors but the difficulty is 3) Answer: E
when all analysis must be converted into a single All the given three statements I, II and III are the
index. reasons to wait for the official survey data in
Sentence from the given passage - One can spite of existing data from various studies.
analyse the progress of non-income indicators
such as education, health, sanitation, drinking 4) Answer: D
water, and child mortality over time with income Only the question mentioned in option d cannot
or consumption poverty. But, converting all of be answered after reading the given passage
them into an index poses several problems. thoroughly because there is no relevant
Hence option b - I and II are true, is the right information in comparison with the two
answer for this question. committee’s methodologies.
The passage has got only the factual information
2) Answer: C provided by the two committees and nothing
Both the statements II and III are the exact more.
reasons for the need for consumption based

Click Here For Bundle PDF Course | support@guidely.in Page 8 of 12


SBI Clerk & RRB PO Mains PDF Course 2023
ENGLISH Day - 45

Other given questions can be answered from the opposite in meaning and hence the antonyms of
given passage as related information is the given word.
available.
8) Answer: C
5) Answer: A The word C is inappropriate. It should be ‘sight’
Among the three statements given only I is true instead of ‘cite’.
with respect to the NSS and NAS surveys. Sight - the ability to see
Statement II is wrong because - the difference is Cite - to mention something or use somebody’s
huge in India exact words
Statement III is wrong because - there is no such Rare sight - rare view
statement made in the given passage The sentence: July saw record rainfall in many
parts of Himachal Pradesh, Punjab and
6) Answer: D Uttarakhand and the rare sight of the Yamuna
Out of the given options - option ‘d’ is the right nearly spilling into the Red Fort in Delhi.
answer. Compulsory is a synonym of the given
word requisite. 9) Answer: A
Requisite - necessary for a particular purpose The word A is misspelt in the given sentence
All the other four words are mere opposites of and hence found inappropriate.
the word ‘requisite’. The correct spelling is ‘announcement’.
Requisite means important/necessary but The sentence: The announcement by Prime
‘surplus, needless, extra and extraneous means Minister Narendra Modi of a scheme to help the
extra or unimportant or unnecessary. urban poor build houses in cities is a signal that
the government is to undertake one more plan to
7) Answer: E address the problem of a shortage in urban
Option e - disproportionate is the right antonyms housing.
of the given word commensurate which means
measurable. 10) Answer: A
Commensurate - corresponding in size, In the given sentence the word A is misspelt, the
importance, quality correct spelling is consultative.
Corresponding, equivalent, comparable and Consultative - intended to give professional
proportionate have similar meaning to the word advice or recommendations.
commensurate, whereas disproportionate is The sentence: At the Consultative Committee
meeting, Minister for Environment, Forest and

Click Here For Bundle PDF Course | support@guidely.in Page 9 of 12


SBI Clerk & RRB PO Mains PDF Course 2023
ENGLISH Day - 45

Climate Change Bhupendra Yadav spoke of the I am planning to follow my doctor's advice
importance of the conservation of wetlands in diligently so that I can regain my strength and
the country. well-being.

11) Answer: B 14) Answer: D


The given word B is misspelt and hence found Both the statements II and III fit in the given
inappropriate in the sentence. blank in the most appropriate and correct way
The right spelling is ‘surveillance’. possible.
Surveillance - the careful watching of somebody ‘I’ is irrelevant to the context of the sentence and
who may have done something wrong hence discarded.
The sentences:
12) Answer: A The recently married couple has started their
In the given sentence the word A is inappropriate own business, embarking on a new
and must be replaced with a new word to make entrepreneurial journey together.
the sentence correct and meaningful. The recently married couple has started their
‘Expresses’ is wrong according to the context of own business, hoping to build a successful
the sentence instead it must be ‘exposes’. enterprise that supports their financial needs.
Expresses - to say or write your feelings,
opinions 15) Answer: E
Exposes - to show something that is usually All the given three fragments fit in the blank in
hidden/to put somebody/something or yourself in the most appropriate way making the sentence
a situation that could be difficult or dangerous contextually correct and meaningful.
The sentences:
13) Answer: C There are many job openings for this year, but
Both phrases/fragments I and III suit the context my aim is to secure a position that aligns with my
of the incomplete sentence. career goals and skills.
II is inappropriate and meaningless because There are many job openings for this year, but
eating junk cannot regain strength and well my aim is to fit in a challenging and passionate
being. role.
The sentences: There are many job openings for this year, but
I am totally sick and I want to rest and recover so my aim is to find a position in the banking sector
that I can regain my strength and well-being. which is my long term goal.

Click Here For Bundle PDF Course | support@guidely.in Page 10 of 12


SBI Clerk & RRB PO Mains PDF Course 2023
ENGLISH Day - 45

16) Answer: A Error in C - lie must be replaced with ‘lies’ to


Only fragment I fits in the given blank making the make it right
sentence logically and contextually correct. The correct sentence: Amidst this unfolding
To crack(in this context) - succeed in doing discourse, however, a prevailing challenge in the
something Indian context lies in effectively channelling
Both II and III involve ‘breaking something’ which these debates to generate substantive and
is not aligned with the context of the incomplete pertinent contributions.
sentence given as question.
The sentence: 19) Answer: E
She practiced every day and night to crack the The sentence and all its parts A, B, C and D are
challenging dance routine for the upcoming correct and error free. Hence option e - all
performance. correct is the right answer for this question.

17) Answer: B 20) Answer: D


Both the fragments I and II fit in the blank Errors are in parts A and B. The correct parts are
correctly making the sentence complete and C and D.
meaningful. Error in A - One of the reason is incorrect,
III - is contradictory (well-off means to be rich) instead it must be ‘one of the reasons’ (1 out of
The sentences: many is the meaning - many represents multiple
My aunt suffers from knee pain and is planning things and hence it must be a plural word)
to undergo physiotherapy to reduce the pain Error in B - Among the world is incorrect, it must
symptoms. be across the world instead (among is used
My aunt suffers from knee pain and is planning when talking about multiple entities, for eg -
to undergo knee surgery as it was suggested by among the nations, among the states, But world
her family doctor. is just one so ‘across the world’ would be the
correct phrase)
18) Answer: C Among - in or concerning a particular group of
Parts A and C contain errors, the correct parts people or things
are B and D. Across - from one side of something to the other
Error in A - Whatever is incorrect, instead it must The correct sentence: One of the reasons for
be however this sort of Islamophobia to rage unabated
Whatever - any or every; anything or everything across the world is the failure of Muslim societies
However - although something is true

Click Here For Bundle PDF Course | support@guidely.in Page 11 of 12


SBI Clerk & RRB PO Mains PDF Course 2023
ENGLISH Day - 45

to question clerical interpretations that present 22) Answer: B


Islam as an exclusivist and intolerant religion. Part B and C are erroneous and the correct parts
are A and D.
21) Answer: A Error in B - ‘has along lectured all’ (meaningless
Error in parts C and D. The correct or error free and incorrect), instead it must be ‘has all along
parts are A and B. lectured’ (meaning of ‘all along’ is ‘for a long
Error in C - sun crosses the sky is the right time’ or ‘from the start’)
phrase (it happens everyday hence it must be Error in C - necessary is wrong/incorrect instead
crosses instead of cross) it must be necessity
Error in D - ‘be even’ is wrong instead the right Necessary is an adjective and necessity is a
phrase is ‘even be’ (grammatically a correct and noun
better phrase) The correct sentence: Moreover, as a signatory
The correct sentence: The history of civilisation of the Paris Agreement, India has all along
shows that the conclusions drawn from the lectured about the necessity to protect the
simple observation that the sun crosses the sky environment, forest and biodiversity.
every day could even be varied and
contradictory.

Click Here For Bundle PDF Course | support@guidely.in Page 12 of 12


SBI Clerk & RRB PO Mains PDF Course 2023
Reasoning Ability Day - 46 (Eng)

Reasoning Ability
Directions (1-5): Study the following information b) Three
carefully and answer the questions given below. c) One
Words arrangement machine when given an d) Zero
input line of words rearranges them following a e) None of these
particular rule in each step. The following is an
illustration of input and rearrangement. 3) Which of the following word is exactly between
Input: Find word apple dog sun taxi export horse “Fnemy” and “Lite” in step IV?
cow a) Mion
Step I: Gind xord bpple eog tun uaxi fxport iorse b) Nake
dow c) Gox
Step II: Bpple gind eog tun uaxi fxport iorse dow d) Call
xord e) None of these
Step III: Bpple dow gind eog tun fxport iorse uaxi
xord 4) How many vowels are there in step V?
Step IV: Bpple dow eog gind fxport iorse tun uaxi a) Fourteen
xord b) Fifteen
Step V: Bpple dow eog fxport gind iorse tun uaxi c) Thirteen
xord d) Twelve
Step V is the last step e) None of these
Input: Shall Ball Fox Make Lion Enemy Purple
Oxen Kite 5) In which of the following steps “Pxen” and
1) Which of the following letter is the third letter “Qurple” didn’t come adjacent to each other?
from the left of the third word from the right end a) Step II
in step IV? b) Step I
a) T c) Step IV
b) R d) Step III
c) X e) None of these
d) E
e) None of these Directions (6-10): Study the following information
carefully and answer the below questions
2) How many words are there between the words Twelve persons- Arpana, Adita, Bhakati,
“Mion” and “Qurple” in step III? Draupadi, Edwin, Ganika, Ishana, Obelia,
a) Two Rachana, Viransh, Vihaan, and Umarani are

Click Here For Bundle PDF Course | support@guidely.in Page 1 of 10


SBI Clerk & RRB PO Mains PDF Course 2023
Reasoning Ability Day - 46 (Eng)

standing in two staircases viz., Staircase1 and ages of Edwin and Umarani. The one whose age
Staircase2. Each staircase has six steps viz., the is 28 stands north-east of the one whose age is a
lowermost step is numbered as one and the square value. Rachana is 10 years elder than the
topmost step is numbered as six. Staircase1 is to one who stands on the same step of Arpana.
the west of Staircase2. They are of different age Obelia’s age is twice that of Vihaan. Vihaan is
lies between 19 to 50. All the information is not elder
necessarily in the same order. than Arpana but not elder than Bhakati.
Note: If the person's name starts with a vowel 6) How many persons are elder than Rachana?
then he/she stands on odd-numbered steps and a) Four
if the number of letters of the person’s name is b) Two
even then he/she stands on an odd numbered c) Five
staircase. d) Three
Edwin stands two steps above the one whose e) None
age is 40, both are stands at the same staircase.
Bhakati stands three steps above the one who 7) Who among the following person’s age is an
stands two steps below Adita. The person stands odd number?
immediately above Umarani stands two steps I. The one who stands east of Ishana
below the one whose age is 30. The one whose II. The one who stands west of Bhakati
age is 30 stands north-east of Draupadi whose III. The one who stands on the same step of
age is a square value. As many steps above Umarani
Draupadi as below the one whose age is 20. IV. Viransh
Neither Adita’s age nor Edwin’s age is 20. a) Only I and II
Ganika stands two steps below the one whose b) Only III and IV
age is two years more than Arpana. Vihaan’s age c) Only II and IV
is an odd number. Neither Umarani nor Edwin d) Only II and III
stands on the same step of Arpana. The one who e) Only I and III
stands immediately below Ganika has age is 10
years more than Ganika, both are standing at the 8) Who among the following person stands on
same staircase. Only eleven years difference the same step of the one whose age
between the ages of Bhakati and Ganika. Ishana is 22?
stands above Rachana. Draupadi is elder than a) The one who stands on the same step of
the one who stands west of Edwin. The Viransh
difference between the ages of Viransh and b) The one who stands west of Bhakati
Edwin is the same as the difference between the c) The one whose age is 20

Click Here For Bundle PDF Course | support@guidely.in Page 2 of 10


SBI Clerk & RRB PO Mains PDF Course 2023
Reasoning Ability Day - 46 (Eng)

d) Obelia sit between J and the one who belongs to ITBP.


e)Draupadi F belongs to BSF and sits
second to the right of the one who belongs to
9) What is the sum of the age of Umarani, ITBP and second to the left of E
Vihaan, Ganika? who doesn’t sit at the end of the row. The one
a) 90 who belongs to NAVY sits
b) 101 fourth from one of the ends of the row and sits
c) 98 adjacent to the one who
d) 94 belongs to BSF. L sits four places away from the
e) 96 one who belongs to NAVY.
Six persons sit between L and the one who
10) Which of the following statement(s) is/are belongs to ARMY. The one who
true? belongs to RPF sits third to the right of the one
I. Ishana’s age is 43 who belongs to ARMY. Three
II. Draupadi and Rachana stand on different persons sit between the one who belongs to RPF
steps and P who sits immediate
III. Edwin is elder than Adita left of the one who belongs to SSB, who sits at
IV. Obelia stands above Arpana one of the extreme ends. The
a) Only I and III number of persons sitting to the left of the one
b) Only III and IV who belongs to SSB is multiple of eleven.
c) Only I and II 11) Who among the following person sits fourth
d) Only II and IV to the left of the one who sits second to the right
e) All I, II, III and IV of J?
a) The one who sits immediate left of F
Directions (11-15): Study the following b) Q
information carefully and answer the questions c) The one who belongs to NAVY
given below. d) K
Certain number of persons are sitting in a row e) None of these
facing north direction and
they belong to different armed forces. Only three 12) How many persons are sitting in the row?
persons sit between Q a) Thirty-Three
and K who belongs to CRPF. J sits three places b) Twenty-Three
away from K. Four persons c) Twenty
d) Twenty-Two

Click Here For Bundle PDF Course | support@guidely.in Page 3 of 10


SBI Clerk & RRB PO Mains PDF Course 2023
Reasoning Ability Day - 46 (Eng)

e) Twenty-One Directions (16-20): Some question is followed by


some set of statements. Study the following
13) Which of the following statement(s) is/are information carefully and decide which of the
true with respect to the final arrangement? following statement is sufficient to answer the
a) K sits fourth to the right of the one who question.
belongs to NAVY 16) Seven persons are living in seven floored
b) F sits immediate right of J building marked 1 to 7 from bottom to top. Who
c) P sits immediate right of the one who belongs among the following person lives immediately
to Army below Q?
d) E sits third to the left of L Statement:
e) Both C and D Column I Column II
A. R lives on floor D. S lives two floors
14) How many persons sit between the one who number 5. Two below U. T and S are
belongs to ITBP and the one who sits third to the persons are living living on the adjacent
left of Q? between R and S. floor.
a) As many persons sit to the right of the one B. P lives on an even E. T lives immediately
who belongs to ARMY number floor. Two below S. Two persons
b) Eleven persons are living are living between S and
c) Six between Q and P. R.
d) As many persons sit between L and the one C. Two persons are F. Neither R nor S lives
who belongs to RPF living between V and on the adjacent floor of
e) Ten U. T and U are not V. Three persons are
living together. living between V and Q.
15) If F is related to the one who belongs to a) Both BD and CE
NAVY, in the same way L is related to the one b) Only AF
who belongs to CRPF, then who among the c) Both AE and CD
following person is related to the one who d) Only CD
belongs to SSB? e) None of these
a) Q
b) The one who belongs to ARMY 17) Seven persons are sitting in a row facing
c) J north, how many persons are sitting between C
d) The one who belongs to RPF and F?
e) P

Click Here For Bundle PDF Course | support@guidely.in Page 4 of 10


SBI Clerk & RRB PO Mains PDF Course 2023
Reasoning Ability Day - 46 (Eng)

Statement: the boxes.


Column I Column II B. Two boxes are kept E. F and E are not
A. A sits third from the D. F neither sits between E and F, which is sitting together.
left end. Three persons adjacent to A nor E. C kept immediately below C.
are sitting between A sits immediate right of C. Box A is kept at the top. F. Neither C nor D
and B. G. One box is kept between A is kept at the top.
B. Two persons are E. F and E are not and F, which is kept three One box is kept
sitting between A and E, sitting together. places above E which is not between B and E.
who sits second from the adjacent to C.
right end. a) Both BD and CE
C. C sits second to the F. Neither B nor A sits b) Only AF
left of B. One person sits adjacent to C. Three c) Both AE and CD
between D and A. persons are sitting d) Only BF
between C and D, e) None of these
who sits immediate
left of F. 19) In a family of seven persons, how L is related
a) Both BD and CE to C?
b) Only AF Statement:
c) Both AE and CD Column I Column II
d) Only BD A. C is the only D. H is the brother of C.
e) None of these daughter of B, who is
the father of H.
18) Six boxes are kept one above the other in B. C is the wife of A, E. E is the mother of L
the form of a stack. Which of the following box is who is the son-in-law and married to Q.
kept immediately below B? of B.
Statement: C. D is the daughter- F. L is the son of D,
Column I Column II in-law of E, who is the who is the only
A. A and C are kept D. F and B are mother of H. daughter-in-law of B.
together. Two boxes are kept together. B is a) Both BD and CE
kept between C and D. kept three places b) Only AF
below A, which is c) Both AE and CD
not kept adjacent d) Only BD
to E. D is one of e) None of these

Click Here For Bundle PDF Course | support@guidely.in Page 5 of 10


SBI Clerk & RRB PO Mains PDF Course 2023
Reasoning Ability Day - 46 (Eng)

20) Seven persons P, Q, R, S, T, U and V visited One person visited


the water park during a week starting from between Q and U. R
Monday to Sunday but not necessarily in the and T visited on
same order, who among the following person consecutive days.
visited on Thursday? C. R visited on Sunday. F. One person visited
Statement: Three persons visited between T and V.
Column I Column II between R and Q, who
A. S visited on Friday. D. P and U visited visited immediately
Two persons were together. S and T after P
visited between S and visited together but a) Both BD and CE
P. none of them visited b) Only AF
on Thursday c) Both AE and CD
B. Two persons visited E. P visited d) Both AE and CF
between U and V. immediately before Q. e) None of these
Click Here to Get the Detailed Video Solution for the above given Questions
Or Scan the QR Code to Get the Detailed Video Solutions

Answer Key with Explanation

Directions (1-5): The first letter of the word which has the smallest
1) Answer: D place value is placed at the extreme left end and
2) Answer: A the first letter of the word which has the largest
3) Answer: B place value is placed at the extreme right end in
4) Answer: C step II. The first letter of the word which has
5) Answer: D second smallest place value is placed second
The first letters of all the words are changed to from the left end and the first letter of the word
the next letter of the alphabetical series in step I. which has second highest place value is placed

Click Here For Bundle PDF Course | support@guidely.in Page 6 of 10


SBI Clerk & RRB PO Mains PDF Course 2023
Reasoning Ability Day - 46 (Eng)

second from the right end in step III, and so  Bhakati stands three steps above the one
on…, who stands two steps below Adita.
Input: Shall Ball Fox Make Lion Enemy Purple From the above condition, there are two
Oxen Kite possibilities.
Step I: Thall call gox nake mion fnemy qurple
pxen lite
Step II: Call gox nake mion fnemy qurple pxen
lite thall
Step III: Call fnemy gox nake mion pxen lite
qurple thall
Step IV: Call fnemy gox nake mion lite pxen
qurple thall Again we have,
Step V: Call fnemy gox lite mion nake pxen  The person stands immediately above
qurple thall Umarani stands two steps below the one
Directions (6-10): whose age is 30.
6) Answer: A  The one whose age is 30 stands north-
7) Answer: A east of Draupadi whose age is a square
8) Answer: B value.
9) Answer: D
10) Answer: A
Final Arrangement

Again we have,
 As many steps above Draupadi as below
the one whose age is 20.
 Neither Adita’s age nor Edwin’s age is 20.
 Ganika stands two steps below the one
We have,
whose age is two years more than
 Edwin stands two steps above the one
Arpana.
whose age is 40, both are stands at the
 Vihaan’s age is an odd number.
same staircase.

Click Here For Bundle PDF Course | support@guidely.in Page 7 of 10


SBI Clerk & RRB PO Mains PDF Course 2023
Reasoning Ability Day - 46 (Eng)

 Neither Umarani nor Edwin stands on the  The one whose age is 28 stands north-
same step of Arpana. east of the one whose age is a square
value.
 Rachana is 10 years elder than the one
who stands on the same step of Arpana.
 Obelia’s age is twice that of Vihaan.
 Vihaan is elder than Arpana but not elder
than Bhakati.
From the above condition, case 2 is eliminated,
case1 shows the final arrangement.
Again we have,
 The one who stands immediately below
Ganika has age is 10 years more than
Ganika, both are standing at the same
staircase.
 Only eleven years difference between the
ages of Bhakati and Ganika.
 Ishana stands above Rachana.
Directions (11-15):
From the above condition
11) Answer: C
12) Answer: B
13) Answer: D
14) Answer: A
15) Answer: E
Final arrangement

Again we have,
 Draupadi is elder than the one who We have,
stands west of Edwin.  Only three persons sit between Q and K
 The difference between the ages of  K belongs to CRPF
Viransh and Edwin is the same as the  J sits three places away from K
difference between the ages of Edwin and  Four persons sit between J and the one
Umarani. who belongs to ITBP

Click Here For Bundle PDF Course | support@guidely.in Page 8 of 10


SBI Clerk & RRB PO Mains PDF Course 2023
Reasoning Ability Day - 46 (Eng)

 F belongs to BSF
 F sits second to the right of the one who
belongs to ITBP
 F sits second to the left of E
From this condition, there are four possibilities,

Again we have,
 Three persons sit between the one who
belongs of RPF and P
 P sits immediate left of the one who
belongs to SSB, who sits at one of the
extreme ends.
Again we have,
 The number of persons sitting to the left of
 E doesn’t sits at the end of the row
the one who belongs to SSB is multiple of
 The one who belongs to NAVY sits fourth
eleven.
from one of the ends of the row
From this condition, Case2 gets eliminated.
 The one who belongs to NAVY sits
Case4 shows the final arrangement.
adjacent to the one who belongs to BSF
 L sits four places away from the one who
belongs to NAVY
 Six persons sit between L and the one
who belongs to ARMY
 The one who belongs to RPF sits third to
the right of the one who belongs to ARMY
From this condition, Case1 and Case3 gets
eliminated.

Click Here For Bundle PDF Course | support@guidely.in Page 9 of 10


SBI Clerk & RRB PO Mains PDF Course 2023
Reasoning Ability Day - 46 (Eng)

16) Answer: B 19) Answer: B


From combining statement AF, we have: From statement AF, we have:

Thus, L is the nephew of C.

Clearly, S lives immediately below Q.


20) Answer: C
Hence, option B is the correct choice.

17) Answer: D
From BD we have:

Clearly, three persons are sitting between F and


C.

18) Answer: E
From statement CD, we have:
Thus, from both AE and CD, we have V visited
on Thursday.

Box D is kept immediate below B.

Click Here For Bundle PDF Course | support@guidely.in Page 10 of 10


SBI Clerk & RRB PO Mains PDF Course 2023
Quantitative Aptitude Day – 46 (Eng)

Quantitative Aptitude

Directions (01 - 04): Study the following ratio of d:e is 23:16. Find the distance covered
information carefully and answer the questions by boat B in 2t hours in still water?
given below. a) 258
The ratio of speed between boats A and B in still b) 268
water is 2:3. The speed of boat C is 25% more c) 298
than the speed of boat A in still water. The ratio d) 288
of speed between the streams of rivers P and Q e) 388
is 2:3. The speed of boat D is 25% less than the
speed of boat B in still water. The speed of the 3) Boat A starts its journey at 8 a.m. from point m
stream of the river R is 25% less than the speed towards n with the stream, and boat B starts its
of the stream of river P. Boat C covered 49 km journey from n towards m at the same time in
upstream in river Q in 3.5 hours, and boat D river P. They meet at 11 a.m. Find the time taken
covered 115.5 km downstream in river R in 5.5 by boat C to go from n to m and return to m in
hours. the river P?
1) Boat A covers _____ km distance in a) 14.5
downstream of river P in _____ and boat B b) 12.5
covers ______ Km distance upstream of river Q c) 11.5
in _______. d) 16.5
Find which of the following satisfies the blank? e) None of these
I.110,5.5,165,5.5
II. 178,9.4,117,6.5 4) Find the ratio of the distance covered by boat
III. 80,4,54,3 B in river Q upstream in 4 hours and the distance
a) only II covered by boat D in river R downstream in 8
b) Only I hours?
c) only III a) 3:7
d) All true b) 2:5
e) All false c) 7:11
d) 5:11
2) Boat C covers d km of distance downstream in e) None of these
river R in t hours, and boat D covers e km of
distance upstream in river Q in (t + 2) hours. The

Click Here For Bundle PDF Course | support@guidely.in Page 1 of 12


SBI Clerk & RRB PO Mains PDF Course 2023
Quantitative Aptitude Day - 46 (Eng)

Directions (05–08): Study the following total tank fills in K minutes. Find which of the
information carefully and answer the questions following is true?
given below. I. K= 21 and pipes C and M are open.
In a tank, there are four inlet pipes [A, B, C, and II. K= 9 and pipes D and O are open.
D] and three outlet pipes [M, N, and O]. The III. K=39 and pipes C and N open.
efficiency ratio of pipes A and B is 3:5. The a) only I
efficiency of pipe C is double that of pipe D. b) only II and III
Pipes A and M fill the tank in 60 minutes. Pipes B c) only I and II
and C fill the tank in 20/3 minutes. Pipes M and d) Only I and III
N together empty the tank in 12 minutes. The e) all true
time taken by pipe C to fill the tank is half of the
time taken by pipe M to empty the tank. Pipe B 7) Pipe D and pipe M are open and fill the tank in
fills the tank in t minutes. The efficiency of pipe N s minutes when pipe D is working at double
is 20% higher than that of pipe O. The ratio of efficiency. Find the percentage of the full tank
time taken to empty the full tank by M and N is empty by pipe O if it opens for [s+t]/2 minutes?
3:2. a) 82.3%
5) Inlet pipes A and B are open with outlet pipes b) 87.5%
N and O. After some time, all pipes are closed, c) 92.5%
and pipe C is open, and fill the remaining tank in d) 98.3%
5 minutes. Find the difference between the e) None of these
percentage of tank filling by pipe C and the
percentage of tank emptying by pipe N? 8) Pipe A, M, B, and O open in alternative
a) 26.67% minutes in the same sequence and fill the tank in
b) 36.67% k minutes, and pipe A, B, and C open in
c) 46.67% alternative minutes in the same sequence and
d) 56.67% fill the tank in l minutes. Find the value of 2l+
e) 66.67% 7k=?
a) 507.12
6) Pipe A and pipe B are open for some time and b) 558.32
fill 40% of the tank, after that, one inlet pipe and c) 154.32
one outlet pipe are open for some time, and the d) 158.32
e) None of these

Directions (09–12): Study the following information carefully and answer the questions given below.

Click Here For Bundle PDF Course | support@guidely.in Page 2 of 12


SBI Clerk & RRB PO Mains PDF Course 2023
Quantitative Aptitude Day - 46 (Eng)

The bar graph shows the average of dancers and singers, the difference between singers and writers, the
difference between writers and painters, and the average of writers and painters in five different cities [A,
B, C, D, and E].

Note:
I. For cities A, B, and C, Number of singers > Number of dancers > Number of painters > Number of
writers;
II. For cities D and E, Number of dancers > Number of singers > Number of writers > Number of painters.
9) Out of the total singers in A, 50% are classical b) 16
singers. The ratio of classical singers A and B is c) 12
3:4. 40% of the total singers in C and 75% of the d) 19
total singers in D are classical singers. The e) 13
number of classical singers in E is half of the
total classical singers in C and B together. Find 10) The ratio of kathak and Bharatanatyam
the difference between the total classical singers dancers in city A is 5:3, and the ratio of the same
A and E together and the total classical singers B in city C is 5:4. The no. of kathak dancers in city
and D together? B is 2 more than the same in city A and the no.
a) 14 of Bharatanatyam dancers in city D is 6 more

Click Here For Bundle PDF Course | support@guidely.in Page 3 of 12


SBI Clerk & RRB PO Mains PDF Course 2023
Quantitative Aptitude Day - 46 (Eng)

than the same in city C. Find the total kathak a) 6.54


dancers in four cities together is approximately b) 6.68
what percent of total Bharatanatyam dancers in c) 6.72
four cities together? [Only these two types of d) 6.84
dancers are there] e) None of these
a) 123.24%
b) 125.32% 12) The number of players in city C is 25% more
c) 145.32% than the number of painters in City C. The ratio
d) 127.27% of the number of painters and players in city D is
e) 156.32% 2:3. The total number of players in cities C, D,
and A is 123. Find the ratio of the number of
11) Out of the total number of writers in E, 50% painters and players in A?
published 5 books, and the rest published 8 a) 7:11
books. Out of the total writers in D, 25% of b) 3:8
writers published 9 books, and the rest published c) 5:7
7 books. Every writer in C published 6 books. d) 3:11
Find the average number of books published by e) None of these
writers in these three cities?

Directions (13–16): Study the following information carefully and answer the questions given below.
The given pie chart shows the percentage distribution of the number of employees in four different [IT,
HR, marketing, and accounts] departments of a company.

Click Here For Bundle PDF Course | support@guidely.in Page 4 of 12


SBI Clerk & RRB PO Mains PDF Course 2023
Quantitative Aptitude Day - 46 (Eng)

Note:
I. M2-M-132 = 0, and M is a positive number.
II. N,34,35,37,41,44
III. O% of 120 - 8.8 = 20
IV. P,47,71,110,166
IV. The total number of employees of the company is 50×O.
13) Out of the total no. of employees in the HR 14) The ratio of males and females in the
department, 25% use public transport , 25% of accounts department is 2:1 and the ratio of
the remaining use bikes and the rest using a car males and females in the marketing department
to reach the office. The Ratio of the no. of is 7:2. The average age of females in the
employees who use public and private transport accounts and marketing department is 28 years,
in the IT department is 4:3. There is no person and the average age of males in the accounts
using a car to reach office in IT department. Find and marketing department is 32 years. Find the
the no. of employees in the HR department who average age of all the employees of the accounts
use car is what percent of the no. of employees and marketing department?
use private transport in IT department? a) 35.32
a) [p+16.25]% b) 37.25
b) [p+12.25]% c) 39.32
c) [p+20.25]% d) 30.93
d) [M+30.25]% e) 34.22
e) [O+10.25]%

Click Here For Bundle PDF Course | support@guidely.in Page 5 of 12


SBI Clerk & RRB PO Mains PDF Course 2023
Quantitative Aptitude Day - 46 (Eng)

15) The no. of employees in the HR department sold, the ratio of wooden table and plastic table
is increasing by [2M+1]%, and the no. of sold, and the ratio of wooden table and plastic
employees in the IT department is increasing by chair sold. Some values are missing.
[N-3]%. Find the difference between the total no.
of employees in HR and IT together and the total
no. of employees in accounts and marketing
together? [ The no. of employees in the
Marketing and accounts departments is same]
a) 7M
b) 5O
c) 5N
d) 5P
e) None of these

17) The ratio of plastic chairs to wooden chairs


16) 75% of employees in the IT and marketing
sold in A is 4:5. The no. of wooden tables sold in
departments are freshers, and the rest are
shop A is 70% of the no. of wooden chairs sold
experienced. 25% of employees in the HR and
in shop A. Out of the total wooden tables sold in
Accounts departments are freshers, and the rest
shop A, 40% are round tables, and the rest are
are experienced Find the ratio of number of
square tables. Out of the total no. of plastic
freshers and experienced employees?
tables sold in shop A, 60% are round tables, and
a) 57:43
the rest are square tables. Find the difference
b) 55:51
between the total no. of round tables sold and
c) 58:53
the total no. of square tables sold?
d) 57:53
a) 5
e) None of these
b) 6
c) 4
Directions (17–20): Study the following
d) 8
information carefully and answer the questions
e) 3
given below.
In five different shops [A, B, C, D, and E]
18) The ratio of total items sold in shops B and A
Different numbers of tables [wooden and plastic]
is 6:5. The no. of plastic tables sold in shop B is
and chairs [ wooden and plastic] were sold. The
half the no. of plastic chairs sold in the same
given table shows the total number of items [
shop. The no. of wooden chairs sold in shop B is
chair+ table] sold, the ratio of table and chair

Click Here For Bundle PDF Course | support@guidely.in Page 6 of 12


SBI Clerk & RRB PO Mains PDF Course 2023
Quantitative Aptitude Day - 46 (Eng)

approximately 27.77% of the total no. of items b) 30%


sold in shop B. Find the percentage of wooden c) 20%
tables sold in Shop B out of the total no. of items d) 2%
sold. e) None of these
a) 24.33%
b) 29.31% 20) The total no. of items sold in Shop D is 400.
c) 21.33% The number of plastic tables sold in D is 20 less
d) 22.22% than the no. of plastic chairs sold in the same
e) 20.33% shop. The price of the wooden table, plastic
table, wooden chair, and plastic chair is Rs. 900,
19) Out of the total tables sold in shop C, 37.5% Rs. 800, Rs. 720, and Rs. 550, respectively. Find
are wooden tables and out of the total chairs sold the total revenue earned by Shop D?
in shop C, 66.66% are wooden chairs. Find the a) 290000
difference in percentage between the total b) 285000
wooden items sold in shop C and total plastic c) 265000
items sold in shop C? d) 245600
a) 50% e) None of these
Click Here to Get the Detailed Video Solution for the above given Questions
Or Scan the QR Code to Get the Detailed Video Solutions

Answer Key with Explanation

Directions (01 - 04): The speed of boat C is 2x*125/100 = 2.5x.


Let the speeds of boats A and B in still water be The speed of boat D is 3x*75/100 = 2.25x.
2x and 3x, respectively. The speed of the stream of river R is
Speed of streams of rivers P and Q in 2y and 3y 2y*75/100=1.5y.
respectively. Now, 2.5x-3y=49/3.5=14

Click Here For Bundle PDF Course | support@guidely.in Page 7 of 12


SBI Clerk & RRB PO Mains PDF Course 2023
Quantitative Aptitude Day – 46 (Eng)

And 2.25x+1.5y=115.5/5.5=21 So, the required time taken by C to go n and


By solving the above equations, we get x = 8 return m is
and y = 2. = 120/[20+4] + 120/[20-4] = 5 + 7.5 = 12.5 hours.
The speeds of boats A and B in still water are 2*
8 = 16 and 3* 8 = 24 km/hr respectively. 4) Answer: A
The speeds of the streams of rivers P and Q are Distance covered by boat B in river Q in
2*2 = 4 and 3*2 = 6 km/hr, respectively. upstream in 4 hours = [24-6]*4 = 72
The speed of boat C is 2.5* 8 = 20 km/hr Distance covered by boat D in river R in
The speed of boat D is 2.25 * 8 = 18 km/hr Downstream in 8 hours = [18+3] *8 = 168
The speed of the stream of river R is 1.5* 2 = 3 So, ratio is = 72:168=3:7
km/hr.
1) Answer: C Directions (05–08):
For I, downstream speed of boat A in the river P Let, pipes M and N take 3x and 2x minutes to
is 16+ 4 = 20 km/hr empty.
So, 20*5.5=110, So, (1/3x) + (1/2x) = 1/12
The upstream speed of boat B in the river Q is Or, 5/6x=1/12
24-6=18 km/hr Or, x=12*5/6=10
So, 18*5.5=99, So, pipes M and N take 3* 10 = 30 and 2* 10 =
So, it's not satisfying. 20 minutes to empty the tank.
Similarly, we can check others' values as well. The time taken by the pipe O to empty the tank
And we see that only III is satisfied. is (20 * 120)/100 = 24 minutes.
Pipe A fill the tank = [1/(1/60)+(1/30)}]
2) Answer: D =[1/{3/60}]=20
So, d = [20+3]*t and e = [t+2]* [18-6] So, Pipe A fill the tank in 20 minutes.
So, d/e = 23t/12*(t+2) = 23/16 Pipe B fills the tank in 20* 3/5 = 12 minutes=t.
Or, 16t=12t+24 Pipe C fill the tank in [1/(3/20)-(1/12)}]
Or, 4t = 24, t = 6. =[1/{(9-5)/60}]=15 minutes.
So, B covers 12* 24 = 288 km in still water. Pipe D takes 15 * 2 = 30 minutes to fill the tank.
Total capacity of tank = LCM
3) Answer: B 20,15,12,30,30,20,24 = 120
Distance from m to n is = [16+4]*3 + [24-4]*3 = So, the efficiency of pipes A,B,C, D, M, N, and O
120 km. is = 6,10, 8, 4 , 4 , 6, and 5 respectively.
5) Answer: C

Click Here For Bundle PDF Course | support@guidely.in Page 8 of 12


SBI Clerk & RRB PO Mains PDF Course 2023
Quantitative Aptitude Day – 46 (Eng)

In 5 minutes, pipe C fill 8* 5 = 40 units. A, B, and C fill in 3 minutes is = [6+10+8] = 24


Rest amount is 120-40=80 units.
So, four pipes are open for = [80/(6+10-6-5)] = After 15 minutes, 24 * 5 = 120 units of tank will
80/5 = 16 minutes. fill
In 16 minutes, Pipe N is empty (16*6)= 96 units So, l=15
So, percentage difference is = [96/120]*100 – So, 2*15+7*68.16=507.12
[40/120]*100 =80%-33.33%=46.67%
Directions (09–12):
6) Answer: D For A,
Pipes A and B are open for = The total writer and painter in city A is = 25*2 =
[(40*120/100)/(6+10)] = 3 minutes 50.
So, for I, pipe C and M open for 21-3=18 The Number writers in city A is = [50-6]/2 = 44/2
minutes. = 22.
In 18 minutes, they fill = [8-4]*18 = 72 units. The Number of painters in city A is = 50-22=28
72+40*120/100=72+48=120 The Number of Singer in city A is = 22+20=42
So, it is true The Number of Dancers in city A is = 37*2-42 =
Similarly, we can check others values as well. 74-42 = 32.
Only I and III are true Similarly, we can calculate the other value as
well.
7) Answer: B
In total efficiency of D and M is = 2*4-4=4
So, they fill the tank in = 120/4=30 minutes = s.
So, in [30+12]/2 = 21 minutes, pipe O empty =
21*5 = 105 unit tank.
So, percentage = [105/120]*100 = 87.5%

8) Answer: A
Four minutes of pipe A, M, B, and O fill = [6- 9) Answer: C

4+10-5] = 7 units. In A, the number of classical singers is =

After 68 minutes, total 7* 17 = 119 units of tank 42*50/100=21

will be filled. In B, the Number of classical singers is = 21*4/3

So, k=68+1/6=68.16 = 28.

Click Here For Bundle PDF Course | support@guidely.in Page 9 of 12


SBI Clerk & RRB PO Mains PDF Course 2023
Quantitative Aptitude Day – 46 (Eng)

The number of classical singers in C is = = [655/(28+32+38)]


40*40/100=16 =6.68
The number of classical singers in D is = 36 * ¾
= 27 12) Answer: A
The number of classical singers in E is = Number of players in C = 32*125/100=40
[28+16]/2 = 22. Number of players in D =26*3/2=39
So, the difference is = [28+27]- [21+22] = 55-43 Number of players in A = 123-39-40=44
= 12. So, required ratio is = 28:44=7:11

10) Answer: D Directions (13–16):


The number of kathak dancers in A, 32* 5/8 = 20 Note – I. M2-M-132 = 0.
The number of Bharatanatyam dancers in A, 32* Or, M2-12M+11M-132=0
3/8 = 12. Or, (M-12)(M+11)=0 M=12,-11, So M=12
The number of kathak dancers in C, 36*5/9 = 20. II. N,34,35,37,41,44
The number of Bharatanatyam dancers in C, Or, 28, 28+[8-2]=34, 34+[4-3]=35, 35+[5-3]=37,
36*4/9=16 37+[7-3]=41, 41+[4-1]=44
The number of kathak dancers in B, 20+ 2 = 22 III. O% of 120 - 8.8 = 20
The number of Bharatanatyam dancers in B, 38- Or, O=28.8*100/120=24
22=16 IV. P,47,71,110,166
The number of Bharatanatyam dancers in D, P = 36, 36+11*1 = 47, 47+12*2 = 71, 71+13*3 =
16+6=22 110, and so on….
The number of kathak dancers in D = 44-22 = 22 S, total number of employees is 50* 24 = 1200.
So, required percentage
= [(20+20+22+22)/(12+16+16+22)]*100
=127.27%

11) Answer: B
The total number of books published by writers
in cities E,D and C is
=[38*50/100]*5+[38*50/100]*8+[32*25/100]*9+[3 13) Answer: C
The number of employees using cars in HR
2*75/100]*7+28*6
department is = [144*75/100] *75/100=81
=95+152+8*9+24*7+168=655
So, average books published by writers

Click Here For Bundle PDF Course | support@guidely.in Page 10 of 12


SBI Clerk & RRB PO Mains PDF Course 2023
Quantitative Aptitude Day – 46 (Eng)

The number of employees of IT department who So, 7x+5x+10x+8x=300


use private transport is =336*3/7=144 Or, 30x=300, x=10
So, required percentage = [81/144]*100 = So, the number of wooden and plastic tables
56.25% = [p+20.25]% sold in A is 7* 10 = 70 and 5*10=50.
So, total round table sold =
14) Answer: D 70*40/100+50*60/100=28+30=58
So, total female is = [288*1/3] + [432*2/9] = 192. Total square table sold = 120-58 = 62
So, total male is = 288+432-192=528 So, difference is 62-58=4
So, average age is =[192*28+528*32]/[288+432]
=22272/720=30.93 18) Answer: D
The total number of items sold in shop B is =
15) Answer: B 300*6/5=360
2M+1=2*12+1=25, N-3=28-3=25 The number of wooden chairs sold in shop B is =
So, total number of employees in the HR and IT 360*27.77/100=100
departments is Let the number of wooden tables and plastic
=[144+336]*125/100=600 chairs sold in shop B be 2x and 3x.
So, the difference = [288+432]-600 = 120 = 5O. The number of plastic tables sold in shop B is
3x/2=1.5x
16) Answer: A So, 3x+1.5x+2x=360-100=260
The total number of fresher employees Or, x=260/6.5=40
=[144+288] *25/100+[336+432]*75/100 So, the number of wooden tables sold in B is 2*
=108+576=684 40 = 80.
The total experienced employees = 1200-684 = So, required percentage = [80/360]*100=22.22%
516
So, ratio is = 684:516 = 57:43 19) Answer: E
Let the ratio of the number of wooden tables and
17) Answer: C plastic chairs sold in shop C be 3x and 2x,
So, let the number of wooden tables and plastic respectively.
tables sold in A be 7x and 5x, respectively. So, the number of wooden chairs sold in C is
The number of wooden chairs sold in A is [2x/33.33] *66.66=4x
7x*100/70=10x The number of plastic tables sold in C is =
The number of plastic chairs sold in A is = [3x/37.5]*62.5 = 5x.
10x*4/5=8x So, 3x+2x+4x+5x=280

Click Here For Bundle PDF Course | support@guidely.in Page 11 of 12


SBI Clerk & RRB PO Mains PDF Course 2023
Quantitative Aptitude Day – 46 (Eng)

So, x=280/14=20 The number of wooden tables sold in D is 150*


So the total number of wooden items sold in 7/15 = 70.
Shop C is = 7*20=140. The number of plastic tables sold in D is = 150-
In shop C, the total number of plastic items sold 70=80
is 7 * 20 = 140. The number of plastic chairs sold in D is =
So, the required percentage is [140/280]*100- 80+20=100
[140/280]*100 = 0. The number of wooden chairs sold in D is = 250-
100=150
20) Answer: A So, total revenue earned by shop D is
The number of tables sold in D is = 400*3/8=150 =900*70+800*80+720*150+550*100=290000
The number of chairs sold in D is = 400*5/8=250

Click Here For Bundle PDF Course | support@guidely.in Page 12 of 12


SBI Clerk & RRB PO Mains PDF Course 2023
ENGLISH Day - 46

English Language

Directions (1-5): Rearrange the following five c) CEBAD


sentences (A), (B), (C), (D) and (E) in the proper d) DBCAE
sequence to form a coherent paragraph and then e) ECDAB
answer the questions given below.
(A) LK-99’s seemingly simple composition and 2) Which of the following is the third sentence
availability of instructions to synthesise it after rearrangement?
prompted scientists outside academia to test the a) A
material as well. b) B
(B) But the South Koreans and the independent c) C
scientists who worked to verify the claim d) D
published their findings as preprint papers that e) E
were free to read.
(C) The scientific community is now confident 3) Pick the erroneous part in sentence C as your
that the/A material known as LK-99 is not a answer.
room-temperature and ambient-pressure/B a) A
superconductor, bringing to a swift close an b) B
exciting/C episode launching by a group of South c) C
Korean researchers/D. No error /E d) D
(D) The pace of developments was exhilarating, e) E
but there was soon hype and misinformation but
some reports indicated that the South Korean 4) Fill in the given blank in sentence D with an
group had _________ manuscripts explaining appropriate word.
their claim to a journal. a) submitted
(E) There has been no formal conclusion to b) deleted
match the formal announcement that this c) informed
material could transport an electric current with d) announced
no resistance in ambient conditions. e) denoted
1) Which of the following is the correct sequence
of arrangement that forms a meaningful coherent 5) Which of the following is the SYNONYM of the
passage? word AMBIENT as mentioned in the sentence E?
a) BECDA a) remote
b) ABEDC b) far

Click Here For Bundle PDF Course | support@guidely.in Page 1 of 11


SBI Clerk & RRB PO Mains PDF Course 2023
ENGLISH Day - 46

c) detached b) estimate, high


d) isolated c) price, moderate
e) surrounding d) outlay, concessional
e) plan, compromised
Directions (6-10): Each of the following questions
has two blanks, each blank indicating that a 9) The _____ NSO data, showing retail inflation
word/phrase has been omitted. Choose the set accelerating to a 15-month high, comes less than
of words/phrases that best fits the given blanks a week after the Reserve Bank of India (RBI) left
according to the context of the sentence. interest rates ________even as it warned of “a
6) In the _______ for equal rights for all, the substantial increase in headline inflation” in the
Supreme Court of India has taken an important near term.
step by releasing guidelines to take on _______ a) old, undisturbed
gender stereotypes that perpetuate inequalities. b) recent, altered
a) search, powerful c) new, changed
b) quest, harmful d) updated, same
c) denial, harmless e) latest, unchanged
d) terms, beneficial
e) context, injurious 10) The ______ Char Dham road building project
has led to large-scale altering of the mountains
7) When there is no sunlight, a small solar panel with significant chunks carved away, rendering
which is on the Rover will _____ power to charge them ________to upheaval.
the battery for the next 14 days ____light comes. a) planned, resilient
a) generate, until b) completed, liable
b) form, by c) ongoing, vulnerable
c) consume, till d) huge, accessible
d) produce, as e) approved, stable
e) use, unless
Directions (11-15): In the following questions
8) Announced by Prime Minister Narendra Modi three columns are given containing three
during his Independence Day speech, the Sentences/phrases each. In the first column,
scheme, with an _____ of ₹13,000 crore, sentences/phrases are A, B and C, in the second
provides loans of up to a total of ₹3 lakh at a column the sentences/phrases are D, E and F
_________ interest rate of 5%. and in the third column the sentences/phrases
a) investment, low are G,H and I are given. A sentence/phrase from

Click Here For Bundle PDF Course | support@guidely.in Page 2 of 11


SBI Clerk & RRB PO Mains PDF Course 2023
ENGLISH Day - 46

one column may or may not connect with


even their
another sentence/phrase from the other two
intentions for
columns to make a grammatically and
travelling.
contextually correct sentence. Each question has
five options, four of which display the C. Today, a F. passports I. ensure

sequence(s) in which the sentences/phrases can passport are more than safe

be joined to form a grammatically and typically mere passage

contextually correct sentence. If none of the contains documents; through their

options given forms a correct sentence after essential they represent territories.

combination, mark option (e), i.e. “None of these” information the gateway to

as your answer. about its global

11) holder, such exploration and


as their the
COLUMN 1 COLUMN 2 COLUMN 3
embodiment of
A. Passports D. passports G. and a) A-D-H
play a crucial facilitate watermarks, b) B-D-G
role in diplomatic to prevent c) C-F-G
maintaining relationships counterfeitin d) B-E-I
national and g and e) None of these
security and international unauthorized
regulating cooperation by use. 12)
international enabling
COLUMN 1 COLUMN 2 COLUMN 3
individuals to
attend A. Through D. can G. various
conferences, user-friendly conveniently administrative
conduct websites, access activities
business, and mobile apps, information, without the
and digital pay taxes, need for
B. The history E. when rulers H. evidence
communicatio apply for physical
of passports and officials of their
n channels, permits, and presence.
can be traced provided identity,
citizens engage in
back to individuals with citizenship,
ancient times, letters of safe and B. E. that H. the
conduct to sometimes Furthermore, leverages relationship

Click Here For Bundle PDF Course | support@guidely.in Page 3 of 11


SBI Clerk & RRB PO Mains PDF Course 2023
ENGLISH Day - 46

e-governance technology to between both natural encourage contributing to


promotes revolutionize governments wonders and travellers to the well-being
transparency the way and citizens man-made appreciate and of both the
and governments remains a respect the destination
accountability interact with compelling natural world and its
by making citizens, deliver force for and local inhabitants.
government public services, positive cultures,
actions and and change.
B. Social E. or other H. culinary
C. At its core, F. automating I. concerns media also purposes, and tours, and
e-governance routine tasks, must be plays a it encompasses immersive
encompasses sharing data addressed significant a wide range of cultural
the use of across rigorously to role, as experiences, events to
various agencies, and safeguard travellers from exploring captivate
technological utilizing data sensitive share their natural travellers and
tools and analytics, information experiences leave them
platforms to governments and prevent and inspire with lasting
can make more potential memories.
informed breaches.
C. In recent F. seek out I. to ensuring
decisions,
years, breathtaking that tourism
a) A-D-G ecotourism landscapes, remains a
b) B-F-I and cultural historical positive force
c) C-E-H tourism have landmarks, for both
d) C-D-H gained architectural travellers and
e) None of these prominence marvels, and the
as vibrant destinations
13) alternatives they visit.
COLUMN 1 COLUMN 2 COLUMN 3 that prioritize
environmental
A. The allure D. and cultural G. fostering a
a) A-F-I
of tourism lies preservation, sense of
in the these forms of responsibility b) C-D-G
c) B-E-H
exploration of tourism and
d) C-F-H

Click Here For Bundle PDF Course | support@guidely.in Page 4 of 11


SBI Clerk & RRB PO Mains PDF Course 2023
ENGLISH Day - 46

e) None of these
urban reliable, people to
development efficient, and traverse
14)
patterns and environmentally densely
COLUMN 1 COLUMN 2 COLUMN 3 friendly mode populated
of areas.
A. Metro rails, D. growth by G. pivotal role
also known connecting in shaping the a) A-E-I

as subway various parts of future of b) A-F-H

systems, a city and urban mobility c) B-F-G

undergrounds facilitating and d) C-D-I

, or rapid access to sustainability. e) None of these

transit employment
systems, centers, 15)

represent a educational COLUMN 1 COLUMN 2 COLUMN 3


institutions, and
A. Efforts to D. from G. especially
B. The E. exploring H. reduced address child children when certain
development options for travel times, labour have working in groups are
of metro rail renewable and a more gained hazardous marginalized
systems energy efficient use momentum industries such or when child
emerged as a sources, of limited globally, with as mining, labour is
response to regenerative urban space. agriculture, and culturally
the braking manufacturing, accepted.
challenges systems, and to
posed by energy-efficient
B. While E. are engaged H. to create
rapid lighting
progress has in work that is environments
urbanization,
been made in often where
population
many regions, hazardous, children can
growth, and
child labor exploitative, thrive without
C. Metro rails F. provide a I. a fast and remains a and detrimental being
also have the crucial lifeline convenient deeply to their well- subjected to
potential to for modern way for entrenched being and exploitation.
reshape cities, offering a millions of challenge

Click Here For Bundle PDF Course | support@guidely.in Page 5 of 11


SBI Clerk & RRB PO Mains PDF Course 2023
ENGLISH Day - 46

a) to address the problem of a shortage in urban


C. The F. poverty is a I. income to
housing
reasons significant supplement
b) for addressing the problem of a shortage in
behind child driver, as household
urban housing
labour are families earnings
c) for address the problem of the shortage in
complex and struggling to along with
urban housing
multifaceted make ends this lack of
d) by addressing the problem of the shortage in
but meet may rely access to
urban housing
on their quality
e) No changes required
children's education and
social
17) The Union government’s proposal to have a
services also
three-member selection panel with a majority for
contribute.
the executive for the appointment of members of
a) A-D-G
the Election Commission may not subserve the
b) B-D-I
objective of protecting the poll watchdog’s
c) C-F-I
independence.
d) C-E-H
a) by a majority of the executive for the
e) None of these
appointment of members
b) with the majority of the executive for the
Directions (16-20): In each of the questions given
appointment of members
below a phrase in the sentence has been
c) from a majority of the executive for the
highlighted. It may or may not need replacement.
appointment of members
Read the question carefully and choose an
d) through a majority of the executive for the
option that would best replace the highlighted
appointment of members
parts to make the sentences grammatically and
e) No changes required
contextually correct. In case, the given sentence
is correct in its current form and there is no
18) The Opposition’s stated objective of moving
improvement needed, mark (e) as your answer.
a no-confidence motion against Prime Minister
16) The announcement by Prime Minister
Narendra Modi’s government was him to force to
Narendra Modi of a scheme to help the urban
speak on the ongoing ethnic violence in Manipur.
poor build houses in cities is a signal that the
a) was to speak him to force on the ongoing
government is to undertake one more plan to
ethnic violence
addressing the problem of the shortage in urban
b) was force on him to speak him to speak on the
housing.
ongoing ethnic violence

Click Here For Bundle PDF Course | support@guidely.in Page 6 of 11


SBI Clerk & RRB PO Mains PDF Course 2023
ENGLISH Day - 46

c) was forced to speak on the ongoing ethnic e) No changes required


violence
d) was to force him to speak on the ongoing 20) The Jeddah conference on the Ukraine war,
ethnic violence in which 42 nations participated, was one of the
e) No changes required most serious efforts from the world though
finding a way to end the conflict.
19) Ecuador, a relatively peaceful country of 18 a) were one serious effort by the world in
million people until 2017-18, now is only the most b) was one of the most serious efforts by the
dangerous countries in Latin America. world in
a) is now one of the mostly dangerous countries c) was the only serious efforts by the world to
in d) were one of the most serious efforts by the
b) is now one of the most dangerous countries at world to
c) is now one of the most dangerous countries in e) No changes required
d) now is the mostly dangerous countries in
Click Here to Get the Detailed Video Solution for the above given Questions
Or Scan the QR Code to Get the Detailed Video Solutions

Answer Key with Explanation

1) Answer: C sentence E - it is said that there was no formal


The correct sequence of rearrangement is conclusion made so sentence B seems to be a
CEBAD. justification which says that the scientists have
The first sentence would be C which has the published papers which are free to read. Fourth
introduction part of the passage - LK 99 sentence would be A as D seems to be the
superconductor. Next sentence would be E as it concluding sentence of the passage. So, D is
is a continuation of C talking about the followed by A in the correct sequence.
conducting nature of the superconductor. In 2) Answer: B

Click Here For Bundle PDF Course | support@guidely.in Page 7 of 11


SBI Clerk & RRB PO Mains PDF Course 2023
ENGLISH Day - 46

B is the third sentence after rearrangement. Deleted, informed, announced and denoted are
The correct sequence of rearrangement is found inappropriate and hence eliminated.
CEBAD. The sentence: The pace of developments was
The first sentence would be C which has the exhilarating, but there was soon hype and
introduction part of the passage - LK 99 misinformation but some reports indicated that
superconductor. Next sentence would be E as it the South Korean group had submitted
is a continuation of C talking about the manuscripts explaining their claim to a journal.
conducting nature of the superconductor. In
sentence E - it is said that there was no formal 5) Answer: E
conclusion made so sentence B seems to be a Meaning of the word ambient is ‘of the area
justification which says that the scientists have around; on all sides’
published papers which are free to read. Fourth Out of the given words a word with similar
sentence would be A as D seems to be the meaning is ‘surrounding’, all the other words are
concluding sentence of the passage. So, D is opposite in meaning to the given word and
followed by A in the correct sequence. hence are discarded.

3) Answer: D 6) Answer: B
The erroneous part in sentence C is part D. The correct pair of words is quest and harmful.
Launching is incorrect; instead it must be Quest - a long search for something that is
‘launched’ as the action is in the past. difficult to find
The sentence: The scientific community is now Option a (search, powerful) - search would fit in
confident that the material known as LK-99 is not the first blank but powerful isn’t appropriate
a room-temperature and ambient-pressure Option c (denial, harmless) - both the words do
superconductor, bringing to a swift closes an not fit in the given blanks as they are totally
exciting episode launched by a group of South inappropriate to the context of the sentence
Korean researchers. Option d (terms, beneficial) - inappropriate words
Option e (context, injurious) - inappropriate
4) Answer: A words
Submitted is the right word that fits the given The sentence: In the quest for equal rights for
blank in a meaningful and correct way. all, the Supreme Court of India has taken an
Out of the given words ‘submitted’ is the most important step by releasing guidelines to take on
appropriate word that can fill the given blank and harmful gender stereotypes that perpetuate
make the sentence complete and meaningful. inequalities.

Click Here For Bundle PDF Course | support@guidely.in Page 8 of 11


SBI Clerk & RRB PO Mains PDF Course 2023
ENGLISH Day - 46

Out of the given options, option e is the correct


7) Answer: A one containing the right set of words that fill the
Generate and until are the correct words that fill blanks in the best way possible.
in the given two blanks in the sentence. First blank - old, new, updated are less relevant
Solar panels generate power (not consume, or less appropriate when compared to recent
form, use - hence discard options b, c and e) and latest in accordance with the context of the
Out of options a and d, ‘a’ is the appropriate sentence (recent and latest are better words to
option as it has got the right word to fill the fill the first blank)
second blank fo the sentence (‘until light comes’ Second blank - ‘left the rate unchanged’ is the
is correct whereas ‘as light comes’ in incorrect correct phrase (left something means ‘to leave it
according to the grammar rule) the same way it is)
The sentence: When there is no sunlight, a small The sentence: The latest NSO data, showing
solar panel which is on the Rover will generate retail inflation accelerating to a 15-month high,
power to charge the battery for the next 14 days comes less than a week after the Reserve Bank
until light comes. of India (RBI) left interest rates unchanged even
as it warned of “a substantial increase in
8) Answer: D headline inflation” in the near term.
Out of the given five options, option d has the
right pair of words that fit the given blanks in the 10) Answer: C
most appropriate way possible. Ongoing and vulnerable are the correct words
Outlay - money that is spent, especially in order that fit in the given blanks of the sentence.
to start a business or project Vulnerable - The ongoing Char Dham road
Concessional rate - a special price building project has led to large-scale altering of
Concessional is the only word that fits the the mountains with significant chunks carved
second blank correctly (hence the other options away, rendering them vulnerable to upheaval.
are discarded) Out of the given options, option c has the right
The sentence: Announced by Prime Minister pair of words that fit in the context of the
Narendra Modi during his Independence Day sentence making it meaningful and correct.
speech, the scheme, with an outlay of ₹13,000 The sentence: The ongoing Char Dham road
crore, provides loans of up to a total of ₹3 lakh at building project has led to large-scale altering of
a concessional interest rate of 5%. the mountains with significant chunks carved
away, rendering them vulnerable to upheaval.
9) Answer: E

Click Here For Bundle PDF Course | support@guidely.in Page 9 of 11


SBI Clerk & RRB PO Mains PDF Course 2023
ENGLISH Day - 46

11) Answer: D environmental and cultural preservation, these


B-E-I is the correct combination which forms a forms of tourism encourage travellers to
meaningful sentence. appreciate and respect the natural world and
The other options have a mix of fragments which local cultures, fostering a sense of responsibility
do not combine to form a complete meaningful and contributing to the well-being of both the
sentence - hence they are discarded. destination and its inhabitants.
The sentence formed: The history of passports
can be traced back to ancient times, when rulers 14) Answer: E
and officials provided individuals with letters of Analyse the options and the
safe conduct to ensure safe passage through statements/fragments given, none of them
their territories. combine to form a complete meaningful
sentence.
12) Answer: A Hence all the options are eliminated and option e
The fragments A, D and G combine to form a - none of these is the right one.
complete meaningful sentence and hence option
a is the right answer for this question. 15) Answer: C
Even though the rest of the options revolve C-F-I is the right combination of fragments that
around the same topic e-governance, the form a meaningful sentence.
statements do not club together to form a correct The other options are discarded because the
and meaningful sentence. sentences thus formed are incomplete and
The sentence formed: Through user-friendly incorrect.
websites, mobile apps, and digital The sentence formed: The reasons behind child
communication channels, citizens can labour are complex and multifaceted but poverty
conveniently access information, pay taxes, is a significant driver, as families struggling to
apply for permits, and engage in various make ends meet may rely on their children's
administrative activities without the need for income to supplement household earnings along
physical presence. with this lack of access to quality education and
social services also contribute.
13) Answer: B
C-D-G is the right combination. 16) Answer: A
The sentence formed: In recent years, In the given sentence ‘to addressing’ is incorrect
ecotourism and cultural tourism have gained and hence the highlighted phrase needs
prominence as alternatives that prioritize improvement/replacement.

Click Here For Bundle PDF Course | support@guidely.in Page 10 of 11


SBI Clerk & RRB PO Mains PDF Course 2023
ENGLISH Day - 46

The correct replacement phrase/fragment is force him to speak on the ongoing ethnic
option a. violence in Manipur.
To address is correct, ‘for addressing, for
address, by address’ are incorrect and hence the 19) Answer: C
respective options are discarded. In the given sentence - ‘now is only’ is incorrect
The sentence: The announcement by Prime and meaningless hence needs replacement.
Minister Narendra Modi of a scheme to help the The correct replacement phrase is option c.
urban poor build houses in cities is a signal that Ecuador was once a peaceful country but it is
the government is to undertake one more plan to now one of the most dangerous countries.
address the problem of a shortage in urban Option a - mostly in incorrect
housing. Option b - at is incorrect
Option d - ‘now is the mostly’ is incorrect
17) Answer: E The sentence: Ecuador, a relatively peaceful
The given sentence is correct and meaningful. country of 18 million people until 2017-18, is now
The highlighted phrase needs no changes and one of the most dangerous countries in Latin
hence option e is the right answer for this America.
question.
20) Answer: B
18) Answer: D Option b is the correct option as it is the correct
The highlighted phrase is wrong/incorrect and phrase to replace the incorrect part in the given
hence needs replacement. sentence making the sentence correct and
‘Was him to force’ is incorrect instead it must be meaningful.
‘was to force him’ (the word must be shuffled in Jeddah conference (singular) - was is the
order to make it correct and meaningful) relevant word (discard options a and d)
Option d has the right phrase for replacement. Out of b and c - phrase/fragment b is
In the other options the words are misplaced grammatically correct and meaningful
causing grammatical errors and hence are The sentence: The Jeddah conference on the
discarded. Ukraine war, in which 42 nations participated,
The sentence: The Opposition’s stated objective was one of the most serious efforts by the world
of moving a no-confidence motion against Prime in finding a way to end the conflict.
Minister Narendra Modi’s government was to

Click Here For Bundle PDF Course | support@guidely.in Page 11 of 11


SBI Clerk & RRB PO Mains PDF Course 2023
Reasoning Ability Day – 47 (Eng)

Reasoning Ability
Directions (1-5): Study the following information d) Four
carefully and answer the given questions. e) One
Nine boxes viz., I, J, K, L, M, N, O, P, and Q are
placed one above another on ten shelves from 3. Four of the following five are alike in a certain
top to bottom but not necessarily in the same way based on the given arrangement and thus
order. One of the shelves is vacant. form a group. Which one of the following does
Not more than two shelves are between the not belong to the group?
boxes Q and N. Box J is placed four shelves a) OQ
above the box N and only two boxes are placed b) KL
between them. No box is placed between the c) JN
boxes Q and J. The number of shelves above the d) IL
vacant shelf is three less than the number of e) NJ
shelves below the box O. Only two boxes are
placed between the boxes O and K. At least 4. If all the boxes are arranged in the reverse
three boxes are placed between the boxes K and alphabetical order from bottom to top except the
L. Box P and Box L are placed adjacent to each vacant shelf, then which among the following
other. The number of boxes placed above the box(s) remains unchanged in their position?
box P is one less than the number of boxes I. Box K
placed below the box M. More than one box is II. Box P
placed between the box M and the box I. III. Box N
1. What is the position of the vacant shelf as per a) Only II
the final arrangement? b) Both I and III
a) Fourth from the bottom c) Only III
b) Third from the top d) Both II and III
c) Sixth from the bottom e) None of these
d) Sixth from the top
e) Fifth from the top 5. If the box R is placed on the vacant shelf, then
what is the position of J with respect to R?
2. How many boxes are placed between the a) Two boxes above
boxes K and P? b) Immediately below
a) As many boxes between M and K c) Three boxes above
b) Three d) Three boxes below
c) As many boxes between N and L e) Immediately above

Click Here For Bundle PDF Course | support@guidely.in Page 1 of 9


SBI Clerk & RRB PO Mains PDF Course 2023
Reasoning Ability Day – 47 (Eng)

Directions (6-10): Study the following information persons sit between H and F as between I and
carefully and answer the given questions. the one who works in Indian bank. The one who
Ten people viz.- A to J sit in two parallel rows works in UCO bank does not face north direction.
such that five persons sit in each row. They are I doesn’t work in UCO bank.
working in two different categories of banks viz.- 6. Who among the following person sits opposite
Public sector banks - SBI, Indian Overseas Bank to the one who sits second to the left of the one
(IOB), UCO bank, Indian Bank, Canara Bank, who works in UCO bank?
and private sector banks - HDFC, ICICI, Yes a) C
Bank, Axis Bank, and RBL Bank. The persons in b) The one who works in Axis bank
one row face towards the north and the persons c) E
in another row face towards the south such that d) The one who works in Canara bank
persons of both rows face each other. e) B
Note: The people who work in public sector
banks are not sitting opposite to each other and 7. Who among the following person works in
the people who work in private sector banks are RBL bank?
also not sitting opposite to each other. a) The one who sits immediate left of B
C and the one who works in Yes bank are sitting b) A
diagonally opposite ends to each other. J sits c) The one who sits opposite to J
second to the left of the one who works in Yes d) I
bank. One person sits between the one who sits e) The one who sits opposite to H
opposite to J and the one who works in HDFC
bank. The one who works in Axis bank sits 8. Which of the following combination is true as
opposite to the one who sits third to the right of per the final arrangement?
G. G sits opposite to D. The one who works in I. F – SBI
Canara bank sits second to the right of the one II. B – RBL bank
who works in ICICI bank. Two persons sit III. I – Canara bank
between B and the one who sits opposite to the a) Only I
one who works in SBI. The one who works in SBI b) Both II and III
is not sitting opposite to the one who works in c) Only III
Yes bank. The number of persons sitting d) Both I and II
between D and A is two more than the number of e) Both I and III
persons sitting between B and the one who
works in RBL bank. H and the one who works in 9. What is the position of G with respect to the
IOB bank are immediate neighbors. As many one who works in RBL bank?

Click Here For Bundle PDF Course | support@guidely.in Page 2 of 9


SBI Clerk & RRB PO Mains PDF Course 2023
Reasoning Ability Day – 47 (Eng)

a) Immediate left I. PROJECT


b) Second to the right II. WORKING
c) Third to the left III. SERVICE
d) Second to the left a) Both I and II
e) Immediate right b) Only II
c) Both II and III
10. As many persons sit between the one who d) Only I
works in Axis bank and ___ as between ___ and e) All I, II, and III
the one who works in ICICI bank?
a) J and I 13. In the given numbers, all the odd digits are
b) F and C added and then the second digit from the left is
c) H and G subtracted from the resultant within the number,
d) D and E then which of the following number’s resultant is
e) H and I more than nine?
I. 18963254
11. Which of the following has the maximum pair II. 56384621
of digits in the given numbers (both forward and III. 79638521
backward) which has as many digits between a) Only II
them as there are in the number series (The b) Both I and II
number series does not continue after ‘9’ and ‘0’ c) Both II and III
again)? d) Only III
I. 5742942 e) Both I and III
II. 4895495
III. 94847213 14. If in the word “DATASTRUCTURE” each
a) Only II consonant is changed to the previous letter and
b) Both II and III each vowel is changed to the next letter as per
c) Only III the English alphabetical series and then the first
d) Both I and II six letters are arranged in reverse alphabetical
e) Only I order from left to right and last six letters are
arranged in alphabetical order from left to right.
12. In the following word, letters are arranged in Then how many such pairs are there in the new
alphabetical order starting from the left end, then word thus formed, in which two consecutive
which of the following words the position of all letters are same?
the letters changed? a) Two

Click Here For Bundle PDF Course | support@guidely.in Page 3 of 9


SBI Clerk & RRB PO Mains PDF Course 2023
Reasoning Ability Day – 47 (Eng)

b) One Note: The one who got the first rank scored the
c) Four highest marks while the one who got the fifth
d) Three rank scored the lowest marks
e) None Five persons viz., A, B, C, D, and E are studying
for the competitive exams. So they are taking a
15. In the following numbers, the digits at an odd mock test for upcoming exams and scored
position from the right end are added and the different marks in four mock tests. No two
resultant will be subtracted from the sum of persons scored the same mark in the same mock
numbers which are at the even position from the test and none of them got the same position in
right end, then how many numbers attain a value more than one mock test.
which is a multiple of 2? Test 1: D % A @; D $ E &; C * D
I. 25896375 Test 2: D $ C * B; E ! B; A % C
II. 46385723 Test 3: C % A; B * D; E $ A # D
III. 68957612 Test 4: B ! E; A $ D # C @
a) Only III 16. If the person D scored 78 marks and C
b) Only I scored 42 marks, then what is the possible mark
c) Both II and III scored by the person E in test 3?
d) Both I and III I. 52
e) All I, II, and III II. 40
III. 69
Directions (16-20): Study the following a) Only II
information carefully and answer the given b) Only III
questions. c) Both I and II
X * Y means X scored more marks than Y d) Both II and III
X # Y means X scored less marks than Y e) Both I and III
X % Y means X does not score more marks than
Y 17. In test 4, who among the following person
X $ Y means X does not score less marks than Y scored the third lowest mark?
X @ means X does not score the highest mark a) D
X & means X does not score the lowest mark b) E
X ! Y means one person scored a mark between c) B
X and Y, who scored less mark than X d) A
e) C

Click Here For Bundle PDF Course | support@guidely.in Page 4 of 9


SBI Clerk & RRB PO Mains PDF Course 2023
Reasoning Ability Day – 47 (Eng)

18. Who among the following person does not c) Only I


score the lowest mark in the entire test? d) Both I and III
a) D e) All I, II, and III
b) A
c) C 20. If D is related to test 1 and B is related to test
d) E 3 in a certain way, then who among the following
e) B person is related to test 4?
I. C
19. In which of the following test(s) the person A II. E
scored more marks than B? III. B
I. In test 2 a) Only II
II. In test 3 b) Both I and III
III. In test 4 c) Only III
a) Both II and III d) Both II and III
b) Only III e) All I, II, and III

Click Here to Get the Detailed Video Solution for the above given Questions
Or Scan the QR Code to Get the Detailed Video Solutions

Answer Key with Explanation

Directions (1-5): 4. Answer: D


1. Answer: D 5. Answer: C
2. Answer: C Final arrangement:
3. Answer: E
(First box is placed above the second box except
option e)

Click Here For Bundle PDF Course | support@guidely.in Page 5 of 9


SBI Clerk & RRB PO Mains PDF Course 2023
Reasoning Ability Day - 47 (Eng)

 The number of shelves above the vacant


shelf is three less than the number of
shelves below the box O.
 Only two boxes are placed between the
boxes O and K.
 At least three boxes are placed between
the boxes K and L.
 Box P and Box L are placed adjacent to
each other.
After applying the above conditions, case 2 gets
eliminated because only two boxes are placed
between O and K.
We have,
 Not more than two shelves are between
the boxes Q and N.
 Box J is placed four shelves above the
box N and only two boxes are placed
between them.
 No box is placed between the boxes Q
and J.
After applying the above conditions, there are
three possibilities. Again, we have
 The number of boxes placed above the
box P is one less than the number of
boxes placed below the box M.
 More than one box is placed between the
box M and the box I.

Again, we have

Click Here For Bundle PDF Course | support@guidely.in Page 6 of 9


SBI Clerk & RRB PO Mains PDF Course 2023
Reasoning Ability Day - 47 (Eng)

 C and the one who works in Yes bank are


sitting diagonally opposite ends to each
other.
 J sits second to the left of the one who
works in Yes bank.
 One person sits between the one who sits
opposite to J and the one who works in
HDFC bank.
After applying the above conditions, there are
two possibilities.

After applying the above conditions, case 1 and


case 1a get eliminated because more than one
box is placed between the box M and the box I,
hence case 3 shows the final arrangement.

Directions (6-10):
6. Answer: D
7. Answer: C
8. Answer: E
9. Answer: D
10. Answer: E
Again, we have
Final arrangement:
 The one who works in Axis bank sits
opposite to the one who sits third to the
right of G.
 G sits opposite to D.
 The one who works in Canara bank sits
second to the right of the one who works
in ICICI bank.
We have,

Click Here For Bundle PDF Course | support@guidely.in Page 7 of 9


SBI Clerk & RRB PO Mains PDF Course 2023
Reasoning Ability Day - 47 (Eng)

 Two persons sit between B and the one After applying the above conditions, case 1 gets
who sits opposite to the one who works in eliminated because the one who works in UCO
SBI. bank does not face the north direction, hence
 The one who works in SBI is not sitting case 2 shows the final arrangement.
opposite to the one who works in Yes
bank.

Again, we have
 The number of persons sitting between D 11. Answer: C

and A is two more than the number of


persons sitting between B and the one
who works in RBL bank.
 H and the one who works in IOB bank are
immediate neighbors. 12. Answer: B
 As many persons sit between H and F as I. PROJECT - CEJOPRT
between I and the one who works in II. WORKING - GIKNORW
Indian bank. III. SERVICE – CEEIRSV
 The one who works in UCO bank does
not face north direction. 13. Answer: E
 I doesn’t work in UCO bank. I. 18963254 -> 1+9+3+5 = 18 – 8 = 10
II. 56384621 -> 5+3+1 = 9 – 6 = 3

Click Here For Bundle PDF Course | support@guidely.in Page 8 of 9


SBI Clerk & RRB PO Mains PDF Course 2023
Reasoning Ability Day - 47 (Eng)

III. 79638521 -> 7+9+3+5+1 = 25 – 9 = 16 Test 1: D % A @; D $ E &; C * D


> A > D, D > E >, C > D
14. Answer: D C>A>D>E>B
DATASTRUCTURE Test 2: D $ C * B; E ! B; A % C
CBSBRSQVBSVQF D > C > B, E > > B, C > A
SSRCBBQBFQSVV Case 1: D > C > E > A > B
15. Answer: C Case 2: D > E > C > B > A
I. 25896375 Case 1 gets eliminated because in both tests 1
Odd position -> 5+3+9+5 = 22 and 2, B gets the lowest mark, hence case 2
Even position -> 7+6+8+2 = 23 shows the final arrangement.
23 -22 = 1 D>E>C>B>A
II. 46385723 Test 3: C % A; B * D; E $ A # D
Odd position -> 3+7+8+6 = 24 A > C, B > D, D/E > A
Even position -> 2+5+3+4 = 14 B > D/E > D/E > A > C
24 – 14 = 10 B > D > E > A > C (Because E does not score
III. 68957612 second highest mark)
Odd position -> 2+6+5+8 = 21 Test 4: B ! E; A $ D # C @
Even position -> 1+7+9+6 = 23 B > > E, A > D, > C > D
23 – 21 = 2 By comparing other tests positions,
A>C>B>D>E
Directions (16-20):
16. Answer: E
17. Answer: C
18. Answer: A
19. Answer: B
20. Answer: D (Odd positions)

Click Here For Bundle PDF Course | support@guidely.in Page 9 of 9


SBI Clerk & RRB PO Mains PDF Course 2023
Quantitative Aptitude Day – 47 (Eng)

Quantitative Aptitude

Directions (01 - 05): Study the following information carefully and answer the questions given below.
The given pie chart shows the degree distribution of the number of people who take vaccination in five
wards of the city and the bar graphs show the number of people who take 1st dose and both 1st and 2nd
dose. The total number of people who take the vaccination is equal to the number of people who take 1 st
dose and the number of people who take both `the 1st and 2nd doses together.

Click Here For Bundle PDF Course | support@guidely.in Page 1 of 10


SBI Clerk & RRB PO Mains PDF Course 2023
Quantitative Aptitude Day - 47 (Eng)

Note: village A formed a right angle. The total number of people vaccinated in ward C is 6000.
1) The total number of people who take 1st dose 2) Find the ratio between the number of people
from ward F is 20% more than the number of who take 1st dose from wards C and D together
people who take 1st dose of vaccine from ward E and the total number of people take both 1st and
and a total of 12500 people take the vaccination 2nd doses from wards D and E together?
from ward F. Find the sum of the number of a) 2:3
people who take both 1st and 2nd doses of b) 1:2
vaccination from wards A and F together? c) 2:9
a) 11500 d) 7:5
b) 12500 e) 5:7
c) 12900
d) 16500 3) Out of the total number of people who take
e) 14500 both doses of vaccine from ward C, 2*[a+b]% of
people also take booster doses. Find the number

Click Here For Bundle PDF Course | support@guidely.in Page 2 of 10


SBI Clerk & RRB PO Mains PDF Course 2023
Quantitative Aptitude Day - 47 (Eng)

of people who take booster doses of the c) 6542


vaccine? d) 6555
a) 540 e) None of these
b) 640
c) 520 5) Find the percentage of the number of people
d) 580 who take both 1st dose and 2nd dose of vaccine
e) None of these in all five wards of the city?
a) 45.83%
4) Find the value of 2z+3m-2n+3200-a% of b) 49.36%
300=? c) 66.35%
a) 6185 d) 98.32%
b) 6250 e) None of these

Directions (06 - 10): Study the following information carefully and answer the questions given below.
The given table shows the total number of laptops [i3+i5+i7] sold by five companies and the number of i3
generation laptops sold, and some statements about the number of i5 and i7 generation laptops sold by
five companies.

Click Here For Bundle PDF Course | support@guidely.in Page 3 of 10


SBI Clerk & RRB PO Mains PDF Course 2023
Quantitative Aptitude Day - 47 (Eng)

6) Find the difference between the number of i7 9) Find the ratio between the number of i5
generation laptops sold by companies C and D generation laptops sold by companies B and C
together and the number of i7 generation laptops together and by companies D and E together?
sold by companies A and E together? a) 7:15
a) 1100 b) 2:9
b) 1600 c) 7:11
c) 1200 d) 4:9
d) 1050 e) None of these
e) 1300
10) M= Difference between the number of i3 and
7) The ratio of the number laptops sold with the i5 generation laptop sold by company B.N=
graphics card and without graphics cards of i3, Difference between the number of i5 and i7
i5, and i7 generation by company B is 3:1,2:3 generation laptops sold by company C.Find the
and 5:3. Find the difference between the number relation between M and N?
of laptops sold by company B with the graphics a) 3M+2N=4800
card and without a graphics card? b) M+N=1500
a) 420 c) 2N+M=2400
b) 440 d) 4M+5N=3200
c) 460 e) None of these
d) 490
e) 340 Directions (11 - 15): A and B are missing the
number of two series. Find the relation between
8) The number of i3 generation laptops sold by A and B?
company F is 25% more than the number of i3 11)
generation laptops sold by company E. If the SERIES I – 15,32,66,134, A
total number of laptops sold by company F is SERIES II -44,84,124,164, B
6500 then find the total number of i5 and i7 a) A+B=478
generation laptops sold by companies A and F b) 3A-2B=21
together? c) A-B=66
a) 8250 d) A+4B=628
b) 8825 e) A+2B=567
c) 5620
d) 8752 12)
e) None of these SERIES I –6840,4896,3360,A,1320

Click Here For Bundle PDF Course | support@guidely.in Page 4 of 10


SBI Clerk & RRB PO Mains PDF Course 2023
Quantitative Aptitude Day - 47 (Eng)

SERIES II -165,195,225,255, B 16) There are two bags. Bag A contains 4 red, 5
a) A-B=2135 blue, and 6 black shirts. Bag B contains 9 shirts
b) A+2B=2685 out of that 3 red shirts. The ratio of blue and
c) A-5B=1245 black shirts is 2:1.
d) A+B=2469 Quantity I: Find the probability of getting 2 blue
e) A-7B=542 shirts from bag B when 2 shirts are randomly
picked up from bag B?
13) Quantity II: Find the sum of the probability of
SERIES I –132,156,182,210, A getting one red shirt from each bag when one
SERIES II -3,7,25,121, B shirt is randomly picked up from each bag?
a) A+2B=1256 Quantity III: Find the sum of the probability of
b) A+B=961 getting one black shirt from bag A and one blue
c) 3A+3B=2546 shirt from bag B when one shirt is picked up
d) 2A+3B=1864 randomly from each bag?
e) None of these a) Quantity I < Quantity II< Quantity III
b) Quantity I < Quantity II >Quantity III
14) c) Quantity I < Quantity II <Quantity III
SERIES I –156,300,496,752, A d) Quantity I < Quantity II =Quantity III
SERIES II -253,299,345,391, B e) Quantity I > Quantity II <Quantity III
a) A-B=256
b) A-2B=356 17) The ratio of efficiency of B and C is 5:3. A
c) A-3B=125 can complete the work in (d+5) days and B can
d) A+B=1254 complete the work in (d+2) days. C takes 5 days
e) None of these more than A.
Quantity I: Find the value of 8d-4?
15) Quantity II: Find the value of 6.5d+15?
SERIES I –15,37.5,112.5,506.25, A Quantity III: Find the value of 4.5d+32?
SERIES II -1404,1716,2028,2340, B a) Quantity I < Quantity II <Quantity III
a) A+B=5183.25 b) Quantity I < Quantity II =Quantity III
b) A+2B=8953.25 c) Quantity I > Quantity II >Quantity III
c) B-A=125.25 d) Quantity I < Quantity II> Quantity III
d) A-B=215.35 e) Quantity I > Quantity II< Quantity III
e) None of these
18)

Click Here For Bundle PDF Course | support@guidely.in Page 5 of 10


SBI Clerk & RRB PO Mains PDF Course 2023
Quantitative Aptitude Day - 47 (Eng)

Quantity I: The ratio of the speed of the boat in Quantity III: A container contains 40 L milk. 20 L
still water and the speed of the stream is 5:2. water is added to that container then 40%
Boat cover 70 km downstream in 2 hours find the mixture is taken out and 15 L water is added.
time taken by the boat to covers 86 km Find the final amount of milk in the mixture?
upstream? a) Quantity I = Quantity II=Quantity III
Quantity II: The ratio of downstream and b) Quantity I < Quantity II< Quantity III
upstream is 5:3. The speed of the stream is 4 c) Quantity I < Quantity II =Quantity III
km/hr. Find the time taken by boat to cover 68 d) Quantity I > Quantity II< Quantity III
km and return? e) Quantity I < Quantity II >Quantity III
Quantity III: The speed of the boat in still water is
20 km/hr. The speed of the stream is 80% less 20)
than the speed of the boat. Find the time taken Quantity I: The ratio of age of A and B after 5
by boat to cover 98 km upstream? years is 5:3. A is 5 years older than C who is 5
a) Quantity I > Quantity II= Quantity III years elder than B. if the age of C is 15 years
b) Quantity I > Quantity II> Quantity III then find the average age of A, B, and C?
c) Quantity I < Quantity II> Quantity III Quantity II: The sum of the age of x, y, and z is
d) Quantity I < Quantity II <Quantity III 48 years. The age of x is 18 years old. If the ratio
e) Quantity I < Quantity II =Quantity III of age of y and z is 3:2 find the age of z?
Quantity III: The age of P is 20 years. The age of
19) Q is 25% more than the Age of P. R is 7 years
Quantity I: The ratio of milk and water in the younger than Q. Find the average age of P, Q,
mixture is 5:3. Find the amount of milk if the and R?
difference between milk and water is 12 liters. a) Quantity I > Quantity II< Quantity III
Quantity II: A mixture contains 85 liters of milk b) Quantity I > Quantity II> Quantity III
and water in a ratio of 3:2. 20 L water and 25 L c) Quantity I < Quantity II =Quantity III
milk are added to the mixture. Find the difference d) Quantity I < Quantity II >Quantity III
between milk and water in the mixture? e) Quantity I < Quantity II <Quantity III

Click Here For Bundle PDF Course | support@guidely.in Page 6 of 10


SBI Clerk & RRB PO Mains PDF Course 2023
Quantitative Aptitude Day – 47 (Eng)

Click Here to Get the Detailed Video Solution for the above given Questions
Or Scan the QR Code to Get the Detailed Video Solutions

Answer Key with Explanation

Directions (01 - 05): The sum of the number of people who takes
According to the question, both 1st and 2nd dose of vaccination from ward A
So, 18a=900, so, a=5 and F together is
60b=3600-900-8*5-10*5=180 = [12500-(8000*120/100) +10000] =12900
Or, b=180/60=3
So, 300=6000, 3600=72000, 10=200. 2) Answer: D
So, 2n+7z=150*200=30000 and Required ratio =
n+z=50*200=10000 [2000+5000]:[3000+2000]=7000:5000=7:5
So, by solving the equations we get, n=8000 and
z=2000. 3) Answer: B
2m+n=90*200=18000, m=[18000-8000]/2=5000 Number of people take booster dose is =
4000*[2*(5+3)]/100
=640

4) Answer: A
2z+3m-2n+3200-a% of 300
=2*2000+3*5000-2*8000+3200-5*300/100
=4000+15000-16000+3200-15=6185

5) Answer: A
The total number of people in the five wards is
1) Answer: C
= [18000+30000+6000+8000+10000] =72000

Click Here For Bundle PDF Course | support@guidely.in Page 7 of 10


SBI Clerk & RRB PO Mains PDF Course 2023
Quantitative Aptitude Day – 47 (Eng)

The total number of people take both doses


= [10000+14000+4000+3000+2000] =33000 8) Answer: B
Required percentage = [33000/72000] The number of i3 generation laptops sold by
*100=45.83% company F is = 1500*125/100=1875
So, the total number of i5 and i7 generation
Directions (06 - 10): laptops sold by company F is =6500-1875=4625
So, the required sum is =
4625+2400+1800=8825

9) Answer: A
Required ratio =
[1800+1000]:[3000+3000]=2800:6000=7:15

10) Answer: A
M= Difference between the number of i3 and i5
generation laptops sold by company B=1800-
1200=600
N- Difference between the number of i5 and i7
generation laptops sold by company C=2500-
1000=1500
6) Answer: C So, 3M+2N=600*3+2*1500=1800+3000=4800
Required difference = [2500+3000]-
[1800+2500]=5500-4300=1200 11) Answer: C
SERIES I –
7) Answer: D
The number of laptops sold with a graphics card
is
= [1200*3/4+1800*2/5+1000*5/8] SERIES II –
=900+720+625=2245
The number of laptops sold without graphics
cards is
=4000-2245=1755 So, A=270, B=204, A-B=270-204=66
So, the required difference = 2245-1755=490

Click Here For Bundle PDF Course | support@guidely.in Page 8 of 10


SBI Clerk & RRB PO Mains PDF Course 2023
Quantitative Aptitude Day – 47 (Eng)

12) Answer: D B=437


SERIES I – A-B=1076-437=639

15) Answer: A
SERIES I –
SERIES II –

A=2531.25
B=285, A=2184, A+B=2184+295=2469
SERIES II –

13) Answer: B
SERIES I –

B=2652, A+B=2531.25+2652=5183.25

16) Answer: C
A=240
The number of blue shirts in bag B is 6*2/3=4
SERIES II –
The number of black shirts in bag B is 6-4=2
Quantity I
Required probability is = [4C2/9C2]=6/36=0.16
Quantity II
B=721
Required probability is =4C1/15C1 +
A+B=240+721=961
3C1/9C1=0.26+0.33=0.59
Quantity III
14) Answer: E
Required probability is =6C1/15C1 +
SERIES I –
4C1/9C1=0.4+0.44=0.84
Quantity I < Quantity II < Quantity III

A=1076 17) Answer: D


SERIES II – So, we can say,(d+5+5)/(d+2)=5/3,
Or, 3d+30=5d+10, d=20/2=10
Quantity I 8*10-4=76
Quantity II 6.5*10+15=80

Click Here For Bundle PDF Course | support@guidely.in Page 9 of 10


SBI Clerk & RRB PO Mains PDF Course 2023
Quantitative Aptitude Day – 47 (Eng)

Quantity III 4.5*10+32=45+32=77 Required difference = [(85*3/5)+25]-


Quantity I < Quantity II> Quantity III [(85*2/5)+20]=76-54=22
Quantity III
18) Answer: C The amount of milk is 40*60/100=24 liters.
Quantity I Quantity I > Quantity II <Quantity III
Downstream speed is 70/2=35 km/hr.
So, 7 unit = 35, 1 unit =5 20) Answer: A
The speed of the boat is 5*5=25 km/hr and the Quantity I
speed of the stream is 5*2=10 km/hr. 5/3= (C+5+5)/(C+5-5)
Required time = 86/[25-10]=5.73 hr Age of C is 15 years
Quantity II Age of A is 15+5=20, age of B is 15-5=10
The speed of the boat is 4*[8/2]=16 km/hr. So, the average age is [20+15+10]=45/3=15
Required time is = 68/20 +68/12=9.06 hr years.
Quantity III Quantity II
Speed of stream is 20*20/100=4 km/hr Age of z is =[48-18]*2/5=30*2/5=12 years
Required time = 98/16=6.125 hr Quantity III
Quantity I < Quantity II >Quantity III Age of P is 20
Age of Q is 20*125/100=25
19) Answer: D Age of R is 25-7=18 years.
Quantity I The average age of P, Q, and R are
Amount of milk is 5*[12/2]=30 liters [25+18+20]/3=21 years.
Quantity II Quantity I > Quantity II< Quantity III

Click Here For Bundle PDF Course | support@guidely.in Page 10 of 10


SBI Clerk & RRB PO Mains PDF Course 2023
ENGLISH Day - 47

English Language

Directions (1-5): In the question given below, terror attack in accepted the
three columns are given. Columns 1 and 3 Jammu and 20 per cent
contain different independent sentences. (3 in Kashmir's waiting list
each column) and Column 2 contains 3
Sunjwan area demand
connectors that may or may not join the given
surfaced on
sentences in the other two columns. Mark the
Saturday,
option which provides the correct sequences of showing
joined sentences as your answer.
A. A2E, B1F, C3D
1.
B. A2D, B3E, C1F
Column I Column II Column III
C. A3E. B1D, C2F

A) Niti Aayog 1. after D) terrorists D. A1F, B3D, C2E,

Vice mounted the E. None of the above is correct match

Chairperson attack on
Rajeev Kumar CISF 2.

has resigned personnel Column I Column II Column III


from his post with grenades
A) Amid rising 1. since D) 2014 at the
and assault
power beginning of
rifles.
demand in the financial
the country year to nine
B) The long 2. and E) will be days as
protest by the replaced by against the
Lok Rakshak economist Centre’s
Dal (LRD) Suman K. mandated 24
aspirants, an Bery from days’ worth of
arm of the May 1. stocks.
Gujarat police,
B) Media 2. which E) a continued
ended on
reports have heat wave,
Friday
suggested India is staring
C) CCTV 3. how F) the state that coal at an
footage of a government inventories electricity

Click Here For Bundle PDF Course | support@guidely.in Page 1 of 12


SBI Clerk & RRB PO Mains PDF Course 2023
ENGLISH Day - 47

had dipped to crisis now lodged in spread


the lowest heightened by a separate
a coal hostel and
shortage at there is no
over 150 need for
power plants hospitalization.

C) The coal 3. due to F) is less than


stock position the regulatory B) As of now, 2. Which E) is slightly
at the Central requirement of the Covid -19 higher than
Election 66.32 MT as situation in infections
Authority of April 21. Tamil Nadu is recorded in
(CEA) under control the previous
supervised but we don’t days
173 power want any rise
plants stood in cases and
at 21.93
C) The state 3. Hence F) we are
million tonnes
reported 286 now doing all
(MT),
active cases the
A. A2E, B1F, C3D necessary
B. A2D, B3E, C1F surveillance
C. A3E, B1D, C2F and all the
D. A1F, B3D, C2E, district
E. None of the above is correct match collectors
have been
3. directed to
Column I Column II Column III ensure that
people must
A) Students of 1. However D) we will be
follow Covid-
IIT Madras taking
appropriate
who have stringent
manners
been tested measures to
positive are prevent the
A. A2E, B1F, C3D

Click Here For Bundle PDF Course | support@guidely.in Page 2 of 12


SBI Clerk & RRB PO Mains PDF Course 2023
ENGLISH Day - 47

B. A2D, B3E, C1F


daughter-in-
C. A3E. B1D, C2F law home.
D. A1F, B3D, C2E
A. A2E, B1F, C3D
E. None of the above is correct match
B. A2D, B3E, C1F
C. A3E. B1D, C2F
4.
D. A1F, B3D, C2E,
Column I Column II Column III
E. None of the above is correct match

A) Promote 1. In an attempt D) the Yogi


sports talent to government 5.

and hone the has prioritised Column I Column II Column III


skills of providing
A) A Ghaziabad 1. as D)
players sports
student died demanding
coming from facilities to all
under the school
the small the players.
mysterious be shut till
villages and
circumstances the matter is
districts,
disclosed

B) The family 2. when E) he was on


B) Jammu 2. In order to E) the attack
protested over his way to
and Kashmir on a CISF
the young boy’s school on
Police have bus in
death, the bus.
detained a Sunjwan on
few persons Friday.
C) The victim’s 3. while F) he had left
family claimed for school in
C) Do 3. in connection F) a farmer
that their son the morning
something with from
was perfectly and have
unique for the Mandsaur
fine accused the
happiness of district of
authorities of
his only son Madhya
lying about
and daughter- Pradesh hired
his death
in-law, a helicopter to
bring his A. A2E, B1F, C3D
B. A2D, B3E, C1F

Click Here For Bundle PDF Course | support@guidely.in Page 3 of 12


SBI Clerk & RRB PO Mains PDF Course 2023
ENGLISH Day - 47

C. A3E. B1D, C2F of Tamil Nadu after they failed to get a decent
D. A1F, B3D, C2E, sale price. Notably, ________________________
E. None of the above is correct match Therefore,in order to avoid more loss, they
halfway dumped nearly seven tonnes of
Directions (6-10): In each of the following Sampangi flowers in pits.
questions a paragraph has been given with a A. they also requested the government to
line/sentence being omitted/missed. You are intervene by providing MSP to avoid such loss of
required to find out the best possible option from livelihood.
the given options that fits aptly in the blank and B. they had to begin plucking the flowers by 5
makes the paragraph complete and coherent. AM in the morning and with a steep fall in prices.
6. BJP’s campaign for the upcoming Mumbai C. the flowers are brought to Sathyamangalam
civic polls has hit a roadblock after their tour Flower producers society where the price is
vehicle was vandalised by bike-borne assailants determined.
last night. _________________________________ D. the cost of Sampangi flower dropped from Rs
to improve the city’s infrastructure and the 70 per kilo last week to Rs 10 for the day,
rampant corruption and scams, was smashed by shocking farmers.
unknown people, prompting the BJP to suspend E. None of the above
the “poll khol” campaign rally.
A. A few policemen from the State Reserve 8. An encounter broke out between Maoists and
Police Force (SRPF) were deployed to maintain Commando Battalion for Resolute Action
law and order in the area (CoBRA) commandos of CRPF in Chhattisgarh's
B. A visibly aggressive Darekar accused the Shiv Bijapur near Sukma border and one CRPF jawan
Sena of pelting stones, following which the police sustained injuries in the attack. A joint team of
stepped up security in the area CoBRA 210 battalion, Special Task Force (STF)
C. The windshield of the truck, fitted with a giant and District Reserve Guard (DRG)
LED screen to play videos outlining the Shiv personnel______________________.
Sena’s failure A. sustained a bullet injury in their leg and were
D. BJP leaders have handed over the CCTV currently undergoing treatment at
footage which showed a person pelting stones at a field hospital in Basaguda.
the truck. B. were out on a search operation when they
E. None of the above were ambushed.
C. and a grenade launcher was recovered from
7. Nearly seven tonnes of Sampangi flowers the encounter site.
were dumped by farmers in pits in Erode district

Click Here For Bundle PDF Course | support@guidely.in Page 4 of 12


SBI Clerk & RRB PO Mains PDF Course 2023
ENGLISH Day - 47

D. and several live Under Barrel Grenade A. the child's caretaker, who was supposed to be
Launcher (UNGL) cells, explosives, Maoist near the van, was arrested.
literature and other materials were also B. based on the parents' complaint, a case under
recovered from the spot. section 304 A was registered with the
E. None of the above Valasaravakkam police
C. the incident occurred on March 28, when
9. After the recent clashes in Delhi's Jahangirpuri Deekshith was dropped by the school van inside
area, Uttar Pradesh Chief Minister Yogi the school premises
Adityanath chaired a law-and-order review D. in connection with a case where a school van
meeting on Monday night and issued a set of ran over an 8-year-old student within the school
directions. Over the premises.
weekend,__________________. As a result, nine E. None of the above
people were injured and admitted to the hospital
for treatment. Directions (11-15): The given sentence has been
A. violence erupted in Jahangirpuri in Delhi broken up into four different parts. The error, if
during a Hanuman Jayanti procession any, will be in one or more parts of the sentence.
B. the UP government said that directions were Select the option which contains the part/parts of
being issued as the festivals of Eid and Akshay the sentence which has/have an error (spelling,
Tritiya are likely to fall on the same date early grammatical or contextual). If there is no error,
next month. choose option E.
C. no religious procession or marches will be 11.
allowed without proper permission I. In account of the high prices of goods and
D. Religious programmes can happen only at services
designated spots and road routes and traffic will II. the government has decided to do away the
not be blocked. system of
E. None of the above III. monthly publication of inflation data and
launched
10. Sri Venkateshwara Matriculation school in IV. the weekly data regarding the prices of white
Chennai dismissed three staff, including the goods
school principal Lakshmi, ________________ A. I, III and IV
While the driver and student's caretaker were B. I, II and III
arrested earlier, the school authorities have C. II and III
today dismissed three staff, after instructions D. I, III and IV
from the State Department of Education. E. No error

Click Here For Bundle PDF Course | support@guidely.in Page 5 of 12


SBI Clerk & RRB PO Mains PDF Course 2023
ENGLISH Day - 47

12. D. Both II and IV


I. In accordance to the provisions of the statute E. No error
of law
II. governing the municipality it was decided that 15.
any decision I. Considering the critical situation of
III. would be taken with explicit approval of II. the patient, the doctors decided to
IV. the Chairman as well as the committee III. start the operation immediately
members IV. without waiting the Police to arrive.
A. I, III and IV A. Both II and III
B. II and III B. Only IV
C. III and IV C. Only III
D. Only I D. Both I and III
E. No error E. No error

13. Directions (16-20): The following questions given


I. The PMI Index by HSBC has indicated that below have four words that have been
II. the services sector of the country is poised highlighted as they might’ve been placed at the
III. to grow by at least 9 percent by the end wrong positions. One of these words might also
IV. of the year though there are concerns be incorrect and need a replacement. Read the
attached to it as well same carefully and mark the appropriate option
A. Only II as the answer.
B. I, III and IV 16. The (A) underpass said the work is (B)
C. II, III and IV expected to begin in the next five to six months
D. I and III and the (C) authority will be ready in two years
E. No error (D) before that.
A. B-D; assumed
14. B. A-C; after
I. It is known to everybody that C. C-D; order
II. he has never taken such a decision D. A-B; overbridge
III. about his child and it is unlikely that E. No swapping needed; across
IV. he will in the future also.
A. Both I and III 17. Actress Vidya Balan said that someone
B. Both II and III asked her if her (A) upcoming movie 'Jalsa' was
C. Only IV

Click Here For Bundle PDF Course | support@guidely.in Page 6 of 12


SBI Clerk & RRB PO Mains PDF Course 2023
ENGLISH Day - 47

a (B) name on actor Amitabh Bachchan's house (D) impact on some parts of the State make for
(C) who goes by the same (D) biopic. an interesting case study.
A. B-D; which A. A-D; efficacy
B. A-C; that B. B-C; zone
C. A-D; impending C. A-B; no replacement needed
D. B-C; identity D. C-D; distance
E. No swapping needed; no replacement needed E. No swapping needed; effect

18. Union Ayush Minister Sarbananda Sonowal 20. While the figure was even higher among the
(A) inaugurated the Yoga (B) Mahotsav 2022 to poor and lower class (A) majority, at well over
mark 100 days (C) countdown to the 8th (D) 80%, what is (B) required and rather surprising is
International Day of Yoga. that our survey also found that the facility of free
A. A-B; set in motion ration even reached a (C) households of the
B. C-D; target well-off households which may not have really
C. B-C; celebration (D) appalling it.
D. B-D; global A. A-C, B-D; interesting
E. No swapping and replacement needed B. A-D, B-C; families
C. A-B, C-D; essential
19. The (A) epicentre of Uttarakhand to the (B) D. Only C-D; No replacement needed
proximity of the farmers’ (C) movement and its E. No swapping needed
Click Here to Get the Detailed Video Solution for the above given Questions
Or Scan the QR Code to Get the Detailed Video Solutions

Answer Key with Explanation

1. Answer: A
The correct match will be- A2E, B1F, C3D

Click Here For Bundle PDF Course | support@guidely.in Page 7 of 12


SBI Clerk & RRB PO Mains PDF Course 2023
ENGLISH Day - 47

A2E- Niti Aayog Vice Chairperson Rajeev Kumar The correct match will be- A1F, B3D, C2E
has resigned from his post and will be replaced A1F- Students of IIT Madras who have been
by economist Suman K. Bery from May 1. tested positive are now lodged in a separate
B1F- The long protest by the Lok Rakshak Dal hostel and there is no need for hospitalization.
(LRD) aspirants, an arm of the Gujarat police, However, we are now doing all the necessary
ended on Friday after the state government surveillance and all the district collectors have
accepted the 20 per cent waiting list demand. been directed to ensure that people must follow
C3D- CCTV footage of a terror attack in Jammu Covid-appropriate manners.
and Kashmir's Sunjwan area surfaced on B3D- As of now, the Covid -19 situation in Tamil
Saturday, showing how terrorists mounted the Nadu is under control but we don’t want any rise
attack on CISF personnel with grenades and in cases and hence we will be taking stringent
assault rifles. measures to prevent the spread.
Hence option A is the correct answer. C2E- The state reported 286 active cases which
is slightly higher than infections recorded in the
2. Answer: C previous days.
The correct match will be- A3E, B1D, C2F Hence option D is the correct answer.
A3E- Amid rising power demand in the country 4. Answer: B
due to a continued heat wave, India is staring at The correct match will be- A2D, B3E, C1F
an electricity crisis heightened by a coal A2D- In order to promote sports talent and hone
shortage at over 150 power plants. the skills of players coming from the small
B1D- Media reports have suggested that coal villages and districts, the Yogi government has
inventories had dipped to the lowest since 2014 prioritised providing sports facilities to all the
at the beginning of the financial year to nine players.
days as against the Centre’s mandated 24 days’ B3E- Jammu and Kashmir Police have detained
worth of stocks. a few persons in connection with the attack on a
C2F- The coal stock position at the Central CISF bus in Sunjwan on Friday.
Election Authority (CEA) supervised 173 power C1F- In an attempt to do something unique for
plants stood at 21.93 million tonnes (MT), which the happiness of his only son and daughter-in-
is less than the regulatory requirement of 66.32 law, a farmer from Mandsaur district of Madhya
MT as of April 21. Pradesh hired a helicopter to bring his daughter-
Hence option C is the correct answer. in-law home.
Hence option B is the correct answer.
3. Answer: D

Click Here For Bundle PDF Course | support@guidely.in Page 8 of 12


SBI Clerk & RRB PO Mains PDF Course 2023
ENGLISH Day - 47

5. Answer: C tonnes of Sampangi flowers. So the option


The correct match will be- A3E, B1D, C2F which aptly fits in the given blank is D. Others
A3E- A Ghaziabad student died under have no relation with the given blank. Hence
mysterious circumstances while he was on his option D is the correct answer.
way to school on the bus.
B1D- As the family protested over the young 8. Answer: B
boy’s death, demanding the school be shut till The paragraph is about an encounter that broke
the matter is disclosed. out between Maoists and Commando Battalion
C2F- The victim’s family claimed that their son for Resolute Action (CoBRA) commandos of
was perfectly fine when he had left for school in CRPF in which one CRPF jawan had injuries in
the morning and have accused the authorities of the attack. Also a joint team of armed forces
lying about his death. were attacked when they were out on a search
Hence option C is the correct answer. operation. So the option which aptly fits in the
given blank is B. Others have no relation with the
6. Answer: C given blank. Hence option B is the correct
The paragraph is all about the BJP’s campaign answer.
for the upcoming Mumbai civic polls in which the
windshield of the campaign truck, fitted with a 9. Answer: A
giant LED screen, was smashed by bike-borne The paragraph is about after the recent violence
men, prompting the BJP to suspend the “poll in Delhi’s Jahangirpuri, UP CM Yogi Adityanath
khol” campaign rally. So the option which aptly issued a set of directions regarding religious
fits in the given blank is C. Others have no processions in the state because of the recent
relation with the given blank. Hence option C is clashes in Delhi's Jahangirpuri area. So the
the correct answer. option which aptly fits in the given blank is A.
Others have no relation with the given blank.
7. Answer: D Hence option A is the correct answer.
The paragraph is all about being unable to get a
decent sale price, further resulting in losses, 10. Answer: D
farmers dumped nearly seven tonnes of The paragraph is about a recent development in
Sampangi flowers in the Erode district of Tamil a case where an 8-year-old boy, a student of
Nadu. The cost of Sampangi flowers dropped Class 2, was crushed to death by the school van
from Rs 70 per kilo last week to Rs 10 which within the school premises; the school
shocked the farmers resulting in dumping the authorities have today dismissed three staff,

Click Here For Bundle PDF Course | support@guidely.in Page 9 of 12


SBI Clerk & RRB PO Mains PDF Course 2023
ENGLISH Day - 47

including the school principal, after instructions provisions of the law governing the municipality.
from the State Department of Education.So the In accordance to is used in order to imply a
option which aptly fits in the given blank is A. reference to any person whereas in accordance
Others have no relation with the given blank. with is used to make reference to any statute or
Hence option D is the correct answer. law, as is the case with this given context.
All the other fragments are correct in all aspects
11. Answer: B and therefore no correction is required in the
Fragment I is incorrect since ‘in account of’ is not other
the correct usage in the given context in which it fragments. The correct sentence would have
is implied that because of the high prices and on been: In accordance with the provisions of the
account of should be used to imply this. statute of law governing the municipality it was
Fragment II is incorrect since it implies that the decided that any decision would be taken with
government has cancelled the monthly explicit approval of the Chairman as well as the
publication of data system and do away with committee members.
refers to this. With is not there in the given So, option D is the correct choice among the
fragment along with do away Fragment III is also given options.
incorrect since there is a problem with the tense
used in the later part of the sentence. The past 13. Answer: E
perfect tense is used in the former part of the The given fragments of the sentence are in order
sentence and the same should be used in this grammatically and contextually with a
part also whereas past tense has been used meaningful sentence being formed out of the
here. It should have been has launched The parts. Therefore no correction is required for any
correct sentence would have been: On account of the fragments. This makes option E the right
of the high prices of goods and services the choice among the given options.
government has decided to do away with the
system of monthly publication of inflation data 14. Answer: C
and has launched the weekly data regarding the The error lies in the last part of the sentence
prices of white goods. This makes option B the since there is problem with the usage of verb in
right choice among the given options. this part. Two auxiliaries can be used with one
principal verb only in case when the principal
12. Answer: D. verb form is appropriate to both the auxiliaries.
Fragment I is incorrect since according to the Here, both the verb forms are not appropriate to
given context, it is implied that according to the the auxiliaries since one is in future tense

Click Here For Bundle PDF Course | support@guidely.in Page 10 of 12


SBI Clerk & RRB PO Mains PDF Course 2023
ENGLISH Day - 47

whereas the other is in present perfect tense. Also, what will be ready? The ‘underpass’ which
Other parts of the sentence are correct and do is placed wrongly in A. So, the best is to
not require any correction. The correct sentence interchange A with C.
would be: It is known to everybody that he has Hence, option (b) is the correct answer.
never taken such a decision about his child and
it is unlikely that he will take in future also. This 17. Answer: A
makes option C the correct choice among the On one simple reading we can find that the word
given options. ‘who’ doesn’t fit anywhere in the sentence so this
definitely needs to change. For house, the
15. Answer: B pronoun ‘which’ would be better.
In the given sentence, the error lies in the The word ‘name’ still looks good in B but the
fragment IV since it does not imply the same as word ‘biopic’ doesn’t go well in D and if these
intended to mean. It is the case that the doctors two words are interchanged, the sentence
started operating on the patient without waiting becomes absolutely correct.
for the police to arrive. ‘Waiting for’ should be Thus, marking (a) as the answer would be crisp!
used in this case to imply the same whereas ‘for’
has been omitted in this part of the sentence. 18. Answer: E
Other parts are correct and do not require any There is no need of words to be interchanged or
correction. The correct sentence would be: swapped as the given sentence is pretty easy
Considering the critical situation of the patient, and understood.
the doctors decided to start the operation Therefore, the correct answer is (e).
properly without waiting for the police to arrive.
This makes option B the correct choice among 19. Answer: C
the given options. The closeness of the state from the area where
the movement was taking place is being
16. Answer: B mentioned.
If the work is said to begin in the coming six So, the words ‘epicentre’ and ‘proximity’ need to
months, how can it be completed two years be changed and the words given make complete
back? So, we can see that the word ‘before’ has sense so there is no need of any replacement.
wrongly been used and should be replaced by Therefore, the correct answer should be option
‘after’. (c).

20. Answer: A

Click Here For Bundle PDF Course | support@guidely.in Page 11 of 12


SBI Clerk & RRB PO Mains PDF Course 2023
ENGLISH Day - 47

The findings reflect that the ration availability So, the word ‘appalling’ which is quite extreme in
was done to majority of the people in need but meaning and is used for horrifying and shocking
some places it was seen that it had reached situations is a misfit in the sentence.
where it wasn’t even needed also. This is an Therefore, the correct answer should be option
element of surprize for the observers and also (a).
interesting as an observation.

Click Here For Bundle PDF Course | support@guidely.in Page 12 of 12


SBI Clerk & RRB PO Mains PDF Course 2023
Reasoning Ability Day – 48 (Eng)

Reasoning Ability
Directions (1-5): Study the following information C. The one who lives two floors below K
carefully and answer the below questions. D. D
Eight persons – B, D, G, H, K, M, Q, and T are E. None of these
from the same family of three generations living
in an eight-storey building marked 1 to 8 from 3) Who among the following person is D’s son-in-
bottom to top respectively. No two persons live law?
on the same floor. Two married couples are in A. The one who lives immediately above H
the family. Q lives four floors above D’s son-in- B. C
law but does not live on the topmost floor. G is C. The one who lives two floors below G
the only daughter of D and lives two floors away D. Both a and c
from her husband. The number of persons living E. None of these
below G is two more than the number of persons
living above D. H is the only daughter of Q’s 4) How many persons are living between K and
brother. T is the mother of Q and mother-in-law H’s aunty?
of G. Three persons are living between T and H. A. As many persons are living between H and T
B is the sibling of H. The number of persons B. Two
living between D and T is one less than the C. As many persons are living between D and K
number of persons living below B. The gender of D. As many persons are living between B and T
B and D is not the same. K’s only son lives on E. None of these
any floor above his daughter.
1) How K is related to the one who lives 5) Which of the following statement is not true?
immediately above B? A. K lives three floors above T
A. Brother-in-law B. Three persons are living between D and B
B. Son C. Two persons are living below M
C. Father-in-law D. G lives on floor number 4
D. Wife E. All the above statements are true
E. None of these
Directions (6-10): Study the following information
2) Who among the following person lives carefully and answer the below questions.
immediately above G’s husband? In a certain code language,
A. The one who lives immediately below H’s ‘___(1)___ Beetle Flea Mantis’ means ‘G@27
mother __(4)__ B#07 D#32’
B. T

Click Here For Bundle PDF Course | support@guidely.in Page 1 of 8


SBI Clerk & RRB PO Mains PDF Course 2023
Reasoning Ability Day – 48 (Eng)

‘Termite Moth Cicada Aphid’ means ‘E@25 C. F#08


___(5)___ D#04 C@05’ D. G@07
‘Ladybug ___(2)___ Wasp Bee’ means ‘E@19 E. H#07
F#48 B#39 A@07’
‘Ant ___(3)___ Grasshopper Fly’ means ‘A@21 10) Which of the following word represents
F#33 ___(6)__ A@31’ “D#07 B#39 A@31”?
6) What does "Ladybug" represent in a code A. Bettle Bee Fly
language? B. Termite Mantis Ant
A. A@07 C. Wasp Ladybug Fly
B. B#39 D. Grasshopper Bee Beetle
C. F#48 E. Beetle Wasp Fly
D. E@19
E. None of the above Directions (11-15): Study the following
information carefully and answer the questions
7) Which of the following code represents in given below.
__(5)__? A certain number of persons are sitting in a linear
A. B#21 row facing towards south direction. Only six
B. C@21 persons sit between N and L, who sits fourth
C. B@22 from one of the extreme ends of the row. J sits
D. D#23 second to the right of N. The number of persons
E. None of the above sitting between L and J is two less than the
number of persons sitting between J and I. I
8) Which of the following word represented in doesn’t sit five places away from N. As many
___(3)___? persons sit to the left of I as to the right of O, who
A. Whitefly sits second to the left of L. At least two persons
B. Butterfly sit between I and H, who sits four places away
C. Horsefly from G. G sits to the right of J. Neither less than
D. Cockroach three nor more than four persons are sitting
E. Mosquito between J and G. Only one person sits between
G and M, who sits seventh to the right of K. K
9) What does "Bumblebee" represent in a code doesn’t sit adjacent to N.
language? 11) How many persons sit in the row?
A. H@09 A. 24
B. G#07 B. 27

Click Here For Bundle PDF Course | support@guidely.in Page 2 of 8


SBI Clerk & RRB PO Mains PDF Course 2023
Reasoning Ability Day – 48 (Eng)

C. 25 E. None of these
D. 26
E. None of these Direction (16-20): In the following paragraph
some statements are given in bold. You have to
12) Who among the following person sits sixth to take these statements to be true while answering
the right of M? the questions asked.
A. K The person explain his project to his mentor, The
B. The one who sits second to the right of N mentor asked that “Some Backend tables are
C. The one who sits three places away from I created” then the student answered “ All
D. The one who sits two places away from L Backend tables are filled with data”, then the
E. None of these mentor told If “ Only a few errors are occurred”,
then the student answered that “ some errors are
13) Which of the following statement(s) is/are rectified” and “ All rectified is done in Backend
true with respect to the final arrangement? tables” and finally the mentor told that “No Data
A. N sits adjacent to I is repeated”.
B. Only three persons sit between K and O 16) Which of the following conclusions are true
C. G sits two places away from H with respect to the given statements?
D. All are true I. No Backend is Repeated
E. None is true II. All Data can be Rectified
III. All Errors can never occur
14) Who among the following person sits IV. Some Create is Data is a possibility
between K and G? A. Both I and III follow
A. L B. Both II and IV follow
B. J C. Both II and III follow
C. I D. All follows
D. Both (b) and (c) E. None follows
E. Both (a) and (b)
17) Which of the following statement is
15) How many persons sit between I and the one necessary to satisfy “No Backend is the
who sits second to the right of O? program”?
A.19 A. Only a few errors is the program
B. 20 B. No Rectangular is Program
C. 21 C. Some created is Program
D. 22 D. Some Data is Program

Click Here For Bundle PDF Course | support@guidely.in Page 3 of 8


SBI Clerk & RRB PO Mains PDF Course 2023
Reasoning Ability Day – 48 (Eng)

E. All Program is Repeated A. Some Data is not an Error


B. No Occurred is Create
18) Which of the following conclusion is/are C. Some Backend is Repeated
definitely false with respect to the given D. All Data can never be Create
statement? E. Some Rectified can be Create
A. Some Rectified is Data
B. All Data can be Rectified 20) Which of the following statement is needed
C. Some Error is Backend to conclude “Some Backend is not a compile”?
D. No Backend is Repeated A. All Occurred is Compile
E. None of the statements is false B. No Rectified is Compile
C. Some Data is Compile
19) Find the conclusion which satisfies the given D. Some Create is Compile
statements? E. None of the above
Click Here to Get the Detailed Video Solution for the above given Questions
Or Scan the QR Code to Get the Detailed Video Solutions

Answer Key with Explanation

Direction (1-5):
1) Answer: C
2) Answer: A
3) Answer: D
4) Answer: D
5) Answer: C

Click Here For Bundle PDF Course | support@guidely.in Page 4 of 8


SBI Clerk & RRB PO Mains PDF Course 2023
Reasoning Ability Day - 48 (Eng)

We have: Again, we have:


 Q lives four floors above D’s son-in-law  H is the only daughter of Q’s brother.
but does not live on the topmost floor.  B is the sibling of H.
 G is the only daughter of D and lives two  T is the mother of Q and mother-in-law of
floors away from her husband. G.
Since, G is the only daughter of D, thus That means, Q is the either sister-in-law
D’s son-in-law is the husband of G. or brother-in-law of G.
 The number of persons living below G is Based on the above given information we have:
two more than the number of persons
living above D.
That means, in case (1) Q lives on floor
number 5, in case (2) Q lives on floor
number 6, in case (3) Q lives on floor
number 7.
Based on the above given information we have:

Again, we have:
 Three persons are living between T and
H.
 The number of persons living between D
and T is one less than the number of
persons living below B.

Click Here For Bundle PDF Course | support@guidely.in Page 5 of 8


SBI Clerk & RRB PO Mains PDF Course 2023
Reasoning Ability Day - 48 (Eng)

Since, H is the only daughter of her Since, from the above statement, K must
parent, thus B must be the son. be married to T, and M must be married
 The gender of B and D is not the same. to G.
That means, in case (1) T lives on floor Thus, in case (2) M lives on floor number
number 2, in case (2) T lives on floor 2, case (1) is not valid.
number 5, case (3) is not valid. Based on the above given information we have:
Based on the above given information we have:

Case (1) is not valid as K’s only son lives on any


Case (3) is not valid as the number of persons floor above his daughter.
living between D and T is one less than the For Blood Relation:
number of persons living below B.
For Blood Relation:

Directions (6-10):
6) Answer: D
Again, we have: 7) Answer: A
 K’s only son lives on any floor above his 8) Answer: C
daughter. 9) Answer: D

Click Here For Bundle PDF Course | support@guidely.in Page 6 of 8


SBI Clerk & RRB PO Mains PDF Course 2023
Reasoning Ability Day - 48 (Eng)

10) Answer: E From the above conditions, there are two


For Symbol Arrangement: Total letters possibilities
countEven# Odd@
For Number Arrangement: The sum of the place
value of the last and first letters of the word in
the English alphabetical series.
For Letter Arrangement:
The total number of letters between the first and
last letters is taken and the resultant number is Again we have,

changed to the corresponding letter as per the  The number of persons sitting between L
and J is two less than the number of
alphabetical series. For example: In between
letters count is 2B, 3C and so on. persons sitting between J and I.
 I doesn’t sit five places away from N.
(1) ButterflyG@27
(2) BeetleD#07 From the above conditions, there are three

(3) MothB#21 possibilities.

(4) WhiteflyF#48
(5) HorseflyF#33
(6) GrasshopperI@25
Directions (11-15):
11) Answer: C
12) Answer: C
13) Answer: E
14) Answer: B Again we have,
15) Answer: A  As many persons sit to the left of I as to
Final arrangement: the right of O, who sits second to the left
of L.
 At least two persons sit between I and H,

We have, who sits four places away from G.

 Only six persons sit between N and L,  G sits to the right of J.

who sits fourth from one of the extreme  Neither less than three nor more than four

ends of the row. persons are sitting between J and G.

 J sits second to the right of N. From the above conditions, case 1 and case 2
get eliminated.

Click Here For Bundle PDF Course | support@guidely.in Page 7 of 8


SBI Clerk & RRB PO Mains PDF Course 2023
Reasoning Ability Day - 48 (Eng)

19) Answer: E
20) Answer: B
Some Backend tables are created
All Backend tables are filled with data
Only a few errors are occurred
some errors are rectified
All rectified is done in Backend tables
Again we have, No Data is repeated
 Only one person sits between G and M, From the above statement, the following diagram
who sits seventh to the right of K. is obtained.
 K doesn’t sit adjacent to N.
Hence, case 1a shows the final arrangement.

Directions (16-20):
16) Answer: A
17) Answer: E
18) Answer: B

Click Here For Bundle PDF Course | support@guidely.in Page 8 of 8


SBI Clerk & RRB PO Mains PDF Course 2023
Quantitative Aptitude Day – 48 (Eng)

Quantitative Aptitude

Direction (1-5): Study the following data carefully and answer the questions:
Graph given below shows the total balls faced, strike rate, number of 4’s hit, and total runs scored by
hitting 6’s by five players A, B, C, D, and E in first match of the tournament.

Note:
1. Some of the values are mentioned in the above graph as it cannot be shown using the graph.
2. Bottom and top of the line shows the number of 4’s hit, and total number of balls faced by each player
respectively.
3. Bottom and top of the rectangular box shows the runs scored by hitting sixes and strike of each player
respectively.
4. Total runs scored by a player = Number of runs scored by hitting 4’s + Number of runs scored by
hitting 6’s + Number of runs scored by running between the wickets.
5. Strike rate is equal to the total number of runs scored by a player for every 100 balls he faced.
6. There is no other method of scoring runs other than hitting 4’s, 6’s and running between the wickets.

Click Here For Bundle PDF Course | support@guidely.in Page 1 of 17


SBI Clerk & RRB PO Mains PDF Course 2023
Quantitative Aptitude Day - 48 (Eng)

1) If player A faced total ‘x’ balls, ‘x + 40’ balls, scored by A and E, and ____ percent
and ‘2x – 20’ balls respectively in 2nd, 3rd, and respectively are scored by running 2’s.
4thmatch of the tournament and scored runs at Which of the following value will fill the blank in
the strike rate of 50, 60, and 70 respectively and the same order?
his average runs in all the 3 matches lies a) 2 and 10
between 45 and 50.What will be the average b) 4 and 5
number of balls faced by him in all the 4 matches c) 3 and 40
and his strike rate in all the 4 matches d) 1 and 50
respectively? e) 1 and 20
[‘x’ is an integer multiple of 5.]
a) 65 and 71 4) If running between the wickets includes 1’s,
b) 70 and 77 2’s, and 3’s and number of 1’s, 2’s, and 3’s ran
c) 75 and 61 by player D is ‘x’, ‘x – 6’, and ‘x – 8’ respectively.
d) 60 and 71 Quantity I: Value of the two closest perfect
e) 66 and 51 squares of ‘x’.
Quantity II: Total runs scored by any player by
2) If running between the wickets includes 1’s running ‘x’ number of 1’s and one 3’s.
and 2’s and player D scored majority of his runs a) Quantity I > Quantity II
by running 2’s while his share of runs scored by b) Quantity I ≥ Quantity II
hitting 6’s is the lowest, the which of the following c) Quantity I ≤ Quantity II
regarding the number of 2’s ran by him between d) Quantity I < Quantity II
the wickets is TRUE? e) Quantity I = Quantity II or relationship cannot
Note: By any method of scoring runs, player D be determined.
did not score same runs by any method.
a) A prime number 5) If running between the wickets includes 1’s,
b) ‘1’ less than the perfect square of an integer. 2’s, and 3’s and player C scores runs by running
c) ‘1’ more than the multiple of 5. all the possible methods. Also, player C ran
d) ‘1’ more than the perfect cube of an integer. maximum possible number of 1’s, then total runs
e) Lies between 15 and 20. scored by him by running 1’s and by hitting 4’s
together is ___ x% of total runs scored by him
3) If running between the wickets includes 1’s and also total number of dot balls faced by him is
and 2’s and total 1’s ran by players A and E ‘y’.
together is 10 and difference between total 2’s
ran by players A and E is ___. If out total runs

Click Here For Bundle PDF Course | support@guidely.in Page 2 of 17


SBI Clerk & RRB PO Mains PDF Course 2023
Quantitative Aptitude Day - 48 (Eng)

Note: Dot balls are those in which no runs are selected for post C is 12.5% of those available
scored. for post C and the total number of candidates
Which of the following is correct pair of value of available for posts A, B and C together is 196.
‘x’ and ‘y’ respectively? 6) If the ratio of male to female candidates
I: 64 available for post C is 15: 13 and the ratio of
II: 50 male to female candidates eligible for post C is 3:
III: 82 4, then find that what per cent of total male
a) (II, I) candidates,who are available for post C, are
b) (II, III) eligible for post C?
c) (I, II) a) (x + 8)%
d) (I, III) b) 4z%
e) (III, I) c) (y + 5)%
d) Both (a) and (b)
Direction (6-9): Study the following data carefully e) All (a), (b) and (c)
and answer the questions:
The data given below is related to the total 7) If the ratio of male to female candidates
number of available candidates, the number of available for post A is , the ratio of male to
eligible candidates (out of total available female candidate available for post B is (2y – 1):
candidates) and the number of selected (x + 1) and the ratio of male to female candidates
candidates (out of total eligible candidates) for available for post C is (x – 3): z, then find the
three different posts A, B and C in a company. average of the number of male candidates
The number of candidates available for posts A available for posts A, B and C?
and Care 5x and 4y respectively and the number a) 41
of candidates available for post B is 10 more b) 45
than 125% of those available for post C. The c)42
number ofcandidates eligible for posts B and C d) 43
are (x + y – 2) and (x + 2) respectively. Number e) 44
of candidates eligible for post A is 15, out of
which candidates were selected for post A. 8)The ratio of male to female candidates eligible
Number of candidates selected for post B is 2 for post B is 1: 2 and the ratio of male to female
more than those selected for post A and the total candidates selected for post B is 3: 4. If the
number of candidates selected for posts A, B and number of male candidates selected for post B is
C together is 19. The number of candidates M% of those eligible for post B and the number

Click Here For Bundle PDF Course | support@guidely.in Page 3 of 17


SBI Clerk & RRB PO Mains PDF Course 2023
Quantitative Aptitude Day - 48 (Eng)

of female candidates selected for post B is N% of total available candidates for post D are eligible
those eligible for post B, then find the value of for post D and of total eligible candidates
[(M – N) * 10]1/3? for post D are selected for post D, then find the
a) 4 average of the number of candidates selected for
b) 7 posts B, C and D?
c)3 a) 8
d) 6 b) 9
e) 5 c)6
d) 10
9)The average of the number of candidates e) 7
available for posts B, C and D is 72. If 30% of

Direction (10-13): Study the following data carefully and answer the questions:
Four persons A, B, C and D are hired to complete a work and each of them completes a different part to
complete the whole work.
Table given below shows the following data:

Note:
1: The time, in which B alone can complete the whole work, is 8 hours more than that, in which C alone
can complete the whole work.
10) Which of the following statements is/are c) Only Q and R
true? d) None is true
P: A, B and D together can complete the whole e) Only R
work in hours.
Q: Ratio of C’s efficiency to D’s efficiency is 9: 4. 11) If A works with of its original efficiency
R: B alone can complete the whole work in 30 and B works with of its original
hours with 75% of its original efficiency. efficiency, then find the time, in which A and B
a) Only Q together can complete the work with their new
b) Only P and Q efficiencies?

Click Here For Bundle PDF Course | support@guidely.in Page 4 of 17


SBI Clerk & RRB PO Mains PDF Course 2023
Quantitative Aptitude Day - 48 (Eng)

a) (x + 2) hours e) 12: 17
b) (y – 5) hours
c)3(y – x) hours 13) If A works with (x * y)% of its original
d) Both (a) and (b) efficiency, B works with [100 + 4(y – x)%] of its
e) Both (a) and (c) original efficiency and C works with (100 + 4y)%
of its original efficiency, then in what time A, B
12) If A, B and E together can complete the and C together can complete the work with their
whole work in hours, then find the ratio of new efficiencies?
time, in which C and E together can complete the a) 8 hours
whole work to the time, in which D and E b) 5 hours
together can complete the whole work? c)7 hours
a) 72: 97 d) 4 hours
b) 16: 27 e) 9 hours
c)36: 47
d) 6: 7

Direction (14-17): Study the following data carefully and answer the questions:
Data given below is related to the number of boundaries (Fours + Sixes) hit by two teams A and B in 4
cricket matches.
Pie chart given below shows the percentage distribution of total boundaries hit by both the teams
together in each match.

Click Here For Bundle PDF Course | support@guidely.in Page 5 of 17


SBI Clerk & RRB PO Mains PDF Course 2023
Quantitative Aptitude Day - 48 (Eng)

Pie chart given below shows the percentage distribution of the number of boundaries hit by team A in
each match.

Note:
1: In 1st match, the ratio of the number of boundaries hit by team A to the number of boundaries hit by
team B is 7: 3.
2: Total number of boundaries hit by team B in all the 4 matches together is 40.
14) If the ratio of the number of boundaries hit by and 5th matches is 10, then find that which of the
team A to team B in 2nd match is p: q and the following statements is/are true?
ratio of the number if boundaries hit by team A to P: Average of the number of boundaries hit by
team B in 4th match is m: n, then find the value of team B in 3rd, 4th and 5th matches is 14.
? Q: Total number of boundaries hit by team B in

a) 48.75 1st and 2nd matches together is not equal to that

b) 43.75 in 5th match.

c) 46.75 R: Difference between the number of boundaries

d) 44.75 hit by teams A and B in 5th match is 8.

e) 42.75 a) Only P
b) Only P and R

15) If the average of the total number of c)All are true

boundaries hit by both the teams together in 3rd, d) None is true

4th and 5th matches is 24 and the average of the e) Only P and Q

number of boundaries hit by team A in 3rd, 4th

Click Here For Bundle PDF Course | support@guidely.in Page 6 of 17


SBI Clerk & RRB PO Mains PDF Course 2023
Quantitative Aptitude Day - 48 (Eng)

16) If the ratio of the number of 4s to the number 18)


of 6s hit by team B in 2nd match is 2: 1 and the
ratio of the number of 4s to the number of 6s hit
by team B in 3rd match is 1: 2, then find the
difference between the total number of 4s hit by
a) P-M, Q-M, Q-N, R-M, R-O
team B in 2nd and 3rd matches together and the
b) P-M, P-N, Q-N, R-M, R-O
total number of 6s hit by team B in 2nd and 3rd
c)P-M, P-N, Q-M, Q-O, R-N
matches together?
d) P-N, P-O, Q-N, R-M, R-O
a) 1
e) P-M, P-N, Q-N, R-N, R-O
b) 2
c)3
19)
d) 4
e) 5

17) If the ratio of 4s to 6s hit by team A in 1st


match is 5: 2, the ratio of 4s to 6s hit by team A
a) P-M, Q-M, Q-N, R-M, R-O
in 2nd match is 2: 1, the ratio of 4s to 6s hit by
b) P-M, P-N, Q-N, R-M, R-O
team B in 3rd match is 1: 1 and the ratio of 4s to
c)P-M, P-N, Q-M, Q-O, R-N
6s hit by team A in 4th match is 2: 3, the find the
d) P-M, P-O, Q-N, Q-O, R-N
average number of 6s hit by team A in each of
e) P-M, P-N, Q-N, R-N, R-O
the 4 matches?
a) 6
20)
b) 5
c)9
d) 7
e) 8
a) P-M, P-O, Q-M, R-N
Direction (18-20): In each question, two columns b) P-M, P-O, Q-N, R-M
A and B are given. Three equations P, Q and R c)P-M, P-O, Q-O, R-O
are given in column A and their roots are given in d) P-M, P-O, Q-M, R-O
column B. Find the correct combination of the e) P-M, P-O, Q-N, R-O
equations with their roots.

Click Here For Bundle PDF Course | support@guidely.in Page 7 of 17


SBI Clerk & RRB PO Mains PDF Course 2023
Quantitative Aptitude Day – 48 (Eng)

Click Here to Get the Detailed Video Solution for the above given Questions
Or Scan the QR Code to Get the Detailed Video Solutions

Answer Key with Explanation

Direction (1-5): Total runs scored by running between the


Tabulating the data given in the graph: wickets = 45 – 38 = 7
Player C:
Runs scored = 100 * (70/100) = 70
Runs scored by hitting 4’s = 8 * 4 = 32
Total runs scored by hitting 4’s and 6’s = 32 + 30
= 62
Total 6’s hit = 30/6 = 5
Player A: Total runs scored by running between the
Runs scored = 80 * (60/100) = 48 wickets = 70 – 62 = 8
Runs scored by hitting 4’s = 5 * 4 = 20 Player D:
Total runs scored by hitting 4’s and 6’s = 20 + 12 Runs scored = 120 * (50/100) = 60
= 32 Runs scored by hitting 4’s = 3 * 4 = 12
Total 6’s hit = 12/6 = 2 Total runs scored by hitting 4’s and 6’s = 12 + 6
Total runs scored by running between the = 18
wickets = 48 – 32 = 16 Total 6’s hit = 6/6 = 1
Player B: Total runs scored by running between the
Runs scored = 75 * (60/100) = 45 wickets = 60 – 18 = 42
Runs scored by hitting 4’s = 5 * 4 = 20 Player E:
Total runs scored by hitting 4’s and 6’s = 20 + 18 Runs scored = 80 * (50/100) = 40
= 38 Runs scored by hitting 4’s = 4 * 4 = 16
Total 6’s hit = 18/6 = 3

Click Here For Bundle PDF Course | support@guidely.in Page 8 of 17


SBI Clerk & RRB PO Mains PDF Course 2023
Quantitative Aptitude Day – 48 (Eng)

Total runs scored by hitting 4’s and 6’s = 16 + 12 Total balls faced in all the 4 matches = 80 + x +
= 28 (x + 40) + (2x – 20) = 100 + 4x = 300
Total 6’s hit = 12/6 = 2 Total runsscored in all the 4 matches = 48 +
Total runs scored by running between the (x/2) + 3(x + 40)/5 + 7(2x – 20)/10 = 48 + 25 + 54
wickets = 40 – 28 = 12 + 56 = 183
Average number of balls faced by A in all the 4
matches = 300/4 = 75
Strike rate of A in all the 4 matches = (183/300) *
100 = 61
Hence, 75 and 61 respectively is the answer.

1. Answer: C 2. Answer: D
Total runs scored by A in 2nd match = x * Number of runs scored by D by hitting 4’s = 12
(50/100) = x/2 Number of runs scored by D by hitting 6’s = 6
Total runs scored by A in 3r match = (x + 40) * Number of runs scored by D by running between
(60/100) = 3(x + 40)/5 the wickets = 42
Total runs scored by A in 4thmatch = (2x – 20) * Since, he scored minimum number of runs by
(70/100) = 7(2x – 20)/10 hitting 6’s and also number of runs scored by
According to the question: him by running 2’s is maximum. Which means
he must score more than 6 runs by running 1’s.
Minimum possible runs scored by D by running
1’s = 7
Number of 2’s ran by D = (42 – 7)/2 = 17.5
[Invalid]
1320<25x+100<1520
Second minimum possible runs scored by D by
1220<25x<1420
running 1’s = 8
48.8<x<56.8
Number of 2’s ran by D = (42 – 8)/2 = 17 [Valid]
Possible values of ‘x’ = (49, 50, 51, 52, 53, 54,
Number of runs scored by D by running 2’s = 17
55, and 56)
* 2 = 34 [Which is maximum among all the
Out of above values of ‘x’, 50 is the only value in
method by which he can scored runs.]
which we get the number of runs scored by A in
Hence, number of 2’s ran by D = 17
2nd, 3rd, and 4th match as an integer.
17 is a prime number.
So, x = 50

Click Here For Bundle PDF Course | support@guidely.in Page 9 of 17


SBI Clerk & RRB PO Mains PDF Course 2023
Quantitative Aptitude Day – 48 (Eng)

17 is ‘1’ less than the perfect square of an


integer. 4. Answer: C
17 is ‘1’ more than the multiple of 5. Total runs scored by D by running between the
17 is not ‘1’ more than the perfect cube of an wickets = 42
integer. 1 * x + 2 * (x – 6) + 3 * (x – 8) = 42
17 lies between 15 and 20. x + 2x – 12 + 3x – 24 = 42
Hence, option (d) is correct. 6x = 78
x = 13
3. Answer: E Quantity I:
Total runs scored by A by running between the Value of ‘x’ = 13
wickets = 16 Two closest perfect square to 13 are 9 and 16.
Total runs scored by E by running between the So, Quantity I = 9 and 16
wickets = 12 Quantity II:
Total runs scored by A = 48 Total runs scored by any player by running ‘x’
Total runs scored by A by running 2’s = of number of 1’s and one 3’s = x * 1 + 3 * 1
48 = 10 =x+3
Total 2’s ran by A = 10/2 = 5 = 16
Total runs scored by running 1’s by A = Total 1’s So, Quantity II = 16
ran by A = 16 – 10 = 6 When Quantity I = 9 and Quantity II = 16, then
Sum of total 1’s ran by A and E together = 10 Quantity I < Quantity II.
Total runs scored by running 1’s by A = Total 1’s When Quantity I = 16 and Quantity II = 16, then
ran by A = 10 – 6 = 4 Quantity I = Quantity II.
Total runs scored by E by running 2’s = 12 – 4 = Hence, Quantity I ≤ Quantity II
8
Total 2’s ran by A = 8/2 = 4 5. Answer: B
Difference between total 2’s ran by A and E = 5 – Since player C ran maximum possible number of
4=1 1’s and also, he scored runs by running 2’s and
Total runs scored by E = 40 3’s, then we need to take minimum possible
Percent of total runs scored by E while running number of 2’s and 3’s ran by him.
2’s = (8/40) * 100 Minimum possible number of 2’s and 3’s ran by
= 20% player C are ‘1’ each.
Hence, the values that can fill the blanks in the Total runs scored by C while running between
same order are 1 and 20 respectively. the wickets = 8

Click Here For Bundle PDF Course | support@guidely.in Page 10 of 17


SBI Clerk & RRB PO Mains PDF Course 2023
Quantitative Aptitude Day – 48 (Eng)

Total runs scored by C by running 2’s and 3’s = And the number of candidates available for post
1*2+1*3=5 C = 4y
Total runs scored by C by running 1’s = 8 – 5 = 3 So,
Total 1’s ran by C = 3/1 = 3 12.5% of 4y = 7
Total runs scored by C by running 1’s and by y = 14
hitting 4’s together = 3 + 32 = 35 The number of candidates available for post C =
x% = (35/70) * 100 4 * 14 = 56
x% = 50% The number of candidates available for post B =
x = 50 125% of 56 + 10 = 80
Total scoring balls faced by C = Total 4’s and 6’s Since, the total number of candidates available
hit + Total number 1’s, 2’s, and 3’s ran = (5 + 8) for posts A, B and C together is 196.
+ (3 + 1 + 1) = 13 + 5 = 18 So, the number of candidates available for post
Total balls faced by C = 100 A = 196 – (56 + 80) = 60
Total dot balls faced by C = y = 100 – 18 Since,
y= 82 5x = 60
Hence, correct pair of values of (x, y) = (50, 82) So, x = 12

Direction (6-9):
Since, the number of candidates eligible for post
A is 15, out of which candidates were
selected for post A.
6. Answer: D
So, the number of candidates selected for post A
The number of candidates available for post C =
= of 15 = 5
56
Since, the number of candidates selected for So, the number of male candidates available for
post B is 2 more than those selected for post A. post C = = 30
So, the number of candidates selected for post B
The number of candidates eligible for post C =
=5+2=7
14
Since, the total number of candidates selected
So, the number of male candidates eligible for
for posts A, B and C together is 19.
post C = =6
So, the number of candidates selected for post C
Required percentage = = 20% = (x +
= 19 – 5 – 7 = 7
8)% = 4z%
Since, the number of candidates selected for
post C is 12.5% of those available for post C.

Click Here For Bundle PDF Course | support@guidely.in Page 11 of 17


SBI Clerk & RRB PO Mains PDF Course 2023
Quantitative Aptitude Day – 48 (Eng)

7. Answer: C And, N = = 25%


Since, the ratio of male to female candidates So, the value of [(M – N) * 10]1/3 = [(37.5 – 25) *
available for post A 10]1/3 = 5

So, the number of male candidates available for 9. Answer: A

post A = = 36 Since, the average of the number of candidates


available for posts B, C and D is 72.
Since, the ratio of male to female candidates
So, the number of candidates available for post
available for post B:
D = (3 * 72) – (80 + 56) = 80
(2y – 1): (x + 1) = 27: 13
The number of candidates eligible for post D =
So, the number of male candidates available for
30% of 80 = 24
post B = = 54
And the number of candidates selected for post
Since, the ratio of male to female candidates
D= of 24 = 10
available for post C:
Since, the number of candidates selected for
(x – 3): z = (12 – 3): 5 = 9: 5
post B = 7
So, the number of male candidates available for
And the number of candidates selected for post
post C = = 36
C=7
Required average = = 42
So, the required average = =8

8. Answer: E
Direction (10-13):
Since, the ratio of male to female candidates
The time taken by B to complete x% part of work
eligible for post B is 1: 2.
= 2.4 hours
So, the number of male candidates eligible for
So, the time, in which B alone can complete the
post B = =8
whole work:
And the number of female candidates eligible for
post B = 24 – 8 = 16
Since, the ratio of male to female candidates The time taken by C alone to complete 4x% part

selected for post B is 3: 4. of work = 6.4 hours

So, the number of male candidates selected for So, the time, in which C alone can complete the

post B = =3 whole work:

And the number of female candidates selected


for post B = 7 – 3 = 4
Now, M = = 37.5%

Click Here For Bundle PDF Course | support@guidely.in Page 12 of 17


SBI Clerk & RRB PO Mains PDF Course 2023
Quantitative Aptitude Day – 48 (Eng)

Since, the time, in which B alone can complete


the whole work, is 8 hours more than that, in 10. Answer: A
which C alone can complete the whole work. From P:
So, Part of work done by A, B and D together in 1
hour:

x = 10
Since, So, the time, in which A, B and C together can
(2y + 5) + x + 4x + y = 100 complete the whole work:
So,
(2y + 5) + 10 + 40 + y = 100
So, P is not true.
y = 15
From Q:
Since, the part of work completed by A in 10.5
Ratio of C’s efficiency to D’s efficiency = 36: 16 =
hours = (2y + 5)% = 35%
9: 4
So, the time, in which A alone can complete the
So, Q is true.
whole work:
From R:
The time, in which B alone can complete the
Since, the part of work completed by B in 2.4 whole work with its original efficiency = 24 hours
hours = x% = 10% So, the time, in which B alone can complete the
So, the time, in which B alone can complete the whole work with 75% of its original efficiency =
whole work: = 32 hours
So, R is not true.

Since, the part of work completed by C in 6.4 Hence, only Q is true.

hours = 4x% = 40%


So, the time, in which C alone can complete the 11. Answer: A

whole work: The time, in which A alone can complete the


work with its new efficiency:

Since, the part of work completed by D in 5.4


hours = y% = 15% The time, in which B alone can complete the

So, the time, in which D alone can complete the work with its new efficiency:

whole work:

Click Here For Bundle PDF Course | support@guidely.in Page 13 of 17


SBI Clerk & RRB PO Mains PDF Course 2023
Quantitative Aptitude Day – 48 (Eng)

The part of work completed by A and B together The time, in which B alone will complete the
in 1 hour = whole work with its new efficiency:
So, the time, in which A and B together can
complete the work with their new efficiencies =
If C works with (100 + 4y)% of its original
12 hours = (x + 2) hours
efficiency:
The time, in which C alone will complete the
12. Answer: E
whole work with its new efficiency:
Since, A, B and E together can complete the
whole work in hours.
So, Now, the part of work completed by A, B and C
together in 1 hour with their new efficiencies =

So, the time, in which A, B and C together will


E = 18 complete the whole work with their new
Now, the part of work complete by C and E efficiencies = 5 hours
together in 1 hour =
So, the time, in which C and E together can Direction (14-17):
complete the whole work: Let the total number of boundaries hit by both
the teams together in 1st, 2nd, 3rd and 4th matches
are 30x, 24x, 18xa and 28x respectively.
The part of work complete by D and E together
Also let the number of boundaries hit by team A
in 1 hour =
in 1st, 2nd, 3rd and 4th matches are 35y, 30y, 10y
So, the time, in which D and E together can
and 25y respectively.
complete the whole work = 12 hours
Since, in 1st match, the ratio of the number of
Required ratio = = 12: 17
boundaries hit by team A to the number of
boundaries hit by team B is 7: 3.
13. Answer: B
So,
If A works with (x * y)% of its original efficiency:
The time, in which A alone will complete the
whole work with its new efficiency: 15y = 30x – 35y
50y = 30x
5y = 3x ------------------(1)
If B works with [100 + 4(y – x)]% of its original
Since, the total number of boundaries hit by
efficiency:
team B in all the 4 matches together is 40.

Click Here For Bundle PDF Course | support@guidely.in Page 14 of 17


SBI Clerk & RRB PO Mains PDF Course 2023
Quantitative Aptitude Day – 48 (Eng)

So, So, the number of boundaries hit by team A in


(30x + 24x + 18x + 28x) – (35y + 30y + 10y + 5th match = (3 * 10) – (6 + 15) = 9
25y) = 40 And the number of boundaries hit by team B in
100x – 100y = 40 5th match = 26 – 9 = 17
5x – 5y = 2 ------------------(2) From P:
From equations (1) and (2): Average of the number of boundaries hit by team
5x – 3x = 2 B in 3rd, 4th and 5th matches:
x=1
From equation (1):
So, P is true.
y = 0.6
From Q:
Total number of boundaries hit by team B in 1st
and 2nd matches together = 9 + 6 = 15
The number of boundaries hit by team B in 5th
match = 17
Total number of boundaries hit by team B in 1st
14. Answer: A and 2nd matches together ≠ The number of
The ratio of the number of boundaries hit by boundaries hit by team B in 5th match
team A to team B in 2nd match: So, Q is true.
p: q = 18: 6 = 3: 1 From R:
The ratio of the number of boundaries hit by Difference between the number of boundaries hit
team A to team B in 4th match: by teams A and B in 5th match:
m: n = 15: 13 17 – 9 = 8
So, the value of = = 48.75 So, R is true.
Hence, all are true.
15. Answer: C
Since, the average of the total number of 16. Answer: B
boundaries hit by both the teams together in 3rd, The number of 4s hit by team B in 2nd match =
4th and 5th matches is 24. =4
So, the total number of boundaries hit by both The number of 6s hit by team B in 2nd match = 6
the teams together in 5th match = (3 * 24) – (18 + –4=2
28) = 26 The number of 4s hit by team B in 3rd match =
Since, the average of the number of boundaries =4
hit by team A in 3rd, 4th and 5th matches is 10.

Click Here For Bundle PDF Course | support@guidely.in Page 15 of 17


SBI Clerk & RRB PO Mains PDF Course 2023
Quantitative Aptitude Day – 48 (Eng)

The number of 6s hit by team B in 3rd match = 12


–4=8 19. Answer: D
Required difference = (2 + 8) – (4 + 4) = 2 From equation P:

17. Answer: A
4x2 – 25x + 39 = 0
The number of 6s hit by team A in 1st match =
4x2 – 12x – 13x + 39 = 0
=6
4x(x – 3) – 13(x – 3) = 0
The number of 6s hit by team A in 2nd match =
x = 3, 13/4
=6
From equation Q:
The number of 6s hit by team A in 3rd match =
=3
The number of 6s hit by team A in 4th match = 4y2 – 15y + 9 = 0

=9 4y2 – 12y – 3y + 9 = 0
4y(y – 3) – 3(y – 3) = 0
Required average = =6
y = 3, 3/4
From equation R:
18. Answer: B
From equation P:
4x2 – 23x + 30 = 0 4z2 – 23z + 15 = 0
4x2 – 8x – 15x + 30 = 0 4z2 – 20z – 3z + 15 = 0
4x(x – 2) – 15(x – 2) = 0 4z(z – 5) – 3(z – 5) = 0
x = 2, 15/4 z = 5, 3/4
From equation Q: The correct combination: P-M, P-O, Q-N, Q-O,
4y2 – 27y + 38 = 0 R-N
4y2 – 8y – 19y + 38 = 0
4y(y – 2) – 19(y – 2) = 0 20. Answer: E
y = 2, 19/4 From equation P:
From equation R: 10x2 + 30x + 10 = 7x2 + 9x + 1000
8z2 – 42z + 45 = 0 3x2 + 21x – 990 = 0
8z2 – 12z – 30z + 45 = 0 x2 + 7x – 330 = 0
4z(2z – 3) – 15(2z – 3) = 0 x2 – 15x + 22x – 330 = 0
z = 3/2, 15/4 x(x – 15) + 22(x – 15) = 0
The correct combination: P-M, P-N, Q-N, R-M, x = -22, 15
R-O From equation Q:

Click Here For Bundle PDF Course | support@guidely.in Page 16 of 17


SBI Clerk & RRB PO Mains PDF Course 2023
Quantitative Aptitude Day – 48 (Eng)

y2 – 27y + 300 = 450 – 32y z2 + 89z + 1320 = 57z + 1100


y2 + 5y – 150 = 0 z2 + 32z + 220 = 0
y2 + 15y – 10y – 150 = 0 z2 + 22z + 10z + 220 = 0
y(y + 15) – 10(y + 15) = 0 z(z + 22) + 10(z + 22) = 0
y = 10, -15 z = -10, -22
From equation R: The correct combination: P-M, P-O, Q-N, R-O

Click Here For Bundle PDF Course | support@guidely.in Page 17 of 17


SBI Clerk & RRB PO Mains PDF Course 2023
ENGLISH Day - 48

English Language

Directions (1-4): In each of the following 3. The Congress party seems to have ________
sentences there are two blank spaces. Below to the ________ to somehow keep the BJP out of
each sentence there are five pairs of words power in Maharashtra.
denoted by numbers (a), (b), (c), (d) and (e). A. Compared, Success
Find out which pair of words can be filled up in B. Replaced, Structure
the blanks in the sentence in the same sequence C. Fallen, Failure
to make the sentence meaningfully complete. D. Reorganized, Victory
1. FCRA guidelines ________ that registered E. Succumbed, Temptation
associations are required to submit an online
annual report with ________ copies of income 4. If we study the lives of great men we are ____
and expenditure statement, receipts and that we too can achieve greatness and, when we
payment account, balance sheet among other die, leave behind our ______.
documents for every financial year within nine A. Convinced, children
months of the closure of the financial year. B. Reminded, footmarks
A. Presume, Hundreds C. Conveyed, followers
B. Allow, Mutilated D. Commanded, belongings
C. Mandate, Scanned E. Proclaimed, Memories
D. Warned, Million
E. Haunted, Horrendous Directions (5): Which of the phrases A,B,C,D &
E given below each statement should replace
2. The document is still a vague, open-ended the phrase printed in bold in the sentence to
mess that says very little while asking a ton of make it grammatically or structurally correct.
____, giving the commission leeway to do more- 5. While the stock market was bouncing back
or-less whatever it wants when the commenting from its 2002 low, U.S. families are still reeling
____ closes in a few months. from the recent recession; between 2001 to
A. Questions, period 2004, typical household savings plummeted
B. Comments, time nearly 25% and the median household debt rose
C. Money, window by a third.
D. Time, website A. was bouncing back from its 2002 low, U.S.
E. Questions, window families are still reeling from the recent
recession; between

Click Here For Bundle PDF Course | support@guidely.in Page 1 of 10


SBI Clerk & RRB PO Mains PDF Course 2023
ENGLISH Day - 48

B. bounced back from its 2002 low, U.S. families 7. Climate change is a growing extreme (a) to
are still reeling from the recent recession, from our food system, with its impact becoming
C. has bounced back from its 2002 low, U.S. increasingly evident. Rising temperatures,
families are still reeling from the recent changing precedent (b) patterns, and threat
recession; between (c) weather events, among other effects, are
D. has bounced back from its 2002 low, U.S. already reducing agricultural yields
families are still reeling from the recent and disrupting (d) food supply chains.
recession; from A. a-d; extreme
E. bounced back from its 2002 low, U.S. families B. b-c; disrupting
are still reeling from the recent recession; from C. a-c; precedent
D. b-d; threat
Directions (6-10): In the following questions, a E. No interchange and replacement required
sentence has been given with four highlighted
words. You have to identify whether the 8. The Andaman and Nicobar islands
highlighted word is used in a correct position or and rejoining (a) areas have been strengthening
not. Also one of the highlighted words is (b) rainfall due to experiencing (c) south-westerly
contextually inappropriate. You are first required winds in the lower tropospheric (d) levels, the
to interchange the words having incorrect weather office said.
position then replace the contextually A. a-d; experiencing
inappropriate word with the correct one from the B. b-c; rejoining
given options. C. a-c; tropospheric
6. On Monday, Prime Minister Narendra Modi D. b-d; strengthening
held bilateral talks with his Nepalese counterfeit E. No interchange and replacement required
(a) Sher Bahadur Deuba in Lumbini – the
birthplace of Gautam Buddha. During the 9. Spokesperson of the Ministry Arindam Bagchi
discussion, they talked about the ways said: "We categorically (a) reject the farcical
to cooperation (b) ongoing strengthen (b) resolution passed by the National Assembly
(c) between the nation and develop new areas in of Pakistan on the subject (c) of
the multifaceted (d) bilateral partnership. the delimitation (d) exercise in the Indian Union
A. a-d; strengthen Territory of Jammu and Kashmir.”
B. b-c; counterfeit A. a-d; farcical
C. a-c; cooperation B. b-c; subject
D. b-d; multifaceted C. a-c; delimitation
E. No interchange and replacement required D. b-d; categorically

Click Here For Bundle PDF Course | support@guidely.in Page 2 of 10


SBI Clerk & RRB PO Mains PDF Course 2023
ENGLISH Day - 48

E. No interchange and replacement required Until Allopathy took long strides in the early
decades of the 20th century, Ayurveda
10. Southwest monsoon advanced (a) into the commanded its place. Yet Indian scholars have
Andaman and Nicobar islands on Monday, the not strategized the recovery of this science of
India Meteorological Department (IMD) said life. Our science of life — Ayur Veda — is
here, largely (b) the start of the four- potentially a champion of Indian civilizational
month seasoning (c) rains that are crucial to revival and deserves our serious attention. Its
the signalling (d) agri-dependent economy. concept of positive health care must be fully
A. a-d; advanced discovered. Who does not wish to live well and
B. b-c; signalling long? When I began to work, I was 20 and it was
C. a-c; largely considered to be too early. My employers retired
D. b-d; seasoning its staff and executives at 55. Today, those
E. No interchange and replacement required retired at 60 complain about how to occupy
themselves. Most of them are fit to continue
Directions (11-17): There is information in the working. Death at 71 or 72 is considered
form of passage given below. Some questions tragically young. At Independence time, 60 was
have been framed on the same which can be an acceptable age to wind up from life. Give or
solved by a careful reading of the passage. You take, life span expectation has jumped by 20
must read the given details carefully and choose years in better-to-do families. The new challenge
the best option for each question as your before Ayurveda, or any other school of
answer. treatment, is how to make the lengthening of
Ayurveda suffered a decline in the late 19th lives more interesting, and more readily
century in the absence of research and enjoyable. Ayurveda and its Rasayana have
development. Ayurveda was the backbone of rejuvenation.
health care through ancient and medieval times, Similarly, Naturopathy can work wonders. I have
until the 19th century, because it was a discipline witnessed my cousin being made to lose weight
for ensuring longevity rather than merely treating and how. When her therapy began, she was 18
the symptoms of illness. As scholars have years old weighing 216 lbs. In twelve months, the
claimed it was a science of life; not merely to weight came down to 130 lbs. The medicines
cure the sick but also to make the healthy were few, the diet was strict for six days a week.
healthier; Chavanprash is an elementary Once a week she could have even a bar of
example while Rasayana or rejuvenation is its chocolate. In a country with rampant poverty,
advanced face. Homeopathy can be a good alternative. In some
persons, a few drops of its medicines work

Click Here For Bundle PDF Course | support@guidely.in Page 3 of 10


SBI Clerk & RRB PO Mains PDF Course 2023
ENGLISH Day - 48

wonders. All for a few rupees — no pain, no A. Ayurveda was the backbone of healthcare
injections, no surgeries. Germans invented it and through ancient and medieval times and came to
they must be generously complimented. India in the 19th century
Unfortunately, hardly anyone has pursued it with B.Ayurveda has provided holistic health to
research. Hence this wonderful cure has not mankind since ancient times, it needs a little
gone very far. Nevertheless, the system support to become an alternative to allopathy
deserves encouragement and greater patronage. C.Most people in India are poor and that is the
Incidentally, Homeopathy is so named because it reason alternative branches of medicine are not
is not an antagonistic treatment. It is sympathetic just needed but necessary
in the sense that the medicine aggravate D.Homeopathy, Naturopathy and Ayurveda are
symptoms of the ailment in the hope that they ancient secrets of the civilisation and proper
get pushed out of the body. Most other schools research can help alleviate the use of Allopathy
of medical treatment are known to be completely
antagonistic which means attacking the E.Each individual wants to live long but what is
symptoms in the hope of killing them. Ayurveda more important is that, one should focus more on
and Naturopathy are different in the sense that living a healthier life
one takes a broad view of the patient's whole life
or ayu. Naturopathy again leans on nature and 12. In what context has the phrase ‘in step
tries to bring the patients' living in step with what with’ been used in the passage?
nature expected. Antagonist treatment can A.People like to live in the comfort of their homes
provoke reactions. Homeopathy is different from B.Naturopathy is in harmony with the nature
others in this regard. C.People wish to get all their treatments in a
Ayurveda must be given the credit for innovating hurry
the vocation of nursing. Both male and female D.The people wish to get natural treatments as
nursing began centuries ago although the they heal quickly
practice took a long time to spread. Indeed, E.Other than the given options
Ayurveda has a record of innovativeness,
sustaining a whole Indian civilization with its 13. Which of the following can be inferred from
holistic approach to life. __________, more the passage?
confidence in its practitioners is necessary; (i) The Greeks have contributed to Indian
investors would come given this ingredient. medicine since ages so much so that the life
11. Which of the given options provides the best expectancy has gone up
central theme for the passage given above? (ii) Most of the alternative branches of medicine
work on the symptoms and not just the problem

Click Here For Bundle PDF Course | support@guidely.in Page 4 of 10


SBI Clerk & RRB PO Mains PDF Course 2023
ENGLISH Day - 48

(iii) Renowned medicine practitioners have only (ii) Homeopathy gets its name from the way in
worked on gastronomy and treatments by which it is executed
working on the stomach (iii) Homeopathy is an all for a few rupees - no
A.Only (i) pain, no injections, no surgeries
B.Both (i) and (ii) A. Only (i)
C.Only (ii) B. Both (ii) and (iii)
D.Both (ii) and (iii) C. Only (ii)
E.None D. Both (iii) and (i)
E. All are true
14. What does one understand by the phrase
‘sympathetic treatment’ as used in the passage? 17. Which of the given adverbs will fill the blank
A.A treatment that goes against the body of the given in the passage above?
patient A.Furthermore
B.The use of reverse psychology where the B.Consequently
patient is treated for the problem he doesn’t have C.However
C.Using drugs that block the production of a D.Moreover
particular substance in one’s body E.Thus
D.A treatment that tries to intensify the symptoms
so that they can be pushed out of the patient’s Directions (18-20): In each question below, two
body sentences have been given. Three starters have
E.Not discussed in the passage been given below the sentences. You are
required to select the appropriate starter that
15. Which of the given parts of the sentence would combine the two sentences by being
highlighted above has an error in it? placed in the beginning of either of the sentences
A.It is sympathetic in the sense without changing the meaning of the sentences
B.that the medicine aggravate 18.
C.symptoms of the ailment in a) That will benefit from Trump’s successes and
D.the hope that they get others that will see gains if he fails
E.pushed out of the body. b) Diversify your investments and consider
mitigating your risks by taking some positions
16. Which of the given options is not true as (i) Assuming…
given in the passage? (ii) Make sure to…
(i) Naturopathy made nursing the most sought- (iii) There is a need to…
after job

Click Here For Bundle PDF Course | support@guidely.in Page 5 of 10


SBI Clerk & RRB PO Mains PDF Course 2023
ENGLISH Day - 48

A.Only (i) C.Only (ii)


B.Both (ii) and (iii) D.Both (i) and (iii)
C.Only (iii) E.All (i), (ii) and (iii)
D.None matches
E.All (i), (ii) and (iii) 20.
a) This kind of success takes effort, investment,
19. and commitment.
a) The change in living quarters, it is important to b) Which is why, when a multinational giant
acknowledge boundaries in the work comes knocking at the door with an impressive
environment. offer, it’s not easy—or always right—to walk
b) For example, in the workplace, I need to away
respect my father as my boss, but he also needs (i) Whatsoever…
to respect my professional opinion even if it (ii) The reality…
differs from his. (iii) Sustaining…
(i) But despite… A.Only (ii)
(ii) Instead of… B.Both (ii) and (i)
(iii) Understanding the… C.Only (iii)
A.Only (i) D.All (i), (ii) and (iii)
B.Only (iii) E.None of the above
Click Here to Get the Detailed Video Solution for the above given Questions
Or Scan the QR Code to Get the Detailed Video Solutions

Answer Key with Explanation

1. Answer: C Meaning: Presume - To believe something to be


The correct answer is C - Mandate, Scanned true based on its likeliness to something,
Mutilated - To injure or disfigure, Mandate -

Click Here For Bundle PDF Course | support@guidely.in Page 6 of 10


SBI Clerk & RRB PO Mains PDF Course 2023
ENGLISH Day - 48

Official Command, Horrendous - Extremely


unpleasant or horrifying 6. Answer: B
Highlighted words “cooperation” and “strengthen
2. Answer: A ” fail to impart any meaning to the current
‘Questions’ gives proper meaning to the position, so we need to interchange them in
sentence when put in the first blank and period order to make the sentence contextually correct.
refers to a length of time. Also the highlighted word “counterfeit” which
means “a fraudulent imitation of something else”
3. Answer: E is contextually inappropriate, so we need to
The correct answer is E - Succumbed, replace it with the correct word i.e. “counterpart”
Temptation which means “a person or thing that corresponds
Meaning: Succumbed - To accept defeat, to or has the same function as another person or
Temptation - The desire to do something thing in a different place or situation”.
4. Answer: B Hence option B is the correct answer.
"Reminded, footmarks" is the correct use.
REMINDED: To take note of something. 7. Answer: C
FOOTMARK: Sign of presence. Highlighted words “extreme” and “threat” fail to
impart any meaning to the current position, so
5. Answer: D we need to interchange them in order to make
In the original sentence, "While the stock market the sentence contextually correct.
was bouncing back" implies that something else Also the highlighted word “precedent” which
was taking place simultaneously in the past, but means “an earlier event or action that is
the rest of the sentence is in present tense regarded as an example or guide to be
("U.S. families are still reeling"). In addition, the considered in subsequent similar
phrasing "between 2001 to 2004" is incorrect; circumstances.” is contextually inappropriate, so
the correct idiom is either "between 2001 and we need to replace it with the correct word i.e.
2004 " or"from 2001 to 2004" (D) CORRECT. “precipitation” which means “rain, snow, sleet, or
This choice remedies the mis-matched tenses by hail that falls to or condenses on the ground”.
pairing the present perfect "has bounced back," Hence option C is the correct answer.
which indicates an action began in the past and
has continued into the present, with the present 8. Answer: B
tense "are still reeling." In addition, it uses the Highlighted words “experiencing” and
correct idiom ("from X to Y"). “strengthening” fail to impart any meaning to the

Click Here For Bundle PDF Course | support@guidely.in Page 7 of 10


SBI Clerk & RRB PO Mains PDF Course 2023
ENGLISH Day - 48

current position, so we need to interchange them correct word i.e. “seasonal” which means
in order to make the sentence contextually “relating to or characteristic of a particular
correct. season of the year.”.
Also the highlighted word “rejoining” which Hence option D is the correct answer.
means “join together again; reunite.” is
contextually inappropriate, so we need to 11. Answer: B
replace it with the correct word i.e. “adjoining” The entire passage talks about the impact of
which means “(of a building, room, or piece of Ayurveda and other branches of medicine as
land) next to or joined with”. they can help in the holistic development of an
Hence option B is the correct answer. individual but the benefits of these disciplines
has become unfruitful because very few people
9. Answer: E know about it and there is a lot of scope in
The highlighted words are contextually correct research in these areas so that it can gain
as well as at their correct positions, thus no acceptance as an alternative to allopathy.
interchange and replacement is required. Hence The first option looks like an additional
option E is the correct answer. information and it doesn’t answer our question.
Farcical- relating to or resembling farce, The third option is one of the aspects discussed
especially because of absurd or ridiculous in the passage and not the central idea. The
aspects. fourth option talks about alleviation of allopathy
Delimitation- the action of fixing the boundary or completely which is not the ambition of the
limits of something. author so it can be ruled out. The fifth option is
not relevant to the question.
10. Answer: D So, the best is to go with option (b) as the
Highlighted words “signalling” and “largely” fail to answer.
impart any meaning to the current position, so
we need to interchange them in order to make 12. Answer: B
the sentence contextually correct. The sentence in which the phrase has been
Also the highlighted word “seasoning” which used hints at naturopathy which works in tandem
means “salt, herbs, or spices added to food to with the nature. The phrase ‘in step with’ means
enhance the flavour or the process of adjusting in agreement with someone or something.
the moisture content of wood to make it more Here, the second option explains the usage of
suitable for use as timber.” is contextually the phrase and all the other options are
inappropriate, so we need to replace it with the superfluous.

Click Here For Bundle PDF Course | support@guidely.in Page 8 of 10


SBI Clerk & RRB PO Mains PDF Course 2023
ENGLISH Day - 48

Thus, the correct answer is option (b). Incidentally, Homeopathy is so named because
it is not an antagonistic treatment. It is
13. Answer: C sympathetic in the sense that the medicine
The first statement makes no sense as; the aggravates symptoms of the ailment in the hope
passage has not described anything about the that they get pushed out of the body.
Greeks. We can’t infer things that are not based Thus, the correct answer is option (d).
on details given in the passage.
The second statement can be inferred because it 15. Answer: B
has been mentioned in the passage that most The sentence carries a very simple error of
medicines, except Homeopathy attack the subject-verb agreement. The
symptoms of the problems to eradicate them. verb aggravate which means to make something
So, this statement makes complete sense. worse or more serious has been wrongly used in
The third statement is a completely new idea its plural form. The subject medicine is singular
and nothing of the sort finds its mention in the and should be followed by a singular
passage. verb aggravates.
So, the only statement that is correct can be So, the best would be to mark option (b) as the
found in option (c). answer.

14. Answer: D 16. Answer: A


Homeopathy as described in the passage is a The first statement is incorrect as the passage
school of medicine that uses sympathetic says that, ayurveda made nursing popular way
treatment in which, as explained the symptoms back it was known to everyone and not
are directly worked upon so that they can be naturopathy. So, (i) is false.
intensified and brought to light before they can The second and the third options are true as
be completely alleviated. both find mention in the third paragraph of the
The first option doesn’t make any sense as, it passage.
doesn’t match with the given meaning. The So, the best is to mark option (a) as the answer.
second option is mere wordplay and can be
eliminated for being superfluous. There’s no clue 17. Answer: C
about the third option in the entire passage. The The usages of the options given above can be
fourth option can be upheld from the lines from understood as:
the third paragraph given below: ‘Furthermore’ can be translated in simple words
as; ‘also’ and ‘in addition.’ It is properly used

Click Here For Bundle PDF Course | support@guidely.in Page 9 of 10


SBI Clerk & RRB PO Mains PDF Course 2023
ENGLISH Day - 48

when there is need only to add one more reason benefit from Trump's successes and
to those before-mentioned. others that will see gains if he fails
‘Consequently’ means as a result of something.
‘However’ is used in place of ‘still’ as in: 19. Answer: A
something is useful still it needs recognition… The best starter that connects the two given
the same condition can be seen in the line with sentences meaningfully is (i). The sentence after
the blank and the previous one as well. connecting would be:
‘Moreover’ means: also; in addition. It is a  But despite the change in living quarters,
synonym to furthermore. it is important to acknowledge boundaries
Out of the given options, option (c) is the best for in the work environment. For example, in
the blank above. the workplace, I need to respect my father
as my boss, but he also needs to respect
18. Answer: B my professional opinion even if it differs
Any of the given starters if placed before the first from his.
sentence won’t make any sense and there would
be a need of other words as well to join the two 20. Answer: C
statements. The sentences can be combined meaningfully
Also, we can use ‘assuming’ to connect the when ‘Sustaining’ starts the first sentence and
sentences but in between so this gets ruled out. the second sentence continues the idea.
The sentences after being connected would be: Other starters do not combine the sentences
 Make sure to diversify your investments well and hence can be ruled out. The correct
and consider mitigating your risks by sentence is given below:
taking some positions that will benefit  Sustaining this kind of success takes
from Trump's successes and others that effort, investment, and commitment.
will see gains if he fails Which is why, when a multinational giant
 There is a need to diversify your comes knocking at the door with an
investments and consider mitigating your impressive offer, it’s not easy—or always
risks by taking some positions that will right—to walk away

Click Here For Bundle PDF Course | support@guidely.in Page 10 of 10


SBI Clerk & RRB PO Mains PDF Course 2023
Reasoning Ability Day – 49 (Eng)

Reasoning Ability
Directions (1-5): Study the following information 2) What is the sum of the areas of a single room
carefully and answer the question given below in flat I of floor number 2 and flat II of floor
In a building, there are four floors numbered 1 to number 1?
4 from bottom to top respectively. Each floor has a) 3312m2
two flats- flat I and flat II from west to east b) 3232m2
respectively. The area of each flat is 8352m2. c) 3132m2
Each flat has different number of rooms and the d) 3322m2
area of each room is equal and a whole number. e) 3222m2
The total number of rooms in none of the floors is
more than 30. 3) Which of the following statement(s) is/are not
The number of rooms in one of the flats of floor false with respect to the final arrangement?
number 2 is a square number and is four more a) The number of rooms in flat I of floor number 4
than the number of rooms in flat II of floor is even number
number 4. The difference between the number of b) The area of each room in flat II of floor number
rooms in flat I of floor number 1 and 3 is six. The 2 is 512 m2
sum of rooms in flat II of floor number 1 and flat I c) The sum of rooms in flat II of floor numbers 1
of floor number 4 is thirteen. The number of and 3 is 10.
rooms in flat I of floor number 4 is multiple of d) Both a and b
three. The difference between the number of e) Both b and c
rooms in flat I of floor number 2 and flat II of floor
number 3 is less than three. The sum of the area 4) Which of the following flat has an area of
of one room in flat I and one room in flat II of floor 1044m2?
number 3 is 1740m2. a) Flat I, floor number 3
1) What is the area of each room in flat II of floor b) Flat II, floor number 1
number 3? c) Flat II, floor number 2
a) 348m2 d) Flat I, floor number 4
b) 1392m2 e) Flat I, floor number 2
c) 522m2
d) 692m2 5) What is the total number of rooms in the
e) 1044m2 building?
a) 87
b) 97
c) 90

Click Here For Bundle PDF Course | support@guidely.in Page 1 of 11


SBI Clerk & RRB PO Mains PDF Course 2023
Reasoning Ability Day – 49 (Eng)

d) 77 Statement II: S@N$R; S&D#Q


e) 80 Who among the following is the son of L?
I:S@N$R%B$L%C
Directions (6-8): Each of the questions below II: Y@L%C&G; Q&D
consists of a question and two statements III: N$R&L; G#L
numbered I and II given below it. A. Only i
A#B means B is the father of A B. Only ii
A@B means A is the daughter of B C. Either only i or only ii
A&B means B is the son of A D. Either only ii or only iii
A$B means A is the mother of B E. Only iii
A%B means A is the husband of B
Along with the given two statements, three 8)
statements are given you have to decide whether Statement I: L%C$G; M#R%B$L
the data provided along with the three Statement II: S@N$R; S&D#Q
statements are sufficient to answer the question. Who among the following is the father of Q?
The statements i, ii, iii given below can be used I:T&Q%S$I@Q
along with the above two statements I, II but II: O@F$Q&D; W%N$S
cannot be combined together. The statements I, III: N&R%B$L; D#Q%S
II above are combined. A. Only i
6) B. Only ii
Statement I:L%C$G; M#R%B$L C. Either only i or only ii
Statement II: S@N$R; S&D#Q D. Either only ii or only iii
How is M related to S? E. None of these
I:K@H%M; F$Q#T
II: V@L#R%B$M Directions (9-10): Each of the questions below
III: S&J#Q; B$L#R&M consists of a question and two statements
A. Only ii numbered I and II given below it.
B. Only i Statement 1:In a certain coded language,
C. Either only i or only ii “Mexican people are talented” is coded as “ar mg
D. Either only i or only iii owxn”, “Roasted chicken are unhealthy” is coded
E. Only iii as “nwgkow md”, “Tall people eat heavy” is
coded as “oezdva mg”.
7) Statement 2:In a certain coded language, “Most
Statement I: L%C$G; M#R%B$L eaten fried chicken” is coded as “kf md ov ci”,

Click Here For Bundle PDF Course | support@guidely.in Page 2 of 11


SBI Clerk & RRB PO Mains PDF Course 2023
Reasoning Ability Day – 49 (Eng)

“Heavy industry eat most” is coded as “varjoekf”, B. Only III


“Mind working are intelligent” is coded as “kd nr C. Either only I or only III
otow”. D. Either only II or only III
Along with the given two statements I, II, three E. None of these
statements i, ii, iii are given
you have to decide whether the data provided Directions (11-15): Study the following
along with the three statements are sufficient to information carefully and answer the below
answer the question. The statements given questions
below can be used along with the above two Unknown number of persons were born in
statements but cannot be combined together. different months of two different years either
The statements I, II above are combined. 2020 or 2021. Information about some of them is
9) What is the code for “Working people eat known.
vegan” in the given coded language? Note: No two known persons were born in the
I: In a certain coded language, “Most boys like same month of different years.
vegan” is coded as “lf iokfbw”. P was born four months after I where both were
II: In a certain coded language, “Working heavy born in different years. I was born in the month
are vegan” is coded as “nr owvaho”. which has an even number of days. S was born
III: In a certain coded language, “Mind game immediately before P. Only four persons were
unhealthy vegan” is coded as “ot ns gkcw”. born between R and S. G was born two months
A. Only I before R but was born in an even numbered
B. Only II year. N was born in the month which has less
C. Either only II or only III than 30 days. As many persons born before N is
D. Either only I or only III same as after M. Only two persons were born
E. None of these between O and M. The number of persons born
between L and Q is same as the number of
10) Which of the following determines the code persons born between O and L. J was born in the
“zdardpvw” in the given coded language? adjacent month of M but both were born in
I: In a certain coded language, “Brilliant talented different years. H was born five months after J.
unhealthy army” is coded as “gkvwxndp”. 11) In which among the following Month and year
II: In a certain coded language, “Mexican chicken Q was born?
roasted army” is coded as “md arvwnw”. a) June - 2021
III: In a certain coded language, “Brilliant tall b) February - 2020
talented army” is coded as “zddpvwxn”. c) April- 2021
A. Only I d) November - 2020

Click Here For Bundle PDF Course | support@guidely.in Page 3 of 11


SBI Clerk & RRB PO Mains PDF Course 2023
Reasoning Ability Day – 49 (Eng)

e) September -2021 e) Seven

12) How many known persons were born Directions (16-20): Study the following
between the one who was born immediately after information carefully and answer the question
I and the one who was born three months before given below
M? Input: @ C 6 * Z A Z # @ 8 L 7 & M 9 K L Q 5 ! 3
a) Six 51$8E0©HF4&U7#1
b) Five Step I: @ 8 _ * Z A Z # @ 8 _ _ & M 9 K L _ R ! 3
c) Ten 51$8E0©H__&U7#1
d) Seven Step II: _ # 7 U _ G 6 H _ 0 E 8 $ 1 5 3 _ R 7 L _
e) Eight 9M&_98@#Z_Z*_8@
Step III: _ # 6 F _ B 7 _ © 1 V 9 $ 0 _ _! M _ G F
13) Who among the following person was born in 8 _! H 8 9 @ # U Z U _ Y _ @
March 2021? Step III gives the final output.
a) S 16) What comes in place of the blank spaces in
b) The one who was born three months after N step II from the right end?
c) P a) &1KMD©! A
d) The one who was born immediately before Q b) 1©&! MKAD
e) L c) MK@*$1DA
d) DAMK! ©&1
14) Who among the following person was born e) DMAK©! &1
seven months after G?
a) P 17) What comes in place of the blank spaces in
b) Unknown person step III from the left end?
c) The one who was born in August 2021 a) O3E246H&9
d) R b) 0D4236H$7
e) The one who was born in December 2020 c) 0#C426H#9
d) O3D246H&9
15) How many Unknown persons were born in e) 03D462H*9
the month which has an odd number of days?
a) Five 18) If all the symbols and even numbers are
b) Six dropped in step III, then which of the following
c) Eleven element is eighth from the right end?
d) Four a) F

Click Here For Bundle PDF Course | support@guidely.in Page 4 of 11


SBI Clerk & RRB PO Mains PDF Course 2023
Reasoning Ability Day – 49 (Eng)

b) H e) D9M76G
c) 9
d) G 20) Which of the following element is tenth from
e) D the left end if all the odd numbers in step II are
dropped?
19) What comes in place of the blank spaces in a) 8
step I from the left end? b) 0
a) D8M67G c) E
b) G7D69M d) H
c) M6D74G e) ©
d) G6M89D
Click Here to Get the Detailed Video Solution for the above given Questions
Or Scan the QR Code to Get the Detailed Video Solutions

Answer Key with Explanation

Directions (1-5):
1) Answer: B
2) Answer: C
3) Answer: C
4) Answer: E
5) Answer: B
Final arrangement:

We have,

Click Here For Bundle PDF Course | support@guidely.in Page 5 of 11


SBI Clerk & RRB PO Mains PDF Course 2023
Reasoning Ability Day – 49 (Eng)

 The number of rooms in one of the flats of


floor number 2 is a square number and is
four more than the number of rooms in
flat II of floor number 4.
 The total number of none of the floors is
more than 30.
From above conditions, we have two
possibilities:

Again we have,
 The difference between the number of
Again we have,
rooms in flat I of floor number 2 and flat II
 The difference between the number of
of floor number 3 is less than three.
rooms in flat I of floor number 1 and 3 is
 The sum of the area of one room in flat I
six.
and one room in flat II of floor number 3 is
 The sum of rooms in flat II of floor number
1740m2.
1 and flat I of floor number 4 is thirteen.
While applying the above conditions, case 1, 1.a
 The number of rooms in flat I of floor
and 2 gets eliminated. Thus case 2.a gives the
number 4 is multiple of three.
final arrangement.

Click Here For Bundle PDF Course | support@guidely.in Page 6 of 11


SBI Clerk & RRB PO Mains PDF Course 2023
Reasoning Ability Day – 49 (Eng)

6) Answer: D
From the given two statements:
Hence, option D is correct.

7) Answer: B
From the given two statements:

Using statement I:

Using statement iI:

Using statement iiI:

Click Here For Bundle PDF Course | support@guidely.in Page 7 of 11


SBI Clerk & RRB PO Mains PDF Course 2023
Reasoning Ability Day – 49 (Eng)

Hence, option B is correct.

8) Answer: E
Using statement II:
From the given two statements:

Therefore, “Working people eat vegan”is coded


as“nr mg oe ho”.
Hence, option B is correct.

None of the statements determine the father of 10) Answer: C


Q. From the given statements, the following were
Hence, option E is correct. determined.

9) Answer: B
From the given statements, the following were
determined.

Click Here For Bundle PDF Course | support@guidely.in Page 8 of 11


SBI Clerk & RRB PO Mains PDF Course 2023
Reasoning Ability Day – 49 (Eng)

Therefore, “zd ar dp vw”is determined as“Tall


Mexican Brilliant Army”.
Hence, option C is correct.

Directions (11-15):
11) Answer: C
12) Answer: B
13) Answer: D
14) Answer: E
15) Answer: E

Using statement I:
I: In a certain coded language, “Brilliant talented
unhealthy army” is coded as
“gkvwxndp”.

Therefore, “zd ar dp vw”is determined as“Tall


Mexican Brilliant Army”.
Using statement III:
III: In a certain coded language, “Brilliant tall
talented army” is coded as “zd dp vw xn”.

We have,

Click Here For Bundle PDF Course | support@guidely.in Page 9 of 11


SBI Clerk & RRB PO Mains PDF Course 2023
Reasoning Ability Day – 49 (Eng)

 P was born four months after I  N was born in the month which has
where both were born in different less than 30 days.
years.  As many persons born before N is
 I was born in the month which has same as after M.
an even number of days.  Only two persons were born
 S was born immediately before P. between O and M.
 Only four persons were born From the above conditions, Case1a is formed.
between R and S. No two known persons were born in the same
From the above conditions there are two month of different years, hence Case2 gets
possibilities. eliminated.

Again we have,
 The number of persons born
Again we have,
between L and Q is the same as
 G was born two months before R
but was born in an even numbered the number of persons born

year. between O and L.

Click Here For Bundle PDF Course | support@guidely.in Page 10 of 11


SBI Clerk & RRB PO Mains PDF Course 2023
Reasoning Ability Day – 49 (Eng)

 J was born in the adjacent month 18) Answer: B -> F B 7 C 1 V 9 M G F H H 9 U Z


of M but both were born in different UY9
years. 19) Answer: E
 H was born five months after J. 20) Answer: A -> # U & G 6 H © 0 E 8 $ ! R L K
From the statement, H was born five months M&M8@#ZAZ*D8@
after J. Case1a gets eliminated. Hence Case1 Step 1: The number which is immediately
shows the final arrangement. followed by a symbol and immediately preceded
by a consonant, then replace the consonant with
its immediate next letter and add two to that
number and then interchange both.
@8D*ZAZ#@89M&M9KL7R!351$
8E0©H6G&U7#1
Step 2: The second half of the elements is
written in reverse order from left to right followed
by the first half of the element in reverse order.
1#7U&G6H©0E8$153!R7LK
9M&M98@#ZAZ*D8@
Step 3:
 For numbers: The odd number is
subtracted by one and even number is
added by one.
 For letters: Vowels are replaced by its
opposite letter as per the English
alphabetical series and consonants are
replaced by its fifth preceding letter as per
the English alphabetical series.
 If a symbol is between the letters, then

Directions (16-20): the symbol is replaced with its immediate

16) Answer: D preceding symbol

17) Answer: C 0#6F#B7C©1V9$042!M6GF8H!H


89@#UZU#Y9@

Click Here For Bundle PDF Course | support@guidely.in Page 11 of 11


SBI Clerk & RRB PO Mains PDF Course 2023
Quantitative Aptitude Day – 49 (Eng)

Quantitative Aptitude

Directions (1-5): Read the following information carefully and answer the questions based on it.
The data given depicts total people who visited blood banks, A, B, C, D, E and F for blood donation in
one week time.

Note: Both ‘x’ and ‘y’ are distinct natural numbers less than 12.
Total people visited = (People with O +ve blood group) + (People with O -ve blood group)
Total people who visited all the blood banks within the week are, 1800.
Total people with O +ve blood group in B: total people visited in blood bank B = 3:7
Total people with O -ve blood group in F: total people visited in blood bank F = 15:19
1) Number of people visited with O –ve blood 2) Find the average number of people visited
group in blood banks A and B together are what with O –ve blood group in blood banks B, C, D
percent of number of people visited with O +ve and E together.
blood group in blood banks A, C and F together? A.95
A.37.5% B.100
B.25% C.85
C.66.66% D.80
D.23.5% E.None of these
E.None of these
3) Find the ratio of total number of people visited
with O +ve blood group in all the blood banks
together and total number of people visited with

Click Here For Bundle PDF Course | support@guidely.in Page 1 of 13


SBI Clerk & RRB PO Mains PDF Course 2023
Quantitative Aptitude Day - 49 (Eng)

O –ve blood group in all the blood banks C.1:3


together. D.2:3
A.1:3 E.None of these
B.5:2
C.7:2 5) Compute the difference between the number
D.1:2 of people with O +ve blood group in blood banks
E.None of these B, C and Dtogetherto that of blood banks A, E
and Ftogether.
4) Find the ratio of total number of people who A.10
visited blood banks A and B to those who visited B.30
blood banks C and E. C.14
A.2:9 D.15
B.7:3 E. None of these

Directions (6-10): Study the information and answer the following questions.
Percentage distribution of the total valid votes of six villages is given in the pie chart.
In every village, only two candidates are fighting in an election.

NOTE:-
Winning candidates of A, C, and E got 60% of valid votes.
Winning candidates of B, F, and D got 70% of valid votes.
Winning candidate of D got 3000 votes more than the losing candidate.

Click Here For Bundle PDF Course | support@guidely.in Page 2 of 13


SBI Clerk & RRB PO Mains PDF Course 2023
Quantitative Aptitude Day - 49 (Eng)

6) Find the total population [approx] of village C if 9) Percentage of invalid votes in villages A, B
12.5% of the vote is invalid, 10% of the people and C is 10%, 20%, and 20% respectively then,
did not cast the vote and 20% of the people are find the total votes cast in villages A, B and C
not eligible to cast the vote. together.
A.18532 A.32122
B.11330 B.30000
C.47362 C.25002
D.12360 D.36000
E.15873 E.14202

7) 10)
Quantity: I Winning candidate votes of village A Quantity: I Find the difference between winning
is what percent of the winning candidate votes of candidate votes of village E and winning
village C? candidate votes of village F.
Quantity: II Losing candidate votes of village B is Quantity: II Find the difference between losing
what percent of losing candidate votes of village candidate votes of village E and F together and
D? losing candidate votes of village A and B
A. Quantity: I < Quantity: II together.
B. Quantity: I ≥ Quantity: II A. Quantity: I < Quantity: II
C. Quantity: II ≥ Quantity: I B. Quantity: I ≥ Quantity: II
D. Quantity: I > Quantity: II C. Quantity: II ≥ Quantity: I
E. Quantity I = Quantity II or relation can't be D. Quantity: I > Quantity: II
established E. Quantity I = Quantity II or relation can't be
established
8) Find the difference between total winning
candidate votes of A, C, and E and total winning Directions (11-15): Study the following
candidate votes of B, D, and F. information and answer the given questions.
A.8912 There were 3 classes in a school 8th, 9th and
B.6702 10th in 2019. The number of boys in the 9th class
C.5950 is 20% more than the number of boys in 8th class.
D.2325 The number of girls in 9th class is 50% more than
E.6225 the number of girls in 10th class. The total
number of boys in 8th and 9th class and the total
number of girls in 9th and 10th class is 128. The

Click Here For Bundle PDF Course | support@guidely.in Page 3 of 13


SBI Clerk & RRB PO Mains PDF Course 2023
Quantitative Aptitude Day - 49 (Eng)

sum of the boys in 8th class and girls in 10th class attend the annual exam is 20% which is 75%
is 56. The number of boys in 10th class is 17 more than the number of girls who didn’t attend
more than the number of girls in 8th class. The the annual exam, then find the total number of
total number of students in all three classes is girls in class 11 in 2020(No new admission has
181. done)?
11) Number of boys in class 11 is 80% of the A.8
number of boys in class 10. 75% of boys in class B.10
11 is equal to 50% of the girls in class 11. Find C.12
the percentage of girls in class 11? D.14
A.54% E.None of these
B.56%
C.58% 14) The number of boys in class 8 in 2019 was
D.60% 20% less than the number of boys in class 7 in
E.62% 2018. In 2018, 60% of the students got passed
and promoted in each gender from Class 7. In
12) On a certain day, 25% of the total students in 2019, class 8 had some new admissions. If the
class 11 were absent. The ratio of the boys to girls who joined as new admission in class 8 in
girls present is 11: 19. The total number of girls 2019 were 60% of the boys who joined as new
enrolled in class 11 is 65. The number of boys admission in class 8 in 2019, then find the
who were absent is 14 more than that of girls. difference between the number of girls who
The number of students in class 11 is what appeared for the annual exam of class 7 in 2018
percentage more/less than the number of and the number of girls in class 10 in 2019?
students in class 8 and 9 together? A.2
A.7.69% B.3
B.6.25% C.4
C.5.69% D.5
D.4.75% E.None of these
E.None of these
15) The average number of students in class 8,
13) In 2019, in class 10, some students did not 11 and 12 is 80. The ratio of the number of boys
attend the annual exam due to Covid19. The in class 11 to class 12 is 5:7 and the total
number of students who passed in the annual number of students in class 11 is 25% less than
exam is 11 less than the total number of students the total number of students in class 12. If the
in that class. If the number of boys who didn’t number of girls in class 12 is 50% of the number

Click Here For Bundle PDF Course | support@guidely.in Page 4 of 13


SBI Clerk & RRB PO Mains PDF Course 2023
Quantitative Aptitude Day - 49 (Eng)

of the boys in class 11 and class 12 together, 18) Arun is traveling from Z to Y at speed of 8
then find the percentage of the girls in class 11? km/h and returns back at point Z at speed of 12
A.33.33% km/h, while Himanshi started fromZ to Y and
B.38.73% returned to Z at speed of 15 km/h. Total journey
C.44.42% time of Himanshi is 17 hours less than that of
D.48.71% Arun, then find the distance between Z and Y?
E.None of these A.240 km
B.120 km
16) Two inlet pipes A and B and one outlet pipe C.180 km
C are connected to a tank. Pipes A and C D.360 km
together can fill a tank in 22.5 minutes, while E.None of these
pipes B and C together can fill the same tank in
90 minutes and ratio of efficiency of pipe A to B 19) Ratio of A’s present age to B’s present age is
is 2: 1 respectively. If pipes A and B works with 5: 6. Ratio of A’s age after 2 years to B’s age
125% and 83(1/3) % of their efficiencies after 4 years will be 4: 5. A and B entered into a
respectively, then in what time pipes A and B partnership and B invested Rs.2000 more than
together will fill the tank? A. If A withdraws his capital when he gets 35
A.12 minutes years old and B withdraws his capital when he
B.6 minutes gets 40 years old and ratio of A’s share to B’s
C.15 minutes share is 9: 8, then find the investment of B?
D.9 minutes A.Rs.16000
E.18 minutes B.Rs.20000
C.Rs.14000
17) Length, breadth and height of a cuboid is in D.Rs.24000
ratio of 3:2:1, and its total surface area is E.Rs.18000
792cm2.Height of cylinder is same as length of
cuboid where its radius is same as difference 20) If the ratio of the marked price to the cost
between height and breadth of cuboid. Find the price of the mobile is 6:5 and the shopkeeper
volume of cylinder (in cm3). Use π = 3. offers a discount of 8% on the marked price of
A.1728 cm3 the mobile. If the difference between the selling
B.5832 cm3 and marked price of the mobile is Rs.2400, then
C.3888 cm3 find the profit earned by the shopkeeper.
D.1944 cm3 A.Rs.2400
E.None of these B.Rs.2500

Click Here For Bundle PDF Course | support@guidely.in Page 5 of 13


SBI Clerk & RRB PO Mains PDF Course 2023
Quantitative Aptitude Day - 49 (Eng)

C.Rs.2600 E.None of these


D.Rs.2800
Click Here to Get the Detailed Video Solution for the above given Questions
Or Scan the QR Code to Get the Detailed Video Solutions

Answer Key with Explanation

Directions (1-5): Total people visited = 400


In blood bank F, In blood bank D,
Given that, Total people visited = 220
Total people with O –ve blood group: total People with O –ve blood group = (200/1100) *
people visited = 15:19 220 = 40
Also, Total people visited = (People with O +ve Thus, People with O +ve blood group = 220 – 40
blood group) + (People with O -ve blood group) = 180
19k = 80 +15k In blood bank C,
People with O +ve blood group = 4k = 80 Total people visited = 320
Thus, k = 20 Also, People with O –ve blood group = (3/8) *320
Hence, People with O -ve blood group = 15k = = 120
300 Thus, People with O +ve blood group = 200
Total people visited = 380 But,
In blood bank E, 25x + 20y = 200
Given that, i.e. 5x + 4y = 40
Total people with O –ve blood group = 40% Given that,
So, 40% = 160 x and y are natural numbers; x, y < 12
1% = 4 Now we’ll try various values that can fit the
Now, People with O +ve blood group = 60% = equation
240 For, x = 1

Click Here For Bundle PDF Course | support@guidely.in Page 6 of 13


SBI Clerk & RRB PO Mains PDF Course 2023
Quantitative Aptitude Day – 49 (Eng)

y = 8.75 Total people who visited all the blood banks


For, x = 2 within the week = 1800
y = 7.5 Total people who visited blood bank A = 1800 –
For, x = 3 (140 + 320 + 220 + 400 + 380) = 340
y = 6.25 People with O –ve blood group who visited blood
For, x = 4 bank A = 200
y=5 People with O +ve blood group who visited blood
For, x = 5 bank A = 140
y = 3.75
For, x = 6
y = 2.5
For, x = 7
y = 1.25
For, x = 8
1. Answer: C
y=0
According to thequestion,
Thus, beyond x = 8, the values of y will occur in
People with O –ve blood group visited in blood
–ve
bank A = 200
So, we get only one value
People with O –ve blood group visited in blood
If, x = 4 and y = 5
bank B = 80
In blood bank B,
Total people visited with O –ve blood group =
People with O –ve blood group = 15x+ 4y = 80
280
According to the question,
People with O +ve blood group visited in blood
The total number of people visited with O +ve
bank A = 140
blood group in B: total number of people visited
People with O +ve blood group visitedin blood
in blood bank B = 3:7
bank C = 200
People with O +ve blood group visited in blood
7k-3k =4k => 80
bank F = 80
So, k = 20
Total people visited with O +ve blood group =
Thus,
420
People with O +ve blood group = 3k = 60
% difference = (280/420) x 100 = (200/3)%
Total people visited = 140
= 66.66%
Now,
Hence, answer is option C

Click Here For Bundle PDF Course | support@guidely.in Page 7 of 13


SBI Clerk & RRB PO Mains PDF Course 2023
Quantitative Aptitude Day – 49 (Eng)

According to thequestion,
2. Answer: B People with O +ve blood group in blood bank B
According tothe question, = 60
People with O –ve blood group visited in blood People with O +ve blood group in blood bank C
bank B = 80 = 200
People with O –ve blood group visited in blood People with O +ve blood group in blood bank D
bank C = 120 = 180
People with O –ve blood group visited in blood People with O +ve blood group in blood bank A
bank D = 40 = 140
People with O –ve blood group visited in blood People with O +ve blood group in blood bankE=
bank E = 160 240
Average = 400/4 = 100 People with O +ve blood group in blood bank F =
Hence, answer is option B 80
Required difference = 460 - 440 = 20
3. Answer: E Hence, answer is option E
According to thequestion,
Total number of people visited with O +ve blood Directions (6-10):
group in all the blood banks together = 900 Total valid votesin all the villages together are
Total number of people visited with O -ve blood 100x
group in all the blood banks together= 900 Total valid votes in village D = 100x*15/100 =
Required ratio= 1:1 15x
Hence, answer is option E Winning candidate get = 15x * 70/100 = 105x/10
votes
4. Answer: D Losing candidate get = 15x * 30/100 = 45x/10
According tothe question, votes
Total number ofpeople who visited blood banks So, according to the question,
A and B = 480 105x/10 - 45x/10 = 3000
Total number ofpeople who visited blood banks Or, 60x = 30000
C and E = 720 Or, x = 500
Required ratio = 2:3 Total valid votes in all villages is = 100 * 500 =
Hence, answer is option D 50000
Total Valid votes in village A = 50000 * 18/100 =
5. Answer: E 9000

Click Here For Bundle PDF Course | support@guidely.in Page 8 of 13


SBI Clerk & RRB PO Mains PDF Course 2023
Quantitative Aptitude Day – 49 (Eng)

Total votes got by Winning candidate in A = Required difference = [5400+6000+7500] -


9000 * 60/100 = 5400 [4200+5250+3500] = 5950
So total votes of losing candidate in A = 9000 -
5400 = 3600 9. Answer: B
In A, 90% of votes casted = 9000
So, total casted votes in A = 9000 * 100/90 =
10000
In B, 80% of votes casted = 6000
So, total casted votes in B = 6000 * 100/80 =
7500
In C, 80% of votes casted =10000
Total votes casted in C = 10000 * 100/80 =
12500
Total votes casted = 10000+12500+7500 =
30000
6. Answer: E
Let the total population of the village is 100x
10. Answer: D
Eligible voters = 80x
Quantity I
Total cast votes = 80x * 90/100 = 72x
Required difference = 7500-3500 = 4000
Total valid votes = 72x * 87.5/100 = 63x
Quantity II
So, 63x = 10000
Required difference = [5000+1500]-[3600+1800]
Or, x = 10000/63
= 1100
So, the total population in village C = 100 *
Quantity I > Quantity II
10000/63 = 15873

Directions (11-15):
7. Answer: D
Let, the number of boys in class 8 = 5x
Quantity I:
Number of boys in class 9 = 5x × 120/100 = 6x
Required percentage = [5400/6000] * 100 = 90%
Let, the number of girls in class 10= 2 y.
Quantity II:
Number of the girls in class 9 = 2y × 150/100 =
Required percentage = [1800/2250] * 100 = 80%
3y
Quantity I > Quantity II
The total number of boys in class 8 and 9 and
the total number of girls in class 9 and 10 are
8. Answer: C
128.

Click Here For Bundle PDF Course | support@guidely.in Page 9 of 13


SBI Clerk & RRB PO Mains PDF Course 2023
Quantitative Aptitude Day – 49 (Eng)

5x + 6x + 2y + 3y = 128 Percentage of girls in class 11 = 100 × 42/70 =


⇒ 11x + 5y = 128 --- (1) 60%
Sum of the boys in class 8 and girls in class 10
is 56. 12. Answer: A
5x + 2y = 56 --- (2) Let, the number of boys and girls present be 11x
From equation (1) and (2), and 19x.
3x = 24 The total number of present students in class
⇒x=8 11= (100 – 25) % of the total students in class 11
Put the value of x in equation (1), ⇒ 11x + 19x = 75% of the total students in class
5 × 8 + 2y = 56 11
⇒ 2y = 16 ⇒ The total students in class 11 = 30x × 100/75
⇒y=8 = 40x
Let, the number of girls in class 8 is z. The total number of students absent = 40x – 30x
Number of boys in class 10 = z + 17 = 10x
The total number of students in all three classes The number of boys who were absent is 14 more
= 181 than that of girls.
⇒ 40 + z + 72 + z + 17 + 16 = 181 So, the number of girls absent = p.
⇒ 2z = 36 Number of boys absent = p + 14
⇒ z = 18 The total number of students absent = p + p + 14
= 2p + 14
⇒ 10x = 2p +14
⇒ p = (10x – 14)/2
The total number of the girls enrolled in class 11
= 65
⇒ 19x + p = 65
11. Answer: D ⇒ 19x + (10x – 14)/2 = 65
Number of boys in class 11 = 35 × 80/100 = 28 ⇒ 38x + 10x – 14 = 130
According to the Question, ⇒ x = 144/48 = 3
75% of the boys in class 11 = 50% of the girls in The number of students in class 11 = 40x = 40 ×
class 11 3 = 120
⇒ 28 × 75/100 = girls in class 11 × 50/100 Number of students in class 8 and 9 together =
⇒ Girls in class 11 = 42 58 + 72 = 130
The total students in class 11 = 28 + 42 = 70

Click Here For Bundle PDF Course | support@guidely.in Page 10 of 13


SBI Clerk & RRB PO Mains PDF Course 2023
Quantitative Aptitude Day – 49 (Eng)

Required percentage = 100 × (130 – 120)/130 = Number of boys who took new admission for
100 × 10/130 = 7.69% class 8 = 40 – 30 = 10
Number of girls who took new admission for
13. Answer: C class 8 = 10 × 60/100 = 6
Number of students passed in the final exam = Number of girls passed and promoted = 18 – 6 =
51 – 11 = 40 --- (1) 12
Number of boys who didn’t attend the final exam ⇒ 0.6x = 12
= 35 × 20/100 = 7 ⇒ x = 20
Number of boys who attend the final exam = 35 Number of girls in class 7 in 2018 = 20
– 7 = 28 Difference between the number of girls who
Number of girls who didn’t attend the final exam appeared for the final exam of class 7 in 2018
= 100 × 7/175 = 4 and girls in class 10 in 2019 = 20 – 16 = 4
Number of girls who attend the final exam = 16 –
4 = 12 15. Answer: D
The total number of students who attend the final The total number of the students in class 8,
exam = 40 --- (2) 11 and 12=80 × 3 = 240
From statement (1) and (2), The total number of the students in class 11 and
All students passed in the final exam. 12=240 – 58 = 182
So, the total number of girls in class 11 in 2020 = Let, the number of boys in class 11 and 12 is 5x
12 and 7x respectively.
Number of the girls in class 12=50% of the
14. Answer: C number of the boys in class 11 and 12 together
The number of boys in class 8 in 2019 =80% of ⇒Number of the girls in class 12 = (5x + 7x) ×
the boys in class 7 in 2018 50/100 = 12x × 50/100 = 6x
⇒ 40 = 80% of the boys in class 7 in 2018 Let, the number of girls in class 11 be ‘g’.
⇒ The number of boys in class7 in 2018 = 100 × According to the Question,
40/80 = 50 5x + g = (7x + 6x) × 75/100
Number of boys passed and promoted = 50 × ⇒ 20x + 4g = 39x
60/100 = 30 ⇒ 19x - 4g = 0 --- (1)
Let the number of girls in class 7 in 2018 be x. Now,
Number of girls passed and promoted = x × The total number of students in class 11 and 12
60/100 = 0.6x together = 182
⇒ 5x + g + 7x + 6x = 182

Click Here For Bundle PDF Course | support@guidely.in Page 11 of 13


SBI Clerk & RRB PO Mains PDF Course 2023
Quantitative Aptitude Day – 49 (Eng)

⇒ 18x + g = 182 --- (2) Time taken by pipe B alone to fill the tank = 2 *
From equation (1) and (2), 15 = 30 minutes
91x = 728 Time taken by pipe B alone, when works with
⇒x=8 83(1/3) % efficiency:
Put the value of x in equation (1), 30 * (300/250) = 36 minutes
19 × 8 – 4g = 0 Part of tank filled by pipes A and B together with
⇒ g = 152/4 = 38 their new efficiencies in 1 minutes = (1/A) + (1/B)
Number of girls in class 11= 38 = (1/12) + (1/36) = 1/9
The total number of students in class 11 = 5x + So, time taken by pipes A and B together to fill
38 = 5 × 8 + 38 = 78 the tank with their new efficiencies = 9 minutes
Required percentage = 100 × 38/78 = 48.71%
17. Answer: D
16. Answer: D Let length breadth and height of cuboid = 3a, 2a,
Since, ratio of efficiency of pipe A to B is 2: 1 a respectively
respectively. Now,
So, ratio between their times will be = 1: 2 2 x (3a x 2a + 2a x a + a x 3a) = 792
Let time taken by pipe A alone and pipe B alone 11a2 = 396
to fill the tank is ‘x’ minutes and ‘2x’ minutes So, value of a = 6 cm
respectively. So, height of cylinder = 3 x 6 = 18 cm
Also let time taken by pipe C alone to empty the Radius of cylinder = 2a – a = a = 6 cm
tank is ‘y’ minutes. Volume of cylinder = 3 x 6 x 6 x 18 = 1944 cm3
So, (1/x) – (1/y) = (1/22.5) -------------------(1) Hence answer is option D
And, (1/2x) – (1/y) = (1/90) -------------------(2)
From equations (1) and (2): 18. Answer: A
(1/x) – (1/2x) = (1/22.5) – (1/90) Average speed of Arun = (2 x 8 x 12) / (12 + 8) =
1/2x = 1/30 48/5 km/h
x = 15 Let journey time of Himanshi = a hours
Time taken by pipe A alone to fill the tank = 15 So, journey time of Arun = (a + 17) hours
minutes Now,
Time taken by pipe A alone, when works with 48/5 x (a + 17) = 2 x 15 x a
125% efficiency: 16a + 272 = 50a
15 * (100/125) = 12 minutes 34a = 272
So, value of a = 272/34 = 8

Click Here For Bundle PDF Course | support@guidely.in Page 12 of 13


SBI Clerk & RRB PO Mains PDF Course 2023
Quantitative Aptitude Day – 49 (Eng)

So, distance between Z and Y = 2 x 15 x 8 = 240 So,


km [p * 5]/ [(p + 2000) * 4] = 9/8
Hence answer is option A 40p = 36p + 72000
p = 18000
19. Answer: B So, B’s investment = 18000 + 2000 = Rs.20000
Let A’s present age B’s present age are ‘5x’
years and ‘6x’ years respectively. 20. Answer: C
So, (5x + 2)/ (6x + 4) = 4/5 CP of mobile = 5x
25x + 10 = 24x + 16 MP of mobile = 6x
x=6 SP of mobile = 6x * 92/100 = 5.52x
A’s present age = 5 * 6 = 30 years 6x – 5.52x = 2400
And A will get 35 years old after 5 years. x = 5000
B’s present age = 6 * 6 = 36 years SP = 5.52 * 5000 = 27600
And B will get 40 years old after 4 years. CP = 5 * 5000 = 25000
Let A’s investment and B’s investment are ‘p’ Profit = 27600 – 25000 = Rs.2600
rupees and (p + 2000) rupees.

Click Here For Bundle PDF Course | support@guidely.in Page 13 of 13


SBI Clerk & RRB PO Mains PDF Course 2023
ENGLISH Day - 49

English Language

Directions (1-5): The sentences given below 3. While all the companies are global leaders in
have been divided into five parts. Out of the five their (A)/ own right, the meeting with General
parts, one part has an error in it. Identify the Atomics head (B)/acquire significance as the
erroneous part and mark that option as your Indian Navy is already operating two (C)/
answer Predator MQ-9 unmanned aerial vehicles for
1. In the backdrop of India acquiring 30 Predator building maritime (D)/ domain awareness from
drones from (A)/the US for muscling over the Gulf of Aden to Lombok Straits in Indonesia (E).
horizon military capability, (B)/Prime Minister A. A
Narendra Modi is meeting the head (C)/of armed B. B
drone manufacturer General Atomics along with C. C
(D)/ four other top US company CEOs in D. D
Washington on September 23. (E). E. E
A. A
B. B 4. With Biden administration giving a green
C. C (A)/signal for India to acquire Predator drone, it
D. D (B)/ is only a matter of time when the Indian
E. E (C)/Navy brings up the proposal before (D)/the
Defence Acquisition Committee (DAC) for the
2. It is quite evident from the line-up that PM process to take off. (E)
Modi’s meetings with the (A)/ CEOs are with the A. A
specific intention of India be a key part of global B. B
(B)/resilient chain, a military power which C. C
matches over the (C)/ capabilities in the Indo- D. D
Pacific and speed up non-conventional energy E. E
(D)/needs to go beyond the climate change
commitments for climate justice (E). 5. The U.K. approach contrasts with the U.S.,
A. A which (A)/said that fully vaccinated foreigners
B. B would (B)/ be welcome from early November, as
C. C long as (C)/ they have had a full course of one of
D. D the medication (D)/ listed for emergency use by
E. E the World Health Organization (E)
A. A

Click Here For Bundle PDF Course | support@guidely.in Page 1 of 9


SBI Clerk & RRB PO Mains PDF Course 2023
ENGLISH Day - 49

B. B forces and hospitals on this, __________ (12) is


C. C affected by the perception of harassment and
D. D legal complications. The Ministry’s latest move
E. E seeks to overcome __________ (13) by rewarding
socially minded individuals who offer immediate
Directions (6-10): In the passage given below assistance and rush a victim with certain kinds of
there are blanks and they are numbered. Fill with injuries to hospital, with ₹ 5,000 and a certificate
the options given below the passage against of recognition for saving a life.
each of the respective numbers. Choose the 6.
correct option to fill in the blank that can make A. Compassionate
the paragraph meaningful and complete B. Streaming
coherently. C. Staggering
The initiative of the Ministry of Road Transport D. Horrified
and Highways to award Good Samaritans who E. Partiality
save lives of road accident victims with a cash
prize is a welcome attempt to reduce India’s 7.
__________ (9) annual death toll from mishaps. A. Fatalities
Ranking third among 20 nations that have the B. Unscrupulous
highest number of accidents, India fares far C. Exploit
worse on an important metric — cases to D. Compassion
__________ (10) ratio — compared to the U.S. E. Stream
and Japan, which have more recorded crashes
but fewer deaths. During 2020, even with 8.
severely disrupted mobility due to COVID-19, A. Lax
National Crime Records Bureau data show B. Grim
1,33,715 lives were lost in 1,20,716 cases C. Underscore
attributed to negligence relating to road D. Suspicion
accidents. Under the Motor Vehicles law, a Good E. Obligation
Samaritan voluntarily helps an accident victim
with no expectation of payment or reward, and 9.
has no legal __________ (11) to record his A. Convalescent
involvement or aid the investigation in the case. B. Extraordinary
In spite of an entire chapter being added to the C. Tame
Motor Vehicles Act last year to sensitise police D. Altruism

Click Here For Bundle PDF Course | support@guidely.in Page 2 of 9


SBI Clerk & RRB PO Mains PDF Course 2023
ENGLISH Day - 49

E. Inclusive day Meanwhile, southwest


monsoon reportedly
10. entered Karntaka on
A. Complacency June 9 through
B. Deport Madikeri and Karwar
C. Reticence districts.
D. Scrutinize
E. Suspension A. 1-a, 2-b, 3-c
B. 1-b, 2-c, 3-a
Directions (11-15): Given below are a few C. 1-c, 2-a, 3-b
questions with two columns in each of them. You D. 1-a, 2-c, 3-b
have to match the first column with the second E. None of the above
column in order to make sentence contextually
correct. If none of the options are correct then 12.
choose option E as your answer. Column 1 Column 2
11. 1.Mother Dairy and a.by higher fodder
Column 1 Column 2 Amul have hiked milk costs, robust demand,
1.The IMD has issued a.while Shirali and Kota prices multiple times in and some impact due
a yellow alert for the in Uttara Kannada the last year. The price to reports of lumpy
coastal Karnataka, district received 4 cm hikes were induced skin.
Dakshina Kannada and rainfall. 2.Congress general b. and accused the
Uttara Kannada secretary Jairam Narendra Modi-led
districts as isolated Ramesh claimed on central government of
heavy Friday that India is on pitting one cooperative
2.Coastal Karnataka b. this June in several the brink of a milk crisis against the other for
region received a good parts of Tamil Nadu, electoral benefits.
amount of rainfall as including Chennai.
Mangaluru in Dakshina 3.The dairy industry c.of fodder shortage
Kannada received 5 has expressed concern and other factors have
cm rainfall, over low milk forced raw milk prices
3.The mercury has c.rains are expected in procurement and said to remain high.
crossed 40 degrees these districts between the prevailing situation
Celsius for the eighth June 10 and June 12. A. 1-a, 2-b, 3-c

Click Here For Bundle PDF Course | support@guidely.in Page 3 of 9


SBI Clerk & RRB PO Mains PDF Course 2023
ENGLISH Day - 49

B. 1-b, 2-c, 3-a 1.The Jharkhand State a.since its recruitment


C. 1-c, 2-a, 3-b Student Union (JSSU) policy in 2021 was set
D. 1-a, 2-c, 3-b has again called a aside by the Jharkhand
E. None of the above strike (bandh) for two High Court in
days that is December 2022.
13.
Column 1 Column 2 2.The bandh has been b.scheduled from June
1.High waves were a.is expected to organised to protest 10 till June 11.
observed at Tithal Beach reach up to 45 to 55 against the 60:40 ratio-
in Gujarat's Valsad on the knots on June 10, based recruitment
Arabian Sea coast, in 11 and 12. 3.The Hemant Soren c.policy of the Chief
anticipation of Cyclone government was Minister Hemant
Biparjoy. As a forced to bring a new Soren-led state
precautionary measure, recruitment policy in government.
2.Several districts in b.Tithal Beach has 2022 based on the
Kerala, including, been closed to 60:40 ratio formula,
Thiruvananthapuram, tourists until June A. 1-a, 2-b, 3-c
Kollam, Pathanamthitta, 14. B. 1-b, 2-c, 3-a
Alappuzha, C. 1-c, 2-a, 3-b
3.The IMD has said the c.Kottayam, Idukki, D. 1-a, 2-c, 3-b
wind speed due to Kozhikode, and E. None of the above
Cyclone Biparjoy, Kannur were put on
yellow alert on 15.
Friday. Column 1 Column 2
1.The students a.on April 18, followed
A. 1-a, 2-b, 3-c organised torchlight by a bandh on April
B. 1-b, 2-c, 3-a processions 19.
C. 1-c, 2-a, 3-b (mashaljulus) at the
D. 1-a, 2-c, 3-b block level
E. None of the above 2.Earlier, in April 2023, b. and reclaiming
Jharkhand’s students executive control over
14. union staged a 72-hour service-related
Column 1 Column 2 protest matters.

Click Here For Bundle PDF Course | support@guidely.in Page 4 of 9


SBI Clerk & RRB PO Mains PDF Course 2023
ENGLISH Day - 49

17.
3.The Centre issued an c.against the 60:40 A. Germany would not “preach or teach” India
ordinance on May 19, recruitment policy. about its position on Russia’s war in Ukraine.
establishing the B. For the success of this platform, we require all
National Capital Civil countries to be present.
Service Authority C. If the Russian action goes unchecked, I think
A. 1-a, 2-b, 3-c it will be really devastating for all of us.
B. 1-b, 2-c, 3-a D. Mr. Plotner’s visit coincides with a number of
C. 1-c, 2-a, 3-b other visits to Delhi carrying a similar message
D. 1-a, 2-c, 3-b on Russia.
E. None of the above E. All the statements are related to each other

Directions (16-19): Five sentences have been 18.


given in each question below one of which is an A. The BJP had constituted a five-member team
odd man out. You are required to read the same in the aftermath of the violence in which eight
carefully and choose the option with the odd people were burnt alive.
sentence as your answer B. The five members on the panel include Rajya
16. Sabha member and former Uttar Pradesh
A. On March 29, Assam and Meghalaya partially Director General of Police Brajlal.
resolved a 50-year-old dispute along their 884.9 C. Following an order of the Calcutta High Court,
km boundary. the Central Bureau of Investigation (CBI) has
B. Meghalaya was carved out of Assam as an taken over the probe into the Birbhum violence.
autonomous State in 1972. D. West Bengal BJP chief Sukanta Majumdar —
C. It was facilitated by Home Minister Amit Shah met BJP president J.P. Nadda and presented the
who urged the States to resolve their boundary report on Wednesday.
disputes by August 15, 2022. E. Claiming that “state-sponsored extortion,
D. According to the partial boundary deal, Assam goonda tax and cut-money” were the main
will get 18.51 sq. km of the 36.79 sq. km reasons for the violence, the report said the
disputed area. Centre should intervene.
E. Apart from Meghalaya, the other States are
Arunachal Pradesh, Mizoram, Nagaland, Tripura 19.
and West Bengal. A. A World Cup like no other in its 92-year history
will take shape this week.

Click Here For Bundle PDF Course | support@guidely.in Page 5 of 9


SBI Clerk & RRB PO Mains PDF Course 2023
ENGLISH Day - 49

B. Qatar will be the top-seeded team in Group A, D. The entry list for the forthcoming Federation
taking position A1 in the schedule. Cup athletics has been released.
C. Qatar’s debut opens the tournament, Nov. 21 E. It means in the group stage Qatar avoids the
at Al Bayt Stadium. world’s top-ranked teams, from Nos. 1 to 7
Click Here to Get the Detailed Video Solution for the above given Questions
Or Scan the QR Code to Get the Detailed Video Solutions

Answer Key with Explanation

1. Answer: B Hence, option (c) is correct.


The correct usage in part (B) should be
‘muscling up’ that means ‘working hard on 4. Answer: A
improving the military powers’ or ‘strengthening’. The correct usage should be ‘the Biden
The other parts are grammatically correct. administration’ as the article is important here to
signify that there is a mention of a specific
2. Answer: B administration.
The correct usage here should be ‘being a key 5. Answer: D
part of’ which means ‘India is the centre of the The correct usage in D should be ‘one of the
global resilient chain, having indefinite power’ so medications' as per the rule that says ‘one of the’
it ought to work more on the climate change should always be followed by a plural subject.
policies. Hence, the correct answer is option (d).

3. Answer: C 6. Answer: C
The correct usage should be 'the meeting Sol. “Staggering” will fill in the blank to make the
acquires’ as per the subject verb agreement rule. paragraph meaningful and complete coherently.
Acquired could also have been correct but Compassionate: feeling or showing sympathy
unnecessarily violating the tense is not needed. and concern for others.

Click Here For Bundle PDF Course | support@guidely.in Page 6 of 9


SBI Clerk & RRB PO Mains PDF Course 2023
ENGLISH Day - 49

Streaming: a method of transmitting or receiving Obligation: an act or course of action to which a


data (especially video and audio material) over a person is morally or legally bound; a duty or
computer network as a steady, continuous flow, commitment.
allowing playback to start while the rest of the
data is still being received. 9. Answer: D
Staggering: deeply shocking; astonishing. Sol. “Altruism” will fill in the blank to make the
Horrified: filled with horror; extremely shocked. paragraph meaningful and complete coherently.
Partiality: unfair bias in favour of one person or Convalescent: (of a person) recovering from an
thing; favouritism. illness or medical treatment.
Extraordinary: very unusual or remarkable.
7. Answer: A Tame: (of an animal) not dangerous or
Sol. “fatalities” will fill in the blank to make the frightened of people; domesticated.
paragraph meaningful and complete coherently. Altruism: disinterested and selfless concern for
Fatalities: an occurrence of death by accident, in the well-being of others.
war, or from disease. Inclusive: including all the services or items
Unscrupulous: having or showing no moral normally expected or required.
principles; not honest or fair.
Exploit: make full use of and derive benefit from 10. Answer: C
(a resource). Sol. “Reticence” will fill in the blank to make the
Compassion: sympathetic pity and concern for paragraph meaningful and complete coherently.
the sufferings or misfortunes of others. Complacency: a feeling of smug or uncritical
Stream: a small, narrow river. satisfaction with oneself or one's achievements.
Deport: expel (a foreigner) from a country,
8. Answer: E typically on the grounds of illegal status or for
Sol. “Obligation” will fill in the blank to make the having committed a crime.
paragraph meaningful and complete coherently. Reticence: the quality of being reticent; reserve.
Lax: not sufficiently strict, severe, or careful. Scrutinize:examine or inspect closely and
Grim: depressing or worrying to consider. thoroughly.
Underscore: underline (something): emphasize Suspension: the action of suspending someone
Suspicion: a feeling or thought that something is or something or the condition of being
possible, likely, or true. suspended.

11. Answer: C

Click Here For Bundle PDF Course | support@guidely.in Page 7 of 9


SBI Clerk & RRB PO Mains PDF Course 2023
ENGLISH Day - 49

Here, the first part of the column one should be Here, the first part of column one should be
matched with the third part of column two as matched with the second part of column two as
both connects contextually which is based on both are connected by the context of strike held
IMD and its expectations and second part of by JSSU in Jharkhand and the second part of
column one should be matched with the first part column one should be matched with the third
of column two as both are connected as they are part of column two as both are connected by the
based on the areas with amount of rainfall context of bandh due to 60:40 ration and the
received and final match should be the third part third part of column one should be matched with
of the first column and the second part of the the first part of column two as both are
second column which is connected as it shows connected by the ruling made by High court in
the mercury readings in the area. So, option C is 2021 against the rule of 60:40 rule by Hemant
the correct answer. Soren government. So, option B is the correct
answer.
12. Answer: A
Here, the all part of column one should be 15. Answer: D
matched with the column in front of them. So, Here, the first part of the column one should be
option A is the correct answer. matched with the first part of column two as both
are conncted by the context of torchlight
13. Answer: B procession carried out by the students and the
Here, the first part of column one should be second part of the column two should be
matched with the second part of the column two matched with the third part of the column two as
as both are connected by the context of high both are connected by the past example of
wave observed at Tithal beach and the second protest made by students against the recruitment
part of the first column should be matched with policy and the final part of the column one
the third part of column two as both are should be matched with the second part of
connected by the context of the yellow alert in column two as both are connected by ordinance
few areas of kerala and the third part of column released by the centre to reclaim executive over
one should be matched with the first part of the service related matters. So, option D is the
column two as few information is given about correct answer.
Biparjoy which is a cyclone. So, option B is the
correct answer. 16. Answer: E
Statements other than the last one are talking
14. Answer: B about the border land dispute between the states

Click Here For Bundle PDF Course | support@guidely.in Page 8 of 9


SBI Clerk & RRB PO Mains PDF Course 2023
ENGLISH Day - 49

of Assam and Meghalaya and all the statements 18. Answer: C


provide details w.r.t that. The sentences discuss the five-member team
The last sentence however looks open-ended. It and the members and their report and findings
says ‘apart from Meghalaya’ which is unclear on the matter. However, the third sentence talks
that in what sense. about the same incident but with a different
So, the best answer is option (e). angle to it that now the SC has ordered a CBI
enquiry into the matter.
17. Answer: B So, (c) carries a different theme.
Only statement (b) is general in nature where it
can’t be understood about which occasion is the 19. Answer: D
being present of all members being discussed. The sentences are all discussing the football
However, all the other sentences are related to World Cup going on in Qatar but one of these
the visits being made to India by various sentences is talking about a different tournament
countries’ representatives with the Russia- which is the Federation Cup Athletics. So, this
Ukraine crisis in the backdrop. sentence is the odd one out.
Hence, the correct answer is option (b). Therefore, the best is mark option (d) as the
answer.

Click Here For Bundle PDF Course | support@guidely.in Page 9 of 9


SBI Clerk & RRB PO Mains PDF Course 2023
Reasoning Ability Day – 50 (Eng)

Reasoning Ability
Directions (1-5): Study the following information P©Q means truck P is at point Q and moves
carefully and answer the given questions straight through the junction to reach a certain
There are four trucks A, B, C, and D. These point
trucks travel from one point to another point. P@Q means truck Q is at point P and moves to
Also, there are four points L, M, N, and O the right from the junction to reach a certain point
connected through a common junction. 1. O&B; C*M. The distance between the junction
The direction of these points with respect to the and all the given points is equal and the distance
junction is as follows: is 12m. If truck C travels 5m to the west from its
L is in the north direction of the junction final position and truck B travels 5m to the east
M is in the west direction of the junction from its final position, then what is the shortest
N is in the east direction of the junction distance between truck C and truck B?
O is in the south direction of the junction a) 13m
Note: While traveling from one point to another b) 18m
all the trucks must pass through the junction. c) 25m
The following codes are given to denote the d) 22m
direction of movement of the trucks. e) 26m
P$Q means truck Q is at a certain point and
moves straight through the junction to reach 2. If M&A#N$C, then what is the direction of the
point P final position of truck C with respect to the initial
P&Q means truck Q is at a certain point and position of truck A?
moves to the right from the junction to reach a) Southwest
point P b) Northeast
P%Q means truck P is at a certain point and c) Southeast
moves to the left from the junction to reach point d) Northwest
Q e) None of these
P#Q means truck P is at a certain point and
moves straight through the junction to reach 3. O@B__L; D®M, which of the following option
point Q should come in the blank to satisfy the final
P®Q means truck P is at a certain point and position of B is northeast of the final position of
moves to the right from the junction to reach D?
point Q a) #
P*Q means truck P is at point Q and moves to b) ®
the left from the junction to reach a certain point c) $

Click Here For Bundle PDF Course | support@guidely.in Page 1 of 11


SBI Clerk & RRB PO Mains PDF Course 2023
Reasoning Ability Day – 50 (Eng)

d) % rt bn sa cd
e) * III. Wild soft pen – yj ol VI. India freedom – bn
pr km
4. Which of the following options denotes the Lion made bar – jk df lo Vast is pure – lu gh er
initial position of C is northwest of the final a) I and IV are sufficient
position of A? b) II and VI are sufficient
a)N@A©M&C c) III and V are sufficient
b) M&A#N$C d) II and IV are sufficient
c) O$C®N&A e) I and VI are sufficient
d) Both a and c
e) None of these 7. Which of the following statements are required
to find the code for the phrase “Matric education
5. If ‘L$B%N&D’, then trucks B and D can reach best”?
which among the following points respectively? Column 1 Column 2
a) Point N and Point O I. Come let’s go – qr ed IV. Soon do it – po cl
b) Only point L vc xz
c) Point M and Point N Education is all – lk bm Educate others – es rf
d) Only point N mv
e) Point M and Point O II. Best of luck – hj tb fs V. She love matric – gf
Study matric – gh jm jm uo
Directions (6-10): Study the following information Latest best education –
carefully and answer the given questions nm lk tb
6. Which of the following statements are required III. Latest result fast – VI. Latest result out-
to find the code for the phrase “Awesome wild pd vb nm nm pd jd
King”? Matric fool fast – vb jm Run fast – vb tv
Column 1 Column 2 tf
I. Food is awesome – ui IV. Wild Lion India – bn a) I and IV are sufficient
gh rt jk op b) II and VI are sufficient
King do wonders – ew Lion is King – jk gh ew c) III and V are sufficient
mv cx d) II and IV are sufficient
II. Peacock is beautiful V. King is powerful – e) I and VI are sufficient
– ab gh yu ew nm gh
India is Awesome – gh Most keep secret – qw

Click Here For Bundle PDF Course | support@guidely.in Page 2 of 11


SBI Clerk & RRB PO Mains PDF Course 2023
Reasoning Ability Day – 50 (Eng)

8. Which of the following statements are required Sweet corn – la sa Rabbit be run – sd gd
to find the code for the phrase “Pets Love Dog”? lp
Column 1 Column 2 a) I and IV are sufficient
I. Love is Blind – bm uy IV. Flowers are lovable b) II and VI are sufficient
ti – jp qw ag c) III and V are sufficient
Pets are Lovable – gd Dog is Blind – zx bm ti d) II and IV are sufficient
jp qw e) None of these
II. Run that dog – zx cx V. I hate pets – gd yu
pd bv 10. Which of the following statements are
Dogs are beautiful – zx Hot foods – lk as required to find the code for the phrase “Most
bm jp expected event”?
III. God is love – ti uy VI. Make that hot – al lk Column 1 Column 2
qh za I. We Best most – ko cr IV. Are you done – qp
Tap water – tm bp Dogs are pets – zx gd pd wo ei
jp Best is expected –pd ko Work is done – ei pd
a) I and IV are sufficient tb zm
b) II and VI are sufficient II. Expect the V. Work load – gh fn
c) III and V are sufficient unexpected – xn dm sk Dream your future – al
d) II and V are sufficient Best is coming – ko hl av wj
e) I and VI are sufficient pd
III. Pay later – vb cm VI. Event is soft - um
9. Which of the following statements are required Most is well – pd crzp ko ef
to find the code for the phrase “Be that Kind”? Most men soft –ef cr
Column 1 Column 2 po
I. Rabbit is Kind – vb IV. Okay that Kind – ko a) I and IV are sufficient
gd ty cv ty b) II and VI are sufficient
Be that beautiful – cb Lost world all – tm qw c) III and V are sufficient
sd cv mu d) II and IV are sufficient
II. Kind that enough – V. Nose cut – shjt e) I and VI are sufficient
cv ty pk Bring that beautiful –bg
Be world – sd tm cy cb Directions (11-15): Study the following
III. God is great – vbrp VI. Angel is Beautiful – information carefully and answer the given
gm vb cb hj questions

Click Here For Bundle PDF Course | support@guidely.in Page 3 of 11


SBI Clerk & RRB PO Mains PDF Course 2023
Reasoning Ability Day – 50 (Eng)

Eight persons- A, B, C, D, E, F, G, and H wrote a b) The one who likes Jasmine


novel in different months- January, March, April, c) The one who wrote immediately after E
June, July, September, November, and d) C
December of the same year. Each of them likes e) D
different flowers Lily, Lotus, Jasmine, Sunflower,
Hibiscus, Rose, Marigold, and Dahlia. 13. Four of the following five are alike in a certain
The one who likes Marigold wrote the novel five way based on the given arrangement and thus
months before G, who wrote the novel in the form a group. Which one of the following does
month with an even number of days. No months not belong to the group?
are between the months in which G and C wrote a) E
the novel. Two persons wrote between C and the b) H
one who likes Rose. Nobody wrote the novel c) D
between the one who likes Rose and B, who d) F
likes Hibiscus. The one who likes Sunflower and e) C
B wrote the novel at a gap of two months. A
neither wrote the novel in the month with an odd 14. If all the persons are arranged in alphabetical
number of days nor likes Marigold. A wrote the order from January to December, then how many
novel immediately after H, who likes Dahlia. persons remain in the same position?
Neither A nor G likes Lily. F neither likes Lily nor a) One
Marigold. The one who likes Jasmine flower b) Two
wrote three persons after D. C and the one who c) Three
likes lotus did not write the novel in consecutive d) Four
months. e) None
11. How many persons wrote the novel between
E and the one who likes Marigold? 15. If A is related to the one who likes Rose and
a) One D is related to the one who likes Hibiscus in a
b) Two certain way, then in the same way who among
c) As many persons wrote between F and C the following person is related to the one who
d) As many persons wrote before H likes Lotus?
e) Both b and c a) A
b) B
12. Who among the following person wrote the c) F
novel in November month? d) E
a) The one who likes Lotus e) C

Click Here For Bundle PDF Course | support@guidely.in Page 4 of 11


SBI Clerk & RRB PO Mains PDF Course 2023
Reasoning Ability Day – 50 (Eng)

Directions (16-20): Study the following d) N


information carefully and answer the below e) Q
questions
Eight persons- J, K, L, M, N, O, P, and Q are 17. How many persons sit between K and the
sitting at the circular table facing the centre. Each one who has Cello Pen?
of them has different brands of pen viz., Parker, a) One
Reynolds, Rorito, Classmate, Montex, Linc, b) Two
Luxor, and Cello. All the information is not c) Three
necessarily in the same order. d) Four
Note: The consecutively alphabetically named e) Both b and d
persons are not sitting adjacent to each other.
Not more than two persons sit between L and N, 18. What is the position of Q with respect to the
when counted from the right of L. The one who one who has Rorito pen?
has Reynolds pen sits second to the right of N. M a) Second to the right
and the one who has Reynolds pen are sitting b) Second to the left
adjacent to each other. Only one person sits c) Third to the right
between M and O who has Luxor Pen. The one d) Third to the left
who has Rorito pen is an immediate neighbour of e) Immediate right
the one who sits opposite to O. K who has
Montex pen sits third to the right of P. The 19. Who among the following person has Linc
number of persons sitting between P and Q is Pen?
two less than the number of persons sitting a) The one who sits immediate right of O
between J and the one who has Classmate pen, b) M
when counted to the left of P and right of J. c) The one who sits opposite to N
Neither Q nor M has Cello pen. Q neither has d) J
Parker pen nor sits opposite to the one who has e) The one who sits immediate left of M
Parker pen.
16. If the position of L and O are interchanged in 20. The number of persons from the right of J to
the same way the position of J and N are ___ is the same as the number of persons from
interchanged, then who among the following the left of N to ____.
person sits third to the left of L? a) P, O
a) P b) L, K
b) K c) K, Q
c) M d) P, M

Click Here For Bundle PDF Course | support@guidely.in Page 5 of 11


SBI Clerk & RRB PO Mains PDF Course 2023
Reasoning Ability Day – 50 (Eng)

e) None of these
Click Here to Get the Detailed Video Solution for the above given Questions
Or Scan the QR Code to Get the Detailed Video Solutions

Answer Key with Explanation

Directions (1-5):
1. Answer: E

2. Answer: C
3. Answer: B

Click Here For Bundle PDF Course | support@guidely.in Page 6 of 11


SBI Clerk & RRB PO Mains PDF Course 2023
Reasoning Ability Day - 50 (Eng)

Directions (6-10):
6. Answer: D
By combining II and IV we get

7. Answer: C
By combining III and V, we get

4. Answer: C

8. Answer: A
By combining I and IV, we get

5. Answer: D
9. Answer: D
By combining II and IV we get,

10. Answer: E
By combining I and VI we get,

Click Here For Bundle PDF Course | support@guidely.in Page 7 of 11


SBI Clerk & RRB PO Mains PDF Course 2023
Reasoning Ability Day - 50 (Eng)

From the above conditions, we have four


possibilities,

Directions (11-15):
11. Answer: E
12. Answer: B
13. Answer: C
(All the persons wrote the novel in the month
with an odd number of days except option c)
14. Answer: B
15. Answer: D
Final arrangement:
Again we have,
 Nobody wrote the novel between the one
who likes Rose and B, who likes Hibiscus.
 The one who likes Sunflower and B wrote
with a gap of two months.
 A neither wrote the novel in the month
with an odd number of days nor likes
Marigold.
 A wrote the novel immediately after H,
who likes Dahlia.
We have, We cannot place the one who likes Sunflower in
 The one who likes Marigold wrote the case 4. Hence it is eliminated.
novel five months before G, who wrote We cannot place A and H in case 3. Hence it is
the novel in the month with an even eliminated.
number of days.
 No months are between the months in
which G and C wrote the novel. Two
persons wrote between C and the one
who likes Rose.

Click Here For Bundle PDF Course | support@guidely.in Page 8 of 11


SBI Clerk & RRB PO Mains PDF Course 2023
Reasoning Ability Day - 50 (Eng)

Directions (16-20):
16. Answer: D
17. Answer: E
18. Answer: B
19. Answer: A
20. Answer: C
Final arrangement:

Again we have,
 Neither A nor G likes Lily.
 F neither likes Lily nor Marigold.
 The one who likes Jasmine flower wrote
three persons after D.
 E and the one who likes lotus did not write
the novel in consecutive months.
E and the one who likes Lotus went in adjacent
months in case 1. Hence it is eliminated. Thus,
case 2 gives the final arrangement.
We have,
 Not more than two persons sit between L
and N, when counted from the right of L.
 The one who has Reynolds pen sits
second to the right of N.

Click Here For Bundle PDF Course | support@guidely.in Page 9 of 11


SBI Clerk & RRB PO Mains PDF Course 2023
Reasoning Ability Day - 50 (Eng)

 M and the one who has Reynolds pen are  Only one person sits between M and O
sitting adjacent to each other. who has Luxor Pen.
From the above conditions, we have three  The one who has Rorito pen is an
possibilities, immediate neighbour of the one who sits
opposite to O.
 K who has Montex pen sits third to the
right of P.
We cannot place O in case 3. Hence it is
eliminated.

Again we have,
 The number of persons sitting between P
and Q is two less than the number of
persons sitting between J and the one
who has Classmate pen, when counted to
Again we have, the left of P and right of J.
 Neither Q nor M has Cello pen.

Click Here For Bundle PDF Course | support@guidely.in Page 10 of 11


SBI Clerk & RRB PO Mains PDF Course 2023
Reasoning Ability Day - 50 (Eng)

 Q neither has Parker pen nor sits


opposite to the one who has Parker pen.
Q does not sit opposite to the one who has
Parker pen. Hence case 2 is eliminated. Thus,
case 1 gives the final arrangement.

Click Here For Bundle PDF Course | support@guidely.in Page 11 of 11


SBI Clerk & RRB PO Mains PDF Course 2023
Quantitative Aptitude Day – 50 (Eng)

Quantitative Aptitude

Directions (01 - 05): Study the following information carefully and answer the questions given below.
The below funnel shows the number of visitors in two parks in five months.

Note- The number of visitors in August in park A is 12000. The number of visitors in the months of
October and November is 16000 and 10000 respectively in park A.
1)Find the difference between the total number b) 19:11
of visitors in park A in September and December c) 17:11
together and the total number of visitors in park d) 14:11
B in August and October together? e) 19:11
a) 1
b) 2 3) If the number of visitors increases by 20% in
c) 0 park A in August from the previous month. If the
d) 500 total number of visitors in July in both parks A &
e) 1000 B together is 25400 then find the increased
percentage of visitors in park B in August?
2) Find the ratio between the total number of a) 5.64%
visitors in September in both parks together and b) 3.89%
the total number of visitors in November in both c) 8.35%
parks together? d) 7.68%
a) 11:12 e) None of these

Click Here For Bundle PDF Course | support@guidely.in Page 1 of 8


SBI Clerk & RRB PO Mains PDF Course 2023
Quantitative Aptitude Day - 50 (Eng)

difference between the volume of cylinders 1 and


4) The ratio of male and female visitors in park A 4?
in December is 3:2 and the ratio of male and a) 11253
female visitors in December in park B is 5:3. then b) 12453
Find the total number of female visitors in c) 10296
December? d) 12380
a) 8550 e) 12357
b) 8450
c) 6500 7) The ratio of the radius of cylinder2 and the
d) 7520 sphere is 2:3. Find the total surface area of
e) None of these cylinder2 and the sphere?
a) 7452
5) In which month difference between the visitors b) 6842
from both parks is more? c) 9852
a) Oct d) 7832
b) Nov e) 9684
c) Dec
d) Aug 8) Find the difference between thecurved surface
e) None of these area between cylinder 1 and cylinder 2?
a) 458
Directions (06 - 10): Study the following b) 528
information carefully and answer the questions c) 658
given below. d) 985
The volume of cylinder 1 is 11088 m. The total e) None of these
surface area of cylinder 2 is 2288 m and the
surface area of cylinder 3 is 1760. The ratio of 9) The ratio of the height of a cone and cylinder 3
the height of cylinder 1 and cylinder 2 is 3:2. The is 3:2 and the radius of the cone is 4 m more
radius of cylinder 1 is 14. The radius of cylinder 2 than cylinder 1 then find the volume of the cone?
is 30% less than the radius of cylinder 3. The a) 7128
height of cylinder 3 is 16.67% more than the b) 7542
height of cylinder 2. c) 7865
6) The ratio of the radius of cylinder 2 and d) 8652
cylinder 4 is 7:9. Height of cylinder 4 is 3 m more e) None of these
than the height of cylinder 1 then find the

Click Here For Bundle PDF Course | support@guidely.in Page 2 of 8


SBI Clerk & RRB PO Mains PDF Course 2023
Quantitative Aptitude Day - 50 (Eng)

10) Find the difference between the volume of c) 12360


cylinder 2 and cylinder 3? d) 10352
a) 10208 e) None of these
b) 11254

Directions (11 - 15): Study the following information carefully and answer the questions given below.
The given table shows the total population, the Ratio of literate male and illiterate male population out
total male population, percentage of illiterate females out of the total female population of five different
villages. The bar graphs show the percentage of males out of the total population of the village.

11) Find the difference between the number of the number of male illiterates in villages C and D
female literates in villages A and B together and together?

Click Here For Bundle PDF Course | support@guidely.in Page 3 of 8


SBI Clerk & RRB PO Mains PDF Course 2023
Quantitative Aptitude Day - 50 (Eng)

a) 4250 15) Find the difference between the total literate


b) 4750 males and total illiterate females in all villages
c) 4460 together?
d) 6250 a) 16256
e) 6230 b) 16526
c) 15276
12) Out of total literate males, 60% are d) 16286
graduates in village E. The ratio of the number of e) None of these
female graduates and female non-graduates out
of total literate females is 5:3. Find the total 16) An item is sold for Rs.8000 then their profit is
graduates in village E. X%. When an item is sold at (X+15)% profit then
a) 3254 the ratio of the selling price of two cases is 1.12.
b) 3650 Find the selling price when the item is sold at
c) 3846 15% profit?
d) 3892 a) Rs.7520
e) 3648 b) Rs.8520
c) Rs.7360
13) The ratio of the number of males in villages A d) Rs.4560
and F is 2:3. Male in village F is 20% more than e) Rs.6520
female, than find female in village F?
a) 9210 17) A Car starts its journey at 9 A.M. and
b) 9000 reachesitsdestination after 6 hours. Another car
c) 9450 starts its journey after 1st car and reaches its
d) 9540 destination before 1 hour of 1st car. If the
e) None of these distance is 240 km and the speed of the second
car is 50% more than the speed of the first car
14) M= Total illiterate people in village C, then find the time when the second car starts his
N=Total literate people in village D. Find the journey?
value of M+N=? a) 9.45 A.M
a) 17852 b) 11 AM
b) 19852 c) 9.30 AM
c) 16852 d) 10 A.M
d) 14822 e) 10.30 AM
e) None of these

Click Here For Bundle PDF Course | support@guidely.in Page 4 of 8


SBI Clerk & RRB PO Mains PDF Course 2023
Quantitative Aptitude Day - 50 (Eng)

18) The sum of two numbers is 54 and the sum kg Assam Tea and sold at a 20% profit. Find the
of the reciprocal of the number is 3/40. If we value of x if Mixture Tea is sold at Rs.5700?
interchange the digit of the lowest then we get M. a) 15
D is the sum of M and twice of the highest b) 14
number, and E is the difference between the c) 25
square of the sum of two numbers and the d) 20
square of the difference of two numbers then find e) None of these
the value of D+E?
a) 2452 20) A class consists of 5 girls and 7 boys. 5
b) 2982 students are randomly chosen from the class.
c) 2652 Find the probability that at least 3 boys are
d) 2872 there?
e) None of these a) 546/792
b) 852/792
19) The rate of Darjeeling tea is Rs.6000 per kg c) 654/792
and the Rate of Assam Tea is Rs.4000 per kg. A d) 777/792
shopkeeper mixed x kg Darjeeling tea with x+10 e) None of these
Click Here to Get the Detailed Video Solution for the above given Questions
Or Scan the QR Code to Get the Detailed Video Solutions

Answer Key with Explanation

Directions (01 - 05): Or, 4X+Z-21Y=10000, Z=10000-


So, X=12000, 48000+42000=4000
Or, 21Y-2X-Y=16000, 10Y-X=8000, So, the number of visitors in park A in August, till
Y=20000/10=2000 September, till October, till November, and till
December is

Click Here For Bundle PDF Course | support@guidely.in Page 5 of 8


SBI Clerk & RRB PO Mains PDF Course 2023
Quantitative Aptitude Day – 50 (Eng)

=12000,2*12000+2000=26000,21*2000=42000, The difference in October is =16000-


4*12000+4000=52000,5*12000+2*2000=64000 10000=6000
respectively. so, The difference between the visitors from both
number of visitors in park A in August, parks in october month has the highest.
September, October, November, and December
is Directions (06 - 10):
=12000,26000-12000=14000,42000- For cylinder 1 height is h.
26000=16000,52000-42000= 10000, 64000- So, 22*14*14*h/7=11088
52000=12000 respectively. Or, h=11088*7/(22*14*14)=18
Similarly, we can calculate the values for park B. The height of cylinder 2 is =18*2/3=12, the
1) Answer: C radius is r.
Required difference = [14000+12000]- So, 2*22/7*r*(r+12)=2288
[16000+10000]=0 Or, r2+12r=2288*7/44=364, r2+12r-364=0.
So, r=-26,14, r=14.
2) Answer: D The radius of cylinder 3 is 14*100/70=20
Required ratio is = [14000+14000] : The height of cylinder 3 is 12*116.67/100=14
[10000+12000] 6) Answer: C
=28000:22000=14:11 The radius of cylinder 4 is 14*9/7=18
Height of cylinder 4 is 18+3=21
3) Answer: B The volume of the cylinder is =
The number of visitors to park A in July is 22/7*18*18*21=21384
=12000*100/120=10000 So, the difference is =21384-11088=10296
The number of visitors in the park in August is =
25400-10000=15400 7) Answer: D
So, increase percentage is The radius of the sphere is 14*3/2=21
=[600/15400]*100=3.89% The total surface area of the sphere is =
4*22/7*21*21=5544
4) Answer: A The total surface area of the sphere and
The number of female visitors is = cylinder2 is = 5544+2288=7832
12000*2/5+10000*3/8
=4800+3750=8550 8) Answer: B
Curved surface area cylinder1 = 2*[22/7]
5) Answer: A *14*18=1584

Click Here For Bundle PDF Course | support@guidely.in Page 6 of 8


SBI Clerk & RRB PO Mains PDF Course 2023
Quantitative Aptitude Day – 50 (Eng)

Curved surface area cylinder2 =2*[22/7] Total graduates in village E is =


*14*12=1056 3920*60/100+2464*5/8=3892
Required difference = 1584-1056=528
13) Answer: B
9) Answer: A Total male in F is = 7200*3/2=10800
The height of the cone is 14*3/2=21 The number of females in F is
The radius of the cone is 14+4=18 10800*100/120=9000
So, volume is = 1/3*[22/7] *[18*18] *21=7128
14) Answer: E
10) Answer: A M= Total illiterate people in village
Volume of cylinder2 is = 22/7*[14*14] *12=7392 C=1200+2520=3720, N=Total literate people in
Volume of cylinder3 is = 22/7*[20*20] *14=17600 village D=7020+6912=13932
Required difference = [17600-7392] =10208 So, M+N=13932+3720=17652

Directions (11 - 15): 15) Answer: E


The total number of populations in village A is Required difference
12000 =[4500+4000+4400+7020+3920]-
The male population is 12000*60/100=7200 [1200+1100+2520+1728+1056]=16236
The female population is 12000*40/100=4800
Male literate is 7200*5/8=4500 16) Answer: C
Male illiterate is 7200-4500=2700 so, selling price of 2nd case is
Female illiterate is 4800*25/100=1200 8000*112/100=8960
Female literate is 4800-1200=3600 so, the difference in selling price is 8960-
Similarly, we can calculate the other village also. 8000=960
difference of profit percentage is [X+15]%-
X%=15%
so, 15%=960, 100%=[960/15]*100=6400
so, the required selling price is
=6400*115/100=Rs.7360
11) Answer: C
Required difference = [3600+4400]- 17) Answer: D
[2340+1200]=4460 Speed of 1st car is 240/6=40 km/r
Speed of 2nd car is 40*150/100=60 km/hr
12) Answer: D

Click Here For Bundle PDF Course | support@guidely.in Page 7 of 8


SBI Clerk & RRB PO Mains PDF Course 2023
Quantitative Aptitude Day – 50 (Eng)

So, the time taken by the second car is 240/60= 19) Answer: A
4 hours. Total cost price =
So, the 2nd car starts his journey 1 hour after the [6000x+4000*(x+10)]=10000x+40000
1st car i.e., 10 A.M Total selling price =
5700*[x+x+10]=57000+11400x
18) Answer: B So, [10000x+40000]*120/100=57000+11400x
Let the number be a and b and a>b Or, 1200000x-1140000x=5700000-4800000
So, a+b=54, and 1/a + 1/b =3/40 Or, x=900000/60000=15
So, ab=54*40/3=720
Or, a(54-a)=720 20) Answer: A
Or, a2-54a+720=0, a=30 so, b=24 Required probability is
Now, M=42, D=42+2*30=102 =[(7C3*5C2)/12C5+(7C4*5C1)/12C5+7C5/12C5]
E=[30+24]2-[30-24]2=4*30*24=2880 =35*10/792+35*5/792+21/792=350/792+175/79
So, E+D =2880+102=2982 2+21/792=546/792

Click Here For Bundle PDF Course | support@guidely.in Page 8 of 8


SBI Clerk & RRB PO Mains PDF Course 2023
ENGLISH Day - 50

English Language

Directions (01-05): The following questions given 1. Choose the option that gives the best
below carry sentences that have been taken rearrangement for the fourth sentence after
from a paragraph. These sentences have been rearrangement.
jumbled within themselves and need to be A.ABDC
rearranged in order to fit into a coherent B.BDCA
paragraph. C.DBCA
A. multiple airports, not to forget gargantuan D.ACBD
statues and grand temples (A)/ which was E.CDAB
constructed in about five years, India does
boast (B)/ now of many modern expressways, 2. Which of the following would be the opening
high-speed trains and (C)/ While not all of them sentence after rearrangement?
may have come to fruition as quickly as the A.A
expressway, (D). B.E
B. As a young minister for public works in (A)/ the C.C
project conceived by an earlier D.D
government (B)/ Maharashtra, Mr Gadkari had E.B
set in motion (C)/ the first Shiv Sena-BJP
government in (D). 3. Which of the given sequences best arranges
C. take its place in the ranks (A)/ while these the third sentence in the paragraph?
fancy projects promise to help India (B)/ a rosy A.CBDA
future, reality becomes a speck in the B.BDAC
distance (C)/ of the developed economies and C. No rearrangement needed
focus our vision on (D). D.ABCD
D. modern intercity highway, a showpiece that E.DBCA
came to (A)/ Pune became fully operational as
India’s first (B)/ Twenty years ago, the 4. What is the correct sequence of the whole
expressway connecting Mumbai and (C)/ be paragraph from the given options?
associated in the public mind with BJP leader A.DBCAE
Nitin Gadkari (D). B.AEDCB
E. Since then, the country has seen a flurry of C.BECAD
big-ticket projects, each grander than the last. D.DBEAC
E.BEDCA

Click Here For Bundle PDF Course | support@guidely.in Page 1 of 13


SBI Clerk & RRB PO Mains PDF Course 2023
ENGLISH Day - 50

5. Which of the following is the last sentence in world for centuries, embracing people of different
the sequence after rearrangement? origins, languages, religions and cultures is not a
A.B difficult task. However, the one thing that
B.E undoubtedly made India what it is today is the
C.C people of this great nation — their resilience,
D.A their aspirations and their unconditional love for
E.D the country.
The 75th anniversary of Independence is the
Directions (6-12): Read the passage given below opportune occasion to commemorate the great
carefully and answer the following questions. journey of India and its people. It is also an
More than seven decades ago, when India opportunity to renew our pledge of patriotism. A
began its journey as a newly-born democracy host of initiatives have been undertaken to
based on universal adult suffrage, naysayers celebrate the Azadi Ka Amrit Mahotsav and one
predicted its premature demise. A country of among them is the Har Ghar Tiranga campaign.
people distinguished more by their differences Every Indian is encouraged to hoist the tricolor at
than commonalities had a lot going against it. their home from August 13-15 to mark 75 years
India had nothing in common; its birth was of independence.
preceded by unprecedented bloodshed and The event has significance beyond the
religious animosity, a fact that immediate excitement it has generated. A nation
further befuddled theorists and political is primarily made of its people, who do not know
scientists. However, India has not only survived each other personally. Yet they find that they are
the initial forebodings but also thrived as a connected through an invisible sense of
vibrant and stable democratic nation, while its familiarity in the face of all the visible
neighbors have notwithstanding their shared dissimilarities. The invisible and abstract
commonalities. One may wonder how India has familiarity or connectedness takes a concrete
managed to achieve the incredible feat. form in national symbols and the national
Though there is no single answer to this anthem. The foremost among these symbols is
question, I believe there is something distinctly the national flag. The national flag covers all the
Indian — about the land and its people — that lines that separate one from the other and holds
defies any explanation. For a land that believes all Indians together.
and During British rule, the dream of raising our own
practises the philosophy of “vasudhaiva flag on our soil inspired people. We have come a
kutumbakam”, a land that whole-heartedly long way since then. However, for today’s fast-
welcomed people from different parts of the paced society, an event that took place more

Click Here For Bundle PDF Course | support@guidely.in Page 2 of 13


SBI Clerk & RRB PO Mains PDF Course 2023
ENGLISH Day - 50

than seven decades ago may appear to be a I. To show every Indian the struggle to make the
thing of ______________ . Today, young people country independent.
may find Gandhiji, Netaji Subhas Chandra Bose II. To celebrate the journey of India and its
or Bhagat Singh as characters of a distant past people.
cloistered in history books. If they do so, it is not III. To renew the pledge of nationalism.
their fault. We are at fault. We must ensure that A. Only II
the people of India, especially the young minds, B. Only III
learn about the glorious past to feel proud of C. Only I
their nation. D. Both I and II
6. What incredible feat India achieved as per the E. Both II and III
author of the passage?
I. India becoming a stable democratic nation. 9. What is the main reason for bringing “Har
II. India became an independent nation with the Ghar Tiranga” campaign?
support of many freedom fighters while A.The initiative aims to encourage people to
sacrificing their lives. bring our flag into our homes and hoist it to mark
III. India not only became independent but also the 75th year of India’s Independence.
developed its economy. B.The idea behind the initiative is to invoke the
A. Only I feeling of patriotism in the hearts of the people
B. Only II and to promote awareness about the Indian
C. Only III National Flag.
D. Both I and II C.Har Ghar Tiranga is aimed to redefine our
E. Both I and III relationship with our flag, and, in effect, with our
nation and its past.
7. What is the meaning of “vasudhaiva D.All are correct
kutumbakam”? E.Both A and B
A. Guest is God
B. Unity in diversity 10. What do you think the author said that “we
C. Let the entire world be happy are at fault” at the end of the passage?
D. The World Is One Family A.Because we failed to narrate the story of how
E. None of the above India won the battle against the mightiest colonial
power with non-violence.
8. Why, according to the author, is the 75th B.Because we failed to show the importance of
anniversary of Independence an auspicious our soil to the younger generation.
occasion ?

Click Here For Bundle PDF Course | support@guidely.in Page 3 of 13


SBI Clerk & RRB PO Mains PDF Course 2023
ENGLISH Day - 50

C.Because we failed to teach the youth about B. Because the cumbersome package weighed
sacrificing their own lives to protect our mother over three hundred pounds, it took two men to
land and its kids. move it.
D.None is correct C. Squad members, already decked out
E.Both B and C in cumbersome chemical suits, put on masks and
rubber gloves.
11. Which of the following words is an antonym D. The organization has promised to
of “befuddled”? eliminate cumbersome and unnecessary
A.Baffled bureaucracy.
B.Enlightened E. You are under no cumbersome to pay
C.Perplexed immediately.
D.Puzzled
E.Bewildered 14. Fastidious
A. It thrives in light sandy soil, and is
12. Which of the following words fit aptly in the not fastidious, coming from sandy, gravelly
blank given in the passage? places.
I. antiquity B. The carriages are shabby, the management
II. novelty is sloppy and the trains are slow.
III. obsolescence C. I'm usually so meticulous, and check and re-
A.Only II check all the knots I tie.
B.Only III D. The finicky student gets upset when her
C.Only I teacher switches up the organized schedule.
D.Both I and II E. The designer is a perfectionist and that shows
E.Both II and III in his eyewear designs.

Directions (13-17): The following questions have 15. Impasse


a word that has been given in bold using which A. Parents of teens frequently find themselves at
the sentences have been formed. You must read an impasse over a variety of issues.
the same carefully and identify the sentence B. The little car slowed to a standstill, but Katie
where the given word has not been used continued spinning the tires.
correctly to mark the same as your answer. C. As the truck came to a halt in front of the
13. Cumbersome house, she stared up at it in awe.
A. This is going to be a lot D. The use of the battleship is to checkmate the
more cumbersome than my usual costume. activities of the enemy's battleship.

Click Here For Bundle PDF Course | support@guidely.in Page 4 of 13


SBI Clerk & RRB PO Mains PDF Course 2023
ENGLISH Day - 50

E. All the given words are correctly used by alphabets, and are given below the paragraph
in a random order. Rearrange these words in an
16. Rampage appropriate order so that they fit the blanks and
A. Indian elephants have a tendency to raid illicit give out grammatically meaningful paragraphs.
stills and go on the rampage through villages. 18. The crisis in Ukraine is ________the power
B. In 1975, he was asked to write the score for a calculus in Europe and in the rest of the world.
New York story about a man who goes on a It’s the first major ________war that Europe is
killing rampage. experiencing after the Second World War.
C. The rampage between him and his wife grew Currently, what seems to be more disturbing is
closer and closer and he daily discovered fresh the determination of two Nordic nations —
spiritual treasures in her. Finland and Sweden — to join the North Atlantic
D. The Japanese Self Defense Force used the Treaty Organization (NATO). After years of
Super X3 to stop Godzilla's rampage before he _______in Scandinavian countries, the sudden
can reach the nuclear power plant to refuel. decision of the two neighbours of the Russian
E. Soldiers went on a rampage, pillaging stores Federation to join the NATO is heightening
and shooting. tension between Moscow and the rest of the
West. _______threats from Moscow, Helsinki and
17. Uproar Stockholm have moved their applications to be a
A. Far from there being an uproar in investment, part of the world’s largest security alliance. The
the real level remains sluggish. time chosen by the two nations to be part of the
B. Cade ignored the uproar and gave proper NATO security orbit is creating _______across
attention to his meal. Europe.
C. Uproar over the art piece began after it was A) rebalancing
found to be a fake. B) neutrality
D. But news of the deaths has caused uproar on C) ripples
the set, with cast members complaining they D) despite
were not consulted. E) devastating
E. This caused a minor uproar, as my husband A. AEBDC
slipped it into a conversation about something B. BACED
else. C. CBADE
D. DABCE
Directions (18-22): In the following question, a E. ABECD
paragraph is given from which five words have
been edited out. These words are represented

Click Here For Bundle PDF Course | support@guidely.in Page 5 of 13


SBI Clerk & RRB PO Mains PDF Course 2023
ENGLISH Day - 50

19. Last month, India ______an additional USD Buddhist place-of-interest framework. There had
500 million credit line to help Sri Lanka import been_______ for decades among the Nepali
fuel as the country has been struggling to pay for citizens regarding India taking away the cream of
imports after its foreign exchange reserves Buddhist tourism circuit, while ignoring the
________sharply in recent times, causing a birthplace. Meanwhile, Lumbini became a place
devaluation of its currency and spiralling inflation. of interest for China which has been using all
On Friday, the mission here said that an Indian ______ possible to increase its strength in Nepal.
ship ______with urgent relief supplies like rice, A) amend
medicines and milk powder for the people of B) angst
crisis-hit Sri Lanka is scheduled to reach C) avenues
Colombo on Sunday. The assistance from D) integrate
people of India complements ________efforts E) hopped
undertaken by the Government of India by A. ABCDE
standing with their _______in Sri Lanka, the High B. BADCE
Commission said in a press release on Friday. C. EADBC
A) multi-pronged D. CDEAB
B) extended E. DEABC
C) plummeted
D) brethren 21. Urban greening can have a _______impact
E) laden on the lives of city dwellers. It can improve air
A. DAECB quality by cooling and purifying the air, ground-
B. BCEAD level ozone and greenhouse effect; _______the
C. ABCDE heat island effect, thereby reducing air-
D. BACDE conditioning needs, which in turn reduces the
E. CDEAB amount of fossil fuels required to generate
electricity. The ability of urban forests to
20. Prime Minister Narendra Modi, ______ across ________ atmospheric carbon dioxide and act as
to Nepal to mark Buddha Purnima celebrations in a long-term carbon sink is a crucial factor in
Lumbini earlier this week. This one-day visit addressing the effects of global warming.
marks a renewed effort to _______ties with a However, the amount of carbon sequestered by
long-trusted partner and friend in the region. trees depends upon the density of the wood,
Lumbini is the birthplace of Buddha and for many size, health and tree species. Trees ______can
years India and Nepal have not managed to also break rain velocity, reduce runoff flow rates
cooperate and_______ it within the larger and their roots_______ soils against erosion,

Click Here For Bundle PDF Course | support@guidely.in Page 6 of 13


SBI Clerk & RRB PO Mains PDF Course 2023
ENGLISH Day - 50

hence, reduce flooding and sedimentation in has been _______several times in the long
streams. history of science. Most of the scientific
A) stabilize community members are interested in
B) canopies communicating with their peer group through
C) substantial scholarly publications. Unfortunately, to digest
D) sequester this scholarly publication is beyond the capacity
E) mitigates of the common man. A good scientist is not
A. BADCE necessarily a good communicator. Nobel
B. ABCDE laureate Paul Dirac was _____shy and could not
C. CBAED face reporters. He had even thought of refusing
D. CEDBA the Nobel Prize in order to ______publicity.
E. BEDAC A) shun
B) achingly
22. _______society is built on materials of C) debated
science. But unless the common man D) burgeoning
understands what the scientists discover in our E) contemporary
laboratories, science becomes meaningless. A. EDCBA
Communicating science to the public-whether B. CBAED
through the media, museums, or outreach - is C. ABCDE
now a ______global industry. The question of D. BCEDA
who should communicate science to the public E. BACDE
Click Here to Get the Detailed Video Solution for the above given Questions
Or Scan the QR Code to Get the Detailed Video Solutions

Answer Key with Explanation

1. Answer: C

Click Here For Bundle PDF Course | support@guidely.in Page 7 of 13


SBI Clerk & RRB PO Mains PDF Course 2023
ENGLISH Day - 50

For the question above, we first need to D mentions Nitin Gadkari and B says ‘… as a
rearrange the sentences amongst themselves. young minister…’ which means DB forms a pair
Sentence B can start with A. Sentence C can and B comes second. E comes next as it says
start with B. D would start with part C and A that since then, bigger projects have been able
would start with D. If we rearrange all the to see the light of the day. BE is a link. So, DBE
sentences completely, we find the sequences is sorted.
below: We can see that E is a small sentence and is
B → ADCB, C → BADC, D → CBAD and A already arranged. So, the correct answer should
→ DBCA. E is already in the correct sequence. be option (c).
Checking the options and matching with these
sequences we find that only one sequence ie 4. Answer: D
given in (c) matches with sentence A. So, this The starting parts of the lines can help us
will be the answer and a hint that this will come understand the introductory sentence ie; D as it
fourth in the sequence. talks about the twenty-year-old Mumbai-Pune
Had the sequences been confusing, we would expressway. The other sentences can only be
have no choice left but to rearrange all the put after this one as they take the idea forward.
paragraph before marking the answer. D mentions Nitin Gadkari and B says ‘… as a
young minister…’ which means DB forms a pair
2. Answer: D and B comes second. E comes next as it says
The starting parts of the lines can help us that since then, bigger projects have been able
understand the introductory sentence ie; D as it to see the light of the day. BE is a link. So, DBE
talks about the twenty-year-old Mumbai-Pune is sorted.
expressway. The other sentences can only be This much is enough as we have just one option
put after this one as they take the idea forward. that fits which is option (d).
So, the best is to mark option (d) as the answer.
5. Answer: C
3. Answer: C The starting parts of the lines can help us
The starting parts of the lines can help us understand the introductory sentence ie; D as it
understand the introductory sentence ie; D as it talks about the twenty-year-old Mumbai-Pune
talks about the twenty-year-old Mumbai-Pune expressway. The other sentences can only be
expressway. The other sentences can only be put after this one as they take the idea forward.
put after this one as they take the idea forward. D mentions Nitin Gadkari and B says ‘… as a
young minister…’ which means DB forms a pair

Click Here For Bundle PDF Course | support@guidely.in Page 8 of 13


SBI Clerk & RRB PO Mains PDF Course 2023
ENGLISH Day - 50

and B comes second. E comes next as it says India and its people. It is also an opportunity to
that since then, bigger projects have been able renew our pledge of patriotism.” So from the
to see the light of the day. BE is a link. So, DBE above quoted lines we can easily say that both II
is sorted. and III statements are correct. Hence option E is
E says grander projects and A says many (of the correct answer.
those projects mentioned in E) might not have
been successful. This makes A the fifth sentence 9. Answer: D
and the remaining C the last. According to the passage our relationship with
So, the correct answer is option (c). the flag had been more formal and institutional
rather than personal. The Har Ghar Tiranga
6. Answer: A campaign launched by Prime Minister Narendra
Refer to the first paragraph- “However, India has Modi aims to change this forever. The initiative
not only survived the initial forebodings but also aims to encourage people to bring our flag into
thrived as a vibrant and stable democratic our homes and hoist it to mark the 75th year of
nation, while its neighbors have floundered India’s Independence. All the three options
notwithstanding their shared commonalities. One revolve around the same theme, and shows the
may wonder how India has managed to achieve aim, idea and initiative.
this incredible feat.” From the above quoted lines Hence option D is the correct answer.
we can say that option A is the correct answer.
10. Answer: A
7. Answer: D Refer to the last paragraph- “However, for
Refer to the second paragraph- “For a land that today’s fast-paced society, an event that took
believes and practises the philosophy of place more than seven decades ago may appear
“vasudhaiva kutumbakam”, a land that whole- to be a thing of antiquity. Today, young people
heartedly welcomed people from different parts may find Gandhiji, Netaji Subhas Chandra Bose
of the world for centuries”. So from the above or Bhagat Singh as characters of a distant past
quoted lines we can easily say that option D is cloistered in history books.” From the above
the correct answer. quoted line the most appropriate answer is A.

8. Answer: E 11. Answer: B


Refer to the third paragraph- “The 75th The highlighted word “befuddled” means “unable
anniversary of Independence is the opportune to think clearly; confused” and the synonyms for
occasion to commemorate the great journey of

Click Here For Bundle PDF Course | support@guidely.in Page 9 of 13


SBI Clerk & RRB PO Mains PDF Course 2023
ENGLISH Day - 50

the same are “Perplexed, Baffled, Puzzled and who are very (particular about minute
Bewildered”. details: perfectionists) and too (concerned about
Enlightened is its antonym, hence option B is the cleanliness: finicky)
correct answer. It is evidently clear that the words in (a), (c), (d)
and (e) are all synonyms of fastidious and their
12. Answer: C usages in the respective sentences are correct.
The word that fits aptly in the blank is “antiquity” The word sloppy however, used for someone
which means “the state of being something old” careless and unsystematic; excessively casual.
or “the ancient past, especially the period of This is an antonym to fastidious.
classical and other human civilizations before Therefore, the correct answer would be option
the Middle Ages”. Remaining words are inapt, (b).
hence option C is the correct answer.
15. Answer: E
13. Answer: E The word given on the top impasse is used to
Cumbersome is used to define things that are describe a situation that has reached a deadlock
slow, complicated and inefficient. Something that and is not able to move either forward or
is large and heavy to carry. backward. When things are stuck.
The first sentence talks about a heavy costume The words given in all the four sentences carry
that’s difficult to carry so that uses the word the same meanings as given above.
correctly. The second sentence also indicates at So, the correct answer should be option (e).
something too big to handle by a single person
which upholds the meaning of the given word. 16. Answer: C
The cumbersome chemical suits also go in line The word rampage means violent, move through
with the explained meaning. The fourth sentence a place in a violent and uncontrollable manner,
is correct as well. destruction.
The last sentence must use the word The first sentence describes the destruction
‘compulsion’ in order for it to be correct as done by elephants through the villages. The
otherwise it would be wrong. second sentence discusses violence carried out
Thus, the correct answer is option (e). by a man.
The fourth sentence talks about destruction done
14. Answer: B by Godzilla on his way to the nuclear plant.
The word Fastidious is used for people who pay The last sentence also uses the word in the
(careful attention to detail: meticulous), those given context. The third sentence however,

Click Here For Bundle PDF Course | support@guidely.in Page 10 of 13


SBI Clerk & RRB PO Mains PDF Course 2023
ENGLISH Day - 50

carries a positive tone as; it describes the sudden decision of the two neighbours of the
calming relationship between the husband and Russian Federation to join the NATO is
the wife for which a word like ‘harmony’ can fit heightening tension between Moscow and the
well. rest of the West. Despite threats from Moscow,
Therefore, the correct answer is option (c). Helsinki and Stockholm have moved their
applications to be a part of the world’s largest
17. Answer: A security alliance. The time chosen by the two
An Uproar means a disorder, chaos or a protest. nations to be part of the NATO security orbit is
The first sentence doesn’t discuss anything creating ripples across Europe.
related to that and hence, the given word has
incorrectly been used in the same. 19. Answer: B
The other sentences correctly use the word as The correct answer should be: BCEAD
all the sentences talk about some kind of Hint: If we check the last blank it says “India by
disturbances and chaos. standing with their _______in Sri Lanka,” and if
Thus, the correct answer is option (a). we check the options, we can see the word
“brethren” which is ‘plural of brother’ will be
18. Answer: A perfectly suitable here. The rest of the words do
Hint: If we check the last blank, it needs a verb not make any sense in the last line.
after the word “creating” so thatthe word “ripples” Last month, India extended an additional USD
fits there perfectly. Only in (a) we can find such 500 million credit line to help Sri Lanka import
an option. The words suggested by other options fuel as the country has been struggling to pay for
for the last blank are grammatically and imports after its foreign exchange
contextually incorrect. reserves plummeted sharply in recent times,
The correct answer should be: AEBDC causing a devaluation of its currency and
The crisis in Ukraine is rebalancing the power spiralling inflation. On Friday, the mission here
calculus in Europe and in the rest of the world. said that an Indian ship laden with urgent relief
It’s the first major devastating war that Europe is supplies like rice, medicines and milk powder for
experiencing after the Second World War. the people of crisis-hit Sri Lanka is scheduled to
Currently, what seems to be more disturbing is reach Colombo on Sunday. The assistance from
the determination of two Nordic nations — people of India complements multi-
Finland and Sweden — to join the North Atlantic pronged efforts undertaken by the Government
Treaty Organization (NATO). After years of India by standing with their brethren in Sri
of neutrality in Scandinavian countries, the

Click Here For Bundle PDF Course | support@guidely.in Page 11 of 13


SBI Clerk & RRB PO Mains PDF Course 2023
ENGLISH Day - 50

Lanka, the High Commission said in a press rain velocity”. The rest of the options are not
release on Friday. correct for this blank & no other option puts the
word “canopies” in this blank.
20. Answer: C Urban greening can have a substantial impact
The correct answer is: EADBC on the lives of city dwellers. It can improve air
Hint: If we check the first blank of the passage, quality by cooling and purifying the air, ground-
“Narendra Modi, ______ across to Nepal to” from level ozone and greenhouse effect; mitigates the
this we can infer that our PM is travelling heat island effect, thereby reducing air-
somewhere. The word “hopped” means to rush conditioning needs, which in turn reduces the
or to go quickly, which perfectly fits the blank. amount of fossil fuels required to generate
Rest options do not make sense. electricity. The ability of urban forests
Prime Minister Narendra Modi, hopped across to to sequester atmospheric carbon dioxide and act
Nepal to mark Buddha Purnima celebrations in as a long-term carbon sink is a crucial factor in
Lumbini earlier this week. This one-day visit addressing the effects of global warming.
marks a renewed effort to amend ties with a However, the amount of carbon sequestered by
long-trusted partner and friend in the region. trees depends upon the density of the wood,
Lumbini is the birthplace of Buddha and for size, health and tree species.
many years India and Nepal have not managed Trees canopies can also break rain velocity,
to cooperate and integrate it within the larger reduce runoff flow rates and their
Buddhist place-of-interest framework. There had roots stabilize soils against erosion, hence,
been angst for decades among the Nepali reduce flooding and sedimentation in streams.
citizens regarding India taking away the cream of
Buddhist tourism circuit, while ignoring the 22. Answer: A
birthplace. Meanwhile, Lumbini became a place Hint: The first blank needs an adjective, so the
of interest for China which has been using words like “contemporary”, & “burgeoning” are
all avenues possible to increase its strength in chosen first. But the word burgeoning means
Nepal. flourishing and contemporary means modern.
The word contemporary is more appropriate
21. Answer: D here as no other option provides an adjective for
The correct answer is: CEDBA the first blank.
Hint: The word canopies means the uppermost The correct answer is: EDCBA
branches of the trees in a forest, so this word will Contemporary society is built on materials of
go with the blank “Trees_____ can also break science. But unless the common man

Click Here For Bundle PDF Course | support@guidely.in Page 12 of 13


SBI Clerk & RRB PO Mains PDF Course 2023
ENGLISH Day - 50

understands what the scientists discover in our communicating with their peer group through
laboratories, science becomes meaningless. scholarly publications. Unfortunately, to digest
Communicating science to the public-whether this scholarly publication is beyond the capacity
through the media, museums, or outreach - is of the common man. A good scientist is not
now a burgeoning global industry.The question necessarily a good communicator. Nobel
of who should communicate science to the laureate Paul Dirac was achingly shy and could
public has been debated several times in the not face reporters. He had even thought of
long history of science. Most of the scientific refusing the Nobel Prize in order
community members are interested in to shun publicity.

Click Here For Bundle PDF Course | support@guidely.in Page 13 of 13

You might also like